the  
5221 results



$1,100 per month for 10 years, if the account earns 2% per year
$1,100 per month for 10 years, if the account earns 2% per year What the student or parent is asking is: If they deposit $1,100 per month in a savings/investment account every month for 10 years, and they earn 2% per year, how much will the account be worth after 10 years? Deposits are monthly. But interest crediting is annual. What we want is to match the two based on interest crediting time, which is annual or yearly. 1100 per month. * 12 months in a year = 13,200 per year in deposit Since we matched interest crediting period with deposits, we now want to know: If they deposit $13,200 per year in a savings/investment account every year for 10 years, and they earn 2% per year, how much will the account be worth after 10 years? This is an annuity, which is a constant stream of payments with interest crediting at a certain period. [SIZE=5][B]Calculate AV given i = 0.02, n = 10[/B] [B]AV = Payment * ((1 + i)^n - 1)/i[/B][/SIZE] [B]AV =[/B]13200 * ((1 + 0.02)^10 - 1)/0.02 [B]AV =[/B]13200 * (1.02^10 - 1)/0.02 [B]AV =[/B]13200 * (1.2189944199948 - 1)/0.02 [B]AV =[/B]13200 * 0.21899441999476/0.02 [B]AV = [/B]2890.7263439308/0.02 [B]AV = 144,536.32[/B]

$1.40 pays for 30 minutes of parking. How long can you park for with $2.80?
$1.40 pays for 30 minutes of parking. How long can you park for with $2.80? Immediately, I see that $2.80 is $1.40 * 2 Which means, if $1.40 pays for 30 minutes of parking $1.40 * 2 = $2.80 means $2.80 pays for 30 minutes * 2 = [B]60 minutes or 1 hour [/B] [I]Double the rate means double the time you can park[/I]

$100 fee plus $30 per month. Write an expression that describes the cost of a gym membership after m
$100 fee plus $30 per month. Write an expression that describes the cost of a gym membership after m months. Set up the cost function C(m) where m is the number of months you rent: C(m) = Monthly membership fee * m + initial fee [B]C(m) = 30m + 100[/B]

$100 is invested in a bank account that gives an annual interest rate of 3%, compounded monthly. How
$100 is invested in a bank account that gives an annual interest rate of 3%, compounded monthly. How much money will be in the account after 7 years? 7 years * 12 months per year = 84 periods. Using our [URL='https://www.mathcelebrity.com/compoundint.php?bal=100&nval=84&int=3&pl=Monthly']compound interest calculator[/URL], we get an account balance of: [B]123.34[/B]

$1000 is invested with interest at a rate of 15% per year for 9 years. Find the amount you would hav
$1000 is invested with interest at a rate of 15% per year for 9 years. Find the amount you would have, if it’s continuously compounded Using [URL='https://www.mathcelebrity.com/simpint.php?av=&p=1000&int=15&t=9&pl=Continuous+Interest']our balance calculator[/URL], we get: [B]$3,857.43[/B]

$13 in the bank. You write a check for $17. What is your balance?
$13 in the bank. You write a check for $17. What is your balance? When you write a check, it's a debit against your account, which means we subtract. So we start with $13. We subtract $17 Our balance is $13 - $17 = [B]-$4[/B]

$2,030.00 was invested at 10% per annum compounded annually. What interest has been earned (in dolla
$2,030.00 was invested at 10% per annum compounded annually. What interest has been earned (in dollars correct to the nearest cent) after 5 years? Using our [URL='http://www.mathcelebrity.com/compoundint.php?bal=2030&nval=5&int=10&pl=Annually']compound interest calculator[/URL], we get: [B]3,269.34[/B]

$3.75 in quarters and nickles in her car. The number of nickles is fifteen more than the number of q
$3.75 in quarters and nickels in her car. The number of nickels is fifteen more than the number of quarters. How many of each type of coin does she have? Let the number of nickels be n, and the number of quarters be q. We know nickels are 0.05, and quarters are 0.25. We're given: [LIST=1] [*]n = q + 15 [*]0.05n + 0.25q = 3.75 [/LIST] Substituting (1) into (2), we get: 0.05(q + 15) + 0.25q = 3.75 0.05q + 0.75 + 0.25q = 3.75 Combine like term: 0.3q + 0.75 = 3.75 [URL='https://www.mathcelebrity.com/1unk.php?num=0.3q%2B0.75%3D3.75&pl=Solve']Typing this equation into our calculator[/URL], we get: [B]q = 10[/B] Substituting q = 10 into Equation (1), we get: n = 10 + 15 [B]n = 25[/B]

$45 and you add $2.25 each day
$45 and you add $2.25 each day Let d be the number of days. Our Cost function C(d) is: [B]C(d) = 2.25d + 45[/B]

$500 is deposited into a savings account. The bank offers a 3.5% interest rate and the money is left
$500 is deposited into a savings account. The bank offers a 3.5% interest rate and the money is left in the account for 5 years. How much interest is earned in this situation? Using our [URL='https://www.mathcelebrity.com/compoundint.php?bal=5000&nval=5&int=3.5&pl=Annually']compound interest calculator[/URL], we get interest earned as: [B]938.43[/B]

$6500 is 7/10 of a number. What is the number
The number is $9,285 from our [URL='http://www.mathcelebrity.com/perc.php?num=+5&den=+8&num1=6500&pct1=70&pcheck=2&pct2=70&den1=80&idpct1=10&hltype=1&idpct2=90&pct=82&decimal=+65.236&astart=12&aend=20&wp1=20&wp2=30&pof1=40&pof2=20&pl=Calculate']percentage-decimal-fraction calculator[/URL].

$8000 are invested in a bank account at an interest rate of 10 percent per year. Find the amount in
$8000 are invested in a bank account at an interest rate of 10 percent per year. Find the amount in the bank after 5 years if interest is compounded annually Using our [URL='https://www.mathcelebrity.com/compoundint.php?bal=8000&nval=5&int=10&pl=Annually']compound interest with balance calculator[/URL], we get: [B]12,884.08[/B]

$96 less x dollars
$96 less x dollars The word [I]less[/I] means we subtract, so we have: [B]$96 - $x or $(96 - x)[/B]

%60 of the freshman ate pizza at lunch today. If 180 freshman ate pizza, how many freshman are enro
60% of the freshman ate pizza [URL='http://www.mathcelebrity.com/community/x-apple-data-detectors://3']at lunch today[/URL]. If 180 freshman ate pizza, how many freshman are enrolled at our school? 60% of x = 180 We write this as 0.6x = 180 Divide each side by 0.6 to isolate x. We get x = 300 freshman

π Digits
Free π Digits Calculator - Calculates PI (π) to a set number of decimal places using the Gauss-Legendre Algorithm.

(10+x)-y=10+(x-y)
Removing the parentheses since there is nothing to distribute, we have: 10 + x - y = 10 + x - y This is true!

(2,3)(4,5)(6,7)(8,9) represents a function
(2,3)(4,5)(6,7)(8,9) represents a function Domain is the x-values: x = (2, 4, 6, 8) Range is the y-values: y = (3, 5, 7, 9) The function y, or f(x) is: y = x + 1 where x = (2, 4, 6, 8) Test this function for x = 2: y = 2 + 1 y = 3 Test this function for x = 4: y = 4 + 1 y = 5 Test this function for x = 6: y = 6 + 1 y = 7 Test this function for x = 8: y = 8 + 1 y = 9

(3,-4) lies on the line with equation 3x-2y=k, find k
(3,-4) lies on the line with equation 3x-2y=k, find k Plug in our values: 3(3) -2(-4) = k 9 + 8 = k k = [B]17[/B]

(3,3) radius of 4
(3,3) radius of 4 We have a circle with center (3,3) with a radius of 4. [URL='https://www.mathcelebrity.com/eqcircle.php?h=3&k=3&r=4&calc=1&d1=-1&d2=2&d3=3&d4=2&ceq=%28x+%2B+3%29%5E2+%2B+%28y+-+2%29%5E2+%3D+16&pl=Calculate']Use our circle equation calculator to get the general form and standard form.[/URL]

(4x - 20)/8 = 9y for x
(4x - 20)/8 = 9y for x Cross multiply: 4x - 20 = 8 * 9y 4x - 20 = 72y Add 20 to each side to isolate x: 4x - 20 + 20 = 72y + 20 Cancel the 20 on the left side, we get: 4x = 72y + 20 Divide each side by 4: 4x/4 = (72y + 20)/4 Cancel the 4 on the left side: x = [B](72y + 20)/4[/B]

(98)^2 no calculator mental math
Round 98 to the nearest 10. This is 100. Call this a. Calculate b which is a - our original number: b = 100 - 98 b = 2 We can use this formula: (a - b)^2 = a(a- 2b) + b^2 Given a = 100 and b = 2, we have: (100 - 2)^2 = 100(100 - 2(2)) + 2^2 98^2 = 100(96) + 4 98^2 = 9600 + 4 98^2 = [B]9,604[/B] [MEDIA=youtube]8lQdxVVo-Ps[/MEDIA]

(A intersection B) U (A intersection B')
(A intersection B) U (A intersection B') This is the [B]Universal Set U[/B]. Everything that isn't A and isn't B is everything else.

(A^C)^C
(A^C)^C we read this as The complement of the Complement of Set A The complement of set A is anything NOT in A. But the complement of the complement is anything NOT NOT IN A. Which is just [B]A [MEDIA=youtube]-eo-Bq6qZVM[/MEDIA][/B]

(Sqrt(24) + sqrt(96))/2
[SIZE=6][B](Sqrt(24) + sqrt(96))/2[/B] [B][/B] [B]Simplify sqrt(24)[/B] 24 = 6 * 4 where 4 is the perfect square sqrt(24) = 2 * sqrt(6) [B]Simplify sqrt(96)[/B] 96 = 16 * 6 where 16 is the perfect square sqrt(96) = 4 * sqrt(6) Simplified, we have: (2 * sqrt(6) + 4 * sqrt(6))/2 6 * sqrt(6)/2 [B]3 * sqrt(6)[/B][/SIZE] [SIZE=6] [/SIZE] [MEDIA=youtube]h-4eZOFUR4I[/MEDIA]

+÷+(-)
+÷+(-) Parentheses first( (-) 12 - 6y So we have: +÷+12 - 6y 2y + 5/6y + 12 - 6y Combine like terms: [B]5/6y - 4y + 12[/B]

-10 times the quantity y minus 4
-10 times the quantity y minus 4 The quantity y minus 4: y - 4 10 times this quantity: [B]10(y - 4) [/B]

-11, -9, -7, -5, -3 What is the next number? What is the 200th term in this sequence?
-11, -9, -7, -5, -3 What is the next number? What is the 200th term in this sequence? We see that Term 1 is -11, Term 2 is -9, so we do a point slope equation of (1,-11)(2,-9) to get the [URL='https://www.mathcelebrity.com/search.php?q=%281%2C-11%29%282%2C-9%29']nth term of the formula[/URL] f(n) = 2n - 13 The next number is the 6th term: f(6) = 2(6) - 13 f(6) = 12 - 13 f(6) = [B]-1 [/B] The 200th term is: f(200) = 2(200) - 13 f(200) = 400 - 13 f(200) = [B]387[/B]

-2 <= x +4 < 9
-2 <= x +4 < 9 Subtract 4 from each piece: -2 - 4 <= x < 5 Simplify: [B]-6 <= x < 5 [/B] To find the interval notation, we set up our notation: [LIST] [*]The left side has a solid bracket, since we have an equal sign: [*]The right side has an open parentheses, since we have no equal sign [*][B][-6, 5)[/B] [/LIST]

-2 times the quantity q minus 3
-2 times the quantity q minus 3 q minus 3: q - 3 -2 times the quantity: -2(q - 3)

-2 times the quantity t plus 7
-2 times the quantity t plus 7 The key word here is quantity. In this case, the quantity is t plus 7 t + 7 -2 times the quantity means we multiply -2 times the quantity t + 7 [B]-2(t + 7) [MEDIA=youtube]nUWLUPfX52k[/MEDIA][/B]

-28 is the solution to the sum of a number p and 21
-28 is the solution to the sum of a number p and 21 The sum of a number p and 21: p + 21 The phrase [I]is the solution to[/I] means an equation, so we set p + 21 equal to -28: [B]p + 21 = -28 [/B] If the problem asks you to solve for p, then we [URL='https://www.mathcelebrity.com/1unk.php?num=p%2B21%3D-28&pl=Solve']type this into our search engine[/URL] and we get: p = [B]-49[/B]

-3x to the negative one power
-3x to the negative one power Raising to a negative power means taking 1 over the same expression to the positive power" (-3x)^-1 = 1/-3x = [B]-1/3x[/B]

-3x<= -9 or 5+x<6
-3x<= -9 or 5+x<6 Take each piece: -3x<= -9 Divide each side by -3: x>=3 Now take 5 + x < 6 5 + x < 6 Subtract 5 from each side: x < 1 Joining together the two inequalities, we have: x<1 or x>=3 Use our [URL='http://www.mathcelebrity.com/interval-notation-calculator.php?num=x%3C1orx%3E%3D3&pl=Show+Interval+Notation']interval notion calculator[/URL] to find the interval notation of this compound inequality

-5n - 5n - 5 = 5
-5n - 5n - 5 = 5 Solve for [I]n[/I] in the equation -5n - 5n - 5 = 5 [SIZE=5][B]Step 1: Group the n terms on the left hand side:[/B][/SIZE] (-5 - 5)n = -10n [SIZE=5][B]Step 2: Form modified equation[/B][/SIZE] -10n - 5 = + 5 [SIZE=5][B]Step 3: Group constants:[/B][/SIZE] We need to group our constants -5 and 5. To do that, we add 5 to both sides -10n - 5 + 5 = 5 + 5 [SIZE=5][B]Step 4: Cancel 5 on the left side:[/B][/SIZE] -10n = 10 [SIZE=5][B]Step 5: Divide each side of the equation by -10[/B][/SIZE] -10n/-10 = 10/-10 n = [B]- 1 [URL='https://www.mathcelebrity.com/1unk.php?num=-5n-5n-5%3D5&pl=Solve']Source[/URL][/B]

-65 times the difference between a number and 79 is equal to the number plus 98
-65 times the difference between a number and 79 is equal to the number plus 98 The phrase [I]a number[/I] means an arbitrary variable. Let's call it x. The first expression, [I]the difference between a number and 79[/I] means we subtract 79 from our arbitrary variable of x: x - 79 Next, -65 times the difference between a number and 79 means we multiply our result above by -65: -65(x - 79) The phrase [I]the number[/I] refers to the arbitrary variable x earlier. The number plus 98 means we add 98 to x: x + 98 Now, let's bring it all together. The phrase [I]is equal to[/I] means an equation. So we set -65(x - 79) equal to x + [B]98: -65(x - 79) = x + 98[/B] <-- This is our algebraic expression If the problem asks you to take it a step further and solve for x, then you [URL='https://www.mathcelebrity.com/1unk.php?num=-65%28x-79%29%3Dx%2B98&pl=Solve']type this equation into our search engine[/URL], and you get: x = [B]76.31818[/B]

-g + 3/4a = y for a
-g + 3/4a = y for a Add g to each side: -g + g + 3/4a = y + g Cancel the g terms on the left side: 3/4a = y + g Cross multiply: 3a = 4(y + g) Divide each side by 3 to isolate a: 3a/3 = 4(y + g)/3 a = [B]4(y + g)/3[/B]

-g+F/A=h^3 for A
-g+F/A=h^3 for A Add g to each side: -g + g+F/A=h^3 + g Cancel the g's on the left side: F/A = h^3 + g Cross multiply: F = A(h^3 + g) Divide each side by (h^3 + g) F/(h^3 + g) = A(h^3 + g)/(h^3 + g) Cancel (h^3 + g) on the right side: A = [B]F/(h^3 + g)[/B]

-n = n
-n = n Add n to each side: -n + n = n + n Cancel the n's on the left side: 0 = 2n Only number that solves this is [B]n = 0[/B]

-x squared
-x squared We take -x and raise it to the 2nd power: (-x)^2 = -x * -x = [B]x^2[/B]

0,7,14,21 What is the next number? What is the 1000th term?
0,7,14,21 What is the next number? What is the 1000th term? We're adding 7 to the last term, so we get a next term of: 21 + 7 = [B]28 [/B] For our nth term, we notice a pattern for the nth term of: 7n - 7 [LIST] [*]n = 1 --> 7(1) - 7 = 0 [*]n = 2 --> 7(2) - 7 = 7 [*]n = 3 --> 7(3) - 7 = 14 [/LIST] For n = 1000, we have: 7(1000) - 7 = 7000 - 7 = [B]6993[/B]

1 - n = n - 1
1 - n = n - 1 Solve for [I]n[/I] in the equation 1 - n = n - 1 [SIZE=5][B]Step 1: Group variables:[/B][/SIZE] We need to group our variables -n and n. To do that, we subtract n from both sides -n + 1 - n = n - 1 - n [SIZE=5][B]Step 2: Cancel n on the right side:[/B][/SIZE] -2n + 1 = -1 [SIZE=5][B]Step 3: Group constants:[/B][/SIZE] We need to group our constants 1 and -1. To do that, we subtract 1 from both sides -2n + 1 - 1 = -1 - 1 [SIZE=5][B]Step 4: Cancel 1 on the left side:[/B][/SIZE] -2n = -2 [SIZE=5][B]Step 5: Divide each side of the equation by -2[/B][/SIZE] -2n/-2 = -2/-2 n = [B]1 [URL='https://www.mathcelebrity.com/1unk.php?num=1-n%3Dn-1&pl=Solve']Source[/URL][/B]

1 apartment equals 10 window each window cost $79.30 there are 30 apartments what is the total for r
1 apartment equals 10 window each window cost $79.30 there are 30 apartments what is the total for replacing all the windows. 30 apartments * 10 windows per apartment * $79.30 per window = [B]$23,790[/B]

1 box is used every 1.5 days. How many are used in 242 days?
1 box is used every 1.5 days. How many are used in 242 days? Set up a proportion of boxes to days where b is the number of boxes used for 242 days: 1/1.5 = b/242 To solve this proportion for b, we [URL='https://www.mathcelebrity.com/prop.php?num1=1&num2=b&den1=1.5&den2=242&propsign=%3D&pl=Calculate+missing+proportion+value']type it in our search engine[/URL] and we get: b = [B]161.3333[/B]

1 Die Roll
Free 1 Die Roll Calculator - Calculates the probability for the following events in the roll of one fair dice (1 dice roll calculator or 1 die roll calculator):
* Probability of any total from (1-6)
* Probability of the total being less than, less than or equal to, greater than, or greater than or equal to (1-6)
* The total being even
* The total being odd
* The total being a prime number
* The total being a non-prime number
* Rolling a list of numbers i.e. (2,5,6)
* Simulate (n) Monte Carlo die simulations.
1 die calculator

1 egg is 2 dollars. Then how much 2 eggs?
1 egg is 2 dollars. Then how much 2 eggs? 2 eggs * 2 dollars / 1 egg = [B]$4[/B]

1 hour and 54 minutes after 7:30
1 hour and 54 minutes after 7:30 Let's take the easy and lazy way to solve this. 1 hour and 54 minutes is 6 minutes short of 2 hours So we add 2 hours to 7:30: 7:30 + 2 hours = 9:30 Then we subtract off the 6 minutes: 9:30 - 6 minutes = [B]9:24[/B]

1 integer is 7 times another. If the product of the 2 integers is 448, then find the integers.
1 integer is 7 times another. If the product of the 2 integers is 448, then find the integers. Let the first integer be x and the second integer be y. We have the following two equations: [LIST=1] [*]x = 7y [*]xy = 448 [/LIST] Substitute (1) into (2), we have: (7y)y = 448 7y^2 = 448 Divide each side by 7 y^2 = 64 y = -8, 8 We use 8, since 8*7 = 56, and 56*8 =448. So the answer is [B](x, y) = (8, 56)[/B]

1 multiplied by b squared multiplied by c squared
1 multiplied by b squared multiplied by c squared b squared means we raise b to the power of 2: b^2 c squared means we raise c to the power of 2: c^2 b squared multiplied by c squared b^2c^2 1 multiplied by b squared multiplied by c squared means we multiply 1 by b^2c^2 1b^2c^2 Multiplying by 1 can be written by [U][I]removing[/I][/U] the 1 since it's an identity multiplication: [B]b^2c^2[/B]

1 over 14 cubed
1 over 14 cubed 14 cubed means we raise 14 to the power of 3: 14^3 1 over 14 cubed is written as: 1/14^3 To simplify this, we [URL='https://www.mathcelebrity.com/powersq.php?sqconst=+6&num=14%5E3&pl=Calculate']evaluate 14^3[/URL] = 2744 So we have: [B]1/2744[/B]

1 over 2 times the sum of x and y
1 over 2 times the sum of x and y The sum of x and y x + y 2 times the sum of x and y 2(x + y) 1 over 2 times the sum of x and y [B]1/2(x + y)[/B]

1 person is born every 5 seconds. How many people are born in 1 minute?
1 person is born every 5 seconds. How many people are born in 1 minute? Set up the chain: 1 person / 5 seconds * 60 seconds / 1 minute Since 60/5 is 12, and the seconds cancel, we have: [B]12 people / minute[/B]

1 rabbit saw 6 elephants while going to the river. Every elephant saw 2 monkeys going towards the ri
1 rabbit saw 6 elephants while going to the river. Every elephant saw 2 monkeys going towards the river. Every monkey holds 1 parrot in their hands. How many animals are going towards the river? [LIST] [*]1 rabbit = 1 [*]6 elephants per rabbit = 6 [*]2 monkeys per elephant = 12 [*]1 parrot per monkey = 12 parrots [*][B]31 total animals[/B] [/LIST]

1 rabbit saw 9 elephants while going to the river. Every elephant saw 3 monkeys going to the river.
1 rabbit saw 9 elephants while going to the river. Every elephant saw 3 monkeys going to the river. Each monkey had 1 tortoise in each hand. How many animals are going to the river? Trick question: The elephants [U]are not[/U] going to the river. So 1 rabbit goes to the river 3 monkeys go to the river, each holding a tortoise in [B]each hand[/B]. 2 hands per money times 3 monkeys = 6 tortoises So we have 1 rabbit + 3 monkeys + 6 tortoises = [B]10 animals[/B]

1 year from now Mike will be 40 years old. The current sum of the ages of Mike and John is 89. How o
1 year from now Mike will be 40 years old. The current sum of the ages of Mike and John is 89. How old is John right now? If Mike will be 40 1 year from now, then he is: 40 - 1 = 39 years old today. And if the current sum of Mike and John's age is 89, then we use j for John's age: j + 39 = 89 [URL='https://www.mathcelebrity.com/1unk.php?num=j%2B39%3D89&pl=Solve']Type this equation into our search engine[/URL], and we get: [B]j = 50[/B]

1 year from now Paul will be 49 years old. The current sum of the ages of Paul and Sharon is 85. How
1 year from now Paul will be 49 years old. The current sum of the ages of Paul and Sharon is 85. How old is Sharon right now? If Paul will be 49 years old 1 year from now, this means today, he is 49 - 1 = 48 years old. Let Sharon's age be s. Then from the current sum of Paul and Sharon's ages, we get: s + 49 = 85 [URL='https://www.mathcelebrity.com/1unk.php?num=s%2B49%3D85&pl=Solve']Type this equation into our search engine[/URL], and get: s = [B]36[/B]

1, 1/2, 1/3, 1/4, 1/5 What is the next number? What is the 89th term of the sequence?
1, 1/2, 1/3, 1/4, 1/5 What is the next number? What is the 89th term of the sequence? Formula for nth term is 1/n Next number is n = 5, so we have [B]1/5[/B] With n = 89, we have [B]1/89[/B]

1, 1/2, 1/4, 1/8, 1/16 The next number in the sequence is 1/32. What is the function machine you wou
1, 1/2, 1/4, 1/8, 1/16 The next number in the sequence is 1/32. What is the function machine you would use to find the nth term of this sequence? Hint: look at the denominators We notice that 1/2^0 = 1/1 = 1 1/2^1 = 1/2 1/2^2 = 1/4 1/2^3 = 1/8 1/2^4 = 1/32 So we write our explicit formula for term n: f(n) = [B]1/2^(n - 1)[/B]

1, 1/2, 1/4, 1/8, 1/16, ...
At first glance, we see powers of 2 in the denominator of every term except the first one. But if we remember 2^0 = 1, we get our breakthrough. 1/2^0 = 1/1 = 1 Therefore, we stagger the powers of 2 by 1 less than the term we are on: a(n) = [B]1/2^(n - 1) [MEDIA=youtube]Ua-arUukOew[/MEDIA][/B]

1, 4, 9, 16, 25 What is the next number? What is the 50th term?
1, 4, 9, 16, 25 What is the next number? What is the 50th term? We see that 1^2 = 1, 2^2 = 4, 3^2 = 9, 4^2 = 16, 5^2 = 25 We build a formula for the nth term: f(n) = n^2 The next number means n = 6th term: f(6) = 6^2 = [B]36 [/B] The 50th term means n = 50: f(50) = 50^2 = [B]2500[/B]

1, 8, 27, 64 What is the 10th term?
1, 8, 27, 64 What is the 10th term? We see the following pattern: 1^3 = 1 2^3 = 8 3^3 = 27 4^3 = 64 We build our sequence function using this pattern: f(n) = n^3 With n = 10, we have: f(10) = 10^3 f(10) = [B]1,000[/B]

1, 9, 25, 49, .......... What is next
1, 9, 25, 49, .......... What is next 1^2 = 1 3^2 = 9 5^2 = 25 7^2 = 49 So this pattern takes odd numbers and squares them. Our next odd number is 9: 9^2 = [B]81[/B]

1.25, 2, 2.75, 3.5 What is the 100th term?
1.25, 2, 2.75, 3.5 What is the 100th term? The formula of nth term is: f(n) = 0.75n + 0.5 So the 100th term is: f(100) = 0.75(100) + 0.5 f(100) = 75 + 0.5 f(100) = [B]75.5[/B]

1/2 of a number decreased by twice a number
1/2 of a number decreased by twice a number [LIST=1] [*]The phrase [I]a number[/I] means an arbitrary variable, let's call it x. [*]1/2 of a number: x/2 [*]Twice a number means we multiply x by 2: 2x [*]The phrase [I]decreased by[/I] means we subtract [/LIST] [B]x/2 - 2x[/B]

1/2 of the product x and y
1/2 of the product x and y The product x and y: xy 1/2 of the product: [B]xy/2[/B]

1/2 of x and 10 is 30. Find the x.
1/2 of x and 10 is 30. Find the x. x and 10 means we add: x + 10 1/2 of this: 1/2(x + 10) The phrase is means equal to, so we set 1/2(x + 10) equal to 30 for our algebraic expression [B]1/2(x + 10) = 30[/B]

1/2 the difference of x and 4
1/2 the difference of x and 4 The difference of x and 4: x - 4 1/2 of the difference means we divide x -4 by 2: [B](x - 4)/2[/B]

1/2 the quantity of x plus y
1/2 the quantity of x plus y The quantity of x plus y x + y 1/2 the quantity means we multiply x + y by 1/2: [B](x + y)/2[/B]

1/2, 3, 5&1/2, 8......203 What term is the number 203?
1/2, 3, 5&1/2, 8......203 What term is the number 203? We see the following pattern: 1/2 = 2.5*1 - 2 3 = 2.5*2 - 2 5&1/2 = 2.5*3 - 2 8 = 2.5*4 - 2 We build our function f(n) = 2.5n - 2 Set 2.5n - 2 = 203 Using our [URL='https://www.mathcelebrity.com/1unk.php?num=2.5n-2%3D203&pl=Solve']equation solver[/URL], we get: n = [B]82[/B]

1/2a-10b=c solve for a
1/2a-10b=c solve for a Multiply each side of the equation by 2: 2/2a - 2(10)b = 2c Simplify: a - 20b = 2c Add 20b to each side: a - 20b + 20b = 2c + 20b Cancel the 20b on the left side: [B]a = 2c + 20b [/B] You can also factor out a 2 on the left side for another version of this answer: [B]a = 2(c + 10b)[/B]

1/3 a number increased by 10 times by that same number
1/3 a number increased by 10 times by that same number The phrase [I]a number[/I] means an arbitrary variable, let's call it x. 1/3 a number 1/3 * x = x/3 That same number means the same arbitrary variable as above: x 10 times that same number: 10x The phrase [I]increased by[/I] means we add: [B]x/3 + 10x [MEDIA=youtube]29TGt3i28jw[/MEDIA][/B]

1/3 of students at a school are boys. If there are 600 students at the school, how many are girls?
1/3 of students at a school are boys. If there are 600 students at the school, how many are girls? If 1/3 are boys, then the number of boys is: 600 * 1/3 600/3 We [URL='https://www.mathcelebrity.com/fraction.php?frac1=600%2F3&frac2=3%2F8&pl=Simplify']type this fraction into our search engine to simplify[/URL], and we get: 200 Now we need to find how many girls are at the school: Girls = Total Students - Boys Girls = 600 - 200 Girls = [B]400[/B]

1/3 of the sum of 4 and p
1/3 of the sum of 4 and p The sum of 4 and p: 4 + p 1/3 of this sum [B](4 + p)/3[/B]

1/3 of the sum of a number and 2 plus 5 is -20
1/3 of the sum of a number and 2 plus 5 is -20 The phrase [I]a number[/I] means an arbitrary variable, let's call it x. x the sum of a number and 2: x + 2 1/3 of the sum of a number and 2 1/3(x + 2) 1/3 of the sum of a number and 2 plus 5 1/3(x + 2) + 5 The phrase [I]is[/I] means equal to, so we set 1/3(x + 2) + 5 equal to -20: [B]1/3(x + 2) + 5 = -20[/B]

1/3 times q plus 5 equal q minus 4
1/3 times q plus 5 equal q minus 4 1/3 times q plus 5: (q + 5)/3 q minus 4: q - 4 The word [I]equal[/I] means we set (q + 5)/3 equal to q - 4: [B](q + 5)/3 = q - 4[/B]

1/3c increased by the square root of d
1/3c increased by the square root of d square root of d: sqrt(d) 1/3c increased by the square root of d [B]1/3c + sqrt(d)[/B]

1/4 of the difference of 6 and a number is 200
1/4 of the difference of 6 and a number is 200 Take this [B]algebraic expression[/B] in 4 parts: [LIST=1] [*]The phrase [I]a number[/I] means an arbitrary variable, let's call it x [*]The difference of 6 and a number means we subtract x from 6: 6 - x [*]1/4 of the difference means we divide 6 - x by 4: (6 - x)/4 [*]Finally, the phrase [I]is[/I] means an equation, so we set (6 - x)/4 equal to 200 [/LIST] [B](6 - x)/4 = 200[/B]

1/4 of the sum of 6 and 3n
[U]The sum of 6 and 3n:[/U] 6 + 3n [U]1/4 of that sum[/U] [B]1/4(6 + 3n)[/B]

1/4 of the sum of n and 40
1/4 of the sum of n and 40. The sum of n and 40 n + 40 1/4 of this: (n + 40)/4

1/5 of the sum of the number u and 2
1/5 of the sum of the number u and 2 The sum of the number u and 2 means we add 2 to u: u + 2 1/5 of the sum: [B](u + 2)/5[/B]

1/6 times the sum k and 5
1/6 times the sum k and 5 The sum k and 5 (k + 5) 1/6 times the sum k and 5 (k + 5)/6

1/9 of all sales were for cash. If cash sales were $59,000, what were the total sales?
1/9 of all sales were for cash. If cash sales were $59,000, what were the total sales? Let sales be s. We're given: s/9 = 59000 To solve this proportion for s, we [URL='https://www.mathcelebrity.com/prop.php?num1=s&num2=59000&den1=9&den2=1&propsign=%3D&pl=Calculate+missing+proportion+value']type it in our search engine[/URL] and we get: s = [B]531000[/B]

1/a + 1/b = 1/2 for a
1/a + 1/b = 1/2 for a Subtract 1/b from each side to solve this literal equation: 1/a + 1/b - 1/b = 1/2 - 1/b Cancel the 1/b on the left side, we get: 1/a = 1/2 - 1/b Rewrite the right side, using 2b as a common denominator: 1/a = (b - 2)/2b Cross multiply: a(b - 2) = 2b Divide each side by (b - 2) a = [B]2b/(b - 2)[/B]

1/n^2 = 3/192
1/n^2 = 3/192 Cross multiply: 192 * 1 = 3 * n^2 3n^2 = 192 Divide each side by 3: 3n^2/3 = 192/3 Cancel the 3's on the left side: n^2 = 64 Take the square root of both sides: n = [B]8 or -8[/B]

10 divided by the sum of 4 and u
10 divided by the sum of 4 and u Take this algebraic expression in parts: The sum of 4 and u means we add 4 to u: 4 + u Next, we divide 10 by this sum: [B]10/(4 + u)[/B]

10 is twice the sum of x and 5
10 is twice the sum of x and 5 The sum of x and 5 means we add: x + 5 Twice the sum means we multiply by 2: 2(x + 5) The word [I]is[/I] means an equation, so we set 2(x + 5) equal to 10 [B]2(x + 5) = 10[/B]

10 less than half the sum of a and b
Sum of a and b a + b half the sum of a and b (a + b)/2 10 less than that (a + b)/2 - 10

10 minus z to the 8th
10 minus z to the 8th z to the 8th z^8 10 minus this: 10 - z^8

10 more dogs than cows
Let d be dogs and c be cows. Then d - 10 = c or c + 10 = d

10 more than a number z, divided by k
10 more than a number z, divided by k The phrase [I]a number[/I] means an arbitrary variable, lets call it x. 10 more than a number means we add 10 to x: x + 10 We divide this quantity by k: [B](x + 10)/k[/B]

10 students play tennis, 5 students play soccer, and 4 students play both. How many students are in
10 students play tennis, 5 students play soccer, and 4 students play both. How many students are in the class? We want Tennis + Soccer - Both 10 + 5 - 4 [B]11[/B] students in the class

10 times a number is 420
10 times a number is 420 A number denotes an arbitrary variable, let's call it x. 10 times a number: 10x The phrase is means equal to, so we set 10x equal to 420 [B]10x = 420 <-- This is our algebraic expression [/B] If you want to solve for x, use our [URL='http://www.mathcelebrity.com/1unk.php?num=10x%3D420&pl=Solve']equation calculator[/URL] We get x = 42

10 times the first of 2 consecutive even integers is 8 times the second. Find the integers
10 times the first of 2 consecutive even integers is 8 times the second. Find the integers. Let the first integer be x. Let the second integer be y. We're given: [LIST=1] [*]10x = 8y [*]We also know a consecutive even integer means we add 2 to x to get y. y = x + 2 [/LIST] Substitute (1) into (2): 10x = 8(x + 2) Multiply through: 10x = 8x + 16 To solve for x, [URL='https://www.mathcelebrity.com/1unk.php?num=10x%3D8x%2B16&pl=Solve']we type this equation into our search engine[/URL] and we get: [B]x = 8[/B] Since y = x + 2, we plug in x = 8 to get: y = 8 + 2 [B]y = 10 [/B] Now, let's check our work. Does x = 8 and y = 10 make equation 1 hold? 10(8) ? 8(10) 80 = 80 <-- Yes!

10 times the square of a number w divided by 12
10 times the square of a number w divided by 12 The square of a number w w^2 10 times this 10w^2 Divided by 12 [B]10w^2/12[/B]

10 x 12 divided by 9
10 x 12 divided by 9 12/9 1.3333333 Then multiply by 10: [B]13.33333333[/B]

10% of the days in June were sunny. How many days in June were sunny?
10% of the days in June were sunny. How many days in June were sunny? June has 30 days. We [URL='https://www.mathcelebrity.com/perc.php?num=+5&den=+8&num1=+16&pct1=+80&pct2=10&den1=30&pcheck=3&pct=+82&decimal=+65.236&astart=+12&aend=+20&wp1=20&wp2=30&pl=Calculate']type in 10% of 30[/URL] in our search engine: [B]3 days[/B]

10, 1,000, 100,000, 10,000,000 What power of 10 is the 80th term?
10, 1,000, 100,000, 10,000,000 What power of 10 is the 80th term? We see the following pattern 10^1 = 10 10^3 = 1000 10^5 = 100,000 10^7 = 10,000,000 f(n) = 10^(2n - 1) We build the 80th term: f(80) = 10^(2(80) - 1) f(80) = 10^(160 - 1) f(80) = 10^[B]159[/B]

100, 75, 50, 25, 0, -25 What is the next number? What is the 100th term?
100, 75, 50, 25, 0, -25 What is the next number? What is the 100th term? Using point slope, we get (1, 100)(2, 75) Our [URL='https://www.mathcelebrity.com/search.php?q=%281%2C+100%29%282%2C+75%29&x=0&y=0']series function becomes[/URL] f(n) = -25n + 125 The next term is the 7th term: f(7) = -25(7) + 125 f(7) = -175 + 125 f(7) = [B]-50 [/B] The 100th term is found by n = 100: f(100) = -25(100) + 125 f(100) = -2500 + 125 f(100) = [B]-2375[/B]

100n = 100
100n = 100 Solve for [I]n[/I] in the equation 100n = 100 [SIZE=5][B]Step 1: Divide each side of the equation by 100[/B][/SIZE] 100n/100 = 100/100 n = [B]1[/B]

104 subtracted from the quantity 6 times r is the same as r
104 subtracted from the quantity 6 times r is the same as r The quantity 6 times r means we multiply 6 by r: 6r 104 subtracted from 6r is written as: 6r - 104 [B]The phrase [I]is the same as[/I] means we have an equation. So we set 6r - 104 equal to r 6r - 104 = r[/B]

108 times a, reduced by 147 is k subtracted from 56
108 times a, reduced by 147 is k subtracted from 56 Take this algebraic expression in pieces: Step 1: 108 times a: 108a Step 2: Reduced by means subtract, so we subtract 47 from 108a: 108a - 47 Step 3: ksubtracted from 56: 56 - k Step 4: The phrase [I]is[/I] means equal to, so we set 108a - 47 equal to 56 - k [B]108a - 47 = 56 - k [MEDIA=youtube]KrY6uzKeeB0[/MEDIA][/B]

1089 Number Trick
Free 1089 Number Trick Calculator - Demonstrates the 1089 number trick for a 3 digit number that you enter

10ac-x/11=3 for a
10ac-x/11=3 for a Add x/11 to each side of the equation to isolate a: 10ac - x/11 + x/11 = 3 + x/11 Cancelling the x/11 on the left side, we get: 10ac = 3 + x/11 Divide each side by 10c to isolate a: 10ac/10c = 3 + x/11 Cancelling the 10c on the left side, we get: a = [B]3/10c + x/110c[/B]

10n - 9n + 8n - 7n + 6n = 10 - 9 + 8 - 7 + 6
10n - 9n + 8n - 7n + 6n = 10 - 9 + 8 - 7 + 6 Solve for [I]n[/I] in the equation 10n - 9n + 8n - 7n + 6n = 10 - 9 + 8 - 7 + 6 [SIZE=5][B]Step 1: Group the n terms on the left hand side:[/B][/SIZE] (10 - 9 + 8 - 7 + 6)n = 8n [SIZE=5][B]Step 2: Group the constant terms on the right hand side:[/B][/SIZE] 10 - 9 + 8 - 7 + 6 = 8 [SIZE=5][B]Step 3: Form modified equation[/B][/SIZE] 8n = + 8 [SIZE=5][B]Step 4: Divide each side of the equation by 8[/B][/SIZE] 8n/8 = 8/8 n = [B]1[/B]

10n = 0.5
10n = 0.5 Solve for [I]n[/I] in the equation 10n = .5 [SIZE=5][B]Step 1: Divide each side of the equation by 10[/B][/SIZE] 10n/10 = .5/10 n = [B]0.05 [URL='https://www.mathcelebrity.com/1unk.php?num=10n%3D.5&pl=Solve']Source[/URL][/B]

11 more then n
The Phrase more then means add, so we have: n + 11

11 to the power of 6 multiply 11 to the power of 3
11 to the power of 6 multiply 11 to the power of 3 Take this in parts. [U]Step 1: 11 to the power of 6 means we raise 11 to the 6th power using exponents:[/U] 11^6 [U]Step 2: 11 to the power of 3 means we raise 11 to the 3rd power using exponents:[/U] 11^3 [U]Step 3: Multiply each term together:[/U] 11^6 * 11^3 [U]Step 4: Simplify[/U] Because we have 2 numbers that are the same, in this case, 11, we can add the exponents together when multiplying: 11^(6 + 3) [B]11^9 [MEDIA=youtube]gCxVq7LqyHk[/MEDIA][/B]

110 subtracted from the product of 244 and w is the product of r and 177 increased by 266
110 subtracted from the product of 244 and w is the product of r and 177 increased by 266 The product of 244 and w: 244w 110 subtracted from the product of 244 and w 244w - 110 the product of r and 177 177r the product of r and 177 increased by 266 177r + 266 The word [I]is[/I] means equal to, so we set 244w - 110 equal to 177r + 266 [B]244w - 110 = 177r + 266[/B]

12 Hour Clock Conversion
Free 12 Hour Clock Conversion Calculator - This calculator performs the following two conversions:
1) Takes a time in 24 hour clock (military time) format and converts it to a 12 hour clock format (AM/PM)
2) Takes a time in 12 hour clock format and converts it to military time (12 hour clock format)

12 is multiplied by some number, that product is reduced by 9, and the total is equal to 37
12 is multiplied by some number, that product is reduced by 9, and the total is equal to 37 The phrase [I]some number[/I] means an arbitrary variable, let's call it x. 12 multiplied by this number: 12x The product of 12x is reduced by 9 12x - 9 The phrase [I]the total is equal to[/I] means an equation, so we set 12x - 9 equal to 37: [B]12x - 9 = 37[/B]

12 is subtracted from d and the result is tripled
12 is subtracted from d and the result is tripled 12 is subtracted from d: d - 12 the result is tripled means we multiply d - 12 by 3 [B]3(d - 12)[/B]

12 is subtracted from d and the result is tripled.
12 is subtracted from d and the result is tripled. 12 is subtracted from d: d - 12 The result is tripled means we multiply d - 12 by 3 [B]3(d - 12) [MEDIA=youtube]1xqWstiIDP0[/MEDIA][/B]

12 laps in 18 minutes . What is the average time per lap?
12 laps in 18 minutes . What is the average time per lap? 18/12 = [B]1.5 minutes per lap[/B]

12 plus 6 times a number is 9 times the number
12 plus 6 times a number is 9 times the number The phrase [I]a number [/I]means an arbitrary variable. Let's call it x. 6 times a number is written as: 6x 12 plus 6 times the number means we add 6x to 12: 12 + 6x 9 times a number is written as: 9x The phrase [I]is[/I] means an equation, so we set 12 + 6x equal to 9x [B]12 + 6x = 9x <-- This is our algebraic expression[/B] [B][/B] If the problem asks you to solve for x, then you [URL='https://www.mathcelebrity.com/1unk.php?num=12%2B6x%3D9x&pl=Solve']type this expression into our search engine[/URL] and you get: x = [B]4[/B]

12 plus the product of 4 and a number is greater than 72
A number means an arbitrary variable, let's call it x. The product of 4 and a number is 4x. 12 plus that product is 4x + 12 Is greater than means an inequality, so we set the entire expression greater than 72 4x + 12 > 72

12 students want pancakes and 14 students want waffles. What is the ratio of the number of students
12 students want pancakes and 14 students want waffles. What is the ratio of the number of students who want pancakes to the total number of students? 12/14 is the initial ratio. However, we can simplify this. [URL='https://www.mathcelebrity.com/fraction.php?frac1=12%2F14&frac2=3%2F8&pl=Simplify']So we type 12/14 into our search engine and choose simplify.[/URL] We get: 6/7

12.66 g of calcium are heated in air,17.73 g of calcium oxide is formed.The percent oxygen in the co
12.66 g of calcium are heated in air,17.73 g of calcium oxide is formed.The percent oxygen in the compound is? 17.73g Calcium oxide - 12.66g Calcium = 5.07g Oxygen Mass of the element / Mass of the compound * 100% = 5.07g O / 17.73g CaO2 X 100% = 28.6% of Oxygen in the compound [B][U]Check Your Work [/U][/B] Mass of the element / Mass of the compound X 100% = 12.66g Ca ÷ 17.73g CaO2 X 100% = 71.4% Ca 71.4% Ca + 28.6% O = 100% CaO2

1225 people live in a village,329 are men and 404 are women. how many are children
1225 people live in a village,329 are men and 404 are women. how many are children We can have either men, women, or children. We have the following equation where children are "c". 239 + 404 + c = 1225 To solve for c, we [URL='https://www.mathcelebrity.com/1unk.php?num=239%2B404%2Bc%3D1225&pl=Solve']type this equation into our search engine[/URL] and we get: c = [B]582[/B]

13 is the product of 5p and 5
13 is the product of 5p and 5 the product of 5p and 5 means we multiply 5p by 5: 5p * 5 25p The word [I]is[/I] means equal to, so we set 25p equal to 13 [B]13 = 25p 25p = 13[/B]

13 minutes to answer 4 problems. how many minutes would it take to answer 22 questions
13 minutes to answer 4 problems. how many minutes would it take to answer 22 questions? Set up a proportion of time to problems where m is the number of minutes it would take for 22 questions: 13/4 = m/22 [URL='https://www.mathcelebrity.com/prop.php?num1=13&num2=m&den1=4&den2=22&propsign=%3D&pl=Calculate+missing+proportion+value']Type this proportion into the search engine[/URL], and we get: m = [B]71.5[/B]

13 times the sum of x and 9y
13 times the sum of x and 9y The sum of x and 9y means we add 9y to x: x + 9y Now multiply this sum by 13: [B]13(x + 9y)[/B]

132 is 393 multiplied by y
132 is 393 multiplied by y 393 multiplied by y 393y The word [I]is[/I] means equal to, so we set 393y equal to 132 as our algebraic expression [B]393y = 132 [/B] If you need to solve for y, use our [URL='http://www.mathcelebrity.com/1unk.php?num=393y%3D132&pl=Solve']equation calculator[/URL]

14 increased by twice Carlos’s age. Use the variable c to represent Carlos age
14 increased by twice Carlos’s age. Use the variable c to represent Carlos age Twice means me multiply a by 2: 2a 14 increased by twice Carlos's age means we add 2a to 14: [B]14 + 2a[/B]

14 is the what of 1/14?
14 is the what of 1/14? [B]Denominator[/B]

14 oranges $3.78
14 oranges $3.78 Using our [URL='http://www.mathcelebrity.com/unit-cost-calculator.php?num=14orangesfor3.78&pl=Calculate+Unit+Cost']unit cost calculator[/URL], we get: [B]$0.27 per orange[/B]. You could also enter in the search engine: 14 oranges for $3.78

149 cars are waiting to take a ferry across the channel each ferry can only hold 18 cars how many tr
149 cars are waiting to take a ferry across the channel each ferry can only hold 18 cars how many trips will it take to get all the cars across Number of trips = Total Cars / Cars Per ferry trip Number of trips = 149/18 Number of trips = 8.28 trips We round up to the next integer and we have [B]9 trips[/B]

15 added to a number is 16 times the number
15 added to a number is 16 times the number [LIST=1] [*]The phrase [I]a number[/I] means an arbitrary variable, let's call it x. [*]15 added to a number: 15 + x [*]16 times the number: 16x [*]The phrase [I]is[/I] means equal to. So we set 15 + x equal to 16x [/LIST] [B]15 + x = 16x[/B]

15 added to the quotient of 8 and a number is 7.
15 added to the quotient of 8 and a number is 7. Take this algebraic expression in pieces: [LIST] [*]The phrase [I]a number[/I] means an arbitrary variable. Let's call it x. [*]The quotient of 8 and a number: 8/x [*]15 added to this quotient: 8/x + 15 [*]The word [I]is[/I] means an equation, so we set 8/x + 15 equal to 7 [/LIST] [B]8/x + 15 = 7[/B]

15 mins into fraction of an hour
15 mins into fraction of an hour [URL='https://www.mathcelebrity.com/timecon.php?quant=1&pl=Calculate&type=hour']An hour is 60 minutes[/URL], so we have the fraction: 15/60 But we can simplify this fraction. We [URL='https://www.mathcelebrity.com/fraction.php?frac1=15%2F60&frac2=3%2F8&pl=Simplify']type in 15/60 into our search engine[/URL], click Simplify, and we get: [B]1/4[/B]

15 of the 45 students have brown eyes. What fraction of the students have brown eyes
15 of the 45 students have brown eyes. What fraction of the students have brown eyes? 15/45 can be [URL='https://www.mathcelebrity.com/fraction.php?frac1=15%2F45&frac2=3%2F8&pl=Simplify']simplified using our fraction simplify calculator[/URL]: [B]1/3[/B]

15 out of 18 students agreed. What percentage did not?
18 - 15 = 3 student disagreed. 3/18 is the fraction of student who disagreed. [URL='http://www.mathcelebrity.com/perc.php?num=3&den=18&pcheck=1&num1=16&pct1=80&pct2=70&den1=80&idpct1=10&hltype=1&idpct2=90&pct=82&decimal=+65.236&astart=12&aend=20&wp1=20&wp2=30&pl=Calculate']Converting to a percentage using our fraction to decimal calculator, we get:[/URL] 16.67%

15y + 13/c = m for y
15y + 13/c = m for y Subtract 13/c from each side to isolate the y term: 15y + 13/c - 13/c = m - 13/c Cancel the 13/c on the left side and we get 15y = m - 13/c Now, divide each side by 15 to isolate y: 15y/15 = (m - 13/c)/15 Cancel the 15 on the left side, and we get: y = [B](m - 13/c)/15[/B]

16 decreased by 3 times the sum of 3 and a number
16 decreased by 3 times the sum of 3 and a number Take this algebraic expression in parts: [LIST] [*]The phrase [I]a number[/I] means an arbitrary variable. Let's call it x. [*]The sum of 3 and a number: 3 + x [*]3 times the sum: 3(3 + x) [*]16 decreased by... means we subtract 3(3 + x) from 16 [/LIST] [B]3(3 + x) from 16[/B]

17 decreased by three times d equals c
17 decreased by three times d equals c three times d means we multiply d by 3: 3d 17 decreased by three times d means we subtract 3d from 17 17 - 3d The word [I]equals[/I] means an equation, so we set 17 - 3d equal to c: [B]17 - 3d = c[/B]

17 multiplied by the quantity 9 minus 5
17 multiplied by the quantity 9 minus 5 The quantity 9 minus 5: 9 - 5 17 multiplied by the quantity means we wrap 9 - 5 in parentheses: [B]17(9 - 5)[/B]

17 people over the maximum capacity
17 people over the maximum capacity Let the maximum capacity be c. We have: [B]c + 17[/B]

175 out of 200 students have a cell phone. What fraction of the students have a cell phone?
175 out of 200 students have a cell phone. What fraction of the students have a cell phone? 175 out of 200 can be written as: 175/200 This can be simplified, so we [URL='https://www.mathcelebrity.com/fraction.php?frac1=175%2F200&frac2=3%2F8&pl=Simplify']type it in our math engine[/URL] and we get: [B]7/8[/B]

175 students separated into n classes is 25
175 students separated into n classes is 25 [U]Divide 175 by n[/U] 175/n [U]The word [I]is[/I] means equal to, so set this expression equal to 25[/U] 175/n = 25 [U]Cross multiply[/U] 25n = 175 [U]Divide each side by 25[/U] [B]n = 7[/B]

18 divided by the difference of t and 9
The difference of t and 9 is: t - 9 Now, we set up a quotient with 18 as the numerator and t - 9 as the denominator 18 ------ t - 9

18 multiplied by the quantity of 11 plus r
18 multiplied by the quantity of 11 plus r The quantity of 11 plus r is written as: 11 + r 18 multiplied by the [I]quantity[/I] means we take 18 and multiply it by the term 11 + r [B]18(11 + r) [MEDIA=youtube]2GYjQTjt8qM[/MEDIA][/B]

19 decreased by the absolute value of c
19 decreased by the absolute value of c Take this algebraic expression in parts: [LIST] [*]Absolute value of c: |c| [*]19 decreased by the absolute value of c is found by subtracting |c| from 19 [/LIST] [B]19 - |c|[/B]

19 increased by twice Greg’s score use the variable g to represent Greg’s score
19 increased by twice Greg’s score use the variable g to represent Greg’s score Use g for Greg's score g Twice g means we multiply g by 2: 2g 19 increased by means we add 2g to 19 [B]2g + 19 [MEDIA=youtube]E9a_U7z-fHE[/MEDIA][/B]

19 increased by twice Vanessa's age
19 increased by twice Vanessa's age Let Vanessa's age be a. Twice means we multiply a by 2: 2a The phrase [I]increased by[/I] means we add 2a to 19: [B]19 + 2a[/B]

2 Asset Portfolio
Free 2 Asset Portfolio Calculator - Given a portfolio with 2 assets, this determines the expected return (mean), variance, and volatility (standard deviation) of the portfolio.

2 baseball players hit 60 home runs combined last season. The first player hit 3 more home runs than
2 baseball players hit 60 home runs combined last season. The first player hit 3 more home runs than twice a number of home runs the second player hit. how many home runs did each player hit? Declare variables: Let the first players home runs be a Let the second players home runs be b We're given two equations: [LIST=1] [*]a = 2b + 3 [*]a + b = 60 [/LIST] To solve this system of equations, we substitute equation (1) into equation (2) for a: 2b + 3 + b = 60 Using our math engine, we [URL='https://www.mathcelebrity.com/1unk.php?num=2b%2B3%2Bb%3D60&pl=Solve']type this equation[/URL] in and get: b = [B]19 [/B] To solve for a, we substitute b = 19 into equation (1): a = 2(19) + 3 a = 38 + 3 a = [B]41[/B]

2 buses leave at 5:30am, 1 comes every 18 minutes and one comes every 15 minutes when will they both
2 buses leave at 5:30am, 1 comes every 18 minutes and one comes every 15 minutes when will they both come at the same time again We want the Least Common Multiple (LCM) of 15 and 18. LCM(15, 18) Enter this into the [URL='http://www.mathcelebrity.com/gcflcm.php?num1=15&num2=18&num3=&pl=LCM']search engine[/URL], and we get: [B]90 minutes[/B]

2 cards have different expressions written on them.: 5y - 2 and 3y + 10. for what value of y do the
2 cards have different expressions written on them.: 5y - 2 and 3y + 10. for what value of y do the 2 cards represent the same number? If they have the same number, we set them equal to each other and solve for y: 5y - 2 = 3y + 10 To solve for y, we [URL='http://5y - 2 = 3y + 10']type this expression in our search engine [/URL]and we get: y = [B]6[/B]

2 coins are tossed. Find the probability of getting 1 head and 1 tail
2 coins are tossed. Find the probability of getting 1 head and 1 tail We can either flip HT or TH. Let's review probabilities: [LIST] [*]HT = 1/2 * 1/2 = 1/4 <-- We multiply since each event is independent [*]TH = 1/2 * 1/2 = 1/4 <-- We multiply since each event is independent [/LIST] P(1 H, 1 T) = P(HT) + P(TH) P(1 H, 1 T) = 1/4 + 1/4 P(1 H, 1 T) = 2/4 [URL='https://www.mathcelebrity.com/fraction.php?frac1=2%2F4&frac2=3%2F8&pl=Simplify']Using our fraction simplifier[/URL], we can reduce 2/4 to 1/2 P(1 H, 1 T) = [B]1/2[/B]

2 consecutive even integers that equal 118
Let x be the first even integer. That means the next consecutive even integer must be x + 2. Set up our equation: x + (x + 2) = 118 Group x terms 2x + 2 = 118 Subtract 2 from each side 2x = 116 Divide each side by 2 x = 58 Which means the next consecutive even integer is 58 + 2 = 60 So our two consecutive even integers are [B]58, 60[/B] Check our work: 58 + 60 = 118

2 consecutive odd integers such that their product is 15 more than 3 times their sum
2 consecutive odd integers such that their product is 15 more than 3 times their sum. Let the first integer be n. The next odd, consecutive integer is n + 2. We are given the product is 15 more than 3 times their sum: n(n + 2) = 3(n + n + 2) + 15 Simplify each side: n^2 + 2n = 6n + 6 + 15 n^2 + 2n = 6n + 21 Subtract 6n from each side: n^2 - 4n - 21 = 0 [URL='https://www.mathcelebrity.com/quadratic.php?num=n%5E2-4n-21%3D0&pl=Solve+Quadratic+Equation&hintnum=+0']Type this problem into our search engine[/URL], and we get: n = (-3, 7) If we use -3, then the next consecutive odd integer is -3 + 2 = -1. So we have [B](-3, -1)[/B] If we use 7, then the next consecutive odd integer is 7 + 2 = 9. So we have [B](7, 9)[/B]

2 dice are rolled what is the probability that doubles are rolled less than 11
2 dice are rolled what is the probability that doubles are rolled less than 11 List out the doubles with a sum less than 11: (1, 1) (2, 2) (3, 3) (4, 4) (5, 5) The probability of each double is 1/6 * 1/6 = 1/36. We have 5 of them, so we have 5*1/36 = [B]5/36[/B]

2 dice roll
Free 2 dice roll Calculator - Calculates the probability for the following events in a pair of fair dice rolls:
* Probability of any sum from (2-12)
* Probability of the sum being less than, less than or equal to, greater than, or greater than or equal to (2-12)
* The sum being even
* The sum being odd
* The sum being a prime number
* The sum being a non-prime number
* Rolling a list of numbers i.e. (2,5,6,12)
* Simulate (n) Monte Carlo two die simulations. 2 dice calculator

2 fair sided die are rolled. How many ways can the dice be rolled to sum exactly 6?
2 fair sided die are rolled. How many ways can the dice be rolled to sum exactly 6? [URL='https://www.mathcelebrity.com/2dice.php?gl=1&pl=6&opdice=1&rolist=+2%2C3%2C9%2C10&dby=2%2C3%2C5&ndby=4%2C5&montect=+100']Using our 2 dice calculator[/URL], we get the following options: [LIST] [*]2,4 [*]3,3 [*]4,2 [*]1,5 [*]5,1 [/LIST] The probability of rolling a sum of 6 is [B]5/36[/B]

2 is added to c, then the result is multiplied by 7
2 is added to c, then the result is multiplied by 7 2 is added to c: c + 2 The result is multiplied by 7: [B]7(c + 2)[/B]

2 less than 3 times n is 4 more than n
2 less than 3 times n is 4 more than n 3 times n: 3n 2 less than 3 times n 3n - 2 4 more than n: n + 4 The word [I]is[/I] means equal to, so we set 3n - 2 equal to n + 4: [B]3n - 2 = n + 4[/B]

2 Lines Intersection
Free 2 Lines Intersection Calculator - Enter any 2 line equations, and the calculator will determine the following:
* Are the lines parallel?
* Are the lines perpendicular
* Do the lines intersect at some point, and if so, which point?
* Is the system of equations dependent, independent, or inconsistent

2 more than twice the sum of 10 and a number
2 more than twice the sum of 10 and a number The phrase [I]a number[/I] means an arbitrary variable, let's call it x. x The sum of 10 and a number means we add x to 10: 10 + x Twice the sum means we multiply 10 + x by 2: 2(10 + x) 2 more than twice the sum means we add 2 to 2(10 + x): [B]2(10 + x) + 2[/B]

2 movie tickets and 3 snacks are $24. 3 movie tickets and 4 snacks are $35. How much is a movie tick
2 movie tickets and 3 snacks are $24. 3 movie tickets and 4 snacks are $35. How much is a movie ticket and how much is a snack? Let a movie ticket cost be m, and a snack cost be s. We have: 2m + 3s = 24.3 3m + 4s = 35 Using the [URL='http://www.mathcelebrity.com/simultaneous-equations.php?term1=2m+%2B+3s+%3D+24.3&term2=3m+%2B+4s+%3D+35&pl=Cramers+Method']simultaneous equations calculator[/URL], we get: m = $7.8 s = $2.9

2 number Word Problems
Free 2 number Word Problems Calculator - This calculator handles word problems in the format below:
* Two numbers have a sum of 70 and a product of 1189 What are the numbers?
* Two numbers have a sum of 70. Their difference 32


2 numbers add to 200. The first is 20 less than the second.
2 numbers add to 200. The first is 20 less than the second. Let the first number be x and the second number be y. We're given: [LIST=1] [*]x + y = 200 [*]x = y - 20 [/LIST] Plug (2) into (1) (y - 20) + y = 200 Group like terms: 2y - 20 = 200 [URL='https://www.mathcelebrity.com/1unk.php?num=2y-20%3D200&pl=Solve']Typing this equation into our search engine[/URL], we get: [B]y = 110[/B] <-- This is the larger number Plug y = 110 into Equation (2) to get the smaller number: x = 110 - 20 [B]x = 90[/B] <-- This is the smaller number Let's check our work for Equation (1) using x = 90, and y = 110 90 + 110 ? 200 200 = 200 <-- Good, our solutions check out for equation (1) Let's check our work for Equation (2) using x = 90, and y = 110 90 = 110 - 20 90 = 90 <-- Good, our solutions check out for equation (2)

2 numbers that add up makes 5 but multiplied makes -36
2 numbers that add up makes 5 but multiplied makes -36 Let the first number be x and the second number be y. We're given two equations: [LIST=1] [*]x + y = 5 [*]xy = -36 [/LIST] Rearrange equation (1) by subtracting y from each side: [LIST=1] [*]x = 5 - y [*]xy = -36 [/LIST] Substitute equation (1) for x into equation (2): (5 - y)y = -36 5y - y^2 = -36 Add 36 to each side: -y^2 + 5y + 36 = 0 We have a quadratic equation. To solve this, we [URL='https://www.mathcelebrity.com/quadratic.php?num=-y%5E2%2B5y%2B36%3D0&pl=Solve+Quadratic+Equation&hintnum=0']type it in our search engine and solve[/URL] to get: y = ([B]-4, 9[/B]) We check our work for each equation: [LIST=1] [*]-4 + 9 = -5 [*]-4(9) = -36 [/LIST] They both check out

2 numbers that are equal have a sum of 60
2 numbers that are equal have a sum of 60 Let's choose 2 arbitrary variables for the 2 numbers x, y Were given 2 equations: [LIST=1] [*]x = y <-- Because we have the phrase [I]that are equal[/I] [*]x + y = 60 [/LIST] Because x = y in equation (1), we can substitute equation (1) into equation (2) for x: y + y = 60 Add like terms to get: 2y = 60 Divide each side by 2: 2y/2 = 60/2 Cancel the 2's and we get: y = [B]30 [/B] Since x = y, x = y = 30 x = [B]30[/B]

2 pens and 1 eraser cost $35 and 3 pens and 4 erasers cost $65. X represents the cost of 1 pen and Y
2 pens and 1 eraser cost $35 and 3 pens and 4 erasers cost $65. X represents the cost of 1 pen and Y represents the cost of 1 eraser. Write the 2 simultaneous equations and solve. Set up our 2 equations where cost = price * quantity: [LIST=1] [*]2x + y = 35 [*]3x + 4y = 65 [/LIST] We can solve this one of three ways: [LIST] [*][URL='https://www.mathcelebrity.com/simultaneous-equations.php?term1=2x+%2B+y+%3D+35&term2=3x+%2B+4y+%3D+65&pl=Substitution']Substitution Method[/URL] [*][URL='https://www.mathcelebrity.com/simultaneous-equations.php?term1=2x+%2B+y+%3D+35&term2=3x+%2B+4y+%3D+65&pl=Elimination']Elimination Method[/URL] [*][URL='https://www.mathcelebrity.com/simultaneous-equations.php?term1=2x+%2B+y+%3D+35&term2=3x+%2B+4y+%3D+65&pl=Cramers+Method']Cramer's Rule[/URL] [/LIST] No matter which method we choose, we get the same answer: [LIST] [*][B]x (cost of 1 pen) = 15[/B] [*][B]y (cost of 1 eraser) = 5[/B] [/LIST]

2 times a number added to another number is 25. 3 times the first number minus the other number is 2
2 times a number added to another number is 25. 3 times the first number minus the other number is 20. Let the first number be x. Let the second number be y. We're given two equations: [LIST=1] [*]2x + y = 25 [*]3x - y = 20 [/LIST] Since we have matching opposite coefficients for y (1 and -1), we can add both equations together and eliminate a variable. (2 + 3)x + (1 - 1)y = 25 + 20 Simplifying, we get: 5x = 45 [URL='https://www.mathcelebrity.com/1unk.php?num=5x%3D45&pl=Solve']Typing this equation into the search engine[/URL], we get: [B]x = 9[/B] To find y, we plug in x = 9 into equation (1) or (2). Let's choose equation (1): 2(9) + y = 25 y + 18 = 25 [URL='https://www.mathcelebrity.com/1unk.php?num=y%2B18%3D25&pl=Solve']Typing this equation into our search engine[/URL], we get: [B]y = 7[/B] So we have (x, y) = (9, 7) Let's check our work for equation (2) to make sure this system works: 3(9) - 7 ? 20 27 - 7 ? 20 20 = 20 <-- Good, we match!

2 times a number equals that number plus 5
2 times a number equals that number plus 5 The phrase [I]a number[/I] means an arbitrary variable, let's call it x. 2 times a number means we multiply 2 by x: 2x That number plus 5 means we add 5 to the number x x + 5 The phrase [I]equals[/I] means we set both expressions equal to each other [B]2x = x + 5[/B] <-- This is our algebraic expression If you want to take this further and solve this equation for x, [URL='https://www.mathcelebrity.com/1unk.php?num=2x%3Dx%2B5&pl=Solve']type this expression in the search engine[/URL] and we get: [B]x = 5[/B]

2 times a number minus 4 times another number is 6. The sum of 2 numbers is 8. Find the 2 numbers
2 times a number minus 4 times another number is 6. The sum of 2 numbers is 8. Find the 2 numbers. Let the first number be x, and the second number be y. We're given two equations: [LIST=1] [*]2x - 4y = 6 [*]x + y = 8 [/LIST] Using our simultaneous equation calculator, there are 3 ways to solve this: [LIST] [*][URL='https://www.mathcelebrity.com/simultaneous-equations.php?term1=2x+-+4y+%3D+6&term2=x+%2B+y+%3D+8&pl=Substitution']Substitution[/URL] [*][URL='https://www.mathcelebrity.com/simultaneous-equations.php?term1=2x+-+4y+%3D+6&term2=x+%2B+y+%3D+8&pl=Elimination']Elimination[/URL] [*][URL='https://www.mathcelebrity.com/simultaneous-equations.php?term1=2x+-+4y+%3D+6&term2=x+%2B+y+%3D+8&pl=Cramers+Method']Cramer's Rule[/URL] [/LIST] They all give the same answers: (x, y) = [B](6.3333333, 1.6666667)[/B]

2 times a number subtracted by x
2 times a number subtracted by x The phrase [I]a number[/I] means an arbitrary variable, let's call it n. n 2 times a number means we multiply n by 2: 2n The phrase [I]subtracted by[/I] means we subtract 2n from x: [B]x - 2n[/B]

2 times as many dimes as quarters and they have a combined value of 180 cents, how many of each coin
2 times as many dimes as quarters and they have a combined value of 180 cents, how many of each coin does he have? Let d be the number of dimes. Let q be the number of quarters. We're given two equations: [LIST=1] [*]d = 2q [*]0.1d + 0.25q = 180 [/LIST] Substitute (1) into (2): 0.1(2q) + 0.25q = 180 0.2q + 0.25q = 180 [URL='https://www.mathcelebrity.com/1unk.php?num=0.2q%2B0.25q%3D180&pl=Solve']Typing this equation into the search engine[/URL], we get: [B]q = 400[/B] Now substitute q = 400 into equation 1: d = 2(400) [B]d = 800[/B]

2 times b squared minus 6
2 times b squared minus 6 b squared means we raise b to the 2nd power: b^2 2 times b squared 2b^2 Minus 6: [B]2b^2 - 6[/B]

2 times half of a number
A number means an arbitrary variable, let's call it x. Half of x means we divide x by 2, or multiply by 0.5 x/2 2 times half x is written: [B]2(x/2)[/B] If we simplify by cancelling the 2's, we just get x.

2 times the quantity x minus 1 is 12
2 times the quantity x minus 1 is 12 The quantity x minus 1 is written as: x - 1 2 times this quantity: 2(x - 1) The word [I]is[/I] means an equation, so we set 2(x - 1) equal to 12: [B]2(x - 1) = 12[/B]

2 times the sum of 1 and some number is 30. What is the number?
2 times the sum of 1 and some number is 30. What is the number? We let the phrase "some number" equal the variable x. The sum of 1 and some number is: x + 1 2 times the sum: 2(x + 1) The word "is" means equal to, so we set [B]2(x + 1) = 30[/B]

2 times the sum of 3 and 5 divided by 10
2 times the sum of 3 and 5 divided by 10 The sum of 3 and 5 is written as: 3 + 5 2 times this sum: 2(3 + 5) Then, we divide this by 10: [B]2(3 + 5)/10[/B] [B][/B] If the problem asks you to simplify this, you [URL='https://www.mathcelebrity.com/order-of-operations-calculator.php?num=2%283%2B5%29%2F10&pl=Perform+Order+of+Operations']type this expression into our search engine[/URL] and get: [B]1.6[/B]

2 times the sum of 3x and 5
2 times the sum of 3x and 5 the sum of 3x and 5 3x + 5 2 times the sum: [B]2(3x + 5)[/B]

2 times the sum of 7 times a number and 4
2 times the sum of 7 times a number and 4 This is an algebraic expression. Let's take it in 4 parts: [LIST=1] [*]The phrase [I]a number[/I] means an arbitrary variable, let's call it x. [*]7 times a number means we multiply x by 7: 7x [*]The sum of 7 times a number and 4 means we add 4 to 7x: 7x + 4 [*]Finally, we multiply the sum in #3 by 2 [/LIST] Build our final algebraic expression: [B]2(7x + 4)[/B]

2 times the sum of a number and 3 is equal to 3x plus 4
2 times the sum of a number and 3 is equal to 3x plus 4 The phrase [I]a number[/I] means an arbitrary variable, let's call it x. x The sum of a number and 3 means we add 3 to x: x + 3 2 times this sum means we multiply the quantity x + 3 by 2 2(x + 3) 3x plus 4 means 3x + 4 since the word plus means we use a (+) sign 3x + 4 The phrase [I]is equal to[/I] means an equation, where we set 2(x + 3) equal to 3x + 4 [B]2(x + 3) = 3x + 4[/B]

2 times the sum of a number x and 5
2 times the sum of a number x and 5 The sum of a number x and 5 means we add 5 to x: x + 5 2 times the sum: [B]2(x + 5)[/B]

2 times the sum of x and 7 plus 10
2 times the sum of x and 7 plus 10 The sum of x and 7 means we add 7 to x x + 7 2 times the sum means we multiply the quantity x + 7 by 2 2(x + 7) Plus 10 means we add 10 to the 2(x + 7): [B]2(x + 7) + 10[/B]

2 tons of dirt cost $280.00. What is the price per pound?
2 tons of dirt cost $280.00. What is the price per pound? We know that 1 ton = 2000 pounds. So 2 tons = 2*2000 = 4,000 pounds We rewrite this as 4,000 pounds of dirt cost $280.00. We set up a proportion where p is the price per one pound: 4000/280 = 1/p [URL='https://www.mathcelebrity.com/prop.php?num1=4000&num2=1&den1=280&den2=p&propsign=%3D&pl=Calculate+missing+proportion+value']Plugging this in our search engine[/URL], we get: p = [B]0.07 or 7 cents per pound.[/B]

2 traffic lights are turned on at the same time. 1 blinks every 4 seconds and. the other blinks ever
2 traffic lights are turned on at the same time. 1 blinks every 4 seconds and. the other blinks every 6 seconds. In 60 seconds how many times will they blink at the same time? We want the [URL='https://www.mathcelebrity.com/gcflcm.php?num1=4&num2=6&num3=&pl=LCM']least common multiple of 4 and 6[/URL] which is 12. So ever 12 seconds, both lights blink together: [LIST=1] [*]12 [*]24 [*]36 [*]48 [*]60 [/LIST] So our answer is [B]5 times[/B]

2, 4, 6, 8....1000. What term is the number 1000?
2, 4, 6, 8....1000. What term is the number 1000? Formula for nth term is 2n If 2n = 1000, then dividing each side by 2, we see that: 2n/2 = 1000/2 n = [B]500[/B]

2-thirds of the sum of 5 and a plus the product of 3 and z
2-thirds of the sum of 5 and a plus the product of 3 and z The sum of 5 and a 5 + a 2-thirds of this sum: 2(5 + a)/3 The product of 3 and z: 3z The word [I]plus[/I] means we add the two terms together: [B]2(5 + a)/3 + 3z[/B]

2/3 of a number 17 is at least 29
2/3 of a number 17 is at least 29 The phrase [I]a number[/I] means an arbitrary variable, let's call it x: x 2/3 of a number means we multiply x by 2/3: 2x/3 The phrase [I]is at least[/I] also means greater than or equal to, so we set up the inequality: [B]2x/3 >= 29[/B]

2/5 the cube of a number
2/5 the cube of a number The phrase [I]a number[/I] means an arbitrary variable, let's call it x. The cube of a number means we raise x to the power of 3: x^3 2/5 of the cube means we multiply x^3 by 2/5: [B](2x^3)/5[/B]

20 percent of my class is boys. There are 30 boys in class. How many girls in my class
20 percent of my class is boys. There are 30 boys in class. How many girls in my class? Let c be the number of people in class. Since 20% = 0.2, We're given: 0.2c = 30 [URL='https://www.mathcelebrity.com/1unk.php?num=0.2c%3D30&pl=Solve']Type this equation into our search engine[/URL], we get: c = 150 Since the class is made up of boys and girls, we find the number of girls in the class by this equation: Girls = 150 - 30 Girls = [B]120[/B]

20 teachers made a bulk purchase of some textbooks. The teachers received a 24% discount for the bul
20 teachers made a bulk purchase of some textbooks. The teachers received a 24% discount for the bulk purchase, which originally cost $5230. Assuming the cost was divided equally among the teachers, how much did each teacher pay? [U]Calculate Discount Percent:[/U] If the teachers got a 24% discount, that means they paid: 100% - 24% = 76% [URL='https://www.mathcelebrity.com/perc.php?num=+5&den=+8&num1=+16&pct1=+80&pct2=+90&den1=+80&pct=76&pcheck=4&decimal=+65.236&astart=+12&aend=+20&wp1=20&wp2=30&pl=Calculate']76% as a decimal[/URL] = 0.76 (Discount Percent) [U]Calculate discount price:[/U] Discount Price = Full Price * (Discount Percent) Discount Price = 5230 * 0.76 Discount Price = 3974.80 Price per teacher = Discount Price / Number of Teachers Price per teacher = 3974.80 / 20 Price per teacher = [B]$198.74[/B]

20% of a number is x. What is 100% of the number? Assume x>0.
20% of a number is x. What is 100% of the number? Assume x>0. Let the number be n. We're given: 0.2n = x <-- Since 20% = 0.2 To find n, we multiply each side of the equation by 5: 5(0.2)n = 5x n = [B]5x[/B]

20% of Kay’s pencils in her pencil case are broken. What is the chance of taking a pencil that is no
20% of Kay’s pencils in her pencil case are broken. What is the chance of taking a pencil that is not broken from the case if she picks one at random? This means 100% - 20% = 80% of pencils are not broken So the probability of drawing a pencil which is [I]not broken[/I] is 80%

200 apples at $69.99 how much is each apple
$69.99 per apple / 200 apples We want the price per apple. Divide top and bottom by 200 $0.35 per apple.

200 feet shorter than the height of a light house
200 feet shorter than the height of a light house Let the height of a light house be h: h 200 fee shorter mean we subtract 200 from h: [B]h - 200[/B]

2000 people attended a baseball game 1300 of the people attending supported the home team while 700
2000 people attended a baseball game 1300 of the people attending supported the home team while 700 supported the visiting team what percentage of people attending supported the home team We want the percentage of 1300 out of 2000. [URL='https://www.mathcelebrity.com/perc.php?num=1300&den=2000&pcheck=1&num1=+16&pct1=+80&pct2=+35&den1=+90&pct=+82&decimal=+65.236&astart=+12&aend=+20&wp1=20&wp2=30&pl=Calculate']We go to our search engine and type 1300 out of 2300 as a percent[/URL] and we get: [B]65%[/B]

21 apples and 49 pears. What is the largest number of baskets to make and still use up all the fruit
21 apples and 49 pears. What is the largest number of baskets to make and still use up all the fruit We use our [URL='https://www.mathcelebrity.com/gcflcm.php?num1=21&num2=49&num3=&pl=GCF+and+LCM']greatest common factor calculator for GCF(21, 49)[/URL] to get: GCF(21, 49) = 7 This means with [B]7 baskets[/B]: [LIST] [*]We divide 21 apples by 7 to get 3 apples per basket [*]We divide 49 pears by 7 to get 7 pears per basket [/LIST]

21 the total of 21 and
21 the total of 21 and? Let the number we want be n. We have: 21 = 21 + n n must be [B]0[/B], since 21 = 21

217 times u, reduced by 180 is the same as q
217 times u, reduced by 180 is the same as q. Take this algebraic expression pieces: Step 1: 217 times u We multiply the variable u by 217 217u Step 2: reduced by 180 Subtract 180 from 217u 217u - 180 The phrase [I]is the same as[/I] means an equation, so we set 217u - 180 equal to q [B]217u - 180 = q[/B]

2200 dollars is placed in an account with an annual interest rate of 7.25%. How much will be in the
2200 dollars is placed in an account with an annual interest rate of 7.25%. How much will be in the account after 29 years [URL='https://www.mathcelebrity.com/compoundint.php?bal=2200&nval=29&int=7.25&pl=Annually']Using our compound interest calculator[/URL], with an initial balance of 2,200, 29 years for time, and 7.25% annual interest rate, we get: [B]16,747.28[/B]

223 subtracted from the quantity 350 times a is equal to b
223 subtracted from the quantity 350 times a is equal to b Take this algebraic expression in parts: [LIST] [*]the quantity 350 times a: 350a [*]223 subtracted from the quantity: 350a - 223 [*]The phrase [I]is equal to[/I] means an equation, so we set 350a - 223 equal to b [/LIST] [B]350a - 223 = b[/B]

225 lines per second how many per minute
225 lines per second how many per minute There are 60 seconds in 1 minute, so we have: 225 lines 60 seconds ---------- * -------------- 1 second 1 minute Cancel the second from top and bottom, and we have: [B]13,500 lines --------------- 1 minute[/B]

231 is 248 subtracted from the quantity h times 128
231 is 248 subtracted from the quantity h times 128 Let's take this algebraic expression in parts: [LIST=1] [*]h times 128: 128h [*]24 subtracted from this: 128h - 248 [*]The word [I]is[/I] means an equation, so we set 128h - 248 equal to 231 [/LIST] [B]128h - 248 = 231[/B] <-- This is our algebraic expression If the problem asks you to solve for h, then you [URL='https://www.mathcelebrity.com/1unk.php?num=128h-248%3D231&pl=Solve']type in this equation into our search engine[/URL] and get: h = [B]3.742[/B]

237 what is the place value of 3
237 what is the place value of 3 Place value for integers with no decimals from right to left is: 7 is the ones digit 3 is the [B]tens digit[/B]

24 coloring books, 60 crayons, and 84 markers can be packaged into at most how many identical packag
24 coloring books, 60 crayons, and 84 markers can be packaged into at most how many identical packages? How many of each would each package contain? First, determine the greatest common factor (GCF) of 24, 60, and 84 using our [URL='http://www.mathcelebrity.com/gcflcm.php?num1=24&num2=60&num3=84&pl=GCF']GCF calculator[/URL]. GCF(24, 60, 84) = 12 So we have 12 identical packages. Now, figure out how many coloring books, crayons, and markers for each package [LIST] [*]24/12 = 2 coloring books [*]60/12 = 5 crayons [*]84/12 = 7 markers [/LIST] [B]So we have 12 identical packages, each containing 2 coloring books, 5 crayons, and 7 markers[/B]

24 students in a class took an algebra test and 19 of them earned a B or better. What percent of stu
24 students in a class took an algebra test and 19 of them earned a B or better. What percent of students earned a B or better? Using our [URL='http://www.mathcelebrity.com/perc.php?num=19&den=24&pcheck=1&num1=16&pct1=80&pct2=70&den1=80&idpct1=10&hltype=1&idpct2=90&pct=82&decimal=+65.236&astart=12&aend=20&wp1=20&wp2=30&pl=Calculate']percentage calculator[/URL], we have: 19/24 = [B]79.1667%[/B]

249 equals 191 times c, decreased by 199
249 equals 191 times c, decreased by 199 [U]Take this in pieces:[/U] 191 times c: 191c The phrase [I]decreased by[/I] means we subtract 199 from 191c: 191c - 199 We set this expression equal to 249: [B]191c - 199 = 249[/B] <-- This is our algebraic expression If you want to solve for c, type this equation into the search engine and we get: [B]c = 2.346[/B]

25 boxes are loaded on a truck. If each box weighs 22 kg, what is the total weight of the load
25 boxes are loaded on a truck. If each box weighs 22 kg, what is the total weight of the load? Total Weight = Number of boxes * weight per box Total Weight = 25 * 22 kg Total Weight = [B]550 kg[/B]

250 students have iPhones. This is one third of the population. How many students are there in total
250 students have iPhones. This is one third of the population. How many students are there in total? Let the population be p. We're given: 1/3p = 250 Cross multiply: p = 250 * 3 p = [B]750[/B]

26 students 15 like vanilla 16 like chocolate. 3 do not like either flavour. How many like both vani
26 students 15 like vanilla 16 like chocolate. 3 do not like either flavour. How many like both vanilla and chocolate Define our people: [LIST=1] [*]We have Vanilla Only [*]Chocolate Only [*]Both Vanilla and Chocolate [*]Neither Vanilla Nor Chocolate [*]Add up 1-4 to get our total [/LIST] Total = Vanilla Only + Chocolate Only - Vanilla and Chocolate + Neither 26 = 15 + 16 - V&C + 3 26 = 34 - V&C Subtract 34 from each side -V&C = -8 Multiply each side by -1 [B]V&C = 8[/B]

2755 students were surveyed: 896 chose a falcon, 937 chose a ram, and 842 chose a panther. The remai
2755 students were surveyed: 896 chose a falcon, 937 chose a ram, and 842 chose a panther. The remaining did not vote. What is the probability that the students choice was a panther? P(Panther) = Panther Choices / Total Students P(Panther) = 842/2,755 P(Panther) = [B]0.3056[/B]

28 students in class and 16 are boys what is percent of girls?
28 students in class and 16 are boys what is percent of girls? Calculate the number of girls: Girls = Total Students - Boys Girls = 28 - 16 Girls = 12 The percent of girls is found by this formula: Percent of Girls = 100 * Number of Girls / Number of Students Percent of Girls = 100 * 12 / 28 Percent of Girls = 1,200 / 28 Percent of Girls = [B]42.86%[/B]

2900 dollars is placed in an account with an annual interest rate of 9%. Hoe much will be in the acc
2900 dollars is placed in an account with an annual interest rate of 9%. Hoe much will be in the account after 13 years to the nearest cent Using our [URL='https://www.mathcelebrity.com/compoundint.php?bal=2900&nval=13&int=9&pl=Annually']compound interest with balance calculator[/URL], we get: [B]8,890.83[/B]

2900 dollars is placed in an account with annual interest rate of 9%. How much will be in the accoun
2900 dollars is placed in an account with annual interest rate of 9%. How much will be in the account after 13 years, round to the nearest cent Using our [URL='https://www.mathcelebrity.com/compoundint.php?bal=2090&nval=13&int=9&pl=Annually']compound interest calculato[/URL]r, we get a balance of: [B]6,407.53[/B]

298 is the same as c and 230 more
[I]Is the same as[/I] means equal to. 230 more means we add 230. Set up this equation: c + 230 = 298 To solve for c if needed, visit our [URL='http://www.mathcelebrity.com/1unk.php?num=c%2B230%3D298&pl=Solve']calculator[/URL]. c = 68

2ade?(ae)
2ade[IMG]https://fonts.gstatic.com/s/e/notoemoji/14.0/2797/72.png[/IMG](ae) 2ade/ae the ae terms cancel, so we have: [B]2d[/B]

2consecutiveevenintegerssuchthatthesmalleraddedto5timesthelargergivesasumof70
2 consecutive even integers such that the smaller added to 5 times the larger gives a sum of 70. Let the first, smaller integer be x. And the second larger integer be y. Since they are both even, we have: [LIST=1] [*]x = y - 2 <-- Since they're consecutive even integers [*]x + 5y = 70 <-- Smaller added to 5 times the larger gives a sum of 70 [/LIST] Substitute (1) into (2): (y - 2) + 5y = 70 Group like terms: (1 + 5)y - 2 = 70 6y - 2 = 70 [URL='https://www.mathcelebrity.com/1unk.php?num=6y-2%3D70&pl=Solve']Typing 6y - 2 = 70 into our search engine[/URL], we get: [B]y = 12 <-- Larger integer[/B] Plugging this into Equation (1) we get: x = 12 - 2 [B]x = 10 <-- Smaller Integer[/B] So (x, y) = (10, 12)

2d = (a - b)/(b - c) for d
2d = (a - b)/(b - c) for d Divide each side by 2 to isolate d: 2d/2 = (a - b)/2(b - c) Cancel the 2's on the left side, we get: d = [B](a - b)/2(b - c)[/B]

2m - n/3 = 5m for n
2m - n/3 = 5m for n Subtract 2m from each side of the equation: 2m-n/3 - 2m = 5m - 2m -n/3 = 3m Multiply each side of the equation by -3 to isolate n: -3 * -n/3 = -3 * 3m n = [B]-9m[/B]

2n + 1 = n + 10
2n + 1 = n + 10 Solve for [I]n[/I] in the equation 2n + 1 = n + 10 [SIZE=5][B]Step 1: Group variables:[/B][/SIZE] We need to group our variables 2n and n. To do that, we subtract n from both sides 2n + 1 - n = n + 10 - n [SIZE=5][B]Step 2: Cancel n on the right side:[/B][/SIZE] n + 1 = 10 [SIZE=5][B]Step 3: Group constants:[/B][/SIZE] We need to group our constants 1 and 10. To do that, we subtract 1 from both sides n + 1 - 1 = 10 - 1 [SIZE=5][B]Step 4: Cancel 1 on the left side:[/B][/SIZE] n = [B]9[/B]

2n + 10 = 3n + 5
2n + 10 = 3n + 5 Solve for [I]n[/I] in the equation 2n + 10 = 3n + 5 [SIZE=5][B]Step 1: Group variables:[/B][/SIZE] We need to group our variables 2n and 3n. To do that, we subtract 3n from both sides 2n + 10 - 3n = 3n + 5 - 3n [SIZE=5][B]Step 2: Cancel 3n on the right side:[/B][/SIZE] -n + 10 = 5 [SIZE=5][B]Step 3: Group constants:[/B][/SIZE] We need to group our constants 10 and 5. To do that, we subtract 10 from both sides -n + 10 - 10 = 5 - 10 [SIZE=5][B]Step 4: Cancel 10 on the left side:[/B][/SIZE] -n = -5 [SIZE=5][B]Step 5: Divide each side of the equation by -1[/B][/SIZE] -1n/-1 = -5/-1 n = [B]5[/B]

2n + 8 - n = 20
2n + 8 - n = 20 Solve for [I]n[/I] in the equation 2n + 8 - n = 20 [SIZE=5][B]Step 1: Group the n terms on the left hand side:[/B][/SIZE] (2 - 1)n = n [SIZE=5][B]Step 2: Form modified equation[/B][/SIZE] n + 8 = + 20 [SIZE=5][B]Step 3: Group constants:[/B][/SIZE] We need to group our constants 8 and 20. To do that, we subtract 8 from both sides n + 8 - 8 = 20 - 8 [SIZE=5][B]Step 4: Cancel 8 on the left side:[/B][/SIZE] n = [B]12[/B]

2n + 8 = 24
2n + 8 = 24 Solve for [I]n[/I] in the equation 2n + 8 = 24 [SIZE=5][B]Step 1: Group constants:[/B][/SIZE] We need to group our constants 8 and 24. To do that, we subtract 8 from both sides 2n + 8 - 8 = 24 - 8 [SIZE=5][B]Step 2: Cancel 8 on the left side:[/B][/SIZE] 2n = 16 [SIZE=5][B]Step 3: Divide each side of the equation by 2[/B][/SIZE] 2n/2 = 16/2 n = [B]8[/B]

2n - 1&1/2n = 59
2n - 1&1/2n = 59 1&1/2n = 3/2n or 1.5n So we have: 2n - 1.5n = 59 Solve for [I]n[/I] in the equation 2n - 1.5n = 59 [SIZE=5][B]Step 1: Group the n terms on the left hand side:[/B][/SIZE] (2 - 1.5)n = 0.5n [SIZE=5][B]Step 2: Form modified equation[/B][/SIZE] 0.5n = + 59 [SIZE=5][B]Step 3: Divide each side of the equation by 0.5[/B][/SIZE] 0.5n/0.5 = 59/0.5 n = [B]118[/B]

2n - 7 = 0
2n - 7 = 0 Solve for [I]n[/I] in the equation 2n - 7 = 0 [SIZE=5][B]Step 1: Group constants:[/B][/SIZE] We need to group our constants -7 and 0. To do that, we add 7 to both sides 2n - 7 + 7 = 0 + 7 [SIZE=5][B]Step 2: Cancel 7 on the left side:[/B][/SIZE] 2n = 7 [SIZE=5][B]Step 3: Divide each side of the equation by 2[/B][/SIZE] 2n/2 = 7/2 n = [B]3.5[/B]

2n = 4n
2n = 4n Solve for [I]n[/I] in the equation 2n = 4n [SIZE=5][B]Step 1: Group variables:[/B][/SIZE] We need to group our variables 2n and 4n. To do that, we subtract 4n from both sides 2n - 4n = 4n - 4n [SIZE=5][B]Step 2: Cancel 4n on the right side:[/B][/SIZE] -2n = 0 [SIZE=5][B]Step 3: Divide each side of the equation by -2[/B][/SIZE] -2n/-2 = 0/-2 n = [B]0[/B]

2x - b/y = 4c for y
2x - b/y = 4c for y Subtract 2x from each side: 2x - 2x - b/y = 4c - 2x Cancel the 2x's on the left side and we get: -b/y = 4c - 2x Cross multiply: -b = y(4c - 2x) Divide each side by (4c - 2x): -b/(4c - 2x) = y(4c - 2x)/(4c - 2x) Cancel the (4c - 2x) on the right side and we get: [B]y = -b/(4c - 2x) [/B]

2x decreased by 15 is equal to -27
2x decreased by 15 is equal to -27 The phrase [I]decreased by[/I] 15 means we subtract 15 from 2x: 2x - 15 The phrase [I]is equal to[/I] means an equation, so we set 2x - 15 equal to -27 [B]2x - 15 = -27 [/B] <-- This is our algebraic expression To solve this, [URL='https://www.mathcelebrity.com/1unk.php?num=2x-15%3D-27&pl=Solve']type 2x - 15 = -27 into the search engine[/URL].

2x increased by 3 times a number
2x increased by 3 times a number The phrase [I]a number[/I] means an arbitary variable, let's call it x. 3 times a number means we multiply x by 3: 3x The phrase [I]increased by[/I] means we add 3x to 2x: 2x + 3x Simplifying, we get: [B]5x[/B]

2x plus 4 increased by 15 is 57
2x plus 4 increased by 15 is 57 Take this algebraic expression in parts: [LIST] [*]2x plus 4: 2x + 4 [*][I]Increased by[/I] means we add 15 to 2x + 4: 2x + 4 + 15 = 2x + 19 [*]The word [I]is[/I] means an equation, so we set 2x + 19 equal to 57: [/LIST] Our final algebraic expression is: [B]2x + 19 = 57 [/B] To solve for x, we [URL='https://www.mathcelebrity.com/1unk.php?num=2x%2B19%3D57&pl=Solve']type this equation into our search engine [/URL]and we get x = [B]19[/B]

2x plus 8, quantity squared
2x plus 8, quantity squared 2x plus 8 means we add 8 to 2x: 2x + 8 Squaring the quantity means we raise it to the power of 2: [B](2x + 8)^2[/B]

2x/5 - 9y = 6 for x
2x/5 - 9y = 6 for x Add 9y to each side to isolate the x term: 2x/5 - 9y + 9y = 9y + 6 Cancel the 9y's on the left side: 2x/5 = 9y + 6 Multiply each side by 5: 2x * 5/5 = 5(9y + 6) Cancel the 5's on the left side and we get: 2x = 5(9y + 6) Divide each side by 2 to isolate x: 2x/2 = 5/2(9y + 6) Cancel the 2's on the left side and we get our final literal equation of: x = [B]5/2(9y + 6)[/B]

2x/5 - 9y = 6 for x
2x/5 - 9y = 6 for x Add 9y to each side of the equation: 2x/5 - 9y + 9y = 6 + 9y Cancel the 9y's on the left side to get: 2x/5 = 6 + 9y Multiply each side of the equation by 5: 5(2x/5) = 5(6 + 9y) Cancel the 5's on the left side to get 2x = 5(6 + 9y) Divide each side of the equation by 2: 2x/2 = 5/2(6 + 9y) Cancel the 2's on the left side to get: x = [B]5/2(6 + 9y)[/B]

2x^2+4x < 3x+6
2x^2+4x < 3x+6 Subtract 3x from both sides: 2x^2 + x < 6 Subtract 6 from both sides 2x^2 + x - 6 < 0 Using our [URL='http://www.mathcelebrity.com/quadratic.php?num=2x%5E2%2Bx-6&pl=Solve+Quadratic+Equation&hintnum=+0']quadratic calculator[/URL], we get: x < 1.5 and x < -2 When we take the intersection of these, it's [B]x < 1.5[/B]

2y divided by the sum of 3x and 5
2y divided by the sum of 3x and 5 The sum of 3x and 5 means we add 5 to 3x: 3x + 5 2y divided by the sum of 3x and 5: [B]2y/(3x + 5)[/B]

3 adults and 4 children must pay $136. 2 adults and 3 children must pay $97.
3a + 4c = 136 2a + 3c = 97 [URL='http://www.mathcelebrity.com/simultaneous-equations.php?term1=3a+%2B+4c+%3D+136&term2=2a+%2B+3c+%3D+97&pl=Cramers+Method']Using any of the 3 methods here[/URL]: [B]a = 20 c = 19[/B]

3 boys share 100 in the ratio 1:2:2. how much each boy will get?
3 boys share 100 in the ratio 1:2:2. how much each boy will get? Given the ratio 1 : 2 : 2, calculate the expected number of items from a population of 100 A ratio of 1 : 2 : 2 means that for every of item A, we can expect 2 of item B and 2 of item c Therefore, our total group is 1 + 2 + 2 = 5 [SIZE=5][B]Calculate Expected Number of Item A:[/B][/SIZE] Expected Number of Item A = 1 x 100/5 Expected Number of Item A = 100/5 Using our [URL='http://mathcelebrity.com/gcflcm.php?num1=100&num2=5&pl=GCF']GCF Calculator[/URL], we see this fraction can be reduced by 5 Expected Number of Item A = 20/1 Expected Number of Item A = [B]20[/B] [SIZE=5][B]Calculate Expected Number of Item B:[/B][/SIZE] Expected Number of Item B = 2 x 100/5 Expected Number of Item B = 200/5 Using our [URL='http://mathcelebrity.com/gcflcm.php?num1=200&num2=5&pl=GCF']GCF Calculator[/URL], we see this fraction can be reduced by 5 Expected Number of Item B = 40/1 Expected Number of Item B = [B]40[/B] [SIZE=5][B]Calculate Expected Number of Item C:[/B][/SIZE] Expected Number of Item C = 2 x 100/5 Expected Number of Item C = 200/5 Using our [URL='http://mathcelebrity.com/gcflcm.php?num1=200&num2=5&pl=GCF']GCF Calculator[/URL], we see this fraction can be reduced by 5 Expected Number of Item C = 40/1 Expected Number of Item C = [B]40[/B] [B]Final Answer:[/B] (A, B, C) =[B] (20, 40, 40)[/B] for 1:2:2 on 100 people

3 cartons of eggs for $5 what if the cost of 8 cartons
3 cartons of eggs for $5 what if the cost of 8 cartons Set up a proportion of cartons of eggs to price where p is the price of 8 cartons: 3/5 = 8/p [URL='https://www.mathcelebrity.com/prop.php?num1=3&num2=8&den1=5&den2=p&propsign=%3D&pl=Calculate+missing+proportion+value']Type this proportion into our search engine[/URL] and we get: p = [B]13.33[/B]

3 cases of fresh apples that cost $21.95 per case with 20% off and a 7.5% sales tax
3 cases of fresh apples that cost $21.95 per case with 20% off and a 7.5% sales tax Figure out the total cost before the discount: Total Cost before discount = Cases * Price per case Total Cost before discount = 3 cases * $21.95 per case Total Cost before discount = $65.85 Now, find the discounted value of the apples: Discounted Apple Price = Total Cost before discount * (1 - discount percent) Discounted ApplesPrice = $65.85 * (1 - 0.2) <-- 20% is the same as 0.2 Discounted ApplesPrice = $65.85 * 0.8 Discounted ApplesPrice = $52.68 Now, apply the sales tax to this discounted value to get the total bill: Total Bill = Discounted Apple Price * (1 + tax rate) Total Bill = $52.68 * (1 + .075) <-- 7.5% = 0.075 Total Bill = $52.68 * 1.075 Total Bill = [B]$56.63[/B]

3 children painted a local park track of 5000m, Alex painted 70m, Dell painted 15m, and Tony painted
3 children painted a local park track of 5000m, Alex painted 70m, Dell painted 15m, and Tony painted 35m. This pattern continues to the end of the track. What percentage of the park did each child paint? 70 + 15 + 35 = 120 When we take[URL='https://www.mathcelebrity.com/modulus.php?num=5000mod120&pl=Calculate+Modulus'] 5000 divided by 120[/URL], we get: 41 remainder 80 So we have: [LIST] [*]Alex: 70 * 41 = 2870 [*]Dell: 15 * 41 = 615 [*]Tony: 35 * 41 = 1435 [/LIST] Now Alex goes next, and paints the full 70. So he has: 2870 + 70 = 2940 Dell goes next, and paints the last 10 615 + 10 = 625 Now for percentages: [LIST] [*]Alex: 2940/5000 = [B]58.8%[/B] [*]Dell: 625/5000 = [B]12.5%[/B] [*]Tony: 1435/5000 = [B]28.7%[/B] [/LIST]

3 consecutive odd integers such that thrice the middle is 15 more than the sum of the other 2
3 consecutive odd integers such that thrice the middle is 15 more than the sum of the other 2. [LIST] [*]Let the first integer be n [*]The next odd one (middle) is n + 2. [*]The next odd one is n + 4 [/LIST] We are given 3(n + 2) = n + n + 4 + 15. Simplifying, we get: 3n + 6 = 2n + 19 [URL='http://www.mathcelebrity.com/1unk.php?num=3n%2B6%3D2n%2B19&pl=Solve']Plugging that problem[/URL] into our search engine, we get n = 13. So the next odd integer is 13 + 2 = 15 The next odd integer is 15 + 2 = 17

3 friends went to dinner and their total bill was $19.50. They each brought $8. If a tip is typicall
3 friends went to dinner and their total bill was $19.50. They each brought $8. If a tip is typically between 15% and 20% of the bill do they have enough for dinner and tip? 15% tip: 19.50 * 1.15 = 22.43 20% tip: 19.50 * 1.2 = 23.4 3 friends each brought $8, so they have: 3 * 8 = $24 So the $24 [B]will cover both a 15% tip and a 20% tip[/B]

3 is subtracted from square of a number
3 is subtracted from square of a number The phrase [I]a number[/I] means an arbitrary variable, let's call it x: x Square of a number means we raise x to the 2nd power: x^2 3 is subtracted from square of a number [B]x^2 - 3[/B]

3 is subtracted from the square of x
3 is subtracted from the square of x Let's take this algebraic expression in two parts: Part 1: The square of x means we raise x to the power of 2: x^2 Part 2: 3 is subtracted means we subtract 3 from x^2 [B]x^2 - 3[/B]

3 less than a number times itself
3 less than a number times itself The phrase [I]a number[/I] means an arbitrary variable, let's call it x: x Itself means the same variable as above. So we have: x * x x^2 3 less than this means we subtract 3 from x^2: [B]x^2 - 3[/B]

3 more than the product of 7 and a number x is less than 26
The product of 7 and a number x is written as 7x. 3 more than that product is written as 7x + 3. Finally, that entire expression is less than 26, so we have: 7x + 3 < 26 as our algebraic expression.

3 pack of bouncy balls for $0.96 or 9 pack of bouncy balls for $3.15
3 pack of bouncy balls for $0.96 or 9 pack of bouncy balls for $3.15 Item 1, the 3 pack is the better buy shown on our [URL='http://www.mathcelebrity.com/betterbuy.php?p1=0.96&p2=3.15&q1=3&q2=9&pl=Better+Buy']better buy calculator[/URL].

3 people can build a shed in 8 hours, how long would it take 5 people
3 people can build a shed in 8 hours, how long would it take 5 people We set up a proportion of people to hours where h is the number of hours for 5 people: 3/8 = 5/h [URL='https://www.mathcelebrity.com/prop.php?num1=3&num2=5&den1=8&den2=h&propsign=%3D&pl=Calculate+missing+proportion+value']Using our proportion calculator[/URL], we get: 13.3333 hours But what if the problem asks for minutes? Then we say 8 hours = 60 * 8 = 480 minutes. We set up the proportion where m is the number of minutes: 3/480 = 5/m In this case, [URL='https://www.mathcelebrity.com/prop.php?num1=3&num2=5&den1=480&den2=m&propsign=%3D&pl=Calculate+missing+proportion+value']we use our search engine again[/URL] and get: m = 800

3 per ride r plus $10 to get into the park
3 per ride r plus $10 to get into the park Cost function C(r) where r is rides: C(r) = Rate per ride * number of rides + admission cost [B]C(r) = 3r + 10[/B]

3 pound bag for $11.25
We need to find the unit cost, which is the cost of one unit of measurement. The measurement unit is in pounds. If it costs $11.25 for a 3 pound bag, how much is it per pound? $11.25/3 = $3.75 per pound.

3 salads, 4 main dishes, and 2 desserts
3 salads, 4 main dishes, and 2 desserts Total meal combinations are found by multiplying each salad, main dish, and dessert using the fundamental rule of counting. The fundamental rule of counting states, if there are a ways of doing one thing, b ways of doing another thing, and c ways of doing another thing, than the total combinations of all the ways are found by a * b * c. With 3 salads, 4 main dishes, and 2 desserts, our total meal combinations are: 3 * 4 * 2 = [B]24 different meal combinations.[/B]

3 subtracted from the cost of the book
3 subtracted from the cost of the book Let the cost of the book be b. We have: [B]b - 3[/B]

3 times a number increased by 1 is between -8 and 13
3 times a number increased by 1 is between -8 and 13. Let's take this algebraic expression in [U]4 parts[/U]: Part 1 - The phrase [I]a number[/I] means an arbitrary variable, let's call it x: x Part 2 - 3 times this number means we multiply x by 3: 3x Part 3 - Increased by 1 means we add 1 to 3x: 3x + 1 The phrase [I]between[/I] means we have an inequality: [B]-8 <= 3x + 1 <=13[/B]

3 times a number is 3 more a number
3 times a number is 3 more a number The phrase [I]a number[/I] means an arbitrary variable, let's call it x. 3 times a number: 3x 3 more than a number means we add 3 to x: x + 3 The word [I]is[/I] means and equation, so we set 3x equal to x + 3 [B]3x = x + 3[/B]

3 times larger than the sum of 4 and 9
The sum of 4 and 9: 4 + 9 3 times larger than this sum [B]3(4 + 9) <-- This is our algebraic expression [/B] Evaluating this amount: 3(13) [B]39[/B]

3 times the difference between t and y
3 times the difference between t and y Difference between t and y t - y 3 times this difference: [B]3(t - y)[/B]

3 times the difference of a and b is equal to 4 times c
3 times the difference of a and b is equal to 4 times c [U]The difference of a and b:[/U] a - b [U]3 times the difference of a and b:[/U] 3(a - b) [U]4 times c:[/U] 4c The phrase [I]is equal to[/I] means an equation. So we set 3(a - b) equal to 4c: [B]3(a - b) = 4c[/B]

3 times the difference of x and 5 is 15
The difference of x and 5 means we subtract: x - 5 3 times the difference means we multiply (x - 5) by 3 3(x - 5) Is, means equal to, so we set our expression equal to 15 [B]3(x - 5) = 15 [/B] If you need to take it one step further to solve for x, use our [URL='http://www.mathcelebrity.com/1unk.php?num=3%28x-5%29%3D15&pl=Solve']equation calculator[/URL]

3 times the quantity 2 decreased by x is 9
3 times the quantity 2 decreased by x is 9 The quantity 2 decreased by x. The phrase [I]decreased by[/I] means we subtract: 2 - x 3 times the quantity: 3(2 - x) The word [I]is[/I] means equal to, so we set 3(2 - x) equal to 9: [B]3(2 - x) = 9 [MEDIA=youtube]Hzyt_GajZA4[/MEDIA][/B]

3 times the square of a number x minus 12
3 times the square of a number x minus 12. Build the algebraic expression piece by piece: [LIST] [*]Square of a number x: x^2 [*]3 times this: 3x^2 [*]Minus 12: [B]3x^2 - 12[/B] [/LIST]

3 times the sum of 2 decreased by x is 9
3 times the sum of 2 decreased by x is 9 2 decreased by x: 2 - x 3 times the sum means we multiply 2 - x by 3: 3(2 - x) The phrase [I]is 9[/I] means equal to, so we set 3(2 - x) equal to 9: [B]3(2 - x) = 9[/B]

3 times the sum of twice k and 8
3 times the sum of twice k and 8 Twice k means we multiply k by 2: 2k The sum of twice k and 8: 2k + 8 3 times the sum: [B]3(2k + 8)[/B]

3 times the sum of x and 9y
3 times the sum of x and 9y The sum of x and 9y means we add 9y to x: x + 9y Now we take this sum, and multiply by 3 to get our final algebraic expression: 3(x + 9y)

3 times the width plus 2 times the length
3 times the width plus 2 times the length Let w be the width Let l be the length We have an algebraic expression of: [B]3w + 2l[/B]

3 times x minus y is 5 times the sum of y and 2 times x
3 times x minus y is 5 times the sum of y and 2 times x Take this algebraic expression in pieces: 3 times x: 3x Minus y means we subtract y from 3x 3x - y The sum of y and 2 times x mean we add y to 2 times x y + 2x 5 times the sum of y and 2 times x: 5(y + 2x) The word [I]is[/I] means an equation, so we set 3x - y equal to 5(y + 2x) [B]3x - y = 5(y + 2x)[/B]

3 to the power of 2 times 3 to the power of x equals 3 to the power of 7
3 to the power of 2 times 3 to the power of x equals 3 to the power of 7. Write this out: 3^2 * 3^x = 3^7 When we multiply matching coefficients, we add exponents, so we have: 3^(2 + x) = 3^7 Therefore, 2 + x = 7. To solve this equation for x, we [URL='https://www.mathcelebrity.com/1unk.php?num=2%2Bx%3D7&pl=Solve']type it into our search engine[/URL] and we get: x = [B]5[/B]

3 tons of compost cost $3,600.00. What is the price per pound?
3 tons of compost cost $3,600.00. What is the price per pound? 1 ton = 2000 pounds, so 3 tons is: 3 * 2000 = 6000 pounds Price per pound = Cost / Total pounds Price per pound = 3600 / 6000 Price per pound = [B]$0.60 per pound[/B]

3 unknowns using Cramers Rule
Free 3 unknowns using Cramers Rule Calculator - Solves for 3 unknowns with equations in the form ax + by + cz = d using Cramers Method.

3, 6, 12, 24, 48 What is the function machine for this sequence?
3, 6, 12, 24, 48 What is the function machine for this sequence? We see the following pattern: 3 * 2^0 = 3 3 * 2^1 = 6 3 * 2^2 = 12 3 * 2^3 = 24 3 * 2^4 = 48 Our function machine for term n is: [B]f(n) = 3 * 2^(n - 1)[/B]

3, 8, 13, 18, .... , 5008 What term is the number 5008?
3, 8, 13, 18, .... , 5008 What term is the number 5008? For term n, we have a pattern: f(n) = 5(n - 1) + 3 Set this equal to 5008 5(n - 1) + 3 = 5008 Using our [URL='https://www.mathcelebrity.com/1unk.php?num=5%28n-1%29%2B3%3D5008&pl=Solve']equation solver,[/URL] we get: n = [B]1002[/B]

3-By-1 Mental Math Multiplication
987 x 9 ------ Break apart 987: 900 + 80 + 7 So we have: 900 * 9 = 8100 80 * 9 = 720 7 * 9 = 63 Add them up and we get [B]8,883[/B] [MEDIA=youtube]4PfdF3co6HI[/MEDIA]

3-dimensional points
Free 3-dimensional points Calculator - Calculates distance between two 3-dimensional points
(x1, y1, z1) and (x2, y2, z2) as well as the parametric equations and symmetric equations

3.50 per pound. you bought 18.25 worth of strawberries
3.50 per pound. you bought 18.25 worth of strawberries The question asks for unit cost. Unit Cost = Total Cost / Total Quantity Unit Cost = 18.25 / 3.50 Unit Cost = [B]5.21[/B]

3/4 a number b divided by 5
3/4 a number b divided by 5 3/4 a number b: 3b/4 Divided by 5: 3b/4/5 We multiply top and bottom by 5 to remove the double fraction: 3b*5/4 [B]15b/4[/B]

3/4 of the students went skiing.there are 24 students in the class. How’s many went?
3/4 of the students went skiing.there are 24 students in the class. How’s many went? We want 3/4 of 24. We [URL='https://www.mathcelebrity.com/fraction.php?frac1=24&frac2=3/4&pl=Multiply']type 3/4 of 24 into our search engine[/URL] and get: [B]18 students[/B]

3/5 of workers at a company have enrolled in the 403(b) program. If 24 workers have enrolled in the
3/5 of workers at a company have enrolled in the 403(b) program. If 24 workers have enrolled in the program, how many workers are employed at this company? We read this as 3/5 of the total workers employed at the company equals 24. Let w be the number of workers. We have the following equation: 3/5w = 24 Run [URL='http://www.mathcelebrity.com/1unk.php?num=3%2F5w%3D24&pl=Solve']3/5w = 24[/URL] through the search engine, we get [B]w = 40[/B].

3/8 of income is rent. 360 is rent. How much is annual income
3/8 of income is rent. 360 is rent. How much is annual income Let i equal the annual income. We're told the following: 3i/8 = 360 To solve for i, we [URL='https://www.mathcelebrity.com/prop.php?num1=3i&num2=360&den1=8&den2=1&propsign=%3D&pl=Calculate+missing+proportion+value']type this in our math engine[/URL] and we get: i = [B]960 [MEDIA=youtube]8gue05tlEGQ[/MEDIA][/B]

30 increased by 3 times the square of a number
Let "a number" equal the arbitrary variable x. The square of that is x^2. 3 times the square of that is 3x^2. Now, 30 increased by means we add 3x^2 to 30 30 + 3x^2

30 increased by 3 times the square of a number
30 increased by 3 times the square of a number The phrase [I]a number[/I] means an arbitrary variable, let's call it x x The square of a number means we raise x to the power of 2: x^2 3 times the square: 3x^2 The phrase [I]increased by[/I] means we add 3x^2 to 30: [B]30 + 3x^2[/B]

30 is equal to thrice y decreased by z
30 is equal to thrice y decreased by z Thrice y means we multiply y by 3: 3y Decreased by z means we subtract z from 3y 3y - z The phrase [I]is[/I] means an equal to, so we set up an equation where 3y - z is equal to 30 [B]3y - z = 30[/B]

30 people are selected randomly from a certain town. If their mean age is 60.5 and ? = 4.6, find a 9
30 people are selected randomly from a certain town. If their mean age is 60.5 and ? = 4.6, find a 95% confidence interval for the true mean age, ?, of everyone in the town.

30% larger then 1/3 of twice q
30% larger then 1/3 of twice q Take this algebraic expression in 3 parts: [LIST=1] [*]Twice q means multiply q by 2: 2q [*]1/3 of twice q means we multiply 2q in Step 1 by 1/3: 2q/3 [*]30% larger means we multiply 2q/3 in step 2 by 1.3, since 30% = 0.3: 1.3(2q/3) [/LIST] [B]1.3(2q/3)[/B]

300 reduced by 5 times my age is 60
300 reduced by 5 times my age is 60 Let my age be a. We have: 5 times my age = 5a 300 reduced by 5 times my age means we subtract 5a from 300: 300 - 5a The word [I]is[/I] means an equation, so we set 300 - 5a equal to 60 to get our final algebraic expression: [B]300 - 5a = 60 [/B] If you have to solve for a, you [URL='https://www.mathcelebrity.com/1unk.php?num=300-5a%3D60&pl=Solve']type this equation into our search engine[/URL] and you get: a = [B]48[/B]

309 is the same as 93 subtracted from the quantity f times 123
309 is the same as 93 subtracted from the quantity f times 123. The quantity f times 123: 123f Subtract 93: 123f - 93 The phrase [I]is the same as[/I] means an equation, so we set 123f - 93 equal to 309 [B]123f - 93 = 309[/B] <-- This is our algebraic expression If you wish to solve for f, [URL='https://www.mathcelebrity.com/1unk.php?num=123f-93%3D309&pl=Solve']type this equation into the search engine[/URL], and we get f = 3.2683.

31,29,24,22,17 what comes next
31,29,24,22,17 what comes next We see that each sequence term alternates between subtracting 2 and subtracting 5. Since the last term, 17, was found by subtracting 5, our next term is found by subtracting 2 from 17: 17 - 2 = [B]15[/B]

32 girls and 52 boys were on an overseas learning trip . They were divided into as many groups as po
32 girls and 52 boys were on an overseas learning trip . They were divided into as many groups as possible where the number of groups of girls and the number of groups of boys is the same .how many boys and how many girls were in each group We want a number such that our total members divided by this number equals our group size. We take the greatest common factor (32,52) = 4 Therefore, we have: [LIST] [*][B]32/4 = 8 girls in each group[/B] [*][B]52/4 = 13 boys in each group[/B] [/LIST]

324 times z, reduced by 12 is z
324 times z, reduced by 12 is z. Take this algebraic expression in pieces: 324 [I]times[/I] z means we multiply 324 by the variable z. 324z [I]Reduced by[/I] 12 means we subtract 12 from 324z 324z - 12 The word [I]is[/I] means we have an equation, so we set 324z - 12 equal to z [B]324z - 12 = z [/B] <-- This is our algebraic expression

331 students went on a field trip. Six buses were filled and 7 students traveled in cars. How many s
331 students went on a field trip. Six buses were filled and 7 students traveled in cars. How many students were in each bus? The number of students who went on the bus is 331 - 7 in the car = 324 324 students on the bus / 6 buses = [B]54 per bus[/B]

331 students went on a field trip. Six buses were filled and 7 students traveled in cars. How many s
331 students went on a field trip. Six buses were filled and 7 students traveled in cars. How many students were In bus? Subtract the 7 students in cars, and we're left with: 331 - 7 = 324 students in buses. 324 students / 6 buses = [B]54 students in each bus[/B].

331 students went on a field trip. Six buses were filled and 7 students traveled in cars. How many s
331 students went on a field trip. Six buses were filled and 7 students traveled in cars. How many students were in each bus? Calculate the students in buses: Students in buses = Total Students - Students in Cars Students in buses = 331 - 7 Students in buses = 324 Calculate the students in each bus Students in each bus = Students in buses / Number of Buses Students in each bus = 324 / 6 Students in each bus = [B]54[/B]

339 equals 303 times w, minus 293
339 equals 303 times w, minus 293 Take this algebraic expression in pieces: 303 times w: 303w Minus 293: 303w - 293 The phrase [I]equals[/I] means we have an equation. We set 303w - 293 = 339 [B]303w - 293 = 339[/B] <-- This is our algebraic expression To solve for w, [URL='https://www.mathcelebrity.com/1unk.php?num=303w-293%3D339&pl=Solve']we type this equation into our search engine[/URL] to get: [B]w = 2.086[/B]

35 added to n is greater than or equal to the sum of k and 21
35 added to n is greater than or equal to the sum of k and 21 Take this algebraic expression in 3 parts: [LIST=1] [*]35 added to n means we have a sum: n + 35 [*]The sum of k and 21 means we add 21 to k: k +21 [*]The phrase [I]greater than or equal to[/I] means an inequality using this sign (>=), so we write this as follows: [/LIST] [B]n + 35 >= k + 21[/B]

35 m/s for 40 s. how far does it travel?
35 m/s for 40 s. how far does it travel? This is a distance problem. The formula to relate, distance, rate, and time is: d = rt We are given r = 35 m/s and t = 40s. We want d d = 35 m/s * 40s d = [B]1,400 meters[/B]

35% of the houses are blue. Write the percent that do not live in blue houses as a decimal and a fra
35% of the houses are blue. Write the percent that do not live in blue houses as a decimal and a fraction in simplest form The percent that do not live in blue houses is found by: Not in blue = 100% - 35% Not in blue = 65% [URL='https://www.mathcelebrity.com/perc.php?num=+5&den=+8&num1=+16&pct1=+80&pct2=+90&den1=+80&pct=65&pcheck=4&decimal=+65.236&astart=+12&aend=+20&wp1=20&wp2=30&pl=Calculate']Typing 65% in our search engine[/URL], we see that the decimal and fraction is: [LIST] [*]65% as a decimal: [B]0.65[/B] [*]65% as a fraction in simplest form: [B]13/20[/B] [/LIST]

36 PAGES AND IT 3/8CM THICK, WHAT IS THE THICKNESS OF 1 PAGE?
36 PAGES AND IT 3/8CM THICK, WHAT IS THE THICKNESS OF 1 PAGE? Set up a proportion in pages to cm: 36 pages /3/8cm = 1 page/x cm Cross multiply: 36x = 3/8 Divide each side by 36 x = 3/(8 * 36) x = 1/(8*12) x = [B]1/96 cm[/B]

36% of the pupils in class 2 are boys the remaining 16 are girls how many pupils are in class 2?
36% of the pupils in class 2 are boys the remaining 16 are girls how many pupils are in class 2? This means 100% - 36% = 64% of the class are girls. And if the class size is s, then we have: 64% of s = 16 Or, written as a decimal: 0.64s = 16 To solve this equation for s, we [URL='https://www.mathcelebrity.com/1unk.php?num=0.64s%3D16&pl=Solve']type it into our search engine[/URL] and we get: s = [B]25[/B]

365 subtracted from the quantity q times 146 is the same as w
[U]q times 146:[/U] 146q [U]365 subtracted from that:[/U] 146q - 365 [U]Is the same as means equal to, so we have:[/U] [B]146q - 365 = w[/B]

38 books into 8 boxes. 6 left. How many books in each box
38 books into 8 boxes. 6 left. How many books in each box Let the number of books in each box be b. We have the following relation: 8b + 6 = 38 to solve this equation, we [URL='https://www.mathcelebrity.com/1unk.php?num=8b%2B6%3D38&pl=Solve']type it in our search engine[/URL] and we get: b = [B]4[/B]

3abc^4/12a^3(b^3c^2)^2 * 8ab^-4c/4a^2b
3abc^4/12a^3(b^3c^2)^2 * 8ab^-4c/4a^2b Expand term 1: 3abc^4/12a^3(b^3c^2)^2 3abc^4/12a^3b^6c^4 Now simplify term 1: 3/12 = 1/4 c^4 terms cancel Subtract powers from variables since the denominator powers are higher: b^(6 - 1) = b^5 a^(3 - 1) = a^2 1/4a^2b^5 Now simplify term 2: 8ab^-4c/4a^2b 8/4 = 2 2c/a^(2 - 1)b^(1 - -4) 2c/ab^5 Now multiply simplified term 1 times simplified term 2: 1/4a^2b^5 * 2c/ab^5 (1 * 2c)/(4a^2b^5 * ab^5) 2c/4a^(2 + 1)b^(5 + 5) 2c/4a^3b^10 2/4 = 1/2, so we have: [B]c/2a^3b^10[/B]

3f,subtract g from the result, then divide what you have by h
3f,subtract g from the result, then divide what you have by h Take this algebraic expression in pieces: 3f subtract g means we subtract the variable g from the expression 3f: 3f - g Divide what we have by h, means we take the result above, 3f - g, and divide it by h: [B](3f - g)/h[/B]

3k^3 = rt for t
3k^3 = rt for t This is a literal equation. Let's divide each side of the equation by r, to isolate t: 3k^3/r = rt/r Cancel the r's on the right side, and we get: t = [B]3k^3/r[/B]

3timesanumberdecreasedby3
A necklace chain costs $15. Beads cost $2.50 each. You spend a total of $30 on a necklace and beads before tax. How many beads did you buy in addition to the necklace? Let the number of beads be b. We're given the following equation: 2.5b + 15 = 30 To solve for b, we [URL='https://www.mathcelebrity.com/1unk.php?num=2.5b%2B15%3D30&pl=Solve']type this equation into our search engine[/URL] and we get: b = [B]6[/B]

3x less than 2 times the sum of 2x and 1 is equal to the sum of 2 and 5
3x less than 2 times the sum of 2x and 1 is equal to the sum of 2 and 5 This is an algebraic expression. Let's take this algebraic expression in 5 parts: [LIST=1] [*]The sum of 2x and 1 means we add 1 to 2x: 2x + 1 [*]2 times the sum of 2x and 1: 2(2x + 1) [*]3x less than the sum of 2x and 1 means we subtract 3x from 2(2x + 1): 2(2x + 1) - 3x [*]The sum of 2 and 5 means we add 5 to 2: 2 + 5 [*]Finally, the phrase [I]equal[/I] means an equation, so we set #3 equal to #4 [/LIST] Our algebraic expression is: [B]2(2x + 1) - 3x = 2 + 5[/B] Now, some problems may ask you to simplify. In this case, we multiply through and group like terms: 4x + 2 - 3x = 7 [B]x + 2 = 7 <-- This is our simplified algebraic expression [/B] Now, what if the problem asks you to solve for x, [URL='https://www.mathcelebrity.com/1unk.php?num=x%2B2%3D7&pl=Solve']you type this into our search engine[/URL] and get: x =[B] 5 [MEDIA=youtube]3hzyc2NPCGI[/MEDIA][/B]

3x over 27 equals 2x minus 2 over 15
3x over 27 equals 2x minus 2 over 15 3x over 27: 3x/27 2x minus 2 over 15: (2x - 2)/15 Set them equal to each other: 3x/27 = (2x - 2)/15

3x to the power 2n
3x to the power 2n We take the expression 3x raise it to the power of 2n [B](3x)^2n[/B]

4 adults and 3 children cost $40. Two adults and 6 children cost $38
4 adults and 3 children cost $40. Two adults and 6 children cost $38 Givens and Assumptions: [LIST] [*]Let the number of adults be a [*]Let the number of children be c [*]Cost = Price * Quantity [/LIST] We're given 2 equations: [LIST=1] [*]4a + 3c = 40 [*]2a + 6c = 38 [/LIST] We can solve this system of equations 3 ways [LIST] [*][URL='https://www.mathcelebrity.com/simultaneous-equations.php?term1=4a+%2B+3c+%3D+40&term2=2a+%2B+6c+%3D+38&pl=Substitution']Substitution Method[/URL] [*][URL='https://www.mathcelebrity.com/simultaneous-equations.php?term1=4a+%2B+3c+%3D+40&term2=2a+%2B+6c+%3D+38&pl=Elimination']Elimination Method[/URL] [*][URL='https://www.mathcelebrity.com/simultaneous-equations.php?term1=4a+%2B+3c+%3D+40&term2=2a+%2B+6c+%3D+38&pl=Cramers+Method']Cramer's Rule[/URL] [/LIST] No matter what method we use, we get: [LIST] [*][B]a = 7[/B] [*][B]c = 4[/B] [/LIST]

4 consecutive integers such that the sum of the first 3 integers is equal to the 4th
4 consecutive integers such that the sum of the first 3 integers is equal to the 4th Let n be our first consecutive integer. [LIST=1] [*]n [*]n + 1 [*]n + 2 [*]n + 3 [/LIST] The sum of the first 3 integers is equal to the 4th: n + n + 1 + n + 2 = n + 3 Simplify by grouping like terms: (n + n + n) + (1 + 2) = n + 3 3n + 3 = n + 3 3n = n n = 0 n = 0 n + 1 = 1 n + 2 = 2 n + 3 = 3 Check our work: 0 + 1 +2 ? 3 3 = 3 Our final answer is [B](0, 1, 2, 3}[/B]

4 divided by sin60 degrees
4 divided by sin60 degrees. We can write as 4/sin(60). [URL='https://www.mathcelebrity.com/anglebasic.php?entry=60&coff=&pl=sin']Using our trigonometry calculator[/URL], we see sin(60) = sqrt(3)/2. So we have 4/sqrt(3)/2. Multiplying by the reciprocal we have: 4*2/sqrt(3) [B]8/sqrt(3)[/B]

4 less than the sum of 7 and 5
4 less than the sum of 7 and 5 Write this out [B](7 + 5) - 4[/B] Simplified 12 - 4 [B]8[/B]

4 machines can complete a job in 6 hours how long will it take 3 machines to complete the same jobs?
4 machines can complete a job in 6 hours how long will it take 3 machines to complete the same jobs? Set up a proportion of machines to hours where h is the number of hours that 3 machines take: 4/6 = 3/h [URL='https://www.mathcelebrity.com/prop.php?num1=4&num2=3&den1=6&den2=h&propsign=%3D&pl=Calculate+missing+proportion+value']Type this proportion into our search engine[/URL] and we get: h = [B]4.5[/B]

4 minus 3p equals 36
4 minus 3p equals 36 4 minus 3p: 4 - 3p The phrase [I]equals[/I] means an equation, so we set 4 - 3p equal to 36: [B]4 - 3p = 36[/B]

4 multiplied by the cube of p is reduced by 5
4 multiplied by the cube of p is reduced by 5 The cube of p means we raise p to the 3rd power: p^3 4 multiplied by the cube of p 4p^3 reduced by 5: [B]4p^3 - 5[/B]

4 rectangular strips of wood, each 30 cm long and 3 cm wide, are arranged to form the outer section
4 rectangular strips of wood, each 30 cm long and 3 cm wide, are arranged to form the outer section of a picture frame. Determine the area inside the wooden frame. Area inside forms a square, with a length of 30 - 3 - 3 = 24. We subtract 3 twice, because we account for 2 rectangular strips with a width of 3. Area of a square is side * side. So we have 24 * 24 = [B]576cm^2[/B]

4 teaspons of vegetable oil and 6 teaspoons of vinegar. 20 teaspoons of vegetable oil to how many te
4 teaspons of vegetable oil and 6 teaspoons of vinegar. 20 teaspoons of vegetable oil to how many teaspoons of vinegar? Set up a proportion where x is the number of teaspoons of vinegar in the second scenario: 4/6 = 20/x [URL='http://www.mathcelebrity.com/prop.php?num1=4&num2=20&den1=6&den2=x&propsign=%3D&pl=Calculate+missing+proportion+value']Plug that expression into the search engine to get[/URL] [B]x = 30[/B]

4 times 8 to the sixth power
4 times 8 to the sixth power 8 to the 6th power: 8^6 4 times this amount: 4 * 8^6 To evaluate this expression, we [URL='https://www.mathcelebrity.com/order-of-operations-calculator.php?num=4%2A8%5E6&pl=Perform+Order+of+Operations']type it in our search engine[/URL] and we get: 1,048,576

4 times a number added to 8 times a number equals 36
4 times a number added to 8 times a number equals 36 Let [I]a number[/I] be an arbitrary variable, let us call it x. 4 times a number: 4x 8 times a number: 8x We add these together: 4x + 8x = 12x We set 12x equal to 36 to get our final algebraic expression of: [B]12x = 36 [/B] If the problem asks you to solve for x, you [URL='https://www.mathcelebrity.com/1unk.php?num=12x%3D36&pl=Solve']type this algebraic expression into our search engine[/URL] and get: x = [B]3[/B]

4 times a number cubed decreased by 7
4 times a number cubed decreased by 7 A number is denoted as an arbitrary variable, let's call it x x Cubed means raise x to the third power x^3 Decreased by 7 means subtract 7 x^3 - 7

4 times a number is the same as the number increased by 78
4 times a number is the same as the number increased by 78. Let's take this algebraic expression in parts: [LIST=1] [*]The phrase [I]a number[/I] means an arbitrary variable, let's call it x. [*]4 times a number is written as 4x [*]The number increased by 78 means we add 78 to x: x + 78 [*]The phrase [I]the same as[/I] mean an equation, so we set #2 equal to #3 [/LIST] [B]4x = x + 78[/B] <-- This is our algebraic expression If the problem asks you to take it a step further, then [URL='https://www.mathcelebrity.com/1unk.php?num=4x%3Dx%2B78&pl=Solve']we type this equation into our search engine [/URL]and get: x = 26

4 times b increased by 9 minus twice y
4 times b increased by 9 minus twice y Take this algebraic expression in parts: Step 1: 4 times b means we multiply the variable b by 4: 4b Step 2: Increased by 9 means we add 9 to 4b: 4b + 9 Step 3: Twice y means we multiply the variable y by 2: 2y Step 4: The phrase [I]minus[/I] means we subtract 2y from 4b + 9 [B]4b + 9 - 2y[/B]

4 times of the sum of the cubes of x and y
4 times of the sum of the cubes of x and y The cube of x means we raise x to the 3rd power: x^3 The cube of y means we raise y to the 3rd power: y^3 The sum of the cubes means we add: x^3 + y^3 4 times the sum of the cubes: [B]4(x^3 + y^3)[/B]

4 times the difference of 6 times a number and 7
4 times the difference of 6 times a number and 7 The phrase [I]a number[/I] means an arbitrary variable, let's call it x. x 6 times a number 6x The difference of 6x and 7 means we subtract 7 from 6x: 6x - 7 Now we multiply this difference by 4: [B]4(6x - 7)[/B]

4 times the number of cows plus 2 times the number of ducks
4 times the number of cows plus 2 times the number of ducks Let c be the number of cows. Let d be the number of ducks. We've got an algebraic expression below: [B]4c + 2d[/B]

4 times the quantity of a number plus 6
4 times the quantity of a number plus 6 The phrase [I]a number[/I] means an arbitrary variable, let's call it x. x The word [I]plus[/I] means we addd 6 to x x + 6 The phrase [I]4 times the quantity [/I]means we multiply x + 6 by 4 [B]4(x + 6)[/B]

4 times the sum of 10 and twice x
4 times the sum of 10 and twice x Twice x means we multiply x by 2: 2x The sum of 10 and twice x: 10 + 2x Now multiply this sum by 4: [B]4(10 + 2x)[/B]

4 times the sum of 3 plus x squared
4 times the sum of 3 plus x squared x squared means we raise x to the power of 2: x^2 3 plus x squared: 3 + x^2 4 times the sum of 3 plus x squared 3(3 + x^2)

4 times the sum of q and p
The sum of q and p is written q + p 4 times the sum of q and p is written as: [B]4(q + p)[/B]

4 unknowns using Cramers Rule
Free 4 unknowns using Cramers Rule Calculator - Solves for 4 unknowns with equations in the form aw + bx + cy + dz = e using Cramers Method.

4/5 of the sum of k and 3
4/5 of the sum of k and 3 The sum of k and 3 means we add 3 to k: k + 3 4/5 of the sum means we multiply 4/5 times the sum k + 3: [B]4(k + 3)/5[/B]

40% of the days in September were sunny how many days were sunny?
40% of the days in September were sunny how many days were sunny? September has 30 days. So we [URL='https://www.mathcelebrity.com/perc.php?num=+5&den=+8&num1=+16&pct1=+80&pct2=40&den1=30&pcheck=3&pct=+82&decimal=+65.236&astart=+12&aend=+20&wp1=20&wp2=30&pl=Calculate']type 40% of 30 in our search engine[/URL]. We get: [B]12 days[/B]

400 reduced by 3 times my age is 214
400 reduced by 3 times my age is 214 Let my age be a. We have: 3 times my age: 3a 400 reduced by 3 times my age: 400 - 3a The word [I]is[/I] means an equation. So we set 400 - 3a equal to 214 400 - 3a = 214 Now if you want to solve this equation for a, we [URL='https://www.mathcelebrity.com/1unk.php?num=400-3a%3D214&pl=Solve']type it in the search engin[/URL]e and we get; a = [B]62[/B]

41% of the passengers on the plane are men. 36% of them are women and 11% of them are boys. The rema
41% of the passengers on the plane are men. 36% of them are women and 11% of them are boys. The remaining 30 passengers are girls. How many passengers are on the plane? Add up the percents: 41% + 36% + 11% = 88% This means that (100% - 88% = 12%) are girls. So if the total amount of passengers on the plane is p, we write 12% s 0.12, and we have: 0.12p = 30 [URL='https://www.mathcelebrity.com/1unk.php?num=0.12p%3D30&pl=Solve']Type this equation into our search engine[/URL], and we get: p = [B]250[/B]

414 people used public pool. Daily prices are $1.75 for children and $2.00 for adults. Total cost wa
414 people used public pool. Daily prices are $1.75 for children and $2.00 for adults. Total cost was $755.25. How many adults and children used the pool Let the number of children who used the pool be c, and the number of adults who used the pool be a. We're given two equations: [LIST=1] [*]a + c = 414 [*]2a + 1.75c = 755.25 [/LIST] We have a simultaneous equations. You can solve this any of 3 ways below: [LIST] [*][URL='https://www.mathcelebrity.com/simultaneous-equations.php?term1=a+%2B+c+%3D+414&term2=2a+%2B+1.75c+%3D+755.25&pl=Substitution']Substitution Method[/URL] [*][URL='https://www.mathcelebrity.com/simultaneous-equations.php?term1=a+%2B+c+%3D+414&term2=2a+%2B+1.75c+%3D+755.25&pl=Elimination']Elimination Method[/URL] [*][URL='https://www.mathcelebrity.com/simultaneous-equations.php?term1=a+%2B+c+%3D+414&term2=2a+%2B+1.75c+%3D+755.25&pl=Cramers+Method']Cramer's Rule[/URL] [/LIST] Whichever method you choose, you get the same answer: [LIST] [*][B]a = 123[/B] [*][B]c = 291[/B] [/LIST]

45 students, 12 taking spanish, 15 taking chemistry, 5 taking both spanish and chemistry. how many s
45 students, 12 taking spanish, 15 taking chemistry, 5 taking both spanish and chemistry. how many students are not taking either? Let S be the number of students taking spanish and C be the number of students taking chemistry: We have the following equation relating unions and intersections: P(C U S) = P(C) + P(S) - P(C and S) P(C U S) = 15 + 12 - 5 P(C U S) = 22 To get people that aren't taking either are, we have: 45 - P(C U S) 45 - 22 [B]23[/B]

45 water balloons were given to 9 children. If each child received the same number of water balloons
45 water balloons were given to 9 children. If each child received the same number of water balloons, how many water balloons did each child receive? Water Balloons per child = Total Water Balloons / Number of Children Water Balloons per child = 45/9 Water Balloons per child = [B]5[/B]

45.62% of Ricks home is lit at night with fluorescent bulbs and the remaining with LED lights. what
45.62% of Ricks home is lit at night with fluorescent bulbs and the remaining with LED lights. what is the percentage is lit with LED lights? Since the remaining is lit with LED lights, fluorescent and LED make up 100% of the lighting. So we have: Percentage of LED lights = 100% - 45.62% Percentage of LED lights = [B]54.38%[/B]

450 people attended a concert at the center. the center was 3/4 full. what is the capacity of the mu
450 people attended a concert at the center. the center was 3/4 full. what is the capacity of the music center. Let the capacity be c. We're given: 3c/4 = 450 To solve this equation, we [URL='https://www.mathcelebrity.com/prop.php?num1=3c&num2=450&den1=4&den2=1&propsign=%3D&pl=Calculate+missing+proportion+value']type it in our search engine[/URL] and we get: c = [B]600[/B]

46 people showed up to the party. There were 8 less men than women present. How many men were there?
46 people showed up to the party. There were 8 less men than women present. How many men were there? Let the number of men be m. Let the number of women be w. We're given two equations: [LIST=1] [*]m = w - 8 [I](8 less men than women)[/I] [*]m + w = 46 [I](46 showed up to the party)[/I] [/LIST] Substitute equation (1) into equation (2) for m: w - 8 + w = 46 To solve for w in this equation, we [URL='https://www.mathcelebrity.com/1unk.php?num=w-8%2Bw%3D46&pl=Solve']type in the equation into our search engine [/URL]and we get: w = 27 To solve for men (m), we substitute w = 27 into equation (1): m = 27 - 8 m = [B]19[/B]

46% of bullied students report notifying an adult at school about the incident. How much of the perc
46% of bullied students report notifying an adult at school about the incident. How much of the percentage did not notify an adult? When it comes to bullying, either a student notified or did not notify. If 46% notified, then: 100% - 46% = [B]54% did not notify[/B]

48 is the difference of Chrissys height and 13 .
48 is the difference of Chrissys height and 13 . Let Chrissy's height = h. The difference of the height and 13 is h - 13. We set this expression equal to 48: [B]h - 13 = 48 [/B] Note: To solve this, [URL='http://www.mathcelebrity.com/1unk.php?num=h-13%3D48&pl=Solve']paste this problem into the search engine[/URL].

4800$ salary spent 12% on clothes 20% on house rent how much money is she left with
4800$ salary spent 12% on clothes 20% on house rent how much money is she left with 12% on clothes plus 20% on house rent = 32% total spendings. If she spent 32%, that means she's left with: 100% - 32% = 68% So we want 68% of 4800. We [URL='https://www.mathcelebrity.com/perc.php?num=+5&den=+8&num1=+16&pct1=+80&pct2=68&den1=4800&pcheck=3&pct=+82&decimal=+65.236&astart=+12&aend=+20&wp1=20&wp2=30&pl=Calculate']type [I]68% of 4800 [/I]into our search engine[/URL] and we get: [B]3,264[/B]

4d/a - 9 = g for a
4d/a - 9 = g for a Add 9 to each side: 4d/a - 9 + 9 = g + 9 Cancel the 9's on the left side and we get: 4d/a = g + 9 Cross multiply: 4d = a(g + 9) Divide each side of the equation by (g + 9) to isolate a: 4d/(g + 9) = a(g + 9)/(g + 9) Cancel the (g + 9) on the right side, and we get: a = [B]4d/(g + 9)[/B]

4n - 8 = n + 1
4n - 8 = n + 1 Solve for [I]n[/I] in the equation 4n - 8 = n + 1 [SIZE=5][B]Step 1: Group variables:[/B][/SIZE] We need to group our variables 4n and n. To do that, we subtract n from both sides 4n - 8 - n = n + 1 - n [SIZE=5][B]Step 2: Cancel n on the right side:[/B][/SIZE] 3n - 8 = 1 [SIZE=5][B]Step 3: Group constants:[/B][/SIZE] We need to group our constants -8 and 1. To do that, we add 8 to both sides 3n - 8 + 8 = 1 + 8 [SIZE=5][B]Step 4: Cancel 8 on the left side:[/B][/SIZE] 3n = 9 [SIZE=5][B]Step 5: Divide each side of the equation by 3[/B][/SIZE] 3n/3 = 9/3 n = [B]3[/B]

4subtractedfrom6timesanumberis32
4 subtracted from 6 times a number is 32. Take this algebraic expression in pieces. The phrase [I]a number[/I] means an arbitrary variable, let's call it x. x 6 times this number means we multiply by x by 6 6x 4 subtracted from this expression means we subtract 4 6x - 4 The phrase [I]is[/I] means an equation, so we set 6x - 4 equal to 32 [B]6x - 4 = 32 [/B] If you need to solve this equation, [URL='https://www.mathcelebrity.com/1unk.php?num=6x-4%3D32&pl=Solve']type it in the search engine here[/URL].

5 -8| -2n|=-75
Subtract 5 from each side: -8|-2n| = -80 Divide each side by -8 |-2n| = 10 Since this is an absolute value equation, we need to setup two equations: -2n = 10 -2n = -10 Solving for the first one by dividing each side by -2, we get: n = -5 Solving for the second one by dividing each side by -2, we get: n = 5

5 added to x is 11
x + 5 = 11 for the algebraic expression. Plug that into the [URL='http://www.mathcelebrity.com/1unk.php?num=x%2B5%3D11&pl=Solve']search engine[/URL], and solve for x. x = 6.

5 added to xis 11
5 added to x means we use the plus sign for a sum. x + 5 "is" means equals, so we set that equal to 11. x + 5 = 11 <-- This is our algebraic expression.

5 books and 5 bags cost $175. What is the cost of 2 books and 2 bags
5 books and 5 bags cost $175. What is the cost of 2 books and 2 bags Let the cost of each book be b and the cost of each bag be c. We're given 5b + 5c = 175 We can factor this as: 5(b + c) = 175 Divide each side of the equation by 5, we get: (b + c) = 35 The problem asks for 2b + 2c Factor out 2: 2(b + c) we know from above that (b + c) = 35, so we substitute: 2(35) [B]70[/B]

5 Card Poker Hand
Free 5 Card Poker Hand Calculator - Calculates and details probabilities of the 10 different types of poker hands given 1 player and 1 deck of cards.

5 chocolates cost $25. What will 15 chocolates cost?
5 chocolates cost $25. What will 15 chocolates cost? Set up a proportion of chocolates to cost, where p is the price of 15 chocolates: 5/25 = 15/p Using our [URL='http://www.mathcelebrity.com/prop.php?num1=5&num2=15&den1=25&den2=p&propsign=%3D&pl=Calculate+missing+proportion+value']proportion calculator[/URL], we get [B]p = 75[/B]

5 diminished by twice the sum of a and b
5 diminished by twice the sum of a and b Take this algebraic expression in parts: [LIST] [*]The sum of a and b: a + b [*]Twice the sum means we multiply a + b by 2: 2(a + b) [*]5 diminished by twice the sum means we subtract 2(a + b) from 5 [/LIST] [B]5 - 2(a + b)[/B]

5 girls share 4 sandwiches. What fraction of the sandwich does each girl get
5 girls share 4 sandwiches. What fraction of the sandwich does each girl get? We want to know sandwiches per girls. So we divide: 4 sandwiches per 5 girls [B]4/5[/B]

5 is one-fourth of a number c
5 is one-fourth of a number c [LIST] [*]A number c is just written as c [*]one-fourth of c means we multiply c by 1/4: c/4 [*]The phrase [I]is[/I] means equal to, so we set c/4 equal to 5 [/LIST] [B]c/4 = 5[/B]

5 kids are sharing 3 cupcakes. How much will they each get
5 kids are sharing 3 cupcakes. How much will they each get 3 cupcakes / 5 kids = [B]3/5 cupcake per kid or 0.6 cupcake per kid[/B]

5 more than the reciprocal of a number
5 more than the reciprocal of a number Take this algebraic expression in pieces: The phrase [I]a number[/I] means an arbitrary variable, let's call it x. x The reciprocal of this number means we divide 1 over x: 1/x 5 more means we add 5 to 1/x [B]1/x + 5[/B]

5 more than twice the cube of a number
5 more than twice the cube of a number. Take this algebraic expression in pieces. The phrase [I]a number[/I] means an arbitrary variable, let's call it x. x The cube of a number means we raise it to a power of 3 x^3 Twice the cube of a number means we multiply x^3 by 2 2x^3 5 more than twice the cube of a number means we multiply 2x^3 by 5 5(2x^3) Simplifying, we get: 10x^3

5 more than twice the cube of a number
5 more than twice the cube of a number The phrase [I]a number[/I] means an arbitrary variable, let's call it x: x The cube of a number means we raise x to the power of 3: x^3 Twice the cube means we multiply x^3 by 2 2x^3 Finally, 5 more than twice the cube means we add 5 to 2x^3: [B]2x^3 + 5[/B]

5 more then the sum of x and y
The sum of x and y is denoted x + y. 5 more then the is denoted x + y + 5.

5 shirts. 3 pants and 8 shoes how many outfits can you wear
5 shirts. 3 pants and 8 shoes how many outfits can you wear Using the fundamental rule of counting, we can have: 5 shirts * 3 pants * 8 shoes = [B]120 different outfits[/B]

5 squared minus a number x
5 squared minus a number x 5 squared is written as 5^2 Minus a number x means we subtract the variable x [B]5^2 - x[/B]

5 subtracted from 3 times a number is 44
5 subtracted from 3 times a number is 44. The problem asks for an algebraic expression. The phrase [I]a number[/I] means an arbitrary variable, let's call it x. 3 times this number is 3x. 5 subtracted from this is written as 3x - 5. The phrase [I]is[/I] means an equation, so we set 3x - 5 equal to 44 [B]3x - 5 = 44[/B]

5 times a number increased by 4 is divided by 6 times the same number
5 times a number increased by 4 is divided by 6 times the same number Take this algebraic expression in parts. Part 1: 5 times a number increased by 4 [LIST] [*]The phrase [I]a number[/I] means an arbitrary variable, let's call it x: x [*]5 times the number means multiply x by 5: 5x [*][I]Increased by 4[/I] means we add 4 to 5x: 5x + 4 [/LIST] Part 2: 6 times the same number [LIST] [*]From above, [I]a number[/I] is x: x [*]6 times the number means we multiply x by 6: 6x [/LIST] The phrase [I]is divided by[/I] means we have a quotient, where 5x + 4 is the numerator, and 6x is the denominator. [B](5x + 4)/6x[/B]

5 times a number is 4 more than twice a number
5 times a number is 4 more than twice a number The phrase [I]a number[/I] means an arbitrary variable, let's call it x. x 5 times a number: 5x Twice a number means we multiply x by 2: 2x 4 more than twice a number 2x + 4 The word [I]is[/I] means equal to, so we set 5x equal to 2x + 4 [B]5x = 2x + 4[/B]

5 times a number is that number minus 3
5 times a number is that number minus 3 The phrase [I]a number[/I] means an arbitrary variable. Let's call it x. [LIST] [*]5 times a number: 5x [*]That number means we use the same number from above which is x [*]That number minus 3: x - 3 [*]The phrase [I]is[/I] means an equation, so we set 5x equal to x - 3 [/LIST] [B]5x = x - 3[/B]

5 times g reduced by the square of h
5 times g reduced by the square of h Take this algebraic expression in pieces: [LIST=1] [*]5 times g means we multiply g by 5: 5g [*]The square of h means we raise h to the 2nd power: h^2 [*]5 times g reduced by the square of h means we subtract h^2 from 5g: [/LIST] [B]5g - h^2[/B]

5 times the product of 2 numbers a and b
5 times the product of 2 numbers a and b The product of 2 numbers a and be means we multiply the variables together: ab 5 times the product means we multiply ab by 5: [B]5ab[/B]

5 times the sum of 3 times a number and -5
5 times the sum of 3 times a number and -5 The phrase [I]a number[/I] means an arbitrary variable, let's call it x: x 3 times a number means we multiply x by 3: 3x the sum of 3 times a number and -5 means we add -5 to 3x: 3x - 5 5 times the sum means we multiply 3x - 5 by 5: [B]5(3x - 5)[/B]

5 times the total of 60 and x
5 times the total of 60 and x The total of 60 and x means we add: 60 + x 5 times the total means we multiply the sum by 5 5(60 + x)

5 years ago Kevin was 3 times as old as Tami. Now he is twice as old as she is. How is each now?
5 years ago Kevin was 3 times as old as Tami. Now he is twice as old as she is. How is each now? Let Kevin's age be k. Let Tami's age be t. We're given the following equations: [LIST=1] [*]k - 5 = 3(t - 5) [*]k = 2t [/LIST] Plug equation (2) into equation (1) for k: 2t - 5 = 3(t - 5) We p[URL='https://www.mathcelebrity.com/1unk.php?num=2t-5%3D3%28t-5%29&pl=Solve']lug this equation into our search engine[/URL] and we get: t = [B]10. Tami's age[/B] Now plug t = 10 into equation (2) to solve for k: k = 2(10) k =[B] 20. Kevin's age[/B]

5, 14, 23, 32, 41....1895 What term is the number 1895?
5, 14, 23, 32, 41....1895 What term is the number 1895? Set up a point slope for the first 2 points: (1, 5)(2, 14) Using [URL='https://www.mathcelebrity.com/search.php?q=%281%2C+5%29%282%2C+14%29&x=0&y=0']point slope formula, our series function[/URL] is: f(n) = 9n - 4 To find what term 1895 is, we set 9n - 4 = 1895 and solve for n: 9n - 4 = 1895 Using our [URL='https://www.mathcelebrity.com/1unk.php?num=9n-4%3D1895&pl=Solve']equation solver[/URL], we get: n = [B]211[/B]

5,10,15,20 What is the next number? What is the 100th term?
5,10,15,20 What is the next number? What is the 100th term? Increment is by 5, so next number is 20 + 5 = [B]25[/B] Formula for nth number is 5 * n With n = 100, we have 5 * 100 = [B]500[/B]

5/8 Of a class are boys. what fraction of the class are girls
5/8 Of a class are boys. what fraction of the class are girls? The total class equals 1. Since 5/8 are boys, we subtract 5/8 from 1: 1 - 5/8 But we can write 1 as 8/8. So we have 8/8 - 5/8 [URL='https://www.mathcelebrity.com/fraction.php?frac1=8%2F8&frac2=5%2F8&pl=Subtract']Type this fraction operation into our search engine[/URL] and we get: [B]3/8[/B] are girls

50 is more than the product of 4 and w
50 is more than the product of 4 and w Take this algebraic expression in pieces: The product of 4 and w mean we multiply the variable w by 4: 4w The phrase [I]is more than[/I] means an inequality using the (>) sign, where 50 is greater than 4w: [B]50 > 4w[/B]

5000 union members of a financially troubled company accepted a 17% pay cut. The company announced t
5000 union members of a financially troubled company accepted a 17% pay cut. The company announced that this would save them approximately $108 million annually. Based on this information, calculate the average annual pay of a single union member Let the full salary of the union members be s. Since 17% is 0.17, We're given: 0.17s = 108000000 To solve this equation, we [URL='https://www.mathcelebrity.com/1unk.php?num=0.17s%3D108000000&pl=Solve']type it in our search engine[/URL] and we get: s = 635,294,117.65 Calculate the average annual pay of a single union member: Average Pay = Total Pay / Number of Union Members Average Pay = 635,294,117.65 / 5000 Average Pay = [B]127,058.82[/B]

508 people are there, the daily price is $1.25 for kids and $2.00 for adults. The receipts totaled $
508 people are there, the daily price is $1.25 for kids and $2.00 for adults. The receipts totaled $885.50. How many kids and how many adults were there? Assumptions: [LIST] [*]Let the number of adults be a [*]Let the number of kids be k [/LIST] Given with assumptions: [LIST=1] [*]a + k = 508 [*]2a + 1.25k = 885.50 (since cost = price * quantity) [/LIST] Rearrange equation (1) by subtracting c from each side to isolate a: [LIST=1] [*]a = 508 - k [*]2a + 1.25k = 885.50 [/LIST] Substitute equation (1) into equation (2): 2(508 - k) + 1.25k = 885.50 Multiply through: 1016 - 2k + 1.25k = 885.50 1016 - 0.75k = 885.50 To solve for k, we [URL='https://www.mathcelebrity.com/1unk.php?num=1016-0.75k%3D885.50&pl=Solve']type this equation into our search engine[/URL] and we get: k = [B]174[/B] Now, to solve for a, we substitute k = 174 into equation 1 above: a = 508 - 174 a = [B]334[/B]

52 card deck what is the probability of being dealt a two
52 card deck what is the probability of being dealt a two? There are four 2's in a deck, so we have 4/52. 4/52 is reducible to [B]1/13[/B]

52% of a town's households have children and 25% have pets. If 12% have both, what percent have neit
52% of a town's households have children and 25% have pets. If 12% have both, what percent have neither Let C represent households with children. Let P represents households with pets. We have the formula to determine households with Children or Pets as C U P (C Union P) or (C or P): C U P = C + P - (C and P) C U P = 52% + 25% - 12% C U P = 65% Now, if we want to find what percent have neither, we use (C U P)': (C U P)' = 100% - (C U P) (C U P)' = 100% - 65% (C U P)' = [B]35%[/B]

54 is the sum of 15 and Vidyas score
54 is the sum of 15 and Vidyas score. Let Vida's score be s. The sum of 15 and s: s + 15 When they say "is", they mean equal to, so we set s + 15 equal to 54. Our algebraic expression is below: [B]s + 15 = 54 [/B] To solve this equation for s, use our [URL='http://www.mathcelebrity.com/1unk.php?num=s%2B15%3D54&pl=Solve']equation calculator[/URL]

54 is the sum of 24 and Julies score. Use the variable J to represent Julies score.
54 is the sum of 24 and Julies score. Use the variable J to represent Julies score. Sum of 24 and Julie's score: 24 + J The phrase [I]is[/I] means an equation, so we set 24 + J equal to 54 to get an algebraic expression: [B]24 + J = 54[/B]

54% of students got an F, 15% of students got a D, 19% of students got a B and 12% of students got a
54% of students got an F, 15% of students got a D, 19% of students got a B and 12% of students got an A. if there were 26 students, how many got an F? First, we check our total percentages: 54% + 15% + 19% + 12% = 100% <-- Good, we have our full sample set F Students = 54% * 26 F Students = 14.04 -->[B] 14[/B]

55 foot tall tree casts a shadow that is 32 feet long, a nearby woman is 5.5 feet tall. What is the
55 foot tall tree casts a shadow that is 32 feet long, a nearby woman is 5.5 feet tall. What is the length of shadow she will cast? Set up a proportion of height to shadow length where s is the shadow length of the woman: 55/32 = 5.5/s [URL='https://www.mathcelebrity.com/prop.php?num1=55&num2=5.5&den1=32&den2=s&propsign=%3D&pl=Calculate+missing+proportion+value']Typing this proportion into our search engine[/URL], we get: s = [B]3.2[/B]

56 is the sum of 20 and Donnie's savings. Use the variable d to represent Donnie's savings
56 is the sum of 20 and Donnie's savings. Use the variable d to represent Donnie's savings The sum of 20 and Donnie's savings using [I]d[/I] to represent Donnie's savings: 20 + d The word [I]is[/I] means equal to, so we set 20 + d equal to 56: [B]20 + d = 56[/B]

59 is the difference of vanessas height and 20
59 is the difference of vanessas height and 20. Let h be Vanessa's height. We have the difference of h and 20: h - 20 The phrase [I]is[/I] means equal to, so we set h - 20 equal to 59 [B]h - 20 = 59[/B]

59 is the sum of 16 and Donnie's saving. Use the variable d to represent Donnie's saving.
59 is the sum of 16 and Donnie's saving. Use the variable d to represent Donnie's saving. The phrase [I]the sum of[/I] means we add Donnie's savings of d to 16: d + 16 The phrase [I]is[/I] means an equation, so we set d + 16 equal to 59 d + 16 = 59 <-- [B]This is our algebraic expression[/B] Now, if the problem asks you to solve for d, then you[URL='https://www.mathcelebrity.com/1unk.php?num=d%2B16%3D59&pl=Solve'] type the algebraic expression into our search engine to get[/URL]: d = [B]43[/B]

5n - 5 = 85
5n - 5 = 85 Solve for [I]n[/I] in the equation 5n - 5 = 85 [SIZE=5][B]Step 1: Group constants:[/B][/SIZE] We need to group our constants -5 and 85. To do that, we add 5 to both sides 5n - 5 + 5 = 85 + 5 [SIZE=5][B]Step 2: Cancel 5 on the left side:[/B][/SIZE] 5n = 90 [SIZE=5][B]Step 3: Divide each side of the equation by 5[/B][/SIZE] 5n/5 = 90/5 n = [B]18[/B]

5^(n - 1) = 15,625
5^(n - 1) = 15,625 We know 5^6 = 15,625, so we have: n - 1 = 6 Add 1 to each side: n - 1 + 1 = 6 + 1 Cancel the 1's on the left side: n = [B]7[/B]

6 books cost 9.24. How much is 1 book
6 books cost 9.24. How much is 1 book Set up a proportion of cost to books where x is the cost for 1 book: 9.24/6 = x/1 To solve this proportion for x, we [URL='https://www.mathcelebrity.com/prop.php?num1=9.24&num2=x&den1=6&den2=1&propsign=%3D&pl=Calculate+missing+proportion+value']type it in our search engine[/URL] and we get: x = [B]1.54[/B]

6 boys have a mean age of 10 years and 14 girls have a mean age of 5 work out the mean age of the 20
6 boys have a mean age of 10 years and 14 girls have a mean age of 5 work out the mean age of the 20 children [U]Calculate Sum of boys ages:[/U] Sum of boys ages/6 = 10 Cross multiply, and we get: Sum of boys ages = 6 * 10 Sum of boys ages = 60 [U]Calculate Sum of girls ages:[/U] Sum of girls ages/14 = 5 Cross multiply, and we get: Sum of girls ages = 14 * 5 Sum of girls ages = 70 Average of 20 children is: Average of 20 children = (Sum of boys ages + sum of girls ages)/20 Average of 20 children = (60 + 70)/20 Average of 20 children = 130/20 Average of 20 children = [B]6.5 years[/B]

6 diminished by twice x is at most 8
6 diminished by twice x is at most 8 Twice x means we multiply x by 2: 2x 6 diminished by twice x means we subtract 2x from 6: 6 - 2x The phrase [I]is at most[/I] is an inequality using the sign <=, so we have: [B]6 - 2x <= 8[/B]

6 is divided by square of a number
6 is divided by square of a number The phrase [I]a number [/I]means an arbitrary variable, let's call it x. x the square of this means we raise x to the power of 2: x^2 Next, we divide 6 by x^2: [B]6/x^2[/B]

6 is one third of a number s
6 is one third of a number s A number s is written as s: s One third of a number s means we multiply s by 1/3 s/3 The word [I]is[/I] means equal to, so we set s/3 equal to 6 [B]s/3 = 6[/B]

6 mph, 2 hours what is the distance
6 mph, 2 hours what is the distance Distance = Rate * Time Distance = 6 mph * 2 hours Distance = [B]12 miles [/B] You can also use our [URL='http://www.mathcelebrity.com/drt.php?d=+&r=+6&t=+2&pl=Calculate+the+missing+Item+from+D%3DRT']distance-rate-time calculator[/URL]

6 numbers have a mean of 4. What is the total of the 6 numbers?
6 numbers have a mean of 4. What is the total of the 6 numbers? Mean = Sum of numbers / Count of numbers Plug our Mean of 4 and our count of 6 into this equation: 4 = Sum/Total of Numbers / 6 Cross multiply: Sum/Total of Numbers = 6 * 4 Sum/Total of Numbers = [B]24[/B]

6 plus twice the sum of a number and 7.
6 plus twice the sum of a number and 7. The phrase [I]a number[/I] mean an arbitrary variable, let's call it x. The sum of a number and 7 means we add 7 to the variable x. x + 7 Twice the sum means we multiply the sum by 2: 2(x + 7) 6 plus means we add 6 to 2(x + 7) [B]6 + 2(x + 7)[/B]

6 red marbles 9 green marbles and 5 blue marbles two marbles are drawn without replacement what is t
6 red marbles 9 green marbles and 5 blue marbles two marbles are drawn without replacement what is the probability of choosing a green and then a blue marble First draw: there are 6 red + 9 green + 5 blue = 20 marbles We draw 9 possible green out of 20 total marbles = 9/20 Second draw: We don't replace, so we have 6 red + 8 green + 5 blue = 19 marbles We draw 5 possible blue of out 19 total marbles = 5/19 Our total probability, since each event is independent, is: [URL='https://www.mathcelebrity.com/fraction.php?frac1=9%2F20&frac2=5%2F19&pl=Multiply']9/20 * 5/19[/URL] = [B]9/76[/B]

6 sided die probability to roll a odd number or a number less than 6
6 sided die probability to roll a odd number or a number less than 6 First, we'll find the set of rolling an odd number. [URL='https://www.mathcelebrity.com/1dice.php?gl=1&opdice=1&pl=Odds&rolist=+2%2C3%2C4&dby=+2%2C3%2C5&montect=+100']From this dice calculator[/URL], we get: Odd = {1, 3, 5} Next, we'll find the set of rolling less than a 6. [URL='https://www.mathcelebrity.com/1dice.php?gl=4&pl=6&opdice=1&rolist=+2%2C3%2C4&dby=+2%2C3%2C5&montect=+100']From this dice calculator[/URL], we get: Less than a 6 = {1, 2, 3, 4, 5} The question asks for [B]or[/B]. Which means a Union: {1, 3, 5} U {1, 2, 3, 4, 5} = {1, 2, 3, 4, 5} This probability is [B]5/6[/B]

6 subtracted from the product of 5 and a number is 68
6 subtracted from the product of 5 and a number is 68 Take this algebraic expression in parts. The phrase [I]a number[/I] means an arbitrary variable, let's call it x. x The product of 5 and this number is: 5x We subtract 6 from 5x: 5x - 6 The phrase [I]is[/I] means an equation, so we set 5x - 6 equal to 68 [B]5x - 6 = 68[/B]

6 times a number multiplied by 3 all divided by 4
6 times a number multiplied by 3 all divided by 4 Take this algebraic expression in parts: [LIST] [*]The phrase [I]a number[/I] means an arbitrary variable, let's call it x [*]6 times a number: 6x [*]Multiplied by 3: 3(6x) = 18x [*]All divided by 4: 18x/4 [/LIST] We can simplify this: We type 18/4 into our search engine, simplify, and we get 9/2. So our answer is: [B]9x/2[/B]

6 times a number, x, is at least 22.
6 times a number, x, is at least 22. 6 times a number x: 6x The phrase [I]is at least[/I] means greater than or equal to. So we have an inequality: [B]6x >= 22[/B] <-- This is our algebraic expression [URL='https://www.mathcelebrity.com/interval-notation-calculator.php?num=6x%3E%3D22&pl=Show+Interval+Notation']To solve this for x, paste this into the search engine[/URL] and we get: [B]x >= 3.666667[/B]

6 times j squared minus twice j squared
6 times j squared minus twice j squared j squared means we raise the variable j to the power of 2: j^2 6 times j squared means we multiply j^2 by 6: 6j^2 Twice j squared means we multiply j^2 by 2: 2j^2 The word [I]minus[/I] means we subtract 2j^2 from 6j^2 6j^2 - 2j^2 So if you must simplify, we group like terms and get: (6 - 2)j^2 [B]4j^2[/B]

6 times the quantity 17b minus 19
6 times the quantity 17b minus 19 the quantity 17b minus 19 17b - 19 6 times the quantity 17b minus 19 6(17b - 19)

6 times the reciprocal of a number equals 2 times the reciprocal of 7. What is the number
6 times the reciprocal of a number equals 2 times the reciprocal of 7. What is the number We've got two algebraic expressions here. Let's take it in parts: Term 1: The phrase [I]a number[/I] means an arbitrary variable, let's call it x. The reciprocal is 1/x Multiply this by 6: 6/x Term 2: Reciprocal of 7: 1/7 2 times this: 2/7 We set these terms equal to each other: 6/x = 2/7 [URL='https://www.mathcelebrity.com/prop.php?num1=6&num2=2&den1=x&den2=7&propsign=%3D&pl=Calculate+missing+proportion+value']Type this proportion into the search engine[/URL], and we get: [B]x = 21[/B]

6 times the reciprocal of a number equals 3 times the reciprocal of 7 .
6 times the reciprocal of a number equals 3 times the reciprocal of 7 . This is an algebraic expression. Let's take it in parts: The phrase [I]a number[/I] means an arbitrary variable, let's call it x. x The reciprocal of a number x means we divide 1 over x: 1/x 6 times the reciprocal means we multiply 6 by 1/x: 6/x The reciprocal of 7 means we divide 1/7 1/7 3 times the reciprocal means we multiply 1/7 by 3: 3/7 Now, the phrase [I]equals[/I] mean an equation, so we set 6/x = 3/7 [B]6/x = 3/7[/B] <-- This is our algebraic expression If the problem asks you to solve for x, then [URL='https://www.mathcelebrity.com/prop.php?num1=6&num2=3&den1=x&den2=7&propsign=%3D&pl=Calculate+missing+proportion+value']we type this proportion in our search engine[/URL] and get: x = 14

6 times the sum of a number and 3 is equal to 42. What is this number?
6 times the sum of a number and 3 is equal to 42. What is this number? The phrase [I]a number[/I] means an arbitrary variable, let's call it x. The sum of a number and 3 means we add 3 to x: x + 3 6 times the sum: 6(x + 3) The word [I]is[/I] means an equation, so we set 6(x + 3) equal to 42 to get our [I]algebraic expression[/I] of: [B]6(x + 3) = 42[/B] [B][/B] If the problem asks you to solve for x, then [URL='https://www.mathcelebrity.com/1unk.php?num=6%28x%2B3%29%3D42&pl=Solve']you type this equation into our search engine[/URL] and you get: x = [B]4[/B]

6 times the sum of a number and 5 is 16
6 times the sum of a number and 5 is 16 A number represents an arbitrary variable, let's call it x x The sum of x and 5 x + 5 6 times the sum of x and 5 6(x + 5) Is means equal to, so set 6(x + 5) equal to 16 [B]6(x + 5) = 16 <-- This is our algebraic expression Solve for x[/B] Multiply through: 6x + 30 = 16 Subtract 30 from each side: 6x - 30 + 30 = 16 - 30 6x = -14 Divide each side by 6 6x/6 = -14/6 Simplify this fraction by dividing top and bottom by 2: x = [B]-7/3 [MEDIA=youtube]oEx5dsYK7DY[/MEDIA][/B]

6 times y divided by x squared
6 times y divided by x squared 6 times y: 6y x squared means we raise x to the power of 2: x^2 The phrase [I]divided by[/I] means we have a fraction: [B]6y/x^2[/B]

6 times y divided by x squared
6 times y divided by x squared 6 times y: 6y x squared means we raise x to the power of 2: x^2 The phrase [I]divided by[/I] means we divide 6y by x^2: [B]6y/x^2[/B]

6 years from now Cindy will be 25 years old. in 12 years, the sum of the ages of Cindy and Jose will
6 years from now Cindy will be 25 years old. in 12 years, the sum of the ages of Cindy and Jose will be 91. how old is Jose right now? Let c be Cindy's age and j be Jose's age. We have: c + 6 = 25 This means c = 19 using our [URL='https://www.mathcelebrity.com/1unk.php?num=c%2B6%3D25&pl=Solve']equation calculator[/URL]. We're told in 12 years, c + j = 91. If Cindy's age (c) is 19 right now, then in 12 years, she'll be 19 + 12 = 31. So we have 31 + j = 91. Using our [URL='https://www.mathcelebrity.com/1unk.php?num=31%2Bj%3D91&pl=Solve']equation calculator[/URL], we get [B]j = 60[/B].

60 is the sum of 22 and Helenas height. Use the variable h to represent Helenas height.
60 is the sum of 22 and Helenas height. Use the variable h to represent Helenas height. If height is represented by h, we have: 22 and h 22 + h When they say "is the sum of", we set 22 + h equal to 60 [B]22 + h = 60[/B]

60 percent of a number minus 17 is -65
60 percent of a number minus 17 is -65 Using our [URL='https://www.mathcelebrity.com/perc.php?num=+5&den=+8&num1=+16&pct1=+80&pct2=+90&den1=+80&pct=60&pcheck=4&decimal=+65.236&astart=+12&aend=+20&wp1=20&wp2=30&pl=Calculate']percent to decimal calculator[/URL], we see that 60% is 0.6, so we have: 0.6 The phrase [I]a number[/I] means an arbitrary variable, let's call it x. So 60% of a number is: 0.6x Minus 17: 0.6x - 17 The word [I]is[/I] means an equation, so we set 0.6x - 17 equal to -65 to get our algebraic expression of: [B]0.6x - 17 = -65[/B] [B][/B] If you want to solve for x in this equation, you [URL='https://www.mathcelebrity.com/1unk.php?num=0.6x-17%3D-65&pl=Solve']type it in our search engine and you get[/URL]: [B]x = -80[/B]

63 oranges is shared among 3 boys in the ratio of 2:3:4 how many oranges will the second boy receive
63 oranges is shared among 3 boys in the ratio of 2:3:4 how many oranges will the second boy receive? Total sharing is 2 + 3 + 4 = 10. [LIST] [*]Boy 2 = 3/10 * 63 = [B]18.9 oranges[/B] [/LIST]

64 divided by the cube of y
64 divided by the cube of y The cube of y means y raised to the 3rd power: y^3 64 divided by this: [B]64/y^3[/B]

64 is 4 times the difference between Sarah's age, a, and 44. Assume Sarah is older than 44.
64 is 4 times the difference between Sarah's age, a, and 44. Assume Sarah is older than 44. The phrase [I]difference between[/I] means we subtract 44 from a: a - 44 The phrase [I]64 is[/I] means an equation, so we set a - 44 equal to 64 [B]a - 44 = 64 <-- This is our algebraic expression [/B] If you want to solve for a, then we [URL='https://www.mathcelebrity.com/1unk.php?num=a-44%3D64&pl=Solve']type this expression into our search engine[/URL] and get: [B]a = 108[/B]

64 is 4 times the difference between Sarah’s age a, and 44.Assume Sarah is older than 44
64 is 4 times the difference between Sarah’s age a, and 44.Assume Sarah is older than 44 Difference between Sarah's age (a) and 44 (Assuming Sarah is older than 44): a - 44 4 times the difference: 4(a - 44) The word [I]is[/I] means equal to, so we set 4(a - 44) equal to 64 to get our algebraic expression: [B]4(a - 44) = 64[/B] If the problem asks you to solve for a, we [URL='https://www.mathcelebrity.com/1unk.php?num=4%28a-44%29%3D64&pl=Solve']type this equation into our search engine[/URL] and we get: a = [B]60[/B]

66 decreased by Janelle's savings is 15
66 decreased by Janelle's savings is 15 Let Janelle's savings be s. 66 decreased by s is: 66 - s The word [I]is[/I] means equal so we set 66 - s equal to 15 [B]66 - s = 15[/B]

67 less than the quantity 96 times q
67 less than the quantity 96 times q 96 times q: 96q 67 less than the quantity 96 times q [B]96q - 67[/B]

6700 dollars is placed in an account with an annual interest rate of 8%. How much will be in the acc
6700 dollars is placed in an account with an annual interest rate of 8%. How much will be in the account after 24 years, to the nearest cent? [URL='https://www.mathcelebrity.com/intbal.php?startbal=6700&intrate=8&bstart=1%2F1%2F2000&bend=1%2F1%2F2024&pl=Annual+Credit']Using our balance with interest calculator[/URL], we get: [B]$42,485.94[/B]

6700 dollars is placed in an account with an annual interest rate of 8%. show much will be in the ac
6700 dollars is placed in an account with an annual interest rate of 8%. show much will be in the account after 24 years, to the nearest cent ? Using our compound interest calculator, we get: [B]42,485.91 [MEDIA=youtube]0C25FB_4004[/MEDIA][/B]

6700 dollars is placed in an account with an annual interest rate of 8.25%. How much will be in the
6700 dollars is placed in an account with an annual interest rate of 8.25%. How much will be in the account after 28 years, to the nearest cent? Using our [URL='https://www.mathcelebrity.com/compoundint.php?bal=6700&nval=28&int=8.25&pl=Annually']balance with interest calculator[/URL], we get: 61,667.47

69 divided by the sum of 10 and y
69 divided by the sum of 10 and y The sum of 10 and y 10 + y 69 divided by this [B]69/(10 + y)[/B]

7 added to the quotient of 2x and 4
7 added to the quotient of 2x and 4 The quotient of 2x and 4: 2x/4 7 added to 2x/4: [B]2x/4 + 7[/B]

7 and 105 are successive terms in a geometric sequence. what is the term following 105?
7 and 105 are successive terms in a geometric sequence. what is the term following 105? Geometric sequences are set up such that the next term in the sequence equals the prior term multiplied by a constant. Therefore, we express the relationship in the following equation: 7k = 105 where k is the constant [URL='https://www.mathcelebrity.com/1unk.php?num=7k%3D105&pl=Solve']Type this equation into our search engine[/URL] and we get: k = 15 The next term in the geometric sequence after 105 is found as follows: 105*15 = [B]1,575[/B]

7 black shirts 5 white shirts 10 gray shirts one is chosen at random, what is the probability that i
7 black shirts 5 white shirts 10 gray shirts one is chosen at random, what is the probability that it is not gray [U]Find the total shirts:[/U] Total shirts = Black Shirts + White Shirts + Gray Shirts Total shirts = 7 + 5 + 10 Total shirts = 22 [U]Calculate the probability of choosing a gray shirt:[/U] P(Gray) = Number of Gray shirts / Total Shirts P(Gray) = 10/22 We can simplify this fraction. We [URL='https://www.mathcelebrity.com/fraction.php?frac1=10%2F22&frac2=3%2F8&pl=Simplify']type in 10/22 into our search engine, choose simplify[/URL], and we get: P(Gray) = [B]5/11[/B]

7 is 1/4 of some number
7 is 1/4 of some number The phrase [I]some number[/I] means an arbitrary variable, let's call it x. 1/4 of this is written as: x/4 The word [I]is[/I] means an equation, so we set x/4 equal to 7: [B]x/4 = 7[/B]

7 minus a number all divided by 4
7 minus a number all divided by 4 The phrase [I]a number[/I] means an arbitrary variable, let's call it x. x 7 minus a number 7 - x All divided by 4: [B](7 - x)/4[/B]

7 multiplied by the quantity 7 take away 6
7 multiplied by the quantity 7 take away 6 Take this algebraic expression in pieces: [LIST] [*]7 take away 6: 7 - 6 [*]7 multiplied by the quantity: [B]7(7 - 6)[/B] [/LIST] This is our algebraic expression. If you need to evaluate this expression, we [URL='https://www.mathcelebrity.com/order-of-operations-calculator.php?num=7%287-6%29&pl=Perform+Order+of+Operations']type it in the search engine[/URL] and we get; [B]7[/B]

7 out of every 30 students ride their bikes to school. There are 720 students. How many ride their b
7 out of every 30 students ride their bikes to school. There are 720 students. How many ride their bikes Set up a proportion of students who ride their bike to total students where r is the number of students who ride their bikes: 7/30 = r/720 To solve this proportion for r, we [URL='https://www.mathcelebrity.com/prop.php?num1=7&num2=r&den1=30&den2=720&propsign=%3D&pl=Calculate+missing+proportion+value']type it in our calculation engine and we get:[/URL] r = [B]168[/B]

7 plus the quantity of 9 increased by a number
7 plus the quantity of 9 increased by a number The phrase [I]a number[/I] means an arbitrary variable, let's call it x: x 9 increased by a number means we add 9 to x 9 + x 7 plus this quantity means we add (9 + x) to 7 [B]7 + (9 + x)[/B]

7 plus the quotient of 12 and x is 2
7 plus the quotient of 12 and x is 2 The quotient of 12 and x: 12/x 7 plus the quotient of 12 and x: 7 + 12/x The word [I]is[/I] means equal to, so we set 7 + 12/x equal to 2: [B]7 + 12/x = 2[/B]

7 salads, 10 main dish, 6 dessert
7 salads, 10 main dish, 6 dessert Using the Fundamental Rule of Counting, we have: 7 * 10 * 6 = 420 possible meals

7 subtracted from x cubed
7 subtracted from x cubed x cubed means x raised to the 3rd power x^3 7 subtracted from this [B]x^3 - 7[/B]

7 times a number and 2 is equal to 4 times a number decreased by 8
7 times a number and 2 is equal to 4 times a number decreased by 8 The phrase [I]a number[/I] means an arbitrary variable, let's call it x. x 7 times a number: 7x and 2 means we add 2: 7x + 2 4 times a number 4x decreased by 8 means we subtract 8: 4x - 8 The phrase [I]is equal to[/I] means an equation, so we set 7x + 2 equal to 4x - 8: [B]7x + 2 = 4x - 8[/B]

7 times a number increased by 4 times the number
7 times a number increased by 4 times the number Let [I]a number[/I] and [I]the number[/I] be an arbitrary variable. Let's call it x. We have an algebraic expression. Let's take it in pieces: [LIST] [*]7 times a number: 7x [*]4 times the number: 4x [*]The phrase [I]increased by[/I] means we add 4x to 7x: [*]7x + 4x [*]Simplifying, we get: (7 + 4)x [*][B]11x[/B] [/LIST]

7 times a number is the same as 12 more than 3 times a number
7 times a number is the same as 12 more than 3 times a number The phrase [I]a number[/I] means an arbitrary variable, let's call it x. [B][U]Algebraic Expression 1:[/U][/B] 7 times a number means we multiply 7 by x: 7x [B][U]Algebraic Expression 2:[/U][/B] 3 times a number means we multiply 3 by x: 3x 12 more than 3 times a number means we add 12 to 3x: 3x + 12 The phrase [I]is the same as[/I] means an equation, so we set 7x equal to 3x + 12 [B]7x = 3x + 12[/B] <-- Algebraic Expression

7 times a positive number n is decreased by 3, it is less than 25
7 times a positive number n is decreased by 3, it is less than 25 7 times a positive number n: 7n Decreased by 3: 7n - 3 The phrase [I]it is less than [/I]means an inequality. So we relate 7n - 3 less than 25 using the < sign to get our algebraic expression of: [B]7n - 3 < 25[/B]

7 times the cube of the sum of x and 8
7 times the cube of the sum of x and 8 Take this algebraic expression in 3 parts: [LIST=1] [*]The sum of x and 8 means we add 8 to x: x + 8 [*]The cube of this sum means we raise the sum to the 3rd power: (x + 8)^3 [*]7 times this cubed sum means we multiply (x + 8)^3 by 7: [/LIST] [B]7(x + 8)^3[/B]

7 times the number of lions plus 4 times the number of tigers
7 times the number of lions plus 4 times the number of tigers Let the number of lions be l Let the number of tigers be t We have an algebraic expression of: [B]7l + 4t[/B]

7 times the quantity of 3 times a number reduced by 10
7 times the quantity of 3 times a number reduced by 10 The phrase [I]a number[/I] means an arbitrary variable. Let's call it x. x 3 times a number: 3x Reduced by 10 means we subtract 10: 3x - 10 7 times this quantity: [B]7(3x - 10)[/B]

7 times the quantity of a plus b
7 times the quantity of a plus b The quantity of a plus b: a + b 7 times this quantity: [B]7(a + b)[/B]

7 ½ decimal
7 ½ decimal Convert the mixed fractions 7 & 1/2 to an improper fraction [URL='https://www.mathcelebrity.com/fraction.php?frac1=7%261%2F2&frac2=3%2F8&pl=Simplify']7 1/2[/URL] = 15/2 Convert the improper fraction to a decimal: [URL='https://www.mathcelebrity.com/perc.php?num=15&den=2&pcheck=1&num1=16&pct1=80&pct2=70&den1=80&idpct1=10&hltype=1&idpct2=90&pct=82&decimal=+65.236&astart=12&aend=20&wp1=20&wp2=30&pl=Calculate']15/2[/URL] = [B]7.5[/B]

7, 10, 15, 22 What is the next number in the sequence? What is the 500th term?
7, 10, 15, 22 What is the next number in the sequence? What is the 500th term? We see that: 1^2 + 6 = 7 2^2 + 6 = 10 3^3 + 6 = 15 4^2 + 6 = 22 We build our function as f(n) = n^2 + 6 Next term in the sequence is f(5) f(5) = 5^2 + 6 f(5) = 25 + 6 f(5) = [B]31 [/B] Calculate the 500th term: f(500) = 500^2 + 6 f(500) = 250,000 + 6 f(500) = [B]250,006[/B]

7.5 degrees C to -5.21 degrees C. Kayla and Jason were discussing the changes in temperature. Kayla
7.5 degrees C to -5.21 degrees C. Kayla and Jason were discussing the changes in temperature. Kayla said, "Wow, I can't believe the temperature fell 12.71 degrees C." Jason said, "What are you talking about? The temperature only fell 2.29 degrees C." Who is correct [B]Kayla[/B] is correct. The temperature change TC is: TC = High Temp - Low Temp TC = 7.5 - (-5.21) TC = 7.5 + 5.21 ([I]Since a minus negative is positive)[/I] TC = 12.71

7/4 yards represents 4/5 of the full length of rope. Find the length of the entire jump rope.
7/4 yards represents 4/5 of the full length of rope. Find the length of the entire jump rope. Let the entire jump rope length be l. We're given the proportion: 4l/5 = 7/4 We type this in our search engine and our [URL='https://www.mathcelebrity.com/prop.php?num1=4l&num2=7&den1=5&den2=4&propsign=%3D&pl=Calculate+missing+proportion+value']proportion calculator[/URL] solves for l to get: l = [B]2.1875 yards[/B]

7100 dollars is placed in an account with an annual interest rate of 7.75%. How much will be in the
7100 dollars is placed in an account with an annual interest rate of 7.75%. How much will be in the account after 30 years, to the nearest cent? Using our [URL='https://www.mathcelebrity.com/compoundint.php?bal=7100&nval=30&int=7.75&pl=Annually']balance with compound interest calculator[/URL], we get: 66,646.40

7100 dollars is placed in an account with an interest of 7.75%. How much will be in the account afte
7100 dollars is placed in an account with an interest of 7.75%. How much will be in the account after 30 years to the nearest cent? Using our [URL='https://www.mathcelebrity.com/compoundint.php?bal=7100&nval=30&int=7.75&pl=Annually']balance with interest calculator[/URL], we get: [B]$66,646.40[/B]

72 pounds and increases by 3.9 pounds per month
72 pounds and increases by 3.9 pounds per month Let m be the number of months. We write the algebraic expression below: [B]3.9m + 72[/B]

75% of a ship’s cargo was destroyed by an on-board fire. The captain of the ship sold the remaining
75% of a ship’s cargo was destroyed by an on-board fire. The captain of the ship sold the remaining cargo, which was slightly damaged, for 25% of its real value and received $1400. What was the value of the cargo before the fire? (Do not include the $ sign or commas in the answer) So 25% of the cargo is left. This was sold at 25% of value. Let the starting value be s: We have 0.25 * 0.25 * s = 1400 0.0625s = 1400 To solve for s, we [URL='https://www.mathcelebrity.com/1unk.php?num=0.0625s%3D1400&pl=Solve']type this equation into our search engine[/URL] and we get: s = [B]22400[/B]

756.218 which digit is in the thousands place
756.218 which digit is in the thousands place Starting from the right of the decimal, we have the tenths place, hundreds place, thousands place. So [B]8[/B] is the thousands place

76 decreased by twice a number. Use the variable n to represent the unknown number
76 decreased by twice a number. Use the variable n to represent the unknown number. Twice a number (n) means we multiply the unknown number n by 2: 2n 76 decreased by twice a number means we subtract 2n from 76 using the (-) operator [B]76 - 2n[/B]

76 subtracted from p is equal to the total of g and 227
76 subtracted from p is equal to the total of g and 227 We've got two algebraic expressions. Take them in pieces: Part 1: 76 subtracted from p We subtract 76 from the variable p p - 76 Part 2: The total of g and 227 The total means a sum, so we add 227 to g g + 227 Now the last piece, the phrase [I]is equal to[/I] means an equation. So we set both algebraic expressions equal to each other: [B]p - 76 = g + 227[/B]

7700 dollars is placed in an account with an annual interest rate of 5.75%. How much will be in the
7700 dollars is placed in an account with an annual interest rate of 5.75%. How much will be in the [URL='https://www.mathcelebrity.com/compoundint.php?bal=7700&nval=5.75&int=24&pl=Annually']Using our compound balance interest calculator[/URL], we get: [B]$26,525.61[/B]

7700 dollars is placed in an account with an annual interest rate of 5.75%. How much will be in the
7700 dollars is placed in an account with an annual interest rate of 5.75%. How much will be in the account after 24 years, to the nearest cent? [URL='https://www.mathcelebrity.com/compoundint.php?bal=7700&nval=24&int=5.75&pl=Annually']Using our balance with interest calculator[/URL], we get: [B]$29,459.12[/B]

78 times the quantity p minus 3
78 times the quantity p minus 3 The quantity p minus 3: p - 3 78 times this quantity: [B]78(p - 3)[/B]

7900 dollars is placed in an account with an annual interest rate of 5.5%. How much will be in the a
7900 dollars is placed in an account with an annual interest rate of 5.5%. How much will be in the account after 11 years, to the nearest cent? Using our [URL='https://www.mathcelebrity.com/compoundint.php?bal=7900&nval=11&int=5.5&pl=Annually']compound interest calculator[/URL], we get: [B]14,236.53[/B]

7n + 4 + n - 5 = 63
7n + 4 + n - 5 = 63 Solve for [I]n[/I] in the equation 7n + 4 + n - 5 = 63 [SIZE=5][B]Step 1: Group the n terms on the left hand side:[/B][/SIZE] (7 + 1)n = 8n [SIZE=5][B]Step 2: Group the constant terms on the left hand side:[/B][/SIZE] 4 - 5 = -1 [SIZE=5][B]Step 3: Form modified equation[/B][/SIZE] 8n - 1 = + 63 [SIZE=5][B]Step 4: Group constants:[/B][/SIZE] We need to group our constants -1 and 63. To do that, we add 1 to both sides 8n - 1 + 1 = 63 + 1 [SIZE=5][B]Step 5: Cancel 1 on the left side:[/B][/SIZE] 8n = 64 [SIZE=5][B]Step 6: Divide each side of the equation by 8[/B][/SIZE] 8n/8 = 64/8 n = [B]8[/B]

8 bags weigh 14 pounds. how much do 20 bags weigh
8 bags weigh 14 pounds. how much do 20 bags weigh Set up a proportion of bags to pounds where p is the number of pounds for 20 bags: 8/14 = 20/p We [URL='https://www.mathcelebrity.com/prop.php?num1=8&num2=20&den1=14&den2=p&propsign=%3D&pl=Calculate+missing+proportion+value']type this proportion in our calculator[/URL] and we get: p = [B]35[/B]

8 bricklayers can build a wall in 10 days. How long would it take 5 bricklayers to build?
8 bricklayers can build a wall in 10 days. How long would it take 5 bricklayers to build? Set up a proportion of bricklayers to building time where t is the amount of time it takes 5 bricklayers to build a wall: 8/10 = 5/t To solve this proportion for t, we [URL='https://www.mathcelebrity.com/prop.php?num1=8&num2=5&den1=10&den2=t&propsign=%3D&pl=Calculate+missing+proportion+value']type it in our search engine[/URL] and we get: t = [B]6.25 days[/B]

8 increased by the product of a number and 7 is greater than or equal to -18
Take this in parts: First, the phrase, "a number" means we pick an arbitrary variable, let's call it x. The product of a number and 7 is 7x. 8 increased by the product of 7x means we add them together. 7x + 8 Finally that entire expression is greater than [U]or equal to[/U] -18 [B]7x + 8 >=-18[/B]

8 is subtracted from the square of x
8 is subtracted from the square of x Take this algebraic expression in parts: [LIST] [*]The square of x means we raise x to the power of 2: x^2 [*]8 subtracted from the square of x is found by subtracting 8 from x^2 [/LIST] [B]x^2 - 8[/B]

8 less than x is 31
8 less than X means X - 8. The word is means equal to. So we have: X - 8 = 31

8 less thantriplethedifferenceof2xand6
8 less than triple the difference of 2x and 6 The [I]difference[/I] of 2x and 6 means we [B]subtract[/B] 6 from 2x 2x - 6 [I]Triple[/I] this difference means we [B]multiply by 3[/B] 3(2x - 6) 8 [I]less[/I] means we [B]subtract 8 from this expression 3(2x - 6) - 8[/B]

8 more than the product of x and 2 equals 4
8 more than the product of x and 2 equals 4 The product of x and 2: 2x 8 more than this, means we add 8: 2x + 8 Set this equal to 4: [URL='https://www.mathcelebrity.com/1unk.php?num=2x%2B8%3D4&pl=Solve']2x + 8 = 4[/URL] <-- Algebraic expression to solve for x, type this into the search engine and we get [B]x = -2[/B].

8 more than twice a number is less than 6 more than the number
8 more than twice a number is less than 6 more than the number. This is an algebraic expression, let's take it in pieces... The phrase [I]a number[/I] means an arbitrary variable, let's call it x. 8 more than twice a number: Twice a number means multiply x by 2: 2x Then add 8: 2x + 8 6 more than the number, means we add 6 to x x + 6 The phrase [I]is less than[/I] means an inequality, where we set 2x + 8 less than x + 6 [B]2x + 8 < x + 6[/B]

8 sweets are shared among 4 pupils. how many does each pupil get
8 sweets divided by 4 pupils = 2 sweets per pupil. We can also write this as a proportion: 8 sweets x sweets ---------- = ------------ 4 pupils 1 pupil Express this as 8/4 = x/1. [URL='http://www.mathcelebrity.com/prop.php?num1=8&num2=x&den1=4&den2=1&propsign=%3D&pl=Calculate+missing+proportion+value']Enter that into the search engine[/URL] x = 2

8 taken away from y
8 taken away from y This is an algebraic expression. The phrase [I]taken away[/I] means we subtract 8 from y: [B]y - 8[/B]

8 times 4 plus m squared
8 times 4 plus m squared m squared means we raise m to the power of 2 m^2 4 plus m squared: 4 + m^2 8 times 4 plus m squared [B]8(4 + m^2)[/B]

8 times the difference of 5y and 3
8 times the difference of 5y and 3 The difference of 5y and 3 means we subtract 3 from 5y: 5y - 3 8 times the difference means we multiply (5y - 3) by 8: [B]8(5y - 3)[/B]

8 times the difference of a number and 2 is the same as 3 times the sum of the number and 3. What is
8 times the difference of a number and 2 is the same as 3 times the sum of the number and 3. What is the number? Let the number be n. We're given two expressions: [LIST=1] [*]8(n - 2) [I]difference means we subtract[/I] [*]3(n + 3) [I]sum means we add[/I] [/LIST] The phrase [I]is the same as[/I] mean an equation. So we set the first expression equal to the second expression: 8(n - 2) = 3(n + 3) To solve this equation for n, we [URL='https://www.mathcelebrity.com/1unk.php?num=8%28n-2%29%3D3%28n%2B3%29&pl=Solve']type it in our search engine[/URL] and we see that: n =[B] 5[/B]

8 times the quantity x plus y
8 times the quantity x plus y The quantity x plus y: x + y 8 times the quantity: [B]8(x + y)[/B]

8 times the sum of 5 times a number and 9
8 times the sum of 5 times a number and 9 Take this algebraic expression in parts: The phrase [I]a number[/I] means an arbitrary variable, let's call it x. 5 times a number means: 5x The sum of this and 9 means we add 9 to 5x: 5x + 9 Now we multiply 8 times this sum: [B]8(5x + 9)[/B]

8 to the power of 8 divided by 8
8 to the power of 8 divided by 8 8 to the power of 8 8^8 Divided by 8 [B]8^8/8[/B]

8 to the power of x over 2 to the power of y
8 to the power of x over 2 to the power of y Step 1: 8 to the power of x means we take 8 and raise it to an exponent of x: 8^x Step 2: 2 to the power of y means we take 2 and raise it to an exponent of y: 2^y Step 3: The word [I]over[/I] means a quotient, also known as divided by, so we have: [B]8^x/2^y [MEDIA=youtube]SPQKOt5EoqA[/MEDIA][/B]

8 years from now a girls age will be 5 times her present age whats is the girls age now
8 years from now a girls age will be 5 times her present age whats is the girls age now. Let the girl's age now be a. We're given: a + 8 = 5a [URL='https://www.mathcelebrity.com/1unk.php?num=a%2B8%3D5a&pl=Solve']Typing this equation into our search engine[/URL], we get: [B]a = 2[/B]

8,11,14,17,20 What is the next number? What is the 150th term?
8,11,14,17,20 What is the next number? What is the 150th term? We're adding by 3 to the last number in the sequence, so we have the next number as: 20 + 3 = [B]23 [/B] For the nth term, we have a formula of this: 3n + 5 3(1) + 5 = 8 3(2) + 5 = 11 3(3) + 5 = 14 With n = 150, we have: 3(150) + 5 = 450 + 5 = [B]455[/B]

80 people 40% were women 12 were children. How many men?
80 people 40% were women 12 were children. How many men? Calculate the number of women: 40% of 80 is 32. 12 were children, so the women and children = 32 + 12 = 44. Which means the men = 80 - 44 = [B]36[/B]

8300 dollars is placed in an account with an annual interest rate of 6.5%. How much will be in the a
8300 dollars is placed in an account with an annual interest rate of 6.5%. How much will be in the account after 14 years, to the nearest cent? [URL='https://www.mathcelebrity.com/compoundint.php?bal=8300&nval=14&int=6.5&pl=Annually']Using our balance with interest calculator[/URL], we get: [B]$20,043.46[/B]

85 bags of nuts are to be divided among 18 friends. Each bag contains 15 nuts. How many nuts will ea
85 bags of nuts are to be divided among 18 friends. Each bag contains 15 nuts. How many nuts will each friend get? [B][U]Calculate the total nuts:[/U][/B] Total Nuts = Total Bags * Nuts Per Bag Total Nuts = 85 * 15 Total Nuts = 1,275 [B][U]Figure out how many nuts each person gets:[/U][/B] Nuts per person = Total Nuts / Friends Nuts per person = 1,275 / 18 Nuts per person = [B]70.83[/B]

9 divided by the quantity x plus y
9 divided by the quantity x plus y The quantity x plus y: x + y 9 divided by this quantity: [B]9/(x + y)[/B]

9 divided by the sum of x and 4 is equal to 6 divided by x minus 4
9 divided by the sum of x and 4 is equal to 6 divided by x minus 4. Build our two algebraic expressions first: 9 divided by the sum of x and 4 9/(x + 4) 6 divided by x minus 4 6/(x - 4) The phrase [I]is equal to[/I] means and equation, so we set the algebraic expressions equal to each other: [B]9/(x + 4) = 6/(x - 4) <-- This is our algebraic expression[/B] [B][/B] If the problem asks you to solve for x, we cross multiply: 9(x - 4) = 6(x + 4) To solve for x, we [URL='https://www.mathcelebrity.com/1unk.php?num=9%28x-4%29%3D6%28x%2B4%29&pl=Solve']type this equation into our search engine[/URL] and we get: x = [B]20[/B]

9 friends brought 178 tickets How many more ticket would they have to buy for all of them could have
9 friends brought 178 tickets How many more ticket would they have to buy for all of them could have the same amount? If we take [URL='https://www.mathcelebrity.com/modulus.php?num=178mod9&pl=Calculate+Modulus']178 mod 9[/URL] to find the remainder, we get 7. If we buy the 7 more (remainder) tickets, we have: 9 friends - 7 remainder = 2 tickets To prove our work, we add the 2 tickets + 178 tickets = 180 tickets 180 tickets / 9 friends = 20 tickets per friends So our answer is [B]2 tickets[/B]

9 friends were paid $385 to clean up the local lake. How much does each friend receive
9 friends were paid $385 to clean up the local lake. How much does each friend receive Each friend gets: Total Payment / Number of friends $385/9 [B]$42.78[/B]

9 is one-third of a number x
9 is one-third of a number x A number x can be written as x x one-third of a number x means we multiply x by 1/3: x/3 The phrase [I]is[/I] means an equation, so we set 9 equal to x/3 to get our final algebraic expression of: [B]x/3 = 9[/B] If the problem asks you to solve for x, you [URL='https://www.mathcelebrity.com/prop.php?num1=x&num2=9&den1=3&den2=1&propsign=%3D&pl=Calculate+missing+proportion+value']type this algebraic expression into our search engine[/URL] and you get: [B]x = 27[/B]

9 is the sum of 7 and twice a number
9 is the sum of 7 and twice a number The phrase [I]a number[/I] means an arbitrary variable, let's call it x. Twice a number means we multiply x by 2: 2x The sum of 7 and twice a number 7 + 2x The word [I]is[/I] mean equal to, so we set 7 + 2x equal to 9: [B]7 + 2x = 9[/B]

9 is the sum of thrice x and y
9 is the sum of thrice x and y Thrice x means multiply x by 3: 3x Sum of this and y: 3x + y Now we set this expression equal to 9: [B]3x + y = 9[/B]

9 less than 5 times a number is 3 more than 2x
9 less than 5 times a number is 3 more than 2x The phrase [I]a number[/I] means an arbitrary variable, let's call it x: x 5 times a number means we multiply x by 5: 5x 9 less than 5x means we subtract 9 from 5x: 5x - 9 3 more than 2x means we add 3 to 2x: 2x + 3 The word [I]is[/I] means an equation, so we set 5x - 9 equal to 2x + 3: [B]5x - 9 = 2x + 3 <-- This is our algebraic expression[/B] [B][/B] If you want to solve for x, [URL='https://www.mathcelebrity.com/1unk.php?num=5x-9%3D2x%2B3&pl=Solve']type this equation into the search engine[/URL], and we get: x = [B]4[/B]

9 less than the product of the profit, p, and 6
9 less than the product of the profit, p, and 6 [U]The product of the profit p and 6:[/U] 6p [U]9 less than the product:[/U] [B]6p - 9[/B]

9 less than twice x is twice y
9 less than twice x is twice y Twice x means we multiply x by 2: 2x 9 less than Twice x means we subtract 9 from 2x 2x - 9 Twice y means we multiply y by 2: 2y The word [I]is[/I] means equal to, so we set 2x - 9 equal to 2y: [B]2x - 9 = 2y[/B]

9 rulers cost the same as 11 erasers. One eraser cost 0.09 cents. What is the cost of 1 ruler
9 rulers cost the same as 11 erasers. One eraser cost 0.09 cents. What is the cost of 1 ruler Let the cost of a ruler be r. We're given: 9r = 11(0.09) 9r = 0.99 Divide each side by 9 and we get: r = [B]0.11[/B]

9 subtracted from the product of 3 and a number is greater than or equal to 16
9 subtracted from the product of 3 and a number is greater than or equal to 16 [LIST=1] [*]The phrase [I]a number[/I] means an arbitrary variable, let's call it x [*]The product of 3 and a number means we multiply 3 times x: 3x [*]9 subtracted from the product: 3x - 9 [*]The phrase is greater than or equal to means an inequality. So we set up an inequality with >= for the greater than or equal to sign in relation to 3x - 9 and 16 [/LIST] Our algebraic expression (inequality) becomes: [B]3x - 19 >= 16[/B]

9 times a number is that number minus 10
9 times a number is that number minus 10 The phrase [I]a number[/I] means we define a random/arbitrary variable, let's call it x: x 9 times a number means we multiply x by 9: 9x The phrase [I]that number[/I] refers back to the original arbitrary variable we defined above, which is x: x That number minus 10 means we subtract 10 from x: x - 10 The word [I]is[/I] means equal to, so we set 9x equal to x - 10 [B]9x = x - 10[/B]

9 times a number is that number minus 3
9 times a number is that number minus 3 Let [I]a number[/I] be an arbitrary variable, let's call it x. We're given: 9 times a number is 9x The number minus 3 is x - 3 The word [I]is[/I] means an equation, so we set 9x equal to x - 3 to get our [I]algebraic expression[/I]: [B]9x = x - 3[/B] To solve for x, we type this equation into our search engine and we get: x = [B]-0.375 or -3/8[/B]

9 times x is twice the sum of x and 5
9 times x is twice the sum of x and 5 9 times x: 9x the sum of x and 5 x + 5 twice the sum of x and 5 2(x + 5) The phrase [I]is[/I] means equal to, so we set 9x equal to 2(x + 5) [B]9x = 2(x + 5)[/B]

9 workers were hired to harvest potatoes from a field. each is given a plot which is 11*7 feet in si
9 workers were hired to harvest potatoes from a field. each is given a plot which is 11*7 feet in size. what is the total area of the field The area of each plot is 11*7 = 77 With 9 workers, the total area of the field is: 9 * 77 = [B]693 sq feet[/B]

9, 3, 1, 1/3, 1/9 What is the next number in this sequence? What is the function machine for this se
9, 3, 1, 1/3, 1/9 What is the next number in this sequence? What is the function machine for this sequence? We see the following pattern in this sequence: 9 = 9/3^0 3 = 9/3^1 1 = 9/3^2 1/3 = 9/3^3 1/9 = 9/3^4 Our function machine formula is: [B]f(n) = 9/3^(n - 1) [/B] Next term is the 6th term: f(6) = 9/3^(6 - 1) f(6) = 9/3^5 f(6) = 9/243 f(6) = [B]1/27[/B]

90% of the day in September were sunny. How many day were sunny?
90% of the day in September were sunny. How many day were sunny? September has 30 days, so we have: Sunny Days = 90% * 30 [URL='https://www.mathcelebrity.com/perc.php?num=+5&den=+8&num1=+16&pct1=+80&pct2=90&den1=30&pcheck=3&pct=+82&decimal=+65.236&astart=+12&aend=+20&wp1=20&wp2=30&pl=Calculate']Sunny Days[/URL] = [B]27[/B]

9000 dollars is placed in an account with an annual interest rate of 8%. How much will be in the acc
9000 dollars is placed in an account with an annual interest rate of 8%. How much will be in the account after 17 years, to the nearest cent? Using our [URL='http://www.mathcelebrity.com/compoundint.php?bal=9000&nval=17&int=8&pl=Annually']compound interest accumulated balance calculator[/URL], we get: [B]$33,300.16[/B]

963 animals on a farm, 159 sheep and 406 cows and pigs. How many are pigs?
963 animals on a farm, 159 sheep and 406 cows and pigs. How many are pigs? Set up equation to represent the total animals on the farm Total Animals = Cows + Pigs + Sheep Now plug in what is given 963 = 406 + Pigs + 159 Simplify: Pigs + 565 = 963 Subtract 565 from each side [B]Pigs = 398[/B]

993 cold drinks bottles are to be placed in crates. Each crate can hold 9 bottles. How many crates w
993 cold drinks bottles are to be placed in crates. Each crate can hold 9 bottles. How many crates would be needed and how many bottles will remain? Let c equal the number of crates 9 bottles per crate * c = 993 9c = 993 Solve for [I]c[/I] in the equation 9c = 993 [SIZE=5][B]Step 1: Divide each side of the equation by 9[/B][/SIZE] 9c /9 = 993/9 c = 110.33333333333 Since we can't have fractional crates, we round up 1 to the next full crate c = [B]111[/B]

9x is subtracted from the sum of 3y and 4
9x is subtracted from the sum of 3y and 4 The sum of 3y and 4 3y + 4 9x is subtracted from the sum of 3y and 4 [B]3y + 4 - 9x[/B]

A $1,000 deposit is made at a bank that pays 12% compounded monthly. How much will you have in your
A $1,000 deposit is made at a bank that pays 12% compounded monthly. How much will you have in your account at the end of 10 years? Using our [URL='https://www.mathcelebrity.com/compoundint.php?bal=10000&nval=120&int=12&pl=Monthly']compound interest calculator[/URL] with time = 10 years * 12 months per year = 120, we get: [B]33,003.87[/B]

A $1,000 investment takes a 10% loss each year. What will be the value 3 years?
A $1,000 investment takes a 10% loss each year. What will be the value 3 years? 10% is 0.1. Our Balance function B(y) where y is the number of years since the start is: B(y) = 1000(1 - 0.1)^y B(y) = 1000(0.9)^y We want to know B(3): B(3) = 1000(0.9)^3 B(3) = 1000(0.729) B(3) = [B]729[/B]

A $480 TV was put on sale for 30% off. It didn't sell, so the price was lowered an additional percen
A $480 TV was put on sale for 30% off. It didn't sell, so the price was lowered an additional percent off the sale price, making the new sale price $285.60. What was the second percent discount that was given? Let the second discount be d. We're given: 480 * (1 - 0.3)(1 - d) = 285.60 480(0.7)(1 - d) = 285.60 336(1 - d) = 285.60 336 - 336d = 285.60 [URL='https://www.mathcelebrity.com/1unk.php?num=336-336d%3D285.60&pl=Solve']Type this equation into our search engine[/URL] to solve for d and we get: d = [B]0.15 or 15%[/B]

A $650 television costs $702 after sales tax is figured in. What is the sales tax percentage?
A $650 television costs $702 after sales tax is figured in. What is the sales tax percentage? [U]Calculate Sales Tax Amount:[/U] Sales Tax Amount = Total Bill - Original Cost Sales Tax Amount = 702 - 650 Sales Tax Amount = 52 [U]Calculate Sales Tax Percentage:[/U] Sales Tax Percentage = 100% * Sales Tax Amount / Original Cost Sales Tax Percentage = 100% * 52 / 650 Sales Tax Percentage = 100% * 0.08 Sales Tax Percentage = [B]8%[/B]

A $654,000 property is depreciated for tax purposes by its owner with the straight-line depreciation
A $654,000 property is depreciated for tax purposes by its owner with the straight-line depreciation method. The value of the building, y, after x months of use is given by y = 654,000 ? 1800x dollars. After how many months will the value of the building be $409,200? We want to know x for the equation: 654000 - 1800x = 409200 To solve this equation for x, we [URL='https://www.mathcelebrity.com/1unk.php?num=654000-1800x%3D409200&pl=Solve']type it in our math engine[/URL] and we get: x = [B]136 months[/B]

A $675 stereo receiver loses value at a rate of about $18 per month The equation y = 675 - 18x repre
A $675 stereo receiver loses value at a rate of about $18 per month The equation y = 675 - 18x represents the value of the receiver after x months. Identify and interpret the x- and y-intercepts. Explain how you can use the intercepts to help you graph the equation y = 675 - 18x The y-intercept is found when x is 0: y = 675 - 18(0) y = 675 - 0 y = 675 The x-intercept is found when y is 0: 0 = 675 - 18x [URL='https://www.mathcelebrity.com/1unk.php?num=675-18x%3D0&pl=Solve']Typing this equation into our search engine[/URL], we get: x = 37.5

A $750 television is on sale for 30% off. There is a 7% sales tax on the television. How much do you
A $750 television is on sale for 30% off. There is a 7% sales tax on the television. How much do you pay? 30% off: 750(1 - 0.3) 750(0.7) = 525 Now, add 7% sales tax 525 * (1.07) = [B]561.75[/B]

a +?b +?c =?180 for b
a +?b +?c =?180 for b We have a literal equation. Subtract (a + c) from each side of the equation to isolate b: a + b + c - (a + c) = 180 - (a + c) The (a + c) cancels on the left side, so we have: [B]b = 180 - (a + c)[/B] or, distributing the negative sign: [B]b = 180 - a - c[/B]

A 1.5 inch tall preying mantis will sometimes hold its ground and attempt to fight a person who is 6
A 1.5 inch tall preying mantis will sometimes hold its ground and attempt to fight a person who is 6 feet tall. If a person who is 6 feet tall is engaged in a battle with an animal that was proportionally as tall as the person is to the preying mantis, how tall would the animal be? In terms of inches, [URL='https://www.mathcelebrity.com/linearcon.php?quant=6&pl=Calculate&type=foot']6 feet = 72 inches[/URL] Set up a proportion of height of smaller creature to larger creature where h is the heigh of the animal 1.5/72 = 72/h Using our [URL='https://www.mathcelebrity.com/proportion-calculator.php?num1=1.5&num2=72&den1=72&den2=h&propsign=%3D&pl=Calculate+missing+proportion+value']proportion calculator[/URL], we get: h = 3456 inches In terms of feet, we have [URL='https://www.mathcelebrity.com/linearcon.php?quant=3456&pl=Calculate&type=inch']3456 inches[/URL] = [B]288 feet[/B]

A 10 ounce serving of energy drink contains about 190 mg of caffeine approximately how much caffeine
A 10 ounce serving of energy drink contains about 190 mg of caffeine approximately how much caffeine is in a 25 ounce of energy drink? Set up a proportion of caffeine to ounces where c is the amount of caffeine in a 25 ounce drink: 190/10 = c/25 [URL='https://www.mathcelebrity.com/prop.php?num1=190&num2=c&den1=10&den2=25&propsign=%3D&pl=Calculate+missing+proportion+value']Type this proportion into the search engine[/URL] and we get: c = [B]475[/B]

A 100 point test contains a total of 20 questions. The multiple choice questions are worth 3 points
A 100 point test contains a total of 20 questions. The multiple choice questions are worth 3 points each and short response questions are worth 8 points each. Write a system of linear equations that represents this situation Assumptions: [LIST] [*]Let m be the number of multiple choice questions [*]Let s be the number of short response questions [/LIST] Since total points = points per problem * number of problems, we're given 2 equations: [LIST=1] [*][B]m + s = 20[/B] [*][B]3m + 8s = 100[/B] [/LIST] We can solve this system of equations 3 ways: [LIST] [*][URL='https://www.mathcelebrity.com/simultaneous-equations.php?term1=m+%2B+s+%3D+20&term2=3m+%2B+8s+%3D+100&pl=Substitution']Substitution Method[/URL] [*][URL='https://www.mathcelebrity.com/simultaneous-equations.php?term1=m+%2B+s+%3D+20&term2=3m+%2B+8s+%3D+100&pl=Elimination']Elimination Method[/URL] [*][URL='https://www.mathcelebrity.com/simultaneous-equations.php?term1=m+%2B+s+%3D+20&term2=3m+%2B+8s+%3D+100&pl=Cramers+Method']Cramer's Rule[/URL] [/LIST] No matter which method we choose, we get: [B]m = 12, s = 8[/B]

A 12 feet ladder leans against the side of a house. The bottom of the ladder is 9 feet from the side
A 12 feet ladder leans against the side of a house. The bottom of the ladder is 9 feet from the side of the house. How high is the top of the ladder from the ground? If necessary, round your answer to the nearest tenth. We have a right triangle, where 12 is the hypotenuse. [URL='https://www.mathcelebrity.com/pythag.php?side1input=&side2input=9&hypinput=12&pl=Solve+Missing+Side']Using our right triangle calculator[/URL], we get: side = [B]7.9[/B]

a 12 sided die is rolled find the probability of rolling a number greater than 7
a 12 sided die is rolled find the probability of rolling a number greater than 7 We assume this is a fair die, not loaded. This means each side 1-12 has an equal probability of 1/12 of being rolled. The problem asks, P(Roll > 7) Greater than 7 means our sample space is {8, 9, 10, 11, 12} If each of these 5 faces have an equal probability of being rolled, then we have: P(Roll > 7) = P(Roll = 8) + P(Roll = 9) + P(Roll = 10) + P(Roll = 11) + P(Roll = 12) P(Roll > 7) = 1/12 + 1/12 + 1/12 + 1/12 + 1/12 P(Roll > 7) =[B] 5/12[/B]

A 12% acid solution is made by mixing 8% and 20% solutions. If the 450 ml of the 12% solution is req
A 12% acid solution is made by mixing 8% and 20% solutions. If the 450 ml of the 12% solution is required, how much of each solution is required? Component Unit Amount 8% Solution: 0.08 * x = 0.08x 20% Solution: 0.2 * y = 0.2y 12% Solution: 0.12 * 450 = 54 We add up the 8% solution and 20% solution to get two equations: [LIST=1] [*]0.08x + 0.2y = 54 [*]x + y = 450 [/LIST] We have a simultaneous set of equations. We can solve it using three methods: [LIST] [*][URL='https://www.mathcelebrity.com/simultaneous-equations.php?term1=0.08x+%2B+0.2y+%3D+54&term2=x+%2B+y+%3D+450&pl=Substitution']Substitution Method[/URL] [*][URL='https://www.mathcelebrity.com/simultaneous-equations.php?term1=0.08x+%2B+0.2y+%3D+54&term2=x+%2B+y+%3D+450&pl=Elimination']Elimination Method[/URL] [*][URL='https://www.mathcelebrity.com/simultaneous-equations.php?term1=0.08x+%2B+0.2y+%3D+54&term2=x+%2B+y+%3D+450&pl=Cramers+Method']Cramer's Rule[/URL] [/LIST] No matter which method we choose, we get the same answer: [LIST] [*][B]x = 300 ml[/B] [*][B]y = 150 ml[/B] [/LIST]

A 12-ounce bottle of shampoo lasts Enrique 16 weeks. How long would you expect an 18-ounce bottle of
A 12-ounce bottle of shampoo lasts Enrique 16 weeks. How long would you expect an 18-ounce bottle of the same brand to last him? Set up a proportion of ounces to weeks were w is the amount of weeks an 18-ounce bottle will last: 12/16 = 18/w We [URL='https://www.mathcelebrity.com/prop.php?num1=12&num2=18&den1=16&den2=w&propsign=%3D&pl=Calculate+missing+proportion+value']type this proportion in our search engine to solve for w[/URL] and we get: w = [B]24[/B]

A 12-sided die is rolled. The set of equally likely outcomes is {1,2,3,4,5,6,7,,8,9,10,11,12}. Find
A 12-sided die is rolled. The set of equally likely outcomes is {1,2,3,4,5,6,7,,8,9,10,11,12}. Find the probability of rolling a number less than 6. We want a {1, 2, 3, 4, 5} P(X < 6) =[B] 5/12[/B]

A 12-sided die is rolled. The set of equally likely outcomes is {1,2,3,4,5,6,7,8,9,10,11,12}. Find t
A 12-sided die is rolled. The set of equally likely outcomes is {1,2,3,4,5,6,7,8,9,10,11,12}. Find the probability of rolling a number less than 6. We have 12 outcomes. Less than 6 means 1, 2, 3, 4, 5. Our probability P(x < 6) is: P(x < 6) = [B]5/12[/B]

A 124-inch length of ribbon is to be cut into three pieces. The longest piece is to be 36 inches lo
A 124-inch length of ribbon is to be cut into three pieces. The longest piece is to be 36 inches longer than the shortest piece, and the third piece is to be half the length of the longest piece. Find the length of each piece of ribbon. [LIST] [*]Let the longest piece be l. [*]The shortest piece is s = l - 36 [*]The third medium piece m = 0.5l [/LIST] We know s + m + l = 124. Now substitute for s and m (l - 36) + 0.5l + l = 124 Combine like terms: 2.5l - 36 = 124 Type [URL='http://www.mathcelebrity.com/1unk.php?num=2.5l-36%3D124&pl=Solve']2.5l - 36 = 124 into our search engine[/URL], we get l = [B]64[/B] Shortest piece s = 64 - 36 = [B]28[/B] Medium piece m = 0.5(64) = [B]32[/B]

A 128 ounce carton of milk states that there are 20 servings. How many ounces are in a serving?
A 128 ounce carton of milk states that there are 20 servings. How many ounces are in a serving? We divide 128 ounces by 20 servings to get ounces per serving: 128 ounces / 20 servings [B]6.4 ounces / serving[/B]

A 13 ft. ladder is leaning against a building 12 ft. up from the ground. How far is the base of the
A 13 ft. ladder is leaning against a building 12 ft. up from the ground. How far is the base of the ladder from the building? This is a classic 5-12-13 pythagorean triple, where the hypotenuse is 13, and the 2 sides are 5 and 12. The building and the ground form a right triangle. [URL='https://www.mathcelebrity.com/pythag.php?side1input=&side2input=12&hypinput=13&pl=Solve+Missing+Side']You can see the proof here[/URL]...

A 13ft ladder leans against the side of a house. The bottom of the ladder is 10ft from the side of t
A 13ft ladder leans against the side of a house. The bottom of the ladder is 10ft from the side of the house. How high is the top of the ladder from the ground? If necessary, round your answer to the nearest tenth. We have a right triangle. Hypotenuse = 13, one leg = 10. We use our [URL='https://www.mathcelebrity.com/pythag.php?side1input=&side2input=10&hypinput=13&pl=Solve+Missing+Side']Pythagorean theorem Calculator to solve for the other leg[/URL]: s = [B]8.3066[/B]

A 15 feet piece of string is cut into two pieces so that the longer piece is 3 feet longer than twic
A 15 feet piece of string is cut into two pieces so that the longer piece is 3 feet longer than twice the shorter piece. If the shorter piece is x feet long, find the lengths of both pieces. If the shorter piece is x, the longer piece is 20 - x We also are given 15 - x = 2x + 3 Add x to each side: 3x + 3 = 15 Using our [URL='http://www.mathcelebrity.com/1unk.php?num=3x%2B3%3D15&pl=Solve']equation calculator[/URL], we get a shorter piece of: [B]x = 4[/B] The longer piece is: 15 - x 15 - 4 [B]11[/B]

A 16 pound roast will feed 24 people. If the largest roast you can buy is 12 pounds. How many people
A 16 pound roast will feed 24 people. If the largest roast you can buy is 12 pounds. How many people can you feed? Set up a proportion of roast pounds to people fed, where p is the number of people fed on a 12 pound roast: 16/24 = 12/p [URL='https://www.mathcelebrity.com/prop.php?num1=16&num2=12&den1=24&den2=p&propsign=%3D&pl=Calculate+missing+proportion+value']Run this through our proportion calculator[/URL] by typing 16/24 = 12/p into our search engine. We get [B]p = 18[/B]. A 12 pound roast will feed 18 people.

A 1975 comic book has appreciated 8% per year and originally sold for $0.26. What will the comic boo
A 1975 comic book has appreciated 8% per year and originally sold for $0.26. What will the comic book be worth in 2020 Calculate the number of years: 2020 - 1975 = 45 Set up the accumulation function A(t) where t is the number of years since 1975: A(t) = 0.26(1.08)^t We want A(45) A(45) = 0.26(1.08)^45 A(45) = 0.26 * 32.9045 A(45) = [B]8.30[/B]

A 2-quart carton of sour cream costs $7.96. What is the price per pint?
A 2-quart carton of sour cream costs $7.96. What is the price per pint? Using our [URL='https://www.mathcelebrity.com/liqm.php?quant=2&pl=Calculate&type=quart']conversion calculator[/URL]: 2 quarts = 4 pints $7.96/4 pints = [B]$1.99 per pint[/B]

A 20 feet piece of string is cut into two pieces so that the longer piece is 5 feet longer than twic
A 20 feet piece of string is cut into two pieces so that the longer piece is 5 feet longer than twice the shorter piece. If the shorter piece is x feet long, find the lengths of both pieces. If the shorter piece is x, the longer piece is 20 - x We also are given 20 - x = 2x + 5 Add x to each side: 3x + 5 = 20 Using our [URL='http://www.mathcelebrity.com/1unk.php?num=3x%2B5%3D20&pl=Solve']equation calculator[/URL], we get a shorter piece of: [B]x = 5 [/B] The longer piece is: 20 - x 20 - 5 [B]15[/B]

A 20 oz. bottle of Pepsi costs $1.60 in 2010 and $1.85 in 2014. What is the percent of increase? Rou
A 20 oz. bottle of Pepsi costs $1.60 in 2010 and $1.85 in 2014. What is the percent of increase? Round to the nearest percent if necessary. We [URL='https://www.mathcelebrity.com/markup.php?p1=1.60&m=&p2=1.85&pl=Calculate']type 1.60 to 1.85 percent increase in our search engine[/URL] and we get: [B]15.63% percent increase[/B]

a 24 sheet of aluminum is to be cut into 2.5 strips. how wide is the remaining wasted piece of alumi
a 24 sheet of aluminum is to be cut into 2.5 strips. how wide is the remaining wasted piece of aluminum Divide 24 by 2.5 to get number of sheets: 24/2.5 = 9.6 So we have 9 full sheets. Which means each strip is [B]0.6 wide[/B]

A 3 gallon bottle of bleach cost $16.32. What is the price per cup?
A 3 gallon bottle of bleach cost $16.32. What is the price per cup? We're given 16.32 / 3 gallons Divide the top and bottom of the fraction by 3 to get the cost per gallon: 16.32/3 = 5.44 gallon Using our [URL='https://www.mathcelebrity.com/liqm.php?quant=1&pl=Calculate&type=gallon']measurement converter[/URL], we see that: 1 gallon = 16 cups So 5.44 /16 cups=[B]$0.34 per cup[/B]

A 3 hour river cruise goes 15 km upstream and then back again. The river has a current of 2 km an ho
A 3 hour river cruise goes 15 km upstream and then back again. The river has a current of 2 km an hour. What is the boat's speed and how long was the upstream journey? [U]Set up the relationship of still water speed and downstream speed[/U] Speed down stream = Speed in still water + speed of the current Speed down stream = x+2 Therefore: Speed upstream =x - 2 Since distance = rate * time, we rearrange to get time = Distance/rate: 15/(x+ 2) + 15 /(x- 2) = 3 Multiply each side by 1/3 and we get: 5/(x + 2) + 5/(x - 2) = 1 Using a common denominator of (x + 2)(x - 2), we get: 5(x - 2)/(x + 2)(x - 2) + 5(x + 2)/(x + 2)(x - 2) (5x - 10 + 5x + 10)/5(x - 2)/(x + 2)(x - 2) 10x = (x+2)(x-2) We multiply through on the right side to get: 10x = x^2 - 4 Subtract 10x from each side: x^2 - 10x - 4 = 0 This is a quadratic equation. To solve it, [URL='https://www.mathcelebrity.com/quadratic.php?num=x%5E2-10x-4%3D0&pl=Solve+Quadratic+Equation&hintnum=+0']we type it in our search engine[/URL] and we get: Speed of the boat in still water =X=5 +- sq. Root of 29 kmph We only want the positive solution: x = 5 + sqrt(29) x = 10.38 [U]Calculate time for upstream journey:[/U] Time for upstream journey = 15/(10.38 - 2) Time for upstream journey = 15/(8.38) Time for upstream journey = [B]1.79[/B] [U]Calculate time for downstream journey:[/U] Time for downstream journey = 15/(10.38 + 2) Time for downstream journey = 15/(12.38) Time for downstream journey = [B]1.21[/B]

A 3-digit security code can use the numbers 0-9. How many possible combinations are there if the num
A 3-digit security code can use the numbers 0-9. How many possible combinations are there if the numbers can be repeated [0-9] * [0-9] * [0-9] 10 * 10 * 10 = [B]1,000 combinations[/B]

A 3-foot stick casts a shadow of 8 feet. If at the same time a tree casts a shadow of 15 feet, how t
A 3-foot stick casts a shadow of 8 feet. If at the same time a tree casts a shadow of 15 feet, how tall is the tree? Set up a proportion of height to shadow length where t is the height of a tree: 3/8 = t/15 Using our [URL='https://www.mathcelebrity.com/proportion-calculator.php?num1=3&num2=t&den1=8&den2=15&propsign=%3D&pl=Calculate+missing+proportion+value']proportion calculator,[/URL] we get: t = [B]5.625[/B]

A 3-gallon bucket of paint costs $87.12. What is the price per quart?
A 3-gallon bucket of paint costs $87.12. What is the price per quart? 3 gallons equals 12 quarts with our [URL='https://www.mathcelebrity.com/liqm.php?quant=3&pl=Calculate&type=gallon#quart']conversion calculator[/URL]. We divide 87.12 for 12 quarts by 12: [URL='https://www.mathcelebrity.com/perc.php?num=87.12&den=12&pcheck=1&num1=16&pct1=80&pct2=70&den1=80&idpct1=10&hltype=1&idpct2=90&pct=82&decimal=+65.236&astart=12&aend=20&wp1=20&wp2=30&pl=Calculate']87.12 / 12[/URL] = [B]$7.26 per quart[/B]

A 5 foot ladder is leaning against a wall. If the bottom of the ladder is 3 feet from the base of th
A 5 foot ladder is leaning against a wall. If the bottom of the ladder is 3 feet from the base of the wall, how high up the wall is the top of the ladder? The answer is [B]4[/B]. Since we have a right triangle, with special ratio 3-4-5. The ladder represents the hypotenuse.

A 50-foot pole and a 70-foot pole are 30 feet apart. If you were to run a line between the tops of t
A 50-foot pole and a 70-foot pole are 30 feet apart. If you were to run a line between the tops of the two poles, what is the minimum length of cord you would need? The difference between the 70 foot and 50 foot pole is: 70 - 50 = 20 foot height difference. So we have a right triangle, with a height of 20, base of 30. We want to know the hypotenuse. Using our [URL='https://www.mathcelebrity.com/pythag.php?side1input=20&side2input=30&hypinput=&pl=Solve+Missing+Side']Pythagorean theorem calculator to solve for hypotenuse[/URL], we get: hypotenuse = [B]36.06 feet[/B]

A 50-pound bowling ball and an 8-pound bowling ball are dropped from a tall building. Which ball wil
A 50-pound bowling ball and an 8-pound bowling ball are dropped from a tall building. Which ball will hit first? [B]They will land at the same time[/B] [B]How fast something falls due to gravity is determined by a number known as the "acceleration of gravity", which is 9.81 m/s^2 at the surface of our Earth. In one second, [I]any object[/I]’s downward velocity will increase by 9.81 m/s because of gravity. This is just the way gravity works - it accelerates everything at exactly the same rate.[/B]

A 5L juice container has 3.6L of juice left. What percentage has been used?
A 5L juice container has 3.6L of juice left. What percentage has been used? What has been used? 5L - 3.6L = 1.4L Now, the [URL='http://www.mathcelebrity.com/perc.php?num=1.4&den=5.4&pcheck=1&num1=16&pct1=80&pct2=70&den1=80&idpct1=10&hltype=1&idpct2=90&pct=82&decimal=+65.236&astart=12&aend=20&wp1=20&wp2=30&pl=Calculate']percentage used[/URL] is 1.4L/5.4L = [B]25.93%[/B]

A 6-sided die is rolled once. What is the probability of rolling a number less than 4?
A 6-sided die is rolled once. What is the probability of rolling a number less than 4? Using our [URL='https://www.mathcelebrity.com/1dice.php?gl=4&pl=4&opdice=1&rolist=+2%2C3%2C4&dby=+2%2C3%2C5&montect=+100']one dice calculator[/URL], we get: P(N < 4) = [B]1/2[/B]

A 6000 seat theater has tickets for sale at $24 and $40. How many tickets should be sold at each pri
A 6000 seat theater has tickets for sale at $24 and $40. How many tickets should be sold at each price for a sellout performance to generate a total revenue of $188,800? Let x be the number of $24 tickets, and y be the number of $40 tickets. We have: [LIST=1] [*]24x + 40y = 188,800 [*]x + y = 6,000 [*]Rearrange (2) to solve for x: x = 6000 - y [*]Plug in (3) to (1): [/LIST] 24(6000 - y) + 40y = 188800 144,000 - 24y + 40y = 188,800 16y + 144,000 = 188,800 Subtract 144,000 from each side: 16y = 44,800 Divide each side by 16 y = 2,800 ($40 tickets) Plug this into (2) x + 2,800 = 6000 Subtract 2,800 from each side: x = 3,200 ($24 tickets)

A 7 by 5 photo was enlarged. The length and width were enlarged 125%. Find the perimeter of the phot
A 7 by 5 photo was enlarged. The length and width were enlarged 125%. Find the perimeter of the photo. Enlarge length 125%: 7 * 1.25 = 8.75 Enlarge width 125%: 5 * 1.25 = 6.25 Perimeter of the enlarged photo is 2l + 2w, so we have: P = 2(8.75) + 2(6.25) P = 17.5 + 12.5 P = [B]30[/B]

A 7-foot piece of cotton cloth costs $3.36. What is the price per inch?
A 7-foot piece of cotton cloth costs $3.36. What is the price per inch? Using [URL='https://www.mathcelebrity.com/linearcon.php?quant=7&pl=Calculate&type=foot']our length converter[/URL], we see that: 7 feet = 84 inches So $3.36 for 84 inches. We [URL='https://www.mathcelebrity.com/perc.php?num=3.36&den=84&pcheck=1&num1=16&pct1=80&pct2=70&den1=80&idpct1=10&hltype=1&idpct2=90&pct=82&decimal=+65.236&astart=12&aend=20&wp1=20&wp2=30&pl=Calculate']divide $3.36 by 84[/URL] to get the cost per inch: $3.36/84 = [B]0.04 per inch[/B]

A 74 inch rake is Leaning against a wall. The top of the rake hits the wall 70 inches above the grou
A 74 inch rake is Leaning against a wall. The top of the rake hits the wall 70 inches above the ground. How far is the bottom of the rake from the base of the wall? We have a right triangle. Hypotenuse is the rake length fo 74 inches. One of the legs is 70. We [URL='https://www.mathcelebrity.com/pythag.php?side1input=&side2input=70&hypinput=74&pl=Solve+Missing+Side']use our right triangle calculator to solve for the other leg[/URL]: [B]24 inches[/B]

a 9-foot rope is cut into two pieces one piece is x feet express the length of the other piece in te
a 9-foot rope is cut into two pieces one piece is x feet express the length of the other piece in terms of x Piece 1 + Piece 2 = 9 Piece 1 = x x + Piece 2 = 9 Subtracting x from each side, we get: x - x + Piece 2 = 9 - x Cancel the x's on the left side, we get: Piece 2 = [B]9 - x [/B] Check our work: x + 9 - x ? 9 9 = 9

A 98-inch piece of wire must be cut into two pieces. One piece must be 10 inches shorter than the ot
A 98-inch piece of wire must be cut into two pieces. One piece must be 10 inches shorter than the other. How long should the pieces be? The key phrase in this problem is [B]two pieces[/B]. Declare Variables: [LIST] [*]Let the short piece length be s [*]Let the long piece length be l [/LIST] We're given the following [LIST=1] [*]s = l - 10 [*]s + l = 98 (Because the two pieces add up to 98) [/LIST] Substitute equation (1) into equation (2) for s: l - 10+ l = 98 Group like terms: 2l - 10 = 98 Solve for [I]l[/I] in the equation 2l - 10 = 98 [SIZE=5][B]Step 1: Group constants:[/B][/SIZE] We need to group our constants -10 and 98. To do that, we add 10 to both sides 2l - 10 + 10 = 98 + 10 [SIZE=5][B]Step 2: Cancel 10 on the left side:[/B][/SIZE] 2l = 108 [SIZE=5][B]Step 3: Divide each side of the equation by 2[/B][/SIZE] 2l/2 = 108/2 l = [B]54[/B] To solve for s, we substitute l = 54 into equation (1): s = 54 - 10 s = [B]44[/B] Check our work: The shorter piece is 10 inches shorter than the longer piece since 54 - 44 = 10 Second check: Do both pieces add up to 98 54 + 44 ? 98 98 = 98

a = v^2/r for r
a = v^2/r for r Start by cross multiplying to get r out of the denominator: ar = v^2 Divide each side of the equation by a to isolate r: ar/a = v^2/a Cancel the a's on the left side, and we get: r = [B]v^2/a[/B]

A = {1, 3, 5, 7, 9} B = {2, 4, 6, 8, 10} C = {1, 5, 6, 7, 9} A ? (B ? C) =
A = {1, 3, 5, 7, 9} B = {2, 4, 6, 8, 10} C = {1, 5, 6, 7, 9} A ? (B ? C) = B ? C = {6} A ? (B ? C) = [B]{} or the empty set[/B]

A b = b a is an example of the property called
A b = b a is an example of the property called the [B]commutative property of multiplication[/B].

A bacteria population increases every hour. At 12pm there are 5 cells. At 1pm there are 10 cells. At
A bacteria population increases every hour. At 12pm there are 5 cells. At 1pm there are 10 cells. At 2pm there are 20 cells. At 3pm there are 40 cells. If this pattern continues, how many cells will there be at 7pm? The bacteria cells double each hour in the example above. From 3pm to 7pm, we have 4 hours, meaning 4 doubling periods. Which is 2 * 2 * 2 * 2 or 2^4. So we have: 40 * 2^4 40 * 16 = [B]640 cells[/B]

A bag contains 10 red balls, 10 green balls and 6 white balls. Two balls are drawn at random from th
A bag contains 10 red balls, 10 green balls and 6 white balls. Two balls are drawn at random from the bag without replacement. What is the probability that they are of different colours? [LIST] [*]The key phrase here is [I]without replacement[/I]. [*]First, it's easier to find the probability of both colors matching, and then subtracting that from 1. [/LIST] We want 1 - (P(Red-Red) + P(Green-Green) + P(White-White)). So we have the following: [U]Find the probability of both colors matching[/U] P(Red-Red) = 10/26 * 9/25 = 0.138462 P(Green-Green) = 10/26 * 9/25 = 0.138462 P(White-White) = 6/26 * 5/25 = 0.046154 P(Red-Red) + P(Green-Green) + P(White-White) = 0.13846 + 0.13846 + 0.046154 = 0.3231 Now, we want to take the complement of this probability which is no colors matching, so we have: P(Both Different Colors) = 1 - P(Same Colors) P(Both Different Colors) = 1 - 0.3231 P(Both Different Colors) = [B]0.6769[/B]

A bag contains 120 marbles. Some are red and the rest are black. There are 19 red marbles for every
A bag contains 120 marbles. Some are red and the rest are black. There are 19 red marbles for every black marble. How many red marbles are in the bag? Let the red marbles be r Let the black marbles be b. A 19 to 1 red to black is written as: r = 19b We're also given: b + r = 120 Substitute r = 19b into this equation and we get: b + 19b = 120 Combine like terms: 20b = 120 To solve this equation, [URL='https://www.mathcelebrity.com/1unk.php?num=20b%3D120&pl=Solve']we type it in our search engine [/URL]and we get: b = 6 Since r = 19b, we substitute b = 6 into this equation to solve for r: r = 19(6) r = [B]114[/B]

A bag contains 120 marbles. Some are red and the rest are black. There are 19 red marbles for every
A bag contains 120 marbles. Some are red and the rest are black. There are 19 red marbles for every black marble. How many red marbles are in the bag? Using our [URL='https://www.mathcelebrity.com/ratio.php?simpratio=100%3A350&rs=19%3A1&rtot=120&pl=Calculate+Ratio']ratio calculator[/URL], we get: [LIST] [*]Red = 114 [*]Black = 6 [/LIST]

A bag contains 19 balls numbered 1 through 19. What is the probability that a randomly selected ball
A bag contains 19 balls numbered 1 through 19. What is the probability that a randomly selected ball has an even number? Even numbers in the bag are {2,4,6,8,10,12,14,16,18} So we have 9 total even numbers. Therefore, the probability of drawing an even number is [B]9/19[/B]

A bag contains 2 red marbles, 3 blue marbles, and 4 green marbles. What is the probability of choosi
A bag contains 2 red marbles, 3 blue marbles, and 4 green marbles. What is the probability of choosing a blue marble, replacing it, drawing a green marble, replacing it, and then drawing a red marble? Calculate total marbles in the bag: Total marbles in the bag = Red Marbles + Blue Marbles + Green Marbles Total marbles in the bag = 2 + 3 + 4 Total marbles in the bag = 9 [U]First choice, blue marble[/U] P(blue) = Total Blue Marbles / Total Marbles in the bag P(blue) = 3/9 [URL='https://www.mathcelebrity.com/fraction.php?frac1=3%2F9&frac2=3%2F8&pl=Simplify']Using our fraction simplifier[/URL], we see: P(blue) = 1/3 [U]Second choice, green marble with all the marbles back in the bag after replacement[/U] P(green) = Total Green Marbles / Total Marbles in the bag P(green) = 4/9 [U]Third choice, red marble with all the marbles back in the bag after replacement[/U] P(red) = Total Red Marbles / Total Marbles in the bag P(red) = 2/9 Since each event is independent, we multiply each probability: P(blue, green, red) = P(blue) * P(green) * P(red) P(blue, green, red) = 1/3 * 4/9 * 2/9 P(blue, green, red) = [B]8/243[/B]

A bag contains 3 black, 4 red, 3 yellow, and 2 green marbles. What is the probability of drawing a b
A bag contains 3 black, 4 red, 3 yellow, and 2 green marbles. What is the probability of drawing a black and then a red marble out of the bag without replacing the black marble before drawing the red marble? The phrase [U][B]without replacement[/B][/U] is a huge clue on this problem. Take each draw and calculate the probability. Draw 1: P(Drawing a red) P(Drawing a red) = Total Red marbles n the jar / Total marbles in the jar P(Drawing a red) = 4/12 4/12 simplifies to 1/3 using a common factor of 4: P(Drawing a red) = 1/3 Draw 2: P(Drawing a black) P(Drawing a black) = Total Black marbles in the jar / Total marbles in the jar [I]We drew one red marble already. Without replacement means we do not put it back. Therefore, we have 12 - 1 = 11 marbles left in the jar.[/I] P(Drawing a black) = 3/11 The question asks, what is the the following probability: P(Drawing a Red, Drawing a Black) Because each draw is [I][U]independent[/U], [/I]we multiply each draw probability together: P(Drawing a Red, Black) = P(Drawing a Red) * P(Drawing a Black) P(Drawing a Red, Black) = 1/3 * 3/11 P(Drawing a Red, Black) = [B]1/11[/B]

A bag contains 3 red marbles and 4 blue marbles. a marble is taken at random and replaced. another m
A bag contains 3 red marbles and 4 blue marbles. a marble is taken at random and replaced. Another marble is taken from the bag. Work out the probability that the two marbles taken from the bag are the same color. [LIST] [*]Total number of marbles in the bag is 3 + 4 = 7. [*]The problem asks for the probability of (RR) [I]or[/I] (BB). [*]It's worthy to note we are replacing the balls after each draw, which means we always have 7 to draw from [/LIST] Since each draw is independent, we take the product of each event for the total event probability. P(RR) = 3/7 * 3/7 = 9/49 P(BB) = 4/7 * 4/7 = 16/49 We want to know P(RR) + P(BB) P(RR) + P(BB) = 9/49 + 16/49 = 25/49 [MEDIA=youtube]26F9vjsgNGs[/MEDIA]

A bag contains 3 white balls and 2 black balls. Another bag contains 2 white and 4 black balls. A ba
A bag contains 3 white balls and 2 black balls. Another bag contains 2 white and 4 black balls. A bag and a ball are picked random. Then probability of that the ball will be white is: Probability that you pick any bag is 0.5. Bag 1 White Ball = 0.5(3/5) = 3/10 = 0.3 Bag 2 White Ball = 0.5(2/6) = 1/6 = 0.16667 Add them both: 0.3 + 0.16667 = [B]0.46667[/B]

A bag contains 5 blue marbles, 6 red marbles, and 4 green marbles. You select one marble at random f
A bag contains 5 blue marbles, 6 red marbles, and 4 green marbles. You select one marble at random from the bag. What is P(blue) P(blue) = Number of blue marbles / Total Marbles P(blue) = 5 / (5 + 6 + 4) P(blue) = 5/15 We can reduce this. So we [URL='https://www.mathcelebrity.com/fraction.php?frac1=5%2F15&frac2=3%2F8&pl=Simplify']type in 5/15 into our search engine, choose simplify[/URL], and we get: P(blue) = [B]1/3[/B]

A bag contains 6 red balls and 7 green balls. You plan to select 4 balls at random. Determine the pr
A bag contains 6 red balls and 7 green balls. You plan to select 4 balls at random. Determine the probability of selecting 4 green balls. Assuming draw without replacement of the balls, we have: [LIST=1] [*]Selection 1: 7 green out of 13 balls [*]Selection 2: 6 green out of 12 balls [*]Selection 3: 5 green out of 11 balls [*]Selection 4: 4 green out of 10 balls [/LIST] Since each draw is independent, we multiply each probability of green: P(GGGG) = 7/13 * 6/12 * 5/11 * 4/10 P(GGGG) = 840/17,160 P(GGGG) = [B]0.05[/B]

A bag contains 666 red balls, 444 green balls, and 333 blue balls. If we choose a ball, then another
A bag contains 666 red balls, 444 green balls, and 333 blue balls. If we choose a ball, then another ball without putting the first one back in the bag, what is the probability that the first ball will be green and the second will be red? [U]Calculate total number of balls to start:[/U] Total Balls = Red Balls + Green Balls + Blue Balls Total Balls = 666 + 444 + 333 Total Balls = 1,443 [U]Calculate the probability of drawing a green ball on the first pick:[/U] P(Green) = Green Balls / Total Balls P(Green) = 444/1443 P(Green) = 0.30769 [U]Calculate the probability of drawing a red ball on the second pick (without replacement):[/U] Total Balls decrease by 1, since we do not replace. So Total Balls = 1,443 - 1 = 1,442 P(Red) = Red Balls / Total Balls P(Red) = 666/1442 P(Red) = 0.46186 Now, we want the probability of Green, Red in that order. Since each event is independent, we multiply the event probabilities P(Green, Red) = P(Green) * P(Red) P(Green, Red) = 0.30769 * 0.46186 P(Green, Red) = [B]0.14211[/B]

A bag contains 7 red, 9 white, and 4 blue marbles. Find the probability of picking 3 blue marbles if
A bag contains 7 red, 9 white, and 4 blue marbles. Find the probability of picking 3 blue marbles if each marble is NOT returned to the bag before the next marble is picked. The problem states we will have no replacement. [LIST] [*]First draw probability is 4 blue marbles out of (7 red + 9 white + 4 blue) = 20 marbles (4/20) [*]Second draw probability is 3 blue marbles out of (7 red + 9 white + 3 blue) = 19 marbles (3/19) [*]Third draw probability is 2 blue marbles out of (7 red + 9 white + 2 blue) = 18 marbles (2/18) [/LIST] Each draw is independent, so we multiply the three draws together: 4/20 * 3/19 * 2/18 24/6840 [B]0.0035[/B]

A bag contains 8 marbles of different colors. How many ways unique ways or orders can you select 3 m
A bag contains 8 marbles of different colors. How many ways unique ways or orders can you select 3 marbles? We want the combinations formula, 8 choose 3, or 8C3. [URL='https://www.mathcelebrity.com/permutation.php?num=8&den=3&pl=Combinations']Type 8 C 3 into our search engine and we get:[/URL] [B]56 unique ways[/B]

A bag contains tiles, 3 tiles are red. 6 tiles are green, and 3 tiles are blue. A tile will be rando
A bag contains tiles, 3 tiles are red. 6 tiles are green, and 3 tiles are blue. A tile will be randomly selected from the bag . What is the probability that the tile selected will be green P(green) = Number of green tiles / Total Tiles P(green) = 6 / (3 + 6 + 3) P(green) = 6 / 12 We can simplify this fraction. We [URL='https://www.mathcelebrity.com/fraction.php?frac1=6%2F12&frac2=3%2F8&pl=Simplify']type in 6/12 into our search engine, pick simplify[/URL], and we get: P(green) = [B]1/2 or 0.5[/B]

A bag has 3 red, 5 blue, and 2 yellow pieces of candy. What is the theoretical probability of drawin
A bag has 3 red, 5 blue, and 2 yellow pieces of candy. What is the theoretical probability of drawing a blue piece of candy P(Blue) = Blue Candies / Total Candies P(Blue) = 5 / (3 + 5 + 2) P(Blue) = 5 / 10 We can [URL='https://www.mathcelebrity.com/search.php?q=5%2F10&x=0&y=0']simplify this using our GCF calculator[/URL] and we get: P(Blue) = [B]1/2[/B]

A bag of fertilizer covers 300 square feet of lawn. Find how many bags of fertilizer should be purch
A bag of fertilizer covers 300 square feet of lawn. Find how many bags of fertilizer should be purchased to cover a rectangular lawn 290 feet by 150 feet. The area of a rectangle is length * width, so we have: A = 290 * 150 A = 43,500 sq ft. Now, to find the number of bags needed for a 300 square feet per bag of fertilizer, we have: Bags Needed = Total Square Feet of Lawn / Square Feet covered per bag Bags Needed = 43,500 / 300 Bags Needed = [B]145[/B]

A bag of marbles is said to contain 50 marbles to the nearest ten. What is the greatest number of ma
A bag of marbles is said to contain 50 marbles to the nearest ten. What is the greatest number of marbles that could be in the bag and what is the least number of marbles that could be in the bag The key word in this problem is [I][B]nearest ten[/B][/I]. The nearest ten below 50 starts at 45. Why? Because the last digit is 5. At 5, we round up to the nearest ten. Therefore, the least number of marbles in the bag is 45 since it rounds up to 50 for the nearest ten The greatest number above 50 rounded to the nearest ten is 54, because less than 5 on the last digit means we round down. Therefore, the greatest number of marbles in the bag is 54 since it rounds down to 50 when the last digit is less than 5 Answer: {[B]45, 54} [MEDIA=youtube]-cl_OHA8-yc[/MEDIA][/B]

A bag of quarters and nickels is worth $8.30. There are two less than three times as many quarters a
A bag of quarters and nickels is worth $8.30. There are two less than three times as many quarters as nickels. How many of the coins must be quarters? Assumptions and givens: [LIST] [*]Let the number of quarters be q [*]Let the number of nickels be n [/LIST] We have two equations: [LIST=1] [*]0.05n + 0.25q = 8.30 [*]n = 3q - 2 [I](Two less than Three times)[/I] [/LIST] Plug in equation (2) into equation (1) for q to solve this system of equations: 0.05(3q - 2) + 0.25q = 8.30 To solve this equation for q, we [URL='https://www.mathcelebrity.com/1unk.php?num=0.05%283q-2%29%2B0.25q%3D8.30&pl=Solve']type it in our search engine[/URL] and we get: q = [B]21[/B]

A bag of rice says to mix 3 cups of rice with 2 cups of water. How much water would be needed to mix
A bag of rice says to mix 3 cups of rice with 2 cups of water. How much water would be needed to mix with 2 cups of rice? Set up a proportion of cups of rice to water where w is the amount of water needed for 2 cups of rice: 3/2 = 2/w To solve this proportion for w, we [URL='https://www.mathcelebrity.com/prop.php?num1=3&num2=2&den1=2&den2=w&propsign=%3D&pl=Calculate+missing+proportion+value']type it in our search engine[/URL] and we get: w = [B]1.33333[/B]

A baker determined the annual profit in dollars from selling pies using p(n ) = 52n - 0.05n^2, where
A baker determined the annual profit in dollars from selling pies using p(n ) = 52n - 0.05n^2, where n is the number of pies sold. What is the annual profit if the baker sells 700 pies? p(700) = 52(700) - 0.05(700)^2 p(700) = 36400 - 0.05 * 490000 p(700) = 36400 - 24500 p(700) = [B]11900[/B]

A baker determined the annual profit in dollars from selling pies using p(n) = 52n - 0.05n^2 , where
A baker determined the annual profit in dollars from selling pies using p(n) = 52n - 0.05n^2 , where n is the number of pies sold. What is the annual profit if the baker sells 400 pies? p(400) = 52(400) - 0.05(400)^2 p(400) = 20800 - 0.05(160000) p(400) = 20800 - 8000 p(400) = [B]12800[/B]

A baker makes 387 cupcakes. They are sold in packs of six. How many full packs can be made? How many
A baker makes 387 cupcakes. They are sold in packs of six. How many full packs can be made? How many cupcakes are leftover? Full packs = Lowest Rounded Integer of (Total cupcakes / packs) Full packs = Lowest Rounded Integer of 387/6 Full Packs = Lowest Rounded Integer of 64.5 Full Packs = [B]64[/B] Leftover = [URL='https://www.mathcelebrity.com/modulus.php?num=387mod6&pl=Calculate+Modulus']387 mod 6[/URL] Leftover = [B]3[/B]

A baker needs 25 kilograms of flour for his bakeshop that is good for a week. If a bag of flour weig
A baker needs 25 kilograms of flour for his bakeshop that is good for a week. If a bag of flour weighs 6 kilograms and is sold only per bag, how many bags of flour should he buy? 25 kilograms of flour /6 kilograms per bag = 4.1667 bags If they only sell full bags, then we round up to [B]5 bags[/B]

A bakery has a fixed cost of $119.75 per a day plus $2.25 for each pastry. The bakery would like to
A bakery has a fixed cost of $119.75 per a day plus $2.25 for each pastry. The bakery would like to keep its daily costs at or below $500 per day. Which inequality shows the maximum number of pastries, p, that can be baked each day. Set up the cost function C(p), where p is the number of pastries: C(p) = Variable Cost + Fixed Cost C(p) = 2.25p + 119.75 The problem asks for C(p) at or below $500 per day. The phrase [I]at or below[/I] means less than or equal to (<=). [B]2.25p + 119.75 <= 500[/B]

A bakery offers a sale price of $3.50 for 4 muffins. What is the price per dozen?
A bakery offers a sale price of $3.50 for 4 muffins. What is the price per dozen? 1 dozen = 12 muffins What this problem is really asking, $3.50 for 4 muffins. Let p be the price for 12 muffins (1 dozen). Set up a proportion of cost to muffins. 3.50/4 = p/12 Using our math engine, we [URL='https://www.mathcelebrity.com/prop.php?num1=3.50&num2=p&den1=4&den2=12&propsign=%3D&pl=Calculate+missing+proportion+value']type this proportion into our search box[/URL] and get: p = [B]10.5 muffins [MEDIA=youtube]ccY7yDkKvzs[/MEDIA][/B]

A bakery received a shipment of 4,554 peaches. If it takes 9 peaches to bake a peach pie, how many p
A bakery received a shipment of 4,554 peaches. If it takes 9 peaches to bake a peach pie, how many peach pies can the bakery make? Total Pies = Total Peaches / 9 Total Pies = 4,554 / 9 Total Pies = [B]506[/B]

A bakery sells 5800 muffins in 2010. The bakery sells 7420 muffins in 2015. Write a linear model tha
A bakery sells 5800 muffins in 2010. The bakery sells 7420 muffins in 2015. Write a linear model that represents the number y of muffins that the bakery sells x years after 2010. Find the number of muffins sold after 2010 through 2015: 7,420 - 5,800 = 1,620 Now, since the problem states a linear sales model, we need to determine the sales per year: 1,620 muffins sold since 2010 / 5 years = 324 muffins per year. Build our linear model: [B]y = 5,800 + 324x [/B] Reading this out loud, we start with 5,800 muffins at the end of 2010, and we add 324 more muffins for each year after 2010.

A ball is dropped from a height of 12 feet and returns to a height that is one-half of the height fr
A ball is dropped from a height of 12 feet and returns to a height that is one-half of the height from which it fell. The ball continues to bounce half the height of the previous bounce each time. How far will the ball have traveled when it hits the ground for the fifth time? Take the top of the bounces one at a time: [LIST=1] [*]Ball is dropped 12 feet and it bounces up to 6 feet [*]Ball drops 6 feet back down and bounces up to 3 feet up [*]Ball drops 3 feet back down and bounces up to 1.5 feet up [*]Ball drops 1.5 feet down and bounces up to 0.75 feet up [*]Return down after Bounce 5 is 0.75 feet down [/LIST] [U]Total distance travelled:[/U] 12 + 6 + 6 + 3 + 3 + 1.5 + 1.5 + 0.75 + 0.75 [B]34.5 feet [MEDIA=youtube]OvDp4Y3vOPY[/MEDIA][/B]

A ball was dropped from a height of 6 feet and began bouncing. The height of each bounce was three-f
A ball was dropped from a height of 6 feet and began bouncing. The height of each bounce was three-fourths the height of the previous bounce. Find the total vertical distance travelled by the all in ten bounces. The height of each number bounce (n) is shown as: h(n) = 6(0.75)^n We want to find h(10) h(n) = 6(0.75)^n Time Height 0 6 1 4.5 2 3.375 3 2.53125 4 1.8984375 5 1.423828125 6 1.067871094 7 0.8009033203 8 0.6006774902 9 0.4505081177 10 0.3378810883 Adding up each bounce from 1-10, we get: 16.98635674 Since vertical distance means both [B]up and down[/B], we multiply this number by 2 to get: 16.98635674 * 2 = 33.97271347 Then we add in the initial bounce of 6 to get: 33.97271347 + 6 = [B]39.97271347 feet[/B]

A bamboo tree grew 3 inches per day. How many days will it take the tree to grow 144 inches? Choose
A bamboo tree grew 3 inches per day. How many days will it take the tree to grow 144 inches? Choose the correct equation to represent this situation. Let the number of days be d. We have the equation: 3d = 144 To solve this equation for d, we [URL='https://www.mathcelebrity.com/1unk.php?num=3d%3D144&pl=Solve']type it in our search engine[/URL] and we get: d = [B]48[/B]

A band came on stage at 9:55 and preformed for 2hours and 27 minutes what time did there performance
A band came on stage at 9:55 and preformed for 2hours and 27 minutes what time did there performance end? 2 hours from 9:55 means we add 2 hours to the hour of 9: 9 + 2 = 11 11:55 Now we add 27 minutes to this time: 5 more minutes gets us to 12:00 PM 27 -5 = 22 minutes So we add 22 more minutes to get [B]12:22 PM[/B]

A bank charges a service fee of $7.50 per month for a checking account. A bank account has $85.00. I
A bank charges a service fee of $7.50 per month for a checking account. A bank account has $85.00. If no money is deposited or withdrawn except the service charge, how many months until the account balance is negative? Let m be the number of months. Our balance is denoted by B(m): B(m) = 85 - 7.5m The question asks when B(m) is less than 0. So we set up an inequality: 85 - 7.5m < 0 To solve this inequality for m, [URL='https://www.mathcelebrity.com/1unk.php?num=85-7.5m%3C0&pl=Solve']we type it in our search engine[/URL] and we get: m > 11.3333 We round up to the next whole integer and get [B]m = 12[/B]

A barn contains cows, ducks, and a 3-legged dog named Tripod. There are twice as many cows as ducks
A barn contains cows, ducks, and a 3-legged dog named Tripod. There are twice as many cows as ducks in the barn and a total of 313 legs. How many ducks are there in the barn? [LIST] [*]Let the number of ducks be d. Duck legs = 2 * d = 2d [*]Number of cows = 2d. Cow legs = 4 * 2d = 8d [*]1 dog Tripod has 3 legs [/LIST] Total legs: 2d + 8d + 3 = 313 Solve for [I]d[/I] in the equation 2d + 8d + 3 = 313 [SIZE=5][B]Step 1: Group the d terms on the left hand side:[/B][/SIZE] (2 + 8)d = 10d [SIZE=5][B]Step 2: Form modified equation[/B][/SIZE] 10d + 3 = + 313 [SIZE=5][B]Step 3: Group constants:[/B][/SIZE] We need to group our constants 3 and 313. To do that, we subtract 3 from both sides 10d + 3 - 3 = 313 - 3 [SIZE=5][B]Step 4: Cancel 3 on the left side:[/B][/SIZE] 10d = 310 [SIZE=5][B]Step 5: Divide each side of the equation by 10[/B][/SIZE] 10d/10 = 310/10 d = [B]31[/B] [URL='https://www.mathcelebrity.com/1unk.php?num=2d%2B8d%2B3%3D313&pl=Solve']Source[/URL]

A baseball card that was valued at $100 in 1970 has increased in value by 8% each year. Write a func
A baseball card that was valued at $100 in 1970 has increased in value by 8% each year. Write a function to model the situation the value of the card in 2020.Let x be number of years since 1970 The formula for accumulated value of something with a percentage growth p and years x is: V(x) = Initial Value * (1 + p/100)^x Set up our growth equation where 8% = 0.08 and V(y) for the value at time x and x = 2020 - 1970 = 50, we have: V(x) = 100 * (1 + 8/100)^50 V(x) = 100 * (1.08)^50 V(x) = 100 * 46.9016125132 V(x) = [B]4690.16[/B]

a baseball park charges $4.50 per admission ticket. the park has already sold 42 tickets. how many m
a baseball park charges $4.50 per admission ticket. the park has already sold 42 tickets. how many more tickets would they need to sell to earn at least $441? Let the number of tickets above 42 be t. A few things to note on this question: [LIST] [*]The phrase [I]at least[/I] means greater than or equal to, so we have an inequality. [*]Earnings = Price * Quantity [/LIST] We're given: Earnings = 4.50 * 42 + 4.5t >= 441 Earnings = 189 + 4.5t >= 441 We want to solve this inequality for t: Solve for [I]t[/I] in the inequality 189 + 4.5t ? 441 [SIZE=5][B]Step 1: Group constants:[/B][/SIZE] We need to group our constants 189 and 441. To do that, we subtract 189 from both sides 4.5t + 189 - 189 ? 441 - 189 [SIZE=5][B]Step 2: Cancel 189 on the left side:[/B][/SIZE] 4.5t ? 252 [SIZE=5][B]Step 3: Divide each side of the inequality by 4.5[/B][/SIZE] 4.5t/4.5 ? 252.4.5 [B]t ? 56[/B]

A baseball player gets 12 hits in 40 at bats. What percent are hits, and what percent are not hits?
A baseball player gets 12 hits in 40 at bats. What percent are hits, and what percent are not hits? Percent Hits = 12/40 Using our [URL='http://www.mathcelebrity.com/perc.php?num=12&den=40&pcheck=1&num1=16&pct1=80&pct2=70&den1=80&idpct1=10&hltype=1&idpct2=90&pct=82&decimal=+65.236&astart=12&aend=20&wp1=20&wp2=30&pl=Calculate']percent calculator[/URL], we get [B]30%[/B] Since you either get a hit or you don't, we subtract 30% from 100% to find the percent of not hits: Percent Not Hits = 100 - 30% = [B]70%[/B]

A baseball player gets 3 hits in the first 15 games of the season. If he continues hitting at the sa
A baseball player gets 3 hits in the first 15 games of the season. If he continues hitting at the same rate, how many hits will he get in the first 45 games? We set up a proportion of hits to games where h is the number of hits the player gets in 45 games: 3/15 = h/45 [URL='https://www.mathcelebrity.com/prop.php?num1=3&num2=h&den1=15&den2=45&propsign=%3D&pl=Calculate+missing+proportion+value']Enter this into our search engine[/URL], and we get [B]h = 9[/B].

A baseball player gets 7 hits in the first 15 games of the season. If he continues hitting at the sa
A baseball player gets 7 hits in the first 15 games of the season. If he continues hitting at the same rate, how many hits will he get in the first 45 games? Let's find the proportion of hits to games. Using h as the number of hits in 45 games, we have: 7/15 = h/45 Using our [URL='http://www.mathcelebrity.com/prop.php?num1=7&num2=h&den1=15&den2=45&propsign=%3D&pl=Calculate+missing+proportion+value']proportion calculator[/URL], we get h = 21

a baseball player has 9 hits in his first 60 at bats. how many consecutive hits would he need to bri
a baseball player has 9 hits in his first 60 at bats. how many consecutive hits would he need to bring his average up to 0.400? Let the amount of consecutive hits needed be h. We have: hits / at bats = Batting Average Plugging in our numbers, we get: (9 + h)/60 = 0.400 Cross multiply: 9 + h = 60 * 0.4 9 + h = 24 To solve this equation for h, [URL='https://www.mathcelebrity.com/1unk.php?num=9%2Bh%3D24&pl=Solve']we type it in our search engine[/URL] and we get: h = [B]15[/B]

A baseball team has 25 total players consisting of 15 position players and 10 pitchers. How many dif
A baseball team has 25 total players consisting of 15 position players and 10 pitchers. How many different ways are there to arrange the batting order of 9 starting players if only one pitcher is used at a time and the pitcher always bats last. (This means that 8 players are taken from the position players and one pitcher is then added at the end of the lineup.) First 8 positions: [URL='https://www.mathcelebrity.com/permutation.php?num=15&den=8&pl=Permutations']15P8[/URL] = 259,459,200 For the pitcher, we can have 10 different possibilities for the 9th player: 259,459,200 x 10 = [B]2,594,592,000 ways[/B]

A baseball team won 40% of its first 30 games. The team then won 65% of its next 60 games. Approxima
A baseball team won 40% of its first 30 games. The team then won 65% of its next 60 games. Approximately what percent of the games did the team win? Using our percentage calculators, we type the following statements into our search engine and get: [URL='https://www.mathcelebrity.com/perc.php?num=+5&den=+8&num1=+16&pct1=+80&pct2=45&den1=30&pcheck=3&pct=+82&decimal=+65.236&astart=+12&aend=+20&wp1=20&wp2=30&pl=Calculate']45% of 30[/URL] = 13.5 [URL='https://www.mathcelebrity.com/perc.php?num=+5&den=+8&num1=+16&pct1=+80&pct2=65&den1=60&pcheck=3&pct=+82&decimal=+65.236&astart=+12&aend=+20&wp1=20&wp2=30&pl=Calculate']65% of 60[/URL] = 39 For a total of 52.5 games won The team played 30 + 60 = 90 games. So we want to know the pecent: [URL='https://www.mathcelebrity.com/perc.php?num=52&den=90&pcheck=1&num1=16&pct1=80&pct2=70&den1=80&idpct1=10&hltype=1&idpct2=90&pct=82&decimal=+65.236&astart=12&aend=20&wp1=20&wp2=30&pl=Calculate']52/90[/URL] = [B]57.78%[/B]

A basket of goods was valued at $45.40 in January 2011. The inflation rate for the year was 4%. What
A basket of goods was valued at $45.40 in January 2011. The inflation rate for the year was 4%. What is the expected cost of the basket of goods in January 2012? Write your answer to the nearest cent. 2012 cost = 2011 cost * (1 + I/100) 2012 cost = 45.40 * (1 + 4/100) 2012 cost = 45.40 * (1 + 0.04) 2012 cost = 45.40 * (1.04) 2012 cost = [B]47.22[/B]

A beach volleyball court is 10 yards wide and 17 yards long. The rope used for the boundary line cos
A beach volleyball court is 10 yards wide and 17 yards long. The rope used for the boundary line costs $2.00 per yard. How much would it cost to buy a new boundary line for the court? [U]Approach:[/U] [LIST] [*]A volleyball court is shaped as a rectangle. [*]And the boundary line runs on the perimeter of the rectangle. [*]So we want the perimeter of the rectangle [/LIST] Using our [URL='https://www.mathcelebrity.com/rectangle.php?l=17&w=10&a=&p=&pl=Calculate+Rectangle']rectangle calculator with length = 17 and width = 10[/URL], we have: P = [B]54[/B]

A bedroom set that normally sells for $1100 is on sale for 15% off. If sales tax rate is 2%, what is
A bedroom set that normally sells for $1100 is on sale for 15% off. If sales tax rate is 2%, what is the total price of the bedroom set if it is bought while on sale? [U]Calculate the sale price:[/U] Sale Price = Normal Price * (1 - Sales Percentage) [U]With our sales percentage of 15% = 0.15, we have:[/U] Sale Price = 1100 * (1 - 0.15) Sale Price = 1100 * (0.85) Sale Price = 935 [U]Calculate post tax amount:[/U] Post tax amount = Sale Price * (1 + Tax Percentage) [U]With our tax percentage of 2% = 0.02, we have:[/U] Post tax amount = 935 * (1 + 0.02) Post tax amount = 935 * (1.02) Post tax amount = [B]$953.70[/B]

a bell ring every 15 seconds another bell ring 30 seconds.at 3:00 pm the 2 bells ring simultaneously
a bell ring every 15 seconds another bell ring 30 seconds.at 3:00 pm the 2 bells ring simultaneously.at what time will the bells ring again at the same time The [URL='https://www.mathcelebrity.com/gcflcm.php?num1=15&num2=30&num3=&pl=GCF+and+LCM']Least Common Multiple (LCM)[/URL] of 15 and 30 is 30: Therefore, 30 seconds from now, 3:00, is when the 2 bells will ring simultaneously. We add 30 seconds to 3:00 and get: 3:00 and 30 seconds.

A bell rings every 18 seconds, while another bell rings every 60 seconds. At 5:00 pm the two ring si
A bell rings every 18 seconds, while another bell rings every 60 seconds. At 5:00 pm the two ring simultaneously. At what time will be the bell ring again at the same time. We want the Least Common Multiple of 18 and 60. Using our [URL='https://www.mathcelebrity.com/gcflcm.php?num1=18&num2=60&num3=&pl=GCF+and+LCM']least common multiple of 18 and 60[/URL] is [B]180 [/B] 180/18 = 10 (18 second periods) 180/60 = 3 (60 second periods) 180 seconds = 3 minutes So the next time the bells ring simultaneously is 5:00 + 3 = [B]5:03 pm[/B]

A bicycle helmet is priced at $18.50. If it is on sale for 10% off and there is 7% sales tax, how mu
A bicycle helmet is priced at $18.50. If it is on sale for 10% off and there is 7% sales tax, how much will it cost after tax? [U]Calculate percent off first:[/U] 10% off means 90% off the price $18.50 * (1 - 0.1) $18.50 * (0.9) = 16.65 [U]Now, add 7% sales tax to the discounted price[/U] Price after sales tax = Discounted Price * 1.07 Price after sales tax = 16.65(1.07) [B]Price after sales tax = 17.82[/B]

A bicycle store costs $1500 per month to operate. The store pays an average of $60 per bike. The ave
A bicycle store costs $1500 per month to operate. The store pays an average of $60 per bike. The average selling price of each bicycle is $80. How many bicycles must the store sell each month to break even? Profit = Revenue - Cost Let the number of bikes be b. Revenue = 80b Cost = 60b + 1500 Break even is when profit equals 0, which means revenue equals cost. Set them equal to each other: 60b + 1500 = 80b We [URL='https://www.mathcelebrity.com/1unk.php?num=60b%2B1500%3D80b&pl=Solve']type this equation into our search engine[/URL] and we get: b = [B]75[/B]

A bicycle store costs $2750 per month to operate. The store pays an average of $45 per bike. The a
A bicycle store costs $2750 per month to operate. The store pays an average of $45 per bike. The average selling price of each bicycle is $95. How many bicycles must the store sell each month to break even? Let the number of bikes be b. Set up our cost function, where it costs $45 per bike to produce C(b) = 45b Set up our revenue function, where we earn $95 per sale for each bike: R(b) = 95b Set up our profit function, which is how much we keep after a sale: P(b) = R(b) - C(b) P(b) = 95b - 45b P(b) = 50b The problem wants to know how many bikes we need to sell to break-even. Note: break-even means profit equals operating cost, which in this case, is $2,750. So we set our profit function of 50b equal to $2,750 50b = 2750 [URL='https://www.mathcelebrity.com/1unk.php?num=50b%3D2750&pl=Solve']We type this equation into our search engine[/URL], and we get: b = [B]55[/B]

a bicycle store costs $3600 per month to operate. The store pays an average of $60 per bike. the ave
a bicycle store costs $3600 per month to operate. The store pays an average of $60 per bike. the average selling price of each bicycle is $100. how many bicycles must the store sell each month to break even? Cost function C(b) where b is the number of bikes: C(b) = Variable Cost + Fixed Cost C(b) = Cost per bike * b + operating cost C(b) = 60b + 3600 Revenue function R(b) where b is the number of bikes: R(b) = Sale price * b R(b) = 100b Break Even is when Cost equals Revenue, so we set C(b) = R(b): 60b + 3600 = 100b To solve this equation for b, we [URL='https://www.mathcelebrity.com/1unk.php?num=60b%2B3600%3D100b&pl=Solve']type it in our math engine[/URL] and we get: b = [B]90[/B]

A bicycle wheel is one meter around. If the spikes are 4 centimeters apart, how many spokes are on t
A bicycle wheel is one meter around. If the spikes are 4 centimeters apart, how many spokes are on the wheel altogether? 1 meter = 100 cm per our [URL='https://www.mathcelebrity.com/linearcon.php?quant=1&pl=Calculate&type=meter']conversions calculator[/URL] 100 cm for the whole circle / 4 cm for each spike = [B]25 spikes[/B]

A bike is purchased for $200 and sold for $150. Determine the percentage of profit or loss.
A bike is purchased for $200 and sold for $150. Determine the percentage of profit or loss. [U]Since sale price is less than purchase price, we have a loss:[/U] Loss = Sale Price - Purchase Price Loss = 150 - 200 Loss = -50 [U]Calculate percent loss:[/U] Percent Loss = 100% * Loss / Purchase Price Percent Loss = 100% * -50/200 Percent Loss = 100% *- 1/4 Percent Loss = [B]-25%[/B]

A bill at a resturant came to $95.75. There is 7.5% sales tax added on. You want to leave a 20% tip
A bill at a resturant came to $95.75. There is 7.5% sales tax added on. You want to leave a 20% tip to the total bill, after tax. How much money will you need to leave for the bill altogether? Since the tip is [I]after tax[/I], we have: Total Bill = Pre-tax Bill * (1 + Sales Tax Percent) * (1 + Tip Percent) Total Bill = $95.75 * (1 + 0.07) * (1 + 0.2) Total Bill = $95.75 * 1.07 * 1.2 Total Bill = [B]$122.94[/B]

A binomial probability experient is conducted with the given parameters. Compute the probability of
A binomial probability experient is conducted with the given parameters. Compute the probability of x successes in the n independent trials of the experiment. n = 40, p = 0.05, x = 2 P(2) = Answer is [B]0.2777[/B]. Using Excel formula of =BINOMDIST(2,40,0.05,FALSE) or using our [URL='http://www.mathcelebrity.combinomial.php?n=+40&p=0.05&k=2&t=+5&pl=P%28X+%3D+k%29']binomial probability calculator[/URL]

A bird was sitting 12 meters from the base of an oak tree and flew 15 meters to reach the top of the
A bird was sitting 12 meters from the base of an oak tree and flew 15 meters to reach the top of the tree. How tall is the tree? So we have a [U]right triangle[/U]. Hypotenuse is 15. Base is 12. We want the length of the leg. The formula for a right triangle relation of sides is a^2 + b^2 = c^2 where c is the hypotenuse and a, b are the sides Rearranging this equation to isolate a, we get a^2 = c^2 - b^2 Taking the square root of both sides, we get a = sqrt(c^2 - b^2) a = sqrt(15^2 - 12^2) a = sqrt(225 - 144) a = sqrt(81) a = [B]9 meters[/B]

A blue dice and a red dice are tossed what is the probability that a 6 will appear on both dice
A blue dice and a red dice are tossed what is the probability that a 6 will appear on both dice Each event is independent. P(Blue dice 6) = 1/6 P(Red Dice 6) = 1/6 P(Blue 6, Red 6) = 1/6 * 1/6 = [B]1/36[/B]

A blue die and a green die are rolled. Find the probability that the blue and green are both less th
A blue die and a green die are rolled. Find the probability that the blue and green are both less than 6 P(Blue not 6) = 5/6 P(Green not 6) = 5/6 Each one is independent of the other, so the probability that both are less than 6 is: P(Both not 6) = P(Blue not 6) x P(Green not 6) P(Both not 6) = 5/6 * 5/6 P(Both not 6) = [B]25/36 = 0.6944[/B]

A boa constrictor is 18 inches long at birth and grows 8 inches per year. Write an equation that rep
A boa constrictor is 18 inches long at birth and grows 8 inches per year. Write an equation that represents the length y (in feet) of a boa constrictor that is x years old. 8 inches per year = 8/12 feet = 2/3 foot [B]y = 18 + 2/3x[/B]

A board must be cut into three pieces that are the same length. If it takes five minutes for each cu
A board must be cut into three pieces that are the same length. If it takes five minutes for each cut, how long will it take to saw the board into three pieces that are the same size? Three equal pieces means only 2 cuts on the board: 2 cuts * 5 minutes per cut = [B]10 minutes[/B]

A boat can carry 582 passengers to the base of a waterfall. A total of 13,105 people ride the boat t
A boat can carry 582 passengers to the base of a waterfall. A total of 13,105 people ride the boat today. All the rides are full except for the first ride. How many rides are given? 582 passengers on the boat Let r be the number of rides So we want to find r when: 582r = 13105 To solve for r, we [URL='https://www.mathcelebrity.com/1unk.php?num=582r%3D13105&pl=Solve']type this equation into our math engine[/URL] and we get: r = 22.517 If we round this down, setting 0.517 rides as the first ride, we get: r = [B]22 [MEDIA=youtube]0J2YRPzKsoU[/MEDIA][/B]

A boat costs 14950 and decrease in value by 7% per year how much will the boat be worth after 8 yea
A boat costs 14950 and decrease in value by 7% per year how much will the boat be worth after 8 years? If a boat decreases in value 7% in value, then our new value each year is 100% - 7% = 93%. So we have a B(y) function where B(y) is the value of the boat after y years: B(y) = 14,950 * (1 - 0.07)^y Simplifying, we get: B(y) = 14,950 * (0.93)^y The problem asks for B(8) B(8) = 14,950 * (0.93)^7 B(8) = 14,950 * 0..6017 B(8) = [B]8,995.43[/B]

A boat is marked up 1/5 of the original price. The original price was $50. What is the new price of
A boat is marked up 1/5 of the original price. The original price was $50. What is the new price of the boat 1/5 of 50 equals 10. So we add the markup of 10 to the original price of 50: 50 + 10 = [B]$60[/B]

a boat traveled 336 km downstream with the current. The trip downstream took 12 hours. write an equa
a boat traveled 336 km downstream with the current. The trip downstream took 12 hours. write an equation to describe this relationship We know the distance (d) equation in terms of rate (r) and time (t) as: d = rt We're given d = 336km and t = 12 hours, so we have: [B]336 km = 12t [/B] <-- this is our equation Divide each side by 12 to solve for t: 12t/12 = 336/12 t = [B]28 km / hour[/B]

A boat traveled at a constant speed for 32 hours, covering a total distance of 597 kilometers. To th
A boat traveled at a constant speed for 32 hours, covering a total distance of 597 kilometers. To the nearest hundredth of a kilometer per hour, how fast was it going? Distance = Rate * Time We're given t = 32, and d = 597. Using our [URL='https://www.mathcelebrity.com/drt.php?d=+597&r=+&t=32&pl=Calculate+the+missing+Item+from+D%3DRT']distance, rate, and time calculator[/URL], we get: r = [B]18.656 km/hr[/B]

A book cost $8.50 without tax. If the tax rate is 7%, what is the total cost of the book including t
A book cost $8.50 without tax. If the tax rate is 7%, what is the total cost of the book including tax? 8.50 * 1.07 = [B]$9.10[/B]

A book is discounted 45%. If the original price is $40, what is the new price?
A book is discounted 45%. If the original price is $40, what is the new price? 45% discount means we pay 100% - 45% = 55% 40 * 55% = [B]22[/B]

A book publishing company has fixed costs of $180,000 and a variable cost of $25 per book. The books
A book publishing company has fixed costs of $180,000 and a variable cost of $25 per book. The books they make sell for $40 each. [B][U]Set up Cost Function C(b) where b is the number of books:[/U][/B] C(b) = Fixed Cost + Variable Cost x Number of Units C(b) = 180,000 + 25(b) [B]Set up Revenue Function R(b):[/B] R(b) = 40b Set them equal to each other 180,000 + 25b = 40b Subtract 25b from each side: 15b = 180,000 Divide each side by 15 [B]b = 12,000 for break even[/B]

a book which was marked at $84 was sold for $75.60 .calculate the percent discount
a book which was marked at $84 was sold for $75.60 .calculate the percent discount Using our [URL='https://www.mathcelebrity.com/markup.php?p1=84&m=&p2=+75.60&pl=Calculate']markdown calculator[/URL], we get: [B]10% markdown/percent discount[/B]

A bookstore was selling books for 50% off. A shelf in the store had a sign that said "Books on this
A bookstore was selling books for 50% off. A shelf in the store had a sign that said "Books on this shelf take an additional 25% off." Leta picked out books from the discount shelf that had a regular price of $100. How much did Leta pay for the discounted books? 100 with 50% discount is $40 $50 with a 25% discount is $12.50 off $50 - $12.50 = [B]$37.50[/B]

A Bouquet of lillies and tulips has 12 flowers. Lillies cost $3 each, and tulips cost $2 each. The b
A Bouquet of lillies and tulips has 12 flowers. Lillies cost $3 each, and tulips cost $2 each. The bouquet costs $32. Write and solve a system of linear equations to find the number of lillies and tulips in the bouquet. Let l be the number of lillies and t be the number of tulips. We're given 2 equations: [LIST=1] [*]l + t = 12 [*]3l + 2t = 32 [/LIST] With this system of equations, we can solve it 3 ways. [LIST] [*][URL='https://www.mathcelebrity.com/simultaneous-equations.php?term1=l+%2B+t+%3D+12&term2=3l+%2B+2t+%3D+32&pl=Substitution']Substitution Method[/URL] [*][URL='https://www.mathcelebrity.com/simultaneous-equations.php?term1=l+%2B+t+%3D+12&term2=3l+%2B+2t+%3D+32&pl=Elimination']Elimination Method[/URL] [*][URL='https://www.mathcelebrity.com/simultaneous-equations.php?term1=l+%2B+t+%3D+12&term2=3l+%2B+2t+%3D+32&pl=Cramers+Method']Cramers Rule[/URL] [/LIST] No matter which method we choose, we get: [LIST] [*][B]l = 8[/B] [*][B]t = 4[/B] [/LIST] [B]Now Check Your Work For Equation 1[/B] l + t = 12 8 + 4 ? 12 12 = 12 [B]Now Check Your Work For Equation 2[/B] 3l + 2t = 32 3(8) + 2(4) ? 32 24 + 8 ? 32 32 = 32

A bowl contains 45 oranges. If ? of the oranges are bad; how many are good?
A bowl contains 45 oranges. If ? of the oranges are bad; how many are good? Using our [URL='https://www.mathcelebrity.com/fraction.php?frac1=1&frac2=2%2F3&pl=Subtract']fraction operator calculator[/URL], we see that: 1 - 2/3 = 1/3 of the oranges are good. We want 1/3 of 45. [URL='https://www.mathcelebrity.com/fraction.php?frac1=45&frac2=1/3&pl=Multiply']Typing this expression into our search engine[/URL], we get: [B]15 good oranges[/B]

A bowler knocks down at least 6 pins 70 percent of the time. Out of 200 rolls, how many times can yo
A bowler knocks down at least 6 pins 70 percent of the time. Out of 200 rolls, how many times can you predict the bowler will knock down at least 6 pins? Expected Value of (knocking down at least 6 pins) = number of rolls * probability of knocking down at least 6 pins Expected Value of (knocking down at least 6 pins) = 200 * 0.7 Expected Value of (knocking down at least 6 pins) = [B]140[/B]

A box
A box contains 4 plain pencils and 4 pens. A second box contains 5 color pencils and 3 crayons. One item from each box is chosen at random. What is the probability that a pen from the first box and a crayon from the second box are selected? [LIST] [*]First box, P(pen) = 4/8 = 1/2 = 0.5 [*]Second box, P(crayon) = 3/8 [/LIST] Since each event is independent, we have: P(Pen from Box 1) * P(Crayon from Box 2) = 1/2 * 3/8 = [B]3/16 or 0.1875[/B]

A box contains 10 bells. There are 6 red bells and the rest are silver. What is the probability of p
A box contains 10 bells. There are 6 red bells and the rest are silver. What is the probability of picking two bells of the same color if the bell is replaced after each pick? If there are 6 red bells, then we have 10 - 6 = 4 silver bells. The problem asks for the probability of picking two bells of the same color. Which mean we have 2 scenarios: [LIST=1] [*]Silver, Silver [*]Red, Red [/LIST] Find the probability of Silver, Silver: Since each draw is independent, and we replace the bells, we have a 4/10 chance of picking silver. [URL='https://www.mathcelebrity.com/fraction.php?frac1=4%2F10&frac2=3%2F8&pl=Simplify']Simplified, this is 2/5[/URL]. (2/5)(2/5) = 4/25 Find the probability of Red, Red: Since each draw is independent, and we replace the bells, we have a 6/10 chance of picking silver. [URL='https://www.mathcelebrity.com/fraction.php?frac1=6%2F10&frac2=3%2F8&pl=Simplify']Simplified, this is 3/5[/URL]. (3/5)(3/5) = 9/25 Because we want Silver, Silver [B][U]or[/U][/B] Red, Red, we add the two probabilities. 4/25 + 9/25 = [B]13/25[/B]

A box contains 22 red apples and 3 green apples. Three apples are selected at random, one after the
A box contains 22 red apples and 3 green apples. Three apples are selected at random, one after the other, without replacement. please show the steps. (a) The first two apples are green. What is the probability that the third apple is red? (b) What is the probability that exactly two of the three apples are red? a) You have 22 red apples left and 1 green left leaving 23 total apples left. Therefore, probability of red is [B]P(R) = 22/23[/B] b) Determine our sample space to select exactly two red apples in three picks. [LIST=1] [*]RRG [*]RGR [*]GRR [/LIST] [U]Now determine the probabilities of each event in the sample space[/U] P(RRG) = 22/25 * 21/24 * 3/23 = 0.1004 P(RGR) = 22/25 * 3/24 * 21/23 = 0.1004 P(GRR) = 3/25 * 22/24 * 21/23 = 0.1004 [U]We want the sum of the three probabilities[/U] P(RRG) + P(RGR) + P(GRR) = 0.1004 + 0.1004 + 0.1004 P(RRG) + P(RGR) + P(GRR) = 3(0.1004) P(RRG) + P(RGR) + P(GRR) = [B]0.3012[/B]

A box contains 4 plain pencils and 4 pens. A second box contains 5 color pencils and 3 crayons. One
A box contains 4 plain pencils and 4 pens. A second box contains 5 color pencils and 3 crayons. One item from each box is chosen at random. What is the probability that a pen from the first box and a crayon from the second box are selected? [LIST] [*]First box, P(pen) = 4/8 = 1/2 = 0.5 [*]Second box, P(crayon) = 3/8 [/LIST] Since each event is independent, we have: P(Pen from Box 1) * P(Crayon from Box 2) = 1/2 * 3/8 = [B]3/16 or 0.1875[/B]

A box contains 4 red jellies, 6 blue jellies and 5 yellow jellies. What is the probability that a j
A box contains 4 red jellies, 6 blue jellies and 5 yellow jellies. What is the probability that a jelly chosen randomly from the box is not red? Calculate Total Jellies: Total Jellies = Red Jellies + Blue Jellies + Yellow Jellies Total Jellies = 4 + 6 + 5 Total Jellies = 15 Not choosing a red jelly means choosing a blue [B]or[/B] yellow jelly P(not red jelly) = P(blue) + P(Yellow) P(not red jelly) = (Blue Jelly + Yellow Jelly) / Total Jellies P(not red jelly) = (6 + 5)/15 P(not red jelly) = [B]11/15[/B]

A box contains 5 black and 2 white balls. 2 balls are drawn without replacement. Find the probabilit
A box contains 5 black and 2 white balls. 2 balls are drawn without replacement. Find the probability of drawing 2 black balls. First draw probability of black is: Total Balls in box = Black balls + white balls Total Balls in Box = 5 + 2 Total Balls in Box = 7 P(Black) = Black Balls / Total balls in box P(Black) = 5/7 Second draw probability of black (with no replacement) is: Total Balls in box = Black balls + white balls Total Balls in Box = 4 + 2 Total Balls in Box = 6 P(Black) = Black Balls / Total balls in box P(Black) = 4/6 Using our [URL='https://www.mathcelebrity.com/fraction.php?frac1=4%2F6&frac2=3%2F8&pl=Simplify']fraction simplifier[/URL], we see that 4/6 is: 2/3 Since each event is independent, we can multiply them to find the probability of drawing 2 black balls: P(Black, Black) = 5/7 * 2/3 [URL='https://www.mathcelebrity.com/fraction.php?frac1=5%2F7&frac2=2%2F3&pl=Multiply']P(Black, Black)[/URL] = 10/21 [MEDIA=youtube]HEa_G3nwgUQ[/MEDIA]

A box contains 5 plain pencils and 3 pens. A second box contains 2 color pencils and 2 crayons . One
A box contains 5 plain pencils and 3 pens. A second box contains 2 color pencils and 2 crayons. One item from each box is chosen at random. What is the probability that a plain pencil from the first box and a color pencil from the second box are selected [U]Calculate the probability of a plain pencil in the first box:[/U] P(plain pencil in the first box) = Total Pencils / Total Objects P(plain pencil in the first box) = 5 pencils / (5 pencils + 3 pens) P(plain pencil in the first box) = 5/8 [U]Calculate the probability of a color pencil in the first box:[/U] P(color in the second box) = Total Pencils / Total Objects P(color in the second box) = 2 pencils / (2 pencils + 2 crayons) P(color in the second box) = 2/4 We can simplify this. [URL='https://www.mathcelebrity.com/fraction.php?frac1=2%2F4&frac2=3%2F8&pl=Simplify']Type 2/4 into our search engine[/URL] and we get 1/2 Now the problem asks for the probability that a plain pencil from the first box and a color pencil from the second box are selected. Since each event is independent, we multiply them together to get our answer: P(plain pencil in the first box, color in the second box) = P(plain pencil in the first box) * P(color in the second box) P(plain pencil in the first box, color in the second box) = 5/8 * 1/2 P(plain pencil in the first box, color in the second box) = [B]5/16[/B]

A box contains 5 plain pencils and 7 pens. A second box contains 4 color pencils and 4 crayons. One
A box contains 5 plain pencils and 7 pens. A second box contains 4 color pencils and 4 crayons. One item from each box is chosen at random. What is the probability that a plain pencil from the first box and a color pencil from the second box are selected? Probability of plain pencil from first box: 5/(5 + 7) = 5/12 Probability of color pencil from second box: 4/(4 + 4) = 4/8 = 1/2 Probability of both events together: Since each event is independent, we multiply probabilities: 5/12 * 1/2 = [B]5/24[/B]

A box contains 6 yellow, 3 red, 5 green, and 7 blue colored pencils. A pencil is chosen at random, i
A box contains 6 yellow, 3 red, 5 green, and 7 blue colored pencils. A pencil is chosen at random, it is not replaced, then another is chosen. What is the probability of choosing a red followed by a green? We have 6 + 3 + 5 + 7 = 21 total pencils P(Red on the first draw) = Total Red / Total pencils P(Red on the first draw) = 3/21 [URL='https://www.mathcelebrity.com/fraction.php?frac1=3%2F21&frac2=3%2F8&pl=Simplify']P(Red on the first draw)[/URL] = 1/7 We're drawing without replacement, this means on the next draw, we have 21 - 1 = 20 pencils P(Green on the second draw) = Total Green / Total pencils P(Green on the second draw) = 5/20 [URL='https://www.mathcelebrity.com/fraction.php?frac1=5%2F20&frac2=3%2F8&pl=Simplify']P(Green on the second draw) [/URL]= 1/4 Since each event is independent, we have: P(Red on first, green on second) = P(Red on First) * P(green on second) P(Red on first, green on second) = 1/7 * 1/4 P(Red on first, green on second) = [B]1/28[/B]

A box had x pencils. Then 6 pencils were removed from the box. The box now has 54 pencils.
A box had x pencils. Then 6 pencils were removed from the box. The box now has 54 pencils. Removed means we subtract from the total. So Our equation is: x - 6 = 54 To solve this equation for x, we [URL='https://www.mathcelebrity.com/1unk.php?num=x-6%3D54&pl=Solve']type it in our search engine [/URL]and we get: x = [B]60[/B]

A box has y oranges. 5 oranges are rotten and removed. The remaining oranges are placed equally into
A box has y oranges. 5 oranges are rotten and removed. The remaining oranges are placed equally into 2 containers . Each container has ______oranges Remove 5 rotten oranges means we subtract 5 from y: y - 5 If each of the two remaining boxes contains an equal amount of the remaining oranges, we have: [B](y - 5)/2[/B] oranges in each box

A box is filled with 10 green cards, 4 blue cards, and 4 brown cards. A card is chosen at random fr
A box is filled with 10 green cards, 4 blue cards, and 4 brown cards. A card is chosen at random from the box. What is the probability that it is a green or a brown card? Calculate Total Cards: 10 green cards + 4 blue cards + 4 brown cards = 18 cards "Or", means either or, so we want P(Green) + P(Brown) [U]Find P(Green)[/U] P(Green) = Green Cards / Total Cards P(Green) = 10/18 <-- Simplify by dividing top and bottom by 2 P(Green)= 5/9 <-- Simplify by dividing top and bottom by 2 Find P(Brown) P(Brown) = Brown Cards / Total Cards P(Brown) = 4/18 <-- Simplify by dividing top and bottom by 2 P(Brown)= 2/9 <-- Simplify by dividing top and bottom by 2 P(Green) + P(Brown) = 5/9 + 2/9 P(Green) + P(Brown) = [B]7/9[/B]

A box is filled with 5 blue cards,2 red cards, and 5 yellow cards. A card is chosen at random from t
A box is filled with 5 blue cards,2 red cards, and 5 yellow cards. A card is chosen at random from the box. What is the probability that it is a blue or a yellow card? Write your answer as a fraction in simplest form. We want P(B) + P(Y) P(B) = 5/12 P(Y) = 5/12 P(B) + P(Y) = 5/12 + 5/12 = 10/12 Reduce this fraction using 2 as our common factor: [B]5/6[/B]

A box of goldfish food can feed 3 fish for 4 weeks. How long will the box last if there are 7 goldfi
A box of goldfish food can feed 3 fish for 4 weeks. How long will the box last if there are 7 goldfish? 4 weeks = 7 * 4 = 28 days One box = 3 fish * 28 days = 84 days 84 days / 7 goldfish = [B]12 days[/B]

A box of pencils weights 3.25 grams. If the teacher orders 14 boxes, how much would the pencils weig
A box of pencils weights 3.25 grams. If the teacher orders 14 boxes, how much would the pencils weigh? Total Weight = Number of Boxes * Weight per box Total Weight = 14 * 3.25 Total Weight = [B]45.5 grams[/B]

A boy has 6 toys he loses 3. How many does the boy have?
A boy has 6 toys he loses 3. How many does the boy have? We subtract 3 from 6 since losing means less, and we have: 6 - 3 = [B]3[/B]

A boy has m mangoes. He sells three of them write down an expression to represent how much he now ha
A boy has m mangoes. He sells three of them write down an expression to represent how much he now has. When the boy sells the mangos, he has less. So we subtract 3 from m to get: [B]m - 3[/B]

A boy is 10 years older than his brother. In 4 years he will be twice as old as his brother. Find th
A boy is 10 years older than his brother. In 4 years he will be twice as old as his brother. Find the present age of each? Let the boy's age be b and his brother's age be c. We're given two equations: [LIST=1] [*]b = c + 10 [*]b + 4 = 2(c + 4) [/LIST] Substitute equation (1) into equation (2): (c + 10) + 4 = 2(c + 4) Simplify by multiplying the right side through and grouping like terms: c + 14 = 2c + 8 [URL='https://www.mathcelebrity.com/1unk.php?num=c%2B14%3D2c%2B8&pl=Solve']Type this equation into our search engine[/URL] and we get: c = [B]6[/B] Now plug c = 6 into equation (1): b = 6 + 10 b = [B]16[/B]

A boy is 6 years older than his sister. In 3 years time he will be twice her age. What are their pre
A boy is 6 years older than his sister. In 3 years time he will be twice her age. What are their present ages? Let b be the boy's age and s be his sister's age. We're given two equations: [LIST=1] [*]b = s + 6 [*]b + 3 = 2(s + 3) [/LIST] Plug in (1) to (2): (s + 6) + 3 = 2(s + 3) s + 9 = 2s + 6 [URL='https://www.mathcelebrity.com/1unk.php?num=s%2B9%3D2s%2B6&pl=Solve']Plugging this equation into our search engine[/URL], we get: [B]s = 3[/B] We plug s = 3 into Equation (1) to get the boy's age (b): b = 3 + 6 [B]b = 9[/B]

A boy is 6 years younger than his sister. If he is (x-9) years old, how long will it take for his si
A boy is 6 years younger than his sister. If he is (x-9) years old, how long will it take for his sister to be x years old? If the boy is x - 9 years old, and he's 6 years younger than his sister, than the sister is older by 6 years. Sister's Age = x - 9 + 6 Sister's Age = x - 3 In order to be x years old, we must add 3 years: x - 3 + 3 = x So in [B]3 years, [/B]the sister will be x years old.

a boy purchased a party-length sandwich 57 inches long. he wants to cut it into three pieces so that
a boy purchased a party-length sandwich 57 inches long. he wants to cut it into three pieces so that the middle piece is 6inches longer than the shortest piece and the shortest piece is 9 inches shorter than the longest price. how long should the three pieces be? Let the longest piece be l. The middle piece be m. And the short piece be s. We have 2 equations in terms of the shortest piece: [LIST=1] [*]l = s + 9 (Since the shortest piece is 9 inches shorter, this means the longest piece is 9 inches longer) [*]m = s + 6 [*]s + m + l = 57 [/LIST] We substitute equations (1) and (2) into equation (3): s + (s + 6) + (s + 9) = 57 Group like terms: (1 + 1 + 1)s + (6 + 9) = 57 3s + 15 = 57 To solve for s, we [URL='https://www.mathcelebrity.com/1unk.php?num=3s%2B15%3D57&pl=Solve']type this equation into our search engine[/URL] and we get: s = [B]14 [/B] [U]Plug s = 14 into equation 2 to solve for m:[/U] m = 14 + 6 m = [B]20 [/B] [U]Plug s = 14 into equation 1 to solve for l:[/U] l = 14 + 9 l = [B]23 [/B] Check our work for equation 3: 14 + 20 + 23 ? 57 57 = 57 <-- checks out [B][/B]

A boy spent one-half of his money for a book and one-third of his money for a pen. The remaining $2.
A boy spent one-half of his money for a book and one-third of his money for a pen. The remaining $2.25 he saved. How much money did he originally have? Find out what percent of money was spent Using a common denominator of 6, we have 1/2 + 1/3 = 3/6 + 2/6 = 5/6. Therefore, 1/6 of his money is left to save. Let the boy's original money be x. We have: x/6 = 2.25 Cross multiply, we get x = [B]13.50[/B]

A brand new car that is originally valued at $25,000 depreciates by 8% per year. What is the value o
A brand new car that is originally valued at $25,000 depreciates by 8% per year. What is the value of the car after 6 years? The Book Value depreciates 8% per year. We set up a depreciation equation: BV(t) = BV(0) * (1 - 0.08)^t The Book Value at time 0 BV(0) = 25,000. We want the book value at time 6. BV(6) = 25,000 * (1 - 0.08)^6 BV(6) = 25,000 * 0.92^6 BV(6) = 25,000 * 0.606355 BV(6) = [B]15,158.88[/B]

A broken clock that loses 12 minutes every hour is set at 12:00 noon at the same time a normal clock
A broken clock that loses 12 minutes every hour is set at 12:00 noon at the same time a normal clock has its time set to 12:00 noon. When the broken clock reaches 12:00 midnight, what will the normal clock read? Set up a proportion normal clock to broken clock: 60/48 = n/12 Using our [URL='https://www.mathcelebrity.com/proportion-calculator.php?num1=60&num2=n&den1=48&den2=12&propsign=%3D&pl=Calculate+missing+proportion+value']proportion calculator,[/URL] we get: n = [B]15 hours [/B] 12:00 AM plus 15 hours = [B]3 pm[/B]

A builder needs 36 nails to finish a projects. If the nails come in packages of 3, how many packages
A builder needs 36 nails to finish a projects. If the nails come in packages of 3, how many packages should the builder purchase? Packages needed = Total Nails / Nails per package Packages needed = 36/3 Packages needed = [B]12[/B]

A bunny population is doubling every 2 years. There are currently 45 bunnies. How many will there be
A bunny population is doubling every 2 years. There are currently 45 bunnies. How many will there be in 10 years? Find the number of doubling periods: Number of Doubling periods = Time / Doubling period Number of Doubling periods = 10/2 Number of Doubling periods = 5 Create a function to determine the amount of bunnies after each doubling period: B(n) = 45 * 2^n Since we calculated 5 doubling periods, we want B(5): B(5) = 45 * 2^5 B(5) = 45 * 32 B(5) = [B]1,440[/B]

A bus can at most be filled with 36 passengers. If there are currently 25 passengers there already.
A bus can at most be filled with 36 passengers. If there are currently 25 passengers there already. How many passengers can be added at most? We have 36 - 25 existing passengers = [B]11 passengers can be added at most[/B]

A bus holds 45 students. How many buses were taken on a field trip if 13 students travels by car and
A bus holds 45 students. How many buses were taken on a field trip if 13 students travels by car and total of 320 students went on a trip? [U]Find the number of students who went on the bus:[/U] Number of students who went on the bus = Total students on field trip - students who traveled by car Number of students who went on the bus = 320 - 13 Number of students who went on the bus = 307 Calculate the number of buses needed: Number of buses needed = Number of students who went on the bus / Bus Capacity Number of buses needed = 307 / 45 Number of buses needed = 6.822 We round up for a full bus to get [B]7 buses[/B]

A bus is carrying 135 passengers, which is 3/4 of the capacity of the bus. What is the capacity of t
A bus is carrying 135 passengers, which is 3/4 of the capacity of the bus. What is the capacity of the bus Let the capacity of the bus be c. We're given: 3c/4 = 135 To solve for c, we [URL='https://www.mathcelebrity.com/prop.php?num1=3c&num2=135&den1=4&den2=1&propsign=%3D&pl=Calculate+missing+proportion+value']type this equation into our search engine [/URL]and we get: c = [B]180[/B]

A bus ride cost 1.50. A 30 day pass cost $24. Write an inequallity to show that the 30 day pass is t
A bus ride cost 1.50. A 30 day pass cost $24. Write an inequallity to show that the 30 day pass is the better deal Let the number of days be d. We have the inequality below where we show when the day to day cost is greater than the monthly pass: 1.5d > 24 To solve this inequality for d, we [URL='https://www.mathcelebrity.com/interval-notation-calculator.php?num=1.5d%3E24&pl=Show+Interval+Notation']type it in our search engine[/URL] and we get: [B]d > 16[/B]

A bus usually takes 25 minutes. Because of traffic the trip took 7 minutes longer. How long did the
A bus usually takes 25 minutes. Because of traffic the trip took 7 minutes longer. How long did the trip take? The word [I]longer[/I] means we add, so we have: 25 + 7 = [B]32 minutes[/B]

A business owner spent $4000 for a computer and software. For bookkeeping purposes, he needs to post
A business owner spent $4000 for a computer and software. For bookkeeping purposes, he needs to post the price of the computer and software separately. The computer costs 4 times as much as the software. What is the cost of the software? Let c be the cost of the computer and s be the cost of the software. We have two equations: [LIST=1] [*]c + s = 4000 [*]c = 4s [/LIST] Substitute (2) into (1) (4s) + s = 4000 Using our [URL='http://www.mathcelebrity.com/1unk.php?num=4s%2Bs%3D4000&pl=Solve']equation solver[/URL], we get [B]s = 800[/B]. Substitute this into Equation (2), we get: c = 4(800) [B]c = 3,200[/B]

A cab charges $5 for the ride plus $1.25 per mile. How much will a 53 mile trip cost?
A cab charges $5 for the ride plus $1.25 per mile. How much will a 53 mile trip cost? We set up our cost function C(m) where m is the number of miles: C(m) = 1.25m + 5 The problem asks for C(53): C(53) = 1.25(53) + 5 C(53) = 66.25 + 5 C(53) = [B]$71.25[/B]

A cab company charges $5 per cab ride, plus an additional $1 per mile driven , How long is a cab rid
A cab company charges $5 per cab ride, plus an additional $1 per mile driven , How long is a cab ride that costs $13? Let the number of miles driven be m. Our cost function C(m) is: C(m) = Cost per mile * m + cab cost C(m) = 1m + 5 The problem asks for m when C(m) = 13: 1m + 5 = 13 To solve this equation for m, [URL='https://www.mathcelebrity.com/1unk.php?num=1m%2B5%3D13&pl=Solve']we type it in our search engine[/URL] and we get: m = [B]8[/B]

A cab company charges $5 per cab ride, plus an additional $3 per mile driven. How long is a cab ride
A cab company charges $5 per cab ride, plus an additional $3 per mile driven. How long is a cab ride that costs $17? Let m be the number of miles driven. We setup the cost equation C(m): C(m) = Cost per mile driven * miles driven + ride cost C(m) = 3m + 5 The questions asks for m when C(m) is 17: 3m + 5 = 17 To solve this equation for m, we [URL='https://www.mathcelebrity.com/1unk.php?num=3m%2B5%3D17&pl=Solve']type it in our search engine[/URL] and we get: m = [B]4[/B]

A cable company charges $75 for installation plus $20 per month. Another cable company offers free i
A cable company charges $75 for installation plus $20 per month. Another cable company offers free installation but charges $35 per month. For how many months of cable service would the total cost from either company be the same [U]Set ups the cost function for the first cable company C(m) where m is the number of months:[/U] C(m) = cost per month * m + installation fee C(m) = 20m + 75 [U]Set ups the cost function for the second cable company C(m) where m is the number of months:[/U] C(m) = cost per month * m + installation fee C(m) = 35m The problem asks for m when both C(m) functions are equal. So we set both C(m) functions equal and solve for m: 20m + 75 = 35m To solve for m, we [URL='https://www.mathcelebrity.com/1unk.php?num=20m%2B75%3D35m&pl=Solve']type this equation into our search engine[/URL] and we get: m = [B]5[/B]

A cake is to be divided into 8 equal parts. After division, each equal portion is again divided into
A cake is to be divided into 8 equal parts. After division, each equal portion is again divided into 2 equal individual parts. How big is each of the new equal parts? 1 cake * 8 parts * 2 parts = 16 parts. So each slice is 1/16 of a cake.

A camel can drink 15 gallons of water in 10 minutes. At this rate, how much water can the camel drin
A camel can drink 15 gallons of water in 10 minutes. At this rate, how much water can the camel drink in 8 minutes? Set up a proportion of gallons of water to time where g is the number of gallons of water in 8 minutes. 15/10 = g/8 [URL='https://www.mathcelebrity.com/prop.php?num1=15&num2=g&den1=10&den2=8&propsign=%3D&pl=Calculate+missing+proportion+value']Run this problem through our proportion calculator[/URL] to get [B]g = 12.[/B]

A camel can drink 15 gallons of water in 10 minutes. At this rate, how much water can the camel drin
A camel can drink 15 gallons of water in 10 minutes. At this rate, how much water can the camel drink in 14 minutes? Set up a proportion of gallons of water over minutes where g is the number of gallons the camel can drink in 14 minutes: 15/10 = g/14 [URL='https://www.mathcelebrity.com/prop.php?num1=15&num2=g&den1=10&den2=14&propsign=%3D&pl=Calculate+missing+proportion+value']Typing this proportion into our search engine[/URL], we get: [B]g = 21[/B]

A camera normally cost for $450 is on sale for $315 what is the discount rate as the percentage on t
A camera normally cost for $450 is on sale for $315 what is the discount rate as the percentage on the camera Using our [URL='https://www.mathcelebrity.com/markup.php?p1=450&m=&p2=+315&pl=Calculate']markdown calculator[/URL], we get: [B]-30%[/B]

A candidate for mayor wants to gauge potential voter reaction to an increase recreational services b
A candidate for mayor wants to gauge potential voter reaction to an increase recreational services by estimating the proportion of voter who now use city services. If we assume that 50% of the voters require city recreational services, what is the probability that 40% or fewer voters in a sample of 100 actually will use these city services? First, let's do a test on the proportion using our [URL='http://www.mathcelebrity.com/proportion_hypothesis.php?x=+40&n=+100&ptype=%3D&p=+0.5&alpha=+0.05&pl=Proportion+Hypothesis+Testing']proportion hypothesis calculator[/URL]: We get Z = -2 Now use the [URL='http://www.mathcelebrity.com/zscore.php?z=p%28z%3C-2%29&pl=Calculate+Probability']z-score calculator[/URL] to get P(z<-2) = [B]0.02275[/B]

A candlestick burns at a rate of 0.2 inches per hour. After eight straight hours of burning, the can
A candlestick burns at a rate of 0.2 inches per hour. After eight straight hours of burning, the candlestick is 13.4 inches tall. Write and solve a linear equation to find the original height of the candle. Let h equal the number of hours the candlestick burns. We have a candlestick height equation of C. C = 13.4 + 0.2(8) <-- We need to add back the 8 hours of candlestick burning C = 13.4 + 1.6 C = [B]15 inches[/B]

A car drives 3 feet the first second, 9 feet in the next second, and 27 feet in the third second. If
A car drives 3 feet the first second, 9 feet in the next second, and 27 feet in the third second. If the pattern stays the same, how far will the car have traveled after 5 seconds, in feet? Our pattern is found by the distance function D(t), where we have 3 to the power of the time (t) in seconds as seen below: D(t) = 3^t The problem asks for D(5): D(5) = 3^5 [URL='https://www.mathcelebrity.com/powersq.php?sqconst=+6&num=3%5E5&pl=Calculate']D(5)[/URL] = [B]243[/B]

A car is bought for $2400 and sold one year later $1440 find the loss as a percentage of the cost pr
A car is bought for $2400 and sold one year later $1440 find the loss as a percentage of the cost price. (2400 - 1440)/2400 960/2400 0.4 As a percentage, we multiply by 100 to get [B]40%[/B]

A car is purchased for $24,000 . Each year it loses 30% of its value. After how many years will t
A car is purchased for $24,000 . Each year it loses 30% of its value. After how many years will the car be worth $7300 or less? (Use the calculator provided if necessary.) Write the smallest possible whole number answer. Set up the depreciation equation D(t) where t is the number of years in the life of the car: D(t) = 24,000 * (1 - 0.3)^t D(t) = 24000 * (0.7)^t The problem asks for D(t)<=7300 24000 * (0.7)^t = 7300 Divide each side by 24000 (0.7)^t = 7300/24000 (0.7)^t= 0.30416666666 Take the natural log of both sides: LN(0.7^t) = -1.190179482215518 Using the natural log identities, we have: t * LN(0.7) = -1.190179482215518 t * -0.35667494393873245= -1.190179482215518 Divide each side by -0.35667494393873245 t = 3.33687437943 [B]Rounding this up, we have t = 4[/B]

A car is purchased for $19000. After each year, the resale value decreases by 30% . What will the re
A car is purchased for $19000. After each year, the resale value decreases by 30% . What will the resale value be after 4 years? Set up a book value function B(t) where t is the number of years after purchase date. If an asset decreases by 30%, we subtract it from the original 100% of the starting value at time t: B(t) = 19,000(1-0.3)^t Simplifying this, we get: B(t) = 19,000(0.7)^t <-- I[I]f an asset decreases by 30%, it keeps 70% of it's value from the prior period[/I] The problem asks for B(4): B(4) = 19,000(0.7)^4 B(4) = 19,000(0.2401) B(4) = [B]4,561.90[/B]

A car is purchased for 27,000$. After each year the resale value decreases by 20%. What will the res
A car is purchased for 27,000$. After each year the resale value decreases by 20%. What will the resale value be after 3 years? If it decreases by 20%, it holds 100% - 20% = 80% of the value each year. So we have an equation R(t) where t is the time after purchase: R(t) = 27,000 * (0.8)^t The problem asks for R(3): R(3) = 27,000 * (0.8)^3 R(3) = 27,000 * 0.512 R(3) = [B]13.824[/B]

A car is traveling 60 km per hour. How many hours will it take for the car to reach a point that is
A car is traveling 60 km per hour. How many hours will it take for the car to reach a point that is 180 km away? Rate * Time = Distance so we have t for time as: 60t = 180 To solve this equation for t, we [URL='https://www.mathcelebrity.com/1unk.php?num=60t%3D180&pl=Solve']type it in the search engine[/URL] and we get: t = [B]3[/B]

a car is traveling 75 kilometers per hour. How many meters does the car travel in one minute
a car is traveling 75 kilometers per hour. How many meters does the car travel in one minute convert from Kilometers to meters 1 kilometer = 1000 meters 75 kilometers = 1000 meters * 75 = 75000 meters convert from hours to minutes 1 hour = 60 minutes, the car travels: 75,000 meters / 60 minutes = [B]1,250 meters / minute[/B]

A car is traveling on a freeway at 50 mph with the cruise control set at 50 mph. Another car is trav
A car is traveling on a freeway at 50 mph with the cruise control set at 50 mph. Another car is traveling at 90 mph with the cruise control set at 90 mph. Which car has a higher acceleration? Acceleration means a change in speed. Neither car has a change in speed, [B]so both cars have the same acceleration which is 0[/B]

a car is worth 24000 and it depreciates 3000 a year how long till it costs 9000
a car is worth 24000 and it depreciates 3000 a year how long till it costs 9000 Let y be the number of years. We want to know y when: 24000 - 3000y = 9000 Typing [URL='https://www.mathcelebrity.com/1unk.php?num=24000-3000y%3D9000&pl=Solve']this equation into our search engine[/URL], we get: y = [B]5[/B]

A car rents $35 per day plus 15 cents per mile driven
A car rents $35 per day plus 15 cents per mile driven Set up the cost function C(m) where m is the number of miles driven: C(m) = Cost per mile * m + Daily Fee [B]C(m) = 0.15m + 35[/B]

A car repair bill was $441. This included $153 for parts and four hours of labor . Find the hourly r
A car repair bill was $441. This included $153 for parts and four hours of labor . Find the hourly rate I was charge for labor Subtract the cost of parts from the total repair bill to get the labor cost: Labor Cost = Total Bill - Parts Cost Labor Cost = 441 - 153 Labor Cost = 288 Labor Cost can be broken down into Labor divided by hours Hourly Labor Rate = Labor Cost / Labor Hours Hourly Labor Rate = = 288 / 4 Hourly Labor Rate = [B]72[/B]

A car salesman earns $800 per month plus a 10% commission on the value of sales he makes for the mon
A car salesman earns $800 per month plus a 10% commission on the value of sales he makes for the month. If he is aiming to earn a minimum of $3200 a month, what is the possible value of sales that will enable this? to start, we have: [LIST] [*]Let the salesman's monthly sales be s. [*]With a 10% discount as a decimal of 0.1 [*]The phrase [I]a minimum[/I] also means [I]at least[/I] or [I]greater than or equal to[/I]. This tells us we want an inequality [*]We want 10% times s + 800 per month is greater than or equal to 3200 [/LIST] We want the inequality: 0.1s + 800 >= 3200 To solve for s, we [URL='https://www.mathcelebrity.com/1unk.php?num=0.1s%2B800%3E%3D3200&pl=Solve']type this inequality into our search engine[/URL] and we get: [B]s >= 24000[/B]

A car travels 16 m/s and travels 824 m. How long was the car moving?
A car travels 16 m/s and travels 824 m. How long was the car moving? Distance = Rate * Time, so we have: 824m = 16m/s * t Using our [URL='https://www.mathcelebrity.com/drt.php?d=+824&r=16&t=&pl=Calculate+the+missing+Item+from+D%3DRT']distance calculator[/URL], we get: [B]51.5 seconds[/B]

A car travels 71 feet each second.How many feet does it travel in 12 seconds?
A car travels 71 feet each second.How many feet does it travel in 12 seconds? Distance = Rate * Time We're given a rate of 71 feet per second and a time of 12 seconds. So we plug this in: Distance = 71 feet/second * 12 seconds [URL='https://www.mathcelebrity.com/drt.php?d=+&r=71&t=+12&pl=Calculate+the+missing+Item+from+D%3DRT']Distance[/URL] = [B]852 feet[/B]

A car travels at 40 kilometers per hour. Write an expression for the distance traveled after h hours
A car travels at 40 kilometers per hour. Write an expression for the distance traveled after h hours. Distance = rate * time, so we have: Distance = 40km/h * h Distance = [B]40h[/B]

a car was bought for $24300 and sold at a loss of $2290. Find the selling price.
a car was bought for $24300 and sold at a loss of $2290. Find the selling price. A loss means the car was sold for less than the buying price. Let the selling price be S. we have: 24300 - S = 2290 [URL='https://www.mathcelebrity.com/1unk.php?num=24300-s%3D2290&pl=Solve']Typing this equation into our search engine[/URL], we get: s = [B]22,010[/B]

A car who’s original value was $25600 decreases in value by $90 per month. How Long will it take bef
A car who’s original value was $25600 decreases in value by $90 per month. How Long will it take before the cars value falls below $15000 Let m be the number of months.We have our Book Value B(m) given by: B(m) = 25600 - 90m We want to know when the Book value is less than 15,000. So we have an inequality: 25600 - 90m < 15000 Typing [URL='https://www.mathcelebrity.com/1unk.php?num=25600-90m%3C15000&pl=Solve']this inequality into our search engine and solving for m[/URL], we get: [B]m > 117.78 or m 118 months[/B]

A car worth $43,000 brand new, depreciates at a rate of $2000 per year. What is the formula that des
A car worth $43,000 brand new, depreciates at a rate of $2000 per year. What is the formula that describes the relationship between the value of the car (C) and the time after it has been purchased (t)? Let t be the number of years since purchase. Depreciation means the value decreases, so we have: [B]C = 43000 - 2000t[/B]

a card is chosen at a random from a deck of 52 cards. it is then replaced and a second card is chose
a card is chosen at a random from a deck of 52 cards. it is then replaced and a second card is chosen. what is the probability of getting a jack and then an eight? Calculate the probability of drawing a jack from a full deck There are 4 jacks in a deck of 52 cards P(J) = 4/52 P(J) = 1/13 <-- We simplify 4/52 by dividing top and bottom of the fraction by 4 Calculate the probability of drawing an eight from a full deck There are 4 eights in a deck of 52 cards. We[I] replaced[/I] the first card giving us 52 cards to choose from. P(8) = 4/52 P(8) = 1/13 <-- We simplify 4/52 by dividing top and bottom of the fraction by 4 Since each event is independent, we multiply: P(J, 8) = P(J) * P(8) P(J, 8) = 1/13 * 1/13 P(J, 8) = [B]1/169[/B]

a card is drawn at random from a standard 52 card deck. find the probability that the card is not a
a card is drawn at random from a standard 52 card deck. find the probability that the card is not a king. There are 4 kings in a standard 52 card deck. To not get a king, we'd have 52 - 4 = 48 possible cards. The probability of not drawing a King is 48/52. But we can simplify this. So we [URL='https://www.mathcelebrity.com/fraction.php?frac1=48%2F52&frac2=3%2F8&pl=Simplify']type the fraction 48/52 into our search engine[/URL], and get: [B]12/13[/B]

A card is drawn from a pack of 52 cards. The probability that the card drawn is a red card is
A card is drawn from a pack of 52 cards. The probability that the card drawn is a red card is The deck is split evenly between red and black cards. So we have 52/2 = 26 red cards P(Red) = # of Red Cards / Total Deck Cards P(Red) = 26/52 We can simplify this fraction. [URL='https://www.mathcelebrity.com/fraction.php?frac1=26%2F52&frac2=3%2F8&pl=Simplify']Using our fraction calculator[/URL], we get: P(Red) = [B]1/2 or 0.5[/B]

A card is drawn from a standard deck of 52 cards. What is the probability of drawing an ace or a 6
A card is drawn from a standard deck of 52 cards. What is the probability of drawing an ace or a 6? There are 4 Ace's in a standard 52 card deck. There are 4 6's as well. So we have 4 + 4 = 8 possible cards out of 52: 8/52 To simplify, [URL='https://www.mathcelebrity.com/fraction.php?frac1=8%2F52&frac2=3%2F8&pl=Simplify']we type this into our search engine[/URL] and we get: [B]2/13[/B]

A card is picked from a deck of 52 cards. Find the probability of getting a black ace or a red queen
A card is picked from a deck of 52 cards. Find the probability of getting a black ace or a red queen. In a standard deck of 52 cards, we have: [LIST] [*]2 black Aces with probability 2/52 = 1/26 [URL='https://www.mathcelebrity.com/fraction.php?frac1=2%2F52&frac2=3%2F8&pl=Simplify']using our fraction simplifier[/URL] [*]2 red Queens with probability 2/52 = 1/26 [URL='http://using our fraction simplifier']using our fraction simplifier[/URL] [/LIST] The problems asks for P(Red Queen Or Black Ace). Or means we add, so we have: P(Red Queen Or Black Ace) = P(Red Queen) + P(Black Ace) P(Red Queen Or P Black Ace) = 1/26 + 1/26 P(Red Queen Or P Black Ace) = 2/26 P(Red Queen Or P Black Ace) = [B]1/13[/B] [URL='https://www.mathcelebrity.com/fraction.php?frac1=2%2F26&frac2=3%2F8&pl=Simplify']using our fraction simplifier[/URL]

a card selected from a deck of 52 cards what is the probability it is a black card or face card
a card selected from a deck of 52 cards what is the probability it is a black card or face card Facts: [LIST] [*]Half the cards in the deck are black (26/52) [*]There are 12 face cards (K, Q, J) in a deck (12/52) [*]Black and Face = 6/52 (Duplicates from above) [/LIST] P(Black or Face) = P(Black) + P(Face) - P(Black And Face) P(Black or Face) = 26/52 + 12/52 - 6/52 P(Black or Face) = 32/52 We can simplify this. We use our [URL='https://www.mathcelebrity.com/fraction.php?frac1=32%2F52&frac2=3%2F8&pl=Simplify']fraction simplifier[/URL] to get: P(Black or Face) = [B]8/13[/B]

a carnival charges $6 admission and $2.50 per ride. You have $50 to spend at the carnival. Which of
a carnival charges $6 admission and $2.50 per ride. You have $50 to spend at the carnival. Which of the following inequalities represents the situation if r is the number of rides? We set up our inequality using less than or equal to, since our cash is capped at $50. We use S for our : Cost per ride * r + Admission <= 50 Plugging in our numbers, we get: 2.50r + 6 <= 50 [B][/B] Now, if the problem asks you to put this in terms of r, then [URL='https://www.mathcelebrity.com/1unk.php?num=2.50r%2B6%3C%3D50&pl=Solve']we plug this inequality into our search engine[/URL] and we get: r <= 17.6 Since we cannot do fractional rides, we round down to 17: [B]r <= 17[/B]

A carnival charges a $15 admission price. Each game at the carnival costs $4. How many games would a
A carnival charges a $15 admission price. Each game at the carnival costs $4. How many games would a person have to play to spend at least $40? Let g be the number of games. The Spend function S(g) is: S(g) = Cost per game * number of games + admission price S(g) = 4g + 15 The problem asks for g when S(g) is at least 40. At least is an inequality using the >= sign: 4g + 15 >= 40 To solve this inequality for g, we type it in our search engine and we get: g >= 6.25 Since you can't play a partial game, we round up and get: [B]g >= 7[/B]

A carpenter bought a piece of wood that was 43.32 centimeters long. Then she sawed 5.26 centimeters
A carpenter bought a piece of wood that was 43.32 centimeters long. Then she sawed 5.26 centimeters off the end. How long is the piece of wood now? When you saw off the end, the length decrease. So we subtract: New length = Original length - Sawed piec New length = 43.32 - 5.26 New length = [B]38.06[/B]

A carpenter wants to cut a 45 inch board into two pieces such that the longer piece will be 7 inches
A carpenter wants to cut a 45 inch board into two pieces such that the longer piece will be 7 inches longer than the shorter. How long should each piece be Let the shorter piece of board length be s. Then the larger piece is: [LIST] [*]l = s + 7 [/LIST] And we know that: Shorter Piece + Longer Piece = 25 Substituting our values above, we have: s + s + 7 = 45 to solve this equation for s, we type it in our search engine and we get: s = [B]19[/B] Plugging this into our equation for l above means that: l = 19 + 7 l =[B] 26[/B]

A carpet cleaner charges $75 to clean the first 180 sq ft of carpet. There is an additional charge
A carpet cleaner charges $75 to clean the first 180 sq ft of carpet. There is an additional charge of 25¢ per square foot for any footage that exceeds 180 sq ft and $1.30 per step for any carpeting on a staircase. A customers cleaning bill was $253.95. This included the cleaning of a staircase with 14 steps. In addition to the staircase, how many square feet of carpet did the customer have cleaned? Calculate the cost of the staircase cleaning. Staircase cost = $1.30 * steps Staircase cost = $1.30 * 14 Staircase cost = $18.20 Subtract this from the cost of the total cleaning bill of $253.95. We do this to isolate the cost of the carpet. Carpet cost = $253.95 - $18.20 Carpet cost = $235.75 Now, the remaining carpet cost can be written as: 75 + $0.25(s - 180) = $235.75 <-- were s is the total square foot of carpet cleaned Multiply through and simplify: 75 + 0.25s - 45 = $235.75 Combine like terms: 0.25s + 30 = 235.75 [URL='https://www.mathcelebrity.com/1unk.php?num=0.25s%2B30%3D235.75&pl=Solve']Type this equation into our search engine[/URL] to solve for s, and we get: s = [B]823[/B]

A car’s purchase price is $24,000. At the end of each year, the value of the car is three-quarters o
A car’s purchase price is $24,000. At the end of each year, the value of the car is three-quarters of the value at the beginning of the year. Write the first four terms of the sequence of the car’s value at the end of each year. three-quarters means 3/4 or 0.75. So we have the following function P(y) where y is the number of years since purchase price: P(y) = 24000 * 0.75^y First four terms: P(1) = 24000 * 0.75 = [B]18000[/B] P(2) = 18000 * 0.75 = [B]13500[/B] P(3) = 13500 * 0.75 = [B]10125[/B] P(4) = 10125 * 0.75 = [B]7593.75[/B]

A case of meatball costs $19.09. There are 320 meatballs in a case. How much does 1 meatball cost?
A case of meatball costs $19.09. There are 320 meatballs in a case. How much does 1 meatball cost? Cost per meatball = Cost of case / Total Meatballs per case Cost per meatball = $19.09 / 320 Cost per meatball = [B]$0.06 per meatball[/B]

a cash prize of $4600 is to be awarded at a fundraiser. if 2300 tickets are sold at $7 each, find th
a cash prize of $4600 is to be awarded at a fundraiser. if 2300 tickets are sold at $7 each, find the expected value. Expected Value E(x) is: E(x) = Probability of winning * Winning Price - Probability of losing * Ticket Price [U]Since only 1 cash price will be given, 2299 will be losers:[/U] E(x) = 4600 * (1/2300) - 2299/2300 * 7 E(x) = 2 - 0.99956521739 * 7 E(x) - 2 - 7 E(x) = [B]-5[/B]

A cash register contains $5 bills and $20 bills with a total value of $180 . If there are 15 bills t
A cash register contains $5 bills and $20 bills with a total value of $180 . If there are 15 bills total, then how many of each does the register contain? Let f be the number of $5 dollar bills and t be the number of $20 bills. We're given the following equations: [LIST=1] [*]f + t = 15 [*]5f + 20t = 180 [/LIST] We can solve this system of equations 3 ways. We get [B]t = 7[/B] and [B]f = 8[/B]. [LIST] [*][URL='https://www.mathcelebrity.com/simultaneous-equations.php?term1=f+%2B+t+%3D+15&term2=5f+%2B+20t+%3D+180&pl=Substitution']Substitution Method[/URL] [*][URL='https://www.mathcelebrity.com/simultaneous-equations.php?term1=f+%2B+t+%3D+15&term2=5f+%2B+20t+%3D+180&pl=Elimination']Elimination Method[/URL] [*][URL='https://www.mathcelebrity.com/simultaneous-equations.php?term1=f+%2B+t+%3D+15&term2=5f+%2B+20t+%3D+180&pl=Cramers+Method']Cramers Method[/URL] [/LIST]

A cashier has 44 bills, all of which are $10 or $20 bills. The total value of the money is $730. How
A cashier has 44 bills, all of which are $10 or $20 bills. The total value of the money is $730. How many of each type of bill does the cashier have? Let a be the amount of $10 bills and b be the amount of $20 bills. We're given two equations: [LIST=1] [*]a + b = 44 [*]10a + 20b = 730 [/LIST] We rearrange equation 1 in terms of a. We subtract b from each side and we get: [LIST=1] [*]a = 44 - b [*]10a + 20b = 730 [/LIST] Now we substitute equation (1) for a into equation (2): 10(44 - b) + 20b = 730 Multiply through to remove the parentheses: 440 - 10b + 20b = 730 Group like terms: 440 + 10b = 730 Now, to solve for b, we [URL='https://www.mathcelebrity.com/1unk.php?num=440%2B10b%3D730&pl=Solve']type this equation into our search engine[/URL] and we get: b = [B]29 [/B] To get a, we take b = 29 and substitute it into equation (1) above: a = 44 - 29 a = [B]15 [/B] So we have [B]15 ten-dollar bills[/B] and [B]29 twenty-dollar bills[/B]

A cashier has a total of 52 bills in her cash drawer. There are only $10 bills and $5 bills in her
A cashier has a total of 52 bills in her cash drawer. There are only $10 bills and $5 bills in her drawer. The value of the bills is $320. How many $10 bills are in the drawer? Let f be the amount of $5 bills in her drawer. Let t be the amount of $10 bills in her drawer. We're given two equations: [LIST=1] [*]f + t = 52 [*]5f + 10t = 320 [/LIST] We have a system of equations. We can solve this 3 ways below: [LIST] [*][URL='https://www.mathcelebrity.com/simultaneous-equations.php?term1=f+%2B+t+%3D+52&term2=5f+%2B+10t+%3D+320&pl=Substitution']Substitution Method[/URL] [*][URL='https://www.mathcelebrity.com/simultaneous-equations.php?term1=f+%2B+t+%3D+52&term2=5f+%2B+10t+%3D+320&pl=Elimination']Elimination Method[/URL] [*][URL='https://www.mathcelebrity.com/simultaneous-equations.php?term1=f+%2B+t+%3D+52&term2=5f+%2B+10t+%3D+320&pl=Cramers+Method']Cramers Rule[/URL] [/LIST] No matter what method we choose, we get: f = 40 and t = 12 So the answer for how many $10 bills are in the drawer is [B]12[/B]. Let's check our work for equation 1: 40 + 12 ? 52 52 = 52 <-- Confirmed Let's check our work for equation 2: 5(40) + 10(12) ? 320 200 + 120 ? 320 320 = 320 <-- Confirmed

A catering service offers 3 appetizers, 6 main courses, and 4 desserts. A customer is to select 2 ap
A catering service offers 3 appetizers, 6 main courses, and 4 desserts. A customer is to select 2 appetizers, 3 main courses, and 3 desserts for a banquet. In how many ways can this be done? We use the combinations formula, and since each event is independent of the others, we multiply: 2 appetizers, 3 main courses, and 3 desserts = [URL='https://www.mathcelebrity.com/permutation.php?num=3&den=2&pl=Combinations']3C2[/URL] * [URL='https://www.mathcelebrity.com/permutation.php?num=6&den=3&pl=Combinations']6C3[/URL] * [URL='https://www.mathcelebrity.com/permutation.php?num=4&den=3&pl=Combinations']4C3[/URL] 2 appetizers, 3 main courses, and 3 desserts = 3 * 20 * 4 2 appetizers, 3 main courses, and 3 desserts = [B]240[/B]

A catering service offers 4 appetizers, 12 main courses, and 9 desserts. A customer is to select 3 a
A catering service offers 4 appetizers, 12 main courses, and 9 desserts. A customer is to select 3 appetizers, 10 main courses, and 5 desserts for a banquet. In how many ways can this be done? We use combinations, so we have: [LIST] [*][URL='https://www.mathcelebrity.com/permutation.php?num=4&den=3&pl=Combinations']4C3 appetizers[/URL] = 4 [*][URL='https://www.mathcelebrity.com/permutation.php?num=12&den=10&pl=Combinations']12C10 main courses[/URL] = 66 [*][URL='https://www.mathcelebrity.com/permutation.php?num=9&den=5&pl=Combinations']9C5 desserts[/URL] = 126 [/LIST] We multiply each of these together to get our total combinations: 4 * 66 * 126 = [B]33,264[/B]

A celebrity 50,000 followers on Instagram. The number of follower increases 45% each year. How many
A celebrity 50,000 followers on Instagram. The number of follower increases 45% each year. How many followers will they have after 8 years? We set up a growth equation for followers F(y), where y is the number of years passed since now: F(y) = 50000 * (1.45)^y <-- since 45% is 0.45 The problem asks for F(8): F(8) = 50000 * 1.45^8 F(8) = 50000 * 19.5408755063 F(8) = [B]977,044[/B]

A cell phone company charges 8$ per minute. How much do you pay for 60 minutes?
A cell phone company charges 8$ per minute. How much do you pay for 60 minutes? Calculate the total bill: Total Bill = Cost per minute * number of minutes Total Bill = $8 * 60 Total Bill = [B]$480[/B]

A cell phone company charges a monthly rate of $12.95 and $0.25 a minute per call. The bill for m mi
A cell phone company charges a monthly rate of $12.95 and $0.25 a minute per call. The bill for m minutes is $21.20. Write an equation that models this situation. Let m be the number of minutes. We have the cost equation C(m): [B]0.25m + 12.95 = $21.20[/B]

A cell phone costs $20 for 400 minutes and $2 for each extra minute. Gina uses 408 minutes. How m
A cell phone costs $20 for 400 minutes and $2 for each extra minute. Gina uses 408 minutes. How much will it cost? Set up the cost function for minutes (m) if m is greater than or equal to 400 C(m) = 20 + 2(m - 400) For m = 408, we have: C(408) = 20 + 2(408 - 400) C(408) = 20 + 2(8) C(408) = [B]36[/B]

A cell phone plan charges $1.25 for the first 400 minutes and $0.25 for each additional minute, x. W
A cell phone plan charges $1.25 for the first 400 minutes and $0.25 for each additional minute, x. Which represents the cost of the cell phone plan? Let C(x) be the cost function where x is the number of minutes we have: [B]C(x) = 1.25(min(400, x)) + 0.25(Max(0, 400 - x))[/B]

A cell phone plan costs $20 a month and includes 200 free minutes. Each additional minute costs 5 ce
A cell phone plan costs $20 a month and includes 200 free minutes. Each additional minute costs 5 cents. If you use your cell phone for at least 200 minutes a month, write a function C(x) that represents the total cost per x minutes. We add the flat rate per month to 5% of the number of minutes [U]over[/U] 200: [B]C(x) = 20 + 0.05(x - 200)[/B]

A cell phone provider is offering an unlimited data plan for $70 per month or a 5 GB plan for $55 pe
A cell phone provider is offering an unlimited data plan for $70 per month or a 5 GB plan for $55 per month. However, if you go over your 5 GB of data in a month, you have to pay an extra $10 for each GB. How many GB would be used to make both plans cost the same? Let g be the number of GB. The limited plan has a cost as follows: C = 10(g - 5) + 55 C = 10g - 50 + 55 C = 10g + 5 We want to set the limited plan equal to the unlimited plan and solve for g: 10g + 5 = 70 Solve for [I]g[/I] in the equation 10g + 5 = 70 [SIZE=5][B]Step 1: Group constants:[/B][/SIZE] We need to group our constants 5 and 70. To do that, we subtract 5 from both sides 10g + 5 - 5 = 70 - 5 [SIZE=5][B]Step 2: Cancel 5 on the left side:[/B][/SIZE] 10g = 65 [SIZE=5][B]Step 3: Divide each side of the equation by 10[/B][/SIZE] 10g/10 = 65/10 g = [B]6.5[/B] Check our work for g = 6.5: 10(6.5) + 5 65 + 5 70

A cellular offers a monthly plan of $15 for 350 min. Another cellular offers a monthly plan of $20 f
A cellular offers a monthly plan of $15 for 350 min. Another cellular offers a monthly plan of $20 for 425 min. Which company offers the better plan? Let's figure out the unit cost of minutes per dollar: [LIST=1] [*]Plan 1: 350 minutes / $15 = 23.33 minutes per dollar [*]Plan 2: 425 minutes / $20 = 21.25 minutes per dollar [/LIST] [B]Plan 2 is better, because you get more minutes per dollar.[/B]

A cellular phone company charges a $49.99 monthly fee for 600 free minutes. Each additional minute c
A cellular phone company charges a $49.99 monthly fee for 600 free minutes. Each additional minute cost $.35. This month you used 750 minutes. How much do you owe? Calculate the excess minutes over the standard plan: Excess Minutes = 750 - 600 Excess Minutes = 150 Calculate additional cost: 150 additional minutes * 0.35 per additional minutes = $52.50 Add this to the standard plan cost of $49.99 $52.50 + $49.99 = [B]$102.49[/B]

A cellular phone company charges a $49.99 monthly fee for 600 free minutes. Each additional minute c
A cellular phone company charges a $49.99 monthly fee for 600 free minutes. Each additional minute costs $.35. This month you used 750 minutes. How much do you owe [U]Find the overage minutes:[/U] Overage Minutes = Total Minutes - Free Minutes Overage Minutes = 750 - 600 Overage Minutes = 150 [U]Calculate overage cost:[/U] Overage Cost = Overage Minutes * Overage cost per minute Overage Cost = 150 * 0.35 Overage Cost = $52.5 Calculate total cost (how much do you owe): Total Cost = Monthly Fee + Overage Cost Total Cost = $49.99 + $52.50 Total Cost = [B]$102.49[/B]

A cereal box has dimensions of 12" x 3" x 18". How many square inches of cardboard are used in its c
A cereal box has dimensions of 12" x 3" x 18". How many square inches of cardboard are used in its construction? A cereal box is a rectangular solid. The volume formula is V = lwh. Substituting these values of the cereal box in, we have: V = 12(3)(18) V = [B]648 cubic inches[/B]

A certain culture of the bacterium Streptococcus A initially has 8 bacteria and is observed to doubl
A certain culture of the bacterium Streptococcus A initially has 8 bacteria and is observed to double every 1.5 hours.After how many hours will the bacteria count reach 10,000. Set up the doubling times: 0 | 8 1.5 | 16 3 | 32 4.5 | 64 6 | 128 7.5 | 256 9 | 512 10.5 | 1024 12 | 2048 13.5 | 4096 15 | 8192 16.5 | 16384 So at time [B]16.5[/B], we cross 10,000 bacteria.

A certain group of woman has a 0.69% rate of red/green color blindness. If a woman is randomly selec
A certain group of woman has a 0.69% rate of red/green color blindness. If a woman is randomly selected, what is the probability that she does not have red/green color blindness? 0.69% = 0.0069. There exists a statistics theorem for an event A that states: P(A) + P(A') = 1 where A' is the event not happening In this case, A is the woman having red/green color blindness. So A' is the woman [U][B][I]not[/I][/B][/U][I] having red/green color blindness[/I] So we have: 0.0069 + P(A') = 1 Subtract 0.0069 from each side, we get: P(A') = 1 - 0.0069 P(A') = [B]0.9931[/B]

A certain Illness is spreading at a rate of 10% per hour. How long will it take to spread to 1,200 p
A certain Illness is spreading at a rate of 10% per hour. How long will it take to spread to 1,200 people if 3 people initially exposes? Round to the nearest hour. Let h be the number of hours. We have the equation: 3 * (1.1)^h = 1,200 Divide each side by 3: 1.1^h = 400 [URL='https://www.mathcelebrity.com/natlog.php?num=1.1%5Eh%3D400&pl=Calculate']Type this equation into our search engine [/URL]to solve for h: h = 62.86 To the nearest hour, we round up and get [B]h = 63[/B]

A certain number added to its square is 30
Let x be the number. We have: x^2 + x = 30 Subtract 30 from each side: x^2 + x - 30 = 0 Using our [URL='http://www.mathcelebrity.com/quadratic.php?num=x%5E2%2Bx-30%3D0&pl=Solve+Quadratic+Equation&hintnum=+0']quadratic calculator[/URL], we get potential solutions of: [B]x = 5 or x = -6[/B] Check 5: 5 + 5^2 = 5 + 25 = [B]30[/B] Check -6 -6 + -6^2 = -6 + 36 = [B]30[/B]

A certain race is a distance of 26 furlongs. How far is the race in (a) miles? (b) yards?
A certain race is a distance of 26 furlongs. How far is the race in (a) miles? (b) yards? [URL='https://www.mathcelebrity.com/linearcon.php?quant=26&pl=Calculate&type=furlong']We type in [I]26 furlongs[/I] into our search engine[/URL] and we get: [LIST] [*][B]3.25 miles[/B] [*][B]5,720 yards[/B] [/LIST]

A certain species of fish costs $3.19 each. You can spend at most $35. How many of this type of f
A certain species of fish costs $3.19 each. You can spend at most $35. How many of this type of fish can you buy for your aquarium? Let the number of fish be f. We have the following inequality where "at most" means less than or equal to: 3.19f <= 35 [URL='https://www.mathcelebrity.com/interval-notation-calculator.php?num=3.19f%3C%3D35&pl=Show+Interval+Notation']Typing this inequality into our search engine[/URL], we get: f <= 10.917 Since we need a whole number of fish, we can buy a maximum of [B]10 fish[/B].

A certain textbook cost $94. If the price increases each year by 3% of the previous year's price, fi
A certain textbook cost $94. If the price increases each year by 3% of the previous year's price, find the price after 7 years. Using our [URL='https://www.mathcelebrity.com/apprec-percent.php?num=acertaintextbookcost94.ifthepriceincreaseseachyearby3%ofthepreviousyearspricefindthepriceafter7years&pl=Calculate']appreciation calculator[/URL], we get: [B]115.61[/B]

A certain type of an elephant would weigh 8 to 12 tons. How many pounds would it weigh?
A certain type of an elephant would weigh 8 to 12 tons. How many pounds would it weigh? 1 ton = 2000 pounds. 8 tons = 8 * 2000 = 16000 pounds 12 tons = 12 * 20000 = 24000 pounds So the elephant weighs between 16000 and 24000 pounds

A chalkboard is 3 feet tall and 4 feet long. What is its perimeter
A chalkboard is 3 feet tall and 4 feet long. What is its perimeter A chalkboard is a rectangle. So the perimeter is: 2l + 2w Using [URL='https://www.mathcelebrity.com/rectangle.php?l=4&w=3&a=&p=&pl=Calculate+Rectangle']our rectangle calculator[/URL], we get: P = [B]14[/B]

A checking account is set up with an initial balance of $2400 and $200 are removed from the account
A checking account is set up with an initial balance of $2400 and $200 are removed from the account each month for rent right and equation who solution is the number of months and it takes for the account balance to reach 1000 200 is removed, so we subtract. Let m be the number of months. We want the following equation: [B]2400 - 200m = 1000 [/B] Now, we want to solve this equation for m. So [URL='https://www.mathcelebrity.com/1unk.php?num=2400-200m%3D1000&pl=Solve']we type it in our search engine[/URL] and we get: m = [B]7[/B]

A cheetah travels at a rate of 90 feet per second. The distance d traveled by the cheetah is a func
A cheetah travels at a rate of 90 feet per second. The distance d traveled by the cheetah is a function of seconds traveled t. Write a rule for the function. How far will the cheetah travel in 25 seconds? Distance, or D(t) is expressed as a function of rate and time below: Distance = Rate x Time For the cheetah, we have D(t) as: D(t) = 90ft/sec(t) The problem asks for D(25): D(25) = 90(25) D(25) = [B]2,250 feet[/B]

A chest of treasure was hidden in the year 64 BC and found in 284 AD. For how long was the chest hid
A chest of treasure was hidden in the year 64 BC and found in 284 AD. For how long was the chest hidden BC stands for Before Christ. Year 0 is when Christ was born. AD stands for After Death On a number line, the point of Christ's birth is 0. So BC is really negative AD is positive So we have: 284 - -64 284 + 64 [B]348 years[/B]

A chicken farm produces ideally 700,000 eggs per day. But this total can vary by as many as 60,000 e
A chicken farm produces ideally 700,000 eggs per day. But this total can vary by as many as 60,000 eggs. What is the maximum and minimum expected production at the farm? [U]Calculate the maximum expected production:[/U] Maximum expected production = Average + variance Maximum expected production = 700,000 + 60,000 Maximum expected production = [B]760,000[/B] [U]Calculate the minimum expected production:[/U] Minimum expected production = Average - variance Minimum expected production = 700,000 - 60,000 Minimum expected production = [B]640,000[/B]

A child's bedroom is rectangular in shape with dimensions 17 feet by 15 feet. How many feet of wallp
A child's bedroom is rectangular in shape with dimensions 17 feet by 15 feet. How many feet of wallpaper border are needed to wrap around the entire room? A rectangle has an Perimeter (P) of: P = 2l + 2w We're given l = 17 and w = 15. So we have: P = 2(17) + 2(15) P = 34 + 30 P = [B]64[/B]

A circle has a center at (6, 2) and passes through (9, 6)
A circle has a center at (6, 2) and passes through (9, 6) The radius (r) is found by [URL='https://www.mathcelebrity.com/slope.php?xone=6&yone=2&slope=+2%2F5&xtwo=9&ytwo=6&pl=You+entered+2+points']using the distance formula[/URL] to get: r = 5 And the equation of the circle is found by using the center (h, k) and radius r as: (x - h)^2 + (y - k)^2 = r^2 (x - 6)^2 + (y - 2)^2 = 5^2 [B](x - 6)^2 + (y - 2)^2 = 25[/B]

A circular balloon is inflated with air flowing at a rate of 10cm3/s. How fast is the radius of the
A circular balloon is inflated with air flowing at a rate of 10cm3/s. How fast is the radius of the ballon increasing when the radius is 2cm? [U]The volume (V) of the balloon with radius (r) is:[/U] V = 4/3?r^3 [U]Differentiating with respect to t, we get:[/U] dV/dt = 4/3? * 3r^2 * dr/dt dV/dt = 4?r^2 * dr/dt The rate of change of the volume is: dV/dt = 10cm^3s^?1 [U]So, we find dr/dt:[/U] dr/dt = 1/4?r^2 * dV/dt dr/dt = 10/4?r^2 dr/dt = 5/2?r^2 Therefore, dr/dt(2cm) is: dr/dt(2cm) = 5/2?(2)^2 dr/dt(2cm) = 5/2?4 dr/dt(2cm) = [B]5?/8[/B]

A city doubles its size every 48 years. If the population is currently 400,000, what will the popula
A city doubles its size every 48 years. If the population is currently 400,000, what will the population be in 144 years? Calculate the doubling time periods: Doubling Time Periods = Total Time / Doubling Time Doubling Time Periods = 144/48 Doubling Time Periods = 3 Calculate the city population where t is the doubling time periods: City Population = Initital Population * 2^t Plugging in our numbers, we get: City Population = 400,000 * 2^3 City Population = 400,000 * 8 City Population = [B]3,200,000[/B]

A city has a population of 240,000 people. Suppose that each year the population grows by 7.25%. Wha
A city has a population of 240,000 people. Suppose that each year the population grows by 7.25%. What will the population be after 9 years? Let's build a population function P(t), where t is the number of years since right now. P(t) = 240,000(1.0725)^t <-- 7.25% as a decimal is 0.0725 The question asks for P(9) P(9) = 240,000(1.0725)^9 P(9) = 240,000(1.87748) P(9) = [B]450,596[/B]

A city has a population of 240,000 people. Suppose that each year the population grows by 8%. What w
A city has a population of 240,000 people. Suppose that each year the population grows by 8%. What will the population be after 5 years? [U]Set up our population function[/U] P(t) = 240,000(1 + t)^n where t is population growth rate percent and n is the time in years [U]Evaluate at t = 0.08 and n = 5[/U] P(5) = 240,000(1 + 0.08)^5 P(5) = 240,000(1.08)^5 P(5) = 240,000 * 1.4693280768 [B]P(5) = 352638.73 ~ 352,639[/B]

A city has a population of 260,000 people. Suppose that each year the population grows by 8.75% . W
A city has a population of 260,000 people. Suppose that each year the population grows by 8.75% . What will the population be after 12 years? Use the calculator provided and round your answer to the nearest whole number. Using our [URL='http://www.mathcelebrity.com/population-growth-calculator.php?num=acityhasapopulationof260000people.supposethateachyearthepopulationgrowsby8.75%.whatwillthepopulationbeafter12years?usethecalculatorprovidedandroundyouranswertothenearestwholenumber&pl=Calculate']population growth calculator,[/URL] we get P = [B]711,417[/B]

A city’s budget is $8,000,000. It actually spends $12,000,000.What is the city’s deficit?
A city’s budget is $8,000,000. It actually spends $12,000,000.What is the city’s deficit? Deficit = Budget - Spend Deficit = $8,000,000 - $12,000,000 Deficit = [B]-$4,000,000[/B]

A city’s population in the 1987 was 125,524. In 2007 the population was 436,884. Determine the popul
A city’s population in the 1987 was 125,524. In 2007 the population was 436,884. Determine the population rate of increase or decrease [U]Find the population change:[/U] Population Change = New Population - Old Population Population Change = 436,884 - 125,524 Population Change = 311,360 [U]Since the population change increased, we calculate the rate of increase:[/U] Rate of increase = 100% * Population Change / Starting Population Rate of increase = 100% * 311,360 / 125,524 Rate of increase = 100% * 2.48 Rate of increase = [B]248%[/B]

a class has 24 people and 1/6 of them have blue eues. what fraction of the class has blue eyes, usin
a class has 24 people and 1/6 of them have blue eues. what fraction of the class has blue eyes, using 24 as the denominator Blue eyes = 1/6 * 24 Blue Eyes = 24/6 Blue Eyes = 4 Since 6*4 = 24, we have: 24/6 * 4/4 = [B]96/24[/B]

A class has 35 boys and girls. There are 7 more girls than boys. Find the number of girls and boys i
A class has 35 boys and girls. There are 7 more girls than boys. Find the number of girls and boys in the class Let the number of boys be b and the number of girls be g. We're given two equations: [LIST=1] [*]b + g = 35 [*]g = b + 7 (7 more girls means we add 7 to the boys) [/LIST] To solve for b, we substitute equation (2) into equation (1) for g: b + b + 7 = 35 To solve for b, we type this equation into our search engine and we get: b = [B]14[/B] Now, to solve for g, we plug b = 14 into equation (2) above: g = 14 + 7 g = [B]21[/B]

A class is made up of 6 boys and 12 girls. Half of the girls wear glasses. A student is selected at
A class is made up of 6 boys and 12 girls. Half of the girls wear glasses. A student is selected at random from the class. What is the probability that the student is a girl with glasses? 1/2 of 6 is 3. So we want the probability we pick any of the 3 girls wearing glasses. We have a total of 6 + 12 = 18 people. Our probability is [B]3/18, or 1/6[/B].

A class of n students was raising money for a field trip. They have earned $800 so far. Each student
A class of [I]n[/I] students was raising money for a field trip. They have earned $800 so far. Each student plans to work [I]x[/I] more hours at a wage of [I]y[/I] dollars per hour. When they are done, how much money will they have earned? Class of n students * x more hours worked * y dollars per hour = xyn Total dollars earned includes the $800 already earned: $800 + xyn

A classroom fish tank contains x goldfish. The tank contains 4 times as many guppies as goldfish. En
A classroom fish tank contains x goldfish. The tank contains 4 times as many guppies as goldfish. Enter an equation that represents the total number of guppies, y, in the fish tank. The phrase [I]4 times as many[/I] means we multiply the goldfish (x) by 4 to get the number of guppies (y): [B]y = 4x[/B]

A classroom had x students. Then 9 of them went home. There are now 27 students in the classroom.
A classroom had x students. Then 9 of them went home. There are now 27 students in the classroom. Take this one piece at a time: [LIST] [*]We start with x students [*]9 of them went home. This means we have 9 less students. So we subtract 9 from x: x - 9 [*]The phrase [I]there are now[/I] means an equation, so we set x - 9 equal to 27 [/LIST] x - 9 = 27 To solve this equation for x, we [URL='https://www.mathcelebrity.com/1unk.php?num=x-9%3D27&pl=Solve']type it in our search engine[/URL] and we get: x = [B]36[/B]

A clothing store buys shirts for n dollars and then marks them up 50%. To reward their employees, th
A clothing store buys shirts for [I]n[/I] dollars and then marks them up 50%. To reward their employees, the store gives a 50% discount to all employees. How much does an employee pay for a shirt? 50% = 0.5 Markup cost = (1 + 0.5)n Markup cost = 1.5n 50% discount: 1.5n/2 = [B]0.75n[/B]

A coat is on sale for 35% off. The regular price of the coat is p. Write and simplify and expression
A coat is on sale for 35% off. The regular price of the coat is p. Write and simplify and expression to represent the sale price of the coat. Show your work. The Sale price of the coat is: S = p(1 - 0.35) <-- Since 35% is 0.35 as a decimal [B]S = 0.65p[/B]

A coat normally costs $100. First, there was a 20% discount. Then, later, it was marked down 30% off
A coat normally costs $100. First, there was a 20% discount. Then, later, it was marked down 30% off of the discounted priced. How much does the coat cost now? Calculate discounted price: Discounted Price = Full Price * (1 - Discount Percentage) Discounted Price = 100 * (1 - 0.20) <-- Since 20% = 0.2 Discounted Price = 100 * (0.80) Discounted Price = 80 Now calculate marked down price off the discount price: Markdown Price = Discount Price * (1 - Markdown Percentage) Markdown Price = 80 * (1 - 0.30) <-- Since 30% = 0.3 Markdown Price = 80 * (0.70) Markdown Price = [B]56[/B]

A coffee franchise is opening a new store. The company estimates that there is a 75% chance the sto
A coffee franchise is opening a new store. The company estimates that there is a 75% chance the store will have a profit of $45,000, a 10% chance the store will break even, and a 15% chance the store will lose $2,500. Determine the expected gain or loss for this store. Calculate the expected value E(x). Expected value is the sum of each event probability times the payoff or loss: E(x) = 0.75(45,000) + 0.1(0) + 0.15(-2,500) <-- Note, break even means no profit and no loss and a loss is denoted with a negative sign E(x) = 33,750 + 0 - 375 E(x) = [B]33,375 gain[/B]

A coin is tossed 3 times. a. Draw a tree diagram and list the sample space that shows all the possib
A coin is tossed 3 times. a. Draw a tree diagram and list the sample space that shows all the possible outcomes [URL='https://www.mathcelebrity.com/cointoss.php?hts=+HTHTHH&hct=+2&tct=+1&fct=+5>=no+more+than&nmnl=+2&htpick=heads&tossct=3&montect=3&calc=5&pl=Calculate+Probability']type in "toss a coin 3 times" and pick the probability option[/URL].

A coin is tossed and a die is rolled. Find the probability pf getting a head and a number greater th
A coin is tossed and a die is rolled. Find the probability pf getting a head and a number greater than 4. Since each event is independent, we multiply the probabilities of each event. P(H) = 0.5 or 1/2 P(Dice > 4) = P(5) + P(6) = 1/6 + 1/6 = 2/6 = 1/3 P(H) AND P(Dice > 4) = 1/2 * 1/3 = [B]1/6 [MEDIA=youtube]ofsbmHmQmjs[/MEDIA][/B]

a collection of 7 pencils, every week 3 more pencils are added How many weeks will it take to have 3
a collection of 7 pencils, every week 3 more pencils are added How many weeks will it take to have 30 pencils? Set up a function, P(w), where w is the number of weeks, and P(w) is the total amount of pencils after w weeks. We have: P(w) = 3w + 7 We want to know what w is when P(w) = 30 3w + 7 = 30 [URL='https://www.mathcelebrity.com/1unk.php?num=3w%2B7%3D30&pl=Solve']Typing this equation into our search engine[/URL], we get: w = 7.6667 We round up to the nearest integer, so we get [B]w = 8[/B]

A collection of nickels and dime has a total value of $8.50. How many coins are there if there are 3
A collection of nickels and dime has a total value of $8.50. How many coins are there if there are 3 times as many nickels as dimes. Let n be the number of nickels. Let d be the number of dimes. We're give two equations: [LIST=1] [*]n = 3d [*]0.1d + 0.05n = 8.50 [/LIST] Plug equation (1) into equation (2) for n: 0.1d + 0.05(3d) = 8.50 Multiply through: 0.1d + 0.15d = 8.50 [URL='https://www.mathcelebrity.com/1unk.php?num=0.1d%2B0.15d%3D8.50&pl=Solve']Type this equation into our search engine[/URL] and we get: [B]d = 34[/B] Now, we take d = 34, and plug it back into equation (1) to solve for n: n = 3(34) [B]n = 102[/B]

A college student earned $6000 during summer vacation working as a waiter in a popular restaurant. T
A college student earned $6000 during summer vacation working as a waiter in a popular restaurant. The student invested part of the money at 8% and the rest at 6%. If the student received a total of $418 in interest at the end of the year, how much was invested at 8%? [URL='https://www.mathcelebrity.com/split-fund-interest-calculator.php?p=6000&i1=8&i2=6&itot=418&pl=Calculate']Using our split fund interest calculator[/URL], we get: [B]$2,900 invested at 8%[/B] $3,100 invested at 6%

A college student earns $21 per day delivering advertising brochures door-to-door, plus 50 cents for
A college student earns $21 per day delivering advertising brochures door-to-door, plus 50 cents for each person he interviews. How many people did he interview on a day when he earned $61.50 Let each person interviewed be p. We have an earnings equation E(p): E(p) = 0.5p + 21 The problems asks for p when E(p) = 61.50 0.5p + 21 = 61.50 To solve for p, we [URL='https://www.mathcelebrity.com/1unk.php?num=0.5p%2B21%3D61.50&pl=Solve']type this equation in our search engine[/URL] and we get: p = [B]81[/B]

A colony of 100 bacteria doubles in size every 34 hours. What will the population be 68 hours from n
A colony of 100 bacteria doubles in size every 34 hours. What will the population be 68 hours from now T(0) = 100 T(34) = 100 * 2 = 200 T(68) = 200 * 2 = [B]400[/B]

A colony of 995 bacteria doubles in size every 206 minutes. What will the population be 618 minutes
A colony of 995 bacteria doubles in size every 206 minutes. What will the population be 618 minutes from now? Calculate the doubling time periods: Doubling Time Periods = Total Minutes From Now / Doubling Period in Minutes Doubling Time Periods = 618/206 Doubling Time Periods = 3 Calculate the new population using the doubling time formula below where t is the number of doubling periods: Population = Initial Population * 2^2 Population = 995 * 2^3 Population = 995 * 8 Population = [B]7,960[/B]

A combination lock open with the correct 4 letter code. Each wheel roared through letters A-L. How m
A combination lock open with the correct 4 letter code. Each wheel roared through letters A-L. How many different 4 letter codes are possible A-L = 12 letters Possible combinations is found by: 12 * 12 * 12 * 12 = [B]20,736 combinations[/B]

A combined total of $34,000 is invested in two bonds that pay 4% and 8.5% simple interest. The annua
A combined total of $34,000 is invested in two bonds that pay 4% and 8.5% simple interest. The annual interest is $2,350. How much is invested in each bond Using our split fund interest calculator, we get; [LIST] [*][B]12,000[/B] in Fund 1 [*][B]22,000[/B] in Fund 2 [/LIST]

A combined total of $40,000 is invested in two bonds that pay 6% and 9% simple interest. The annual
A combined total of $40,000 is invested in two bonds that pay 6% and 9% simple interest. The annual interest is $3,210.00. How much is invested in each bond? Using our [URL='https://www.mathcelebrity.com/split-fund-interest-calculator.php?p=40000&i1=6&i2=9&itot=3210&pl=Calculate']split fund interest calculator[/URL], we get: Fund 1 at 6% = [B]13,000[/B] Fund 2 at 9% =[B] 27,000[/B]

A comet passes earth every 70 years. another comet passes earth every 75 years of both comets pass e
A comet passes earth every 70 years. another comet passes earth every 75 years of both comets pass earth this year how many years will it be before they pass on the same year again. We want the least common multiple of (70, 75). [URL='https://www.mathcelebrity.com/gcflcm.php?num1=70&num2=75&num3=&pl=GCF+and+LCM']Using our LCM calculator[/URL], we find the answer is [B]1,050 years[/B]

A committee consisting of 4 faculty members and 5 students is to be formed. Every committee position
A committee consisting of 4 faculty members and 5 students is to be formed. Every committee position has the same duties and voting rights. There are 12 faculty members and 15 students eligible to serve on the committee. In how many ways can the committee be formed? We'll use combinations, so we have: [LIST] [*][URL='https://www.mathcelebrity.com/permutation.php?num=12&den=4&pl=Combinations']12 faculty members choose 4 faculty members --> 12 C 4[/URL] = 495 [*][URL='https://www.mathcelebrity.com/permutation.php?num=15&den=5&pl=Combinations']15 students choose 5 students --> 15 C 5[/URL] = 3,003 [/LIST] To get the total committees, we multiply the total faculty member choices by the total student choices: Total committees = total faculty members * total students Total committees = 495 * 3,003 Total committees = [B]1,486,485[/B]

A compact disc is designed to last an average of 4 years with a standard deviation of 0.8 years. Wha
A compact disc is designed to last an average of 4 years with a standard deviation of 0.8 years. What is the probability that a CD will last less than 3 years? Using our [URL='http://www.mathcelebrity.com/probnormdist.php?xone=3&mean=4&stdev=0.8&n=1&pl=P%28X+%3C+Z%29']Z-score and Normal distribution calculator[/URL], we get: [B]0.10565[/B]

A company charges $7 for a T-Shirt and ships and order for $22. A school principal ordered a number
A company charges $7 for a T-Shirt and ships and order for $22. A school principal ordered a number of T-shirts for the school store. The total cost of the order was $1,520. Which equation can be used to find the number one f shirts ordered? Set up the cost equation C(f) where f is the number of shirts: C(f) = Cost per shirt * f + Shipping We're given C(f) = 1520, Shipping = 22, and cost per shirt is 7, so we have: [B]7f + 22 = 1520 [/B] To solve for f, we [URL='https://www.mathcelebrity.com/1unk.php?num=7f%2B22%3D1520&pl=Solve']type this equation in our search engine[/URL] and we get: f = [B]214[/B]

A company delivered 498 packages on Monday. On Tuesday they delivered 639 and on Wednesday they deli
A company delivered 498 packages on Monday. On Tuesday they delivered 639 and on Wednesday they delivered 436. How many packages did the company deliver in all? Add up all the packages: 498 + 639 + 436 = [B]1573[/B]

A company had sales of $19,808 million in 1999 and $28,858 million in 2007. Use the Midpoint Formula
A company had sales of $19,808 million in 1999 and $28,858 million in 2007. Use the Midpoint Formula to estimate the sales in 2003 2003 is the midpoint of 1999 and 2007, so we use our [URL='https://www.mathcelebrity.com/mptnline.php?ept1=19808&empt=&ept2=28858&pl=Calculate+missing+Number+Line+item']midpoint calculator[/URL] to get: [B]24,333[/B] sales in 2003

A company has 12,600 employees. Of these, 1/4 drive alone to work, 1/6 car pool, 1/8 use public tran
A company has 12,600 employees. Of these, 1/4 drive alone to work, 1/6 car pool, 1/8 use public transportation, 1/9 cycle, and the remainder use other methods of transportation. How many employees use each method of transportation? Find the remainder fraction: Remainder = 1 - (1/4 + 1/6 + 1/8 + 1/9) The least common multiple of 4, 6, 8, 9 is 72. So we divide 72 by each fraction denominator to get our multiplier: 1/4 = 18/72 1/6 = 12/72 1/8 = 9/72 1/9 = 8/72 Add those all up: (18 + 12 + 9 + 8)/72 47/72 Now subtract the other methods out from 1 to get the remainder of who use other methods: Remainder = 1 - 47/72 Since 1 = 72/72, we have: (72 - 47)/72 [B]25/72[/B]

A company has 3,100 employees and is expected to grow at a rate of 0.04 for the next six years. How
A company has 3,100 employees and is expected to grow at a rate of 0.04 for the next six years. How many employees will they have in 6 years? Round to the nearest whole number. We build the following exponential equation: Final Balance = Initial Balance * (1 + growth rate)^time Final Balance = 3100(1.04)^6 Final Balance = 3100 * 1.2653190185 Final Balance = 3922.48895734 The problem asks us to round to the nearest whole number. Since 0.488 is less than 0.5, we round [U]down.[/U] Final Balance = [B]3,922[/B]

A company has 81 employees of whom x are members of a union how many are not in the union
A company has 81 employees of whom x are members of a union how many are not in the union You can either be a union member or a non-union member. This is our sample space. If we have 81 employees and x are union members, this means that: Non-Union membes = [B]81 - x[/B]

A company has 95 employees. 4 get sacked. 57 quit. 52 are hired. How many employees are there now?
A company has 95 employees. 4 get sacked. 57 quit. 52 are hired. How many employees are there now? Take this in pieces [LIST=1] [*]We start with 95 employees [*]4 get sacked (fired). So we subtract: 95 - 4 = 91 [*]57 quit. We subtract: 91 - 57 = 34 [*]52 are hired. We add: 34 + 52 = [B]86 employees[/B] [/LIST]

A company has a fixed cost of $26,000 / month when it is producing printed tapestries. Each item tha
A company has a fixed cost of $26,000 / month when it is producing printed tapestries. Each item that it makes has its own cost of $34. One month the company filled an order for 2400 of its tapestries, selling each item for $63. How much profit was generated by the order? [U]Set up Cost function C(t) where t is the number of tapestries:[/U] C(t) = Cost per tapestry * number of tapestries + Fixed Cost C(t) = 34t + 26000 [U]Set up Revenue function R(t) where t is the number of tapestries:[/U] R(t) = Sale Price * number of tapestries R(t) = 63t [U]Set up Profit function P(t) where t is the number of tapestries:[/U] P(t) = R(t) - C(t) P(t) = 63t - (34t + 26000) P(t) = 63t - 34t - 26000 P(t) = 29t - 26000 [U]The problem asks for profit when t = 2400:[/U] P(2400) = 29(2400) - 26000 P(2400) = 69,600 - 26,000 P(2400) = [B]43,600[/B]

A company has a fixed cost of $34,000 and a production cost of $6 for each unit it manufactures. A u
A company has a fixed cost of $34,000 and a production cost of $6 for each unit it manufactures. A unit sells for $15 Set up the cost function C(u) where u is the number of units is: C(u) = Cost per unit * u + Fixed Cost C(u) = [B]6u + 34000[/B] Set up the revenue function R(u) where u is the number of units is: R(u) = Sale price per unit * u R(u) = [B]15u[/B]

a company has revenue given by R(x)=500x dollars and total cost given by C(x)=48,000 100x dollars, w
a company has revenue given by R(x)=500x dollars and total cost given by C(x)=48,000 + 100x dollars, where x is the number of units produced and sold. How many units will give a profit Profit P(x) is given by: R(x) - C(x) So we have: P(x) = 500x - (100x + 48,000) P(x) = 500x - 100x - 48,000 P(x) = 400x - 48,000 A profit is found when P(x) > 0, so we have: 400x - 48000 > 0 To solve this inequality, [URL='https://www.mathcelebrity.com/1unk.php?num=400x-48000%3E0&pl=Solve']we type it into our search engine [/URL]and we get: [B]x > 120[/B]

A company is planning to manufacture a certain product. The fixed costs will be $474778 and it will
A company is planning to manufacture a certain product. The fixed costs will be $474778 and it will cost $293 to produce each product. Each will be sold for $820. Find a linear function for the profit, P , in terms of units sold, x . [U]Set up the cost function C(x):[/U] C(x) = Cost per product * x + Fixed Costs C(x) = 293x + 474778 [U]Set up the Revenue function R(x):[/U] R(x) = Sale Price * x R(x) = 820x [U]Set up the Profit Function P(x):[/U] P(x) = Revenue - Cost P(x) = R(x) - C(x) P(x) = 820x - (293x + 474778) P(x) = 820x - 293x - 474778 [B]P(x) = 527x - 474778[/B]

a company made a profit of $4 million per month for 8 months, then lost $10 million per month for 4
a company made a profit of $4 million per month for 8 months, then lost $10 million per month for 4 months. What was their result for the year? Profits = 4 million per month * 8 months = 32,000,000 Losses = 10 million per month * 4 months = 40,000,000 Calculate results for the year: Result for the year = Profits - Losses Result for the year = 32,000,000 - 40,000,000 Result for the year = [B]8,000,000[/B]

A company makes a puzzle that is made of 53 small plastic cubes. The puzzles are shipped in boxes th
A company makes a puzzle that is made of 53 small plastic cubes. The puzzles are shipped in boxes that each contain 52 puzzles. That boxes are loaded into trucks that each contain 53 boxes. What is the total number of small plastic cubes in each truck? 1 truck has 53 boxes, and each box contains 52 puzzles, and each puzzle has 53 small plastic cubes. We have 53 * 52 * 53 = [B]146,068 plastic cubes[/B]

A company makes toy boats. Their monthly fixed costs are $1500. The variable costs are $50 per boat.
A company makes toy boats. Their monthly fixed costs are $1500. The variable costs are $50 per boat. They sell boats for $75 a piece. How many boats must be sold each month to break even? [U]Set up Cost function C(b) where t is the number of tapestries:[/U] C(b) = Cost per boat * number of boats + Fixed Cost C(b) = 50b + 1500 [U]Set up Revenue function R(b) where t is the number of tapestries:[/U] R(b) = Sale Price * number of boats R(b) = 75b [U]Break even is where Revenue equals Cost, or Revenue minus Cost is 0, so we have:[/U] R(b) - C(b) = 0 75b - (50b + 1500) = 0 75b - 50b - 1500 = 0 25b - 1500 = 0 To solve for b, we [URL='https://www.mathcelebrity.com/1unk.php?num=25b-1500%3D0&pl=Solve']type this equation in our math engine[/URL] and we get: b = [B]60[/B]

A company now has 4900 employees nationwide. It wishes to reduce the number of employees by 300 per
A company now has 4900 employees nationwide. It wishes to reduce the number of employees by 300 per year through retirements, until its total employment is 2560. How long will this take? Figure out how many reductions are needed 4900 - 2560 = 2340 We want 300 per year for retirements, so let x equal how many years we need to get 2340 reductions. 300x = 2340 Divide each side by 300 x = 7.8 years. If we want full years, we would do 8 full years

A company ordered 325 boxes of pens. Each box has 12 cases in it. Each case holds 24 pens. How many
A company ordered 325 boxes of pens. Each box has 12 cases in it. Each case holds 24 pens. How many pens did the company order 325 boxes Each box has 12 cases Each case has 24 pens We have 325 boxes * 12 cases/box * 24 pens / box 325 * 12 * 24 = [B]93,600 pens[/B]

A company specializes in personalized team uniforms. It costs the company $15 to make each uniform a
A company specializes in personalized team uniforms. It costs the company $15 to make each uniform along with their fixed costs at $640. The company plans to sell each uniform for $55. [U]The cost function for "u" uniforms C(u) is given by:[/U] C(u) = Cost per uniform * u + Fixed Costs [B]C(u) = 15u + 640[/B] Build the revenue function R(u) where u is the number of uniforms: R(u) = Sale Price per uniform * u [B]R(u) = 55u[/B] Calculate break even function: Break even is where Revenue equals cost C(u) = R(u) 15u + 640 = 55u To solve for u, we [URL='https://www.mathcelebrity.com/1unk.php?num=15u%2B640%3D55u&pl=Solve']type this equation into our search engine[/URL] and we get: u = [B]16 So we break even selling 16 uniforms[/B]

A company that makes superhero capes has 6,300 yards of fabric left. It uses 70 yards of fabric each
A company that makes superhero capes has 6,300 yards of fabric left. It uses 70 yards of fabric each day. How many days will it take the company to use all the fabric? 6300 yards / 70 yards per day = [B]90 days[/B]

A company that manufactures lamps has a fixed monthly cost of $1800. It costs $90 to produce each l
A company that manufactures lamps has a fixed monthly cost of $1800. It costs $90 to produce each lamp, and the selling price is $150 per lamp. Set up the Cost Equation C(l) where l is the price of each lamp: C(l) = Variable Cost x l + Fixed Cost C(l) = 90l + 1800 Determine the revenue function R(l) R(l) = 150l Determine the profit function P(l) Profit = Revenue - Cost P(l) = 150l - (90l + 1800) P(l) = 150l - 90l - 1800 [B]P(l) = 60l - 1800[/B] Determine the break even point: Breakeven --> R(l) = C(l) 150l = 90l + 1800 [URL='https://www.mathcelebrity.com/1unk.php?num=150l%3D90l%2B1800&pl=Solve']Type this into the search engine[/URL], and we get [B]l = 30[/B]

A companys cost function is C(x) = 16x2 + 900 dollars, where x is the number of units. Find th
A companys cost function is C(x) = 16x^2 + 900 dollars, where x is the number of units. Find the marginal cost function. Marginal Cost is the derivative of the Cost function. [B]C'(x) = 32x[/B]

A company’s number of personnel on active duty (not on sick leave or vacation leave) during the peri
A company’s number of personnel on active duty (not on sick leave or vacation leave) during the period 2000 - 2013 can be approximated by the cubic model f(x) = 2.5x^3 - 15x^2 - 80x + 1025, where x = 0 corresponds to 2000. Based on the model, how many personnel were on active duty in 2010? What is the domain of f? If x = 0 corresponds to 2000, when 2010 is 2010 - 2000 = 10. We want f(10): f(10) = 2.5(10)^3 - 15(10)^2 - 80(10) + 1025 f(10) = 2.5(1000) - 15(100) - 800 + 1025 f(10) = 2500 - 1500 - 800 + 1025 f(10) = [B]1,225[/B]

a computer is purchased for 800 and each year the resale value decreases by 25% what will be the res
a computer is purchased for 800 and each year the resale value decreases by 25% what will be the resale value after 4 years Let the resale in year y be R(y). We have: R(y) = 800 * (1 - 0.25)^y R(y) = 800 * (0.75)^y The problem asks for R(4): R(4) = 800 * (0.75)^4 R(4) = 800 * 0.31640625 R(4) = [B]$253.13[/B]

A computer randomly generates a whole number from 1 to 25. Find the probability that the computer ge
A computer randomly generates a whole number from 1 to 25. Find the probability that the computer generates a multiple of 5 [URL='https://www.mathcelebrity.com/factoriz.php?num=25&pl=Show+Factorization']Multiples of 5[/URL]: {1, 5, 25} So we have the probability of a random number multiple of 5 is [B]3/25[/B]

A computer screen has a diagonal dimension of 19 inches and a width of 15 inches. Approximately what
A computer screen has a diagonal dimension of 19 inches and a width of 15 inches. Approximately what is the height of the screen? We have a right triangle, with hypotenuse of 19, and width of 15. [URL='https://www.mathcelebrity.com/pythag.php?side1input=&side2input=15&hypinput=19&pl=Solve+Missing+Side']Using our right triangle calculator, we get [/URL][B]height = 11.662[/B]

A computer was on sale. The original cost of the computer was $900. It’s on sale for 5/6 the price.
A computer was on sale. The original cost of the computer was $900. It’s on sale for 5/6 the price. How much is the computer now? We want 5/6 of 900. We [URL='https://www.mathcelebrity.com/fraction.php?frac1=900&frac2=5/6&pl=Multiply']type this in our search engine[/URL] and we get: [B]750[/B]

A construction company can remove 2/3 tons of dirt from a construction site each hour. How long wil
A construction company can remove 2/3 tons of dirt from a construction site each hour. How long will it take them to remove 30 tons of dirt from the site? Let h be the number of hours. We have the following equation: 2/3h = 30 Multiply each side by 3: 2(3)h/3 = 30 * 3 Cancel the 3 on the left side: 2h = 90 [URL='https://www.mathcelebrity.com/1unk.php?num=2h%3D90&pl=Solve']Type 2h = 90 into the search engine[/URL], we get [B]h = 45[/B].

A construction company needs to remove 24 tons of dirt from a construction site. They can remove 3/8
A construction company needs to remove 24 tons of dirt from a construction site. They can remove 3/8 ton s of dirt each hour. How long will I it take to remove the dirt? Let h be the number of hours it takes, we have: 3/8h = 24 Multiply each side by 8/3 h = 24(8)/3 24/3 = 8, so we have: h = 8(8) h = [B]64 hours[/B]

A construction crew has just built a new road. They built 43.75 kilometers of road at a rate of 7 ki
A construction crew has just built a new road. They built 43.75 kilometers of road at a rate of 7 kilometers per week. How many weeks did it take them? Let w = weeks 7 kilometers per week * w = 43.75 To solve for w, we divide each side of the equation by 7: 7w/7 = 43.75/7 Cancel the 7's, we get: w = [B]6.25 [/B]

A construction crew has just built a new road. They built 8.75 kilometers of road in 7 weeks. At wha
A construction crew has just built a new road. They built 8.75 kilometers of road in 7 weeks. At what rate did they build the road? Rate = Km of road / weeks Rate = 8.75 km / 7 weeks Rate = [B]1.25 km per week[/B]

A construction crew must build a road in 10 months or they will be penalized $500,000. It took 10 wo
A construction crew must build a road in 10 months or they will be penalized $500,000. It took 10 workers 6 months to build half of the road. How many additional workers must be added to finish the road in the remaining 4 months? Calculate unit rate per one worker: 10 workers * 6 months = 60 months for one worker Calculate workers needed: 60 months / 4 months = 15 workers Calculate additional workers needed: Additional workers needed = New workers needed - Original workers needed Additional workers needed = 15 - 10 Additional workers needed = [B]5 additional workers[/B]

A construction worker can lift 220 lb, while an architect can only lift 40 lb. The construction work
A construction worker can lift 220 lb, while an architect can only lift 40 lb. The construction worker can lift how many times what the architect can lift? [URL='https://www.mathcelebrity.com/perc.php?num=220&den=40&pcheck=1&num1=16&pct1=80&pct2=70&den1=80&idpct1=10&hltype=1&idpct2=90&pct=82&decimal=+65.236&astart=12&aend=20&wp1=20&wp2=30&pl=Calculate']We divide 220 by 40 to get the multiplier:[/URL] 220/40 = [B]5.5 times[/B]

A contractor needs to buy nails to build a house. The nails come in small boxes and large boxes. Eac
A contractor needs to buy nails to build a house. The nails come in small boxes and large boxes. Each small box has 100 nails and each large box has 350 nails. How many nails are there in 6 small boxes and 3 large boxes? How many nails are there in a small boxes and l large boxes? [U]Calculate Small Nails[/U] Small Nails = Small Nail Boxes * Nails per Box Small Nails = 6 * 100 Small Nails = 600 [U] Calculate Large Nails[/U] Large Nails = Large Nail Boxes * Nails per Box LargeNails = 3 * 350 Large Nails = 1,050 [U]Calculate Total Nails[/U] Total Nails = Small Nails + Large Nails Total Nails = 600 + 1,050 Total Nails = [B]1,650[/B]

A contractor’s crew can frame 3 houses in a week. How long will it take them to frame 54 houses if t
A contractor’s crew can frame 3 houses in a week. How long will it take them to frame 54 houses if they frame the same number each week? 54 houses / 3 houses per week = [B]18 weeks[/B]

A cook has 2 3/4 pounds of ground beef.How many quarter-pound burgers can he make?
A cook has 2 3/4 pounds of ground beef.How many quarter-pound burgers can he make? 2 & 3/4 [URL='https://www.mathcelebrity.com/fraction.php?frac1=2%263%2F4&frac2=3%2F8&pl=Simplify']converts to 11/4 in our fraction converter[/URL]. A quarter-pound burger is 1/4 of a pound. 11/4 = 1/4 * 11, so the cook can make [B]11 quarter-pound burgers[/B]

a cookie jar contains 3 vanilla, 2 chocolate chip, and 7 gingersnap cookies. Of one cookie is taken
a cookie jar contains 3 vanilla, 2 chocolate chip, and 7 gingersnap cookies. Of one cookie is taken at random from the jar, what is the probability that it will be a vanilla cookie? Total cookies = 3 + 2 + 7 Total cookies = 12 P(Vanilla) = Vanilla Cookies / Total Cookies P(Vanilla) = 3/12 Simplified dividing top and bottom by 3, we have: P(Vanilla) = [B]1/4[/B]

A cookie jar contains 8 cookies. There are 4 children. How many cookies will each child receive?
A cookie jar contains 8 cookies. There are 4 children. How many cookies will each child receive? Cookies Per child = Total Cookies / Total Children Cookies Per child = 8/4 Cookies Per child = [B]2[/B]

A cookie recipe uses 10 times as much flour as sugar. If the total amount of these ingredients is 8
A cookie recipe uses 10 times as much flour as sugar. If the total amount of these ingredients is 8 1/4 cups, how much flour and how much sugar would it be? Let f be the number of cups of sugar. And let f be the number of cups of flour. We're given two equations: [LIST=1] [*]f = 10s [*]s + f = 8 & 1/4 [/LIST] Substitute (1) into (2): s + 10s = 8 & 1/4 11fs= 33/4 <-- 8 & 1/4 = 33/4 Cross multiply: 44s = 33 Divide each side by 44: s= 33/44 Divide top and bottom by 11 and we get s [B]= 3/4 or 0.75[/B] Now substitute this into (1): f = 10(33/44) [B]f = 330/44 or 7 & 22/44 or 7.5[/B]

A copy machine makes 28 copies per minute. how many copies does it make in 3 minutes and 45 seconds
A copy machine makes 28 copies per minute. how many copies does it make in 3 minutes and 45 seconds? 45 seconds = 45/60 = 3/4 of a minute. 3/4 = 0.75 So we have 3.75 minutes. Set up a proportion of copies to minutes where c is the number of copies made in 3 minutes and 45 seconds: 28/1 = c/3.75 [URL='https://www.mathcelebrity.com/prop.php?num1=28&num2=c&den1=1&den2=3.75&propsign=%3D&pl=Calculate+missing+proportion+value']Typing this proportion into our calculator[/URL], we get: c = [B]105[/B]

A copy machine makes 44 copies per minute. How many copies does it make in 5 minutes and 45 seconds
A copy machine makes 44 copies per minute. How many copies does it make in 5 minutes and 45 seconds Set up a proportion of copies to minutes where c is the number of copies for 5 minutes and 45 seconds. [URL='https://www.mathcelebrity.com/fraction.php?frac1=45%2F60&frac2=3%2F8&pl=Simplify']Since 45 seconds[/URL] is: 45/60 = 3/4 of a minute, we have: 5 minutes and 45 seconds = 5.75 minutes 44/1 = c/5.75 To solve this proportion, we [URL='https://www.mathcelebrity.com/prop.php?num1=44&num2=c&den1=1&den2=5.75&propsign=%3D&pl=Calculate+missing+proportion+value']type it in our search engine[/URL] and we get: c = [B]253[/B]

A corn refining company produces corn gluten cattle feed at a variable cost of $84 per ton. If fixe
A corn refining company produces corn gluten cattle feed at a variable cost of $84 per ton. If fixed costs are $110,000 per month and the feed sells for $132 per ton, how many tons should be sold each month to have a monthly profit of $560,000? [U]Set up the cost function C(t) where t is the number of tons of cattle feed:[/U] C(t) = Variable Cost * t + Fixed Costs C(t) = 84t + 110000 [U]Set up the revenue function R(t) where t is the number of tons of cattle feed:[/U] R(t) = Sale Price * t R(t) = 132t [U]Set up the profit function P(t) where t is the number of tons of cattle feed:[/U] P(t) = R(t) - C(t) P(t) = 132t - (84t + 110000) P(t) = 132t - 84t - 110000 P(t) = 48t - 110000 [U]The question asks for how many tons (t) need to be sold each month to have a monthly profit of 560,000. So we set P(t) = 560000:[/U] 48t - 110000 = 560000 [U]To solve for t, we [URL='https://www.mathcelebrity.com/1unk.php?num=48t-110000%3D560000&pl=Solve']type this equation into our search engine[/URL] and we get:[/U] t =[B] 13,958.33 If the problem asks for whole numbers, we round up one ton to get 13,959[/B]

A couple is opening a savings account for a newborn baby. They start with $3450 received in baby gif
A couple is opening a savings account for a newborn baby. They start with $3450 received in baby gifts. If no depositts or withdrawals are made, what is the balance of the account if it earns simple interest at 6% for 18 years? Using [URL='https://www.mathcelebrity.com/simpint.php?av=&p=3450&int=6&t=18&pl=Simple+Interest']our simple interest calculator[/URL], we get: [B]7,176[/B]

A coupon that was mailed to preferred customers of video village rentals is good for 15% on any vide
A coupon that was mailed to preferred customers of video village rentals is good for 15% on any video that is bought. How much savings is there using the coupon to purchase a $22 video? Savings = Full Price * Coupon Amount Savings = $22 * 0.15 Savings = $3.30

A crate contains 300 coins and stamps. The coins cost $3 each and the stamps cost $1.5 each. The tot
A crate contains 300 coins and stamps. The coins cost $3 each and the stamps cost $1.5 each. The total value of the items is $825. How many coins are there? Let c be the number of coins, and s be the number of stamps. We're given: [LIST=1] [*]c + s = 300 [*]3c + 1.5s = 825 [/LIST] We have a set of simultaneous equations, or a system of equations. We can solve this 3 ways: [LIST=1] [*][URL='https://www.mathcelebrity.com/simultaneous-equations.php?term1=c+%2B+s+%3D+300&term2=3c+%2B+1.5s+%3D+825&pl=Substitution']Substitution Method[/URL] [*][URL='https://www.mathcelebrity.com/simultaneous-equations.php?term1=c+%2B+s+%3D+300&term2=3c+%2B+1.5s+%3D+825&pl=Elimination']Elimination Method[/URL] [*][URL='https://www.mathcelebrity.com/simultaneous-equations.php?term1=c+%2B+s+%3D+300&term2=3c+%2B+1.5s+%3D+825&pl=Cramers+Method']Cramers Method[/URL] [/LIST] No matter which way we pick, we get: s = 50 c = [B]250[/B]

A credit plan charges interest rate of 36% compounded monthly. Find the effective rate.
A credit plan charges interest rate of 36% compounded monthly. Find the effective rate. [U]Calculate Monthly Nominal Rate:[/U] Monthly Nominal Rate = Annual Rate / 12 months per year Monthly Nominal Rate = 36%/12 Monthly Nominal Rate = 3% [U]Since there are 12 months in a year, we compound 12 times to get the effective rate below:[/U] Effective Rate = (1 + Monthly Nominal Rate as a Decimal)^12 - 1 Since 3% = 0.03, we have: Effective Rate = 100% * ((1 + 0.03)^12 - 1) Effective Rate = 100% * ((1.03)^12 - 1) Effective Rate = 100% * (1.42576088685 - 1) Effective Rate = 100% * (0.42576088685) Effective Rate = [B]42.58%[/B]

A cube has an edge that is x cm long. What is the capacity of C(x)?
A cube has an edge that is x cm long. What is the capacity of C(x)? Capacity is another word for volume, or the amount an object will hold. Given a side x, the capacity (volume) of a cube is: C(x) = [B]x^3[/B]

A cube is 1 meter long.What is the total length of all its edges?
A cube is 1 meter long.What is the total length of all its edges? A cube has 12 edges. 12 edges x 1 meter for each edge = [B]12 meters[/B]

A cubical storage box has edges that are 2 feet 4 inches long. What is the volume of the storage box
A cubical storage box has edges that are 2 feet 4 inches long. What is the volume of the storage box? Since 1 foot = 12 inches, we have: 2 feet 4 inches = 2(12) + 4 2 feet 4 inches = 24 + 4 2 feet 4 inches = 28 inches We type [URL='https://www.mathcelebrity.com/cube.php?num=28&pl=Side&type=side&show_All=1']cube side = 28[/URL] into our search engine to get: V = [B]21952 cubic inches[/B]

A cubicle is 6 1?2 feet by 8 3?4 feet. What is the area of the cubicle?
A cubicle is 6 1?2 feet by 8 3?4 feet. What is the area of the cubicle? Area of a cube is length times width: A = 8 & 3/4 * 6 & 1/2 We need to convert these to improper fractions. [LIST] [*]8 & 3/4 [URL='https://www.mathcelebrity.com/fraction.php?frac1=8%263%2F4&frac2=3%2F8&pl=Simplify']converted to an improper fraction[/URL] is 35/4 [*]6 & 1/2 [URL='https://www.mathcelebrity.com/fraction.php?frac1=6%261%2F2&frac2=3%2F8&pl=Simplify']converted to an improper fraction[/URL] is 13/2 [/LIST] Multiply the improper fractions together: A = 35/4 * 13/2 [URL='https://www.mathcelebrity.com/fraction.php?frac1=35%2F4&frac2=13%2F2&pl=Multiply']Using our fraction multiplier[/URL], we get: [B]455/8 sq ft[/B] If you want to convert this to a mixed fraction, we [URL='https://www.mathcelebrity.com/fraction.php?frac1=455%2F8&frac2=3%2F8&pl=Simplify']type this in our calculator [/URL]and get: [B]56 & 7/8 sq ft[/B]

A culture of bacteria doubles every hour. If there are 500 bacteria at the beginning, how many bacte
A culture of bacteria doubles every hour. If there are 500 bacteria at the beginning, how many bacteria will there be after 9 hours? Assumptions and givens; [LIST] [*]h is the number of hours. [*]B(h) is the number of bacteria at time h [*]B(0) is the starting bacteria amount [*]Doubling means multiplying by 2, so we have: [/LIST] B(h) = B(0) * 2^h We want h = 9, so we have: B(9) = 500 * 2^9 B(9) = 500 * 512 B(9) = [B]256,000[/B]

A cup of coffee cost $4 and a cup of tea cost $3.50. If ray has $40 and has bought 6 cups of coffee,
A cup of coffee cost $4 and a cup of tea cost $3.50. If ray has $40 and has bought 6 cups of coffee, find the maximum cups of tea he can buy [U]Calculate total coffee spend:[/U] Total coffee spend = Cost per Cup of Coffee * Cups of Coffee Total coffee spend = 4 * 6 Total coffee spend = 24 [U]Calculate remaining amount to be spent on tea:[/U] Remaining tea money = Starting Money - Total Coffee spend Remaining tea money = 40 - 24 Remaining tea money = 16 [U]Calculate cups of tea Ray can buy:[/U] Cups of tea Ray can buy = Remaining Tea money / Cost per cup of tea Cups of tea Ray can buy = 16/3.50 Cups of tea Ray can buy = 4.57142857143 Since Ray can't buy partial cups, we round down and we get: Cups of tea Ray can buy = [B]4[/B]

A cup of coffee costs $1.75. A monthly unlimited coffee card costs $25.00. Which inequality represe
A cup of coffee costs $1.75. A monthly unlimited coffee card costs $25.00. Which inequality represents the number x of cups of coffee you must purchase for the monthly card to be a better deal? Let c be the number of cups. We want to know how many cups (x) where: 1.75x > 25 Using our [URL='https://www.mathcelebrity.com/interval-notation-calculator.php?num=1.75x%3E25&pl=Show+Interval+Notation']inequality solver[/URL], we see: [B]x > 14.28[/B]

A cup that is filled with equal parts red, green, and blue dye spills half of its contents. Enough g
A cup that is filled with equal parts red, green, and blue dye spills half of its contents. Enough green dye is then poured into the cup to fill it again. What is the ratio of red to green to blue dye now? Original Cup: [LIST=1] [*]Blue [*]Green [*]Red [/LIST] Spilled Cup [LIST=1] [*]Empty [*]Blue [*]Green [*]Red [/LIST] Refilled Cup [LIST=1] [*]Green [*]Blue [*]Green [*]Red [/LIST] [B]1:4:1[/B]

A cupe-shaped tank has edge lengths of 1/2 foot. Sheldon used the expression s to the power of 3 = v
A cupe-shaped tank has edge lengths of 1/2 foot. Sheldon used the expression s to the power of 3 = v to find the volume. What was the volume of the tank? 1/2 foot = 6 inches v = (6)^3 v = [B]216 cubic inches[/B]

A customer buys 5 pounds of apples at $0.79 a pound. They hand the cashier a $10 bill. How much chan
A customer buys 5 pounds of apples at $0.79 a pound. They hand the cashier a $10 bill. How much change will they get back? Calculate the total bill: Total Bill = Pounds of Apples * Cost per pound Total Bill = 5 * 0.79 Total Bill = $3.95 Calculate Change: Change = Money Offered - Total Bill Change = $10 - $3.95 Change = [B]$6.05[/B]

A customer withdrew $100 from a bank account. The customer then deposited $33 the next day. Write an
A customer withdrew $100 from a bank account. The customer then deposited $33 the next day. Write and then evaluate an expression to show the net effect of these transactions. Withdrawals are negative since we take money away Deposits are positive since we add money So we have: [LIST] [*]100 withdrawal = -100 [*]33 deposit = +33 [/LIST] Our balance is: -100 + 33 = [B]-67 net[/B]

A dad gave his 3 sons each the same amount of money in an envelope. He took $20 from one son for get
A dad gave his 3 sons each the same amount of money in an envelope. He took $20 from one son for getting a D on a math test and he gave another son an extra $35 for doing extra chores. Combined, the sons had $81. Figure out how much each son had. Let x, y, and z be the money each son received. To begin, x = y = z. But then, Dad took 20 from son X and gave it to son Y. So now, x = y - 20 Next, he gave Son Z an extra $35 for chores So z is now y + 35 since y and z used to be equal Combined, they all have 81. x + y + z = 181 But with the changes, it is: (y - 20) + y + (y + 35) Combine like terms: 3y - 20 + 35 = 81 3y + 15 = 81 Subtract 15 from each side: 3y = 66 Divide each side by 3 to isolate y y = 22 Since x = y - 20, x = 2 Since z = y + 35, we have z = 57 Checking our work, 2 + 22 + 57 = 81.

A daily pass costs $62. A season ski pass costs $450. The skier would have to rent skis with eithe
A daily pass costs $62. A season ski pass costs $450. The skier would have to rent skis with either pass for $30 per day. How many days would the skier have to go skiing in order to make the season pass less expensive than the daily passes? Let d be the number of days the skier attends. Calculate the daily cost: Daily Total Cost = Daily Cost + Rental Cost Daily Total Cost = 62d + 30d Daily Total Cost = 92d Calculate Season Cost: Season Total Cost = Season Fee + Rental Cost Season Total Cost = 450 + 30d Set the daily total cost and season cost equal to each other: 450 + 30d = 92d [URL='https://www.mathcelebrity.com/1unk.php?num=450%2B30d%3D92d&pl=Solve']Typing this equation into the search engine[/URL], we get d = 7.258. We round up to the next full day of [B]8[/B]. Now check our work: Daily Total Cost for 8 days = 92(8) = 736 Season Cost for 8 days = 30(8) + 450 = 240 + 450 = 710. Therefore, the skier needs to go at least [B]8 days[/B] to make the season cost less than the daily cot.

A delivery man had 3,456 bottles of water in his truck. The bottles were packages in cases. There we
A delivery man had 3,456 bottles of water in his truck. The bottles were packages in cases. There were 48 bottles in each case. The driver delivered 192 bottles to a store. How many cases were in the truck after delivery? Total cases in the truck before delivery 3,456/48 = 72 cases Driver delivers 192 bottles to the store. 192 bottles / 48 bottles per case = 4 cases 72 cases before delivery - 4 cases for delivery = 68 cases after delivery

A department store buys 100 shirts at a cost of $600 and sells them at a selling price of 10 each fi
A department store buys 100 shirts at a cost of $600 and sells them at a selling price of 10 each find the percentage mark up Find Unit Cost: Unit Cost = Cost / Number of Shirts Unit Cost = 600 / 100 Unit Cost = 6 With a selling price of 10, our markup percentage is: Markup % = 100 * (New Price - Old Price)/Old Price Markup % = 100 * (10 - 6)/6 Markup % = 100 * 4/6 Markup % = 400/6 Markup % = [B]66.67%[/B]

A desk and a chair together cost $1550. If the cost of the chair is n, what is the cost of the desk?
A desk and a chair together cost $1550. If the cost of the chair is n, what is the cost of the desk? Let the cost of the desk be d. WE know that: d + n = 1550 Therefore, d = [B]1550 - n[/B]

A desk drawer contains 10 blue pencils, 7 red pencils, and 8 green pencils. Without looking, you dra
A desk drawer contains 10 blue pencils, 7 red pencils, and 8 green pencils. Without looking, you draw out a pencil and then draw out a second pencil without returning the first pencil. What is the probability that the first pencil and the second pencil are both green? We are drawing without replacement. Take each draw probability: [LIST=1] [*]First draw, we have a total of 10 + 7 + 8 = 25 pencils to choose from. P(Green) = 8/25 [*]Next draw, we only have 24 total pencils, and 7 green pencils since we do not replace. Therefore, we have P(Green)= 7/24 [/LIST] Since both events are independent, we have: P(Green) * P(Green) = 8/25 * 7/24 P(Green) * P(Green) = 56/600 Using our [URL='http://www.mathcelebrity.com/gcflcm.php?num1=56&num2=600&num3=&pl=GCF']GCF Calculator[/URL], we see the greatest common factor of 56 and 600 is 8. So we divide top and bottom of the fraction by 8. [B]P(Green) * P(Green) = 7/75[/B]

A dice has six sides. The dice is rolled once. What is the probability that a six will be the result
A dice has six sides. The dice is rolled once. What is the probability that a six will be the result. P(6) = [B]1/6[/B]

A die and a coin are tossed. What is the probability of getting a 6 and a tail?
A die and a coin are tossed. What is the probability of getting a 6 and a tail? Roll a 6: P(6) = 1/6 Flip a tail: P(T) = 1/2 Probability of getting a 6 and a tail: Since both events are independent, we have: P(6 and T) = P(6) * P(T) P(6 and T) = 1/6 * 1/2 P(6 and T) = [B]1/12[/B]

A direct variation includes the points ( – 5, – 20) and (n,8). Find n.
A direct variation includes the points ( – 5, – 20) and (n,8). Find n. Slopes are proportional for rise over run. Set up a proportion of x's to y's: -5/n = -20/8 To solve this proportion for n, we [URL='https://www.mathcelebrity.com/prop.php?num1=-5&num2=-20&den1=n&den2=8&propsign=%3D&pl=Calculate+missing+proportion+value']type it in our math engine[/URL] and we get: n = [B]2[/B]

A discount store buys a shipment of fish bowls at a cost of $3.80 each. The fish bowls will be sold
A discount store buys a shipment of fish bowls at a cost of $3.80 each. The fish bowls will be sold for $5.76 apiece. What is the mark-up, as a percentage? Using our [URL='https://www.mathcelebrity.com/markup.php?p1=3.80&m=&p2=5.76&pl=Calculate']markup calculator[/URL], we get: [B]51.58% markup[/B]

A dish company needs to ship an order of 893 glass bowls. If each shipping box can hold 19 bowls, ho
A dish company needs to ship an order of 893 glass bowls. If each shipping box can hold 19 bowls, how many boxes will the company need? Number of boxes needed = Total bowls / Glass bowls per box Number of boxes needed = 893/19 Number of boxes needed = [B]47[/B]

A diving board is 10 feet long and 1 foot wide. What is its area?
A diving board is 10 feet long and 1 foot wide. What is its area? A diving board is a rectangle. And the area of a rectangle is: A = lw Plugging in our numbers, we get: A = 10(1) A = [B]10 sq feet[/B]

A dog and a cat together cost $100. If the price of the dogs $90 more than the cat, what is the cost
A dog and a cat together cost $100. If the price of the dogs $90 more than the cat, what is the cost of the cat? Set up givens and equations [LIST] [*]Let the cost of the dog be d [*]Let the cost of the cat be c [/LIST] We're given 2 equations: [LIST=1] [*]c + d = 100 [*]d = c + 90 [/LIST] Substitute equation (2) into equation (1) for d c + c + 90 = 100 Using our [URL='https://www.mathcelebrity.com/1unk.php?num=c%2Bc%2B90%3D100&pl=Solve']math engine[/URL], we see that: c = [B]5 [/B] Substitute c = 5 into equation (2) above: d = 5 + 90 d = [B]95[/B]

A dog on a 20-foot long leash is tied to the middle of a fence that is 100 feet long. The dog ruined
A dog on a 20-foot long leash is tied to the middle of a fence that is 100 feet long. The dog ruined the grass wherever it could reach. What is the area of the grass that the dog ruined. The leash forms a circle where the dog can get to. A = pi(r)^2 A = 3.1415(20)^2 A = 3.1415 * 400 A = 1256 square feet The fence blocks off half the circle where the dog can move to, so we have a half-circle area: A = 1256/2 A = [B]628 square feet[/B]

A dog walker charges a flat rate of $6 per walk plus an hourly rate of $30. How much does the dog wa
A dog walker charges a flat rate of $6 per walk plus an hourly rate of $30. How much does the dog walker charge for a 3 hour walk? Set up the cost equation C(h) where h is the number of hours: C(h) = Hourly rate * h + flat rate C(h) = 30h + 6 The question asks for C(h) when h = 3: C(3) = 30(3) + 6 C(3) = 90 + 6 C(3) = [B]96[/B]

A dormitory manager buys 38 bed sheets and 61 towels for $791.50. The manager get another 54 bed she
A dormitory manager buys 38 bed sheets and 61 towels for $791.50. The manager get another 54 bed sheets and 50 towels for $923 from the same store. What is the cost of one bed sheet and one towel? Let s be bed sheets and t be towels. We have two equations: [LIST=1] [*]38s + 61t = 791.50 [*]54s + 50t = 923 [/LIST] Using our [URL='http://www.mathcelebrity.com/simultaneous-equations.php?term1=38s+%2B+61t+%3D+791.50&term2=54s+%2B+50t+%3D+923&pl=Cramers+Method']system of equations calculator,[/URL] we get: [LIST] [*]s = 12 [*]t = 5.5 [/LIST]

A drawer is filled with 9 black shirts , 6 white shirts, and 5 gray shirts one shirt is chosen at ra
A drawer is filled with 9 black shirts , 6 white shirts, and 5 gray shirts one shirt is chosen at random from the drawer find the probability that it is not a white shirt P(Not White) = P(Black or Gray) P(Black or Gray) = (Total Black + Total Gray)/Total Shirts P(Black or Gray) = (9 + 5)/(9 + 6 + 5) P(Black or Gray) = 14/20 Simplifying this [URL='https://www.mathcelebrity.com/fraction.php?frac1=14%2F20&frac2=3%2F8&pl=Simplify']using our fraction simplify calculator[/URL], we get: P(Black or Gray) = [B]7/10, or 0.7 or 70%[/B]

A dress is on sale for $33. This is 3/5 of the regular price. What is the regular price?
A dress is on sale for $33. This is 3/5 of the regular price. What is the regular price? Original price is p. We have: 3p/5 = 33 Cross multiply using our [URL='http://www.mathcelebrity.com/prop.php?num1=3p&num2=33&den1=5&den2=1&propsign=%3D&pl=Calculate+missing+proportion+value']proportion calculator[/URL], we get [B]p = 55[/B].

A dresser has a length of 24 inches. What is the length of the dresser in centimeters?
A dresser has a length of 24 inches. What is the length of the dresser in centimeters? [SIZE=5][B]Convert 24 inches to centimeters[/B][/SIZE] centimeters = 2.54 x inches centimeters = 2.54 x 24 centimeters = [B]60.96[/B]

A driver drove at a speed of 42 mph for z hours. How far did the driver go?
A driver drove at a speed of 42 mph for z hours. How far did the driver go? Distance = Rate * Time, so we have: Distance = [B]42z[/B]

A driver drove at a speed of 56 mph for z hours. How far did the driver go?
A driver drove at a speed of 56 mph for z hours. How far did the driver go? Distance = Rate * time So we have: Distance = 56 mph * z Distance = [B]56z[/B]

A driver drove at a speed of 58 mph for t hours. How far did the driver go?
A driver drove at a speed of 58 mph for t hours. How far did the driver go? Since distance = rate * time, we have distance D of: [B]D = 58t[/B]

A factory put cakes into boxes of 6 how many boxes can they fill with 3285
A factory put cakes into boxes of 6 how many boxes can they fill with 3285 3285 cakes / 6 cakes per box = [B]547.5 boxes[/B]

A Fahrenheit thermometer shows that the temperature is 15 degrees below zero. Enter the integer that
A Fahrenheit thermometer shows that the temperature is 15 degrees below zero. Enter the integer that represents the temperature in degrees Fahrenheit. Below zero means negative in Fahrenheit, so we have: [B]-15[/B]

A fair charges an admission fee of 4 dollars for each person. Let C be the cost of admission (in d
A fair charges an admission fee of 4 dollars for each person. Let C be the cost of admission (in dollars) for P people. Write an equation relating C to P. [B]C = 4P[/B]

A fair coin is tossed 4 times. a) How many outcomes are there in the sample space? b) What is the pr
A fair coin is tossed 4 times. a) How many outcomes are there in the sample space? b) What is the probability that the third toss is heads, given that the first toss is heads? c) Let A be the event that the first toss is heads, and B be the event that the third toss is heads. Are A and B independent? Why or why not? a) 2^4 = [B]16[/B] on our [URL='http://www.mathcelebrity.comcointoss.php?hts=+HTHTHH&hct=+2&tct=+1&fct=+5>=no+more+than&nmnl=+2&htpick=heads&tossct=+4&calc=5&montect=+500&pl=Calculate+Probability']coin toss calculator[/URL] b) On the link above, 4 of those outcomes have H and H in toss 1 and 3. So it's [B]1/4 or 0.25[/B] c) [B]Yes, each toss is independent of each other.[/B]

A fair die is rolled. What is the probability of rolling a 3 or a 6?
A fair die is rolled. What is the probability of rolling a 3 or a 6? P(3 or 6) can be written as: P(3) + P(6) A fair die means all faces have an equal probability of 1/6 P(3) = 1/6 P(6) = 1/6 P(3 or 6) = P(3) + P(6) P(3 or 6) = 1/6 + 1/6 P(3 or 6) = 2/6 [URL='https://www.mathcelebrity.com/fraction.php?frac1=2%2F6&frac2=3%2F8&pl=Simplify']Using our fractions simplifier for 2/6[/URL], we get: P(3 or 6) = [B]1/3[/B]

A fair six-sided die is rolled. Describe the sample space.
A fair six-sided die is rolled. Describe the sample space. [B]{1, 2, 3, 4, 5, 6}[/B]

A fake coin has heads on both sides, if the coin tossed once, what is the probability of getting a h
A fake coin has heads on both sides, if the coin tossed once, what is the probability of getting a head? Since you always flip a head, we have: P(Head) = [B]1 or 100%[/B]

A family buys airline tickets online. Each ticket costs $167. The family buys travel insurance with
A family buys airline tickets online. Each ticket costs $167. The family buys travel insurance with each ticket that costs $19 per ticket. The Web site charges a fee of $16 for the entire purchase. The family is charged a total of $1132. How many tickets did the family buy? Let t be the number of tickets. We have the following equation with ticket price, insurance, and flat fee: 167t + 19t + 16 = 1132 Combine like terms: 186t + 16 = 1132 Using our [URL='http://www.mathcelebrity.com/1unk.php?num=186t%2B16%3D1132&pl=Solve']equation calculator[/URL], we have: [B]t = 6[/B]

A family decides to rent a canoe for an entire day. The canoe rental rate is $50 for the first three
A family decides to rent a canoe for an entire day. The canoe rental rate is $50 for the first three hours and then 20$ for each additional hour. Suppose the family can spend $110 for the canoe rental. What is the maximum number of hours the family can rent the canoe? IF we subtract the $50 for the first 3 hours, we get: 110 - 50 = 60 remaining Each additional hour is 20, so the max number of hours we can rent the canoe is $60/20 = 3 hours additional plus the original 3 hours is [B]6 hours[/B]

A family has 4 children. Give the sample space in regards to the genders of the children
A family has 4 children. Give the sample space in regards to the genders of the children. Children can either be male or female. Therefore, the sample space is 2 * 2 * 2 * 2 = 16 possible combinations. [LIST=1] [*]MMMM [*]MMMF [*]MMFM [*]MFMM [*]FMMM [*]MMFF [*]MFFM [*]FFMM [*]MFMF [*]FMFM [*]MMMF [*]FMMM [*]FFFM [*]MFFF [*]FMMF [*]FFFF [/LIST] [MEDIA=youtube]W0bthXg-368[/MEDIA]

A family is renting an apartment. For 2007, the rent is $1376 per month. The monthly rent in 2007
A family is renting an apartment. For 2007, the rent is $1376 per month. The monthly rent in 2007 is 7.5% higher than the monthly rent in 2006. Determine the monthly rent in 2006. 7.5% as a decimal is 0.075 To increase a value by 7.5%, we multiply by 1.075 [U]Calculate Rent Increase[/U] R(2007) = R(2006) * 1.075 R(2007) = 1376 * 1.075 R(2007) = [B]1,479.20[/B]

A family is taking a cross-country trip of 3000 miles by car. They are bringing two spare tires with
A family is taking a cross-country trip of 3000 miles by car. They are bringing two spare tires with them and want all six tires to go an equal distance. How many miles will each tire go? 3000 * 4 tires = 12,000 miles traveled 12,000 / 6 tires = [B]2,000 miles[/B]

A family of 4 spent $78 at the exhibition. They spent $22 on rides and the rest on entrance fees. Ho
A family of 4 spent $78 at the exhibition. They spent $22 on rides and the rest on entrance fees. How much was the entrance fee per person We need to find the entrance fee per person. So subtract the cost of rides from the total spend: Entrance Fees = Total Spend - Cost of Rides Entrance Fees = 78 - 22 Entrance Fees = 56 [U]Now find the entrance fee per person:[/U] Entrance Fee Per Person = Entrance Fees / Total People in the Family Entrance Fee Per Person =56/4 Entrance Fee Per Person = [B]14[/B]

A family of four used about 11,370 gallons of water in their home last month. There were 30 days in
A family of four used about 11,370 gallons of water in their home last month. There were 30 days in the month. About how many gallons of water did each person use each day? 11370 gallons of water / (30 days in a month * 4 people) 11370 gallons of water / (120 people days) 94.75 [B]gallons[/B]

A family room measures 15.6 feet long and 18.4 feet wide. What is the area of the room?
A family room measures 15.6 feet long and 18.4 feet wide. What is the area of the room? The room is rectangular. So our area A = lw. Using our [URL='https://www.mathcelebrity.com/rectangle.php?l=15.6&w=18.4&a=&p=&pl=Calculate+Rectangle']rectangle calculator[/URL], we get: A = [B]287.04 square feet[/B]

a family went to a baseball game. the cost to park the car was $5 AND THE COST PER TICKET WAS $21. W
a family went to a baseball game. the cost to park the car was $5 AND THE COST PER TICKET WAS $21. WRITE A LINEAR FUNCTION IN THE FORM Y=MX+B, FOR THE TOTAL COST OF GOING TO THE BASEBALL GAME,Y, AND THE TOTAL NUMBER PEOPLE IN THE FAMILY,X. We have: [B]y = 21x + 5[/B] Since the cost of each ticket is $21, we multiply this by x, the total number of people in the family. We add 5 as the cost to park the car, which fits the entire family, and is a one time cost.

A farmer bought a number of pigs for $232. However, 5 of them died before he could sell the rest at
A farmer bought a number of pigs for $232. However, 5 of them died before he could sell the rest at a profit of 4 per pig. His total profit was $56. How many pigs did he originally buy? Let p be the purchase price of pigs. We're given: [LIST] [*]Farmer originally bought [I]p [/I]pigs for 232 which is our cost C. [*]5 of them died, so he has p - 5 left [*]He sells 4(p - 5) pigs for a revenue amount R [*]Since profit is Revenue - Cost, which equals 56, we have: [/LIST] Calculate Profit P = R - C Plug in our numbers: 4(p - 5) - 232 = 56 4p - 20 - 232 = 56 To solve for p, [URL='https://www.mathcelebrity.com/1unk.php?num=4p-20-232%3D56&pl=Solve']we type this equation into our search engine[/URL] and we get: p = [B]77[/B]

A farmer has 165 feet of fencing material in which to enclose a rectangular grazing area. He wants t
A farmer has 165 feet of fencing material in which to enclose a rectangular grazing area. He wants the length x to be greater than 50 feet and the width y to be no more than 20 feet. Write a system to represent this situation. Perimeter of a rectangle: P = 2l + 2w We have P = 165 and l = x --> x>50 and width y <= 20. Plug these into the perimeter formula [B]165 = 2x + 2y where x > 50 and y <= 20[/B]

A farmer has a total of 200 ducks and cows in his barn. If he has n cows, how many total legs are th
A farmer has a total of 200 ducks and cows in his barn. If he has n cows, how many total legs are there in the barn? (Make sure you include the farmer.) [LIST] [*]Number of cows = n [*]Legs per cow = 4 [*]Cows legs = 4n [*]Number of ducks = 200 - n [*]Legs per duck = 2 [*]Number of ducks legs = (200 - n) x 2 = 400 - 2n [*]Farmers legs = 2 [/LIST] Total legs = Cows legs + Ducks Legs + Farmers Legs Total legs = 4n + 400 - 2n + 2 Total legs = [B]2n + 402[/B]

A farmer has c chickens. She sells 3 of them for $6 each and the rest she sells for $5 each. How muc
A farmer has c chickens. She sells 3 of them for $6 each and the rest she sells for $5 each. How much money will she receive? Total money received = 3 * 6 + 5(c - 3) Total money received = 18 + 5c - 15 Total money received = [B]5c + 3[/B]

A farmer is taking her eggs to the market in a cart, but she hits a pothole, which knocks over all
A farmer is taking her eggs to the market in a cart, but she hits a pothole, which knocks over all the containers of eggs. Though she is unhurt, every egg is broken. So she goes to her insurance agent, who asks her how many eggs she had. She says she doesn't know, but she remembers somethings from various ways she tried packing the eggs. When she put the eggs in groups of two, three, four, five, and six there was one egg left over, but when she put them in groups of seven they ended up in complete groups with no eggs left over. What can the farmer figure from this information about the number of eggs she had? Is there more than one answer? We need a number (n) that leaves a remainder of 1 when divided by 2, 3, 4, 5, 6 but no remainder when divided by 7. 217 + 84 = [B]301[/B]. Other solutions are multiples of 3 x 4 x 5 x 7, but we want the lowest one here.

A Farmer Sell products at the market in 38- pound crates. If he sells 100 crates . How many pounds o
A Farmer Sell products at the market in 38- pound crates. If he sells 100 crates . How many pounds of produce has he sold [U]Calculate the pounds of produce:[/U] Pounds of Produce = Number of Crates * pounds per crate Pounds of Produce = 100 crates * 38 pounds per crates Pounds of Produce = [B]3,800 pounds of produce[/B]3

A farmer sold 250 of his sheep, bought 35 and then bought 68. If he now has 190, how many did he beg
A farmer sold 250 of his sheep, bought 35 and then bought 68. If he now has 190, how many did he begin with? Let's start his count with x. We have: x - 250 + 35 + 68 = 190 Using our [URL='http://www.mathcelebrity.com/1unk.php?num=x-250%2B35%2B68%3D190&pl=Solve']equation solver[/URL], we get x = [B]337[/B]

A farmer was 1/3 of his land to grow corn, a quarter of his land to grow lettuce, and 12.5% of his l
A farmer was 1/3 of his land to grow corn, a quarter of his land to grow lettuce, and 12.5% of his land to grow green beans. He uses the remaining 7 acres to grow wheat.How many total acres does the farmer own? Convert all land portions to fractions or decimals. We will do fractions: [LIST] [*]1/3 for corn [*][I]A quarter[/I] means 1/4 for lettuce [*]12.5% is 12.5/100 or 1/8 for green beans [/LIST] Now add all these up: 1/3 + 1/4 + 1/8 We need a common factor for 3, 4, and 8. Using our [URL='https://www.mathcelebrity.com/gcflcm.php?num1=3&num2=4&num3=8&pl=LCM']LCM Calculator[/URL], we get 24. 1/3 = 8/24 1/4 = 6/24 18 = 3/24 Add them all up: (8 + 6 + 3)/24 17/24 This means 17/24 of the land is used for everything but wheat. Wheat occupies (24-17)/24 = 7/24 of the land. We'll use a for the number of acres on the farm. 7a/24 = 7 [B]a = 24[/B]

A father is K years old and his son is M years younger. The sum of their ages is 53.
A father is K years old and his son is M years younger. The sum of their ages is 53. Father's Age = K Son's Age = K - M and we know K + (K - M) = 53 Combine like terms: 2K - M = 53 Add M to each side: 2K - M + M = 53 + M Cancel the M's on the left side, we get: 2K = 53+ M Divide each side by 2: 2K/2 = (53 + M)/2 Cancel the 2's on the left side: K = [B](53 + M)/2[/B]

A faucet drips 15 milliliters of water every 45 minutes. How many milliliters of water will it drip
A faucet drips 15 milliliters of water every 45 minutes. How many milliliters of water will it drip in 3 hours? 3 hours = 60 * 3 = 180 minutes 180 minutes / 45 minutes = 4 So the faucet drips 15 milliliters 4 times 15 * 4 = [B]60 milliliters[/B]

A financial advisor has invested $7000 in two accounts. If one account contains x dollars, express t
A financial advisor has invested $7000 in two accounts. If one account contains x dollars, express the amount in the second account in terms of x The other account contains: [B]7000 - x[/B]

A financial analyst computed the ROI for all companies listed on the NYSE. She found that the mean o
A financial analyst computed the ROI for all companies listed on the NYSE. She found that the mean of this distribution was 10% with standard deviation of 5%. She is interested in examining further those companies whose ROI is between 14% and 16% of the approximately 1,500 companies listed on the exchange, how many are of interest of her? First, use our [URL='http://www.mathcelebrity.com/zscore.php?z=p%280.14%3Cz%3C0.16%29&pl=Calculate+Probability']z-score calculator[/URL] to get P(0.14 < z < 0.16) = 0.007889 Divide that by 2 for two-tail test to get0.003944729 Use the NORMSINV(0.003944729) in Excel to get the Z value of 2.66 Therefore, the companies of interest are 2.66 * 1500 * 0.10 = [B]399[/B]

A firm wants to know with a 98% level of confidence if it can claim that the boxes of detergent it s
A firm wants to know with a 98% level of confidence if it can claim that the boxes of detergent it sells contain more than 500g of detergent. From past experience the firm knows that the amount of detergent in the boxes is normally distributed. The firm takes a random sample of n =25 and finds that X = 520 g and s = 75g. What's your final conclusion? (Ho: u = 500; Ha: u > 500) [URL='http://www.mathcelebrity.com/mean_hypothesis.php?xbar=520&n=25&stdev=75&ptype==&mean=500&alpha=0.02&pl=Mean+Hypothesis+Testing']Perform a hypothesis testing of the mean[/URL] [B]Yes, accept null hypothesis[/B]

A first number plus twice a second number is 10. Twice the first number plus the second totals 29. F
A first number plus twice a second number is 10. Twice the first number plus the second totals 29. Find the numbers. Let the first number be x. Let the second number be y. We are given the following two equations: [LIST=1] [*]x + 2y = 10 [*]2x + y = 29 [/LIST] We can solve this 3 ways using: [LIST=1] [*][URL='https://www.mathcelebrity.com/simultaneous-equations.php?term1=x+%2B+2y+%3D+10&term2=2x+%2By+%3D+29&pl=Substitution']Substitution[/URL] [*][URL='https://www.mathcelebrity.com/simultaneous-equations.php?term1=x+%2B+2y+%3D+10&term2=2x+%2By+%3D+29&pl=Elimination']Elimination[/URL] [*][URL='https://www.mathcelebrity.com/simultaneous-equations.php?term1=x+%2B+2y+%3D+10&term2=2x+%2By+%3D+29&pl=Cramers+Method']Cramers Rule[/URL] [/LIST] Using any of the 3 methods, we get the same answers of [B](x, y) = (16, -3)[/B]

A first number plus twice a second number is 10. Twice the first number plus the second totals 35. F
A first number plus twice a second number is 10. Twice the first number plus the second totals 35. Find the numbers. [U]The phrase [I]a number[/I] means an arbitrary variable[/U] A first number is written as x A second number is written as y [U]Twice a second number means we multiply y by 2:[/U] 2y [U]A first number plus twice a second number:[/U] x + 2y [U]A first number plus twice a second number is 10 means we set x + 2y equal to 10:[/U] x + 2y = 10 [U]Twice the first number means we multiply x by 2:[/U] 2x [U]Twice the first number plus the second:[/U] 2x + y [U]Twice the first number plus the second totals 35 means we set 2x + y equal to 35:[/U] 2x + y = 35 Therefore, we have a system of two equations: [LIST=1] [*]x + 2y = 10 [*]2x + y = 35 [/LIST] Since we have an easy multiple of 2 for the x variable, we can solve this by multiply the first equation by -2: [LIST=1] [*]-2x - 4y = -20 [*]2x + y = 35 [/LIST] Because the x variables are opposites, we can add both equations together: (-2 + 2)x + (-4 + 1)y = -20 + 35 The x terms cancel, so we have: -3y = 15 To solve this equation for y, we [URL='https://www.mathcelebrity.com/1unk.php?num=-3y%3D15&pl=Solve']type it in our search engine[/URL] and we get: y = [B]-5 [/B] Now we substitute this y = -5 into equation 2: 2x - 5 = 35 To solve this equation for x, we[URL='https://www.mathcelebrity.com/1unk.php?num=2x-5%3D35&pl=Solve'] type it in our search engine[/URL] and we get: x = [B]20[/B]

A first number plus twice a second number is 11. Twice the first number plus the second totals 34. F
A first number plus twice a second number is 11. Twice the first number plus the second totals 34. Find the numbers. Let the first number be x and the second number be y. We're given: [LIST=1] [*]x + 2y = 11 [*]2x + y = 34 [/LIST] Using our simultaneous equations calculator, we have 3 methods to solve this: [LIST=1] [*][URL='https://www.mathcelebrity.com/simultaneous-equations.php?term1=x+%2B+2y+%3D+11&term2=2x+%2B+y+%3D+34&pl=Substitution']Substitution Method[/URL] [*][URL='https://www.mathcelebrity.com/simultaneous-equations.php?term1=x+%2B+2y+%3D+11&term2=2x+%2B+y+%3D+34&pl=Elimination']Elimination Method[/URL] [*][URL='https://www.mathcelebrity.com/simultaneous-equations.php?term1=x+%2B+2y+%3D+11&term2=2x+%2B+y+%3D+34&pl=Cramers+Method']Cramer's Rule[/URL] [/LIST] All 3 methods give the same solution: [LIST] [*][B]x = 19[/B] [*][B]y = -4[/B] [/LIST]

A first number plus twice a second number is 14. Twice the first number plus the second totals 40. F
A first number plus twice a second number is 14. Twice the first number plus the second totals 40. Find the numbers. [B][U]Givens and assumptions:[/U][/B] [LIST] [*]Let the first number be x. [*]Let the second number be y. [*]Twice means multiply by 2 [*]The phrases [I]is[/I] and [I]totals[/I] mean equal to [/LIST] We're given two equations: [LIST=1] [*]x + 2y = 14 [*]2x + y = 40 [/LIST] To solve this system, we can take a shortcut, and multiply the top equation by -2 to get our new system: [LIST=1] [*]-2x - 4y = -28 [*]2x + y = 40 [/LIST] Now add both equations together (-2 _ 2)x (-4 + 1)y = -28 + 40 -3y = 12 To solve this equation, we [URL='https://www.mathcelebrity.com/1unk.php?num=-3y%3D12&pl=Solve']type it in our search engine[/URL] and we get: y = [B]-4 [/B] We substitute this back into equation 1 for y = -4: x + 2(-4) = 14 x - 8 = 14 To solve this equation for x, we [URL='https://www.mathcelebrity.com/1unk.php?num=x-8%3D14&pl=Solve']type it in our search engine[/URL] and we get: x = [B]22[/B]

A first number plus twice a second number is 22. Twice the first number plus the second totals 28. F
A first number plus twice a second number is 22. Twice the first number plus the second totals 28. Find the numbers. Let the first number be x. Let the second number be y. We're given two equations: [LIST=1] [*]x + 2y = 22 <-- Since twice means multiply by 2 [*]2x + y = 28 <-- Since twice means multiply by 2 [/LIST] We have a set of simultaneous equations. We can solve this three ways [LIST] [*][URL='https://www.mathcelebrity.com/simultaneous-equations.php?term1=x+%2B+2y+%3D+22&term2=2x+%2B+y+%3D+28+&pl=Substitution']Substitution Method[/URL] [*][URL='https://www.mathcelebrity.com/simultaneous-equations.php?term1=x+%2B+2y+%3D+22&term2=2x+%2B+y+%3D+28&pl=Elimination']Elimination Method[/URL] [*][URL='https://www.mathcelebrity.com/simultaneous-equations.php?term1=x+%2B+2y+%3D+22&term2=2x+%2B+y+%3D+28&pl=Cramers+Method']Cramers Rule[/URL] [/LIST] No matter which method we use, we get the same answer: [LIST] [*][B]x = 11 & 1/3[/B] [*][B]y = 5 & 1/3[/B] [/LIST]

A first number plus twice a second number is 3. Twice the first number plus the second totals 24.
A first number plus twice a second number is 3. Twice the first number plus the second totals 24. Let the first number be x. Let the second number be y. We're given: [LIST=1] [*]x + 2y = 3 <-- Because [I]twice[/I] means multiply by 2 [*]2x + y = 24 <-- Because [I]twice[/I] means multiply by 2 [/LIST] We have a system of equations. We can solve it any one of three ways: [LIST] [*][URL='https://www.mathcelebrity.com/simultaneous-equations.php?term1=x+%2B+2y+%3D+3&term2=2x+%2B+y+%3D+24&pl=Substitution']Substitution Method[/URL] [*][URL='https://www.mathcelebrity.com/simultaneous-equations.php?term1=x+%2B+2y+%3D+3&term2=2x+%2B+y+%3D+24&pl=Elimination']Elimination Method[/URL] [*][URL='https://www.mathcelebrity.com/simultaneous-equations.php?term1=x+%2B+2y+%3D+3&term2=2x+%2B+y+%3D+24&pl=Cramers+Method']Cramer's Rule[/URL] [/LIST] No matter which way we choose, we get: [LIST] [*]x = [B]15[/B] [*]y = [B]-6[/B] [/LIST]

A first number plus twice a second number is 6. Twice the first number plus the second totals 15. Fi
A first number plus twice a second number is 6. Twice the first number plus the second totals 15. Find the numbers. Let the first number be x. Let the second number be y. We're given two equations: [LIST=1] [*]x + 2y = 6 [*]2x + y = 15 [/LIST] Multiply the first equation by -2: [LIST=1] [*]-2x - 4y = -12 [*]2x + y = 15 [/LIST] Now add them -2x + 2x - 4y + y = -12 + 15 -3y = 3 Divide each side by -3: y = 3/-3 y =[B] -1[/B] Plug this back into equation 1: x + 2(-1) = 6 x - 2 = 6 To solve for x, we [URL='https://www.mathcelebrity.com/1unk.php?num=x-2%3D6&pl=Solve']type this equation into our search engine[/URL] and we get: x = [B]8[/B]

A first number plus twice a second number is 7
A first number plus twice a second number is 7 Let the first number be x. Let the second number be y. We're given: [LIST] [*]A first number is x [*]A second number is y [*]Twice the second number means we multiply y by 2: 2y [*][I]Plus [/I]means we add x to 2y: x + 2y [*]The phrase [I]is[/I] means an equation, so we set x + 2y equal to 7 [/LIST] [B]x + 2y = 7[/B]

A first number plus twice a second number is 7. Twice the first number plus the second totals 23. Fi
A first number plus twice a second number is 7. Twice the first number plus the second totals 23. Find the numbers Let the first number be a and the second number be b. We have: [LIST=1] [*]a + 2b = 7 [*]2a + b = 23 [/LIST] Rearrange (1) into (3) (3) a = 7 - 2b Substitute (3) into (2): 2(7 - 2b) + b = 23 Multiply through: 14 - 4b + b = 23 Combine like terms: 14 - 3b = 23 Subtract 14 from each side: -3b = 9 Divide each side by -3 [B]b = -3[/B] Substitute this into (3) a = 7 - 2b a = 7 - 2(-3) a = 7 + 6 [B]a = 13[/B] [B](a, b) = (13, -3)[/B]

A flea is very small, but can jump very high. For example, a flea that is 1/8 inch tall can jump 12
A flea is very small, but can jump very high. For example, a flea that is 1/8 inch tall can jump 12 inches in height. If a child who is 4 feet tall had the ability to jump like a flea, how high could she jump? Set up a proportion of height to jump height where j is the jump height of the child: 1/8/12 = 4/j Using our [URL='https://www.mathcelebrity.com/proportion-calculator.php?num1=0.125&num2=4&den1=12&den2=j&propsign=%3D&pl=Calculate+missing+proportion+value']proportion calculator[/URL], we get: j = [B]384 feet[/B]

A flower bed is to be 3 m longer than it is wide. The flower bed will an area of 108 m2 . What will
A flower bed is to be 3 m longer than it is wide. The flower bed will an area of 108 m2 . What will its dimensions be? A flower bed has a rectangle shape, so the area is: A = lw We are given l = w + 3 Plugging in our numbers given to us, we have: 108 = w(w + 3) w^2 + 3w = 108 Subtract 108 from each side: w^2 + 3w - 108 = 0 [URL='https://www.mathcelebrity.com/quadratic.php?num=w%5E2%2B3w-108%3D0&pl=Solve+Quadratic+Equation&hintnum=+0']Type this problem into our search engine[/URL], and we get: w = (9, -12) Since length cannot be negative, w = 9. And l = 9 + 3 --> l = 12 So we have [B](l, w) = (12, 9)[/B] Checking our work, we have: A = (12)9 A = 108 <-- Match!

A food truck sells salads for $6.50 each and drinks for $2.00 each. The food trucks revenue from sel
A food truck sells salads for $6.50 each and drinks for $2.00 each. The food trucks revenue from selling a total of 209 salads and drinks in one day was $836.50. How many salads were sold that day? Let the number of drinks be d. Let the number of salads be s. We're given two equations: [LIST=1] [*]2d + 6.50s = 836.50 [*]d + s = 209 [/LIST] We can use substitution to solve this system of equations quickly. The question asks for the number of salads (s). Therefore, we want all expressions in terms of s. Rearrange Equation 2 by subtracting s from both sides: d + s - s = 209 - s Cancel the s's, we get: d = 209 - s So we have the following system of equations: [LIST=1] [*]2d + 6.50s = 836.50 [*]d = 209 - s [/LIST] Substitute equation (2) into equation (1) for d: 2(209 - s) + 6.50s = 836.50 Multiply through to remove the parentheses: 418 - 2s + 6.50s = 836.50 To solve this equation for s, we [URL='https://www.mathcelebrity.com/1unk.php?num=418-2s%2B6.50s%3D836.50&pl=Solve']type it into our search engine and we get[/URL]: s = [B]93[/B]

A football gained 52 yards during the possession. In the next 3 possessions they gained the same amo
A football gained 52 yards during the possession. In the next 3 possessions they gained the same amount of yards each time. If they gained a total of 256 yards, write and solve an equation for how many yards they gained in each of the last 3 possessions. Subtract 52 initial yards 256 - 52 = 204 Now, divide 204 by 3 possessions 204/3 = [B]68 yards[/B]

A football team gained 4 yards on a play,lost 8 on the next play ,then gained 2 yards on the third p
A football team gained 4 yards on a play,lost 8 on the next play ,then gained 2 yards on the third play write and addition expression Gains are expressed with positives (+) and losses are expressed with negatives (-): [LIST] [*]Gained 4 years: +4 [*]Lost 8 on the next play: -8 [*]Gained 2 yards on the third play: +2 [/LIST] Expression: [B]+4 - 8 + 2 = -2[/B]

A football team gained 8 yards on a first down, lost 12 yards on the second down, and gained 16 yard
A football team gained 8 yards on a first down, lost 12 yards on the second down, and gained 16 yards on the third down. How many yards did the team gain or lose? Assumptions: [LIST] [*]We reflect gains by adding [*]We reflect losses by subtracting [/LIST] Plays: [LIST] [*]Gain of 8 = +8 [*]Loss of 12 = -12 [*]Gain of 16 = +16 [/LIST] Net Gain/Loss +8 - 12 + 16 [B]+12 (gain)[/B]

A football team loses 27 yards total during its first 3 plays. On average, what is the yards per pl
A football team loses 27 yards total during its first 3 plays. On average, what is the yards per play for these 3 plays? A loss of yards means negative yardage. Average Yards per play = Total Yards / Total plays Average Yards per play = -27/3 Average Yards per play = -[B]9 or 9 yard loss[/B]

A football team loses 3 yards, gains 12 yards, gains 10 yards, and then loses 15 yards. Did the team
A football team loses 3 yards, gains 12 yards, gains 10 yards, and then loses 15 yards. Did the team gain yards or lose yards overall? How many yards We have: Gains - Losses -3 + 12 + 10 - 15 (12 + 10) - (3 + 15) 22 - 18 [B]4 yard gain[/B]

A football team lost 7 yards each play for four consecutive plays. Represent the team’s total change
A football team lost 7 yards each play for four consecutive plays. Represent the team’s total change in position for the four plays as an integer. A net loss in yardage for 7 yards is written as -7 4 plays * -7 yards equals [B]-28[/B]

a football team won 3 more games than it lost.the team played 11 games.how many did it win?
a football team won 3 more games than it lost.the team played 11 games.how many did it win? Let wins be w. Let losses be l. We're given two equations: [LIST=1] [*]w = l + 3 [*]l + w = 11 [/LIST] Plug equation (1) into equation (2) to solve for l: l + (l + 3) = 11 Group like terms: 2l + 3 = 11 [URL='https://www.mathcelebrity.com/1unk.php?num=2l%2B3%3D11&pl=Solve']Typing this equation into our search engine[/URL], we get: l = 4 To solve for w, we plug in l = 4 above into equation (1): w = 4 + 3 w = [B]7[/B]

A fraction has a value of 3/4. If 7 is added to the numerator, the resulting fraction is equal to th
A fraction has a value of 3/4. If 7 is added to the numerator, the resulting fraction is equal to the reciprocal of the original fraction. Find the original fraction. Let the fraction be x/y. We're given two equations: [LIST=1] [*]x/y = 3/4 [*](x + 7)/y = 4/3. [I](The reciprocal of 3/4 is found by 1/(3/4)[/I] [/LIST] Cross multiply equation 1 and equation 2: [LIST=1] [*]4x = 3y [*]3(x + 7) = 4y [/LIST] Simplifying, we get: [LIST=1] [*]4x = 3y [*]3x + 21 = 4y [/LIST] If we divide equation 1 by 4, we get: [LIST=1] [*]x = 3y/4 [*]3x + 21 = 4y [/LIST] Substitute equation (1) into equation (2) for x: 3(3y/4) + 21 = 4y 9y/4 + 21 = 4y Multiply the equation by 4 on both sides to eliminate the denominator: 9y + 84 = 16y To solve this equation for y, we type it in our math engine and we get: y = [B]12 [/B] We then substitute y = 12 into equation 1 above: x = 3 * 12/4 x = 36/4 x = [B]9 [/B] So our original fraction x/y = [B]9/12[/B]

A framed print measures 80cm by 65cm. The frame is 5cm wide. Find the area of the unframed print
A framed print measures 80cm by 65cm. The frame is 5cm wide. Find the area of the unframed print. We subtract 5 cm from the length and the width to account for the frame: Unframed Length: 80 - 5 = 75 Unframed Width: 65 - 5 = 60 Area of the unframed rectangle is: A = lw A = 75(60) A = [B]4,500 sq cm[/B]

A fruit basket contains 2 red apples and 2 green apples. What is the ratio of the number of red appl
A fruit basket contains 2 red apples and 2 green apples. What is the ratio of the number of red apples to the total number of apples? 2:2 = [B]1:1[/B] simplified

A fruit basket contains 458 apples And 139 oranges.How many more apples are there in the basket?
A fruit basket contains 458 apples And 139 oranges.How many more apples are there in the basket? We want Apples - Oranges: Apples - Oranges = 458 - 139 Apples - Oranges = [B]319[/B]

A fuel injection system is designed to last 18 years, with a standard deviation of 1.4 years. What i
A fuel injection system is designed to last 18 years, with a standard deviation of 1.4 years. What is the probability that a fuel injection system will last less than 15 years? Using our [URL='https://www.mathcelebrity.com/probnormdist.php?xone=15&mean=18&stdev=14&n=1&pl=P%28X+%3C+Z%29']z-score calculator[/URL], we see that: P(X < 15) = [B]0.416834[/B]

A furniture company plans to have 25 employees at its corporate headquarters and 25 employees at eac
A furniture company plans to have 25 employees at its corporate headquarters and 25 employees at each store it opens. Let s represent the number of stores and m represent the total number of employees. There is only one corporate headquarters. So we have the number of employees (m) as: m = Store Employees + Corporate Employees Each store has 25 employees. Total store employees equal 25 per store times the number of stores (s). [B]m = 25s + 25[/B]

A game show has 74 categories. There are 9,785 points in each category. How many points are there in
A game show has 74 categories. There are 9,785 points in each category. How many points are there in total on the game show? Total points = Total categories * points per category Total points = 9785 * 74 Total points = [B]724,090[/B]

A garden has a length that is three times its width. If the width is n feet and fencing cost $8 per
A garden has a length that is three times its width. If the width is n feet and fencing cost $8 per foot, what is the cost of the fencing for the garden? Garden is a rectangle which has Perimeter P of: P = 2l + 2w l = 3w P = 2(3w) + 2w P = 6w + 2w P = 8w Width w = n, so we have: P = 8n Cost = 8n * 8 = [B]64n dollars[/B]

A garden table and a bench cost $977 combined. The garden
A garden table and a bench cost $977 combined. The garden table costs $77 more than the bench. What is the cost of the bench? Let the garden table cost be g and the bench cost be b. We're given [LIST=1] [*]b + g = 977 [*]g = b + 77 <-- The phrase [I]more than[/I] means we add [/LIST] Substitute (2) into (1): b + (b + 77) = 977 Combine like terms: 2b + 77 = 977 [URL='https://www.mathcelebrity.com/1unk.php?num=2b%2B77%3D977&pl=Solve']Typing this equation into our search engine[/URL], we get: [B]b = $450[/B]

A gardener plants flowers in the following order: carnations,daffodils, larkspurs, tiger lillies, an
A gardener plants flowers in the following order: carnations,daffodils, larkspurs, tiger lillies, and zinnias. if the gardener planted 47 plants, what kind of flower did he plant last? Let c be carnations, d be daffodils, l be larkspurs, t be tiger lillies, and z be zinnias. The order goes as follows: c, d, l, t, z. So each cycle of plants counts as 5 plants. We know that 9 * 5 = 45. So the gardener plants 9 full cycles. Which means they have 47 - 45 = 2 plans left over. In the order above, the second plant is the daffodil. So the gardener planted the [B]daffodil[/B] last. Now, can we shortcut this problem? Yes, using modulus. 47 plants, with 5 plants per cycle, we do [URL='https://www.mathcelebrity.com/modulus.php?num=47mod5&pl=Calculate+Modulus']47 mod 5 through our calculator[/URL], and get 2. So we have 2 plants left over, and the daffodil is the second plant.

A gardener wants to fence a circular garden of diameter 21cm. Find the length of the rope he needs t
A gardener wants to fence a circular garden of diameter 21cm. Find the length of the rope he needs to purchase, if he makes 2round of fence Also find the cost of the rope, if it costs Rs4 per meter (take pie as 22/7) Circumference of a circle = Pi(d). Given Pi = 22/7 for this problem, we have: C = 22/7(21) C = 22*3 [B]C = 66[/B]

A gas tank contains 5.3 gallons. The capacity of the tank is 12.5 gallons. How much will it cost to
A gas tank contains 5.3 gallons. The capacity of the tank is 12.5 gallons. How much will it cost to fill the tank at $1.30 per gallon. [U]Calculate the empty portion of the gas tank:[/U] Empty portion = Capacity - Gas gallons remaining Empty portion = 12.5 - 5.3 Empty portion (in gallons) = 7.2 [U]Calculate the cost to fill the tank:[/U] Cost to fill the tank = Empty portion (in gallons) * cost per gallon Cost to fill the tank = 7.2 * $1.30 Cost to fill the tank = [B]$9.36[/B]

A gasoline tank is leaking at a rate of n gallons in t hours. If the gasoline cost $2 per gallon, wh
A gasoline tank is leaking at a rate of n gallons in t hours. If the gasoline cost $2 per gallon, what is the value of the gasoline that will be lost in m minutes? n gallons / t hours = n/t gallons per hour are leaking The value of the gas that leaks each hour is $2, so we have: 2n/t dollar per hour is leaking Value per minute means we divide by 60: 2n/60t Dividing top and bottom by 2 to simplify, we have: n/30t Given m minutes, we multiply to get: [B]nm/30t[/B]

A giant burrito required 75.75 lb of cheese. About how many 12-lb boxes of cheese did the cooks use?
A giant burrito required 75.75 lb of cheese. About how many 12-lb boxes of cheese did the cooks use? 75.75/12 = [B]6.31 boxes [/B] We can round up to 7 full boxes

A giant tortoise can live 175 years in captivity. The gastrotrich, which is a small aquatic animal,
A giant tortoise can live 175 years in captivity. The gastrotrich, which is a small aquatic animal, has a life-span of only 3 days (72 hours). If a gastrotrich died after 36 hours, a giant tortoise that lived 87.5 yeas would live proportionally the same because they both would have died halfway through their life-span. How long would a giant tortoise live if it lived proportionally the same as a gastrotrich that died after 50 hours? Set up a proportion of hours lived to lifespan where n is the number of years the giant tortoise lives: 50/72 = n/175 Using our [URL='https://www.mathcelebrity.com/proportion-calculator.php?num1=50&num2=n&den1=72&den2=175&propsign=%3D&pl=Calculate+missing+proportion+value']proportion calculator[/URL], we get: n = [B]121.5[/B]

A girl is pedaling her bicycle at a velocity of 0.10 km/hr. How far will she travel in two hours?
A girl is pedaling her bicycle at a velocity of 0.10 km/hr. How far will she travel in two hours? The distance formula is: d = rt We're given a rate (r) of 0.10km/hr We're given time (t) of 2 hours Plug these values into the distance formula and we get: d= 0.1 * 2 d = [B]0.2km [MEDIA=youtube]w80E_YM-tDA[/MEDIA][/B]

A girl is three years older than her brother. If their combined age is 35 years, how old is each
A girl is three years older than her brother. If their combined age is 35 years, how old is each Let the girl's age be g. Let the boy's age be b. We're given two equations: [LIST=1] [*]g = b + 3 ([I]Older means we add)[/I] [*]b + g = 35 [/LIST] Now plug in equation (1) into equation (2) for g: b + b + 3 = 35 To solve for b, we [URL='https://www.mathcelebrity.com/1unk.php?num=b%2Bb%2B3%3D35&pl=Solve']type this equation into our search engine[/URL] and we get: b = [B]16 [/B] Now, to solve for g, we plug in b = 16 that we just solved for into equation (1): g = 16 + 3 g = [B]19[/B]

A girl makes 12 foul shots for every eight that she misses how many shots did you make if she shot 1
A girl makes 12 foul shots for every eight that she misses how many shots did you make if she shot 125 foul shots This means she makes 12/20 We want to know x shots, if 12/20 = x/125. [URL='http://www.mathcelebrity.com/prop.php?num1=12&num2=x&den1=20&den2=125&propsign=%3D&pl=Calculate+missing+proportion+value']Enter this proportion into the search engine[/URL] to get [B]x = 75[/B]

A golf ball is selected at random from a golf bag. If the golf bag contains 5 type A balls, 4 type B
A golf ball is selected at random from a golf bag. If the golf bag contains 5 type A balls, 4 type B balls, and 9 type C balls, find the probability that the golf ball is not a type A ball Not Type A means Type B or Type C. Our total balls = 5 + 4 + 9 = 18 P(B or C) = P(B) + P(C) P(B or C) = 4/18 + 9/18 P(B or C) = [B]13/18[/B]

A Government antipollution spokeperson asserts that more than 80% of the plants in the Glassboro are
A Government antipollution spokeperson asserts that more than 80% of the plants in the Glassboro area meet the antipollution standards. An antipollution advocate does not believe the government claim. She takes a random sample of published data on pollution emission for 64 plants in the area and finds that 56 plants meet the pollution standards. Do the sample data support the government claim at the 1% level of significance? (H0: ρ=0.8; Ha: ρ>0.8) [URL='http://www.mathcelebrity.com/proportion_hypothesis.php?x=+56&n=64&ptype==&p=+0.8&alpha=+0.01&pl=Proportion+Hypothesis+Testing']Perform a hypothesis testing of a proportion[/URL] [B]Accept null hypothesis[/B]

A grandmother deposited $5,000 in an account that pays 8% per year compounded annually when her gran
A grandmother deposited $5,000 in an account that pays 8% per year compounded annually when her granddaughter was born. What will the value of the account be when the granddaughter reaches her 16th birthday? We have the accumulation function A(t) = 5,000(1.08)^t. For t = 16, we have: A(16) = 5,000(1.08)^16 A(16) = 5,000*3.42594264333 A(16) = [B]17,129.71[/B]

A grocer is selling oranges at 3 for $2. How much would it cost to buy a dozen oranges?
A grocer is selling oranges at 3 for $2. How much would it cost to buy a dozen oranges? Set up a proportion of oranges per cost where c is the cost of a dozen oranges: 3/2 = 12/c <-- A dozen equals 12 [URL='https://www.mathcelebrity.com/prop.php?num1=3&num2=12&den1=2&den2=c&propsign=%3D&pl=Calculate+missing+proportion+value']Typing this proportion into our search engine[/URL], we get: c = [B]8[/B]

A grocery store is selling 6 cans of cat food for $3. Find the cost of a can of cat food
A grocery store is selling 6 cans of cat food for $3. Find the cost of a can of cat food Unit Cost = Cost / Quantity Unit Cost = $3 / 6 Unit cost = [B]0.50 per can[/B]

A grocery store sells 6 pounds of apples for $12. What is the unit price of the apples?
A grocery store sells 6 pounds of apples for $12. What is the unit price of the apples? Unit Price = Cost/Quantity Unit Price = 12/6 [B]Unit Price = $2/lb[/B]

A grocery store sells chili peppers at $2.04 for a dozen. At this rate, what's the cost per pepper?
A grocery store sells chili peppers at $2.04 for a dozen. At this rate, what's the cost per pepper? A dozen = 12 peppers, so our cost per pepper is: Cost per pepper = Cost per dozen / 12 per dozen Cost per pepper = 2.04/12 Cost per pepper = [B]0.17[/B]

A group of 30 students from your school is part of the audience for a TV game show. The total number
A group of 30 students from your school is part of the audience for a TV game show. The total number of people in the audience is 120. What theoretical probability of 5 students from your school being selected as contestants out of 9 possible contestant spots? We want the probability a student from your school is chosen out of total students times total ways to choose students from your school: [U]a) P(5 students being selected):[/U] 5/30 * 4/(120 - 30) 5/30 * 4/90 20/2700 [URL='https://www.mathcelebrity.com/fraction.php?frac1=20%2F2700&frac2=3%2F8&pl=Simplify']Simplifying this fraction[/URL], we get: 1/135 [U]b) Total Ways 9 students can be picked from your school:[/U] 9/120 [URL='https://www.mathcelebrity.com/fraction.php?frac1=9%2F120&frac2=3%2F8&pl=Simplify']Simplifying this fraction[/URL], we get: 3/40 Divide a by b: 1/135 / 3/40 40/405 [URL='https://www.mathcelebrity.com/fraction.php?frac1=40%2F405&frac2=3%2F8&pl=Simplify']Simplifying[/URL], we get: [B]8/81[/B]

A group of 4 adults and 5 children is visiting an amusement park. Admission is $15 per adult and $9
A group of 4 adults and 5 children is visiting an amusement park. Admission is $15 per adult and $9 per child. Find the total cost of admission for the group. Set up the cost function for adults and children: C(a, c) = 15a + 9c We want the cost for 4 adults and 5 children C(4, 5) = 15(4) + 9(5) C(4, 5) = 60 + 45 C(4, 5) = [B]105[/B]

A group of campers have 250 pounds of food. They plan to eat 12 pounds a day. How many days will it
A group of campers have 250 pounds of food. They plan to eat 12 pounds a day. How many days will it take them to eat the food. Write your answer in a linear equation. Let the number of days be d. We have the following equation: 12d = 250 To solve for d, we [URL='https://www.mathcelebrity.com/1unk.php?num=12d%3D250&pl=Solve']type this equation in our search engine[/URL] and we get: d = [B]20.833[/B]

A group of people was surveyed to determine what newspaper they read. 80% of those interviewed read
A group of people was surveyed to determine what newspaper they read. 80% of those interviewed read the New York Times, while 50% read U.S.A. Today. If 35% read both papers, what percent read neither paper? New York Times: 80% - 35% for both = 45% USA Today: 50% - 35% for both = 15% 45% + 15% + 35% = 95% Which means 100% - 95% = [B]5% read neither[/B]

A group of people were asked if they had run a red light in the last year. 497 responded "yes", and
A group of people were asked if they had run a red light in the last year. 497 responded "yes", and 223 responded "no". Find the probability that if a person is chosen at random, they have run a red light in the last year. P(Run Red Light) = yes answers / total answers P(Run Red Light) = 497 / (497 + 223) P(Run Red Light) = 497 / 720 P(Run Red Light) = [B]0.6903[/B]

A group of scientists studied the effect of a chemical on various strains of bacteria. Strain A star
A group of scientists studied the effect of a chemical on various strains of bacteria. Strain A started with 6000 cells and decreased at a constant rate of 2000 cells per hour after the chemical was applied. Strain B started with 2000 cells and decreased at a constant rate of 1000 cells per hour after the chemical was applied. When will the strains have the same number of cells? Explain. Set up strain equations where h is the number of hours since time 0: [LIST] [*]Strain A: 6000 - 2000h [*]Strain B: 2000 - 1000h [/LIST] Set them equal to each other 6000 - 2000h = 2000 - 1000h Using our [URL='http://www.mathcelebrity.com/1unk.php?num=6000-2000h%3D2000-1000h&pl=Solve']equation solver[/URL], we see that [B]h = 4[/B]

a group of students and teachers are going on a field trip. one ninth of the group will fit on 1/3 o
a group of students and teachers are going on a field trip. one ninth of the group will fit on 1/3 of a school bus how many buses are needed to transport the entire group 1/9g = 1/3b We want to find g, so we multiply each side through by 9 g = 9/3b Simplify: g = 3b, so we need [B]3 buses[/B]

A group of students at a school takes a history test. The distribution is normal with a mean of 25,
A group of students at a school takes a history test. The distribution is normal with a mean of 25, and a standard deviation of 4. (a) Everyone who scores in the top 30% of the distribution gets a certificate. (b) The top 5% of the scores get to compete in a statewide history contest. What is the lowest score someone can get and still go onto compete with the rest of the state? (a) [URL='http://www.mathcelebrity.com/zcritical.php?a=0.70&pl=Calculate+Critical+Z+Value']Top 30% is 70% percentile[/URL] Inverse of normal distribution(0.7) = -0.5244005 Plug into z-score formula, -0.5244005 = (x - 25)/4 [B]x = 22.9024[/B] (b) [URL='http://www.mathcelebrity.com/zcritical.php?a=0.95&pl=Calculate+Critical+Z+Value']Top 5% is 95% percentile[/URL] Inverse of normal distribution(0.95) = 1.644853627 Plug into z-score formula, 1.644853627 = (x - 25)/4 [B]x = 31.57941451[/B]

A group of workers can plant 54 acres in 6 days. What is their rate in acres per day?
A group of workers can plant 54 acres in 6 days. What is their rate in acres per day? Acres per day = 54 acres / 6 days = [B]9 acres per day[/B]

A group of workers can plant 72 acres in 8 days what is the rate in acres per a day
A group of workers can plant 72 acres in 8 days what is the rate in acres per a day Acres per day = Total Acres / Total Days Acres per day = 72/8 Acres per day =[B] 9[/B]

A guitar that normally cost n dollars is on sale for 20% off. The tax is 8%. What is the total cost
A guitar that normally cost n dollars is on sale for 20% off. The tax is 8%. What is the total cost of the guitar including tax? Discount Amount = 0.2n Total paid after discount = n - 0.2n = 0.8n Tax amount: 0.8n * 0.08 = 0.064n After tax amount: 0.8n + 0.64n = [B]0.864n[/B]

A gym charges a $30 sign-up fee plus $20 per month. You have a $130 gift card for the gym. When doe
A gym charges a $30 sign-up fee plus $20 per month. You have a $130 gift card for the gym. When does the total spent on your gym membership exceed the amount of your gift card? Subtract the sign up fee of $30 from your gift card amount: $130 - $30 = $100 Since each month costs $20, we have $100/$20 = 5 months. So if you go for [B]more than 5 months[/B], you'll exceed your gift card.

A gym has 18 exercise stations, including 2 rowing machines. What is the probability that a randoml
A gym has 18 exercise stations, including 2 rowing machines. What is the probability that a randomly selected exercise station will be a rowing machine? The probability is 2/18. We can simplify this fraction. Divide top and bottom by 2: [B]1/9[/B]

A gym has yoga classes. Each class has 11 students. If there are c classes, write an equation to rep
A gym has yoga classes. Each class has 11 students. If there are c classes, write an equation to represent the total number of students s taking yoga. Total students is the number of classes times the number of students in each class: [B]s = 11c[/B]

A gym has yoga classes. Each class has 14 students. If there are c classes write an equation to repr
A gym has yoga classes. Each class has 14 students. If there are c classes write an equation to represent the total number of students s taking yoga s = students per class * number of classes [B]s = 14c[/B]

A gym membership has a $50 joining fee plus charges $17 a month for m months
A gym membership has a $50 joining fee plus charges $17 a month for m months Build a cost equation C(m) where m is the number of months of membership. C(m) = Variable Cost * variable units + Fixed Cost C(m) = Months of membership * m + Joining Fee Plugging in our numbers and we get: [B]C(m) = 17m + 50 [MEDIA=youtube]VGXeqd3ikAI[/MEDIA][/B]

A heating company charges $60 per hour plus $54 for a service call. Let n be the number of hours t
A heating company charges $60 per hour plus $54 for a service call. Let n be the number of hours the technician works at your house. The cost function C(n) where n is the number of hours is: C(n) = Hourly Rate * hours + Service Call Charge [B]C(n) = 60n + 54[/B]

A helicopter blade does 3206 full turns in 7 minutes , work out the number of full turns per minute
A helicopter blade does 3206 full turns in 7 minutes , work out the number of full turns per minute 3206 full turns / 7 minutes [URL='https://www.mathcelebrity.com/fraction.php?frac1=3206%2F7&frac2=3%2F8&pl=Simplify']Divide the fraction by 7 to get turns per minute[/URL] [B]458 turns per minute[/B]

A helicopter is flying at an altitude of 785 feet. It descends 570 feet, and then ascends 595 feet.
A helicopter is flying at an altitude of 785 feet. It descends 570 feet, and then ascends 595 feet. Write an expression to represent this situation. Then determine and interpret the sum. [LIST] [*]Start at +785 feet [*]Descend 570 feet means using a minus sign -570 [*]Ascend 595 feet means using a plus sign +595 [/LIST] [U]Calculate the sum:[/U] +785 - 570 + 595 [B]+810[/B]

A helicopter rose vertically 300 m and then flew west 400 m how far was the helicopter from it’s sta
A helicopter rose vertically 300 m and then flew west 400 m how far was the helicopter from it’s starting point? The distance forms a right triangle. We want the distance of the hypotenuse. Using our [URL='http://www.mathcelebrity.com/pythag.php?side1input=300&side2input=400&hypinput=&pl=Solve+Missing+Side']right triangle calculator[/URL], we get a distance of [B]500[/B]. We also could use a shortcut on this problem. If you divide 300 and 400 by 100, you get 3 and 4. Since we want the hypotenuse, you get the famous 3-4-5 triangle ratio. So the answer is 5 * 100 = 500.

A high school graduating class is made up of 440 students. There are 168 more girls than boys. How m
A high school graduating class is made up of 440 students. There are 168 more girls than boys. How many boys are in the class? Let b be the number of boys and g be the number of girls. We're given 2 equations: [LIST=1] [*]b + g = 440 [*]g = b + 168 [/LIST] Substitute (2) into (1) b + (b + 168) = 440 Combine like terms: 2b + 168 = 440 [URL='https://www.mathcelebrity.com/1unk.php?num=2b%2B168%3D440&pl=Solve']Type this equation into the search engine[/URL], and we get: [B]b = 136[/B]

A high school with 1000 students offers two foreign language courses : French and Japanese. There ar
A high school with 1000 students offers two foreign language courses : French and Japanese. There are 200 students in the French class roster, and 80 students in the Japanese class roster. We also know that 30 students enroll in both courses. Find the probability that a random selected student takes neither foreign language course. Let F be the event a student takes French and J be the event a student takes Japanese P(F) = 200/1000 = 0.2 P(J) = 80/1000 = 0.08 P(F ? J) = 30/1000 = 0.03 From our [URL='http://www.mathcelebrity.com/probunion2.php?pa=+0.2&pb=0.08+&paintb=+0.03&aub=+&pl=Calculate']two event calculator[/URL], we get P(F U J) = 0.25 So we want P(F U J)^C = 1 - P(F U J) = 1 - 0.25 = [B]0.75[/B]

A hill rises 60 ft for every horizontal 96 ft. Find the slope.
A hill rises 60 ft for every horizontal 96 ft. Find the slope. Slope = Rise / Run Slope = 60/96 Using [URL='https://www.mathcelebrity.com/fraction.php?frac1=60%2F96&frac2=3%2F8&pl=Simplify']our fraction simplifier, we reduce 60/96 [/URL]to [B]5/8[/B]

A holiday in Florida costs $876. A holiday in Bali costs $394. How much more expensive is the Florid
A holiday in Florida costs $876. A holiday in Bali costs $394. How much more expensive is the Florida holiday? We want the difference between Florida holiday costs and Bali holiday costs: $876 - $394 = [B]$482[/B]

A home is to be built on a rectangular plot of land with a perimeter of 800 feet. If the length is 2
A home is to be built on a rectangular plot of land with a perimeter of 800 feet. If the length is 20 feet less than 3 times the width, what are the dimensions of the rectangular plot? [U]Set up equations:[/U] (1) 2l + 2w = 800 (2) l = 3w - 20 [U]Substitute (2) into (1)[/U] 2(3w - 20) + 2w = 800 6w - 40 + 2w = 800 [U]Group the w terms[/U] 8w - 40 = 800 [U]Add 40 to each side[/U] 8w = 840 [U]Divide each side by 8[/U] [B]w = 105 [/B] [U]Substitute w = 105 into (2)[/U] l = 3(105) - 20 l = 315 - 20 [B]l = 295[/B]

A hoodie sold for d dollars. Now, the new price of the hoodie can be represented by 1.3d. Which desc
A hoodie sold for d dollars. Now, the new price of the hoodie can be represented by 1.3d. Which description could explain what happened to the price of the hoodie? We can rewrite this as: d(1 + 0.3) And in this format, we see that the [B]hoodie was increased by 30% [/B]which is also 1.3

a horse and a saddle cost $5,000. if the horse cost 4 times as much as the saddle, what was the cost
a horse and a saddle cost $5,000. if the horse cost 4 times as much as the saddle, what was the cost of each? Let the cost of the horse be h, and the cost of the saddle be s. We're given: [LIST=1] [*]h + s = 5000 [*]h = 4s [/LIST] Substitute equation (2) into equation (1): 4s + s = 5000 [URL='https://www.mathcelebrity.com/1unk.php?num=4s%2Bs%3D5000&pl=Solve']Type this equation into the search engine[/URL], we get: [B]s = 1,000[/B] Substitute s = 1000 into equation (2): h = 4(1000) [B]h = 4,000[/B]

A hot air balloon at 1120 feet descends at a rate of 80 feet per minute. Let y represent the height
A hot air balloon at 1120 feet descends at a rate of 80 feet per minute. Let y represent the height and let x represent the number of minutes the balloon descends. Descending means we subtract height, so we have: [B]y = 1120 - 80x[/B]

A hot dog costs $3 and a corn dog costs $1.50. If $201 was collected, write a mathematical sentence
A hot dog costs $3 and a corn dog costs $1.50. If $201 was collected, write a mathematical sentence to represent this information Assumptions: [LIST] [*]Let the number of corn dogs be c [*]Let the number of hot dogs be h [/LIST] Since cost = price * quantity, we have: [B]1.50c + 3h = 201[/B]

A house costs 3.5 times as much as the lot. Together they sold for $135,000. Find the cost of each
A house costs 3.5 times as much as the lot. Together they sold for $135,000. Find the cost of each. Let the house cost be h, and the lot cost be l. We have the following equations: [LIST=1] [*]h = 3.5l [*]h + l = 135,000 [/LIST] Substitute (1) into (2) 3.5l + l = 135,000 Combine like terms: 4.5l = 135,000 Divide each side by 4.5 to isolate l [B]l = 30,000[/B] Substitute this back into equation (1) h = 3.5(30,000) [B]h = 105,000[/B]

A house has 8 rooms one quarter of the rooms are bedrooms how many of the rooms are bedrooms
A house has 8 rooms one quarter of the rooms are bedrooms how many of the rooms are bedrooms 1/4 * 8 = 8/4 = [B]2 rooms are bedrooms[/B]

A house painting company charges $376 plus $12 per hour. Another painting company charges $280 plus
A house painting company charges $376 plus $12 per hour. Another painting company charges $280 plus $15 per hour. How long is a job for which companies will charge the same amount? Set up the cost function C(h) where h is the number of hours. Company 1: C(h) = 12h + 376 Company 2: C(h) = 15h + 280 To see when the companies charge the same amount, set both C(h) functions equal to each other. 12h + 376 = 15h + 280 To solve for h, we [URL='https://www.mathcelebrity.com/1unk.php?num=12h%2B376%3D15h%2B280&pl=Solve']type this equation into our search engine[/URL] and we get: h = [B]32[/B]

A house painting company charges $376 plus $12 per hour. Another painting company charges $280 plus
A house painting company charges $376 plus $12 per hour. Another painting company charges $280 plus $15 per hour. How long is a job for which both companies will charge the same amount? [U]Set up the cost function for the first company C(h) where h is the number of hours:[/U] C(h) = Hourly Rate * h + flat rate C(h) = 12h + 376 [U]Set up the cost function for the first company C(h) where h is the number of hours:[/U] C(h) = Hourly Rate * h + flat rate C(h) = 15h + 280 The problem asks how many hours will it take for both companies to charge the same. So we set the cost functions equal to each other: 12h + 376 = 15h + 280 Plugging this equation [URL='https://www.mathcelebrity.com/1unk.php?num=12h%2B376%3D15h%2B280&pl=Solve']into our search engine and solving for h[/URL], we get: h = [B]32[/B]

A house rental company charges a $700 for a week stay plus an additional $4 per night for a roll awa
A house rental company charges a $700 for a week stay plus an additional $4 per night for a roll away bed. Your family rents a house for a week and pays $756. How many roll away beds did they rent? Roll Away Beds = (Total Rental Price - Weekly Charge)/Per night bed fee Plugging in our numbers, we get: Roll Away Beds = (756 - 700)/4 Roll Away Beds = 56/4 Roll Away Beds = [B]14[/B]

A house sold for $200,000 and the real estate agent earned a commission of $10,200.00. Find the comm
A house sold for $200,000 and the real estate agent earned a commission of $10,200.00. Find the commission rate. Commission Rate = 100 * Commission Amount / Sale Price Commission Rate = 100 * 10200/20000 Commission Rate = 100 * 0.051 Commission Rate = [B]5.51%[/B]

A house valued at 70,000 in 1989 increased in value to 125,000 in 2000. Find a function which gives
A house valued at 70,000 in 1989 increased in value to 125,000 in 2000. Find a function which gives the value of the house, v, as a function of y, the number of years after 1989. Let's determine the years: 2000 - 1989 = 11 Let's determine the change in value: 125,000 - 70,000 = 55,000 Assuming a linear progression, we have: 55,000/11 = 5,000 per year increase [B]y = 70,000 + 5,000v[/B] where v is the number of years after 1989 Plug in 11 to check our work y = 70,000 + 5,000(11) y = 70,000 + 55,000 y = 125,000

A hungry Emperor penguin was swimming 21 feet below sea level. It swam up 9 feet to eat a fish. What
A hungry Emperor penguin was swimming 21 feet below sea level. It swam up 9 feet to eat a fish. What is the position of the penguin now relative to sea level? 21 feet below is written as -21 9 feet up is written as +9 So we have -21 + 9 = [B]-12 or 12 feet below sea level[/B]

a hypothesis test is to be performed.Determine the null and alternative hypotheses.
The mean credit card debt among households in one state is $8400. A hypothesis test is to be performed to decide whether the mean credit card debt for households in the formerly affluent town of Rich-No-More differs from the mean credit card debt for the state.

A hypothetical population consists of eight individuals ages 14,15,17,20,26,27,28, and 30 years. Wh
A hypothetical population consists of eight individuals ages 14,15,17,20,26,27,28, and 30 years. What is the probability of selecting a participant who is at least 20 years old? At least 20 means 20 or older, so our selection of individuals is: {20, 26, 27, 28, 30} This is 5 out of a possible 8, so we have [URL='http://www.mathcelebrity.com/perc.php?num=5&den=8&pcheck=1&num1=16&pct1=80&pct2=70&den1=80&idpct1=10&hltype=1&idpct2=90&pct=82&decimal=+65.236&astart=12&aend=20&wp1=20&wp2=30&pl=Calculate']5/8 of 0.625, which is 62.5%[/URL]

A is 0 and AR=19 what is the midpoint
A is 0 and AR=19 what is the midpoint [URL='https://www.mathcelebrity.com/mptnline.php?ept1=0&empt=&ept2=19&pl=Calculate+missing+Number+Line+item']Using our midpoint calculator, with one point at 0, and the other point at 19[/URL], we get the midpoint M: M = [B]19/2 or 9.5[/B]

a is 2 years older than b who is twice as old as c. if the total ages of a,b and c is 42, then how o
a is 2 years older than b who is twice as old as c. if the total ages of a,b and c is 42, then how old is b We're given 3 equations: [LIST=1] [*]a = b + 2 [*]b = 2c [*]a + b + c = 42 [/LIST] Substituting equation (2) into equation (1), we have: a = 2c + 2 Since b = 2c, we substitute both of these into equation (3) to get: 2c + 2 + 2c + c = 42 To solve for c, we [URL='https://www.mathcelebrity.com/1unk.php?num=2c%2B2%2B2c%2Bc%3D42&pl=Solve']type this equation into our math engine[/URL] and we get: c = 8 Now take c = 8 and substitute it into equation (2) above: b = 2(8) b = [B]16[/B]

A is the set of factors of 12
A is the set of factors of 12 Type in [URL='https://www.mathcelebrity.com/factoriz.php?num=12&pl=Show+Factorization']factor 12[/URL] into our math engine and we get: A = {[B]1, 2, 3, 4, 6, 12[/B]}

A is the set of integers greater than or equal to -5 and less than or equal to -2
A is the set of integers greater than or equal to -5 and less than or equal to -2 [B]-5 <= A <= -2[/B]

A is the set of odd integers between 4 and 12
A is the set of odd integers between 4 and 12 Let A be the set of odd numbers between 4 and 12: [B]A = {5, 7, 9, 11}[/B]

A jar contains 7 red marbles, 8 green marbles, and 6 blue marbles. What is the probability that you
A jar contains 7 red marbles, 8 green marbles, and 6 blue marbles. What is the probability that you draw 4 green marbles in a row if you do not replace the marbles after each draw? The key phrase in this problem is [I]do not replace[/I]. [U]Draw #1:[/U] P(Green) = Total Green Marbles in the Jar / Total Marbles in the Jar Total Green Marbles in the Jar = 8 Total Marbles in the Jar = 7 red + 8 green + 6 blue = 21 P(Green) = 8/21 [U]Draw #2:[/U] P(Green) = Total Green Marbles in the Jar / Total Marbles in the Jar Total Green Marbles in the Jar = 8 - 1 = 7 Total Marbles in the Jar = 7 red + 7 green + 6 blue = 20 P(Green) = 7/20 [U]Draw #3:[/U] P(Green) = Total Green Marbles in the Jar / Total Marbles in the Jar Total Green Marbles in the Jar = 7 - 1 = 6 Total Marbles in the Jar = 7 red + 6 green + 6 blue = 19 P(Green) = 6/19 [U]Draw #4:[/U] P(Green) = Total Green Marbles in the Jar / Total Marbles in the Jar Total Green Marbles in the Jar = 6 - 1 = 5 Total Marbles in the Jar = 7 red + 5 green + 6 blue = 18 P(Green) = 5/18 We want P(Green, Green, Green, Green) Because each draw is [U][B]independent[/B][/U] of all other draws, we multiply each draw to get the final probability P(Green, Green, Green, Green) = P(Green on Draw 1) * P(Green on Draw 2) * P(Green on Draw 3) * P(Green on Draw 4) * P(Green, Green, Green, Green) = 8/21 * 7/20 * 6/19 * 5/18 P(Green, Green, Green, Green) = 1680/143640 Using our [URL='https://www.mathcelebrity.com/fraction.php?frac1=1680%2F143640&frac2=3%2F8&pl=Simplify']fraction simplifier[/URL], we get: P(Green, Green, Green, Green) = [B]2/171 [MEDIA=youtube]b2C_D4_d0Ug[/MEDIA][/B]

A jar contains 80 nickels and dimes worth $6.40. How many of each kind of coin are in the jar?
A jar contains 80 nickels and dimes worth $6.40. How many of each kind of coin are in the jar? Using our [URL='http://www.mathcelebrity.com/coin-word-problem.php?coinvalue=6.40&cointot=80&coin1=nickels&coin2=dimes&pl=Calculate+Coin+Quantities']coin combination word problem calculator[/URL], we get: [LIST] [*][B]48 dimes[/B] [*][B]32 nickels[/B] [/LIST]

a jar contains a $5 note, two $10 notes, a $20 note and a $50 note. if 2 notes are taken out by rand
a jar contains a $5 note, two $10 notes, a $20 note and a $50 note. if 2 notes are taken out by random, find the probability that their sum is $15 To get a sum of $15, we'd need to pull the $5 and the $10. Since both events are indepdenent, we have: P($5 or 10) or P(whatever is not pulled in the first pull) First Pull: 2/4 (We can pull either a $10 or a $5, so 2 choices out of 4 bills) Second Pull: 1/3 <-- since there are only 3 bills and 1 bill to pull Each pull is independent, so we multiply: 2/4 * 1/3 = 2/12 We can simply this, so [URL='https://www.mathcelebrity.com/fraction.php?frac1=2%2F12&frac2=3%2F8&pl=Simplify']we type this fraction in our search engine[/URL] and we get: [B]1/6[/B]

A jet left Nairobi and flew east at an average speed of 231 mph. A passenger plane left four hours l
A jet left Nairobi and flew east at an average speed of 231 mph. A passenger plane left four hours later and flew in the same direction but with an average speed of 385 mph. How long did the jet fly before the passenger plane caught up? Jet distance = 231t Passenger plane distance = 385(t - 4) 385(t - 4) = 231t 385t - 1540 = 231t Subtract 231t from each side 154t = 1540 [URL='https://www.mathcelebrity.com/1unk.php?num=154t%3D1540&pl=Solve']Type 154t = 1540[/URL] into the search engine, we get [B]t = 10. [/B] Check our work: Jet distance = 231(10) = 2,310 Passenger plane distance = 385(10 - 4) = 385 * 6 = 2,310

A jet plane traveling at 550 mph over takes a propeller plane traveling at 150 mph that had a 3 hour
A jet plane traveling at 550 mph over takes a propeller plane traveling at 150 mph that had a 3 hours head start. How far from the starting point are the planes? Use the formula D = rt where [LIST] [*]D = distance [*]r = rate [*]t = time [/LIST] The plan traveling 150 mph for 3 hours: Time 1 = 150 Time 2 = 300 Time 3 = 450 Now at Time 3, the other plane starts Time 4 = 600 Time 5 = 750 Time 6 = 450 + 150t = 550t Subtract 150t 400t = 450 Divide each side by 400 t = 1.125 Plug this into either distance equation, and we get: 550(1.125) = [B]618.75 miles[/B]

A jet travels 832 km in 5 hours. At this rate, how far could the jet fly in 12 hours? What is the ra
A jet travels 832 km in 5 hours. At this rate, how far could the jet fly in 12 hours? What is the rate of speed of the jet? Distance = rate * time. We're given D = 832 and t = 5. Using our [URL='https://www.mathcelebrity.com/drt.php?d=+832&r=+&t=+5&pl=Calculate+the+missing+Item+from+D%3DRT']drt calculator[/URL], we solve or rate to get: [B]r = 166.4[/B] The problems asks for a distance D when t = 12 hours and r = 166.4 from above. Using our [URL='https://www.mathcelebrity.com/drt.php?d=&r=+166.4&t=+12&pl=Calculate+the+missing+Item+from+D%3DRT']drt calculator solving for d[/URL], we get: d = [B]1,996.8 km[/B]

A jet travels at 485 miles per hour. Which equation represents the distance, d, that the jet will tr
A jet travels at 485 miles per hour. Which equation represents the distance, d, that the jet will travel in t hours. The distance formula is: d = rt We're given r = 485, so we have: [B]d = 485t[/B]

A job pays $56 for 8 hours of work. how much money does the job pay per hour
A job pays $56 for 8 hours of work. how much money does the job pay per hour Hourly Wage = Total Wages / Total Hours Worked $56/8 = [B]$7 per hour[/B].

A jug holds 1.2 litres of orange juice. All of the juice is poured equally into six glasses. How muc
A jug holds 1.2 litres of orange juice. All of the juice is poured equally into six glasses. How much orange is in each glass? 1.2 litres / 6 glasses = [B]0.2 litres[/B] in each glass.

A kilogram of chocolate costs8 dollars. Sally buys p kilograms. Write an equation to represent the t
A kilogram of chocolate costs8 dollars. Sally buys p kilograms. Write an equation to represent the total cost c that Sally pays. c[B] = 8p[/B]

A kilogram of rice costs $2.05.what is the cost of 30kg of the rice?
A kilogram of rice costs $2.05.what is the cost of 30kg of the rice? We multiply the price for 1 kilogram by the 30 kilograms total: $2.05 * 30 = [B]$61.50[/B]

A kitchen measures 5 yd by 6 yd. How much would it cost to install new linoleum in the kitchen if th
A kitchen measures 5 yd by 6 yd. How much would it cost to install new linoleum in the kitchen if the linoleum costs $2 per square foot? The kitchen has an area of 5yd x 6yd = 30 sq yards. If the linoleum costs $2 per square foot, we have 30 sq yards / $2 per square foot = [B]$15[/B]

A ladder 25 feet long is leaning against a wall. If the base of the ladder is 7 feet from the wall,
A ladder 25 feet long is leaning against a wall. If the base of the ladder is 7 feet from the wall, how high up the wall does the ladder reach? We have a right triangle, where the ladder is the hypotenuse, and we want the measurement of one leg. Set up the pythagorean theorem with these given items using our P[URL='https://www.mathcelebrity.com/pythag.php?side1input=&side2input=7&hypinput=25&pl=Solve+Missing+Side']ythagorean Theorem Calculator[/URL]. We get Side 1 = [B]24 feet.[/B]

A ladder is 25 ft long. The ladder needs to reach to a window that is 24 ft above the ground. How fa
A ladder is 25 ft long. The ladder needs to reach to a window that is 24 ft above the ground. How far away from the building should the bottom of the ladder be placed? We have a right triangle, where the ladder is the hypotenuse, and the window side is one side. Using our right triangle and the [URL='https://www.mathcelebrity.com/pythag.php?side1input=&side2input=24&hypinput=25&pl=Solve+Missing+Side']pythagorean theorem calculator[/URL], we get a length of [B]7 ft [/B]for the ladder bottom from the wall.

A ladder rests 2.5 m from the base of a house. If the ladder is 4 m long, how far up the side of the
A ladder rests 2.5 m from the base of a house. If the ladder is 4 m long, how far up the side of the house will the ladder reach? We have a right triangle with the hypotenuse as 4, the one leg as 2.5 We want to solve for the other leg length. We use our [URL='https://www.mathcelebrity.com/pythag.php?side1input=&side2input=2.5&hypinput=4&pl=Solve+Missing+Side']right triangle solver[/URL] to get [B]3.122[/B]

A lady walks into a store and steals $100 bill from the register without the owners knowledge. She c
A lady walks into a store and steals $100 bill from the register without the owners knowledge. She comes back 5 minutes later and buys $70 worth of goods with the $100 bill. The owner gives her $30 in change. How much did the owner lose? [LIST=1] [*]After the lady steals $100, the owner is down -$100. [*]The lady comes back, and buys $70 worth of goods. At this point, the owner has -$100 + $70 = $-30. [*]Next, the owner gives the lady another $30 in change, making the owner's loss -$30 - $30 = [B]-$60[/B]. [/LIST]

a landscaper buys 1 gallon of plant fertilizer. he uses 1/5 of the fertilizer, and then divides the
a landscaper buys 1 gallon of plant fertilizer. he uses 1/5 of the fertilizer, and then divides the rest into 3 smaller bottles. how many gallons does he put into each bottle? First, we find the remaining fraction of fertilizer after using 1/5. [URL='https://www.mathcelebrity.com/fraction.php?frac1=1&frac2=1%2F5&pl=Subtract']Using our fraction calculator[/URL], we see: 1 - 1/5 = 4/5 To find the amount of fertilizer per bottle, we then [URL='https://www.mathcelebrity.com/fraction.php?frac1=4%2F5&frac2=3&pl=Divide']divide 4/5 by 3 and we get[/URL]: [B]4/15 gallon per bottle[/B]

A laptop is purchased for $1700. After each year, the resale value decreases by 25%. What will be th
A laptop is purchased for $1700. After each year, the resale value decreases by 25%. What will be the resale value after 5 years? [U]Let R(t) be the Resale value at time t:[/U] R(t) = 1,700(1 - 0.25)^t [U]We want R(5)[/U] R(5) = 1,700(1 - 0.25)^5 R(5) =1,700(0.75)^5 R(5) =1,700 * 0.2373 R(5) = [B]$403.42[/B]

A large bag of candy contains 84 blue candies, 96 red candies, and 120 yellow candies. What percent
A large bag of candy contains 84 blue candies, 96 red candies, and 120 yellow candies. What percent of the candies are red? [U]Calculate total candies:[/U] Total Candies = Blue Candies + Red Candies + Yellow Candies Total Candies = 84 + 96 + 120 Total Candies = 300 [U]Now calculate the red candy percentage:[/U] Red Candy Percent = 100 * Red Candies / Total Candies Red Candy Percent = 100 * 96 / 300 Red Candy Percent = 9600 / 300 Red Candy Percent = [B]32%[/B]

a large fry has 120 more calories than a small. 5 large fries is the same amount of calories as 7 sm
a large fry has 120 more calories than a small. 5 large fries is the same amount of calories as 7 small. How many calories does each size fry have? Let the number of calories in large fries be l. Let the number of calories in small fries be s. We're given two equations: [LIST=1] [*]l = s + 120 [*]5l = 7s [/LIST] Substitute equation (1) into equation (2): 5(s + 120) = 7s [URL='https://www.mathcelebrity.com/1unk.php?num=5%28s%2B120%29%3D7s&pl=Solve']Type this equation into the search engine[/URL] and we get: s = [B]300[/B] Substitute s = 300 into equation (1): l = 300 + 120 l = [B]420[/B]

A large storage container is filled with 44.9 quarts of water. One quart of water is equivalent to 3
A large storage container is filled with 44.9 quarts of water. One quart of water is equivalent to 32 fluid ounces. How many fluid ounces of water are stored in the container? Round your answer to the nearest whole number. 44.9 quarts * 32 fluid ounce / quart = 1,436.8 if we found to the nearest whole number, we round up since 0.8 is greater than 0.5, so we get: [B]1,437 fluid ounces[/B]

A laundry basket contains 30 socks, of which 9 are black. What is the probability that a randomly s
A laundry basket contains 30 socks, of which 9 are black. What is the probability that a randomly selected sock will be black? P(Black) = 9/30 Simplifying, we can divide top and bottom by 3: [B]3/10 3/10 as a percentage is 30%[/B]

A lead pipe 20 ft long is 3/8 inch thick and has an inner diameter of 3 inches. Find the volume of l
A lead pipe 20 ft long is 3/8 inch thick and has an inner diameter of 3 inches. Find the volume of lead in it. A lead pipe is a cylinder. We want the volume of a cylinder. Convert 20ft to inches: 20ft = 12(20) = 240 inches Find the inner radius: 1/2 * inner diameter 1/2 * 3 = 3/2 Now add the thickness for the total radius 3/2 + 3/8 = 12/8 + 3/8 = 15/8 Find volume of the lead where volume = pi r^2 h Lead vol (V) = Overall volume - inner volume Lead Vol = pi(15/8)^2(240) - pi(3/2)^2(240) Lead Vol = 240pi(225/64 - 9/4) 9/4 = 144/64 Lead Vol = 240pi(225/64 - 144/64) Lead Vol = 240pi(81/64) [B]Lead Vol = 303.75pi[/B]

a licence plate that has 3 numbers from 0 to 9 followed by 2 letters
a licence plate that has 3 numbers from 0 to 9 followed by 2 letters How many license plate combinations can we form? We multiply as follows: [LIST] [*][0-9] = 10 possible digits (D) [*]A-Z = 26 possible letters (L) [/LIST] The problem asks for this: DDDLL So we have: 10 * 10 * 10 * 26 * 26 = [B]676,000[/B] plates

a license plate has 3 letters followed by 4 numbers
a license plate has 3 letters followed by 4 numbers There are 26 letters A-Z and 10 numbers 0-9. So we have: 26 * 26 * 26 * 10 * 10 * 10 * 10 [B]175,760,000 different license plate combinations[/B]

A license plate is made up of 2 letter and 3 single digit numbers
A license plate is made up of 2 letter and 3 single digit numbers. There are 26 letters (A-Z). And there are 10 single digit numbers [0-9]. So our total combinations are: Letter - Letter - Number - Number - Number 26 * 26 * 10 * 10 * 10 = [B]676,000[/B]

A light bulb consumes 17100 watt-hours in 4 days and 18 hours. How many watt-hours does it consume a
A light bulb consumes 17100 watt-hours in 4 days and 18 hours. How many watt-hours does it consume a day? Since one day equals 24 hours, we have: 4 days and 18 hours equals: 4(24) + 18 hours 96 + 18 hours 114 hours Therefore, we have a proportion, where w is the number of watt-hours in a 24-hour period. 17,100 watt-hours/114 hours = w/24 [URL='https://www.mathcelebrity.com/prop.php?num1=17100&num2=w&den1=114&den2=24&propsign=%3D&pl=Calculate+missing+proportion+value']Typing 1711/114 = w/24 into our calculator[/URL], we get: [B]w = 3,600[/B]

A light flashes every 2 minutes a, second light flashes every 7 minutes, and a third light flashes e
A light flashes every 2 minutes a, second light flashes every 7 minutes, and a third light flashes every 8 minutes. If all lights flash together at 8 P.M., what is the next time of day they will all flash together [URL='https://www.mathcelebrity.com/gcflcm.php?num1=2&num2=7&num3=8&pl=LCM']We use our least common multiple calculator[/URL] to see when the 3 numbers have a common multiple: LCM of (2, , 8) = 56 minutes So this means we add 56 minutes to 8:00 P.M. and we get [B]8:56 P.M.[/B]

a lighthouse blinks every 12 minutes. A second lighthouse blinks every 10 minutes if they both blink
a lighthouse blinks every 12 minutes. A second lighthouse blinks every 10 minutes if they both blink at 8:10 P.M at what time will they next blink together We want the least common multiple of (10, 12). This will be the next time each number times a multiple equals the same number. [URL='https://www.mathcelebrity.com/gcflcm.php?num1=10&num2=12&num3=&pl=GCF+and+LCM']Typing in LCM 10,12 into our search engine[/URL], we get: 60 So if we start at 8:10, and 60 minutes later is when both lighthouses blink. 60 minutes equals 1 hour. So we add 1 hour to 8:10, we have [B]9:10[/B]

A lighthouse blinks every 12 minutes.A second lighthouse blinks every 10 minutes.If they both blink
A lighthouse blinks every 12 minutes.A second lighthouse blinks every 10 minutes.If they both blink at 8:10 P.M., at what time will they next blink together? We want to know the least common multiple, so that 12 and 10 intervals meet again.[URL='https://www.mathcelebrity.com/gcflcm.php?num1=10&num2=12&num3=&pl=GCF+and+LCM'] We type in LCM(10,12) into our search engine[/URL] and we get 60. 60 minutes is 1 hour, so we add this to 8:10 to get [B]9:10[/B]

A limo costs $85 to rent for 3 hours plus a 7% sales tax. What is the total cost to rent the limo fo
A limo costs $85 to rent for 3 hours plus a 7% sales tax. What is the total cost to rent the limo for 6 hours? Determine the number of 3 hour blocks: 3 hour blocks = Total Rental Time / 3 3 hour blocks = 6 hours / 3 3 hour blocks = 2 With 7% = 0.07, we have: Total Cost = $85 * / 3 hours * 2 (3 hour blocks) * 1.07 Total Cost = 85 * 2 * 1.07 Total Cost = [B]181.9[/B]

A line has a slope of 1/2 and a run of 50. Find the rise of the line.
A line has a slope of 1/2 and a run of 50. Find the rise of the line. Slope = Rise/Run We're given a run of 50, so let the rise be r. We have: r/50 = 1/2 To solve this proportion, we [URL='https://www.mathcelebrity.com/prop.php?num1=r&num2=1&den1=50&den2=2&propsign=%3D&pl=Calculate+missing+proportion+value']type it in our search engine[/URL] and we get: r = [B]25[/B]

A line has a slope of 7 and a y-intercept of -4. What is its equation in slope intercept form
A line has a slope of 7 and a y-intercept of -4. What is its equation in slope intercept form The slope-intercept equation for a line: y = mx + b where m is the slope Given m = 7, we have: y = 7x + b The y-intercept is found by setting x to 0: y = 7(0) + b y = 0 + b y = b We're given the y-intercept is -4, so we have: b = -4 So our slope-intercept equation is: [B]y = 7x - 4[/B]

A line in the xy-plane passes through the origin and has a slope of 4/5. What points lie on that lin
A line in the xy-plane passes through the origin and has a slope of 4/5. What points lie on that line. Our line equation is: y = mx + b We're given: m = 4/5 (x, y) = (0, 0) So we have: 0 = 4/5(0) + b 0 = 0 + b b = 0 Therefore, our line equation is: y = 4/5x [URL='https://www.mathcelebrity.com/function-calculator.php?num=y%3D4%2F5x&pl=Calculate']Start plugging in values here to get a list of points[/URL]

A line joins A (1, 3) to B (5, 8). (a) (i) Find the midpoint of AB.
A line joins A (1, 3) to B (5, 8). (a) (i) Find the midpoint of AB. We type in (1,3),(5,8) to our search engine. We [URL='https://www.mathcelebrity.com/slope.php?xone=1&yone=3&slope=+2%2F5&xtwo=5&ytwo=8&pl=You+entered+2+points']choose our midpoint of 2 points calculator,[/URL] and we get: [B](3, 11/2)[/B]

A line passes through the point -3,4 and has a slope of -5
A line passes through the point -3,4 and has a slope of -5 Using our [URL='http://A line passes through the point -3,4 and has a slope of -5']point slope calculator[/URL], we get a line equation of: y = -5x - 11

A line segment has the endpoints S(10, 7) and T(2, 7). Find the coordinates of its midpoint M.
A line segment has the endpoints S(10, 7) and T(2, 7). Find the coordinates of its midpoint M. [URL='https://www.mathcelebrity.com/slope.php?xone=2&yone=7&slope=+&xtwo=10&ytwo=7&bvalue=+&pl=You+entered+2+points']Using our midpoint calculator[/URL], we get: M = [B](6, 7)[/B]

A line segment is 26 centimeters long. If a segment, x centimeters, is taken, how much of the line s
A line segment is 26 centimeters long. If a segment, x centimeters, is taken, how much of the line segment remains? This means the leftover segment has a length of: [B]26 - x[/B]

a lion can run 72 feet in one second how far can the lion run in one minute
a lion can run 72 feet in one second how far can the lion run in one minute? Using our [URL='https://www.mathcelebrity.com/timecon.php?quant=1&pl=Calculate&type=minute']time conversions calculator by typing [I]1 minute[/I] into our search engine[/URL], we see: 1 minute = 60 seconds So 72 feet per second * 60 seconds / minute = [B]4,320 feet / minute[/B]

A loaf of bread has 35 slices. Ann eats 8 slices, Betty eats 6 slices, Carl eats 5, and Derrick eats
A loaf of bread has 35 slices. Ann eats 8 slices, Betty eats 6 slices, Carl eats 5, and Derrick eats 9 slices. What fraction of the loaf is left? [U]Calculate total slices eaten:[/U] Slices eaten = Ann + Betty + Carl + Derrick Slices eaten = 8 + 6 + 5 + 9 Slices eaten = 28 [U]Calculate remaining slices:[/U] Remaining slices = Slices in loaf - Slices Eaten Remaining slices = 35 - 28 Remaining slices = 7 [U]Calculate fraction of the loaf left:[/U] Fraction left = Remaining Slices / Slices in Loaf Fraction left = 7/35 We can simplify this fraction. We [URL='https://www.mathcelebrity.com/fraction.php?frac1=7%2F35&frac2=3%2F8&pl=Simplify']type 7/35 into our search engine[/URL], choose simplify, and we get: Fraction left = [B]1/5[/B]

A local bank charges 19 per month plus 3 cents per check. The credit union charges7 per month plus
A local bank charges 19 per month plus 3 cents per check. The credit union charges7 per month plus 7 cents per check. How many checks should be written each month to make the credit union a better deal? Set up the cost function B(c) for the local bank where c is the number of checks: B(c) = 0.03c + 19 Set up the cost function B(c) for the credit union where c is the number of checks: B(c) = 0.07c + 7 We want to find out when: 0.07c + 7 < 0.03c + 19 [URL='https://www.mathcelebrity.com/1unk.php?num=0.07c%2B7%3C0.03c%2B19&pl=Solve']Typing this inequality into our search engine[/URL], we get: c < 300

A local college classifies its students by major, year (Freshman, Sophomore, Junior, Senior) and sex
A local college classifies its students by major, year (Freshman, Sophomore, Junior, Senior) and sex (M, F). If the college offers 20 majors, how many combinations are possible? We have 20 majors, 4 grade levels, and 2 sexes. The total combinations = 20 * 4 * 2 = [B]160[/B]

A local Dunkin’ Donuts shop reported that its sales have increased exactly 16% per year for the last
A local Dunkin’ Donuts shop reported that its sales have increased exactly 16% per year for the last 2 years. This year’s sales were $80,642. What were Dunkin' Donuts' sales 2 years ago? Declare variable and convert numbers: [LIST] [*]16% = 0.16 [*]let the sales 2 years ago be s. [/LIST] s(1 + 0.16)(1 + 0.16) = 80,642 s(1.16)(1.16) = 80,642 1.3456s = 80642 Solve for [I]s[/I] in the equation 1.3456s = 80642 [SIZE=5][B]Step 1: Divide each side of the equation by 1.3456[/B][/SIZE] 1.3456s/1.3456 = 80642/1.3456 s = 59930.142687277 s = [B]59,930.14[/B]

A local radio station sells time slots for programs in 20-minute intervals. If the station operates
A local radio station sells time slots for programs in 20-minute intervals. If the station operates 24 hours per day, what is the total number of 20-minute time slots the radio station can sell for Thursday and Friday? Thursday and Friday = 2 days With 24 hours per day, we have 24 * 2 = 48 hours for Thursday and Friday. Since 20 minutes is 1/3 of an hour, then we have 3 20-minute time slots per hour. 3 20-minute time slots * 48 hours = [B]144[/B] total 20-minute time slots

A local shop sold 499 hamburgers and cheese burgers. There were 51 fewer cheese burgers sold. How ma
A local shop sold 499 hamburgers and cheese burgers. There were 51 fewer cheese burgers sold. How many hamburgers were sold? Let h = number of hamburgers sold and c be the number of cheeseburgers sold. We have two equations: (1) c = h - 51 (2) c + h = 499 Substitute (1) into (2) h - 51 + h = 499 Combine like terms 2h - 51 = 499 Add 51 to both sides 2h = 550 Divide each side by 2 to isolate h [B]h = 275[/B]

A local sports centre charges $8 per visit. For an annual membership fee of$45, the cost per visit i
A local sports centre charges $8 per visit. For an annual membership fee of$45, the cost per visit is only $5.50. What is the least number of visits needed in a year in order for the membership to be a better deal? Set up the cost for the visitors plan C(v) where v is the number of visits: C(v) = 8v Set up the cost for the membership plan C(v) where v is the number of visits: C(v) = 5v + 45 The problem asks for v where: 5v + 45 < 8v [URL='https://www.mathcelebrity.com/1unk.php?num=5v%2B45%3C8v&pl=Solve']Type this inequality into our search engine[/URL] and get: v > 15 This means, the least number of visits is 1 more which is [B]16[/B]

A lottery offers 1 $1000 prize and 5 $100 prizes. 1000 tickets are sold. Find the expectation if a p
A lottery offers 1 $1000 prize and 5 $100 prizes. 1000 tickets are sold. Find the expectation if a person buys 1 ticket for $5. Set up the expected values E(x): for the 1,000 price: E(x) = (1000 - 5) * 1/1000 = 995/1000 For the 5 $100 prizes: E(x) = (100 - 5) * 5/1000 = 475/1000 For the losing ticket. With 6 winning tickets, we have 1000 - 6 = 994 losing tickets: E(x) = -3 * 994/1000 = -2982/1000 We get our total expected value by adding all of these expected values up. Since they all have the same denominator, we add numerators: E(x) = (995 + 475 - 2982)/1000 E(x) = -1512/1000 E(x) = [B]-1.51[/B]

A lottery uses a container with 25 identical balls numbered 1 through 25, from which three balls are
A lottery uses a container with 25 identical balls numbered 1 through 25, from which three balls are selected. What is the theoretical probability that the number 13 is picked first? P(1st ball being 13) = [B]1 /25[/B]

A luncheon for 14 guests cost $468.00. What was the average cost per guest?
A luncheon for 14 guests cost $468.00. What was the average cost per guest? Average Cost per Guest = Total Cost / Number of Guests Average Cost per Guest = $468 / 14 Average Cost per Guest = [B]$33.43[/B]

a machine has a first cost of 13000 an estimated life of 15 years and an estimated salvage value of
a machine has a first cost of 13000 an estimated life of 15 years and an estimated salvage value of 1000.what is the book value at the end of 9 years? Using [URL='https://www.mathcelebrity.com/depsl.php?d=&a=13000&s=1000&n=15&t=9&bv=&pl=Calculate']our straight line depreciation calculator[/URL], we get a book value at time 9, B9 of: [B]5,800[/B]

A machine prints 230 movie posters each hour. Write and solve an equation to find the number of hour
A machine prints 230 movie posters each hour. Write and solve an equation to find the number of hours it takes the machine to print 1265 posters. Let h be the number of hours. We're given the following expression for the printing output of the machine: 230h The questions asks for how long (h) to print 1265 posters, so we setup the equation: 230h = 1265 To solve for h, we [URL='https://www.mathcelebrity.com/1unk.php?num=230h%3D1265&pl=Solve']type this equation into our math engine[/URL] and we get: h = [B]5.5 hours[/B]

A machine shop employee earned $642 last week. She worked 40hours at her regular rate and 9 hours at
A machine shop employee earned $642 last week. She worked 40hours at her regular rate and 9 hours at a time and a half rate. Find her regular hourly rate. Let the regular hourly rate be h. We're given: 40h + 40(1.5)(h - 40) = 642 Multiply through and simplify: 40h + 60h - 2400 = 642 100h - 2400 = 642 [URL='https://www.mathcelebrity.com/1unk.php?num=100h-2400%3D642&pl=Solve']To solve for h, we type this equation into our search engine[/URL] and we get: h = [B]30.42[/B]

A magic box has pennies in it that double every minute. If the box takes a full hour to become compl
A magic box has pennies in it that double every minute. If the box takes a full hour to become completely full, how long does it take for the box to become half full? At the hour mark, it's 100% full. Half full means 50%. Since it doubles every minute, then at the [B]59th minute[/B], it's half full.

A mail courier charges a base fee of $4.95 plus $11.90 per package being delivered. If x represents
A mail courier charges a base fee of $4.95 plus $11.90 per package being delivered. If x represents the number of packages delivered, which of the following equations could be used to find y, the total cost of mailing packages? Set up the cost function y = C(x) [B]C(x) = 4.95 + 11.90x[/B]

A man bought a mobile phone for $800 and sold it for $1000. What was his profit as a percentage of t
A man bought a mobile phone for $800 and sold it for $1000. What was his profit as a percentage of the cost price Calculate Profit: Profit = Sales Price - Cost Profit = 1000 - 800 Profit = 200 Calculate profit percentage: Profit Percentage = Profit * 100 / Cost Profit Percentage = 800 * 100 / 200 Profit Percentage = [B]400%[/B]

A man invested part of $15,000 at 12% and the remainder at 8%. If his annual income from the investm
A man invested part of $15,000 at 12% and the remainder at 8%. If his annual income from the investments is $1456, how much does he have invested at each rate? Using our [URL='https://www.mathcelebrity.com/split-fund-interest-calculator.php?p=15000&i1=12&i2=8&itot=1456&pl=Calculate']split fund interest calculator[/URL], we get: [LIST] [*]Fund 1 Investment @ 12% = [B]6,400[/B] [*]Fund 2 Investment @ 8% =[B] [B]8,600[/B][/B] [/LIST]

A man invests $5,200, part at 4% and the balance at 3%. If his total income for the two investments
A man invests $5,200, part at 4% and the balance at 3%. If his total income for the two investments is $194, how much money did he invest at each rate? Using our [URL='https://www.mathcelebrity.com/split-fund-interest-calculator.php?p=5200&i1=4&i2=3&itot=194&pl=Calculate']split fund interest calculator[/URL], we get: [LIST] [*][B]Fund 1 = $3,800[/B] [*][B]Fund 2 = $1,400[/B] [/LIST]

A man is 5 years older than his wife, and the daughter age is half of the mother, and if you add the
A man is 5 years older than his wife, and the daughter age is half of the mother, and if you add their ages is equal 100 Let the man's age be m. Let the wife's age be w. Let the daughter's age be d. We're given: [LIST=1] [*]m = w + 5 [*]d = 0.5m [*]d + m + w = 100 [/LIST] Rearrange equation 1 in terms of w my subtracting 5 from each side: [LIST=1] [*]w = m - 5 [*]d = 0.5m [*]d + m + w = 100 [/LIST] Substitute equation (1) and equation (2) into equation (3) 0.5m + m + m - 5 = 100 We [URL='https://www.mathcelebrity.com/1unk.php?num=0.5m%2Bm%2Bm-5%3D100&pl=Solve']type this equation into our search engine[/URL] to solve for m and we get: m = [B]42 [/B] Now, substitute m = 42 into equation 2 to solve for d: d = 0.5(42) d = [B]21 [/B] Now substitute m = 42 into equation 1 to solve for w: w = 42 - 5 w = [B]37 [/B] To summarize our ages: [LIST] [*]Man (m) = 42 years old [*]Daughter (d) = 21 years old [*]Wife (w) = 37 years old [/LIST]

A man is four time as old as his son. How old is the man if the sum of their ages is 60?
A man is four time as old as his son. How old is the man if the sum of their ages is 60? Let the son's age be a. Then the man's age is 4a. If the sum of their ages is 60, we have: a + 4a = 60 To solve this equation for a, we [URL='https://www.mathcelebrity.com/1unk.php?num=a%2B4a%3D60&pl=Solve']type it in our math engine[/URL] and we get: a = 12 Therefore, the man's age is: 4(12) = [B]48[/B]

A man is four times as old as his son. In five years time he will be three times as old. Find their
A man is four times as old as his son. In five years time he will be three times as old. Find their present ages. Let the man's age be m, and the son's age be s. We have: [LIST=1] [*]m = 4s [*]m + 5 = 3(s + 5) [/LIST] Substitute (1) into (2) 4s + 5 = 3s + 15 Use our [URL='http://www.mathcelebrity.com/1unk.php?num=4s%2B5%3D3s%2B15&pl=Solve']equation calculator[/URL], and we get [B]s = 10[/B]. m = 4(10) [B]m = 40[/B]

A man is swimming at 10 feet below sea level. Then he descends 1 feet and then ascends 5 feet. What
A man is swimming at 10 feet below sea level. Then he descends 1 feet and then ascends 5 feet. What is the man's new location? 10 feet below sea level is written as -10 He descends 1 foot. Since descending is negative depth, we have: -10 - 1 = -11 Then he ascends 5 feet, so we add: -11 + 5 = [B]-6 or 6 feet below sea level[/B]

A man left a warehouse at 9:00 a.m. and travels 150 km to reach his home. What is the speed if he a
A man left a warehouse at 9:00 a.m. and travels 150 km to reach his home. What is the speed if he arrives at 11:00 a.m.? [LIST] [*]His trip took 2 hours (11 - 9) [*]He traveled 150 km in 2 hours [*]His speed is measured in km per hour [/LIST] If we have 150km/2 hours, we want his speed in km per hour Divide top and bottom by 2 [B]75km/hr[/B]

A man purchased 20 tickets for a total of $225. The tickets cost $15 for adults and $10 for children
A man purchased 20 tickets for a total of $225. The tickets cost $15 for adults and $10 for children. What was the cost of each ticket? Declare variables: [LIST] [*]Let a be the number of adult's tickets [*]Let c be the number of children's tickets [/LIST] Cost = Price * Quantity We're given two equations: [LIST=1] [*]a + c = 20 [*]15a + 10c = 225 [/LIST] Rearrange equation (1) in terms of a: [LIST=1] [*]a = 20 - c [*]15a + 10c = 225 [/LIST] Now that I have equation (1) in terms of a, we can substitute into equation (2) for a: 15(20 - c) + 10c = 225 Solve for [I]c[/I] in the equation 15(20 - c) + 10c = 225 We first need to simplify the expression removing parentheses Simplify 15(20 - c): Distribute the 15 to each term in (20-c) 15 * 20 = (15 * 20) = 300 15 * -c = (15 * -1)c = -15c Our Total expanded term is 300-15c Our updated term to work with is 300 - 15c + 10c = 225 We first need to simplify the expression removing parentheses Our updated term to work with is 300 - 15c + 10c = 225 [SIZE=5][B]Step 1: Group the c terms on the left hand side:[/B][/SIZE] (-15 + 10)c = -5c [SIZE=5][B]Step 2: Form modified equation[/B][/SIZE] -5c + 300 = + 225 [SIZE=5][B]Step 3: Group constants:[/B][/SIZE] We need to group our constants 300 and 225. To do that, we subtract 300 from both sides -5c + 300 - 300 = 225 - 300 [SIZE=5][B]Step 4: Cancel 300 on the left side:[/B][/SIZE] -5c = -75 [SIZE=5][B]Step 5: Divide each side of the equation by -5[/B][/SIZE] -5c/-5 = -75/-5 c = [B]15[/B] Recall from equation (1) that a = 20 - c. So we substitute c = 15 into this equation to solve for a: a = 20 - 15 a = [B]5[/B]

A man stands at point p, 45 metres from the base of a building that is 20 metres high. Find the angl
A man stands at point p, 45 metres from the base of a building that is 20 metres high. Find the angle of elevation of the top of the building from the man. Draw a right triangle ABC where Side A is from the bottom of the building to the man and Side B is the bottom of the building to the top of the building. Using right triangle calculations, we want Angle A which is the angle of elevation. [URL='http://www.mathcelebrity.com/righttriangle.php?angle_a=&a=20&angle_b=&b=45&c=&pl=Calculate+Right+Triangle']Angle of Elevation[/URL] which is [B]23.9625°[/B]

A man's age (a) 10 years ago is 43
A man's age (a) 10 years ago is 43 [U]10 years ago means we subtract 10 from a:[/U] a - 10 [U]The word [I]is[/I] means an equation. So we set a - 10 equal to 43 to get our algebraic expression[/U] [B]a - 10 = 43[/B] If the problem asks you to solve for a, [URL='https://www.mathcelebrity.com/1unk.php?num=a-10%3D43&pl=Solve']we type this equation into our search engine[/URL] and we get: a = 53

A man's age (a) 10 years ago is 43.
A man's age (a) 10 years ago is 43. Years ago means we subtract [B]a - 10 = 43 [/B] If the problem asks you to solve for a, we type this equation into our math engine and we get: Solve for [I]a[/I] in the equation a - 10 = 43 [SIZE=5][B]Step 1: Group constants:[/B][/SIZE] We need to group our constants -10 and 43. To do that, we add 10 to both sides a - 10 + 10 = 43 + 10 [SIZE=5][B]Step 2: Cancel 10 on the left side:[/B][/SIZE] a = [B]53[/B]

a mans age (a) ten years ago
a mans age (a) ten years ago The problem asks for an algebraic expression for age. The phrase [I]ago[/I] means before now, so they were younger. And younger means we [B]subtract[/B] from our current age: [B]a - 10[/B]

A manufacturer has a monthly fixed cost of $100,000 and a production cost of $10 for each unit produ
A manufacturer has a monthly fixed cost of $100,000 and a production cost of $10 for each unit produced. The product sells for $22/unit. The cost function for each unit u is: C(u) = Variable Cost * Units + Fixed Cost C(u) = 10u + 100000 The revenue function R(u) is: R(u) = 22u We want the break-even point, which is where: C(u) = R(u) 10u + 100000 = 22u [URL='https://www.mathcelebrity.com/1unk.php?num=10u%2B100000%3D22u&pl=Solve']Typing this equation into our search engine[/URL], we get: u =[B]8333.33[/B]

A manufacturer has a monthly fixed cost of $100,000 and a production cost of $12 for each unit produ
A manufacturer has a monthly fixed cost of $100,000 and a production cost of $12 for each unit produced. The product sells for $20/unit [U]Cost Function C(u) where u is the number of units:[/U] C(u) = cost per unit * u + fixed cost C(u) = 12u + 100000 [U]Revenue Function R(u) where u is the number of units:[/U] R(u) = Sale price * u R(u) = 20u Break even point is where C(u) = R(u): C(u) = R(u) 12u + 100000 = 20u To solve for u, we [URL='https://www.mathcelebrity.com/1unk.php?num=12u%2B100000%3D20u&pl=Solve']type this equation into our search engine[/URL] and we get: u = [B]12,500[/B]

A manufacturer has a monthly fixed cost of $100,000 and a production cost of $14 for each unit produ
A manufacturer has a monthly fixed cost of $100,000 and a production cost of $14 for each unit produced. The product sells for $20/unit. Let u be the number of units. We have a cost function C(u) as: C(u) = Variable cost * u + Fixed Cost C(u) = 14u + 100000 [U]We have a revenue function R(u) with u units as:[/U] R(u) = Sale Price * u R(u) = 20u [U]We have a profit function P(u) with u units as:[/U] Profit = Revenue - Cost P(u) = R(u) - C(u) P(u) = 20u - (14u + 100000) P(u) = 20u - 14u - 100000 P(u) = 6u - 1000000

A manufacturer has a monthly fixed cost of $25,500 and a production cost of $7 for each unit produce
A manufacturer has a monthly fixed cost of $25,500 and a production cost of $7 for each unit produced. The product sells for $10/unit. Set up cost function where u equals each unit produced: C(u) = 7u + 25,500 Set up revenue function R(u) = 10u Break Even is where Cost equals Revenue 7u + 25,500 = 10u Plug this into our [URL='http://www.mathcelebrity.com/1unk.php?num=7u%2B25500%3D10u&pl=Solve']equation calculator[/URL] to get [B]u = 8,500[/B]

A manufacturer has a monthly fixed cost of $52,500 and a production cost of $8 for each unit produce
A manufacturer has a monthly fixed cost of $52,500 and a production cost of $8 for each unit produced. The product sells for $13/unit. Using our [URL='http://www.mathcelebrity.com/cost-revenue-profit-calculator.php?fc=52500&vc=8&r=13&u=20000%2C50000&pl=Calculate']cost-revenue-profit calculator[/URL], we get the following: [LIST] [*]P(x) = 55x - 2,500 [*]P(20,000) = 47,500 [*]P(50,000) = 197,500 [/LIST]

a manufacturing company has a debt to equity ratio of 3 to 2. if the company has a debt of $12 milli
a manufacturing company has a debt to equity ratio of 3 to 2. if the company has a debt of $12 million, how much does it have in equity? Set up a proportion of debt to equity 3/2 = 12/x Using our [URL='http://www.mathcelebrity.com/prop.php?num1=3&num2=12&den1=2&den2=x&propsign=%3D&pl=Calculate+missing+proportion+value']proportion calculator[/URL], we get: x = 8

A marathon runner took 2 hours and 15 minutes to complete the race. During that time he spent 50 min
A marathon runner took 2 hours and 15 minutes to complete the race. During that time he spent 50 minutes in the lead. Write down, in its simplest form, the fraction of time he spent in the lead. [U]Calculate total race time in minutes[/U] [URL='https://www.mathcelebrity.com/timecon.php?quant=2&pl=Calculate&type=hour']2 hours[/URL] = 120 minutes 120 minutes + 15 minutes = 135 minutes [U]Calculate fraction of lead time[/U] Fraction of lead time = Time spent in lead / total race time Fraction of lead time = 50/135 Simplifying this fraction, we get: [URL='https://www.mathcelebrity.com/fraction.php?frac1=50%2F135&frac2=3%2F8&pl=Simplify']Fraction of lead time[/URL] = [B]10/27[/B]

A Math Quiz has 5 multiple choice option. Each question has four options. Find the number of possibl
[SIZE=6][B]A quiz has 5 questions with 4 answer choices each find the number of possible outcomes[/B] [B][/B] [B]We have 4 * 4 * 4 * 4 * 4 = 1024 outcomes[/B][/SIZE]

A math teacher bought 40 calculators at $8.20 each and a number of other calculators costing$2.95 ea
A math teacher bought 40 calculators at $8.20 each and a number of other calculators costing$2.95 each. In all she spent $387. How many of the cheaper calculators did she buy Let the number of cheaters calculators be c. Since amount equals price * quantity, we're given the following equation: 8.20 * 40 + 2.95c = 387 To solve this equation for c, we [URL='https://www.mathcelebrity.com/1unk.php?num=8.20%2A40%2B2.95c%3D387&pl=Solve']type it in our search engine [/URL]and we get: c = [B]20[/B]

A Math teacher gives one test a week to his class of 31 students. Estimate the number of tests the t
A Math teacher gives one test a week to his class of 31 students. Estimate the number of tests the teacher will mark in 39 weeks. 31 students * 1 test per week * 39 weeks = [B]1,209 tests[/B]

A math test is worth 100 points and has 38 problems. Each problem is worth either 5 points or 2 poin
A math test is worth 100 points and has 38 problems. Each problem is worth either 5 points or 2 points. How many problems of each point value are on the test? Let's call the 5 point questions m for multiple choice. Let's call the 2 point questions t for true-false. We have two equations: [LIST=1] [*]m + t = 38 [*]5m + 2t = 100 [/LIST] Rearrange (1) to solve for m - subtract t from each side: 3. m = 38 - t Now, substitute (3) into (2) 5(38 - t) + 2t = 100 190 - 5t + 2t = 100 Combine like terms: 190 - 3t = 100 Using our [URL='http://www.mathcelebrity.com/1unk.php?num=190-3t%3D100&pl=Solve']equation solver[/URL], we get [B]t = 30[/B]. Plugging t = 30 into (1), we get: 30 + t = 38 Using our [URL='http://www.mathcelebrity.com/1unk.php?num=m%2B30%3D38&pl=Solve']equation solver[/URL] again, we get [B]m = 8[/B]. Check our work for (1) 8 + 30 = 38 <-- Check Check our work for (2) 5(8) + 2(30) ? 100 40 + 60 ? 100 100 = 100 <-- Check You could also use our [URL='http://www.mathcelebrity.com/simultaneous-equations.php?term1=m+%2B+t+%3D+38&term2=5m+%2B+2t+%3D+100&pl=Cramers+Method']simultaneous equations calculator[/URL]

A mathematician has 8 favorite paintings and only 6 wall hooks to hang the paintings. How many diffe
A mathematician has 8 favorite paintings and only 6 wall hooks to hang the paintings. How many different ways can she hang the paintings? 8 paintings taken 6 at a time is written as: [URL='https://www.mathcelebrity.com/permutation.php?num=8&den=6&pl=Permutations']8P6[/URL] = [B]20,160[/B]

A mechanic charges $45 per hour and parts cost $125. Write an expression for the total if the mechan
A mechanic charges $45 per hour and parts cost $125. Write an expression for the total if the mechanic works h hours. Set up the cost function C(h) where h is the number of hours worked: C(h) = Hourly Rate * h + parts C(h) = [B]45h + 125[/B]

A mechanic charges $50 to inspect your heater, plus $80 per hour to work on it. You owe the mechani
A mechanic charges $50 to inspect your heater, plus $80 per hour to work on it. You owe the mechanic a total of $310. Write and solve an equation to find the amount of time h (in hours) the mechanic works on your heater. We calculate the cost function C(h) as: C(h) = Hourly Rate * hours + Flat Fee Inspection C(h) = 80h + 50 <-- this is our cost equation Now, we want to solve for h when C(h) = 310 80h + 50 = 310 [URL='https://www.mathcelebrity.com/1unk.php?num=80h%2B50%3D310&pl=Solve']We type this equation into our search engine[/URL] and we get: h = [B]3.25[/B]

A mechanic will charge a new customer $45.00 for an initial diagnosis plus $20 an hour of labor. How
A mechanic will charge a new customer $45.00 for an initial diagnosis plus $20 an hour of labor. How long did the mechanic work on a car if he charged the customer $165? We set up a cost function C(h) where h is the number of hours of labor: C(h) = Hourly Labor Rate * h + Initial Diagnosis C(h) = 20h + 45 The problem asks for the number of hours if C(h) = 165. So we set our cost function C(h) above equal to 165: 20h + 45 = 165 To solve for h, [URL='https://www.mathcelebrity.com/1unk.php?num=20h%2B45%3D165&pl=Solve']we plug this equation into our search engine[/URL] and we get: h = [B]6[/B]

A medium orange has 70 calories. This is 10 calories less then 1/4 of the calories in a sugar krunch
A medium orange has 70 calories. This is 10 calories less then 1/4 of the calories in a sugar krunchy. How many calories are in a sugar crunchy? Let s = calories in a sugar crunch. Let o = 70 be the calories in a medium orange. Set up the equation: o = 1/4s - 10 70 = 1/4s - 10 Add 10 to each side 1/4s = 80 Multiply each side by 4 [B]s = 320[/B]

A members-only speaker series allows people to join for $16 and then pay $1 for every event attended
A members-only speaker series allows people to join for $16 and then pay $1 for every event attended. What is the maximum number of events someone can attend for a total cost of $47? Subtract the join fee from the total cost: $47 - $16 = $31 Now divide this number by the cost per event: $31 / $1 = [B]31 events[/B]

A metal block is made of nickel and copper. The weight of metals in the block are in a ratio of 2:9.
A metal block is made of nickel and copper. The weight of metals in the block are in a ratio of 2:9. The weight of the block is 407 pounds. What is the weight of the nickel? Using our [URL='https://www.mathcelebrity.com/ratio.php?simpratio=100%3A350&rs=2%3A9&rtot=407&pl=Calculate+Ratio']ratio calculator[/URL], we get: [B]74 pounds[/B]

A meter is defined as the distance light travels in 1/299,792,458 of a second. How many meters does
A meter is defined as the distance light travels in 1/299,792,458 of a second. How many meters does light travel in 1/8 of a second? 1/8 second / 1/299,792,458 299,792,458/8 = [B]37,474,057.25 meters[/B]

A Middleweight UFC fighter weighs between 170 lbs and 185 lbs.
A Middleweight UFC fighter weighs between 170 lbs and 185 lbs. Let w be the UFC fighter's weight: We have a compound inequality. Right side includes 185 lbs. because between means includes 185lbs. Left side includes 170 lbs. because between means includes 17lb0s [B]170 <= w <= 185[/B]

A milk booth sells 445 litres of milk in a day. How many litres of milk will it sell in 4 years
A milk booth sells 445 litres of milk in a day. How many litres of milk will it sell in 4 years Calculate the number of days in 4 years: Days in 4 years = Days in 1 year * 4 Days in 4 years = 365 * 4 Days in 4 years = 1,460 Calculate litres of milk sold in 4 years: Litres of milk sold in 4 years = Litres of milk sold in 1 day * Days in 4 years Litres of milk sold in 4 years = 445 * 1,460 Litres of milk sold in 4 years = [B]649,700 litres[/B]

A monster energy drink has 164 mg of caffeine. Each hour your system reduces the amount of caffeine
A monster energy drink has 164 mg of caffeine. Each hour your system reduces the amount of caffeine by 12%. Write an equation that models the amount of caffeine that remains in your body after you drink an entire monster energy. Set up a function C(h) where he is the number of hours after you drink the Monster energy drink: Since 12% as a decimal is 0.12, we have: C(h) = 164 * (1 - 0.12)^h <-- we subtract 12% since your body flushes it out [B]C(h) = 164 * (0.88)^h[/B]

a more than b is greater than 6
a more than b is greater than 6 a more than b: b + a Is greater than 6 means an inequality using the > sign: [B]b + a > 6[/B]

A mother duck lines her 13 ducklings up behind her. In how many ways can the ducklings line up
A mother duck lines her 13 ducklings up behind her. In how many ways can the ducklings line up? In position one, we can have any of the 13 ducks. In position two, we can have 12 ducks, since one has to occupy position one. We subtract 1 each time until we fill up all 13 positions. We have: 13 * 12 * 11 * ... * 2 * 1 Or, 13!. [URL='https://www.mathcelebrity.com/factorial.php?num=13!&pl=Calculate+factorial']Typing 13! into our search engine[/URL], we get [B]6,227,020,800[/B] ways the ducklings can line up behind the mother duck.

A mother gives birth to a 10 pound baby. Every 2 months, the baby gains 5 pounds. If x is the age o
A mother gives birth to a 10 pound baby. Every 2 months, the baby gains 5 pounds. If x is the age of the baby in months, then y is the weight of the baby in pounds. Find an equation of a line in the form y = mx + b that describes the baby's weight. If the baby gains 5 pounds every 2 months, then they gain 5/2 = 2.5 pounds per month. Let x be the number of months old for the baby, we have: The baby starts at 10 pounds. And every month (x), the baby's weight increases 2.5 pounds. Our equation is: [B]y = 2.5x + 10[/B]

A mother gives birth to a 6 pound baby. Every 4 months, the baby gains 4 pounds. If x is the age of
A mother gives birth to a 6 pound baby. Every 4 months, the baby gains 4 pounds. If x is the age of the baby in months, then y is the weight of the baby in pounds. Find an equation of a line in the form y = mx b that describes the baby's weight. The baby gains 4 pounds every month, where x is the number of months since birth. The baby boy starts life (time 0) at 6 pounds. So we have [B]y = 4x + 6[/B]

A mother gives birth to a 7 pound baby. Every 3 months, the baby gains 2 pounds. If x is the age of
A mother gives birth to a 7 pound baby. Every 3 months, the baby gains 2 pounds. If x is the age of the baby in months, then y is the weight of the baby in pounds. Find an equation of a line in the form y = mx + b that describes the baby's weight. Every month, the baby gains 2/3 of a pound. So we have: [B]y = 2/3x + 7 [/B] The baby starts off with 7 pounds. So we add 7 pounds + 2/3 times the number of months passed since birth.

A motorboat travels 408 kilometers in 8 hours going upstream and 546 kilometers in 6 hours going dow
A motorboat travels 408 kilometers in 8 hours going upstream and 546 kilometers in 6 hours going downstream. What is the rate of the boat in still water and what is the rate of the current? [U]Assumptions:[/U] [LIST] [*]B = the speed of the boat in still water. [*]S = the speed of the stream [/LIST] Relative to the bank, the speeds are: [LIST] [*]Upstream is B - S. [*]Downstream is B + S. [/LIST] [U]Use the Distance equation: Rate * Time = Distance[/U] [LIST] [*]Upstream: (B-S)6 = 258 [*]Downstream: (B+S)6 = 330 [/LIST] Simplify first by dividing each equation by 6: [LIST] [*]B - S = 43 [*]B + S = 55 [/LIST] Solve this system of equations by elimination. Add the two equations together: (B + B) + (S - S) = 43 + 55 Cancelling the S's, we get: 2B = 98 Divide each side by 2: [B]B = 49 mi/hr[/B] Substitute this into either equation and solve for S. B + S = 55 49 + S = 55 To solve this, we [URL='https://www.mathcelebrity.com/1unk.php?num=49%2Bs%3D55&pl=Solve']type it in our search engine[/URL] and we get: S = [B]6 mi/hr[/B]

A motorist pays $4.75 per day in tolls to travel to work. He also has the option to buy a monthly pa
A motorist pays $4.75 per day in tolls to travel to work. He also has the option to buy a monthly pass for $80. How many days must he work (i.e. pass through the toll) in order to break even? Let the number of days be d. Break even means both costs are equal. We want to find when: 4.75d = 80 To solve for d, we [URL='https://www.mathcelebrity.com/1unk.php?num=4.75d%3D80&pl=Solve']type this equation into our search engine[/URL] and we get: d = 16.84 days We round up to an even [B]17 days[/B].

A mountain with a base 6684 feet below sea level rises 22304 feet. What is the elevation above sea l
A mountain with a base 6684 feet below sea level rises 22304 feet. What is the elevation above sea level of its peak? Below sea level is written as a negative, so we have: -6684 + 22304 = [B]15,620 feet above sea level[/B]

A movie started at 11:28 am and it ended at 2:49 pm. How long was the movie?
A movie started at 11:28 am and it ended at 2:49 pm. How long was the movie? Using our [URL='http://www.mathcelebrity.com/elaptime.php?num1=11%3A28&check1=1&num2=2%3A49&check2=2&pl=Calculate+Elapsed+Time']elapsed time calculator[/URL], we have [B]3 hours and 21 minutes[/B].

A movie theater charges $7 for adults and $3 for seniors on a particular day when 324 people paid an
A movie theater charges $7 for adults and $3 for seniors on a particular day when 324 people paid an admission the total receipts were 1228 how many were seniors and how many were adults? Let the number of adult tickets be a. Let the number of senior tickets be s. We're given two equations: [LIST=1] [*]a + s = 324 [*]7a + 3s = 1228 [/LIST] We have a set of simultaneous equations we can solve using 3 methods: [LIST] [*][URL='https://www.mathcelebrity.com/simultaneous-equations.php?term1=a+%2B+s+%3D+324&term2=7a+%2B+3s+%3D+1228&pl=Substitution']Substitution Method[/URL] [*][URL='https://www.mathcelebrity.com/simultaneous-equations.php?term1=a+%2B+s+%3D+324&term2=7a+%2B+3s+%3D+1228&pl=Elimination']Elimination Method[/URL] [*][URL='https://www.mathcelebrity.com/simultaneous-equations.php?term1=a+%2B+s+%3D+324&term2=7a+%2B+3s+%3D+1228&pl=Cramers+Method']Cramer's Rule[/URL] [/LIST] No matter what method we choose, we get: [LIST] [*][B]a = 64[/B] [*][B]s = 260[/B] [/LIST]

A movie theater charges 7.00 for adults and 2.00 for seniors citizens. On a day when 304 people paid
A movie theater charges 7.00 for adults and 2.00 for seniors citizens. On a day when 304 people paid for admission, the total receipt were 1118. How many who paid were adults ? How many were senior citizens? Let a be the number of adult tickets. Let s be the number of senior citizen tickets. We're given two equations: [LIST=1] [*]a + s = 304 [*]7a + 2s = 1118 [/LIST] We can solve this system of equations three ways: [LIST] [*][URL='https://www.mathcelebrity.com/simultaneous-equations.php?term1=a+%2B+s+%3D+304&term2=7a+%2B+2s+%3D+1118&pl=Substitution']Substitution Method[/URL] [*][URL='https://www.mathcelebrity.com/simultaneous-equations.php?term1=a+%2B+s+%3D+304&term2=7a+%2B+2s+%3D+1118&pl=Elimination']Elimination Method[/URL] [*][URL='https://www.mathcelebrity.com/simultaneous-equations.php?term1=a+%2B+s+%3D+304&term2=7a+%2B+2s+%3D+1118&pl=Cramers+Method']Cramer's Method[/URL] [/LIST] No matter which way we choose, we end up with the same answer: [LIST] [*]a = [B]102[/B] [*]s = [B]202[/B] [/LIST]

A movie theater has a seating capacity of 143. The theater charges $5.00 for children, $7.00 for stu
A movie theater has a seating capacity of 143. The theater charges $5.00 for children, $7.00 for students, and $12.00 of adults. There are half as many adults as there are children. If the total ticket sales was $ 1030, How many children, students, and adults attended? Let c be the number of children's tickets, s be the number of student's tickets, and a be the number of adult's tickets. We have 3 equations: [LIST=1] [*]a + c + s = 143 [*]a = 0.5c [*]12a + 5c + 7s =1030 [/LIST] Substitute (2) into (1) 0.5c + c + s = 143 1.5c + s = 143 Subtract 1.5c from each side 4. s = 143 - 1.5c Now, take (4) and (2), and plug it into (3) 12(0.5c) + 5c + 7(143 - 1.5c) = 1030 6c + 5c + 1001 - 10.5c = 1030 Combine like terms: 0.5c + 1001 = 1030 Use our [URL='http://www.mathcelebrity.com/1unk.php?num=0.5c%2B1001%3D1030&pl=Solve']equation calculator[/URL] to get [B]c = 58[/B]. Plug this back into (2) a = 0.5(58) [B]a = 29 [/B] Now take the a and c values, and plug it into (1) 29 + 58 + s = 143 s + 87 = 143 Using our [URL='http://www.mathcelebrity.com/1unk.php?num=s%2B87%3D143&pl=Solve']equation calculator[/URL] again, we get [B]s = 56[/B]. To summarize, we have: [LIST] [*]29 adults [*]58 children [*]56 students [/LIST]

A mug has 3 inch diameter and is 3.5 inches tall how much water can it hold
A mug has 3 inch diameter and is 3.5 inches tall how much water can it hold A mug is a cylinder. If the diameter is 3, then the radius is 3/2 = 1.5. Using our cylinder volume calculator, we get: [B]V = 7.875pi or 24.74 cubic inches[/B]

A music app charges $2 to download the app plus $1.29 per song download. Write and solve a linear eq
A music app charges $2 to download the app plus $1.29 per song download. Write and solve a linear equation to find the total cost to download 30 songs Set up the cost function C(s) where s is the number of songs: C(s) = cost per song * s + download fee Plugging in our numbers for s = 30 and a download fee of $2 and s = 1.29, we have: C(30) = 1.29(30) + 2 C(30) = 38.7 + 2 C(30) = [B]40.7[/B]

A music app charges $2 to download the app plus $1.29 per song downloaded
A music app charges $2 to download the app plus $1.29 per song downloaded Let d be the number of downloads. The cost function C(d) is: C(d) = cost per download * d + download fee [B]C(d) = 1.29d + 2[/B]

A music app charges $2 to download the app plus $1.29 per song downloaded. Write and solve a linear
A music app charges $2 to download the app plus $1.29 per song downloaded. Write and solve a linear equation to find the total cost to download 30 songs. Let the number of songs be s. And the cost function be C(s). We have: C(s) = Price per song downloaded * s + app download charge C(s) = 1.29s + 2 The problem asks for C(30): C(3) = 1.29(30) + 2 C(3) = 38.7 +2 C(3) = $[B]40.7[/B]

a music app charges $5 to download the app plus $1.25 per song downloaded. write linear equation to
a music app charges $5 to download the app plus $1.25 per song downloaded. write linear equation to calculate the cost for x number of songs With x songs, our Cost equation C(x) is: C(x) = cost per download * x downloads + app download fee [B]C(x) = 1.25x + 5[/B]

A music app charges 2$ to download the app plus 1.29$ per song download. Write and solve linear equa
A music app charges 2$ to download the app plus 1.29$ per song download. Write and solve linear equation and a linear equation to find the total cost to download 30 songs Set up the equation C(d) where d is the number of downloads: C(d) = cost per download * d + download fee Plugging in our numbers, we get: C(d) = 1.29d + 2 The problem asks for C(30): C(30) = 1.29(30) + 2 C(30) = 38.7 + 2 C(30) = [B]40.70[/B]

A music camp with 50 students decided to break the students into barbershop quartets to see which co
A music camp with 50 students decided to break the students into barbershop quartets to see which combination of four students sounded the best. How many different barbershop quartets can be made with 50 students so that each possible combinations of four is tried? We want 50 combinations of 4. [URL='https://www.mathcelebrity.com/permutation.php?num=50&den=4&pl=Combinations']50C4 [/URL]= 230,300

A music teacher budgets $150 for new books. The minimum cost of a new book is $12. How many books ca
A music teacher budgets $150 for new books. The minimum cost of a new book is $12. How many books can she buy? 150 / 12 per book = 12.5 So the teacher can buy [B]12 full books[/B].

A national political party has a budget of $30,000,000 to spend on the inauguration of the new presi
A national political party has a budget of $30,000,000 to spend on the inauguration of the new president. 16% of the costs will be paid to personnel, 12% of the costs will go toward food, and 10% will go to decorations. How much money will go for personnel, food, and decorations? [LIST] [*]Personnel Costs = 0.16 * 30,000,000 = $4,800,000 [*]Food Costs = 0.12 * 30,000,000 = $3,600,000 [*]Decoration Costs = 0.10 * 30,000,000 = $3,000,000 [/LIST]

A necklace chain costs $15. Beads cost $2.75 each. You spend a total of $28.75 on a necklace and bea
A necklace chain costs $15. Beads cost $2.75 each. You spend a total of $28.75 on a necklace and beads before tax. How many beads did you buy in addition to the necklace? [U]Calculate the amount left to spend on beads:[/U] Bead Spend = Total Spend - Necklace Cost Bead Spend = $28.75 - $15 Bead Spend = $13.75 [U]Calculate the number of beads you bought:[/U] Beads Bought = Bead Spend / Cost Per Bead Beads Bought = $13.75 / $2.75 Beads Bought = [B]$5[/B]

A new car worth $24,000 is depreciating in value by $3,000 per year , how many years till the cars v
A new car worth $24,000 is depreciating in value by $3,000 per year , how many years till the cars value will be $9,000 We have a flat rate depreciation each year. Set up the function D(t) where t is the number of years of depreciation: D(t) = 24000 - 3000t The problem asks for the time (t) when D(t) = 9000. So we set D(t) = 9000 24000 - 3000 t = 9000 To solve for t, [URL='https://www.mathcelebrity.com/1unk.php?num=24000-3000t%3D9000&pl=Solve']we plug this function into our search engine[/URL] and we get: t = [B]5[/B]

A new car worth $30,000 is depreciating in value by $3,000 per year. After how many years will the c
A new car worth $30,000 is depreciating in value by $3,000 per year. After how many years will the cars value be $9,000 Step 1, the question asks for Book Value. Let y be the number of years since purchase. We setup an equation B(y) which is the Book Value at time y. B(y) = Sale Price - Depreciation Amount * y We're given Sale price = $30,000, depreciation amount = 3,000, and B(y) = 9000 30000 - 3000y = 9000 To solve for y, we [URL='https://www.mathcelebrity.com/1unk.php?num=30000-3000y%3D9000&pl=Solve']type this in our math engine[/URL] and we get: y = [B]7 [/B] To check our work, substitute y = 7 into B(y) B(7) = 30000 - 3000(7) B(7) = 30000 - 21000 B(7) = 9000 [MEDIA=youtube]oCpBBS7fRYs[/MEDIA]

A new company is projecting its profits over a number of weeks. They predict that their profits each
A new company is projecting its profits over a number of weeks. They predict that their profits each week can be modeled by a geometric sequence. Three weeks after they started, the company's projected profit is $10,985.00 Four weeks after they started, the company's projected profit is $14,280.50 Let Pn be the projected profit, in dollars, n weeks after the company started tracking their profits. a. What is the common ratio of the sequence? b. Calculate the initial value c. Construct a recurrence relation that can be used to model the value of Pn a. 14,280.50/10,985.00 = [B]1.3[/B] b. 3 weeks ago, the Initial value is 10,985/1.3^3 = [B]$5,000 c. Pn = 5000 * 1.3^n[/B]

A new company president is said to have caused the company "to do a 180." Before the new president,
A new company president is said to have caused the company "to do a 180." Before the new president, the company was losing money. What is the company most likely doing under the new president? A 180 is a completely different direction. Since 180 degrees means the other way, a half-circle, a switch in direction. This means if the company was losing money, after doing a "180", they're making money.

a new savings account starts at $700 at a rate of 1.2% yearly. how much money will be in the account
a new savings account starts at $700 at a rate of 1.2% yearly. how much money will be in the account after 8 years? Using our [URL='https://www.mathcelebrity.com/compoundint.php?bal=700&nval=8&int=1.2&pl=Annually']balance and interest calculator with annual (yearly) compounding[/URL], we have: [B]770.09[/B]

A non-profit organization is having a couple’s banquet for a fundraiser. The banquet hall will only
A non-profit organization is having a couple’s banquet for a fundraiser. The banquet hall will only hold 250 people. The President, Vice-President, two volunteers, and a guest speaker will be working the event. How many couples will be able to attend the banquet? We subtract the 5 people working the event to get: 250 - 5 = 245 A couple is 2 people, so we have 245/2 = 122.5 We round down to [B]122 couples[/B].

a number added to 5 minus p
a number added to 5 minus p The phrase [I]a number[/I] means an arbitrary variable, let's call it x. We add 5 minus p to this number x: [B]x + 5 - p[/B]

a number added to the product of y and x
a number added to the product of y and x Since we're already using the variables x and y, we choose another arbitrary variable for the phrase [I]a number.[/I] a The product of y and x isL xy Then add a: [B]a + xy[/B]

A number cube is rolled and a coin is tossed. The number cube and the coin are fair. What is the pro
A number cube is rolled and a coin is tossed. The number cube and the coin are fair. What is the probability that the number rolled is greater than 3 and the coin toss is heads? Write your answer as a fraction in simplest form Let's review the vitals of this question: [LIST] [*]The probability of heads on a fair coin is 1/2. [*]On a fair die, greater than 3 means either 4, 5, or 6. Any die roll face is a 1/6 probability. [*]So we have a combination of outcomes below: [/LIST] Outcomes [LIST=1] [*]Heads and 4 [*]Heads and 5 [*]Heads and 6 [/LIST] For each of the outcomes, we assign a probability. Since the coin flip and die roll are independent, we multiply the probabilities: [LIST=1] [*]P(Heads and 4) = 1/2 * 1/6 = 1/12 [*]P(Heads and 5) = 1/2 * 1/6 = 1/12 [*]P(Heads and 6) = 1/2 * 1/6 = 1/12 [/LIST] Since we want any of those events, we add all three probabilities 1/12 + 1/12 + 1/12 = 3/12 This fraction is not simplified. S[URL='https://www.mathcelebrity.com/fraction.php?frac1=3%2F12&frac2=3%2F8&pl=Simplify']o we type this fraction into our search engine, and choose Simplify[/URL]. We get a probability of [B]1/4[/B]. By the way, if you need a decimal answer or percentage answer instead of a fraction, we type in the following phrase into our search engine: [URL='https://www.mathcelebrity.com/perc.php?num=1&den=4&pcheck=1&num1=+16&pct1=+80&pct2=+35&den1=+90&pct=+82&decimal=+65.236&astart=+12&aend=+20&wp1=20&wp2=30&pl=Calculate']1/4 to decimal[/URL] Alternative Answers: [LIST] [*]For a decimal, we get [B]0.25[/B] [*]For a percentage, we get [B]25%[/B] [/LIST]

a number increased by 6
The phrase [I]a number[/I] means an arbitrary variable. We can pick any letter a-z except for i and e. Let's choose x. The phrase [I]increased by[/I] means we add 6 to x [B]x +6[/B]

a number increased by 8 and then tripled
a number increased by 8 and then tripled The phrase [I]a number[/I] means an arbitrary variable, let's call it x: x Increased by 8 means we add 8 to x: x + 8 Then tripled means we multiply the expression x + 8 by 3: [B]3(x + 8)[/B]

a number is twice another number
a number is twice another number The phrase [I]a number[/I] means an arbitrary variable, let's call it x The phrase [I]another number [/I]means another arbitrary variable, let's call it y Twice means we multiply y by 2: 2y The phrase [I]is [/I]means an equation, so we set x equal to 2y: [B]x = 2y[/B]

A number K is doubled and then increased by 3
A number K is doubled and then increased by 3 K is doubled means we multiply K by 2: 2K Increased by 3 means we add: [B]2K + 3[/B]

A number m is no less than -8 and fewer than 9.
A number m is no less than -8 and fewer than 9. No less than means greater than or equal to: m >= -8 Fewer than 9 means less than 9: m < 9 Combine these two inequalities to get [B]-8 <= m < 9[/B]

A number multiplied by 6 and divided by 5 give four more than a number?
A number multiplied by 6 and divided by 5 give four more than a number? A number is represented by an arbitrary variable, let's call it x. Multiply by 6: 6x Divide by 5 6x/5 The word "gives" means equals, so we set this equal to 4 more than a number, which is x + 4. 6x/5 = x + 4 Now, multiply each side of the equation by 5, to eliminate the fraction on the left hand side: 6x(5)/5 = 5(x + 4) The 5's cancel on the left side, giving us: 6x = 5x + 20 Subtract 5x from each side [B]x = 20[/B] Check our work from our original equation: 6x/5 = x + 4 6(20)/5 ? 20 + 4 120/5 ?24 24 = 24 <-- Yes, we verified our answer

A number n diminished by 8 gives 12
A number n diminished by 8 gives 12 A number n can be written as n: n Diminished by means we subtract, so we subtract 8 from n: n - 8 The word [I]gives[/I] means an equation, so we set n - 8 equal to 12: [B]n - 8 = 12[/B]

A number n is no less than 2 and no more than 49.
A number n is no less than 2 and no more than 49. This is a compound inequality. Let's break it into parts. Step 1: No more than 49 means 49 or less. Or, less than or equal to 49 <= 49 Step 2: no less than 2 means 2 or greater. Or, greater than or equal to 2 >=2 Writing this in terms of the number n, we have: [B]2 <= n <= 49[/B]

a number of bacteria b tripled
a number of bacteria b tripled The word [I]tripled[/I] means we multiply by 3, so we have: [B]3b[/B]

A number of dogs are to equally share a bag of dog food. If there are n dogs in the group and one do
A number of dogs are to equally share a bag of dog food. If there are [I]n[/I] dogs in the group and one dog eats its share, what percent of the bag is left? Fraction of the bag left is: (n - 1)/n Multiply by 100 to get a percentage: [B]100(n - 1)/n[/B]

a number of pennies splits into 4 equal groups
a number of pennies splits into 4 equal groups The phrase [I]a number[/I] means an arbitrary variable, let's call it x. We take x and divide it by 4 to get 4 equal groups: [B]x/4[/B]

A number p subtracted by its double is 10
A number p subtracted by its double is 10 The double of a number means we multiply p by 2: 2p A number p is subtracted by its double p - 2p The phrase [I]is[/I] means equal to, so we set p - 2p equal to 10: [B]p - 2p = 10[/B]

A number t is no less than 30 and fewer than 40.
A number t is no less than 30 and fewer than 40. This is a compound inequality. Take it in 3 parts: Step 1: fewer than 40 means less than (does not include 40) t < 40 Step 2: no less than 30 means greater than or equal to t >= 30 Step 3: Combine these 2 statements into one compound inequality: [B]30 <= t < 40[/B]

A number y increased by itself
A number y increased by itself increased by itself means we add the variable y to itself to get our final algebraic expression of: [B]y + y [/B] [I]If[/I] the problem asks you to simplify, we group like terms and get: [B]2y[/B]

A numerical pass code is required to open a car door. The pass code is five digits long and uses the
A numerical pass code is required to open a car door. The pass code is five digits long and uses the digits 0-9. Numbers may be repeated in the pass code. How many different pass codes exist? 0-9 is 10 digits. Since digits can repeat, we use the fundamental rule of counting to get: 10 * 10 * 10 * 10 * 10 = [B]100,000 different pass codes[/B]

A pack of 12 tortillas cost $3.24. What is the price per tortilla?
A pack of 12 tortillas cost $3.24. What is the price per tortilla? Price per tortilla = Total Cost / Total Tortillas Price per tortilla = $3.24/12 Price per tortilla = [B]$0.27[/B]

A pack of 36 black sharp tip markers costs $34.49. What is the price of one marker?
A pack of 36 black sharp tip markers costs $34.49. What is the price of one marker? Set up unit cost: 34.49/36 = [B]$0.96 per marker[/B]

A package of cookies cost $4.00 it has 20 cookies how much does each cookie cost?
A package of cookies cost $4.00 it has 20 cookies how much does each cookie cost? We want the cost per cookie: $4.00/20 cookies = [B]$0.20 or 20 cents per cookie[/B]

a package of soccer accessories costs $25 for cleats, $14 for shin guards , and $12 for a ball. Writ
a package of soccer accessories costs $25 for cleats, $14 for shin guards , and $12 for a ball. Write two equivalent expressions for the total cost of 9 accessory package. Then find the cost. Let c be the number of cleats, s be the number of shin guards, and b be the number of balls. We have the following cost function for 9 accessory packages: [B]9(25c + 14s + 12b)[/B] But if we multiply through, we get an equivalent expression: [B]225c + 126s + 108b[/B]

A package that is heavier than 11 lbs and 8 oz will have a label that says HEAVY on it. Gloria packe
A package that is heavier than 11 lbs and 8 oz will have a label that says HEAVY on it. Gloria packed 6 flowerpots to send to her customers. Each of the flowerpots weighs 1 lb and 12 oz. The packing material weighs 5 oz. Will her package be labeled as HEAVY? Calculate weight of flowerpots: Flowerpot weight = Weight per flowerpot * number of flowerpots Flowerpot weight = 1 lb 12 oz * 6 Flowerpot weight = 6 lb and 72 oz Since 72oz = 72/16 = 4 lbs and 8 oz, we have: Flowerpot weight = 6 lb 8 oz + 4 lbs and 8 oz = 12 lb 16 oz Since 16oz = 1 lb, we have: 13lb Add in the 5 oz of packing material, we have: 13lb 5 oz Since this is greater than 11lb 8oz, the package [B]will be labeled as HEAVY[/B]

A packing machine can package 236 first aid kit each hour. At this rate, find the number of first ai
A packing machine can package 236 first aid kit each hour. At this rate, find the number of first aid kit package in 24 hours Total First Aid Kits = Kits Per Hour * Number of Hours Total First Aid Kits = 236 * 24 Total First Aid Kits = [B]5,664[/B]

a painter is painting a circular sculpture. The sculpture has a radius of 5 meters. How much paint s
a painter is painting a circular sculpture. The sculpture has a radius of 5 meters. How much paint should she use to paint the sculpture Area of a circle (A) is: A = ?r² Substituting r = 5 into this formula, we get: A = ? * 5² A = [B]25?[/B]

A painter rented a wallpaper steamer at 9 a.m. and returned it at 4 p.m. He paid a total of $28.84.
A painter rented a wallpaper steamer at 9 a.m. and returned it at 4 p.m. He paid a total of $28.84. What was the rental cost per hour? 9am to 4pm is 7 hours. Cost per hour = Total Cost / Hours Cost per hour = 28.84 / 7 Cost per hour = [B]$4.12[/B]

A pair of dice are rolled. Find the probability for P(Not 2 or Not 12).
A pair of dice are rolled. Find the probability for P(Not 2 or Not 12). P(Not 2 or Not 12) = 1 - P(2) - P(12) P(Not 2 or Not 12) = 1 - [URL='https://www.mathcelebrity.com/2dice.php?gl=1&pl=2&opdice=1&rolist=+2%2C3%2C9%2C10&dby=2%2C3%2C5&ndby=4%2C5&montect=+100']1/36[/URL] - [URL='https://www.mathcelebrity.com/2dice.php?gl=1&pl=12&opdice=1&rolist=+2%2C3%2C9%2C10&dby=2%2C3%2C5&ndby=4%2C5&montect=+100']1/36[/URL] P(Not 2 or Not 12) = 34/36 [URL='https://www.mathcelebrity.com/fraction.php?frac1=34%2F36&frac2=3%2F8&pl=Simplify']P(Not 2 or Not 12)[/URL] = [B]17/18[/B]

A pair of dice is cast. what is the probablitly that the sum is less than 5
A pair of dice is cast. what is the probablitly that the sum is less than 5? Using our [URL='http://www.mathcelebrity.com/2dice.php?gl=4&pl=5&opdice=1&rolist=+2%2C3%2C9%2C10&dby=2%2C3%2C5&ndby=4%2C5&montect=+100']two dice calculator[/URL], we get 1/6 or 16.67%

A pair of dice is rolled. Find the probability of rolling a sum of not less than 5
A pair of dice is rolled. Find the probability of rolling a sum of not less than 5. The phrase [I]not less than[/I] also means greater than or equal to. So we [URL='https://www.mathcelebrity.com/2dice.php?gl=3&pl=5&opdice=1&rolist=+&dby=&ndby=&montect=+']use our 2 dice calculator for a sum roll of 5 or greater[/URL] and we get: [B]5/6[/B]

A pair of jeans are priced at $129.99 there is a discount of 20% and sales tax of 8% what is the fin
A pair of jeans are priced at $129.99 there is a discount of 20% and sales tax of 8% what is the final cost [U]Calculate discounted price:[/U] Discounted price = Full price * (100% - discount percent) Discounted price = 129.99 * (100% - 20%) Discounted price = 129.99 * 80% Since 80% = 0.8, we have: Discounted price = 129.99 * 0.8 Discounted price = 103.99 [U]Calculate after tax cost:[/U] Tax Rate = Tax percent/100 Tax Rate = 8/100 Tax Rate = 0.08 After Tax cost = Discounted price * (1 + Tax rate) After Tax cost = 103.99 * (1 + 0.08) After Tax cost = 103.99 * 1.08 After Tax cost = [B]112.31[/B]

A pair of numbers has an HCF (Highest Common Factor) of 3, and an LCM (Lowest Common Multiple) of
A pair of numbers has an HCF (Highest Common Factor) of 3, and an LCM (Lowest Common Multiple) of 45 . If one of the numbers in the pair is 15 , what is the other number? [LIST=1] [*]Prime Factorization for 15 is 3 * 5 [*]Prime Factorization for 9 is 3 * 3 [*]LCM of (9, 15) = 35 [/LIST] [URL='https://www.mathcelebrity.com/gcflcm.php?num1=9&num2=15&num3=&pl=GCF+and+LCM']Check out this link here to see the details[/URL]

A pair of shoes cost $250. The price was decreased by 20%. A week later shoes were mark down again b
A pair of shoes cost $250. The price was decreased by 20%. A week later shoes were mark down again by 25%. What is the final price of the shoes? 20% is 0.2. 25% is 0.25. A decrease is a reduction, so we have: Final Price = 250 * (1 - 0.2) * (1 - 0.25) Final Price = 250 * 0.8 * 0.75 Final Price = [B]150[/B]

A pair of standard dice is rolled, how many possible outcomes are there
A pair of standard dice is rolled, how many possible outcomes are there? We want the number of outcomes in the sample space. The first die has 6 possibilities 1-6. The second die has 6 possibilities 1-6. Our sample space count is 6 x 6 = [B]36 different outcomes [/B] [LIST=1] [*](1, 1) [*](1, 2) [*](1, 3) [*](1, 4) [*](1, 5) [*](1, 6) [*](2, 1) [*](2, 2) [*](2, 3) [*](2, 4) [*](2, 5) [*](2, 6) [*](3, 1) [*](3, 2) [*](3, 3) [*](3, 4) [*](3, 5) [*](3, 6) [*](4, 1) [*](4, 2) [*](4, 3) [*](4, 4) [*](4, 5) [*](4, 6) [*](5, 1) [*](5, 2) [*](5, 3) [*](5, 4) [*](5, 5) [*](5, 6) [*](6, 1) [*](6, 2) [*](6, 3) [*](6, 4) [*](6, 5) [*](6, 6) [/LIST]

A Pairs of fair dice is tossed. What is the probability of not getting a sum 7 or 8?
A Pairs of fair dice is tossed. What is the probability of not getting a sum 7 or 8? Not a 7 or 8 means 2 - 6 or 9 - 12 [URL='https://www.mathcelebrity.com/2dice.php?gl=5&pl=6&opdice=1&rolist=+&dby=&ndby=&montect=+']Using our 2-dice calculator[/URL], P(2 - 6) = 5/12 [URL='https://www.mathcelebrity.com/2dice.php?gl=3&pl=9&opdice=1&rolist=+&dby=&ndby=&montect=+']Using our 2-dice calculator[/URL], P(9 - 12) = 5/18 Since the sum could be either of these, we add probabilities: P(Not a 7 or 8) = P(2 - 6) + P(9 - 12) P(Not a 7 or 8) = 5/12 + 5/18 [URL='https://www.mathcelebrity.com/fraction.php?frac1=5%2F12&frac2=5%2F18&pl=Add']P(Not a 7 or 8) [/URL]= [B]25/36[/B]

a paper boy delivers thirteen paper to an apartment complex. if these deliveries compose one-seventh
a paper boy delivers thirteen paper to an apartment complex. if these deliveries compose one-seventh of his route, how many papers does he deliver Let d be the total number of deliveries the paper boy makes on the route. d We're given, d/7 = 13 d = 13 * 7 d = [B]91 [MEDIA=youtube]HRviz-3fn5c[/MEDIA][/B]

A parabola has a Vertex at (4,-2) and a Focus at (6,-2). Find the equation of the parabola
A parabola has a Vertex at (4,-2) and a Focus at (6,-2). Find the equation of the parabola and the lotus rectum. Equation of a parabola given the vertex and focus is: ([I]x[/I] – [I]h[/I])^2 = 4[I]p[/I]([I]y[/I] – [I]k[/I]) The vertex (h, k) is 4, -2 The distance is p, and since the y coordinates of -2 are equal, the distance is 6 - 4 = 2. So p = 2 Our parabola equation becomes: (x - 4)^2 = 4(2)(y - -2) [B](x - 4)^2 = 8(y + 2)[/B] Latus rectum of a parabola is 4p, where p is the distance between the vertex and the focus LR = 4p LR = 4(2) [B]LR = 8[/B]

A parallelogram has a perimeter of 48 millimeters. Two of the sides are each 20 millimeters long. Wh
A parallelogram has a perimeter of 48 millimeters. Two of the sides are each 20 millimeters long. What is the length of each of the other two sides? 2 sides * 20 mm each is 40 mm subtract this from the perimeter of 48: 48 - 40 = 8 Since the remaining two sides equal each other, their length is: 8/2 = [B]4mm[/B]

A parallelogram has a perimeter of 54 centimeters. Two of the sides are each 17 centimeters long. Wh
A parallelogram has a perimeter of 54 centimeters. Two of the sides are each 17 centimeters long. What is the length of each of the other two sides? A parallelogram is a rectangle bent on it's side. So we have the perimeter formula P below: P = 2l + 2w We're given w = 17 and P = 54. So we plug this into the formula for perimeter: 2l + 2(17) = 54 2l + 34 = 54 Using our [URL='https://www.mathcelebrity.com/1unk.php?num=2l%2B34%3D54&pl=Solve']equation calculator[/URL], we get [B]l = 10[/B].

A park bench is 6 feet long. Convert the length to inches
A park bench is 6 feet long. Convert the length to inches We [URL='https://www.mathcelebrity.com/linearcon.php?quant=6&pl=Calculate&type=foot']type in 6 feet into our search engine[/URL]. We get: 6 feet = [B]72 inches[/B]

A parking garage charges $5 plus $2 per hour. You have $16 to spend for parking. How many hours can
A parking garage charges $5 plus $2 per hour. You have $16 to spend for parking. How many hours can you park? Subtract the flat rate to get the amount you have for hourly parking: 16 - 5 = 11 So we divide 11 dollars to park by 2 dollars per hour to get: 11/2 [B]5.5 hours[/B]

A parking lot has seventy-one parking spaces numbered from 1 to 71. There are no cars in the parking
A parking lot has seventy-one parking spaces numbered from 1 to 71. There are no cars in the parking lot when Jillian pulls in and randomly parks. What is the probability that the number on the parking space where she parks is greater than or equal to 31? Greater than or equal to means including 31 all the way through 71 31-71 is 40 spaces P(s>=31) = [B]40/71[/B]

A parking lot has sixty-eight parking spaces numbered from 1 to 68. There are no cars in the parking
A parking lot has sixty-eight parking spaces numbered from 1 to 68. There are no cars in the parking lot when Jillian pulls in and randomly parks. What is the probability that the number on the parking space where she parks is greater than or equal to 21? We want P(X>=21). This is also found by taking 1 - P(X <= 20). P(X<=20) = 20/68. Reduced using a [URL='http://www.mathcelebrity.com/gcflcm.php?num1=20&num2=68&num3=&pl=GCF']GCF of 4[/URL], we get 5/17. P(X >=21) = 1 - 5/17 = [B]12/17[/B]

A parking meter contains 27.05 in quarters and dimes. All together there are 146 coins. How many of
A parking meter contains 27.05 in quarters and dimes. All together there are 146 coins. How many of each coin are there? Let d = the number of dimes and q = the number of quarters. We have two equations: (1) d + q = 146 (2) 0.1d + 0.25q = 27.05 Rearrange (1) into (3) solving for d (3) d = 146 - q Substitute (3) into (2) 0.1(146 - q) + 0.25q = 27.05 14.6 - 0.1q + 0.25q = 27.05 Combine q's 0.15q + 14.6 = 27.05 Subtract 14.6 from each side 0.15q = 12.45 Divide each side by 0.15 [B]q = 83[/B] Plugging that into (3), we have: d = 146 - 83 [B]d = 63[/B]

A party rental company has chairs and tables for rent. The total cost to rent 5 chairs and 3 tables
A party rental company has chairs and tables for rent. The total cost to rent 5 chairs and 3 tables is $37. The total cost to rent 2 chairs and 6 tables is $64. What is the cost to rent each chair and each table? Let c be the cost of renting one chair and t be the cost of renting one table. We're given two equations: [LIST=1] [*]5c + 3t = 37 [*]2c + 6t =64 [/LIST] We have a system of equations. Using our system of equations calculator, we can solve this problem any of 3 ways below: [LIST] [*][URL='https://www.mathcelebrity.com/simultaneous-equations.php?term1=5c+%2B+3t+%3D+37&term2=2c+%2B+6t+%3D+64&pl=Elimination']Elimination Method[/URL] [*][URL='https://www.mathcelebrity.com/simultaneous-equations.php?term1=5c+%2B+3t+%3D+37&term2=2c+%2B+6t+%3D+64&pl=Substitution']Substitution Method[/URL] [*][URL='https://www.mathcelebrity.com/simultaneous-equations.php?term1=5c+%2B+3t+%3D+37&term2=2c+%2B+6t+%3D+64&pl=Cramers+Method']Cramer's Rule[/URL] [/LIST] All 3 methods give the same answer: [LIST] [*][B]Chairs (c) cost $1.25[/B] [*][B]Tables (t) cost $10.25[/B] [/LIST]

A passenger train left station A at 6:00 P.M. Moving with the average speed 45 mph, it arrived at st
A passenger train left station A at 6:00 P.M. Moving with the average speed 45 mph, it arrived at station B at 10:00 p.m. A transit train left from station A 1 hour later than the passenger train, but it arrived at the station B at the same time with the passenger train. What was the average speed of the transit train? [U]Passenger Train[/U] [LIST] [*]45 miles per hour and it got there in 4 hours. [/LIST] Using our formula D = rt where: [LIST] [*]D = Distance [*]r = rate [*]t = time [/LIST] [LIST] [*]D = rt [*]D = 45(4) [*]D = 180 miles from Station A to Station B [/LIST] Transit Train [LIST] [*]It has to go the same distance, 180 miles, so D = 180 [*]It made it there in 3 hours. This is r [*]We want to solve for t [/LIST] D = rt 180 = 3r Divide each side by 3 [B]r = 60 miles per hour[/B]

A patient’s temperature was 103°. The temperature then fell by 4° and later rose by 2°. What was the
A patient’s temperature was 103°. The temperature then fell by 4° and later rose by 2°. What was the patient’s final temperature Start with 103 Fell by 4 means we subtract 4: 103 - 4 = 99 Rose by 2 means we add 2L 99 + 2 = [B]101[/B]

A pawn broker buys a tv and a computer for $600. He sells the computer at a markup of 30% and the tv
A pawn broker buys a tv and a computer for $600. He sells the computer at a markup of 30% and the tv at a markup of 20%. If he makes a profit of $165 on the sale of the two items, what did he pay for the computer? Let c be the price of the computer and t be the price of the tv. WE have: [LIST=1] [*]c + t = 600 [*]c(1.3) + t(1.2) = 765 <-- (600 + 165 profit) [/LIST] Using our [URL='http://www.mathcelebrity.com/simultaneous-equations.php?term1=c+%2B+t+%3D+600&term2=1.3c+%2B+1.2t+%3D+765&pl=Cramers+Method']simultaneous equation calculator[/URL], we get: [B]c = 450[/B] t = 150

A peanut vendor has initial start up costs of $7600 and variable costs of $0.70 per bag of peanuts.
A peanut vendor has initial start up costs of $7600 and variable costs of $0.70 per bag of peanuts. What is the cost function? We set up the cost function C(b) where b is the number of bags: C(b) = Cost per bag * b + Start up costs Plugging in our numbers, we get: [B]C(b) = 0.70b + 7600[/B]

A penny has a diameter of 19 millimeters. What is the radius of the penny.
A penny has a diameter of 19 millimeters. What is the radius of the penny. D = 2r To solve for r, we divide each side by 2: r = D/2 Plugging in D = 19, we get: r = [B]19/2 or 9.5[/B]

A person has $13,000 invested in stock A and stock B. Stock A currently sells for $20 a share and
A person has $13,000 invested in stock A and stock B. Stock A currently sells for $20 a share and stock B sells for $90 a share. If stock B triples in value and stock A goes up 50%, his stock will be worth $33,000. How many shares of each stock does he own? Set up the given equations, where A is the number of shares for Stock A, and B is the number of shares for Stock B [LIST=1] [*]90A + 20B = 13000 [*]3(90A) + 1.5(20B) = 33000 <-- [I]Triple means multiply by 3, and 50% gain means multiply by 1.5[/I] [/LIST] Rewrite (2) by multiplying through: 270A + 30B = 33000 Using our simultaneous equations calculator, we get [B]A = 100 and B = 200[/B]. Click the links below to solve using each method: [LIST] [*][URL='https://www.mathcelebrity.com/simultaneous-equations.php?term1=90A+%2B+20B+%3D+13000&term2=270A+%2B+30B+%3D+33000&pl=Substitution']Substitution Method[/URL] [*][URL='https://www.mathcelebrity.com/simultaneous-equations.php?term1=90A+%2B+20B+%3D+13000&term2=270A+%2B+30B+%3D+33000&pl=Elimination']Elimination Method[/URL] [*][URL='https://www.mathcelebrity.com/simultaneous-equations.php?term1=90A+%2B+20B+%3D+13000&term2=270A+%2B+30B+%3D+33000&pl=Cramers+Method']Cramers Method[/URL] [/LIST] Check our work using equation (1) 90(100) + 20(200) ? 13,000 9000 + 4000 ? 13,000 13000 = 13000

A person invested 30,000 in stocks and bonds. Her investment in bonds is 2000 more than 1-third her
A person invested 30,000 in stocks and bonds. Her investment in bonds is 2000 more than 1-third her investments in stocks. How much did she invest in stocks? How much did she invest in bonds? Let the stock investment be s, and the bond investment be b. We're given: [LIST=1] [*]b + s = 30000 [*]b = 1/3s + 2000 [/LIST] Plug in (2) to (1): 1/3s + 2000 + s = 30000 Group like terms: (1/3 + 1)s + 2000 = 30000 Since 1 = 3/3, we have: 4/3s + 2000 = 30000 Subtract 2000 from each side: 4/3s + 2000 - 2000 = 30000 - 2000 Cancel the 2000's on the left side, we get: 4/3s = 28000 [URL='https://www.mathcelebrity.com/1unk.php?num=4%2F3s%3D28000&pl=Solve']Typing this equation into our calculator[/URL], we get: s = [B]21,000[/B]

A person invests $500 in an account that earns a nominal yearly rate of 4%. How much will this inves
A person invests $500 in an account that earns a nominal yearly rate of 4%. How much will this investment be worth in 10 years? If the interest was applied four times per year (known as quarterly compounding), calculate how much the investment would be worth after 10 years. Using our [URL='https://www.mathcelebrity.com/compoundint.php?bal=500&nval=10&int=4&pl=Annually']compound interest calculator[/URL], $500 @ 4% for 10 years is: $[B]740.12 [/B] Using [URL='https://www.mathcelebrity.com/compoundint.php?bal=500&nval=40&int=4&pl=Quarterly']quarterly compounding in our compound interest calculator[/URL], we have 10 years * 4 quarters per year = 40 periods, so we have: [B]$744.43[/B]

A person invests $9400 in an account at 5% interest compound annually. When will the value of the in
A person invests $9400 in an account at 5% interest compound annually. When will the value of the investment be $12,800. Let's take it one year at a time: Year 1: 9,400(1.05) = 9,870 Year 2: 9,870(1.05) = 10,363.50 Year 3: 10,363.50(1.05) = 10,881.68 Year 4: 10.881.68(1.05) = 11,425.76 Year 5: 11,425.76(1.05) = 11,997.05 Year 6: 11,997.05(1.05) = 12.596.90 Year 7: 12,596.90(1.05) = 13,226.74 So it take [B][U]7 years[/U][/B] to cross the $12,800 amount.

A person is earning 600 per day to do a certain job. Express the total salary as a function of the n
A person is earning 600 per day to do a certain job. Express the total salary as a function of the number of days that the person works. Set up the salary function S(d) where d is the number of days that the person works: S(d) = Daily Rate * d [B]S(d) = 600d[/B]

A person paid $60 for a vase at an estate auction. She resold it to an antiques dealer for $50. What
A person paid $60 for a vase at an estate auction. She resold it to an antiques dealer for $50. What was her profit or loss She lost, since the sale price was less than the purchase price. The loss is calculated as: 50 - 60 = [B]-$10[/B]

A person places $230 in an investment account earning an annual rate of 6.8%, compounded continuousl
A person places $230 in an investment account earning an annual rate of 6.8%, compounded continuously. Using the formula V = Pe^{rt}V=Pe^rt, where V is the value of the account in t years, P is the principal initially invested, e is the base of a natural logarithm, and r is the rate of interest, determine the amount of money, to the nearest cent, in the account after 20 years Using our [URL='http://www.mathcelebrity.com/simpint.php?av=&p=230&int=6.8&t=20&pl=Continuous+Interest']continuous compounding calculator[/URL], we get: V = [B]896.12[/B]

A person places $96300 in an investment account earning an annual rate of 2.8%, compounded continuou
A person places $96300 in an investment account earning an annual rate of 2.8%, compounded continuously. Using the formula V=PertV = Pe^{rt} V=Pe rt , where V is the value of the account in t years, P is the principal initially invested, e is the base of a natural logarithm, and r is the rate of interest, determine the amount of money, to the nearest cent, in the account after 7 years. Substituting our given numbers in where P = 96,300, r = 0.028, and t = 7, we get: V = 96,300 * e^(0.028 * 7) V = 96,300 * e^0.196 V = 96,300 * 1.21652690533 V = [B]$117,151.54[/B]

A person that runs for 15 minutes burns 180 calories. If someone burns 300 calories, how long did tg
A person that runs for 15 minutes burns 180 calories. If someone burns 300 calories, how long did tgey run for Set up a proportion of minutes to calories where m is the number of minutes per 300 calories: 15/180 = m/300 To solve for m, [URL='https://www.mathcelebrity.com/prop.php?num1=15&num2=m&den1=180&den2=300&propsign=%3D&pl=Calculate+missing+proportion+value']we type this proportion into our search engine[/URL] and we get: m = [B]25[/B]

A person who had $1,000,000 gave $100 to charity. How much must a student who has $100 give to chari
A person who had $1,000,000 gave $100 to charity. How much must a student who has $100 give to charity to give proportionally the same as the millionaire? Set up a proportion of wealth owned to donation where x is the amount the student gives: 1000000/100 = 100/x Using our [URL='https://www.mathcelebrity.com/proportion-calculator.php?num1=1000000&num2=100&den1=100&den2=x&propsign=%3D&pl=Calculate+missing+proportion+value']proportion calculator[/URL], we get: [B]x = 0.01[/B]

A person will devote 31 years to be sleeping and watching tv. The number of years sleeping will exce
A person will devote 31 years to be sleeping and watching tv. The number of years sleeping will exceed the number of years watching tv by 19. How many years will the person spend on each of these activities Let s be sleeping years and t be tv years, we have two equations: [LIST=1] [*]s + t = 31 [*]s = t + 19 [/LIST] Substitute (2) into (1) (t + 19) + t = 31 Combine like terms: 2t + 19 = 31 Using our [URL='http://www.mathcelebrity.com/1unk.php?num=2t%2B19%3D31&pl=Solve']equation solver[/URL], we get [B]t = 6[/B]. Using equation (2), we have s = 6 + 19 s = [B]25[/B]

A person works 46 hours in one week and earns 440 dollars. They get time and a half for over 40 hour
A person works 46 hours in one week and earns 440 dollars. They get time and a half for over 40 hours. What is their hourly salary? Let the hourly rate be r. Since time and a half is 1.5 the hourly rate, We're given: 40r + 6(1.5r) = 440 40r + 9r = 440 to solve this equation for r, we type it in our search engine and we get: r = [B]$8.98[/B]

A pet store has 10 rabbits. What’s the probability that they have at least 1 female rabbit.
A pet store has 10 rabbits. What’s the probability that they have at least 1 female rabbit. At least 1 female rabbit means we [U]must[/U] have a female rabbit First, we calculate the probability of 0 females A rabbit can be either male or female with equal probabilities of 1/2 or 0.5. Since each birth is independent, we can multiply to get the probability of all males: P(MMMMMMMMMM) = 1/2 * 1/2 * 1/2 * 1/2 * 1/2 * 1/2 * 1/2 * 1/2 * 1/2 * 1/2 P(MMMMMMMMMM) = 1/1024 Then, we subtract this probability from 1 to get the probability of [B]at least[/B] one female: P(At least one F) = 1 - 1/1024 Since 1 = 1024/1024, we have: P(At least one F) = (1024 - 1)/1024 P(At least one F) = [B]1023/1024[/B]

A pet supply chain called pet city has 15 hamsters and 12 gerbils for sale at its seaside location.
A pet supply chain called pet city has 15 hamsters and 12 gerbils for sale at its seaside location. At its livingston location there are 19 hamsters and 10 gerbils. Which location has a lower ratio of hamsters to gerbils? Seaside ratio 15/12 = 1.25 Livingston ratio 19/10 = 1.9 Since 1.25 < 1.9, Seaside has the lower ratio of hamsters to gerbils

A Petri dish contains 2000. The number of bacteria triples every 6 hours. How many bacteria will exi
A Petri dish contains 2000. The number of bacteria triples every 6 hours. How many bacteria will exist after 3 days? Determine the amount of tripling periods: [LIST] [*]There are 24 hours in a day. [*]24 hours in a day * 3 days = 72 hours [*]72 hours / 6 hours tripling period = 12 tripling periods [/LIST] Our bacteria population function B(t) where t is the amount of tripling periods. Tripling means we multiply by 3, so we have: B(t) = 2000 * 3^t with t = 12 tripling periods, we have: B(12) = 2000 * 3^12 B(12) = 2000 * 531441 B(12) = [B]1,062,882,000[/B]

A phone company charges a $30 usage fee $15 per 1GB of data. Write an expression that describes the
A phone company charges a $30 usage fee $15 per 1GB of data. Write an expression that describes the monthly charge and use d to represent data We multiply gigabyte fee by d and add the usage fee: [B]15d + 30[/B]

A phone company offers two monthly charge plans. In Plan A, there is no monthly fee, but the custome
A phone company offers two monthly charge plans. In Plan A, there is no monthly fee, but the customer pays 8 cents per minute of use. In Plan B, the customer pays a monthly fee of $1.50 and then an additional 7 cents per minute of use. For what amounts of monthly phone use will Plan A cost more than Plan B? Set up the cost equations for each plan. The cost equation for the phone plans is as follows: Cost = Cost Per Minute * Minutes + Monthly Fee Calculate the cost of Plan A: Cost for A = 0.08m + 0. <-- Since there's no monthly fee Calculate the cost of Plan B: Cost for B = 0.07m + 1.50 The problem asks for what amounts of monthly phone use will Plan A be more than Plan B. So we set up an inequality: 0.08m > 0.07m + 1.50 [URL='https://www.mathcelebrity.com/1unk.php?num=0.08m%3E0.07m%2B1.50&pl=Solve']Typing this inequality into our search engine[/URL], we get: [B]m > 150 This means Plan A costs more when you use more than 150 minutes per month.[/B]

A photographer snapped 224 photos over a period of 15 days. At this rate, how many would he take in
A photographer snapped 224 photos over a period of 15 days. At this rate, how many would he take in 45 days? Set up a proportion of photos to days where p is the number of photos snapped in 45 days: 224/15 = p/45 To solve this proportion for p, we [URL='https://www.mathcelebrity.com/prop.php?num1=224&num2=p&den1=15&den2=45&propsign=%3D&pl=Calculate+missing+proportion+value']type it in our search engine[/URL] and we get; p = [B]672[/B]

A piece of gym equipment which cost 450 including vat last year is now selling at 500 excluding vat.
A piece of gym equipment which cost 450 including vat last year is now selling at 500 excluding vat. Calculate the percentage increase. Increase = (New Price - Old Price)/Old Price Increase = (500-450)/450 50/450 = 0.1111 To get the percentage, multiply by 100 [B]11.11%[/B]

A piece of pipe is 144 inches long. After 4 pieces, each 33 inches long are cut, what length of pipe
A piece of pipe is 144 inches long. After 4 pieces, each 33 inches long are cut, what length of pipe is left? Calculate the length of cut pipe: 4 pieces * 33 inches per piece = 132 inches The remaining pipe is found by subtracting the original pipe length by the cut pipe length: 144 - 132 = [B]12 inches[/B]

a piece of ribbon is y cm long. Ashley cut it into 4 equal pieces to do her 4 sisters hair. write an
a piece of ribbon is y cm long. Ashley cut it into 4 equal pieces to do her 4 sisters hair. write an expression for the amount of ribbon used for each sister We take y cm and divide it equal among 4 sisters: [B]y/4[/B]

A piggy bank contains $90.25 in dimes and quarters. Which equation represents this scenario? Let x r
A piggy bank contains $90.25 in dimes and quarters. Which equation represents this scenario? Let x represent the number of dimes, and let y represent the number of quarters. Since amount = cost * quantity, we have: [B]0.1d + 0.25q = 90.25[/B]

A pile of coins, consisting of quarters and half dollars, is worth 11.75. If there are 2 more quarte
A pile of coins, consisting of quarters and half dollars, is worth 11.75. If there are 2 more quarters than half dollars, how many of each are there? Let h be the number of half-dollars and q be the number of quarters. Set up two equations: (1) q = h + 2 (2) 0.25q + 0.5h = 11.75 [U]Substitute (1) into (2)[/U] 0.25(h + 2) + 0.5h = 11.75 0.25h + 0.5 + 0.5h = 11.75 [U]Group h terms[/U] 0.75h + 0.5 = 11.75 [U]Subtract 0.5 from each side[/U] 0.75h = 11.25 [U]Divide each side by h[/U] [B]h = 15[/B] [U]Substitute h = 15 into (1)[/U] q = 15 + 2 [B]q = 17[/B]

A plain chocolate bar weights 40 grams. It contains 12 grams of fat. A milk chocolate bar is 30 gram
A plain chocolate bar weights 40 grams. It contains 12 grams of fat. A milk chocolate bar is 30 grams. It contains 10 grams of fat. Which chocolate bar has the highest proportion of fat? Calculate the proportion of fat grams for each chocolate bar: [LIST] [*]Plain chocolate: 12/40 = 3/10 = 30% [*]Milk chocolate: 10/30 = 1/3 = 33% [/LIST] [B]Milk chocolate[/B] has the higher proportion of fat grams.

A plane is flying at an altitude of 45,000 feet. It begins to drop in altitude 3,000 feet per minute
A plane is flying at an altitude of 45,000 feet. It begins to drop in altitude 3,000 feet per minute. What is the slope in this situation? Set up a graph where minutes is on the x-axis and altitude is on the y-axis. [LIST=1] [*]Minute 1 = (1, 42,000) [*]Minute 2 = (2, 39,000) [*]Minute 3 = (3, 36,000) [*]Minute 4 = (4, 33,000) [/LIST] You can see for every 1 unit move in x, we get a -3,000 unit move in y. Pick any of these 2 points, and [URL='https://www.mathcelebrity.com/slope.php?xone=1&yone=42000&slope=+2%2F5&xtwo=2&ytwo=39000&bvalue=+&pl=You+entered+2+points']use our slope calculator[/URL] to get: Slope = -[B]3,000[/B]

A plane takes off at 11:53 and lands at 9 minutes to 2. How long is the flight?
A plane takes off at 11:53 and lands at 9 minutes to 2. How long is the flight? 9 minutes to 2 is 1:51 11:53 to 1:53 is exactly 2 hours. 1:51 is 2 minutes less than 1:53. So we have [B]1 hour and 58 minutes[/B]

A plant is 15 cm high and grows 4.5 cm every month. How many months will it take until the plant is
A plant is 15 cm high and grows 4.5 cm every month. How many months will it take until the plant is 27.5 cm We set up the height function H(m) where m is the number of months since now. We have: H(m) = 4.5m + 15 We want to know when H(m) = 27.5, so we set our H(m) function equal to 27.5: 4.5m + 15 = 27.5 To solve for m, we [URL='https://www.mathcelebrity.com/1unk.php?num=4.5m%2B15%3D27.5&pl=Solve']type this equation into our search engine[/URL] and we get: m = 2.78 So we round up to [B]3 whole months[/B]

A playground requires 2,459 pounds of sand to cover the ground. If the sand comes in 60-pound bags,
A playground requires 2,459 pounds of sand to cover the ground. If the sand comes in 60-pound bags, how many bags are needed Number of bags = Total Weight of Sand / Pounds per bag Number of bags = 2459/60 Number of bags = 40.9833

A playing card is 7 centimeters wide and 10 centimeters tall. What is its area?
A playing card is 7 centimeters wide and 10 centimeters tall. What is its area? A playing card has a rectangle shape, so the area is l x w. A = l x w A = 10 cm x 7 cm A =[B] 70 cm^2[/B]

A plumber charges $45 for a house call plus $25 for each hour worked.Let h represent the number of h
A plumber charges $45 for a house call plus $25 for each hour worked.Let h represent the number of hours worked. Write the expression that shows how much a plumber charges for a job. Then find how much the plumbers charges for a job lasting 4 hours [U]Set up the cost function C(h) where h is the number of hours:[/U] C(h) = Hours worked * hourly rate + house call fee [B]C(h) = 25h + 45 <-- This is the expression for how much the plumber charges for a job [/B] [U]Now determine how much the plumber charges for a job lasting 4 hours[/U] We want C(4) C(4) = 25(4) + 45 C(4) = 100 + 45 C(4) = [B]$145[/B]

A plumber charges $50 to visit a house plus $40 for every hour of work.
A plumber charges $50 to visit a house plus $40 for every hour of work. Set up the cost function in terms of hours (h) using the flat fee of $50 [B]C(h) = 40h + 50[/B]

A plumber makes a starting $36,000 a year. They get paid semimonthly. They have a health insurance p
A plumber makes a starting $36,000 a year. They get paid semimonthly. They have a health insurance premium of $74.28 and $25 in union dues each paycheck. 1. What is their semimonthly salary? Calculate the number of semi-monthly periods per year: Semi-monthly periods per year = 12 Months per year * 2 Semi-monthly periods per year = 24 Calculate semi-monthly salary amount: Semi-monthly salary amount = Annual Salary / Semi-monthly periods per year Semi-monthly salary amount = $36,000 / 24 Semi-monthly salary amount = $1,500 Now, calculate the net pay each semimonthly period: Net pay = Semi-monthly salary amount - Health Insurance Premium - Union Dues Net pay = $1,500 - $74.28 - $25 Net pay = [B]$1,400.72[/B]

A police officer is trying to catch a fleeing criminal. The criminal is 20 feet away from the cop, r
A police officer is trying to catch a fleeing criminal. The criminal is 20 feet away from the cop, running at a rate of 5 feet per second. The cop is running at a rate of 6.5 feet per second. How many seconds will it take for the police officer to catch the criminal? Distance = Rate * Time [U]Criminal:[/U] 5t + 20 [U]Cop[/U]: 6.5t We want to know when their distances are the same (cop catches criminal). So we set the equations equal to each other: 5t + 20 = 6.5t To solve this equation, [URL='https://www.mathcelebrity.com/1unk.php?num=5t%2B20%3D6.5t&pl=Solve']we type it in our search engine[/URL] and we get: t = 13.333 seconds

A pollster selected 4 of 7 people. How many different groups of 4 are possible?
A pollster selected 4 of 7 people. How many different groups of 4 are possible? We want to use the combinations formula. [URL='https://www.mathcelebrity.com/permutation.php?num=7&den=4&pl=Combinations']So we type 7C4 into our search engine[/URL]. This is also known as 7 choose 4. We get [B]35[/B] different groups.

A pool is 5 meters wide and 21 meter long what is the area of the pool?
A pool is 5 meters wide and 21 meter long what is the area of the pool? A pool is a rectangle. So the area for a rectangle is: A = lw [I]where l is the length and w is the width.[/I] [URL='https://www.mathcelebrity.com/rectangle.php?l=21&w=5&a=&p=&pl=Calculate+Rectangle']Plugging in our width of 5 and length of 21 to our rectangle calculator[/URL], we get: A = [B]105 m^2[/B]

A population grows at 6% per year. How many years does it take to triple in size?
A population grows at 6% per year. How many years does it take to triple in size? With a starting population of P, and triple in size means 3 times the original, we want to know t for: P(1.06)^t = 3P Divide each side by P, and we have: 1.06^t = 3 Typing this equation into our search engine to solve for t, we get: t = [B]18.85 years[/B] Note: if you need an integer answer, we round up to 19 years

A population of 200 doubles in size every hour. What is the rate of growth of the population after 2
A population of 200 doubles in size every hour. What is the rate of growth of the population after 2.5 hours? Time 1: 400 Time 2: 800 Time 3: 1200 (Since it's only 1/2 of a period)

A population of wolves on an island starts at 5 if the population doubles every 10 years, what will
A population of wolves on an island starts at 5 if the population doubles every 10 years, what will be the population in 90 years? If the population doubles every 10 years, we have 90/10 = 9 doubling periods. Our population function P(t) is where t is the doubling period P(t) = 5(2^t) The problem asks for P(9): P(9) = 5(2^9) P(9) = 5(512) P(9) = [B]2,560[/B]

A postcard is 4 inches tall and 5 inches wide. What is its area?
A postcard is 4 inches tall and 5 inches wide. What is its area? A postcard is a rectangle. The area is 4 x 5 = [B]20 square inches[/B]

A pot of soup, currently 66°C above room temperature, is left out to cool. If that temperature diffe
A pot of soup, currently 66°C above room temperature, is left out to cool. If that temperature difference decreases by 10% per minute, then what will the difference be in 17 minutes? We set up the temperature function T(m), where m is the number of minutes of cooling. With 10% = 0.1, we have: T(m) = 66 * (1 - 0.10)^m The problem asks for T(17) [U]and[/U] the difference temperature: T(17) = 66 * 0.9^17 T(17) = 66 * 0.16677181699 T(17) = [B]11.01C[/B] [B][/B] [U]Calculate the difference in temperature[/U] Difference = Starting Temperature - Ending Temperature Difference = 66 - 11.01 Difference = 66 - 11.01 = [B]54.99 ~ 55[/B]

A pound of chocolate costs 6 dollars. Greg buys p pounds. Write an equation to represent the total c
A pound of chocolate costs 6 dollars. Greg buys p pounds. Write an equation to represent the total cost c that Greg pays Since cost = price * quantity, we have: [B]c = 6p[/B]

A pound of chocolate costs 6 dollars. Ryan buys p pounds. Write an equation to represent the total c
A pound of chocolate costs 6 dollars. Ryan buys p pounds. Write an equation to represent the total cost c that Ryan pays Since cost = Price * Quantity, we have: [B]c = 6p[/B]

A pound of chocolate costs 7 dollars. Hong buys p pounds . Write an equation to represent the total
A pound of chocolate costs 7 dollars. Hong buys p pounds . Write an equation to represent the total cost c that Hong pays Our equation is the cost of chocolate multiplied by the number of pounds: [B]c = 7p[/B]

A pound of coffee costs $14.99. What is the cost per ounce?
A pound of coffee costs $14.99. What is the cost per ounce? 1 pound = 16 ounces. So we have: $14.99/16 ounces [B]$0.94 per ounce[/B]

A pound of popcorn is popped for a class party. The popped corn is put into small popcorn boxes that
A pound of popcorn is popped for a class party. The popped corn is put into small popcorn boxes that each hold 120 popped kernels. There are 1,600 kernels in a pound of unpopped popcorn. If all the boxes are filled except for the last box, how many boxes are needed and how many popped kernels are in the last partially filled box? Using modulus calculator, we know [URL='https://www.mathcelebrity.com/modulus.php?num=1600mod120&pl=Calculate+Modulus']1600 mod 120[/URL] gives us [B]13 full boxes[/B] of unpopped popcorn. We also know that 13*120 = 1,560. Which means we have 1,600 - 1,560 = [B]40[/B] popped kernels left in the last box. FB Live: [URL='https://www.facebook.com/plugins/video.php?href=https%3A%2F%2Fwww.facebook.com%2FMathCelebrity%2Fvideos%2F10156733590718291%2F&show_text=0&width=560']https://www.facebook.com/plugins/video.php?href=https://www.facebook.com/MathCelebrity/videos/10156733590718291/&show_text=0&width=560[/URL]

A pound-and-a-half of candy costs $1.65. Find the cost of one pound
A pound-and-a-half of candy costs $1.65. Find the cost of one pound 1.65/1.5 = [B]$1.10 per pound[/B]

A pretzel factory has daily fixed costs of $1100. In addition, it costs 70 cents to produce each bag
A pretzel factory has daily fixed costs of $1100. In addition, it costs 70 cents to produce each bag of pretzels. A bag of pretzels sells for $1.80. [U]Build the cost function C(b) where b is the number of bags of pretzels:[/U] C(b) = Cost per bag * b + Fixed Costs C(b) = 0.70b + 1100 [U]Build the revenue function R(b) where b is the number of bags of pretzels:[/U] R(b) = Sale price * b R(b) = 1.80b [U]Build the revenue function P(b) where b is the number of bags of pretzels:[/U] P(b) = Revenue - Cost P(b) = R(b) - C(b) P(b) = 1.80b - (0.70b + 1100) P(b) = 1.80b = 0.70b - 1100 P(b) = 1.10b - 1100

A principal of $3300 is invested at 3.25% interest, compounded annually. How much will the investmen
A principal of $3300 is invested at 3.25% interest, compounded annually. How much will the investment be worth after 10 years? [URL='https://www.mathcelebrity.com/compoundint.php?bal=3300&nval=10&int=3.25&pl=Annually']Using our balance calculator with compound interest[/URL], we get: [B]$4,543.75[/B]

A printer can print 25 pages per minute. At this rate, how long will it take to print 2000 pages?
A printer can print 25 pages per minute. At this rate, how long will it take to print 2000 pages? Let m be the number of minutes it takes to print 2,000 pages. We have the equation: 25m = 2000 [URL='https://www.mathcelebrity.com/1unk.php?num=25m%3D2000&pl=Solve']Type this equation into our search engine[/URL], and we get: m = 80

a printer charges a $30 setup fee plus $2.00 per ticket. Write an algebraic expression for the cost
a printer charges a $30 setup fee plus $2.00 per ticket. Write an algebraic expression for the cost of t tickets. What is the cost of 225 tickets? Algebraic Expression: Cost per ticket * t + set up fee [B]2t + 30[/B] How much for t = 225? 2(225) + 30 450 + 30 [B]480[/B]

A printer prints 2 photos each minute. Let P be the number of photos printed in M minutes. Write an
A printer prints 2 photos each minute. Let P be the number of photos printed in M minutes. Write an equation relating P to M. Set up the equation P(M). [B]P(M) = 2M[/B] Read this as for every minute that goes by, 2 photos are printed.

A private high school charges $36,400 for tuition, but this figure is expected to rise 10% per year.
A private high school charges $36,400 for tuition, but this figure is expected to rise 10% per year. What will tuition be in 10 years? Let the tuition be T(y) where y is the number of years from now. We've got: T(y) = 36400 * (1.1)^y The problem asks for T(10) T(10) = 36400 * (1.1)^10 T(10) = 36400 * 2.5937424601 T(10) = [B]94,412.23[/B]

A private high school charges $52,200 for tuition, but this figure is expected to rise 7% per year.
A private high school charges $52,200 for tuition, but this figure is expected to rise 7% per year. What will tuition be in 3 years? We have the following appreciation equation A(y) where y is the number of years: A(y) = Initial Balance * (1 + appreciation percentage)^ years Appreciation percentage of 7% is written as 0.07, so we have: A(3) = 52,200 * (1 + 0.07)^3 A(3) = 52,200 * (1.07)^3 A(3) = 52,200 * 1.225043 A(3) = [B]63,947.25[/B]

A private jet flies the same distance in 4 hours that a commercial jet flies in 2 hours. If the spee
A private jet flies the same distance in 4 hours that a commercial jet flies in 2 hours. If the speed of the commercial jet was 154 mph less than 3 times the speed of the private jet, find the speed of each jet. Let p = private jet speed and c = commercial jet speed. We have two equations: (1) c = 3p - 154 (2) 4p =2c Plug (1) into (2): 4p = 2(3p - 154) 4p = 6p - 308 Subtract 4p from each side: 2p - 308 = 0 Add 308 to each side: 2p = 308 Divide each side by 2: [B]p = 154[/B] Substitute this into (1) c = 3(154) - 154 c = 462 - 154 [B]c = 308[/B]

A problem states: "There are 9 more children than parents in a room. There are 25 people in the room
A problem states: "There are 9 more children than parents in a room. There are 25 people in the room in all. How many children are there in the room?" Let the number of children be c. Let the number of parents be p We're given: [LIST=1] [*]c = p + 9 [I](9 more children than parents)[/I] [*]c + p = 25 [/LIST] to solve this system of equations, we plug equation (1) into equation (2) for c: (p + 9) + p = 25 Group like terms: 2p + 9 = 25 To solve this equation for p, we [URL='https://www.mathcelebrity.com/1unk.php?num=2p%2B9%3D25&pl=Solve']type it in our search engine[/URL] and we get: p = [B]8[/B]

A professor assumed there was a correlation between the amount of hours people were expose to sunlig
A professor assumed there was a correlation between the amount of hours people were expose to sunlight and their blood vitamin D level. The null hypothesis was that the population correlation was__ a. Positive 1.0 b. Negative 1.0 c. Zero d. Positive 0.50 [B]c. Zero[/B] Reason: Since the professor wanted to assume a correlation (either positive = 1.0 or negative = -1.0), then we take the other side of that assumption for our null hypothesis and say that there is no correlation (Zero)

A professor wants to test all possible pairwise comparisons among three means. If we need to maintai
A professor wants to test all possible pairwise comparisons among three means. If we need to maintain an experiment-wise alpha of 0.05, what is the error rate per comparison after applying Bonferroni correction? We are given: [LIST] [*]? = 0.05 [*]n = 3 [/LIST] Bonferroni Correction = ?/n Bonferroni Correction = 0.05/3 Bonferroni Correction = [B]0.01666666667[/B]

A project requires a $5000 investment. It pays out $1000 at year 1, $2000 at year 2, $3000 at year 3
A project requires a $5000 investment. It pays out $1000 at year 1, $2000 at year 2, $3000 at year 3. The discount rate is 5%. Should you invest? Using our [URL='https://www.mathcelebrity.com/npv.php?matrix1=0%2C-5000%0D%0A1%2C1000%0D%0A2%2C2000%0D%0A3%2C3000&irr=5&pl=NPV']NPV calculator,[/URL] we get: NPV = 357.94. Because NPV > 0, we [B]should invest [MEDIA=youtube]jXvwCTDwQ1o[/MEDIA][/B]

A promotional deal for long distance phone service charges a $15 basic fee plus $0.05 per minute for
A promotional deal for long distance phone service charges a $15 basic fee plus $0.05 per minute for all calls. If Joe's phone bill was $60 under this promotional deal, how many minutes of phone calls did he make? Round to the nearest integer if necessary. Let m be the number of minutes Joe used. We have a cost function of: C(m) = 0.05m + 15 If C(m) = 60, then we have: 0.05m + 15 = 60 [URL='https://www.mathcelebrity.com/1unk.php?num=0.05m%2B15%3D60&pl=Solve']Typing this equation into our search engine[/URL], we get: m = [B]900[/B]

A property sold for $198,000 with a listing commission of 8%. The selling office is to receive 40% o
A property sold for $198,000 with a listing commission of 8%. The selling office is to receive 40% of the total commission. How much will the listing salesperson receive if she is paid 60% of the amount retained by listed broker. [U]Calculate commission amount:[/U] Commission amount = Sale Price * Commission Percent Commission amount = 198,000 * 8% [URL='https://www.mathcelebrity.com/perc.php?num=+5&den=+8&num1=+16&pct1=+80&pct2=8&den1=198000&pcheck=3&pct=+82&decimal=+65.236&astart=+12&aend=+20&wp1=20&wp2=30&pl=Calculate']Commission amount [/URL]= 15,840 [U]Calculate listing salesperson commission amount:[/U] Listing salesperson commission amount = Commission Amount * Listing salesperson Percent Listing salesperson commission amount = 15,840 * 60% [URL='https://www.mathcelebrity.com/perc.php?num=+5&den=+8&num1=+16&pct1=+80&pct2=60&den1=15840&pcheck=3&pct=+82&decimal=+65.236&astart=+12&aend=+20&wp1=20&wp2=30&pl=Calculate']Listing salesperson commission amount[/URL] = [B]9,504[/B]

A quantity x is at least 10 and at most 20
A quantity x is at least 10 and at most 20 The phrase [I]at most[/I] means less than or equal to The phrase [I]at least[/I] means greater than or equal to. So we have the following inequality [B]10 <= x <= 20[/B]

A quarter of a number is greater than or equal to 38
A quarter of a number is greater than or equal to 38. The phrase [I]a number[/I] means an arbitrary variable, let's call it x. A quarter of a number means 1/4, so we have: x/4 The phrase [I]is greater than or equal to[/I] means an inequality, so we use the >= sign in relation to 38: [B]x/4 >= 38 <-- This is our algebraic expression [/B] If you want to solve this inequality, [URL='https://www.mathcelebrity.com/prop.php?num1=x&num2=38&propsign=%3E%3D&den1=4&den2=1&pl=Calculate+missing+proportion+value']we type it in the search engine[/URL] to get: x >= [B]152[/B]

A quarter of the learners in a class have blond hair and two thirds have brown hair. The rest of the
A quarter of the learners in a class have blond hair and two thirds have brown hair. The rest of the learners in the class have black hair. How many learners in the class if 9 of them have blonde hair? Total learners = Blond + Brown + Black Total Learners = 1/4 + 2/3 + Black Total Learners will be 1, the sum of all fractions 1/4 + 2/3 + Black = 1 Using common denominators of 12, we have: 3/12 + 8/12 + Black = 12/12 11/12 + Black = 12/12 Subtract 11/12 from each side: Black = 1/12 Let t be the total number of people in class. We are given for blondes: 1/4t = 9 Multiply each side by 4 [B]t = 36[/B] Brown Hair 2/3(36) = 24 Black Hair 1/12(36) = 3

A random sample of 100 light bulbs has a mean lifetime of 3000 hours. Assume that the population sta
A random sample of 100 light bulbs has a mean lifetime of 3000 hours. Assume that the population standard deviation of the lifetime is 500 hours. Construct a 95% confidence interval estimate of the mean lifetime. [B]2902 < u < 3098[/B] using our [URL='http://www.mathcelebrity.com/normconf.php?n=100&xbar=3000&stdev=500&conf=95&rdig=4&pl=Large+Sample']confidence interval for the mean calculator[/URL]

A random sample of 144 with a mean of 100 and a standard deviation of 70 is known from a population
A random sample of 144 with a mean of 100 and a standard deviation of 70 is known from a population of 1,000. What is the 95% confidence interval for the unknown population? [URL='http://www.mathcelebrity.com/normconf.php?n=144&xbar=100&stdev=70&conf=95&rdig=4&pl=Large+Sample']Large Sample Confidence Interval Mean Test[/URL] [B]88.5667 < u < 111.4333[/B]

A random sample of 149 students has a test score average of 61 with a standard deviation of 10. Find
A random sample of 149 students has a test score average of 61 with a standard deviation of 10. Find the margin of error if the confidence level is 0.99. (Round answer to two decimal places) Using our [URL='https://www.mathcelebrity.com/normconf.php?n=149&xbar=61&stdev=10&conf=99&rdig=4&pl=Large+Sample']confidence interval of the mean calculator[/URL], we get [B]58.89 < u < 63.11[/B]

A random sample of 25 customers was chosen in CCP MiniMart between 3:00 and 4:00 PM on a Friday afte
A random sample of 25 customers was chosen in CCP MiniMart between 3:00 and 4:00 PM on a Friday afternoon. The frequency distribution below shows the distribution for checkout time (in minutes). Checkout Time (in minutes) | Frequency | Relative Frequency 1.0 - 1.9 | 2 | ? 2.0 - 2.9 | 8 | ? 3.0 - 3.9 | ? | ? 4.0 - 5.9 | 5 | ? Total | 25 | ? (a) Complete the frequency table with frequency and relative frequency. (b) What percentage of the checkout times was less than 3 minutes? (c)In what class interval must the median lie? Explain your answer. (d) Assume that the largest observation in this dataset is 5.8. Suppose this observation were incorrectly recorded as 8.5 instead of 5.8. Will the mean increase, decrease, or remain the same? Will the median increase, decrease or remain the same? Why? (a) [B]Checkout Time (in minutes) | Frequency | Relative Frequency 1.0 - 1.9 | 2 | 2/25 2.0 - 2.9 | 8 | 8/25 3.0 - 3.9 | 10 (since 25 - 5 + 8 + 2) = 10 | 10/25 4.0 - 5.9 | 5 | 5/25 Total | 25 | ?[/B] (b) (2 + 8)/25 = 10/25 = [B]40%[/B] c) [B]3.0 - 3.9[/B] since 2 + 8 + 10 + 5 = 25 and 13 is the middle value which occurs in the 3.0 - 3.9 interval (d) [B]Mean increases[/B] since it's a higher value than usual. Median would not change as the median is the most frequent distribution and assuming the 5.8 is only recorded once.

A random sample of 40 adults with no children under the age of 18 years results in a mean daily leis
A random sample of 40 adults with no children under the age of 18 years results in a mean daily leisure time of 5.22 hours, with a standard deviation of 2.31 hours. A random sample of 40 adults with children under the age of 18 results in a mean daily leisure time of 4.44 hours, with a standard deviation of 1.74 hours. Construct and interpret a 90% confidence interval for the mean difference in leisure time between adults with no children and adults with children left parenthesis mu 1 minus mu 2 right parenthesis (?1 - ?2). Using our confidence interval for [URL='http://www.mathcelebrity.com/meandiffconf.php?n1=+40&xbar1=+5.22&stdev1=+2.31&n2=+40&xbar2=+4.44&stdev2=1.74&conf=+90&pl=Mean+Diff+Conf.+Interval+%28Large+Sample%29']difference of means calculator[/URL], we get: [B]0.0278 < ?1 - ?2 < 1.5322[/B]

A random sample of 40 adults with no children under the age of 18 years results in a mean daily leis
A random sample of 40 adults with no children under the age of 18 years results in a mean daily leisure time of 5.22 hours, with a standard deviation of 2.31 hours. A random sample of 40 adults with children under the age of 18 results in a mean daily leisure time of 4.29 hours, with a standard deviation of 1.58 hours. Construct and interpret a 90% confidence interval for the mean difference in leisure time between adults with no children and adults with children (u1 - u2) What is the interpretation of this confidence interval? A. There is 90% confidence that the difference of the means is in the interval. Conclude that there is insufficient evidence of a significant difference in the number of leisure hours B. There is a 90% probability that the difference of the means is in the interval. Conclude that there is a significant difference in the number of leisure hours C. There is 90% confidence that the difference of the means is in the interval. Conclude that there is a significant difference in the number of leisure hours D. There is a 90% probability that the difference of the means is in the interval. Conclude that there is insufficient evidence of a significant difference in the number of leisure hours 0.2021 < u1 - u2 < 1.6579 using our [URL='http://www.mathcelebrity.com/meandiffconf.php?n1=+40&xbar1=+5.22&stdev1=2.31&n2=40&xbar2=4.29&stdev2=1.58&conf=+90&pl=Mean+Diff+Conf.+Interval+%28Large+Sample%29']difference of means confidence interval calculator[/URL] [B]Choice D There is a 90% probability that the difference of the means is in the interval. Conclude that there is insufficient evidence of a significant difference in the number of leisure hours[/B]

A random sample of n = 10 flash light batteries with a mean operating life X=5 hr. And a sample stan
A random sample of n = 10 flash light batteries with a mean operating life X=5 hr. And a sample standard deviation S = 1 hr. is picked from a production line known to produce batteries with normally distributed operating lives. What's the 98% confidence interval for the unknown mean of the working life of the entire population of batteries? [URL='http://www.mathcelebrity.com/normconf.php?n=10&xbar=5&stdev=1&conf=98&rdig=4&pl=Small+Sample']Small Sample Confidence Interval for the Mean test[/URL] [B]4.1078 < u < 5.8922[/B]

A random sample of STAT200 weekly study times in hours is as follows: 2 15 15 18 30 Find the sam
A random sample of STAT200 weekly study times in hours is as follows: 2 15 15 18 30 Find the sample standard deviation. (Round the answer to two decimal places. Show all work.) [B]9.98[/B] using [URL='http://www.mathcelebrity.com/statbasic.php?num1=+2,15,15,18,30&num2=+0.2,0.4,0.6,0.8,0.9&pl=Number+Set+Basics']our standard deviation calculator[/URL]

A random variable X follows the uniform distribution with a lower limit of 670 and an upper limit
A random variable X follows the uniform distribution with a lower limit of 670 and an upper limit a. Calculate the mean and standard deviation of this distribution. (Round intermediate calculation for standard deviation to 4 decimal places and final answer to 2 decimal places.) Using our [URL='http://www.mathcelebrity.com/uniform.php?a=+670&b=+770&x=+680&t=+3&pl=PDF']uniform distribution calculator[/URL], we get: [B]Mean = 720 Standard deviation = 28.87 [/B] b. What is the probability that X is less than 730? (Do not round intermediate calculations. Round your answer to 2 decimal places.) Using our [URL='http://www.mathcelebrity.com/uniform.php?a=+670&b=+770&x=+730&t=+3&pl=CDF']uniform distribution calculator[/URL], we get: [B]0.6[/B]

A rational expression is undefined when what is 0?
A rational expression is undefined when what is 0? The [B]denominator[/B]. Because division by zero is undefined.

A rational number is such that when you multiply it by 7/3 and subtract 3/2 from the product, you ge
A rational number is such that when you multiply it by 7/3 and subtract 3/2 from the product, you get 92. What is the number? Let the rational number be x. We're given: 7x/3 - 3/2 = 92 Using a common denominator of 3*2 = 6, we rewrite this as: 14x/6 - 9/6 = 92 (14x - 9)/6 = 92 Cross multiply: 14x - 9 = 92 * 6 14x - 9 = 552 To solve for x, we [URL='https://www.mathcelebrity.com/1unk.php?num=14x-9%3D552&pl=Solve']type this equation into our search engine [/URL]and we get: x = [B]40.07[/B]

A real estate agency receives 3.5% commission on the first $100,000 of a sale and 2.25% on the remai
A real estate agency receives 3.5% commission on the first $100,000 of a sale and 2.25% on the remainder. How much commission is received on the sale of a $450,000 property? Calculate commission on first $100,000 (Commission 1): Commission 1 = $100,000 * 0.035 Commission 1 = $3,500 Calculate commission on the remainder (Commission 2): Commission 2 = 0.025 * ($450,000 - $100,000) Commission 2 = 0.025 * ($350,000) Commission 2 = $8,750 Calculate Total Commission: Total Commission = Commission 1 + Commission 2 Total Commission = $3,500 + $8,750 Total Commission = [B]$12,250[/B]

A real estate agent has $920 to spend on newspaper ads. Each ad costs $6. After buying as many ads a
A real estate agent has $920 to spend on newspaper ads. Each ad costs $6. After buying as many ads as she can afford, how much money will the real estate agent have left over? We want to know the remainder of 920/6. We can type 920 mod 6 into our search engine and get: [URL='https://www.mathcelebrity.com/modulus.php?num=920mod6&pl=Calculate+Modulus']920 mod 6[/URL] = [B]2[/B]

A real estate agent sells a house for $229,605. A sales commission of 6% is charged. The agent gets
A real estate agent sells a house for $229,605. A sales commission of 6% is charged. The agent gets 45% of this commission. How much money does the agent get? The agents Commission (C) is: C = Sale price * sales commission percent * agent commission percent Since 6% = 0.06 and 45% = 0.45, we have: C = 229605 * 0.06 * 0.45 C = [B]6,199.34[/B]

A realtor makes an annual salary of $25000 plus a 3% commission on sales. If a realtor's salary is $
A realtor makes an annual salary of $25000 plus a 3% commission on sales. If a realtor's salary is $67000, what was the amount of her sales? Total post-salary pay = $67,000 - $25,000 = $42,000 Let Sales be s. So 0.03s = $42,000 Divide each side by 0.03 s = $1,400,000

A recent survey showed that 44% of recent college graduates named flexible hours as their most desir
A recent survey showed that 44% of recent college graduates named flexible hours as their most desire employment benefit. In a graduating class of 870 college students, how many would you expect to rank flexible hours as their top priority in job benefits? Using our [URL='http://www.mathcelebrity.com/perc.php?num=+5&den=+8&num1=+16&pct1=+80&pct2=44&den1=870&pcheck=3&pct=+82&decimal=+65.236&astart=+12&aend=+20&wp1=20&wp2=30&pl=Calculate']percentage calculator[/URL], 44% of 870 = 382.8 ~ [B]383[/B]

A recent survey showed that 49% of recent college graduates named flexible hours as their most desir
A recent survey showed that 49% of recent college graduates named flexible hours as their most desire employment benefit. In a graduating class of 820 college students, how many would you expect to rank flexible hours as their top priority in job benefits? 49% of 820, using our [URL='http://www.mathcelebrity.com/perc.php?num=+5&den=+8&num1=+16&pct1=+80&pct2=49&den1=820&pcheck=3&pct=+82&decimal=+65.236&astart=+12&aend=+20&wp1=20&wp2=30&pl=Calculate']percentage calculator[/URL], we get: 401.8 ~ [B]402[/B]

a recipe of 20 bread rolls requires 5 tablespoons of butter. How many tablespoons of butter are need
a recipe of 20 bread rolls requires 5 tablespoons of butter. How many tablespoons of butter are needed for 30 bread rolls? Set up a proportion of bread rolls per tablespoons of butter where t is the amount of tablespoons of butter needed for 30 bread rolls: 20/5 = 30/t Cross multiply our proportion: Numerator 1 * Denominator 2 = Denominator 1 * Numerator 2 20t = 30 * 5 20t = 150 Divide each side of the equation by 20: 20t/20 = 150/20 Cancel the 20's on the left side and we get: t = [B]7.5[/B]

A recipe that makes 25 oatmeal cookies calls for 2.5 cups of oats and one cup of sugar. Jerry needs
A recipe that makes 25 oatmeal cookies calls for 2.5 cups of oats and one cup of sugar. Jerry needs to make 195 cookies for his school party. How many cups of oats will he need? Set up a proportion of oats to cookies where c is the number of cups needed to make 195 cookies 2.5/25 = c/195 Using our [URL='https://www.mathcelebrity.com/proportion-calculator.php?num1=2.5&num2=c&den1=25&den2=195&propsign=%3D&pl=Calculate+missing+proportion+value']proportion calculator,[/URL] we get: c = [B]19.5[/B]

A recipie calls for 2 tablespoons of olive oil for every 3 servings. How much olive oil will be nee
A recipe calls for 2 tablespoons of olive oil for every 3 servings. How much olive oil will be needed for 6 servings? Set up a proportion of tablespoons to servings: 2/3 = o/6 where o is the number of tablespoons per 6 servings. [URL='https://www.mathcelebrity.com/prop.php?num1=2&num2=o&den1=3&den2=6&propsign=%3D&pl=Calculate+missing+proportion+value']Type 2/3 = o/6 into our search engine[/URL], and we get [B]o = 4[/B].

a rectangle has a length of x-7 and a width of x + 5. Write an expression that represents the area o
a rectangle has a length of x-7 and a width of x + 5. Write an expression that represents the area of the rectangle in terms of x. Area of a rectangle (A) with length(l) and width (w) is expressed as follows: A = lw Plugging in our values given above, we have: [B]A = (x - 7)(x + 5)[/B]

A rectangle has a length that is 8.5 times its width. IF the width is n, what is the perimeter of th
A rectangle has a length that is 8.5 times its width. IF the width is n, what is the perimeter of the rectangle. w = n l = 8.5n P = 2(8.5n) + 2n P = 17n + 2n P = [B]19n[/B]

A RECTANGLE HAS A PERIMETER OF 196 CENTIMETERS. IF THE LENGTH IS 6 TIMES ITS WIDTH FIND TH DIMENSION
A RECTANGLE HAS A PERIMETER OF 196 CENTIMETERS. IF THE LENGTH IS 6 TIMES ITS WIDTH FIND TH DIMENSIONS OF THE RECTANGLE? Whoa... stop screaming with those capital letters! But I digress... The perimeter of a rectangle is: P = 2l + 2w We're given two equations: [LIST=1] [*]P = 196 [*]l = 6w [/LIST] Plug these into the perimeter formula: 2(6w) + 2w = 196 12w + 2w = 196 [URL='https://www.mathcelebrity.com/1unk.php?num=12w%2B2w%3D196&pl=Solve']Plugging this equation into our search engine[/URL], we get: [B]w = 14[/B] Now we put w = 14 into equation (2) above: l = 6(14) [B]l = 84 [/B] So our length (l), width (w) of the rectangle is (l, w) = [B](84, 14) [/B] Let's check our work by plugging this into the perimeter formula: 2(84) + 2(14) ? 196 168 + 28 ? 196 196 = 196 <-- checks out

a rectangle has an area of 238 cm 2 and a perimeter of 62 cm. What are its dimensions?
a rectangle has an area of 238 cm 2 and a perimeter of 62 cm. What are its dimensions? We know the rectangle has the following formulas: Area = lw Perimeter = 2l + 2w Given an area of 238 and a perimeter of 62, we have: [LIST=1] [*]lw = 238 [*]2(l + w) = 62 [/LIST] Divide each side of (1) by w: l = 238/w Substitute this into (2): 2(238/w + w) = 62 Divide each side by 2: 238/w + w = 31 Multiply each side by w: 238w/w + w^2 = 31w Simplify: 238 + w^2 = 31w Subtract 31w from each side: w^2 - 31w + 238 = 0 We have a quadratic. So we run this through our [URL='https://www.mathcelebrity.com/quadratic.php?num=w%5E2-31w%2B238%3D0&pl=Solve+Quadratic+Equation&hintnum=+0']quadratic equation calculator[/URL] and we get: w = (14, 17) We take the lower amount as our width and the higher amount as our length: [B]w = 14 l = 17 [/B] Check our work for Area: 14(17) = 238 <-- Check Check our work for Perimeter: 2(17 + 14) ? 62 2(31) ? 62 62 = 62 <-- Check

A rectangle is cut in half to create two squares that each has an area of 25. What is the perimeter
[SIZE=4]A rectangle is cut in half to create two squares that each has an area of 25. What is the perimeter of the original rectangle? A. 20 B. 25 C. 30 D. 50[/SIZE] [SIZE=4]Area of a square: A = s^2 We're given A = 25: s^2= 25 s = 5 This means the rectangle width is 5. The rectangle length is 2(5) = 10. Perimeter of a rectangle: P = 2l + 2w P = 2(10) + 2(5) P = 20 + 10 P = [B]30 (choice C)[/B] [B][/B] [B][MEDIA=youtube]tKpS1gQY68o[/MEDIA][/B][/SIZE]

A rectangle shaped parking lot is to have a perimeter of 506 yards. If the width must be 100 yards b
A rectangle shaped parking lot is to have a perimeter of 506 yards. If the width must be 100 yards because of a building code, what will the length need to be? Perimeter of a rectangle (P) with length (l) and width (w) is: 2l + 2w = P We're given P = 506 and w = 100. We plug this in to the perimeter formula and get: 2l + 2(100) = 506 To solve this equation for l, we [URL='https://www.mathcelebrity.com/1unk.php?num=2l%2B2%28100%29%3D506&pl=Solve']type it in our search engine[/URL] and we get: l = [B]153[/B]

A rectangular field is to be enclosed with 1120 feet of fencing. If the length of the field is 40 fe
A rectangular field is to be enclosed with 1120 feet of fencing. If the length of the field is 40 feet longer than the width, then how wide is the field? We're given: [LIST=1] [*]l = w + 40 [/LIST] And we know the perimeter of a rectangle is: P = 2l + 2w Substitute (1) into this formula as well as the given perimeter of 1120: 2(w + 40) + 2w = 1120 Multiply through and simplify: 2w + 80 + 2w = 1120 Group like terms: 4w + 80 = 1120 [URL='https://www.mathcelebrity.com/1unk.php?num=4w%2B80%3D1120&pl=Solve']Typing this equation into our search engine[/URL], we get: [B]w = 260[/B]

A rectangular field is twice as long as it is wide. If the perimeter is 360 what are the dimensions?
A rectangular field is twice as long as it is wide. If the perimeter is 360 what are the dimensions? We are given or know the following about the rectangle [LIST] [*]l = 2w [*]P = 2l + 2w [*]Since P = 360, we have 2l + 2w = 360 [/LIST] Since l = 2w, we have 2l + (l) = 360 3l = 360 Divide by 3, we get [B]l = 120[/B] Which means w = 120/2 [B]w = 60[/B]

A rectangular fish tank has a base that is 8 inches by 7 inches. How much water will it take to a de
A rectangular fish tank has a base that is 8 inches by 7 inches. How much water will it take to a depth of 5 inches The volume (V) or a rectangular solid is: V = lwh Using l = 8, w = 7, and h = 5, we have: V = 8(7)(5) V = [B]280 cubic inches[/B]

A rectangular football pitch has its length equal to twice its width and a perimeter of 360m. Find i
A rectangular football pitch has its length equal to twice its width and a perimeter of 360m. Find its length and width. The area of a rectangle (A) is: A = lw --> where l is the length and w is the width We're given l = 2w, so we substitute this into the Area equation: A = (2w)w A = 2w^2 We're given the area of the pitch is 360, so we set: 2w^2 = 360 We [URL='https://www.mathcelebrity.com/1unk.php?num=2w%5E2%3D360&pl=Solve']type this equation into our search engine[/URL], follow the links, and get: w = [B]6*sqrt(5) [/B] Now we take this, and substitute it into this equation: 6*sqrt(5)l = 360 Dividing each side by 6*sqrt(5), we get: l = [B]60/sqrt(5)[/B]

A rectangular garden is 5 ft longer than it is wide. Its area is 546 ft2. What are its dimensions?
A rectangular garden is 5 ft longer than it is wide. Its area is 546 ft2. What are its dimensions? [LIST=1] [*]Area of a rectangle is lw. lw = 546ft^2 [*]We know that l = w + 5. [/LIST] Substitute (2) into (1) (w + 5)w = 546 w^2 + 5w = 546 Subtract 546 from each side w^2 + 5w - 546 = 0 Using the positive root in our [URL='http://www.mathcelebrity.com/quadratic.php?num=w%5E2%2B5w-546%3D0&pl=Solve+Quadratic+Equation&hintnum=+0']quadratic calculator[/URL], we get [B]w = 21[/B]. This means l = 21 + 5. [B]l = 26[/B]

A rectangular hotel room is 4 yards by 5 yards. The owner of the hotel wants to recarpet the room wi
A rectangular hotel room is 4 yards by 5 yards. The owner of the hotel wants to recarpet the room with carpet that costs $76.00 per square yard. How much will it cost to recarpet the room? $ The area of a rectangle is length * width, so we have: A = 5 yards * 4 yards A = 20 square yards Total cost = Cost per square yard * total square yards Total Cost = $76 * 20 Total Cost = [B]$1520[/B]

A rectangular house is 68 yards wide and 112 yards long. What is its perimeter?
A rectangular house is 68 yards wide and 112 yards long. What is its perimeter? The perimeter of a rectangle is: P = 2l + 2w Plugging in our length of 112 and our width of 68, we get: P = 2(112) + 2(68) P = 224 + 136 P = [B]360[/B]

A rectangular parking lot has a perimeter of 152 yards. If the length of the parking lot is 12 yards
A rectangular parking lot has a perimeter of 152 yards. If the length of the parking lot is 12 yards greater than the width. What is the width of the parking lot? The perimeter of a rectangle is: 2l + 2w = P. We're given 2 equations: [LIST=1] [*]2l + 2w = 152 [*]l = w + 12 [/LIST] Substitute equation (2) into equation (1) for l: 2(w + 12) + 2w = 152 2w + 24 + 2w = 152 Combine like terms: 4w + 24 = 152 To solve for w, we [URL='https://www.mathcelebrity.com/1unk.php?num=4w%2B24%3D152&pl=Solve']type this equation into our search engine[/URL] and we get: w =[B] 32[/B]

A rectangular piece of paper has the dimensions of 10 inches by 7 inches.What is the perimeter of th
A rectangular piece of paper has the dimensions of 10 inches by 7 inches.What is the perimeter of the piece of paper Using our [URL='https://www.mathcelebrity.com/rectangle.php?l=10&w=7&a=&p=&pl=Calculate+Rectangle']rectangle calculator[/URL], we get perimeter P: P = [B]34[/B]

A rectangular room is 3 times as long as it is wide, and its perimeter is 56 meters
A rectangular room is 3 times as long as it is wide, and its perimeter is 56 meters Given l = length and w = width, The perimeter of a rectangle is 2l + 2w, we have: [LIST=1] [*]l = 3w [*]2l + 2w = 56 [/LIST] Substitute equation (1) into equation (2) for l: 2(3w) + 2w = 56 6w + 2w = 56 To solve this equation for w, we [URL='https://www.mathcelebrity.com/1unk.php?num=6w%2B2w%3D56&pl=Solve']type it in our math engine[/URL] and we get: w = [B]7 [/B] To solve for l, we substitute w = 7 into equation (1): l = 3(7) l = [B]21[/B]

A rectangular room is 3 times as long as it is wide, and its perimeter is 56 meters.
A rectangular room is 3 times as long as it is wide, and its perimeter is 56 meters. We're given the following: [LIST] [*]l = 3w [/LIST] We know the Perimeter (P) of a rectangle is: P = 2l + 2w Substituting l = 3w and P = 56 into this equation, we get: 2(3w) + 2w = 56 Multiplying through, we get: 6w + 2w = 56 (6 +2)w = 56 8w = 56 [URL='https://www.mathcelebrity.com/1unk.php?num=8w%3D56&pl=Solve']Typing this equation into our search engine[/URL], we get: [B]w = 7[/B] Substitute w = 7 into l = 3w, we get: l = 3(7) [B]l = 21[/B]

A rectangular room is 3 times as long as it is wide, and its perimeter is 56 meters. Find the dimens
A rectangular room is 3 times as long as it is wide, and its perimeter is 56 meters. Find the dimensions of the room. We're given two items: [LIST] [*]l = 3w [*]P = 56 [/LIST] We know the perimeter of a rectangle is: 2l + 2w = P We plug in the given values l = 3w and P = 56 to get: 2(3w) + 2w = 56 6w + 2w = 56 To solve for w, we [URL='https://www.mathcelebrity.com/1unk.php?num=6w%2B2w%3D56&pl=Solve']plug this equation into our search engine[/URL] and we get: w = [B]7 [/B] To solve for l, we plug in w = 7 that we just found into the given equation l = 3w: l = 3(7) l = [B]21 [/B] So our dimensions length (l) and width (w) are: (l, w) = [B](21, 7)[/B]

A rectangular room is 3 times as long as it is wide, and its perimeter is 56 meters. Find the dimens
A rectangular room is 3 times as long as it is wide, and its perimeter is 56 meters. Find the dimension of the room. We're given: l = 3w The Perimeter (P) of a rectangle is: P = 2l + 2w With P = 56, we have: [LIST=1] [*]l = 3w [*]2l + 2w = 56 [/LIST] Substitute equation (1) into equation (2) for l: 2(3w) + 2w = 56 6w + 2w = 56 To solve this equation for w, we [URL='https://www.mathcelebrity.com/1unk.php?num=6w%2B2w%3D56&pl=Solve']type it in our search engine[/URL] and we get: w = [B]7 [/B] Now we plug w = 7 into equation (1) above to solve for l: l = 3(7) l = [B]21[/B]

A rectangular room is 3 times as long as it is wide, and its perimeter is 64 meters. Find the dimens
A rectangular room is 3 times as long as it is wide, and its perimeter is 64 meters. Find the dimension of the room. We're given: [LIST] [*]l = 3w [*]P = 64 [/LIST] We also know the perimeter of a rectangle is: 2l + 2w = P We plugin l = 3w and P = 64 into the perimeter equation: 2(3w) + 2w = 64 Multiply through to remove the parentheses: 6w + 2w = 64 To solve this equation for w, we type it in our search engine and we get: [B]w = 8[/B] To solve for l, we plug w = 8 into the l = 3w equation above: l = 3(8) [B]l = 24[/B]

A rectangular room is 4 times as long as it is wide, and its perimeter is 80 meters. Find the dimens
A rectangular room is 4 times as long as it is wide, and its perimeter is 80 meters. Find the dimension of the room The perimeter of a rectangle is P = 2l + 2w. We're given two equations: [LIST=1] [*]l = 4w [*]2l + 2w = 80. <-- Since perimeter is 80 [/LIST] Plug equation (1) into equation (2) for l: 2(4w) + 2w = 80 8w + 2w = 80 [URL='https://www.mathcelebrity.com/1unk.php?num=8w%2B2w%3D80&pl=Solve']Plugging this equation into our search engine[/URL], we get: w = [B]10[/B] To get l, we plug w = 10 into equation (1): l = 4(10) l = [B]40[/B]

A rental truck costs $49.95+$0.59 per mile and another costs $39.95 plus $0.99, set up an equation t
A rental truck costs $49.95+$0.59 per mile and another costs $39.95 plus $0.99, set up an equation to determine the break even point? Set up the cost functions for Rental Truck 1 (R1) and Rental Truck 2 (R2) where m is the number of miles R1(m) = 0.59m + 49.95 R2(m) = 0.99m + 39.95 Break even is when we set the cost functions equal to one another: 0.59m + 49.95 = 0.99m + 39.95 [URL='https://www.mathcelebrity.com/1unk.php?num=0.59m%2B49.95%3D0.99m%2B39.95&pl=Solve']Typing this equation into the search engine[/URL], we get [B]m = 25[/B].

A repair bill for a car is $648.45. The parts cost $265.95. The labor cost is $85 per hour. Write an
A repair bill for a car is $648.45. The parts cost $265.95. The labor cost is $85 per hour. Write and solve an equation to find the number of hours spent repairing the car. Let h be the number of hours spent repairing the car. We set up the cost function C(h): C(h) = Labor Cost per hour * h + Parts Cost We're given C(h) = 648.85, parts cost = 265.95, and labor cost per hour of 85, so we have: 85h + 265.95 = 648.85 To solve this equation, we [URL='https://www.mathcelebrity.com/1unk.php?num=85h%2B265.95%3D648.85&pl=Solve']type this into our search engine[/URL] and we get: h = [B]4.5[/B]

A repair bill for your car is $553. The parts cost $265. The labor cost is $48 per hour. Write and s
A repair bill for your car is $553. The parts cost $265. The labor cost is $48 per hour. Write and solve an equation to find the number of hours of labor spent repairing the car Set up the cost equation C(h) where h is the number of labor hours: C(h) = Labor Cost per hour * h + Parts Cost We're given C(h) = 553, Parts Cost = 265, and Labor Cost per Hour = 48. So we plug these in: 48h + 265 = 553 To solve this equation for h, we [URL='https://www.mathcelebrity.com/1unk.php?num=48h%2B265%3D553&pl=Solve']type it in our math engine[/URL] and we get: h = [B]6 hours[/B]

A repair will require 3 hours at $40 per hour. How much will the total/labor cost be for this job?
A repair will require 3 hours at $40 per hour. How much will the total/labor cost be for this job? Total cost = Hourly Labor Rate * hours Total cost = $40 * 3 Totaal cost = [B]$120[/B]

a repairman charged $93.06. The price included 2 hours of labor and a $40 service charge. How much d
a repairman charged $93.06. The price included 2 hours of labor and a $40 service charge. How much does the repairman charge per hour? Subtract the service charge: 93.06 - 40 = 53.06 53.06/2 hours = [B]$26.53 per hour[/B].

A researcher believed that there was a difference in the amount of time boys and girls at 7th grade
A researcher believed that there was a difference in the amount of time boys and girls at 7th grade studied by using a two-tailed t test. Which of the following is the null hypothesis? a. Mean of hours that boys studied per day was equal to mean of hours that girls studied per day b. Mean of hours that boys studied per day was greater than mean of hours that girls studied per day c. Mean of hours that boys studied per day was smaller than mean of hours that girls studied per day d. Mean of hours that boys studied per day was smaller than or equal to mean of hours that girls studied per day [B]a. Mean of hours that boys studied per day was equal to mean of hours that girls studied per day[/B] Reason is that in hypothesis testing, you take a position other than what is assumed or what is being tested as the null hypothesis

A researcher posed a null hypothesis that there was no significant difference between boys and girls
A researcher posed a null hypothesis that there was no significant difference between boys and girls on a standard memory test. He randomly sampled 100 girls and 120 boys in a community and gave them the standard memory test. The mean score for girls was 70 and the standard deviation of mean was 5.0. The mean score for boys was 65 and the standard deviation of mean was 6.0. What's the absolute value of the difference between means? |70 -65| = |5| = 5

A researcher posed a null hypothesis that there was no significant difference between boys and girls
A researcher posed a null hypothesis that there was no significant difference between boys and girls on a standard memory test. He randomly sampled 100 girls and 100 boys in a community and gave them the standard memory test. The mean score for girls was 70 and the standard deviation of mean was 5.0. The mean score for boys was 65 and the standard deviation of mean was 5.0. What is the standard error of the difference in means? [B]0.707106781187[/B] using our [URL='http://www.mathcelebrity.com/meandiffconf.php?n1=+100&xbar1=70&stdev1=5&n2=+100&xbar2=65&stdev2=5&conf=+99&pl=Hypothesis+Test']difference of means calculator[/URL]

A researcher posed a null hypothesis that there was no significant difference between boys and girls
A researcher posed a null hypothesis that there was no significant difference between boys and girls on a standard memory test. He randomly sampled 100 girls and 100 boys in a community and gave them the standard memory test. The mean score for girls was 70 and the standard deviation of mean was 5.0. The mean score for boys was 65 and the standard deviation of mean was 5.0. What's the t-value (two-tailed) for the null hypothesis that boys and girls have the same test scores? t = 7.07106781187 using our [URL='http://www.mathcelebrity.com/meandiffconf.php?n1=+100&xbar1=70&stdev1=5&n2=+100&xbar2=65&stdev2=5&conf=+99&pl=Hypothesis+Test']difference of means calculator[/URL]

A restaurant bill comes to $28.35. Find the total cost if the tax is 6.25% and a 20% tip is left on
A restaurant bill comes to $28.35. Find the total cost if the tax is 6.25% and a 20% tip is left on the amount before tax. Calculate the tip: Tip = Bill Before Tax * Tip % Tip = $28.35 * 20% Tip = $5.67 Calculate Tax: Tax = Bill without Tip * Tax % Tax = $28.35 * 6.25% Tax = $1.77 Total Cost = Bill + Tax + Tip Total Cost = $28.35 + $5.67 + $1.77 Total Cost = [B]$35.79[/B]

A restaurant chain sells 15,000 pizzas each month, and 70% of the pizzas are topped with pepperoni.
A restaurant chain sells 15,000 pizzas each month, and 70% of the pizzas are topped with pepperoni. Of these, 2/3 also have peppers. How many pizzas have pepperoni and peppers? We multiply the pizzas sold by the percentage of pepperoni times the fraction of peppers. Since 70% is 7/10, we have: Pizzas with pepperoni and peppers = 15,000 * 7/10 * 2/3 7/10 * 2/3 = 14/30. [URL='https://www.mathcelebrity.com/fraction.php?frac1=14%2F30&frac2=3%2F8&pl=Simplify']Using our fraction simplifier calculator[/URL], we can reduce this to 7/15 Pizzas with pepperoni and peppers = 15,000 * 7/15 Pizzas with pepperoni and peppers = [B]7,000[/B]

A restaurant earns $1073 on Friday and $1108 on Saturday. Write and solve an equation to find the am
A restaurant earns $1073 on Friday and $1108 on Saturday. Write and solve an equation to find the amount x (in dollars) the restaurant needs to earn on Sunday to average $1000 per day over the three-day period. Let Sunday's earnings be s. With 3 days, we divide our sum of earnings for 3 days by 3 to get our 1,000 average, so we have: (1073 + 1108 + s)/3 = 1000 Cross multiply: 1073 + 1108 + s = 1000 * 3 1073 + 1108 + s = 3000 To solve for s, we [URL='https://www.mathcelebrity.com/1unk.php?num=1073%2B1108%2Bs%3D3000&pl=Solve']type this equation into our search engine[/URL] and we get: s = [B]819[/B]

A restaurant has 8 pizza toppings to choose from. How many different 2 topping pizzas are possible?
A restaurant has 8 pizza toppings to choose from. How many different 2 topping pizzas are possible? We want 8 combinations of 2, denoted as 8 C 2, or 8 choose 2. [URL='https://www.mathcelebrity.com/permutation.php?num=8&den=2&pl=Combinations']Typing 8 C 2 into the search engine[/URL], we get [B]28[/B] different 2 topping pizzas

A restaurant is going to raise all their prices by 5%. If the current price of an item is p dollars,
A restaurant is going to raise all their prices by 5%. If the current price of an item is p dollars, write an expression for the price after the increase. 5% = 0.05 as a decimal. New price = Old Price * (1 + decimal increase) New price = p * (1 + 0.05) New price = [B]1.05p[/B]

A restaurant is going to raise all their prices by 7%. If the current price of an item is p dollars
A restaurant is going to raise all their prices by 7%. If the current price of an item is p dollars, write an expression for the price after the increase. A 7% increase on price means we multiply the current price of p by 1.07. So our algebraic expression is: [B]1.07p[/B]

A restaurant is open for 10 ½ hours during the day. The restaurant has 5 ½ families coming and leavi
A restaurant is open for 10 ½ hours during the day. The restaurant has 5 ½ families coming and leaving every hour. A family has 4 members. How many people have visited the restaurant during the day? [U]Given:[/U] [LIST] [*]10 & 1/2 = 10.5 hours [*]5 & 1/2 = 5.5 families [/LIST] Total Visitors = Hours Open * Families per hour * member per family Total Visitors = 10.5 * 5.5 * 4 Total Visitors = [B]231[/B]

A restaurant offers 20 appetizers and 40 main courses, how many ways can a person order a two course
A restaurant offers 20 appetizers and 40 main courses, how many ways can a person order a two course meal? Using the fundamental rule of counting, we can have: 20 appetizers * 40 main courses = [B]800 possible two-course meals[/B]

A restaurant offers a special pizza with any 5 toppings. If the restaurant has 17 topping from which
A restaurant offers a special pizza with any 5 toppings. If the restaurant has 17 topping from which to choose, how many different special pizzas are possible? We have 17 choose 5, or 17C5. [URL='https://www.mathcelebrity.com/permutation.php?num=17&den=5&pl=Combinations']Type this into the search engine[/URL], and we get [B]6,188[/B] different special pizzas available.

A restaurant offers the following options: Starter – soup or salad Main – chicken, fish or vegetar
A restaurant offers the following options: [LIST] [*]Starter – soup or salad [*]Main – chicken, fish or vegetarian [*]Dessert – ice cream or cake [/LIST] How many possible different combinations of starter, main and dessert are there? Using the fundamental rule of counting, we have: 2 starters * 3 main courses * 2 desserts = [B]12 different combinations [MEDIA=youtube]-N9j7FQ8Le4[/MEDIA][/B]

A retired couple invested $8000 in bonds. At the end of one year, they received an interest payment
A retired couple invested $8000 in bonds. At the end of one year, they received an interest payment of $584. What was the simple interest rate of the bonds? For simple interest, we have: Balance * interest rate = Interest payment 8000i = 584 Divide each side of the equation by 8000 to isolate i: 8000i/8000 = 584/8000 Cancelling the 8000's on the left side, we get: i = 0.073 Most times, interest rates are expressed as a percentage. Percentage interest = Decimal interest * 100% Percentage interest = 0.073 * 100% Multiplying by 100 is the same as moving the decimal point two places right: Percentage interest = [B]7.3%[/B]

A revenue function is R(x) = 22x and a cost function is C(x) = -9x + 341. The break-even point is
A revenue function is R(x) = 22x and a cost function is C(x) = -9x + 341. The break-even point is Break even is when C(x) = R(x). So we set them equal and solve for x: -9x + 341 = 22x Typing[URL='https://www.mathcelebrity.com/1unk.php?num=-9x%2B341%3D22x&pl=Solve'] this equation into our search engine[/URL], we get: x = [B]11[/B]

A right triangle has legs of 9 feet and 12 feet. How long is the hypotenuse?
A right triangle has legs of 9 feet and 12 feet. How long is the hypotenuse? A common right triangle ratio is 3:4:5 9 = 3 * 3 12 = 3 * 4 3 * 5 = 15, so we have [B]15 feet[/B]

A river is rising at a rate of 3 inches per hour. If the river rises more than 2 feet, it will excee
A river is rising at a rate of 3 inches per hour. If the river rises more than 2 feet, it will exceed flood stage. How long can the river rise at this rate without exceeding flood stage? Let the number of inches be i. Remember 12 inches to a foot, so we have 2 feet = 12*2 = 24 inches. [LIST] [*]Inequality: 3i <= 24. (since more than means the river can go [U]up to[/U] 2 feet or 24 inches [/LIST] To solve the inequality for I, we [URL='https://www.mathcelebrity.com/interval-notation-calculator.php?num=3i%3C%3D24&pl=Show+Interval+Notation']type it in our search engine[/URL] and we get: [B]I <= 8 This means after 8 hours, the river will flood[/B]

A road construction team built a 114 mile road over a period of 19 months what was their average bui
A road construction team built a 114 mile road over a period of 19 months what was their average building distance per a month Average building distance = miles built / months of building Average building distance = 114/19 Average building distance = [B]6 miles per month[/B]

a rocket is propelled into the air. its path can be modelled by the relation h = -5t^2 + 50t + 55, w
a rocket is propelled into the air. its path can be modeled by the relation h = -5t^2 + 50t + 55, where t is the time in seconds, and h is height in metres. when does the rocket hit the ground We set h = 0: -5t^2 + 50t + 55 = 0 Typing this quadratic equation into our search engine to solve for t, we get: t = {-1, 11} Time can't be negative, so we have: t = [B]11[/B]

A rocket travels at a rate of 160 meters in 3 seconds. What is the speed of the rocket in meters per
A rocket travels at a rate of 160 meters in 3 seconds. What is the speed of the rocket in meters per second? 160 meters /3 seconds = [B]53.333333333 meters per second[/B]

A roller coaster begins at 90 feet above ground level. Then it descends 105 feet. Find the height of
A roller coaster begins at 90 feet above ground level. Then it descends 105 feet. Find the height of the coaster after the first descent. 90 feet above and then we descend 105 feet, meaning we subtract: 90 - 105 = -15. We read this [B]15 feet below ground level[/B]

A roller coaster begins at 90 feet above ground level. Then it descends 105 feet. Find the height of
A roller coaster begins at 90 feet above ground level. Then it descends 105 feet. Find the height of the roller coaster after the first descent. 90 feet above ground level is written as +90 Descending 105 feet means we subtract 105 feet to get: +90 - 105 = [B]-15 or 15 feet below ground level[/B]

A roller coaster car carriers 32 people every 10 minutes there are 572 people in line in front of Ru
A roller coaster car carriers 32 people every 10 minutes there are 572 people in line in front of Ruben how long will it take for Ruben to ride the roller coaster 527/32 = 17.875 Which means on the 18th ride, Ruben will get a seat. 18 rides * 10 minutes per ride = [B]180 minutes, or 3 hours.[/B]

a roller coaster has 6 trains. each train has 3 cars, and each car seats 4 people. write and simplif
a roller coaster has 6 trains. each train has 3 cars, and each car seats 4 people. write and simplify an expression including units to find the total number of people that can ride the roller coaster at one time 6 trains * 3 cars per train * 4 people per car = [B]72 people[/B]

A roof drops 4 feet for every 12 feet forward. Determine the slope of the roof.
A roof drops 4 feet for every 12 feet forward. Determine the slope of the roof. Slope = Rise or Drop / Run Slope = 4/12 We can simplify this fraction. We [URL='https://www.mathcelebrity.com/fraction.php?frac1=4%2F12&frac2=3%2F8&pl=Simplify']type 4/12 into our search engine[/URL] and get: Slope. = [B]1/3[/B]

A room is 15 ft long and 12 feet wide. What are the length and width of the room in yards?
A room is 15 ft long and 12 feet wide. What are the length and width of the room in yards? Since 3 feet = 1 yard, we have: Length in yards = Length in feet / 3 Length in yards = 15/3 Length in yards = 5 Width in yards = Width in feet / 3 Width in yards = 12/3 Width in yards = 4

A salary after a 4.6% increase, of the original salary is x dollars
A salary after a 4.6% increase, of the original salary is x dollars 4.6% is also written as 0.046. Our formula for the new salary S is: S = (1 + 0.046)x [B]S = 1.046x[/B]

A sales clerk receives a monthly salary of $750 plus a commission of 4% on all sales over $3900. Wha
A sales clerk receives a monthly salary of $750 plus a commission of 4% on all sales over $3900. What did the clerk earn the month that she sold $12,800 in merchandise? [U]Calculate Commission Sales Eligible Amount:[/U] Commission Sales Eligible Amount = Sales - 3900 Commission Sales Eligible Amount = 12800 - 3900 Commission Sales Eligible Amount = 8900 [U]Calculate Commission Amount:[/U] Commission Amount = Commission Sales Eligible Amount * Commission Percent Commission Amount = [URL='https://www.mathcelebrity.com/perc.php?num=+5&den=+8&num1=+16&pct1=+80&pct2=4&den1=8900&pcheck=3&pct=+82&decimal=+65.236&astart=+12&aend=+20&wp1=20&wp2=30&pl=Calculate']8900 * 4%[/URL] Commission Amount = 356 [U]Calculate total earnings:[/U] Total Earnings = Base Salary + Commission Amount Total Earnings = 750 + 356 Total Earnings = [B]1106[/B]

A Sales Manager buys antacid in bottles by the gross. If he goes through 3 bottles of antacid everyd
A Sales Manager buys antacid in bottles by the gross. If he goes through 3 bottles of antacid everyday, how long will the gross last? [U][B]Calculate a gross[/B][/U] [URL='https://en.wikipedia.org/wiki/Gross_(unit)#:~:text=A%20gross%20refers%20to%20a,cubic%20dozen%2C%20123).']A gross equals[/URL] 144 because 1 gross = 12 dozen 12 dozen * 12 items/dozen = 144 [B] [U]Answer[/U][/B] Days lasted = Total Bottles / Bottles per day Days lasted = 144 bottles / 3 days [B]Days lasted = 48 days[/B]

A salesperson drove 9 hours. How long will he have driven t hours later?
Set up a function where t is the number of hours driven, and f(t) is the distance driven after t hours: [B]f(t) = 9t[/B]

A salesperson earns a commission of $364 for selling $2600 in merchandise. Find the commission rate.
A salesperson earns a commission of $364 for selling $2600 in merchandise. Find the commission rate. Write your answer as a percentage. Commission percentage = Commission Amount / Sales Price Commission percentage = 364 / 2600 Commission percentage = 0.14 Multiply by 100 to get the percentage: 0.14 * 100 = [B]14%[/B]

A salesperson receives a base salary of $300 per week and a commission of 15% on all sales over $5,0
A salesperson receives a base salary of $300 per week and a commission of 15% on all sales over $5,000. If x represents the salesperson’s weekly sales, express the total weekly earnings E(x) as a function of x and simplify the expression. Then find E(2,000) and E(7,000) and E(10,000). 15% as a decimal is written as 0.15. Build our weekly earnings function E(x) = Commission + Base Salary E(x) = 0.15(Max(0, x - 5000)) + 300 Now find the sales salary for 2,000, 7,000, and 10,000 in sales E(2,000) = 0.15(Max(0,2000 - 5000)) + 300 E(2,000) = 0.15(Max(0,-3000)) + 300 E(2,000) = 0.15(0) + 300 [B]E(2,000) = 300 [/B] E(7,000) = 0.15(Max(0,7000 - 5000)) + 300 E(7,000) = 0.15(Max(0,2000)) + 300 E(7,000) = 0.15(2,000) + 300 E(7,000) = 300 + 300 [B][B]E(7,000) = 600[/B][/B] E(10,000) = 0.15(Max(0,10000 - 5000)) + 300 E(10,000) = 0.15(Max(0,5000)) + 300 E(10,000) = 0.15(5,000) + 300 E(10,000) = 750+ 300 [B][B]E(10,000) = 1,050[/B][/B]

A salesperson works 40 hours per week at a job where he has two options for being paid. Option A is
A salesperson works 40 hours per week at a job where he has two options for being paid. Option A is an hourly wage of $24. Option B is a commission rate of 4% on weekly sales. How much does he need to sell this week to earn the same amount with the two options? Option A payment function: 24h With a 40 hour week, we have: 24 * 40 = 960 Option B payment function with sales amount (s): 0.04s We want to know the amount of sales (s) where Option A at 40 hours = Option B. So we set both equal to each other: 0.04s = 960 To solve this equation for s, we [URL='https://www.mathcelebrity.com/1unk.php?num=0.04s%3D960&pl=Solve']type it in our math engine[/URL] and we get: s = [B]24,000[/B]

A sample of 71 college students yields a mean annual income of $3595. Assuming that ? = $898, find t
A sample of 71 college students yields a mean annual income of $3595. Assuming that ? = $898, find the margin of error in estimating µ at the 99% level of confidence

A sample of 71 college students yields a mean annual income of $3595. Assuming that ? = $898, find t
A sample of 71 college students yields a mean annual income of $3595. Assuming that ? = $898, find the margin of error in estimating µ at the 99% level of confidence

A savings account earns 15% interest annually. What is the balance after 8 years in the savings acco
A savings account earns 15% interest annually. What is the balance after 8 years in the savings account when the initial deposit is 7500 Using our [URL='https://www.mathcelebrity.com/compoundint.php?bal=7500&nval=8&int=15&pl=Annually']compound interest with balance calculator,[/URL] we get a balance of: [B]22,942.67[/B]

A school bus can safely carry as many as 76 students. If 19 students are already on the bus, how man
A school bus can safely carry as many as 76 students. If 19 students are already on the bus, how many more can board the bus? Remaining boarders = total capacity - students on the bus Remaining boarders = 76 - 19 Remaining boarders = [B]57[/B]

A school conducts 27 tests in 36 weeks. Assume the school conducts tests at a constant rate. What is
A school conducts 27 tests in 36 weeks. Assume the school conducts tests at a constant rate. What is the slope of the line that represents the number of tests on the y-axis and the time in weeks on the x-axis? Slope is y/x,so we have 27/36. [URL='https://www.mathcelebrity.com/fraction.php?frac1=27%2F36&frac2=3%2F8&pl=Simplify']Using our fraction simplifier[/URL], we can reduce 27/36 to 3/4. So this is our slope. [B]3/4[/B]

A school conducts 27 tests in 36 weeks. Assume the school conducts tests at a constant rate. What is
A school conducts 27 tests in 36 weeks. Assume the school conducts tests at a constant rate. What is the slope of the line that represents the number of tests on the y-axis and the time in weeks on the x-axis? Slope = Rise/Run or y/x Since tests are on the y-axis and time is on the x-axis, we have: Slope = 27/36 We can simplify this, so we [URL='https://www.mathcelebrity.com/fraction.php?frac1=27%2F36&frac2=3%2F8&pl=Simplify']type in 27/36 into our search engine[/URL], and get: [B]Slope = 3/4[/B]

A school dance committee is to consist of 2 freshmen, 3 sophomores, 4 juniors, and 5 seniors. If 6 f
A school dance committee is to consist of 2 freshmen, 3 sophomores, 4 juniors, and 5 seniors. If 6 freshmen, 9 sophomores, 7 juniors, and 7 seniors are eligible to be on the committee, in how many ways can the committee be chosen? We want combinations for freshmen, sophomores, juniors, and seniors. [LIST] [*]Freshmen choices: [URL='https://www.mathcelebrity.com/permutation.php?num=6&den=2&pl=Combinations']6 C 2[/URL] = 15 [*]Sophomore choices: [URL='https://www.mathcelebrity.com/permutation.php?num=9&den=3&pl=Combinations']9 C 3[/URL] = 84 [*]Junior choices: [URL='https://www.mathcelebrity.com/permutation.php?num=7&den=4&pl=Combinations']7 C 4[/URL] = 35 [*]Senior choices: [URL='https://www.mathcelebrity.com/permutation.php?num=7&den=5&pl=Combinations']7 C 5 [/URL]= 21 [/LIST] The number of committees we can choose is the product of combinations of freshmen, sophomores, juniors, and seniors. Total Committees = Freshmen choices * Sophomore choices * Junior choices * Senior choices Total Committees = 15 * 84 * 35 * 21 Total Committees = [B]926,100[/B]

A school dance had 675 cookies each student took 3 cookies and there were 15 cookies leftover how ma
A school dance had 675 cookies each student took 3 cookies and there were 15 cookies leftover how many students attended the dance Let each student be s. We have: 3s + 15 = 675 To solve this equation for s, [URL='https://www.mathcelebrity.com/1unk.php?num=3s%2B15%3D675&pl=Solve']we type it in our search engine[/URL] and we get: s = [B]220[/B]

A school has a boy to girl ratio of 6:7. If there are 288 boys, how many girls are there?
A school has a boy to girl ratio of 6:7. If there are 288 boys, how many girls are there? 6 boys/7 girls = 288 boys/g girls Using our [URL='https://www.mathcelebrity.com/proportion-calculator.php?num1=6&num2=288&den1=7&den2=g&propsign=%3D&pl=Calculate+missing+proportion+value']proportion calculator[/URL], we get: g = [B]336 [MEDIA=youtube]P7x_u9oAkEc[/MEDIA][/B]

A school spent $150 on advertising for a breakfast fundraiser. Each plate of food was sold for $8.00
A school spent $150 on advertising for a breakfast fundraiser. Each plate of food was sold for $8.00 but cost the school $2.00 to prepare. After all expenses were paid, the school raised $2,400 at the fundraiser. Which equation can be used to find x, the number of plates that were sold? Set up the cost equation C(x) where x is the number of plates sold: C(x) = Cost per plate * x plates C(x) = 2x Set up the revenue equation R(x) where x is the number of plates sold: R(x) = Sales price per plate * x plates C(x) = 8x Set up the profit equation P(x) where x is the number of plates sold: P(x) = R(x) - C(x) P(x) = 8x - 2x P(x) = 6x We're told the profits P(x) for the fundraiser were $2,400, so we set 6x equal to 2400 and solve for x: 6x = 2400 To solve this equation for x, we [URL='https://www.mathcelebrity.com/1unk.php?num=6x%3D2400&pl=Solve']type it in our math engine[/URL] and we get: x =[B]400 plates[/B]

A school theater group is selling candy to raise funds in order to put on their next performance. Th
A school theater group is selling candy to raise funds in order to put on their next performance. The candy cost the group $0.20 per piece. Plus, there was a $9 shipping and handling fee. The group is going to sell the candy for $0.50 per piece. How many pieces of candy must the group sell in order to break even? [U]Set up the cost function C(p) where p is the number of pieces of candy.[/U] C(p) = Cost per piece * p + shipping and handling fee C(p) = 0.2p + 9 [U]Set up the Revenue function R(p) where p is the number of pieces of candy.[/U] R(p) = Sale price * p R(p) = 0.5p Break-even means zero profit or loss, so we set the Cost Function equal to the Revenue Function 0.2p + 9 = 0.5p To solve this equation for p, we [URL='https://www.mathcelebrity.com/1unk.php?num=0.2p%2B9%3D0.5p&pl=Solve']type it in our math engine[/URL] and we get: p = [B]30[/B]

A school wants to buy a chalkboard that measures 1 yard by 2 yards. The chalkboard costs $27.31 per
A school wants to buy a chalkboard that measures 1 yard by 2 yards. The chalkboard costs $27.31 per square yard. How much will the chalkboard cost? Area of a chalkboard is denoted as : A = lw Given 1 yard width and 2 years length of the chalkboard, we have: A = 2(1) A = 2 square yards Therefore, total cost is: Total Cost = $27.31 * square yards Total Cost = $27.31(2) Total Cost = [B]$54.62[/B]

A scuba diver is 30 feet below the surface of the water 10 seconds after he entered the water and 10
A scuba diver is 30 feet below the surface of the water 10 seconds after he entered the water and 100 feet below the surface after 40 seconds later. At what rate is the scuba diver going deeper down in the water If we take these as coordinates on a graph, where y is the depth and x is the time, we calculate our slope or rate of change where (x1, y1) = (10, 30) and (x2, y2) = (40, 100) Rate of change = (y2 - y1)/(x2 - x1) Rate of change = (100 - 30)/(40 - 10) Rate of change = 70/30 Rate of change =[B] 2.333 feet per second[/B]

A scuba diver swam 96ft under the sea and then went back up 34ft. What is the depth of the diver at
A scuba diver swam 96ft under the sea and then went back up 34ft. What is the depth of the diver at this point? 96 - 34 = [B]62 ft[/B]

A secret number is added to 6. The total is multiplied by 5 to get 50. What is the secret number?
A secret number is added to 6. The total is multiplied by 5 to get 50. What is the secret number? Take this algebraic expression in pieces: [LIST] [*]Let the secret number be n. [*]Added to means we add 6 to n: n + 6 [*]The total is multiplied by 5: 5(n + 6) [*]The phrase [I]to get[/I] means equal to, so we set 5(n + 6) equal to 50 [/LIST] 5(n + 6) = 50 To solve this equation for n, we type it in our search engine and we get: n = [B]4[/B]

A segment has an endpoint at (2, 1). The midpoint is at (5, 1). What are the coordinates of the othe
A segment has an endpoint at (2, 1). The midpoint is at (5, 1). What are the coordinates of the other endpoint? The other endpoint is (8,1) using our [URL='http://www.mathcelebrity.com/mptnline.php?ept1=2&empt=5&ept2=&pl=Calculate+missing+Number+Line+item']midpoint calculator.[/URL]

A Septon said that he has a collection of 1,000,000 stones in his house. How many stones is that in
A Septon said that he has a collection of 1,000,000 stones in his house. How many stones is that in base 10? The 1 is in decimal position 7, or 6th power. [URL='https://www.mathcelebrity.com/powersq.php?sqconst=+6&num=7%5E6&pl=Calculate']1 * 7^6[/URL] = [B]117,649[/B]

A Septonian said he had eaten .5 of his cookie while an American said he had eaten .5 of his cookie.
A Septonian said he had eaten .5 of his cookie while an American said he had eaten .5 of his cookie. Who ate more? Why? [LIST] [*]The Septon ate 5 groups of 1/7 or 5/7 [*]The American ate 5 groups of 1/10 or 5/10 which is 1/2 [/LIST] [URL='https://www.mathcelebrity.com/fraction.php?frac1=5/7&frac2=5/10&pl=Compare']5/7 is larger[/URL], so the Septoian ate more.

A Septonian said that his exact weight is 250.346 pounds (base 7). Translate this into base 10.
A Septonian said that his exact weight is 250.346 pounds (base 7). Translate this into base 10. Start the conversion: 2 * 7^2 + 5 * 7^1 + 0 * 7^0 + 3 * 1/7 + 4 * (1/7)^2 + 6 * (1/7)^3 2 * 49 + 5 * 7 + 0 * 1 + 3/7 + 4 * 1/49 + 6 * 1/343 98 + 35 + 3/7 + 4/49 + 6/343 [B]133 & 181/343[/B]

A Septonian won the lottery in the United States and won $1,000,000. How many dollars in that in bas
A Septonian won the lottery in the United States and won $1,000,000. How many dollars in that in base 7? Using our [URL='https://www.mathcelebrity.com/binary.php?num=1000000&check1=7&bchoice=7&pl=Convert']base change calculator[/URL], we get: 1,000,000 in base 7 = [B]11,333,311[/B]

A service charges a $1.95 flat rate plus $0.05 per mile . Jason only has $12 to spend on a a ride
A service charges a $1.95 flat rate plus $0.05 per mile. Jason only has $12 to spend on a a ride. Set up the cost equation C(m) where m is the number of miles: C(m) = 0.05m + 1.95 The problems asks for the number of miles (m) when C(m) = 12: 0.05m + 1.95 = 12 [URL='https://www.mathcelebrity.com/1unk.php?num=0.05m%2B1.95%3D12&pl=Solve']Typing this equation into our search engine[/URL], we get: m = [B]201[/B]

A set has a cardinality of 9. How many proper subsets does the set have?
A set has a cardinality of 9. How many proper subsets does the set have? The set has 2^9 = [B]512 proper subsets[/B]

A set of 19 scores has a mean of 6.3. A new score of 8 is then included in the data set. What is th
A set of 19 scores has a mean of 6.3. A new score of 8 is then included in the data set. What is the new mean? We know the mean formula is: Sum of scores / Number of Scores = Mean We're given mean = 6.3 and number of scores = 19, so we have: Sum of scores / 19 = 6.3 Cross multiply: Sum of scores = 19 * 6.3 Sum of scores = 119.7 Now a new score is added of 8, so we have: Sum of scores = 119.7 + 8 = 127.7 Number of scores = 19 + 1 = 20 So our new mean is: Mean = Sum of scores / Number of Scores Mean = 127.7/20 Mean = [B]6.385[/B] [COLOR=rgb(0, 0, 0)][SIZE=5][FONT=arial][B][/B][/FONT][/SIZE][/COLOR]

A set of 4 consecutive integers adds up to 314. What is the least of the 4 integers?
A set of 4 consecutive integers adds up to 314. What is the least of the 4 integers? First integer is x. The next 3 are x + 1, x + 2, and x + 3. Set up our equation: x + (x + 1) + (x + 2) + (x + 3) = 314 Group x terms and group constnats (x + x + x + x) + (1 + 2 + 3) = 314 Simplify and combine 4x + 6 = 314 [URL='http://www.mathcelebrity.com/1unk.php?num=4x%2B6%3D314&pl=Solve']Enter this in the equation solver[/URL] [B]x = 77[/B]

A set of 6 wooden chairs costs $444. A set of 8 metal chairs costs $720. How much more do the metal
A set of 6 wooden chairs costs $444. A set of 8 metal chairs costs $720. How much more do the metal chairs cost per chair? [U]Wooden Chair Unit Cost:[/U] Unit Cost = Total Cost / Quantity Unit Cost = 444/6 Unit Cost = 74 [B][/B] [U]Metal Chair Unit Cost:[/U] Unit Cost = Total Cost / Quantity Unit Cost = 720/8 Unit Cost = 90 [B][B][/B][/B] Find the difference (how much more) Difference = Metal Chair Unit Cost - Wooden Chair Unit Cost Difference = 90 - 74 Difference = [B]16[/B]

A set of data has a range of 30. The least value in the set of data is 22. What is the greatest valu
A set of data has a range of 30. The least value in the set of data is 22. What is the greatest value in the set of data? High Value - Low Value = Range Let the high value be h. We're given: h - 22 = 30 We [URL='https://www.mathcelebrity.com/1unk.php?num=h-22%3D30&pl=Solve']type this equation into our search engine[/URL] and we get: h = [B]52[/B]

A sewing class has 205 yards off a bric to make quilts. Each quilt requires 7 yards off a bric. How
A sewing class has 205 yards off a bric to make quilts. Each quilt requires 7 yards off a bric. How much will remain after all the quilts are made? Calculate the number of full quilts: 205/7 = 29.2857 so 29 full quilts. 29 * 7 = 203 205 - 203 = [B]2 yards remaining[/B]. You can also use the [URL='http://www.mathcelebrity.com/modulus.php?num=205mod7&pl=Calculate+Modulus']modulus calculator[/URL]

A ship is traveling at an average velocity of 28 miles per hour. How far will the ship travel in two
A ship is traveling at an average velocity of 28 miles per hour. How far will the ship travel in two days? 28 miles/1 hour * 24 hours/1 day * 2 days 28 * 24 * 2 = [B]1,344 miles[/B]

A shipping service charges $0.43 for the first ounce and $0.29 for each additional ounce of package
A shipping service charges $0.43 for the first ounce and $0.29 for each additional ounce of package weight. Write an equation to represent the price P of shipping a package that weighs x ounces, for any whole number of ounces greater than or equal to 1. Set up the price function P(x) [B]P(x) = 0.43 + 0.29(x - 1)[/B]

A shirt costs 15 and jeans costs 25. write an expressions for the costs of j jeans and s shirts
A shirt costs 15 and jeans costs 25. write an expressions for the costs of j jeans and s shirts. Cost equals quantity times price, so we have the total cost C: [B]C(s, j) = 15s + 25j[/B]

A shoe store was having a sale where 2 pairs of Brand A shoes cost $23.10 and 3 pairs of Brand B sho
A shoe store was having a sale where 2 pairs of Brand A shoes cost $23.10 and 3 pairs of Brand B shoes cost $35.85. Which brand is the better buy? [URL='https://www.mathcelebrity.com/betterbuy.php?p1=23.10&p2=35.85&q1=2&q2=3&pl=Better+Buy']Using our better buy calculator[/URL]: [SIZE=5][B]Calculate Unit Price[/B][/SIZE] Unit Price = Price/Quantity [SIZE=5][B]Calculate Unit Price 1:[/B][/SIZE] Unit Price Brand A = P1/Q1 Unit Price Brand A = 23.10/2 Unit Price Brand A = 11.55 [SIZE=5][B]Calculate Unit Price 2:[/B][/SIZE] Unit Price Brand B = P2/Q2 Unit Price Brand B = 35.85/3 Unit Price Brand B = 11.95 Since Brand A's Unit price is lower, [B]Brand A is the better buy [MEDIA=youtube]Q16iZn6Uer8[/MEDIA][/B]

a shop has a sale of 1/5 off all items in stock. if the original price of a dress is £45, what would
a shop has a sale of 1/5 off all items in stock. if the original price of a dress is £45, what would be its sale price? [URL='https://www.mathcelebrity.com/fraction.php?frac1=45&frac2=1/5&pl=Multiply']1/5 of 45[/URL] = 9 45 - 9 = [B]36[/B]

A shopkeeper buys a box of 20 cans of cola for $10. He sells the cans for 65 cents each. Work out hi
A shopkeeper buys a box of 20 cans of cola for $10. He sells the cans for 65 cents each. Work out his percentage profit. [U]Calculate Revenue[/U] Revenue = Sale price per can * number of cans Revenue = 0.65 * 20 Revenue = 13 [U]Calculate Profit given a cost of $10:[/U] Profit = Revenue - Cost Profit = 13 - 10 Profit = 3 [U]Calculate Percentage Profit:[/U] Percentage Profit = Profit/Revenue * 100% Percentage Profit = 3/13 * 100% Percentage Profit = 0.23076923076 * 100% Percentage Profit = [B]23.08%[/B]

A shopper paid $51.93 including tax for an item marked $48.99. What would she pay for another item m
A shopper paid $51.93 including tax for an item marked $48.99. What would she pay for another item marked $75? Set up a proportion, assuming identical tax rates: 51.93/48.99 = 75/x where x is the after-tax amount Using our [URL='http://www.mathcelebrity.com/prop.php?num1=51.93&num2=75&den1=48.99&den2=x&propsign=%3D&pl=Calculate+missing+proportion+value']proportion calculator[/URL], we get: [B]x = 70.75[/B]

A single card is drawn from a standard 52 card deck. What is the possibility that the card drawn is
A single card is drawn from a standard 52 card deck. What is the possibility that the card drawn is either a 4 or a 6 There are 4 (4's) and 4 (6's) in a standard 52 card deck. P(4 or 6) = P(4) + P(6) P(4 or 6) = 4/52 + 4/52 P(4 or 6) = 8/52 We can simplify this fraction by [URL='https://www.mathcelebrity.com/fraction.php?frac1=8%2F52&frac2=3%2F8&pl=Simplify']typing it in our search engine and choosing simplify[/URL]: P(4 or 6) = [B]2/13[/B]

A skier is trying to decide whether or not to buy a season ski pass. A daily pass costs $75. A seas
A skier is trying to decide whether or not to buy a season ski pass. A daily pass costs $75. A season ski pass costs $350. The skier would have to rent skis with either pass for $20 per day. How many days would the skier have to go skiing in order to make the season pass less expensive than the daily passes? Let d be the number of days: Daily Plan cost: 75d + 20d = 95d Season Pass: 350 + 20d We want to find d such that 350 + 20d < 95d Subtract 20d from each side 75d > 350 Divide each side by 75 d > 4.66667 [B]d = 5[/B]

A skydiver falls 144 feet in three seconds. How far does the skydiver fall per second?
A skydiver falls 144 feet in three seconds. How far does the skydiver fall per second? 144 feet/3 seconds Divide top and bottom by 3 to get feet per second [B]48 feet per second[/B]

A small theater had 6 rows of 24 chairs each. An extra 7 chairs have just been brought in. How many
A small theater had 6 rows of 24 chairs each. An extra 7 chairs have just been brought in. How many chairs are in the theater now? Calculate the chairs in the theater before the extra chairs were brought in: Chairs in theater (before) = Rows * Chairs per row Chairs in theater (before) = 6 * 24 Chairs in theater (before) = 144 To get the chairs in the theater after the extra, we have: Chairs in theater = Chairs in theater (before) + extra chairs Chairs in theater = 144 + 7 Chairs in theater = [B]151[/B]

A small theater had 9rows of 24 chairs each. An extra 8 chairs have just been brought in. How many c
A small theater had 9 rows of 24 chairs each. An extra 8 chairs have just been brought in. How many chairs are in the theater now? [LIST=1] [*]9 rows x 24 chairs per row = 216 chairs. [*]Add an extra 8 chairs, we get 216 + 8 = [B]224 chairs[/B] [/LIST]

A soccer team has picked its five best players to take part in penalty kicks to determine the winner
A soccer team has picked its five best players to take part in penalty kicks to determine the winner of a soccer match that is tied. Each of the five players will get one shot against the opposing team's goalie. The coach needs to decide the order in which the five players will take their shots. How many possible ways are there to arrange the five players? First shot, 5 players can take the shot. Next shot is 4, then 3, then 2, then 1 5! = 5 x 4 x 3 x 2 x 1 = [B]120 ways[/B]

A soccer team is buying T-shirts to sell as a fundraiser. The team pays a flat fee of $35 for a logo
A soccer team is buying T-shirts to sell as a fundraiser. The team pays a flat fee of $35 for a logo design plus $7.00 per T-shirt. Set up the cost function C(t) where t is the number of t-shirts: C(t) = Cost per t-shirt * number of t-shirts + Flat Fee [B]C(t) = 7t + 35[/B]

A soccer team lost 30% of their games and drew 15% and they played 67 games. How many games did they
A soccer team lost 30% of their games and drew 15% and they played 67 games. How many games did they win If they lost 30% and they drew (tied) 15%, then they won the following: Wins = 100% - Losses - Drew Wins = 100% - 30% - 15% Wins = 55% So we want 55% of 67. We [URL='https://www.mathcelebrity.com/perc.php?num=+5&den=+8&num1=+16&pct1=+80&pct2=55&den1=67&pcheck=3&pct=+82&decimal=+65.236&astart=+12&aend=+20&wp1=20&wp2=30&pl=Calculate']type this expression into our search engine[/URL] and we get: [B]36.85 ~ 37 games[/B]

A social networking site currently has 38,000 active members per month, but that figure is dropping
A social networking site currently has 38,000 active members per month, but that figure is dropping by 5% with every month that passes. How many active members can the site expect to have in 7 months? Setup an equation S(m) where m is the number of months that pass: S(m) = 38000 * (1 - 0.05)^t S(m) = 38000 * (0.95)^t The problem asks for S(7): S(7) = 38000 * (0.95)^7 S(7) = 38000 * (0.69833729609) S(7) = 26,536.82 We round down to a full person and get: S(7) = [B]26,536[/B]

A soda cost $100. What is the cost of y sodas?
A soda cost $100. What is the cost of y sodas? Total cost = price * quantity Total Cost = [B]100y[/B]

A soft drink costs $1.65, and each refill for the drink costs $0.95. If you have $4.50, how many ref
A soft drink costs $1.65, and each refill for the drink costs $0.95. If you have $4.50, how many refills can you purchase? Subtract the first drink: 4.50 - 1.65 = 2.85 Now, we need to find out how many refills we get for 2.85. 2.85/0.95 = [B]3 refills[/B]

A softball player had 13 hits in 25 times at bat. What percent of her times at bat resulted in hits?
We take the ratio of hits to at bats: 13/25 = 0.52 To get the percent, we multiply by 100: 100 * 0.52 = 52%

A software company, in 3 consecutive years, makes profits of -3 million dollars, 10 million dollars,
A software company, in 3 consecutive years, makes profits of -3 million dollars, 10 million dollars, and -2 million dollars. What was its profit over the 3 year period? Profit = -3,o00,000 + 10,000,000 - 2,000,000 Profit = [B]5,000,000[/B]

a son is 1/4 of his fathers age. the difference in their ages is 30. what is the fathers age.
a son is 1/4 of his fathers age. the difference in their ages is 30. what is the fathers age. Declare variables: [LIST] [*]Let f be the father's age [*]Let s be the son's age [/LIST] We're given two equations: [LIST=1] [*]s = f/4 [*]f - s = 30. [I]The reason why we subtract s from f is the father is older[/I] [/LIST] Using substitution, we substitute equaiton (1) into equation (2) for s: f - f/4 = 30 To remove the denominator/fraction, we multiply both sides of the equation by 4: 4f - 4f/4 = 30 *4 4f - f = 120 3f = 120 To solve for f, we divide each side of the equation by 3: 3f/3 = 120/3 Cancel the 3's on the left side and we get: f = [B]40[/B]

A spacecraft with a volume of 800 cubic feet is leaking air at a rate of .4 cubic feet every n minut
A spacecraft with a volume of 800 cubic feet is leaking air at a rate of .4 cubic feet every [I]n[/I] minutes. How many minutes until the spacecraft has no air? 800 cubic feet / .4 cubic feet every n minutes = 2000 (n minute parts) Total time = [B]2000n[/B]

A spherical water tank holds 11,500ft^3 of water. What is the diameter?
A spherical water tank holds 11,500ft^3 of water. What is the diameter? The tank holding amount is volume. And the volume of a sphere is: V = (4pir^3)/3 We know that radius is 1/2 of diameter: r =d/2 So we rewrite our volume function: V = 4/3(pi(d/2)^3) We're given V = 11,500 so we have: 4/3(pi(d/2)^3) = 11500 Multiply each side by 3/4 4/3(3/4)(pi(d/2)^3) = 11,500*3/4 Simplify: pi(d/2)^3 = 8625 Since pi = 3.1415926359, we divide each side by pi: (d/2)^3 = 8625/3.1415926359 (d/2)^3 = 2745.42 Take the cube root of each side: d/2 = 14.0224 Multiply through by 2: [B]d = 28.005[/B]

A spider has 8 legs. How many legs are there for k spiders?
A spider has 8 legs. How many legs are there for k spiders? We multiply legs per spider * spiders [B]8k[/B]

A spinner has 10 equally sized sections, 2 are gray 8 are blue. The spinner is spun twice. What is t
A spinner has 10 equally sized sections, 2 are gray 8 are blue. The spinner is spun twice. What is the probability that the first spin lands on blue and the second lands on gray? P(blue) = Blue sections / Total Sections P(blue) = 8/10 [URL='https://www.mathcelebrity.com/fraction.php?frac1=8%2F10&frac2=3%2F8&pl=Simplify']Reducing this using our simplified fraction calculator[/URL], we get: P(blue) = 4/5 P(gray) = Gray sections / Total Sections P(blue) = 2/10 [URL='https://www.mathcelebrity.com/fraction.php?frac1=2%2F10&frac2=3%2F8&pl=Simplify']Reducing this using our simplified fraction calculator[/URL], we get: P(gray) = 1/5 We want the probability of blue,gray. Since each spin is independent, we multiply the two probabilities to get our answer: P(blue, gray) = P(blue) * P(gray) P(blue, gray) = 4/5 * 1/5 P(blue, gray) = [B]4/25[/B]

A spinner has 3 equal sections labelled A, B, C. A bag contains 3 marbles: 1 grey, 1 black, and 1 w
A spinner has 3 equal sections labelled A, B, C. A bag contains 3 marbles: 1 grey, 1 black, and 1 white. The pointer is spun and a marble is picked at random. a) Use a tree diagram to list the possible outcomes. [LIST=1] [*][B]A, Grey[/B] [*][B]A, Black[/B] [*][B]A, White[/B] [*][B]B, Grey[/B] [*][B]B, Black[/B] [*][B]B, White[/B] [*][B]C, Grey[/B] [*][B]C, Black[/B] [*][B]C, White[/B] [/LIST] b) What is the probability of: i) spinning A? P(A) = Number of A sections on spinner / Total Sections P(A) = [B]1/3[/B] --------------------------------- ii) picking a grey marble? P(A) = Number of grey marbles / Total Marbles P(A) = [B]1/3[/B] --------------------------------- iii) spinning A and picking a white marble? Since they're independent events, we multiply to get: P(A AND White) = P(A) * P(White) P(A) was found in i) as 1/3 Find P(White): P(White) = Number of white marbles / Total Marbles P(White) = 1/3 [B][/B] Therefore, we have: P(A AND White) = 1/3 * 1/3 P(A AND White) = [B]1/9[/B] --------------------------------- iv) spinning C and picking a pink marble? Since they're independent events, we multiply to get: P(C AND Pink) = P(C) * P(Pink) Find P(C): P(C) = Number of C sections on spinner / Total Sections P(C) = 1/3 [B][/B] Find P(Pink): P(Pink) = Number of pink marbles / Total Marbles P(Pink) = 0/3 [B][/B] Therefore, we have: P(C AND Pink) = 1/3 * 0 P(C AND Pink) = [B]0[/B]

A spinner has 6 equal sections, of which 2 are green. If you spin the spinner once, what is the prob
A spinner has 6 equal sections, of which 2 are green. If you spin the spinner once, what is the probability that it will land on a green section? Write your answer as a fraction or whole number. P(green) = Total Green / Total spaces P(green) = 2/6 We can simplify this fraction. So we [URL='https://www.mathcelebrity.com/fraction.php?frac1=2%2F6&frac2=3%2F8&pl=Simplify']type 2/6 into our search engine[/URL], choose Simplify, and we get: P(green) = [B]1/3[/B]

A spinner is divided into 4 equal sections numbered 1 to 4. The theoretical probability of the spinn
A spinner is divided into 4 equal sections numbered 1 to 4. The theoretical probability of the spinner stopping on 3 is 25%. Which of the following is most likely the number of 3s spun in 10,000 spins? We want Expected Value of s spins. Set up the expected value formula for any number 1-4 E(s) = 0.25 * n where n is the number of spins. Using s = 3, n = 10,000, we have: E(10,000) = 0.25 * 10,000 E(10,000) = [B]2,500[/B]

A sports store near Big Bear Lake is having a 20% off sale on all water skis. What will the sale pri
A sports store near Big Bear Lake is having a 20% off sale on all water skis. What will the sale price be for water skis which regularly sell for $248? [U]Calculate Sale Price:[/U] Sale Price = Full Price * (1 - sale discount) Sale Price = 248 * (1 - 0.2) <-- since 20% is 0.2 Sale Price = 248 * (0.8) Sale Price = [B]198.40[/B]

A sports tournament has c teams. Each team has 17 players. Using c, write
A sports tournament has c teams. Each team has 17 players. Using c, write an expression for the total number of players in the tournament. Total Players = Total Teams * Players Per Team Total Players =[B] 17c[/B]

A sports tournament has d teams. Each team has 14 players. Using d, write an expression for the tota
A sports tournament has d teams. Each team has 14 players. Using d, write an expression for the total number of players in the tournament. Tournament Players = Players per team * Number of Teams Tournament Players = [B]14d[/B]

A sprinter runs 400 meters in 54 seconds. What is the runners average running rate in meters per sec
A sprinter runs 400 meters in 54 seconds. What is the runners average running rate in meters per second? 400 meters/54 seconds = [B]7.407 meters per second[/B].

A square has a perimeter of 24 inches. What is the area of the square?
A square has a perimeter of 24 inches. What is the area of the square? Perimeter of a square = 4s where s = the length of a side. Therefore, we have: 4s = P 4s = 24 Using our equation solver, [URL='https://www.mathcelebrity.com/1unk.php?num=4s%3D24&pl=Solve']we type in 4s = 24[/URL] and get: s = 6 The problems asks for area of a square. It's given by A = s^2 Plugging in s = 6, we get: A = 6^2 A = 6 * 6 A = [B]36 [/B] Now if you want a shortcut in the future, type in the shape and measurement you know. Such as: [I][URL='https://www.mathcelebrity.com/square.php?num=24&pl=Perimeter&type=perimeter&show_All=1']square perimeter = 24[/URL][/I] From the link, you'll learn every other measurement about the square.

A square of an integer is the integer. Find the integer.
A square of an integer is the integer. Find the integer. Let the integer be n. The square means we raise n to the power of 2, so we have: n^2 = n Subtract n from each side: n^2 - n = n - n n^2 - n = 0 Factoring this, we get: n(n - 1) = 0 So n is either [B]0 or 1[/B].

A stack of boards is 21 inches high. Each board is 1¾ inches thick. How many boards are there?
A stack of boards is 21 inches high. Each board is 1¾ inches thick. How many boards are there? We want 21 / 1 & 3/4 Using our [URL='https://www.mathcelebrity.com/fraction.php?frac1=21&frac2=1%263%2F4&pl=Divide']fraction operation calculator[/URL], we get: [B]12 boards[/B]

A stack of lumber is 8 feet wide, 5 feet high, and 2 feet long. Give the volume of the stack
A stack of lumber is 8 feet wide, 5 feet high, and 2 feet long. Give the volume of the stack The lumber stack is a rectangular solid. The Volume V is found from the length (l), width (w), and height (h) by: V = lwh Plugging in our given values, we get: V = 2 * 8 * 5 V = [B]80 cubic feet[/B]

A standard die is rolled. Find the probability that the number rolled is greater than 3
A standard die is rolled. Find the probability that the number rolled is greater than 3. Using our [URL='http://www.mathcelebrity.com/1dice.php?gl=2&pl=3&opdice=1&rolist=+2%2C3%2C4&dby=+2%2C3%2C5&montect=+100']dice calculator[/URL], the probability is [B]1/2 or 0.5[/B]

A standard volleyball court has an area of 1800ft. The length is 60. What is the width of the volley
A standard volleyball court has an area of 1800ft. The length is 60. What is the width of the volleyball court Plugging [URL='https://www.mathcelebrity.com/rectangle.php?l=60&w=&a=1800&p=&pl=Calculate+Rectangle']this into our rectangle calculator[/URL] and we get: w = [B]30[/B]

A state park charges a $6.00 entry fee plus $7.50 per night of camping. Write an algebraic expressio
A state park charges a $6.00 entry fee plus $7.50 per night of camping. Write an algebraic expression for the cost in dollars of entering the park and camping for n nights. The cost in dollars C is found below: [B]C = 7.50n + 6[/B]

A state park charges a $6.00 entry fee plus $7.50 per night of camping. Write an algebraic expressio
A state park charges a $6.00 entry fee plus $7.50 per night of camping. Write an algebraic expression for the cost in dollars of entering the park and camping for n nights. We write this as: cost per night of camping * n nights + entry fee [B]7.50n + 6[/B]

A stick that is ten feet tall casts a shadow of 12 feet. If a tree has a 96 foot shadow, how tall is
A stick that is ten feet tall casts a shadow of 12 feet. If a tree has a 96 foot shadow, how tall is the tree? Set up a proportion of wood height to shadow length where h is the height of the tree: 10/12 = h/96 Using our [URL='https://www.mathcelebrity.com/proportion-calculator.php?num1=10&num2=h&den1=12&den2=96&propsign=%3D&pl=Calculate+missing+proportion+value']proportion calculator[/URL], we get: h = [B]80 feet[/B]

a stone mason builds 7 houses in 3 days. How many days does it take to build 11 houses?
a stone mason builds 7 houses in 3 days. How many days does it take to build 11 houses? The build rate of houses per days is proportional. Set up a proportion of [I]houses to days[/I] where d is the number of days it takes to build 11 houses: 7/3 = 11/d Cross multiply: Numerator 1 * Denominator 2 = Denominator 1 * Numerator 2 7d = 11 * 3 7d = 33 Divide each side of the equation by 7: 7d/7 = 33/7 d = [B]4.7142857142857[/B]

A storage box has a volume of 56 cubic inches. The base of the box is 4 inches by 4 inches. What is
A storage box has a volume of 56 cubic inches. The base of the box is 4 inches by 4 inches. What is the height of the box? The volume of the box is l x w x h. We're given l and w = 4. So we want height: 56 = 4 x 4 x h 16h = 56 [URL='https://www.mathcelebrity.com/1unk.php?num=16h%3D56&pl=Solve']Type this equation into our search engine[/URL] and we get: h = [B]3.5[/B]

A store had 600 pounds of feed. After delivering equal amounts to 4 farmers, there are 60 pounds lef
A store had 600 pounds of feed. After delivering equal amounts to 4 farmers, there are 60 pounds left. How many pounds did each farmer receive? If there were 60 pounds left, then the store had 600 - 60 = 540 pounds delivered. 540 pounds delivered in equal amounts to 4 farmers means each farmer got: 540/4 = [B]135 pounds of feed[/B]

A store is offering a 11% discount on all items. Write an equation relating the final price
A store is offering a 11% discount on all items. Write an equation relating the final price 11% discount means we pay 100% - 11% = 89% of the full price. Since 89% as a decimal is 0.89. With a final price f and an original price p, we have: [B]F = 0.89p[/B]

A store is offering a 15% discount on all items. Write an equation relating the sale price S for an
A store is offering a 15% discount on all items. Write an equation relating the sale price S for an item to its list price L If we give a discount of 15%, then we pay 100% - 15% = 85% of the list price. 85% as a decimal is 0.85, So we have: L = 0.85S

A store is offering a 18% discount on all items. Write an equation relating the sale price S for an
A store is offering a 18% discount on all items. Write an equation relating the sale price S for an item to its list price L. 18% discount means we subtract 18% (0.18) as a decimal, from the 100% of the price: S = L(1 - 0.18) [B]S = 0.82L[/B]

A store manager must calculate the total number of winter hats available to sell in the store from a
A store manager must calculate the total number of winter hats available to sell in the store from a starting number of 293. In the past month, the store sold 43 blue hats, 96 black hats, 28 red hats, and 61 pink hats. The store received a shipment of 48 blue hats, 60 black hats, 18 red hats, and 24 pink hats. How many total hats does the store have for sale? [LIST=1] [*]We start with 293 hats [*]We calculate the hats sold: (43 + 96 + 28 + 61) = 228 [*]We subtract Step 2 from Step 1 to get remaining hats before the shipment: 293 - 228 = 65 [*]Now we calculate the number of hats received in the shipment: (48 + 60 + 18 + 24) = 150 [*]We add Step 4 to Step 3: 65 + 150 = [B]215 hats for sale[/B] [/LIST]

A store owner bought 240 cartons of eggs. The owner sold 5/8 of the eggs and set aside 5 cartons. Ho
A store owner bought 240 cartons of eggs. The owner sold 5/8 of the eggs and set aside 5 cartons. How many cartons of eggs did the owner have left to sale? If the owner sold 5/8 of the eggs, they have 1 - 5/8 left. 1 = 8/8, so we have 8/8 - 5/8 = 3/8 left 3/8 (240 cartons) = 90 cartons remaining The owner set aside 5 cartons. We're left with 90 cartons - 5 cartons = [B]85 cartons[/B]

a store sells a certain toaster oven for 35. The store offers a 30% discount and charges 8% sales ta
a store sells a certain toaster oven for 35. The store offers a 30% discount and charges 8% sales tax. How much will the toaster oven cost? [U]Calculate discounted price:[/U] Discounted Price = Full Price * (1 - Discount Percent) Since 30% = 0.3, we have Discounted Price = 35 * (1 - 0.3) Discounted Price = 35 * 0.7 Discounted Price = 24.5 Calculate after-tax amount: After-tax amount = Discounted Price * (1 + Tax Percent) Since 8% = 0.08, we have Discounted Price = 24.5 * (1 + 0.08) Discounted Price = 24.5 * 1.08 Discounted Price = [B]26.46[/B]

A store sells about $45 a day 7 days a week about how many gigs is my the stores sell in 4 weeks
A store sells about $45 a day 7 days a week about how many gigs is my the stores sell in 4 weeks 4 weeks = 7 * 4 = 28 days. $45 per day * 28 days = [B]$1,260[/B]

A store sells books for 50% off on Sundays. The store advertises that on Easter Sunday the store tak
A store sells books for 50% off on Sundays. The store advertises that on Easter Sunday the store takes an additional 25% off. What would a pile of books cost on Easter Sunday that normally sell for $100 on a Thursday? 50% off means we'd pay 100% - 50% = 50% An additional 25% off means we'd pay 100% - 25% = 75% Build this percentage paid stack below 100 * 50% * 75% = [B]37.50[/B]

A store sells small notebooks for $6 and large notebooks for $12. If a student buys 6 notebooks and
A store sells small notebooks for $6 and large notebooks for $12. If a student buys 6 notebooks and spends $60, how many of each did he buy? Let the amount of small notebooks be s. Let the amount of large notebooks be l. We're given two equations: [LIST=1] [*]l + s = 6 [*]12l + 6s = 60 [/LIST] Multiply equation (1) by -6 [LIST=1] [*]-6l - 6s = -36 [*]12l + 6s = 60 [/LIST] Now add equation 1 to equation 2: 12l - 6l + 6s - 6s = 60 - 36 Cancel the 6s terms, and we get: 6l = 24 To solve for l, we [URL='https://www.mathcelebrity.com/1unk.php?num=6l%3D24&pl=Solve']type this equation into our search engine[/URL] and we get: l = [B]4 [/B] Now substitute this into equation 1: 4 + s = 6 To solve for s, [URL='https://www.mathcelebrity.com/1unk.php?num=4%2Bs%3D6&pl=Solve']we type this equation into our search engine[/URL] and we get: s = [B]2[/B]

A straight line has the equation ax + by=23. The points (5,-2) and (1,-5) lie on the line. Find the
A straight line has the equation ax + by=23. The points (5,-2) and (1,-5) lie on the line. Find the values of a and b. plug in both points and form 2 equations: [LIST=1] [*]5a - 2b = 23 [*]1x - 5b = 23 [/LIST] We can solve this simultaneous equations any one of three ways: [LIST] [*][URL='https://www.mathcelebrity.com/simultaneous-equations.php?term1=5a+-+2b+%3D+23&term2=1a+-+5b+%3D+23&pl=Substitution']Substitution Method[/URL] [*][URL='https://www.mathcelebrity.com/simultaneous-equations.php?term1=5a+-+2b+%3D+23&term2=1a+-+5b+%3D+23&pl=Elimination']Elimination Method[/URL] [*][URL='https://www.mathcelebrity.com/simultaneous-equations.php?term1=5a+-+2b+%3D+23&term2=1a+-+5b+%3D+23&pl=Cramers+Method']Cramer's Rule[/URL] [/LIST] No matter which method we choose, we get the same answer: [LIST] [*][B]a = 3[/B] [*][B]b = -4[/B] [/LIST]

A straight road to the top of a hill is 2500 feet long and makes an angle of 12 degrees with the hor
A straight road to the top of a hill is 2500 feet long and makes an angle of 12 degrees with the horizontal. Find the height of the hill. Height = Distance * Sin(Horizon Angle) Height = 2500 * [URL='http://www.mathcelebrity.com/anglebasic.php?entry=12&coff=&pl=sin']Sin(12)[/URL] Height = 2500 * 0.207911691 Height = [B]519.78 feet[/B]

a string measures 20 inches is cut into pieces. Let z represent the length of one of the resulting p
a string measures 20 inches is cut into pieces. Let z represent the length of one of the resulting pieces. express length of the second piece in terms of the length z of the first pice Second piece length = [B]20 - z[/B]

A student and the marine biologist are together at t = 0. The student ascends more slowly than the m
A student and the marine biologist are together at t = 0. The student ascends more slowly than the marine biologist. Write an equation of a function that could represent the student's ascent. Please keep in mind the slope for the marine biologist is 12. Slope means rise over run. In this case, rise is the ascent distance and run is the time. 12 = 12/1, so for each second of time, the marine biologist ascends 12 units of distance If the student ascends slower, than the total distance gets reduced by an unknown factor, let's call it c. So we have the student's ascent function as: [B]y(t) = 12t - c[/B]

a student has $50 in saving and earns $40 per week. How long would it take them to save $450
a student has $50 in saving and earns $40 per week. How long would it take them to save $450 Set up the savings function S(w), where w is the number of weeks. The balance, S(w) is: S(w) = Savings Per week * w + Initial Savings S(w) = 40w + 50 The problems asks for how many weeks for S(w) = 450. So we have; 40w + 50 = 450 To solve for w, we[URL='https://www.mathcelebrity.com/1unk.php?num=40w%2B50%3D450&pl=Solve'] type this equation in our search engine[/URL] and we get: w = [B]10[/B]

A student has an average mark of 68 from 10 tests. What mark must be gained in the next test to rais
A student has an average mark of 68 from 10 tests. What mark must be gained in the next test to raise their average to 70? This is a missing average problem. We use our [URL='http://www.mathcelebrity.com/missingaverage.php?num=68%2C68%2C68%2C68%2C68%2C68%2C68%2C68%2C68%2C68&avg=70&pl=Calculate+Missing+Score']missing average calculator[/URL]. The student's next score must be a [B]90[/B].

A student hypothesized that girls in his class had the same blood pressure levels as boys. The proba
A student hypothesized that girls in his class had the same blood pressure levels as boys. The probability value for his null hypothesis was 0.15. So he concluded that the blood pressures of the girls were higher than boys'. Which kind of mistake did he make? a. Type I error b. Type II error c. Type I and Type II error d. He did not make any mistake [B]d. He did not make any mistake[/B] [I]p value is high, especially using a significance level of 0.05[/I]

A student posed a null hypothesis that during the month of September, the mean daily temperature of
A student posed a null hypothesis that during the month of September, the mean daily temperature of Boston was the same as the mean daily temperature of New York. His alternative hypothesis was that mean temperatures in these two cities were different. He found the P value of his null hypothesis was 0.56. Thus, he could conclude: a. In September, Boston was colder than New York b. In September, Boston was warmer than New York c. He may reject the null hypothesis d. He failed to reject the null hypothesis [B]d. He failed to reject the null hypothesis[/B] [I]Higher p value tells us we cannot reject the null hypothesis[/I]

A student walks 1500 meters to school in 30 minutes. What is their average speed in meters per minut
A student walks 1500 meters to school in 30 minutes. What is their average speed in meters per minute? 1500 meters / 30 minutes Divide top and bottom by 30 [B]50 meters / minute[/B]

A submarine at an elevation of -185 meters descends to 3 times that elevation. Then, it elevates 90
A submarine at an elevation of -185 meters descends to 3 times that elevation. Then, it elevates 90 meters. What is the submarines new elevation? 3 times the current elevation is: 3 * -185 = -555 Elevating 90 meters means we have a positive change: -555 + 90 = [B]-465[/B]

A submarine dove 132.58 meters to reach a resting depth of 700 meter below sea level. What was it's
A submarine dove 132.58 meters to reach a resting depth of 700 meter below sea level. What was it's original depth Below sea level is a negative amount. So they end up at -700. To go back up toward sea level, we'd be at: -700 + 132.58 = -567.42 Negative numbers mean below sea level, so the original depth was [B]567.42 meters below sea level[/B]

A submarine hovers at 240 meters below sea level. If it descends 160 meters and then ascends 390 met
A submarine hovers at 240 meters below sea level. If it descends 160 meters and then ascends 390 meters, what is its new position? 240 meters below sea level means a negative number, so we start with: -240 Descending 160 meters means our depth decreases, so we subtract: -240 - 160 = -400 Ascends means our depth increases, so we add: -400 + 390 = [B]-10 or 10 feet below sea level [MEDIA=youtube]ngToCpLBgH4[/MEDIA][/B]

a submarine is 450 feet below sea level. It descends 300 feet. What is its new position?
a submarine is 450 feet below sea level. It descends 300 feet. What is its new position? We start at 450 feet below sea level. We descend another 300 feet, so we're now at: -450 - 300 = -750 Negative depth means below sea level, so the submarine is at [B]750 feet below sea level[/B]

A submarine is 75 feet below sea level. It descends another 25 feet every 10 seconds for 3 minutes.
A submarine is 75 feet below sea level. It descends another 25 feet every 10 seconds for 3 minutes. What integer represents the submarines current location? Assumptions and givens: [LIST] [*]Let m be the number of minutes [*]10 seconds is 1/6 of a minute, 6 (10) seconds blocks per minute * 3 minutes = 18 (10 second blocks) [*]Below sea level is a negative number [/LIST] [U]Current depth:[/U] -25(18) - 75 -450 - 75 [B]-525[/B]

A submarine sits at –300 meters in relation to sea level. Then it descends 115 meters. What is its n
A submarine sits at –300 meters in relation to sea level. Then it descends 115 meters. What is its new position in relation to sea level? Descending means we go down in sea level, so we subtract: -300 - 115 = [B]-415 or 415 meters below sea level[/B]

A suitcase contains nickels, dimes and quarters. There are 2&1/2 times as many dimes as nickels and
A suitcase contains nickels, dimes and quarters. There are 2&1/2 times as many dimes as nickels and 5 times the number of quarters as the number of nickels. If the coins have a value of $24.80, how many nickels are there in the suitcase? Setup number of coins: [LIST] [*]Number of nickels = n [*]Number of dimes = 2.5n [*]Number of quarters = 5n [/LIST] Setup value of coins: [LIST] [*]Value of nickels = 0.05n [*]Value of dimes = 2.5 * 0.1n = 0.25n [*]Value of quarters = 5 * 0.25n = 1.25n [/LIST] Add them up: 0.05n + 0.25n + 1.25n = 24.80 Solve for [I]n[/I] in the equation 0.05n + 0.25n + 1.25n = 24.80 [SIZE=5][B]Step 1: Group the n terms on the left hand side:[/B][/SIZE] (0.05 + 0.25 + 1.25)n = 1.55n [SIZE=5][B]Step 2: Form modified equation[/B][/SIZE] 1.55n = + 24.8 [SIZE=5][B]Step 3: Divide each side of the equation by 1.55[/B][/SIZE] 1.55n/1.55 = 24.80/1.55 n = [B]16[/B] [B] [URL='https://www.mathcelebrity.com/1unk.php?num=0.05n%2B0.25n%2B1.25n%3D24.80&pl=Solve']Source[/URL][/B]

A sum of money doubles in 20 years on simple interest. It will get triple at the same rate in: a.
A sum of money doubles in 20 years on simple interest. It will get triple at the same rate in: a. 40 years b. 50 years c. 30 years d. 60 years e. 80 years Simple interest formula if we start with 1 dollar and double to 2 dollars: 1(1 + i(20)) = 2 1 + 20i = 2 Subtract 1 from each side: 20i = 1 Divide each side by 20 i = 0.05 Now setup the same simple interest equation, but instead of 2, we use 3: 1(1 + 0.05(t)) = 3 1 + 0.05t = 3 Subtract 1 from each side: 0.05t = 2 Divide each side by 0.05 [B]t = 40 years[/B]

A super deadly strain of bacteria is causing the zombie population to double every day. Currently, t
A super deadly strain of bacteria is causing the zombie population to double every day. Currently, there are 25 zombies. After how many days will there be over 25,000 zombies? We set up our exponential function where n is the number of days after today: Z(n) = 25 * 2^n We want to know n where Z(n) = 25,000. 25 * 2^n = 25,000 Divide each side of the equation by 25, to isolate 2^n: 25 * 2^n / 25 = 25,000 / 25 The 25's cancel on the left side, so we have: 2^n = 1,000 Take the natural log of each side to isolate n: Ln(2^n) = Ln(1000) There exists a logarithmic identity which states: Ln(a^n) = n * Ln(a). In this case, a = 2, so we have: n * Ln(2) = Ln(1,000) 0.69315n = 6.9077 [URL='https://www.mathcelebrity.com/1unk.php?num=0.69315n%3D6.9077&pl=Solve']Type this equation into our search engine[/URL], we get: [B]n = 9.9657 days ~ 10 days[/B]

A survey of 75 people found 45 like rabbits, 32 like hamsters, and 15 like both animals. How many pe
A survey of 75 people found 45 like rabbits, 32 like hamsters, and 15 like both animals. How many people like neither animal? People who like either rabbits or hamsters = Rabbit liners + hamster likes - both likers People who like either rabbits or hamsters = 45 + 32 - 15 People who like either rabbits or hamsters = 62 People who like neither = 75 - People who like either rabbits or hamsters People who like neither = 775 - 62 People who like neither = [B]13[/B]

A survey of 950 college students found that 85% of the men and 90% of the women identified math as t
A survey of 950 college students found that 85% of the men and 90% of the women identified math as their favorite subject. If altogether 834 students reported math to be their favorite subject how many men and women participated in the survey Let m be the number of men and w be the number of women. We are given 2 equations [LIST=1] [*]m + w = 950 [*]0.85m + 0.90w = 834 [/LIST] Using our [URL='http://www.mathcelebrity.com/simultaneous-equations.php?term1=m+%2B+w+%3D+950&term2=0.85m+%2B+0.90w+%3D+834&pl=Cramers+Method']simultaneous equations calculator[/URL], we get: [LIST] [*]m = [B]420[/B] [*]w = [B]530[/B] [/LIST]

A survey was conducted that asked 1007 people how many books they had read in the past year. Results
A survey was conducted that asked 1007 people how many books they had read in the past year. Results indicated that x overbarequals11.3 books and sequals16.6 books. Construct a 90% confidence interval for the mean number of books people read. Interpret the interval. x bar = 11.3 s = 16.6 n = 1007 [URL='https://www.mathcelebrity.com/normconf.php?n=1007&xbar=11.3&stdev=16.6&conf=90&rdig=4&pl=Not+Sure']We use our confidence interval calculator[/URL] and get [B]10.4395 < u < 12.1605[/B]. [B][I]We interpret this as: If we repeated experiments, the proportion of such intervals containing u would be 90%[/I][/B]

A survey was given to 120 6th grade students at middle school. It showed the 42 students said they l
A survey was given to 120 6th grade students at middle school. It showed the 42 students said they like playing at the park. What % of the students said they like playing there? We want [I][URL='https://www.mathcelebrity.com/perc.php?num=42&den=120&pcheck=1&num1=+16&pct1=+80&pct2=+35&den1=+90&pct=+82&decimal=+65.236&astart=+12&aend=+20&wp1=20&wp2=30&pl=Calculate']42 is what percent of 120[/URL][/I] Using our calculator above, we get [B]35%[/B]

A sweater costs $40. That is 5 times as much as a shirt. What is the price of the shirt?
A sweater costs $40. That is 5 times as much as a shirt. What is the price of the shirt? State this as an equation. Let the price of the shirt be s. 5 times as much means we multiply s by 5: 5s = 40 [URL='https://www.mathcelebrity.com/1unk.php?num=5s%3D40&pl=Solve']Type this equation into the search engine[/URL], we get: s = [B]8[/B]

A sweater that you love costs $32. You really want the sweater but only have $35. If there’s a sales
A sweater that you love costs $32. You really want the sweater but only have $35. If there’s a sales tax of 4% on the item, do you have enough to buy the sweater? Calculate after-tax amount: After tax amount = Sale Price * (1 + sales tax percent) After tax amount = 32 * (1 + 0.04) <-- Since 4% = 0.04 After tax amount = 32 * (1.04) After tax amount = $33.28 [B]Yes[/B], since $33.28 is less than or equal to $35, you have enough to buy the sweater.

A tablet of Tylenol contains 35 mg of the active ingredient acetaminophen. If you take 140 mg of ac
A tablet of Tylenol contains 35 mg of the active ingredient acetaminophen. If you take 140 mg of acetaminophen, how many tablets did you take? 140/35 = 4 tablets

A tank has 800 liters of water. 12ml of water leaks from the tank every second.how long does it take
A tank has 800 liters of water. 12ml of water leaks from the tank every second.how long does it take for the tank to be empty Assumptions and givens: [LIST] [*]Let the number of seconds be s. [*]An empty tank means 0 liters of water. [*]Leaks mean we subtract from the starting volume. [/LIST] We have the following relation: 800 - 12s = 0 To solve this equation for s, we [URL='https://www.mathcelebrity.com/1unk.php?num=800-12s%3D0&pl=Solve']type it in our search engine[/URL] and we get: s = 66.67 seconds

A tank used 22 gallons of gas to go 17.6 miles. How many miles per gallon did the tank use?
A tank used 22 gallons of gas to go 17.6 miles. How many miles per gallon did the tank use? 17.6 miles / 22 gallons = [B]0.8 miles per gallon[/B]

A taxi cab in Chicago charges $3 per mile and $1 for every person. If the taxi cab ride for two peop
A taxi cab in Chicago charges $3 per mile and $1 for every person. If the taxi cab ride for two people costs $20. How far did the taxi cab travel. Set up a cost function C(m) where m is the number of miles driven: C(m) = cost per mile * m + per person fee [U]Calculate per person fee:[/U] per person fee = $1 per person * 2 people per person fee = $2 [U]With a cost per mile of $3 and per person fee of $2, we have:[/U] C(m) = cost per mile * m + per person fee C(m) = 3m + 2 The problem asks for m when C(m) = 20, so we set 3m + 2 equal to 20: 3m + 2 = 20 To solve this equation for m, we [URL='https://www.mathcelebrity.com/1unk.php?num=3m%2B2%3D20&pl=Solve']plug it in our search engine[/URL] and we get: m = [B]6[/B]

A taxi cab in nyc charges a pick up fee of $5.00 . The customer must also pay $2.59 for each mile th
A taxi cab in nyc charges a pick up fee of $5.00 . The customer must also pay $2.59 for each mile that the taxi must drive to reach their destination. Write an equation Set up a cost function C(m) where m is the number of miles: C(m) = Mileage Charge * m + pick up fee [B]C(m) = 2.59m + 5[/B]

A taxi charges a flat rate of $1.50 with an additional charge of $0.80 per mile. Samantha wants to s
A taxi charges a flat rate of $1.50 with an additional charge of $0.80 per mile. Samantha wants to spend less than $12 on a ride. Which inequality can be used to find the distance Samantha can travel? Set up the travel cost equation where m is the number of miles: C(m) = 0.8m + 1.50 If Samantha wants to spend less than 12 per ride, we have an inequality where C(m) < 12: [B]0.8m + 1.50 < 12[/B]

A taxi charges a flat rate of $1.50 with an additional charge of $0.80 per mile. Samantha wants to s
A taxi charges a flat rate of $1.50 with an additional charge of $0.80 per mile. Samantha wants to spend less than $12 on a ride. Which inequality can be used to find the distance Samantha can travel? [LIST] [*]Each ride will cost 1.50 + 0.8x where x is the number of miles per trip. [*]This expression must be less than 12. [/LIST] [U]Setup the inequality:[/U] 1.5 + 0.8x < 12 [U]Subtracting 1.5 from each side of the inequality[/U] 0.8x < 10.5 [U]Simplifying even more by dividing each side of the inequality by 0.8, we have:[/U] [B]x < 13.125[/B]

A taxi charges a flat rate of $1.75, plus an additional $0.65 per mile. If Erica has at most $10 to
A taxi charges a flat rate of $1.75, plus an additional $0.65 per mile. If Erica has at most $10 to spend on the cab ride, how far could she travel? Set up a cost function C(m), where m is the number of miles: C(m) = Cost per mile * m + flat rate C(m) = 0.65m + 1.75 The problem asks for m when C(m) = 10 0.65m + 1.75 = 10 [URL='https://www.mathcelebrity.com/1unk.php?num=0.65m%2B1.75%3D10&pl=Solve']Typing this equation into the search engine[/URL], we get: m = [B]12.692 miles[/B]

A taxi charges a flat rate of $1.75, plus an additional $0.65 per mile. If Erica has at most 10$ to
A taxi charges a flat rate of $1.75, plus an additional $0.65 per mile. If Erica has at most 10$ to spend on the cab ride, how far could she travel Set up a cost function C(m), where m is the number of miles Erica can travel. We have: C(m) = 0.65m + 1.75 If C(m) = 10, we have: 0.65m + 1.75 = 10 [URL='https://www.mathcelebrity.com/1unk.php?num=0.65m%2B1.75%3D10&pl=Solve']Typing this equation into our search engine[/URL], we get: m = 12.69 miles If the problem asks for complete miles, we round down to 12 miles.

A taxi charges a flat rate of $1.75, plus an additional $0.65 per mile. If Erica has at most 10$ to
A taxi charges a flat rate of $1.75, plus an additional $0.65 per mile. If Erica has at most 10$ to spend on the cab ride, how far could she travel? Set up the cost function C(m) where m is the number of miles: C(m) = 0.65m + 1.75 If Erica has $10, then C(m) = 10, so we have: 0.65m + 1.75 = 10 [URL='https://www.mathcelebrity.com/1unk.php?num=0.65m%2B1.75%3D10&pl=Solve']Typing this equation into the search engine[/URL], we get m = 12.69 if the answer asks for whole number, then we round down to m = 12

A taxi charges a flat rate of 1.75, plus an additional 0.65 per mile. If Erica has at most 10 to spe
A taxi charges a flat rate of 1.75, plus an additional 0.65 per mile. If Erica has at most 10 to spend on the cab ride, how far could she travel? Setup an equation where x is the number of miles traveled: 0.65x + 1.75 = 10 Subtract 1.75 from each side: 0.65x = 8.25 Divide each side by 0.65 [B]x = 12.69 miles[/B] If we do full miles, we round down to 12. [MEDIA=youtube]mFqUe2mjX-w[/MEDIA]

A taxi service charges an initial fee of $3 plus $1.80 per mile. How far can you travel for $12?
A taxi service charges an initial fee of $3 plus $1.80 per mile. How far can you travel for $12? Given m for miles, we have the equation: 1.80m + 3 = 12 We [URL='https://www.mathcelebrity.com/1unk.php?num=1.80m%2B3%3D12&pl=Solve']type this equation into our search engine[/URL] to solve for m and we get: m = [B]5[/B]

A teacher assumed that the average of grades for a math test was 80. Imagine 20 students took the te
A teacher assumed that the average of grades for a math test was 80. Imagine 20 students took the test and the 95% confidence interval of grades was (83, 90). Can you reject the teacher's assumption? a. Yes b. No c. We cannot tell from the given information [B]a. Yes[/B] [I]At the 0.05 significance level, yes since 80 is not in the confidence interval.[/I]

A teacher had 54 pencils gave p pencils to each of his s students. How many pencils does he have lef
A teacher had 54 pencils gave p pencils to each of his s students. How many pencils does he have left? If the teacher gave p pencils to each of his s students, then he gave away ps total pencils. He's left with: [B]54 - ps[/B]

A teacher hypothesized that in her class, grades of girls on a chemistry test were the same as grade
A teacher hypothesized that in her class, grades of girls on a chemistry test were the same as grades of boys. If the probability value of her null hypothesis was 0.56, it suggested: a. We failed to reject the null hypothesis b. Boys' grades were higher than girls' grades c. Girls' grades were higher than boys' grades d. The null hypothesis was rejected [B]a. We failed to reject the null hypothesis[/B] Due to a high probability.

a teacher puts 1125 marbles into 9 containers to put the same number of marbles into each container
a teacher puts 1125 marbles into 9 containers to put the same number of marbles into each container how many marbles does the teacher put into each container marbles per container = Total marbles / total containers marbles per container = 1125/9 marbles per container = [B]125[/B]

A teacher who makes $54000 per year $8400 in taxes, and $1200 in union does what fraction of the tea
A teacher who makes $54000 per year $8400 in taxes, and $1200 in union does what fraction of the teacher’s income does she have left Calculate Net Income: Net income = Earnings - Taxes - Union Dues Net income = 54000 - 8400 - 1200 Net income = 44,400 Net Income Percent = 100% * Net Income / Earnings Net Income Percent = 100% * 44,400/54,000 Net Income Percent = 100% * 0.8222 Net Income Percent = [B]82.22%[/B]

A teacher’s salary was $3300 after she had received an increase of 10%. Calculate the teacher’s sala
A teacher’s salary was $3300 after she had received an increase of 10%. Calculate the teacher’s salary if she has received an increase of 20% instead. First, we need to find the starting salary. Let the starting salary be s. Since 10% as a decimal is 0.10, We're given: s*(1.10) = 3300 1.10s = 3300 To solve for s, [URL='https://www.mathcelebrity.com/1unk.php?num=1.10s%3D3300&pl=Solve']we type this equation into our search engine[/URL] and we get: s = [B]3000[/B] The problem asks for the new salary if the teacher's starting salary was increased by 20%. 20% as a decimal is 0.20, so we have: 3000(1.2) = $[B]3,600[/B]

A television sells for $750. Instead of paying the total amount at the time of the purchase, the sam
A television sells for $750. Instead of paying the total amount at the time of the purchase, the same television can be bought by paying $100 down and $50 a month for 14 months. How much is saved by paying the total amount at the time of the purchase? Option 2: 100 + 50(14) 100 + 700 800 800 - 750 = [B]$50 saved [MEDIA=youtube]XAixLxvelcg[/MEDIA][/B]

A test has three true-false questions. Find the total number of ways you can answer the three questi
A test has three true-false questions. Find the total number of ways you can answer the three questions We can either choose T or F. So we have: Question 1: 2 choies Question 2: 2 choices Question 3: 2 choices 2 * 2 * 2 = [B]8 choices [/B] [LIST=1] [*][B]TTT[/B] [*][B]TTF[/B] [*][B]TFT[/B] [*][B]FTT[/B] [*][B]FTF[/B] [*][B]FFT[/B] [*][B]TFF[/B] [*][B]FFF[/B] [/LIST]

A test has twenty questions worth 100 points . The test consist of true/false questions worth 3 poin
A test has twenty questions worth 100 points . The test consist of true/false questions worth 3 points each and multiple choice questions worth 11 points each . How many multiple choice questions are on the test? Set up equations where t = true false and m = multiple choice: [LIST=1] [*]t + m = 20 [*]3t + 11m = 100 [/LIST] Use our [URL='http://www.mathcelebrity.com/simultaneous-equations.php?term1=t+%2B+m+%3D+20&term2=3t+%2B+11m+%3D+100&pl=Cramers+Method']simultaneous equation calculator[/URL]: [B]t = 15, m = 5[/B]

A test has twenty questions worth 100 points total. the test consists of true/false questions worth
A test has twenty questions worth 100 points total. the test consists of true/false questions worth 3 points each and multiple choice questions worth 11 points each. How many true/false questions are on the test? Let m be the number of multiple choice questions and t be the number of true/false questions. We're given: [LIST=1] [*]m + t = 20 [*]11m + 3t = 100 [/LIST] We can solve this system of equations 3 ways below: [LIST] [*][URL='https://www.mathcelebrity.com/simultaneous-equations.php?term1=m+%2B+t+%3D+20&term2=11m+%2B+3t+%3D+100&pl=Substitution']Substitution Method[/URL] [*][URL='https://www.mathcelebrity.com/simultaneous-equations.php?term1=m+%2B+t+%3D+20&term2=11m+%2B+3t+%3D+100&pl=Elimination']Elimination Method[/URL] [*][URL='https://www.mathcelebrity.com/simultaneous-equations.php?term1=m+%2B+t+%3D+20&term2=11m+%2B+3t+%3D+100&pl=Cramers+Method']Cramer's Rule[/URL] [/LIST] No matter which method we choose, we get the following answers: [LIST] [*][B]m = 5[/B] [*][B]t = 15[/B] [/LIST] Check our work in equation 1: 5 + 15 ? 20 [I]20 = 20[/I] Check our work in equation 2: 11(5) + 3(15) ? 100 55 + 45 ? 100 [I]100 = 100[/I]

A test has twenty questions worth 100 points. The test consists of True/False questions worth 3 poin
A test has twenty questions worth 100 points. The test consists of True/False questions worth 3 points each and multiple choice questions worth 11 points each. How many multiple choice questions are on the test? Let the number of true/false questions be t. Let the number of multiple choice questions be m. We're given two equations: [LIST=1] [*]m + t = 20 [*]11m + 3t = 100 [/LIST] We have a set of simultaneous equations. We can solve this using 3 methods: [LIST=1] [*][URL='https://www.mathcelebrity.com/simultaneous-equations.php?term1=1m+%2B+t+%3D+20&term2=11m+%2B+3t+%3D+100&pl=Substitution']Substitution Method[/URL] [*][URL='https://www.mathcelebrity.com/simultaneous-equations.php?term1=1m+%2B+t+%3D+20&term2=11m+%2B+3t+%3D+100&pl=Elimination']Elimination Method[/URL] [*][URL='https://www.mathcelebrity.com/simultaneous-equations.php?term1=1m+%2B+t+%3D+20&term2=11m+%2B+3t+%3D+100&pl=Cramers+Method']Cramer's Rule[/URL] [/LIST] No matter which method we pick, we get the same answer: [LIST] [*][B]m = 5[/B] [*][B]t = 15[/B] [/LIST]

A text message plan costs $7 per month plus $0.28 per text. Find the monthly cost for x text message
A text message plan costs $7 per month plus $0.28 per text. Find the monthly cost for x text messages. We set up the cost function C(x) where x is the number of text messages per month: C(x) = Cost per text * x + Monthly cost Plugging in our given numbers, we get: [B]C(x) = 0.28x + 7[/B]

a textbook store sold a combined total of 296 sociology and history text books in a week. the number
a textbook store sold a combined total of 296 sociology and history text books in a week. the number of history textbooks sold was 42 less than the number of sociology textbooks sold. how many text books of each type were sold? Let h = history book and s = sociology books. We have 2 equations: (1) h = s - 42 (2) h + s = 296 Substitute (1) to (2) s - 42 + s = 296 Combine variables 2s - 42 = 296 Add 42 to each side 2s = 338 Divide each side by 2 s = 169 So h = 169 - 42 = 127

A textbook store sold a combined total of 307 biology and chemistry textbooks in a week. The number
A textbook store sold a combined total of 307 biology and chemistry textbooks in a week. The number of chemistry textbooks sold was 71 less than the number of biology textbooks sold. How many textbooks of each type were sold? Let b be the number of biology books and c be the number of chemistry books. We have two equations: [LIST=1] [*]b + c = 307 [*]c = b - 71 [/LIST] Substitute (2) into (1) for c b + (b - 71) = 307 Combine like terms: 2b - 71 = 307 Using our [URL='http://www.mathcelebrity.com/1unk.php?num=2b-71%3D307&pl=Solve']equation solver[/URL], we get: [B]b = 189[/B] Now substitute that into (2): c = 189 - 71 [B]c = 118[/B]

A theater has 1200 seats. Each row has 20 seats. Write and solve an equation to find the number x of
A theater has 1200 seats. Each row has 20 seats. Write and solve an equation to find the number x of rows in the theater. Let x be the number of rows in the theater: x = Total Seats / Seats per row x = 1200/20 x = [B]60[/B]

A theater is 3/4 full. When 96 people leave, the theater is only 35% full. How many seat are there
A theater is 3/4 full. When 96 people leave, the theater is only 35% full. How many seats are there? Let the full capacity of seats in the theater be s. We're given: 3/4s - 96 = 0.35s (Since 35% is 0.35) We also know that 3/4 = 0.75, so let's use this to have decimals: 0.75s - 96 = 0.35s To solve for s, we [URL='https://www.mathcelebrity.com/1unk.php?num=0.75s-96%3D0.35s&pl=Solve']type this equation into our search engine[/URL] and we get: s = [B]240[/B]

A theatre contains 459 seats and the ticket prices for a recent play were $53 for adults and $16 for
A theatre contains 459 seats and the ticket prices for a recent play were $53 for adults and $16 for children. If the total proceeds were $13,930 for a sold- out matinee, how many of each type of ticket were sold? Let a be the number of adult tickets and c be the number of children tickets. We have the following equations: [LIST=1] [*]a + c =459 [*]53a + 16c = 13930 [/LIST] Using our [URL='http://www.mathcelebrity.com/simultaneous-equations.php?term1=a%2Bc%3D459&term2=53a+%2B+16c+%3D+13930&pl=Cramers+Method']simultaneous equation calculator[/URL], we have: [B]a = 178 c = 281[/B]

A theatre sold 67 tickets to a show. 43 tickets were for children up to 12 years old. How many ticke
A theatre sold 67 tickets to a show. 43 tickets were for children up to 12 years old. How many tickets were for people older than 12 67 tickets - 43 tickets for 12 and under = [B]24 tickets[/B] for people older than 12

A thermometer has a range of 1.5 degrees of the temperature, what is the maximum and minimum at 87.4
A thermometer has a range of 1.5 degrees of the temperature, what is the maximum and minimum at 87.4 degrees Range = Max - Min Divide this by 2 to get the lesser half and larger half: Half-Range = 1.5/2 Half-Range = 0.75 [U]Our Maximum temperature is:[/U] Max Temp = Current Temp + Half-Range Max Temp = 87.4 + 0.75 Max Temp = [B]88.15 [/B] [U]Our Minimum temperature is:[/U] Min Temp = Current Temp - Half-Range Min Temp = 87.4 - 0.75 Min Temp = [B][B]86.65[/B][/B]

A third of a pizza is 400 calories. How many calories in the whole pizza?
A third of a pizza is 400 calories. How many calories in the whole pizza? Let c be the number of calories in the whole pizza. WE have: c/3 = 400 To solve this proportion for c, we [URL='https://www.mathcelebrity.com/prop.php?num1=c&num2=400&den1=3&den2=1&propsign=%3D&pl=Calculate+missing+proportion+value']type it in our search engine[/URL] and get: c = [B]1,200[/B]

A three digit number, if the digits are unique
A three digit number, if the digits are unique [LIST=1] [*]For our first digit, we can start with anything but 0. So we have 9 options [*]For our second digit, we can use anything but 9 since we want to be unique. So we have 9 options [*]For our last digit, we can use anything but the first and second digit. So we have 10 - 2 = 8 options [/LIST] Our total 3 digit numbers with all digits unique is found by the fundamental rule of counting: 9 * 9 * 8 = [B]648 possible 3 digit numbers[/B]

a times b divided by the quantity a minus b
a times b divided by the quantity a minus b a times b: ab a minus b: a - b Now divide a times b by a minus b: [B]ab/(a - b)[/B]

A times r squared multiplied by h
A times r squared multiplied by h r squared means we raise r to the power of 2: r^2 a times r squared: ar^2 Multiplied by h: [B]ahr^2[/B]

A tire repair shop charges $5 for tool cost and $2 for every minute the worker spends on the repair.
A tire repair shop charges $5 for tool cost and $2 for every minute the worker spends on the repair. A) Write an equation of the total cost of repair, $y, in terms of a total of x minutes of repair. y = Variable Cost + Fixed Cost y = Cost per minute of repair * minutes of repair + Tool Cost [B]y = 2x + 5[/B]

A toad croaks every 8seconds and a frog croaks every 6 seconds .They both croak at the same .After h
A toad croaks every 8seconds and a frog croaks every 6 seconds .They both croak at the same .After how many seconds will they next croak at the same time again. We want the least common multiple of 8 and 6. We type in [URL='https://www.mathcelebrity.com/gcflcm.php?num1=6&num2=8&num3=&pl=GCF+and+LCM']LCM(6, 8) into our search engine[/URL] and we get [B]24[/B]

A toffee jar contains 225 toffees . How many toffees will be there in 62 such toffee jars ?
A toffee jar contains 225 toffees . How many toffees will be there in 62 such toffee jars ? Total Toffees = Toffee per jar * number of jars Total Toffees = 225 * 62 Total Toffees = [B]13,950 toffees[/B]

a tomato plant has 5 leaves on day 1, 7 on day 3 and 9 on day 5. what is the rate it is growing?
a tomato plant has 5 leaves on day 1, 7 on day 3 and 9 on day 5. what is the rate it is growing? Day 1 = 5 leaves Day 3 = 7 leaves Day 5 = 9 leaves [B]Growth Rate = 2 leaves per day[/B]

A tortoise is walking in the desert. It walks at a speed of 5 meters per minute for 12.5 meters. For
A tortoise is walking in the desert. It walks at a speed of 5 meters per minute for 12.5 meters. For how many minutes does it walk? Distance formula (d) for a rate (r) and time (t) is: d = rt We're given d = 12.5 and r = 5 12.5 = 5t 5t = 12.5 Solve for t. Divide each side of the equation by 5: 5t/5 = 12.5/5 Cancel the 5's on left side and we get: t = [B]2.5[/B]

a total of $4000 is invested: part at 10% and the remainder at 15%. How much is invested at each rat
a total of $4000 is invested: part at 10% and the remainder at 15%. How much is invested at each rate if the annual interest is $430? Using our [URL='https://www.mathcelebrity.com/split-fund-interest-calculator.php?p=4000&i1=10&i2=15&itot=430&pl=Calculate']split fund interest calculator[/URL], we get: [LIST] [*][B]3,400[/B] @ 10% [*][B]600[/B] @ 15% [/LIST]

A total of $4300 was invested, part of it at 6% interest and the remainder at 9%. If the total yearl
A total of $4300 was invested, part of it at 6% interest and the remainder at 9%. If the total yearly interest amounted to $315, how much was invested at each rate? Using our [URL='https://www.mathcelebrity.com/split-fund-interest-calculator.php?p=4300&i1=6+&i2=9&itot=315&pl=Calculate']split fund interest calculator[/URL], we get: [LIST] [*][B]Fund 1: 2,400[/B] [*][B]Fund 2: 1,900[/B] [/LIST]

A total of $6,000 was invested, a portion at 6% and the remainder at 8%. The total amount of interes
A total of $6,000 was invested, a portion at 6% and the remainder at 8%. The total amount of interest earned was $450. How much was invested at each rate? Using our split fund interest calculator, we get: [LIST] [*][B]1500 in 6% fund[/B] [*][B]4500 in 8% fund[/B] [/LIST]

A total of $7000 is invested: part at 7% and the remainder at 9%. How much is invested at each rate
A total of $7000 is invested: part at 7% and the remainder at 9%. How much is invested at each rate if the annual interest is $550? Using our [URL='https://www.mathcelebrity.com/split-fund-interest-calculator.php?p=7000&i1=7&i2=9&itot=550&pl=Calculate']split fund interest calculator[/URL], we get: [LIST] [*][B]Fund 1: $4,000[/B] [*][B]Fund 2: $3,000[/B] [/LIST]

a total of 6000 is invested part at 8% and the remainder at 13%. how much is invested at each rate i
a total of 6000 is invested part at 8% and the remainder at 13%. how much is invested at each rate if the annual interest is 710 Using our [URL='https://www.mathcelebrity.com/split-fund-interest-calculator.php?p=6000&i1=8&i2=13&itot=710&pl=Calculate']split fund interest calculator[/URL], we get: 1,400 4,600

A total of 7000 is invested part at 7% and the reminder at 11% .how much is invested at each rate of
A total of 7000 is invested part at 7% and the reminder at 11% .how much is invested at each rate of the annual interest is 640 Using our [URL='https://www.mathcelebrity.com/split-fund-interest-calculator.php?p=7000&i1=7&i2=11&itot=640&pl=Calculate']split fund interest rate calculator[/URL], we get: [LIST] [*]Fund 1 = [B]3250[/B] [*]Fund 2 = [B]3750[/B] [/LIST]

A tow truck charges a service fee of $50 and an additional fee of $1.75 per mile. What distance was
A tow truck charges a service fee of $50 and an additional fee of $1.75 per mile. What distance was Marcos car towed if he received a bill for $71 Set up a cost equation C(m) where m is the number of miles: C(m) = Cost per mile * m + Service Fee Plugging in the service fee of 50 and cost per mile of 1.75, we get: C(m) = 1.75m + 50 The question asks for what m is C(m) = 71. So we set C(m) = 71 and solve for m: 1.75m + 50 = 71 Solve for [I]m[/I] in the equation 1.75m + 50 = 71 [SIZE=5][B]Step 1: Group constants:[/B][/SIZE] We need to group our constants 50 and 71. To do that, we subtract 50 from both sides 1.75m + 50 - 50 = 71 - 50 [SIZE=5][B]Step 2: Cancel 50 on the left side:[/B][/SIZE] 1.75m = 21 [SIZE=5][B]Step 3: Divide each side of the equation by 1.75[/B][/SIZE] 1.75m/1.75 = 21/1.75 m = [B]12[/B]

A town has a population of 12000 and grows at 5% every year. What will be the population after 12 ye
A town has a population of 12000 and grows at 5% every year. What will be the population after 12 years, to the nearest whole number? We calculate the population of the town as P(t) where t is the time in years since now. P(t) = 12000(1.05)^t The problem asks for P(12) P(12) = 12000(1.05)^12 P(12) = 12000(1.79585632602) P(12) = [B]21550[/B] <- nearest whole number

A town has a population of 25,000 and grows at 7.7% every 4 months. What will be the population afte
A town has a population of 25,000 and grows at 7.7% every 4 months. What will be the population after 6 years? [LIST] [*]1 year = 12 months [*]12 months / 4 months = 3 compounding periods per year [*]3 compounding periods per year * 6 years = 18 compounding periods [/LIST] So we have our population growth as follows: 25,000(1.077)^18 25,000 * 3.8008668804 95,021.67 ~ [B]95,021[/B]

A town has a population of 50,000. Its rate increases 8% every 6 months. Find the population after 4
A town has a population of 50,000. Its rate increases 8% every 6 months. Find the population after 4 years. Every 6 months means twice a year. So we have 4 years * twice a year increase = 8 compounding periods. Our formula for compounding an initial population P at time t is P(t) at a compounding percentage i: P(t) = P * (1 + i)^t Since 8% is 0.08 as a decimal and t = 4 *2 = 8, we have: P(8) = 50000 * (1.08)^8 P(8) = 50000 * 1.85093 P(8) = 92,546.51 Since we can't have a partial person, we round down to [B]92,545[/B]

A towns population is currently 500. If the population doubles every 30 years, what will the populat
A towns population is currently 500. If the population doubles every 30 years, what will the population be 120 years from now? Find the number of doubling times: 120 years / 30 years per doubling = 4 doubling times Set up our growth function P(n) where n is the number of doubling times: P(n) = 500 * 2^n Since we have 4 doubling times, we want P(4): P(4) = 500 * 2^4 P(4) = 500 * 16 P(4) = [B]8,000[/B]

A toy company makes "Teddy Bears". The company spends $1500 for factory expenses plus $8 per bear. T
A toy company makes "Teddy Bears". The company spends $1500 for factory expenses plus $8 per bear. The company sells each bear for $12.00 each. How many bears must this company sell in order to break even? [U]Set up the cost function C(b) where b is the number of bears:[/U] C(b) = Cost per bear * b + factory expenses C(b) = 8b + 1500 [U]Set up the revenue function R(b) where b is the number of bears:[/U] R(b) = Sale Price per bear * b R(b) = 12b [U]Break-even is where cost equals revenue, so we set C(b) equal to R(b) and solve for b:[/U] C(b) = R(b) 8b + 1500 = 12b To solve for b, we [URL='https://www.mathcelebrity.com/1unk.php?num=8b%2B1500%3D12b&pl=Solve']type this equation into our search engine[/URL] and we get: b = [B]375[/B]

A toy factory makes 5,000 teddy bears per day. The supervisor randomly selects 10 teddy bears from a
A toy factory makes 5,000 teddy bears per day. The supervisor randomly selects 10 teddy bears from all 5,000 teddy bears and uses this sample to estimate the mean weight of teddy bears and the sample standard deviation. How many degrees of freedom are there in the estimate of the standard deviation? DF = n - 1 DF = 10 - 1 [B]DF = 9[/B]

A tractor tire has a radius of 24 inches. If the tire rotates one time around, about how many inches
A tractor tire has a radius of 24 inches. If the tire rotates one time around, about how many inches of ground will it cover? Use 3.14 for pi. A tractor tire is a circle. We want the circumference, which is the distance around the tire. C = 2pir C = 2(3.1415)24 [B]C ~ 150.8[/B]

A train leaves San Diego at 1:00 PM. A second train leaves the same city in the same direction at 3
A train leaves San Diego at 1:00 PM. A second train leaves the same city in the same direction at 3:00 PM. The second train travels 30mph faster than the first. If the second train overtakes the first at 6:00 PM, what is the speed of each of the two trains? Distance = Rate x Time Train 1: d = rt t = 1:oo PM to 6:00 PM = 5 hours So we have d = 5r Train 2: d = (r + 30)t t = 3:oo PM to 6:00 PM = 3 hours So we have d = 3(r + 30) Set both distances equal to each other since overtake means Train 2 caught up with Train 1, meaning they both traveled the same distance: 5r = 3(r + 30) Multiply through: 3r + 90 = 5r [URL='https://www.mathcelebrity.com/1unk.php?num=3r%2B90%3D5r&pl=Solve']Run this equation through our search engine[/URL], and we get [B]r = 45[/B]. This is Train 1's Speed. Train 2's speed = 3(r + 30). Plugging r = 45 into this, we get 3(45 + 30). 3(75) [B]225[/B]

A train ticket is 8 centimeters tall and 10 centimeters long. What is its area?
A train ticket is 8 centimeters tall and 10 centimeters long. What is its area? The ticket is a rectangle. The area is: A = lw Plugging in our numbers, we get: A = (8)(10) A = 80

A train traveled at 66km an hour for four hours. Find the distance traveled
A train traveled at 66km an hour for four hours. Find the distance traveled Distance = Rate * Time Distance = 66km/hr * 4 hours Distance = [B]264 miles[/B]

A trapezoid has one base that is 120% of the length of the other base. The two sides are each 1/2 th
A trapezoid has one base that is 120% of the length of the other base. The two sides are each 1/2 the length of the smaller base. If the perimeter of the trapezoid is 54.4 inches, what is the length of the smaller base of the trapezoid? Setup measurements: [LIST] [*]Small base = n [*]Large base = 1.2n [*]sides = n/2 [*]Perimeter = n + 1.2n + 0.5n + 0.5n = 54.4 [/LIST] Solve for [I]n[/I] in the equation n + 1.2n + 0.5n + 0.5n = 54.4 [SIZE=5][B]Step 1: Group the n terms on the left hand side:[/B][/SIZE] (1 + 1.2 + 0.5 + 0.5)n = 3.2n [SIZE=5][B]Step 2: Form modified equation[/B][/SIZE] 3.2n = + 54.4 [SIZE=5][B]Step 3: Divide each side of the equation by 3.2[/B][/SIZE] 3.2n/3.2 = 54.4/3.2 n = [B]17[/B] [URL='https://www.mathcelebrity.com/1unk.php?num=n%2B1.2n%2B0.5n%2B0.5n%3D54.4&pl=Solve']Source[/URL]

A traveler is walking on a moving walkway in an airport. the traveler must walk back on the walkway
A traveler is walking on a moving walkway in an airport. the traveler must walk back on the walkway to get a bag he forgot. the traveler's ground speed is 2 ft/s against the walkway and 6 ft/s with the walkway. what is the traveler's speed off the walkway? What is the speed of the moving walkway. We have two equations, where w is the speed of the walkway and t is the speed of the traveler. [LIST=1] [*]t - w = 2 [*]t + w = 6 [*]Rearrange (1) to solve for t: t = w + 2 [/LIST] Now plug (3) into (2) (w + 2) + w = 6 Combine like terms: 2w + 2 = 6 Using our [URL='http://www.mathcelebrity.com/1unk.php?num=2w%2B2%3D6&pl=Solve']equation solver[/URL], we get [B]w = 2[/B] Plug this into (1) t - 2 = 2 Add 2 to each side [B]t = 4[/B]

A tree grows 35 cm in 2 years. If it continues to grow at the same rate determine how long it would
A tree grows 35 cm in 2 years. If it continues to grow at the same rate determine how long it would take to grow 85 cm We set up a proportion of cm to years where y is the number of years it takes to grow 85 cm: 35/2 = 85/y To solve this proportion for y, [URL='https://www.mathcelebrity.com/prop.php?num1=35&num2=85&den1=2&den2=y&propsign=%3D&pl=Calculate+missing+proportion+value']we type it in our search engine[/URL] and we get: [B]y = 4.86[/B]

A tree is 23.1 feet tall. What is its height in meters ? Use the following conversion: 1 meter is 3.
A tree is 23.1 feet tall. What is its height in meters ? Use the following conversion: 1 meter is 3.3 feet 23.1 feet * 1 meter / 3.3 feet = [B]7 meters[/B]

A trench is 40 feet long and the trencher goes 2 feet per minute. How long does it take to dig the
A trench is 40 feet long and the trencher goes 2 feet per minute. How long does it take to dig the trench? 2 feet per minute * x minutes = 40 feet Divide each side by 2 [B]x = 20 minutes[/B]

a triangle has side lengths of 12,16, and 20 centimeters. is it a right triangle?
a triangle has side lengths of 12,16, and 20 centimeters. is it a right triangle? First, we see if we can simplify. So we [URL='https://www.mathcelebrity.com/gcflcm.php?num1=12&num2=16&num3=20&pl=GCF']type GCF(12,16,20) [/URL]and we get 4. We divide the 3 side lengths by 4: 12/4 = 3 16/4 = 4 20/4 = 5 And lo and behold, we get a Pythagorean Triple of 3, 4, 5. So [B]yes, this is a right triangle[/B].

A truck driver took 7 hours and 45 minutes to travel 426.25 miles. What was the average speed of the
A truck driver took 7 hours and 45 minutes to travel 426.25 miles. What was the average speed of the truck driver? 45/60 = 0.75 of an hour 7 hours and 45 minutes = 7.75 hours 426.25 miles / 7.75 hours miles = [B]55 miles per hour[/B]

A turtle and rabbit are in a race to see who is the first to reach a point 100 feet away. The turtle
A turtle and rabbit are in a race to see who is the first to reach a point 100 feet away. The turtle travels at a constant speed of 20 feet per minute for the entire 100 feet. The rabbit travels at a constant speed of 40 feet per minute for the first 50 feet, stops for 3 minutes, and then continuous at a constant speed of 40 feet per minute for the last 50 feet. (i) Determine which animal won the race. (ii). By how much time the animal won the race. (iii) Explain one life lesson from the race. We know the distance formula is: d = rt For the turtle, he has a rate (r) of 20 feet / minute and distance (d) of 100. We want to solve for time: [URL='https://www.mathcelebrity.com/drt.php?d=+100&r=+20&t=&pl=Calculate+the+missing+Item+from+D%3DRT']Using our distance rate time calculator solving for t[/URL], we get: t = 5 The rabbit has 3 parts of the race: Rabbit Part 1: Distance (d) = 50 and rate (r) = 40 [URL='https://www.mathcelebrity.com/drt.php?d=50&r=40&t=+&pl=Calculate+the+missing+Item+from+D%3DRT']Using our distance rate time calculator solving for t[/URL], we get: t = 1.25 Rabbit Part 2: The rabbit stops for 3 minutes (t = 3) Rabbit Part 3: Distance (d) = 50 and rate (r) = 40 [URL='https://www.mathcelebrity.com/drt.php?d=50&r=40&t=+&pl=Calculate+the+missing+Item+from+D%3DRT']Using our distance rate time calculator solving for t[/URL], we get: t = 1.25 Total time for the rabbit from the 3 parts is (t) = 1.25 + 3 + 1.25 Total time for the rabbit from the 3 parts is (t) = 5.5 [LIST] [*](i) The [B]turtle won[/B] the race because he took more time to finish and they both started at the same time [*](ii) We subtract the turtles time from the rabbit's time: 5.5 - 5 = [B]0.5 minutes which is also 30 seconds[/B] [*](iii) [B]Slow and Steady wins the race[/B] [/LIST]

A tv is originally priced at $69.99 is reduced to $42.50. Find the % decrease in price
A tv is originally priced at $69.99 is reduced to $42.50. Find the % decrease in price Using our [URL='https://www.mathcelebrity.com/markup.php?p1=+69.99&m=+&p2=42.50&pl=Calculate']markdown calculator[/URL], we get: [B]-39.28%[/B]

A TV that usually sells for $192.94 is on sale for 15% off. If sales tax on the TV is 6%, what is th
A TV that usually sells for $192.94 is on sale for 15% off. If sales tax on the TV is 6%, what is the price of the TV, including tax? Find the discounted price: 15% off of 192.94 Discounted Price = 192.94 * (1 - 0.15) <-- 15% as a decimal is 0.15, and 1 is 100%, so we subtract to get 85% of the original price Discounted Price =192.94(0.85) Discounted Price = $164 Now, add in the sales tax of 6% to the Discounted Price Price after sales tax = Discounted Price * 1.06 Price after sales tax = $164 * 1.06 [B]Price after sales tax = $173.84[/B]

A typical adult has an average IQ score of 105 with a standard deviation of 20. If 20 randomly selec
A typical adult has an average IQ score of 105 with a standard deviation of 20. If 20 randomly selected adults are given an IQ test, what is the probability that the sample mean scores will be between 85 and 125 points? For x = 125, our z-score and probability is seen [URL='http://www.mathcelebrity.com/probnormdist.php?xone=+125&mean=+105&stdev=+20&n=+1&pl=P%28X+%3C+Z%29']here[/URL] Z = 1 P(x < 1) = 0.841345 For x = 85, our z-score and probability is seen [URL='http://www.mathcelebrity.com/probnormdist.php?xone=+85&mean=+105&stdev=+20&n=+1&pl=P%28X+%3C+Z%29']here[/URL] Z = -1 P(x < -1) = 0.158655 So what we want is the probability between these values:
0.841345 - 0.158655 = [B]0.68269[/B]

A typical human adult weighs 150 pounds, while a human newborn weighs approximately 7 pounds. An adu
A typical human adult weighs 150 pounds, while a human newborn weighs approximately 7 pounds. An adult female Western Grey Kangaroo weighs about 30 kilograms and gives birth to babies who are approximately one gram. If human babies were proportionally the same weight as adults as Western Grey Kangaroos babies, how much would a human newborn weigh? Set up a proportion of adult weight to baby weight where n is the weight of a human baby: 30000/1 = 150/n Using our [URL='https://www.mathcelebrity.com/proportion-calculator.php?num1=30000&num2=150&den1=1&den2=n&propsign=%3D&pl=Calculate+missing+proportion+value']proportion calculator,[/URL] we get: n = [B]0.005 or 1/200 of a pound[/B]

A typist is paid a basic wage of $22.50 per hour for a 40-hour week. Calculate the typist's basic we
A typist is paid a basic wage of $22.50 per hour for a 40-hour week. Calculate the typist's basic weekly wage Basic Weekly Wage = Hourly Rate * Hours Worked Basic Weekly Wage = $22.50 * 40 Basic Weekly Wage = [B]$900[/B]

A U ? = A
A U ? = A Let x ? [I]S[/I], where [I]S[/I] is the universal set. First we show that if A ? Ø ? A. Let x ? A ? Ø. Then x ? A or x ? Ø. by definition of the empty set, x cannot be an element in Ø. So by assumption, x ? A ? Ø, x must be in A. So A ? Ø ? A. Next, we show that A ? A ? Ø. This is true because the set resulting from the union of two sets contains both of the sets forms the union Since A ? Ø ? A and A ? A ? Ø, we have that A ? Ø = A.

A used automobile dealership recently reduced the price of a used compact car by 6%. If the price of
A used automobile dealership recently reduced the price of a used compact car by 6%. If the price of the car before discount was $18,100, find the discount and the new price. Using our [URL='http://www.mathcelebrity.com/markup.php?p1=&m=+6&p2=++18100&pl=Calculate']discount calculator[/URL], we get: [B]Discount = $1,086 New Price = $17,014[/B]

A used automobile dealership recently reduced the price of a used compact car by 6%. If the price of
A used automobile dealership recently reduced the price of a used compact car by 6%. If the price of the car before discount was 18,400, find the discount and the new price. First, find the discount amount: Discount Amount = 6% * 18,400 = [B]1,104 [/B] [U]Calculate discounted price:[/U] Discounted Price = Full Price - Discount Amount Discounted Price = 18,400 - 1,104 Discounted Price = 18,400 - 1,104 = [B]17,296[/B]

A used book store also started selling used CDs and videos. In the first week, the store sold a comb
A used book store also started selling used CDs and videos. In the first week, the store sold a combination of 40 CDs and videos. They charged $4 per CD and $6 per video and the total sales were $180. Determine the total number of CDs and videos sold Let c be the number of CDs sold, and v be the number of videos sold. We're given 2 equations: [LIST=1] [*]c + v = 40 [*]4c + 6v = 180 [/LIST] You can solve this system of equations three ways: [LIST] [*][URL='https://www.mathcelebrity.com/simultaneous-equations.php?term1=c+%2B+v+%3D+40&term2=4c+%2B+6v+%3D+180&pl=Substitution']Substitution Method[/URL] [*][URL='https://www.mathcelebrity.com/simultaneous-equations.php?term1=c+%2B+v+%3D+40&term2=4c+%2B+6v+%3D+180&pl=Elimination']Elimination Method[/URL] [*][URL='https://www.mathcelebrity.com/simultaneous-equations.php?term1=c+%2B+v+%3D+40&term2=4c+%2B+6v+%3D+180&pl=Cramers+Method']Cramers Rule[/URL] [/LIST] No matter what method we choose, we get [B]c = 30, v = 10[/B]. Now let's check our work for both given equations for c = 30 and v = 10: [LIST=1] [*]30 + 10 = 40 <-- This checks out [*]4c + 6v = 180 --> 4(30) + 6(10) --> 120 + 60 = 180 <-- This checks out [/LIST]

A used book store also started selling used CDs and videos. In the first week, the store sold a comb
A used book store also started selling used CDs and videos. In the first week, the store sold a combination of 40 CDs and videos. They charged $4 per CD and $6 per video and the total sales were $180. Determine the total number of CDs and videos sold. Let the number of cd's be c and number of videos be v. We're given two equations: [LIST=1] [*]c + v = 40 [*]4c + 6v = 180 [/LIST] We can solve this system of equations using 3 methods: [LIST] [*][URL='https://www.mathcelebrity.com/simultaneous-equations.php?term1=c+%2B+v+%3D+40&term2=4c+%2B+6v+%3D+180&pl=Substitution']Substitution Method[/URL] [*][URL='https://www.mathcelebrity.com/simultaneous-equations.php?term1=c+%2B+v+%3D+40&term2=4c+%2B+6v+%3D+180&pl=Elimination']Elimination Method[/URL] [*][URL='https://www.mathcelebrity.com/simultaneous-equations.php?term1=c+%2B+v+%3D+40&term2=4c+%2B+6v+%3D+180&pl=Cramers+Method']Cramer's Rule[/URL] [/LIST] No matter which method we choose, we get the same answer: [B]c = 30 v = 10[/B]

A used bookstore sells paperback fiction books in excellent condition for $2.50 and in fair conditio
A used bookstore sells paperback fiction books in excellent condition for $2.50 and in fair condition for $0.50. Write an expression for the cost of buying x excellent-condition paperbacks and f fair-condition paperbacks. Cost = Price * Quantity, so we have: [B]2.50x + 0.50f[/B]

A varies directly as B and inversely as C.
A varies directly as B and inversely as C. There exists a constant k such that: [B]a = kb/c [/B] Inversely means we divide by and directly means we multiply by

a varies directly with b and inversely with c
a varies directly with b and inversely with c Direct variation means we multiply. Inverse variation means we divide. There exists a constant k such that: [B]a = kb/c[/B]

A vehicle purchased for $25,000 depreciates at a constant rate of 5%. Determine the approximate valu
A vehicle purchased for $25,000 depreciates at a constant rate of 5%. Determine the approximate value of the vehicle 11 years after purchase. Round to the nearest whole dollar. Depreciation at 5% means it retains 95% of the value. Set up the depreciation equation to get Book Value B(t) at time t. B(t) = $25,000 * (1 - 0.05)^t Simplifying, this is: B(t) = $25,000 * (0.95)^t The problem asks for B(11) B(11) = $25,000 * (0.95)^11 B(11) = $25,000 * 0.5688 B(11) = [B]$14,220[/B]

A vendor sells h hot dogs and s sodas. If a hot dog costs twice as much as a soda, and if the vendor
A vendor sells h hot dogs and s sodas. If a hot dog costs twice as much as a soda, and if the vendor takes in a total of d dollars, how many cents does a soda cost? Let the cost of the soda be p. So the cost of a hot dog is 2p. The total cost of hot dogs: 2hp The total cost of sodas: ps The total cost of both equals d. So we set the total cost of hots dogs plus sodas equal to d: 2hp + ps = d We want to know the cost of a soda (p). So we have a literal equation. We factor out p from the left side: p(2h + s) = d Divide each side of the equation by (2h + s) p(2h + s)/(2h + s) = d/(2h + s) Cancel the (2h + s) on the left side, we get: p = [B]d/(2h + s[/B])

A vertical line that passes through the point (3, -2). Identify TWO additional points on the line.
A vertical line that passes through the point (3, -2). Identify TWO additional points on the line. A vertical line runs straight up, so the x-coordinate is always the same. We use x = 3 and any y point: (3, -1) (3, 0) (3, 1)

A vial contains 150 cc of penicillin. How many 5 cc injections can be administered from the vial?
A vial contains 150 cc of penicillin. How many 5 cc injections can be administered from the vial? 150cc / 5cc per injection = [B]30 injections[/B]

A video store charges a monthly membership fee of $7.50, but the charge to rent each movie is only $
A video store charges a monthly membership fee of $7.50, but the charge to rent each movie is only $1.00 per movie. Another store has no membership fee, but it costs $2.50 to rent each movie. How many movies need to be rented each month for the total fees to be the same from either company? Set up a cost function C(m) where m is the number of movies you rent: C(m) = Rental cost per movie * m + Membership Fee [U]Video Store 1 cost function[/U] C(m) = 1m + 7.5 Video Store 2 cost function: C(m) = 2.50m We want to know when the costs are the same. So we set each C(m) equal to each other: m + 7.5 = 2.50m To solve this equation for m, [URL='https://www.mathcelebrity.com/1unk.php?num=m%2B7.5%3D2.50m&pl=Solve']we type it in our search engine[/URL] and we get: m = [B]5[/B]

A virus is spreading exponentially. The initial amount of people infected is 40 and is increasing at
A virus is spreading exponentially. The initial amount of people infected is 40 and is increasing at a rate of 5% per day. How many people will be infected with the virus after 12 days? We have an exponential growth equation below V(d) where d is the amount of days, g is the growth percentage, and V(0) is the initial infected people: V(d) = V(0) * (1 + g/100)^d Plugging in our numbers, we get: V(12) = 40 * (1 + 5/100)^12 V(12) = 40 * 1.05^12 V(12) = 40 * 1.79585632602 V(12) = 71.8342530409 or [B]71[/B]

A washer and a dryer cost 600 combined. The cost of the washer is 3 times the cost of the dryer. Wha
A washer and a dryer cost 600 combined. The cost of the washer is 3 times the cost of the dryer. What is the cost of the dryer? Let w be the cost of the washer. Let d be the cost of the dryer. We have 2 given equations: [LIST=1] [*]w + d = 600 [*]w = 3d [/LIST] Substitute (2) into (1) (3d) + d = 600 4d = 600 [URL='http://www.mathcelebrity.com/1unk.php?num=4d%3D600&pl=Solve']Run it through our equation calculator[/URL], to get [B]d = 150[/B].

A watch was bought for $250 and sold for $375. What was the profit on the sale of the watch?
A watch was bought for $250 and sold for $375. What was the profit on the sale of the watch? Profit = Revenue (Sales) - Cost Profit = $375 - $250 Profit = [B]$125[/B]

A water tank holds 236 gallons but is leaking at a rate of 3 gallons per week. A second water tank h
A water tank holds 236 gallons but is leaking at a rate of 3 gallons per week. A second water tank holds 354 gallons but is leaking at a rate if 5 gallons per week. After how many weeks will the amount of water in the two tanks be the same Let w be the number of weeks of leaking. We're given two Leak equations L(w): [LIST=1] [*]L(w) = 236 - 3w [*]L(w) = 354 - 5w [/LIST] When the water in both tanks is the same, we can set both L(w) equations equal to each other: 236 - 3w = 354 - 5w To solve this equation for w, we [URL='https://www.mathcelebrity.com/1unk.php?num=236-3w%3D354-5w&pl=Solve']type it in our search engine[/URL] and we get: w = [B]59[/B]

a well driller charges $9.00 per foot for the first 10 feet, 9.10 per foot for the next 10 feet, $9.
a well driller charges $9.00 per foot for the first 10 feet, 9.10 per foot for the next 10 feet, $9.20 per foot for the next 10 feet, and so on, at a price increase of $0.10 per foot for succeeding intervals of 10 feet. How much does it cost to drill a well to a depth of 150 feet? Set up the cost function C(f) where f is the number of feet: Cost = 9(10) + 9.1(10) + 9.2(10) + 9.3(10) + 9.4(10) + 9.5(10) + 9.6(10) + 9.7(10) + 9.8(10) + 9.9(10) + 10(10) + 10.1(10) + 10.2(10) + 10.3(10) + 10.4(10) Cost = [B]1,455[/B]

A wide receiver sprints at a speed of 8.6 feet per second. How many feet would he expect the wide re
A wide receiver sprints at a speed of 8.6 feet per second. How many feet would he expect the wide receiver to run in 25 seconds? 8.6 feet per second * 25 seconds = [B]212.5 feet[/B]

A wildlife reserve has a population of 180 elephants. A group of researchers trapped 60 elephants an
A wildlife reserve has a population of 180 elephants. A group of researchers trapped 60 elephants and recorded their vital statistics. Of the trapped elephants, 12 were female. If that rate holds true for the entire population of 180 elephants, how many female elephants are on the wildlife reserve? Set up a proportion of female to trapped elephants: 12/60 = f/180 Using our [URL='http://www.mathcelebrity.com/prop.php?num1=12&num2=f&den1=60&den2=180&propsign=%3D&pl=Calculate+missing+proportion+value']proportion calculator[/URL], we see that f = [B]36[/B]

A woman dies at the age of 100 and her son is 35 years old how old was she when she gave birth to hi
A woman dies at the age of 100 and her son is 35 years old how old was she when she gave birth to him. 35 years ago meant she was 100 - 35 = [B]65 years[/B].

A woman is one-half as old as her mother. The sum of their ages is 75. What are their ages?
A woman is one-half as old as her mother. The sum of their ages is 75. What are their ages? Let the woman's age be w. Let the mother's age be m. We're given two equations: [LIST=1] [*]w = m/2 [*]m + w = 75 [/LIST] Substitute equation (1) into equation (2) for w: m + m/2 = 75 To solve for m, we [URL='https://www.mathcelebrity.com/1unk.php?num=m%2Bm%2F2%3D75&pl=Solve']type this equation into our search engine [/URL]and we get: m = [B]50 [/B] To solve for w, we plug m = 50 into equation (1): w = 50/2 w = [B]25[/B]

A woman walked for 5 hours, first along a level road, then up a hill, and then she turned around and
A woman walked for 5 hours, first along a level road, then up a hill, and then she turned around and walked back to the starting point along the same path. She walks 4mph on level ground, 3 mph uphill, and 6 mph downhill. Find the distance she walked. Hint: Think about d = rt, which means that t = d/r. Think about each section of her walk, what is the distance and the rate. You know that the total time is 5 hours, so you know the sum of the times from each section must be 5. Let Level distance = L and hill distance = H. Add the times it took for each section of the walk: L/4 + H /3 + H/6 + L/4 = 5 The LCD of this is 12 from our [URL='http://www.mathcelebrity.com/gcflcm.php?num1=4&num2=3&num3=6&pl=LCM']LCD Calculator[/URL] [U]Multiply each side through by our LCD of 12[/U] 3L + 4H + 2H + 3L = 60 [U]Combine like terms:[/U] 6L + 6H = 60 [U]Divide each side by 3:[/U] 2L + 2H = 20 The woman walked [B]20 miles[/B]

A woman whose income for the year was $42,800 paid $10,700 in taxes. What percent of her income did
A woman whose income for the year was $42,800 paid $10,700 in taxes. What percent of her income did she pay in taxes? Tax Percent = Tax Amount / Income * 100% Tax Percent = $10,700 / $42,800 * 100% Tax Percent = 0.25 * 100% Tax Percent = [B]25%[/B]

A wood screw advances 1/16 inch for each complete turn. How far will the screw advance in 8 complete
A wood screw advances 1/16 inch for each complete turn. How far will the screw advance in 8 complete turns 1/6 inch per turn x 8 complete turns = 8/16. Enter 8/16 into the search engine. Choose simplify. Using our [URL='http://www.mathcelebrity.com/fraction.php?frac1=8%2F16&frac2=3%2F8&pl=Simplify']simplify fraction calculator,[/URL] we get 8/16 simplified is [B]1/2[/B] of a turn.

a writer can write a novel at a rate of 3 pages per 5 hour work. if he wants to finish the novel in
a writer can write a novel at a rate of 3 pages per 5 hour work. if he wants to finish the novel in x number of pages, determine a function model that will represent the accumulated writing hours to finish his novel if 3 pages = 5 hours, then we divide each side by 3 to get: 1 page = 5/3 hours per page So x pages takes: 5x/3 hours Our function for number of pages x is: [B]f(x) = 5x/3[/B]

A yard is 33.21 meters long and 17.6 meters wide. What length of fence must be purchased to enclose
A yard is 33.21 meters long and 17.6 meters wide. What length of fence must be purchased to enclose the entire yard? The yard is a rectangle. The perimeter of a rectangle is: P = 2l + 2w where l is the length and w is the width. Evaluating, using our [URL='https://www.mathcelebrity.com/rectangle.php?l=33.21&w=17.6&a=&p=&pl=Calculate+Rectangle']rectangle calculator[/URL], we get P = [B]101.62[/B]

A yard with dimensions of 15m x 10m has a flower garden in the middle. The flower garden has a dimen
A yard with dimensions of 15m x 10m has a flower garden in the middle. The flower garden has a dimensions of 4m x 7m. What Is the area of the yard without the flower garden? Find the area of the yard: AY = l x w AY = 15 x 10 AFY= 150 Find the area of the flower garden: AFG = l x w AFG = 7 x 14 AFG = 28 Take the area of the remaining piece of the flower garden: ARP = AY - AFG A = 150 - 28 [B]A = 122[/B]

A yardstick casts a shadow of 8 inches. At the same time, a tree casts a shadow of 52 feet. How tall
A yardstick casts a shadow of 8 inches. At the same time, a tree casts a shadow of 52 feet. How tall is the tree? Setup a proportion of height to shadow distance where h is the height of the tree: 36/8 = h/52 Using our [URL='https://www.mathcelebrity.com/proportion-calculator.php?num1=36&num2=h&den1=8&den2=52&propsign=%3D&pl=Calculate+missing+proportion+value']proportion calculator[/URL], we get: h = [B]234 feet[/B]

A yoga member ship costs $16 and additional $7 per class. Write a linear equation modeling the cost
A yoga member ship costs $16 and additional $7 per class. Write a linear equation modeling the cost of a yoga membership? Set up the cost function M(c) for classes (c) [B]M(c) = 16 + 7c[/B]

A young dad, who was a star football player in college, set up a miniature football field for his fi
A young dad, who was a star football player in college, set up a miniature football field for his five-year-old young daughter, who was already displaying an unusual talent for place-kicking. At each end of the mini-field, he set up goal posts so she could practice kicking extra points and field goals. He was very careful to ensure the goalposts were each straight up and down and that the crossbars were level. On each set, the crossbar was six feet long, and a string from the top of each goalpost to the midpoint between them on the ground measured five feet. How tall were the goalposts? How do you know this to be true? The center of each crossbar is 3 feet from each goalpost. We get this by taking half of 6, since midpoint means halfway. Imagine a third post midway between the two goal posts. It has the same height as the two goalposts. From the center post, the string from the top of a goalpost to the base of the center post, and half the crossbar form and right triangle with hypotenuse 5 feet and one leg 3 feet. [URL='https://www.mathcelebrity.com/pythag.php?side1input=&side2input=3&hypinput=5&pl=Solve+Missing+Side']Using the Pythagorean Theorem[/URL], the other leg -- the height of each post -- is 4 feet.

A young snake measures 0.23 meters long. During the course of his lifetime,he will grow to be 13 tim
A young snake measures 0.23 meters long. During the course of his lifetime,he will grow to be 13 times his current length what will be his length be when he is full grown Full Grown Length = Current Length * Growth Multiplier Full Grown Length = 0.23 * 13 Full Grown Length = [B]2.99 meters[/B]

A zoo has 15 Emperor penguins. The Emperor penguins make up 30% percent of all the penguins at the z
A zoo has 15 Emperor penguins. The Emperor penguins make up 30% percent of all the penguins at the zoo. How many penguins live at the zoo? Let p be the total number penguins at the zoo. We're told: 30% of p = 15 Since 30% = 0.3, we have: 0.3p = 15 Solve for [I]p[/I] in the equation 0.3p = 15 [SIZE=5][B]Step 1: Divide each side of the equation by 0.3[/B][/SIZE] 0.3p/0.3 = 15/0.3 p = [B]50[/B]

a ^5 x a ^2 without exponents
a ^5 x a ^2 without exponents When we multiply the same variable or number, we add exponents, so we have: a^(5 + 2) a^7 To write a variable raised to an exponent without exponents, we break it up. The formula to do this is: a^n = a times itself n times a^7 = [B]a * a * a * a * a * a * a[/B]

A ________ ________ is the value of a statistic that estimates the value of a parameter.
A ________ ________ is the value of a statistic that estimates the value of a parameter. [B]Point Estimate[/B]. A point [B]estimate[/B] is a single [B]value[/B] (statistic) used to [B]estimate[/B] a population [B]value[/B]([B]parameter[/B])

a/m - b = c for m
a/m - b = c for m Add b to both sides: a/m - b + b = c + b Cancel b on both sides: a/m = c + b Multiply each side by m: am/m = m(c + b) Cancel the m's on the left side: a = m(c + b) Divide each side by (c + b) a/(c + b) = m(c + b)/(c + b) Cancel the (c + b) on the right side, and we get: m[B] = a/(c + b)[/B]

A=2(l+w) for w
Multiply through using the distributive property, so we have: A = 2l + 2w Subtract 2l from each side 2w = A - 2l Divide each side by w w = (A - 2l)/2 [MEDIA=youtube]Nm-tYD4aEY4[/MEDIA]

A=a+b+c+d÷4 for c
A=a+b+c+d÷4 for c Assume A and a are different variables: Cross multiply: a + b + c + d = 4A Subtract a, b, and d from each side: a + b + c + d - (a + b + d) = 4A - (a + b + d) Cancel the a + b + d on the left side [B]c = 4A - a - b - d[/B]

A={2,8,1} and B={4,3,1}.find the Cartesian product A×B.
A={2,8,1} and B={4,3,1}.find the Cartesian product A×B. Click [URL='http://www.mathcelebrity.com/cartprod.php?num1=2%2C8%2C1&num2=4%2C3%2C1&pl=Cartesian+Product']here[/URL] to find the answer

Aaron bought a bagel and 3 muffins for $7.25. Bea bought a bagel and 2 muffins for $6. How much is b
Aaron bought a bagel and 3 muffins for $7.25. Bea bought a bagel and 2 muffins for $6. How much is bagel and how much is a muffin? Let b be the number of bagels and m be the number of muffins. We have two equations: [LIST=1] [*]b + 3m = 7.25 [*]b + 2m = 6 [/LIST] Subtract (2) from (1) [B]m = 1.25 [/B] Plug this into (2), we have: b + 2(1.25) = 6 b + 2.5 = 6 Subtract 2.5 from each side [B]b = 3.5[/B]

Aaron bought a guitar for n dollars. The tax in his state is 6%. What is the total cost of the guita
Aaron bought a guitar for n dollars. The tax in his state is 6%. What is the total cost of the guitar including tax? Sale price is n Tax on sale is 0.06n Add them together n + 0.06n = [B]1.06n[/B]

Aaron buys a bag of cookies that contains 8 chocolate chip cookies, 6 peanut butter cookies,7 sugar
Aaron buys a bag of cookies that contains 8 chocolate chip cookies, 6 peanut butter cookies,7 sugar cookies and 6 oatmeal raisin cookies. What it’s the probability that Aaron randomly selects a peanut butter cookie from the bag, eats it,, then randomly selects another peanut butter cookie? First draw out of the bag is a peanut butter cookie: P(PB) = Total Peanut Butter Cookies / Total Cookies P(PB) = 6/27 Second draw out of the bag is a peanut butter cookie, but we have one less since Aaron ate one: P(PB) = Total Peanut Butter Cookies - 1 / Total Cookies - 1 P(PB) = (6 - 1)/(27 - 1) P(PB) = 5/26 Now, since each event is independent, we multiply them to see the probability of choosing a peanut butter cookie, eating it, then reaching in and choosing another peanut butter cookie: P(PB, PB) = 6/27 * 5/26 [URL='https://www.mathcelebrity.com/fraction.php?frac1=6%2F27&frac2=5%2F26&pl=Multiply']P(PB, PB)[/URL] = [B]5/117[/B]

Aaron is staying at a hotel that charges $99.95 per night plus tax for a room. A tax of 8% is applie
Aaron is staying at a hotel that charges $99.95 per night plus tax for a room. A tax of 8% is applied to the room rate, and an additional onetime untaxed fee of $5.00 is charged by the hotel. Which of the following represents Aaron’s total charge, in dollars, for staying [I]x[/I] nights? [LIST] [*]The Room cost equals 99.95 times x where x is the number of rooms [*]We express an 8% tax by multiplying the room cost by 1.08 [*]Finally, we add on $5, which is [I]untaxed[/I] [/LIST] [I][/I] Take this in pieces: Room Cost: 99.95x Tax on Room 1.08(99.95x) Add on $5 which is untaxed: [B]1.08(99.95x) + 5[/B]

ab/d + c = e for d
ab/d + c = e for d I know this is a literal equation because we are asked to solve for a variable [U]in terms of[I] another variable [/I][/U] Subtract c from each side to isolate the d term: ab/d + c - c = e - c Cancel the c's on the left side and we get: ab/d = e - c Cross multiply: ab = d(e - c) Divide each side of the equation by (e - c): ab/(e - c)= d(e - c)/(e - c) Cancel the (e - c) on the right side, and we get: d = [B]ab/(e - c)[/B]

Abbey knew that the combination for her locker had the numbers 36, 12, 8, and 40, but she couldn't r
Abbey knew that the combination for her locker had the numbers 36, 12, 8, and 40, but she couldn't remember the right order of the numbers. How many different possibilities are there for the lock combination using the four numbers? First number could be 4 choices, then 3, then 2, then 1. So we have: 4! = 4 x 3 x 2 x 1 = [B]24 possibilities[/B]

About 3/5th of the registered voters participated in 2016 election. California has 25 million regist
About 3/5th of the registered voters participated in 2016 election. California has 25 million registered voters. Find the number of registered voters who participated in 2016 election. 3[URL='https://www.mathcelebrity.com/fraction.php?frac1=25000000&frac2=3/5&pl=Multiply']/5 of 25000000[/URL] = [B]15,000,000[/B]

About the Book
Free Traffic Frenzy: How To Get 450,000 Website Visitors is my 2nd book. It covers 11 years of traffic building tips. I show you all the juicy details and secrets behind the growth of my website to 450,000 monthy visitors...without spending a dime. [URL='https://www.amazon.com/dp/B07BR6G5KC']Check it out on Amazon[/URL].

Absolute Difference
Free Absolute Difference Calculator - Calculates the absolute difference between 2 numbers

absolute value of x is less than or equal to 4
absolute value of x is less than or equal to 4 Absolute value of x: |x| Set up an inequality where this is less than or equal to 4: [B]|x| <= 4 [/B] <-- This is our algebraic expression To solve this, we have the following compound inequality: -4 < x < 4

Absolute value of x less than 8
These are now available as shortcuts. You can type any number or variable in the following forms: [LIST] [*]Absolute value of x less than 8 [*]Absolute value of x less than or equal to 8 [*]Absolute value of x greater than 8 [*]Absolute value of x greater than or equal to 8 [*]Absolute value of x equal to 8 [/LIST]

Acceleration
Free Acceleration Calculator - Solves for any of the 4 items in the acceleration equation including initial velocity, velocity, and time.

According to the American Bureau of Labor Statistics, you will devote 32 years to sleeping and eatin
According to the American Bureau of Labor Statistics, you will devote 32 years to sleeping and eating. The number of years sleeping will exceed the number of years eating by 24. Over your lifetime, how many years will you spend on each of these activities? Assumptions: [LIST] [*]Let years eating be e [*]Let years sleeping be s [/LIST] We're given: [LIST=1] [*]s = e + 24 [*]e + s = 32 [/LIST] To solve this system of equations, we substitute equation (1) into equation (2) for s: e + e + 24 = 32 To solve this equation for e, we [URL='https://www.mathcelebrity.com/1unk.php?num=e%2Be%2B24%3D32&pl=Solve']type it in our math engine[/URL] and we get: e = [B]4 [/B] Now, we take e = 4 and substitute it into equation (1) to solve for s: s = 4 + 24 s = [B]28[/B]

Accounting Rate of Return
Free Accounting Rate of Return Calculator - Given an initial investment and a set of returns, this calculates the Accounting Rate of Return

Accuracy and Precision
Free Accuracy and Precision Calculator - Given an integer or decimal, this determines the precision and accuracy (scale)

Activity Method Depreciation
Free Activity Method Depreciation Calculator - Calculates the following: Depreciable Base, Depreciation per Unit, Depreciation for Period

acw+cz=y for a
acw+cz=y for a Solve this literal equation: Subtract cz from each side: acw + cz - cz = y - cz Cancel the cz on the left side: acw = y - cz Divide each side by cw to isolate a: acw/cw = (y - cz)/cw Cancel cw on the left side: [B]a = (y - cz)/cw[/B]

Adam ate 1/5 of a cake and Matt ate the rest. What fraction did Matt eat
Adam ate 1/5 of a cake and Matt ate the rest. What fraction did Matt eat? The rest of the cake is 1 - 1/5. 1 as a fraction is 5/5. So we have: 5/5 - 1/5 Using our f[URL='https://www.mathcelebrity.com/fraction.php?frac1=5%2F5&frac2=1%2F5&pl=Subtract']raction calculator[/URL], we get [B]4/5[/B]

Adam drove the 10 miles to school at a speed of 60 mph. On his way home, due to traffic, his speed w
Adam drove the 10 miles to school at a speed of 60 mph. On his way home, due to traffic, his speed was 30 mph. What was his average speed for the round trip to school and back? D = rt To school: 60 miles in 60 minutes = 10 miles in 10 minutes To home: 30 miles in 60 minutes = 10 miles in 20 minutes Total time: 10 + 20 = 30 minutes or 0.5 hours With a speed of s, we have: d = st 20 = 0.5s Divide each side by 2: s = [B]40 mph[/B]

Adam has 20 sweets he eats a quarter of them how many does he have left?
Adam has 20 sweets he eats a quarter of them how many does he have left? A quarter means 1/4. There's 2 ways you can approach this problem. [B][U]Approach #1:[/U][/B] Adam eats a quarter, or 1/4 of the sweets. So he eats: 20 * 1/4 = 5 Remaining sweets = Total Apples - Eaten Apples Remaining sweets = 0 - 5 Remaining sweets= 15 [U][B]Approach #2:[/B][/U] If Adam eats 1/4 of the sweets, this means he has: 1 - 1/4 sweets remaining. Since 1 equals 4/4, we have: 4/4 - 1/4 = 3/4 Therefore, he has 20 * 3/4 sweets remaining. This is 60/4, or [B]15[/B]

Adam took money from his savings account to use as spending money on a trip to San Antonio. On Monda
Adam took money from his savings account to use as spending money on a trip to San Antonio. On Monday, he spent half his money. On Tuesday, he sp ent half of what was left. On Wednesday, he again spent half of his remaining money. On Thursday, he work up with very little money left, but again spent half of it. If Adam started the vacation with n dollars, how much money did he have at the end of Thursday? [LIST] [*]Start with: n [*]Monday: n * 1/2 = n/2 [*]Tuesday: n/2 * 1/2 = n/4 [*]Wednesday: n/4 * 1/2 = n/8 [*]Thursday: n/8 * 1/2 = [B]n/16[/B] [/LIST]

Adam, Bethany, and Carla own a painting company. To paint a particular home, Adam estimates it woul
Adam, Bethany, and Carla own a painting company. To paint a particular home, Adam estimates it would take him 4 days. Bethany estimates 5.5 days. Carla estimates 6 days. How long would it take them to work together to paint the house. Our combined work function for time (t) using a = Adam's time, b = Bethany's time, and c = Carla's time is: 1/a + 1/b + 1/c = 1/t Plugging in a, b, and c, we get: 1/4 + 1/5.5 + 1/6 = 1/t 0.25 + 0.181818 + 0.1667 = 1/t 1/t = 0.59848 t = [B]1.67089 days[/B]

Add 2 and z, then subtract y from the result
Add 2 and z, then subtract y from the result Add 2 and z 2 + z Subtract y from the result: [B]2 + z - y[/B]

add 3 and 9, then multiply n by the result
add 3 and 9, then multiply n by the result Add 3 and 9 3 + 9 12 Then multiply n by the result: [B]12n[/B]

Add 3 to 6, subtract w from the result, then triple what you have
Add 3 to 6, subtract w from the result, then triple what you have Add 3 to 6; 3 + 6 Subtract w from the result; 3 + 6 - w Triple what you have (means multiply by 3): [B]3(3 + 6 - w)[/B]

add 4 to the sum of 5 and z
add 4 to the sum of 5 and z The sum of 5 and z 5 + z Add 4 to this 4 + (5 + z) Simplified: [B]9 + z[/B]

Add 5 and 6 and then multiply by 3
Add 5 and 6 and then multiply by 3 Add 5 and 6: (5 + 6) Then multiply by 3: [B]3(5 + 6) [/B] If you want to evaluate this term, then we [URL='https://www.mathcelebrity.com/distributive-property.php?a=3&b=5&c=6&pl=Distributive']type it into the math engine[/URL] and we get: [B]33[/B]

Add 5 to p, then divide the sum by 4
Add 5 to p, then divide the sum by 4 Add 5 to p: p + 5 Divide the sum by 4: [B](p + 5)/4 [/B] note: B[I]ecause this is a sum, we wrap it in parentheses to divide the sum by a number[/I]

Add 5 to the sum of 2x and y
Add 5 to the sum of 2x and y The sum of 2x and y means we add y to 2x: 2x + y Add 5 [B]2x + y + 5[/B]

add 7 and 2, raise the result to the 6th power, then add what you have to s
add 7 and 2, raise the result to the 6th power, then add what you have to s Add 7 and 2: 7 + 2 Simplify this, we get:9 Raise the result to the 6th power: 9^6 [URL='https://www.mathcelebrity.com/powersq.php?sqconst=+6&num=9%5E6&pl=Calculate']Simplifying this using our exponent calculator[/URL], we get: 531,441 Now, we add what we have (our result) to s to get our final algebraic expression: [B]s + 531,441[/B]

Add 7 to a, and divide the sum by b
Add 7 to a, and divide the sum by b Add 7 to a: a + 7 Divide the sum by b: [B](a + 7)/b[/B]

add 8 and 10 then divide u
add 8 and 10 then divide u Add 8 and 10 8 + 10 Divide by u (8 + 10)/u Simplified, it is 18/u

Add 8 and 7, and then multiply by 2.
Add 8 and 7, and then multiply by 2. Add 8 and 7: 8 + 7 Then multiply by 2: 2(8 + 7) If you want to evaluate this order of operations, then [URL='https://www.mathcelebrity.com/distributive-property.php?a=2&b=8&c=7&pl=Distributive']type it in our search engine[/URL] to get: [B]30[/B]

Add all the whole numbers 1 through 100
Add all the whole numbers 1 through 100 [URL='https://www.mathcelebrity.com/inclusnumwp.php?num1=1&num2=100&pl=Sum']Using our inclusive number word problem calculator[/URL], we get: 5,050

Add b and 9, then add c to the result
Add b and 9, then add c to the result Add b and 9 b + 9 Add c to the result [B]b + 9 + c[/B]

add c and b, multiply the result by a, then double what you have
add c and b, multiply the result by a, then double what you have Take this algebraic expression in pieces: [LIST] [*]add c and b: c + b [*]Multiply the result by a: a(c + b) [*]Double what you have means take the last step result, and multiply it by 2: [/LIST] [B]2a(c + b)[/B]

add c to b, subtract d from the result, then double what you have
add c to b, subtract d from the result, then double what you have Add c to b: b + c Subtract d from the result: b + c - d Double what you have means multiply the entire expression by 2: [B]2(b + c - d)[/B]

add c to d, multiply a by the result, then divide what you have by b
add c to d, multiply a by the result, then divide what you have by b Add c to d: d + c Multiply a by the result: a(d + c) then divide what you have by b: [B]a(d + c)/b[/B]

add c to d, then multiply 9 by the result
add c to d, then multiply 9 by the result Add c to d c + d Multiply 9 by the result: [B]9(c + d)[/B]

add d to 5, raise the result to the 9th power, then subtract what you have from 2
add d to 5, raise the result to the 9th power, then subtract what you have from 2 Add d to 5: d + 5 Raise the result to the 9th power means we raise (d + 5) to the 9th power using an exponent: (d + 5)^9 the subtract what we have (the result) from 2: [B]2 - (d + 5)^9[/B]

add f and g, then triple the result
add f and g, then triple the result Add f and g f + g Triple the result means we multiply f + g by 3 [B]3(f + g)[/B]

Add h and 7, then triple the result
Add h and 7, then triple the result Add h and 7 h + 7 Triple the result: [B]3(h + 7)[/B]

add p to 7, add the result to 10, then multiply 4 by what you have
add p to 7, add the result to 10, then multiply 4 by what you have Add p to 7: p + 7 Add the result to 10: p + 7 + 10 p + 17 <-- combine like terms Then multiply 4 by what you have: [B]4(p + 17)[/B]

Add q and t, subtract s from the result, then multiply by r
Add q and t, subtract s from the result, then multiply by r Take this algebraic expression in parts: [LIST] [*]Add q and t: q + t [*]Subtract s from the result: q + t - s [*]Multiply by r means we multiply the entire expression by r: [/LIST] [B]r(q + t - s)[/B]

Add q to p, add a to the result, then divide r by what you have
Add q to p, add a to the result, then divide r by what you have Add q to p: p + q Add a to the result: p + q + a Then divide r by what you have: [B]r/(p + q + a)[/B]

add r and q, divide the result by s, then triple what you have
add r and q, divide the result by s, then triple what you have Add r and q: r + q Divide the result by s. The result above is r + q, so we have: (r + q)/s Triple what you have means we multiply the expression above by 3: [B]3(r + q)/s[/B]

add r and s, add the result to q, then subtract what you have from p
add r and s, add the result to q, then subtract what you have from p Take this algebraic expression in 3 parts: [LIST=1] [*]Add r and s: r + s [*]Add the result to q: r + s + q [*]Subtract what we have from p: [/LIST] [B]p - (r + s + q)[/B]

add r to 3, triple the result, then divide s by what you have
add r to 3, triple the result, then divide s by what you have Take this algebraic expression in parts: [LIST=1] [*]Add r to 3: 3 + r [*]Triple the result means multiply the result above by 3: 3(3 + r) [*]Then divide s by what you have. [B]s/3(3 + r)[/B] [/LIST]

Add r to the difference of s and t, then add q to the result
Add r to the difference of s and t, then add q to the result the difference of s and t s - t Add r to the difference of s and t s - t + r Add r to the difference of s and t, then add q to the result [B]s - t + r + q[/B]

add s and 2 then subtract 4
add s and 2 then subtract 4 Add s and 2 s + 2 Subtract 4: [B]s + 2 - 4[/B]

add s and t, multiply the result by u, then add r to what you have
add s and t, multiply the result by u, then add r to what you have. Take this algebraic expression in 3 parts: [LIST=1] [*]Add s and t: s + t [*]Multiply the result by u means me multiply (s + t) times u: u(s + t) [*]Then add r to what you have. [I]what you have means the result in #2.[/I] [/LIST] [B]u(s + t) + r[/B]

add s to r, double the result
add s to r, double the result Add s to r: r + s Double the result means multiply r + s by 2: [B]2(r + s)[/B]

add s to r, subtract q from the result
add s to r, subtract q from the result Add s to r: r + s Subtract q from the result: [B]r + s - q[/B]

add s to v, multiply the result by u, then multiply t by what you have
add s to v, multiply the result by u, then multiply t by what you have Take this algebraic expression in parts: [LIST] [*]Add s to v: v + s [*]Multiply the result by u: u(v + s) [*]Then multiply t by what you have: [/LIST] [B]tu(v + s)[/B]

add t and r and double the result
add t and r and double the result Add t and r: t + r Double the result means multiply by 2: [B]2(t + r)[/B]

add the sum of 9 and q to r
add the sum of 9 and q to r The sum of 9 and q 9 + q Add this to r [B]9 + q + r[/B]

add u and t divide s by the result then triple what you have
add u and t divide s by the result then triple what you have Take this algebraic expression in parts: [LIST] [*]Add u and t: u + t [*]Divide s by the result: s/(u + t) [*]Triple what you have means we you multiply s/(u + t) by 3 [/LIST] [B]3s/(u + t)[/B]

Add u and w, triple the result, then add what you have to v
Add u and w, triple the result, then add what you have to v Add u and w u + w Triple the result means multiply the sum of u and w by 3: 3(u + w) Then add what you have to v: [B]v + 3(u + w)[/B]

add v to t, add the result to u, then subtract what you have from s
add v to t, add the result to u, then subtract what you have from s Add v to t: t + v Add the result to u: t + v + u Then subtract what you have from s: [B]s - (t + v + u)[/B]

add w and u, subtract t from the result
add w and u, subtract t from the result Add w and u: w + u Subtract t from the result: [B]w + u - t[/B]

add w to t, add u to the result, then divide what you have by v
add w to t, add u to the result, then divide what you have by v Take this algebraic expression in parts: [LIST] [*]Add w to t: t + w [*]Add u to the result: t + w + u [*]Divide what you have by v: [/LIST] ([B]t + w + u)/v[/B]

add w to u, triple the result, then add v to what you have
add w to u, triple the result, then add v to what you have Take this algebraic expression in parts: [LIST] [*]add w to u: w + u [*]triple the result means we multiply w + u by 3: 3(w + u) [*]Then add v to what you have [/LIST] [B]3(w + u) + v[/B]

add x and 3; then multiply by y
Add x and 3 x + 3 Then multiply by y (they mean the total) [B]y(x + 3)[/B]

Addition and Multiplication Tables (Times Tables)
Free Addition and Multiplication Tables (Times Tables) Calculator - Shows the color coded addition or multiplication table entries and answer for any 2 numbers 1-15.

Addition Equality Property
Free Addition Equality Property Calculator - Demonstrates the Addition Equality Property Numerical Properties

Addition Property Of Inequality
Free Addition Property Of Inequality Calculator - Demonstrates the Addition Property Of Inequality. Numerical Properties

Additive Identity Property
Free Additive Identity Property Calculator - Displays the line by line proof for the additive identity property Numerical Properties

Additive Inverse Property
Free Additive Inverse Property Calculator - Demonstrates the Additive Inverse property using a number. A + (-A) = 0 Numerical Properties

ADG,BEH,CFI,___,___,___
ADG,BEH,CFI,___,___,___ Looking at this pattern, we see: [LIST=1] [*]the first term starts with A and increments by 1 letter [*]the second term starts with D and increments by 1 letter [*]the third term starts with G and increments by 1 letter [/LIST] So terms 4, 5, and 6 are: [LIST] [*][B]DGJ[/B] [*][B]EHK[/B] [*][B]FIL[/B] [/LIST]

Admir works at a coffee shop and earns $9/hour he also works at a grocery store and earns $15/hour.
Admir works at a coffee shop and earns $9/hour he also works at a grocery store and earns $15/hour. Last week he earned $500 dollars. Write an equation that represents the situation. [LIST] [*]Let c be the hours Admir works at the coffee shop. [*]Let g be the hours Admir works at the grocery store. [/LIST] Since earnings equal hourly rate times hours, We have the following equation: [B]9c + 15g = 500[/B]

Admission to a baseball game is $2.00 for general admission and $5.50 for reserved seats. The recei
Admission to a baseball game is $2.00 for general admission and $5.50 for reserved seats. The receipts were $3577.00 for 1197 paid admissions. How many of each ticket were sold? Let g be the number of tickets for general admission Let r be the number of tickets for reserved seats We have two equations: [LIST=1] [*]g + r = 1197 [*]2g + 5.50r = 3577 [/LIST] We can solve this a few ways, but let's use substitution using our [URL='https://www.mathcelebrity.com/simultaneous-equations.php?term1=g+%2B+r+%3D+1197&term2=2g+%2B+5.50r+%3D+3577&pl=Substitution']simultaneous equations calculator[/URL]: [LIST] [*][B]r = 338[/B] [*][B]g = 859[/B] [/LIST]

admission to the school fair is $2.50 for students and $3.75 for others. if 2848 admissions were col
admission to the school fair is $2.50 for students and $3.75 for others. if 2848 admissions were collected for a total of 10,078.75, how many students attended the fair Let the number of students be s and the others be o. We're given two equations: [LIST=1] [*]o + s = 2848 [*]3.75o + 2.50s = 10078.75 [/LIST] Since we have no coefficients for equation 1, let's solve this the fast way using substitution. Rearrange equation 1 by subtracting o from each side to isolate s [LIST=1] [*]o = 2848 - s [*]3.75o + 2.50s = 10078.75 [/LIST] Now substitute equation 1 into equation 2: 3.75(2848 - s) + 2.50s =10078.75 To solve for s, we [URL='https://www.mathcelebrity.com/1unk.php?num=3.75%282848-s%29%2B2.50s%3D10078.75&pl=Solve']type this equation into our search engine[/URL] and we get: s = [B]481[/B]

Adrienne brings home $1580 per month. She spends $316 on food. What is the fraction of what she spen
Adrienne brings home $1580 per month. She spends $316 on food. What is the fraction of what she spends on food Using our [URL='https://www.mathcelebrity.com/fraction.php?frac1=316%2F1580&frac2=3%2F8&pl=Simplify']fraction calculator[/URL], we see that: 316/1580 = [B]1/5[/B]

Affine Cipher
Free Affine Cipher Calculator - Builds the Affine Cipher Translation Algorithm from a string given an a and b value

After 20 minutes, Juan had completed 12 questions, which is 0.7 of his assignment. What percent of t
After 20 minutes, Juan had completed 12 questions, which is 0.7 of his assignment. What percent of the assignment has Juan NOT completed? We know that 0.7 as a percentage is: 0.7 * 100% = 70% In this problem, we have either or. Juan either completed the question or DID NOT complete the question. 100% of questions has one of two classifications - Completed or not completed. This means Juan did not complete the following amount of questions: 100% - 70% = [B]30%[/B]

After 5 years, a car is worth $22,000. It’s value decreases by $1,500 a year, which of the following
After 5 years, a car is worth $22,000. It’s value decreases by $1,500 a year, which of the following equations could represent this situation? Group of answer choices Let y be the number of years since 5 years. Our Book value B(y) is: [B]B(y) = 22,000 - 1500y[/B]

After a 33 percent reduction, you purchase a television for $281.40. What was the televisions price
After a 33 percent reduction, you purchase a television for $281.40. What was the televisions price before the reduction? Using our [URL='http://www.mathcelebrity.com/markup.php?p1=++281.40&m=+33&p2=&pl=Calculate']markup/markdown calculator[/URL], we get: Original Sale Price = [B]$374.26[/B]

After a long journey, you finally arrive at the edge o a deep gorge where there are two identical br
After a long journey, you finally arrive at the edge o a deep gorge where there are two identical bridges from which to choose your path to the other side. One bridge is safe, while the other is very dangerous and has caused the deaths of hundreds of travelers. The owner of the first bridge is a talking rat, while the owner of the second bridge is a talking frog. Friends told you before you left that one of the bridge owners always tells the truth, while the other always lies. You are allowed one question to ask of either the frog or the rat to find out which bridge is the safe bridge. What is the question that you would ask? [B]Ask the frog the following question: "If I were to ask the rat which bridge is the same bridge, which one would he point to?" [/B] If the frog is the truth teller, he would tell you that the rat would point to the dangerous bridge. If the frog is the liar, the truth telling rat would point out the safe bridge, but the lying frog would tell you he said the dangerous bridge. In both situations, the dangerous bridge would be pointed to. Take the other bridge.

After an explosion of an oxygen tank, three astronauts were forced to move into the lunar lander tha
After an explosion of an oxygen tank, three astronauts were forced to move into the lunar lander that was attached to their spacecraft. If the lunar lander had enough oxygen to last 2 people for 6 days, is there enough oxygen for 3 astronauts if their journey back to earth will take 4 days? 2 people * 6 days = 12 days of oxygen for 1 person. 12 days of oxygen per person / 3 astronauts = [B]4 days of oxygen The answer is yes, there IS ENOUGH oxygen[/B]

after buying some tickets for $19.00, Ann has $18.00 left. How much money did Ann have to beginwith
After buying some tickets for $19.00, Ann has $18.00 left. How much money did Ann have to begin with? Let the beginning amount be b. We're given: b - 19 = 18. <-- [I]We subtract 19 because a purchase is a spend reducing the original amount[/I] To solve for b, we [URL='https://www.mathcelebrity.com/1unk.php?num=b-19%3D18&pl=Solve']type the equation b - 19 = 18 into our search engine [/URL]and we get: b = [B]37[/B]

After paying 7 dollars for the pie, Keith has 64 dollars left. How much money did he have before buy
After paying 7 dollars for the pie, Keith has 64 dollars left. How much money did he have before buying the pie? we add the 7 dollars back in to find Keith's original total t: t = 64 + 7 t = [B]$71[/B]

Age Difference
Free Age Difference Calculator - Determines the ages for an age difference word problem.

Age now and then
I brute forced this and got a wrong answer, logic tells me is right. I tried the calculator here but maybe messed up the equation using another users problem as an example. Having no luck. Problem: Jacob is 4 times the age of Clinton. 8 years ago Jacob was 9 times the age of Clinton. How old are they now and how old were they 8 years ago?

Age now and then
she wrote it down wrong! The 9 should have been a 10. So I tried 4c-8=40c-80 in the equation solver and it also came back with C=2 which was the same answer you got before?

Age now and then
she wrote it down wrong! The 9 should have been a 10. So I tried 4c-8=40c-80 in the equation solver and it also came back with C=2 which was the same answer you got before? [QUOTE="Drew, post: 1161, member: 63"]she wrote it down wrong! The 9 should have been a 10. So I tried 4c-8=40c-80 in the equation solver and it also came back with C=2 which was the same answer you got before?[/QUOTE] oh and my brute force answer was 12-48 and 8 years earlier was 4-40

Age now and then
I read it wrong before. Here you go: Jacob is 4 times the age of Clinton. 8 years ago Jacob was 10 times the age of Clinton. How old are they now and how old were they 8 years ago? [LIST=1] [*]j = 4c [*]j - 8 = 10(c - 8) [/LIST] Substitute (1) into (2) (4c) - 8 = 10c - 80 [URL='http://www.mathcelebrity.com/1unk.php?num=4c-8%3D10c-80&pl=Solve']Equation solver[/URL] gives us [B]c = 12[/B] which means j = 4(12) = [B]48[/B]. 8 years ago,[B] j = 40 and c = 4[/B] which holds the 10x rule.

Age now and then
[QUOTE="math_celebrity, post: 1163, member: 1"]I read it wrong before. Here you go: Jacob is 4 times the age of Clinton. 8 years ago Jacob was 10 times the age of Clinton. How old are they now and how old were they 8 years ago? [LIST=1] [*]j = 4c [*]j - 8 = 10(c - 8) [/LIST] Substitute (1) into (2) (4c) - 8 = 10c - 80 [URL='http://www.mathcelebrity.com/1unk.php?num=4c-8%3D10c-80&pl=Solve']Equation solver[/URL] gives us [B]c = 12[/B] which means j = 4(12) = [B]48[/B]. 8 years ago,[B] j = 40 and c = 4[/B] which holds the 10x rule.[/QUOTE] Thank you, I see what I did wrong!

Age now problems
The age of the older of the two boys is twice that of the younger. 5 years ago it was three times that of the younger. Find the age of each

Age now problems
Age of the older boy is o, younger boy is y. We have the following equations: [LIST=1] [*]o = 2y [*]o - 5 = 3(y - 5) [/LIST] Plug (1) into (2) (2y) - 5 = 3y - 15 Plug this into our [URL='http://www.mathcelebrity.com/1unk.php?num=2y-5%3D3y-15&pl=Solve']equation solver[/URL], we get: [B]y = 10[/B] Plug that into (1), we get: o = 2(10), [B]o = 20[/B]

Age now problems
A father is three times as old as the son, and the daughter is 3 years younger than the son. If the sum of their ages 3 years ago was 63 Find the present age of the father

Age now problems
Let f be the age of the father and d be the age of the daughter and s be the age of the son. We have: [LIST=1] [*]f = 3s [*]d = s - 3 [*]d - 3 + f - 3 + s - 3 = 63 [/LIST] Simplify (3) d + f + s - 9 = 63 d + f + s = 72 Now, substitute (1) and (2) into the modified (3) (s - 3) + 3s + s = 72 Combine like terms: 5s - 3 = 72 Add 3 to each side 5s = 75 Divide each side by 5 s = 15 We want f, so we substitute s = 15 into (1) f = 3(15) [B]f = 45[/B]

Ages are consecutive integers. The sum of ages are 111. What are the ages
Ages are consecutive integers. The sum of ages are 111. What are the ages In the search engine, we type [I][URL='http://www.mathcelebrity.com/consecintwp.php?num=111&pl=Sum']sum of 2 consecutive integers is 111[/URL][/I]. We get [B]55 and 56[/B].

Ahmad has a jar containing only 5-cent and 20-cent coins. In total there are 31 coins with a total v
Ahmad has a jar containing only 5-cent and 20-cent coins. In total there are 31 coins with a total value of $3.50. How many of each type of coin does Ahmad have? Let the number of 5-cent coins be f. Let the number of 20-cent coins be t. We're given two equations: [LIST=1] [*]f + t = 31 [*]0.05f + 0.2t = 3.50 [/LIST] We can solve this simultaneous system of equations 3 ways: [LIST] [*][URL='https://www.mathcelebrity.com/simultaneous-equations.php?term1=f+%2B+t+%3D+31&term2=0.05f+%2B+0.2t+%3D+3.50&pl=Substitution']Substitution Method[/URL] [*][URL='https://www.mathcelebrity.com/simultaneous-equations.php?term1=f+%2B+t+%3D+31&term2=0.05f+%2B+0.2t+%3D+3.50&pl=Elimination']Elimination Method[/URL] [*][URL='https://www.mathcelebrity.com/simultaneous-equations.php?term1=f+%2B+t+%3D+31&term2=0.05f+%2B+0.2t+%3D+3.50&pl=Cramers+Method']Cramers Rule[/URL] [/LIST] No matter which method we choose, we get: [LIST] [*][B]f = 18[/B] [*][B]t = 13[/B] [/LIST]

Ahmed was born in 530 B.C.E. and lived for 60 years, in which year did he die?
Ahmed was born in 530 B.C.E. and lived for 60 years, in which year did he die? In B.C.E., the year decreases as time goes on until we get to year 0. So we have the year of death as: 530 - 60 = [B]470 B.C.E.[/B]

Ailyah had $24 to spend on seven pencils. After buying them she had $10. How much did each pencil co
The seven pencils cost $24 - $10 = $14. $14 / 7 pencils = [B]$2 per pencil[/B].

Al's Rentals charges $25 per hour to rent a sailboard and a wetsuit. Wendy's Rentals charges $20 per
Al's Rentals charges $25 per hour to rent a sailboard and a wetsuit. Wendy's Rentals charges $20 per hour plus $15 extra for a wetsuit. Find the number of hours for which the total charges for both companies would be the same. Al's Rentals Cost Equation C(h) where h is the number of hours you rent a sailboard and wetsuit: C(h) = 25h Wendy's Rentals Cost Equation C(h) where h is the number of hours you rent a sailboard and wetsuit: C(h) = 20h + 15 We want to set both cost equation equal to each other, and solve for h: 20h + 15 = 25h [URL='https://www.mathcelebrity.com/1unk.php?num=20h%2B15%3D25h&pl=Solve']Typing this equation into our search engine[/URL], we get: h = [B]3[/B]

Alan is y years old. Beth is 3 years old than Alan.Write an expression for how old Beth is?
Alan is y years old. Beth is 3 years old than Alan.Write an expression for how old Beth is? The word [I]older[/I] means we add 3 to Alan's age of y. So Beth's age is: [B]y + 3[/B]

Alana puts $700.00 into an account to use for school expenses. The account earns 8% interest, compou
Alana puts $700.00 into an account to use for school expenses. The account earns 8% interest, compounded annually. How much will be in the account after 4 years? We use our [URL='https://www.mathcelebrity.com/compoundint.php?bal=700&nval=8&int=4&pl=Annually']balance with interest calculator[/URL] and we get: [B]$958[/B]

Alberto and Willie each improved their yards by planting daylilies and ivy. They bought their suppli
Alberto and Willie each improved their yards by planting daylilies and ivy. They bought their supplies from the same store. Alberto spent $64 on 3 daylilies and 8 pots of ivy. Willie spent $107 on 9 daylilies and 7 pots of ivy. What is the cost of one daylily and the cost of one pot of ivy? Assumptions: [LIST] [*]Let d be the cost of one daylily [*]Let i be the cost of one pot of ivy [/LIST] Givens: [LIST=1] [*]3d + 8i = 64 [*]9d + 7i = 107 [/LIST] To solve this system of equations, you can use any of three methods below: [LIST] [*][URL='https://www.mathcelebrity.com/simultaneous-equations.php?term1=3d+%2B+8i+%3D+64&term2=9d+%2B+7i+%3D+107&pl=Substitution']Substitution Method[/URL] [*][URL='https://www.mathcelebrity.com/simultaneous-equations.php?term1=3d+%2B+8i+%3D+64&term2=9d+%2B+7i+%3D+107&pl=Elimination']Elimination Method[/URL] [*][URL='https://www.mathcelebrity.com/simultaneous-equations.php?term1=3d+%2B+8i+%3D+64&term2=9d+%2B+7i+%3D+107&pl=Cramers+Method']Cramer's Method[/URL] [/LIST] No matter what method we use, we get the same answer: [LIST] [*][B]d = 8[/B] [*][B]i = 5[/B] [/LIST] [B][MEDIA=youtube]K1n3niERg-U[/MEDIA][/B]

Alberto’s salary was $1500 greater than 5 times Nick’s salary. Write an equation stating Alberto’s a
Alberto’s salary was $1500 greater than 5 times Nick’s salary. Write an equation stating Alberto’s and Nick’s salaries in terms of x and y. Let x be Alberto's salary. Let y be Nick's salary. We have: Let's break this down: [LIST=1] [*]5 times Nick's salary (y), means we multiply the variable y by 5 [*]$1500 greater means we add $1500 to 5y [/LIST] [B]x = 5y - 1500[/B]

Alec had c caramels. Then, Alecs sister took 85 of the caramels. Choose the expression that shows th
Alec had c caramels. Then, Alecs sister took 85 of the caramels. Choose the expression that shows the number of caramels Alec has left. Alec starts with c caramels. His sister took 85. The word [I]took[/I] means subtract, so we have: [B]c - 85[/B]

alex burns approximately 650 calories in a 1.25-hour game. Daniellle enjoys kickboxing and burns app
alex burns approximately 650 calories in a 1.25-hour game. Daniellle enjoys kickboxing and burns approximately 420 calories in 45 minute class. who burns calories at the higher rate? We want a calories to minutes measure. [LIST] [*][URL='https://www.mathcelebrity.com/timecon.php?quant=1.25&pl=Calculate&type=hour']1.25 hours[/URL] = 75 minutes [/LIST] Alexa's unit calorie burn: 650/75 = 8.67 Danielle's unit calorie burn: 420/45 = 9.33 So [B]Danielle[/B] burns calories at a higher rate.

Alex rode his bike to school at a speed of 12 mph. He then walked home at a speed of 5 mph. What was
Alex rode his bike to school at a speed of 12 mph. He then walked home at a speed of 5 mph. What was Alex's average speed for his trip to school and back? Say the distance was 1 mile from school to home D = rt To school 1 = 12t t = 1/12 From school: 1 = 5t t = 1/5 1/2(1/12 + 1/5) [URL='https://www.mathcelebrity.com/fraction.php?frac1=1%2F24&frac2=1%2F10&pl=Add']1/24 + 1/10[/URL] = 17/120 120 = Average speed * 17 Average speed = 120/17 = [B]7.06 mph[/B]

Alex says all factors of 16 are even why is she wrong
Alex says all factors of 16 are even why is she wrong. [URL='https://www.mathcelebrity.com/factoriz.php?num=16&pl=Show+Factorization']Type in factor 16[/URL] into our search engine. We get the following factor of 16: 1, 2, 4, 8, 16 [B]All of these are even [I]except[/I] 1, which is odd. This is why Alex is wrong.[/B]

Alex sells cars at Keith Palmer ford. He earns 400 dollars a week plus 150 dollars per car he sells.
Alex sells cars at Keith Palmer ford. He earns 400 dollars a week plus 150 dollars per car he sells. If he earned 1450 dollars last week, how many cars did he sell? Subtract the base salary of $400 $1,450 - 400 =$1,050 Divide this by 150 per car $1,050/$150 = [B]7 cars[/B]

Alexandra was given a gift card for a coffee shop. Each morning, Alexandra uses the card to buy one
Alexandra was given a gift card for a coffee shop. Each morning, Alexandra uses the card to buy one cup of coffee. The original amount of money on the gift card was $45 and each cup of coffee costs $2.50. Write an equation for A(x),A(x), representing the amount money remaining on the card after buying xx cups of coffee. We start with 45, and each cup of coffee decreases our balance by 2.50, so we subtract: [B]A(x) = 45 - 2.50x[/B]

Alexis is working at her schools bake sale. Each mini cherry pie sells for $4 and each mini peach pi
Alexis is working at her schools bake sale. Each mini cherry pie sells for $4 and each mini peach pie sells for $3. Alexis sells 25 pies and collects $84. How many pies of each kind does she sell? Let each cherry pie be c and each peach pie be p. We have the following equations: [LIST=1] [*]c + p = 25 [*]4c + 3p = 84 [/LIST] You can solve this system of equations 3 ways. [URL='https://www.mathcelebrity.com/simultaneous-equations.php?term1=c%2Bp%3D25&term2=4c+%2B+3p+%3D+84&pl=Substitution']Substitution Rule[/URL] [URL='https://www.mathcelebrity.com/simultaneous-equations.php?term1=c%2Bp%3D25&term2=4c+%2B+3p+%3D+84&pl=Elimination']Elimination Rule[/URL] [URL='https://www.mathcelebrity.com/simultaneous-equations.php?term1=c%2Bp%3D25&term2=4c+%2B+3p+%3D+84&pl=Cramers+Method']Cramers Rule[/URL] No matter which way you choose, you get [B]c = 9 and p = 16[/B].

Alfred carries a load of 12 kilograms. He finds it heavy so he removes a weight of 4 kilograms. What
Alfred carries a load of 12 kilograms. He finds it heavy so he removes a weight of 4 kilograms. What is the weight of the remaining load? Removes means he subtract weight. So we have: 12 kilograms - 4 kilograms = [B]8 kilograms[/B]

Algebra Master (Polynomials)
Free Algebra Master (Polynomials) Calculator - Given 2 polynomials this does the following:
1) Polynomial Addition
2) Polynomial Subtraction

Also generates binomial theorem expansions and polynomial expansions with or without an outside constant multiplier.

algebraic expression for the sum of x and double the value of y
algebraic expression for the sum of x and double the value of y Double the value of y means we multiply y by 2: 2y The sum of x and 2y means we add 2y to x: [B]x + 2y[/B]

Algebraic Expressions
Free Algebraic Expressions Calculator - This calculator builds algebraic expressions based on word representations of numbers using the four operators and the words that represent them(increased,product,decreased,divided,times) Also known as Mathematical phrases

Algebraic Substitutions
Free Algebraic Substitutions Calculator - Given an algebraic statement with variables [a-z], this calculator takes a set of given substitution values, i.e., x=2,y=3,z=4, and evaluates your statement using the substitution values.

algexpress: letthefirstnumberequalx.thesecondnumberis3morethantwicethefirstnumber.expressthesecondnu
Let the first number equal x. The second number is 3 more than twice the first number. Express the second number in terms of the first number x. [LIST] [*]Let the second number be y. [*]Twice means multiply by 2 [*]3 more than means we add 3 [/LIST] So we have the following algebraic expression: [B]y = 2x + 3[/B]

Ali buys 6 sunglasses which cost $1.84 each, calculate the total cost.
Ali buys 6 sunglasses which cost $1.84 each, calculate the total cost. Total Cost = Quantity * Price Total Cost = 6 * $1.84 Total Cost = [B]$11.04[/B]

Ali needs to make a total of 90 deliveries this week. So far he has completed 72 of them. What perce
Ali needs to make a total of 90 deliveries this week. So far he has completed 72 of them. What percentage of his total deliveries has Ali completed [URL='https://www.mathcelebrity.com/perc.php?num=72&den=90&pcheck=1&num1=16&pct1=80&pct2=70&den1=80&idpct1=10&hltype=1&idpct2=90&pct=82&decimal=+65.236&astart=12&aend=20&wp1=20&wp2=30&pl=Calculate']Enter 72/90 into our search engine and choose the percentage option[/URL] and we get [B]80%[/B].

Ali runs each lap in 6 minutes. He will run at least 11 laps today. What are the possible numbers of
Ali runs each lap in 6 minutes. He will run at least 11 laps today. What are the possible numbers of minutes he will run today? Let m be the number of minutes. The phrase [I]at least[/I] means an inequality, also known as greater than or equal to. So we have: m >= 11*6 [B]m >= 66 You can read this as Ali will run 66 or more minutes today. Or at least 66 minutes. Or greater than or equal to 66 minutes[/B]

Ali spent $60 at the grocery store. Of this amount, he spent $51 on fruit. What percentage of the to
Ali spent $60 at the grocery store. Of this amount, he spent $51 on fruit. What percentage of the total did he spend on fruit? 51/60 = 0.85 Multiply 0.85 by 100 to get the percentage 0.85 * 100 = [B]85%[/B]

Alice is 3 years younger than Barbara, and Barbara is 5 years younger than Carol. Together the siste
Alice is 3 years younger than Barbara, and Barbara is 5 years younger than Carol. Together the sisters are 68 years old. How old is Barbara? Let a be Alice's age, b be Barbara's age, and c be Carol's age. We have 3 given equations: [LIST=1] [*]a = b - 3 [*]b = c - 5 [*]a + b + c = 68 [/LIST] Rearrange (2) c = b + 5 Now plug in (1) and (2) revised into (3). We want to isolate for b. a + b + c = 68 (b - 3) + b + (b + 5) = 68 Combine like terms: (b + b + b) + (5 - 3) = 68 3b + 2 = 68 Run this through our [URL='https://www.mathcelebrity.com/1unk.php?num=3b%2B2%3D68&pl=Solve']equation calculator[/URL], and we get b = [B]22[/B]

Alice is a machinist in a shirt factory. For the first 180 shirts she is paid $2.20 and then $2.90 p
Alice is a machinist in a shirt factory. For the first 180 shirts she is paid $2.20 and then $2.90 per garment thereafter. What are her gross wages for a week in which she produces 240 shirts? Calculate commission on the first 180 shirts (Commission 1): Commission 1 = Shirts (up to 180) * $2.20 Commission 1 = 180 * $2.20 Commission 1 = $396 Calculate commission on the rest of the shirts about 180 (Commission 2): Commission 2 = Shirts Above 180 * $2.90 Commission 2 = (240 - 180) * $2.90 Commission 2 = 60 * $2.90 Commission 2 = $174 Calculate Total Commission: Total Commission = Commission 1 + Commission 2 Total Commission = $396 + $174 Total Commission = [B]$570[/B]

Alice is making a sandwich to pack in her lunch. She has 2 different kinds of bread, 3 cheeses, 4 lu
Alice is making a sandwich to pack in her lunch. She has 2 different kinds of bread, 3 cheeses, 4 lunch meats, and 2 condiments to choose from. Assuming she uses one of each of bread, cheese, meat, and condiment, how many different sandwiches can she make? We use the Fundamental Rule of Counting [LIST] [*]Bread: 2 [*]Cheeses: 3 [*]Lunch Meats: 4 [*]Condiments: 2 [/LIST] 2 * 3 * 4 * 2 = [B]48 different sandwiches[/B]

Alicia deposited $41 into her checking account. She wrote checks for $31 and $13. Now her account ha
Alicia deposited $41 into her checking account. She wrote checks for $31 and $13. Now her account has a balance of $81. How much did she have in her account to start with? We start with a balance of b. Depositing 41 means we add to the account balance: b + 41 Writing checks for 31 and 13 means we subtract from the account balance: b + 41 - 31 - 13 The final balance is 81, so we set b + 41 - 31 - 13 equal to 81: b + 41 - 31 - 13 = 81 To solve for b, we [URL='https://www.mathcelebrity.com/1unk.php?num=b%2B41-31-13%3D81&pl=Solve']type this equation into our math engine[/URL] and we get: b = [B]84[/B]

Alisha is 5 years younger than her brother. If the age of her brother is y years then age of Alisha
Alisha is 5 years younger than her brother. If the age of her brother is y years then age of Alisha in terms of her brother Younger means we subtract. If her brother is y years of age, then Alisha is: [B]y - 5[/B]

Aliyah had $24 to spend on seven pencils after buying them she had $10 how much did each pencil cost
Aliyah had $24 to spend on seven pencils after buying them she had $10 how much did each pencil cost? If Aliyah had $24 to spend, and $10 left over, then she spent $24 - $10 = $14 on pencils Find the cost per pencil: Cost per pencil = Pencil Spend / Number of Pencils Cost per pencil = 14/7 Cost per pencil = [B]$2[/B]

Aliyah had $24 to spend on seven pencils. After buying them she had $1. How much did each pencil cos
Aliyah had $24 to spend on seven pencils. After buying them she had $1. How much did each pencil cost? Let each pencil cost p. We're given the following equation: 7p + 1 = 24 [URL='https://www.mathcelebrity.com/1unk.php?num=7p%2B1%3D24&pl=Solve']Type this equation into our search engine[/URL] and we get: p = [B]$3.29[/B]

Aliyah had $24 to spend on seven pencils. After buying them she had $10. How much did each pencil co
Aliyah had $24 to spend on seven pencils. After buying them she had $10. How much did each pencil cost? Let p be the number of pencils. We're given the following equation: 7p + 10 = 24 To solve this equation for p, we [URL='https://www.mathcelebrity.com/1unk.php?num=7p%2B10%3D24&pl=Solve']type it in our math engine[/URL] and we get: p = [B]2 [/B]

Aliyah had $24 to spend on seven pencils. After buying them she had $10. How much did each pencil co
Aliyah had $24 to spend on seven pencils. After buying them she had $10. How much did each pencil cost? Let the number of pencils be p. We have: 7p + 10 = 24 To solve this equation for p, we [URL='https://www.mathcelebrity.com/1unk.php?num=7p%2B10%3D24&pl=Solve']type it in our math engine[/URL] and we get: p = [B]2[/B]

Aliyah has $24 to spend on 7 pencils. After buying them she had $10. How much did each pencil cost?
Aliyah has $24 to spend on 7 pencils. After buying them she had $10. How much did each pencil cost? Let the cost of each pencil be p. The phrase [I]leftover[/I] means we add to the cost of the pencils after buying them. We're given the equation: 7p + 10 = 24 To solve for p, we [URL='https://www.mathcelebrity.com/1unk.php?num=7p%2B10%3D24&pl=Solve']type this equation into our search engine[/URL] and we get: p = [B]2[/B]

All real numbers that are less than equal to -1 or greater than 5
We have two expressions here, so we need a union since we have the word [U]or[/U]. First, All real numbers less than or equal to -1 is x <= -1. All real numbers greater than 5 is x > 5 So we have x <= -1 U x > 5 [MEDIA=youtube]boOueZTCSuU[/MEDIA]

all real numbers y greater than or equal to 12
all real numbers y greater than or equal to 12 Greater than or equal to means we use the sign >= [B]y >= 12[/B]

All squares are rectangles and all rectangles are parallelograms, therefore all squares are parallel
All squares are rectangles and all rectangles are parallelograms, therefore all squares are parallelograms. Is this true? [B]Yes.[/B] This is similar to A implies B and B implies C so A implies C also known as transitive property

All the colors of the rainbow
All the colors of the rainbow You can find this with the acronym: ROYGBIV [LIST=1] [*][B]Red[/B] [*][B]Orange[/B] [*][B]Yellow[/B] [*][B]Green[/B] [*][B]Blue[/B] [*][B]Indigo[/B] [*][B]Violet[/B] [/LIST] [U]Written as a set, we have 7 elements:[/U] {[B]Red, Orange, Yellow, Green, Blue, Indigo, Violet[/B]}

Allan built an additional room onto his house. The length of the room is 3 times the width. The peri
Allan built an additional room onto his house. The length of the room is 3 times the width. The perimeter of the room is 60 feet. What is the length of the room A room is a rectangle. We know the perimeter of a rectangle is: P = 2l + 2w We're given two equations: [LIST=1] [*]l = 3w [*]P = 60 [/LIST] Plug (1) and (2) into our rectangle perimeter formula: 2(3w) + w = 60 6w + w = 60 [URL='https://www.mathcelebrity.com/1unk.php?num=6w%2Bw%3D60&pl=Solve']Type this equation into our search engine[/URL] to solve for w: w = 8.5714 Now plug w = 8.5714 into equation 1 to solve for l: l = 3(8.5714) l = [B]25.7142[/B]

Allison can pay her gym membership fee monthly but if she pays for her entire year at one she gets a
Allison can pay her gym membership fee monthly but if she pays for her entire year at one she gets a $53 discount her discounted bill at the end of the year was 463 what is her monthly fee Her full annual bill is found by adding the discounted annual bill to the discount amount: Full annual bill = Discounted annual bill + discount amount Full annual bill = 463 + 53 Full annual bill = 516 Her monthly gym membership is found by the following calculation: Monthly Gym Membership = Full Annual Bill / 12 Monthly Gym Membership = 516 / 12 Monthly Gym Membership = [B]$43[/B]

Alonzo needs to buy some pencils. Brand A has a pack of 36 pencils for $8.52. Brand B has a pack of
Alonzo needs to buy some pencils. Brand A has a pack of 36 pencils for $8.52. Brand B has a pack of 48 pencils for $9.98. Find the unit price for each brand. Then state which brand is the better buy based on the unit price. Round your answers to the nearest cent. Using our [URL='http://www.mathcelebrity.com/betterbuy.php?p1=8.52&p2=9.98&q1=36&q2=48&pl=Better+Buy']better buy calculator[/URL], we see the following unit prices: [LIST] [*][B]Brand A = $0.24[/B] [*][B]Brand B = $0.21[/B]. [*][B]Brand B has the better unit price by 3 cents.[/B] [/LIST]

Alonzo runs each lap in 4 minutes. He will run at most 32 minutes today. What are the possible numbe
Alonzo runs each lap in 4 minutes. He will run at most 32 minutes today. What are the possible numbers of laps he will run today? 32 minutes / 4 minutes per lap =[B] 8 laps maximum[/B]. He can also run less than 8 laps if his lap time gets slower.

Alorah joins a fitness center. She pays for a year plus a joining fee of $35. If the cost for the en
Alorah joins a fitness center. She pays for a year plus a joining fee of $35. If the cost for the entire year is $299, how much will she pay each month? We set up the cost function C(m) where m is the number of months of membership: C(m) = cost per month * m + joining fee Plugging in our numbers from the problem with 12 months in a year, we get: 12c + 35 = 299 To solve this equation for c, we [URL='https://www.mathcelebrity.com/1unk.php?num=12c%2B35%3D299&pl=Solve']type it in our search engine [/URL]and we get: c = [B]22[/B]

Alvin is 12 years younger than Elga. The sum of their ages is 60 . What is Elgas age?
Alvin is 12 years younger than Elga. The sum of their ages is 60 . What is Elgas age? Let a be Alvin's age and e be Elga's age. We have the following equations: [LIST=1] [*]a = e - 12 [*]a + e = 60 [/LIST] Plugging in (1) to (2), we get: (e - 12) + e = 60 Grouping like terms: 2e - 12 = 60 Add 12 to each side: 2e = 72 Divide each side by 2 [B]e = 36[/B]

Alvin is 34 years younger than Elga. Elga is 3 times older than Alvin. What is Elgas age?
Alvin is 34 years younger than Elga. Elga is 3 times older than Alvin. What is Elgas age? Let a be Alvin's age. Let e be Elga's age. We're given: [LIST=1] [*]a = e - 34 [*]e = 3a [/LIST] Substitute (2) into (1): a = 3a - 34 [URL='https://www.mathcelebrity.com/1unk.php?num=a%3D3a-34&pl=Solve']Typing this equation into the search engine[/URL], we get a = 17 Subtitute this into Equation (2): e = 3(17) e = [B]51[/B]

Alvin planted t fewer trees than Danielle. Danielle planted 56 trees. Write an expression that shows
Alvin planted t fewer trees than Danielle. Danielle planted 56 trees. Write an expression that shows how many trees Alvin planted. The word [I]fewer[/I] means we subtract, so we have Alvin's tree planting of: [B]56 - t[/B]

Alvin, an HR Director has 12 employees in his department. There are still 3 positions to be filled u
Alvin, an HR Director has 12 employees in his department. There are still 3 positions to be filled up. What fraction of the entire staff are still available? If 3 spots need to be filled, then we have 12 - 3 = 9 people still available. [URL='https://www.mathcelebrity.com/fraction.php?frac1=9%2F12&frac2=3%2F8&pl=Simplify']9/12 =[/URL] 3[B]/4[/B]

Alya loves to read. She reads 90 pages in half an hour. How many pages does she read per minute?
Alya loves to read. She reads 90 pages in half an hour. How many pages does she read per minute? Set up a proportion of pages to minutes. Since 30 minutes is a half hour, we have the number of pages (p) for 1 minute as: 90/30 = p/1 To solve this proportion for p, [URL='https://www.mathcelebrity.com/prop.php?num1=90&num2=p&den1=30&den2=1&propsign=%3D&pl=Calculate+missing+proportion+value']we type it in our search engine[/URL] and we get: p = [B]3[/B]

Alyssa had 87 dollars to spend on 6 books. After buying them she had 15 dollars . How much did each
Alyssa had 87 dollars to spend on 6 books. After buying them she had 15 dollars . How much did each book cost ? Let b be the cost of each book. We're given: 87 - 6b = 15 [URL='https://www.mathcelebrity.com/1unk.php?num=87-6b%3D15&pl=Solve']Typing this equation into search engine[/URL], we get: [B]b = 12[/B]

Alyssa has $952 and is spending $27 each week (w) for math tutoring write an algebraic expression to
Alyssa has $952 and is spending $27 each week (w) for math tutoring write an algebraic expression to model the situation Alyssa's balance is found by using this expression: [B]952 - 27w[/B]

Amanda runs each lap in 4 minutes. She will run less than 44 minutes today. What are the possible nu
Amanda runs each lap in 4 minutes. She will run less than 44 minutes today. What are the possible number of laps she will run today? Notes for this problem: [LIST] [*]Let laps be l. [*]Lap time = Time per lap * number of laps (l) [*]Less than means we have an inequality using the < sign [/LIST] We have the inequality: 4l < 44 To solve this inequality for l, we [URL='https://www.mathcelebrity.com/interval-notation-calculator.php?num=4l%3C44&pl=Show+Interval+Notation']type it in our math engine[/URL] and we get: [B]l < 11[/B]

Amanda spent 2/5 of her time after school doing homework and ¼ of her remaining time riding her bike
Amanda spent 2/5 of her time after school doing homework and ¼ of her remaining time riding her bike. If she rode her bike for 45 minutes in a week, how much time did she devote to homework in the same week If Amanda spent 2/5 of her time after school doing homework, she has 1 - 2/5 time left over. We convert 1 to a fraction using a denominator of 5, we get: 5/5 - 2/5 = 3/5 And Amanda spent 1/4 of 3/5 of her time bike riding, which means she spent: 1(3)/4(5) = 3/20 of her time. If the total time after school is t, we have: 3t/20 = 45 [URL='https://www.mathcelebrity.com/prop.php?num1=3t&num2=45&den1=20&den2=1&propsign=%3D&pl=Calculate+missing+proportion+value']Typing in 3t/20 = 45 to our search engine[/URL], we get t = 300. So Amanda has 300 total minutes after school, which means she spent 2/5(300) = [B]120 minutes (2 hours)[/B] doing homework.

Amar goes to the dance class every fourth day. Karan goes to the dance class every fifth day. Both m
Amar goes to the dance class every fourth day. Karan goes to the dance class every fifth day. Both met at the dance class today. After how many days will they meet at the dance class again? We want the least common multiple of 4 and 5. We type in [URL='https://www.mathcelebrity.com/gcflcm.php?num1=4&num2=5&num3=&pl=GCF+and+LCM']LCM(4,5)[/URL] into our search engine and we get [B]20. So 20 days from now, Amar and Karen will meet again.[/B]

Amara currently sells televisions for company A at a salary of $17,000 plus a $100 commission for ea
Amara currently sells televisions for company A at a salary of $17,000 plus a $100 commission for each television she sells. Company B offers her a position with a salary of $29,000 plus a $20 commission for each television she sells. How many televisions would Amara need to sell for the options to be equal? Let the number of tv's be t. Set up the salary function S(t): S(t) = Commision * tv's sold + Salary Company A: S(t) = 100t + 17,000 Company B: S(t) = 20t + 29,000 The problem asks for how many tv's it takes to make both company salaries equal. So we set the S(t) functions equal to each other: 100t + 17000 = 20t + 29000 [URL='https://www.mathcelebrity.com/1unk.php?num=100t%2B17000%3D20t%2B29000&pl=Solve']Type this equation into our search engine[/URL] and we get: t = [B]150[/B]

Amount you spend if you buy a shirt for $20 and jeans for j dollars
Amount you spend if you buy a shirt for $20 and jeans for j dollars We want an algebraic expression for our total spend. We add the $20 for a shirt plus j for the jeans: [B]20 + j[/B]

Amy and ryan operate a car dealing and repair service. For a car detailing (full wash outside and in
Amy and ryan operate a car dealing and repair service. For a car detailing (full wash outside and inside. Amy charges 40$ and Ryan charges 50$ . In addition they charge a hourly rate. Amy charges $35/h and ryan charges $30/h. How many hours does amy and ryan have to work to make the same amount of money? Set up the cost functions C(h) where h is the number of hours. [U]Amy:[/U] C(h) = 35h + 40 [U]Ryan:[/U] C(h) = 30h + 50 To make the same amount of money, we set both C(h) functions equal to each other: 35h + 40 = 30h + 50 To solve for h, we [URL='https://www.mathcelebrity.com/1unk.php?num=35h%2B40%3D30h%2B50&pl=Solve']type this equation into our search engine[/URL] and we get: h = [B]2[/B]

Amy deposits 4000 into an account that pays simple interest at a rate of 6% per year. How much inter
Amy deposits 4000 into an account that pays simple interest at a rate of 6% per year. How much interest will she be paid in the first 4 years? Using our [URL='http://www.mathcelebrity.com/simpint.php?av=&p=4000&int=6&t=4&pl=Simple+Interest']simple interest calculator[/URL], we get an accumulated value of 4,960 Interest Paid = Accumulated Value - Principal Interest Paid = 4960 - 4000 Interest Paid = [B]960[/B]

Amy has n decks of cards. Each deck has 52 cards in it. Using n, write an expression for the total
Amy has n decks of cards. Each deck has 52 cards in it. Using n, write an expression for the total number of cards Amy has. [B]52n[/B]

An 8 yard gain and a 3 yard loss results in what kind of gain or loss?
An 8 yard gain and a 3 yard loss results in what kind of gain or loss? [LIST=1] [*]Start with 0 yards [*]Gains means we add, so we have 0 + 8 = 8 [*]Losses mean we subtract, so we have 8 - 3 = 5 [/LIST] The overall result is a [B]5 yard gain[/B]

An air horn goes off every 48 seconds,another every 80 seconds. At 5:00 pm the two go off simultaneo
An air horn goes off every 48 seconds,another every 80 seconds. At 5:00 pm the two go off simultaneously. At what time will the air horns blow again at the same time? We want to find the least common multiple of (48, 80). So we [URL='https://www.mathcelebrity.com/gcflcm.php?num1=48&num2=80&num3=&pl=GCF+and+LCM']type this in our search engine[/URL], and we get: 240. So 240 seconds is our next common meeting point for each air horn. When we [URL='https://www.mathcelebrity.com/timecon.php?quant=240&pl=Calculate&type=second']type 240 seconds into our search engine[/URL], we get 4 minutes. We add the 4 minutes to the 5:00 pm time to get [B]5:04 pm[/B].

An airplane carries 500 passengers 45% are men, 20% are children. The number of women in the airplan
An airplane carries 500 passengers 45% are men, 20% are children. The number of women in the airplane is If we assume the sample space is either men, women, or children to get 100% of the passengers, we have: PercentWomen = 100% - Men - Children PercentWomen = 100% - 45% - 20% PercentWomen = 35% Calculate Women passengers Women passengers = Total passengers * Percent Women Women passengers = 500 * 35% Women passengers = [B]175[/B]

An airplane is flying at 38,800 feet above sea level. The airplane starts to descend at a rate of 18
An airplane is flying at 38,800 feet above sea level. The airplane starts to descend at a rate of 1800 feet per minute. Let m be the number of minutes. Which of the following expressions describe the height of the airplane after any given number of minutes? Let m be the number of minutes. Since a descent equals a [U]drop[/U] in altitude, we subtract this in our Altitude function A(m): [B]A(m) = 38,800 - 1800m[/B]

An airport has 13 scheduled arrivals every day. This week, 28 private planes also landed there. How
An airport has 13 scheduled arrivals every day. This week, 28 private planes also landed there. How many planes flew into the airport this week? A week has 7 days. 13 scheduled arrivals per day times 7 days = 91 scheduled planes Next, we add 28 private planes: 91 + 28 = [B]119 planes[/B]

An alligators tail length, T, varies directly as its body length, B. An alligator with a body length
An alligators tail length, T, varies directly as its body length, B. An alligator with a body length of 5.6 feet has a tail length of 4.9 feet. What is the tail length of an alligator whos body length is 4.8 feet Set up a proportion of T/B 5.6/4.9 = t/4.8 Using our [URL='http://www.mathcelebrity.com/prop.php?num1=5.6&num2=t&den1=4.9&den2=4.8&propsign=%3D&pl=Calculate+missing+proportion+value']proportion calculator[/URL], we get [B]t = 5.49[/B].

An Amazon delivery package receives a bonus if he delivers a package to a costumer in 20 minutes plu
An Amazon delivery package receives a bonus if he delivers a package to a costumer in 20 minutes plus or minus 5 minutes. Which inequality or equation represents the drivers allotted time (x) to receive a bonus 20 plus 5 minutes = 25 minutes 20 minus 5 minutes = 15 minutes So we have the inequality: [B]15 <= x <= 25[/B]

An analysis of the final test scores for Managerial Decision Making Tools reveals the scores follow
An analysis of the final test scores for Managerial Decision Making Tools reveals the scores follow the normal probability distribution. The mean of the distribution is 75 and the standard deviation is 8. The instructor wants to award an "A" to students whose score is in the highest 10 percent. What is the dividing point for those students who earn an "A"? Top 10% is equivalent to the 90th percentile. Using our [URL='http://www.mathcelebrity.com/percentile_normal.php?mean=+75&stdev=8&p=+90&pl=Calculate+Percentile']percentile calculator[/URL], the 90th percentile cutoff point is [B]85.256[/B]

An ancient Greek was said to have lived 1/4 of his live as a boy, 1/5 as a youth, 1/3 as a man, and
An ancient Greek was said to have lived 1/4 of his live as a boy, 1/5 as a youth, 1/3 as a man, and spent the last 13 years as an old man. How old was he when he died? Set up his life equation per time lived as a boy, youth, man, and old man 1/4 + 1/5 + 1/3 + x = 1. Using our [URL='http://www.mathcelebrity.com/gcflcm.php?num1=4&num2=3&num3=5&pl=LCM']LCM Calculator[/URL], we see the LCM of 3,4,5 is 60. This is our common denominator. So we have 15/60 + 12/60 + 20/60 + x/60 = 60/60 [U]Combine like terms[/U] x + 47/60 = 60/60 [U]Subtract 47/60 from each side:[/U] x/60 = 13/60 x = 13 out of the 60 possible years, so he was [B]60 when he died[/B].

An angle is 30 degrees less than 5 times it's complement. Find the angle.
An angle is 30 degrees less than 5 times it's complement. Find the angle. Let the angle be a. The complement of a is 90 - a. We're given the following equation: a = 5(90 - a) - 30 <-- Less means we subtract Multiplying though, we get: a = 450 - 5a - 30 a = 420 - 5a [URL='https://www.mathcelebrity.com/1unk.php?num=a%3D420-5a&pl=Solve']Typing this equation into our search engine[/URL], we get: a =[B] 70[/B]

An auto repair bill is $126 for parts and $35 for each hour of labor. If h is the number of hours of
An auto repair bill is $126 for parts and $35 for each hour of labor. If h is the number of hours of labor, express the amount of the repair bill in terms of number of hours of labor. Set up cost function, where h is the number of hours of labor: [B]C(h) = 35h + 136[/B]

An auto repair bill was $563. This includes $188 for parts and $75 for each hour of labor. Find the
An auto repair bill was $563. This includes $188 for parts and $75 for each hour of labor. Find the number of hours of labor Let the number of hours of labor be h. We have the cost function C(h): C(h) = Hourly Labor Rate * h + parts Given 188 for parts, 75 for hourly labor rate, and 563 for C(h), we have: 75h + 188 = 563 To solve this equation for h, we [URL='https://www.mathcelebrity.com/1unk.php?num=75h%2B188%3D563&pl=Solve']type it in our search engine[/URL] and we get: h = [B]5[/B]

An avocado is not ripe until 4 days after picking and will go bad after 7 days after picking. Repres
An avocado is not ripe until 4 days after picking and will go bad after 7 days after picking. Represent the days the avocado is ripe Our sweet spot for ripeness is 4 days or more but not more than 7 days. Using d as our days, we have the following inequality: [B]4 <= d <= 7[/B]

an earthworm moves at distance of 45cm in 90 seconds what is the speed
an earthworm moves at distance of 45cm in 90 seconds what is the speed Using our [URL='https://www.mathcelebrity.com/drt.php?d=45&r=+&t=90&pl=Calculate+the+missing+Item+from+D%3DRT']distance, rate, time calculator[/URL], we have: Rate = [B]1/2cm or 0.5cm per second[/B]

An eccentric millionaire decided to give away $1,000,000 if Janelle took one die and rolled a "4". H
An eccentric millionaire decided to give away $1,000,000 if Janelle took one die and rolled a "4". He wanted Janelle to have a better than 1 in 6 chance of winning, so before she rolled the die he told her that she could roll the die 3 times. If any roll was a "4", she would win the million dollars. What are Janelle's chances of winning the million dollars? Chance of winning each roll is 1/6. Which means the chances of losing each roll are 1 - 1/6 = 5/6 Calculate the probability of 3 straight losing rolls: P(Lose) = P(Loser) * P(Loser) * P(Loser) = 5/6 * 5/6 * 5/6 = 125/216 P(Win) = 1 - P(Lose) P(Win) = 1 - 125/216 P(Win) = 216/216 -125/216 P(Win) = [B]91/216[/B]

An eccentric millionaire has 5 golden hooks from which to hang her expensive artwork. She wants to h
An eccentric millionaire has 5 golden hooks from which to hang her expensive artwork. She wants to have enough paintings so she can change the order of the arrangement each day for the next 41 years. (The same five paintings are okay as long as the hanging order is different.) What is the fewest number of paintings she can buy and still have a different arrangement every day for the next 41 years? 365 days * 41 years + 10 leap year days = 14,975 days what is the lowest permutations count of n such that nP5 >= 14,975 W[URL='https://www.mathcelebrity.com/permutation.php?num=9&den=5&pl=Permutations']e see that 9P5[/URL] = 15,120, so the answer is [B]9 paintings[/B]

An electrician starts off the day with 600 feet of copper wiring on his truck. During the course of
An electrician starts off the day with 600 feet of copper wiring on his truck. During the course of the day he uses pieces 100, 82, 25, and 40 feet long. The next day, he purchases another 400 feet and puts it on his truck and later in the day uses pieces of 41, 39, and 44 feet long. How many feet of wiring are still on the truck at the end of the second day? If the electrician uses pieces, we subtract. If he purchases pieces, we add. So we have: 600 - 100 - 82 - 25 - 40 + 400 - 41 - 39 - 44 = [B]629 feet[/B]

An elevator can hold less than 2700 pounds of extra weight. If an average person weighs 150 pounds,
An elevator can hold less than 2700 pounds of extra weight. If an average person weighs 150 pounds, what is the maximum number of people (p) that can be on the elevator at one time? Total weight = average weight per person * Number of people Total weight = 150p We know from the problem that: 150p < 2700 We want to solve this inequality for p. Divide each side of the inequality by 150: 150p/150 < 2700/150 Cancel the 150's on the left side and we get: p < [B]18[/B]

An elevator can safely lift at most 4400 1bs. A concrete block has an average weight of 41 lbs. What
An elevator can safely lift at most 4400 1bs. A concrete block has an average weight of 41 lbs. What is the maximum number of concrete blocks that the elevator can lift? Total blocks liftable = Lift Max / Weight per block Total blocks liftable = 4400 / 41 Total blocks liftable = 107.31 We round down to whole blocks and we get [B]107[/B]

An elevator has a maximum weight of 3000 pounds. How many 150-pound people can safely ride the eleva
An elevator has a maximum weight of 3000 pounds. How many 150-pound people can safely ride the elevator? (Use "p" to represent the number of people) Maximum means less than or equal to. We have the inequality: 150p <= 3000 To solve this inequality for p, we [URL='https://www.mathcelebrity.com/interval-notation-calculator.php?num=150p%3C%3D3000&pl=Show+Interval+Notation']type it in our search engine[/URL] and we get: [B]p <= 20[/B]

An employee earns $7.00 an hour for the first 35 hours worked in a week and $10.50 for any hours ove
An employee earns $7.00 an hour for the first 35 hours worked in a week and $10.50 for any hours over 35. One weeks paycheck (before deductions) was for $308.00. How many hours did the employee work? Let's first check to see if the employee worked overtime: Regular Hours: 35 * 7 = 245 Since the employee made $308, they worked overtime. Let's determine how much overtime money was made: 308 - 245 = 63 Now, to calculate the overtime hours, we divide overtime pay by overtime rate 63/10.50 = 6 Now figure out the total hours worked in the week: Total Hours = Regular Pay Hours + Overtime Hours Total Hours = 35 + 6 [B]Total Hours = 41[/B]

An equilateral triangle has three sides of equal length. What is the equation for the perimeter of a
An equilateral triangle has three sides of equal length. What is the equation for the perimeter of an equilateral triangle if P = perimeter and S = length of a side? P = s + s + s [B]P = 3s[/B]

An estate has 6 houses and each house has x lighting fittings which need 1 lamp each, and y fittings
An estate has 6 houses and each house has x lighting fittings which need 1 lamp each, and y fittings which need 3 lamps each. Write a formula to find z, the total number of lamps needed on the estate. z = 6(x * 1 + 3 * y) z = [B]6(x + 3y)[/B]

An executive in an engineering firm earns a monthly salary plus a Christmas bonus of 6400 dollars. I
An executive in an engineering firm earns a monthly salary plus a Christmas bonus of 6400 dollars. If she earns a total of 87400 dollars per year, what is her monthly salary in dollars? Calculate the annual salary without bonus: Annual Salary = Total Pay - Christmas Bonus Annual Salary = 87400 - 6400 Annual Salary = 81000 Now calculate the monthly salary. [I]Note: there are 12 months in a year[/I]: Monthly Salary = Annual Salary / 12 Monthly Salary = 81000/12 [URL='https://www.mathcelebrity.com/fraction.php?frac1=81000%2F12&frac2=3%2F8&pl=Simplify']Monthly Salary[/URL] = [B]6750[/B]

An executive invests $21,000, some at 8% and the rest at 7% annual interest. If he receives an annua
An executive invests $21,000, some at 8% and the rest at 7% annual interest. If he receives an annual return of $1,600, how much is invested at each rate? Using our [URL='http://www.mathcelebrity.com/split-fund-interest-calculator.php?p=21000&i1=8&i2=7&itot=1600&pl=Calculate']split fund calculator[/URL], we get: [LIST] [*][B]Fund 1 = 13,000[/B] [*][B]Fund 2 = 8,000[/B] [/LIST]

An executive invests $22,000, some at 7% and the rest at 6% annual interest. If he receives an annua
An executive invests $22,000, some at 7% and the rest at 6% annual interest. If he receives an annual return of $1,420, how much is invested at each rate Using our [URL='https://www.mathcelebrity.com/split-fund-interest-calculator.php?p=22000&i1=7&i2=6&itot=1420&pl=Calculate']split fund interest calculator[/URL], we get: [LIST] [*][B]10,000[/B] @ 7% [*][B]12,000[/B] @ 6% [/LIST]

An executive invests $23,000, some at 8% and some at 4% annual interest. If he receives an annual re
An executive invests $23,000, some at 8% and some at 4% annual interest. If he receives an annual return of $1,560, how much is invested at each rate? Let x be the amount invested at 8% and y be the amount invested at 4%. We have two equations: [LIST=1] [*]x + y = 23,000 [*]0.08x + 0.04y = 1,560 [/LIST] Using our [URL='http://www.mathcelebrity.com/simultaneous-equations.php?term1=x+%2B+y+%3D+23000&term2=0.08x+%2B+0.04y+%3D+1560&pl=Cramers+Method']system of equations calculator[/URL], we get: [B]x = 16,000 y = 7,000[/B]

An executive invests $29,000, some at 8% and the rest at 6% annual interest. If he receives an annua
An executive invests $29,000, some at 8% and the rest at 6% annual interest. If he receives an annual return of $2,020, how much is invested at each rate? Using our [URL='http://www.mathcelebrity.com/split-fund-interest-calculator.php?p=29000&i1=8&i2=6&itot=2020&pl=Calculate']split fund calculator[/URL], we get: [LIST] [*]Fund 1: $14,000 [*]Fund 2: $15,000 [/LIST]

An experienced accountant can balance the books twice as fast as a new accountant. Working together
An experienced accountant can balance the books twice as fast as a new accountant. Working together it takes the accountants 10 hours. How long would it take the experienced accountant working alone? Person A: x/2 job per hour Person B: 1/x job per hour Set up our equation: 1/x + 1/(2x) = 1/10 Multiply the first fraction by 2/2 to get common denominators; 2/(2x) + 1/(2x) = 1/10 Combine like terms 3/2x = 1/10 Cross multiply: 30 = 2x Divide each side by 2: [B]x = 15[/B]

An ice cream shop carries 6 ice cream flavors, 3 sauces, and 4 toppings. If a sundae has one scoop o
An ice cream shop carries 6 ice cream flavors, 3 sauces, and 4 toppings. If a sundae has one scoop of ice cream, one sauce, and one topping, how many different sundaes can be created? Using the rule of counting, we have: We have 6 possible ice cream flavors * 3 possible sauces * 4 possible toppings = [B]72 possible sundaes[/B]

An indoor water park has two giant buckets that slowly fill with 1000 gallons of water before dumpin
An indoor water park has two giant buckets that slowly fill with 1000 gallons of water before dumping it on the people below. One bucket dumps water every 18 minutes. The other bucket dumps water every 21 minutes. It is currently 1:15 P.M. and both buckets dumped water 5 minutes ago. Find the next two times that both buckets dump water at the same time. We want to find the Least Common Multiple between 18 minutes and 21 minutes. This shows us when both bucket dumping cycles happen simultaneously. So we[URL='https://www.mathcelebrity.com/gcflcm.php?num1=18&num2=21&num3=&pl=GCF+and+LCM'] type in LCM(18,21) into our search engine and we get[/URL]: LCM(18, 21) = 126 This means, in 126 minutes, both buckets will dump water. Since 60 minutes is in an hour, we find out how many full hours we have. To find the full hours and remainder, we [URL='https://www.mathcelebrity.com/modulus.php?num=126mod60&pl=Calculate+Modulus']type in 126 mod 60[/URL] into our search engine and we get: 6. This means 126 minutes is 2 hours and 6 minutes. Find the next bucket dumping time: [LIST=1] [*]We start at 1:15 PM [*]Add 2 hours and we get 3:15 PM [*]Add 6 minutes and we get [B]3:21 PM[/B] [/LIST]

An initial deposit of $50 is now worth $400. The account earns 5.2% interest compounded continuously
An initial deposit of $50 is now worth $400. The account earns 5.2% interest compounded continuously. Determine how long the money has been in the account. [URL='https://www.mathcelebrity.com/simpint.php?av=400&p=50&int=5.2&t=&pl=Continuous+Interest']Using our continuous interest compound calculator solving for t[/URL], we get: t =[B] 39.99 periods[/B]

An interior designer charges $100 to visit a site, plus $55 to design each room. Identify a function
An interior designer charges $100 to visit a site, plus $55 to design each room. Identify a function that represents the total amount he charges for designing a certain number of rooms. What is the value of the function for an input of 6, and what does it represent? [U]Set up the cost function C(r) where r is the number of room to design:[/U] C(r) = Cost per room * r + Site Visit Fee C(r) = 55r + 100 [U]Now, the problem asks for an input of 6, which is [I]the number of rooms[/I]. So we want C(6) which is the [I]cost to design 6 rooms[/I]:[/U] C(6) = 55(6) + 100 C(6) = 330 + 100 C(6) = [B]430[/B]

An investor has $300,000 to invest, part at 12% and the remainder in a less risky investment at 7%.
An investor has $300,000 to invest, part at 12% and the remainder in a less risky investment at 7%. If the investment goal is to have an annual income of $27,000, how much should be put in each investment? Using our [URL='https://www.mathcelebrity.com/split-fund-interest-calculator.php?p=300000&i1=12&i2=7&itot=27000&pl=Calculate']split-fund interest calculator[/URL], we get: [LIST] [*][B]$120,000 in the 12% Fund[/B] [*][B]$180,000 in the 7% Fund[/B] [/LIST]

An investor invests $1000. Part of the investment is made at 5% interest and part of the investment
An investor invests $1000. Part of the investment is made at 5% interest and part of the investment is made at 10% interest. How much should be invested at 5% so the total interest in the first year is $80? Using our [URL='https://www.mathcelebrity.com/split-fund-interest-calculator.php?p=1000&i1=5&i2=10&itot=80&pl=Calculate']split fund interest calculator[/URL], we get: [B]$400[/B]

An irregular pentagon is a five sided figure. The two longest sides of a pentagon are each three tim
An irregular pentagon is a five sided figure. The two longest sides of a pentagon are each three times as long as the shortest side. The remaining two sides are each 8m longer than the shortest side. The perimeter of the Pentagon is 79m. Find the length of each side of the Pentagon. Let long sides be l. Let short sides be s. Let medium sides be m. We have 3 equations: [LIST=1] [*]2l + 2m + s = 79 [*]m = s + 8 [*]l = 3s [/LIST] Substitute (2) and (3) into (1): 2(3s) + 2(s + 8) + s = 79 Multiply through and simplify: 6s + 2s + 16 + s = 79 9s + 16 = 79 [URL='https://www.mathcelebrity.com/1unk.php?num=9s%2B16%3D79&pl=Solve']Using our equation calculator[/URL], we get [B]s = 7[/B]. This means from Equation (2): m = 7 + 8 [B]m = 15 [/B] And from equation (3): l = 3(7) [B]l = 21[/B]

An isosceles triangles non-congruent angle is 16 more than twice the congruent ones. What is the mea
An isosceles triangles non-congruent angle is 16 more than twice the congruent ones. What is the measure of all 3 angles? Let the congruent angles measurement be c. And the non-congruent angle measurement be n. We're given: [LIST=1] [*]n = 2c + 16 <-- Twice means we multiply by 2, and more than means we add 16 [*]2c + n = 180 <-- Since the sum of angles in an isosceles triangle is 180 [/LIST] Substitute (1) into (2): 2c + (2c + 16) = 180 Group like terms: 4c + 16 = 180 [URL='https://www.mathcelebrity.com/1unk.php?num=4c%2B16%3D180&pl=Solve']Typing this equation into our search engine[/URL], we get: [B]c = 41[/B] Substituting this value into Equation 1, we get n = 2(41) + 16 n = 82 + 16 [B]n = 98[/B]

An item cost $370 before tax, and the sales tax is 25.90 what is the percentage?
An item cost $370 before tax, and the sales tax is 25.90 what is the percentage? Sales Tax = Tax Amount/Original Bill Sales Tax = 25.90/370 Sales Tax = 0.07 Multiply by 100 to convert to a percent, we have[B] 7%[/B]

An item costs $470 before tax, and the sales tax is $14.10. Find the sales tax rate. Write your answ
An item costs $470 before tax, and the sales tax is $14.10. Find the sales tax rate. Write your answer as a percentage. Sales Tax Percent = 100% * Sales Tax / Before Tax Amount Sales Tax Percent = 100% * 14.10 / 470 Sales Tax Percent = 100% * 0.03 Sales Tax Percent = [B]3%[/B]

An oil tank contains 220.2 gallons of oil......
An oil tank contains 220.2 gallons of oil. Whenever the amount of oil drops below 90 gallons, an alarm sounds. If 145.3 gallons are pumped into a delivery truck, how many gallons must be pumped back into the tank in order to shut off the alarm? I'm doing a remedial math course and I need help with a lot of questions..

An operation is defined by a*b=3a-b.Calculate the exact value of 2*3
An operation is defined by a*b=3a-b.Calculate the exact value of 2*3 We're given a = 2 and b = 3. So the operator says: 3(2) - 3 6 - 3 [B]3[/B]

An orchard has 378 orange trees. The number of rows exceeds the number of trees per row by 3. How ma
An orchard has 378 orange trees. The number of rows exceeds the number of trees per row by 3. How many trees are there in each row? We have r rows and t trees per row. We're give two equations: [LIST=1] [*]rt = 378 [*]r = t + 3 [/LIST] Substitute equation (2) into equation (1) for r: (t + 3)t = 378 Multiply through: t^2 + 3t = 378 We have a quadratic equation. To solve this equation, we [URL='https://www.mathcelebrity.com/quadratic.php?num=t%5E2%2B3t%3D378&pl=Solve+Quadratic+Equation&hintnum=+0']type it in our search engine [/URL]and we get: t = 18 and t = -21 Since t cannot be negative, we get trees per row (t): [B]t = 18[/B]

An orchard has 816 apple trees. The number of rows exceeds the number of trees per row by 10. How ma
An orchard has 816 apple trees. The number of rows exceeds the number of trees per row by 10. How many trees are there in each row? Let the rows be r and the trees per row be t. We're given two equations: [LIST=1] [*]rt = 816 [*]r = t + 10 [/LIST] Substitute equation (2) into equation (1) for r: (t + 10)t = 816 t^2 + 10t = 816 Subtract 816 from each side of the equation: t^2 + 10t - 816 = 816 - 816 t^2 + 10t - 816 = 0 We have a quadratic equation. To solve this, we [URL='https://www.mathcelebrity.com/quadratic.php?num=t%5E2%2B10t-816%3D0&pl=Solve+Quadratic+Equation&hintnum=+0']type it in our search engine [/URL]and we get: t = (24, -34) Since the number of trees per row can't be negative, we choose [B]24[/B] as our answer

An ordinary fair die is rolled twice. The face value of the rolls is added together. Compute the pro
An ordinary fair die is rolled twice. The face value of the rolls is added together. Compute the probability of the following events: Event A: The sum is greater than 6. Event B: The sum is divisible by 5 or 6 or both. [URL='http://www.mathcelebrity.com/2dice.php?gl=2&pl=6&opdice=1&rolist=+2%2C3%2C9%2C10&dby=2%2C3%2C5&ndby=4%2C5&montect=+100']Sum greater than 6[/URL] = [B]7/12[/B] Sum is divisible by 5 or 6 or both This means a sum of 5, a sum of 6, a sum of 10, or a sum of 12. [URL='http://www.mathcelebrity.com/2dice.php?gl=1&pl=5&opdice=1&rolist=+2%2C3%2C9%2C10&dby=2%2C3%2C5&ndby=4%2C5&montect=+100']Sum of 5[/URL] = 1/9 or 4/36 [URL='http://www.mathcelebrity.com/2dice.php?gl=1&pl=6&opdice=1&rolist=+2%2C3%2C9%2C10&dby=2%2C3%2C5&ndby=4%2C5&montect=+100']Sum of 6[/URL] = 5/36 [URL='http://www.mathcelebrity.com/2dice.php?gl=1&pl=10&opdice=1&rolist=+2%2C3%2C9%2C10&dby=2%2C3%2C5&ndby=4%2C5&montect=+100']Sum of 10[/URL] = 1/12 or 3/36 [URL='http://www.mathcelebrity.com/2dice.php?gl=1&pl=12&opdice=1&rolist=+2%2C3%2C9%2C10&dby=2%2C3%2C5&ndby=4%2C5&montect=+100']Sum of 12[/URL] = 1/36 Adding all these up, we get: (4 + 5 + 3 + 1)/36 [B]13/36[/B]

An organic juice bottler ideally produces 215,000 bottle per day. But this total can vary by as much
An organic juice bottler ideally produces 215,000 bottle per day. But this total can vary by as much as 7,500 bottles. What is the maximum and minimum expected production at the bottling company? Our production amount p is found by adding and subtracting our variance amount: 215,000 - 7,500 <= p <= 215,000 + 7,500 [B](min) 207,500 <= p <=222,500 (max)[/B]

Ana has 24 carrots, 12 cucumbers, and 36 radishes. She wants to make identical vegetable baskets usi
Ana has 24 carrots, 12 cucumbers, and 36 radishes. She wants to make identical vegetable baskets using all of the vegetables. What is the greatest number of baskets she can make The key to solving this problem is asking what is the common factor between the 3 numbers. We want the greatest common factor or GCF [URL='https://www.mathcelebrity.com/gcflcm.php?num1=12&num2=24&num3=36&pl=GCF']GCF(12, 24, 36) [/URL]= [B]12[/B] We divide up our 12 baskets into carrots, cucumbers, and radishes. Each basket of the 12 baskets has the following: [LIST=1] [*]12 cucumbers / GCF of 12 = [B]1 cucumber per basket[/B] [*]24 carrots / GCF of 12 = [B]2 carrots per basket[/B] [*]36 radishes / GCF of 12 = [B]3 radishes per basket[/B] [/LIST] [B][MEDIA=youtube]D1KTOP0h2P4[/MEDIA][/B]

Ana was y years old 7 years ago. Represent her age twenty years from now
Ana was y years old 7 years ago. Represent her age twenty years from now twenty years from now, means we add 7 years to get to now and another 20 years to get to twenty years from now: y + 7 + 20 [B]y + 27[/B]

Anagram for July 5, 2019
Unscramble the anagram: [B]ANENXPOITLE [/B] Hint: Think finance

Ande has 8 pints of milk. If he drinks 1/4 of a pint of milk each day, how long will the 8 pints of
Ande has 8 pints of milk. If he drinks 1/4 of a pint of milk each day, how long will the 8 pints of milk last him? Milk Days = Total Pints of Milk / pints drank per day Milk Days = 8 / 1/4 Dividing by a fraction is the same as multiplying by it's reciprocal. The [URL='https://www.mathcelebrity.com/fraction.php?frac1=1%2F4&frac2=3%2F8&pl=Reciprocal']reciprocal of[/URL] 1/4 is 4/1, so we have: 8 * 4/1 = [B]32 days[/B]

Andrea has 6 hours to spend training for an upcoming race. She completes her training by running ful
Andrea has 6 hours to spend training for an upcoming race. She completes her training by running full speed the distance of the race and walking back the same distance to cool down. If she runs at a speed of 7mph and walks back at a speed of 3mph, how long should she plan to spend walking back? Let the distance be d. Running full speed one way, 7d Walking back the opposite way, 3d And we know 7d + 3d = 6 hours 10d = 6 hours d =3/5 hour

Andrea has one hour to spend training for an upcoming race she completes her training by running ful
Andrea has one hour to spend training for an upcoming race she completes her training by running full speed in the distance of the race and walking back the same distance to cool down if she runs at a speed of 9 mph and walks back at a speed of 3 mph how long should she plan on spending to walk back Let r = running time. Let w = walking time We're given two equations [LIST=1] [*]r + w = 1 [*]9r = 3w [/LIST] Rearrange equation (1) by subtract r from each side: [LIST=1] [*]w = 1 - r [*]9r = 3w [/LIST] Now substitute equation (1) into equation (2): 9r = 3(1 - r) 9r = 3 - 3r To solve for r, [URL='https://www.mathcelebrity.com/1unk.php?num=9r%3D3-3r&pl=Solve']we type this equation into our search engine[/URL] and we get: r = 0.25 Plug this into modified equation (1) to solve for w, and we get: w = 1. 0.25 [B]w = 0.75[/B]

Angad was thinking of a number. Angad adds 20 to it, then doubles it and gets an answer of 53. What
Angad was thinking of a number. Angad adds 20 to it, then doubles it and gets an answer of 53. What was the original number? The phrase [I]a number[/I] means an arbitrary variable, let's call it n. [LIST] [*]Start with n [*]Add 20 to it: n + 20 [*]Double it means we multiply the expression by 2: 2(n + 20) [*]Get an answer of 53: means an equation, so we set 2(n + 20) equal to 53 [/LIST] 2(n + 20) = 53 To solve for n, we [URL='https://www.mathcelebrity.com/1unk.php?num=2%28n%2B20%29%3D53&pl=Solve']type this equation into our search engine[/URL] and we get: n = [B]6.5[/B]

Angelica’s financial aid stipulates that her tuition cannot exceed $1000. If her local community col
Angelica’s financial aid stipulates that her tuition cannot exceed $1000. If her local community college charges a $35 registration fee plus $375 per course, what is the greatest number of courses for which Angelica can register? We set up the Tuition function T(c), where c is the number of courses: T(c) = Cost per course * c + Registration Fee T(c) = 35c + 375 The problem asks for the number of courses (c) where her tuition [I]cannot exceed[/I] $1000. The phrase [I]cannot exceed[/I] means less than or equal to, or no more than. So we setup the inequality for T(c) <= 1000 below: 35c + 375 <= 1000 To solve this inequality for c, we [URL='https://www.mathcelebrity.com/1unk.php?num=35c%2B375%3C%3D1000&pl=Solve']type it in our search engine and we get[/URL]: c <= 17.85 Since we cannot have fractional courses, we round down and get: c[B] <= 17[/B]

Angie and Kenny play online video games. Angie buy 2 software packages and 4 months of game play. Ke
Angie and Kenny play online video games. Angie buy 2 software packages and 4 months of game play. Kenny buys 1 software package and 1 month of game play. Each software package costs $25. If their total cost is $155, what is the cost of one month of game play. Let s be the cost of software packages and m be the months of game play. We have: [LIST] [*]Angie: 2s + 4m [*]Kenny: s + m [/LIST] We are given each software package costs $25. So the revised equations above become: [LIST] [*]Angie: 2(25) + 4m = 50 + 4m [*]Kenny: 25 + m [/LIST] Finally, we are told their combined cost is 155. So we add Angie and Kenny's costs together: 4m + 50 + 25 + m = 155 Combine like terms: 5m + 75 = 155 [URL='http://www.mathcelebrity.com/1unk.php?num=5m%2B75%3D155&pl=Solve']Typing this into our search engine[/URL], we get [B]m = 16[/B]

Angie is 11, which is 3 years younger than 4 times her sister's age.
Angie is 11, which is 3 years younger than 4 times her sister's age. Let her sister's age be a. We're given the following equation: 4a - 3 = 11 To solve for a, we [URL='https://www.mathcelebrity.com/1unk.php?num=4a-3%3D11&pl=Solve']type this equation into our math engine[/URL] and we get: [B]a = 3.5[/B]

Angie knew 90% of the answers on a worksheet. What fraction of the answers did she know?
Angie knew 90% of the answers on a worksheet. What fraction of the answers did she know? We [URL='https://www.mathcelebrity.com/perc.php?num=+5&den=+8&num1=+16&pct1=+80&pct2=+90&den1=+80&pct=90&pcheck=4&decimal=+65.236&astart=+12&aend=+20&wp1=20&wp2=30&pl=Calculate']type [I]90% as a fraction[/I] into our search engine[/URL] and we get: [B]9/10[/B]

Angle Ratio for a Triangle
Free Angle Ratio for a Triangle Calculator - Given an angle ratio for a triangle of a:b:c, this determines the angle measurements of the triangle.

Angular Momentum
Free Angular Momentum Calculator - Solves for any of the 4 variables in the angular momentum equation, L, V, M, and R

Ann purchased 8 packs of grape gum, 12 packs of cherry gum, and 6 packs of strawberry gum. If there
Ann purchased 8 packs of grape gum, 12 packs of cherry gum, and 6 packs of strawberry gum. If there are 6 pieces in each pack, how many pieces of gum did Ann purchased? Total Pieces = Packs x pieces per pack Total Pieces = (8 + 12 + 6) x 6 Total Pieces = 26 x 6 Total Pieces = [B]156[/B]

Ann took a taxi home from the airport. The taxi fare was $2.10 per mile, and she gave the driver a t
Ann took a taxi home from the airport. The taxi fare was $2.10 per mile, and she gave the driver a tip of $5 Ann paid a total of $49.10. Set up the cost function C(m) where m is the number of miles: C(m) = Mileage Rate x m + Tip 2.10m + 5 = 49.10 [URL='https://www.mathcelebrity.com/1unk.php?num=2.10m%2B5%3D49.10&pl=Solve']Type 2.10m + 5 = 49.10 into the search engine[/URL], and we get [B]m = 21[/B].

Anna has 50 coins in her piggy bank. She notices that she only has dimes and pennies. If she has exa
Anna has 50 coins in her piggy bank. She notices that she only has dimes and pennies. If she has exactly four times as many pennies as dimes, how many pennies are in her piggy bank? Let d be the number of dimes, and p be the number of pennies. We're given: [LIST=1] [*]d + p = 50 [*]p = 4d [/LIST] Substitute (2) into (1) d + 4d = 50 [URL='https://www.mathcelebrity.com/1unk.php?num=d%2B4d%3D50&pl=Solve']Type that equation into our search engine[/URL]. We get: d = 10 Now substitute this into Equation (2): p = 4(10) [B]p = 40[/B]

Anna is collecting boxes of cereal to deliver in a food bank. The volume of each cereal box is 324 c
Anna is collecting boxes of cereal to deliver in a food bank. The volume of each cereal box is 324 cubic inches. The picture shows the cereal boxes she has collected so far. A large delivery box holds three times as many boxes as Anna collected. About what is the volume of the delivery box? The picture has 12 cereal boxes. Since the delivery box holds three times as many cereal boxes as Anna collected, the delivery box holds 12 * 3 = 36 cereal boxes. With each cereal box having a volume of 324 cubic inches, we have the total volume as: V = 324 cubic inches * 36 cereal boxes V = [B]11,664 cubic inches[/B]

Anna made $60 babysitting. She spent 4/5 of the money on new shoes. How much money does she have lef
Anna made $60 babysitting. She spent 4/5 of the money on new shoes. How much money does she have left? [U]Calculate shoe spend[/U] [URL='https://www.mathcelebrity.com/fraction.php?frac1=60&frac2=4/5&pl=Multiply']4/5 of 60[/URL] = 48 [U]Calculate leftover money[/U] Leftover money = Babysitting money - shoe spend Leftover money = 60 - 48 Leftover money = [B]12[/B]

Anna painted 1/6 of a wall, Eric painted 1/5 of the wall, and Meadow painted 1/4 of the wall. There
Anna painted 1/6 of a wall, Eric painted 1/5 of the wall, and Meadow painted 1/4 of the wall. There are now 3910 square feet left to paint. How many square feet did Anna paint? [URL='https://www.mathcelebrity.com/gcflcm.php?num1=4&num2=5&num3=6&pl=LCM']Using 60 as a common denominator through least common multiple[/URL], we get: 1/6 = 10/60 1/5 = 12/60 1/4 = 15/60 10/60 + 12/60 + 15/60 = 37/60 Remaining part of the wall is 60/60 - 37[B]/[/B]60 = 23/60 3910/23 = 170 for each 1/60 of a wall Anna painted 1/6 or 10/60 of the wall. So we multiply 170 * 10 = [B]1,700 square feet[/B]

Anna paints a fence in 4 hours wile her brother paints it in 5 hours. If they work together, how lon
Anna paints a fence in 4 hours wile her brother paints it in 5 hours. If they work together, how long will it take them to paint the fence? Set up unit rates per hour: [LIST] [*]Anna paints 1/4 of a fence per hour [*]Brother paints 1/5 of a fence per hour [*]Combined, they paint [URL='https://www.mathcelebrity.com/fraction.php?frac1=1%2F4&frac2=1%2F5&pl=Add']1/4 + 1/5[/URL] = 9/20 of a fence per hour [/LIST] Setup a proportion of time to hours where h is the number of hours needed to paint the fence 9/20 of a fence the first hour 18/20 of a fence the second hour 2/20 is left. Each 1/20 of the fence takes 60/9 = 6 & 2/3 minutes 6 & 2/3 minutes * 2 = 13 & 1/3 minutes Final time is: [B]2 hours and 13 & 1/3 minutes[/B]

Anna’s age increased by 3 times her age, the result is 72
Anna’s age increased by 3 times her age, the result is 72. Let a be Anna's age. We have: a + 3a = 72 Combine like terms: (1 + 3)a = 72 4a = 72 [URL='https://www.mathcelebrity.com/1unk.php?num=4a%3D72&pl=Solve']Type 4a = 72 into our calculator[/URL], and we get [B]a = 18[/B].

Anne wants to make a platform that is 7 feet wide and 10 feet long. If she uses boards that measure
Anne wants to make a platform that is 7 feet wide and 10 feet long. If she uses boards that measure 6 inches wide by 2 feet long, how many boards will she need to complete the job? Area of platform which is a rectangle: A = lw A = 10 * 7 A = 70 Area of boards which are rectangles: A = lw A = 2 * 6 A = 12 We divide our platform area by our board area to get the number of boards needed: Boards needed = Platform Area / Board Area Boards needed = 70/12 Boards needed = 5.83333 We round up if we want full boards to be [B]6[/B]

Annie got a new video game. She scored 152 points on the first level, 170 points on the second level
Annie got a new video game. She scored 152 points on the first level, 170 points on the second level, 188 points on the third level, and 206 points on the fourth level. What kind of sequence is this? This is an [URL='https://www.mathcelebrity.com/sequenceag.php?num=152%2C170%2C188%2C206&n=10&pl=Calculate+Series&a1=&d=']arithmetic series as seen on our calculator[/URL]:

Annuity that pays 6.6% compounded monthly. If $950 is deposited into this annuity every month, how m
Annuity that pays 6.6% compounded monthly. If $950 is deposited into this annuity every month, how much is in the account after 7 years? How much of this is interest? Let's assume payments are made at the end of each month, since the problem does not state it. We have an annuity immediate formula. Interest rate per month is 6.6%/12 = .55%, or 0.0055. 7 years * 12 months per year gives us 84 deposits. Using our [URL='http://www.mathcelebrity.com/annimmpv.php?pv=&av=&pmt=950&n=84&i=0.55&check1=1&pl=Calculate']present value of an annuity immediate calculator[/URL], we get the following: [LIST=1] [*]Accumulated Value After 7 years = [B]$101,086.45[/B] [*]Principal = 79,800 [*]Interest Paid = (1) - (2) = 101,086.45 - 79,800 = [B]$21,286.45[/B] [/LIST]

Annulus
Free Annulus Calculator - Calculates the area of an annulus and the equation of the annulus using the radius of the large and small concentric circles.

Answer an electrician charges a base fee of $75. plus a $50 for each hour of work. The minimum the e
Answer an electrician charges a base fee of $75. plus a $50 for each hour of work. The minimum the electrician charges is $175. Create a table that shows the amount the electrician charges for 1,2,3, and 4 hours of work. The hourly cost for h hours worked is C(h): C(h) = Max(175, 50h + 75) 1 hour cost: C(1) = Max(175, 50(1) + 75) C(1) = Max(175, 50 + 75) C(1) = Max(175, 125) [B]C(1) = 175[/B] 2 hour cost: C(2) = Max(175, 50(2) + 75) C(2) = Max(175, 100 + 75) C(2) = Max(175, 175) [B]C(2) = 175[/B] 3 hour cost: C(3) = Max(175, 50(3) + 75) C(3) = Max(175, 150 + 75) C(3) = Max(175, 225) [B]C(3) = 225[/B] 4 hour cost: C(4) = Max(175, 50(4) + 75) C(4) = Max(175, 200 + 75) C(4) = Max(175, 275) [B]C(4) = 275[/B]

Antilog
Free Antilog Calculator - Calculates the antilog of a number using a base.

Apply for IELTS certificate online ( whatsapp : +237680047619 ) Get ielts band 7,8,9 online .
We are engaged in the production of registered TOEFL, IELTS, ESOL, CELTA / DELTA and other English certificates. Please note that our IELTS & TOEFL certificates are original and are registered in the database and can be verified. After your order has been placed, it only takes a few days for us to receive your data in the system. Once your data is captured in the system, it will be displayed forever on the IELTS or TOEFL website. legit and verifiable forever. We can help you to get IELTS and other certification without you taking the exams, The certificate is registered. This certificate for admission to the university and any type of immigration. We register your results in every ielts center around the world. All our certificates are original and British Council certified IELTS is the high-stakes English test for study, migration or work

Approximate Square Root Using Exponential Identity
Free Approximate Square Root Using Exponential Identity Calculator - Calculates the square root of a positive integer using the Exponential Identity Method

Approximately 2,800 red blood cells are created in the bone marrow each second. How many red blood c
Approximately 2,800 red blood cells are created in the bone marrow each second. How many red blood cells would be created in .03125 seconds? 2800 red blood cells / second * .01325 seconds = [B]87.5 red blood cells[/B]

April, May and June have 90 sweets between them. May has three-quarters of the number of sweets that
April, May and June have 90 sweets between them. May has three-quarters of the number of sweets that June has. April has two-thirds of the number of sweets that May has. How many sweets does June have? Let the April sweets be a. Let the May sweets be m. Let the June sweets be j. We're given the following equations: [LIST=1] [*]m = 3j/4 [*]a = 2m/3 [*]a + j + m = 90 [/LIST] Cross multiply #2; 3a = 2m Dividing each side by 2, we get; m = 3a/2 Since m = 3j/4 from equation #1, we have: 3j/4 = 3a/2 Cross multiply: 6j = 12a Divide each side by 12: a = j/2 So we have: [LIST=1] [*]m = 3j/4 [*]a = j/2 [*]a + j + m = 90 [/LIST] Now substitute equation 1 and 2 into equation 3: j/2 + j + 3j/4 = 90 Multiply each side by 4 to eliminate fractions: 2j + 4j + 3j = 360 To solve this equation for j, we [URL='https://www.mathcelebrity.com/1unk.php?num=2j%2B4j%2B3j%3D360&pl=Solve']type it in our search engine[/URL] and we get: j = [B]40[/B]

Arc Length and Area of a Sector of a Circle
Free Arc Length and Area of a Sector of a Circle Calculator - Calculates the arc length of a circle and the area of the sector of a circle

Area Conversions
Free Area Conversions Calculator - This calculator converts between the following area measurements:
acre
hectare
square inch
square foot
square yard
square mile
square millimeter
square meter
square kilometer

area of a rectangle
area of a rectangle Let l be the length and w be the width of a rectangle. The Area (A) is: A = [B]lw[/B]

Ariel was playing basketball. 1 of her shots went in the hoop. 2 of her shots did not go in the hoop
Ariel was playing basketball. 1 of her shots went in the hoop. 2 of her shots did not go in the hoop. How many shots were there in total? Total shots = Shots made + shots missed Total shots = 1 + 2 Total shots = [B]3[/B]

Arithmetic and Geometric and Harmonic Sequences
Free Arithmetic and Geometric and Harmonic Sequences Calculator - This will take an arithmetic series or geometric series or harmonic series, and an optional amount (n), and determine the following information about the sequence
1) Explicit Formula
2) The remaining terms of the sequence up to (n)
3) The sum of the first (n) terms of the sequence Also known as arithmetic sequence, geometric sequence, and harmonic sequence

Arithmetic Annuity
Free Arithmetic Annuity Calculator - Calculates the Present Value, Accumulated Value (Future Value), First Payment, or Arithmetic Progression of an Increasing or Decreasing Arithmetic Annuity Immediate.

Arizona became a state in 1912. This was 5 years after Oklahoma became a state. Which equation can b
Arizona became a state in 1912. This was 5 years after Oklahoma became a state. Which equation can be used to find the year Oklahoma became a state? In what year did Oklahoma become a state? Let o be the year Oklahoma became a state: o = 1912 - 5 o = [B]1907[/B]

Arnie bought some bagels at 20 cents each. He ate 4, and sold the rest at 30 cents each. His profit
Arnie bought some bagels at 20 cents each. He ate 4, and sold the rest at 30 cents each. His profit was $2.40. How many bagels did he buy? Let x be the number of bagels Arnie sold. We have the following equation: 0.30(x - 4) - 0.20(4) = 2.40 Distribute and simplify: 0.30x - 1.20 - 0.8 = 2.40 Combine like terms: 0.30x - 2 = 2.40 Add 2 to each side: 0.30x = 4.40 Divide each side by 0.3 [B]x = 14.67 ~ 15[/B]

Arthur had $90. He spent $40 and gave $20 to his brother. What fraction of Arthur's money is left
Arthur had $90. He spent $40 and gave $20 to his brother. What fraction of Arthur's money is left? Arthur starts with $90. He gives away $40, so now he has $90 - $40 = $50. Next, he gives $20 to his brother, so now he has $50 - $20 = $30. So Arthur has 30/90 left. [URL='https://www.mathcelebrity.com/fraction.php?frac1=30%2F90&frac2=3%2F8&pl=Simplify']We type 30/90 into our search engine[/URL] and simplify to get: [B]1/3[/B]

Arvin is twice as old as Cory. The sum of their ages is 42. What are their ages?
Arvin is twice as old as Cory. The sum of their ages is 42. What are their ages? Let Arvin's age be a. Let Cory's age be c. We're given two equations: [LIST=1] [*]a = 2c [*]a + c = 42 [/LIST] Plug equation (1) into equation (2): 2c + c = 42 [URL='https://www.mathcelebrity.com/1unk.php?num=2c%2Bc%3D42&pl=Solve']Plug this into our search engine[/URL] and we get: [B]c = 14[/B] Now, we plug c = 14 into equation 1 to solve for a: a = 2(14) [B]a = 28[/B]

As a salesperson you will earn $600 per month plus a commission of 20% of sales. Find the minimum am
As a salesperson you will earn $600 per month plus a commission of 20% of sales. Find the minimum amount of sales you need to make in order to receive a total income of at least $1500 per month. Let the amount of sales be s. The phrase [I]at least[/I] means greater than or equal to. Since 20% is 0.2, We want to know when: 0.20s + 600 >= 1500 We [URL='https://www.mathcelebrity.com/1unk.php?num=0.20s%2B600%3E%3D1500&pl=Solve']type this inequality into our search engine to solve for s[/URL] and we get: s >= [B]4500[/B]

As a salesperson, Lauren earns a base salary of $94 per week plus a commission of 10% of sales. If s
As a salesperson, Lauren earns a base salary of $94 per week plus a commission of 10% of sales. If she had $90 in sales last week, what was her total pay? [B][U]Use the Base plus Commission formula above[/U][/B] Salary = Base Salary + 10%(Total Sales) Salary = $94 + 0.1(90) Salary = $94 + $9 Salary = [B]$103[/B]

As a salesperson, you are paid $50 per week plus $2 per sale. This week you want your pay to be at l
As a salesperson, you are paid $50 per week plus $2 per sale. This week you want your pay to be at least $100. What is the minimum number of sales you must make to earn at least $100? Set up the inequality where s is the amount of sales you make: 50 + 2s >= 100 We use >= because the phrase [I]at least[/I] 100 means 100 or more Subtract 50 from each side: 2s >= 50 Divide each side by 2 [B]s >= 25[/B]

As the sample size increases, we assume:
As the sample size increases, we assume: a. ? increases b. ? increases c. The probability of rejecting a hypothesis increases d. Power increases [B]d. Power increases[/B] [LIST] [*]Power increases if the standard deviation is smaller. [*]If the difference between the means is bigger, the power is bigger. [*]Sample size also increases power [/LIST]

Ashley age is 2 times Johns age. The sum of their ages is 63. What is Johns age?
Ashley age is 2 times Johns age. The sum of their ages is 63. What is Johns age? Let Ashley's age be a. Let John's age be j. We have two equations: [LIST=1] [*]a = 2j [*]a + j = 63 [/LIST] Now substitute (1) into (2) (2j) + j = 63 Combine like terms: 3j = 63 [URL='http://www.mathcelebrity.com/1unk.php?num=3j%3D63&pl=Solve']Typing 3j = 63 into our search engine[/URL], we get [B]j = 21[/B]

Ashley deposited $4000 into an account with 2.5% interest, compounded semiannually. Assuming that no
Ashley deposited $4000 into an account with 2.5% interest, compounded semiannually. Assuming that no withdrawals are made, how much will she have in the account after 10 years? Semiannual means twice a year, so 10 years * 2 times per year = 20 periods. We use this and [URL='https://www.mathcelebrity.com/compoundint.php?bal=4000&nval=20&int=2.50&pl=Semi-Annually']plug the numbers into our compound interest calculator[/URL] to get: [B]$5,128.15[/B]

ason decided that he will sell his stocks if their values per share (x) goes below $5 or above $15.
Jason decided that he will sell his stocks if their values per share (x) goes below $5 or above $15. Write a compound inequality represents the values at which Jason will sell his stocks? Below $5 is also known as less than $5: x < 5 Above $15 is also known as greater than $15 x > 15 We write the compound inequality: [B]x < 5 U x > 15[/B]

Associative Property
Free Associative Property Calculator - Demonstrates the associative property using 3 numbers. Covers the Associative Property of Addition and Associative Property of Multiplication. Also known as the Associative Law of Addition and Associative Law of Multiplication Numerical Properties

Assume that a balloon is spherical shaped. If you have a balloon with the radius of 3, what’s the vo
Assume that a balloon is spherical shaped. If you have a balloon with the radius of 3, what’s the volume? Using our [URL='https://www.mathcelebrity.com/sphere.php?num=3&pl=Radius']sphere calculator[/URL], we get Volume (V): V = [B]36pi or 113.0973[/B]

Assume that in your Abnormal Psychology class you have earned test scores of 74%, 78%, and 63%, and
Assume that in your Abnormal Psychology class you have earned test scores of 74%, 78%, and 63%, and only one test remains. If you need a mean score of 80% to earn a B for you final grade, is it possible for you to accomplish this? Assume there is no extra credit. Show work and explain why or why not. Hint: you're taking 4 tests total. Using our [URL='https://www.mathcelebrity.com/missingaverage.php?num=74%2C78%2C63&avg=80&pl=Calculate+Missing+Score']missing average calculator with our 3 given scores and target average[/URL], we get: A 4th score needed of 105. Since the most you can score on an exam is 100, [B][I]this is impossible[/I][/B].

Assume that you make random guesses for 5 true-or-false questions
Assume that you make random guesses for 5 true-or-false questions. (a) What is the probability that you get all 5 answers correct? (Show work and write the answer in simplest fraction form) (b) What is the probability of getting the correct answer in the 5th question, given that the first four answers are all wrong? (Show work and write the answer in simplest fraction form) (c) If event A is “Getting the correct answer in the 5th question” and event B is “The first four answers are all wrong”. Are event A and event B independent? Please explain. (a) Correct Answer on each one is 1/2 or 0.5. Since all are independent events, we have: (1/2)^5 = [B]1/32[/B] (b) We have [B]1/2[/B] (1/2)^4 * 1/2/((1/2)^4) c) [B]Independent since you could have gotten correct or wrong on any of the 4 and the probability does not change[/B]

Assume the speed of vehicles along a stretch of I-10 has an approximately normal distribution with a
Assume the speed of vehicles along a stretch of I-10 has an approximately normal distribution with a mean of 71 mph and a standard deviation of 8 mph. a. The current speed limit is 65 mph. What is the proportion of vehicles less than or equal to the speed limit? b. What proportion of the vehicles would be going less than 50 mph? c. A new speed limit will be initiated such that approximately 10% of vehicles will be over the speed limit. What is the new speed limit based on this criterion? d. In what way do you think the actual distribution of speeds differs from a normal distribution? a. Using our [URL='http://www.mathcelebrity.com/probnormdist.php?xone=65&mean=71&stdev=8&n=+1&pl=P%28X+%3C+Z%29']z-score calculator[/URL], we see that P(x<65) = [B]22.66%[/B] b. Using our [URL='http://www.mathcelebrity.com/probnormdist.php?xone=+50&mean=71&stdev=8&n=+1&pl=P%28X+%3C+Z%29']z-score calculator[/URL], we see that P(x<50) = [B]0.4269%[/B] c. [URL='http://www.mathcelebrity.com/zcritical.php?a=0.9&pl=Calculate+Critical+Z+Value']Inverse of normal for 90% percentile[/URL] = 1.281551566 Plug into z-score formula: (x - 71)/8 = 1.281551566 [B]x = 81.25241252[/B] d. [B]The shape/ trail differ because the normal distribution is symmetric with relatively more values at the center. Where the actual has a flatter trail and could be expected to occur.[/B]

Assume you have a laptop worth 2900. There is a 3 percent chance of it getting lost what’s the fair
Assume you have a laptop worth 2900. There is a 3 percent chance of it getting lost what’s the fair premium insurance? Fair premium Insurance = Price * probability of loss: Fair premium Insurance = 2,900 * 3% Fair premium Insurance = [B]87[/B]

assume your math class has 10 sophomores and 7 juniors. there are 3 female sophomores and 4 male jun
assume your math class has 10 sophomores and 7 juniors. there are 3 female sophomores and 4 male juniors. what is the probability of randomly selecting a student who is a female or a junior [U]Sophomores:[/U] 10 sophomores: 3 female male = 10 - 3 = 7 [U]Juniors:[/U] 7 juniors 4 males female = 7 - 4 = 3 [U]Females:[/U] Total Females = Female Sophomores + Female Juniors Total Females = 3 + 3 Total Females = 6 [U]Total Students:[/U] Total Students = Total Sophomores + Total Juniors Total Students = 10 + 7 Total Students = 17 [U]We want P(female or Junior). We use the formula below to avoid duplicates:[/U] P(female or Junior) = P(female) + P(junior) - P(female and junior) P(female or Junior) = Total Females / Total Students + Total Juniors / Total Students - Total Female Juniors / Total Students P(female or Junior) = 6/17 + 7/17 - 3/17 P(female or Junior) = [B]10/17[/B]

Assuming a standard 52-card deck, what's the probability of dealing three eights in a row when the c
Assuming a standard 52-card deck, what's the probability of dealing three eights in a row when the cards are returned and the deck is shuffled between each draw? There are four (8's) in a standard 52 card deck. The probability of drawing an 8 is: 4/52 [URL='https://www.mathcelebrity.com/fraction.php?frac1=4%2F52&frac2=3%2F8&pl=Simplify']Using our fraction simplifier[/URL], we get: 1/13 Now, with each draw, we replace the deck. So each draw of an 8 has a 1/13 probability. And since each of the three draws is independent, we multiply each probability: 1/13 * 1/13 * 1/13 = [B]1/2197 or 0.00045516613[/B]

ASVAB Exam Questions
Exam Questions from the Armed Services Vocational Aptitude Battery (ASVAB)

At 1:00 pm you have 24 megabytes of a movie and at 1:15 pm you have 96 megabytes of a movie. What is
At 1:00 pm you have 24 megabytes of a movie and at 1:15 pm you have 96 megabytes of a movie. What is the download rate in megabytes per minute? First, find the number of minutes: 1:15 - 1:00 = 15 minutes Next, determine the difference in megabytes 96 - 24 = 72 Finally, determine the download rate: 72 megabytes / 15 minutes = [B]4.8 megabytes per minute [MEDIA=youtube]RCvs3TQMzdM[/MEDIA][/B]

At 2:18 Pm, a parachutist is 4900 feet above the ground. At 2:32 pm, the parachutist is 2100 above t
At 2:18 Pm, a parachutist is 4900 feet above the ground. At 2:32 pm, the parachutist is 2100 above the ground. Find the average rate of change in feet per minute Average Rate of Change = Change in Distance / Change in time Average Rate of Change = (4900 - 2100) / (2:32 - 2:18) Average Rate of Change = 2800 / 14 Average Rate of Change = [B]200 feet per minute[/B]

At 4am the outside temperature was -28 By 4pm it rose 38° What was the temperature at 4pm
At 4am the outside temperature was -28 By 4pm it rose 38° What was the temperature at 4pm -28 + 38 = [B]10 degrees (above zero)[/B]

At 7:00 AM, the temperature started dropping 1 degree Celsius per hour until it reached 30 degrees C
At 7:00 AM, the temperature started dropping 1 degree Celsius per hour until it reached 30 degrees Celsius at 12:00 PM. What was the temperature at 7:00 AM? 7:00 AM to 12:00 PM is 5 hours. 1 degree per hour * 5 hours = 5 degrees. If it's 30 degrees at 12 pm, then it was 30 + 5 = 35 degrees Celsius at 7:00 AM, since it dropped each hour.

at 9:30am you enter a parking garage. It cost $3.25 for each hour to park your car. You leave the ga
at 9:30am you enter a parking garage. It cost $3.25 for each hour to park your car. You leave the garage at 2:00pm. How much does it cost to park? [U]Calculate Hours:[/U] 9:30 am to 10:00 am is 0.5 hours 10 am to 2 pm is 4 hours So our total time is 4.5 hours [U]Calculate Total Cost[/U] Total Cost = Hours * Cost per hour Total Cost = 4.5 * 3.25 Total Cost = [B]$14.63[/B]

at a bakery the cost of one cupcake and 2 slices of pie is $12.40. the cost of 2 cupcakes and 3 slic
at a bakery the cost of one cupcake and 2 slices of pie is $12.40. the cost of 2 cupcakes and 3 slices of pie costs $20.20. what is the cost of one cupcake? Let the number of cupcakes be c Let the number of pie slices be p Total Cost = Unit cost * quantity So we're given two equations: [LIST=1] [*]1c + 2p = 12.40 [*]2c + 3p = 20.20 [/LIST] We can solve this system of equations any one of three ways: [LIST] [*][URL='https://www.mathcelebrity.com/simultaneous-equations.php?term1=1c+%2B+2p+%3D+12.40&term2=2c+%2B+3p+%3D+20.20&pl=Substitution']Substitution Method[/URL] [*][URL='https://www.mathcelebrity.com/simultaneous-equations.php?term1=1c+%2B+2p+%3D+12.40&term2=2c+%2B+3p+%3D+20.20&pl=Elimination']Elimination Method[/URL] [*][URL='https://www.mathcelebrity.com/simultaneous-equations.php?term1=1c+%2B+2p+%3D+12.40&term2=2c+%2B+3p+%3D+20.20&pl=Cramers+Method']Cramer's Rule[/URL] [/LIST] No matter which method we choose, we get the same answer: [LIST] [*][B]c = 3.2[/B] [*]p = 4.6 [/LIST]

At a birthday party, 13 children ate cake, 9 ate ice cream, 7 ate both cake and ice cream, and one c
At a birthday party, 13 children ate cake, 9 ate ice cream, 7 ate both cake and ice cream, and one child had neither cake nor ice cream. How many children were at the party? We have one of three scenarios [LIST=1] [*]A child ate cake (possibly ice-cream) [*]A child ate ice cream (possible cake) [*]A child ate neither [/LIST] We have cake + ice cream + neither - both cake and ice cream. We subtract both cake and ice cream to avoid duplicates 13 + 9 + 1 - 7 = 16 kids at the party

At a carnival, Diane bought 14 packs of 7 tickets each. She also found 9 more tickets on the ground.
At a carnival, Diane bought 14 packs of 7 tickets each. She also found 9 more tickets on the ground. How many tickets did Diane have in all? 14 packs * 7 tickets/pack = 98 tickets Now add the 9 tickets on the ground: 98 + 9 = [B]107 tickets[/B]

At a carnival, the price of an adult ticket is $6 while a child ticket is $4. On a certain day, 30 m
At a carnival, the price of an adult ticket is $6 while a child ticket is $4. On a certain day, 30 more child tickets than adult tickets were sold. If a total of $6360 was collected from the total ticket sale that day, how many child tickets were sold? Let the number of adult tickets be a. Let the number of child tickets be c. We're given two equations: [LIST=1] [*]c = a + 30 [*]6a + 4c = 6360 [/LIST] Substitute equation (1) into equation (2): 6a + 4(a + 30) = 6360 Multiply through to remove parentheses: 6a + 4a + 120 = 6360 T[URL='https://www.mathcelebrity.com/1unk.php?num=6a%2B4a%2B120%3D6360&pl=Solve']ype this equation into our search engine[/URL] to solve for a and we get: a = 624 Now substitute a = 624 back into equation (1) to solve for c: c = 124 + 30 c = [B]154[/B]

At a certain university, 60% of the students enrolled in a math course, 50% are enrolled in an Engli
At a certain university, 60% of the students enrolled in a math course, 50% are enrolled in an English course, and 40% are enrolled in both. What percentage of the students are enrolled in an English course and/or a math course? Let M be a math course, E be an english course, We are given: [LIST] [*]P(M) = 0.6 [*]P(E) = 0.5 [*]P(E AND M) = 0.4 [*]We want P(E U M) [/LIST] Using [URL='http://www.mathcelebrity.com/probunion2.php?pa=0.6+&pb=+0.5&paintb=+0.4&aub=+&pl=Calculate']two event probability[/URL], we get [B]P(E U M) = 0.7[/B]

At a concert there were 25 more women than men. The total number of people at the concert was 139. F
At a concert there were 25 more women than men. The total number of people at the concert was 139. Find the number of women and the number of men at the concert. Let men be m and women be w. We're given two equations. [LIST=1] [*]w = m + 25 [*]m + w = 139 [/LIST] Substitute equation (1) into equation (2): m + m + 25 = 139 To solve for m, we [URL='https://www.mathcelebrity.com/1unk.php?num=m%2Bm%2B25%3D139&pl=Solve']type this equation into our search engine[/URL] and we get: m = [B]57 [/B] To find w, we substitute m = 57 into equation (1): w = 57 + 25 w = [B]82[/B]

At a concert, 20% of the audience members were teenagers. If the number of teenagers at the concert
At a concert, 20% of the audience members were teenagers. If the number of teenagers at the concert was 360, what was the total number of audience members? We're looking for total audience members where [I]20% of what equals 360[/I]? [URL='https://www.mathcelebrity.com/perc.php?num=+5&den=+8&num1=360&pct1=20&pcheck=2&pct2=+70&den1=+80&pct=+82&decimal=+65.236&astart=+12&aend=+20&wp1=20&wp2=30&pl=Calculate']Type this expression into our search engine[/URL] and we get: Audience = [B]1,800[/B]

At a festival, Cherly bought 5 ride tokens and 9 game tokens. She spent $59. Let x represent the cos
At a festival, Cherly bought 5 ride tokens and 9 game tokens. She spent $59. Let x represent the cost of ride tokens and let y represent the cost of game tokens. Write an equation in standard for that can be used to determine how much each type of token costs. We know that: Token Cost + Game Cost = Total Cost Since cost = price * quantity, we have: [B]5x + 9y = 59[/B]

At a football game, a vender sold a combined total of 117 sodas and hot dogs. The number of hot dogs
At a football game, a vender sold a combined total of 117 sodas and hot dogs. The number of hot dogs sold was 59 less than the number of sodas sold. Find the number of sodas sold and the number of hot dogs sold. [U]Let h = number of hot dogs and s = number of sodas. Set up our given equations:[/U] [LIST=1] [*]h + s = 117 [*]h = s - 59 [/LIST] [U]Substitute (2) into (1)[/U] (s - 59) + s = 117 [U]Combine s terms[/U] 2s - 59 = 117 [U]Using our [URL='http://www.mathcelebrity.com/1unk.php?num=2s-59%3D117&pl=Solve']equation solver[/URL], we find:[/U] [B]s = 88 [/B] [U]Plug s = 88 into (2)[/U] h = 88 - 59 [B]h = 29[/B]

At a homecoming football game, the senior class sold slices of pizza for $.75 each and hamburgers fo
At a homecoming football game, the senior class sold slices of pizza for $.75 each and hamburgers for $1.35 each. They sold 40 more slices of pizza than hamburgers, and sales totaled $292.5. How many slices of pizza did they sell Let the number of pizza slices be p and the number of hamburgers be h. We're given two equations: [LIST=1] [*]p = h + 40 [*]1.35h + 0.75p = 292.50 [/LIST] [I]Substitute[/I] equation (1) into equation (2) for p: 1.35h + 0.75(h + 40) = 292.50 1.35h + 0.75h + 30 = 292.50 2.10h + 30 = 292.50 To solve for h, we [URL='https://www.mathcelebrity.com/1unk.php?num=2.10h%2B30%3D292.50&pl=Solve']plug this equation into our search engine[/URL] and we get: h = 125 The problem asks for number of pizza slices sold (p). So we substitute our value above of h = 125 into equation (1): p = 125 + 40 p = [B]165[/B]

At a light bulb factory 4 out of every 25 light bulbs are defective. How many light bulbs would you
At a light bulb factory 4 out of every 25 light bulbs are defective. How many light bulbs would you excpect to be defective out of 350 light bulbs Set up a proportion of light bulbs to defects where d is the number of defects per 350 light bulbs: 4/25 = b/350 [URL='https://www.mathcelebrity.com/prop.php?num1=4&num2=b&den1=25&den2=350&propsign=%3D&pl=Calculate+missing+proportion+value']Type this proportion into our search engine[/URL], and we get: b = [B]56[/B]

At a local fitness center, members pay a $10 membership fee and $3 for each aerobics class. Nonme
At a local fitness center, members pay a $10 membership fee and $3 for each aerobics class. Nonmembers pay $5 for each aerobics class. For what number of aerobics classes will the cost for members and nonmembers be the same? Set up the cost functions where x is the number of aerobics classes: [LIST] [*]Members: C(x) = 10 + 3x [*]Non-members: C(x) = 5x [/LIST] Set them equal to each other 10 + 3x = 5x Subtract 3x from both sides: 2x = 10 Divide each side by 2 [B]x = 5 classes[/B]

At a local fitness center, members pay an $8 membership fee and $3 for each aerobics class. Nonmembe
At a local fitness center, members pay an $8 membership fee and $3 for each aerobics class. Nonmembers pay $5 for each aerobics class. For what number of aerobics classes will the cost for members be equal to nonmembers? Set up two cost equations C(x): [LIST=1] [*]Members: C(x) = 8 + 3x [*]Nonmembers: C(x) = 5x [/LIST] Set the two cost equations equal to each other: 8 + 3x = 5x Subtract 3x from each side 2x = 8 Divide each side by 2 [B]x = 4[/B]

at a party, there are 72 people. The ratio of men to ladies to kids is 4 to 3 to 2.
at a party, there are 72 people. The ratio of men to ladies to kids is 4 to 3 to 2. [LIST] [*]How many men at the party? [*]How many ladies at the party? [*]How many kids at the party? [/LIST] Our total ratio denominator is 4 + 3 + 2 = 9. To find the number of each type of person, we take their ratio divided by their ratio numerator times 72 people at the party [U]Calculate ratios:[/U] [LIST] [*]Men: [URL='https://www.mathcelebrity.com/fraction.php?frac1=4%2F9&frac2=72&pl=Multiply']4/9 * 72[/URL] = [B]32[/B] [*]Ladies: [URL='https://www.mathcelebrity.com/fraction.php?frac1=3%2F9&frac2=72&pl=Multiply']3/9 * 72[/URL] = [B]24[/B] [*]Kids: [URL='https://www.mathcelebrity.com/fraction.php?frac1=2%2F9&frac2=72&pl=Multiply']2/9 * 72[/URL] = [B]16[/B] [/LIST] [U]Check our work:[/U] Men + Ladies + Kids = 32 + 24 + 16 Men + Ladies + Kids = 72 <-- This checks out!

At a rate of 4 gallons per min , how long will it take to fill a 300 gallon swimming pool
At a rate of 4 gallons per min , how long will it take to fill a 300 gallon swimming pool Time to fill = Total Gallons of the Pool / Fill Rate Time to Fill = 300 gallons / 4 gallons per minute Time to Fill = [B]75 minutes[/B]

At a recent motorcycle rally, the number of men exceeded the number of women by 247. If x represents
At a recent motorcycle rally, the number of men exceeded the number of women by 247. If x represents the number of women, write an expression for the number of men. [B]m = x + 247[/B]

At Appliance Market, a salesperson sells a dishwasher for $569. She gets a commission rate of 18 per
At Appliance Market, a salesperson sells a dishwasher for $569. She gets a commission rate of 18 percent. Which expression represents how much she will receive in commission from the sale? Since 18 percent = 0.18, we have: Commission = Sales * Commission Percent Commission = 569 * 0.18 Commission = [B]$102.42[/B]

At Billy’s Baseball Dugout, they are having a sale on merchandise. All bats cost $45.00, sunflower s
At Billy’s Baseball Dugout, they are having a sale on merchandise. All bats cost $45.00, sunflower seeds, $1.50, and cleats $85.00. Write an expression if you bought b bats, s sunflower seeds, and c cleats. Since amount = cost * quantity, we have a cost of: [B]45b + 1.50s + 85c[/B]

At Falling Creek Middle School, they noticed that 3 out of every 4 buses were on time. If there are
At Falling Creek Middle School, they noticed that 3 out of every 4 buses were on time. If there are a total of 32 buses that drop off at this school, how many buses will NOT be on time. If 3/4 are on time, we have 1 - 3/4 are not on time. 1 = 4/4 4/4 - 3/4 = 1/4 are not on time We multiply 1/4(32) = [B]8 buses will NOT be on time[/B].

At midnight in Winnipeg, the temperature was ?23°C. During the next 24 hours, the temperature rose 1
At midnight in Winnipeg, the temperature was ?23°C. During the next 24 hours, the temperature rose 12°C, then dropped 8°C. What was the final temperature? We start with ?23°C Temperatures rising 12°C mean we add: -23 + 12 = -11 Temperatures dropping 8°C mean we subtract: -11 - 8 = [B]-19°C[/B]

At midnight, the temperature was -15 degrees Fahrenheit. By the next morning, the temperature had go
At midnight, the temperature was -15 degrees Fahrenheit. By the next morning, the temperature had gone up to 20 degrees. What was the temperature then? The student meant to say gone up 20 degrees. We start with -15. We add 20 -15 + 20 = [B]5[/B]

At night, the average temperature on the surface of Saturn is -150 C. During the day, the temperatur
At night, the average temperature on the surface of Saturn is -150 C. During the day, the temperature rises 27 C. What is the average temperature on the planet's surface during the day? Rising temperature means we add, so we have: -150+ 27 = [B]-123C[/B]

At Priscilla's school, the final grade for her Calculus course is weighted as follows: Tests: 50% Qu
At Priscilla's school, the final grade for her Calculus course is weighted as follows: Tests: 50% Quizzes: 30% Homework: 20% Priscilla has an average of 87% on her tests, 100% on her quizzes, and 20% on her homework. What is Priscilla's weighted average? Weighted Average gives weights to each percent of the average as follows: Weighted Average = Average * weighting percent Weighted Average = Test Average * Test Weighting + Quiz Average. * Quiz Weighting + Homework Average * Homework Weighting Weighted Average = 87% * 50% + 100% * 30% + 20% * 20% Weighted Average = 43.5% + 30% + 4% Weighted Average = [B]77.5%[/B]

At Sams Club, 32 cans of Coke cost a total of $8.96. What is the cost per can?
At Sams Club, 32 cans of Coke cost a total of $8.96. What is the cost per can? Unit Cost is Total Cost / Number of Units Unit Cost = $8.96/32 Unit Cost = [B]$0.28[/B]

At the 2002 Winter Olympics, Austria won 2 gold medals. This was 1/8 of the total medals Austria won
At the 2002 Winter Olympics, Austria won 2 gold medals. This was 1/8 of the total medals Austria won. How many did Austria win? Let t be the total medals Austria won. We have: 2 = x/8 Cross multiply, we get: x = 2 * 8 x = [B]16[/B]

At the beginning of Jack's diet, he was 257 pounds. If he lost 3 pounds per week, find his weight af
At the beginning of Jack's diet, he was 257 pounds. If he lost 3 pounds per week, find his weight after 12 weeks. A loss of weight means we subtract from Jack's current weight. New Weight = Current Weight - Weight Loss per week * number of weeks New Weight =257 - 3*12 New Weight =257 - 36 New Weight =[B] 221[/B]

At the end of the 2021 NBA season, the NY Knicks had 10 more wins than losses. This NBA season the N
At the end of the 2021 NBA season, the NY Knicks had 10 more wins than losses. This NBA season the NY Knicks played a total of 72 times. Find a solution to this problem and explain. Let w be the number of wins Let l be the number of losses We're given two equations: [LIST=1] [*]w = l + 10 [*]l + w = 72 [/LIST] To solve this system of equations, substitute equation (1) into equation (2) for w: l + l + 10 = 72 To solve this equation for l, we [URL='https://www.mathcelebrity.com/1unk.php?num=l%2Bl%2B10%3D72&pl=Solve']type it in our math engine[/URL] and we get: l = [B]31 [/B] To solve for w, we substitute l = 31 into equation (1): w = 31 + 10 w = [B]41[/B]

At the end of the day, a bakery had 1/2 of a pie left over. The 4 employees each took home the same
At the end of the day, a bakery had 1/2 of a pie left over. The 4 employees each took home the same amount of leftover pie. How much pie did each employee take home? We have 1/2 of the pie eaten, if 1/2 was left over. So 1/2 of a pie divided by 4 employees = [B]1/8 of a pie per person[/B]. To check our work, we have 4 * 1/8 = 4/8 = 1/2 of pie eaten.

At the end of the day, the temperature is -16°C. During the day it dropped 12°C. What was the temper
At the end of the day, the temperature is -16°C. During the day it dropped 12°C. What was the temperature in the morning? Write an equation to represent, then solve and verify your answer let the starting temperature be s. If the temperature dropped, that means we subtract, so we have the following equation: s - 12 = -16 To solve this equation, we [URL='https://www.mathcelebrity.com/1unk.php?num=s-12%3D-16&pl=Solve']type it in our search engine[/URL] and we get: s = [B]-4[/B]

At the end of the week, Francesca had a third of her babysitting money left after spending $14.65 on
At the end of the week, Francesca had a third of her babysitting money left after spending $14.65 on a movie and popcorn and another $1.35 on a pen. How much did she earn babysitting? Let the original amount of money earned for babysitting be b. We're given: [LIST=1] [*]Start with b [*]Spending 14.65 for a movie means we subtract 14.65 from b: b - 14.65 [*]Spending 1.35 on a pen means we subtract another 1.35 from step 2: b - 14.65 - 1.35 [*]Francesca has a third of her money left. So we set step 3 equal to 1/3 of b [/LIST] b - 14.65 - 1.35 = b/3 Multiply each side of the equation by 3 to remove the fraction 3(b - 14.65 - 1.35) = 3b/3 3b - 43.95 - 4.05 = b To solve this equation for b, [URL='https://www.mathcelebrity.com/1unk.php?num=3b-43.95-4.05%3Db&pl=Solve']we type it in our search engine[/URL] and we get: b =[B] 24[/B]

At the movie theater, Celeste bought 2 large drinks and 2 large popcorns for $8.50. She paid with a
At the movie theater, Celeste bought 2 large drinks and 2 large popcorns for $8.50. She paid with a twenty-dollar bill. What is the fewest number of bills and coins that she could have received as change?r of bills and coins that she could have received as change? Calculate change: Change = Amount Paid - Bill Change = $20.00 - $8.50 Change = $11.50 Largest bill we can start with is a 10 dollar bill: $11.50 - 10 = $1.50 Next largest bill is a $1 bill $1.50 - $1 = 0.50 Now we're down to coins. Largest coin(s) we can use are quarters (assuming no half-dollars) 2 quarters equals 0.50 0.50 - 0.50 = 0 [U]Therefore, our answer is:[/U] [B]Ten dollar Bill, 1 dollar bill, and 2 quarters[/B]

At Zabowood’s Gadget Store, some items are paid on instalment basis through credit cards. Clariza wa
[B]A[/B]t Zabowood’s Gadget Store, some items are paid on instalment basis through credit cards. Clariza was able to sell 10 cellphones costing Php 18,000.00 each. Each transaction is payable in 6 months equally divided into 6 equal instalments without interest. Clariza gets 2% commission on the first month for each of the 10 cellphones. Commission decreases by 0.30% every month thereafter and computed on the outstanding balance for the month. How much commission does Clariza receive on the third month? Calculate Total Sales Amount: Calculate Total Sales Amount = 10 cellphones * 18000 per cellphone Calculate Total Sales Amount = 180000 Calculate monthly sales amount installment: monthly sales amount installment = Total Sales Amount / 6 monthly sales amount installment = 180000/6 monthly sales amount installment = 30000 per month Calculate Third Month Commission: Third month commission = First Month Commission - 0.30% - 0.30% Third month Commission = 2% - 0.30% - 0.30% = 1.4% Calculate 3rd month commission amount: 3rd month Commission amount = 1.4% * 30000 3rd month Commission amount = [B]420[/B]

Austin deposited $4000 into an account with 4.8% interest,compounded monthly. Assuming that no
Austin deposited $4000 into an account with 4.8% interest, compounded monthly. Assuming that no withdrawals are made, how much will he have in the account after 4 years? Do not round any intermediate computations, and round your answer to the nearest cent. Using our [URL='http://www.mathcelebrity.com/compoundint.php?bal=40000&nval=4&int=4.8&pl=Annually']balance calculator[/URL], we get: [B]$48,250.87[/B]

Austin has 15 CDs, which is 3 less than his sister has. How many CDs does his sister have?
Austin has 15 CDs, which is 3 less than his sister has. How many CDs does his sister have? Let s be the number of CD's his sister has and a be the number Austin has [LIST=1] [*]a = 15 [*]a = s - 3 [/LIST] Substitute (1) into (2) 15 = s - 3 Add 3 to each side [B]s = 18[/B]

Austin needs $240 to buy a new bike if he can save $16 per week and how many weeks can you purchase
Austin needs $240 to buy a new bike if he can save $16 per week and how many weeks can you purchase the bike? Set up the equation, where w equals the number of weeks needed. We have: 16w = 240 [URL='https://www.mathcelebrity.com/1unk.php?num=16w%3D240&pl=Solve']Typing this into our search engine[/URL], we get [B]w = 15[/B].

Automorphic Number
Free Automorphic Number Calculator - This calculator determines the nth automorphic number

Ava is 4 times as old as Peter. What equation can be used to find Peter’s age?
Ava is 4 times as old as Peter. What equation can be used to find Peter’s age? [U]Assumptions[/U] Let a be Ava's age Let p be Peter's age We're given: a = 4p To find Peter's age, we divide each side of the equation by 4 to get: a/4 = 4p/4 p = [B]a/4[/B]

Ava set her watch 2 seconds behind, every day it sets back 1 second. How many days has it been since
Ava set her watch 2 seconds behind, every day it sets back 1 second. How many days has it been since she last set her watch if it is 41 seconds behind? Right now: Watch is 2 seconds behind [U]Let d be the day after right now[/U] (1)d + 2 = 41 d + 2 = 41 [U]Subtract 2 from each side[/U] [B]d = 39[/B]

ax + b = cx - d
We are solving for x: Subtract b from each side: ax = cx - d - b Subtract cx from each side: ax - cx = -d - b Factor out x from the left side: x(a - c) = -d - b Divide each side by (a - c) x = (-d - b)/(a - c)

ax - mn = mn + bx for x
ax - mn = mn + bx for x Add mn to each side: ax - mn + mn = mn + bx + mn Cancel the mn terms on the left side and we get: ax = bx + 2mn Subtract bx from each side: ax - bx = bx - bx + 2mn Cancel the bx terms on the right side: ax - bx = 2mn Factor out x on the left side: x (a - b) = 2mn Divide each side of the equation by (a - b): x (a - b)/(a - b) = 2mn/(a - b) Cancel the (a - b) on the left side and we get: x = [B]2mn/(a - b)[/B]

a^2 + b62 = c^2 for c
a^2 + b^2 = c^2 for c Take the square root of each side: c = [B]sqrt(a^2 + b^2)[/B]

b bags of beans there are 7 beans in each bag
b bags of beans there are 7 beans in each bag This means we have 7 beans x b bags = 7b beans total beans.

B is the midpoint of AC and BC=5
B is the midpoint of AC and BC=5 Since the midpoint divides a segment into two equal segments, we know that: AB = BC So AB =[B] 5[/B] And AC = 5 + 5 = [B]10[/B]

B is the set of vowels in the English alphabet
B is the set of vowels in the English alphabet B = [B]{a, e, i, o, u} [MEDIA=youtube]-O686SiqLrk[/MEDIA][/B]

b more points than 75
b more points than 75 Let b be the number of points b + 75

B out of 6 is 12
B out of 6 is 12 b out of 6: b/6 The phrase [I]is[/I] means an equation, so we set b/6 equal to 12: [B]b/6 = 12[/B]

b to the fifth power decreased by 7
b to the fifth power decreased by 7 Take this algebraic expression in steps: [LIST] [*]b to the fifth power: b^5 [*]Decreased by 7 means we subtract 7 from b^5: [B]b^5 - 7[/B] [/LIST]

b varies directly as the sum of x and y
b varies directly as the sum of x and y This is a direct variation problem. Direct variation means there exists a constant k such that: [B]b = k(x + y)[/B]

b/3d - h = 343 for b
b/3d - h = 343 for b A literal equation means we solve for one variable in terms of another variable or variables Add h to each side to isolate the b term: b/3d - h + h = 343 + h Cancel the h's on the left side, we get: b/3d = 343 + h Cross multiply: b = [B]3d(343 + h)[/B]

Babylonian Method
Free Babylonian Method Calculator - Determines the square root of a number using the Babylonian Method.

Bacteria in a petra dish doubles every hour. If there were 34 bacteria when the experiment began, wr
Bacteria in a petra dish doubles every hour. If there were 34 bacteria when the experiment began, write an equation to model this. Let h be the number of hours since the experiment began. Our equation is: [B]B(h) = 34(2^h)[/B]

Bakshali Method
Free Bakshali Method Calculator - Calculates the square root of a positive integer using the Bakshali Method

Balance Sheet
Free Balance Sheet Calculator - Given various asset and liability entries, this determines various calculations that can be made from the balance sheet.

Balance with Interest
Free Balance with Interest Calculator - Calculates the final account balance given a beginning balance, interest rate, and interest crediting period.

Balancing Equations
Free Balancing Equations Calculator - Given 4 numbers, this will use the four operations: addition, subtraction, multiplication, or division to balance the equations if possible.

Balls numbered 1 to 10 are placed in a bag. Two of the balls are drawn out at random. Find the proba
Balls numbered 1 to 10 are placed in a bag. Two of the balls are drawn out at random. Find the probability that the numbers on the balls are consecutive. Build our sample set: [LIST] [*](1, 2) [*](2, 3) [*](3, 4) [*](4, 5) [*](5, 6) [*](6, 7) [*](7, 8) [*](8, 9) [*](9, 10) [/LIST] Each of these 9 possibilities has a probability of: 1/10 * 1/9 This is because we draw without replacement. To start, the bag has 10 balls. On the second draw, it only has 9. We multiply each event because each draw is independent. We have 9 possibilities, so we have: 9 * 1/10 * 1/9 Cancelling, the 9's, we have [B]1/10[/B]

Bangladesh, a country about the size of the state of Iowa, but has about half the U.S population, ab
Bangladesh, a country about the size of the state of Iowa, but has about half the U.S population, about 170 million. The population growth rate in Bangladesh is assumed to be linear, and is about 1.5% per year of the base 170 million. Create a linear model for population growth in Bangladesh. Assume that y is the total population in millions and t is the time in years. At any time t, the Bangladesh population at year t is: [B]y = 170,000,000(1.015)^t[/B]

Barbara bought a piece of rope that was 7 1/3 meters long. She cut the rope into 3 equal pieces. How
Barbara bought a piece of rope that was 7 1/3 meters long. She cut the rope into 3 equal pieces. How long is each piece of rope? Using our mixed number converter, we see that: [URL='https://www.mathcelebrity.com/fraction.php?frac1=7%261%2F3&frac2=3%2F8&pl=Simplify']7&1/3[/URL] = 22/3 Split into [URL='https://www.mathcelebrity.com/fraction.php?frac1=22%2F9&frac2=3&pl=Simplify']3 equal pieces[/URL], we have: 22/3 / 3 = 22/9 or 2&4/9

Barbra is buying plants for her garden. She notes that potato plants cost $3 each and corn plants co
Barbra is buying plants for her garden. She notes that potato plants cost $3 each and corn plants cost $4 each. If she plans to spend at least $20 and purchase less than 15 plants in total, create a system of equations or inequalities that model the situation. Define the variables you use. [U]Define variables[/U] [LIST] [*]Let c be the number of corn plants [*]Let p be the number of potato plants [/LIST] Since cost = price * quantity, we're given two inequalities: [LIST=1] [*][B]3p + 4c >= 20 (the phrase [I]at least[/I] means greater than or equal to)[/B] [*][B]c + p < 15[/B] [/LIST]

Barney has $450 and spends $3 each week. Betty has $120 and saves $8 each week. How many weeks will
Barney has $450 and spends $3 each week. Betty has $120 and saves $8 each week. How many weeks will it take for them to have the same amount of money? Let w be the number of weeks that go by for saving/spending. Set up Barney's balance equation, B(w). Spending means we [U]subtract[/U] B(w) = Initial Amount - spend per week * w weeks B(w) = 450 - 3w Set up Betty's balance equation, B(w). Saving means we [U]add[/U] B(w) = Initial Amount + savings per week * w weeks B(w) = 120 + 8w The same amount of money means both of their balance equations B(w) are equal. So we set Barney's balance equal to Betty's balance and solve for w: 450 - 3w = 120 + 8w Add 3w to each side to isolate w: 450 - 3w + 3w = 120 + 8w + 3w Cancelling the 3w on the left side, we get: 450 = 120 + 11w Rewrite to have constant on the right side: 11w + 120 = 450 Subtract 120 from each side: 11w + 120 - 120 = 450 - 120 Cancelling the 120's on the left side, we get: 11w = 330 To solve for w, we divide each side by 11 11w/11 = 330/11 Cancelling the 11's on the left side, we get: w = [B]30 [MEDIA=youtube]ifG_q-utgJI[/MEDIA][/B]

Basal Metabolic Rate (BMR)
Free Basal Metabolic Rate (BMR) Calculator - Given a gender, an age, and a height/weight in inches/pounds or meters/kilograms, this will calculate the Basal Metabolic Rate (BMR)

Base Change Conversions
Free Base Change Conversions Calculator - Converts a positive integer to Binary-Octal-Hexadecimal Notation or Binary-Octal-Hexadecimal Notation to a positive integer. Also converts any positive integer in base 10 to another positive integer base (Change Base Rule or Base Change Rule or Base Conversion)

based on a sample of size 41, a 95% confidence interval for the mean score of all students,µ, on an
based on a sample of size 41, a 95% confidence interval for the mean score of all students,µ, on an aptitude test is from 60 to 66. Find the margin of error

based on a sample of size 41, a 95% confidence interval for the mean score of all students,µ, on an
Did they give you a standard deviation?

Bashar just read that many more car accidents occur within 30 miles of one's home. He decided that h
Bashar just read that many more car accidents occur within 30 miles of one's home. He decided that he would wear his seat belt only when he is driving within 30 miles from his home and not on long trips because it is obviously safer to travel when you are more than 30 miles from your home. Explain why Bashar's logic is flawed. [B]More accidents occur within 30 miles of one's home because that is where a majority of the travel takes place.[/B]

Bashir finds some nickels and pennies under the couch cushions. How much money ( in dollars ) does h
Bashir finds some nickels and pennies under the couch cushions. How much money ( in dollars ) does he have if he has x nickels and y pennies Amount = Cost * Quantity, so we have: [B]0.01y + 0.05x[/B]

Basic m x n Matrix Operations
Free Basic m x n Matrix Operations Calculator - Given 2 matrices |A| and |B|, this performs the following basic matrix operations
* Matrix Addition |A| + |B|
* Matrix Subtraction |A| - |B|
* Matrix Multiplication |A| x |B|
* Scalar multiplication rA where r is a constant.

Basic Math Operations
Free Basic Math Operations Calculator - Given 2 numbers, this performs the following arithmetic operations:
* Addition (Adding) (+)
* Subtraction (Subtracting) (-)
* Multiplication (Multiplying) (x)
* Long division (Dividing) with a remainder (÷)
* Long division to decimal places (÷)
* Partial Sums (Shortcut Sums)
* Short Division
* Duplication and Mediation

Basic Statistics
Free Basic Statistics Calculator - Given a number set, and an optional probability set, this calculates the following statistical items:
Expected Value
Mean = μ
Variance = σ2
Standard Deviation = σ
Standard Error of the Mean
Skewness
Mid-Range
Average Deviation (Mean Absolute Deviation)
Median
Mode
Range
Pearsons Skewness Coefficients
Entropy
Upper Quartile (hinge) (75th Percentile)
Lower Quartile (hinge) (25th Percentile)
InnerQuartile Range
Inner Fences (Lower Inner Fence and Upper Inner Fence)
Outer Fences (Lower Outer Fence and Upper Outer Fence)
Suspect Outliers
Highly Suspect Outliers
Stem and Leaf Plot
Ranked Data Set
Central Tendency Items such as Harmonic Mean and Geometric Mean and Mid-Range
Root Mean Square
Weighted Average (Weighted Mean)
Frequency Distribution
Successive Ratio

Bawi solves a problem that has an answer of x = -4. He first added 7 to both sides of the equal sign
Bawi solves a problem that has an answer of x = -4. He first added 7 to both sides of the equal sign, then divided by 3. What was the original equation [LIST=1] [*]If we added 7 to both sides, that means we had a minus 7 (-7) to start with as a constant. Since subtraction undoes addition. [*]If we divided by 3, this means we multiplied x by 3 to begin with. Since division undoes multiplication [/LIST] So we have the start equation: 3x - 7 If the answer was x = -4, then we plug this in to get our number on the right side of the equation: 3(-4) - 7 -12 - 7 -19 This means our original equation was: [B]3x - 7 = -19[/B] And if we want to solve this to prove our answer, we [URL='https://www.mathcelebrity.com/1unk.php?num=3x-7%3D-19&pl=Solve']type the equation into our search engine [/URL]and we get: x = -4

Bayes Rule
Free Bayes Rule Calculator - Calculates the conditional probabilities of (B given A) of 2 events and a conditional probability event using Bayes Rule

Bearing
Free Bearing Calculator - Takes a bearing and lists the steps needed to get from Point A to Point B using that bearing.

Because of the promotion, attendance exceeded normal by 275 percent. If the normal attendance was 66
Because of the promotion, attendance exceeded normal by 275 percent. If the normal attendance was 6600 people, how many people attended? Find the exceeded multiplier: Exceeded Multiplier = Exceed percent / 100 [URL='https://www.mathcelebrity.com/perc.php?num=+5&den=+8&num1=+16&pct1=+80&pct2=+90&den1=+80&pct=275&pcheck=4&decimal=+65.236&astart=+12&aend=+20&wp1=20&wp2=30&pl=Calculate']275% as a decimal[/URL] = 2.75 We multiply as follows: Exceeded Attendance = Normal Attendance * Exceeded Multiplier Exceeded Attendance = 6600 * 2.75 Exceeded Attendance = [B]18,150[/B]

Before Barry Bonds, Mark McGwire, and Sammy Sosa, Roger Maris held the record for the most home runs
Before Barry Bonds, Mark McGwire, and Sammy Sosa, Roger Maris held the record for the most home runs in one season. Just behind Maris was Babe Ruth. The numbers of home runs hit by these two athletes in their record-breaking seasons form consecutive integers. Combined, the two athletes hit 121 home runs. Determine the number of home runs hit by Maris and Ruth in their record-breaking seasons. We want [URL='https://www.mathcelebrity.com/consecintwp.php?num=121&pl=Sum']the sum of 2 consecutive integers equals 121[/URL]. [B]We get Maris at 61 and Ruth at 60[/B]

Before you is a 3-quart container, a 5-quart container and a sink full of water. There are no markin
Before you is a 3-quart container, a 5-quart container and a sink full of water. There are no markings on either container to show how many quarts are in each one. All you know is that when the 3-quart container is full, it contains three quarts. You also know that when the 5-quart container is full, it contains five quarts. Your task is to place exactly four quarts in the 5-quart container. How would you accomplish this task? [LIST=1] [*]Fill the 3-quart container and pour it into the 5-quart container [*]Fill the 3-quart container and pour as much as you can (2 quarts) into the 5-quart container. This will leave one quart left. [*]Empty the 5-quart container and pour the one quart into the 5-quart [*]Fill the 3-quart and pour it into the 5-quart. Now there are 4 quarts in the 5-quart container. [/LIST] [I]Note: This is the Die Hard 3 movie problem[/I]

Belen can make 15 necklaces in 3 1/2 hours. How many can she make in one hour?
Belen can make 15 necklaces in 3 1/2 hours. How many can she make in one hour? We set up a proportion of necklaces to time, where n is the number of necklaces Belen can make in 1 hour: 3 & 1/2 = 3.5, so we have: 15/3.5 = n/1 [SIZE=3][FONT=Helvetica][COLOR=rgb(34, 34, 34)] To solve this proportion, we [URL='https://www.mathcelebrity.com/prop.php?num1=15&num2=n&den1=3.5&den2=1&propsign=%3D&pl=Calculate+missing+proportion+value']type it in our search engine and we ge[/URL]t: n = [B]4.29 hours[/B][/COLOR][/FONT][/SIZE]

Belle bought 30 pencils for $1560. She made a profit of $180. How much profit did she make on each p
Belle bought 30 pencils for $1560. She made a profit of $180. How much profit did she make on each pencil The cost per pencil is: 1560/30 = 52 Build revenue function: Revenue = Number of Pencils * Sales Price (s) Revenue = 30s The profit equation is: Profit = Revenue - Cost Given profit is 180 and cost is 1560, we have: 30s - 1560 = 180 To solve for s, we [URL='https://www.mathcelebrity.com/1unk.php?num=30s-1560%3D180&pl=Solve']type this equation into our search engine[/URL] and we get: s = 58 This is sales for total profit. The question asks profit per pencil. Profit per pencil = Revenue per pencil - Cost per pencil Profit per pencil = 58 - 52 Profit per pencil = [B]6[/B]

Below are data showing the results of six subjects on a memory test. The three scores per subject ar
Below are data showing the results of six subjects on a memory test. The three scores per subject are their scores on three trials (a, b, and c) of a memory task. Are the subjects getting better each trial? Test the linear effect of trial for the data. A score trial B score trial 2 C Score trial 3 4 6 7 3 7 8 2 8 5 1 4 7 4 6 9 2 4 2 (a) Compute L for each subject using the contrast weights -1, 0, and 1. That is, compute (-1)(a) + (0)(b) + (1)(c) for each subject. (b) Compute a one-sample t-test on this column (with the L values for each subject) you created. Formula t = To computer a one-sample t-test first know the meaning of each letter (a) Each L column value is just -1(Column 1) + 0(Column2) + 1(Column 3) A score trial B score trial 2 C Score trial 3 L = (-1)(a) + (0)(b) + (1)(c) 4 6 7 3 3 7 8 5 2 8 5 3 1 4 7 6 4 6 9 5 2 4 2 0 (b) Mean = (3 + 5 + 3 + 6 + 5 + 0)/6 = 22/6 = 3.666666667 Standard Deviation = 2.160246899 Use 3 as our test mean (3.666667 - 3)/(2.160246899/sqrt(6)) = 0.755928946

Ben can write 153 letters in 3 minutes. At this rate, how many letters can he write in 10 minutes?
Ben can write 153 letters in 3 minutes. At this rate, how many letters can he write in 10 minutes? We set up a proportion of letters to minutes where the number of letters in 10 minutes is l: 153/3 = l/10 We [URL='https://www.mathcelebrity.com/prop.php?num1=153&num2=l&den1=3&den2=10&propsign=%3D&pl=Calculate+missing+proportion+value']type this proportion into a search engine[/URL] and we get: l =[B] 510[/B]

Ben has $4.50 in quarters(Q) and dimes(D). a)Write an equation expressing the total amount of money
Ben has $4.50 in quarters(Q) and dimes(D). a)Write an equation expressing the total amount of money in terms of the number of quarters and dimes. b)Rearrange the equation to isolate for the number of dimes (D) a) The equation is: [B]0.1d + 0.25q = 4.5[/B] b) Isolate the equation for d. We subtract 0.25q from each side of the equation: 0.1d + 0.25q - 0.25q = 4.5 - 0.25q Cancel the 0.25q on the left side, and we get: 0.1d = 4.5 - 0.25q Divide each side of the equation by 0.1 to isolate d: 0.1d/0.1 = (4.5 - 0.25q)/0.1 d = [B]45 - 2.5q[/B]

Ben is 4 times as old as Ishaan and is also 6 years older than Ishaan.
Ben is 4 times as old as Ishaan and is also 6 years older than Ishaan. Let b be Ben's age and i be Ishaan's age. We're given: [LIST=1] [*]b = 4i [*]b = i + 6 [/LIST] Set (1) and (2) equal to each other: 4i = i + 6 [URL='https://www.mathcelebrity.com/1unk.php?num=4i%3Di%2B6&pl=Solve']Typing this equation into our search engine[/URL], we get: [B]i = 2[/B] Substitute this into equation (1): b = 4(2) [B]b = 8 [/B] [I]Therefore, Ishaan is 2 years old and Ben is 8 years old.[/I]

Ben visits the park every 2 days and goes to the library every 5 days. If Ben gets to do both of the
Ben visits the park every 2 days and goes to the library every 5 days. If Ben gets to do both of these today, how many days will pass before Ben gets to do them both on the same day again? To find this, we want the least common multiple (LCM) of 2 and 5. We [URL='https://www.mathcelebrity.com/gcflcm.php?num1=2&num2=5&num3=&pl=GCF+and+LCM']type LCM(2,5) into our search engine[/URL] and we get: [B]10 days [/B] We check our work: 2 days * 5 visits = 10 days 5 days * 2 visits = 10 days

Bending Beams Displacement
Free Bending Beams Displacement Calculator - Calculates the displacement from the bending of beams at one end or both ends

Benny bought 8 new baseball trading cards to add to his collection. The next day his dog ate half of
Benny bought 8 new baseball trading cards to add to his collection. The next day his dog ate half of his collection. There are now only 47 cards left. How many cards did Benny start with? Let b be the number of baseball trading cards Benny started with. We have the following events: [LIST=1] [*]Benny buys 8 new cards, so we add 8 to get b + 8 [*]The dog ate half of his cards the next day, so Benny has (b + 8)/2 [*]We're told he has 47 cards left, so we set (b + 8)/2 equal to 47 [/LIST] (b + 8)/2 = 47 [B][U]Cross multiply:[/U][/B] b + 8 = 47 * 2 b + 8 = 94 [URL='https://www.mathcelebrity.com/1unk.php?num=b%2B8%3D94&pl=Solve']Type this equation into the search engine[/URL], we get [B]b = 86[/B].

Benny bought a soft drink for 2 dollars and 7 candy bars. He spent a total of 27 dollars. How much d
Benny bought a soft drink for 2 dollars and 7 candy bars. He spent a total of 27 dollars. How much did each candy bar cost? [U]Calculate the candy bar spend:[/U] Candy Bar Spend = Total Spend - Soft Drink Candy Bar Spend = 27 - 2 Candy Bar Spend = 25 [U]Calculate the cost of each candy bar:[/U] Cost of each candy bar = Candy Bar Spend / Total Candy Bars Cost of each candy bar = 25 / 7 Cost of each candy bar = [B]3.57[/B]

Benny had 119 dollars to spend on 9 books. After buying them he had 11 dollars. How much did each bo
Benny had 119 dollars to spend on 9 books. After buying them he had 11 dollars. How much did each book cost ? Let each book cost "b". We have: 9b + 11 = 119 Using our [URL='http://www.mathcelebrity.com/1unk.php?num=9b%2B11%3D119&pl=Solve']equation calculator[/URL], we get [B]b = 12[/B].

Benny had 90 dollars to spend on 7 books. After buying them he had 13 dollars. How much did each boo
Benny had 90 dollars to spend on 7 books. After buying them he had 13 dollars. How much did each book cost? Let each book cost be b. We have: 7b + 13 = 90 [URL='https://www.mathcelebrity.com/1unk.php?num=7b%2B13%3D90&pl=Solve']Typing this equation into the search engine[/URL], and you get: [B]b = 11[/B]

Benny opened a bank account. He deposited $92.50 into his account every month for 10 months. He used
Benny opened a bank account. He deposited $92.50 into his account every month for 10 months. He used $36.50 every month to pay for art lessons. After 10 months, he used 1/2 of the total money left in his account to go to a summer camp for artists. What is the total amount of money Benny spent to go to the summer camp? If Benny deposits $92.50 every month and withdraws $36.50 every month, his net deposit each month is: 92.50 - 36.50 = 56 Benny does this for 10 months, so his balance after 10 months is: 56 * 10 = 560 Half of this is: 560/2 = [B]280[/B]

Bernoulli Trials
Free Bernoulli Trials Calculator - Given a success probability p and a number of trials (n), this will simulate Bernoulli Trials and offer analysis using the Bernoulli Distribution. Also calculates the skewness, kurtosis, and entropy

Besides 8 and 1, what is one factor of 8
Besides 8 and 1, what is one factor of 8. Using our [URL='http://www.mathcelebrity.com/factoriz.php?num=8&pl=Show+Factorization']factor calculator[/URL], or entering the shortcut [B]Factor 8[/B], we get the following factors: 1, 2, 4, 8 Excluding 1 and 8, we have [B]2, 4[/B]

Beth is 5 years younger than celeste. Next year, their ages will have a sum equal to 57. How old is
Beth is 5 years younger than celeste. Next year, their ages will have a sum equal to 57. How old is each now? Let b = Beth's age Let c = Celeste's age We are given: [LIST=1] [*]b = c - 5 [*]b + 1 + c + 1 = 57 [/LIST] Substitute (1) into (2) (c - 5) + 1 + c + 1 = 57 Group like terms: 2c - 3 = 57 [URL='https://www.mathcelebrity.com/1unk.php?num=2c-3%3D57&pl=Solve']Type 2c - 3 = 57 into our search engine[/URL], we get [B]c = 30[/B] Substitute c = 30 into Equation (1), we get: b = 30 - 5 [B]b = 25 [/B] Therefore, Beth is 25 and Celeste is 30.

Beth made a trip to the train station and back. On the trip there she traveled 45 km/h and on the re
Beth made a trip to the train station and back. On the trip there she traveled 45 km/h and on the return trip she went 30 km/h. How long did the trip there take if the return trip took six hours? We use the distance formula: D = rt where D = distance, r = rate, and t = time. Start with the return trip: D = 45(6) D = 270 The initial trip is: 270= 30t Divide each side by 30 [B]t = 9 hours[/B]

Better Buy Comparison
Free Better Buy Comparison Calculator - Given two items with a price and quantity, this determines which is the better buy by comparing unit prices. Finds the better deal.

Beverly has $50 to spend at an amusement park. She plans to spend $10 for food, and $15 for admissio
Beverly has $50 to spend at an amusement park. She plans to spend $10 for food, and $15 for admission to the park. Each ride costs $1.50 to ride. Write an inequality to represent the possible number of rides she can ride? First, we subtract the food and admission cost from Beverly's starting balance of $50: Cost available for rides = Starting Balance - Food - Admission Cost available for rides = 50 - 10 - 15 Cost available for rides = 25 Now we set up an inequality for the number of rides (r) that Beverly can ride with the remaining balance: 1.50r <= 25 To solve for r, we [URL='https://www.mathcelebrity.com/interval-notation-calculator.php?num=1.50r%3C%3D25&pl=Show+Interval+Notation']type this inequality into our search engine[/URL] and we get: [B]r <=[/B] [B]16.67[/B]

Bid-Ask
Free Bid-Ask Calculator - This bid ask calculator takes a bid amount and ask amount and calculates the spread and the margin percent

Big John weighs 300 pounds and is going on a diet where he'll lose 3 pounds per week. Write an equat
Big John weighs 300 pounds and is going on a diet where he'll lose 3 pounds per week. Write an equation in slope-intercept form to represent this situation. [LIST] [*]The slope intercept form is y = mx + b [*]y is John's weight [*]x is the number of weeks [*]A 3 pound per week weight loss means -3 as the coefficient m [*]b = 300, John's starting weight [/LIST] [B]y = -3x + 300[/B]

Bike rental shop A charges $20 per kilometre travelled with no additional fee. Bike rental shop B ch
Bike rental shop A charges $20 per kilometre travelled with no additional fee. Bike rental shop B charges only $8 per kilometre travelled, but has a starting charge of $35. If Bob plans to travel 7km by bike, which rental shop should he choose for a better price [U]Shop A Cost function C(k) where k is the number of kilometers used[/U] C(k) = Cost per kilometer * k + Starting Charge C(k) = 20k With k = 7, we have: C(7) = 20 * 7 C(7) = 140 [U]Shop B Cost function C(k) where k is the number of kilometers used[/U] C(k) = Cost per kilometer * k + Starting Charge C(k) = 8k + 35 With k = 7, we have: C(7) = 8 * 7 + 35 C(7) = 56 + 35 C(7) = 91 Bog should choose [B]Shop B[/B] since they have the better price for 7km

Bill and nine of his friends each have a lot of money in the bank. Bill has 10^10 dollars in his acc
Bill and nine of his friends each have a lot of money in the bank. Bill has 10^10 dollars in his acc All nine of Bill's friends pooled together is: 9 * 10^9 Bill's 10^10 can be written as 10 * 10^9 So [B]Bill's is greater[/B]

Bill is q years old. How old will he in 6 years ? How old was he 4 years ago ?
Bill is q years old. How old will he in 6 years ? How old was he 4 years ago ? Start with q years old. In 6 years means we add since it's the future: [B]q + 6[/B] 4 years ago means we subtract since it's in the past: [B]q - 4[/B]

Bill raises the flag 15 feet above the ground. Then he lowers it 8 feet and raises it another 2 feet
Bill raises the flag 15 feet above the ground. Then he lowers it 8 feet and raises it another 2 feet. how far above the ground is the flag now? We tally up the positives and negatives: Raise = +15 Lower = -8 Raise = +2 Total = +15 - 8 + 2 Total = [B]9 feet above the ground[/B]

Bills car rental charges a base fee of 50$ and then $0.20 per mile
Bills car rental charges a base fee of 50$ and then $0.20 per mile. Set up the cost function C(m) where m is the number of miles driven: [B]C(m) = 50 + 0.20m[/B]

Bingo Card Generator
Free Bingo Card Generator Calculator - This program generates the following two types of bingo cards
1) Random Numerical bingo cards 1-75 with a FREE Space.
2) Buzzword Bingo cards which allow you to enter words of your choice to be used on the bingo card.

Binomial Distribution
Free Binomial Distribution Calculator - Calculates the probability of 3 separate events that follow a binomial distribution. It calculates the probability of exactly k successes, no more than k successes, and greater than k successes as well as the mean, variance, standard deviation, skewness and kurtosis.
Also calculates the normal approximation to the binomial distribution with and without the continuity correction factor
Calculates moment number t using the moment generating function

Binomial Multiplication (FOIL)
Free Binomial Multiplication (FOIL) Calculator - Multiplies out the product of 2 binomials in the form (a + b)(c + d) with 1 unknown variable.
This utilizes the First-Outside-Inside-Last (F.O.I.L.) method.

Binominal Probability
If a seed is planted, it has a 90% chance of growing into a healthy plant. If 12 seeds are planted, what is the probability that exactly 4 don't grow? Im seriously confused is it like u multiple the amount of the (0.90) and multiple (0.30) by power depends how any they r right?

Binominal Probability
You want the binomial distribution, where a "success" is that the plant [U]does not[/U] grow. So if the probability that the plant grows is 0.9, the probability it does not grow is 1 - 0.9 = 0.1. We have n = 12, p = 0.1 You want the probability that exactly 4 of 12 do not grow. Use our [URL='http://www.mathcelebrity.com/binomial.php?n=+12&p=+0.1&k=+4&t=+5&pl=P%28X+%3D+k%29']binomial distribution probability calculato[/URL]r to get P(X = 4) = [B]0.0213[/B]

Black-Scholes
Free Black-Scholes Calculator - Calculates the call or put option value of a stock based on inputs related to the option using Black Scholes method.

blair’s bank account was overdrawn by $40. she spent $30 at the grocery store. what is the balance i
blair’s bank account was overdrawn by $40. she spent $30 at the grocery store. what is the balance in her account now? The word [I]overdrawn[/I] means a negative balance. So we start with: -40 Spending 30 at the grocery store means we subtract 30 from our initial balance: -40 - 30 = [B]-70 or $70 overdrawn[/B]

Blake and Tatsu are each assigned a paper for a class they share. Blake decides to write 4 pages at
Blake and Tatsu are each assigned a paper for a class they share. Blake decides to write 4 pages at a time while Tatsu decides to write 7 pages at a time. If they end up writing the same number of pages, what is the smallest number of pages that the papers could have had? We want the least common multiple of 4 and 7, written as LCM(4, 7). Using our [URL='http://www.mathcelebrity.com/gcflcm.php?num1=4&num2=7&num3=&pl=LCM']LCM Calculator[/URL], we get: LCM(4, 7) = [B]28 pages[/B]

Blake writes 4 pages per hour. How many hours will Blake have to spend writing this week in order to
Blake writes 4 pages per hour. How many hours will Blake have to spend writing this week in order to have written a total of 16 pages? [U]Let x = the number of hours Blake needs to write[/U] 4 pages per hour * x hours = 16 [U]Divide each side by 4[/U] [B]x = 4 hours[/B]

Blanca works as a salesperson and earns a base salary of $72 per week plus a commission of 12% of al
Blanca works as a salesperson and earns a base salary of $72 per week plus a commission of 12% of all her sales. If Blanca had $75 in weekly sales, how much did she make? [U]Find the commission on her sales[/U] Commission = Sales * 12% Commission = 75 * 0.12 = 9 [U]Now add in her base salary[/U] Total Salary = Base Salary + Commission Total Salary = 72 + 9 Total Salary = [B]81[/B]

Blueberries are $4.99 a pound. Diego buys b pounds of blueberries and pays $14.95.
Blueberries are $4.99 a pound. Diego buys b pounds of blueberries and pays $14.95. Since price * quantity = cost, we have the equation: 4.99b = 14.95 To solve for b, [URL='https://www.mathcelebrity.com/1unk.php?num=4.99b%3D14.95&pl=Solve']we type this equation into our search engine[/URL] and we get: b = [B]$3.00[/B]

Bob bought 10 note books and 4 pens for 18$. Bill bought 6 notebooks and 4 pens for 12$. Find the pr
Bob bought 10 note books and 4 pens for 18$. Bill bought 6 notebooks and 4 pens for 12$. Find the price of one note book and one pen. Let the price of each notebook be n. Let the price of each pen be p. We're given two equations: [LIST=1] [*]10n + 4p = 18 [*]6n + 4p = 12 [/LIST] Since we have matching coefficients for p, we subtract equation 1 from equation 2: (10 - 6)n + (4 - 4)p = 18 - 12 Simplifying and cancelling, we get: 4n = 6 [URL='https://www.mathcelebrity.com/1unk.php?num=4n%3D6&pl=Solve']Type this equation into our search engine[/URL] and we get: [B]n = 1.5[/B] Now, substitute this value for n into equation (2): 6(1.5) + 4p = 12 Multiply through: 9 + 4p = 12 [URL='https://www.mathcelebrity.com/1unk.php?num=9%2B4p%3D12&pl=Solve']Type this equation into our search engine[/URL] and we get: [B]p = 0.75[/B]

Bob can complete 35 math problems in 5 minutes how many can he complete in 1 minute
Bob can complete 35 math problems in 5 minutes how many can he complete in 1 minute 35 math problems / 5 minutes Divide the top and bottom of the fraction by 5: 35 math problems / 5 minutes =[B] 7 math problems per minute[/B]

Bob fenced in his backyard. The perimeter of the yard is 22 feet, and the length of his yard is 5 fe
Bob fenced in his backyard. The perimeter of the yard is 22 feet, and the length of his yard is 5 feet. Use the perimeter formula to find the width of the rectangular yard in inches: P = 2L + 2W. Plugging our numbers in for P = 22 and L = 5, we get: 22 = 2(5) + 2W 22 = 10 + 2w Rewritten, we have: 10 + 2w = 22 [URL='https://www.mathcelebrity.com/1unk.php?num=10%2B2w%3D22&pl=Solve']Plug this equation into the search engine[/URL], we get: [B]w = 6[/B]

Bob finished reading his book in x days. Each day, he read 4 pages. His book has 28 pages
Bob finished reading his book in x days. Each day, he read 4 pages. His book has 28 pages Our equation for this is found by multiplying pages per day times number of days; 4x = 28 To solve for x, [URL='https://www.mathcelebrity.com/1unk.php?num=4x%3D28&pl=Solve']we type the equation into our search engine[/URL] and we get: x = [B]7[/B]

Bob has 1.5 pounds of nails. If there are 80 nails in a pound, how many nails does Bob have?
Bob has 1.5 pounds of nails. If there are 80 nails in a pound, how many nails does Bob have? 1.5 pounds x 80 nails per pound = [B]120 nails[/B].

Bob has 4/5 of a pound of fudge. He wants to share it with Sue. How much of a pound of fudge will bo
Bob has 4/5 of a pound of fudge. He wants to share it with Sue. How much of a pound of fudge will both of them get? If Bob shares the fudge with Sue, we assume they split equal parts. This means: We take 4/5 total and divide into 2 for 2 people: 4/5/2 This is the same as 4/5 * 1/2 4/10 This fraction is not simplified. Factor of 4 = {1, [U]2[/U], 4} Factors of 10 = {1, [U]2[/U], 5, 10} In both of these lists, we see the greatest common factor is 2. So we divide top and bottom of 4/10 by 2: 4/2 / 10 / 2 [B]2/5 Bob gets 2/5 of a pound of fudge and Sue gets [B]2/5 of a pound of fudge[/B][/B]

Bob has a bookcase with 4 shelves. There are k books on each shelf. Using k, write an expression for
Bob has a bookcase with 4 shelves. There are k books on each shelf. Using k, write an expression for the total number of books. Total Books = Bookcases * shelves per bookcase * books per shelf Total Books = 1 * 4 * k Total Books = [B]4k[/B]

Bob has half as many quarters as dimes. He has $3.60. How many of each coin does he have?
Bob has half as many quarters as dimes. He has $3.60. How many of each coin does he have? Let q be the number of quarters. Let d be the number of dimes. We're given: [LIST=1] [*]q = 0.5d [*]0.25q + 0.10d = 3.60 [/LIST] Substitute (1) into (2): 0.25(0.5d) + 0.10d = 3.60 0.125d + 0.1d = 3.6 Combine like terms: 0.225d = 3.6 [URL='https://www.mathcelebrity.com/1unk.php?num=0.225d%3D3.6&pl=Solve']Typing this equation into our search engine[/URL], we're given: [B]d = 16[/B] Substitute d = 16 into Equation (1): q = 0.5(16) [B]q = 8[/B]

Bob is twice as old as Henry. The sum of their ages is 42. How old is Henry?
Bob is twice as old as Henry. The sum of their ages is 42. How old is Henry? Let Bob's age be b. Let Henry's age be h. We're given two equations: [LIST=1] [*]b = 2h [*]b + h = 42 [/LIST] Substitute b = 2h in equation 1 into equation 2 for b: 2h + h = 42 To solve for h, we [URL='https://www.mathcelebrity.com/1unk.php?num=2h%2Bh%3D42&pl=Solve']type this equation into our search engine[/URL] and we get: h = [B]14[/B]

Bob, Joe, Pete, Tom, Dick, and Tim share s sandwiches. How many sandwiches does each boy get?
Bob, Joe, Pete, Tom, Dick, and Tim share s sandwiches. How many sandwiches does each boy get? We have 6 boys, so they each get: [B]s/6 sandwiches[/B]

Body Mass Index (BMI)
Free Body Mass Index (BMI) Calculator - Solves for the popular health measurement of Body Mass Index or Weight using inches and pounds input or meters and kilos input.
Also calculates the estimated surface area of the body using the Mosteller Formula

Bond Flat Price-Accrued Coupon-Market Price
Free Bond Flat Price-Accrued Coupon-Market Price Calculator - Calculates the flat price, accrued coupon, and market price for a bond between valuation dates using the following methods:
1) Theoretical Method
2) Practical Method
3) Semi-Theoretical Method

Bond Price Formulas
Free Bond Price Formulas Calculator - Given a face value, coupon percent, yield percent, term, and redemption value, this calculates the price of a bond using the four price formulas for bonds
1) Basic
2) Premium/Discount
3) Base
4) Makeham

Bond Yield Rates
Free Bond Yield Rates Calculator - Calculates the yield rate of bonds using the Yield Approximation Method or the Bond Salesman Method.

Bonnita deposited $4,500 into a savings account paying 3% interest compounded continuously. She plan
Bonnita deposited $4,500 into a savings account paying 3% interest compounded continuously. She plans on leaving the account alone for 7 years. How much money will she have at that time? Using our [URL='https://www.mathcelebrity.com/simpint.php?av=&p=4500&int=3&t=7&pl=Continuous+Interest']compound interest calculator[/URL], we get: [B]$5551.55[/B]

Boolean Algebra Multiplication
Free Boolean Algebra Multiplication Calculator - Determines the product of two expressions using boolean algebra.

Boris baked 40 cookies. His family ate m of them
Boris baked 40 cookies. His family ate m of them If his family ate m, that mean we [I]subtract[/I] m from 40. So Boris has the remaining cookies: [B]40 - m[/B]

Boyles Law
Free Boyles Law Calculator - This will solve for any of the 4 items in Boyles Law
P1 x V1 = P2 x V2

Brad has $40 in a savings account. The interest rate is 5%, compounded annually. To the nearest cen
Brad has $40 in a savings account. The interest rate is 5%, compounded annually. To the nearest cent, how much will he have in 3 years? [URL='https://www.mathcelebrity.com/compoundint.php?bal=40&nval=3&int=5&pl=Annually']Using our balance with interest calculator[/URL], we get [B]$46.31[/B].

Braille Translator
Free Braille Translator Calculator - Given a phrase with letters, numbers, and most punctuation symbols, the calculator will perform the following duties:
1) Translate that phrase to Braille
2) Calculate the number of dots in the message
3) Calculate the number of empty spaces in the message

Brandon can shovel his sidewalk in 8 minutes, while his brother can shovel the walk in 12 minutes. I
Brandon can shovel his sidewalk in 8 minutes, while his brother can shovel the walk in 12 minutes. If they work together, how long will it take them to shovel the sidewalk? Set up unit rates: [LIST] [*]Brandon can shovel 1/8 of a sidewalk per minute [*]His brother can shovel 1/12 of a sidewalk per minute [/LIST] Together, they can shovel: [URL='https://www.mathcelebrity.com/fraction.php?frac1=1%2F8&frac2=1%2F12&pl=Add']1/8 + 1/12[/URL] = 5/24 of a sidewalk per minute 1 minute = 60 seconds 5/24 / 60 seconds = 1/x seconds 5/24 * 60 = 1/x 5/1440 = 1/x Using our [URL='https://www.mathcelebrity.com/proportion-calculator.php?num1=5&num2=1&den1=1440&den2=x&propsign=%3D&pl=Calculate+missing+proportion+value']proportion calculator,[/URL] we get: x = 288 288/60 = [B]4 minutes and 48 seconds[/B]

Break Even
Free Break Even Calculator - Given a fixed cost, variable cost, and revenue function or value, this calculates the break-even point

Brenda has already knit 4 centimeters of scarf, and can knit 1 centimeter each night. After 43 night
Brenda has already knit 4 centimeters of scarf, and can knit 1 centimeter each night. After 43 nights of knitting, how many centimeters of scarf will Brenda have knit in total? 1 centimeter per night * 43 nights = 43 centimeters knitted. Add that to the 4 centimeters she started with, and we have: 43 + 4 = [B]47 centimeters[/B]

Brenda invests $1535 in a savings account with a fixed annual interest rate of 3% compounded continu
Brenda invests $1535 in a savings account with a fixed annual interest rate of 3% compounded continuously. What will the account balance be after 8 years Using our [URL='https://www.mathcelebrity.com/simpint.php?av=&p=1535&int=3&t=8&pl=Continuous+Interest']continuous interest balance calculator[/URL], we get: [B]1,951.37 [MEDIA=youtube]vbYV6SYXtvs[/MEDIA][/B]

Brendan bought an aquarium originally priced at $50 but on sale for 50% off. After 12% sales tax, wh
Brendan bought an aquarium originally priced at $50 but on sale for 50% off. After 12% sales tax, what was the total cost? 50% off of 50 means they pay half, or 1/2(50) = 25. Now, this gets taxed at 12%. So we multiply 25 * 1.12 Total Cost = 25(1.12) Total Cost = [B]$28[/B]

brendan sells roses for $7.99 a bunch. at the end of the day he had collected $79.90. how many bunch
Brendan sells roses for $7.99 a bunch. at the end of the day he had collected $79.90. How many bunches did he sell? $79.90 / $7.99 = 10 bunches

Bretschneiders Formula
Free Bretschneiders Formula Calculator - Calculates the area of a quadrilateral using Bretschneiders Formula

Brian's age is 3/4 of Marcus'. The sum of their ages is 14. How old are Marcus and Brian?
Brian's age is 3/4 of Marcus'. The sum of their ages is 14. How old are Marcus and Brian? Let Marcus's age be m. Then Brian's age = 3/4m The sum is: m + 3m/4 = 14 Combine like terms 7m/4 = 14 Cross multiply: 7m = 56 [URL='http://www.mathcelebrity.com/1unk.php?num=7m%3D56&pl=Solve']Plugging this into the search engine[/URL], we get m = 8. So Brian's age = 3(8)/4 = 24/4 = 6

Brice has 1200 in the bank. He wants to save a total of 3000 by depositing 40 per week from his payc
Brice has 1200 in the bank. He wants to save a total of 3000 by depositing 40 per week from his paycheck. How many weeks will it take until he saves 3000? Remaining Savings = 3,000 - 1,200 = 1,800 40 per week * x weeks = 1,800 40x = 1800 Divide each side of the equation by 40 [B]x = 45 weeks[/B]

Bridget can grow 6 flowers with every seed packet. With 4 seed packets, how many total flowers can B
Bridget can grow 6 flowers with every seed packet. With 4 seed packets, how many total flowers can Bridget have in her garden? Set up a proportion of flowers to seed packets where f is the number of flowers for 4 seed packets. We have: 6/1 = f/4 Cross multiply: f(1) = 24 f = 24

Bridget deposited $4500 at 6 percent simple interest. How much money was in the account at the end o
Bridget deposited $4500 at 6 percent simple interest. How much money was in the account at the end of three years? Using our [URL='https://www.mathcelebrity.com/simpint.php?av=&p=4500&int=6&t=3&pl=Simple+Interest']simple interest balance calculator[/URL], we get: $[B]5,310[/B]

Brighthouse charges $120 a month for their basic plan, plus $2.99 for each on demand movie you buy.
Brighthouse charges $120 a month for their basic plan, plus $2.99 for each on demand movie you buy. Write and solve and inequality to find how many on demand movies could you buy if you want your bill to be less than $150 for the month. Let x equal to the number room movie rentals per month. Our inequality is: 120 + 2.99x < 150 To solve for the number of movies, Add 120 to each side 2.99x < 30 Divide each side by 2.99 x < 10.03, which means 10 since you cannot buy a fraction of a movie

Bruno is 3x years old and his son is x years old now. Their combined age is 40 years. How old is Bru
Bruno is 3x years old and his son is x years old now. Their combined age is 40 years. How old is Bruno Combined age means we add, so we have: 3x + x = 40 To solve this equation for x, we [URL='https://www.mathcelebrity.com/1unk.php?num=3x%2Bx%3D40&pl=Solve']type it in our search engine[/URL] and we get: x = 10 This means Bruno is: 3(10) = [B]30[/B]

Bud makes $400 more per month than maxine If their total income is $3600 how much does bud earn per
Bud makes $400 more per month than maxine If their total income is $3600 how much does bud earn per month Let Bud's earnings be b. Let Maxine's earnings be m. We're given two equations: [LIST=1] [*]b = m + 400 [*]b + m = 3600 [/LIST] To solve this system of equations, we substitute equation (1) into equation (2) for b m + 400 + m = 3600 To solve this equation for m, we [URL='https://www.mathcelebrity.com/1unk.php?num=m%2B400%2Bm%3D3600&pl=Solve']type it in our search engine[/URL] and we get: m = 1600 To solve for b, we substitute m = 1600 into equation (1) above: b = 1600 + 400 b = [B]2200[/B]

Budget Line Equation
Free Budget Line Equation Calculator - Solves for any one of the 5 items in the standard budget line equation:
Income (I)
Quantity of x = Qx
Quantity of y = Qy
Price of x = Px
Price of y = Py

Building A is 150 feet shorter than Building B. The height of both building is 1530 feet. Find the h
Building A is 150 feet shorter than Building B. The height of both building is 1530 feet. Find the height of both building A and B. Let a be the height of building A Let b be the height of building B We're given two equations: [LIST=1] [*]a = b - 150 [*]a + b = 1530 [/LIST] To solve this system of equations, we substitute equation (1) into equation (2) for a: (b - 150) + b = 1530 To solve this equation for b, we [URL='https://www.mathcelebrity.com/1unk.php?num=b-150%2Bb%3D1530&pl=Solve']type it in our search engine[/URL] and we get: b = [B]840[/B] To solve for a, we substitute b = 840 into equation (1): a = 840 - 150 a = [B]690[/B]

by + 2/3 = c for y
by + 2/3 = c for y Subtract 2/3 from each side of the literal equation: by + 2/3 - 2/3 = c - 2/3 Cancel the 2/3 on the left side to get: by = c - 2/3 Divide each side by b to isolate y: by/b = (c - 2/3)/b Cancel the b's on the left side to get: y = [B](c - 2/3)/b[/B]

b^2 - 6 = 5an for a
b^2 - 6 = 5an for a Divide each side of the equation by 5n to isolate a: (b^2 - 6)/5n = 5an/5n Cancel the 5n on the right side and we get: a = [B](b^2 - 6)/5n[/B]

C is the midpoint of BD then BC congruent CD
C is the midpoint of BD then BC congruent CD [URL='https://www.mathcelebrity.com/proofs.php?num=cisthemidpointofbd&pl=Prove']True using this proof[/URL]

C times the product b and a
C times the product b and a [U]The product b and a:[/U] ab [U]c times the product:[/U] [B]abc[/B]

C varies directly as d use k as the constant of variation
C varies directly as d use k as the constant of variation Direct variation means we multiply below: [B]C = kd[/B]

C varies directly as the cube of a and inversely as the 4th power of B
C varies directly as the cube of a and inversely as the 4th power of B The cube of a means we raise a to the 3rd power: a^3 The 4th power of B means we raise b to the 4th power: b^4 Varies directly means there exists a constant k such that: C = ka^3 Also, varies inversely means we divide by the 4th power of B C = [B]ka^3/b^4[/B] Varies [I]directly [/I]as means we multiply by the constant k. Varies [I]inversely [/I]means we divide k by the term which has inverse variation. [MEDIA=youtube]fSsG1OB3qdk[/MEDIA]

c varies jointly as the square of q and cube of p
c varies jointly as the square of q and cube of p The square of q means we raise q to the 2nd power: q^2 The cube of p means we raise p to the rdd power: p^3 The phrase [I]varies jointly[/I] means there exists a constant k such that: [B]c = kp^3q^2[/B]

c/a=db/r for a
c/a=db/r for a Cross multiply the proportion: cr = adb Divide each side of the equation by db to isolate a: cr/db = adb/db Cancel the db terms on the left side and we get: a = [B]cr/db[/B]

c=59f-288 for f
c=59f-288 for f Add 288 to each side: c + 288 = 59f - 288 + 288 Cancel the 288 on the right side, we get: 59f = c + 288 Divide each side by 59 to isolate f: 59f/59 = (c + 288)/59 Cancel the 59 on the left side, we get: f = [B](c + 288)/59[/B]

Calculate the simple interest if the principal is 1500 at a rate of 7% for 3 years
Calculate the simple interest if the principal is 1500 at a rate of 7% for 3 years. Using our [URL='http://www.mathcelebrity.com/simpint.php?av=&p=1500&int=7&t=3&pl=Simple+Interest']simple interest calculator[/URL], the total interest earned over 3 years is [B]$315[/B].

Calculate the value of an investment of $15,000 at 6% interest after 7 years.
Calculate the value of an investment of $15,000 at 6% interest after 7 years. Using our [URL='https://www.mathcelebrity.com/compoundint.php?bal=15000&nval=7&int=6.5&pl=Annually']balance calculator[/URL], we get; [B]23,309.80[/B]

Caleb earns points on his credit card that he can use towards future purchases.
Caleb earns points on his credit card that he can use towards future purchases. He earns four points per dollar spent on flights, two points per dollar spent on hotels, and one point per dollar spent on all other purchases. Last year, he charged a total of $9,480 and earned 14,660 points. The amount of money spent on flights was $140 money than twice the amount of money spent on hotels. Find the amount of money spent on each type of purchase.

Caleb earns points on his credit card that he can use towards future purchases.
Let f = dollars spent on flights, h dollars spent on hotels, and p dollars spent on all other purchases. [U]Set up our equations:[/U] (1) 4f + 2h + p = 14660 (2) f + h + p = 9480 (3) f = 2h + 140 [U]First, substitute (3) into (2)[/U] (2h + 140) + h + p = 9480 3h + p + 140 = 9480 3h + p = 9340 [U]Subtract 3h to isolate p to form equation (4)[/U] (4) p = 9340 - 3h [U]Take (3) and (4), and substitute into (1)[/U] 4(2h + 140) + 2h + (9340 - h) = 14660 [U]Multiply through[/U] 8h + 560 + 2h + 9340 - 3h = 14660 [U]Combine h terms and constants[/U] (8 + 2 - 3)h + (560 + 9340) = 14660 7h + 9900 = 14660 [U]Subtract 9900 from both sides:[/U] 7h = 4760 [U]Divide each side by 7[/U] [B]h = 680[/B] [U]Substitute h = 680 into equation (3)[/U] f = 2(680) + 140 f = 1360 + 140 [B]f = 1,500[/B] [U] Substitute h = 680 and f = 1500 into equation (2)[/U] 1500 + 680 + p = 9480 p + 2180 = 9480 [U]Subtract 2180 from each side:[/U] [B]p = 7,300[/B]

Caleb has a complicated and difficult research paper due soon. What should he do to keep from feelin
Caleb has a complicated and difficult research paper due soon. What should he do to keep from feeling overwhelmed and procrastinating? A. work on the paper every day but save the bulk of the work for the night before it's due B. break down the paper into several small steps and start with the smallest one C. write down the deadline for the paper where he can see it every day so he doesn't forget D. work on the hardest parts of the paper first and take multiple breaks until he's finished Caleb wants to avoid both overwhelm and procrastination. Let's review each option: [LIST] [*]A is out because saving a majority of the work will cause overwhelm [U]and[/U] demonstrates procrastination [*]B is a good option as small steps reduce overwhelm [*]C looks nice on paper, but will he follow through with seeing the deadline everyday? [*]D is a good option as well. Finishing the tough parts first makes the rest of the journey seem like a downhill cruise [/LIST] Based on these, I'd take [B]B or D[/B]

Calls-Puts-Option Δ
Free Calls-Puts-Option Δ Calculator - Calculates the call price, put price, and option Δ based on an option under the risk neutral scenario with a 1 year term.

Cam is 3 years older than Lara. If their combined age is 63, determine their ages by solving an appr
Cam is 3 years older than Lara. If their combined age is 63, determine their ages by solving an appropriate pair of equations. Let Cam's age be c. Let Lara's age be l. We're given two equations: [LIST=1] [*]c = l + 3 <-- older means we add [*]c + l = 63 <-- combined ages mean we add [/LIST] Substitute equation (1) into equation (2): l + 3 + l = 63 Combine like terms to simplify our equation: 2l + 3 = 63 To solve for l, [URL='https://www.mathcelebrity.com/1unk.php?num=2l%2B3%3D63&pl=Solve']we type this equation into our search engine[/URL] and we get: l = [B]30[/B] Now, we plug l = 30 into equation (1) to solve for c: c = 30 + 3 c = [B]33[/B]

Cam is 3 years older than Lara. If their combined age is 63, determine their ages by solving an appr
Cam is 3 years older than Lara. If their combined age is 63, determine their ages by solving an appropriate pair of equations. Let Cam's age be c. Let Lara's age be l. We're given two equations: [LIST=1] [*]c = l + 3 (Since older means we add) [*]c + l = 63 [/LIST] To solve this system of equations, we substitute equation (1) into equation (2) for c: l + 3 + l = 63 To solve this equation for l, we [URL='https://www.mathcelebrity.com/1unk.php?num=l%2B3%2Bl%3D63&pl=Solve']type it in our search engine [/URL]and we get: l = [B]30 [/B] Now, we take l = 30 and substitute it in equation (1) to solve for c: c = 30 + 3 c = [B]33[/B]

camille has 7 blouses,2 skirts,3 pair of short pants and 5 pair of jeans.how many different outfits
camille has 7 blouses,2 skirts,3 pair of short pants and 5 pair of jeans.how many different outfits can he wear,assuming that he always wear a belt. Using the fundamental rule of counting, we find total amount of different outfits as follows: 7 blouses * 2 skirts * 3 pair of short pants * 5 pair of jeans = [B]210 outfits[/B].

can 0.2 be the probability of an outcome in a sample space?
can 0.2 be the probability of an outcome in a sample space? Yes. Any probability p is a valid sample space outcome if: [B]0 <= p <= 1[/B]

Can a coefficient of determination be negative? Why or why not?
Can a coefficient of determination be negative? Why or why not? [B]Yes, reasons below[/B] [LIST] [*] predictions that are being compared to the corresponding outcomes have not been derived from a model-fitting procedure using those data [*] where linear regression is conducted without including an intercept [*] Yes, negative values of R2 may occur when fitting non-linear functions to data [/LIST]

can someone help me with how to work out this word problem?
Consider a paper cone, pointing down, with the height 6 cm and the radius 3 cm; there is currently 9/4 (pie) cubic cm of water in the cone, and the cone is leaking at a rate of 2 cubic centimeters of water per second. How fast is the water level changing, in cm per second?

can someone help me with how to work out this word problem?
Have you tried the rate of change formula?

can you continue this pattern 1,5,13,29
can you continue this pattern 1,5,13,29 Looking at the numbers, we see a pattern of the next number as the prior number * 2 and then add 3 With each term as t(n), we find t(n + 1) as: t(n + 1) = [B]2*t(n) + 3[/B] t(2) = 2(1) + 3 = 2 + 3 = 5 t(3) = 2(5) + 3 = 10 + 3 = 13 t(4) = 2(13) + 3 = 26 + 3 = 29 t(5) = 2(29) + 3 = 58 + 3 = [B]61[/B]

Can you solve this word problem? The Wildgrove Middle School cafeteria goes through a lot of peanut
Can you solve this word problem? The Wildgrove Middle School cafeteria goes through a lot of peanut butter. Currently, they have 120 ounces of regular peanut butter in stock. They also have 319 ounces of crunchy peanut butter. How many ounces do they have in total? [U]Calculate Total Peanut Butter Ounces[/U] Total Peanut Butter Ounces = Regular Peanut Butter Ounces + Crunch Peanut Butter Ounces Total Peanut Butter Ounces = 120 + 319 Total Peanut Butter Ounces = [B]439 ounces[/B]

Cardioid
Free Cardioid Calculator - Shows you the area, arc length, polar equation of the horizontal cardioid, and the polar equation of the vertical cardioid

Cards in a pack are either orange or purple. 80% of the cards are orange. Write the ratio of orange
Cards in a pack are either orange or purple. 80% of the cards are orange. Write the ratio of orange cards to purple cards. [URL='https://www.mathcelebrity.com/perc.php?num=+5&den=+8&num1=+16&pct1=+80&pct2=+90&den1=+80&pct=80&pcheck=4&decimal=+65.236&astart=+12&aend=+20&wp1=20&wp2=30&pl=Calculate']80% as a fraction [/URL]is 4/5. Fractions to ratios can be written as numerator : denominator, so we have: [B]4:5[/B]

Carl is taking a math test. There are 10 questions which take 30 seconds each; 15 questions which ta
Carl is taking a math test. There are 10 questions which take 30 seconds each; 15 questions which take 40 seconds each; and 12 questions which take 2 minutes each. Carl pauses for 5 seconds between questions. In addition, he sharpens his pencil twice, which takes 20 seconds each time. The test begins promptly at 10:00 am. When Carl hands in his completed test, what time is it? [U]10 Questions:[/U] [LIST] [*]30 seconds each x 10 questions = 5 minutes [*]10 pauses between questions x 5 seconds per question = 50 seconds [/LIST] [U]15 Questions[/U] [LIST] [*]40 seconds each x 15 questions = 600 seconds, or 10 minutes [*]15 pauses between questions x 5 seconds per question = 75 seconds, or 1 minute, 15 seconds [/LIST] [U]12 Questions[/U] [LIST] [*]2 minutes x 12 questions = 24 minutes [*]12 pauses x 5 seconds per question = 60 seconds, or 1 minute [/LIST] [U]2 Pencil Sharpenings[/U] [LIST] [*]2 pencil sharpening x 20 seconds each = 40 seconds [/LIST] [U]Total Time[/U] 5 minutes, 50 seconds 11 minutes, 15 seconds 25 minutes 40 seconds 41 minutes and 105 seconds But 105 seconds is 1 minute, 45 seconds. So we have 41 minutes, 45 seconds Therefore, it's [B]10:41[/B]

Carlos age increased by is 16 is 62
Carlos age increased by is 16 is 62. Let a be Carlos's age. Increased by 16 means we add 16 a + 16 Now the phrase [I]is[/I] means equal to, so we set [B]a + 16 = 62[/B]

Carlos was asked to write an equivalent equation to 2x/5 = 1 - x. he wrote it as 2x = 1 - 5x. do you
Carlos was asked to write an equivalent equation to 2x/5 = 1 - x. he wrote it as 2x = 1 - 5x. do you agree with his conclusion? explain your answer for x Cross multiply 2x/5 = 1 - x 2x = 5(1 - x) 2x = 5 - 5x I disagree with his conclusion. He forgot to multiply the 5 through to [B]both terms[/B]

Carly grew 50 plants with 25 seed packets. With 37 seed packets, how many total plants can Carly hav
Carly grew 50 plants with 25 seed packets. With 37 seed packets, how many total plants can Carly have in her backyard? Solve using unit rates. Set up a proportion of plants per seed packets where p is the number of plants per 37 seed packets. 50/25 = p/37 Copying and pasting this problem [URL='http://www.mathcelebrity.com/prop.php?num1=50&num2=p&den1=25&den2=37&propsign=%3D&pl=Calculate+missing+proportion+value']into our search engine[/URL], we get [B]p = 74[/B].

Carly has already written 35 of a novel. She plans to write 12 additional pages per month until she
Carly has already written 35 of a novel. She plans to write 12 additional pages per month until she is finished. Write and solve a linear equation to find the total number of pages written at 5 months. Let m be the number of months. We have the pages written function P(m) as: P(m) = 12m + 35 The problem asks for P(5): P(5) = 12(5) + 35 P(5) = 60 + 35 P(5) = [B]95[/B]

Carly has already written 35 pages of a novel. She plans to write 12 additional pages per month unti
Carly has already written 35 pages of a novel. She plans to write 12 additional pages per month until she is finished. Write and solve a linear equation to find the total number of pages written at 5 months. Set up the equation where m is the number of months: pages per month * m + pages written already 12m + 35 The problems asks for m = 5: 12(5) + 35 60 + 35 [B]95 pages[/B]

Carmen has $30 in store bucks and a 25% discount coupon for a local department store. What maximum d
Carmen has $30 in store bucks and a 25% discount coupon for a local department store. What maximum dollar amount can Carmen purchase so that after her store bucks and discount are applied, her total is no more than $60 before sales tax Let the original price be p. p Apply 25% discount first, which is the same as subtracting 0.25: p(1 - 0.25) Subtract 30 for in store buck p(1 - 0.25) - 30 The phrase [I]no more than[/I] means an inequality using less than or equal to: p(1 - 0.25) - 30 <= 60 To solve this inequality for p, we [URL='https://www.mathcelebrity.com/1unk.php?num=p%281-0.25%29-30%3C%3D60&pl=Solve']type it in our math engine[/URL] and we get: [B]p <= 120[/B]

Carmen is serving her child french fries and chicken wings for lunch today. Let f be the number of f
Carmen is serving her child french fries and chicken wings for lunch today. Let f be the number of french fries in the lunch, and let c be the number of chicken wings. Each french fry has 25 calories, and each chicken wing has 100 calories. Carmen wants the total calorie count from the french fries and chicken wings to be less than 500 calories. Using the values and variables given, write an inequality describing this. We have: 25f + 100c < 50 Note: We use < and not <= because it states less than in the problem.

Carol bought a pair of jeans for $12.95 and a belt for $3.79. The sales tax is $1.01. Carol gave the
Carol bought a pair of jeans for $12.95 and a belt for $3.79. The sales tax is $1.01. Carol gave the store clerk a $20.00 bill. How much change should she get back? Calculate total cost: Total cost = Jeans + Belt + Sales Tax Total cost = $12.95 + $3.79 + $1.01 Total cost = $17.75 Calculate Change Change = Carol's payment - Total cost Change = $20 - $17.75 Change = [B]$2.25[/B]

Carol gets 5 each week for allowance. She saves 1 of her allowance. What percent of her allowance do
Carol gets 5 each week for allowance. She saves 1 of her allowance. What percent of her allowance does carol save? [U]Calculate the decimal:[/U] 1/5 = 0.2 [U]Convert the decimal to a percentage[/U] Percentage = Decimal * 100 Percentage = 100 * 0.2 [B]20%[/B]

Carrie had $32 when she got to the carnival. After riding 6 rides, she had $20 left. What was the pr
Carrie had $32 when she got to the carnival. After riding 6 rides, she had $20 left. What was the price for each ride? If Carrie had $20 left, then the rides cost: $32 - $20 = 12 Price per ride = Cost per all rides / Total Rides Price per ride = 12/6 Price per ride = [B]2[/B]

Cars and trucks are the most popular vehicles. last year, the number of cars sold was 39,000 more th
Cars and trucks are the most popular vehicles. last year, the number of cars sold was 39,000 more than 3 times the number of trucks sold. There were 216,000 cars sold last year. Write an equation that can be used to find the number of trucks, t, sold last year. Let c be the number of cars. Let t be the number of trucks. We're given two equations: [LIST=1] [*]c = 3t + 39000 [*]c + t = 216000 [/LIST] Substitute equation (1) into equation (2) for c: 3t + 39000 + t = 216000 To solve this equation for t, [URL='https://www.mathcelebrity.com/1unk.php?num=3t%2B39000%2Bt%3D216000&pl=Solve']we type it in our math engine [/URL]and we get: t = [B]44,250[/B]

Cartesian Product
Free Cartesian Product Calculator - Given a Set A and Set B, this calculates the Cartesian Product A × B

Casey is 26 years old. Her daughter Chloe is 4 years old. In how many years will Casey be double her
Casey is 26 years old. Her daughter Chloe is 4 years old. In how many years will Casey be double her daughter's age Declare variables for each age: [LIST] [*]Let Casey's age be c [*]Let her daughter's age be d [*]Let n be the number of years from now where Casey will be double her daughter's age [/LIST] We're told that: 26 + n = 2(4 + n) 26 + n = 8 + 2n Solve for [I]n[/I] in the equation 26 + n = 8 + 2n [SIZE=5][B]Step 1: Group variables:[/B][/SIZE] We need to group our variables n and 2n. To do that, we subtract 2n from both sides n + 26 - 2n = 2n + 8 - 2n [SIZE=5][B]Step 2: Cancel 2n on the right side:[/B][/SIZE] -n + 26 = 8 [SIZE=5][B]Step 3: Group constants:[/B][/SIZE] We need to group our constants 26 and 8. To do that, we subtract 26 from both sides -n + 26 - 26 = 8 - 26 [SIZE=5][B]Step 4: Cancel 26 on the left side:[/B][/SIZE] -n = -18 [SIZE=5][B]Step 5: Divide each side of the equation by -1[/B][/SIZE] -1n/-1 = -18/-1 n = [B]18[/B] Check our work for n = 18: 26 + 18 ? 8 + 2(18) 44 ? 8 + 36 44 = 44

Cassidy is renting a bicycle on the boardwalk. The rental costs a flat fee of $10 plus an additional
Cassidy is renting a bicycle on the boardwalk. The rental costs a flat fee of $10 plus an additional $7 per hour. Cassidy paid $45 to rent a bicycle. We set up the cost equation C(h) where h is the number of hours of rental: C(h) = hourly rental rate * h + Flat Fee C(h) = 7h + 10 We're told that Cassidy paid 45 to rent a bicycle, so we set C(h) = 45 7h + 10 = 45 To solve for h, we [URL='https://www.mathcelebrity.com/1unk.php?num=7h%2B10%3D45&pl=Solve']type this equation into our math engine[/URL] and we get: h = [B]5[/B]

Catherine has $400 in her checking account. She writes a check for $600. What is the balance in her
Catherine has $400 in her checking account. She writes a check for $600. What is the balance in her account? Writing a check decreases the bank balance. So we have: $400 - $600 = [B]-$200[/B]

Cathy wants to buy a gym membership. One gym has a $150 joining fee and costs $35 per month. Another
Cathy wants to buy a gym membership. One gym has a $150 joining fee and costs $35 per month. Another gym has no joining fee and costs $60 per month. a. In how many months will both gym memberships cost the same? What will that cost be? Set up cost equations where m is the number of months enrolled: [LIST=1] [*]C(m) = 35m + 150 [*]C(m) = 60m [/LIST] Set them equal to each other: 35m + 150 = 60m [URL='http://www.mathcelebrity.com/1unk.php?num=35m%2B150%3D60m&pl=Solve']Pasting the equation above into our search engine[/URL], we get [B]m = 6[/B].

CCP Stat Club is sending a delegate of 2 members to attend the 2015 Joint Statistical Meeting in Sea
CCP Stat Club is sending a delegate of 2 members to attend the 2015 Joint Statistical Meeting in Seattle. There are 10 qualified candidates. How many different ways can the delegate be selected? 10C2 = [B]45[/B] shown on our [URL='http://www.mathcelebrity.com/permutation.php?num=10&den=2&pl=Combinations']Combinations Calculator[/URL]

Ceiling
Free Ceiling Calculator - Calculates the ceiling of a number

center (3, -2), radius = 4
center (3, -2), radius = 4 To see the general form or standard form, you can check out this link: [URL='http://Circle Equations']https://www.mathcelebrity.com/eqcircle.php?h=3&k=-2&r=4&d1=1&d2=1&d3=2&d4=4&calc=1&ceq=&pl=Calculate[/URL]

Centripetal Acceleration
Free Centripetal Acceleration Calculator - Solves for any of the 3 items in the centripetal acceleration formula, centripetal acceleration, rotational speed, and radius.

cereal is on sale for 3.60 for a 9-ounce box. what is the price per ounce?
cereal is on sale for 3.60 for a 9-ounce box. what is the price per ounce? price per ounce = Total cost / ounces price per ounce = 3.60/9 price per ounce = [B]$0.40[/B]

Cevian Triangle Relations
Free Cevian Triangle Relations Calculator - Given a triangle with a cevian, this will solve for the cevian or segments or sides based on inputs

Chain Discounts and Net Cost Price and Net Cost Equivalent
Free Chain Discounts and Net Cost Price and Net Cost Equivalent Calculator - Given a chain discount and an original price, this calculates the total discount and net cost price.

Chance has 3/4 hour left to finish 5 math problems on the test. How much time does she have to spend
Chance has 3/4 hour left to finish 5 math problems on the test. How much time does she have to spend on each problem? 3/4 of an hour in minutes is: 3/4 * 60 = 45 minutes 45 minutes / 5 math problems = [B]9 minutes per problem[/B]

Chang is serving his child french fries and chicken wings for lunch today. Let f be the number of f
Chang is serving his child french fries and chicken wings for lunch today. Let f be the number of french fries in the lunch, and let c be the number of chicken wings. Each french fry has 25 calories, and each chicken wing has 100 calories. Chang wants the total calorie count from the french fries and chicken wings to be less than 600 calories. Using the values and variables given, write an inequality describing this. We have [B]25f + 100c < 600[/B] as our inequality.

Change 1/9 to a decimal
Change 1/9 to a decimal This is the same as 100/9 = 11.1111% or [B]0.1111[/B]

Change Counting
Free Change Counting Calculator - This shows you how to make change using the least amount of bills/coins by taking a bill amount and a cash tendered amount from a customer and figuring out the fastest way to make change. Maximum denomination is $100

Change the base 10 number 100 into base 5
Change the base 10 number 100 into base 5 Using our [URL='https://www.mathcelebrity.com/binary.php?num=100&check1=7&bchoice=5&pl=Convert']base change calculator[/URL], we get: 100 = [B]400 (Base 5)[/B]

Change the base 2 number 1000 into base 10
Change the base 2 number 1000 into base 10 1 * 2^3 + 0 * 2^2 + 0 * 2^1 + 0 * 2^0 8 + 0 + 0 + 0 = [B]8[/B]

Charlene wants to invest $10,000 long enough for it to grow to at least $20000. The compound interes
Charlene wants to invest $10,000 long enough for it to grow to at least $20,000. The compound interest rate is 6% p.a. How many whole number of years does she need to invest the money for so that it grows to her $20,000 target? We want 10,000(1.06)^n = 20,000. But what the problem asks for is how long it will take money to double. We can use a shortcut called the Rule of 72. [URL='https://www.mathcelebrity.com/rule72.php?num=6&pl=Calculate']Using the Rule of 72 at 6%[/URL], we get [B]12 years[/B].

Charles Law
Free Charles Law Calculator - This will solve for any of the 4 items in Charles Law assuming constant pressure
V1 ÷ T1 = V2 ÷ T2

Charlie buys a 40 pound bag of cat food. His cat eats a 1/2 pound of food per day.
Charlie buys a 40 pound bag of cat food. His cat eats a 1/2 pound of food per day. Set up an equation: 1/2x = 40 where x is the number of days Multiply through by 2 [B]x = 80[/B]

Charlie has $2700 in his bank account. He spends $150 a week. How many weeks will have passed when C
Charlie has $2700 in his bank account. He spends $150 a week. How many weeks will have passed when Charlie has $600 in his bank account? Let w be the weeks that pass. We have the following equation for Charlie's balance: 2700 - 150w = 600 To solve for w, we [URL='https://www.mathcelebrity.com/1unk.php?num=2700-150w%3D600&pl=Solve']type this equation into our math engine[/URL] and we get: w = [B]14[/B]

Charmaine’s fish tank has 16 liters of water in it. she plans to add 6 liters per minute until the t
Charmaine’s fish tank has 16 liters of water in it. she plans to add 6 liters per minute until the tank has at least 58 liters. What are the possible numbers of minutes Charmaine could add water? This is an algebraic inequality. The phrase [I]at least[/I] means greater than or equal to. So we have: 6m + 16 >= 58 <-- This is our algebraic expression/inequality. To solve this, [URL='https://www.mathcelebrity.com/1unk.php?num=6m%2B16%3E%3D58&pl=Solve']we type this into our search engine [/URL]and we get: [B]m >= 7[/B]

Charrie found a piece of 8 meters rope. She cuts it into equal length. She made three cuts. How long
Charrie found a piece of 8 meters rope. She cuts it into equal length. She made three cuts. How long is each piece of the rope? Equal length means we divide the length of the rope by the number of equal cuts [B]8/3 or 2 & 2/3 meters[/B]

Chebyshevs Theorem
Free Chebyshevs Theorem Calculator - Using Chebyshevs Theorem, this calculates the following:
Probability that random variable X is within k standard deviations of the mean.
How many k standard deviations within the mean given a P(X) value.

CHEBYSHEVS THEOREM TELLS US THAT WHAT PERCENTAGE LIES BETWEEN 2.25 STANDARD DEVIATIONS?
CHEBYSHEVS THEOREM TELLS US THAT WHAT PERCENTAGE LIES BETWEEN 2.25 STANDARD DEVIATIONS? Using our [URL='http://www.mathcelebrity.com/chebyshev.php?pl=probability&k=2.25&probk=0.75']Chebyshevs Theorem calculator[/URL], we get: P(X - u| < k?) >= [B]0.802469[/B]

Chemical Compounds
Free Chemical Compounds Calculator - Shows details of the chemical compounds including name, formula, and molar mass

cheryl scores 68 out of 80 on her science test, nadia scores 86 out of 120 on her science test and a
cheryl scores 68 out of 80 on her science test, nadia scores 86 out of 120 on her science test and ali scores 120 out of 150 on her science test. who preforms the best in his/her science test Cheryl: [URL='https://www.mathcelebrity.com/perc.php?num=68&den=80&pcheck=1&num1=16&pct1=80&pct2=70&den1=80&idpct1=10&hltype=1&idpct2=90&pct=82&decimal=+65.236&astart=12&aend=20&wp1=20&wp2=30&pl=Calculate']68/80[/URL] = 85% Nadia: [URL='https://www.mathcelebrity.com/perc.php?num=86&den=120&pcheck=1&num1=16&pct1=80&pct2=70&den1=80&idpct1=10&hltype=1&idpct2=90&pct=82&decimal=+65.236&astart=12&aend=20&wp1=20&wp2=30&pof1=&pof2=&pl=Calculate']86/120[/URL] = 71.67% Ali: [URL='https://www.mathcelebrity.com/perc.php?num=+120&den=+150&pcheck=1&num1=16&pct1=80&pct2=70&den1=80&idpct1=10&hltype=1&idpct2=90&pct=82&decimal=+65.236&astart=12&aend=20&wp1=20&wp2=30&pof1=&pof2=&pl=Calculate']120/150[/URL] = 80% [B]Cheryl[/B] has the highest percentage, so she did the best on her test.

Chi-Square Critical Values
Free Chi-Square Critical Values Calculator - Given a probability, this calculates the critical value for the right-tailed and left-tailed tests for the Chi-Square Distribution. CHIINV from Excel is used as well.

Chicken is on sale for $3.90 per pound. If Ms.Gelllar buys 2.25 pounds of chicken, how much will she
Chicken is on sale for $3.90 per pound. If Ms.Gelllar buys 2.25 pounds of chicken, how much will she spend? round to the nearest penny and show your work Total spend = Cost per pound * Number of pounds Total spend = $3.90 * 2.25 pounds Total spend = [B]$8.78[/B] (rounded to 2 digits)

Chinese Remainder Theorem
Free Chinese Remainder Theorem Calculator - Given a set of modulo equations in the form:
x ≡ a mod b
x ≡ c mod d
x ≡ e mod f

the calculator will use the Chinese Remainder Theorem to find the lowest possible solution for x in each modulus equation.
Given that the ni portions are not pairwise coprime and you entered two modulo equations, then the calculator will attempt to solve using the Method of Successive Subsitution

Cholesterol
Free Cholesterol Calculator - Solves for each of the 4 following cholesterol equation items:
1) Total Cholesterol
2) High Density Lipoproteins (HDL) Good Cholesterol
3) Low Density Lipoproteins (LDL) Bad Cholesterol
4) Triglycerides

Choose the best inequality for this scenario. Casey bought a sandwich and a drink for $3.75. If she
Choose the best inequality for this scenario. Casey bought a sandwich and a drink for $3.75. If she has $6.00 to spend, what is the most she can spend on dessert? Let dessert spend be d. We have: d <= $6.00 - $3.75 [B]d <= $2.25[/B]

Choose the equation of a line in standard form that satisfies the given conditions. perpendicular to
Choose the equation of a line in standard form that satisfies the given conditions. perpendicular to 4x + y = 8 through (4, 3). Step 1: Find the slope of the line 4x + y = 8. In y = mx + b form, we have y = -4x + 8. The slope is -4. To be perpendicular to a line, the slope must be a negative reciprocal of the line it intersects with. Reciprocal of -4 = -1/4 Negative of this = -1(-1/4) = 1/4 Using our [URL='https://www.mathcelebrity.com/slope.php?xone=4&yone=3&slope=+0.25&xtwo=3&ytwo=2&bvalue=+&pl=You+entered+1+point+and+the+slope']slope calculator[/URL], we get [B]y = 1/4x + 2[/B]

Choosing coffee or tea; with cream, milk, or honey; served in a glass or plastic cup
Choosing coffee or tea; with cream, milk, or honey; served in a glass or plastic cup Using the fundamental rule of counting: 2 drink types * 3 sweetness * 2 cups = [B]12 possible choices[/B]

Chord
Free Chord Calculator - Solves for any of the 3 items in the Chord of a Circle equation, Chord Length (c), Radius (r), and center to chord midpoint (t).

Chris has 6 cds that he is going to give away. He let his best friend choose 2 of the 6 cds. How man
Chris has 6 cds that he is going to give away. He let his best friend choose 2 of the 6 cds. How many different groups of 2cds are possible? We want 6 choose 2 using combinations. We use combinations because the problem states the word [I]different[/I]. [URL='https://www.mathcelebrity.com/permutation.php?num=6&den=2&pl=Combinations']6 C 2[/URL] using our calculator is [B]15[/B].

Chris walks 12 blocks north and then 16 blocks East. How far is his home from the park
Chris walks 12 blocks north and then 16 blocks East. How far is his home from the park We've got a right triangle. If we divide 12 and 16 by 4, we get: 12/4 = 3 16/4 = 4 Since the hypotenuse is the distance from the home to the park, we have a classic 3-4-5 right triangle. So our hypotenuse is 5*4 = [B]20[/B]

Chris, Alex and Jesse are all siblings in the same family. Alex is 5 years older than chris. Jesse i
Chris, Alex and Jesse are all siblings in the same family. Alex is 5 years older than chris. Jesse is 6 years older than Alex. The sum of their ages is 31 years. How old is each one of them? Set up the relational equations where a = Alex's age, c = Chris's aged and j = Jesse's age [LIST=1] [*]a = c + 5 [*]j = a + 6 [*]a + c + j = 31 [*]Rearrange (1) in terms of c: c = a - 5 [/LIST] [U]Plug in (4) and (2) into (3)[/U] a + (a - 5) + (a + 6) = 31 [U]Combine like terms:[/U] 3a + 1 = 31 [U]Subtract 1 from each side[/U] 3a = 30 [U]Divide each side by 3[/U] [B]a = 10[/B] [U]Plug in 1 = 10 into Equation (4)[/U] c = 10 - 5 [B]c = 5[/B] Now plug 1 = 10 into equation (2) j = 10 + 6 [B]j = 16[/B]

Christopher has $25 000 to invest. He finds a bank who will pay an interest rate of 5.65% p.a compou
Christopher has $25 000 to invest. He finds a bank who will pay an interest rate of 5.65% p.a compounded annually. What will the total balance be after 6 years? Using our [URL='https://www.mathcelebrity.com/compoundint.php?bal=25000&nval=6&int=5.65&pl=Annually']compound interest balance calculator[/URL], we get: [B]34,766.18[/B]

Chuck-a-luck is an old game, played mostly in carnivals and county fairs. To play chuck-a-luck you p
Chuck-a-luck is an old game, played mostly in carnivals and county fairs. To play chuck-a-luck you place a bet, say $1, on one of the numbers 1 through 6. Say that you bet on the number 4. You then roll three dice (presumably honest). If you roll three 4’s, you win $3.00; If you roll just two 4’s, you win $2; if you roll just one 4, you win $1 (and, in all of these cases you get your original $1 back). If you roll no 4’s, you lose your $1. Compute the expected payoff for chuck-a-luck. Expected payoff for each event = Event Probability * Event Payoff Expected payoff for 3 matches: 3(1/6 * 1/6 * 1/6) = 3/216 = 1/72 Expected payoff for 2 matches: 2(1/6 * 1/6 * 5/6) = 10/216 = 5/108 Expected payoff for 1 match: 1(1/6 * 5/6 * 5/6) = 25/216 Expected payoff for 0 matches: -1(5/6 * 5/6 * 5/6) = 125/216 Add all these up: (3 + 10 + 25 - 125)/216 -87/216 ~ [B]-0.40[/B]

Circle Equation
Free Circle Equation Calculator - This calculates the standard equation of a circle and general equation of a circle from the following given items:
* A center (h,k) and a radius r
* A diameter A(a1,a2) and B(b1,b2)
This also allows you to enter a standard or general form equation so that the center (h,k) and radius r can be determined.

Circular Permutation
Free Circular Permutation Calculator - Calculates the following:
Number of ways to arrange n distinct items arranged on a circle

Claire bought 180 candies and 140 pens for goody bags for her birthday. What is the largest number o
Claire bought 180 candies and 140 pens for goody bags for her birthday. What is the largest number of goody bags that Claire can make so that each goody bag has the same number of candies and the same number of pens? (All candies and pens should be used.) We want the greatest common factor of 180 and 140. When we [URL='https://www.mathcelebrity.com/gcflcm.php?num1=140&num2=180&num3=&pl=GCF+and+LCM']run GCF(180,140) in our calculator[/URL], we get 20. We divide our total candies and total pens by our GCF. So each bag has the following: Candies: 180/20 = [B]9 candies[/B] Pens: 140/20 = [B]7 pens[/B]

Claire makes bracelets using blue and red beads.Each bracelet has 20 red beads and 5 blue beads.Writ
Claire makes bracelets using blue and red beads.Each bracelet has 20 red beads and 5 blue beads.Write an ordered pair to represent the number of red beads and blue beads Claire will use to make 8 bracelets. 8 bracelets gives you 8 x 20 = 160 red beads and 8 * 5 = 40 blue beads. The ordered pair is[B] (160, 40)[/B]

Clara can bake 17 cookies with each scoop of flour. With two scoops of flour, how many cookies can C
Clara can bake 17 cookies with each scoop of flour. With two scoops of flour, how many cookies can Clara bake? Set up a proportion where x is the number of cookies per 2 scoops of flour 17 cookies/1 scoop = x cookies/2 scoops [URL='http://www.mathcelebrity.com/prop.php?num1=17&num2=x&den1=1&den2=2&propsign=%3D&pl=Calculate+missing+proportion+value']Running this in the search engine, we get[/URL]: [B]x = 34 cookies[/B]

Clark wants to give some baseball cards to his friends. If he gives 6 cards to each of his friends,
Clark wants to give some baseball cards to his friends. If he gives 6 cards to each of his friends, he will have 5 cards left. If he gives 8 cards to each of his friends, he will need 7 more cards. How many friends is the giving the cards to? Let the number of friends Clark gives his cards to be f. Let the total amount of cards he gives out be n. We're given 2 equations: [LIST=1] [*]6f + 5 = n [*]8f - 7 = n [/LIST] Since both equations equal n, we set these equations equal to each other 6f + 5 = 8f - 7 To solve for f, we [URL='https://www.mathcelebrity.com/1unk.php?num=6f%2B5%3D8f-7&pl=Solve']type this equation into our search engine[/URL] and we get: f = [B]6 [/B] To check our work, we plug in f = 6 into each equation: [LIST=1] [*]6(6) + 5 = 36 + 5 = 41 [*]8(6) - 7 = 48 - 7 = 41 [/LIST] So this checks out. Clark has 41 total cards which he gives to 6 friends.

Class A has 8 pupils and class B has 10 pupils. Both classes sit the same maths test. The mean sco
Class A has 8 pupils and class B has 10 pupils. Both classes sit the same maths test. The mean score for class A is 55. The mean score for both classes is 76. What is the mean score (rounded to 1 DP) in the maths test for class B Mean of the sum equals the sum of the means. U(A + B) = U(A) + U(B) 76 = 55 + U(B) Subtract 55 from each side, we get: [B]U(B) = 21[/B]

Classify Fraction
Free Classify Fraction Calculator - Determines the if a fraction is proper, improper, or whole.

Clock Angle
Free Clock Angle Calculator - Calculate the angle on a clock between the hour and minute hands or how many times on the clock form an angle of (x°) between the minute and hour hand (backwards and forwards). Clock Angle Calculator

Clock Gain and Loss
Free Clock Gain and Loss Calculator - This calculates the gain/loss in seconds of a clock over a certain period of time.

Clock Hands Meet
Free Clock Hands Meet Calculator - Calculates the first time on a clock when the hands will meet or touch again.

Closest Fraction
Free Closest Fraction Calculator - Determines the closest fraction in a list to a target fraction

Coach vega orders 30 bats for the team. He orders 7 oak, 7 maple, 12 ash bats, and and some bamboo b
Coach vega orders 30 bats for the team. He orders 7 oak, 7 maple, 12 ash bats, and and some bamboo bats. Find b, the number of bamboo bats. 30 bats - 7 maple - 7 oak - 12 ash 30 - 26 = [B]4 bamboo bats[/B]

Cody invests $4,734 in a retirement account with a fixed annual interest rate of 4% compounded conti
Cody invests $4,734 in a retirement account with a fixed annual interest rate of 4% compounded continuously. What will the account balance be after 19 years? Using our c[URL='http://www.mathcelebrity.com/simpint.php?av=&p=4734&int=4&t=19&pl=Continuous+Interest']ontinuous interest compounding calculator[/URL], we get: [B]10,122.60[/B]

cody takes about 24,040 breaths a day. how many breaths is that in an hour?
cody takes about 24,040 breaths a day. how many breaths is that in an hour? There are 24 hours in a day, so we divide 24,040 / 24 to get breaths per hour: 24,040 / 24 = [B]1001.67 [/B]

Cofunction
Free Cofunction Calculator - Calculates the cofunction of the 6 trig functions: * sin
* cos
* tan
* csc
* sec
* cot


Coin Amount to Denomination
Free Coin Amount to Denomination Calculator - Takes a money value and using the highest possible bills/coins, constructs the amount using bills and coins.

Coin Combinations
Free Coin Combinations Calculator - Given a selection of coins and an amount, this determines the least amount of coins needed to reach that total.

Coin Denomination Conversions
Free Coin Denomination Conversions Calculator - This caculator converts between the following coin denominations:
* Penny
* Nickel
* Dime
* Quarter
* Half-Dollar
* Dollar

Coin Toss Probability
Free Coin Toss Probability Calculator - This calculator determines the following coin toss probability scenarios
* Coin Toss Sequence such as HTHHT
* Probability of x heads and y tails
* Probability of at least x heads in y coin tosses
* Probability of at least x tails in y coin tosses
* Probability of no more than x heads in y coin tosses
* Probability of no more than x tails in y coin tosses
* (n) Coin Tosses with a list of scenario results displayed
* Monte Carlo coin toss simulation

Coin Total Word Problems
Free Coin Total Word Problems Calculator - This word problem lesson solves for a quantity of two coins totaling a certain value with a certain amount more or less of one coin than another

Coin Values
Free Coin Values Calculator - This calculates the total value of a given amount of:
* Pennies
* Nickels
* Dimes
* Quarters
* Half-Dollars
* Dollars

Cole and Finn are roommates. They paid three months rent and a $200 security deposit when they signe
Cole and Finn are roommates. They paid three months rent and a $200 security deposit when they signed the lease. In total, they paid $1,850. What is the rent for one month? Write an equation and solve it. Equation, let m = rent for one month 3m + 200 = 1,850 Using our [URL='http://www.mathcelebrity.com/1unk.php?num=3m%2B200%3D1850&pl=Solve']Equation Solver[/URL], we get [B]m = 550[/B].

Coles paycheck was $257.20. He put 25% of it into his savings account and used 1/3 of what was left
Coles paycheck was $257.20. He put 25% of it into his savings account and used 1/3 of what was left to pay bills. How much money does he have remaining from his paycheck? 25% is also 1/4. Calculate savings $257.20(0.25) = $64.3 We have 75% left over = $192.90 Coles pays 1/3 of this for bills = $192.90 * 1/3 = $64.30 Subtract the bills: $192.90 - $64.30 = [B]$128.60[/B]

Colin was thinking of a number. Colin divides by 8, then adds 1 to get an answer of 2. What was the
Colin was thinking of a number. Colin divides by 8, then adds 1 to get an answer of 2. What was the original number? Let the number be n. Divide by 8: n/8 Then add 1: n/8 + 1 The phrase [I]get an answer[/I] of means an equation, so we set n/8 + 1 equal to 2: n/8 + 1 = 2 To solve for n, we subtract 1 from each side to isolate the n term: n/8 + 1 - 1 = 2 - 1 Cancel the 1's on the left side, we get: n/8 = 1 Cross multiply: n = 8*1 n = [B]8[/B]

Collatz Conjecture
Free Collatz Conjecture Calculator - Takes any natural number using the Collatz Conjecture and reduces it down to 1.

Collinear Points that form Unique Lines
Free Collinear Points that form Unique Lines Calculator - Solves the word problem, how many lines can be formed from (n) points no 3 of which are collinear.

Combination with Variable
Free Combination with Variable Calculator - Calculates the following:
Solves for r given n and the combination value.
Solves for n given r and the combination value

Combinations with Replacement
Free Combinations with Replacement Calculator - Calculates the following:
How many combinations can we have from a sample of r elements from a set of n distinct objects where order does matter and replacements are not allowed?

Combined Gas Law
Free Combined Gas Law Calculator - This will solve for any of the 6 items in the Combined Gas Law using pressure, volume (Capacity), and temperature.
(P1 x V1)/T1 = (P2 x V2)/T2

Combined Ratio
Free Combined Ratio Calculator - Given a ratio a:b and a ratio b:c, this determines the combined ratio a:c

Committees of 4 men 5 women form a group of 11 men and 11 women.
Committees of 4 men 5 women form a group of 11 men and 11 women. We want combinations. 4 men from 11 men is the combination 11C4. [URL='https://www.mathcelebrity.com/permutation.php?num=11&den=4&pl=Combinations']Using our combinations calculator[/URL], we get: 11C4 = 330 5 women from 11 women is the combination 11C5. [URL='https://www.mathcelebrity.com/permutation.php?num=11&den=5&pl=Combinations']Using our combinations calculator[/URL], we get: 11C5 = 462 We multiply the committee of men times the committee of women: 11C4 * 11C5 = 330 * 432 11C4 * 11C5 = [B]142,560[/B]

Commutative Property
Free Commutative Property Calculator - Demonstrates the commutative property of addition and the commutative property of multiplication using 3 numbers. Numerical Properties

Company a charges $25 plus $0.10 a mile. Company b charges $20 plus $0.15 per mile. How far would yo
Company a charges $25 plus $0.10 a mile. Company b charges $20 plus $0.15 per mile. How far would you need to travel to get each charge to be the same? Let x be the number of miles traveled Company A charge: C = 25 + 0.10x Company B charge: C = 20 + 0.15x Set up an equation find out when the charges are the same. 25 + 0.10x = 20 + 0.15x Combine terms and simplify 0.05x = 5 Divide each side of the equation by 0.05 to isolate x x = [B]100[/B]

Company A rents copy machines for $300 a month plus $0.05 per copy. Company B charges $600 plus $0.0
Company A rents copy machines for $300 a month plus $0.05 per copy. Company B charges $600 plus $0.01 per copy. For which number of copies do the two companies charge the same amount? With c as the number of copies, we have: Company A Cost = 300 + 0.05c Company B Cost = 600 + 0.01c Set them equal to each other 300 + 0.05c = 600 + 0.01c Use our [URL='http://www.mathcelebrity.com/1unk.php?num=300%2B0.05c%3D600%2B0.01c&pl=Solve']equation solver[/URL] to get: [B]c = 7,500[/B]

Compare Raises
Free Compare Raises Calculator - Given two people with a salary and annual raise amount, this determines how long it takes for the person with the lower salary to catch the person with the higher salary.

Compared to the normal distribution, the t distribution has ___ values at the top and ___ at the tai
Compared to the normal distribution, the t distribution has ___ values at the top and ___ at the tails. a. More; less b. More; more c. Less; less d. Less; more [B]d. Less; more[/B] [I]t value chart is wider and flatter[/I]

Comparison of Numbers
Free Comparison of Numbers Calculator - Compares two numbers and checks to see if they are equal to one another, if the first number is greater than the second number, or the first number is less than the second number. Minimum and maximum.

Complementary and Supplementary Angles
Free Complementary and Supplementary Angles Calculator - This calculator determines the complementary and supplementary angle of a given angle that you enter OR it checks to see if two angles that you enter are complementary or supplementary.

Complex Number Operations
Free Complex Number Operations Calculator - Given two numbers in complex number notation, this calculator:
1) Adds (complex number addition), Subtracts (complex number subtraction), Multiplies (complex number multiplication), or Divides (complex number division) any 2 complex numbers in the form a + bi and c + di where i = √-1.
2) Determines the Square Root of a complex number denoted as √a + bi
3) Absolute Value of a Complex Number |a + bi|
4) Conjugate of a complex number a + bi

Composite Number
Free Composite Number Calculator - This calculator determines the nth composite number. Helps you generate composite numbers.

Compound Interest Accumulated Balance
Free Compound Interest Accumulated Balance Calculator - Given an interest rate per annum compounded annually (i), semi-annually, quarterly, monthly, semi-monthly, weekly, and daily, this calculates the accumulated balance after (n) periods

Compound Interest and Annuity Table
Free Compound Interest and Annuity Table Calculator - Given an interest rate (i), number of periods to display (n), and number of digits to round (r), this calculator produces a compound interest table. It shows the values for the following 4 compound interest annuity functions from time 1 to (n) rounded to (r) digits:
vn
d
(1 + i)n
an|
sn|
än|i
sn|i
Force of Interest δn

Compute a 75% Chebyshev interval around the mean for x values and also for y values.
Compute a 75% Chebyshev interval around the mean for [I]x[/I] values and also for [I]y[/I] values. [B][U]Grid E: [I]x[/I] variable[/U][/B] 11.92 34.86 26.72 24.50 38.93 8.59 29.31 23.39 24.13 30.05 21.54 35.97 7.48 35.97 [B][U]Grid H: [I]y[/I] variable[/U][/B] 27.86 13.29 33.03 44.31 16.58 42.43 39.61 25.51 39.14 16.58 47.13 14.70 57.47 34.44 According to Chebyshev's Theorem, [1 - (1/k^2)] proportion of values will fall between Mean +/- (k*SD) k in this case equal to z z = (X-Mean)/SD X = Mean + (z*SD) 1 - 1/k^2 = 0.75 - 1/k^2 = 0.75 - 1= - 0.25 1/k^2 = 0.25 k^2 = 1/0.25 k^2 = 4 k = 2 Therefore, z = k = 2 First, [URL='http://www.mathcelebrity.com/statbasic.php?num1=11.92%2C34.86%2C26.72%2C24.50%2C38.93%2C8.59%2C29.31%2C23.39%2C24.13%2C30.05%2C21.54%2C35.97%2C7.48%2C35.97&num2=+0.2%2C0.4%2C0.6%2C0.8%2C0.9&pl=Number+Set+Basics']determine the mean and standard deviation of x[/URL] Mean(x) = 25.24 SD(x) = 9.7873 Required Interval for x is: Mean - (z * SD) < X < Mean + (z * SD) 25.24 - (2 * 9.7873) < X < 25.24 - (2 * 9.7873) 25.24 - 19.5746 < X < 25.24 + 19.5746 5.6654 < X < 44.8146 Next, [URL='http://www.mathcelebrity.com/statbasic.php?num1=27.86%2C13.29%2C33.03%2C44.31%2C16.58%2C42.43%2C39.61%2C25.51%2C39.14%2C16.58%2C47.13%2C14.70%2C57.47%2C34.44&num2=+0.2%2C0.4%2C0.6%2C0.8%2C0.9&pl=Number+Set+Basics']determine the mean and standard deviation of y[/URL] Mean(y) = 32.29 SD(y) = 9.7873 Required Interval for y is: Mean - (z * SD) < Y < Mean + (z * SD) 32.29 - (2 * 13.1932) < Y < 32.29 - (2 * 13.1932) 32.29 - 26.3864 < Y < 32.29 + 26.3864 5.9036 < X < 58.6764

Computer Storage Conversions
Free Computer Storage Conversions Calculator - This calculator converts between the following computer storage measurements:
* byte
* kilobyte
* megabyte
* gigabyte
* terabyte
* petabyte
* exabyte

Computers R US was selling laptops that had 4GB of memory for $495. You can buy additional memory fo
Computers R US was selling laptops that had 4GB of memory for $495. You can buy additional memory for $97 per GB. If your grandfather gave you $980 to buy a laptop and additional memory, how much memory can you get? Figure out remaining money total after buying the laptop [LIST=1] [*]4GB: 980 - 495 = 485 [*]485/97 = 5 GB [*]4GB + 5GB = [B]9GB[/B] [/LIST]

Concert tickets cost $14.95 each. Which expression represents the total cost of 25 tickets?
Concert tickets cost $14.95 each. Which expression represents the total cost of 25 tickets? Calculate Total Cost: Total Cost = Cost Per Ticket * Number of Tickets Total Cost = $14.95 * 25 Total Cost = [B]$373.75[/B]

Condos in Centerville go up in value by 3% each year. If the Ayala family's condo is now worth $697,
Condos in Centerville go up in value by 3% each year. If the Ayala family's condo is now worth $697,580, what will it be worth in 2 years? Let the condo value in (y) years be C(y). 3% as a decimal is 0.03, so we have: C(y) = 697,850 * (1.03)^y The problem asks for C(2): C(2) = 697,850 * (1.03)^2 C(2) = 697,850 * 1.0609 C(2) = [B]740.349.07[/B]

Confidence Interval for the Mean
Free Confidence Interval for the Mean Calculator - Calculates a (90% - 99%) estimation of confidence interval for the mean given a small sample size using the student-t method with (n - 1) degrees of freedom or a large sample size using the normal distribution Z-score (z value) method including Standard Error of the Mean. confidence interval of the mean

Confidence Interval for Variance and Standard Deviation
Free Confidence Interval for Variance and Standard Deviation Calculator - Calculates a (95% - 99%) estimation of confidence interval for the standard deviation or variance using the χ2 method with (n - 1) degrees of freedom.

Confidence Interval of a Proportion
Free Confidence Interval of a Proportion Calculator - Given N, n, and a confidence percentage, this will calculate the estimation of confidence interval for the population proportion π including the margin of error. confidence interval of the population proportion

Confidence Interval/Hypothesis Testing for the Difference of Means
Free Confidence Interval/Hypothesis Testing for the Difference of Means Calculator - Given two large or two small distriutions, this will determine a (90-99)% estimation of confidence interval for the difference of means for small or large sample populations.
Also performs hypothesis testing including standard error calculation.

Congratulations!! You are hired at Roof and Vinyl Housing Systems. Your starting salary is $45,600 f
Congratulations!! You are hired at Roof and Vinyl Housing Systems. Your starting salary is $45,600 for the year. Each year you stay employed with them your salary will increase by 3.5%. Determine what your salary would be if you worked for the company for 12 years. Set up a function S(y) where y is the number of years after you start at the Roof and Vinyl place. S(y) = 45600 * (1.035)^y <-- Since 3.5% = 0.035 The question asks for S(12): S(12) = 45600 * (1.035)^12 S(12) = 45600 * 1.51106865735 S(12) = [B]68,904.73[/B]

Congruence Modulo n
Free Congruence Modulo n Calculator - Given a possible congruence relation a ≡ b (mod n), this determines if the relation holds true (b is congruent to c modulo n).

Conjugates
Free Conjugates Calculator - Simplifies a fraction with conjugates in the denominator

Conner earned these scores on the first three tests in science this term: 86, 88, and 78. What is th
Conner earned these scores on the first three tests in science this term: 86, 88, and 78. What is the lowest that Conner can earn on the fourth and final test of the term if he wants to have an average of at least 83? Using our [URL='https://www.mathcelebrity.com/missingaverage.php?num=86%2C+88%2C78&avg=83&pl=Calculate+Missing+Score']missing average calculator[/URL], we find that the fourth score must be [B]80[/B]

Connie’s kindergarten class has 24 children. She wants them to get into 4 equal groups. How many chi
Connie’s kindergarten class has 24 children. She wants them to get into 4 equal groups. How many children will she put in each group? We take 24 children divided by 4 equal groups = 24/4 24/4 = [B]6 children per group[/B]

Connor bought 386.21 pounds of peanuts and 331.28 pounds of raisins. How many pounds of snacks did h
Connor bought 386.21 pounds of peanuts and 331.28 pounds of raisins. How many pounds of snacks did he buy in all? Since peanuts [U]and[/U] raising are both snacks, we [URL='http://www.mathcelebrity.com/longdiv.php?num1=386.21&num2=331.28&pl=Add']add the weights[/URL]. Total pounds = 386.21 + 331.28 Total pounds = [B]717.49[/B]

Connor runs 2 mi more each day than David. The sum of the distances they run each week is 56 mi. How
Connor runs 2 mi more each day than David. The sum of the distances they run each week is 56 mi. How far does David run each day? Let Connor's distance be c Let David's distance be d We're given two equations: [LIST=1] [*]c = d + 2 [*]7(c + d) = 56 [/LIST] Simplifying equation 2 by dividing each side by 7, we get: [LIST=1] [*]c = d + 2 [*]c + d = 8 [/LIST] Substitute equation (1) into equation (2) for c d + 2 + d = 8 To solve for d, we [URL='https://www.mathcelebrity.com/1unk.php?num=d%2B2%2Bd%3D8&pl=Solve']type this equation into our calculation engine[/URL] and we get: d = [B]3[/B]

Consecutive Integer Word Problems
Free Consecutive Integer Word Problems Calculator - Calculates the word problem for what two consecutive integers, if summed up or multiplied together, equal a number entered.

Consider a firm that has two assembly lines, 1 and 2, both producing calculator. Assume that you hav
Consider a firm that has two assembly lines, 1 and 2, both producing calculator. Assume that you have purchased a calculator and it turns out to be defective. And the line 1 produces 60% of all calculators produced. L1: event that the calculator is produced on line 1 L2: event that the calculator is produced on line 2 Suppose that your are given the conditional information: 10% of the calculator produced on line 1 is defective 20% of the calculator produced on line 2 is defective Q: If we choose one defective, what is the probability that the defective calculator comes from Line 1 and Line2? L1 = event that the calculator is produced on line 1 = 0.6 L2 = event that the calculator is produced on line 2 = 1 - 0.6 = 0.4 D = Defective D|L1 Defective from Line 1 = 0.1 D|L2 = Defective from Line 2 = 0.20 [U]Defective from Line 1[/U] P(L1|D) = P(L1)P(D/L1) / [ P(L1)P(D/L1) + P(L2)P(D/L2)] P(L1|D) = (.60)(.10) /[(.60)(.10)+ (.40)(.20)] [B]P(L1|D) = 0.4286[/B] [U]Defective from Line 2[/U] P(L2|D) = P(L2)P(D/L2) / [ P(L1)P(D/L1) + P(L2)P(D/L2)] P(L2|D) = (.40)(.20) /[(.60)(.10)+ (.40)(.20)] [B]P(L2|D) = 0.5714[/B]

Consider a probability model consisting of randomly drawing two colored balls from a jar containing
Consider a probability model consisting of randomly drawing two colored balls from a jar containing 2 red and 1 blue balls. What is the Sample Space of this experiment? (assume B= blue and R=red) The sample space is the list of all possible events [LIST] [*]RRB [*]RBR [*]BRR [/LIST]

Consider the case of a manufacturer who has an automatic machine that produces an important part. Pa
Consider the case of a manufacturer who has an automatic machine that produces an important part. Past records indicate that at the beginning of the data the machine is set up correctly 70 percent of the time. Past experience also shows that if the machine is set up correctly it will produce good parts 90 percent of the time. If it is set up incorrectly, it will produce good parts 40 percent of the time. Since the machine will produce 60 percent bad parts, the manufacturer is considering using a testing procedure. If the machine is set up and produces a good part, what is the revised probability that it is set up correctly? [U]Determine our events:[/U] [LIST] [*]C = Correctly Set Machine = 0.7 [*]C|G = Correctly Set Machine And Good Part = 0.9 [*]I = Incorrectly Set Machine = 1 - 0.7 = 0.3 [*]I|G = Incorrectly Set Machine And Good Part = 0.4 [*]B< = BAD PARTS = 0.60 [/LIST] P[correctly set & part ok] = P(C) * P(C|G) P[correctly set & part ok] = 70% * 90% = 63% P[correctly set & part ok] = P(I) * P(I|G) P[incorrectly set & part ok] = 30% *40% = 12% P[correctly set | part ok] = P[correctly set & part ok]/(P[correctly set & part ok] + P[incorrectly set & part ok]) P[correctly set | part ok] = 63/(63+12) = [B]0.84 or 84%[/B]

Consider the following 15 numbers 1, 2, y - 4, 4, 5, x, 6, 7, 8, y, 9, 10, 12, 3x, 20 - The mean o
Consider the following 15 numbers 1, 2, y - 4, 4, 5, x, 6, 7, 8, y, 9, 10, 12, 3x, 20 - The mean of the last 10 numbers is TWICE the mean of the first 10 numbers - The sum of the last 2 numbers is FIVE times the sum of the first 3 numbers (i) Calculate the values of x and y We're given two equations: [LIST=1] [*](x + 6 + 7 + 8 + y + 9 + 10 + 12 + 3x + 20)/10 = 2(1 + 2 + y - 4 + 4 + 5 + x + 6 + 7 + 8 + y)/10 [*]3x - 20 = 5(1 + 2 + y - 4) [/LIST] Let's evaluate and simplify: [LIST=1] [*](x + 6 + 7 + 8 + y + 9 + 10 + 12 + 3x + 20)/10 = (1 + 2 + y - 4 + 4 + 5 + x + 6 + 7 + 8 + y)/5 [*]3x - 20 = 5(y - 1) [/LIST] Simplify some more: [URL='https://www.mathcelebrity.com/polynomial.php?num=x%2B6%2B7%2B8%2By%2B9%2B10%2B12%2B3x%2B20&pl=Evaluate'](x + 6 + 7 + 8 + y + 9 + 10 + 12 + 3x + 20)/10[/URL] = (4x + y + 72)/10 [URL='https://www.mathcelebrity.com/polynomial.php?num=1%2B2%2By-4%2B4%2B5%2Bx%2B6%2B7%2B8%2By&pl=Evaluate'](1 + 2 + y - 4 + 4 + 5 + x + 6 + 7 + 8 + y)/5[/URL] = (2y + x + 29)/5 5(y - 1) = 5y - 5 So we're left with: [LIST=1] [*](4x + y + 72)/10 = (2y + x + 29)/5 [*]3x - 20 = 5y - 5 [/LIST] Cross multiply equations in 1, we have: 5(4x + y + 72) = 10(2y + x + 29) 20x + 5y + 360 = 20y + 10x + 290 We have: [LIST=1] [*]20x + 5y + 360 = 20y + 10x + 290 [*]3x - 20 = 5y - 5 [/LIST] Combining like terms: [LIST=1] [*]10x - 15y = -70 [*]3x - 5y = 15 [/LIST] Now we have a system of equations which we can solve any of three ways: [LIST] [*][URL='https://www.mathcelebrity.com/simultaneous-equations.php?term1=10x+-+15y+%3D+-70&term2=3x+-+5y+%3D+15&pl=Substitution']Substitution Method[/URL] [*][URL='https://www.mathcelebrity.com/simultaneous-equations.php?term1=10x+-+15y+%3D+-70&term2=3x+-+5y+%3D+15&pl=Elimination']Elimination Method[/URL] [*][URL='https://www.mathcelebrity.com/simultaneous-equations.php?term1=10x+-+15y+%3D+-70&term2=3x+-+5y+%3D+15&pl=Cramers+Method']Cramer's Rule[/URL] [/LIST] No matter which method we choose, we get the same answer: (x, y) = [B](-115, -72)[/B]

Consider the formula for the area of a trapezoid: A=12h(a+b) . Is it mathematically simpler to solve
Consider the formula for the area of a trapezoid: A=12h(a+b) . Is it mathematically simpler to solve for a, b, or h? Why? Solve for each of these variables to demonstrate. The variable "h" is the easiest to solve for. Because you only have one step. Let's review: Divide each side of the equation by 12(a + b) h = 12(a + b)/A Solving for "a", we two steps. Divide each side by 12h: A/12h = a + b Subtract b from each side a = A/12h - b Solving for "b" takes two steps as well. Divide each side by 12h: A/12h = a + b Subtract a from each side b = A/12h - a

Construct a confidence interval of the population proportion at the given level of confidence. x = 1
Construct a confidence interval of the population proportion at the given level of confidence. x = 120, n = 300, 99% confidence Round to 3 decimal places as needed [B]0.327 < p < 0.473[/B] using our [URL='http://www.mathcelebrity.com/propconf.php?bign=300&smalln=120&conf=99&pl=Proportion+Confidence+Interval']proportion confidence interval calculator[/URL]

Construct a data set of seven temperature readings where the mean is positive and the median is nega
Construct a data set of seven temperature readings where the mean is positive and the median is negative. [B]{-20,-10.-5,-2,-1,20,40}[/B] [URL='https://www.mathcelebrity.com/statbasic.php?num1=-20%2C-10%2C-5%2C-2%2C-1%2C20%2C40&num2=+0.2%2C0.4%2C0.6%2C0.8%2C0.9&pl=Number+Set+Basics']Using our mean and median calculator[/URL], we see that: [B]Mean = 3.142857 (positive) Median = -2[/B]

Container Arrangements
Free Container Arrangements Calculator - Given a set of items inside a container, this calculates the probability that you draw certain items in the following fashion:
Draw all the items
Draw any of the items
How many ways can you choose m items of a, n items of b, o items of c, etc.

Continued Fraction
Free Continued Fraction Calculator - Shows the continued fraction for a fraction

Continuous Annuity
Free Continuous Annuity Calculator - Determines the Present Value and Accumulated Value of a Continuous Annuity

Conventionally, the null hypothesis is false if the probability value is: a. Greater than 0.05 b. L
Conventionally, the null hypothesis is false if the probability value is: a. Greater than 0.05 b. Less than 0.05 c. Greater than 0.95 d. Less than 0.95 [B]b. Less than 0.05[/B] This is standard in hypothesis testing using p = 0.05

Coordinating Conjunctions
Free Coordinating Conjunctions Calculator - Shows the 7 coordinating conjunctions using the mnemonic FANBOYS. The seven coordinating conjunctions are (FOR, AND, NOR, BUT, OR, YET, SO)

Corvettes are known as sporty cars that can travel at high rates of speed. It is therefore assumed t
Corvettes are known as sporty cars that can travel at high rates of speed. It is therefore assumed that they are much more dangerous than minivans. An owner of a Corvette points out that when statistics are studied, there are far more deaths each year from crashes that involve minivans than crashes that involve Corvettes, so Corvettes, so Corvettes must be safer than minivans. The statistics the Covert owner sites are correct. Is his logic faulty? Why or why not? [B]Faulty.[/B] There are hundreds of times more minivans on the road than Corvettes, so we expect more deaths even if they are the safest car on the road.

Cost of Carry
Free Cost of Carry Calculator - Calculates the cost of carry expressed as the forward price for a position

Cost Recovery Method
Free Cost Recovery Method Calculator - Given a sales price, cost, and set of payments, this determines the gross profit per year based on the cost recovery method.

Counting
Free Counting Calculator - Counts up from a number to another number using a factor
Counts down from one number to another number using a factor. Also known as skip counting.

Counting with Groups of 10 and Leftovers
Free Counting with Groups of 10 and Leftovers Calculator - This calculator finds the total using groups of tens and leftover values.

Country A produces about 7 times the amount of diamonds in carats produce in Country B. If the total
Country A produces about 7 times the amount of diamonds in carats produce in Country B. If the total produced in both countries is 40,000,000 carats, find the amount produced in each country. Set up our two given equations: [LIST=1] [*]A = 7B [*]A + B = 40,000,000 [/LIST] Substitute (1) into (2) (7B) + B = 40,000,000 Combine like terms 8B = 40,000,000 Divide each side by 8 [B]B = 5,000,000[/B] Substitute this into (1) A = 7(5,000,000) [B]A = 35,000,000[/B]

Coupon Comparison
Free Coupon Comparison Calculator - Given a cost of goods, a dollar off coupon, and a percentage off coupon, this calculator will compare the two deals and determine which one is of more value. If the dollar coupon wins, the calculator will project the break even price where the dollar coupon would surpass the percentage coupon

Covariance and Correlation coefficient (r) and Least Squares Method and Exponential Fit
Free Covariance and Correlation coefficient (r) and Least Squares Method and Exponential Fit Calculator - Given two distributions X and Y, this calculates the following:
* Covariance of X and Y denoted Cov(X,Y)
* The correlation coefficient r.
* Using the least squares method, this shows the least squares regression line (Linear Fit) and Confidence Intervals of α and Β (90% - 99%)
Exponential Fit
* Coefficient of Determination r squared r2
* Spearmans rank correlation coefficient
* Wilcoxon Signed Rank test

Cox-Ross-Rubenstein Pricing
Free Cox-Ross-Rubenstein Pricing Calculator - Using the Cox-Ross-Rubenstein method, this calculates the call price and put price of an option.

Craig went bowling with $25 to spend. He rented shoes for $5.25 and paid $4.00 for each game. What w
Craig went bowling with $25 to spend. He rented shoes for $5.25 and paid $4.00 for each game. What was the greatest number of games Craig could have played? Set up the cost function C(g) where g is the number of games Craig plays: C(g) = Game fee * number of games (g) + shoe rental fee C(g) = 4g + 5.25 The problem asks for the maximum number of games Craig can play for $25. So we want an inequality of [I]less than or equal to[/I]. 4g + 5.25 <= 25 [URL='https://www.mathcelebrity.com/1unk.php?num=4g%2B5.25%3C%3D25&pl=Solve']Type this inequality into our search engine[/URL], and we get: g <= 4.9375 We want exact games, so we round this down to [B]4 games[/B].

Cribbage
Free Cribbage Calculator - Calculates the score you would get after the deck card is flipped in a hand of cribbage.

Critical Values for F-test
Free Critical Values for F-test Calculator - Calculate a critical value for the F-Test statistic based on DF1, DF2, and α

Critical Z-values
Free Critical Z-values Calculator - Given a probability from a normal distribution, this will generate the z-score critical value. Uses the NORMSINV Excel function.

Cross Partitions
Free Cross Partitions Calculator - Given a set of partitions, this determines the cross partitions.

Cross Product
Free Cross Product Calculator - Given two vectors A and B in R3, this calculates the cross product A × B as well as determine if the two vectors are parallel

Crypto Scams
I'd like to warn our fans about a crypto scam going around. The site is [URL]https://crypto-fortress.com[/URL]. The scam runs like this... [LIST] [*]You're asked to deposit money, a minimum of $1,000 in BTC. [*]You're given credits on the money from their mining/aribtrage plan. [*]However, when it comes time to cash out after a week, they suddenly tell you, their is some magical agreement (which you never signed nor is on their website) where you now have to pay 25% of your profits to them and you'll get a withdrawal code for the rest. [*]When you press them on how they pay 75% of your profits from a 25% deposit which makes no sense, they tell you that it's how things work. [/LIST]

Crystal is serving pizza at a birthday party for her brother there are 25 people coming to the part
Crystal is serving pizza at a birthday party for her brother there are 25 people coming to the party she wants each each person to have 3 pieces of pizza each pizza has 8 slices how many pizzas should she buy? 25 people * 3 pieces of pizza each = 75 pieces of pizza Each pizza has 8 pieces. 75 pieces / 8 pieces per pizza = 9.375 pizzas. Round up to [B]10[/B] since we want an integer answer.

cscx-cotx*cosx=sinx
cscx-cotx*cosx=sinx A few transformations we can make based on trig identities: [LIST] [*]csc(x) = 1/sin(x) [*]cot(x) = cos(x)/sin(x) [/LIST] So we have: 1/sin(x) - cos(x)/sin(x) * cos(x) = sin(x) (1 - cos^2(x))/sin(x) = sin(x) 1 - cos^2(x) = sin^2(x) This is [B]true[/B] from the identity: sin^2(x) - cos^2(x) = 1

Cube
Free Cube Calculator - Solves for Volume (Capacity), Lateral Area,Surface Area, and the value of a side for a cube.

cube root of a number and 7
cube root of a number and 7 The phrase [I]a number[/I] means an arbitrary variable, let's call it x: x Cube root of a number means we raise x to the 1/3 power: x^1/3 And 7 means we add 7: [B]x^1/3 + 7[/B]

Cube the difference of b and c
Cube the difference of b and c the difference of b and c: b - c Cubing means raising to the power of 3: [B](b - c)^3[/B]

Cubic Equation
Free Cubic Equation Calculator - Solves for cubic equations in the form ax3 + bx2 + cx + d = 0 using the following methods:
1) Solve the long way for all 3 roots and the discriminant Δ
2) Rational Root Theorem (Rational Zero Theorem) to solve for real roots followed by the synthetic div/quadratic method for the other imaginary roots if applicable.

Cuboid
Free Cuboid Calculator - Calculates the volume, surface area, diagonals, and space diagonal for a cuboid

Currently 16 out of every 25 American adults drink coffee every day. In a town with a population of
Currently 16 out of every 25 American adults drink coffee every day. In a town with a population of 7900 adults, how many of these adults would you expect to drink coffee ever We'd multiply 16/25 times 7900: Using our [URL='https://www.mathcelebrity.com/fraction.php?frac1=7900&frac2=16/25&pl=Multiply']fraction multiplication calculator by type 16/25 of 7900[/URL], we get: [B]5056[/B]

Customers arrive at the claims counter at the rate of 20 per hour (Poisson distributed). What is th
Customers arrive at the claims counter at the rate of 20 per hour (Poisson distributed). What is the probability that the arrival time between consecutive customers is less than five minutes? Use the [I]exponential distribution[/I] 20 per 60 minutes is 1 every 3 minutes 1/λ = 3 so λ = 0.333333333 Using the [URL='http://www.mathcelebrity.com/expodist.php?x=+5&l=0.333333333&pl=CDF']exponential distribution calculator[/URL], we get F(5,0.333333333) = [B]0.811124396848[/B]

cx+b/d=y for b
cx+b/d=y for b Subtract cx from each side to isolate b/d: cx - cx + b/d = y - cx Cancel the cx terms on each side: b/d = y - cx Cross multiply: b = [B]d(y - cx)[/B]

d - f^3 = 4a for a
d - f^3 = 4a for a Solve this literal equation for a: Divide each side of the equation by 4: (d - f^3)/4 = 4a/4 Cancel the 4's on the right side, and rewrite with our variable to solve for on the left side: a = [B](d - f^3)/4[/B]

d is h decreased by 301
d is h decreased by 301 h decreased by 301 means we subtract 301 from h h - 301 The phrase [I]is[/I] means equal to, so we set d equal to this expression: [B]d = h - 301[/B]

D is the set of days in the week.
D is the set of days in the week. [B]D = {Sunday, Monday, Tuesday, Wednesday, Thursday, Friday, Saturday}[/B]

d is the set of days of the week
d is the set of days of the week [B]d = {Sunday, Monday, Tuesday, Wednesday, Thursday, Friday, Saturday}[/B]

d squared is greater than or equal to 17
d squared is greater than or equal to 17 d squared means we raise the variable d to the power of 2: d^2 The phrase [I]greater than or equal to[/I] means an inequality. So we set this up using the >= in relation to 17: [B]d^2 >= 17[/B]

d-i-v interest rate relationships
Free d-i-v interest rate relationships Calculator - Calculates d,i, or v based on 1 of the items entered.

D= {a,b,c,d,e,f,g} the cardinality of set D is
D= {a,b,c,d,e,f,g} the cardinality of set D is Cardinality of D, denoted |D|, is the number of items in the set: |D| = [B]7[/B]

Dad is (y) years old. Mom is 5 years younger than Dad. What is the total of their ages
Dad is (y) years old. Mom is 5 years younger than Dad. What is the total of their ages Dad's age: y Mom's age (younger means we subtract): y - 5 The total of their ages is found by adding them together: y + y - 5 Group like terms, and we get: [B]2y - 5[/B]

Dakota needs a total of $400 to buy a new bicycle. He has $40 saved. He earns $15 each week deliveri
Dakota needs a total of $400 to buy a new bicycle. He has $40 saved. He earns $15 each week delivering newspapers. How many weeks will Dakota have to deliver papers to have enough money to buy the bicycle? Let w be the number of weeks of delivering newspapers. We have the equation: 15w + 40 = 400 To solve for w, we [URL='https://www.mathcelebrity.com/1unk.php?num=15w%2B40%3D400&pl=Solve']type this equation into our search engine[/URL] and we get: w = [B]24[/B]

Dale has a bookcase with d shelves. There are 14 books on each shelf. Using d, write an expression f
Dale has a bookcase with d shelves. There are 14 books on each shelf. Using d, write an expression for the total number of books. We multiply the number of shelves by the number of books per shelf. [B]14d[/B]

Dale has a box that contains 20 American quarters and 20 Canadian quarters. If he takes them from th
Dale has a box that contains 20 American quarters and 20 Canadian quarters. If he takes them from the box one at a time, how many must he remove before he is guaranteed to have 5 quarters from the same country? Worst case scenario, Dale picks 4 American and 4 Canadian quarters which guarantees his next pick would be a 5th of either quarter. So the answer is 4 + 4 + 1 = [B]9[/B]

Dan and four friends planned a 32-day hike to the summit of Mt. Everest. After they bought enough fo
Dan and four friends planned a 32-day hike to the summit of Mt. Everest. After they bought enough food to last the five people 32 days, three other friends decided that they also wanted to go along on the adventure. How long will the food last now that eight people are taking the trip? If the food last 32 days for 5 people, it lasts 32 * 5 = 160 days for 8 people. Therefore, for 8 people, the food lasts 160/8 = [B]20 days[/B]

Dan bought 7 new baseball trading cards to add to his collection. The next day his dog ate half of h
Dan bought 7 new baseball trading cards to add to his collection. The next day his dog ate half of his collection. There are now only 26 cards left. How many cards did Dan start with? Let the starting amount of cards be s. We're given: [LIST] [*]Dan bought 7 new cards: s + 7 [*]The dog ate half of his collection. This means he's left with half, or (s + 7)/2 [*]Now, he's got 26 cards left. So we set up the following equation: [/LIST] (s + 7)/2 = 26 Cross multiply: s + 7 = 26 * 2 s + 7 = 52 To solve for s, [URL='https://www.mathcelebrity.com/1unk.php?num=s%2B7%3D52&pl=Solve']we plug this equation into our search engine[/URL] and we get: s = [B]45[/B]

Dan bought 8 new baseball trading cards to add to his collection. The next day his dog ate half of h
Dan bought 8 new baseball trading cards to add to his collection. The next day his dog ate half of his collection. There are now only 30 cards left. How many cards did Dan start with? Let the original collection count of cards be b. So we have (b + 8)/2 = 30 Cross multiply: b + 8 = 30 * 2 b + 8 = 60 [URL='http://www.mathcelebrity.com/1unk.php?num=b%2B8%3D60&pl=Solve']Use the equation calculator[/URL] [B]b = 52 cards[/B]

Dan bought a computer in a state that has a sales tax rate of 7%. If he paid $67.20 sales tax, what
Dan bought a computer in a state that has a sales tax rate of 7%. If he paid $67.20 sales tax, what did the computer cost? Set up the equation for price p: p * 0.07 = 67.20 p = 67.20 / 0.07 p = [B]$960[/B]

Dan has a favorite fast food restaurant where he always orders French fries and a milk shake. If the
Dan has a favorite fast food restaurant where he always orders French fries and a milk shake. If the fries contain 15 grams of fat and the shake contains 9 grams of fat, how many burgers, at 17 grams of fat each, can Dan add to his fries and milkshake if he wants to keep the total fat content of his meal no greater than 69 grams? His original meal is 1 fry and 1 shake. This contains 15 + 9 = 24 grams of fat. To limit his meal to 69 grams of fat, he has 69 - 24 = 45 grams of fat left over. Therefore, he can consume: 17b <= 45 where b is the number of burgers Dividing by 17, we get b = 2.65. Since he does not want to go over 45, he can eat 2 burgers.

Dan makes 11 an hour working at the local grocery store. Over the past year he has saved 137.50 towa
Dan makes 11 an hour working at the local grocery store. Over the past year he has saved 137.50 toward a new pair of retro sneakers. If sneakers cost 240, how many hours will he need to be able to buy the sneakers? Figure out his remaining savings target: 240 - 137.50 = 102.50 Let x equal the number of remaining hours Dan needs to work 11x = 102.50 Divide each side by 11 x = 9.318 We round up for a half-hour to 9.5, or a full hour to 10.

Dan needs 309 programs for the school play on Thursday. How many boxes of programs will he need, giv
Dan needs 309 programs for the school play on Thursday. How many boxes of programs will he need, given that each box contains 41 programs? Each box contains 41 programs, so we divide 309 programs by 41 programs per box to get our boxes: 309/41 using our [URL='https://www.mathcelebrity.com/longdiv.php?num1=309&num2=41&pl=Long%20Division%20%28Decimals%29']division calculator[/URL] is 7.5365. Since we don't have fractional boxes, we round up to the next highest integer. [B]8 boxes[/B]

Dan's school is planning a field trip to an art museum. Bus company A charges a $60 rental fee plus
Dan's school is planning a field trip to an art museum. Bus company A charges a $60 rental fee plus $4 per student. Bus company B charges $150 plus $2 per student. How many students would have to go for the cost to be the same? [U]Set up Company A's cost equation C(s) where s is the number of students[/U] C(s) = Cost per student * s + Rental Fee C(s) = 4s + 60 [U]Set up Company B's cost equation C(s) where s is the number of students[/U] C(s) = Cost per student * s + Rental Fee C(s) = 2s + 150 The problem asks for s where both C(s) equations would be equal. So we set Company A and Company B's C(s) equal to each other: 4s + 60 = 2s + 150 To solve for s, we [URL='https://www.mathcelebrity.com/1unk.php?num=4s%2B60%3D2s%2B150&pl=Solve']type this equation into our search engine[/URL] and we get: s = [B]45[/B]

Dane wrote the letters of “NEW YORK CITY” on cards and placed them in a hat. What is the probability
Dane wrote the letters of “NEW YORK CITY” on cards and placed them in a hat. What is the probability that he will draw the letter “Y” out of the hat? New York City has 11 letters. Our probability of drawing a Y is denoted as P(Y): P(Y) = Number of Y's / Total Letters P(Y) = [B]2/11[/B]

Daniel is 41 inches tall. He is 3/5 as tall as his brother. How tall is his brother?
Daniel is 41 inches tall. He is 3/5 as tall as his brother. How tall is his brother? We set Daniel's brother's height at h. We have: 3h/5 = 41 To solve this equation for h, we [URL='https://www.mathcelebrity.com/prop.php?num1=3h&num2=41&den1=5&den2=1&propsign=%3D&pl=Calculate+missing+proportion+value']type it in our search engine[/URL] and we get: [B]h = 68.3333 or 68 & 1/3[/B]

Daniel is 6cm taller than Kamala. If their total height is 368cm, how tall is Kamala?
Daniel is 6cm taller than Kamala. If their total height is 368cm, how tall is Kamala? Let Daniel's height be d. Let Kamala's height be k. We're given two equations: [LIST=1] [*]d = k + 6 [*]d + k = 368 [/LIST] Substitute equation (1) into equation (2) for d: k + 6 + k = 368 To solve for k, we [URL='https://www.mathcelebrity.com/1unk.php?num=k%2B6%2Bk%3D368&pl=Solve']type this equation into our search engine[/URL] and we get: k = [B]181[/B]

Daniel pays $10 to get into the parking lot and will pay a fee of $2 per hour his car will be left i
Daniel pays $10 to get into the parking lot and will pay a fee of $2 per hour his car will be left in the parking lot. He ending up paying a total of $23 for parking. How many hours was Daniels car left in the parking lot? Calculate the amount of fees for hours: Fees for hours = Total Bill - Entrance fee Fees for hours = 23 - 10 Fees for hours = 13 Calculate the number of hours Daniel parked: Number of hours = Fees for hours / Hourly Rate Number of hours = 13/2 Number of hours = [B]6.5[/B]

Daniel's gas tank holds n gallons of gas. How many pints does the tank hold?
Daniel's gas tank holds n gallons of gas. How many pints does the tank hold? Setup conversion: 1 gallon = 8 pints n gallons = [B]8n[/B] pints

Danna walked along a road. Starting from her house she walked 14 meters due south then walked 8 mete
Danna walked along a road. Starting from her house she walked 14 meters due south then walked 8 meters due north and finally walked 20 meters due south. how far away was Danna from her hours 14 - 8 + 20 = [B]26 miles due south[/B]

Danny buys 5 books at $34 each and pays for them with 10-dollar bills. How many $10 bills did it tak
Danny buys 5 books at $34 each and pays for them with 10-dollar bills. How many $10 bills did it take? Calculate his total bill: Total bill = Number of books * cost per book Total bill = 5 * 34 Total bill = 170 Now calculate the number of 10-dollar bills he used: 10-dollar bills used = Total bill / 10 10-dollar bills used = 170/10 10-dollar bills used = [B]17[/B]

Danny's mom ate 1/6 of an ice cream cake. Danny and his sister want to split the remainder of it. Ho
Danny's mom ate 1/6 of an ice cream cake. Danny and his sister want to split the remainder of it. How much of the cake would each get? If Danny's mom ate 1/6 of the cake, then we have: 1 - 1/6 of the cake left. We [URL='https://www.mathcelebrity.com/fraction.php?frac1=1&frac2=1%2F6&pl=Subtract']use our fraction subtraction calculator[/URL] for 1 - 1/6 to get: 5/6 If Danny and his sister split the remainder, then we divide 5/6 by 2. It's also the same as multiplying 5/6 by 1/2: We [URL='https://www.mathcelebrity.com/fraction.php?frac1=5%2F6&frac2=1%2F2&pl=Multiply']use our fraction multiplication calculator[/URL] to get: [B]5/12 for Danny and his sister[/B]

data below consists of the pulse rates (in beats per minute) of 32 students. Assuming ? = 10.66,
The data below consists of the pulse rates (in beats per minute) of 32 students. Assuming ? = 10.66, obtain a 95.44% confidence interval for the population mean. 80 74 61 93 69 74 80 64 51 60 66 87 72 77 84 96 60 67 71 79 89 75 66 70 57 76 71 92 73 72 68 74

Date and Time Difference
Free Date and Time Difference Calculator - Calculates the difference between two dates using the following methods
1) Difference in dates using year/month/day/hour/minute/second as the primary unit of time
2) Difference in dates in the form of years remaining, months remaining, days remaining, hours remaining, minutes remaining, seconds remaining.

Date Information
Free Date Information Calculator - This calculator takes a date in mm/dd/yyyy format, and gives the following information about it:
* Weekday
* Day number in the year
* Week number in the year
* Number of days in the month containing that date
* Leap Year (Yes or No)
* Zodiac Sign
* Julian Date

Dave has a savings account that pays interest at 3 1/2% per year. His opening balance for May was $1
Dave has a savings account that pays interest at 3 1/2% per year. His opening balance for May was $1374.67. He did not deposit or withdraw money during the month. The interest is calculated daily. How much interest did the account earn in May? First, determine n, which is 31, since May has 31 days. We use our [URL='http://www.mathcelebrity.com/compoundint.php?bal=1374.67&nval=31&int=3.5&pl=Daily']compound interest balance calculator[/URL] to get: [B]1,378.76[/B]

Dave paints a fence in 4 hours while Sara paints the same fence in 2 hours. If they work together, h
Dave paints a fence in 4 hours while Sara paints the same fence in 2 hours. If they work together, how long will it take them to paint the fence? Set up unit rates: [LIST] [*]Dave paints 1/4 of the fence in 1 hour [*]Sara will paint 1/2 of the fence in 1 hour [/LIST] So together, they paint 1/2 + 1/4 = 2/4 + 14 = 3/4 of the fence in one hour. 1 hour = 60 minutes, so we set up a proportion of time to minutes where m is the time in minutes needed to complete 1 full fence: 3/4/60 = 1/m 3/240 = 1/m [URL='https://www.mathcelebrity.com/proportion-calculator.php?num1=3&num2=1&den1=240&den2=m&propsign=%3D&pl=Calculate+missing+proportion+value']Typing this proportion in our math engine[/URL], we get: m = [B]80 minutes[/B] [B]80 minutes is also 1 hour and 20 minutes.[/B]

Dave rented a limousine for his wife's birthday. The hourly rate is $60. They used the limousine for
Dave rented a limousine for his wife's birthday. The hourly rate is $60. They used the limousine for 4 hours, plus Dave gave the driver a 20% tip. How much did he spend in total for the hourly charges plus tip? Hourly Spend = $60 * 4 = $240 Calculate 20% tip 0.2 * $240 = $48 Calculate total: $240 + 48 = [B]$288[/B]

David had 20 pencils. 5 of them are green and 15 are purple. What percentage of pencils were gr
David had 20 pencils. 5 of them are green and 15 are purple. What percentage of pencils were green? Percentage of Green Pencils = 100* Green Pencils / Total Pencils Percentage of Green Pencils = 100* 5/20 Percentage of Green Pencils = 500 / 20 Percentage of Green Pencils = [B]25%[/B]

David has b dollars in his bank account; Claire has three times as much money as David. The sum of t
David has b dollars in his bank account; Claire has three times as much money as David. The sum of their money is $240. How much money does Claire have? David has b Claire has 3b since three times as much means we multiply b by 3 The sum of their money is found by adding David's bank balance to Claire's bank balance to get the equation: 3b + b = 240 To solve for b, [URL='https://www.mathcelebrity.com/1unk.php?num=3b%2Bb%3D240&pl=Solve']we type this equation into our search engine[/URL] and we get: b = 60 So David has 60 dollars in his bank account. Therefore, Claire has: 3(60) = [B]180[/B]

David obtained 5 out of 20 votes in the election. What percentage of the votes did david receive?
David obtained 5 out of 20 votes in the election. What percentage of the votes did david receive? 5/20 is the fraction. You can simplify by dividing top and bottom by 5 to get 1/4 As a decimal, this is 0.25 To get a percentage, multiply the decimal by 100 100 * 0.25 = 25% You can also use our [URL='http://www.mathcelebrity.com/perc.php?num=+5&den=+20&pcheck=1&num1=6500&pct1=70&pct2=70&den1=80&idpct1=10&hltype=1&idpct2=90&pct=82&decimal=+65.236&astart=12&aend=20&wp1=20&wp2=30&pof1=40&pof2=20&pl=Calculate']decimal-percentage-fraction converter[/URL]

David roller skates for 3 1/3 hours with a constant speed of 24 km/h and then for another 1 hour 10
David roller skates for 3 1/3 hours with a constant speed of 24 km/h and then for another 1 hour 10 minutes with constant speed of 12 km/h. What distance did he go? Distance = Rate x Time [U]Part 1 of his trip:[/U] D1 = R1 x T1 D1 = 3 & 1/3 hours * 24 km/h D1 = 80 km [U]Part 2 of his trip:[/U] D2 = R2 x T2 D2 = 1 & 1/6 hours * 12 km/h (Note, 10 minutes = 1/6 of an hour) D2 = 14 km [U]Calculate Total Distance (D)[/U] D = D1 + D2 D = 80 + 14 D = [B]94 km[/B]

Dawn has less than $60. She wants to buy 3 sweaters. What price of sweaters can she afford if all th
Dawn has less than $60. She wants to buy 3 sweaters. What price of sweaters can she afford if all the sweaters are the same price? Let s be the price of each sweater. Write this as an inequality. The phrase [I]less than[/I] means an inequality, so we have the following inequality: 3s < 60 To solve this inequality, we [URL='https://www.mathcelebrity.com/interval-notation-calculator.php?num=3s%3C60&pl=Show+Interval+Notation']type it in our math engine[/URL] and we get: s < [B]20[/B]

Day of Year Calendar
Free Day of Year Calendar Calculator - Shows you the numeric day within a full calendar year and leap year

DeAndre is a spelunker (someone who explores caves). One day DeAndre is exploring a cave that has a
DeAndre is a spelunker (someone who explores caves). One day DeAndre is exploring a cave that has a series of ladders going down into the depths. Every ladder is exactly 10 feet tall, and there is no other way to descend or ascend (the other paths in the cave are flat). DeAndre starts at 186 feet in altitude, and reaches a maximum depth of 86 feet in altitude.Write an equation for DeAndre's altitude, using x to represent the number of ladders DeAndre used (hint: a ladder takes DeAndre down in altitude, so the coefficient should be negative). Set up a function A(x) for altitude, where x is the number of ladders used. Each ladder takes DeAndre down 10 feet, so this would be -10x. And DeAndre starts at 186 feet, so we'd have: [B]A(x) = 186 - 10x[/B]

Deanna has 5-cent stamps and 10-cent stamps. If she has 100 total stamps, what is the value of the s
Deanna has 5-cent stamps and 10-cent stamps. If she has 100 total stamps, what is the value of the stamps? Call the 5-cent stamps n. Value of 5-cent stamps 0.05n Number of 10 cent stamps is: 100 - n Value is 0.10(100 - n) = 10 - 0.10n Add them both: 10 - 0.10n + 0.05n [B]10 - 0.05n[/B]

Debbie baked 32 cookies with 4 scoops of flour. With 10 scoops of flour, how many cookies can Debbie
Debbie baked 32 cookies with 4 scoops of flour. With 10 scoops of flour, how many cookies can Debbie bake? Set up a proportion of cookies to scoops of flour, where c is the number of cookies per 10 scoops of flour: 32/4 = c/10 [URL='https://www.mathcelebrity.com/prop.php?num1=32&num2=c&den1=4&den2=10&propsign=%3D&pl=Calculate+missing+proportion+value']Typing this proportion into our search engine[/URL], we get: c = [B]80[/B]

Debra buys candy that costs 4 per pound. She will spend less than 20 on candy. What are the possible
Debra buys candy that costs 4 per pound. She will spend less than 20 on candy. What are the possible numbers of pounds she will buy? Set up an inequality using less than < and p for pounds: 4p < 20 Divide each side by 4: 4p/4 < 20/4 [B]p < 5[/B]

Decagon
Free Decagon Calculator - Solves for the side, perimeter, and area of a decagon.

Decagonal Number
Free Decagonal Number Calculator - This calculator determines the nth decagonal number

Declining Balance Depreciation
Free Declining Balance Depreciation Calculator - Solves for Depreciation Charge, Asset Value, and Book Value using the Declining Balance Method

Decompose Fraction
Free Decompose Fraction Calculator - Decomposes a Fraction. Shows you the work behind decomposing a fraction

Decrease 12 by a number
Decrease 12 by a number The phrase [I]a number[/I] means an arbitrary variable, let's call it x. We take 12 and decrease it by x, meaning we subtract x from 12: [B]12 - x[/B]

decrease a number by 7 and multiply by 6.
decrease a number by 7 and multiply by 6. The phrase [I]a number[/I] means an arbitrary variable, let's call it x: x Decrease a number by 7: x - 7 Multiply by 6 [B]6(x - 7)[/B]

Dedra took a total of 6 pages of notes during 2 hours of class. After attending 3 hours of class, ho
Dedra took a total of 6 pages of notes during 2 hours of class. After attending 3 hours of class, how many total pages of notes will Dedra have in her notebook? Set up a proportion of pages of notes to hours of class where p equals the number of pages of notes Dedra takes for 3 hours of class: 6/2 = p/3 [URL='https://www.mathcelebrity.com/prop.php?num1=6&num2=p&den1=2&den2=3&propsign=%3D&pl=Calculate+missing+proportion+value']Type this proportion into our search engine[/URL], and we get: p = [B]9[/B]

Del and his 5 friends eat 4 pizzas. If each has the same amount and there is 1/4 of a pizza left ove
Del and his 5 friends eat 4 pizzas. If each has the same amount and there is 1/4 of a pizza left over, how much did each person eat? This means 4 full pizzas - 1/4 of a pizza = 3 & 3/4 pizzas eaten Del and his 5 friends means 6 people total. Since they ate equal amounts, we divide pizzas eaten by total people: 3 & 3/4 / 6 Convert 3 & 3/4 to a mixed fraction: (4*3 + 3)/4 = 15/4 15/4/6 Divide by a fraction is the same as multiply by a reciprocal: 15/4 * 1/6 = [B]15/24 pizzas per person[/B]

Demoivres Theorem
Free Demoivres Theorem Calculator - Using Demoivres Theorem, this calculator performs the following:
1) Evaluates (acis(θ))n
2) Converts a + bi into Polar form
3) Converts Polar form to Rectangular (Standard) Form

DeMorgans Laws
Free DeMorgans Laws Calculator - Demonstrates DeMorgans Laws including the proof

Dennis was getting in shape for a marathon. The first day of the week he ran n miles. Dennis then ad
Dennis was getting in shape for a marathon. The first day of the week he ran n miles. Dennis then added a mile to his run each day. By the end of the week (7 days), he had run a total of 70 miles. How many miles did Dennis run the first day? Setup distance ran for the 7 days: [LIST=1] [*]n [*]n + 1 [*]n + 2 [*]n + 3 [*]n + 4 [*]n + 5 [*]n + 6 [/LIST] Add them all up: 7n + 21 = 70 Solve for [I]n[/I] in the equation 7n + 21 = 70 [SIZE=5][B]Step 1: Group constants:[/B][/SIZE] We need to group our constants 21 and 70. To do that, we subtract 21 from both sides 7n + 21 - 21 = 70 - 21 [SIZE=5][B]Step 2: Cancel 21 on the left side:[/B][/SIZE] 7n = 49 [SIZE=5][B]Step 3: Divide each side of the equation by 7[/B][/SIZE] 7n/7 = 49/7 n =[B] 7 [URL='https://www.mathcelebrity.com/1unk.php?num=7n%2B21%3D70&pl=Solve']Source[/URL][/B]

Density
Free Density Calculator - Solves for any of the 3 items in the Density Formula, Density (D), Mass (M), and Volume (V) (Capacity), with 2 given items.

Deon opened his account starting with $650 and he is going to take out $40 per month. Mai opened up
Deon opened his account starting with $650 and he is going to take out $40 per month. Mai opened up her account with a starting amount of $850 and is going to take out $65 per month. When would the two accounts have the same amount of money? We set up a balance equation B(m) where m is the number of months. [U]Set up Deon's Balance equation:[/U] Withdrawals mean we subtract from our current balance B(m) = Starting Balance - Withdrawal Amount * m B(m) = 650 - 40m [U]Set up Mai's Balance equation:[/U] Withdrawals mean we subtract from our current balance B(m) = Starting Balance - Withdrawal Amount * m B(m) = 850 - 65m When the two accounts have the same amount of money, we can set both balance equations equal to each other and solve for m: 650 - 40m = 850 - 65m Solve for [I]m[/I] in the equation 650 - 40m = 850 - 65m [SIZE=5][B]Step 1: Group variables:[/B][/SIZE] We need to group our variables -40m and -65m. To do that, we add 65m to both sides -40m + 650 + 65m = -65m + 850 + 65m [SIZE=5][B]Step 2: Cancel -65m on the right side:[/B][/SIZE] 25m + 650 = 850 [SIZE=5][B]Step 3: Group constants:[/B][/SIZE] We need to group our constants 650 and 850. To do that, we subtract 650 from both sides 25m + 650 - 650 = 850 - 650 [SIZE=5][B]Step 4: Cancel 650 on the left side:[/B][/SIZE] 25m = 200 [SIZE=5][B]Step 5: Divide each side of the equation by 25[/B][/SIZE] 25m/25 = 200/25 m = [B]8[/B]

Derangements - Subfactorials
Free Derangements - Subfactorials Calculator - Calculates the number of derangements/subfactorial !n.

Derek must choose a 4 digit PIN. Each Digit can be chosen from 0 to 9. Derek does not want to reuse
Derek must choose a 4 digit PIN. Each Digit can be chosen from 0 to 9. Derek does not want to reuse any digits. He also only wants an even number that begins with 5. How many possible PINS could he choose from? [LIST=1] [*]First digit must begin with 5. So we have 1 choice [*]We subtract 1 possible digit from digit 3 to have 8 - 1 = 7 possible digits [*]This digit can be anything other than 5 and the even number in the next step. So we have 0-9 is 10 digits - 2 = 8 possible digits [*]Last digit must end in 0, 2, 4, 6, 8 to be even. So we have 5 choices [/LIST] Our total choices from digits 1-4 are found by multiplying each possible digit choice: 1 * 7 * 8 * 5 = [B]280 possible PINS[/B]

Derivatives
Free Derivatives Calculator - This lesson walks you through the derivative definition, rules, and examples including the power rule, derivative of a constant, chain rule

Determine if the statement below is True or False
Determine if the statement below is True or False If B ? A, then A ? B = B Is this statement True or False? [B]True:[/B] If B ? A, then B ? A So A ? B is the similar elements of both. B contains itself as a subset. So this is [U]true[/U]

Determine the area under the standard normal curve that lies between:
Determine the area under the standard normal curve that lies between: (a) Z = -0.38 and Z = 0.38 (b) Z = -2.66 and Z = 0 (c) Z = -1.04 and Z - 1.67 [B](a) 0.2961 using our [URL='http://www.mathcelebrity.comzscore.php?z=+p%28-0.38%3Cz%3C0.38%29&pl=Calculate+Probability']z score calculator[/URL] (b) 0.4961 using our [URL='http://www.mathcelebrity.com/zscore.php?z=+p%28-2.66%3Cz%3C0%29&pl=Calculate+Probability']z score calculator[/URL] (c) 0.8034 using our [URL='http://www.mathcelebrity.com/zscore.php?z=+p%28-1.04%3Cz%3C1.67%29&pl=Calculate+Probability']z score calculator[/URL][/B]

Determine the formula of the given statement by following the procedures. Choose any number then add
Determine the formula of the given statement by following the procedures. Choose any number then add 2. Multiply your answer to 3 and minus 2 For the phrase [I]choose any number[/I] we can use an arbitrary variable, let's call it x. Add 2: x + 2 Multiply your answer to 3: 3(x + 2) And minus 2 which means we subtract: [B]3(x + 2) - 2[/B]

Determine ux and sigma(x) from the given parameters of the population and sample size u = 76, sigma
Determine ux and sigma(x) from the given parameters of the population and sample size u = 76, sigma = 28, n = 49 ux = ? sigma(x) = ? [B]u = ux = 76[/B] sigma(x) = sigma/sqrt(n) so we have 28/sqrt(49) = 28/7 = [B]4[/B]

Determine whether the random variable is discrete or continuous. In each case, state the possible v
Determine whether the random variable is discrete or continuous. In each case, state the possible values of the random variable. (a) The number of customers arriving at a bank between noon and 1:00 P.M. (i) The random variable is continuous. The possible values are x >= 0. (ii) The random variable is discrete. The possible values are x = 0, 1, 2,... (iii) The random variable is continuous. The possible values are x = 0, 1, 2,... (iv) The random variable is discrete. The possible values are x >= 0. (b) The amount of snowfall (i) The random variable is continuous. The possible values are s = 0, 1, 2,... (ii) The random variable is discrete. The possible values are s >= 0. (iii) The random variable is discrete. The possible values are s = 0, 1, 2,... (iv) The random variable is continuous. The possible values are s >= 0. [B](a) (ii) The random variable is discrete. The possible values are x = 0, 1, 2,... Discrete variables are limited in the values they can take between 9 and ? (b) (iv) The random variable is continuous. The possible values are s >= 0. Snowfall can be a decimal and can vary between 0 and ?[/B]

Determine whether the statement is true or false. If 0 < a < b, then Ln a < Ln b
Determine whether the statement is true or false. If 0 < a < b, then Ln a < Ln b We have a logarithmic property that states: ln(a) - ln(b) = ln (a / b) We're given a < b, so (a / b) < 1 Therefore: ln (a / b) < 0 And since ln(a) - ln(b) = ln (a / b) Then Ln(a) - Ln(b) < 0 Add Ln(b) to each side and we get: Ln(a) - Ln(b) + Ln(b) < 0 + Ln(b) Cancel the Ln(b) on the left side and we get: Ln(a)

Determine whether the statement is true or false. If y = e^2, then y’ = 2e
Determine whether the statement is true or false. If y = e^2, then y’ = 2e e^2 is a constant, and the derivative of a constant is 0. So y' = 0 So this is [B]FALSE[/B]

Determine whether the statement is true or false. You can always divide by e^x
Determine whether the statement is true or false. You can always divide by e^x [B]True. As x --> infinity, 1/e^x approaches 0 but never touches it.[/B]

devaughn,sageis2timessydneysage,thesumoftheiragesis78,whatissydneysage
devaughn,sageis2timessydneysage,thesumoftheiragesis78,whatissydneysage Let d be Devaughn's age. Let s be Sydney's age. We have two equations: [LIST=1] [*]d = 2s [*]d + s = 78 [/LIST] Substitute (1) into (2) 2s + s = 78 3s = 78 Entering [URL='http://www.mathcelebrity.com/1unk.php?num=3s%3D78&pl=Solve']3x = 78 into the search engine[/URL], we get [B]s = 26[/B].

Dewey Decimal System Classification
Free Dewey Decimal System Classification Calculator - Given a 3 digit code, this will determine the class, division, and section of the library book using the Dewey Decimal System.

Deyante made a loss of $79.45 on shirt he sold for $240. What was the price of the shirt?
Deyante made a loss of $79.45 on shirt he sold for $240. What was the price of the shirt? He sold for $240. If he took a loss, that means he bought the shirt for: $240 + 79.45 = [B]$319.45[/B]

Deyjiana reads 30 pages in 25 minutes. If she reads 210 pages at this rate, how long will it take he
Deyjiana reads 30 pages in 25 minutes. If she reads 210 pages at this rate, how long will it take her? Set up a proportion of pages to minutes, were m is the number of minutes it takes to read 210 pages: 30/25 = 210/m To solve this proportion for m, we [URL='https://www.mathcelebrity.com/prop.php?num1=30&num2=210&den1=25&den2=m&propsign=%3D&pl=Calculate+missing+proportion+value']type this proportion into our search engine[/URL] and we get: m = [B]175[/B]

Diana earns $8.50 working as a lifeguard. Write an equation to find Dianas money earned m for any nu
Diana earns $8.50 working as a lifeguard. Write an equation to find Dianas money earned m for any numbers of hours h Set up the revenue function: [B]R = 8.5h[/B]

Diana invested $3000 in a savings account for 3 years. She earned $450 in interest over that time pe
Diana invested $3000 in a savings account for 3 years. She earned $450 in interest over that time period. What interest rate did she earn? Use the formula I=Prt to find your answer, where I is interest, P is principal, r is rate and t is time. Enter your solution in decimal form rounded to the nearest hundredth. For example, if your solution is 12%, you would enter 0.12. Our givens are: [LIST] [*]I = 450 [*]P = 3000 [*]t = 3 [*]We want r [/LIST] 450 = 3000(r)(3) 450 = 9000r Divide each side by 9000 [B]r = 0.05[/B]

Dick invested $9538 in an account at 10% compounded annually. Calculate the total investment after
Dick invested $9538 in an account at 10% compounded annually. Calculate the total investment after 10 years. Round your answer to the nearest penny if necessary. Annual compounding means we don't need to make adjustments to interest rate per compounding period. [URL='https://www.mathcelebrity.com/compoundint.php?bal=9538&nval=10&int=10&pl=Annually']Using our compound interest calculator[/URL], we get our new balance after 10 years of: [B]$24,739.12[/B]

Diego is jogging at a rate of 5mi/h. A function relates how far Deigo jogs to his rate of speed.
Let d be distance and h be hours in time. Set up our function. [LIST] [*]f(h) = d [*][B]f(h) = 5h[/B] [/LIST] Read this out, it says, for every hour Diego jogs, multiply that by 5 to get the distance he jogs.

Diegos savings increased by 9 is 68 . Use the variable to represent Diegos savings.
Diegos savings increased by 9 is 68 . Use the variable to represent Diegos savings. Let Diego's savings be s. The phrase [I]increased by[/I] means add, so we add 9 to s s + 9 The phrase [I]is [/I]means equal to, so we set 2 + 9 = 68 [B]s + 9 = 68[/B]

difference between 2 positive numbers is 3 and the sum of their squares is 117
difference between 2 positive numbers is 3 and the sum of their squares is 117 Declare variables for each of the two numbers: [LIST] [*]Let the first variable be x [*]Let the second variable be y [/LIST] We're given 2 equations: [LIST=1] [*]x - y = 3 [*]x^2 + y^2 = 117 [/LIST] Rewrite equation (1) in terms of x by adding y to each side: [LIST=1] [*]x = y + 3 [*]x^2 + y^2 = 117 [/LIST] Substitute equation (1) into equation (2) for x: (y + 3)^2 + y^2 = 117 Evaluate and simplify: y^2 + 3y + 3y + 9 + y^2 = 117 Combine like terms: 2y^2 + 6y + 9 = 117 Subtract 117 from each side: 2y^2 + 6y + 9 - 117 = 117 - 117 2y^2 + 6y - 108 = 0 This is a quadratic equation: Solve the quadratic equation 2y2+6y-108 = 0 With the standard form of ax2 + bx + c, we have our a, b, and c values: a = 2, b = 6, c = -108 Solve the quadratic equation 2y^2 + 6y - 108 = 0 The quadratic formula is denoted below: y = -b ± sqrt(b^2 - 4ac)/2a [U]Step 1 - calculate negative b:[/U] -b = -(6) -b = -6 [U]Step 2 - calculate the discriminant ?:[/U] ? = b2 - 4ac: ? = 62 - 4 x 2 x -108 ? = 36 - -864 ? = 900 <--- Discriminant Since ? is greater than zero, we can expect two real and unequal roots. [U]Step 3 - take the square root of the discriminant ?:[/U] ?? = ?(900) ?? = 30 [U]Step 4 - find numerator 1 which is -b + the square root of the Discriminant:[/U] Numerator 1 = -b + ?? Numerator 1 = -6 + 30 Numerator 1 = 24 [U]Step 5 - find numerator 2 which is -b - the square root of the Discriminant:[/U] Numerator 2 = -b - ?? Numerator 2 = -6 - 30 Numerator 2 = -36 [U]Step 6 - calculate your denominator which is 2a:[/U] Denominator = 2 * a Denominator = 2 * 2 Denominator = 4 [U]Step 7 - you have everything you need to solve. Find solutions:[/U] Solution 1 = Numerator 1/Denominator Solution 1 = 24/4 Solution 1 = 6 Solution 2 = Numerator 2/Denominator Solution 2 = -36/4 Solution 2 = -9 [U]As a solution set, our answers would be:[/U] (Solution 1, Solution 2) = (6, -9) Since one of the solutions is not positive and the problem asks for 2 positive number, this problem has no solution

Difference of a and b, divided by 2
Difference of a and b, divided by 2. The difference of a and b is written as: a - b We divide this by 2: [B](a - b)/2[/B]

Difference of Proportions Test
Free Difference of Proportions Test Calculator - Calculates a test statistic and conclusion for a hypothesis for the difference of proportions

Difference of Two Squares
Free Difference of Two Squares Calculator - Factors a difference of squares binomial in the form a2 - b2 or multiplies 2 binomials through in the form (ax + by)(ax - by).

Digraph Items
Free Digraph Items Calculator - Given a digraph, this determines the leader, and symmetric matrix.

Dina is twice as old as Andrea. The sum of their age is 72. Find their present ages.
Dina is twice as old as Andrea. The sum of their age is 72. Find their present ages. Let d be Dina's age. Let a be Andrea's age. We're given: [LIST=1] [*]d = 2a <-- Twice means multiply by 2 [*]a + d = 72 [/LIST] Substitute equation (1) into equation (2): a + 2a = 72 [URL='https://www.mathcelebrity.com/1unk.php?num=a%2B2a%3D72&pl=Solve']Type this equation into our search engine[/URL] and we get: [B]a = 24[/B] Substitute a = 24 into equation (1): d = 2(24) [B]d = 48 So Andrea is 24 years old and Dina is 48 years old[/B]

Dina is twice the age of Anton. If Anton is 12, how will you represent the age of Dina?
Dina is twice the age of Anton. If Anton is 12, how will you represent the age of Dina? Twice means multiply by 2, so we have: Dina = 2 * Anton's age Dina = 2 * 12 Dina = [B]24[/B]

Dinh has 4 more patients to care for than Juan. if Dinah has 18 patients to care for how many does J
Dinh has 4 more patients to care for than Juan. if Dinah has 18 patients to care for how many does Juan? Let d = j + 14. Since d = 18, we have: 18 = j + 14 [URL='http://www.mathcelebrity.com/1unk.php?num=j%2B14%3D18&pl=Solve']Plug this into the search engine and we have[/URL] j = 4

Diophantine Equations
Free Diophantine Equations Calculator - Solves for ax + by = c using integer solutions if they exist

Direct Current (Electrical Engineering) Ohms Law
Free Direct Current (Electrical Engineering) Ohms Law Calculator - Enter two of the following items from the DIRECT CURRENT(DC) electrical engineering set of variables, and this will solve for the remaining two:
* I = current(amps.)
* V = Electricity potential of voltage(volts)
* R = resistance(ohms)
* P = power(watts)

distance between -2 and 9 on the number line
distance between -2 and 9 on the number line Distance on the number line is the absolute value of the difference: D = |9 - -2| D = |11| D = [B]11[/B]

Distance Catch Up
Free Distance Catch Up Calculator - Calculates the amount of time that it takes for a person traveling at one speed to catch a person traveling at another speed when one person leaves at a later time.

Distance Rate and Time
Free Distance Rate and Time Calculator - Solves for distance, rate, or time in the equation d=rt based on 2 of the 3 variables being known.

Distributive Property
Free Distributive Property Calculator - Demonstrates the distributive property using 3 numbers. Numerical Properties

Divide 10 by the difference of z and y
[U]The difference of z and y means we subtract y from z[/U] z - y [U]Now, we form a fraction, where 10 is the numerator and z - y is the denominator[/U] 10/(z - y)

Divide 17 by g. Then, subtract 9.
Divide 17 by g. Then, subtract 9. Divide 17 by g 17/g Subtract 9 [B]17/g - 9[/B]

Divide 73 into two parts whose product is 402
Divide 73 into two parts whose product is 40 Our first part is x Our second part is 73 - x The product of the two parts is: x(73 - x) = 40 Multiplying through, we get: -x^2 + 73x = 402 Subtract 40 from each side, we get: -x^2 + 73x - 402 = 0 This is a quadratic equation. To solve this, we type it in our search engine, choose "solve Quadratic", and we get: [LIST=1] [*]x = [B]6[/B] [*]x = [B]67[/B] [/LIST]

divide 8 by 9, then subtract t
divide 8 by 9, then subtract t Divide 8 by 9 8/9 Then subtract t [B]8/9 - t[/B]

divide 8 by t, raise the result to the 7th power
divide 8 by t, raise the result to the 7th power. We take this algebraic expression in two parts: 1. Divide 8 by t 8/t 2. Raise the result to the 7th power. (This means we use an exponent of 7) [B](8/t)^7[/B]

divide a by 8, triple the result, then add 7
divide a by 8, triple the result, then add 7 [LIST] [*]Divide a by 8: a/8 [*]Triple the result means multiply by 3: 3a/8 [*]Then add 7 [/LIST] [B]3a/8 + 7[/B]

Divide a by b, double the result, then multiply c by what you have
Divide a by b, double the result, then multiply c by what you have Take this algebraic expression in parts: [LIST] [*]Divide a by b: a/b [*]Double the result means multiply by 2: 2a/b [*]Then multiply c by what you have: [/LIST] [B]2ac/b[/B]

divide a by c, triple the result, then subtract what you have from b
divide a by c, triple the result, then subtract what you have from b Let's take this algebraic expression in parts: [LIST=1] [*]Divide a by c: a/c [*]Triple the result. This means we multiply a/c by 3: 3a/c [*]Then subtract what you have (the result) from b: b - 3a/c [/LIST] [B]b - 3a/c[/B]

Divide a number by 10. Then, add 10.
Divide a number by 10. Then, add 10. The phrase [I]a number[/I] means an arbitrary variable, let's call it x. Divide the number by 10 mean we have a quotient, of x over 10 x / 10 Then, add 10: [B](x / 10) + 10[/B]

divide b by a, subtract the result from c, then add what you have to d
divide b by a, subtract the result from c, then add what you have to d Take this algebraic expression in 3 parts: [U]1) Divide b by a:[/U] b/a [U]2) Subtract the result from c:[/U] c - b/a [U]3) Then add what you have to d:[/U] [B]c - b/a + d[/B]

divide d by a, add the result to b, then add c
divide d by a, add the result to b, then add c [LIST] [*]Divide d by a: d/a [*]add the result to b: b + d/a [*]Then add c [/LIST] [B]b + d/a + c[/B]

Divide m by 3 and then add 10
Divide m by 3 and then add 10 Divide m by 3: m/3 Then add 10: [B]m/3 + 10[/B]

Divide the difference of 4 and r by 10
Divide the difference of 4 and r by 10 The difference of 4 and r, mean we subtract r from 4: 4 - r Now we divide this expression by 10: [B](4 - r)/10 [/B]

divide the difference of q and s by the sum of p and r
divide the difference of q and s by the sum of p and r Take this algebraic expression in pieces: [LIST] [*]The difference of q and s: q - s [*]The sum of p and r: p + r [*]The word [I]divide[/I] means we divide q - s by p + r [/LIST] [B](q - s)/(p + r)[/B]

Divide the sum of a and b by the square of c
Divide the sum of a and b by the square of c The sum of a and b: a + b The square of c means we raise c to the power of 2: c^2 Divide means we have a quotient, with a + b on top, and c^2 on the bottom: [B](a + b)/c^2[/B]

divide the sum of the square of a and b by thrice c
divide the sum of the square of a and b by thrice c Sum of the squares of a and b is found as follows: [LIST] [*]a squared means we raise a to the power of 2: a^2 [*]b squared means we raise b to the power of 2: b^2 [*]Sum of the squares means we add both terms: a^2 + b^2 [*]Thrice c means we multiply c by 3: 3c [/LIST] Divide means we have a quotient: [B](a^2 + b^2)/3c[/B]

Divide the sum of the squares of a and b by the square of c
Divide the sum of the squares of a and b by the square of c square of a: a^2 square of b: b^2 Sum of the squares of a and b: a^2 + b^2 square of c: c^2 Divide the Sum of the squares of a and b by the square of c: [B](a^2 + b^2)/c^2[/B]

Divide the sum x and y by the difference of subtracting a from b
Divide the sum x and y by the difference of subtracting a from b The sum x and y is written as: x + y The difference of subtracting a from b is written as: b - a We divide and get the algebraic expression: [B](x + y)/(b - a)[/B]

divide u by s multiply the result by v
divide u by s multiply the result by v Divide u by s: u/s Multiply the result by v: [B]uv/s[/B]

divide u by s, then subtract the result from t
divide u by s, then subtract the result from t Divide u by s: u/s Subtract the result from t: [B]t - u/s[/B]

divide u by w add the result to v
divide u by w add the result to v Divide u by w: u/w Add the result to v: [B]v + u/w[/B]

Divide v by the sum of 4 and w
Divide v by the sum of 4 and w The sum of 4 and w means we add w to 4: 4 + w Next, we divide v by this sum to get our final algebraic expression: [B]v/(4 + w)[/B]

Divide x by 2.2, and then add 2.2 to the quotient.
Divide x by 2.2, and then add 2.2 to the quotient. Divide x by 2.2 (This is a quotient): x/2.2 Then add 2.2 to the quotient [B]x/2.2 + 2.2[/B]

Divide x cubed by the quantity x minus 7
Divide x cubed by the quantity x minus 7 x cubed means we raise x to the power of 3: x^3 We divide this by x - 7: [B]x^3/(x - 7)[/B]

Dividend Discount Model
Free Dividend Discount Model Calculator - This calculator determines the present value of dividends using the Dividend Discount Model.

Divisibility
Free Divisibility Calculator - Shows the divisibility of a number by seeing if it is divisible by (2,3,4,5,6,7,8,9,10,11)

Divisibility by 11 no calculator shortcuts
2 rules. If either of them passes, then the number is divisible by 11: [LIST=1] [*]Sum of the odd digits - Sum of the even digits is divisible by 11 [*]Sum of the odd digits - Sum of the even digits = 0 (Ex. 121) [/LIST] [MEDIA=youtube]WpV87es0WAU[/MEDIA]

Division Equality Property
Free Division Equality Property Calculator - Demonstrates the Division Equality Property Calculator Numerical Properties

Division Property Of Inequality
Free Division Property Of Inequality Calculator - Demonstrates the Division Property Of Inequality Numerical Properties

Divya has 70 rocks. She donates half of the rocks to a science center. Then she collects 3 rocks on
Divya has 70 rocks. She donates half of the rocks to a science center. Then she collects 3 rocks on each of her nature hikes. Write an expression to represent the number of rocks Divya has after she collects rocks on n nature hikes. For each hike, we have: [LIST=1] [*]Start with 70 rocks [*]She donates half which is 35, which means she's left with 35 [/LIST] Since each nature hike gives her 3 more rocks, and she goes on n nature hikes, we have the following algebraic expression: [B]3n + 35[/B]

Do the phrases 7 less than a number and a number less than 7 mean the same thing explain
Do the phrases 7 less than a number and a number less than 7 mean the same thing explain No, they are different, here's how: First, the phrase [I]a number[/I] means an arbitrary variable, let's call it x. 7 less than a number means we subtract 7 from x: x - 7 A number less than 7 means we subtract x from 7: 7 - x As you can see: x - 7 <> 7 - x so [B]they are different[/B]

Does (6,5) make the equation y = x true
Does (6,5) make the equation y = x true x =6 and y = 5, so we have: 5 = 6 which is [B]false. So no[/B], it does not make the equation True.

does the equation y= x/3 represent a direct variation? If so, state the value of k
does the equation y= x/3 represent a direct variation? If so, state the value of k [B]Yes[/B], it's a direct variation equation. We rewrite this as: y = 1/3 * x So k = 1/3, and y varies directly as x.

Does the point (0, 3) satisfy the equation y = x?
Does the point (0, 3) satisfy the equation y = x? Plug in our values of x = 0 and y = 3: 3 = 0 This is false, so the point (0,3) does [B]not satisfy[/B] the equation y = x

Does the point (2, 4) satisfy the equation y = 2x?
Does the point (2, 4) satisfy the equation y = 2x? Plug in x = 2 to y = 2x: y = 2(2) y = 4 [B]Yes, the point (2,4) satisfies the equation y = 2x[/B]

Does the point (3, 0) satisfy the equation y = x?
Does the point (3, 0) satisfy the equation y = x? plug in x = 3 and y = 0 into y = x 0 = 3 which is [B]false. No, it doesn't satisfy the equation[/B]

does the point (3,0) line on the line y=3x
does the point (3,0) line on the line y=3x Substitute the x value of (x,y) = (3,0) into y = 3x: y = 3(3) y = 9 Since y = 9 and y <> 0, then no, this point [B]does not[/B] lie on the line

Dollar Weighted Interest Method
Free Dollar Weighted Interest Method Calculator - Solves for Interest Rate, Starting Asset Value, Ending Asset Value, and Expenses using the Dollar Weighted Method.

Dominic bought 3 bags of beans. There are b beans in each bag.
Dominic bought 3 bags of beans. There are b beans in each bag. Total beans = Bags of beans * beans per bag Total beans = [B]3b[/B]

Don wants to bring some sand home from his vacation at the beach. He has a box that is 3 inches wide
Don wants to bring some sand home from his vacation at the beach. He has a box that is 3 inches wide, 4 inches long, and 2 inches tall. How much sand can he fit in the box? We want the volume. The volume of a rectangular solid is found with the formula: V = lwh V = 4 * 3 * 2 V = [B]24 cubic inches[/B]

Donna bought 4 bags of dog treats for $9.36. What is the cost per bag of dog treats?
Donna bought 4 bags of dog treats for $9.36. What is the cost per bag of dog treats? Using our unit cost formula, we get: $9.36/4 [B]$2.34 per bag[/B]

Dora has $35 saved. She earns $9.50 per hour at her job. How many hours must she work to have a tota
Dora has $35 saved. She earns $9.50 per hour at her job. How many hours must she work to have a total of $358 in her savings? Subtract the existing savings from the desired savings to see what we have left: 358 - 35 = 323 Now, at 9.50 per hour, how many hours of work does she need to get 323? Let h be the number of hours. We have: 9.50h = 323 [URL='http://www.mathcelebrity.com/1unk.php?num=9.50h%3D323&pl=Solve']Running this problem through our search engine[/URL], we get [B]h = 34[/B]

Dotty McGinnis starts up a small business manufacturing bobble-head figures of famous soccer players
Dotty McGinnis starts up a small business manufacturing bobble-head figures of famous soccer players. Her initial cost is $3300. Each figure costs $4.50 to make. a. Write a cost function, C(x), where x represents the number of figures manufactured. Cost function is the fixed cost plus units * variable cost. [B]C(x) = 3300 + 4.50x[/B]

double 10, add the result to 9, then add y
double 10, add the result to 9, then add y Double 10 means multiply 10 by 2: 10 * 2 20 Add the result to 9, means we add 20 to 9: 20 + 9 29 Then we add y: [B]29 + y[/B]

double 6 , divide the result by y ,then raise what you have to the 10th power
double 6 , divide the result by y ,then raise what you have to the 10th power Take this in pieces: Double 6 means multiply 6 by 2 --> 6(2) = 12 Divide the result by y: 12/y Then raise what you have to the 10th power: [B](12/y)^10[/B]

Double Declining Balance Depreciation
Free Double Declining Balance Depreciation Calculator - Calculates Depreciation and Book Value using the Double Declining Balance Depreciation Method.

double n, multiply answer by 3
double n, multiply answer by 3 Double n means multiply n by 2 2n Multiply the answer by 3: 3(2n) = [B]6n[/B]

double the quotient of 4 and 7
double the quotient of 4 and 7 The quotient fo 4 and 7: 4/7 Double means multiply by this expression by 2: [B]2(4/7)[/B] If you need to evaluate and simplify this, it's: [B]8/7[/B]

double v, add u, then divide t by what you have
double v, add u, then divide t by what you have Double v means we multiply the variable v by 2: 2v Add u: 2v + u We build a fraction, with t as the numerator, and 2v + u as the denominator [B]t/(2v + u)[/B]

double v, raise the result to the 6th power, then multiply what you have by w
double v, raise the result to the 6th power, then multiply what you have by w Double v means multiply v by 2: 2v Raise the result to the 6th power, means we use an exponent of 6 on 2v: (2v)^6 Then multiply what you have by w, means take the result above, and multiply by w: [B]w(2v)^6[/B]

DoubleClick Search Fundamentals
Exam answers and study guide for the Google DoubleClick Search Fundamentals exam

Dr. Carlson is contemplating the impact of an antibiotic on a particular patient. The patient will t
Dr. Carlson is contemplating the impact of an antibiotic on a particular patient. The patient will take 229 milligrams, and every hour his body will break down 20% of it. How much will be left after 9 hours? Set up the antibiotic remaining function A(h) where h is the number of hours after the patient takes the antibiotic. If the body breaks down 20%, then the remaining is 100% - 20% = 80% 80% as a decimal is 0.8, so we have: A(h) = 229 * (0.8)^h The problems asks for A(9): A(9) = 229 * (0.8)^9 A(9) = 229 * 0.134217728 A(9) = [B]30.74 milligrams[/B]

Dr. Hoffman is contemplating the impact of an antibiotic on a particular patient. The patient will t
Dr. Hoffman is contemplating the impact of an antibiotic on a particular patient. The patient will take 590 milligrams, and every hour his body will break down 30% of it. How much will be left after 8 hours? If necessary, round your answer to the nearest tenth. Set up a function A(h), where h is the number of hours since the patient took the antibiotic. If the body breaks down 30%, it keeps 70%, or 0.7. A(h) = 590(0.70)^h The problem asks for A(8): A(8) = 590(0.70)^8 A(8) =590 * 0.05764801 A(8) = 34.012 hours Rounded to the nearest tenth, it's [B]34.0 hours[/B].

Dunder Mifflin will print business cards for $0.10 each plus setup charge of $15. Werham Hogg offers
Dunder Mifflin will print business cards for $0.10 each plus setup charge of $15. Werham Hogg offers business cards for $0.15 each with a setup charge of $10. What numbers of business cards cost the same from either company Declare variables: [LIST] [*]Let b be the number of business cards. [/LIST] [U]Set up the cost function C(b) for Dunder Mifflin:[/U] C(b) = Cost to print each business card * b + Setup Charge C(b) = 0.1b + 15 [U]Set up the cost function C(b) for Werham Hogg:[/U] C(b) = Cost to print each business card * b + Setup Charge C(b) = 0.15b + 10 The phrase [I]cost the same[/I] means we set both C(b)'s equal to each other and solve for b: 0.1b + 15 = 0.15b + 10 Solve for [I]b[/I] in the equation 0.1b + 15 = 0.15b + 10 [SIZE=5][B]Step 1: Group variables:[/B][/SIZE] We need to group our variables 0.1b and 0.15b. To do that, we subtract 0.15b from both sides 0.1b + 15 - 0.15b = 0.15b + 10 - 0.15b [SIZE=5][B]Step 2: Cancel 0.15b on the right side:[/B][/SIZE] -0.05b + 15 = 10 [SIZE=5][B]Step 3: Group constants:[/B][/SIZE] We need to group our constants 15 and 10. To do that, we subtract 15 from both sides -0.05b + 15 - 15 = 10 - 15 [SIZE=5][B]Step 4: Cancel 15 on the left side:[/B][/SIZE] -0.05b = -5 [SIZE=5][B]Step 5: Divide each side of the equation by -0.05[/B][/SIZE] -0.05b/-0.05 = -5/-0.05 b = [B]100[/B]

During a game, a basketball team scored 38 points worth of two-pointers, 33 points worth of three-po
During a game, a basketball team scored 38 points worth of two-pointers, 33 points worth of three-pointers, and 8 points worth of free throws. How many shots did the basketball team make during the game? shots (Hint: A free throw is worth one point). [U]Calculate two-point shots:[/U] Two-point shots = Two Pointers / 2 Two-point shots = 38/2 Two-point shots = 19 [U]Calculate three-point shots:[/U] Three-point shots = Three Pointers / 2 Three-point shots = 33/3 Three-point shots = 11 [U]Calculate free-throw shots:[/U] Free throw shots = Free throws / 1 Free throw shots = 8/1 Free throw shots = 8 [U]Calculate total shots:[/U] Total shots = Two-point shots + three-point shots + free throws Total shots = 19 + 11 + 8 Total shots = [B]38[/B]

During a performance, a juggler tosses one ball straight upward while continuing to juggle three oth
During a performance, a juggler tosses one ball straight upward while continuing to juggle three others. The height f(t), in feet, of the ball is given by the polynomial function f(t) = ?16t^2 + 26t + 3, where t is the time in seconds since the ball was thrown. Find the height of the ball 1 second after it is tossed upward. We want f(1): f(1) = ?16(1)^2 + 26(1) + 3 f(1) = -16(1) + 26 + 3 f(1) = -16 + 26 + 3 f(1) = [B]13[/B]

During a recent season Miguel Cabrera and Mike Jacobs hit a combined total of 46 home runs. Cabrera
During a recent season Miguel Cabrera and Mike Jacobs hit a combined total of 46 home runs. Cabrera hit 6 more home runs than Jacobs how many home runs did each player hit Let c be Miguel Cabrera's home runs and j be Mike Jacobs home runs. We are given two equations: [LIST=1] [*]c + j = 46 [*]c = j + 6 [/LIST] Substitute (2) into (1) (j + 6) + j = 46 Combine like terms: 2j + 6 = 46 [URL='https://www.mathcelebrity.com/1unk.php?num=2j%2B6%3D46&pl=Solve']Plugging this into our equation calculator[/URL], we get [B]j = 20[/B]. Substitute this into equation (2), we have: c = 20 + 6 [B]c = 26 [/B] Therefore, Mike Jacobs hit 20 home runs and Miguel Cabrera hit 26 home runs.

During the 2016 christmas season,UPS had 14 employees retire, 122 employees were hired and 31 left d
During the 2016 christmas season,UPS had 14 employees retire, 122 employees were hired and 31 left due to illness. If UPS ended the year with 410 employees, how many did they have at the start of the season? Let x be the number of employees at the start of the season. We have: [LIST] [*]-14 since retiring is an employee loss [*]+122 hired since hiring is an employee gain [*]-31 since illness means a leave [/LIST] x - 14 + 122 - 31 = 410 Using our [URL='http://www.mathcelebrity.com/1unk.php?num=x-14%2B122-31%3D410&pl=Solve']equation solver[/URL], we get: [B]x = 333[/B]

During the first lesson on coordinate grids, Audrey is asked by her teacher to find the ordered pair
During the first lesson on coordinate grids, Audrey is asked by her teacher to find the ordered pair (2,5) on the coordinate grid. Starting from (0,0) which movement should Audrey complete first? We move [B]2 spaces right (positive x), and then 5 spaces up (positive 5)[/B]

During the summer, you work 30 hours per week at a gas station and earn $8.75 per hour. You also wor
During the summer, you work 30 hours per week at a gas station and earn $8.75 per hour. You also work as a landscaper for $11 per hour and can work as many hours as you want. You want to earn a total of $400 per week. How many hours, t, must you work as a landscaper? [U]Calculate your gas station salary:[/U] Gas Station Salary = Hours Worked * Hourly Rate Gas Station Salary = 30 * $8.75 Gas Station Salary = $262.50 [U]Now subtract this from the desired weekly earnings of $400[/U] $400 - 262.50 = $137.50 The landscaper makes $11 per hour. And they want to make $137.50 from landscaping. So we have the following equation: 11t = 137.50 Run this through our [URL='https://www.mathcelebrity.com/1unk.php?num=11t%3D137.50&pl=Solve']equation calculator[/URL], and we get t = 12.5 hours.

During your first year on the job, you deposit $2000 in an account that pays 8.5%, compounded contin
During your first year on the job, you deposit $2000 in an account that pays 8.5%, compounded continuously. What will be your balance after 35 years? [URL='https://www.mathcelebrity.com/simpint.php?av=&p=2000&int=8.5&t=35&pl=Continuous+Interest']Using our continuous compound balance calculator[/URL], we get a balance of [B]$39,179.25.[/B]

Dwayne earn $6 for each hour of yard work. After doing a total of 3 hours of yard work, how much mon
Dwayne earn $6 for each hour of yard work. After doing a total of 3 hours of yard work, how much money will Dwayne have earned? We're given the hourly earnings equation below: Hourly Earnings = Hourly Rate * hours worked Hourly Earnings = $6 * 3 Hourly Earnings = [B]$18[/B]

Dwayne has 9 peppermints. Mary has p fewer peppermints than Dwayne. Choose the expression that shows
Dwayne has 9 peppermints. Mary has p fewer peppermints than Dwayne. Choose the expression that shows how many peppermints Mary has. The phrase [I]fewer than[/I] means we subtract: [B]9 - p[/B]

Dylan is playing darts. He hit the bullseye on 5 out of his last 20 tosses. Considering this data, h
Dylan is playing darts. He hit the bullseye on 5 out of his last 20 tosses. Considering this data, how many bullseyes would you expect Dylan to get during his next 16 tosses? We have a proportion of bullseyes to tosses where b is the number of bullseyes for 16 tosses: 5/20 = b/16 [URL='https://www.mathcelebrity.com/prop.php?num1=5&num2=b&den1=20&den2=16&propsign=%3D&pl=Calculate+missing+proportion+value']Type this proportion into our search engine[/URL] and we get: b = [B]4[/B]

Dylans mother tells Dylan he must spend less time playing electronic games. On the weekends he spend
Dylans mother tells Dylan he must spend less time playing electronic games. On the weekends he spends 9.5 hours playing electronic games. If he plays between 13 and 19 hours each week, how many hours does he play games on weekdays? Let x equal the number of hours Dylan plays electronic games per week. [U]Set up our inequality:[/U] 13 <= x <= 19 [U]To see how much he plays during weekdays, subtract off the weekend time[/U] 13 - 9.5 <= x <= 19 - 9.5 [B]3.5 <= x <= 9.5[/B]

Each brick is 14 inches long, 8 inches wide, and 5 inches tall.if they used 16,800 in3 of concrete,
Each brick is 14 inches long, 8 inches wide, and 5 inches tall.if they used 16,800 in3 of concrete, how many bricks did they make? Volume of a brick (V) is: V = l * w * h Plugging in our brick measurements, we get: V = 14 * 8 * 5 V = 560 Calculate number of bricks: Number of Bricks = Total Volume / Volume of one Brick Number of Bricks = 16,800/560 Number of Bricks =[B]30[/B]

Each calendar will selll for $5.00 each. Write an equation to model the total income,y, for selling
Each calendar will selll for $5.00 each. Write an equation to model the total income,y, for selling x calendars income (y) = Price * Quantity [B]y = 5x[/B]

Each carton of juice holds 750 millilitres. There are 24 cartons in each box. How much juice is ther
Each carton of juice holds 750 millilitres. There are 24 cartons in each box. How much juice is there in five boxes 750 millilitres 24 cartons 5 boxes ---------------- * ---------------- * 1 carton 1 box 750 millilitres * 24 cartons * 5 boxes = [B]90,000 millilitres[/B]

each classroom at HCS has a total of 26 desks. After Mr. Sean pond ordered 75 new desks the total nu
each classroom at HCS has a total of 26 desks. After Mr. Sean pond ordered 75 new desks the total number of desks in the school was 543. How many classrooms does the school have? Let d be the number of desks per classroom. We're given an equation: 26d + 75 = 543 To solve for d, [URL='https://www.mathcelebrity.com/1unk.php?num=26d%2B75%3D543&pl=Solve']type this equation into our search engine[/URL] and we get: d = [B]18[/B]

Each of 6 students reported the number of movies they saw in the past year. Here is what they repor
Each of 6 students reported the number of movies they saw in the past year. Here is what they reported. 19, 9, 14, 10, 16, 17. Find the mean number of movies that the students saw. If necessary, round your answer to the nearest tenth. The mean is the average, so we add up the 6 movie scores, and divide by 6. [URL='https://www.mathcelebrity.com/statbasic.php?num1=19%2C+9%2C+14%2C+10%2C+16%2C+17&num2=+0.2%2C0.4%2C0.6%2C0.8%2C0.9&pl=Number+Set+Basics']Mean (Average)[/URL] = Sum of 6 Movie Scores / 6 [URL='https://www.mathcelebrity.com/statbasic.php?num1=19%2C+9%2C+14%2C+10%2C+16%2C+17&num2=+0.2%2C0.4%2C0.6%2C0.8%2C0.9&pl=Number+Set+Basics']Mean (Average)[/URL] = 84 / 6 [URL='https://www.mathcelebrity.com/statbasic.php?num1=19%2C+9%2C+14%2C+10%2C+16%2C+17&num2=+0.2%2C0.4%2C0.6%2C0.8%2C0.9&pl=Number+Set+Basics']Mean (Average)[/URL] = 14.16666667 The problem asks us to round to the nearest tenth, which is the first decimal place. Since the 2nd decimal place, 6 is more than 5, we round the first decimal place up one and remove the rest. [B]14.2[/B]

Each of letters in the word PROPER are on separate cards, face down on the table. If you pick a card
Each of letters in the word PROPER are on separate cards, face down on the table. If you pick a card at random, what is the probability that its letter will be P or R? PROPER has 6 letters in it. It has 2 P's and 2 R's. So we have: Pr(P or R) = Pr(P) + Pr(R) Pr(P or R) = 2/6 + 2/6 Pr(P or R) = 4/6 We can simplify this. So [URL='https://www.mathcelebrity.com/fraction.php?frac1=4%2F6&frac2=3%2F8&pl=Simplify']we type this fraction in our search engine[/URL], choose simplify, and we get: Pr(P or R) = [B]2/3[/B]

Each of the letters in the word PLOTTING are on separate cards, face down on the table. If you pick
Each of the letters in the word PLOTTING are on separate cards, face down on the table. If you pick a card at random, what is the probability that its letter will be T or G? PLOTTING has to 8 letters. It has 2 T'sand 1 G, so we have: P(T or G) = P(T) + P(G) P(T or G) = 2/8 + 1/8 P(T or G) = [B]3/8[/B]

Each pair of skates has 8 wheels.if the skating rink has 87 pairs of skates. How many wheels are at
Each pair of skates has 8 wheels.if the skating rink has 87 pairs of skates. How many wheels are at the rink? Total Wheels = Pairs of Skates * 8 wheels per pair Total Wheels = 87 * 8 Total Wheels = [B]609[/B]

Each piece of candy costs 25 cents. The cost of x pieces of candy is $2.00. Use variable x to transl
Each piece of candy costs 25 cents. The cost of x pieces of candy is $2.00. Use variable x to translate the above statements into algebraic equation. Our algebraic expression is: [B]0.25x = 2 [/B] To solve this equation for x, we [URL='https://www.mathcelebrity.com/1unk.php?num=0.25x%3D2&pl=Solve']type it in our search engine[/URL] and we get: x = [B]8[/B]

Each side of a square is lengthened by 3 inches . The area of this new, larger square is 25 square
Each side of a square is lengthened by 3 inches . The area of this new, larger square is 25 square inches. Find the length of a side of the original square. area of a square is s^2 New square has sides s + 3, so the area of 25 is: (s + 3)^2 = 25 [URL='https://www.mathcelebrity.com/1unk.php?num=%28s%2B3%29%5E2%3D25&pl=Solve']Solving for s[/URL], we get: s = [B]2[/B]

Each tree in an orchard containing 2,650 trees requires 210 grams of fertiliizer. At $1.25 per kilog
Each tree in an orchard containing 2,650 trees requires 210 grams of fertiliizer. At $1.25 per kilogram of fertilizer, how much does it cost to fertilize the orchard? [U]Calculate the total fertilizer needed:[/U] Total fertilizer needed = Number of trees * grams of fertilizer per tree Total fertilizer needed = 2650 * 210 [URL='https://www.mathcelebrity.com/longdiv.php?num1=2650&num2=210&pl=Multiply']Total fertilizer needed[/URL] = 556500 grams [U]1 kilogram = 1000 grams, so we convert our 556500 grams to kilograms:[/U] kilograms of fertilizer = grams of fertilizer / 1000 kilograms of fertilizer = 556500/1000 kilograms of fertilizer = 556.5 [U]Calculate fertilizer cost:[/U] Fertilizer cost = kilograms of fertilizer * cost per kilogram Fertilizer cost = 556.5 * 1.25 Fertilizer cost = [B]695.63[/B]

Each unit on a map of a forest represents 1 mile. To the nearest tenth of a mile, what is the distan
Each unit on a map of a forest represents 1 mile. To the nearest tenth of a mile, what is the distance from a ranger station at (1, 2) on the map to a river crossing at (2, 4) ? We use our 2 point calculator and we get a distance of 2.2361. Since each unit represents 1 mile, we have: 2.2361 units * 1 mile per unit = [B]2.2361 miles[/B]

Ed invests $5,500 into the stock market which earns 2% per year. In 20 years, how much will Ed's inv
Ed invests $5,500 into the stock market which earns 2% per year. In 20 years, how much will Ed's investment be worth if interest is compounded monthly? Round to the nearest dollar. 20 years * 12 months per year = 240 months Using our [URL='https://www.mathcelebrity.com/compoundint.php?bal=5550&nval=240&int=2&pl=Monthly']compound interest calculator[/URL], we get: [B]8,276.87[/B]

Edna plans to treat her boyfriend Curt to dinner for his birthday. The costs of their date options a
Edna plans to treat her boyfriend Curt to dinner for his birthday. The costs of their date options are listed next to each possible choice. Edna plans to allow Curt to choose whether they will eat Mexican food ($25), Chinese food ($15), or Italian food ($30). Next, they will go bowling ($20), go to the movies ($30) or go to a museum ($10). Edna also is deciding between a new wallet ($12) and a cell phone case ($20) as possible gift options for Curt. What is the maximum cost of this date? Edna has 3 phases of the date: [LIST=1] [*]Dinner [*]Event after dinner [*]Gift Option [/LIST] In order to calculate the maximum cost of the date, we take the maximum cost option of all 3 date phases: [LIST=1] [*]Dinner - Max price is Italian food at $30 [*]Event after dinner - Max price is movies at $30 [*]Gift Option - Max price option is the cell phone cast at $20 [/LIST] Add all those up, we get: $30 + $30 + $20 = [B]$80[/B]

Effective Annual Yield Rate
Free Effective Annual Yield Rate Calculator - Figures out the effective annual yield rate of interest entered by compounding daily, weekly, semi-monthly, monthly, quarterly, semi-annually, and continuously.

Eight gallons were poured into a gas tank that was 1/4 full. Now the tank is 3/4 full. How many gall
Eight gallons were poured into a gas tank that was 1/4 full. Now the tank is 3/4 full. How many gallons does a full tank hold? 3/4 - 1/4 = 2/4 8 gallons = 2/4 2/4 = 1/2 8 gallons = 1/2 Multiply by 2: 8 * 2 = 2/2 = [B]16 gallons[/B] = 1 full tank

Eight less then the sum of y and x
The sum of y and x is denoted as: x + y Eight less than that, using the number (8) for eight is: x + y - 8

eight oranges are $1.00 how much would 5 dozen oranges cost?
eight oranges are $1.00 how much would 5 dozen oranges cost? Set up a proportion of oranges to cost where c is the cost for 5 dozen = 60 oranges: 8/1 = 60/c To solve this proportion, [URL='https://www.mathcelebrity.com/prop.php?num1=8&num2=60&den1=1&den2=c&propsign=%3D&pl=Calculate+missing+proportion+value']we type it in our search engine[/URL] and we get: c = [B]7.5[/B]

Eight times the quantity y plus two divided by four
the quantity y plus two y + 2 the quantity y plus two divided by four (y +2)/4 Eight times the quantity y plus two divided by four 8(y +2)/4 8/4 = 2, so we have: [B]2(y +2) or 2y + 4 [MEDIA=youtube]xzwaXi6N1uI[/MEDIA][/B]

Eight workers dug 3/8 of a tunnel in 10 days. If they need to finish the remaining 5/8 of the tunnel
Eight workers dug 3/8 of a tunnel in 10 days. If they need to finish the remaining 5/8 of the tunnel in 3 & 1/3 days, how many more workers must they hire? Eight workers * 10 days = 80 days for one worker If 3/8 of a tunnel took 80 days for one worker, then each 1/8 of a tunnel takes a single worker: 80/3 = 26 & 2/3 days Multiply by 5 for the remaining 5/8 and we get: 133 & 1/3 We need 133 & 1/3 / 3 & 1/3 = 40 workers needed Additional workers = Workers Needed - Original Workers Additional workers = 40 - 8 Additional workers = [B]32[/B]

Eight-ninths of the population p
Eight-ninths of the population p [B]8p/9[/B]

Eighteen times the difference of a number and ten
The phrase [I]a number[/I] means an arbitrary variable. We can pick any letter a-z except for i and e. Let's choose x. The difference of a number and ten x - 10 Eighteen times the difference of a number and ten [B]18(x - 10)[/B]

Eighty percent of the employees at Rowan University have their biweekly Wages deposit directly to th
Eighty percent of the employees at Rowan University have their biweekly Wages deposit directly to their bank by electronic deposit program. Suppose we select a random samples of 8 employees. What is the probability that three of the eight (8) sampled employees use direct deposit program? Use the [I]binomial distribution[/I] [LIST] [*]p = 0.8 [*]n = 8 [*]k = 3 [/LIST] So we want P(X = 3) Using our [URL='http://www.mathcelebrity.com/binomial.php?n=+8&p=+0.8&k=+3&t=+5&pl=P%28X+=+k%29']binomial distribution calculator[/URL], we get P(X = 3) = [B]0.0092[/B]

Elapsed Time
Free Elapsed Time Calculator - This determines the elapsed time between two clock readings.

Electrons have a charge of -1. Protons have a charge of 1. The total charge of an atom is the sum of
Electrons have a charge of -1. Protons have a charge of 1. The total charge of an atom is the sum of its electron charges and proton charges. Find the total charge of an atom with 24 protons and 32 electrons We use sign operations to get: +24 - 32 = [B]-8[/B]

Ellen reads 23 pages in 40 minutes. Rob reads 9 pages in 16 minutes. Who is the faster reader? Ju
Ellen reads 23 pages in 40 minutes. Rob reads 9 pages in 16 minutes. Who is the faster reader? Justify your answer. Compare in terms of pages per minute. Ellen = 23 pages / 40 minutes =0.575 pages per minute Rob = 9 pages / 16 minutes = 0.5625 pages per minute [B]Ellen reads faster.[/B]

Elli is purchasing 2.75 pounds of jelly beans which are priced at 4.85 per pound. How much change sh
Elli is purchasing 2.75 pounds of jelly beans which are priced at 4.85 per pound. How much change should she get back from a 20? The total bill is 4.85 per pound * 2.75 pounds = 13.34 Her change from a 20 is 20 - 13.34 = [B]6.66[/B].

Ellipses
Free Ellipses Calculator - Given an ellipse equation, this calculates the x and y intercept, the foci points, and the length of the major and minor axes as well as the eccentricity.

Elsa took a total of 25 quizzes over the course of 5 weeks. After attending 8 weeks of school this q
Elsa took a total of 25 quizzes over the course of 5 weeks. After attending 8 weeks of school this quarter, how many quizzes will Elsa have taken in total? Assume the relationship is directly proportional. Set up a proportion of quizzes to weeks where q is the number of quizzes taken in 8 weeks. We have: 25/5 = q/8 We [URL='https://www.mathcelebrity.com/prop.php?num1=25&num2=q&den1=5&den2=8&propsign=%3D&pl=Calculate+missing+proportion+value']type this proportion into our search engine[/URL] and we get: q = [B]40[/B]

Emil bought a camera for $268.26, including tax. He made a down payment of $12.00 and paid the balan
Emil bought a camera for $268.26, including tax. He made a down payment of $12.00 and paid the balance in 6 equal monthly payments. What was Emil’s monthly payment for this camera? Calculate remaining balance 268.26 - 12 = 256.26 Determine monthly payment: 256.26/6 = [B]21.36[/B]

Emily buys a car for 9000 sells it for 12000. Whats the profit?
Emily buys a car for 9000 sells it for 12000. Whats the profit? Profit = Sale Price - Purchase Price Profit = 12,000 - 9,000 Profit = [B]3,000[/B]

Emily is three years older than twice her sister Mary’s age. The sum of their ages is less than 30.
Emily is three years older than twice her sister Mary’s age. The sum of their ages is less than 30. What is the greatest age Mary could be? Let e = Emily's age and m = Mary's age. We have the equation e = 2m + 3 and the inequality e + m < 30 Substitute the equation for e into the inequality: 2m + 3 + m < 30 Add the m terms 3m + 3 < 30 Subtract 3 from each side of the inequality 3m < 27 Divide each side of the inequality by 3 to isolate m m < 9 Therefore, the [B]greatest age[/B] Mary could be is 8, since less than 9 [U]does not include[/U] 9.

Equation 2y+5x=40. Interprt the intercepts
Equation 2y+5x=40. Interprt the intercepts Y intercept is when X = 0 2y + 5(0) = 40 2y = 40 Divide each side by 2 [B]y = 20 [/B] X intercept is when Y = 0 2(0) + 5x = 40 5x = 40 Divide each side by 5 [B]x = 8[/B]

Equation and Inequalities
Free Equation and Inequalities Calculator - Solves an equation or inequality with 1 unknown variable and no exponents as well as certain absolute value equations and inequalities such as |x|=c and |ax| = c where a and c are constants. Solves square root, cube root, and other root equations in the form ax^2=c, ax^2 + b = c. Also solves radical equations in the form asqrt(bx) = c. Also solves open sentences and it will solve one step problems and two step equations. 2 step equations and one step equations and multi step equations

Equation of a Plane
Free Equation of a Plane Calculator - Given three 3-dimensional points, this calculates the equation of a plane that contains those points.

Equation of Exchange
Free Equation of Exchange Calculator - Solves for any of the 4 variables in the Equation of Exchange: money, velocity, price, quantity

Equilateral Triangle
Free Equilateral Triangle Calculator - Given a side (a), this calculates the following items of the equilateral triangle:
* Perimeter (P)
* Semi-Perimeter (s)
* Area (A)
* altitudes (ha,hb,hc)
* medians (ma,mb,mc)
* angle bisectors (ta,tb,tc)
* Circumscribed Circle Radius (R)
* Inscribed Circle Radius (r)

Equivalent Annual Cost (EAC)
Free Equivalent Annual Cost (EAC) Calculator - Given 2 Items/machines with an Investment Cost, expected lifetime, and maintenance cost, this will calculate the EAC for each Item/machine as well as draw a conclusion on which project to invest in.

Eric is taking a trip of 245 miles. If he has traveled x miles, represent the remainder of the trip
Eric is taking a trip of 245 miles. If he has traveled x miles, represent the remainder of the trip in terms of x. Remaining distance = [B]245 - x[/B]

eric is twice as old as Shawn. The sum of their ages is 33. How old is Shawn?
eric is twice as old as Shawn. The sum of their ages is 33. How old is Shawn? Let Eric's age be e. Let Shawn's age be s. We're given two equations: [LIST=1] [*]e = 2s [*]e + s = 33 [/LIST] Substitute equation (1) into equation (2) for e so we can solve for s: 2s + s = 33 To solve for s, we [URL='https://www.mathcelebrity.com/1unk.php?num=2s%2Bs%3D33&pl=Solve']type this equation into our search engine[/URL] and we get: s = [B]11[/B]

Erica has $14 and plans to save $5 each week until she has the $64 she needs for a new jacket. Par
Erica has $14 and plans to save $5 each week until she has the $64 she needs for a new jacket. Part A: Write a number sentence describing this situation, using W to stand for the number of weeks Erica needs to save. [B]14 + 5w = 64[/B]

Erica is 5 feet tall and has a shadow of 2 feet. A nearby tree has a shadow of 18 feet. How tall is
Erica is 5 feet tall and has a shadow of 2 feet. A nearby tree has a shadow of 18 feet. How tall is the tree? Set up a proportion of feet tall to shadow height where n is the height of the tree 5/2 = n/18 Using our [URL='https://www.mathcelebrity.com/proportion-calculator.php?num1=5&num2=n&den1=2&den2=18&propsign=%3D&pl=Calculate+missing+proportion+value']proportion calculator[/URL], we get: n =[B]45 feet[/B]

Erik is rolling two regular six-sided number cubes. What is the probability that he will roll an eve
Erik is rolling two regular six-sided number cubes. What is the probability that he will roll an even number on one cube and a prime number on the other? P(Even on first cube) = (2,4,6) / 6 total choices P(Even on first cube) = 3/6 P(Even on first cube) = 1/2 <-- [URL='https://www.mathcelebrity.com/fraction.php?frac1=3%2F6&frac2=3%2F8&pl=Simplify']Using our fraction simplify calculator[/URL] P(Prime on second cube) = (2,3,5) / 6 total choices P(Prime on second cube) = 3/6 P(Prime on second cube) = 1/2 <-- [URL='https://www.mathcelebrity.com/fraction.php?frac1=3%2F6&frac2=3%2F8&pl=Simplify']Using our fraction simplify calculator[/URL] Since each event is independent, we have: P(Even on the first cube, Prime on the second cube) = P(Even on the first cube) * P(Prime on the second cube) P(Even on the first cube, Prime on the second cube) = 1/2 * 1/2 P(Even on the first cube, Prime on the second cube) = [B]1/4[/B]

Erin has 72 stamps in her stamp drawer along with a quarter, two dimes and seven pennies. She has 3
Erin has 72 stamps in her stamp drawer along with a quarter, two dimes and seven pennies. She has 3 times as many 3-cent stamps as 37-cent stamps and half the number of 5-cent stamps as 37-cent stamps. The value of the stamps and coins is $8.28. How many 37-cent stamps does Erin have? Number of stamps: [LIST] [*]Number of 37 cent stamps = s [*]Number of 3-cent stamps = 3s [*]Number of 5-cent stamps = 0.5s [/LIST] Value of stamps and coins: [LIST] [*]37 cent stamps = 0.37s [*]3-cent stamps = 3 * 0.03 = 0.09s [*]5-cent stamps = 0.5 * 0.05s = 0.025s [*]Quarter, 2 dime, 7 pennies = 0.52 [/LIST] Add them up: 0.37s + 0.09s + 0.025s + 0.52 = 8.28 Solve for [I]s[/I] in the equation 0.37s + 0.09s + 0.025s + 0.52 = 8.28 [SIZE=5][B]Step 1: Group the s terms on the left hand side:[/B][/SIZE] (0.37 + 0.09 + 0.025)s = 0.485s [SIZE=5][B]Step 2: Form modified equation[/B][/SIZE] 0.485s + 0.52 = + 8.28 [SIZE=5][B]Step 3: Group constants:[/B][/SIZE] We need to group our constants 0.52 and 8.28. To do that, we subtract 0.52 from both sides 0.485s + 0.52 - 0.52 = 8.28 - 0.52 [SIZE=5][B]Step 4: Cancel 0.52 on the left side:[/B][/SIZE] 0.485s = 7.76 [SIZE=5][B]Step 5: Divide each side of the equation by 0.485[/B][/SIZE] 0.485s/0.485 = 7.76/0.485 s = [B]16[/B] [URL='https://www.mathcelebrity.com/1unk.php?num=0.37s%2B0.09s%2B0.025s%2B0.52%3D8.28&pl=Solve']Source[/URL]

Estimate a 15% tip on a dinner bell of $49.76 by first round the bill amount to the nearest $10
Estimate a 15% tip on a dinner bell of $49.76 by first round the bill amount to the nearest $10. Round the bill to the nearest $10 [LIST] [*]$49.76 is in between ($40, $50) [*]1/2 of that interval is (40 + 50)/2 = 90/2 = 45 [*]Since $49.76 is greater than or equal to 45, we round up to $50 [/LIST] Add a 15% tip 50(1 + 0.15) = 50 + 7.50 = [B]$57.50[/B]

Estimate Square Roots
Free Estimate Square Roots Calculator - Estimates the square root of a number

Estimate Sums
Free Estimate Sums Calculator - Estimates the sum of 2 numbers.

Ethan has $9079 in his retirement account, and Kurt has $9259 in his. Ethan is adding $19per day, wh
Ethan has $9079 in his retirement account, and Kurt has $9259 in his. Ethan is adding $19per day, whereas Kurt is contributing $1 per day. Eventually, the two accounts will contain the same amount. What balance will each account have? How long will that take? Set up account equations A(d) where d is the number of days since time 0 for each account. Ethan A(d): 9079 + 19d Kurt A(d): 9259 + d The problems asks for when they are equal, and how much money they have in them. So set each account equation equal to each other: 9079 + 19d = 9259 + d [URL='https://www.mathcelebrity.com/1unk.php?num=9079%2B19d%3D9259%2Bd&pl=Solve']Typing this equation into our search engine[/URL], we get [B]d = 10[/B]. So in 10 days, both accounts will have equal amounts in them. Now, pick one of the account equations, either Ethan or Kurt, and plug in d = 10. Let's choose Kurt's since we have a simpler equation: A(10) = 9259 + 10 A(10) = $[B]9,269 [/B] After 10 days, both accounts have $9,269 in them.

Euclidean Geometry
Free Euclidean Geometry Calculator - Shows you the area, arc length, polar equation of the horizontal cardioid, and the polar equation of the vertical cardioid

Euclids Algorithm and Euclids Extended Algorithm
Free Euclids Algorithm and Euclids Extended Algorithm Calculator - Given 2 numbers a and b, this calculates the following
1) The Greatest Common Divisor (GCD) using Euclids Algorithm
2) x and y in Bézouts Identity ax + by = d using Euclids Extended Algorithm Extended Euclidean Algorithm

Eulers Formula for Planar Geometry
Free Eulers Formula for Planar Geometry Calculator - This calculator solves for any one of the 3 following items using Eulers Formula for planar geometry:
* Vertices (v)
* Faces (f)
* Edges (e)


Eva earns $72 washing 6 cars. At this rate, how many cars did Eva wash to earn $132?
Eva earns $72 washing 6 cars. At this rate, how many cars did Eva wash to earn $132? Set up a proportion of money to cars washed where c is the number of cars washed for $132 in earnings: 72/6 = 132/c [URL='https://www.mathcelebrity.com/prop.php?num1=72&num2=132&den1=6&den2=c&propsign=%3D&pl=Calculate+missing+proportion+value']Type this proportion into our calculator[/URL], we get: [B]c = 11[/B]

evaluate 16 raised to 1/4
evaluate 16 raised to 1/4 What number raised to the 4th power equals 16? [B]2[/B], since 2 * 2 * 2 * 2 = 16

Evaluate the expression (8C3) (7C6)
Evaluate the expression (8C3) (7C6) [URL='https://www.mathcelebrity.com/permutation.php?num=8&den=3&pl=Combinations']8C3[/URL] = 56 [URL='https://www.mathcelebrity.com/permutation.php?num=7&den=6&pl=Combinations']7C6[/URL] = 7 (8C3) (7C6) = 56 * 7 (8C3) (7C6) = [B]392[/B]

Evan scored 34 points in a basketball game. 21 of the points were from 3-point shots and the rest we
Evan scored 34 points in a basketball game. 21 of the points were from 3-point shots and the rest were free-throws. What expression shows the points scored from free-throws? Calculate the points from free throws (f): f = 34 - 21 f = [B]13[/B]

evelyn needs atleast $112 to buy a new dress. She has already saved $40 . She earns $9 an hour babys
evelyn needs atleast $112 to buy a new dress. She has already saved $40 . She earns $9 an hour babysitting. How many hours will she need to babysit to buy the dress? Let the number of hours be h. We have the earnings function E(h) below E(h) = hourly rate * h + current savings E(h) = 9h + 40 We're told E(h) = 112, so we have: 9h + 40 = 112 [URL='https://www.mathcelebrity.com/1unk.php?num=9h%2B40%3D112&pl=Solve']Typing this equation in our math engine[/URL] and we get: h = [B]8[/B]

Every 10 customers receive a soda. Every 9 customers receive a hot dog. There are 450 customers. How
Every 10 customers receive a soda. Every 9 customers receive a hot dog. There are 450 customers. How many received both a soda and hot dog? Fine the Least Common Multiple of 9 and 10 using our [URL='http://www.mathcelebrity.com/gcflcm.php?num1=9&num2=10&num3=&pl=LCM']LCM Calculator[/URL]: LCM(9, 10) = 90. So every 90 customers receives a soda [U][B]and[/B][/U] a hot dog. We have 90, 180, 270, 360, 450, so [B]5 customers[/B] receive both.

Every 100 seeds of corn he plants, he harvests 84 ears of corn. If he wants to harvest 7200 ears of
Every 100 seeds of corn he plants, he harvests 84 ears of corn. If he wants to harvest 7200 ears of corn, how many seeds must he plant? Set up a proportion seeds to ears: 100/84 = x/7200 where x is the number of seeds needed for 7200 ears of corn. Using our [URL='http://www.mathcelebrity.com/prop.php?num1=100&num2=x&den1=84&den2=7200&propsign=%3D&pl=Calculate+missing+proportion+value']proportion calculator[/URL], we get: [B]x = 8,571.43 ~ 8,572[/B]

Every 6 customers receive a soda, every 8 a hot dog there are 329 customers . how many received both
This is a least common multiple problem. [URL='http://www.mathcelebrity.com/gcflcm.php?num1=6&num2=8&num3=&pl=LCM']The least common multiple of 6 and 8 is 24[/URL] So every 24th person, less than or equal to 329 receives both a soda [U]and[/U] a hot dog. Using our multiples calculator, we find there are [URL='http://www.mathcelebrity.com/multiple.php?num=24&pl=Multiplication+Multiples']13 multiples of 24 less than or equal to 329[/URL]. 24,48,72,96,120,144,168,192,216,240,264,288,312

Expand Master and Build Polynomial Equations
Free Expand Master and Build Polynomial Equations Calculator - This calculator is the ultimate expansion tool to multiply polynomials. It expands algebraic expressions listed below using all 26 variables (a-z) as well as negative powers to handle polynomial multiplication. Includes multiple variable expressions as well as outside multipliers.
Also produces a polynomial equation from a given set of roots (polynomial zeros). * Binomial Expansions c(a + b)x
* Polynomial Expansions c(d + e + f)x
* FOIL Expansions (a + b)(c + d)
* Multiple Parentheses Multiplications c(a + b)(d + e)(f + g)(h + i)


Expanded Notation
Free Expanded Notation Calculator - Writes the expanded notation for a number.

Expected Frequency
Free Expected Frequency Calculator - Given a contingency table (two-way table), this will calculate expected frequencies and then determine a conclusion based on a Χ2 test with critical value test and conclusion.

Expected Value
Free Expected Value Calculator - This lesson walks you through what expected value is, expected value notation, the expected value of a discrete random variable, the expected value of a continuous random variable, and expected value properties.

Explain the relationship between "squaring" a number and finding the "square root" of a number. Use
Explain the relationship between "squaring" a number and finding the "square root" of a number. Use an example to further explain your answer. Squaring a number means raising it to the power of 2 The square root of a number [I]undoes[/I] a square of a number. So square root of x^2 is x x squared is x^2 Let x = 5. x squared = 5^2 = 25 Square root of 25 = square root of 5^2 = 5

Explain the steps you would take to find an equation for the line perpendicular to 4x - 5y = 20 and
Explain the steps you would take to find an equation for the line perpendicular to 4x - 5y = 20 and sharing the same y-intercept Get this in slope-intercept form by adding 5y to each side: 4x - 5y + 5y = 5y + 20 Cancel the 5y's on the left side and we get: 5y + 20 = 4x Subtract 20 from each side 5y + 20 - 20 = 4x - 20 Cancel the 20's on the left side and we get: 5y = 4x - 20 Divide each side by 5: 5y/5 = 4x/5 - 4 y = 4x/5 - 4 So we have a slope of 4/5 to find our y-intercept, we set x = 0: y = 4(0)/5 - 4 y = 0 - 4 y = -4 If we want a line perpendicular to the line above, our slope will be the negative reciprocal: The reciprocal of 4/5 is found by flipping the fraction making the numerator the denominator and the denominator the numerator: m = 5/4 Next, we multiply this by -1: -5/4 So our slope-intercept of the perpendicular line with the same y-intercept is: [B]y = -5x/4 - 4[/B]

Explain why 1/2 and 3/6 are equivalent
Explain why 1/2 and 3/6 are equivalent. Multiply any number by 1, and we get the same number. Multiply 1/2 by 3/3 which is 1 (1 * 3)/(2 *3) = 3/6

Exponential Distribution
Free Exponential Distribution Calculator - Calculates the Probability Density Function (PDF) and Cumulative Density Function (CDF) of the exponential distribution as well as the mean, variance, standard deviation, and entropy.

Exponential Growth
Free Exponential Growth Calculator - This solves for any 1 of the 4 items in the exponential growth equation or exponential decay equation, Initial Value (P), Ending Value (A), Rate (r), and Time (t).

Express cos4? and sin4? in terms of sines and cosines of multiples of ?
Express cos4? and sin4? in terms of sines and cosines of multiples of ?. Using a trignometric identity: cos (2?) = cos^2(?) - sin^2(?) Since 4? = 2*2?, so we have: [B]cos(4?) = cos^2(2?) - sin^2(2?)[/B] Using another trignometric identity, we have: sin(2?) = 2 sin(?) cos(?) Since 4? = 2*2?, so we have: [B]sin(4?) = 2 sin(2?) cos(2?)[/B]

Express the confidence interval 0.039 < p < 0.479 in the form of p ± E.
Express the confidence interval 0.039 < p < 0.479 in the form of p ± E. We find the range of this interval: Range = Upper Bound - Lower Bound Range = 0.479 - 0.039 Range = 0.44 Each piece on opposite sides of p gets: 0.44/2 = 0.22 So our expression becomes [B]p ± 0.22 [MEDIA=youtube]FGZcvcuWCpE[/MEDIA][/B]

Express the fact that x differs from 3 by less than 2/7 as an inequality involving absolute value. S
Express the fact that x differs from 3 by less than 2/7 as an inequality involving absolute value. Solve for x. Let's build this algebraic expression in pieces: The phrase [I]differs from[/I] means a difference. x - 3 By less than 2/7 means we use the < sign compared to 2/7 x - 3 < 2/7 Finally, the problem says we involve absolute value. So we write this as: [B]|x - 3| < 2/7[/B]

ey/n + k = t for y
ey/n + k = t for y Let's take this literal equation in pieces: Subtract k from each side: ey/n + k - k = t - k Cancel the k's on the left side, we have: ey/n = t - k Now multiply each side by n: ney/n = n(t - k) Cancel the n's on the left side, we have: ey = n(t - k) Divide each side by e: ey/e = n(t - k)/e Cancel the e's on the left side, we have: [B]y = n(t - k)/e[/B]

f - g = 1/4b for b
f - g = 1/4b for b Multiply each side of the equation by 4 to remove the 1/4 and isolate b: 4(f - g) = 4/4b 4/4 = 1, so we have: b = [B]4(f - g)[/B] [I]the key to this problem was multiplying by the reciprocal of the constant[/I]

f of X equals three to the x
f of X equals three to the x [B]f(x) = 3^x[/B]

F Test Statistic
Free F Test Statistic Calculator - Calculates the F-test statistic for two populations

F varies directly as g and inversely as r^2
F varies directly as g and inversely as r^2 [U]Givens and assumptions[/U] [LIST] [*]We take a constant of variation called k. [*][I]Varies directly means we multiply our variable term by k[/I] [*][I]Varies inversely means we divide k by our variable term[/I] [/LIST] The phrase varies directly or varies inversely means we have a constant k such that: [B]F = kg/r^2[/B]

f varies jointly with u and h and inversely with the square of y.
f varies jointly with u and h and inversely with the square of y. Variation means we have a constant k. Varies jointly with u and h means we multiply k by hu Varies inversely with the square of y means we divide by y^2 [B]f = khu/y^2[/B]

f(x) = 3x - 1; g(x) = 15 - 3*f(x) The functions f and g are defined above. What is the value of g(2)
f(x) = 3x - 1; g(x) = 15 - 3*f(x) The functions f and g are defined above. What is the value of g(2)? g(2) = 15 - 3 * f(2) f(2) = 3(2) - 1 f(2) = 6 - 1 f(2) = 5 Therefore, with f(2) = 5, g(2) is: g(2) = 15 - 3 * f(2) g(2) = 15 - 3 * 5 g(2) = 15 - 15 g(2) = [B]0[/B]

f(x)=a(b)^x and we know that f(3)=17 and f(7)=3156. what is the value of b
f(x)=a(b)^x and we know that f(3)=17 and f(7)=3156. what is the value of b Set up both equations with values When x = 3, f(3) = 17, so we have a(b)^3 = 17 When x = 7, f(7) = 3156, so we have a(b)^7 = 3156 Isolate a in each equation a = 17/(b)^3 a = 3156/(b)^7 Now set them equal to each other 17/(b)^3 = 3156/(b)^7 Cross Multiply 17b^7 = 3156b^3 Divide each side by b^3 17b^4 = 3156 Divide each side by 17 b^4 = 185.6471 [B]b = 3.6912[/B]

Facebook provides a variety of statistics on its Web site that detail the growth and popularity of t
Facebook provides a variety of statistics on its Web site that detail the growth and popularity of the site. On average, 28 percent of 18 to 34 year olds check their Facebook profiles before getting out of bed in the morning. Suppose this percentage follows a normal distribution with a standard deviation of five percent. a. Find the probability that the percent of 18 to 34-year-olds who check Facebook before getting out of bed in the morning is at least 30. b. Find the 95th percentile, and express it in a sentence. a. P(X >=0.30), calculate the [URL='http://www.mathcelebrity.com/probnormdist.php?xone=+0.30&mean=+0.28&stdev=+0.05&n=+1&pl=P%28X+%3E+Z%29']z-score[/URL] which is: Z = 0.4 P(x>0.4) = [B]0.344578 or 34.46%[/B] b. Inverse Normal (0.95) [URL='http://www.mathcelebrity.com/zcritical.php?a=0.95&pl=Calculate+Critical+Z+Value']calculator[/URL] = 1.644853627 Use NORMSINV(0.95) on Excel 0.28 + 0.05(1.644853627) = [B]0.362242681 or 36.22%[/B]

Factorials
Free Factorials Calculator - Calculates the following factorial items:
* A factorial of one number such as n!
* A factorial of a numerator divided by a factorial of a denominator such as n!m!/a!b!
* Double Factorials such as n!!
* Stirlings Approximation for n!

Factoring and Root Finding
Free Factoring and Root Finding Calculator - This calculator factors a binomial including all 26 variables (a-z) using the following factoring principles:
* Difference of Squares
* Sum of Cubes
* Difference of Cubes
* Binomial Expansions
* Quadratics
* Factor by Grouping
* Common Term
This calculator also uses the Rational Root Theorem (Rational Zero Theorem) to determine potential roots
* Factors and simplifies Rational Expressions of one fraction
* Determines the number of potential positive and negative roots using Descarte’s Rule of Signs

Factorization
Free Factorization Calculator - Given a positive integer, this calculates the following for that number:
1) Factor pairs and prime factorization and prime power decomposition
2) Factors and Proper Factors 3) Aliquot Sum

Factors of 36 between 2 and 12
Factors of 36 between 2 and 12 We type in [I][URL='https://www.mathcelebrity.com/factoriz.php?num=36&pl=Show+Factorization']factors of 36[/URL][/I] into our search engine and we get: {1, 2, 3, 4, 6, 9, 12, 18, 36} The problem asks for factors of 36 between 2 and 12: Between does not mean inclusive, so we have anything greater than 2 and less than 12: [B]{3, 4, 6, 9}[/B]

Faith is 1/5 her mother's age. Their combined ages are 30. How old is faith?
Faith is 1/5 her mother's age. Their combined ages are 30. How old is faith? Let Faith's age be f. Let her mother's age be m. We're given: [LIST=1] [*]f = m/5 [*]f + m = 30 [/LIST] Rearrange (1) by cross-multiplying: m = 5f Substitute this into equation (2): f + 5f = 30 [URL='https://www.mathcelebrity.com/1unk.php?num=f%2B5f%3D30&pl=Solve']Type this equation into our search engine[/URL] and we get: f = [B]5[/B]

Falling Object
Free Falling Object Calculator - Calculates any of the 3 items in the falling object formula, distance (s), acceleration (a), and time (t).

Fantasia decided to paint her circular room which had a diameter of 25 feet. She started painting in
Fantasia decided to paint her circular room which had a diameter of 25 feet. She started painting in the center and when she had painted a circle with a 5-foot diameter, she used one quart of paint. How many more quarts of paint must Fantasia buy to finish her room? The area formula for a circle is: Area = pir^2 Area of full room Radius = D/2 Radius = 25/2 Radius = 12.5 Area = 3.1415 * 12.5 * 12.5 Area = 490.625 Area of 5-foot diameter circle Radius = D/2 Radius = 5/2 Radius = 2.5 Area = 3.1415 * 2.5 * 2.5 Area = 19.625 So 1 quart of paint covers 19.625 square feet Area of unpainted room = Area of Room - Area of 5-foot diameter circle Area of unpainted room = 490.625 - 19.625 Area of unpainted room = 471 Calculate quarts of paint needed: Quarts of paint needed = Area of unpainted Room / square feet per quart of paint Quarts of paint needed = 471/19.625 Quarts of paint needed = [B]24 quarts[/B]

Farah rolls a fair dice and flips a fair coin. What is the probability of obtaining a 5 and a head?
Farah rolls a fair dice and flips a fair coin. What is the probability of obtaining a 5 and a head? Give your answer in its simplest form. Probability of a 5 is 1/6 Probability of a head is 1/2 Since each event is independent, we get the total probability by multiplying both together: P(5,H) = 1/6 * 1/2 P(5,H) = [B]1/12[/B]

Farmer Bob can get 10 gallons of milk from 4 cows. How many gallons of milk can he get from 14 cows?
Farmer Bob can get 10 gallons of milk from 4 cows. How many gallons of milk can he get from 14 cows? Set up a proportion of gallons to cows where g is the number of gallons per 14 cows: 10/4 = g/14 To solve this proportion for g, we[URL='https://www.mathcelebrity.com/prop.php?num1=10&num2=g&den1=4&den2=14&propsign=%3D&pl=Calculate+missing+proportion+value'] type it in our search engine[/URL] and we get: g = [B]35 [/B]

Farmer Yumi has too many plants in her garden. If she starts out with 150 plants and is losing them
Farmer Yumi has too many plants in her garden. If she starts out with 150 plants and is losing them at a rate of 4% each day, how long will it take for her to have 20 plants left? Round UP to the nearest day. We set up the function P(d) where d is the number of days sine she started losing plants: P(d) = Initial plants * (1 - Loss percent / 100)^d Plugging in our numbers, we get: 20 = 150 * (1 - 4/100)^d 20 = 150 * (1 - 0.04)^d Read left to right so it's easier to read: 150 * 0.96^d = 20 Divide each side by 150, and we get: 0.96^d = 0.13333333333 To solve this logarithmic equation for d, we [URL='https://www.mathcelebrity.com/natlog.php?num=0.96%5Ed%3D0.13333333333&pl=Calculate']type it in our search engine[/URL] and we get: d = 49.35 The problem tells us to round up, so we round up to [B]50 days[/B]

Fermats Little Theorem
Free Fermats Little Theorem Calculator - For any integer a and a prime number p, this demonstrates Fermats Little Theorem.

Fibonacci Retracement and Extension Levels
Free Fibonacci Retracement and Extension Levels Calculator - Shows the fibonacci retracement and extension levels

Fibonacci Sequence
Free Fibonacci Sequence Calculator - Generates a list of the first 100 Fibonacci numbers. Also shows how to generate the nth Fibonacci number using Binet's Formula

final cost for a hair cut is 22.00 the tax is 2.00 what is the tax percentage
final cost for a hair cut is 22.00 the tax is 2.00 what is the tax percentage Calculate Tax Percentage Tax Percentage = 100% * Tax / Total Bill Tax Percentage = 100% * 2/22 Tax Percentage = 100% * 1/11 Tax Percentage = [B]9.09%[/B]

Finance
1. Spend 8000 on a new machine. You think it will provide after tax cash inflows of 3500 per year for the next three years. The cost of funds is 8%. Find the NPV, IRR, and MIRR. Should you buy it? 2. Let the machine in number one be Machine A. An alternative is Machine B. It costs 8000 and will provide after tax cash inflows of 5000 per year for 2 years. It has the same risk as A. Should you buy A or B? 3. Spend 100000 on Machine C. You will need 5000 more in net working capital. C is three year MACRS. The cost of funds is 8% and the tax rate is 40%. C is expected to increase revenues by 45000 and costs by 7000 for each of the next three years. You think you can sell C for 10000 at the end of the three year period. a. Find the year zero cash flow. b. Find the depreciation for each year on the machine. c. Find the depreciation tax shield for the three operating years. d. What is the projects contribution to operations each year, ignoring depreciation effects? e. What is the cash flow effect of selling the machine? f. Find the total CF for each year. g. Should you buy it?

Finance
1) [URL='http://www.mathcelebrity.com/npv.php?matrix1=0%2C-8000%0D%0A1%2C3500%0D%0A2%2C3500%0D%0A3%2C3500%0D%0A&irr=8&pl=NPV']Net present value[/URL] = $1,019.85 [URL='http://www.mathcelebrity.com/npv.php?matrix1=0%2C-8000%0D%0A1%2C3500%0D%0A2%2C3500%0D%0A3%2C3500&irr=8&pl=IRR']IRR[/URL] = 14% I need a reinvestment rate from you for [URL='http://www.mathcelebrity.com/mirr.php']MIRR shown here[/URL] Yes, we should pursue the project since NPV > 0 2) [URL='http://www.mathcelebrity.com/npv.php?matrix1=0%2C-8000%0D%0A1%2C5000%0D%0A2%2C5000&irr=8&pl=NPV']Net present value[/URL] = $916.32 Buy A as it has the higher net present value.

Find 2 consecutive numbers such that the sum of twice the smaller number and 3 times the larger numb
Find 2 consecutive numbers such that the sum of twice the smaller number and 3 times the larger number is 73. Let x be the smaller number and y be the larger number. We are given: 2x + 3y = 73 Since the numbers are consecutive, we know that y = x + 1. Substitute this into our given equation: 2x + 3(x + 1) = 73 Multiply through: 2x + 3x + 3 = 73 Group like terms: 5x + 3 = 73 [URL='https://www.mathcelebrity.com/1unk.php?num=5x%2B3%3D73&pl=Solve']Type 5x + 3 = 73 into the search engine[/URL], and we get [B]x = 14[/B]. Our larger number is 14 + 1 = [B]15 [/B] Therefore, our consecutive numbers are[B] (14, 15)[/B]

Find 3 consecutive integers such that the sum of twice the smallest and 3 times the largest is 126
Find 3 consecutive integers such that the sum of twice the smallest and 3 times the largest is 126. Let the first integer be n, the second integer be n + 1, and the third integer be n + 2. We have: Sum of the smallest and 3 times the largest is 126: n + 3(n + 2) = 126 Multiply through: n + 3n + 6 = 126 Group like terms: 4n + 6 = 126 [URL='https://www.mathcelebrity.com/1unk.php?num=4n%2B6%3D126&pl=Solve']Type 4n + 6 = 126 into our calculator[/URL], we get n = 30. Which means the next two integers are 31 and 32. [B]{30, 31, 32}[/B]

Find 3 Even Integers with a sum of 198
Find 3 Even Integers with a sum of 198 Let x be the first even integer. Then y is the next, and z is the third even integer. [LIST=1] [*]y = x + 2 [*]z = x + 4 [*]x + y + z = 198 [/LIST] Substituting y and z into (3): x + x + 2 + x + 4 = 198 Group x terms 3x + 6 = 198 Using our [URL='http://www.mathcelebrity.com/1unk.php?num=3x%2B6%3D198&pl=Solve']equation solver[/URL], we get: [B]x = 64[/B] y = 64 + 2 [B]y= 66[/B] z = 64 + 4 [B]z = 68[/B]

Find a linear function f, given f(16)=-2 and f(-12)=-9. Then find f(0)
Find a linear function f, given f(16)=-2 and f(-12)=-9. Then find f(0). We've got 2 points: (16, -2) and (-12, -9) Calculate the slope (m) of this line using: m = (y2 - y1)/(x2 - x1) m = (-9 - -2)/(-12 - 16) m = -7/-28 m = 1/4 The line equation is denoted as: y = mx + b Let's use the first point (x, y) = (16, -2) -2 = 1/4(16) + b -2 = 4 + b Subtract 4 from each side, and we get: b = -6 So our equation of the line is: y = 1/4x - 6 The questions asks for f(0): y = 1/4(0) - 6 y = 0 - 6 [B]y = -6[/B]

Find an angle (theta) with 0<(theta)<360° or 0<(theta)<(2*pi) that has the same sine value as 80°
Find an angle (theta) with 0<(theta)<360° or 0<(theta)<(2*pi) that has the same sine value as 80°. The sine is positive in Quadrant I and Quadrant II. So we find the reference angle for 80°. It's 180 - 80 = [B]100°[/B]. This is our answer. Sin(80°) = Sin(100°)

Find an equation of the line containing the given pair of points (1,5) and (3,6)
Find an equation of the line containing the given pair of points (1,5) and (3,6). Using our[URL='https://www.mathcelebrity.com/slope.php?xone=1&yone=5&slope=+2%2F5&xtwo=3&ytwo=6&pl=You+entered+2+points'] point slope calculator[/URL], we get: [B]y = 1/2x + 9/2[/B]

Find four consecutive odd numbers which add to 64
Find four consecutive odd numbers which add to 64. Let the first number be x. The next three numbers are: x + 2 x + 4 x + 6 Add them together to get 64: x + (x + 2) + (x + 4) + (x + 6) = 64 Group like terms: 4x + 12 = 64 Using our [URL='http://www.mathcelebrity.com/1unk.php?num=4x%2B12%3D64&pl=Solve']equation calculator[/URL], we get: [B]x = 13[/B] The next 3 odd numbers are: x + 2 = 13 + 2 = 15 x + 4 = 13 + 4 = 17 x + 6 = 13 + 6 = 19 So the 4 consecutive odd numbers which add to 64 are: [B](13, 15, 17, 19)[/B]

Find Mean 106 and standard deviation 10 of the sample mean which is 25
mean of 106 inches and a standard deviation of 10 inches and for sample of size is 25. Determine the mean and the standard deviation of /x

Find Mean 106 and standard deviation 10 of the sample mean which is 25
Do you mean x bar? mean of 106 inches and a standard deviation of 10 inches and for sample of size is 25. Determine the mean and the standard deviation of /x If so, x bar equals the population mean. So it's [B]106[/B]. Sample standard deviation = Population standard deviation / square root of n 10/Sqrt(25) 10/5 [B]2[/B]

Find Mean and standard deviation
one trunk can carry 5068.8 lb. weight of boxes that it carries have a mean of 75lb and a standard deviation of 16 Ib. For Sample size of 64 ,find the mean and standard deviation of /x

Find Mean and standard deviation
one trunk can carry 5068.8 lb. weight of boxes that it carries have a mean of 75lb and a standard deviation of 16 Ib. For Sample size of 64 ,find the mean and standard deviation of /x

Find Necessary Sample Size
The monthly earnings of a group of business students are are normally distributed with a standard deviation of 545 dollars. A researcher wants to estimate the mean monthly earnings of all business students. Find the sample size needed to have a confidence level of 95% and a margin of error of 128 dollars.

Find r in P(7, r)
Find r in P(7, r) Recall the permutations formula: 7! / (7-r!) = 840. We [URL='https://www.mathcelebrity.com/factorial.php?num=7!&pl=Calculate+factorial']run 7! through our search engine[/URL] and we get: [URL='https://www.mathcelebrity.com/factorial.php?num=7!&pl=Calculate+factorial']7![/URL] = 5040 5040 / (7 - r)! = 840 Cross multiply, and we get: 5040/840 = 7 - r! 6 = (7 - r)! Since 6 = 3*2*! = 3!, we have; 3! = (7 - r)! 3 = 7 - r To solve for r, we [URL='https://www.mathcelebrity.com/1unk.php?num=3%3D7-r&pl=Solve']type this equation into our search engine[/URL] and we get: r = [B]4[/B]

Find Requested Confidence Value
The data below consists of the pulse rates (in beats per minute) of 32 students. Assuming ? = 10.66, obtain a 95.44% confidence interval for the population mean. 80 74 61 93 69 74 80 64 51 60 66 87 72 77 84 96 60 67 71 79 89 75 66 70 57 76 71 92 73 72 68 74

Find Requested Value
Physiologists often use the forced vital capacity as a way to assess a person's ability to move air in and out of their lungs. A researcher wishes to estimate the forced vital capacity of people suffering from asthma. A random sample of 15 asthmatics yields the following data on forced vital capacity, in liters. 5.2 4.9 2.9 5.3 3.0 4.0 5.2 5.2 3.2 4.7 3.2 3.5 4.8 4.0 5.1 Use the data to obtain a point estimate of the mean forced vital capacity for all asthmatics

Find the balance if $5000 is invested in an account paying 4.5% interest compounded continuously for
Find the balance if $5000 is invested in an account paying 4.5% interest compounded continuously for 21 years Using our [URL='https://www.mathcelebrity.com/simpint.php?av=&p=5000&int=4.5&t=21&pl=Continuous+Interest']continuous compounding interest calculator[/URL], we get: [B]$12,864.07[/B]

Find the confidence interval specified.
Physiologists often use the forced vital capacity as a way to assess a person's ability to move air in and out of their lungs. A researcher wishes to estimate the forced vital capacity of people suffering from asthma. A random sample of 15 asthmatics yields the following data on forced vital capacity, in liters. 5.1 4.9 4.7 3.1 4.3 3.7 3.7 4.3 3.5 5.2 3.2 3.5 4.8 4.0 5.1 Use the data to obtain a 95.44% confidence interval for the mean forced vital capacity for all asthmatics. Assume that ? = 0.7.

find the difference between a mountain with an altitude of 1,684 feet above sea level and a valley
find the difference between a mountain with an altitude of 1,684 feet above sea level and a valley 216 feet below sea level. Below sea level is the same as being on the opposite side of zero on the number line. To get the difference, we do the following: 1,684 - (-216) Since subtracting a negative is a positive, we have: 1,684 + 216 [B]1,900 feet[/B]

Find the distance between the points (10,7) and (6,10)
Find the distance between the points (10,7) and (6,10). [URL='https://www.mathcelebrity.com/slope.php?xone=10&yone=7&slope=+2%2F5&xtwo=6&ytwo=10&pl=You+entered+2+points']Using our two-points calculator[/URL], we get a distance of [B]5[/B].

Find the elements on the principal diagonal of matrix B
Find the elements on the principal diagonal of matrix B Matrix B: |0 0 8| |-1 3 0| |2 -5 -7| The main diagonal is any entry where row equals column |[B]0[/B] 0 8| |-1 [B]3 [/B] 0| |2 -5 [B]-7[/B]| In this case, it is [B]0, 3, -7[/B]

Find the explicit formula of the sequence 3,12,48
Find the explicit formula of the sequence 3,12,48 We [URL='https://www.mathcelebrity.com/sequenceag.php?num=3,12,48&n=10&pl=Calculate+Series']type in 3,12,48 into our search engine[/URL]. Choose series, and we get: [B]a(n) = 3 * 4^(n - 1)[/B]

Find the final amount of money in an account if $ 3,800 is deposited at 8% interest compounded annua
Find the final amount of money in an account if $ 3,800 is deposited at 8% interest compounded annually and the money is left for 6 years Using our [URL='https://www.mathcelebrity.com/compoundint.php?bal=3800&nval=6&int=8&pl=Annually']compound interest with balance calculator[/URL], we get: [B]$6,030.12[/B]

Find the future value and interest earned if $8806.54 is invested for 9 years at 6% compounded (a) s
Find the future value and interest earned if $8806.54 is invested for 9 years at 6% compounded (a) semiannually and (b) continuously a) 14,992.54 using our [URL='http://www.mathcelebrity.com/compoundint.php?bal=8806.54&nval=18&int=6&pl=Semi-Annually']balance with interest calculator[/URL] b) 15112.08 using our [URL='http://www.mathcelebrity.com/simpint.php?av=&p=8806.54&int=6&t=9&pl=Continuous+Interest']continuous interest balance calculator[/URL]

Find the gradient of the the line with the equation 8x - 4y =12
Find the gradient of the the line with the equation 8x - 4y =12 [URL='https://www.mathcelebrity.com/parperp.php?line1=8x-4y%3D12&line2=6x+-+3y+%3D+18&pl=Slope']Type this equation into our search engine[/URL] and choose "slope" and we get: Slope (gradient) = [B]2[/B]

Find the greatest common factor without a calculator
Find the greatest common factor without a calculator Check out this table stacking method and the product of factors. [MEDIA=youtube]3Zjo0XRD6fw[/MEDIA]

Find the greatest number which divides 845 and 1250
Find the greatest number which divides 845 and 1250 This is the greatest common factor. We [URL='https://www.mathcelebrity.com/gcflcm.php?num1=845&num2=1250&num3=&pl=GCF+and+LCM']type GCF(845,1250) into our search engine [/URL]and we get: [B]5[/B]

Find the largest of three consecutive even integers when six times the first integers is equal to fi
Find the largest of three consecutive even integers when six times the first integers is equal to five times the middle integer. Let the first of the 3 consecutive even integers be n. The second consecutive even integer is n + 2. The third (largest) consecutive even integer is n + 4. We are given 6n = 5(n + 2). Multiply through on the right side, and we get: 6n = 5n + 10 [URL='https://www.mathcelebrity.com/1unk.php?num=6n%3D5n%2B10&pl=Solve']Typing 6n = 5n + 10 into the search engine[/URL], we get n = 10. Remember, n was our smallest of 3 consecutive even integers. So the largest is: n + 4 10 + 4 [B]14[/B]

Find the last digit of 2 raised to the 2020 no calculator
Check out this pattern: 2^1= 2 2^2= 4 2^3 = 8 2^4= 16 2^5 = 32 2^6 = 64 2^7 = 128 2^8 = 256 The last digit repeats itself in blocks of 4 2, 4, 8, 6 We want to know what is the largest number in 1, 2, 3, 4 that divides 2020 without a remainder. LEt's start with 4 and work backwards. 2020/4 = 505 Ever power of 2^4(n) ends in 6, so our answer is [B]6 [MEDIA=youtube]6uX5gwb1jdY[/MEDIA][/B]

Find the last digit of 4^2081 no calculator
Find the last digit of 4^2081 no calculator 4^1= 4 4^2 = 16 4^3 = 64 4^4 = 256 4^5 = 1024 4^6 = 4096 Notice this pattern alternates between odd exponent powers with the result ending in 4 and even exponent powers with the result ending in 6. Since 2081 is odd, the answer is [B]4. [MEDIA=youtube]ueBWAW4XW4Q[/MEDIA][/B]

Find the last digit of 7^2013
Consider the first 8 calculations of 7 to an exponent: [LIST] [*]7^1 = 7 [*]7^2 = 49 [*]7^3 = 343 [*]7^4 = 2,401 [*]7^5 = 16,807 [*]7^6 = 117,649 [*]7^7 = 823,543 [*]7^8 = 5,764,801 [/LIST] Take a look at the last digit of the first 8 calculations: 7, 9, 3, 1, 7, 9, 3, 1 The 7, 9, 3, 1 repeats through infinity. So every factor of 4, the cycle of 7, 9, 3, 1 restarts. Counting backwards from 2013, we know that 2012 is the largest number divisible by 4: 7^2013 = 7^2012 * 7^1 The cycle starts over after 2012. Which means the last digit of 7^2013 = [B]7 [MEDIA=youtube]Z157jj8R7Yc[/MEDIA][/B]

Find the midpoint of the set of points (4,4) and (0,6)
Find the midpoint of the set of points (4,4) and (0,6) We [URL='https://www.mathcelebrity.com/slope.php?xone=4&yone=4&slope=+2%2F5&xtwo=0&ytwo=6&pl=You+entered+2+points']type in (4,4) and (0,6) into our search engine [/URL]and we get: Midpoint = [B](2, 5)[/B]

Find the number of combinations and the number of permutations for 10 objects taken 6 at a time
Find the number of combinations and the number of permutations for 10 objects taken 6 at a time [LIST] [*]Combinations is written as 10 C 6. Using our [URL='http://www.mathcelebrity.com/permutation.php?num=10&den=6&pl=Combinations']combinations calculator[/URL], we get [B]210[/B]. [*]Permutations is written as 10 P 6. Using our [URL='http://www.mathcelebrity.com/permutation.php?num=10&den=6&pl=Permutations']permutations calculator[/URL], we get [B]151,200[/B]. [/LIST]

Find the odd number less than 100 that is divisible by 9, and when divided by 10 has a remainder of
Find the odd number less than 100 that is divisible by 9, and when divided by 10 has a remainder of 7. From our [URL='http://www.mathcelebrity.com/divisibility.php?num=120&pl=Divisibility']divisibility calculator[/URL], we see a number is divisible by 9 if the sum of its digits is divisible by 9. Starting from 1 to 99, we find all numbers with a digit sum of 9. This would be digits with 0 and 9, 1 and 8, 2 and 7, 3 and 6, and 4 and 5. 9 18 27 36 45 54 63 72 81 90 Now remove even numbers since the problem asks for odd numbers 9 27 45 63 81 Now, divide each number by 10, and find the remainder 9/10 = 0 [URL='http://www.mathcelebrity.com/modulus.php?num=27mod10&pl=Calculate+Modulus']27/10[/URL] = 2 R 7 We stop here. [B]27[/B] is an odd number, less than 100, with a remainder of 7 when divided by 10.

find the probability of drawing a 4 or an ace
find the probability of drawing a 4 or an ace. In a 52 card deck, there are 4 (4's) and 4 (Aces), for a total of 8 cards: The probability is 8 cards / 52 total cards: 8/52 using our s[URL='https://www.mathcelebrity.com/fraction.php?frac1=8%2F52&frac2=3%2F8&pl=Simplify']implify fractions calculator[/URL] = [B]2/13[/B]

Find the subset of {a,b,c,d,e}
Find the subset of {a,b,c,d,e} Using our power set calculator, we find [URL='https://www.mathcelebrity.com/powerset.php?num=a%2Cb%2Cc%2Cd%2Ce&pl=Show+Power+Set+and+Partitions']all the 32 subsets of {a,b,c,d,e}[/URL]

Find the total coast of four nights lodging at $62.00 per night with 8 1/2% sales tax.
Find the total coast of four nights lodging at $62.00 per night with 8 1/2% sales tax. [U]Calculate Total lodging cost[/U] Total lodging cost = Nightly Rate * Number of Nights Total lodging cost = 62 * 4 Total lodging cost = 248 [U]Calculate total bill with tax[/U] Total bill with tax = Total bill * (1 + sales tax percent) Total bill with tax = 248 * (1 + 0.85) <-- 8 1/2% = 0.085 as a decimal Total bill with tax = 248 * 1.085 Total bill with tax =[B] $269.08[/B]

find the two square roots of 81
find the two square roots of 81 When we multiply 9 * 9, we get 81 When we multiply -9 * -9, we get 81 So our two square roots of 81 are: [LIST] [*][B]-9, 9[/B] [/LIST]

find the value of $20000 invested for 7 years at an annual interest rate of 2.55% compounded continu
find the value of $20000 invested for 7 years at an annual interest rate of 2.55% compounded continuously Using our [URL='https://www.mathcelebrity.com/simpint.php?av=&p=200000&int=2.55&t=7&pl=Continuous+Interest']compound continuous interest with balance calculator[/URL] we get: [B]239.084.58[/B]

find the value of ? that corresponds to a confidence level of 86.5
find the value of ? that corresponds to a confidence level of 86.5

Find the velocity of a cheetah that runs 100m in 4 seconds
Find the velocity of a cheetah that runs 100m in 4 seconds 100m / 4 seconds Divide top and bottom by 4 [B]25m/second[/B]

Find the volume of the box. The box shows the length is 6 feet, the width is 4 feet, and the height
Find the volume of the box. The box shows the length is 6 feet, the width is 4 feet, and the height is 3 feet. The shape is a rectangular solid. The Volume (V) is shown below: V = lwh V = 6 * 4 * 3 V = [B]72 cubic feet[/B]

Find two consecutive integers if the sum of their squares is 1513
Find two consecutive integers if the sum of their squares is 1513 Let the first integer be n. The next consecutive integer is (n + 1). The sum of their squares is: n^2 + (n + 1)^2 = 1513 n^2 + n^2 + 2n + 1 = 1513 2n^2 + 2n + 1 = 1513 Subtract 1513 from each side: 2n^2 + 2n - 1512 = 0 We have a quadratic equation. We [URL='https://www.mathcelebrity.com/quadratic.php?num=2n%5E2%2B2n-1512%3D0&pl=Solve+Quadratic+Equation&hintnum=+0']type this into our search engine[/URL] and get: n = (-27, 28) Let's take the positive solution. The second integer is: n + 1 28 + 1 = 29

Find two consecutive odd integers such that the sum of their squares is 290
Find two consecutive odd integers such that the sum of their squares is 290. Let the first odd integer be n. The next odd integer is n + 2 Square them both: n^2 (n + 2)^2 = n^2 + 4n + 4 from our [URL='https://www.mathcelebrity.com/expand.php?term1=%28n%2B2%29%5E2&pl=Expand']expansion calculator[/URL] The sum of the squares equals 290 n^2 + n^2 + 4n + 4 = 290 Group like terms: 2n^2 + 4n + 4 = 290 [URL='https://www.mathcelebrity.com/quadratic.php?num=2n%5E2%2B4n%2B4%3D290&pl=Solve+Quadratic+Equation&hintnum=+0']Enter this quadratic into our search engine[/URL], and we get: n = 11, n = -13 Which means the two consecutive odd integer are: 11 and 11 + 2 = 13. [B](11, 13)[/B] -13 and -13 + 2 = -11 [B](-13, -11)[/B]

Find two consecutive positive integers such that the difference of their square is 25
Find two consecutive positive integers such that the difference of their square is 25. Let the first integer be n. This means the next integer is (n + 1). Square n: n^2 Square the next consecutive integer: (n + 1)^2 = n^2 + 2n + 1 Now, we take the difference of their squares and set it equal to 25: (n^2 + 2n + 1) - n^2 = 25 Cancelling the n^2, we get: 2n + 1 = 25 [URL='https://www.mathcelebrity.com/1unk.php?num=2n%2B1%3D25&pl=Solve']Typing this equation into our search engine[/URL], we get: n = [B]12[/B]

Find two consecutive positive integers such that the sum of their squares is 25
Find two consecutive positive integers such that the sum of their squares is 25. Let the first integer be x. The next consecutive positive integer is x + 1. The sum of their squares equals 25. We write this as:: x^2 + (x + 1)^2 Expanding, we get: x^2 + x^2 + 2x + 1 = 25 Group like terms: 2x^2 + 2x + 1 = 25 Subtract 25 from each side: 2x^2 + 2x - 24 = 0 Simplify by dividing each side by 2: x^2 + x - 12 = 0 Using our [URL='http://www.mathcelebrity.com/quadratic.php?num=x%5E2%2Bx-12%3D0&pl=Solve+Quadratic+Equation&hintnum=+0']quadratic calculator[/URL], we get x = 3 or x = -4. The problem asks for positive integers, so we discard -4, and use 3. This means, our next positive integer is 3 + 1 = 4. So we have [B](3, 4) [/B]as our answers. Let's check our work: 3^2 + 4^2 = 9 + 16 = 25

Find two numbers word problems
Free Find two numbers word problems Calculator - Given two numbers with a sum of s where one number is n greater than another, this calculator determines both numbers.

Find x
Find x [IMG]https://mathcelebrity.com/community/data/attachments/0/cong-angles.jpg[/IMG] Since both angles are congruent, we set them equal to each other: 6x - 20 = 4x To solve for x, we [URL='https://www.mathcelebrity.com/1unk.php?num=6x-20%3D4x&pl=Solve']type this equation into our math engine[/URL] and we get: x = [B]10[/B]

Find y if the line through (1, y) and (2, 7) has a slope of 4.
Find y if the line through (1, y) and (2, 7) has a slope of 4. Given two points (x1, y1) and (x2, y2), Slope formula is: slope = (y2 - y1)/(x2 - x1) Plugging in our coordinates and slope to this formula, we get: (7 - y)/(2 - 1) = 4 7 - y/1 = 4 7 - y = 4 To solve this equation for y, w[URL='https://www.mathcelebrity.com/1unk.php?num=7-y%3D4&pl=Solve']e type it in our search engine[/URL] and we get: y = [B]3[/B]

Find y if the line through (1,y) and (4,5) has a slope of 3
Find y if the line through (1,y) and (4,5) has a slope of 3. Slope formula is: m = (y2 - y1)/(x2 - x1) With m = 3, we have: 3 = (5 - y)/(4 - 1) 3 = (5 - y)/3 Cross multiply: 5 - y = 9 Subtract 5 from each side -y = 4 Multiply each side by -1 [B]y = -4[/B]

Finding a 20% tip no calculator
Finding a 20% tip no calculator We have 2 methods to calculate a 20% tip. [LIST=1] [*]Divide by 5 [*]Shift one decimal place left and take the value. Multiply by 2 [/LIST] Example: 180 tip, find a 20% tip: Method 1: 180/5 = 36 Method 2: Move decimal place left = 18 Multiply this value by 2: 18 * 2 = 36 [MEDIA=youtube]UW4GYWfMhsE[/MEDIA]

Finite Field
Free Finite Field Calculator - Demonstrates the addition table and multiplication table for a finite field (Galois Field) of n denoted GF(n).

Finn has 8 toy cars. Dirk has t times as many toy cars as Finn
Finn has 8 toy cars. Dirk has t times as many toy cars as Finn The phrase [I]times as many [/I]means we multiply: [B]8t[/B]

Fiona thinks of a number. fiona halves the number and gets an answer of 72.8. Form an equation with
Fiona thinks of a number. fiona halves the number and gets an answer of 72.8. Form an equation with x from the information Halving means dividing by 2, so our equation is: [B]x/2 = 72.8[/B]

First four exams scores were 78%, 76%, 82% and 84%. What is needed on the final exam to receive a 90
First four exams scores were 78%, 76%, 82% and 84%. What is needed on the final exam to receive a 90% exam average? We need a missing average. [URL='https://www.mathcelebrity.com/missingaverage.php?num=78%2C+76%2C+82%2C84&avg=90&pl=Calculate+Missing+Score']Using our missing average calculator with our 4 test scores and a target average of 90%[/URL], we get: [B]130%[/B]

Fisher Transformation and Fisher Inverse
Free Fisher Transformation and Fisher Inverse Calculator - Given a correlation coefficient (r), this calculates the Fisher Transformation (z).
Given a Fisher Transformation (r), this calculates the Fisher Inverse (r)

Fishers Exact Test
Free Fishers Exact Test Calculator - Given a, b, c, and d, this calculates the probability of any such set of values using Fishers exact Test

Five players are going to be picked to start a basketball game. If there are 13 players on the team,
Five players are going to be picked to start a basketball game. If there are 13 players on the team, how many different combinations of 5 starting players can be made? We want 13 combinations of 5 [URL='https://www.mathcelebrity.com/permutation.php?num=13&den=5&pl=Combinations']13C5[/URL] = [B]1287[/B]

Five sets of ear buds cost $53.15. How much will 9 sets of ear buds cost?
Five sets of ear buds cost $53.15. How much will 9 sets of ear buds cost? Set up a proportion of earbuds to cost where c is the cost of 9 ear buds: 5/53.15 = 9/c To solve this proportion for c, we [URL='https://www.mathcelebrity.com/prop.php?num1=5&num2=9&den1=53.15&den2=c&propsign=%3D&pl=Calculate+missing+proportion+value']type it in our math engine[/URL] and we get: c = [B]95.67[/B]

Five times Kim's age plus 13 equals 58. How old is Kim?
Five times Kim's age plus 13 equals 58. How old is Kim? Let Kim's age be a. We have: Five times Kim's age: 5a Plus 13 means we add 13 5a + 13 Equals 58 means we set the expression 5a + 13 equal to 58 5a + 13 = 58 <-- This is our algebraic expression To solve this equation for a, [URL='https://www.mathcelebrity.com/1unk.php?num=5a%2B13%3D58&pl=Solve']we type it into our search engine[/URL] and get: a = [B]9[/B]

Fixed cost 500 marginal cost 8 item sells for 30
fixed cost 500 marginal cost 8 item sells for 30. Set up Cost Function C(x) where x is the number of items sold: C(x) = Marginal Cost * x + Fixed Cost C(x) = 8x + 500 Set up Revenue Function R(x) where x is the number of items sold: R(x) = Revenue per item * items sold R(x) = 30x Set up break even function (Cost Equals Revenue) C(x) = R(x) 8x + 500 = 30x Subtract 8x from each side: 22x = 500 Divide each side by 22: x = 22.727272 ~ 23 units for breakeven

Flight is $295 and car rental is $39 a day, if a competition charges $320 and $33 a day car rental,
Flight is $295 and car rental is $39 a day, if a competition charges $320 and $33 a day car rental, which is cheaper? Set up cost function where d is the number of days: [LIST] [*]Control business: C(d) = 39d + 295 [*]Competitor business: C(d) = 33d + 320 [/LIST] Set the [URL='http://www.mathcelebrity.com/1unk.php?num=39d%2B295%3D33d%2B320&pl=Solve']cost functions equal to each other[/URL]: We get d = 4.1667. The next integer day up is 5. Now plug in d = 1, 2, 3, 4. For the first 4 days, the control business is cheaper. However, starting at day 5, the competitor business is now cheaper forever.

flip 7 coins How many total outcomes are there
flip 7 coins How many total outcomes are there A flip of a coin has 2 outcomes, heads or tails. Since each outcome is independent of the other outcomes, we multiply each flip by 2 outcomes: Total outcomes = 2 * 2 * 2 * 2 * 2 * 2 * 2 Total outcomes = 2^7 Total outcomes = [B]128[/B]

Floor
Free Floor Calculator - Determines the floor of a number

Following the birth of triplets, the grandparents deposit $30,000 in a college trust fund that earns
Following the birth of triplets, the grandparents deposit $30,000 in a college trust fund that earns 4.5% interest, compounded quarterly. How much will be in the account after 18 years? 18 years = 18 * 4 = 72 quarters. Using our [URL='http://www.mathcelebrity.com/compoundint.php?bal=30000&nval=72&int=4.5&pl=Quarterly']compound interest balance calculator[/URL], we have: [B]$67,132.95[/B]

For a population with ? = 60 and ? = 12, what is the z-score that corresponds to a score of 66?
For a population with ? = 60 and ? = 12, what is the z-score that corresponds to a score of 66? [URL='https://www.mathcelebrity.com/probnormdist.php?xone=66&mean=60&stdev=12&n=1&pl=P%28X+%3C+Z%29']Using our z-score calculator[/URL], we get a probability: [B]0.691462[/B]

for every 10 white cars a dealer sells he sells 7 silver, 6 blue, 5 red, 4 yellow, 3 green, 2 black,
for every 10 white cars a dealer sells he sells 7 silver, 6 blue, 5 red, 4 yellow, 3 green, 2 black, 2 purple and 1 brown car. If he sells 120 cars how many blue cars? [U]Take this in blocks, so each block has:[/U] 10 white + 7 silver + 6 blue + 5 red + 4 yellow + 3 green + 2 black + 2 purple + 1 brown = 40 cars [U]Calculate the number of blocks:[/U] 120 cars / 40 cars = 3 blocks. [U]For 120 cars sold, it takes 3 blocks, which means we multiply:[/U] 6 blue cars per block * 3 blocks = [B]18 blue cars[/B]

For g(x) = 4-5x, determine the input for x when the output of g(x) is -6
For g(x) = 4-5x, determine the input for x when the output of g(x) is -6 We want to know when: 4 - 5x = 6 To solve this equation for x, we [URL='https://www.mathcelebrity.com/1unk.php?num=4-5x%3D6&pl=Solve']type it in our search engine[/URL] and we get: x = [B]-0.4 or -2/5[/B]

For her phone service, Maya pays a monthly fee of $27 , and she pays an additional $0.04 per minu
For her phone service, Maya pays a monthly fee of $27 , and she pays an additional $0.04 per minute of use. The least she has been charged in a month is $86.04 . What are the possible numbers of minutes she has used her phone in a month? Use m for the number of minutes, and solve your inequality for m . Maya's cost function is C(m), where m is the number of minutes used. C(m) = 0.04m + 27 We are given C(m) = $86.04. We want her cost function [I]less than or equal[/I] to this. 0.04m + 27 <= 86.04 [URL='https://www.mathcelebrity.com/1unk.php?num=0.04m%2B27%3C%3D86.04&pl=Solve']Type this inequality into our search engine[/URL], and we get [B]m <= 1476[/B].

For the first 10 seconds of the ride, the height of the coaster can be determined by h(t) = 0.3t^3 -
For the first 10 seconds of the ride, the height of the coaster can be determined by h(t) = 0.3t^3 - 5t^2 + 21t, where t is the time in seconds and h is the height in feet. classify this polynomial by degree and by number of terms. [URL='http://www.mathcelebrity.com/polynomial.php?num=0.3t%5E3-5t%5E2%2B21t&pl=Evaluate']Using our polynomial calculator, we determine[/URL]: [LIST] [*]The degree of the polynomial is 3 [*]There are 3 terms [/LIST]

for the function, h(x) = bx - 22, b is a constant and h(-5) = -7. Find the value of h(5)
for the function, h(x) = bx - 22, b is a constant and h(-5) = -7. Find the value of h(5) h(-5) = -5b - 22 Since we're given h(-5) = -7, we have: -5b - 22 = -7 [URL='https://www.mathcelebrity.com/1unk.php?num=-5b-22%3D-7&pl=Solve']Typing this equation into our search engine[/URL], we get: b = -3 So our h(x) equation is now: h(x) = -3x - 22 The problem asks for h(5): h(5) = -3(5) - 22 h(5) = 15 - 22 h(5) = [B]-37[/B]

For the normal distribution with parameters ? = 4, ? = 3 ; calculate P(x > 1)
For the normal distribution with parameters ? = 4, ? = 3 ; calculate P(x > 1) [URL='https://www.mathcelebrity.com/probnormdist.php?xone=1&mean=4&stdev=3&n=1&pl=P%28X+%3E+Z%29']Using our calculator[/URL], we get P(x > 1) = [B]0.841345[/B]

Forward Rate
Free Forward Rate Calculator - Given two times and two zero-coupon yield rates at those times, this calculates the forward rate.

Foster is centering a photo that is 9/1/2 inches wide on a scrapbook pages that is 10 inches wide. H
Foster is centering a photo that is 9/1/2 inches wide on a scrapbook pages that is 10 inches wide. How far from each side of the pages should he put the picture? Enter your answer as a mixed number. First, determine your margins, which is the difference between the width and photo width, divided by 2. 10 - 9 & 1/2 = 1/2 1/2 / 2 = [B]1/4[/B]

Four children can eat a large pizza in 12 minutes. How long would it take 9 children to eat the same
Four children can eat a large pizza in 12 minutes. How long would it take 9 children to eat the same pizza? (Give your answer in minutes and seconds.) One child can eat the pizza in 4 * 12 minutes = 48 minutes 48 minutes per child / 9 children = 5.3333 minutes 1/3 of a minute = 20 seconds, so we have: [B]5 minutes and 20 seconds[/B]

Four coins are flipped. What is the probability of the coins all landing on heads
Four coins are flipped. What is the probability of the coins all landing on heads The probability of one head is 1/2. Since all 4 flips are independent, we multiply each flip probability: P(HHHH) = 1/2 * 1/2 * 1/2 * 1/2 P(HHHH) = [B]1/16[/B]

Four consecutive integers beginning with n
Four consecutive integers beginning with n consecutive meaning one after another. So we have: [LIST] [*][B]n[/B] [*][B]n + 1[/B] [*][B]n + 2[/B] [*][B]n + 3[/B] [/LIST]

Four cousins were born at two-year intervals. The sum of their ages is 36. What are their ages?
Four cousins were born at two-year intervals. The sum of their ages is 36. What are their ages? So the last cousin is n years old. this means consecutive cousins are n + 2 years older than the next. whether their ages are even or odd, we have the sum of 4 consecutive (odd|even) integers equal to 36. We [URL='https://www.mathcelebrity.com/sum-of-consecutive-numbers.php?num=sumof4consecutiveevenintegersis36&pl=Calculate']type this into our search engine[/URL] and we get the ages of: [B]6, 8, 10, 12[/B]

Four less than five times a number
The phrase [I]a number[/I] means an arbitrary variable. We can pick any letter a-z except for i and e. Let's choose x. 5 times a number: 5x Four less means we subtract 4 from 5x: [B]5x - 4[/B]

Four more then double a number is greater than 2
Four more then double a number is greater than 2 Double a number: A number implies an arbitrary variable, let's call it "x". Double means multiply this by 2 2x Four more than this: 2x + 4 Now, we set this expression as an inequality greater than 2 [B]2x + 4 > 2[/B]

Four times the quantity six plus two
six plus two: 6 + 2 Four times the quantity six plus two [B]4(6 + 2) [/B]<-- This is our algebraic expression If we need to evaluate this, we have: 4(8) [B]32[/B]

Four-fifths of Kayla’s Math Notebook is filled. She has written on 48 pages. How many pages is there
Four-fifths of Kayla’s Math Notebook is filled. She has written on 48 pages. How many pages is there total in the notebook? Let the total pages be p. WE're given: 4p/5 = 48 To solve for p, we[URL='https://www.mathcelebrity.com/prop.php?num1=4p&num2=48&den1=5&den2=1&propsign=%3D&pl=Calculate+missing+proportion+value'] type this equation into our search engine[/URL] and we get: p = [B]60[/B]

Fraction Cancellation Property
Free Fraction Cancellation Property Calculator - Demonstrates the Fraction Cancellation Property also known as Cancellation Property of Fractions Numerical Properties

Fraction with variable x in numerator and 6 in the denominator.
Fraction with variable x in numerator and 6 in the denominator. The numerator is the top of the fraction. The denominator is the bottom of the fraction. [B]x/6[/B]

Fractions and Mixed Numbers
Free Fractions and Mixed Numbers Calculator - Given (improper fractions, proper fraction, mixed numbers, or whole numbers), this performs the following operations:
* Addition (Adding)
* Subtraction (Subtracting)
* Positive Difference (Absolute Value of the Difference)
* Multiplication (Multiplying)
* Division (Dividing: complex fraction division is included)
* Compare Fractions
* Simplifying of proper and improper fractions as well as mixed numbers. Fractions will be reduced down as far as possible (Reducing Fractions).
* Reciprocal of a Fraction
* Find all fractions between two fractions
* reduce a fraction

Francis paid 51.12 for his dinner including tax. The cost of his dinner is 48. What percent is the t
Francis paid 51.12 for his dinner including tax. The cost of his dinner is 48. What percent is the tax? Answer: [B]6.5%[/B] using our [URL='http://www.mathcelebrity.com/tax.php?p=48&tb=51.12&pl=Calculate+Tax']sales tax calculator[/URL].

Frank is a plumber who charges a $35 service charge and $15 per hour for his plumbing services. Find
Frank is a plumber who charges a $35 service charge and $15 per hour for his plumbing services. Find a linear function that expresses the total cost C for plumbing services for h hours. Cost functions include a flat rate and a variable rate. The flat rate is $35 and the variable rate per hour is 15. The cost function C(h) where h is the number of hours Frank works is: [B]C(h) = 15h + 35[/B]

Fred had 156 dollars to spend on 9 books. After buying them he had 12 dollars. How much did each boo
Fred had 156 dollars to spend on 9 books. After buying them he had 12 dollars. How much did each book cost? Subtract the 12 dollars left over from the $156 starting amount: $156 - $12 = $144 Now divide $144 / 9 books to get the cost per book: $144/9 = [B]$16 per book[/B]

Frequency and Wavelength and Photon Energy
Free Frequency and Wavelength and Photon Energy Calculator - Provides the following 3 items using the speed of light and Plancks constant (h):
- Given a frequency of centimeters, feet, meters, or miles the calculator will determine wavelength in Hz, KHz, MHz, GHz
- Given a wavelength of Hz, KHz, MHz, GHz, the calculator will determine frequency in centimeters, feet, meters, or miles
- Calculates photon energy

Frequency Conversions
Free Frequency Conversions Calculator - Converts between the following frequencies:
hertz
millihertz
kilohertz
megahertz
gigahertz
terahertz
rps
rpm
rph

Frequency Distribution Table
Free Frequency Distribution Table Calculator - Determines the classes and frequency distribution using the 2 to k rule.

From 9 names on a ballot, a committee of 4 will be elected to attend a political national convention
From 9 names on a ballot, a committee of 4 will be elected to attend a political national convention. How many different committees are possible? A. 3024 B. 15,120 C. 1512 D. 126 We want unique combinations, so we have 9 choose 4, or 9C4. [URL='https://www.mathcelebrity.com/permutation.php?num=9&den=4&pl=Combinations']Typing this into the search engine[/URL], we get: 9C4 = [B]126 different committees or Answer D [MEDIA=youtube]Pq2YXQn38wY[/MEDIA][/B]

From a group of 10 men and 8 women, how many ways can 2 men and 3 women be chosen for 5 positions
From a group of 10 men and 8 women, how many ways can 2 men and 3 women be chosen for 5 positions? We use combinations. Since men and women are independent, we multiply each result: We want 10 men choose 2 men multiplied by 8 women choose 3 women. [URL='https://www.mathcelebrity.com/permutation.php?num=10&den=2&pl=Combinations']Type 10C2 into our search engine[/URL] and we get 45 [URL='https://www.mathcelebrity.com/permutation.php?num=8&den=3&pl=Combinations']Type 8C3 into our search engine[/URL] and we get 56 Multiply both together: 45 * 56 = [B]2,520 ways[/B]

From a regular deck of 52 playing cards, you turn over a 6 and then a 7. What is the probability tha
From a regular deck of 52 playing cards, you turn over a 6 and then a 7. What is the probability that the next card you turn over will be a face card? Key phrases: 52 card standard deck so you know there's no tricks or missing cards. [U]Calculate the number of face cards in a standard 52 card deck[/U] First, we know that face cards = (J, K, Q) We also know that there are 4 suits (Hearts, Diamonds, Spades, Clubs) Total Face Cards = 3 face card types * 4 possible suits = 12 face cards [U]Calculate total face down cards[/U] First card, you turn over a 6 Next card, you turn over a 7 This means, we have 52 cards - 2 cards from the draws = 50 cards left in the deck which are face down. P(Face Card) = Total Face Cards / Total Cards in the Deck Face Down P(Face Card) = 12/50 Simplifying this fraction [URL='https://www.mathcelebrity.com/fraction.php?frac1=12%2F50&frac2=3%2F8&pl=Simplify']using our math engine[/URL], we get: P(Face Card) = [B]6/25[/B]

From a standard 52 card deck, how many 6-card hands will have 2 spades and 4 hearts?
From a standard 52 card deck, how many 6-card hands will have 2 spades and 4 hearts? We want the product of 13C2 * 13C4 since we have 13 possible spades choose 2 and 13 possible hearts choose 4 [LIST] [*]Spades: 13C2 from our [URL='http://www.mathcelebrity.com/permutation.php?num=13&den=2&pl=Combinations']combinations calculator[/URL] = 78 [*]Hearts: 13C4 from our [URL='http://www.mathcelebrity.com/permutation.php?num=13&den=4&pl=Combinations']combinations calculator[/URL] = 715 [/LIST] (78)(715) = [B]55,770[/B]

Function
Free Function Calculator - Takes various functions (exponential, logarithmic, signum (sign), polynomial, linear with constant of proportionality, constant, absolute value), and classifies them, builds ordered pairs, and finds the y-intercept and x-intercept and domain and range if they exist. Table of Functions Calculator

Functions-Derivatives-Integrals
Free Functions-Derivatives-Integrals Calculator - Given a polynomial expression, this calculator evaluates the following items:
1) Functions ƒ(x).  Your expression will also be evaluated at a point, i.e., ƒ(1)
2) 1st Derivative ƒ'(x)  The derivative of your expression will also be evaluated at a point, i.e., ƒ'(1)
3) 2nd Derivative ƒ''(x)  The second derivative of your expression will be also evaluated at a point, i.e., ƒ''(1)
4)  Integrals ∫ƒ(x)  The integral of your expression will also be evaluated on an interval, i.e., [0,1]
5) Using Simpsons Rule, the calculator will estimate the value of ≈ ∫ƒ(x) over an interval, i.e., [0,1]

Fundamental Rule of Counting
Free Fundamental Rule of Counting Calculator - Given a set of items, this calculates the total number of groups/choices that can be formed using the rule of product.

FV-O/T=A for o
FV-O/T=A for o Add O/T to each side: FV-O/T + O/T = A + O/T We have: A + O/T = FV Subtract A from each side: A - A + O/T = FV + A Cancelling the A's, e have: O/T = FV - A Cross multiply the T: [B]O = T(FV - A)[/B]

f^2+5g=3md for d
f^2+5g = 3md for d Divide each side by 3m to isolate d: (f^2+5g)/3m = 3md/3md Cancel the 3m on the right side and we get: d = [B](f^2+5g)/3m[/B]

g equals 232 subtracted from the quantity 377 times g
g equals 232 subtracted from the quantity 377 times g 377 times g: 377g 232 subtracted from 377 times g: 377g - 232 We set the variable g equal to this expression: [B]g = 377g - 232[/B]

g less than 143 is equal to 39 reduced by w
g less than 143 is equal to 39 reduced by w g less than 143 means we subtract g from 143 143 - g 39 reduced by w means we subtract w from 39 39 - w We set these 2 expressions equal to each other: [B]143 - g = 39 - w[/B]

g times by 5 then add 3
g times by 5 then add 3 The phrase [I]times by [/I]means times or multiplied by: 5g Then add 3 means we add 3 to 5g: [B]5g + 3 [MEDIA=youtube]7KeEWSY1WMg[/MEDIA][/B]

g=1+2a/a
Cancel the a's in the last term: g = 1 + 2 g = 3

Gabe rents a piano for $49 per month. He earns $15 per hour giving piano lessons to students. How ma
Gabe rents a piano for $49 per month. He earns $15 per hour giving piano lessons to students. How many hours of lessons per month must he give to earn a profit of $326? Build a profit function P(h) where h is the number of hours: P(h) = Hourly Rate * Number of Hours (h) - Cost of Piano P(h) = 15h - 49 The problem asks for the number of hours where P(h) = $326 15h - 49 = 326 We take this equation and [URL='https://www.mathcelebrity.com/1unk.php?num=15h-49%3D326&pl=Solve']type it in our search engine[/URL] to solve for h: h = [B]25[/B]

Gamma Constant γ
Free Gamma Constant γ Calculator - This calculator generates 5000 iterations for the development of the gamma constant γ

Gary is buying chips. Each bag costs $3.50. He has $40 to spend. Write an inequality to represent th
Gary is buying chips. Each bag costs $3.50. He has $40 to spend. Write an inequality to represent the number of chip bags, c, he can afford. Gary's spend is found by this inequality: [B]3.50c <= 40 [/B] To solve this inequality, [URL='https://www.mathcelebrity.com/interval-notation-calculator.php?num=3.50c%3C%3D40&pl=Show+Interval+Notation']we type it in our search engine[/URL] and we get: [B]c <= 11.43[/B]

Gayle has 36 coins, all nickels and dimes, worth $2.40. How many dimes does she have?
Gayle has 36 coins, all nickels and dimes, worth $2.40. How many dimes does she have? Set up our given equations using n as the number of nickels and d as the number of dimes: [LIST=1] [*]n + d = 36 [*]0.05n + 0.1d = 2.40 [/LIST] Use our [URL='http://www.mathcelebrity.com/simultaneous-equations.php?term1=n+%2B+d+%3D+36&term2=0.05n+%2B+0.1d+%3D+2.40&pl=Cramers+Method']simultaneous equations calculator[/URL] to get: n = 24 [B]d = 12[/B]

GDP Deflator
Free GDP Deflator Calculator - Calculates the GDP Deflator using nominal and real GDP

Geocache puzzle help
In the first hour, he sold one-half of his sticks, plus one-half of a stick. The next hour, he sold one-third of his remaining sticks plus one-third of a stick. In the third hour, he sold one-fourth of what he had left, plus three-fourths of a stick. The last hour, he sold one-fifth of the remaining sticks, plus one-fifth of a stick. He did not cut up any sticks to make these sales. He returned home with 19 sticks. How many did he originally take to the event?

Geocache puzzle help
Let x equal the number of sticks he started with. We have: The equation is 4/5 * (3/4 * (2/3 * (0.5x - 0.5) -1/3) - 0.75) - 0.2 = 19 Add 0.2 to each side: 4/5 * (3/4 * (2/3 * (0.5x - 0.5) -1/3) - 0.75) = 19.2 Multiply each side by 5/4 (3/4 * (2/3 * (0.5x - 0.5) - 1/3) - 0.75) = 24 Multiply the inside piece first: 2/6x - 2/6 - 1/3 2/6x - 4/6 Now subtract 0.75 which is 3/4 2/6x - 4/6 - 3/4 4/6 is 8/12 and 3/4 is 9/12, so we have: 2/6x - 17/12 Now multiply by 3/4 6/24x - 51/48 = 24 Simplify: 1/4x - 17/16 = 24 Multiply through by 4 x - 17/4 = 96 Since 17/4 = 4.25, add 4.25 to each side x = 100.25 Since he did not cut up any sticks, he has a full stick to start with: So x = [B]101[/B]

Geocache puzzle help
Ok. To go further in this equation. It reads: ...How many did he originally take to the event? Multiply the answer by 3 and reverse the digits. This will give you the answer for ACH in the coordinates. Does that make sense to reverse 303? :-/ Thank you for your help!!

Geocache puzzle help
Let x equal the number of sticks he started with. We have: The equation is 4/5 * (3/4 * (2/3 * (0.5x - 0.5) -1/3) - 0.75) - 0.2 = 19 Add 0.2 to each side: 4/5 * (3/4 * (2/3 * (0.5x - 0.5) -1/3) - 0.75) = 19.2 Multiply each side by 5/4 (3/4 * (2/3 * (0.5x - 0.5) - 1/3) - 0.75) = 24 Multiply the inside piece first: 2/6x - 2/6 - 1/3 2/6x - 4/6 Now subtract 0.75 which is 3/4 2/6x - 4/6 - 3/4 4/6 is 8/12 and 3/4 is 9/12, so we have: 2/6x - 17/12 Now multiply by 3/4 6/24x - 51/48 = 24 Simplify: 1/4x - 17/16 = 24 Multiply through by 4 x - 17/4 = 96 Since 17/4 = 4.25, add 4.25 to each side x = 100.25 Since he did not cut up any sticks, he has a full stick to start with: So x = [B]101[/B]

Geocache puzzle help
Let me post the whole equation paragraph: Brainteaser # 1: Answer for ACH A fellow geocacher decided that he would try to sell some hand-made walking sticks at the local geocaching picnic event. In the first hour, he sold one-half of his sticks, plus one-half of a stick. The next hour, he sold one-third of his remaining sticks plus one-third of a stick. In the third hour, he sold one-fourth of what he had left, plus three-fourths of a stick. The last hour, he sold one-fifth of the remaining sticks, plus one-fifth of a stick. He did not cut up any sticks to make these sales. He returned home with 19 sticks. How many did he originally take to the event? Multiply the answer by 3 and reverse the digits. This will give you the answer for ACH in the coordinates. Make sure to multiply and reverse the digits. What would the answer be?

Geocache puzzle help
Multiply the answer by 3: 101 * 3 = 303 Reverse the digits: 303 reversed is a palindrome, so it's still [B]303[/B].

Geometric Annuity Immediate
Free Geometric Annuity Immediate Calculator - Given an immediate annuity with a geometric progression, this solves for the following items
1) Present Value
2) Accumulated Value (Future Value)
3) Payment

Geometric Distribution
Free Geometric Distribution Calculator - Using a geometric distribution, it calculates the probability of exactly k successes, no more than k successes, and greater than k successes as well as the mean, variance, standard deviation, skewness, and kurtosis.
Calculates moment number t using the moment generating function

geometric mean of 6 and 24
Use the [URL='http://www.mathcelebrity.com/statbasic.php?num1=6%2C24&num2=+0.2%2C0.4%2C0.6%2C0.8%2C0.9&pl=Central+Tendency']geometric mean calculator[/URL]: We get [B]12[/B]

Geometric Mean of a Triangle
Free Geometric Mean of a Triangle Calculator - Given certain segments of a special right triangle, this will calculate other segments using the geometric mean

Geometry Summary
Free Geometry Summary Calculator - This is a table which lists out the formulas for geometric shapes

George and William both ran a one mile race. William won the race with a time of 4 minutes and 30 se
George and William both ran a one mile race. William won the race with a time of 4 minutes and 30 seconds. If George was 480 feet behind William when the race finished, how long did it take George to run the entire mile? (George continued to run at the same pace.) When the race was done, George completed: 5280 feet in a mile - 480 feet = 4800 feet set up a proportion of distance traveled to time where n is the time needed to run the mile 4800/4.5 = 5280/n Using our [URL='https://www.mathcelebrity.com/proportion-calculator.php?num1=4800&num2=5280&den1=4.5&den2=n&propsign=%3D&pl=Calculate+missing+proportion+value']proportion calculator,[/URL] we get: n = 4.95 5280/4800 = 1.1 Setup another proportion with the 1.1 factor of distance to time: 4800 * 1.1/4.5 * 1.1 = 5280/4.95 4.95 = 4 minutes and .95*60 seconds 4.95 = [B]4 minutes and 57 seconds[/B]

George has 600 baseball cards and Joy has one fifth as many baseball cards as George. How many baseb
George has 600 baseball cards and Joy has one fifth as many baseball cards as George. How many baseball cards does joy have? Let j = Joy's cards and g = George's cards. We have the following equation: g = 600 j = 1/5g So j = 600/5 [B]j = 120[/B]

George has a certain number of apples, and Sarah has 4 times as many apples as George. They have a t
George has a certain number of apples, and Sarah has 4 times as many apples as George. They have a total of 25 apples. Let George's apples be g. Let Sarah's apples be s. We're give two equations: [LIST=1] [*]s = 4g [*]g + s = 25 [/LIST] Substitute equation (1) into equation (2) for s: g + 4g = 25 If [URL='https://www.mathcelebrity.com/1unk.php?num=g%2B4g%3D25&pl=Solve']we plug this equation into our search engine[/URL] and solve for g, we get: g = [B]5[/B] Now substitute this into equation 1 for g = 5: s = 4(5) s = [B]20[/B] [B]So George has 5 apples and Sarah has 20 apples[/B]

Georgie joins a gym. she pays $25 to sign up and then $15 each month. Create an equation using this
Georgie joins a gym. she pays $25 to sign up and then $15 each month. Create an equation using this information. Let m be the number of months Georgie uses the gym. Our equation G(m) is the cost Georgie pays for m months. G(m) = Variable Cost * m (months) + Fixed Cost Plug in our numbers: [B]G(m) = 15m + 25[/B]

Georgina has 9 bags of shaved coconuts. If each bag weighs 1/2 pounds what is the total weight of th
Georgina has 9 bags of shaved coconuts. If each bag weighs 1/2 pounds what is the total weight of the bags? We take 9 bags * 1/2 pound per bag = [B]4.5 total pounds[/B].

Germany and Austria have a total of 25 states. Germany has 7 more states than Austria has. Create 2
Germany and Austria have a total of 25 states. Germany has 7 more states than Austria has. Create 2 equations. Let g be the number of German states. Let a be the number of Austrian states. We're given two equations: [LIST=1] [*]a + g = 25 [*]g = a + 7 [/LIST] To solve this system of equations, we substitute equation (2) into equation (1) for g: a + (a + 7) = 25 Combine like terms: 2a + 7 = 25 To solve for a, we[URL='https://www.mathcelebrity.com/1unk.php?num=2a%2B7%3D25&pl=Solve'] type this equation into our search engine[/URL] and we get: [B]a = 9[/B] To solve for g, we plug in a = 9 into equation (2): g = 9 + 7 [B]g = 16[/B]

Gigi’s family left their house and drove 14 miles south to a gas station and then 48 miles east to a
Gigi’s family left their house and drove 14 miles south to a gas station and then 48 miles east to a water park. How much shorter would their trip to the water park have been if they hadn’t stopped at the gas station and had driven along the diagonal path instead? [IMG]https://mathcelebrity.com/community/data/attachments/0/pythag-diagonal.jpg[/IMG] Using our [URL='https://www.mathcelebrity.com/pythag.php?side1input=14&side2input=48&hypinput=&pl=Solve+Missing+Side']Pythagorean theorem calculator[/URL], we see the diagonal route would be: 50 miles The original trip distance was: Original Trip Distance = 14 + 48 Original Trip Distance = 62 miles Diagonal Trip was 50 miles, so the difference is: Difference = Original Trip Distance - Diagonal Distance Difference = 62 - 50 Difference = [B]12 miles[/B]

Gino has 7 whole pineapples. He cuts each whole into 4 equal parts. Write an improper fraction for t
Gino has 7 whole pineapples. He cuts each whole into 4 equal parts. Write an improper fraction for the cut parts of pineapples. Take our whole pineapples divided by the number of equal parts: [B]7/4[/B]

Giovanni is thinking of a number. If he adds 2 to it, then divides that sum by 3, he gets 7. What is
Giovanni is thinking of a number. If he adds 2 to it, then divides that sum by 3, he gets 7. What is the number? Let the number be n: [LIST] [*]n [*]Add 2: n + 2 [*]Divide the sum by 3: (n + 2)/3 [*]The word "gets" means an equation, so we set (n + 2)/3 equal to 7 [/LIST] (n + 2)/3 = 7 Cross multiply: n + 2 = 21 To solve for n, we [URL='https://www.mathcelebrity.com/1unk.php?num=n%2B2%3D21&pl=Solve']type this equation into our search engine[/URL] and we get: n = [B]19[/B]

Given g(a)=a² - 2a - 1 and f(x)=x² - 2x, Find: a) f(a+2)-f(a)/2 b) g(a+h)-g(a)/h
Given g(a)=a² - 2a - 1 and f(x)=x² - 2x: Find: a) f(a+2) - f(a)/2 b) g(a+h) - g(a)/h a) f(a + 2) = (a + 2)^2 - 2(a + 2) f(a + 2) = a^2 + 2a + 4 - 2a - 4 Simplify and combine like terms: the 2a and 4's cancel, so we have: f(a + 2) = a^2 f(a)/2 = (a^2 - 2a)/2 Subtract one from the other, we get: a^2 - a^2/2 - a [B]a) a^2/2 - a ------------------------[/B] b) g(a + h) = (a + h)^2 - 2(a + h) - 1 g(a + h) = a^2 +2ah + h^2 - 2a - 2h - 1 g(a)/2 = (a^2 - 2a - 1)/h g(a)/2 = (a^2 - 2a - 1)/h Subtract one from the other: g(a+h) - g(a)/h a^2 +2ah + h^2 - 2a - 2h - 1 - (a^2 - 2a - 1)/h Multiply through by h [B]a^2h + 2ah^2 + h^3 - 2ah - 2h^2 - h - a^2 + 2a + 1[/B]

Given P(A) = 0.37, find P ( not A )
Given P(A) = 0.37, find P ( not A ) Not A is also written as A'. We use the formula below: P(A') = 1 - P(A) P(A') = 1 - 0.37 P(A') = [B]0.63[/B]

Given that E[Y]=2 and Var [Y] =3, find E[(2Y + 1)^2]
Given that E[Y]=2 and Var [Y] =3, find E[(2Y + 1)^2] Multiply through E[(2Y + 1)^2] = E[4y^2 + 4y + 1] We can take the expected value of each term E[4y^2] + E[4y] + E[1] For the first term, we have: 4E[Y^2] We define the Var[Y] = E[Y^2] - (E[Y])^2 Rearrange this term, we have E[Y^2] = Var[Y] + (E[Y])^2 E[Y^2] = 3+ 2^2 E[Y^2] = 3+ 4 E[Y^2] = 7 So our first term is 4(7) = 28 For the second term using expected value rules of separating out a constant, we have 4E[Y] = 4(2) = 8 For the third term, we have: E[1] = 1 Adding up our three terms, we have: E[4y^2] + E[4y] + E[1] = 28 + 8 + 1 E[4y^2] + E[4y] + E[1] = [B]37[/B]

Given that m is a positive integer and 4^m - 1 = n, which of the following values CANNOT represent n
A. 3 B. 7 C. 63 D. 255 We know that: [LIST] [*]4^1 = 4 [*]4^2 = 16 [*]4^3 = 64 [*]4^4 = 256 [*]4^5 = 1024 [*]4^6 = 4096 [/LIST] Notice they all end in 4 or 6. This continues for to infinity. 4^m will either end in a 4 or a 6 Therefore, 4^m - 1 ends in: [LIST] [*]4 - 1 = 3 [*]6 - 1 = 5 [/LIST] Choices A, C, and D end in 3 or 5. Choice B does not. So [B]Choice B [/B]is the answer. [MEDIA=youtube]iVazxGUHvP8[/MEDIA]

Given that P (A)=0.6, P (B)=0.5, P (A|B) = 0.2, P (C|A)= 0.3 and P (C|B)=0.4. (1) If they are depe
Given that P (A)=0.6, P (B)=0.5, P (A|B) = 0.2, P (C|A)= 0.3 and P (C|B)=0.4. (1) If they are dependent each other, what is P (B | A) = ? (2) If the event C is conditionally dependent upon evens A and B, What's the probability: P (A|C) = ? (1) Bayes Rule: P(B|A) = P(B) * P(A|B) P(B|A) = 0.5 * 0.2 = 0.1 (2) Bayes Rule: P(A|C) = P(A) * P(C|A) P(A|C)= 0.6 * 0.3 = 0.18

Given the function f(x)=3x?9, what is the value of x when f(x)=9
Given the function f(x)=3x?9, what is the value of x when f(x)=9 Plug in our numbers and we get: 3x - 9 = 9 To solve this equation for x, we [URL='https://www.mathcelebrity.com/1unk.php?num=3x-9%3D9&pl=Solve']type it in our search engine[/URL] and we get: x = [B]6[/B]

Given the rectangular prism below, if AB = 6 in., AD = 8 in. and BF = 24, find the length of FD.
Given the rectangular prism below, if AB = 6 in., AD = 8 in. and BF = 24, find the length of FD. [IMG]http://www.mathcelebrity.com/images/math_problem_library_129.png[/IMG] If AB = 6 and AD = 8, by the Pythagorean theorem, we have BD = 10 from our [URL='http://www.mathcelebrity.com/pythag.php?side1input=6&side2input=8&hypinput=&pl=Solve+Missing+Side']Pythagorean Theorem[/URL] Calculator Using that, we have another right triangle which we can use the [URL='http://www.mathcelebrity.com/pythag.php?side1input=10&side2input=24&hypinput=&pl=Solve+Missing+Side']pythagorean theorem[/URL] calculator to get [B]FD = 26[/B]

Given w(x) = 3x + 8, find w(2b + 6).
Given w(x) = 3x + 8, find w(2b + 6). Plug the value of 2b + 6 in for x w(2b + 6) = 3(2b + 6) + 8 Multiply through: w(2b + 6) = 6b + 18 + 8 Group like terms: w(2b + 6) = [B]6b + 26[/B]

Given y= 4/3x what is the constant of proportionality
Given y= 4/3x what is the constant of proportionality Direct variation means the constant of proportionality is y/x. Cross multiplying, we get: y/x = [B]4/3[/B]

Given: 3(2x ? 5) = 15 Prove: x = 5
Given: 3(2x ? 5) = 15 Prove: x = 5 Set x = 5: 3(2(5) - 5) 3(10 - 5) 3(5) 15 So if x = 5, then: 3(2x ? 5) = 15

Given: 9 - 4x = -19 Prove: x = 7
Given: 9 - 4x = -19 Prove: x = 7 Solve for [I]x[/I] in the equation 9 - 4x = - 19 [SIZE=5][B]Step 1: Group constants:[/B][/SIZE] We need to group our constants 9 and -19. To do that, we subtract 9 from both sides -4x + 9 - 9 = -19 - 9 [SIZE=5][B]Step 2: Cancel 9 on the left side:[/B][/SIZE] -4x = -28 [SIZE=5][B]Step 3: Divide each side of the equation by -4[/B][/SIZE] -4x/-4 = -28/-4 x = [B]7[/B]

Given:
Given:

Given: WS bisects
Given: WS bisects

Glen got 48 out of 64 correct in his test. What fraction of the marks did he get wrong? Give your a
Glen got 48 out of 64 correct in his test. What fraction of the marks did he get wrong? Give your answer in its simplest form. Glen got 64 - 48 = 16 questions wrong 16/64 [URL='http://www.mathcelebrity.com/fraction.php?frac1=16%2F64&frac2=3%2F8&pl=Simplify']as a fraction[/URL] is 1/4

Goal is to take at least 10,000 steps per day. According to your pedometer you have walked 5,274 ste
Goal is to take at least 10,000 steps per day. According to your pedometer you have walked 5,274 steps. Write and solve an inequality to find the possible numbers of steps you can take to reach your goal. [U] Subtract off the existing steps (s) from your goal of 10,000[/U] g >= 10000 - 5274 [B]g >= 4726[/B] [I]Note: we use >= since 10,000 steps meets the goal as well as anytihng above it[/I]

Golden Ratio
Free Golden Ratio Calculator - Solves for 2 out of the 3 variables for a segment broken in 2 pieces that satisfies the Golden Ratio (Golden Mean).
(a) Large Segment
(b) Small Segment
(a + b) Total Segment

Google AdWords Advanced Display Exam
Answers and Study Guide for the Google AdWords Advanced Display exam

Google Adwords Video Exam Certification
Questions and answers from the Google Adwords Video Exam Certification

Google Analytics Certification Test Questions
Actual Questions from the Google Analytics Certification Test/Exam.

Google Analytics on Steroids
This software declassifies anonymous website visitors. It tells you their company, what pages they view, and gathers the contact information for decision makers at the company. And I've negotiated a free, 7 day trial for you to try it out. This software is best if you sell or deal with B2B transactions. Enjoy this [URL='https://www.donsevcik.com/identify-anonymous-visitors']Google Analytics on Steroids[/URL].

Google Digital Sales Exam
Exam answers and Study Guide for the Google Digital Sales Exam

Google Educator Certification Test Questions
Questions and Answers from the Google Educator Certification Test

Google Optimize Partner Exam
Exam answers and Study Guide for the Google Optimize Partner Exam

Graham is hiking at an altitude of 14,040 feet and is descending 50 feet each minute.Max is hiking a
Graham is hiking at an altitude of 14,040 feet and is descending 50 feet each minute.Max is hiking at an altitude of 12,500 feet and is ascending 20 feet each minute. How many minutes will it take until they're at the same altitude? Set up the Altitude function A(m) where m is the number of minutes that went by since now. Set up Graham's altitude function A(m): A(m) = 14040 - 50m <-- we subtract for descending Set up Max's altitude function A(m): A(m) = 12500 + 20m <-- we add for ascending Set the altitudes equal to each other to solve for m: 14040 - 50m = 12500 + 20m [URL='https://www.mathcelebrity.com/1unk.php?num=14040-50m%3D12500%2B20m&pl=Solve']We type this equation into our search engine to solve for m[/URL] and we get: m = [B]22[/B]

Grand Mean
Free Grand Mean Calculator - Calculates the grand mean of a set of number sets.

Grandmother, mother and daughter are celebrating 150 years of life. The Mother is 25 years older tha
Grandmother, mother and daughter are celebrating 150 years of life. The Mother is 25 years older than her daughter, but 31 years younger than her mother (the grandmother). How old are the three Let grandmother's age be g. Let mother's age be m. Let daughter's age be d. We're given 3 equations: [LIST=1] [*]m = d + 25 [*]m = g - 31 [*]d + g + m = 150 [/LIST] This means the daughter is: d = 25 + 31 = 56 years younger than her grandmother. So we have: 4. d = g - 56 Plugging in equation (2) and equation(4) into equation (3) we get: g - 56 + g + g - 31 Combine like terms: 3g - 87 = 150 [URL='https://www.mathcelebrity.com/1unk.php?num=3g-87%3D150&pl=Solve']Typing this equation into the search engine[/URL], we get: [B]g = 79[/B] Plug this into equation (2): m = 79 - 31 [B]m = 48[/B] Plug this into equation (4): d = 79 - 56 [B]d = 23[/B]

Gravitational Force
Free Gravitational Force Calculator - Using Sir Isaac Newtons Law of Gravitational Force, this calculator determines the force between two objects with mass in kilograms at a distance apart in meters using the constant of gravity.

Grayson took a total of 16 quizzes over the course of 8 weeks. How many weeks of school will Grayson
Grayson took a total of 16 quizzes over the course of 8 weeks. How many weeks of school will Grayson have to attend this quarter before he will have taken a total of 20 quizzes? Assume the relationship is directly proportional. Set up a proportion of quizzes to weeks, where w is the number of weeks for 20 quizzes: 16/8 = 20/w [URL='https://www.mathcelebrity.com/prop.php?num1=16&num2=20&den1=8&den2=w&propsign=%3D&pl=Calculate+missing+proportion+value']Type this proportion into our search engine[/URL], and we get: w = [B]10[/B]

Greatest Common Factor and Least Common Multiple
Free Greatest Common Factor and Least Common Multiple Calculator - Given 2 or 3 numbers, the calculator determines the following:
* Greatest Common Factor (GCF) using Factor Pairs
* Rewrite Sum using the Distributive Property and factoring out the GCF
* Least Common Multiple (LCM) / Least Common Denominator (LCD) using Factor Pairs
* GCF using the method of Successive Division
* GCF using the Prime Factorization method
* Determine if the numbers are coprime and twin prime

Greatest Common Factors of Monomials
Free Greatest Common Factors of Monomials Calculator - This calculator will determine the Greatest Common Factors of a set of Monomials

Greek Alphabet
Free Greek Alphabet Calculator - Shows you the greek alphabet and symbols used in math and science

Greg runs 120 m in 20 seconds. How far can he run in one minute?
Greg runs 120 m in 20 seconds. How far can he run in one minute? We want to compare seconds to seconds. [URL='https://www.mathcelebrity.com/timecon.php?quant=1&pl=Calculate&type=minute']1 minute[/URL] = 60 seconds Set up a proportion of meters to seconds where m is the meters ran in 60 seconds: 120/20 = m/60 To solve this proportion for m, we [URL='https://www.mathcelebrity.com/prop.php?num1=120&num2=m&den1=20&den2=60&propsign=%3D&pl=Calculate+missing+proportion+value']type it in our search engine[/URL] and we get: m. = [B]360 meters[/B]

Gregg has 8 cards.Half red,half black. He picks 2 cards from the deck.What is the probability both o
Gregg has 8 cards.Half red,half black. He picks 2 cards from the deck.What is the probability both of them are red? Half means 4 cards are red and 4 cards are black. The first draw probability of red is: 4 total red cards out of 8 total cards = 4/8. [URL='https://www.mathcelebrity.com/fraction.php?frac1=4%2F8&frac2=3%2F8&pl=Simplify']Simplified, this is[/URL] 1/2 The second draw is 3 total red cards out of 7 remaining cards. Since 1 red was drawn (4 - 1) = 3 reds left and 1 card was drawn (8 -1) = left 3/7 Since each draw is independent, we multiply the probabilities: 1/2 * 3/7 = [B]3/14[/B]

Gretchen earns $7 per hour at the local pizza shop. If she works 3 hours in an afternoon, how much m
Gretchen earns $7 per hour at the local pizza shop. If she works 3 hours in an afternoon, how much money does she earn? Earnings = Hourly Wage * Hours Worked Earnings = $7 * 3 Earnings = [B]$21[/B]

Gross Domestic Product (GDP)
Free Gross Domestic Product (GDP) Calculator - Solves for all items of the Gross Domestic Product (GDP) equation:
GDP
Consumption (C)
Investment (I)
Government Spending (G)
Exports (X)
Imports (I).

Guadalupe left the restaurant traveling 12 mph. Then, 3 hours later, Lauren left traveling the same
Guadalupe left the restaurant traveling 12 mph. Then, 3 hours later, Lauren left traveling the same direction at 24 mph. How long will Lauren travel before catching up with Guadalupe? Distance = Rate x Time Guadulupe will meet Lauren at the following distance: 12t = 24(t - 3) 12t = 24t - 72 [URL='https://www.mathcelebrity.com/1unk.php?num=12t%3D24t-72&pl=Solve']Typing that equation into our search engine[/URL], we get: t = 6

gy=-g/v+w for g
gy=-g/v+w for g Multiply each side of the equation by v to eliminate fractions: gvy = -g + vw Add g to each side: gvy + g = -g + g + vw Cancel the g's on the right side and we geT: gvy + g = vw Factor out g on the left side: g(vy + 1) = vw Divide each side of the equation by (vy + 1): g(vy + 1)/(vy + 1) = vw/(vy + 1) Cancel the (vy + 1) on the left side and we geT: g = [B]vw/(vy + 1)[/B]

Gym A: $75 joining fee and $35 monthly charge. Gym B: No joining fee and $60 monthly charge. (Think
Gym A: $75 joining fee and $35 monthly charge. Gym B: No joining fee and $60 monthly charge. (Think of the monthly charges paid at the end of the month.) Enter the number of months it will take for the total cost for both gyms to be equal. Gym A cost function C(m) where m is the number of months: C(m) = Monthly charge * months + Joining Fee C(m) = 35m + 75 Gym B cost function C(m) where m is the number of months: C(m) = Monthly charge * months + Joining Fee C(m) = 60m Set them equal to each other: 35m + 75 = 60m To solve for m, [URL='https://www.mathcelebrity.com/1unk.php?num=35m%2B75%3D60m&pl=Solve']we type this equation into our search engine[/URL] and get: m = [B]3[/B]

H minus 6 all cubed
H minus 6 all cubed H minus 6 h - 6 All cubed means raise the entire expression to the 3rd power (h - 6)^3

Hailey worked 32 hours at 8 dollars a hour .Taxes were 1% .How much money was left?
Hailey worked 32 hours at 8 dollars a hour .Taxes were 1% .How much money was left? Calculate earnings: Earnings = Hourly rate * hours worked Earnings = 32 * 8 Earnings = 256 If taxes are 1%, then Hailey ends up with 100% - 1% = 99% Leftover = 256 * 99% Leftover = [B]$253.44[/B]

Hal bought a house in 1995 for $190,000. If the value of the house appreciates at a rate of 4.5 perc
Hal bought a house in 1995 for $190,000. If the value of the house appreciates at a rate of 4.5 percent per year, how much was the house worth in 2006 [U]Calculate year difference:[/U] Year Difference = End Year - Start Year Year Difference = 2006 - 1995 Year Difference = 11 Using our [URL='https://www.mathcelebrity.com/apprec-percent.php?q=a+house+worth+190000+appreciates+4.5%25+for+11+years&pl=Calculate+Appreciation']appreciation calculator[/URL], we get the value of the house in 2006: [B]$308,342.08[/B]

Haley invested $750 into a mutual fund that paid 3.5% interest each year compounded annually. Find t
Haley invested $750 into a mutual fund that paid 3.5% interest each year compounded annually. Find the value of the mutual fund in 15 years. Using our [URL='https://www.mathcelebrity.com/compoundint.php?bal=750&nval=15&int=3.5&pl=Annually']compound interest calculator[/URL], we get: [B]1,256.51[/B]

Half of a pepperoni pizza plus 3/4ths of a ham and pineapple pizza has 765 calories. 1/4th of a pepp
Half of a pepperoni pizza plus 3/4ths of a ham and pineapple pizza has 765 calories. 1/4th of a pepperoni pizza and a whole ham and pineapple pizza contains 745 calories. How many calories are each of the 2 kinds of pizzas individually? Let p be the pepperoni pizza calories and h be the ham and pineapple pizza calories. We're given [LIST=1] [*]0.5p + 0.75h = 765 [*]0.25p + h = 745 [/LIST] With this system of equations, we can solve using 3 methods: [LIST] [*][URL='https://www.mathcelebrity.com/simultaneous-equations.php?term1=0.5p+%2B+0.75h+%3D+765&term2=0.25p+%2B+h+%3D+745&pl=Substitution']Substitution Method[/URL] [*][URL='https://www.mathcelebrity.com/simultaneous-equations.php?term1=0.5p+%2B+0.75h+%3D+765&term2=0.25p+%2B+h+%3D+745&pl=Elimination']Elimination Method[/URL] [*][URL='https://www.mathcelebrity.com/simultaneous-equations.php?term1=0.5p+%2B+0.75h+%3D+765&term2=0.25p+%2B+h+%3D+745&pl=Cramers+Method']Cramer's Rule[/URL] [/LIST] No matter what method we choose, we get: [B]h = 580 p = 660[/B]

half of c subtracted from the sum of a and b
half of c subtracted from the sum of a and b The sum of a and b: a + b half of c means we divide c by 2: c/2 half of c subtracted from the sum of a and b: [B]a + b - c/2[/B]

Half of g multiplied by t squared is equal to d.
Half of g multiplied by t squared is equal to d. Half of g: g/2 t squared: t^2 Half of g multiplied by t squared: gt^2/2 The phrase [I]is equal to[/I] mean we set gt^2/2 equal to d: [B]gt^2/2 = d[/B]

Half of the difference of a and b
Half of the difference of a and b The difference of a and b is written as: a - b Half of the difference means we divide (a - b) by 2: [B](a - b)/2[/B]

half of the sum of 2p and q
half of the sum of 2p and q The sum of 2p and q means we add q to 2p: 2p + q Half of this means we divide the sum by 2: [B](2p + q)/2[/B]

half the difference of x and 3
half the difference of x and 3 The difference of x and 3 means we subtract 3 from x: x - 3 half of the difference means we divide the difference by 2: [B](x - 3)/2[/B]

half the sum of the numbers s, t, and u
half the sum of the numbers s, t, and u The [I]sum [/I]of s, t, and u means we add all 3: s + t + u [I]Half[/I] the sum means we divide the sum by 2: [B](s + t + u)/2[/B]

Half-Life of a Substance
Free Half-Life of a Substance Calculator - Given a half-life (h) of a substance at time t, this determines the new substance size at time tn, otherwise known as decay.

Hall looked at 10 websites every 35 hours. At this rate, how long, in hours, will it take to look at
Hall looked at 10 websites every 35 hours. At this rate, how long, in hours, will it take to look at 6 websites? Set up a proportion of websites to hours where h is the number of hours it takes to look at 6 websites: 10/35 = 6/h To solve this proportion for h, we [URL='https://www.mathcelebrity.com/prop.php?num1=10&num2=6&den1=35&den2=h&propsign=%3D&pl=Calculate+missing+proportion+value']type it in our math engine[/URL] and we get: h = [B]21 hours[/B]

Hannah invested $540 in an account paying an interest rate of 4.7% compounded continuously. Assuming
Hannah invested $540 in an account paying an interest rate of 4.7% compounded continuously. Assuming no deposits or withdrawals are made, how much money, to the nearest hundred dollars, would be in the account after 18 years? [URL='https://www.mathcelebrity.com/simpint.php?av=&p=540&int=4.7&t=18&pl=Continuous+Interest']Using our compound interest balance calculator[/URL], we get: [B]$1,258.37[/B]

Hans rented a truck for one day. There was a base fee of 16.95, and there was an additional charge o
Hans rented a truck for one day. There was a base fee of 16.95, and there was an additional charge of 76 cents for each mile driven. Hans had to pay 152.99 when he returned the truck. For how many miles did he drive the truck? Set up the equation where x is the amount of miles he drove: 0.76x + 16.95 = 152.99 [URL='http://www.mathcelebrity.com/1unk.php?num=0.76x%2B16.95%3D152.99&pl=Solve']Plug this equation into our calculator[/URL]: x = 179 miles

Happy Paws charges $16.00 plus $1.50 per hour to keep a dog during the day. Woof Watchers charges $1
Happy Paws charges $16.00 plus $1.50 per hour to keep a dog during the day. Woof Watchers charges $11.00 plus $2.75 per hour. Complete the equation and solve it to find for how many hours the total cost of the services is equal. Use the variable h to represent the number of hours. Happy Paws Cost: C = 16 + 1.5h Woof Watchers: C = 11 + 2.75h Setup the equation where there costs are equal 16 + 1.5h = 11 + 2.75h Subtract 11 from each side: 5 + 1.5h = 2.75h Subtract 1.5h from each side 1.25h = 5 Divide each side by 1.25 [B]h = 4[/B]

Happy Paws charges $19.00 plus $5.50 per hour to keep a dog during the day. Woof Watchers charges $1
Happy Paws charges $19.00 plus $5.50 per hour to keep a dog during the day. Woof Watchers charges $11.00 plus $6.75 per hour. Complete the equation and solve it to find for how many hours the total cost of the services is equal. Use the variable h to represent the number of hours. [B]Happy Paws cost equation:[/B] 5.50h + 19 [B]Woof Watchers cost equation:[/B] 6.75h + 11 [B]Set them equal to each other:[/B] 5.50h + 19 = 6.75h + 11 Using our [URL='http://www.mathcelebrity.com/1unk.php?num=5.50h%2B19%3D6.75h%2B11&pl=Solve']equation solver[/URL], we get [B]h = 6.4[/B].

Hardy-Weinberg
Free Hardy-Weinberg Calculator - Given a dominant gene frequency probability of p, this displays the Punnet Square Hardy Weinberg frequencies

Hari planted 324 plants in such a way that there were as many rows of plants as there were number of
Hari planted 324 plants in such a way that there were as many rows of plants as there were number of columns. Find the number of rows and columns. Let r be the number of rows and c be the number of columns. We have the area: rc = 324 Since rows equal columns, we have a square, and we can set r = c. c^2 = 324 Take the square root of each side: [B]c = 18[/B] Which means [B]r = 18[/B] as well. What we have is a garden of 18 x 18.

harley had $500 in his bank account at the beginning of the year. he spends $20 each week on food, c
harley had $500 in his bank account at the beginning of the year. he spends $20 each week on food, clothing, and movie tickets. he wants to have more than $100 at the end of summer to make sure he has enough to purchase some new shoes before school starts. how many weeks, w, can harley withdraw money from his savings account and still have more than $100 to buy new shoes? Let the number of weeks be w. Harley needs $100 (or more) for shoes. We have the balance in Harley's account as: 500 - 20w >= 100 To solve this inequality for w, we [URL='https://www.mathcelebrity.com/1unk.php?num=500-20w%3E%3D100&pl=Solve']type it in our search engine[/URL] and we get: [B]w <= 20[/B]

Harry got 42 out of 49 correct in his test. What fraction of the marks did he get correct?
Harry got 42 out of 49 correct in his test. What fraction of the marks did he get correct? The fraction correct is: 42/49 Both the numerator and denominator [URL='http://www.mathcelebrity.com/gcflcm.php?num1=42&num2=49&num3=&pl=GCF']have a common factor[/URL] of 7 Reducing top and bottom by 7, we get: [B]6/7[/B]

Hayden bought 48 new trading cards. Three-fourths of the new cards are baseball cards. How many base
Hayden bought 48 new trading cards. Three-fourths of the new cards are baseball cards. How many baseball cards did Hayden buy? We want 3/4 of 48. We [URL='https://www.mathcelebrity.com/fraction.php?frac1=48&frac2=3/4&pl=Multiply']type this statement into our calculator[/URL] and we get: [B]36[/B]

He charges $1.50 per delivery and then $2 per km he has to drive to get from his kitchen to the deli
He charges $1.50 per delivery and then $2 per km he has to drive to get from his kitchen to the delivery address. Write an equation that can be used to calculate the delivery price and the distance between the kitchen and the delivery address. Use your equation to calculate the total cost to deliver to someone 2.4km away Let k be the number of kilometers between the kitchen and delivery address. Our Delivery equation D(k) is: [B]D(k) = 2k + 1.50[/B] The problem wants to know D(2.4): D(2.4) = 2(2.4) + 1.50 D(2.4) = 4.8 + 1.50 D(2.4) = [B]$6.30[/B]

Heat Index
Free Heat Index Calculator - Given a temperature in Fahrenheit and a relative humidity percentage, this calculates the Heat Index.

heat loss of a glass window varies jointly as the window's area and the difference between the outsi
heat loss of a glass window varies jointly as the window's area and the difference between the outside and the inside temperature. a window 6 feet wide by 3 feet long loses 1,320 btu per hour when the temperature outside is 22 degree colder than the temperature inside. Find the heat loss through a glass window that is 3 feet wide by 5 feet long when the temperature outside is 9 degree cooler than the temperature inside. Find k of the equation: 6*3*22*k = 1320 396k = 1,320 k = 3.33333 [URL='https://www.mathcelebrity.com/1unk.php?num=396k%3D1320&pl=Solve']per our equation solver[/URL] Now, find the heat loss for a 3x5 window when the temperature is 9 degrees cooler than the temperature inside: 3*5*9*3.333333 = [B]450 btu per hour[/B]

Heather has completed six deliveries so far this week she needs to make 40 deliveries for the week w
Heather has completed six deliveries so far this week she needs to make 40 deliveries for the week what percentage of her deliveries has Heather completed? We want [URL='https://www.mathcelebrity.com/perc.php?num=6&den=40&pcheck=1&num1=16&pct1=80&pct2=70&den1=80&idpct1=10&hltype=1&idpct2=90&pct=82&decimal=+65.236&astart=12&aend=20&wp1=20&wp2=30&pl=Calculate']6/40 as a percent[/URL] which is [B]15%[/B]

Height and weight are two measurements used to track a child's development. TheWorld Health Organiza
Height and weight are two measurements used to track a child's development. The World Health Organization measures child development by comparing the weights of children who are the same height and the same gender. In 2009, weights for all 80 cm girls in the reference population had a mean μ = 10.2 kg and standard deviation σ = 0.8 kg. Weights are normally distributed. X ~ N(10.2, 0.8). Calculate the z-scores that correspond to the following weights and interpret them. a. 11 kg
b. 7.9 kg
c. 12.2 kg a. [URL='http://www.mathcelebrity.com/probnormdist.php?xone=+11&mean=10.2&stdev=8&n=+1&pl=1" target="_blank']Answer A[/URL] - Z = 0.1 b. [URL='http://www.mathcelebrity.com/probnormdist.php?xone=+7.9&mean=+10.2&stdev=+8&n=+1&pl=1']Answer B[/URL] - Z = -0.288 c. [URL='http://www.mathcelebrity.com/probnormdist.php?xone=+12.2&mean=+10.2&stdev=+8&n=+1&pl=1']Answer C[/URL] - Z = 0.25

Help
Suppose company A charges a rate of $40 per day and Company B charges a $60 fee plus $40 per day. For what number of days is the cost the same?

Help
Yes, $40 for both. This was my son's 8th grade problem. They are learning integers with special cases, so this makes sense, I hope. Thank you so much for responding. I did have this written, but I wasn't certain it was correct.

Help on problem
[B]I need 36 m of fencing for my rectangular garden. I plan to build a 2m tall fence around the garden. The width of the garden is 6 m shorter than twice the length of the garden. How many square meters of space do I have in this garden? List the answer being sought (words) ______Need_________________________ What is this answer related to the rectangle?_Have_________________________ List one piece of extraneous information____Need_________________________ List two formulas that will be needed_______Have_________________________ Write the equation for width_____________Have_________________________ Write the equation needed to solve this problem____Need____________________[/B]

Help on problem
[B]List the answer being sought (words) ______Area of the garden What is this answer related to the rectangle?_Have_________________________ List one piece of extraneous information____2m tall fence List two formulas that will be needed_______P = 36. P = 2l + 2w Write the equation for width_____________w = 2l - 6 Write the equation needed to solve this problem A = lw, P = 2l + 2w[/B]

Help Plz
There are three siblings in a family. Their ages add to 26. Let nicks age be "x". John in half of micks age and Pip is two thirds of johns age. write an equation and solve. Find each Childs age.

Help Plz
Nick's age: x John's age: x/2 Pip's age = 2/3 * x/2 = x/3 The sum is 26, so we have: x + x/2 + x/3 = 26 Common denominator is (1 * 2 * 3) = 6 6x/6 + 3x/6 + 2x/6 = 26 Combine like terms: 11x/6 = 26 Cross multiply: 11x = 156 x = 14.1818 This doesn't make sense for age. Are you sure you wrote out the problem right?

HELP SOLVE
Perform a one-sample z-test for a population mean. Be sure to state the hypotheses and the significance level, to compute the value of the test statistic, to obtain the P-value, and to state your conclusion. Five years ago, the average math SAT score for students at one school was 475. A teacher wants to perform a hypothesis test to determine whether the mean math SAT score of students at the school has changed. The mean math SAT score for a random sample of 40 students from this school is 469. Do the data provide sufficient evidence to conclude that the mean math SAT score for students at the school has changed from the previous mean of 475? Perform the appropriate hypothesis test using a significance level of 10%. Assume that ? = 73.

HELP SOLVE
A sample mean, sample size, and population standard deviation are given. Use the one-mean z-test to perform the required hypothesis test at the given significance level. x = 20.5, n = 11, ? = 7, H0: µ = 18.7 , Ha: µ ? 18.7 , ? = 0.01

HELP SOLVE
sample mean, sample size, and population standard deviation are given. Use the one-mean z-test to perform the required hypothesis test at the given significance level. x = 3.7, n = 32, ? = 1.8, H0: µ = 4.2 , Ha: µ ? 4.2 , ? = 0.05

HELP SOLVE
A sample mean, sample size, and population standard deviation are given. Use the one-mean z-test to perform the required hypothesis test about the mean, µ, of the population from which the sample was drawn x = 3.26 , S = 0.55, ?N= 9, H0: µ = 2.85, Ha: µ > 2.85 , ? = 0.01

HELP SOLVE
[URL]http://www.mathcelebrity.com/mean_hypothesis.php?xbar=469&n=40&stdev=73&ptype=%3D&mean=475&alpha=0.10&pl=Mean+Hypothesis+Testing[/URL]

HELP SOLVE
[URL]http://www.mathcelebrity.com/mean_hypothesis.php?xbar=3.7&n=3.2&stdev=1.8&ptype=%3D&mean=4.2&alpha=0.05&pl=Mean+Hypothesis+Testing[/URL]

HELP SOLVE
[URL]http://www.mathcelebrity.com/mean_hypothesis.php?xbar=20.5&n=11&stdev=7&ptype=%3D&mean=18.7&alpha=0.01&pl=Mean+Hypothesis+Testing[/URL]

Hemisphere
Free Hemisphere Calculator - Calculates the base circumference, volume, curved surface area, base surface area, total surface area of a hemisphere with radius r

Henrietta hired a tutor to help her improve her math scores. While working with the tutor, she took
Henrietta hired a tutor to help her improve her math scores. While working with the tutor, she took four tests. She scored 10 points better on the second test than she did on the first, 20 points better on the third test than on the first, and 30 points better on the fourth test than on the first. If the mean of these four tests was 70, what was her score on the third test? Givens: [LIST] [*]Let the first test score be s: [*]The second test score is: s + 10 [*]The third test score is: s + 20 [*]The fourth test score is: s + 30 [/LIST] The mean of the four tests is 70, found below: Sum of test scores / Number of Tests = Mean Plugging in our number, we get: (s + s + 10 + s + 20 + s + 30) / 4 = 70 Cross multiply and simplify: 4s + 60 = 70 * 4 4s + 60 = 280 To [URL='https://www.mathcelebrity.com/1unk.php?num=4s%2B60%3D280&pl=Solve']solve this equation for s, we type it in our search engine[/URL] and we get: s = 55 So the third test score: s + 20 = 55 + 20 [B]75[/B]

Heptagonal Number
Free Heptagonal Number Calculator - This calculator determines the nth heptagonal number

Herfindahl Index
Free Herfindahl Index Calculator - Given a market share of a set of companies, this determines the Herfindahl Index and Normalized Herfindahl Index.

Hero cards come in packs of 6. Max has 8 packs of hero cards. He decides to give as many of his frie
Hero cards come in packs of 6. Max has 8 packs of hero cards. He decides to give as many of his friends as he can 9 cards each. How many cards are left over after he does this? Calculate the number of cards Max starts with: 8 packs * 6 cards per pack = 48 total cards If he gives as many friends as he can 9 cards each, we want to know how many left over after giving as many friends as he can 9 cards. So we have: [URL='https://www.mathcelebrity.com/modulus.php?num=48mod9&pl=Calculate+Modulus']48 mod 9[/URL] = [B]3 left over[/B]

Hexagon
Free Hexagon Calculator - This calculator solves for side length (s), Area (A), and Perimeter (P) of a hexagon given one of the 3 entries.

Hexagonal Number
Free Hexagonal Number Calculator - This calculator determines the nth hexagonal number

High and Low Method
Free High and Low Method Calculator - Calculates the variable cost per unit, total fixed costs, and the cost volume formula

High-Low Method
Free High-Low Method Calculator - Calculates Variable Cost per Unit, Total Fixed Cost, and Cost Volume using the High-Low Method

HomeWork Help Please Respond ASAP!!!
[CENTER][B]The Sum of three times a number and 18 is -39. Find the number.[/B][/CENTER] I was always confused with these problems and never understood them. Any help would be much appreciated!! Thank you!

HomeWork Help Please Respond ASAP!!!
The phrase a number means an arbitrary variable, let's call it x. Three times a number: 3x And 18 means we add 18 3x + 18 The word is means equal to, so we set 3x + 18 equal to -39 3x + 18 = -39 This is your algebraic expression. If you want to solve for x, plug it into the [URL='http://www.mathcelebrity.com/1unk.php?num=3x%2B18%3D-39&pl=Solve']search engine[/URL] and you get x = -19

Hong is riding his bicycle. He rides for 22.5 kilometers at a speed of 9 kilometers per hour. For ho
Hong is riding his bicycle. He rides for 22.5 kilometers at a speed of 9 kilometers per hour. For how many hours does he ride? Distance = Rate * Time The problem asks for time. [URL='https://www.mathcelebrity.com/drt.php?d=+22.5&r=+9&t=&pl=Calculate+the+missing+Item+from+D%3DRT']Using our distance rate time calculator[/URL], we get: t = [B]2.5 hours[/B]

Hootsuite Advanced Social Advertising Exam
Exam answers and Study Guide for the Hootsuite Advanced Social Advertising Exam

Hootsuite Platform Certification
Exam Answers and Study Guide for the Hootsuite Platform Certification

Hootsuite Professional Certification
Exam Answers and Study Guide for the Hootsuite Professional Certification

Hootsuite Social Selling Exam
Exam answers and Study Guide for the Hootsuite Social Selling Certification Exam

Hope it's okay to ask this here?
A candy vendor analyzes his sales records and ?nds that if he sells x candy bars in one day, his pro?t(in dollars) is given byP(x) = ? 0.001x2 + 3x ? 1800 (a.) Explain the signi?cance of the number 1800 to the vendor. (b.) What is the maximum pro?t he can make in one day, and how many candy bars must he sell to achieve it? I got 1800 as the amount he starts with, and can't go over. maximum pro?t as 4950 and if I got that right I am getting stuck on how to find how many candy bars. Thanks

Hope it's okay to ask this here?
a) 1800 is the cost to run the business for a day. To clarify, when you plug in x = 0 for 0 candy bars sold, you are left with -1,800, which is the cost of doing business for one day. b) Maximum profit is found by taking the derivative of the profit equation and setting it equal to 0. P'(x) = -0.002x + 3 With P'(x) = 0, we get: -0.002x + 3 = 0 Using our [URL='http://www.mathcelebrity.com/1unk.php?num=-0.002x%2B3%3D0&pl=Solve']equation solver[/URL], we get: x = 1,500 To get maximum profit, we plug in x = 1,500 to our [I]original profit equation[/I] P(1,500) = ? 0.001(1,500)^2 + 3(1,500) ? 1800 P(1,500) = -2,250 + 4,500 - 1,800 P(1,500) = $[B]450[/B]

Horizontal Vertical Lines
Horizontal Vertical Line Calculator - Calculates things horizontal and vertical lines and the various properties

Hose A can fill a pool in 4 hours. Hose B can fill the pool in 2 hours. If both hoses are turned on
Hose A can fill a pool in 4 hours. Hose B can fill the pool in 2 hours. If both hoses are turned on at the same time how long will it take to fill the pool? [LIST] [*]Hose A can fill the pool in 1/4 of the pool an hour [*]Hose B can fill the pool in 1/2 of the pool an hour [/LIST] In one hour using combined effort, we have: [URL='https://www.mathcelebrity.com/fraction.php?frac1=1%2F2&frac2=1%2F4&pl=Add']1/2 + 1/4[/URL] = 3/4 of the pool will be filled. 3/4 of the pool gets filled in 60 minutes. We set up a proportion of proportion filled to time where t is the time to fill the full pool: 3/4/60 = 1/t 3/240 = 1/t Using our [URL='https://www.mathcelebrity.com/proportion-calculator.php?num1=3&num2=1&den1=240&den2=t&propsign=%3D&pl=Calculate+missing+proportion+value']proportion solver[/URL], we get: t = [B]80 minutes or 1 hour and 20 minutes[/B]

How can you rewrite the number 1 as 2 to a power?
How can you rewrite the number 1 as 2 to a power? There exists an identity which says, n^0 = 1 where n is a number. So [B]2^0 = 1[/B]

How long does it take to cook 20 eggs if it takes 10 minutes to cook 4 eggs
How long does it take to cook 20 eggs if it takes 10 minutes to cook 4 eggs Set up a proportion of minutes to eggs where m is the number of minutes it takes for 20 eggs. 10 minutes / 4 eggs = m/20 [URL='https://www.mathcelebrity.com/proportion-calculator.php?num1=10&num2=m&den1=4&den2=20&propsign=%3D&pl=Calculate+missing+proportion+value']Solving for m[/URL], we get: m = 50

How long will it take $3000 to earn $900 interest at 6% simple interest?
How long will it take $3000 to earn $900 interest at 6% simple interest? Set up the simple interest equation for the interest piece: 3000 * 0.06t = 900 To solve for t in this equation, we [URL='https://www.mathcelebrity.com/1unk.php?num=3000%2A0.06t%3D900&pl=Solve']type it in our search engine [/URL]and we get: t = [B]5[/B]

How many 1/4 sheets are there in 5 sheets
How many 1/4 sheets are there in 5 sheets We divide 5 sheets by 1/4 sheets: 5/1/4 However, when we divide by a fraction, it's the same as multiplying by the reciprocal of the fraction: The reciprocal of 1/4 is 4/1, so we have: 5 * 4/1 = 20/1 = [B]20[/B]

How many 8$, tickets can I get for 100$
How many 8$, tickets can I get for 100$ Tickets = Total Money / price per ticket Tickets = 100/8 Tickets = [B]12.5 [/B] If the problem asks for a whole number, this means you cannot have a partial ticket. Therefore, we round down to [B]12 tickets[/B]

How many days are there in 12 weeks? Use the following information to convert this time to days
How many days are there in 12 weeks? Use the following information to convert this time to days 12 weeks * 7 days / week = [B]84 days[/B]

How many dimes must be added to a bag of 200 nickels so that the average value of the coins in the b
How many dimes must be added to a bag of 200 nickels so that the average value of the coins in the bag is 8 cents? 200 nickels has a value of 200 * 0.05 = $10. Average value of coins is $10/200 = 0.05 Set up our average equation, where we have total value divided by total coins: (200 * 0.05 + 0.1d)/(200 + d) = 0.08 Cross multiply: 16 + 0.08d = 10 + 0.1d Using our [URL='http://www.mathcelebrity.com/1unk.php?num=16%2B0.08d%3D10%2B0.1d&pl=Solve']equation solver[/URL], we get: [B]d = 300[/B]

How many distinct 3 letter arrangements can be made using P, R, I, M and E
How many distinct 3 letter arrangements can be made using P, R, I, M and E? We have all unique letters. We want the combination formula 5 Choose 3, or C(5,3). Using our [URL='http://www.mathcelebrity.com/permutation.php?num=5&den=3&pl=Combinations']combinations calculator[/URL], we get 10 unique 3 letter arrangements.

How many hours are there in 720 minutes?
How many hours are there in 720 minutes? 720 minutes * (1 hour / 60 minutes) = [B]12 hours[/B]

How many kobo are there in y naira?
How many kobo are there in y naira? One naira is divided into [B]100 kobo[/B]. So we have [B]100y kobo[/B]

How many license plates can be made consisting of 3 letters followed by 2 digits
How many license plates can be made consisting of 3 letters followed by 2 digits There are 26 letters A-Z and 10 digits 0-9. We have: 26 * 26 * 26 * 10 * 10 = [B]1,757,600 license plates[/B]

How many minutes are there in m hours
How many minutes are there in m hours m hours * 60 minutes per hour = [B]60m[/B]

how many nickels are there in 10 dimes
how many nickels are there in 10 dimes Using our[URL='https://www.mathcelebrity.com/coincon.php?quant=10&type=dime&pl=Calculate'] coin conversions calculator[/URL], we see that: 10 dimes = [B]20 nickels[/B]

How many one-fifths are there in 200?
How many one-fifths are there in 200? Using the rule of dividing by a fraction is the same as multiplying by the reciprocal, we have: 200 / 1/5 = 200 * 5 = [B]1000[/B]

How many palindromes are between 700 and 800?
How many palindromes are between 700 and 800? Numeric palindromes are numbers which read the same backwards and forwards. In this case, the number has to start and end with 7. [LIST=1] [*]707 [*]717 [*]727 [*]737 [*]747 [*]757 [*]767 [*]777 [*]787 [*]797 [/LIST] There are [B]10[/B] palindromes between 700 and 800

How many possible batting orders are there for a baseball team with 9 players?
How many possible batting orders are there for a baseball team with 9 players? 9! = 9 x 8 x 7 x 6 x 5 x 4 x 3 x 2 x 1 = [B]362,880 batting orders[/B]

How many rides per day to reach 150 rides in 90 days?
How many rides per day to reach 150 rides in 90 days? Set up a proportion of rides per day where r is the number or rides per day: 150/90 = r/1 Type [URL='https://www.mathcelebrity.com/prop.php?num1=150&num2=r&den1=90&den2=1&propsign=%3D&pl=Calculate+missing+proportion+value']this proportion into our search engine[/URL] and we get: r = 1.66 7

how many sixths equal one-third
how many sixths equal one-third We have a variable x where we want to solve for in the following equation: x/6 = 1/3 [URL='https://www.mathcelebrity.com/prop.php?num1=x&num2=1&den1=6&den2=3&propsign=%3D&pl=Calculate+missing+proportion+value']Typing this proportion into our math engine[/URL], we get: x = [B]2[/B]

How many straight lines can be formed by 8 points of which 3 are collinear?
The formula is nC2 - rC2 + 1 In this problem: n = 8 and r = 3 [URL='https://www.mathcelebrity.com/permutation.php?num=8&den=2&pl=Combinations']8C2 [/URL]= 28 [URL='https://www.mathcelebrity.com/permutation.php?num=3&den=2&pl=Combinations']3C2[/URL] = 3 Evaluating, we have: 28 - 3 + 1 [B]26 [MEDIA=youtube]B3MGSmXOiY8[/MEDIA][/B]

How many tens are in 930
How many tens are in 930 To find how many tens, we remove the 0 at the end: [B]93 tens[/B]

How many twelfths equal three-sixths?
How many twelfths equal three-sixths? We set up the equation below where x is the number of twelfths in three-sixths: 1/12x = 3/6 Cross multiply, and we get: 12x * 3 = 6 * 1 36x = 6 To solve for x, we [URL='https://www.mathcelebrity.com/1unk.php?num=36x%3D6&pl=Solve']type this in our math engine[/URL] and we get: x = [B]1/6 or 0.16667[/B]

How many ways are there to hang 5 paintings on 5 hooks?
How many ways are there to hang 5 paintings on 5 hooks? 5! = 5 x 4 x 3 x 2 x 1 5! = [B]120[/B]

How many ways can 5 people be seated in 5 seats?
How many ways can 5 people be seated in 5 seats? We have the permutation 5!. Because the first seat can have 5 different people. The next seat has 5 - 1 = 4 people since one person is in the first seat The next seat can have 5 - 2 = 3 people since we have two people in the first two seats The next seat can have 5 - 3 = 2 people since we have three people in the first three seats The next seat can have 5 - 4 = 1 people since we have four people in the first four seats [URL='https://www.mathcelebrity.com/factorial.php?num=5!&pl=Calculate+factorial']Type in 5! into our search engine[/URL], and we get 120.

How many ways can 6 people be arranged around a circular table?
The tip off for this problem is the 2 phrases: [LIST] [*]circular table [*]arranged [/LIST] Whenever you see these 2 phrases together, the problem is asking for a [URL='https://www.mathcelebrity.com/circular-permutation-calculator.php?num=6&pl=Circular+Permutation']circular permutation[/URL] With n = 6: (6 - 1)! 5! 5 x 4 x 3 x 2 x 1 = [B]120 ways[/B] [MEDIA=youtube]4PXvg-UeN5Ao[/MEDIA]

How many ways can a basketball coach choose to the first five player from a group of 15 players
How many ways can a basketball coach choose to the first five player from a group of 15 players We use combinations. We want 15 choose 5. We type this in our search engine and we get: [URL='https://www.mathcelebrity.com/permutation.php?num=15&den=5&pl=Combinations']15C5[/URL] = [B]3,003 different five player rosters[/B]

How many wheels would 9 tricycles have?
How many wheels would 9 tricycles have? Each tricycle has 3 wheels, so the total wheels is found below: Total Wheels = 3 wheels per tricycle * 9 tricycles Total Wheels = [B]27[/B]

how much are you paid by the minute if you get $170 a day
how much are you paid by the minute if you get $170 a day? 170 / day * 1 day / 24 hours * 1 hour / 60 minutes 170 / (60*24) per minut 170 / 1440 [B]11.8 cents per minute[/B]

How MUCH Change would be returned from a $50.00 bill for the purchase of 26 stainless Steel 8-in. bo
How MUCH Change would be returned from a $50.00 bill for the purchase of 26 stainless Steel 8-in. bolts at the Price Of 79.5 cents each? Calculate the Stainless Steel Bolts Cost: Stainless Steel Bolts Cost = Number of Stainless Steel Bolts * Price per bolt Stainless Steel Bolts Cost = 26 * 0.795 Stainless Steel Bolts Cost = $20.67 Calculate the change: Change = Cash Offered - Stainless Steel Bolts Cost Change = $50 - $20.67 Change = [B]$29.33[/B]

How much do 10 pieces of candy cost if 1000 pieces cost 100.00?
How much do 10 pieces of candy cost if 1000 pieces cost 100.00? Set up a proportion of pieces to cost 10/x = 1000/100 Divide the right side by 100 on top and bottom 10/x = 10/1 [B]x = 1[/B]

How much is $100 per month forever at 12% per year worth today?
This is a perpetuity with payments assumed at the end of each month. 12% per year = 12/12 = 1% per month The present value of a perpetuity with payments at the end of the month is: Payment/I Plugging in our values, we get: 100/0.01 10,000 [MEDIA=youtube]FFAJnJyAHjw[/MEDIA]

How much money must be invested to accumulate $10,000 in 8 years at 6% compounded annually?
How much money must be invested to accumulate $10,000 in 8 years at 6% compounded annually? We want to know the principle P, that accumulated to $10,000 in 8 years compounding at 6% annually. [URL='https://www.mathcelebrity.com/simpint.php?av=10000&p=&int=6&t=8&pl=Compound+Interest']We plug in our values for the compound interest equation[/URL] and we get: [B]$6,274.12[/B]

How much money will there be in an account at the end of 10 years if $8000 is deposited at a 7.5% an
How much money will there be in an account at the end of 10 years if $8000 is deposited at a 7.5% annual rate that is compounded continuously? Using our [URL='https://www.mathcelebrity.com/simpint.php?av=&p=8000&int=7.5&t=10&pl=Continuous+Interest']continuous compounding calculator[/URL], we get [B]$16,936[/B].

How much sand is needed to fill a pit that measures 8 meters deep, 10 meters wide, and 15 meters lon
How much sand is needed to fill a pit that measures 8 meters deep, 10 meters wide, and 15 meters long? Explain your answer. The pit is a rectangular solid. The volume is: V = l * w * h V = 15 * 10 * 8 V = [B]1,200 cubic meters[/B]

How much would you need to deposit in an account now in order to have $6000 in the account in 10 yea
How much would you need to deposit in an account now in order to have $6000 in the account in 10 years? Assume the account earns 6% interest compounded monthly. We start with a balance of B. We want to know: B(1.06)^10 = 6000 B(1.79084769654) = 6000 Divide each side of the equation by 1.79084769654 to solve for B B = [B]3,350.37[/B]

How much would you need to deposit in an account now in order to have $6000 in the account in 15 yea
How much would you need to deposit in an account now in order to have $6000 in the account in 15 years? Assume the account earns 8% interest compounded monthly. 8% compounded monthly = 8/12 = 0.6667% per month. 15 years = 15*12 = 180 months We want to know an initial balance B such that: B(1.00667)^180 = $6,000 3.306921B = $6,000 Divide each side by 3.306921 [B]B = $1,814.38[/B]

How old am I if 400 reduced by 3 times my age is 124?
How old am I if 400 reduced by 3 times my age is 124? Let my age be a. We're given an algebraic expression: [LIST] [*]3 times my age means we multiply a by 3: 3a [*]400 reduced by 3 times my age means we subtract 3a from 400: [*]400 - 3a [*]The word [I]is[/I] mean an equation, so we set 400 - 3a equal to 124 [/LIST] 400 - 3a = 124 To solve for a, [URL='https://www.mathcelebrity.com/1unk.php?num=400-3a%3D124&pl=Solve']we type this equation into our search engine[/URL] and we get: a = [B]92[/B]

How old am I if: 210 reduced by 3 times my current age is 4 times my current age?
How old am I if: 210 reduced by 3 times my current age is 4 times my current age? Let your current age be a. We're given: [LIST] [*]210 reduced by 3 times current age = 210 - 3a [*]4 times current age = 4a [*]The word [I]is[/I] means equal to. So we set 210 - 3a equal to 4a [/LIST] 210 - 3a = 4a To solve for a, we [URL='https://www.mathcelebrity.com/1unk.php?num=210-3a%3D4a&pl=Solve']type this equation into our search engine [/URL]and we get: a = [B]30[/B]

How old am I of 400 reduced by 2 times my age is 224
How old am I of 400 reduced by 2 times my age is 224 [LIST=1] [*]Let my age be a. [*]2 times my age: 2a [*]400 reduced by 2 times my age: 400 - 2a [*]The phrase [I]is [/I]means an equation. So we set 400 - 2a equal to 224 for our algebraic expression [/LIST] [B]400 - 2a = 224 [/B] If the problem asks you to solve for a, we [URL='https://www.mathcelebrity.com/1unk.php?num=400-2a%3D224&pl=Solve']type this equation into our search engine[/URL] and we get: a = [B]88[/B]

How old is Ruben if he was 28 years old eleven years ago?
How old is Ruben if he was 28 years old eleven years ago? Let's Ruben's age be a. If he was 28 years old 11 years ago, then his age is expressed as: a - 11 = 28 [URL='https://www.mathcelebrity.com/1unk.php?num=a-11%3D28&pl=Solve']Plugging this into our calculator[/URL], we get: a = [B]39[/B]

How To Get $10 in Free Bitcoin
Here's the platform I've made a small fortune on. If you sign up through this link and buy or sell $100 in Bitcoin, you'll get a free $10 in Bitcoin. [URL='https://www.coinbase.com/join/MathCelebrity']Get $10 Free in Bitcoin[/URL]

How to Shade Fractions
The formula is: Numerator = One color Denominator - Numerator = Another Color [MEDIA=youtube]mJD2acCpol4[/MEDIA]

Hubspot Agency Partner Exam
Exam answers and Study Guide for the Hubspot Agency Partner Exam

HubSpot Certification
Exam Answers for the HubSpot Certification exam

HubSpot Content Marketing Exam
Exam answers and Study Guide for the HubSpot Content Marketing Exam

HubSpot Contextual Marketing Exam
Exam answers and Study Guide for the HubSpot Contextual Marketing Exam

HubSpot Email Marketing Exam
Exam answers and Study Guide for the HubSpot Email Marketing Certification Exam

HubSpot Growth Driven Agency Exam
Exam answers and Study Guide for the HubSpot Growth Driven Agency Exam

HubSpot Inbound Certification Exam
Exam answers and Study Guide for the HubSpot Inbound Certification Exam

HubSpot Inbound Sales Exam
Exam answers and Study Guide for the HubSpot Inbound Sales Certification Exam

Hubspot Sales Enablement Exam
Exam answers and Study Guide for the Hubspot Sales Enablement Exam

HubSpot Sales Software Exam
Exam answers and Study Guide for the HubSpot Sales Software Certification Exam

Hunter puts $300.00 into an account to use for school expenses. The account earns 15% interest, comp
Hunter puts $300.00 into an account to use for school expenses. The account earns 15% interest, compounded annually. How much will be in the account after 10 years? Using our [URL='https://www.mathcelebrity.com/compoundint.php?bal=300&nval=10&int=15&pl=Annually']compound interest calculator[/URL], we get: [B]$1,213.67[/B]

Hyperbola
Free Hyperbola Calculator - Given a hyperbola equation, this calculates:
* Equation of the asymptotes
* Intercepts
* Foci (focus) points
* Eccentricity ε
* Latus Rectum
* semi-latus rectum

Hypergeometric Distribution
Free Hypergeometric Distribution Calculator - Calculates the probability of drawing x objects out of a subgroup of k with n possibilities in a total group of N using the hypergeometric distribution.

Hypothesis Testing for a proportion
Free Hypothesis Testing for a proportion Calculator - Performs hypothesis testing using a test statistic for a proportion value.

Hypothesis testing for the mean
Free Hypothesis testing for the mean Calculator - Performs hypothesis testing on the mean both one-tailed and two-tailed and derives a rejection region and conclusion

I am 12 years old. My brother is 5 years older than me. How old is my brother?
I am 12 years old. My brother is 5 years older than me. How old is my brother? Older means we add, so we have: Brother's age = 12 + 5 Brother's age = [B]17[/B]

I am thinking of a number. I multiply it by 14 and add 13. I get the same answer if I multiply by 5
I am thinking of a number. I multiply it by 14 and add 13. I get the same answer if I multiply by 5 and add 283. What is my number? Let the number be n. We're given two expressions: [LIST=1] [*]Multiply it by 14 and add 13: 14n + 13 [*]Multiply by 5 and add 283: 5n + 283 [/LIST] The phrase [I]I get the same answer[/I] means an equation. So we set expression 1 equal to expression 2: 14n + 13 = 5n + 283 To solve for n, we [URL='https://www.mathcelebrity.com/1unk.php?num=14n%2B13%3D5n%2B283&pl=Solve']type this equation into our search engine[/URL] and we get: n = [B]30[/B]

I am thinking of a number. I multiply it by 14 and add 21. I get the same answer if I multiply by 4
I am thinking of a number. I multiply it by 14 and add 21. I get the same answer if I multiply by 4 and add 141. Let the number be n. We have two expressions: [LIST=1] [*]Multiply by 14 and add 21 is written as: 14n + 21 [*]Multiply by 4 and add 141 is written as: 4n + 141 [/LIST] The phrase [I]get the same expression[/I] means they are equal. So we set (1) and (2) equal to each other and solve for n: 14n + 21 = 4n + 141 [URL='https://www.mathcelebrity.com/1unk.php?num=14n%2B21%3D4n%2B141&pl=Solve']Type this equation into our search engine [/URL]to solve for n and we get: n = [B]12[/B]

I am Thinking of a number. I multiply it by 3 and add 67. I get the same answer If i multiply by 6 s
I am Thinking of a number. I multiply it by 3 and add 67. I get the same answer If i multiply by 6 subtract 8. Let the number be n. We're given two equal expressions: [LIST=1] [*]3n + 67 [*]6n - 8 [/LIST] Set the expressions equal to each other since they give the [B]same answer[/B]: 3n + 67 = 6n - 8 We have an equation. [URL='https://www.mathcelebrity.com/1unk.php?num=3n%2B67%3D6n-8&pl=Solve']Type this equation into our search engine and we get[/URL]: n = [B]25[/B]

I am thinking of a number. I multiply it by 7 and add 25. I get the same answer if I multiply by 3 a
I am thinking of a number. I multiply it by 7 and add 25. I get the same answer if I multiply by 3 and add 93. What is my number? Let the number be n. We're given two expressions: [LIST] [*]Multiply the number by 7: 7n [*]add 25: 7n + 25. <-- Expression 1 [*]Multiply by 3: 3n [*]Add 93: 3n + 93 <-- Expression 2 [*]The phrase [I]get the same answer[/I] means both expression 1 and expression 2 are equal. So we set them equal to each other: [/LIST] 7n + 25 = 3n + 93 To solve for n, we [URL='https://www.mathcelebrity.com/1unk.php?num=7n%2B25%3D3n%2B93&pl=Solve']type this equation into our search engine[/URL] and we get: n = [B]17[/B]

I am thinking of a number.i multiply it by 5 and add 139. I get the same number if I multiply by 13
I am thinking of a number.i multiply it by 5 and add 139. I get the same number if I multiply by 13 and subtract 13.What is my number? Take a number (n): The first operation is multiply 5 times n, and then add 39: 5n + 139 The second operation is multiply 13 times n and subtract 13: 13n - 13 Set both operations equal to each other since they result in [I]the same number[/I] 5n + 139 = 13n - 13 [URL='https://www.mathcelebrity.com/1unk.php?num=5n%2B139%3D13n-13&pl=Solve']Type this equation into our search engine[/URL] and we get: [B]n = 19[/B]

I bought four candles that cost 7.00, 8.00, 9.00, and 20.00. If I have a budget of $50, how much can
I bought four candles that cost 7.00, 8.00, 9.00, and 20.00. If I have a budget of $50, how much can I spend on the last candle? [U]Add up your total spending:[/U] 7 + 8 + 9 + 20 = 44 [U]Determine your remaining budget:[/U] Remaining Budget = Total Budget - Spending Remaining Budget = 50 - 44 Remaining Budget = [B]$6.00[/B]

I had $21 to spend on two notebooks after buying them I had $13 how much did each notebook cost
I had $21 to spend on two notebooks after buying them I had $13 how much did each notebook cost? If you had $13, leftover, this means you spent the following on notebooks: $21 - $13 = $7 Cost per notebook = Total Notebook spend / Total Notebooks Cost per notebook = $7/2 Cost per notebook = [B]$3.50[/B]

I had a brother but my brother had no brothers. how can this be
I had a brother but my brother had no brothers. how can this be Because "I" is a female. To solve trick questions like this, you must expand your theory of constraints. Most people look at this problem and see the word [I]brother [/I]twice and limit themselves to thinking in terms of men.

I have $36 dollars and it goes up by 3 every day how much money would I have after 500 days
I have $36 dollars and it goes up by 3 every day how much money would I have after 500 days We have a balance function B(d) where d is the number of days passed since we first had $36: B(d) = 3d + 36 The problem asks for B(500): B(500) = 3(500) + 36 B(500) = 1500 + 36 B(500) = [B]1536[/B]

I have $789 in the bank and make 1% interest a month. How much money do I have at the end of 6 month
I have $789 in the bank and make 1% interest a month. How much money do I have at the end of 6 months? Our balance is found using our compound interest formula: New Balance = Starting Balance * (1 + i/100)^t With I = 1% and t = 6, we have: New Balance = 789 * (1 + 1/100)^6 New Balance = 789 * (1.01)^6 New Balance = 789 * 1.0615201506 New Balance = [B]837.54[/B]

i have 10 apples i take 5 away then add 5 how much is left ?
i have 10 apples i take 5 away then add 5 how much is left ?' [LIST=1] [*]Start with 10 [*]Take away 5: 10 - 5 = 5 [*]Add 5: 5 + 5 = [B]10[/B] [/LIST]

I have 10 boxes, i gave my Mom 2 today. The next day i gave her 7. How much does Mom have now?
I have 10 boxes, i gave my Mom 2 today. The next day i gave her 7. How much does Mom have now? We start with 10 boxes. We give Mom 2, so Mom has 2 boxes: 2 boxes The next day, we give her 7 more boxes. We add 7 boxes to the 2 boxes Mom has: 2 + 7 = [B]9 boxe[/B]s

I have 20 bills consisting of $5 and $10. If the total amount of my money is $130, how many of each
I have 20 bills consisting of $5 and $10. If the total amount of my money is $130, how many of each bill do i have? Let f be $5 bills and t be $10 bills, we have: f + t = 20 5f + 10t = 130 Using our [URL='http://www.mathcelebrity.com/simultaneous-equations.php?term1=f%2Bt%3D20&term2=5f+%2B+10t+%3D+130&pl=Cramers+Method']system of equation solver[/URL], we get: [LIST] [*][B]f = 14[/B] [*][B]t = 6[/B] [/LIST]

i have 25 pencil cases there are p pencils in each pencil case. how many pencils do i have altogethe
i have 25 pencil cases there are p pencils in each pencil case. how many pencils do i have altogether? Total pencils = Number of cases * pencils per case Total pencils = [B]25p[/B]

I have 6 cakes and I want to divide them between 8 people how much does each person get?
6 cakes for 8 people. Divide by 8 people to get the cakes for each person. 6/8 cake per person. However, this fraction can be simplified. Divide the top and bottom by 2. We get 3/4, or 0.75 cake for each person.

I have 8 cookies. My friend at 7/10 of the 8 cookies. How many cookies did he have.
I have 8 cookies. My friend at 7/10 of the 8 cookies. How many cookies did he have? 8 * (7/10) 56/10 Simplify by dividing the top and bottom by 2: [B]28/5 = 5.6 cookies[/B]

I have saved 24 to buy a game which is three-fourth of the total cost of the game how much does the
I have saved 24 to buy a game which is three-fourth of the total cost of the game how much does the game cost ? Let the cost of the game be c. We're given: 3c/4 = 24 To solve this equation for c, we [URL='https://www.mathcelebrity.com/prop.php?num1=3c&num2=24&den1=4&den2=1&propsign=%3D&pl=Calculate+missing+proportion+value']type it in our search engine[/URL] and we get: c = [B]32[/B]

I invest $3000 at 5% interest a year. So far I have made $600 with simple interest. How many years h
I invest $3000 at 5% interest a year. So far I have made $600 with simple interest. How many years have I been investing? Simple interest is calculated using interest * principal. We have 5% * 3000 = $150 interest per year We take our $600 of total interest and divide it by our interest per year to get the total years: $600 / $150 = [B]4 years[/B]

I make 750 toys in 10 hours how many can I make in 4 minutes
I make 750 toys in 10 hours how many can I make in 4 minutes Convert 10 hours to 4 minutes so we can compare minutes to minutes: 10 hours * 60 hours per minute = 600 minutes Now set up a proportion of toys to minutes where t is the number of toys made in 4 minutes: 750/600 = t/4 [URL='https://www.mathcelebrity.com/prop.php?num1=750&num2=t&den1=600&den2=4&propsign=%3D&pl=Calculate+missing+proportion+value']Type this proportion into our search engine and we get[/URL]: t = [B]5[/B]

I need 3 cans of paint for each room and need to paint 20 rooms.
I need 3 cans of paint for each room and need to paint 20 rooms. [U]Calculate the Total Paint Needed[/U] Total Paint Needed = Total Rooms * Cans of paint per room Total Paint Needed = 20 * 3 Total Paint Needed = [B]60[/B]

I need help for this question. Can someone pls help me?
The simple interests earned on the sum of money for 4 years at 7.5% p.a. exceeds that on the same sum for 3.5 years at 8% p.a. by $90. (a)Find the original sum of money. (b)If the original sum of money accumulates to $4612.50 in 5 months at simple interest, find the interests rate per annum.

I only own blue blankets and red blankets. 8 out of every 15 blankets I have are red.
I only own blue blankets and red blankets. 8 out of every 15 blankets I have are red. If have i 45 blankets, how many are blue? If 8 out of 15 blankets are red, then 15 - 8 = 7 are blue So 7 out of every 15 blankets are blue. Set up a proportion of blue blankets to total blankets where b is the number of blue blankets in 45 blankets 7/15 = b/45 Cross multiply: If 2 proportions are equal, then we can do the following: Numerator 1 * Denominator 2 = Denominator 1 * Numerator 2 15b = 45 * 7 15b = 315 To solve for b, divide each side of the equation by 15: 15b/15 = 315/15 Cancel the 15's on the left side and we get: b = [B]21[/B]

I sold 3 units in 563 attempts. How many did I sell per 100 attempts?
I sold 3 units in 563 attempts. How many did I sell per 100 attempts? Set up a proportion of sales to attempts where s is the number of sales for 100 attempts: 3/563 = s/100 [URL='https://www.mathcelebrity.com/prop.php?num1=3&num2=s&den1=563&den2=100&propsign=%3D&pl=Calculate+missing+proportion+value']Typing this in our search engine[/URL], we get: s = [B]0.532 sales[/B]

I think of a number. I multiply it by 6 and add 3. If my answer is 75, calculate the number I starte
I think of a number. I multiply it by 6 and add 3. If my answer is 75, calculate the number I started with. Let the number be n. Multiply it by 6: 6n Add 3: 6n + 3 If the answer is 75, we set 6n + 3 equal to 75: 6n + 3 = 75 We have an equation. To solve for n, [URL='https://www.mathcelebrity.com/1unk.php?num=6n%2B3%3D75&pl=Solve']we type this equation into our search engine[/URL] and get: [B]n = 12[/B]

Identify a pair of factors of -35 that has a sum of -2
Identify a pair of factors of -35 that has a sum of -2. If we [URL='https://www.mathcelebrity.com/factoriz.php?num=-35&pl=Show+Factorization']type in [I]factor -35[/I] into our search engine[/URL], we see 4 factor pairs. When we add up the factors for each pair, we see [B]7, -5[/B] added together gives us 2.

If
If it takes 7 people 3 hours to dig a 50 cubic foot hole, how long will it take two people to dig the hole? 7 people * 3 hours = 21 hours per person 21 hours per person / 2 people = [B]10.5 hours[/B]

if $7000 is invested at 3% compounded monthly, what is the amount after 4 years
if $7000 is invested at 3% compounded monthly, what is the amount after 4 years 4 years = 12 *4 = 48 months since we're compounding monthly. From our c[URL='https://www.mathcelebrity.com/compoundint.php?bal=3000&nval=48&int=3&pl=Monthly']ompound interest calculator,[/URL] we get: [B]$3,381.98[/B]

If $9000 grows to $9720 in 2 years find the simple interest rate.
If $9000 grows to $9720 in 2 years find the simple interest rate. Simple interest formula is Initial Balance * (1 + tn) = Current Balance We have [LIST] [*]Initial Balance = 9000 [*]Current Balance = 9720 [*]n = 2 [/LIST] Plugging in these values, we get: 9000 * (1 + 2t) = 9720 Divide each side by 9000 1 + 2t = 1.08 Subtract 1 from each sdie 2t = 0.08 Divide each side by 2 t = [B]0.04 or 4%[/B]

If (a - b)/b = 3/7, which of the following must also be true?
If (a - b)/b = 3/7, which of the following must also be true? A) a/b = -4/7 B) a/b = 10/7 C) (a + b)/b = 10/7 D) (a - 2b)/b = -11/7 We can rewrite (a - b)/b as: a/b - b/b = 3/7 Since b/b = 1, we have: a/b - 1 = 3/7 Since -1 = -7/7, we have: a/b - 7/7 = 3/7 Add 7/7 to each side: a/b - 7/7 + 7/7 = 3/7 + 7/7 Cancel the 7/7 on the left side, we get: [B]a/b = 10/7 or Answer B [MEDIA=youtube]PKjLuwoso1U[/MEDIA][/B]

If (x - 1)/3 = k and k = 2, what is the value of x?
If (x - 1)/3 = k and k = 2, what is the value of x? If k = 2, we have: (x - 1)/3 = 2 Cross multiply: x - 1 = 3 * 2 x - 1 = 6 [URL='https://www.mathcelebrity.com/1unk.php?num=x-1%3D6&pl=Solve']Type this equation into the search engine[/URL], we get: [B]x = 7[/B]

If .75 inches on a map are equal to 6 miles, how many miles is one inch equal to?
If .75 inches on a map are equal to 6 miles, how many miles is one inch equal to? Using the unit measurement, we have: 6 miles / 0.75 inches = [B]8 miles per inch[/B]

if 1 person with corona spreads it to 3 people and those people spread it to 3 people, how many peop
if 1 person with corona spreads it to 3 people and those people spread it to 3 people, how many people have corona So the spread rate is 3/1 = 3. If 3 people have it, then they spread it to 3 x 3 = [B]9 people[/B].

If 1/2 cup of milk makes 8 donuts. How much cups it takes to make 28 donuts
If 1/2 cup of milk makes 8 donuts. How much cups it takes to make 28 donuts? Set up a proportion of cups to donuts, where c is the number of cups required to make 28 donuts: 1/2/8 = c/28 Cross multiply: 28(1/2) = 8c 8c = 14 [URL='https://www.mathcelebrity.com/1unk.php?num=8c%3D14&pl=Solve']Plugging this equation into our search engine[/URL], we get: [B]c = 1.75[/B]

If 10% of 400 is decreased by 25, the result is
If 10% of 400 is decreased by 25, the result is? [URL='https://www.mathcelebrity.com/perc.php?num=+5&den=+8&num1=+16&pct1=+80&pct2=10&den1=400&pcheck=3&pct=+82&decimal=+65.236&astart=+12&aend=+20&wp1=20&wp2=30&pl=Calculate']10% of 400 using our search engine[/URL] is 40. The phrase [I]decreased by[/I] means we subtract 25 from 40: 40 - 25 = [B]15[/B]

If 100 people are required to introduce themselves to each other and shake hands with each person on
If 100 people are required to introduce themselves to each other and shake hands with each person one time, how many handshakes will take place? We want 100 choose 2 since we have 2 people per handshake: [URL='https://www.mathcelebrity.com/permutation.php?num=100&den=2&pl=Combinations']100C2[/URL] = [B]4950[/B]

If 100 runners went with 4 bicyclists and 5 walkers, how many bicyclists would go with 20 runners an
If 100 runners went with 4 bicyclists and 5 walkers, how many bicyclists would go with 20 runners and 2 walkers? [U]Set up a joint variation equation, for the 100 runners, 4 bicyclists, and 5 walkers:[/U] 100 = 4 * 5 * k 100 = 20k [U]Divide each side by 20[/U] k = 5 <-- Coefficient of Variation [U]Now, take scenario 2 to determine the bicyclists with 20 runners and 2 walkers[/U] 20 = 2 * 5 * b 20 = 10b [U]Divide each side by 10[/U] [B]b = 2[/B]

If 11 times a number is added to twice the number, the result is 104
If 11 times a number is added to twice the number, the result is 104 Let [I]the number[/I] be an arbitrary variable we call x. 11 times a number: 11x Twice the number (means we multiply x by 2): 2x The phrase [I]is added to[/I] means we add 2x to 11x: 11x + 2x Simplify by grouping like terms: (11 + 2)x = 13x The phrase [I]the result is[/I] means an equation, so we set 13x equal to 104: 13x = 104 <-- This is our algebraic expression To solve this equation for x, [URL='https://www.mathcelebrity.com/1unk.php?num=13x%3D104&pl=Solve']we type it in our search engine[/URL] and we get: x = [B]8[/B]

If 115% of a number is 460, what is 75% of the number
If 115% of a number is 460, what is 75% of the number. Let the number be n. We're given: 115% * n = 460 We write 115% of n as 1.15n, so we have: 1.15n = 460 [URL='https://www.mathcelebrity.com/1unk.php?num=1.15n%3D460&pl=Solve']Using our equation calculator[/URL], we get: n = [B]400 [/B] The problem asks for 75% of this number, so we [URL='https://www.mathcelebrity.com/perc.php?num=+5&den=+8&num1=+16&pct1=+80&pct2=75&den1=400&pcheck=3&pct=+82&decimal=+65.236&astart=+12&aend=+20&wp1=20&wp2=30&pl=Calculate']type in [I]75% of 400[/I] into our search engine[/URL] and get: [B]300[/B]

If 12 times a number is added to twice the number, the result is 112
If 12 times a number is added to twice the number, the result is 112. Let the number be n, so we have: 12n + 2n = 112 Combine like terms 14n = 112 Using our [URL='http://www.mathcelebrity.com/1unk.php?num=14n%3D112&pl=Solve']equation solver[/URL], we get [B]n = 8[/B].

If 12 workers can dig a tunnel in 100 days, how long will it take 20 workers to dig the tunnel?
If 12 workers can dig a tunnel in 100 days, how long will it take 20 workers to dig the tunnel? If one person dug the tunnel, it would take 100 days * 12 workers = 1,200 days. 1200 days / 20 workers = [B]60 days[/B]

If 13,754 people voted for a politician in his first election, 15,420 voted for him in his second el
If 13,754 people voted for a politician in his first election, 15,420 voted for him in his second election, and 8,032 voted for him in the first and second elections, how many people voted for this politician in the first or second election? Let P(A) be the first election votes, P(B) be the second election votes, and P(A ? B) be votes for both the first AND the second elections. We want P(A U B). Use our [URL='http://www.mathcelebrity.com/probunion2.php?pa=+13754&pb=15420&paintb=8032&aub=+&pl=Calculate']two event calculator[/URL] P(A U B) = P(A) + P(B) - P(A ? B) P(A U B) = 13,754 + 15,420 - 8032 P(A U B) = 29,174 - 8,032 P(A U B) = [B]21,142[/B]

If 2 inches is about 5 centimeters, how many inches are in 25 centimeters? Choose the proportions th
If 2 inches is about 5 centimeters, how many inches are in 25 centimeters? Choose the proportions that accurately represent this scenario. We set up a proportion of inches to centimeters where i is the number of inches in 25 centimeters: 2/5 = i/25 To solve this proportion for i, we [URL='https://www.mathcelebrity.com/prop.php?num1=2&num2=i&den1=5&den2=25&propsign=%3D&pl=Calculate+missing+proportion+value']type it in our math engine[/URL] and we get: i = [B]10[/B]

If 2 is added to the numerator and denominator it becomes 9/10 and if 3 is subtracted from the numer
If 2 is added to the numerator and denominator it becomes 9/10 and if 3 is subtracted from the numerator and denominator it become 4/5. Find the fractions. Convert 2 to a fraction with a denominator of 10: 20/2 = 10, so we multiply 2 by 10/10: 2*10/10 = 20/10 Add 2 to the numerator and denominator: (n + 2)/(d + 2) = 9/10 Cross multiply and simplify: 10(n + 2) = 9(d + 2) 10n + 20 = 9d + 18 Move constants to right side by subtracting 20 from each side and subtracting 9d: 10n - 9d = -2 Subtract 3 from the numerator and denominator: (n - 3)/(d - 3) = 4/5 Cross multiply and simplify: 5(n - 3) = 4(d - 3) 5n - 15 = 4d - 12 Move constants to right side by adding 15 to each side and subtracting 4d: 5n - 4d = 3 Build our system of equations: [LIST=1] [*]10n - 9d = -2 [*]5n - 4d = 3 [/LIST] Multiply equation (2) by -2: [LIST=1] [*]10n - 9d = -2 [*]-10n + 8d = -6 [/LIST] Now add equation (1) to equation (2) (10 -10)n (-9 + 8)d = -2 - 6 The n's cancel, so we have: -d = -8 Multiply through by -1: d = 8 Now bring back our first equation from before, and plug in d = 8 into it to solve for n: 10n - 9d = -2 10n - 9(8) = -2 10n - 72 = -2 To solve for n, we [URL='https://www.mathcelebrity.com/1unk.php?num=10n-72%3D-2&pl=Solve']plug this equation into our search engine[/URL] and we get: n = 7 So our fraction, n/d = [B]7/8[/B]

If 2 ounces goes into 100 gallons how many ounces is needed for 3000 gallons
If 2 ounces goes into 100 gallons how many ounces is needed for 3000 gallons? Set up a proportion of ounces to gallons. We set o as the number of ounces for 3000 gallons. 2/100 = o/3000 Using our [URL='http://www.mathcelebrity.com/prop.php?num1=2&num2=o&den1=100&den2=3000&propsign=%3D&pl=Calculate+missing+proportion+value']proportion calculator[/URL], we get [B]o = 60[/B].

If 2 times an integer x is increased by 5
If 2 times an integer x is increased by 5 2 times an integer x: 2x The phrase [I]increased by[/I] means we add, so we add 5 to 2x: [B]2x + 5[/B]

If 200 bacteria triple every 1/2 hour, how much bacteria in 3 hours
If 200 bacteria triple every 1/2 hour, how much bacteria in 3 hours Set up the exponential function B(t) where t is the number of tripling times: B(d) = 200 * (3^t) 3 hours = 6 (1/2 hour) periods, so we have 6 tripling times. We want to know B(6): B(6) = 200 * (3^6) B(6) = 200 * 729 B(6) = [B]145,800[/B]

if 200 is divided in the ratio of 1:3:4 , what is the greatest number
if 200 is divided in the ratio of 1:3:4 , what is the greatest number Determine the ratio denominator by adding up the ratio amounts: 1 + 3 + 4 = 8 So we have the following ratios and ratio amounts with our greatest number in bold: [LIST] [*]1/8 * 200 = 25 [*]3/8 * 200 = 75 [*]4/8 * 200 = [B]100[/B] [/LIST]

If 23.8% of a population is 8,212,000. What is the total population?
If 23.8% of a population is 8,212,000. What is the total population? This can be written as [I]23.8% of x is 8212000 [/I] We [URL='https://www.mathcelebrity.com/perc.php?num=+5&den=+8&num1=+16&pct1=+80&pct2=23.8&den1=8212000&pcheck=3&pct=+82&decimal=+65.236&astart=+12&aend=+20&wp1=20&wp2=30&pl=Calculate']type this expression into our search engine[/URL] and we get: [B]1,954,456[/B]

If 25% of a number b is 25.18. What is 20% of b?
If 25% of a number b is 25.18. What is 20% of b? Using our 25% as 0.25, we have: 0.25b = 25.18 Using our [URL='https://www.mathcelebrity.com/1unk.php?num=0.25x%20%3D%2025.18&pl=Solve']equation calculator[/URL], we get: b = 100.72 The question asks what is [URL='https://www.mathcelebrity.com/perc.php?num=+5&den=+8&num1=+16&pct1=+80&pct2=20&den1=100.72&pcheck=3&pct=+82&decimal=+65.236&astart=+12&aend=+20&wp1=20&wp2=30&pl=Calculate']20% of 100.72[/URL]. Using our calculator, we get: [B]20.144[/B]

If 2x + y = 7 and y + 2z = 23, what is the average of x, y, and z?
If 2x + y = 7 and y + 2z = 23, what is the average of x, y, and z? A. 5 B. 7.5 C. 15 D. 12.25 Add both equations to get all variables together: 2x + y + y + 2z = 23 + 7 2x + 2y + 2z = 30 We can divide both sides by 2 to simplify: (2x + 2y + 2z)/2= 30/2 x + y + z = 15 Notice: the average of x, y, and z is: (x + y + z)/3 But x + y + z = 15, so we have: 15/3 = [B]5, answer A[/B] [MEDIA=youtube]tOCAhhfMCLI[/MEDIA]

if 2z-1 is an odd integer what is the preceding odd integer?
if 2z-1 is an odd integer what is the preceding odd integer? The preceding odd integer is found by subtracting 2: 2z - 1 - 2 [B]2z - 3[/B]

If 3 coins are flipped simultaneously, the probability of having three tails is
If 3 coins are flipped simultaneously, the probability of having three tails is... The probability of flipping a head is 1/2. Since each coin flip is independent, we multiply the probabilities together of the three coin flips: P(HHH) = 1/2 * 1/2 * 1/2 P(HHH) = [B]1/8[/B]

If 3 pounds of apples cost 0.90, how much will 10 pounds cost?
Set up a proportion of apples to cost, where a is the unknown cost amount of apples for 10 pounds 3/0.90 = 10/c Use our [URL='http://www.mathcelebrity.com/prop.php?num1=3&num2=10&den1=0.90&den2=c&propsign=%3D&pl=Calculate+missing+proportion+value']proportion calculator[/URL]: [B]c = 3[/B]

If 3 times a number added to 2 is divided by the number plus 4 the result is 4/3
If 3 times a number added to 2 is divided by the number plus 4 the result is 4/3 Take this in pieces, where "a number" means an arbitrary variable, let's call it "x". [LIST=1] [*]3 times a number --> 3x [*]3 times a number added to 2 --> 3x + 2 [*]The number plus 4 --> x + 4 [*]is divided by --> (3x + 2)/(x + 4) [*]the result is 4/3 --> (3x + 2)/(x + 4) = 4/3 [/LIST]

If 3(c + d) = 5, what is the value of c + d?
If 3(c + d) = 5, what is the value of c + d? A) 3/5 B) 5/3 C) 3 D) 5 Divide each side of the equation by 3 to [U]isolate[/U] c + d 3(c + d)/3 = 5/3 Cancel the 3's on the left side, we get: c + d = [B]5/3, or answer B[/B]

If 3.75 inches on a map are equal to 18.75 miles, how many miles are 5 inches equal to?
If 3.75 inches on a map are equal to 18.75 miles, how many miles are 5 inches equal to? Set up a proportion of inches to miles where m is the number of miles for 5 inches: 3.75/18.75 = 5/m Using our [URL='https://www.mathcelebrity.com/proportion-calculator.php?num1=3.75&num2=5&den1=18.75&den2=m&propsign=%3D&pl=Calculate+missing+proportion+value']proportion calculator,[/URL] we get: m = [B]25 miles[/B]

if 3/15 is equivalent to 45/a, find a
if 3/15 is equivalent to 45/a, find a. Set up the proportion: 3/15 = 45/a [URL='https://www.mathcelebrity.com/prop.php?num1=3&num2=45&den1=15&den2=a&propsign=%3D&pl=Calculate+missing+proportion+value']Type this proportion into our search engine[/URL], and we get: a = [B]225[/B]

If 3/5 of a bun is shared equally between 4 people, what fraction of the bun would each receive?
If 3/5 of a bun is shared equally between 4 people, what fraction of the bun would each receive? [URL='https://www.mathcelebrity.com/fraction.php?frac1=3%2F5&frac2=1%2F4&pl=Multiply']We divide 3/5 by 4[/URL] to get [B]3/20[/B]

if 30% of 40% of x is 18.6, find the value of x
if 30% of 40% of x is 18.6, find the value of x 30% is 0.3 40% is 0.4 So we have: 0.3 * 0.4 * x = 18.6 Simplifying, we get: 0.12x = 18.6 [URL='https://www.mathcelebrity.com/1unk.php?num=0.12x%3D18.6&pl=Solve']Typing this equation into our search engine[/URL], we get: x = [B]155[/B]

If 3000 is invested at an annual interest rate of 5% and compounded annually, find the balance after
If 3000 is invested at an annual interest rate of 5% and compounded annually, find the balance after 2 years. Use our [URL='http://www.mathcelebrity.com/simpint.php?av=&p=3000&int=5&t=2&pl=Compound+Interest']compound interest calculator[/URL], we get: Balance = [B]$3,307.50[/B]

If 3a+5b = 98 and a=11, what is the value of a +b
If 3a+5b = 98 and a=11, what is the value of a +b a = 11: 3(11) + 5b = 98 33 + 5b = 98 Using our [URL='https://www.mathcelebrity.com/1unk.php?num=33%2B5b%3D98&pl=Solve']equation solver[/URL], we get: b = 13 a + b = 11 + 13 a + b = [B]24[/B]

If 3r = 18, what is the value of 6r + 3?
2 ways to do this: [B][U]Method 1[/U][/B] If 3r = 18, what is the value of 6r + 3? A) 6 B) 27 C) 36 D) 39 If [URL='https://www.mathcelebrity.com/1unk.php?num=3r%3D18&pl=Solve']we type in the equation 3r = 18 into our search engine[/URL], we get: r = 6 Take r = 6, and subtitute it into 6r + 3: 6(6) + 3 36 + 3 [B]39 or Answer D [U]Method 2:[/U][/B] 6r + 3 = 3r(2) = 3 We're given 3r = 18, so we have: 18(2) + 3 36 + 3 [B]39 or Answer D [MEDIA=youtube]ty3Nk2al1sE[/MEDIA][/B]

If 3x - y = 12, what is the value of 8^x/2^y
If 3x - y = 12, what is the value of 8^x/2^y We know 8 = 2^3 So using a rule of exponents, we have: (2^3)^x/2^y 2^(3x)/2^y Using another rule of exponents, we rewrite this fraction as: 2^(3x -y) We're given 3x - y = 12, so we have: [B]2^12[/B]

If 4 people have the same 7 shirts, what is the chance that they will wear the same shirt on one day
If 4 people have the same 7 shirts, what is the chance that they will wear the same shirt on one day? [LIST=1] [*]For each person, the probability they all wear the first shirt is 1/4 * 1/4 * 1/4 * 1/4 = 1/256 [*]For each person, the probability they all wear the second shirt is 1/4 * 1/4 * 1/4 * 1/4 = 1/256 [*]For each person, the probability they all wear the third shirt is 1/4 * 1/4 * 1/4 * 1/4 = 1/256 [*]For each person, the probability they all wear the fourth shirt is 1/4 * 1/4 * 1/4 * 1/4 = 1/256 [*]For each person, the probability they all wear the fifth shirt is 1/4 * 1/4 * 1/4 * 1/4 = 1/256 [*]For each person, the probability they all wear the sixth shirt is 1/4 * 1/4 * 1/4 * 1/4 = 1/256 [*]For each person, the probability they all wear the seventh shirt is 1/4 * 1/4 * 1/4 * 1/4 = 1/256 [/LIST] Now, we add up all those probabilities to get our answer, since any of the 7 scenarios above meets the criteria: (1 + 1 + 1 + 1 + 1 + 1 + 1)/256 [B]7/256[/B]

If 4 times a number is added to 9, the result is 49
If 4 times a number is added to 9, the result is 49. [I]A number[/I] means an arbitrary variable, let's call it x. 4 [I]times a number[/I] means we multiply x by 4 4x [I]Added to[/I] 9 means we add 9 to 4x 4x + 9 [I]The result is[/I] means we have an equation, so we set 4x + 9 equal to 49 [B]4x + 9 = 49[/B] <-- This is our algebraic expression To solve this equation, [URL='https://www.mathcelebrity.com/1unk.php?num=4x%2B9%3D49&pl=Solve']we type it in the search engine[/URL] and get x = 10

If 4(x-9)=3x-8x, what is x?
[SIZE=5]If 4(x-9)=3x-8x, what is x? [/SIZE] [SIZE=4]Multiply through: 4x - 36 = 3x - 8x Group like terms: 4x - 36 = -5x [/SIZE] [URL='https://www.mathcelebrity.com/1unk.php?num=4x-36%3D-5x&pl=Solve'][SIZE=4]Typing this equation into the search[/SIZE][/URL][SIZE=4][URL='https://www.mathcelebrity.com/1unk.php?num=4x-36%3D-5x&pl=Solve'] engine[/URL], we get: [B]x = 4[/B][/SIZE]

If 4x+7=xy-6, then what is the value of x, in terms of y
If 4x+7=xy-6, then what is the value of x, in terms of y Subtract xy from each side: 4x + 7 - xy = -6 Add 7 to each side: 4x - xy = -6 - 7 4x - xy = -13 Factor out x: x(4 - y) = -13 Divide each side of the equation by (4 - y) [B]x = -13/(4 - y)[/B]

If 5 is transformed into 11, and 12 is transformed into 25, then what does 15 become?
If 5 is transformed into 11, and 12 is transformed into 25, then what does 15 become? Taking a look at potential patterns, we see: 5 * 2 + 1 = 11 12 * 2 + 1 = 25 Using this formula, we have: 15 * 2 + 1 =[B]31[/B]

If 50 out of 250 people die. How many people died per 10 people
If 50 out of 250 people die. How many people died per 10 people We set up a proportion of deaths to total people where d is the number of deaths for 10 people. We have: 50/250 = d/10 To solve this proportion for d, we [URL='https://www.mathcelebrity.com/prop.php?num1=50&num2=d&den1=250&den2=10&propsign=%3D&pl=Calculate+missing+proportion+value']type it in our search engine[/URL] and we get: d = [B]2[/B]

If 5000 dollars is invested in a bank account at an interest rate of 10 per cent per year, find the
If 5000 dollars is invested in a bank account at an interest rate of 10 per cent per year, find the amount in the bank after 9 years if interest is compounded annually. We assume the interest is compounded at the end of the year. Use the [URL='http://www.mathcelebrity.com/annimmpv.php?pv=&av=&pmt=5000&n=9&i=10&check1=1&pl=Calculate']annuity immediate formula[/URL]: [B]67,897.39[/B]

if 65/100 = 117/x, what is x
if 65/100 = 117/x, what is x Type the [URL='https://www.mathcelebrity.com/prop.php?num1=65&num2=117&den1=100&den2=x&propsign=%3D&pl=Calculate+missing+proportion+value']proportion [I]65/100 = 117/x [/I]into our search engine[/URL] and we get: x = [B]180[/B]

If 7 movie tickets cost $63 what is the unit price of the movie tickets?
If 7 movie tickets cost $63 what is the unit price of the movie tickets? Unit Cost = Total Cost / Total Quantity Unit Cost = 63/7 Unit Cost = [B]$9 per ticket[/B]

If 7 times the square of an integer is added to 5 times the integer, the result is 2. Find the integ
If 7 times the square of an integer is added to 5 times the integer, the result is 2. Find the integer. [LIST] [*]Let the integer be "x". [*]Square the integer: x^2 [*]7 times the square: 7x^2 [*]5 times the integer: 5x [*]Add them together: 7x^2 + 5x [*][I]The result is[/I] means an equation, so we set 7x^2 + 5x equal to 2 [/LIST] 7x^2 + 5x = 2 [U]This is a quadratic equation. To get it into standard form, we subtract 2 from each side:[/U] 7x^2 + 5x - 2 = 2 - 2 7x^2 + 5x - 2 = 0 [URL='https://www.mathcelebrity.com/quadratic.php?num=7x%5E2%2B5x-2%3D0&pl=Solve+Quadratic+Equation&hintnum=+0']Type this problem into our search engine[/URL], and we get two solutions: [LIST=1] [*]x = 2/7 [*]x= -1 [/LIST] The problem asks for an integer, so our answer is x[B] = -1[/B]. [U]Let's check our work by plugging x = -1 into the quadratic:[/U] 7x^2 + 5x - 2 = 0 7(-1)^2 + 5(-1) - 2 ? 0 7(1) - 5 - 2 ? 0 0 = 0 So we verified our answer, [B]x = -1[/B].

If 72 is added to a number it will be 4 times as large as it was originally
If 72 is added to a number it will be 4 times as large as it was originally The phrase [I]a number[/I] means an arbitrary variable. Let's call it x. x 72 added to a number: x + 72 4 times as large as it was originally means we take the original number x and multiply it by 4: 4x Now, the phrase [I]it will be[/I] means an equation, so we set x + 72 equal to 4x to get our final algebraic expression: [B]x + 72 = 4x[/B] [B][/B] If the problem asks you to solve for x, we [URL='https://www.mathcelebrity.com/1unk.php?num=x%2B72%3D4x&pl=Solve']type this equation into our search engine[/URL] and we get: x = [B]24[/B]

If 800 feet of fencing is available, find the maximum area that can be enclosed.
If 800 feet of fencing is available, find the maximum area that can be enclosed. Perimeter of a rectangle is: 2l + 2w = P However, we're given one side (length) is bordered by the river and the fence length is 800, so we have: So we have l + 2w = 800 Rearranging in terms of l, we have: l = 800 - 2w The Area of a rectangle is: A = lw Plug in the value for l in the perimeter into this: A = (800 - 2w)w A = 800w - 2w^2 Take the [URL='https://www.mathcelebrity.com/dfii.php?term1=800w+-+2w%5E2&fpt=0&ptarget1=0&ptarget2=0&itarget=0%2C1&starget=0%2C1&nsimp=8&pl=1st+Derivative']first derivative[/URL]: A' = 800 - 4w Now set this equal to 0 for maximum points: 4w = 800 [URL='https://www.mathcelebrity.com/1unk.php?num=4w%3D800&pl=Solve']Typing this equation into the search engine[/URL], we get: w = 200 Now plug this into our perimeter equation: l = 800 - 2(200) l = 800 - 400 l = 400 The maximum area to be enclosed is; A = lw A = 400(200) A = [B]80,000 square feet[/B]

If 9 is added to 1/3 of a number, the result is 15. What is the number?
If 9 is added to 1/3 of a number, the result is 15. What is the number? The phrase [I]a number[/I] means an arbitrary variable, let's call it x: x 1/3 of a number means we multiply x by 1/3: x/3 9 is added to 1/3 of a number: x/3 + 9 The phrase [I]the result is[/I] means an equation. so we set x/3 + 9 equal to 15 x/3 + 9 = 15 To solve this equation for x, we [URL='https://www.mathcelebrity.com/1unk.php?num=x%2F3%2B9%3D15&pl=Solve']type it in our search engine[/URL] and we get: x = [B]18[/B]

if 9 times a number is decreased by 6, the result is 111
if 9 times a number is decreased by 6, the result is 111 A number means an arbitrary variable, let's call it x. 9 times a number: 9x Decreased by 6 9x - 6 The result is 11, this means we set 9x - 6 equal to 11 [B]9x - 6 = 11 [/B] To solve this equation for x, use our [URL='http://www.mathcelebrity.com/1unk.php?num=9x-6%3D11&pl=Solve']equation calculator[/URL]

If 9/20 of a salad is eaten, how much is leftover?
If 9/20 of a salad is eaten, how much is leftover? The full salad is 1. Using a common denominator, we have 1 = 20/20 [URL='https://www.mathcelebrity.com/fraction.php?frac1=1&frac2=9%2F20&pl=Subtract']So the leftover is 20/20 - 9/20[/URL] = [B]11/20[/B]

if a + b = 2 and a2 - b2 = -4, what is the value of a - b?
if a+b=2 and a2-b2=-4, what is the value of a-b? a^2 - b^2 = -4 Factor this: (a + b)(a - b) = -4 We know from above, (a +b) = 2, so substitute: 2(a - b) = -4 Divide each side by 2 [B](a - b) = -2[/B]

If A and B are independent events with P(A) = 0.2 and P(B) = 0.6, then P(A U B)=
If A and B are independent events with P(A) = 0.2 and P(B) = 0.6, then P(A U B)=? We know the following formula for the probability of 2 events: P(A U B) = P(A) + P(B) - P(A intersection B) We're told A and B are independent, which makes P(A intersection B) = 0. So we're left with: P(A U B) = P(A) + P(B) - P(A intersection B) P(A U B) = 0.2 + 0.6 - 0 P(A U B) = [B]0.8[/B]

if a and b are odd then a + b is even
if a and b are odd then a + b is even Let a and b be positive odd integers of the form: [LIST] [*]a = 2n + 1 [*]b = 2m + 1 [/LIST] a + b = 2n + 1 + 2m + 1 a + b = 2n + 2m + 1 + 1 Combing like terms, we get: a + b = 2n + 2m + 2 a + b = 2(n + m) + 2 Let k = n + m a + b = 2k + 2 [B]Therefore a + b is even[/B]

If a baby weighs 6lbs and14ozs how much does the baby weigh in ounces
If a baby weighs 6lbs and14ozs how much does the baby weigh in ounces Since a pound = 16oz, we have: ounces = 6*16 + 14 ounces = 96 + 14 ounces =[B] 110[/B]

If a car is traveling at a speed of 60 miles per hour, how many hours will it take for the car to tr
If a car is traveling at a speed of 60 miles per hour, how many hours will it take for the car to travel n miles? n miles / 60 miles per hour = [B]n/60 hours[/B]

if a city grows by 12% per month what is the yearly growth rate
if a city grows by 12% per month what is the yearly growth rate We know that there are 12 months in a year. 12% = 0.12 Annual Growth Rate = (1 + Monthly Growth Rate)^12 - 1 Annual Growth Rate = (1 + 0.12)^12 - 1 Annual Growth Rate = (1.12)^12 - 1 Annual Growth Rate = 3.89597599255 - 1 Annual Growth Rate = 2.90 For our percentage, our annual growth rate is the Annual growth rate * 100% 2.90 * 100% = [B]290%[/B]

If a die is rolled, what is the probability that the number rolled will not be a "5"?
If a die is rolled, what is the probability that the number rolled will not be a "5"? Possible rolls: {1, 2, 3, 4, 5, 6} Probability of not a 5 means: {1, 2, 3, 4, 6} P(Not 6) = 1 - P(6) P(Not 6) = 1 - 1/6 P(Not 6) = [B]5/6[/B]

if a divides b, then a divides bc
if a divides b, then a divides bc Suppose a divides b. Then there exists an integer q such that b = aq, so that bc = a(qc) and a divides bc. Suppose that a divides c. Then there exists an integer k such that c = ak, so that bc = a(kb) and a divides bc.

If a is an even integer and b is an odd integer then prove a ? b is an odd integer
If a is an even integer and b is an odd integer then prove a ? b is an odd integer Let a be our even integer Let b be our odd integer We can express a = 2x (Standard form for even numbers) for some integer x We can express b = 2y + 1 (Standard form for odd numbers) for some integer y a - b = 2x - (2y + 1) a - b = 2x - 2y - 1 Factor our a 2 from the first two terms: a - b = 2(x - y) - 1 Since x - y is an integer, 2(x- y) is always even. Subtracting 1 makes this an odd number. [MEDIA=youtube]GDVuQ7bGHx8[/MEDIA]

If a jar of coins contains 50 half-dollars and 120 quarters, what is the monetary value of the coins
If a jar of coins contains 50 half-dollars and 120 quarters, what is the monetary value of the coins? We use our [URL='https://www.mathcelebrity.com/coinvalue.php?p=+&n=+&d=+&q=120&h=+50&dol=+&pl=Calculate+Coin+Value']coin values calculator[/URL], and we get: [B]$55.00[/B]

If a machine produces 100 bags per minute how long will it take to make 40,000
If a machine produces 100 bags per minute how long will it take to make 40,000 100 bags/ per minute = 40,000 bags / m Cross multiply 100m = 40000 [URL='https://www.mathcelebrity.com/1unk.php?num=100m%3D40000&pl=Solve']Type this equation into the search engine[/URL] and we get: m = [B]400[/B]

if a number is added to its square, it equals 20
if a number is added to its square, it equals 20. Let the number be an arbitrary variable, let's call it n. The square of the number means we raise n to the power of 2: n^2 We add n^2 to n: n^2 + n It equals 20 so we set n^2 + n equal to 20 n^2 + n = 20 This is a quadratic equation. So [URL='https://www.mathcelebrity.com/quadratic.php?num=n%5E2%2Bn%3D20&pl=Solve+Quadratic+Equation&hintnum=+0']we type this equation into our search engine[/URL] to solve for n and we get two solutions: [B]n = (-5, 4)[/B]

if a number is added to its square, the result is 72. find the number
if a number is added to its square, the result is 72. find the number. Let the number be n. We're given: n + n^2 = 72 Subtract 72 from each side, we get: n^2 + n - 72 = 0 This is a quadratic equation. [URL='https://www.mathcelebrity.com/quadratic.php?num=n%5E2%2Bn-72%3D0&pl=Solve+Quadratic+Equation&hintnum=+0']We type this equation into our search engine[/URL], and we get: [B]n = 8 and n = -9[/B]

if a number is decreased by 5, and then the result is multiplied by 2, the result is 26
If a number is decreased by 5, and then the result is multiplied by 2, the result is 26 The phrase [I]a number[/I] means an arbitrary variable, let's call it x: x [I]Decreased by[/I] means we subtract 5 from x: x - 5 Multiply the result by 2: 2(x - 5) The result is 26 means we set 2(x - 5) equal to 26: [B]2(x - 5) = 26[/B]

If a number is increased by 16 and then divided by 3, the result is 8
If a number is increased by 16 and then divided by 3, the result is 8. Let x be the number. We have: (x + 16)/3 = 8 Cross multiply x + 16 = 24 Using our equation calculator, we get: [B]x = 8[/B]

if a number is tripled the result is 60
if a number is tripled the result is 60 The phrase [I]a number[/I] means an arbitrary variable, let's call it x. x Triple the number means we multiply by 3: 3x The phrase [I]the result is[/I] means an equation, so we set 3x equal to 60: [B]3x = 60 <-- This is our algebraic expression [/B] If you want to solve this equation, then [URL='https://www.mathcelebrity.com/1unk.php?num=3x%3D60&pl=Solve']you type in 3x = 60 into the search engine[/URL] and get: x = 20

If a person from Septon at .1 of their cookie, what fraction of the cookie did they eat using base 1
If a person from Septon at .1 of their cookie, what fraction of the cookie did they eat using base 10 (1/7)^1 = 1/7

If a person invests $360 In an account that pays 8% interests compounded annually, find the balance
If a person invests $360 In an account that pays 8% interests compounded annually, find the balance after 5 years [B]$528.95[/B] per our [URL='http://www.mathcelebrity.com/intbal.php?startbal=360&intrate=8&bstart=1%2F1%2F2000&bend=1%2F1%2F2005&pl=Annual+Credit']balance calculator[/URL].

If a rock rolled n meters, how many decimeters did it roll?
If a rock rolled n meters, how many decimeters did it roll? Setup conversion 1 meter = 10 decimeters Therefore, n meters is [B]10n[/B] decimeters

If a speedometer indicates that a car is traveling at 65 kilometers per hour, how fast is the car tr
If a speedometer indicates that a car is traveling at 65 kilometers per hour, how fast is the car traveling in miles per hour? (Round to the nearest tenth.) Set up a proportion of miles per kilometers: 0.621/1 = n/65 Using our [URL='https://www.mathcelebrity.com/proportion-calculator.php?num1=0.621&num2=n&den1=1&den2=65&propsign=%3D&pl=Calculate+missing+proportion+value']proportion calculator,[/URL] we get: n = [B]40.365[/B]

If a statistical test result is not significant at the 0.05 level, then we can conclude: a. It is no
If a statistical test result is not significant at the 0.05 level, then we can conclude: a. It is not significant at 0.01 level b. It is not significant at 0.10 level c. It must be significant at 0.01 level d. It must be significant above 0.05 level [B]b. It is not significant at 0.10 level[/B] [I]As significance level gets lower, the test gets more strict.[/I]

if a teachers salary grows by 4% each year. How many years will it take to double.
if a teachers salary grows by 4% each year. How many years will it take to double. We can use the[URL='https://www.mathcelebrity.com/rule72.php?num=4&pl=Calculate+Rule+of+72+Time'] Rule of 72 at 4%[/URL] to get [B]18 years[/B]

if a train travels at 80 mph for 15 mins, what is the distance traveled?
if a train travels at 80 mph for 15 mins, what is the distance traveled? Let d = distance, r = rate, and t = time, we have the distance equation: D = rt Plugging in our values for r and t, we have: D = 80mph * 15 min Remember our speed is in miles per hour, so 15 min equal 1/4 of an hour D = 80mph * 1/4 D = [B]20 miles[/B]

If a tutor charges $35 an hour and works for 286 minutes, what is the dollar amount she is owed?
If a tutor charges $35 an hour and works for 286 minutes, what is the dollar amount she is owed? Dollar Amount Owed = Hourly Rate * Number of Hours Worked Convert Minutes worked to hours worked Hours worked = Minutes Worked / 60 Hours worked = 286 minutes / 60 minutes per hour Hours worked = 4.77 So now back to our main formula... Dollar Amount Owed = Hourly Rate * Number of Hours Worked Dollar Amount Owed = $35 * 4.77 Dollar Amount Owed = [B]$166.95[/B]

If a universal set contains 250 elements, n(A) = 90, n(B) = 125, and n(A ? B) = 35, find n(A ? B)'.
If a universal set contains 250 elements, n(A) = 90, n(B) = 125, and n(A ? B) = 35, find n(A ? B)'. We know from set theory that: n(A U B) = n(A) + n(B) - n(A ? B) Plugging in our given values, we get: n(A U B) = 90 + 125 - 35 n(A U B) = 180 The problem asks for n(A U B)'. This formula is found with: n(A U B)' = n(U) - n(A U B) n(U) is the universal set which is 250, so we have: n(A U B)' = 250 - 180 n(A U B)' = [B]70[/B]

If a+b=16, then what is 3a+3b=
If a+b=16, then what is 3a+3b= Factor 3a + 3b: 3(a + b) Since we know a+b = 16, we have: 3(16) = [B]48[/B]

If a, b, and c are positive integers such that a^b = x and c^b = y, then xy = ?
If a, b, and c are positive integers such that a^b = x and c^b = y, then xy = ? A) ac^b B) ac^2b C) (ac)^b D) (ac)^2b E) (ac)^b^2 xy = a^b * c^b We can use the Power of a Product Rule a^b * c^b = (ac)^b Therefore: xy = [B](ac)^b - Answer C[/B]

If a/b = 2, what is the value of 4b/a?
If a/b = 2, what is the value of 4b/a? 4b/a = 4(b/a) If a/b = 2, then the reciprocal b/a = 1/2. So we have 4(1/2) = [B]2[/B]

If a=-9 and b=-6, show that (a-b) unequal (b-a)
If a=-9 and b=-6, show that (a-b) unequal (b-a) [U]a - b:[/U] a - b = -9 - -6 a - b = -9 + 6 a - b = -3 [U]b - a:[/U] b - a = -6 - -9 b - a = -6 + 9 b - a = 3 [B]Since -3 <> 3, then a - b <> b - a[/B]

if a=3, then find the value of 30-5a.
if a=3, then find the value of 30-5a. Using [URL='https://www.mathcelebrity.com/algsubs.php?num=30-5a&varlist=a%3D3&pl=Evaluate']our algebraic substitution calculator[/URL], we get: a = [B]15[/B]

If all A's are B's, then all B's are A's. Is this true?
If all A's are B's, then all B's are A's. Is this true? [B]No.[/B] Example: All dogs are mammals, but not all mammals are dogs. All squares are rectangles, but not all rectangles are squares.

If an employee starts saving with $750 and increases his savings by 8% each month, what will be his
If an employee starts saving with $750 and increases his savings by 8% each month, what will be his total savings after 10 months? Set up the savings function S(m), where m is the number of months and I is the interest rate growth: S(m) = Initial Amount * (1 + i)^m Plugging in our number at m = 10 months we get: S(10) = 750 * (1 + 0.08)^10 S(10) = 750 * 1.08^10 S(10) = [B]$1,619.19[/B]

If approximately 68% of the scores lie between 40.7 and 59.9 , then the approximate value of the sta
If approximately 68% of the scores lie between 40.7 and 59.9 , then the approximate value of the standard deviation for the distribution, according to the empirical rule, is The empirical rule states 68% of the values lie within 1 standard deviation of the mean. The mean is the midpoint of the interval above: (59.9 + 40.7)/2 = 50.3 Standard deviation is the absolute value of the mean - endpoint |59.9 - 50.3| = [B]9.6[/B]

if a^b=x and c^b=y then xy=?
if a^b=x and c^b=y then xy=? xy = a^b * c^b xy = [B](ac)^b[/B]

if ballons are on sale at 15 for$3, whats the cost for a ballon? a)50cents b)25cost c)20cents d)20 d
if ballons are on sale at 15 for$3, whats the cost for a ballon? a)50cents b)25cost c)20cents d)20 dollars Let c be the cost of 1 balloon. We set up a proportion of balloons to cost: 15/3 = 1/c To solve this proportion for c, we [URL='https://www.mathcelebrity.com/prop.php?num1=15&num2=1&den1=3&den2=c&propsign=%3D&pl=Calculate+missing+proportion+value']type it in our search engine[/URL] and we get: c = [B]0.2 or 20 cents[/B]

If ben recently paid a $3.77 fine for a book that was 13 days late, what is the daily fine?
If ben recently paid a $3.77 fine for a book that was 13 days late, what is the daily fine? Daily Fine = Total Fine / Number of Days Daily Fine = $3.77 / 13 days Daily Fine = [B]$0.29[/B]

If Bill's salary is $25 and he gets a 20¢ commission on every newspaper he sells, how many must he s
If Bill's salary is $25 and he gets a 20¢ commission on every newspaper he sells, how many must he sell to make $47 Set up bills Earnings function E(n) where n is the number of newspapers he sells: E(n) =. Cost per newspaper * number of newspapers sold + base salary E(n) = 0.2n + 25 We're asked to find n when E(n) = 47, so we set E(n) = 47 and solve for n: 0.2n + 25 = 47 Using our [URL='https://www.mathcelebrity.com/1unk.php?num=0.2n%2B25%3D47&pl=Solve']equation solver[/URL], we get: n = [B]110[/B]

If cats cost $15 each, what is the cost of n cats?
If cats cost $15 each, what is the cost of n cats? Cost = Price x Quantity Cost = [B]15n[/B]

If cement cost $4 per cubic foot, how much does a cubic yard of cement cost?
If cement cost $4 per cubic foot, how much does a cubic yard of cement cost? There are 3 feet in a yard 3 x 3 x 3 = 27 cubic feet in a yard Total cost = 27 cubic feet * $4 per cubic foot = [B]$108[/B]

If Dan does not study, he will fail his history test. Dan failed his History test. Do we know that h
If Dan does not study, he will fail his history test. Dan failed his History test. Do we know that he did not study? [B]No.[/B] Dan could have failed for a variety of other reasons, including studying but STILL failing.

If Distance equals Speed times Time (D = S x T), then what does time equal in terms of speed and dis
If Distance equals Speed times Time (D = S x T), then what does time equal in terms of speed and distance? Divide each side by S to isolate T: D/S = S x T/S Cancel the S's on the right side: [B]T = D/S[/B]

If EF = 9x - 17, FG = 17x - 14, and EG = 20x + 17, what is FG?
If EF = 9x - 17, FG = 17x - 14, and EG = 20x + 17, what is FG? By segment addition, we know that: EF + FG = EG Substituting in our values for the 3 segments, we get: 9x - 17 + 17x - 14 = 20x + 17 Group like terms and simplify: (9 + 17)x + (-17 - 14) = 20x - 17 26x - 31 = 20x - 17 Solve for [I]x[/I] in the equation 26x - 31 = 20x - 17 [SIZE=5][B]Step 1: Group variables:[/B][/SIZE] We need to group our variables 26x and 20x. To do that, we subtract 20x from both sides 26x - 31 - 20x = 20x - 17 - 20x [SIZE=5][B]Step 2: Cancel 20x on the right side:[/B][/SIZE] 6x - 31 = -17 [SIZE=5][B]Step 3: Group constants:[/B][/SIZE] We need to group our constants -31 and -17. To do that, we add 31 to both sides 6x - 31 + 31 = -17 + 31 [SIZE=5][B]Step 4: Cancel 31 on the left side:[/B][/SIZE] 6x = 14 [SIZE=5][B]Step 5: Divide each side of the equation by 6[/B][/SIZE] 6x/6 = 14/6 x = [B]2.3333333333333[/B]

If Emma reads 1 page of a book in 44 seconds, how many pages will she read in 15 minutes
If Emma reads 1 page of a book in 44 seconds, how many pages will she read in 15 minutes Convert 15 minutes to seconds: 15 minutes = 60 * 15 = 900 seconds Set up a proportion of pages read to seconds where p is the number of pages read in 900 seconds (15 minutes): 1/44 = p/900 [URL='https://www.mathcelebrity.com/prop.php?num1=1&num2=p&den1=44&den2=900&propsign=%3D&pl=Calculate+missing+proportion+value']Typing this proportion into our search engine[/URL], we get: p = [B]20.45[/B]

If f(1) = 2 and f(n) = nf(n-1)+4 then find the value of f(4)
If f(1) = 2 and f(n) = nf(n-1)+4 then find the value of f(4) Find f(2) f(2) = 2*f(2 - 1) + 4 f(2) = 2*f(1) + 4 f(2) = 2*2 + 4 f(2) = 4 + 4 f(2) = 8 Find f(3) f(3) = 2*f(3 - 1) + 4 f(3) = 2*f(2) + 4 f(3) = 2*8+ 4 f(3) = 16 + 4 f(3) = 20 Find f(4) f(4) = 2*f(4 - 1) + 4 f(4) = 2*f(3) + 4 f(4) = 2*20+ 4 f(4) = 40 + 4 f(3) = [B]44[/B]

If f(x) = (3x + 7)^2, then f(1) = ?
If f(x) = (3x + 7)^2, then f(1) = ? A) 10 B) 16 C) 58 D) 79 E) 100 f(1) = (3(1) + 7)^2 f(1) = (3 + 7)^2 f(1) = 10^2 [B]f(1) = 100 - Choice E[/B]

If f(x) = 3 - 2x and g(x) = 1/x + 5 what is the value of (f/g)(8)?
If f(x) = 3 - 2x and g(x) = 1/x + 5 what is the value of (f/g)(8)? Set up (f/g)(x) (3 - 2x)/(1/x + 5) Now find (f/g)(8) (3 - 2(8))/(1/8+ 5) (3 - 16)/(5.125) -13/5.125 [B]2.5366[/B]

If f(x) = 3x + 1 and g(x) = x^2 + 2x, find x when f(g(x)) = 10
If f(x) = 3x + 1 and g(x) = x^2 + 2x, find x when f(g(x)) = 10 [U]Evaluate f(g(x))[/U] f(g(x)) = 3(x^2 + 2x) + 1 f(g(x)) = 3x^2 + 6x + 1 [U]When f(g(x)) = 10, we have[/U] 10 = 3x^2 + 6x + 1 [U]Subtract 10 from each side:[/U] 3x^2 + 6x - 9 = 0 Divide each side of the equation by 3 x^2 + 2x - 3 = 0 Factor, we have: (x + 3)(x - 1) = 0 So x is either [B]1 or -3[/B]

If f(x) = ax^2 + bx + c and f(0) = 1 and f(-1) = 3, what is a - b
If f(x) = ax^2 + bx + c and f(0) = 1 and f(-1) = 3, what is a - b Evaluate f(0) f(0) = a(0^2) + b(0) + c f(0) = a(0) + b(0) + c f(0) = c Since f(0) = 1, we have c = 1 Evaluate f(-1) f(-1) = a(-1^2) + b(-1) + c f(-1) = a(1) - b + c f(-1) = a - b + c Since f(-1) = 3, we have: a - b + c = -3 We learned above that f(0) = 1, so c = 1. Plug c = 1 into f(-1) a - b + 1 = -3 Subtract 1 from each side: a - b + 1 - 1 = -3 - 1 Cancel the 1's on the left side and we get: a - b = [B]-4[/B]

If f={(4,2),(5,1),(6,2),(7,3),(8,6)} then the range of f is what
If f={(4,2),(5,1),(6,2),(7,3),(8,6)} then the range of f is what The range is the full set of all possible y-values: {1, 2, 2, 3, 6} Remove duplicates, we get: [B]{1, 2, 3, 6}[/B]

If FG = 9, GH = 4x, and FH = 7x, what is GH?
If FG = 9, GH = 4x, and FH = 7x, what is GH? By segment addition, we have: FG + GH = FH Substituting in the values given, we have: 9 + 4x = 7x To solve this equation for x, we [URL='https://www.mathcelebrity.com/1unk.php?num=9%2B4x%3D7x&pl=Solve']type it in our math engine[/URL] and we get: x = 3 The question asks for GH, so with x = 3, we have: GH = 4(3) GH = [B]12[/B]

If first integer is 5y, then the next two consecutive integers are
If first integer is 5y, then the next two consecutive integers are integers increase by 1, so we have: [B]5y + 1[/B] 5y + 1 + 1 = [B]5y + 2[/B]

if flip a coin 4 times, what is the probability of getting all 4 tails
if flip a coin 4 times, what is the probability of getting all 4 tails. P(Tails) = 1/2 Each flip is independent, so we have: [URL='https://www.mathcelebrity.com/cointoss.php?hts=TTTT&hct=+2&tct=+1&fct=+5>=no+more+than&nmnl=+2&htpick=tails&calc=1&montect=500&pl=Calculate+Probability']P(TTTT)[/URL] = [B]1/16[/B]

If Frank’s age is double of Willis’ age and the sum of their ages is 42. What are their ages?
If Frank’s age is double of Willis’ age and the sum of their ages is 42. What are their ages? Let Frank's age be f. Let Willis's age be w. We're given two equations: [LIST=1] [*]f = 2w <-- Double means multiply by 2 [*]f + w = 42 [/LIST] Substitute equation (1) into equation (2): 2w + w = 42 To solve for w, [URL='https://www.mathcelebrity.com/1unk.php?num=2w%2Bw%3D42&pl=Solve']type this equation into our search engine[/URL]. We get: w = [B]14 [/B] Now, take w = 14, and substitute it back into equation (1) to solve for f: f = 2(14) f = [B]28[/B]

If from twice a number you subtract four, the difference is twenty
The phrase [I]a number[/I] means an arbitrary variable. We can pick any letter a-z except for i and e. Let's choose x. Twice a number means we multiply x by 2: 2x Subtract four: 2x - 4 The word [I]is [/I]means equal to. We set 2x - 4 equal to 20 for our algebraic expression: [B]2x - 4 = 20 [/B] If the problem asks you to solve for x: we [URL='https://www.mathcelebrity.com/1unk.php?num=2x-4%3D20&pl=Solve']plug this equation into our calculator [/URL]and get x = [B]12[/B]

if g(x) =5x + 3 and g(a) = 14, then what is the value of a?
if g(x) =5x + 3 and g(a) = 14, then what is the value of a? We set g(a) = 5a + 3 = 14 5a + 3 = 14 Using our [URL='https://www.mathcelebrity.com/1unk.php?num=5a%2B3%3D14&pl=Solve']equation solver[/URL], we get: a = [B]2.2[/B]

If H = {(1,-4), (2,-3), (3,-2), (4,-1),...), What is H(9)?
If H = {(1,-4), (2,-3), (3,-2), (4,-1),...), What is H(9)? It looks like each x-coordinate goes up by 1 and each y-coordinate decreases by 1. Their difference is 5. So we have: H(9) = [B](9, 4)[/B]

If Hailey makes $300 every two weeks, how much will Hailey have at the end of the year?
If Hailey makes $300 every two weeks, how much will Hailey have at the end of the year? 52 weeks in a year, which means we have: 52/2 = 26 two week periods 300 * 26 two week periods = [B]7,800[/B]

If half the number is added to twice the number, the answer is 50
If half the number is added to twice the number, the answer is 50. Let the number be n. Half is also written as 0.5, and twice is written by multiplying by 2. We have: 0.5n + 2n = 50 [URL='https://www.mathcelebrity.com/1unk.php?num=0.5n%2B2n%3D50&pl=Solve']Plugging this equation into our search engine[/URL], we get: [B]n = 20[/B]

if hose a can fill up a swimming pool in 6 hours, hose b in 3 hours, hose c in 2 hours, how long wil
if hose a can fill up a swimming pool in 6 hours, hose b in 3 hours, hose c in 2 hours, how long will it take to fill up the pool using all 3 hoses? Let V be the pool's Volume. Each hour, the hoses fill up this much of the pool: [LIST] [*]Hose A, V/6 of the pool [*]Hose B, V/3 of the pool [*]Hose C, V/2 of the pool [/LIST] Effective fill rate is: V/6 + V/3 + V/2 6V/36 + 12V/36 + 18V/36 36V/36 which is volume units per hour Let t = units / rate t = 1 hour, so we have: t = units / rate t = V (volume units) / V (volume units / hour) t = [B]1 hour[/B]

if i = square root of -1 what is the sum (7 + 3i) + (-8 + 9i)
if i = square root of -1 what is the sum (7 + 3i) + (-8 + 9i) We group like terms, and we get: 7 - 8 + (3 + 9)i Simplifying, we get: [B]-1 + 12i[/B]

If I add 8 to the number and then multiply the result by 6 I get the same answer as when I add 58 to
If I add 8 to the number and then multiply the result by 6 I get the same answer as when I add 58 to a number. Form an equation Let the number be n. We're given: 6(n + 8) = n + 58 Multiply through: 6n + 48 = n + 58 To solve this equation for n, [URL='https://www.mathcelebrity.com/1unk.php?num=6n%2B48%3Dn%2B58&pl=Solve']we type it into our search engine[/URL] and we get: n = [B]2[/B]

if I have 1/4 of a tank of gas. what fractions represents the amount required to fill my gas tank th
if I have 1/4 of a tank of gas. what fractions represents the amount required to fill my gas tank the rest of the way? A full tank of gas is 1. You can also write 1 as 4/4. So we take 4/4 - 1/4 = [B]3/4 tank remaining[/B]

If I have a reading average of 2 hours 30 minutes 0 seconds per 93.25 pages, how long would it take
If I have a reading average of 2 hours 30 minutes 0 seconds per 93.25 pages, how long would it take me to read 58 pgs? Set up a proportion, of reading time to pages where m is the number of minutes it takes you to read 58 pages. 2 hours and 30 minutes is: 60(2) + 30 120 + 30 150 minutes Our proportion is: 150/93.25 = m/58 [URL='https://www.mathcelebrity.com/prop.php?num1=150&num2=m&den1=93.25&den2=58&propsign=%3D&pl=Calculate+missing+proportion+value']Type this proportion into the search engine[/URL], and we get: [B]m = 93.3 minutes, or about 1 hour, 33 minutes[/B]

If I make 40,000 dollars every 15 minutes then how long will it take me to make a million
If I make 40,000 dollars every 15 minutes then how long will it take me to make a million Let f be the number of fifteen minute blocks. We're given: 40000f = 1000000 To solve for f, we [URL='https://www.mathcelebrity.com/1unk.php?num=40000f%3D1000000&pl=Solve']type this equation into our search engine[/URL] and we get: f = 25 Total minutes = Fifteen minute blocks (f) * 15 minutes Total minutes = 25 * 15 Total minutes = [B]375 minutes or 6 hours and 15 minutes[/B]

If i triple the number then subtract 7 the answer is 2. What is the number
If i triple the number then subtract 7 the answer is 2. What is the number Let the number be x. Triple the number: 3x Subtract 7 3x - 7 The answer is 2 means we set: [B]3x - 7 = 2[/B] This is our algebraic expression. To solve this, [URL='https://www.mathcelebrity.com/1unk.php?num=3x-7%3D2&pl=Solve']we type this problem into the search engine[/URL] and get [B]x = 3[/B].

If I wanted to buy 40000 balls and they are each 50 cents how much money do I need?
If I wanted to buy 40,000 balls and they are each 50 cents how much money do I need? Cost = Price * Quantity Cost = $0.50 * 40,000 Cost = [B]$20,000[/B]

If it takes 3 people 4 hours to clean a warehouse, how long will it take 4 people to clean the wareh
If it takes 3 people 4 hours to clean a warehouse, how long will it take 4 people to clean the warehouse? 3 people * 4 hours = 12 hours per person 12 hours per person / 4 people = [B]3 hours[/B]

If it takes 6 hours to paint 5/7 of a truck, how long will it take to paint the whole truck?
If it takes 6 hours to paint 5/7 of a truck, how long will it take to paint the whole truck? 6 hours / 5/7 = [B]42/5 hours or 8 & 2/5 hours[/B]

If Jason have 90 pills and have to take 3 pills a day for 3 weeks how many pills do Jason have left?
If Jason have 90 pills and have to take 3 pills a day for 3 weeks how many pills do Jason have left? 1 week = 7 days 3 weeks = 7 days * 3 = 21 days 3 pills per day * 21 days = 63 pills Subtract the 63 pills from the 90 pills to get: 90 - 63 = [B]27 pills left[/B]

If JK = PQ and PQ = ST, then JK=ST
If JK = PQ and PQ = ST, then JK=ST JK = PQ | Given Substitute ST for PQ since PQ = ST | Substitution [B]JK = ST[/B]

If Jody had $3 more she would have twice as much as Lars together they have $60
If Jody had $3 more she would have twice as much as Lars together they have $60. Let j be Jody's money and l be Lars's money. We have two equations: [LIST=1] [*]j + l = 60 [*]j + 3 = 2l [/LIST] Rearrange (2) to solve for j by subtracting 3 j = 2l - 3 Now substitute this into (1) (2l - 3) + l = 60 Combine like terms 3l - 3 = 60 Enter this into our [URL='http://www.mathcelebrity.com/1unk.php?num=3l-3%3D60&pl=Solve']equation calculator[/URL], and we get: [B]l = 21[/B] Now plug l = 21 into our rearranged equation above: j = 2(21) - 3 j = 42 - 3 [B]j = 39[/B]

if joey has 5 swimsuits, 3 bicycles, and 4 pairs of running shoes, how many ways are there for joey
if joey has 5 swimsuits, 3 bicycles, and 4 pairs of running shoes, how many ways are there for joey to choose Using the Fundamental Rule of Counting, we have: 5 swimsuits * 3 bicycles * 4 pairs of running shoes = [B]60 possible choices[/B]

if Logb(5)=3.56 and logb(8)=4.61 then what is logb(40)
if Logb(5)=3.56 and logb(8)=4.61 then what is logb(40) There exists a logarithmic identity which says log(xy) = log(x) + log(y). Since the two logs above have the same base b, we have: x = 5 and y = 8. So we have: logb(40) = logb(5) + logb(8) logb(40) = 3.56 + 4.61 logb(40) = [B]8.17[/B]

if m is odd and n is odd, then mn is odd.
if m is odd and n is odd, then mn is odd. m = 2k +a where a = 0 or 1 n = 2l + b where b = 0 or 1 mn = (2k + a)(2l + b) = 4kl + 2kb + 2al + ab Since mn is odd, ab = 1 since a = 1 and b = 1

If m% of m is 36, then m is?
If m% of m is 36, then m is? m% = m/100, so we have: m/100 * m = 36 m^2/100 = 36 Cross multiply and we get: m^2 = 3600 We use our [URL='https://www.mathcelebrity.com/radex.php?num=sqrt(3600%2F1)&pl=Simplify+Radical+Expression']radical expressions simplifier[/URL] to get: m = [B]60 [MEDIA=youtube]vlsIbZz4dx4[/MEDIA][/B]

If MN is perpendicular to PQ and the slope of PQ is -4 what is the slope for MN
If MN is perpendicular to PQ and the slope of PQ is -4 what is the slope for MN the slope of a line perpendicular to another line is the negative reciprocal. Therefore: Slope of MN = -1/Slope of PQ Slope of MN = -1/-4 Slope of MN = [B]1/4[/B]

If Mr hernandez has 5 times as many students as Mr daniels and together they have 150 students how m
If Mr hernandez has 5 times as many students as Mr daniels and together they have 150 students how many students do each have? Let h = Mr. Hernandez's students and d = Mr. Daniels students. We are given two equations: (1) h = 5d (2) d + h = 150 Substitute equation (1) into equation (2) d + (5d) = 150 Combine like terms: 6d = 150 Divide each side of the equation by 6 to isolate d d = 25 <-- Mr. Daniels Students Now, plug the value for d into equation (1) h = 5(25) h = 125 <-- Mr. Hernandez students

If n is odd, then 3n + 2 is odd
Look at the Contrapositive: If n is even, then 3n + 2 is even... Suppose that the conclusion is false, i.e., that n is even. Then n = 2k for some integer k. Then we have: 3n + 2 = 3(2k) + 2 3n + 2 = 6k + 2 3n + 2 = 2(3k + 1). Thus 3n + 2 is even, because it equals 2j for an integer j = 3k + 1. So 3n + 2 is not odd. We have shown that ¬(n is odd) ? ¬(3n + 2 is odd), therefore, the contrapositive (3n + 2 is odd) ? (n is odd) is also true.

If n represents an odd integer what represents the previous smaller odd integer
If n represents an odd integer what represents the previous smaller odd integer Each odd integer is 2 away from the last one, so the previous smaller odd integer is found by subtracting 2 from n: [B]n - 2[/B]

If n% of 400 is 260, then what is 20% of n?
If n% of 400 is 260, then what is 20% of n? n% = n/100, so we have: n/100 * 400 = 260 400n/100 = 260 4n = 260 Using our equation calculator, we type this in our math engine and we get: n = 65 [URL='https://www.mathcelebrity.com/perc.php?num=+5&den=+8&num1=+16&pct1=+80&pct2=20&den1=65&pcheck=3&pct=+82&decimal=+65.236&astart=+12&aend=+20&wp1=20&wp2=30&pl=Calculate']20% of 65 [/URL]= [B]13 [MEDIA=youtube]3a83xA5Am-M[/MEDIA][/B]

If n(A)=1200, n(B)=1250 and n(AintersectionB)=320, then n(AUB) is
If n(A)=1200, n(B)=1250 and n(AintersectionB)=320, then n(AUB) is We know that: n(AUB) = n(A) + n(B) - n(AintersectionB) Plugging in our given numbers, we get: n(AUB) = 1200 + 1250 - 320 n(AUB) = [B]2130[/B]

If one addend of x is c, what is the other addend
If one addend of x is c, what is the other addend [B]-c [/B] We have x + c and x - c, so the other addend is [B]-c[/B]

If one calculator costs d dollars, what is the cost, in dollars, of 13 calculators?
If one calculator costs d dollars, what is the cost, in dollars, of 13 calculators? Set up cost function C(n), where n is the number of calculators: C(n) = dn C(13) = [B]13d[/B]

If one half of a number is 24, what is twice the number?
If one half of a number is 24, what is twice the number? Let the number be n. We have: n/2 = 24 Cross multiply, we get n = 48 The problem asks for 2n. 2(48) = [B]96[/B]

If p = log2(x), what is the value of log2(2x^3) in terms of p?
If p = log2(x), what is the value of log2(2x^3) in terms of p? A. 6p B. 2p^3 C. 1 + 3p D. 3 + 3p E. 1 + p^3 log2(2x^3) = log2(2) + log2(x^3) log2(2) = 1, so we have: log2(2x^3) = 1 + 3log2(x) Since we're given log2(x) = p, we have: log2(2x^3) = [B]1 + 3p - Answer C [MEDIA=youtube]-fEkVno3bxs[/MEDIA][/B]

If p is inversely proportional to the square of q, and p is 2 when q is 4, determine p when q is equ
If p is inversely proportional to the square of q, and p is 2 when q is 4, determine p when q is equal to 2. We set up the variation equation with a constant k such that: p = k/q^2 [I](inversely proportional means we divide) [/I] When q is 4 and p is 2, we have: 2 = k/4^2 2 = k/16 Cross multiply: k = 2 * 16 k = 32 Now, the problem asks for p when q = 2: p = 32/2^2 p = 32/4 p = [B]8 [MEDIA=youtube]Mro0j-LxUGE[/MEDIA][/B]

If p+4=2 and q-3=2, what is the value of qp?
If p+4=2 and q-3=2, what is the value of qp? Isolate p by subtracting 4 from each side using our [URL='http://www.mathcelebrity.com/1unk.php?num=p%2B4%3D2&pl=Solve']equation calculator[/URL] p = -2 Isolate q by adding 3 to each side using our [URL='http://www.mathcelebrity.com/1unk.php?num=q-3%3D2&pl=Solve']equation calculator[/URL]: q = 5 pq = (-2)(5) [B]pq = -10[/B]

if p=2x is even, then p^2 is also even
if p=2x is even, then p^2 is also even p^2 = 2 * 2 * x^2 p^2 = 4x^2 This is [B]true [/B]because: [LIST] [*]If x is even, then x^2 is even since two evens multiplied by each other is even and 4x^2 is even [*]If x is odd, the x^2 is odd, but 4 times the odd number is always even since even times odd is even [/LIST]

If power is big, you can assume:
If power is big, you can assume: a. The difference between the means is more likely to be detected b. The significance level set by the researcher must be high c. We increase the probability of type I error d. Your study result will be more likely to be inconclusive [B]b. The significance level set by the researcher must be high[/B]

If QR = 16, RS = 4x ? 17, and QS = x + 20, what is RS?
If QR = 16, RS = 4x ? 17, and QS = x + 20, what is RS? From the segment addition rule, we have: QR + RS = QS Plugging our values in for each of these segments, we get: 16 + 4x - 17 = x + 20 To solve this equation for x, [URL='https://www.mathcelebrity.com/1unk.php?num=16%2B4x-17%3Dx%2B20&pl=Solve']we type it in our search engine[/URL] and we get: x = 7 Take x = 7 and substitute it into RS: RS = 4x - 17 RS = 4(7) - 17 RS = 28 - 17 RS = [B]11[/B]

If Quinn has 4 times as many quarters as nickels and they have a combined value of 525 cents, how ma
If Quinn has 4 times as many quarters as nickels and they have a combined value of 525 cents, how many of each coin does he have? Using q for quarters and n for nickels, and using 525 cents as $5.25, we're given two equations: [LIST=1] [*]q = 4n [*]0.25q + 0.05n = 5.25 [/LIST] Substitute equation (1) into equation (2) for q: 0.25(4n) + 0.05n = 5.25 Multiply through and simplify: n + 0.05n = 5.25 To solve this equation for n, we [URL='https://www.mathcelebrity.com/1unk.php?num=n%2B0.05n%3D5.25&pl=Solve']type it in our search engine[/URL] and we get: n = [B]5 [/B] To get q, we plug in n = 5 into equation (1) above: q = 4(5) q = [B]20[/B]

If sales tax is currently 8.2%, write an algebraic expression representing the amount of sales tax y
If sales tax is currently 8.2%, write an algebraic expression representing the amount of sales tax you would have to pay for an item that costs D dollars. 8.2% is 0.082 as a decimal. So we have: Sales Tax Paid = [B]0.082D[/B]

if sc = hr and hr=ab then sc=ab
if sc = hr and hr=ab then sc=ab sc = hr (given) Since hr = ab, we can substitute ab for hr by substitution: [B]sc = ab[/B]

If sin(26)=x what does cos(64) equal?
If sin(26)=x what does cos(64) equal? Using our cofunction calculator, we see the cofunction of sin(26) = cos(64). Therefore, sin(26) = cos(64), so cos(64) = [B]x[/B]

If Susan is riding her bike, she always wears her helmet. Susan is wearing her helmet. Do we know th
If Susan is riding her bike, she always wears her helmet. Susan is wearing her helmet. Do we know that Susan is riding her bike? [B]No.[/B] Susan may also wear her helmet for other activities like skateboarding.

If Susie sleeps for 8 hours, what fraction of the day is she asleep?
If Susie sleeps for 8 hours, what fraction of the day is she asleep? A day has 24 hours, so Susie slept 8/24 of a day. [URL='https://www.mathcelebrity.com/fraction.php?frac1=8%2F24&frac2=3%2F8&pl=Simplify']Using our fraction simplifier[/URL], we get 8/24 = [B]1/3[/B] of a day.

If Tanya eats the box of donuts before she goes to bed, she'll wake up in a fog the next day. If she
If Tanya eats the box of donuts before she goes to bed, she'll wake up in a fog the next day. If she is in a fog, she will not do well on the SAT test that she will take the next day. If she doesn't do well on that SAT test, she will not get a scholarship to college and will have to pay her own way. Can we conclude that if Tanya eats the box of donuts before she goes to age that she will not get a scholarship to college? [B]Yes [/B] [LIST] [*]IF she eats the donuts, she'll wake up in a fog. [*]IF she is in a fog, she will not do well on the test [*]IF she doesn't do well on the test, she will not get a scholarship [/LIST]

if the blackout begins at 5:20 pm and ended at 7:05 pm how long did the black out last?
if the blackout begins at 5:20 pm and ended at 7:05 pm how long did the black out last? [I]add[/I] 2 hours, and we get: 7:20 [I]Subtract[/I] 15 minutes, and we get: 7:05 2 hours - 15 minutes = [B]1 hour and 45 minutes[/B]

If the circumference of a circular rug is 16? feet, then what is the area of the rug in terms of pi
If the circumference of a circular rug is 16? feet, then what is the area of the rug in terms of pi C = 2pir, so we have: C = 16? 16? = 2?r Divide each side by 2?: r = 16?/2? r = 8 Now, the area of a circle A is denoted below: A = ?r^2 Given r = 8 from above, we have: A = ?(8)^2 A = [B]64?[/B]

If the correlation between two variables is close to minus one, the association is: Strong Moderate
If the correlation between two variables is close to minus one, the association is: Strong Moderate Weak None [B]Strong[/B] - Coefficient near +1 or -1 indicate a strong correlation

If the cost of a bat and a baseball combined is $1.10 and the bat cost $1.00 more than the ball how
Let a be the cost of the ball and b be the cost of the bat: We're given 2 equations: [LIST=1] [*]a + b = 1.10 [*]b = a + 1 [/LIST] Substitute equation (2) into equation (1) for b: a + a + 1 = 1.10 Combine like terms: 2a + 1 = 1.10 Subtract 1 from each side: 2a + 1 - 1 = 1.10 - 1 2a = 0.10 Divide each side by 2: 2a/2 = 0.10/2 a = [B]0.05[/B] [MEDIA=youtube]79q346Hy7R8[/MEDIA]

If the cost of each hat is x dollars, what is the cost of y hats?
If the cost of each hat is [I]x[/I] dollars, what is the cost of [I]y[/I] hats? Cost = Price per unit * Quantity Cost = [B]xy dollars [/B]or [B]$xy[/B]

If the diameter of a circle is n, what is the circumference?
If the diameter of a circle is n, what is the circumference? Diameter of a circle = pi(d) Given d = n, we have: Diameter = pi(n)

If the difference of a number and 4 is multiplied by 3 the result is 19
If the difference of a number and 4 is multiplied by 3 the result is 19 The phrase [I]a number[/I] means an arbitrary variable, let's call it x: x The difference of a number and 4: x - 4 The phrase [I]is multiplied by[/I] means we multiply x - 4 by 3: 3(x - 4) The phrase [I]the result is[/I] means equals, so we set 3(x - 4) equal to 19 [B]3(x - 4) = 19 [MEDIA=youtube]Q8bnVJuWeVk[/MEDIA][/B]

If the distribution of IQ scores is bell-shaped, with a mean of 100 and a standard deviation of 15,
If the distribution of IQ scores is bell-shaped, with a mean of 100 and a standard deviation of 15, then approximately ____% of IQ scores are less than 55? A bell-shaped curved implies a normal distribution. By using our [URL='https://www.mathcelebrity.com/probnormdist.php?xone=55&mean=100&stdev=15&n=1&pl=Empirical+Rule']empirical rule calculator[/URL], we see that: 99.7% of all normal distribution values lie within 3 standard deviations of the mean. This means the percent of scores less than 55 which is 3 standard deviations away from the mean is: 100% - 99.7% = [B]0.3%[/B]

If the equation of a line passes through the points (1, 3) and (0, 0), which form would be used to w
If the equation of a line passes through the points (1, 3) and (0, 0), which form would be used to write the equation of the line? [URL='https://www.mathcelebrity.com/slope.php?xone=1&yone=3&slope=+&xtwo=0&ytwo=0&bvalue=+&pl=You+entered+2+points']Typing (1,3),(0,0) into the search engine[/URL], we get a point-slope form: [B]y - 3 = 3(x - 1)[/B] If we want mx + b form, we have: y - 3 = 3x - 3 Add 3 to each side: [B]y = 3x[/B]

if the input is 3 and the output is 13
if the input is 3 and the output is 13 We write this as: [B]f(3) = 13[/B]

If the max time that John can spend on Client A ($20/hr) in one week is 32 hours, and the min time i
If the max time that John can spend on Client A ($20/hr) in one week is 32 hours, and the min time is 8 hours, while the max time on Client B ($14/hr) is 8 hours and the min 5 hours, what is the RANGE (max, min) of total pay he can earn in one week (40 hours) [LIST] [*]Client A Minimum = 20 x 8 hours = $160 [*]Client A Maximum = 20 x 32 hours = $640 [*]Client B Minimum = 14 x 5 hours = $70 [*]Client B Maximum = 14 x 8 hours = $112 [/LIST] [U]The Total Maximum Pay is found by adding Client A Maximum and Client B Maximum[/U] Total Maximum = Client A Maximum + Client B Maximum Total Maximum = 640 + 112 Total Maximum = 752 [U]The Total Minimum Pay is found by adding Client A Minimum and Client B Minimum[/U] Total Minimum = Client A Minimum + Client B Minimum Total Minimum = 160 + 70 Total Minimum = 230 [U]The Range is the difference between the Total maximum and the Total minimum[/U] Range(Total Maximum, Total Minimum) = Total Maximum - Total Minimum Range(752, 230) = 752 - 230 Range(752, 230) = [B]522[/B]

If the number of professors in a college is P and the number is students S, and there are 14 times a
If the number of professors in a college is P and the number is students S, and there are 14 times as many students as professors 14 times as many means we multiply: [B]S = 14P[/B]

If the original price of an item was $30.00 and Joan only paid $24.00 for it, what percentage discou
If the original price of an item was $30.00 and Joan only paid $24.00 for it, what percentage discount did Joan receive on her purchase? She received 6 dollars off of a 30 dollar purchase, so we have 6/30 = 1/5 = 0.2 = [B]20%[/B]

If the P-value of a hypothesis test is 0.40, you conclude a. You accept the null hypothesis b. You r
If the P-value of a hypothesis test is 0.40, you conclude a. You accept the null hypothesis b. You reject the null hypothesis c. You failed to reject the null hypothesis d. You think there is a significant difference [B]c. You failed to reject the null hypothesis[/B] [I]due to a high p value, especially above 0.05[/I]

If the perimeter of a rectangular field is 120 feet and the length of one side is 25 feet, how wide
If the perimeter of a rectangular field is 120 feet and the length of one side is 25 feet, how wide must the field be? The perimeter of a rectangle P, is denoted as: P = 2l + 2w We're given l = 25, and P = 120, so we have 2(25) + 2w = 120 Simplify: 2w + 50 = 120 [URL='https://www.mathcelebrity.com/1unk.php?num=2w%2B50%3D120&pl=Solve']Typing this equation into our search engine[/URL], we get: [B]w = 35[/B]

If the perimeter of a rectangular sign is 44cm and the width is 2cm shorter than half the length, th
If the perimeter of a rectangular sign is 44cm and the width is 2cm shorter than half the length, then what are the length and width? The perimeter (P) of a rectangle is: 2l + 2w = P We're given P = 44, so we substitute this into the rectangle perimeter equation: 2l + 2w = 44 We're also given w = 0.5l - 2. Substitute the into the Perimeter equation: 2l + 2(0.5l - 2) = 44 Multiply through and simplify: 2l + l - 4 = 44 Combine like terms: 3l - 4 = 44 [URL='https://www.mathcelebrity.com/1unk.php?num=3l-4%3D44&pl=Solve']Type this equation into the search engine[/URL], and we get: [B]l = 16[/B] Substitute this back into the equation w = 0.5l - 2 w = 0.5(16) - 2 w = 8 - 2 [B]w = 6[/B]

if the point (.53,y) is on the unit circle in quadrant 1, what is the value of y?
if the point (.53,y) is on the unit circle in quadrant 1, what is the value of y? Unit circle equation: x^2 + y^2 = 1 Plugging in x = 0.53, we get (0.53)^2 + y^2 = 1 0.2809 + y^2 = 1 Subtract 0.2809 from each side: y^2 = 0.7191 y = [B]0.848[/B]

If the price of cheese is $2.35 per pound, what is the cost of 2.45 pounds of cheese?
If the price of cheese is $2.35 per pound, what is the cost of 2.45 pounds of cheese? Since Cost = Price * Quantity, we have: $2.35 per pound * 2.45 pounds = [B]$5.76[/B]

If the probability of an event occurring is 7%, what is the probability of an event not occurring?
If the probability of an event occurring is 7%, what is the probability of an event not occurring? The probability of all event is 1, or 100%. If we treat the success of an event as p, then q is 1 - p. Using percentages, we have: q = 100% - p Given p = 7%, we have: q = 100% - 7% q = [B]93%[/B]

If the probability of getting struck by lighting each year is 1 in 1,000,000, what is the probabilit
If the probability of getting struck by lighting each year is 1 in 1,000,000, what is the probability that you will not be struck by lightning in one year? Our sample space is either getting struck by lightning or NOT getting struck by lightning. So we have: P(Not getting struck by lightning) = 1 - P(Getting struck by lightning) P(Not getting struck by lightning) = 1 - 1/1,000,000 P(Not getting struck by lightning) = [B]999,999/1,000,000[/B]

If the probability of rain is 15%, what is the probability that it won't rain?
If the probability of rain is 15%, what is the probability that it won't rain? If we assign the probability of raining as event A, then A' (A complement) is the probability it won't rain. Since it either rains or doesn't rain are the only two events. There exists an axiom in statistics that states: P(A) + P(A') = 1 Rearranging this, we get: P(A') = 1 - P(A) If we assign the probability of raining as event A which is 0.15, we get: P(A') = 1 - 0.15 P(A') = [B]0.85[/B]

If the probability of winning is X, what is the probability of losing? (Assume there are no ties.)
If the probability of winning is X, what is the probability of losing? (Assume there are no ties.) This means you can either win or lose. Since all probabilities in the sample space must add up to 1, then we have: P(Winning) + P(Losing) = 1 P(Losing) = 1 - P(Winning) Since P(Winning) = X, we have: P(Losing) = [B]1 - X[/B]

If the probability that you will correctly reject a false null hypothesis is 0.80 at 0.05 significan
If the probability that you will correctly reject a false null hypothesis is 0.80 at 0.05 significance level. Therefore, ? is__ and ? is__. [LIST] [*]? represents the significance level of 0.05 [*]We want the Power of a Test which is 1 - ? = 0.8 so ? = 0.20 [/LIST] Our answer is: [B]0.05, 0.20 [/B]

If the raccoon is innocent, then the opossum is not lying. If the rabbit is being truthful, then the
If the raccoon is innocent, then the opossum is not lying. If the rabbit is being truthful, then the opossum is lying. If the rabbit is lying, then it has something to gain from lying. The owl just found out that the rabbit would gain nothing from lying. Is the raccoon guilty or innocent Why? [B]Guilty. [/B] [LIST] [*][B]The only way the rabbit would be lying is if it had something to gain by lying.[/B] [*][B]The owl said that the rabbit had nothing to gain from lying, therefor the rabbit is telling the truth.[/B] [*][B]Because the rabbit is telling the truth, the opossum is lying.[/B] [*][B]if the raccoon was innocent, the opossum would not be lying.[/B] [/LIST]

if the ratio of 2x to 5y is 3 to 4, what is the ratio of x to y?
if the ratio of 2x to 5y is 3 to 4, what is the ratio of x to y? Set up our given ratio: 2x/5y = 3/4 Cross multiply: 2x * 4 = 5y * 3 8x = 15y Divide each side by 8: 8x/8 = 15y/8 x = 15y/8 Now divide each side by y to find x/y: x/y = 15y/8y x/y =[B] 15/8[/B]

If the ratio of private school students to public school students in a city is 4 to 15 and there is
If the ratio of private school students to public school students in a city is 4 to 15 and there is a total of 18,601 students, how many students are in public schools? Since 4 out of 15 are public school students, this means (15 - 4)/15 = 11/15 are public school students. The total public school students are (11/15) * 18601 = 13,640.73. Rounded up, it is [B]13,641[/B].

If the slope is 6 what would the slope of a line parallel to it be?
If the slope is 6 what would the slope of a line parallel to it be? Our rule for the relation of second lines to first lines with regards to slope is this: [LIST] [*]Parallel lines have the [U]same[/U] slope [*]Perpendicular lines have the [U]negative reciprocal[/U] slope [/LIST] So the slope of the line parallel would also be [B]6[/B]

If the speed of an aeroplane is reduced by 40km/hr, it takes 20 minutes more to cover 1200m. Find th
If the speed of an aeroplane is reduced by 40km/hr, it takes 20 minutes more to cover 1200m. Find the time taken by aeroplane to cover 1200m initially. We know from the distance formula (d) using rate (r) and time (t) that: d = rt Regular speed: 1200 = rt Divide each side by t, we get: r = 1200/t Reduced speed. 20 minutes = 60/20 = 1/3 of an hour. So we multiply 1,200 by 3 3600 = (r - 40)(t + 1/3) If we multiply 3 by (t + 1/3), we get: 3t + 1 So we have: 3600 = (r - 40)(3t + 1) Substitute r = 1200/t into the reduced speed equation: 3600 = (1200/t - 40)(3t + 1) Multiply through and we get: 3600 = 3600 - 120t + 1200/t - 40 Subtract 3,600 from each side 3600 - 3600 = 3600 - 3600 - 120t + 1200/t - 40 The 3600's cancel, so we get: - 120t + 1200/t - 40 = 0 Multiply each side by t: -120t^2 - 40t + 1200 = 0 We've got a quadratic equation. To solve for t, [URL='https://www.mathcelebrity.com/quadratic.php?num=-120t%5E2-40t%2B1200%3D0&pl=Solve+Quadratic+Equation&hintnum=+0']we type this in our search engine[/URL] and we get: t = -10/3 or t = 3. Since time [I]cannot[/I] be negative, our final answer is: [B]t = 3[/B]

If the temperature during the day is 6° and the temperature drops 15° after sunset, what is the temp
If the temperature during the day is 6° and the temperature drops 15° after sunset, what is the temperature at night? A drop in temperature means we subtract, so we have: 6 - 15 = [B]-9 or 9 below zero[/B]

If the temperature is dropping at a rate of 2° per hour, how many hours will take to drop 15°
If the temperature is dropping at a rate of 2° per hour, how many hours will take to drop 15° Hours Needed = Total Temperature Drop / Drop per hour Hours Needed = 15/2 Hours Needed = [B]7.5[/B]

If the third of 6 consecutive numbers is 12, what is their sum?
If the third of 6 consecutive numbers is 12, what is their sum? If 12 is the third of 6 consecutive numbers: First consecutive number is 12 - 2 = 10 Second consecutive number = 12 - 1 = 11 Third consecutive number = 12 Fourth consecutive number = 12 + 1 = 13 Fifth consecutive number = 13 + 1 = 14 Sixth consecutive number = 14 + 1 = 15 The sum of all consecutive numbers is: 10 + 11 + 12 + 13 + 14 + 15 =[B] 75[/B]

if the vertex of a parabola is (4,9) what is the axis of symmetry
if the vertex of a parabola is (4,9) what is the axis of symmetry [B]x = 4[/B]

If there are 10^30 grains of sand on Beach A, how many grains of sand are there on a beach the has 1
If there are 10^30 grains of sand on Beach A, how many grains of sand are there on a beach the has 10 times the sand as Beach A? (Express your answer using exponents.) 10^30 * 10 = 10^(30 + 1) = [B]10^31[/B]

If there are 52 cards in a pack, what is the probability of picking 2 kings in a row when the first
If there are 52 cards in a pack, what is the probability of picking 2 kings in a row when the first card picked is not put back? 4 kings in a deck, and 52 cards in a pack. First draw, the probability of drawing a king is 4/52. Second draw, we have 51 cards left since we do not put the first card back, and only 3 Kings left. So the second draw probability for a King is 3/51. Since each draw is independent, we multiply the first and second draws: 4/52 * 3/51 = [B]12/2652 = 0.0045[/B]

If there are 7 bolts in a widget and 6 widgets in an appliance, how many bolts are in 20 appliances?
If there are 7 bolts in a widget and 6 widgets in an appliance, how many bolts are in 20 appliances? Total Bolts = 7 bolts per widget * 6 widgets per appliance * 20 appliances Total Bolts = 7 * 6 * 20 = [B]840 [MEDIA=youtube]jTQXF5mxzbQ[/MEDIA][/B]

If there are 8 girls entered in a race, how many different ways can the runners place first, second,
If there are 8 girls entered in a race, how many different ways can the runners place first, second, and third? We want 8 choose 3, or 8C3. [URL='https://www.mathcelebrity.com/permutation.php?num=8&den=3&pl=Combinations']Type 8C3 into the search engine[/URL], and we get [B]56[/B] different ways to place first, second, and third.

If there are 9000 seconds in 2.5 hours, how many hours are there in 13,500 seconds?
If there are 9000 seconds in 2.5 hours, how many hours are there in 13,500 seconds? Setup a proportion of hours to seconds where h is the number of hours in 13,500 seconds 2.5/9000 = h/13500 Using our [URL='https://www.mathcelebrity.com/proportion-calculator.php?num1=2.5&num2=h&den1=9000&den2=13500&propsign=%3D&pl=Calculate+missing+proportion+value']proportion calculator[/URL] we get: h = [B]3.75 hours[/B]

If there are n days in a vacation, how many hours are there in a vacation?
If there are n days in a vacation, how many hours are there in a vacation? 1 day = 24 hours n days = [B]24n[/B] hours

If thrice a number is increased by 11,the result is 35. What is the number
If thrice a number is increased by 11,the result is 35. What is the number? [LIST] [*]The phrase [I]a number [/I]means an arbitrary variable. Let's call it x. [*]Thrice means multiply by 3, so we have 3x [*]Increased by 11 means we add 11, so we have 3x + 11 [*]The [I]result is[/I] means an equation, so we set 3x + 11 equal to 35 [/LIST] 3x + 11 = 35 <-- This is our algebraic expression The problem ask us to solve the algebraic expression. [URL='https://www.mathcelebrity.com/1unk.php?num=3x%2B11%3D35&pl=Solve']Typing this problem into our search engine[/URL], we get [B]x = 8[/B].

If twice a number is divided by 7, the result is -28
If twice a number is divided by 7, the result is -28. The phrase [I]a number[/I] means an arbitrary variable, let's call it "x". Twice x means we multiply x by 2: 2x Divide this by 7: 2x/7 We set this equal to -28, and we have our algebraic expression: [B]2x/7 = -28 [/B]

if two angles are supplementary and congruent then they are right angles
if two angles are supplementary and congruent then they are right angles Let the first angle be x. Let the second angle be y. Supplementary angles means their sum is 180: x + y = 180 We're given both angles are congruent, meaning equal. So we set x = y: y + y = 180 To solve for y, we [URL='https://www.mathcelebrity.com/1unk.php?num=y%2By%3D180&pl=Solve']type this equation into our search engine[/URL] and we get: y = [B]90. <-- 90 degrees is a right angle, so this is TRUE[/B]

If two coins are flipped, what is the probability that there will not be two heads?
If two coins are flipped, what is the probability that there will not be two heads? There's only one way to flip 2 coins and get 2 heads: P(HH) = 1/2 * 1/2 = 1/4 Which means the probability of NOT getting 2 heads is: 1 - 1/4 = [B]3/4 [MEDIA=youtube]vNbA7vE361M[/MEDIA][/B]

If two consecutive even numbers are added, the sum is equal to 226. What is the smaller of the two n
If two consecutive even numbers are added, the sum is equal to 226. What is the smaller of the two numbers? Let the smaller number be n. The next consecutive even number is n + 2. Add them together to equal 226: n + n + 2 = 226 Solve for [I]n[/I] in the equation n + n + 2 = 226 [SIZE=5][B]Step 1: Group the n terms on the left hand side:[/B][/SIZE] (1 + 1)n = 2n [SIZE=5][B]Step 2: Form modified equation[/B][/SIZE] 2n + 2 = + 226 [SIZE=5][B]Step 3: Group constants:[/B][/SIZE] We need to group our constants 2 and 226. To do that, we subtract 2 from both sides 2n + 2 - 2 = 226 - 2 [SIZE=5][B]Step 4: Cancel 2 on the left side:[/B][/SIZE] 2n = 224 [SIZE=5][B]Step 5: Divide each side of the equation by 2[/B][/SIZE] 2n/2 = 224/2 n = [B]112 [URL='https://www.mathcelebrity.com/1unk.php?num=n%2Bn%2B2%3D226&pl=Solve']Source[/URL][/B]

If two standard dice are rolled, what is the probability that the sum is 3?
If two standard dice are rolled, what is the probability that the sum is 3? [URL='https://www.mathcelebrity.com/2dice.php?gl=1&pl=3&opdice=1&rolist=+&dby=&ndby=&montect=+']Using our 2 dice calculator[/URL], we get: [B]1/18[/B]

If V is the volume of a cube whose side is s, express s in terms of V:
If V is the volume of a cube whose side is s, express s in terms of V: We know the Volume (V) of a cube with side length s is: V = s^3 Take the cube root of each side: V^1/3 = (s^3)^1/3 s = [B]V^1/3[/B]

If x = 2y/3 and y = 18, what is the value of 2x - 3?
If x = 2y/3 and y = 18, what is the value of 2x - 3? A) 21 B) 15 C) 12 D) 10 Substitute the values into the equation: 2(2y/3) - 3 <-- Given x = 2y/3 Simplifying, we have: 4y/3 - 3 Now substitute y = 18 into this: 4(18)/3 - 3 4(6) - 3 24 - 3 [B]21 or Answer A[/B]

If x = b/a, then ax = b
If x = b/a, then ax = b Yes, because we [I]cross multiply[/I] to get: x = b/a ax = b

If x is divided by 9, the remainder is 5. What is the remainder if 3x is divided by 9?
If x is divided by 9, the remainder is 5. What is the remainder if 3x is divided by 9? pick an integer x where when dividing by 9, we get a remainder of 5. 14/9 gives us a remainder of 5. Now multiply 14 by 3: 14 * 3 = 42 [URL='https://www.mathcelebrity.com/modulus.php?num=42mod9&pl=Calculate+Modulus']42/9 gives a remainder of[/URL] [B]6[/B]

If x represents the first, or the smaller, of two consecutive odd integers, express the sum of the
If x represents the first, or the smaller, of two consecutive odd integers, express the sum of the two integers in terms of x If x is the first of two consecutive odd integers, then we find the next consecutive odd integer by adding 2 to x: x + 2 The sum of the two consecutive odd integers is expressed by x + (x + 2) Simplify by grouping like terms, we get: [B]2x + 2[/B]

If x varies directly with y and x = -3 when y = 12, what is the constant of variation?
If x varies directly with y and x = -3 when y = 12, what is the constant of variation? Using our [URL='https://www.mathcelebrity.com/community/forums/calculator-requests.7/create-thread']variation calculator[/URL], we see the constant of variation (k) is: k =[B] -1/4 or -0.25[/B]

If x/2y = 3/4, what is the value of y/x?
If x/2y = 3/4, what is the value of y/x? Cross multiply this proportion: 4x = 3(2y) 4x = 6y Divide each side by x: 4x/x = 6y/x The x's cancel, and we have: 6y/x = 4 Divide each side by 6: 6y/6x = 4/6 The 6's on the left cancel, we have: y/x = 4/6 We can simplify this. [URL='https://www.mathcelebrity.com/fraction.php?frac1=4%2F6&frac2=3%2F8&pl=Simplify']Type in Simplify 4/6 into the search engine[/URL], and we get 2/3. y/x = [B]2/3[/B]

if x2 is added to x, the sum is 42
If x2 is added to x, the sum is 42. x^2 + x = 42 Subtract 42 from both sides: x^2 + x - 42 = 0 We have a quadratic equation. Using our [URL='http://www.mathcelebrity.com/quadratic.php?num=x%5E2%2Bx-42%3D0&pl=Solve+Quadratic+Equation&hintnum=+0']quadratic equation solver[/URL], we get: [B]x = 6 and x = -7 [/B] Since the problem does not state positive number solutions, they are both answers.

if x^2=y^3, for what value of z does x^{3z}= y^9
if x^2=y^3, for what value of z does x^{3z}= y^9 y^9 = y^3 * y^3, so if we square the right side, we must square the left side for equivalence: x^2 * x^2 = x^4 Therefore, x^4 = y^9 Going back to our problem, x^{3z}= y^9, so 3z = 4 Divide each side by 3 to isolate z, and we have: 3z/3 = 4/3 z = [B]4/3[/B]

If y varies directly as x and inversely as z, then which equation describes the relation?
If y varies directly as x and inversely as z, then which equation describes the relation? Directly means we multiply, inversely means we divide, so we have a constant k such that: [B]y = kx/z[/B]

If y=2x and y=18, what is the value of x
If y=2x and y=18, what is the value of x Since y = 2x [B]and[/B] y = 18, we set 2x equals to 18 since they both equal y 2x = 18 [URL='https://www.mathcelebrity.com/1unk.php?num=2x%3D18&pl=Solve']Type this equation into our search engine[/URL] and we get: x = [B]9[/B]

if you add 35 to twice a number, the result is 17. What is the number?
if you add 35 to twice a number, the result is 17. What is the number? A number is represented by a variable, let's call it "x". Twice a number means we multiply by 2 --> 2x Add 35 2x + 35 Now set that entire expression equal to 17 2x + 35 = 17 [URL='http://www.mathcelebrity.com/1unk.php?num=2x%2B35%3D17&pl=Solve']Plug that into the search engine to solve for x[/URL] [B]x = -9[/B]

if you add 7 to 2x, the result is 17
if you add 7 to 2x, the result is 17 Add 7 to 2x: 2x + 7 The phrase [I]the result is[/I] means an equation, so we set 2x + 7 = 17 [B]2x + 7 = 17 [/B] <-- This is our algebraic expression Now, if you want to solve for x, [URL='https://www.mathcelebrity.com/1unk.php?num=2x%2B7%3D17&pl=Solve']type in 2x + 7 = 17 into the search engine[/URL], and we get [B]x = 5[/B].

If you are running 6 miles per hour, then it takes you 10 minutes to run 1 mile. If you are running
If you are running 6 miles per hour, then it takes you 10 minutes to run 1 mile. If you are running 8 miles per hour, it takes you 7.5 minutes to run a mile. What does your speed have to be for your speed in miles per hour to be equal to your mile time in minutes? From above, we have: [LIST] [*]6mph x 10 minutes = 1 mile [*]8mph x 7.5 minutes = 1 mile [/LIST] Notice that mph x minutes = 60 since there are 60 minutes in 1 hour? So we have x mph x y minutes = 60. Since we want mph and y (minutes) = x (mph), we have x^2 = 60 x = sqrt(60) [B]x = 7.746 mph[/B]

If you arrived at your preschool classroom at 7:35 am and stayed until 10:24 am how much time did yo
If you arrived at your preschool classroom at 7:35 am and stayed until 10:24 am how much time did you spend in the classroom? Using our [URL='https://www.mathcelebrity.com/elaptime.php?num1=7%3A35&check1=1&num2=10%3A24&check2=1&pl=Calculate+Elapsed+Time']elapsed time calculator[/URL], we get: [B]2 hours and 49 minutes[/B]

If you buy 5 packages of noodles at $1.48 each whats the total?
If you buy 5 packages of noodles at $1.48 each whats the total? Total = Packages x Cost Per Package Total = 5 x 1.48 Total = [B]$7.40[/B]

if you buy 50 bales of hay at $3.56 each, and then buy an additional 234 bales at $3.33 each, how mu
if you buy 50 bales of hay at $3.56 each, and then buy an additional 234 bales at $3.33 each, how much do you pay for the entire lot of 284 bales? Since cost = price * quantity, we have: Total lot cost = price(1) of hay * bales(1) of hay + price(2) of hay * bales(2) of hay Total lot cost = 3.56 * 50 + 3.33 * 24 Total lot cost = 178 + 79.92 Total lot cost = [B]257.92[/B]

If you buy a computer directly from the manufacturer for $3,509 and agree to repay it in 36 equal in
If you buy a computer directly from the manufacturer for $3,509 and agree to repay it in 36 equal installments at 1.73% interest per month on the unpaid balance, how much are your monthly payments? How much total interest will be paid? [U]Determine the monthly payment[/U] The monthly payment is [B]$114.87[/B] using our [URL='http://www.mathcelebrity.com/annimmpv.php?pv=3059&av=&pmt=&n=36&i=1.73&check1=1&pl=Calculate']annuity calculator[/URL] [U]Determine the total payments made[/U] Total payment is 36 times $114.87 = $4,135.37 [U]Now determine the total interest paid[/U] Take the total payments of $4,135.37 and subtract the original loan of $3,059 to get interest paid of [B]$1,076.37[/B]

If you can buy 1?3 of a box of chocolates for 6 dollars, how much can you purchase for 4 dollars? Wr
If you can buy 1?3 of a box of chocolates for 6 dollars, how much can you purchase for 4 dollars? Write your answer as a fraction of a box. Set up a proportion of dollars to boxes where b is the number of boxes for $4: 6/1/3 = 4/b Cross multiply: 6b = 4/3 Multiply each side by 1/6 to isolate b: b = 4/18 [URL='https://www.mathcelebrity.com/gcflcm.php?num1=4&num2=18&num3=&pl=GCF+and+LCM']Type in GCF(4,18) into the search engine[/URL]. We get a greatest common factor of 2. Divide 4 and 18 in the fraction by 2. We get the reduced fraction of: [B]b = 2/9[/B]

If you divide my brother's age by 3 and then add 20, you will get my age, which is 31. What is my br
If you divide my brother's age by 3 and then add 20, you will get my age, which is 31. What is my brothers age? Let b be the brother's age. We're given the following relationship for the brother's age and my age: b/3 + 20 = 31 Subtract 20 from each side: b/3 + 20 - 20 = 31 - 20 Cancel the 20's on the left side and we get: b/3 = 11 Cross multiply, and we get: b = 3 * 11 b = [B]33 [/B] Check our work using b = 33 for b/3 + 20 = 31: 33/3 + 20 ? 31 11 + 20 ? 31 31 = 31

If you have $272, and you spend $17 each day, how long would it be until you had no money left?
If you have $272, and you spend $17 each day, how long would it be until you had no money left? Let d be the number of days. We have a balance expression of: 272 - 17d We want to know when the balance is 0, so we set 272 - 17d equal to 0. 272 - 17d = 0 To solve for d, we [URL='http://272 - 17d = 0']type this equation into our search engine[/URL] and we get: d = [B]16[/B]

If you have 65$, how many pots can you buy, the pots are 3x + 10
If you have 65$, how many pots can you buy, the pots are 3x + 10 Set 3x + 10 = 65 and solve for x: 3x + 10 = 65 Plugging this into our equation calculator, we get: x = [B]18.33[/B]

if you have a bag with 7 red balls in it and 3 yellow balls in it, whats the probability of pulling
if you have a bag with 7 red balls in it and 3 yellow balls in it, whats the probability of pulling out a yellow ball P(Yellow) = 3/(3 + 7) P(Yellow) = [B]3/10 or 0.3[/B].

If you how 8 dogs, how many total legs are there? How eyes and ears?
If you how 8 dogs, how many total legs are there? How eyes and ears? [U]Calculate total dog legs:[/U] Dog legs = 4 legs per dog * 8 dogs Dog legs = [B]32 legs [/B] [U]Calculate total dog ears:[/U] Dog ears = 2 ears per dog * 8 dogs Dog ears = 16[B] ears [/B] [U]Calculate total dog eyes:[/U] Dog eyes = 2 eyes per dog * 8 dogs Dog eyes = 16 eyes

If you like me, then I like you
If you like me, then I like you So this is a modus ponens problem. If P, then Q. P is you like me Q is I like you If P, you like me Therefore Q, I like you.

If you multiply me by 33 and subtract 20, the result is 46. Who am I?
If you multiply me by 33 and subtract 20, the result is 46. Who am I? [LIST] [*]Start with the variable x [*]Multiply me by 33 = 33x [*]Subtract 20: 33x - 20 [*]The result is 46, means we set this expression equal to 46: 33x - 20 = 46 [/LIST] Run this through our [URL='http://www.mathcelebrity.com/1unk.php?num=33x-20%3D46&pl=Solve']equation calculator[/URL], and we get: [B]x = 2[/B]

if you own 5 pants, 8 shirts, and 3 jackets how many outfits can you make wearing 1 of each item
if you own 5 pants, 8 shirts, and 3 jackets how many outfits can you make wearing 1 of each item? Using the Fundamental Rule of counting, we have: Total Pants * Total Shirts * Total Jackets 5 * 8 * 3 [B]120 [/B]

If you paid $2.95 for 2.5 pounds of apples,what was the cost per pound?
If you paid $2.95 for 2.5 pounds of apples,what was the cost per pound? Using our [URL='http://www.mathcelebrity.com/unit-cost-calculator.php?num=2poundbagfor2.95&pl=Calculate+Unit+Cost']unit cost calculator[/URL], we get $[B]1.48.[/B]

If you put $1 a day away and every day you add a dollar to the previous days amount, how much would
If you put $1 a day away and every day you add a dollar to the previous days amount, how much would you have after 100 days Day 1, you have 1 Day 2, you have 1 + 1 = 2 Day 3, you have 1 + 2 = 3 So our formula for day n is: D(n) = n D(100) = [B]100[/B]

If you take a Uber and they charge $5 just to show up and $1.57 per mile, how much will it cost you
If you take a Uber and they charge $5 just to show up and $1.57 per mile, how much will it cost you to go 12 miles? (Assume no tip.) a. Create an equation from the information above. b. Identify the slope in the equation? c. Calculate the total cost of the ride? 2. With the same charges as #1, how many miles could you go with $50, if you also gave a $7.50 tip? (Challenge Question! Hint, you only have a $50, exactly, with you a. Cost Equation C(m) for m miles is as follows: [B]C(m) = 1.57m + 5 [/B] b. Slope of the equation is the coefficient for m, which is [B]1.57 [/B] c. Total cost of the ride for m = 12 miles is: C(12) = 1.57(12) + 5 C(12) = 18.84 + 5 C(12) = [B]23.84 [/B] d. If you give a 7.50 tip, we subtract the tip from the $50 to start with a reduced amount: 50 - 7.50 = 42.50 So C(m) = 42.50 1.57m + 5 = 42.50 To solve this equation for m, we [URL='https://www.mathcelebrity.com/1unk.php?num=1.57m%2B5%3D42.50&pl=Solve']type it in our search engine[/URL] and we get: m = 23.89 Since we deal in full miles, we round our answer down to m = [B]23[/B]

If you throw a die for two times, what is the probability that you will get a one on the first throw
If you throw a die for two times, what is the probability that you will get a one on the first throw or a one on the second throw (or both)? [LIST] [*]P(1) on first roll and P(anything on second roll) = 1/6 * 1 = 1/6 [*]P(anything on first roll) and P(1) on second roll = 1 * 1/6 = 1/6 [*]Add those together: 1/6 + 1/6 = 2/6 = [B]1/3[/B] [/LIST]

If you toss a fair coin 6 times, what is the probability of getting all tails?
If you toss a fair coin 6 times, what is the probability of getting all tails? We [URL='https://www.mathcelebrity.com/cointoss.php?hts=TTTTTT&hct=+2&tct=+1&fct=+5>=no+more+than&nmnl=+2&htpick=tails&calc=1&montect=500&pl=Calculate+Probability']type in our search engine [I]TTTTTT [/I]and we get[/URL]: P(TTTTTT) = [B]1/64 or 0.015625[/B]

If you triple a number and then add 10, you get one-half of the original number. What is the number
If you triple a number and then add 10, you get one-half of the original number. What is the number? Let the number be n. We have: 3n + 10 = 0.5n Subtract 0.5n from each side 2.5n + 10 = 0 Subtract 10 from each side: 2.5n = -10 Using our [URL='http://www.mathcelebrity.com/1unk.php?num=2.5n%3D-10&pl=Solve']equation calculator,[/URL] we get: [B]n = -4[/B]

If your parents give you $20 per week and $1.50 per chore, how many chores would you have to do to e
If your parents give you $20 per week and $1.50 per chore, how many chores would you have to do to earn a total of $33.50 that week? Let c be the number of chores. We're given the equation: 1.50c + 20 = 33.50 To solve this equation for c, we [URL='https://www.mathcelebrity.com/1unk.php?num=1.50c%2B20%3D33.50&pl=Solve']type it in our search engine [/URL]and we get: c = [B]9[/B]

Imaginary Numbers
Free Imaginary Numbers Calculator - Calculates the imaginary number i where i = √-1 raised to any integer power as well as the product of imaginary numbers of quotient of imaginary numbers

Imagine a researcher posed a null hypothesis that in a certain community, the average energy expendi
Imagine a researcher posed a null hypothesis that in a certain community, the average energy expenditure should be 2,100 calories per day. He randomly sampled 100 people in that community. After he computed the t value by calculating a two-tailed t-statistic, he found that the probability value was 0.10. Thus, he concluded: a. The average energy expenditure was bigger than 2,100 calories per day b. The average energy expenditure was smaller than 2,100 calories per day c. He could not reject the null hypothesis that the average energy expenditure was 2,100 calories per day d. The average energy expenditure was either more than 2,100 calories per day or less than 2,100 calories per day [B]c. He could not reject the null hypothesis that the average energy expenditure was 2,100 calories per day[/B] [I]p-value is higher than 0.05[/I]

Imagine a researcher wanted to test the effect of the new drug on reducing blood pressure. In this s
Imagine a researcher wanted to test the effect of the new drug on reducing blood pressure. In this study, there were 50 participants. The researcher measured the participants' blood pressure before and after the drug intake. If we want to compare the mean blood pressure from the two time periods with a two-tailed t test, how many degrees of freedom are there? a. 49 b. 50 c. 99 d. 100 [B]a. 49[/B] Degrees of Freedom = n - 1 Degrees of Freedom = 50 - 1 Degrees of Freedom = 49

Imagine that a researcher wanted to know the average weight of 5th grade boys in a high school. He r
Imagine that a researcher wanted to know the average weight of 5th grade boys in a high school. He randomly sampled 5 boys from that high school. Their weights were: 120 lbs., 99 lbs, 101 lbs, 87 lbs, 140 lbs. What's the sample [U][B]standard deviation[/B][/U]? [B]20.79182532[/B] using stdev.s in excel or also found on our [URL='http://www.mathcelebrity.com/statbasic.php?num1=120%2C99%2C101%2C87%2C140&num2=+0.2%2C0.4%2C0.6%2C0.8%2C0.9&pl=Number+Set+Basics#standard_deviation']statistics calculator[/URL]

Imagine that a researcher wanted to know the average weight of 5th grade boys in a high school. He r
Imagine that a researcher wanted to know the average weight of 5th grade boys in a high school. He randomly sampled 5 boys from that high school. Their weights were: 120 lbs., 99 lbs, 101 lbs, 87 lbs, 140 lbs. What's the [B][U]standard error of the mean[/U][/B]? 9.29839 using our [URL='http://www.mathcelebrity.com/statbasic.php?num1=120%2C99%2C101%2C87%2C140&num2=+0.2%2C0.4%2C0.6%2C0.8%2C0.9&pl=Number+Set+Basics#standard_error_of_the_mean']statistics calculator[/URL]

Imagine that a researcher wanted to know the average weight of 5th grade boys in a high school. He r
Imagine that a researcher wanted to know the average weight of 5th grade boys in a high school. He randomly sampled 5 boys from that high school. Their weights were: 120 lbs., 99 lbs, 101 lbs, 87 lbs, 140 lbs. The researcher posed a null hypothesis that the average weight for boys in that high school should be 100 lbs. What is the [B][U]absolute value[/U][/B] of calculated t that we use for testing the null hypothesis? Mean is 109.4 and Standard Deviation = 20.79182532 using our [URL='http://www.mathcelebrity.com/statbasic.php?num1=120%2C99%2C101%2C87%2C140&num2=+0.2%2C0.4%2C0.6%2C0.8%2C0.9&pl=Number+Set+Basics']statistics calculator[/URL] Now use those values and calculate the t-value Abs(t value) = (100 - 109.4)/ 20.79182532/sqrt(5) Abs(tvalue) = [B]1.010928029[/B]

Imagine that the diabetic test accurately indicates the disease in 95% of the people who have it. Wh
Imagine that the diabetic test accurately indicates the disease in 95% of the people who have it. What's the miss rate? Miss Rate = 1 - 0.95 [B]Miss Rate = 0.05 or 5%[/B]

In 16 years, Ben will be 3 times as old as he is right now
In 16 years, Ben will be 3 times as old as he is right now. Let Ben's age today be a. We're given: a + 16 = 3a [URL='https://www.mathcelebrity.com/1unk.php?num=a%2B16%3D3a&pl=Solve']Type this equation into the search engine[/URL], and we get: a = [B]8[/B]

In 1910, the population of math valley was 15,000. If the population is increasing at an annual rate
In 1910, the population of math valley was 15,000. If the population is increasing at an annual rate of 2.4%, what was the population in 1965? 1965 - 1910 = 55 years of growth. P(1965) = 15,000 * (1.024)^55 P(1965) = 15,000 * 3.68551018049 P(1965) = 55282.652707 ~ [B]55,283[/B]

In 1996 Ato Boldon of UCLA ran the 100-meter dash in 9.92 seconds. In 1969 John Carlos of San Jose S
In 1996 Ato Boldon of UCLA ran the 100-meter dash in 9.92 seconds. In 1969 John Carlos of San Jose State ran the 100-yard dash in 9.1 seconds. Which runner had the faster average speed? We [URL='https://www.mathcelebrity.com/linearcon.php?quant=100&type=yard&pl=Calculate']convert yards to meters using our conversion calculator[/URL] and we get: 100 yards = 91.44 meters Now we set up a proportion of time per meter: [LIST] [*]Ato Boldon: 9.92/100 = 0.992 per meter [*]John Carlos: 9.1/91.44 = 0.995 per meter [/LIST] [B]Since Ato Boldon's time was [I]less per meter[/I], he had the faster average speed[/B]

In 2000 a company increased its workforce by 50%. In 2001 it decreased its workforce by 50%. How doe
In 2000 a company increased its workforce by 50%. In 2001 it decreased its workforce by 50%. How does the size of its workforce at the end of 2001 compare with the size of the workforce at the beginning of 2000? Let w be the size of the workforce before any changes. We have: [LIST] [*]w(2000) = w(1999) * 1.5 [I](50% increase is the same as multiplying by 1.5)[/I] [*]w(2001) = w(2000)/1.5 [I](50% decrease is the same as dividing by 1.5)[/I] [/LIST] Substitute the first equation back into the second equation w(2001) = w(1999) * 1.5/1.5 Cancel the 1.5 on top and bottom w(2001) = w(1999) This means the workforce had [B]zero net change[/B] from the beginning of 2000 to the end of 2001.

In 2010 a algebra book cost $125. In 2015 the book cost $205. Whats the linear function since 2010?
In 2010 a algebra book cost $125. In 2015 the book cost $205. Whats the linear function since 2010? In 5 years, the book appreciated 205 - 125 = 80 in value. 80/5 = 16. So each year, the book increases 16 in value. Set up the cost function: [B]C(y) = 16y where y is the number of years since 2010[/B]

In 2010, the population of Greenbow, AL was 1,100 people. The population has risen at at rate of 4%
In 2010, the population of Greenbow, AL was 1,100 people. The population has risen at at rate of 4% each year since. Let x = the number of years since 2010 and y = the population of Greenbow. What will the population of Greenbow be in 2022? P(x) = 1,100(1.04)^x x = 2022 - 2010 x = 12 years We want P(12): P(12) = 1,100(1.04)^12 P(12) = 1,100(1.60103221857) P(12) = [B]1,761.14 ~ 1,761[/B]

In 2016 the geese population was at 750. the geese population is expected to grow at a rate of 12% e
In 2016 the geese population was at 750. the geese population is expected to grow at a rate of 12% each year. What is the geese population in 2022? 12% is also 0.12. We have the population growth function: P(t) = 750(1.12)^t 2022 - 2016 is 6 years of growth. We want P(6). P(6) = 750(1.12)^6 P(6) = 750(1.9738) [B]P(6) = 1,480.36 ~ 1,480[/B]

In 2016, National Textile installed a new textile machine in one of its factories at a cost of $300,
In 2016, National Textile installed a new textile machine in one of its factories at a cost of $300,000. The machine is depreciated linearly over 10 years with a scrap value of $10,000. (a) Find an expression for the textile machines book value in the t?th year of use (0 ? t ? 10) We have a straight line depreciation. Book Value is shown on the [URL='http://www.mathcelebrity.com/depsl.php?d=&a=300000&s=10000&n=10&t=3&bv=&pl=Calculate']straight line depreciation calculator[/URL].

In 203.46, which digit is in the hundredths place?
In 203.46, which digit is in the hundredths place? Moving from the right of the decimal place, we have : tenths place = 3 hundredths place = [B]6[/B]

In 32, what is the value of 2
In 32, what is the value of 2 For place value, starting from the right decimal with no decimals, we have: tens, ones 3 is the tens digit 2 is the ones digit 32 = 3 * 10 + 2 * 1 Which means 2 is the [B]ones digit[/B]

in 5 years, sarah will be old enough to vote in an election. the minimum age for voting is at least
in 5 years, sarah will be old enough to vote in an election. the minimum age for voting is at least 18 years. what can you say about how old she is now? 18 - 5 = [B]13 years old[/B]

in 6th grade, veronica reads 45 books. in 7th grade she reads 63 books. what is the percent change?
in 6th grade, veronica reads 45 books. in 7th grade she reads 63 books. what is the percent change? Percent Change = 100% * (New Value - Old Value)/Old Value Percent Change = 100% * (63 - 45)/45 Percent Change = 100% * 18/45 Percent Change = 100% * 0.4 Percent Change = [B]40%[/B] [B] There is a percentage increase[/B]

In 8 years kelly's age will be twice what it is now. How old is kelly?
In 8 years kelly's age will be twice what it is now. How old is kelly? Let Kelly's age be a. In 8 years means we add 8 to a: a + 8 Twice means we multiply a by 2: 2a The phrase [I]will be[/I] means equal to, so we set a + 8 equal to 2a a + 8 = 2a To solve this equation, we [URL='https://www.mathcelebrity.com/1unk.php?num=a%2B8%3D2a&pl=Solve']type it in our math engine[/URL] and we get: a = [B]8 [/B] [U]Evaluate a = 8 and check our work[/U] 8 + 8 ? 2(8) 16 = 16 [MEDIA=youtube]y4jaQpkaJEw[/MEDIA]

In a basketball game, you make 8 of 20 free throws. If you continue this for the next 50 free throws
In a basketball game, you make 8 of 20 free throws. If you continue this for the next 50 free throws, how many can you expect to make? We set up a [U][I]proportion[/I][/U] of made free throws to attempts. 8/20 = m/50 where m is the number of made free throws in 50 attempts. [URL='https://www.mathcelebrity.com/prop.php?num1=8&num2=m&den1=20&den2=50&propsign=%3D&pl=Calculate+missing+proportion+value']We type 8/20 = m/50 into the search engine[/URL] and get [B]m = 20[/B].

In a bike shop they sell bicycles & tricycles. I counted 80 wheels & 34 seats. How many bicycles & t
In a bike shop they sell bicycles & tricycles. I counted 80 wheels & 34 seats. How many bicycles & tricycles were in the bike shop? Let b be the number or bicycles and t be the number of tricycles. Since each bicycle has 2 wheels and 1 seat and each tricycle has 3 wheels and 1 seat, we have the following equations: [LIST=1] [*]2b + 3t = 80 [*]b + t = 34 [/LIST] We can solve this set of simultaneous equations 3 ways: [LIST] [*][URL='https://www.mathcelebrity.com/simultaneous-equations.php?term1=2b+%2B+3t+%3D+80&term2=b+%2B+t+%3D+34&pl=Substitution']Substitution Method[/URL] [*][URL='https://www.mathcelebrity.com/simultaneous-equations.php?term1=2b+%2B+3t+%3D+80&term2=b+%2B+t+%3D+34&pl=Elimination']Elimination Method[/URL] [*][URL='https://www.mathcelebrity.com/simultaneous-equations.php?term1=2b+%2B+3t+%3D+80&term2=b+%2B+t+%3D+34&pl=Cramers+Method']Cramers Rule[/URL] [/LIST] No matter which method we choose, we get the same answer: [LIST] [*][B]b = 22[/B] [*][B]t = 12[/B] [/LIST]

In a booklet there are 25 tickets. Flame needs 75 tickets . How many booklets he need
In a booklet there are 25 tickets. Flame needs 75 tickets . How many booklets he need 25 tickets per booklet * b = 75 Booklets needed (b) = 75 / 25 Booklets needed (b) = [B]3[/B]

In a car lot there are 38 rows with 25 parking spots in each row. How many parking spots are there?
In a car lot there are 38 rows with 25 parking spots in each row. How many parking spots are there? Total parking spots = Number of Rows * Parking spots per row Total parking spots = 38 * 25 Total parking spots = [B]950[/B]

In a certain Algebra 2 class of 26 students, 18 of them play basketball and 7 of them play baseball.
In a certain Algebra 2 class of 26 students, 18 of them play basketball and 7 of them play baseball. There are 5 students who play neither sport. What is the probability that a student chosen randomly from the class plays both basketball and baseball? Students play either basketball only, baseball only, both sports, or no sports. Let the students who play both sports be b. We have: b + 18 + 7 - 5 = 26 <-- [I]We subtract 5 because we don't want to double count the students who played a sport who were counted already [/I] We [URL='https://www.mathcelebrity.com/1unk.php?num=b%2B18%2B7-5%3D26&pl=Solve']type this equation into our search engine[/URL] and get: b = [B]6[/B]

In a certain lot, there are 16 white, 7 red, 8 blue, and 9 black cars. You randomly pick a set of ke
In a certain lot, there are 16 white, 7 red, 8 blue, and 9 black cars. You randomly pick a set of keys to one of the cars. What is the probability of choosing a set of keys to a blue car? [U]Our total cars are:[/U] Total Cars = White Cars + Red Cars + Blue Cars = Black Cars Total Cars = 16 + 7 + 8 + 9 Total Cars = 40 P(Blue) = Blue Cars / Total Cars P(Blue) = 8/40 Using our [URL='https://www.mathcelebrity.com/fraction.php?frac1=8%2F40&frac2=3%2F8&pl=Simplify']fraction simplify calculator[/URL], we get: P(Blue) = [B]1/5[/B]

in a city, the record monthly high temperature for March is 56°F. The record monthly low for March i
in a city, the record monthly high temperature for March is 56°F. The record monthly low for March is -4°F. What is the range of temperatures for the month of March Range = High - Low Range = 56 - -4 Range = 56 + 4 [I]since double negative is positive[/I] Range = [B]60[/B]

In a class of 30 pupils, 18 take Social Studies and 17 take Technical Drawing, 3 take neither. How m
In a class of 30 pupils, 18 take Social Studies and 17 take Technical Drawing, 3 take neither. How many take both Social Studies and Technical Drawing? Let students who take both be b. We have: 18 + 17 + 3 - b = 30 38 - b = 30 Using our [URL='https://www.mathcelebrity.com/1unk.php?num=38-b%3D30&pl=Solve']equation solver[/URL], we get: b = [B]8[/B]

In a class there are 5 more boys than girls. There are 13 students in all. How many boys are there i
In a class there are 5 more boys than girls. There are 13 students in all. How many boys are there in the class? We start by declaring variables for boys and girls: [LIST] [*]Let b be the number of boys [*]Let g be the number of girls [/LIST] We're given two equations: [LIST=1] [*]b = g + 5 [*]b + g = 13 [/LIST] Substitute equation (1) for b into equation (2): g + 5 + g = 13 Grouping like terms, we get: 2g + 5 = 13 Subtract 5 from each side: 2g + 5 - 5 = 13 - 5 Cancel the 5's on the left side and we get: 2g = 8 Divide each side of the equation by 2 to isolate g: 2g/2 = 8/2 Cancel the 2's on the left side and we get: g = 4 Substitute g = 4 into equation (1) to solve for b: b = 4 + 5 b = [B]9[/B]

In a factory that manufactures tires, a machine responsible for molding the tire has a failure rate
In a factory that manufactures tires, a machine responsible for molding the tire has a failure rate of 0.2%. If 1,000 tires are produced in a day, of which 6 are faulty, what is the difference between the experimental probability and the theoretical probability? Theoretical probability = Failure Rate * Tires Theoretical probability = 0.002 * 1000 Theoretical probability = 2 The experimental probability was given as 6, so the difference is: 6 - 2 = [B]4[/B]

In a family of 4 children, what is the probability that all four will be girls?
In a family of 4 children, what is the probability that all four will be girls? P(G) = 1/2 or 0.5 Since each child is independent, we have: 1/2 * 1/2 * 1/2 * 1/2 or (1/2)^4 [B]1/16 or in decimal form, 0.0625[/B]

In a given year, Houston has good air quality 48% of the days, moderate air quality 41% of the days,
In a given year, Houston has good air quality 48% of the days, moderate air quality 41% of the days, and unhealthy air quality 4% of the days. How many days per year do residents have unhealthy air quality? 4% of 365 days in a year = [B]14.6 days. If we are talking full days, we have 14.[/B]

In a golf tournament Ned was 4 under par on Saturday and 5 over or on Sunday. What is the difference
In a golf tournament Ned was 4 under par on Saturday and 5 over or on Sunday. What is the difference in his score on Sunday compared to his score on Saturday Givens and opening thoughts: [LIST] [*]Think of par as 0 or average. [*]Under par is negative [*]Over par is positive [*]We have 4 under par as -4 [*]We have 5 over par as +5 [/LIST] The difference is found by subtracting: +5 - -4 +5 + 4 [B]9 strokes[/B]

In a hurricane the wind pressure varies directly as the square of the wind velocity. If a wind pres
In a hurricane the wind pressure varies directly as the square of the wind velocity. If a wind pressure is a measure of a hurricane's destruction capacity, what happens to this destructive power when the wind speed doubles? Let P = pressure and v = velocity (wind speed) We are given p = v^2 Double velocity, so we have a new pressure P2: P2 = (2v)^2 P2 = 4v^2 Compare the 2: p = v^2 p = 4v^2 Doubling the wind speed [B]quadruples, or 4 times[/B] the pressure.

In a newspaper, it was reported that yearly robberies in Springfield were up 34% to 134 in 2011 from
In a newspaper, it was reported that yearly robberies in Springfield were up 34% to 134 in 2011 from 2010. How many robberies were there in Springfield in 2010? 2010 robberies = 2011 robberies / 1.34 2010 robberies = 134 / 1.34 2010 robberies = [B]100[/B]

In a newspaper, it was reported that yearly robberies in Springfield were up 40% to 77 in 2012 from
In a newspaper, it was reported that yearly robberies in Springfield were up 40% to 77 in 2012 from 2011. How many robberies were there in Springfield in 2011? Let r be the number of robberies in 2011. We have: Robberies in 2012 = Robberies in 2011 * 1.4 77 = r * 1.4 Divide each side by 1.4 [B]r = 55[/B]

In a newspaper, it was reported that yearly robberies in Springfield were up 50% to 351 in 2013 from
In a newspaper, it was reported that yearly robberies in Springfield were up 50% to 351 in 2013 from 2012. How many robberies were there in Springfield in 2012? Let the robberies in 2012 be r. We're given the following equation: 1.5r = 351 <-- We write a 50% increase as 1.5 To solve this equation for r, we [URL='https://www.mathcelebrity.com/1unk.php?num=1.5r%3D351&pl=Solve']type it into our search engine[/URL] and we get: r = [B]234[/B]

In a paper bag, 7 of the 15 marbles are yellow. In a cloth bag, 2 of the 15 marbles are yellow. If
In a paper bag, 7 of the 15 marbles are yellow. In a cloth bag, 2 of the 15 marbles are yellow. If Tim randomly draws one marble from each bag, what is the probability that they are both yellow? Bag 1 probability of drawing yellow is 7/15 Bag 2 probability of drawing yellow is 2/15 Since each event is independent, we multiply each draw to get our final probability: P(yellow Bag 1)(yellow Bag 2) = P(Yellow Bag 1) * P(Yellow Bag 2) P(yellow Bag 1)(yellow Bag 2) = 7/15 * 2/15 P(yellow Bag 1)(yellow Bag 2) = [B]14/225[/B] [URL='https://www.mathcelebrity.com/fraction.php?frac1=14%2F225&frac2=3%2F8&pl=Simplify']Since we cannot simplify this fraction anymore[/URL], our answer is [B]14/225[/B]

In a population of 100 persons, 40 persons like tea and 30 persons like coffee. 10 persons like both
In a population of 100 persons, 40 persons like tea and 30 persons like coffee. 10 persons like both of them. How many persons like either tea or coffee We don't want to count duplicates, so we have the following formula Tea Or Coffee = Tea + Coffee - Both Tea Or Coffee = 40 + 30 - 10 Tea Or Coffee = [B]60[/B]

In a sample of 80 beetles, 50 beetles had 4 spots each, and the rest had 6 spots each. What was the
In a sample of 80 beetles, 50 beetles had 4 spots each, and the rest had 6 spots each. What was the average number of spots per beetle? Show your work below. Average spots per beetle = Total spots for all beetles / Total beetles Average spots per beetle = (50(4) + 6(80 - 50))/80 Average spots per beetle =(200 + 6(30))/80 Average spots per beetle = (200 + 180)/80 Average spots per beetle = (380)/80 Average spots per beetle = [B]4.75 spots[/B]

In a school auditorium there are 36 seats in each row of seats. How many rows are needed for 716 stu
In a school auditorium there are 36 seats in each row of seats. How many rows are needed for 716 students to each have a seat? Total Rows = total Students / Seats Per row Total Rows = 716/36 Total Rows = 19.89 So we need [B]20 full rows[/B]

In a shipment of 330 animals, 125 were hogs, 68 were sheep, and the rest were cattle. Find the numbe
In a shipment of 330 animals, 125 were hogs, 68 were sheep, and the rest were cattle. Find the number of cattle in the shipment. To find the rest (cattle), we subtract off the hogs and sheep from the total. Cattle = Total Animals - Hogs - Sheep Cattle = 330 - 125 - 68 [B]Cattle = 137[/B]

In a survey of 420 people, 230 use samsung mobile, 180 use iphone, 90 use both ,find the number of p
In a survey of 420 people, 230 use samsung mobile, 180 use iphone, 90 use both ,find the number of people who don't use either of them People who don't use both is: 420 - (230 + 180 - 90) 420 - (320) [B]100[/B]

In a theatre audience of 500 people, 80 percent were adults. How many children were in the audience
In a theatre audience of 500 people, 80 percent were adults. How many children were in the audience? If 80% were adults, this means 100% - 80% = 20% were children. [URL='https://www.mathcelebrity.com/perc.php?num=+5&den=+8&num1=+16&pct1=+80&pct2=20&den1=500&pcheck=3&pct=+82&decimal=+65.236&astart=+12&aend=+20&wp1=20&wp2=30&pl=Calculate']We type the expression 20% of 500 into our search engine[/URL] and get [B]100 children[/B]

In an algebra test, the highest grade was 50 points higher than the lowest grade. The sum of the two
In an algebra test, the highest grade was 50 points higher than the lowest grade. The sum of the two grades was 180. Let the high grade be h and the low grade be l. We're given: [LIST=1] [*]h = l + 50 [*]h + l = 180 [/LIST] Substitute equation (1) into equation (2) for h l + 50 + l = 180 To solve this equation, [URL='https://www.mathcelebrity.com/1unk.php?num=l%2B50%2Bl%3D180&pl=Solve']we type it in our search engine[/URL] and we get: l = [B]65 [/B] Now, we take l = 65 and substitute it into equation (1) to solve for h: h = 65 + 50 h = [B]115[/B]

In base 10 the number 25.12 actually means 20 + 5 + 1/10 + 2/100. What does the base 7 number 25.12
In base 10 the number 25.12 actually means 20 + 5 + 1/10 + 2/100. What does the base 7 number 25.12 mean? 2 groups of 7 5 groups of 1 1 group of 1/7 2 groups of 1/49 (1/7)^2 14 + 5 + 1/7 + 2/49

In base 10, the number .1111... approaches 1/9. What does .111111 base 2 approach in base 10?
In base 10, the number .1111... approaches 1/9. What does .111111 base 2 approach in base 10? Base 2 .11111 means: (1/2)^1 + (1/2)^2 + + (1/2)^3 + (1/2)^4 1/2 + 1/4 + 1/8 + 1/16 [B]This approaches 1[/B]

In Chicago it snowed 65% of the day’s in January. How many days did it NOT snow in January?
In Chicago it snowed 65% of the day’s in January. How many days did it NOT snow in January? If it snowed 65% of the days in January, then it did NOT snow 100% - 65% = 35% of the days in January. Since there are 31 days in January, we have: 35% * 31 = 10.85 days in January. So we round down up to [B]11[/B] days.

In how many ways can a committee consisting of 4 faculty members and 5 students be formed if there a
In how many ways can a committee consisting of 4 faculty members and 5 students be formed if there are 8 faculty members and 9 students eligible to serve on the committee? We have 8 choose 4 * 9 choose 5 written as : 8C4 * 9C5 [LIST] [*][URL='http://www.mathcelebrity.com/permutation.php?num=8&den=4&pl=Combinations']8C4[/URL] = 70 [*][URL='http://www.mathcelebrity.com/permutation.php?num=9&den=5&pl=Combinations']9C5[/URL] = 126 [/LIST] Multiply these together to get [B]8,820[/B]

In how many ways can I arrange the 7 letters A, B, C, D, E, F, G?
In how many ways can I arrange the 7 letters A, B, C, D, E, F, G? [B]5,040[/B] from our [URL='http://www.mathcelebrity.com/wordarrange.php?aword=ABCDEFG&pl=Calculate+Letter+Arrangements']letter arrangement calculator[/URL]

In January 2017 the cost of postage stamps increased from 47 cents to 49 cents. What was the percent
In January 2017 the cost of postage stamps increased from 47 cents to 49 cents. What was the percent of increase? [URL='https://www.mathcelebrity.com/percentage-change-calculator.php?num=thecostofpostagestampsincreasedfrom47centsto49cents.whatwasthepercentofincrease&pl=Calculate']Using our percentage change calculator[/URL], we get: [B]4.26% increase[/B]

In Maricopa County, 5 persons are to be elected to the Board of Supervisors. If 8 persons are candid
In Maricopa County, 5 persons are to be elected to the Board of Supervisors. If 8 persons are candidates, how many different arrangements are possible? We want 8 choose 5, or 8C5. [URL='http://www.mathcelebrity.com/permutation.php?num=8&den=5&pl=Combinations']Typing this into the search engine[/URL] we get [B]56[/B].

In one day, a store sells 14 pairs of jeans. The 14 jeans represent 20% of the total number of items
In one day, a store sells 14 pairs of jeans. The 14 jeans represent 20% of the total number of items sold that day. How many items did the store sell in one day? Explain or show how you got your answer. 14 = 20%s where s is the number of items sold in one day. We can write 20% as 0.2, so we have: 0.2s = 14 [URL='https://www.mathcelebrity.com/1unk.php?num=0.2s%3D14&pl=Solve']Type this equation into the search engine[/URL], and we get: s = [B]70[/B]

In order to select new board members, the French club held an election. 63 out of the 90 members of
In order to select new board members, the French club held an election. 63 out of the 90 members of the French club voted in the election. What percentage of the members voted? Using our [URL='http://www.mathcelebrity.com/perc.php?num=63&den=90&pcheck=1&num1=+16&pct1=+80&pct2=+35&den1=+90&pct=+82&decimal=+65.236&astart=+12&aend=+20&wp1=20&wp2=30&pl=Calculate']percentage calculator[/URL], 63 out of 90 is [B]70%[/B]

In order to test if there is a difference between means from two populations, which of following ass
In order to test if there is a difference between means from two populations, which of following assumptions are NOT required? a. The dependent variable scores must be a continuous quantitative variable. b. The scores in the populations are normally distributed. c. Each value is sampled independently from each other value. d. The two populations have similar means [B]a and d [/B] [I]because b and c [U]are[/U] required[/I]

In planning for a particular job, a painter buys $747 worth of materials. When the job is complet
In planning for a particular job, a painter buys $747 worth of materials. When the job is completed, she returns some unused rollers and brushes for a credit of $38. What was the net amount of her bill? Net bill = Purchase - Returns Net bill = 747 - 38 Net bill = [B]709[/B]

In rolling a die, the event E is getting a number greater than or equal to 3. What is the complement
In rolling a die, the event E is getting a number greater than or equal to 3. What is the complement of the event? The complement E' is everything but the event. So we have: E = P(n >= 3) E' = [B]P(n < 3)[/B]

In simple linear regression the slope and the correlation coefficient will have the same signs True
In simple linear regression the slope and the correlation coefficient will have the same signs True False [B]FALSE[/B] - Only if they are normalized

In Super Bowl XXXV, the total number of points scored was 41. The winning team outscored the losing
In Super Bowl XXXV, the total number of points scored was 41. The winning team outscored the losing team by 27 points. What was the final score of the game? In Super Bowl XXXV, the total number of points scored was 41. The winning team outscored the losing team by 27 points. What was the final score of the game? Let w be the winning team's points, and l be the losing team's points. We have two equations: [LIST=1] [*]w + l = 41 [*]w - l = 27 [/LIST] Add the two equations: 2w = 68 Divide each side by 2 [B]w = 34[/B] Substitute this into (1) 34 + l = 41 Subtract 34 from each side [B]l = 7[/B] Check your work: [LIST=1] [*]34 + 7 = 41 <-- check [*]34 - 7 = 27 <-- check [/LIST] The final score of the game was [B]34 to 7[/B]. You could also use our [URL='http://www.mathcelebrity.com/simultaneous-equations.php?term1=w+%2B+l+%3D+41&term2=w+-+l+%3D+27&pl=Cramers+Method']simultaneous equation solver[/URL].

In the bank you will find $24. 3/4 of it is quarters. How much money is that?
In the bank you will find $24. 3/4 of it is quarters. How much money is that? 24 * 3/4 = 6 * 3 = [B]$18[/B]

In the last year a library bought 237 new books and removed 67 books. There were 5745 books in the l
In the last year a library bought 237 new books and removed 67 books. There were 5745 books in the library at the end of the year. How many books were in the library at the start of the year Let the starting book count be b. We have: [LIST] [*]We start with b books [*]Buying 237 books means we add (+237) [*]Removing 67 books means we subtract (-67) [*]We end up with 5745 books [/LIST] Our change during the year is found by the equation: b + 237 - 67 = 5745 To solve for b, we [URL='https://www.mathcelebrity.com/1unk.php?num=b%2B237-67%3D5745&pl=Solve']type this equation into our search engine[/URL] and we get: b = [B]5575[/B]

In the movie Die Hard: With a Vengeance, in one of the action scenes, the characters Mc Clane and Ca
In the movie Die Hard: With a Vengeance, in one of the action scenes, the characters Mc Clane and Carver were caught in a breathtaking scenario where they need to keep a bomb from exploding, and the only way to prevent explosion is to put exactly four gallons of water on a scale. How would they do it if they only have a five - gallon and a three gallon jug? [LIST=1] [*]Fill the 5-gallon jug all the way. [*]Pour water into the 3 gallon jug until it is full. [*]Now you have 2 gallons in the 5-gallon jug and a full 3 gallons in the 3-gallon jug. [*]Empty the 3-gallon jug. [*]Pour the 2 gallons of water still in the 5-gallon jug into the 3-gallon jug. [*]Now the 3-gallon jug has 2 gallons of water in it, and 1 gallon of empty space. [*]Fill up the 5 gallon jug all the way, and then pour water out of the 5-gallon jug into the 3-gallon jug until the 3-gallon jug is full. [*]This leaves exactly [B]4 gallons[/B] in the 5-gallon jug. [/LIST]

In the past year, Yoko watched 76 movies that she thought were very good. She watched 80 movies ove
In the past year, Yoko watched 76 movies that she thought were very good. She watched 80 movies over the whole year. Of the movies she watched, what percentage did she think were very good? [URL='http://www.mathcelebrity.com/perc.php?num=76&den=80&pcheck=1&num1=16&pct1=80&pct2=70&den1=80&idpct1=10&hltype=1&idpct2=90&pct=82&decimal=+65.236&astart=12&aend=20&wp1=20&wp2=30&pl=Calculate']Enter 76/80 into our search engine to get 95%[/URL].

In the wild, monkeys eat an average of 28 bananas a day with a standard deviation of 2 bananas. One
In the wild, monkeys eat an average of 28 bananas a day with a standard deviation of 2 bananas. One monkey eats only 21 bananas. What is the z-score for this monkey? Is the number of bananas the monkey eats unusually low? Using [URL='https://www.mathcelebrity.com/probnormdist.php?xone=21&mean=28&stdev=2&n=1&pl=P%28X+%3C+Z%29']our z-score calculator[/URL], we get: Z < -3.5 P(Z < -3.5) = 0.499767 Also, this [B]is unusually low as it's more than 3 deviations away from the mean[/B]

In the year 1980, Rick was twice as old as Nancy who was twice as old as Michael. In the year 1992 R
In the year 1980, Rick was twice as old as Nancy who was twice as old as Michael. In the year 1992 Ric, Nancy, and Michael ages added up to 78 years. How old was Ric in 1980? Age in 1980: [LIST] [*]Let Michael's age be m [*]Nancy's age is 2m [*]Rick's age is 2 * 2m = 4m [/LIST] Age in 1992: [LIST] [*]Michael's age = m + 12 [*]Nancy's age is 2m + 12 [*]Rick's age is 2 * 2m = 4m + 12 [/LIST] Total them up: m + 12 + 2m + 12 + 4m + 12 = 78 Solve for [I]m[/I] in the equation m + 12 + 2m + 12 + 4m + 12 = 78 [SIZE=5][B]Step 1: Group the m terms on the left hand side:[/B][/SIZE] (1 + 2 + 4)m = 7m [SIZE=5][B]Step 2: Group the constant terms on the left hand side:[/B][/SIZE] 12 + 12 + 12 = 36 [SIZE=5][B]Step 3: Form modified equation[/B][/SIZE] 7m + 36 = + 78 [SIZE=5][B]Step 4: Group constants:[/B][/SIZE] We need to group our constants 36 and 78. To do that, we subtract 36 from both sides 7m + 36 - 36 = 78 - 36 [SIZE=5][B]Step 5: Cancel 36 on the left side:[/B][/SIZE] 7m = 42 [SIZE=5][B]Step 6: Divide each side of the equation by 7[/B][/SIZE] 7m/7 = 42/7 m = 6 Rick's age = 6 * 4 = [B]24 [URL='https://www.mathcelebrity.com/1unk.php?num=m%2B12%2B2m%2B12%2B4m%2B12%3D78&pl=Solve']Source[/URL] [/B]

In the year 1989, Luke's age was 3 times Rachel's age and Rachel's age was 3 times Dan's age. If Dan
In the year 1989, Luke's age was 3 times Rachel's age and Rachel's age was 3 times Dan's age. If Dan's age was n, how old were Rachel and Luke? Rachel's age = 3 * Dan's age Rachel's age = 3n Luke's age = 3 times Rachel's age Luke's age = 3(3n) Luke's age = [B]9n[/B]

In the year 1999, Hicham El Guerrouj of Morocco set a new world record when he ran a mile in 3 minut
In the year 1999, Hicham El Guerrouj of Morocco set a new world record when he ran a mile in 3 minutes 43.13 seconds. What was his speed in miles per hour? (Round your answer to the nearest hundredth.) 3 minutes = 60 seconds per minute = 180 seconds 180 seconds + 43.13 seconds = 223.13 seconds 223.13 seconds/3600 seconds per hour = 1 mile/n miles Cross multiply: 223.13n = 3600 Using our [URL='https://www.mathcelebrity.com/1unk.php?num=223.13n%3D3600&pl=Solve']equation solver[/URL], we get: n = [B]16.13 miles per hour[/B]

In the year 2000, the population of Rahway, New Jersey, was 26500. Express this number in scientific
In the year 2000, the population of Rahway, New Jersey, was 26500. Express this number in scientific notation 26,500 in [URL='https://www.mathcelebrity.com/scinot.php?num=26500&pl=Convert+to+Number']scientific notation is found using our scientific notation calculator[/URL]: [B]2.65 x 10^4[/B]

In this class of 4/5 students are right handed. if there are 20 right handed students, what is the t
In this class of 4/5 students are right handed. if there are 20 right handed students, what is the total number of students in this class? Let x be the total number of students in the class. We have: 4/5x = 20 Cross multiplying or using our [URL='http://www.mathcelebrity.com/1unk.php?num=4x%3D100&pl=Solve']equation calculator[/URL], we get: 4x = 100 Divide each side by 4 [B]x = 25[/B]

In Trina's desk drawer, there are 15 paper clips and 18 rubber bands. In Kirk's office supply tray,
In Trina's desk drawer, there are 15 paper clips and 18 rubber bands. In Kirk's office supply tray, there are 13 paper clips and 16 rubber bands. Who has a higher ratio of paper clips to rubber bands? Trina: 15/18 Kirk: 13/16 We want common denominators to compare, so we get a greatest common factor (GCF) for 16 and 18. [URL='https://www.mathcelebrity.com/gcflcm.php?num1=16&num2=18&num3=&pl=GCF+and+LCM']Running this through our search engine[/URL], we get GCF(16, 18) = 144 For Trina, 144/18 = 8 For Kirk, 144/16 = 9 We multiply Trina's fraction, top and bottom by 8: 15 * 8 / 18 * 8 120/144 We multiply Trina's fraction, top and bottom by 8: 13 * 8 / 16 * 8 104/144 [B]Trina[/B] has more in her numerator, so her ratio of paper clips to rubber bands is greater.

In which quadrant is the point (2,negative 6) located?
In which quadrant is the point (2,negative 6) located? We have the point (2, -6). It lies in Quadrant IV. to get this, [URL='https://www.mathcelebrity.com/polrectcord.php?num=2%2C-6&pl=Show+Detail#Quadrant']type in (2, -6) to the search engine[/URL], and click "Quadrant".

In x years time, Peter will be 23 years old. How old is he now?
In x years time, Peter will be 23 years old. How old is he now? Let Peter's current age be a. In x years time means we add x to a, so we're given: a + x = 23 We want to find a, s we subtract x from each side to get: a + x - x = 23 - x Cancel the x terms on the left side and we get: a = [B]23 - x[/B]

Ina has $40 in her bank account and saves $8 a week. Ree has $200 in her bank account and spends $12
Ina has $40 in her bank account and saves $8 a week. Ree has $200 in her bank account and spends $12 a week. Write an equation to represent each girl. Let w equal the number of weeks, and f(w) be the amount of money in the account after w weeks: [LIST] [*]Ina: [B]f(w) = 40 + 8w[/B] [LIST] [*]We add because Ina saves money, so her account grows [/LIST] [*]Ree: [B]f(w) = 200 - 12w[/B] [LIST] [*]We subtract because Ree saves [/LIST] [/LIST]

Ina school, out of 300 students, 70% are girls and 30% are boys. if 30 girls leave and no new boy is
Ina school, out of 300 students, 70% are girls and 30% are boys. if 30 girls leave and no new boy is admitted, what is the new% of girls in the school. Current ratios: [LIST] [*]Girls = 70% of 300 = 210 [*]Boys = 30% of 300 = 90 [/LIST] Ratios after girls leave: [LIST] [*]Girls = 210 - 30 = 180 [*]Boys = 90 [*]Total = 180 + 90 = 270 [*]Girls Percent = 180/270 = 2/3 = [B]66 & 2/3%[/B] [/LIST]

Inclusive Number Word Problems
Free Inclusive Number Word Problems Calculator - Given an integer A and an integer B, this calculates the following inclusive word problem questions:
1) The Average of all numbers inclusive from A to B
2) The Count of all numbers inclusive from A to B
3) The Sum of all numbers inclusive from A to B

Income Elasticity of Demand
Free Income Elasticity of Demand Calculator - Calculates the income elasticity of demand using demand changes and income changes. Inputs are demand 1 and demand 0 and income 1 and income 0.

Incremental Cash Flow
Free Incremental Cash Flow Calculator - Given cash inflows, outflows, depreciable amounts, and tax rates, this determines the incremental cash flows.

index form of (5^3)^6
Index form of (5^3)^6 Index form is written as a number raised to a power. Let's simplify by multiply the exponents. Since 6*3 = 18, We have: [B]5^18[/B]

Input Table
Free Input Table Calculator - Given an input table with input and output values, this will determine the operator and rule used to populate the missing values.

Installment Sales Method of Accounting
Free Installment Sales Method of Accounting Calculator - Given a sales price, cost amount, installment payment amount and term, this will show the accounting for the Installment Payment method.

Int Function
Free Int Function Calculator - Determines the integer of a number

Integrals
Free Integrals Calculator - This lesson walks you through the integral definition, rules, and examples

International Unit Conversions
Free International Unit Conversions Calculator - This calculator converts between the following weight measurements:
* International Units (IU)
* mcg
* mg


Interpolation
Free Interpolation Calculator - Given a set of data, this interpolates using the following methods:
* Linear Interpolation
* Nearest Neighbor (Piecewise Constant)
* Polynomial Interpolation

Interval Counting
Free Interval Counting Calculator - Evaluates a set of interval counting statements in the form a(b)c.

Interval Notation and Set Builder Notation
Free Interval Notation and Set Builder Notation Calculator - This calculator translates the following inequality statements to interval notation and set builder notation:
* x < 5
* y <= 5
* z > 5
* a >= 5
* b < 5 or b > 20
* Compound Inequality such as 0 <= c < 4
* |x|<3
* Reverse Interval Notation to Inequality Statement such as (-7,5]
* {x|x<1}
* Word representations of interval notations such as 2 is less than or equal to x is less than or equal to 8

Interval Partition
Free Interval Partition Calculator - Given a partitioned interval, this evaluates the norm (mesh) by calculating each subinterval

Inventory Method
Free Inventory Method Calculator - Takes accounting entries using the FIFO (first in first out) and LIFO (last in first out) inventory methods.

Irregular Verbs
Free Irregular Verbs Calculator - Shows the 168 irregular verbs along with their simple past tense and past participle

Is (1, 3) a solution to the equation y = 3x?
Is (1, 3) a solution to the equation y = 3x? Plug in x = 1 into y = 3x: y = 3(1) y = 3 The answer is [B]yes[/B], (1, 3) is a solution to y = 3x

Is (3,10) a solution to the equation y=4x
Is (3,10) a solution to the equation y=4x Plug in the numbers to check: 10 ? 4(3) 10 <> 12 No, this is [B]not a solution[/B]

Is (9, 6) a solution to the equation y = x - 3?
Is (9, 6) a solution to the equation y = x - 3? The ordered pair (x, y) = (9, 6) Plug in x = 9 into y = x - 3: y = 9 - 3 y = 6 [B]Yes, (9, 6) is a solution to the equation y = x - 3[/B]

Is 20% equivalent to 2/5?
Is 20% equivalent to 2/5? Let's compare fractions to fractions: 20% equals 1/5 from our [URL='https://www.mathcelebrity.com/perc.php?num=+5&den=+8&num1=+16&pct1=+80&pct2=+90&den1=+80&pct=20&pcheck=4&decimal=+65.236&astart=+12&aend=+20&wp1=20&wp2=30&pl=Calculate']percentage-decimal-fraction calculator[/URL]. 1/5 < 2/5 so these fractions are [B][I]not equivalent[/I][/B].

Is 3 standard deviations above the means considered an outlier?
Is 3 standard deviations above the means considered an outlier? [B]Yes.[/B] Using the empirical rule, we know that: [LIST] [*]68% of the values lie within one standard deviation of the mean [*]95% of the values lie within two standard deviations of the mean [/LIST] Anything out side of two standard deviations is considered an outlier.

is 6x a monomial?
[B]Yes[/B]. It's an algebraic expression consisting of one term. The constant is 6, and the variable is x.

Is it correct to word "10% * 50 + 50" as "10% upper 50"?
Is there any other statement in mathematics for this with the "upper" definition? Replace "50" with "x" x + 10% x

Is it correct to word "10% * 50 + 50" as "10% upper 50"?
Yes, it's close to the upper bound. I just wonder what we interpret the below inequality is? y > 10% x + x My two cents: y is greater than 10% upper x What do you say?

Is it correct to word "10% * 50 + 50" as "10% upper 50"?
Yes, that's a way. However, in my case, I need to interpret the expression with the "upper" definition. Could you help me?

Is it correct to word "10% * 50 + 50" as "10% upper 50"?
Yes, then if we are dealing with percentiles, I read that as 10% upper x%

is parallel to the x-axis and has an y-intercept of 3
is parallel to the x-axis and has an y-intercept of 3 Parallel to the x axis means it runs through the y-axis y-intercept of 3 means our equation is [B]y = 3[/B]

Is someone has $1,000,000 in base 2, how much money does she have in base 10?
Is someone has $1,000,000 in base 2, how much money does she have in base 10? 1 is in 7th digit place, so we raise it to the 6th power: [URL='https://www.mathcelebrity.com/powersq.php?sqconst=+6&num=2%5E6&pl=Calculate']1 * 2^6 [/URL]= [B]64[/B]

Is the point (4,7) a solution of the equation yequals15xminus8?
Is the point (4,7) a solution of the equation y equals 15x minus 8? Plug in x = 4: 15(4) - 8 60 - 8 52 Since 52 <> 4, (4,7) is [U][B]not[/B][/U] a solution of the equation y equals 15x minus 8

Is this algebra?
Can anyone answer this equation? You start off with 5 tickets and every 24min you get 1 extra ticket. After you sell your first ticket you have exactly 10min to sell another ticket and so on. How many tickets can you sell before you run out of tickets to sell? Plz give the mathematical equation for others to know also[IMG]https://www.facebook.com/images/emoji.php/v9/f34/1/16/1f914.png[/IMG]

Is this algebra?
When are you selling tickets? That is not stated in the problem.

Isaac invested $5000 at two different rates, 4% and 6.5% if his total interest income was $250, how
Isaac invested $5000 at two different rates, 4% and 6.5% if his total interest income was $250, how much did he invest at each rate? Using our [URL='https://www.mathcelebrity.com/split-fund-interest-calculator.php?p=5000&i1=4&i2=6.5&itot=250&pl=Calculate']split fund calculator[/URL], we have the following investments per fund: Fund 1: [B]$3,000[/B] Fund 2: [B]$2,000[/B]

Isabel earns $7.50 per hour on the weekends. Write and solve an inequality to find how many hours sh
Isabel earns $7.50 per hour on the weekends. Write and solve an inequality to find how many hours she needs to work to earn at least $120. A few things to note: [LIST] [*]Earnings = Rate * time [*]Let h be the number of hours worked [*]The phrase [I]at least[/I] means greater than or equal to, so we have the following inequality. [/LIST] We represent this with the following inequality: 7.5h < 120 To solve this inequality for h, we [URL='https://www.mathcelebrity.com/interval-notation-calculator.php?num=7.5h%3C120&pl=Show+Interval+Notation']type it into our math engine[/URL] and we get: [B]h < 16[/B]

Isabel is making face mask. She spends $50 on supplies and plans on selling them for $4 per mask. Ho
Isabel is making face mask. She spends $50 on supplies and plans on selling them for $4 per mask. How many mask does have to make in order to make a profit equal to $90? [U]Set up the cost function C(m) where m is the number of masks:[/U] C(m) = supply cost C(m) = 50 [U]Set up the cost function R(m) where m is the number of masks:[/U] R(m) = Sale Price * m R(m) = 4m [U]Set up the profit function P(m) where m is the number of masks:[/U] P(m) = R(m) - C(m) P(m) = 4m - 50 The problems asks for profit of 90, so we set P(m) = 90: 4m - 50 = 90 To solve this equation for m, we [URL='https://www.mathcelebrity.com/1unk.php?num=4m-50%3D90&pl=Solve']type it in our search engine[/URL] and we get: m = [B]35[/B]

Isabel will run less than 36 minutes today. So far, she has run 22 minutes. What are the possible nu
Isabel will run less than 36 minutes today. So far, she has run 22 minutes. What are the possible numbers of additional minutes she will run? Set up our inequality. If she ran 22 minutes, we need to find an expression to find out the remaining minutes x + 22 < 36 Subtract 22 from each side: x < 14 Remember, she cannot run negative minutes, so our lower bound is 0, so we have: [B]0 < x < 14 [/B]

Ishaan is 72 years old and William is 4 years old. How many years will it take until Ishaan is only
[SIZE=4]Ishaan is 72 years old and William is 4 years old. How many years will it take until Ishaan is only 5 times as old as William? [U]Express Ishaan and William's age since today where y is the number of years since today, we have:[/U] i = 72+y w = 4+y [U]We want the time for Ishaan age will be 5 times William's age:[/U] i = 5w 72 + y = 5(4 + y) We [URL='https://www.mathcelebrity.com/1unk.php?num=72%2By%3D5%284%2By%29&pl=Solve']plug this equation into our search engine [/URL]and get: y = [B]13[/B] [/SIZE]

Isosceles Triangle
Free Isosceles Triangle Calculator - Given a long side (a) and a short side (b), this determines the following items of the isosceles triangle:
* Area (A)
* Semi-Perimeter (s)
* Altitude a (ha)
* Altitude b (hb)
* Altitude c (hc)


It costs $2.50 to rent bowling shoes. Each game costs $2.25. You have $9.25. How many games can you
It costs $2.50 to rent bowling shoes. Each game costs $2.25. You have $9.25. How many games can you bowl. Writing an equation and give your answer. Let the number of games be g. we have the function C(g): C(g) = cost per game * g + bowling shoe rental C(g) = 2.25g + 2.50 The problem asks for g when C(g) = 9.25 2.25g + 2.50 = 9.25 To solve this equation, we[URL='https://www.mathcelebrity.com/1unk.php?num=2.25g%2B2.50%3D9.25&pl=Solve'] type it in our search engine[/URL] and we get: g = [B]3[/B]

It costs $4.25 per game at the bowling alley plus $1.90 to rent shoes. if Wayne has $20, how many ga
It costs $4.25 per game at the bowling alley plus $1.90 to rent shoes. if Wayne has $20, how many games can he Bowl? Let g be the number of games. The cost for Wayne is: C(g) = Cost per game * number of games + shoe rental 4.25g + 1.90 = C(g) We're given C(g) = 20, so we have: 4.25g + 1.90 = 20 Using our [URL='https://www.mathcelebrity.com/1unk.php?num=4.25g%2B1.90%3D20&pl=Solve']equation solver[/URL] for g, we get: g = 4.25 We need whole games, we we round down to [B]4 games[/B]

it costs $75.00 for a service call from shearin heating and air conditioning company. the charge for
it costs $75.00 for a service call from shearin heating and air conditioning company. the charge for labor is $60.00 . how many full hours can they work on my air conditioning unit and still stay within my budget of $300.00 for repairs and service? Our Cost Function is C(h), where h is the number of labor hours. We have: C(h) = Variable Cost * Hours + Fixed Cost C(h) = 60h + 75 Set C(h) = $300 60h + 75 = 300 [URL='https://www.mathcelebrity.com/1unk.php?num=60h%2B75%3D300&pl=Solve']Running this problem in the search engine[/URL], we get [B]h = 3.75[/B].

It costs a $20 flat fee to rent a lawn mower, plus $5 a day starting with the first day. Let x repre
It costs a $20 flat fee to rent a lawn mower, plus $5 a day starting with the first day. Let x represent the number of days rented, so y represents the charge to the user (in dollars) Set up our function: [B]y = 20 + 5x[/B]

It is estimated that weekly demand for gasoline at new station is normally distributed, with an aver
It is estimated that weekly demand for gasoline at new station is normally distributed, with an average of 1,000 and standard deviation of 50 gallons. The station will be supplied with gasoline once a week. What must the capacity of its tank be if the probability that its supply will be exhausted in a week is to be no more than 0.01? 0.01 is the 99th percentile Using our [URL='http://www.mathcelebrity.com/percentile_normal.php?mean=+1000&stdev=50&p=99&pl=Calculate+Percentile']percentile calculator[/URL], we get [B]x = 1116.3[/B]

It is known that 45% of men snore an 25% of women snore. A doctor looked at these numbers and made t
It is known that 45% of men snore an 25% of women snore. A doctor looked at these numbers and made the following statement: "If you put a man and a woman together, there is a 70% chance that someone is snoring." Explain why the doctor's math is wrong. The doctor added the percents together: 45% + 25% = 70%. Here's why this is incorrect: [LIST] [*]45% of men snore means 100% - 45% = 55% of men do not snore [*]25% of women snore means 100% - 25% = 75% of women do not snore [*]Both men and women not snoring is: 55% * 75% = 41.25% neither of them snore [*]100% - 41.25% = [B]58.75%[/B] somebody is snoring [/LIST]

It is recommended that a ladder be placed 2 feet away from the Wall for every 5 feet of height. How
It is recommended that a ladder be placed 2 feet away from the Wall for every 5 feet of height. How far from the Wall should a 20 foot ladder be placed? Set up a proportion: 2ft away from the wall / 5ft = (x)ft away from the wall / 20ft [URL='http://www.mathcelebrity.com/prop.php?num1=2&num2=x&den1=5&den2=20&propsign=%3D&pl=Calculate+missing+proportion+value']Run this proportion through our calculator by typing[/URL]: 2/5=x/20 x = [B]8 ft[/B]

It takes 3 city snowplows 14 hours to clear 500 miles of road. If the city wants the 500 miles of ro
It takes 3 city snowplows 14 hours to clear 500 miles of road. If the city wants the 500 miles of road to be cleared in 6 hours, how many additional snowplows must they buy? Set up unit rate per plow: 14 hours * 3 plows = 42 hours for one plow to clear 500 miles of road Calculate the amount of plows we need: 42 hours / 6 hours = 7 plows Additional plows = New plows - original plows: Additional plows = 7 - 3 Additional plows = [B]4[/B]

It takes 3/4 of an hour to complete a puzzle. How many puzzles can Cindy finish in 3 hours?
It takes 3/4 of an hour to complete a puzzle. How many puzzles can Cindy finish in 3 hours? We setup a proportion of time to puzzles where p is the number of puzzles Cindy can complete in 3 hours: 3/4/1 = 3/p Dividing by 1 means the same as the original fraction, so we have: 3/4 = 3/p [URL='https://www.mathcelebrity.com/prop.php?num1=3&num2=3&den1=4&den2=p&propsign=%3D&pl=Calculate+missing+proportion+value']Typing this proportion into the search engine[/URL], we get: p = [B]4[/B]

It takes 5 workers 12 hours to unload one truck. How long would it take 6 workers to unload the truc
It takes 5 workers 12 hours to unload one truck. How long would it take 6 workers to unload the truck? 5 workers * 12 hours = 60 hours for one worker. 60 hours for one worker / 6 workers = [B]10 hours[/B]

It takes 60 minutes for 7 people to paint 5 walls. How many minutes does it take 10 people to paint
It takes 60 minutes for 7 people to paint 5 walls. How many minutes does it take 10 people to paint 10 walls? Rate * Time = Output Let "Rate" (r) be the rate at which [B]one person[/B] works. So we have: 7r * 60 = 5 Multiply through and simplify: 420r = 5 Divide each side by 5 to isolate r: r = 1/84 So now we want to find out how many minutes it takes 10 people to paint 10 walls using this rate: 10rt = 10 With r = 1/84, we have: 10t/84 = 10 Cross multiply: 10t = 840 To solve for t, we t[URL='https://www.mathcelebrity.com/1unk.php?num=10t%3D840&pl=Solve']ype this equation into our search engine[/URL] and we get: t = [B]84 minutes[/B]

It takes a crew of 4 painters 12 hours one house. If they wanted to paint the house in 8 hours, how
It takes a crew of 4 painters 12 hours one house. If they wanted to paint the house in 8 hours, how many additional painters must they hire? It takes one painter 4 * 12 hours = 48 hours to paint the house. Now we calculate the unit rate: 48 hours / 8 hours = 6 painters 6 painters - 4 original painters = [B]2 additional painters[/B]

It takes Deanna 7 hours to paint a fence. Who fraction of the fence does she paint in one hour?
It takes Deanna 7 hours to paint a fence. Who fraction of the fence does she paint in one hour? 7 hours for 1 fence = [B]1/7 of the fence per hour[/B]

It takes Spot 2 hours to paint a fence and Steven 4 hours to paint the same fence. If they work toge
It takes Spot 2 hours to paint a fence and Steven 4 hours to paint the same fence. If they work together, how long will it take them to paint the fence? Spot paints 1/2 of a fence in an hour Steven paints 1/4 of a fence in an hour Together, in an hour, they paint 1/2 + 1/4 of a fence in an hour 1/2 = 2/4, so we have 2/4 + 1/4 = 3/4 of a fence in an hour Meaning they take another 20 minutes to pain the last 1/4 of the fence [B]1 hour + 20 minutes[/B] is the total time it takes

It took 3.5 gallons of paint to cover a wall that is 985 square feet. How many gallons will it take
It took 3.5 gallons of paint to cover a wall that is 985 square feet. How many gallons will it take to cover a wall that is 6501 square feet? Set up a proportion of gallons of paint to square feet where n is the number of gallons of paint to cover 6501 square feet 3.5/985 = n/6501 Using our [URL='https://www.mathcelebrity.com/proportion-calculator.php?num1=3.5&num2=n&den1=985&den2=6501&propsign=%3D&pl=Calculate+missing+proportion+value']proportion calculator[/URL], we get: n = [B]23.1[/B]

Itachi want’s to buy apples that weights 5.7 pounds. The apples is priced at $1.58 per pound how muc
Itachi want’s to buy apples that weights 5.7 pounds. The apples is priced at $1.58 per pound how much does the apples costs Cost = price per pound * number of pounds Cost = 1.58 * 5.7 Cost = [B]9.01[/B]

j - m/4 = 4k for m
j - m/4 = 4k for m Multiply each side by 4: 4j - 4m/4 = 4(4k) Simplify: 4j - m = 16k Add m to each side: 4j - m + m = 16k + m The m's cancel on the left side, so we have: 4j = 16k + m Subtract 16k from each side: 4j - 16k = 16k - 16k + m The 16k cancels on the right side, so we're left with: [B]m = 4j - 16k or 4(j - 4k)[/B]

Jack and Jill have a magic pail of beans. The number of beans in the pail doubles every second. If
Jack and Jill have a magic pail of beans. The number of beans in the pail doubles every second. If the pail is full after 10 seconds, when was the pail half full? Explain your answer. [LIST] [*]At time 0, we have n beans [*]At time 1, we have 2n beans [*]At time 2, we have 4n beans [*]At time 3, we have 8n beans [*]At time 4, we have 16n beans [*]At time 5, we have 32n beans [*]At time 6, we have 64n beans [*]At time 7, we have 128n beans [*]At time 8, we have 256n beans [*]At time 9, we have 512n beans [*]At time 10, we have 1024n beans [/LIST] 1/2 of 1024 is 512, so at [B]Time 9[/B], the pail is half full.

Jack bought 7 tickets for a movie. He paid $7 for each adult ticket and $4 for each child ticket. Ja
Jack bought 7 tickets for a movie. He paid $7 for each adult ticket and $4 for each child ticket. Jack spent $40 for the tickets Let a = Number of adult tickets and c be the number of child tickets. [LIST=1] [*]7a + 4c = 40 [*]a + c = 7 [*]Rearrange (2): a = 7 - c [/LIST] Now substitute a in (3) into (1): 7(7 - c) + 4c = 40 49 - 7c + 4c = 40 49 - 3c = 40 Add 3c to each side and subtract 40: 3c = 9 Divide each side by 3: [B]c = 3 [/B] Substitute c = 3 into Equation (2) a + 3 = 7 Subtract 3 from each side: [B]a = 4[/B]

Jack bought a car for $17,500. The car loses $750 in value each year. Which equation represents the
Jack bought a car for $17,500. The car loses $750 in value each year. Which equation represents the situation? Let y be the number of years since Jack bought the car. We have a Book value B(y): [B]B(y) = 17500 - 750y[/B]

Jack has 34 bills and coins in 5’s and 2’s. The total value is $116. How many 5 dollar bills does he
Jack has 34 bills and coins in 5’s and 2’s. The total value is $116. How many 5 dollar bills does he have? Let the number of 5 dollar bills be f. Let the number of 2 dollar bills be t. We're given two equations: [LIST=1] [*]f + t = 34 [*]5f + 2t = 116 [/LIST] We have a system of equations, which we can solve 3 ways: [LIST=1] [*][URL='https://www.mathcelebrity.com/simultaneous-equations.php?term1=f+%2B+t+%3D+34&term2=5f+%2B+2t+%3D+116&pl=Substitution']Substitution Method[/URL] [*][URL='https://www.mathcelebrity.com/simultaneous-equations.php?term1=f+%2B+t+%3D+34&term2=5f+%2B+2t+%3D+116&pl=Elimination']Elimination Method[/URL] [*][URL='https://www.mathcelebrity.com/simultaneous-equations.php?term1=f+%2B+t+%3D+34&term2=5f+%2B+2t+%3D+116&pl=Cramers+Method']Cramer's Rule[/URL] [/LIST] No matter which method we choose, we get the same answers: [LIST] [*][B]f = 16[/B] [*][B]t = 18[/B] [/LIST]

Jack is making snack bags. He has 18 baby carrots and 42 pretzels. He wants to divide them equally a
Jack is making snack bags. He has 18 baby carrots and 42 pretzels. He wants to divide them equally among the bags. What is the greatest number of snack bags he can make? Find the [URL='http://www.mathcelebrity.com/gcflcm.php?num1=18&num2=42&num3=&pl=GCF']Greatest Common Factor[/URL] of (18, 42) = 6 6 bags for 18 carrots = 3 carrots per bag 6 bags for 42 pretzels = 7 pretzels per bag [B]6 bags is the answer[/B]

Jack reads 90 pages of a book in six hours. What is the average number of pages he read each hour
Jack reads 90 pages of a book in six hours. What is the average number of pages he read each hour 90 pages / 6 hour = 90/6 Type [URL='https://www.mathcelebrity.com/fraction.php?frac1=90%2F6&frac2=3%2F8&pl=Simplify']90/6 in our search engine, click simplify[/URL], and we get: [B]15 pages per hour[/B]

Jack scored a mean of 15 points per game in his first 3 basketball games. In his 4th grade, he score
Jack scored a mean of 15 points per game in his first 3 basketball games. In his 4th grade, he scored 27 points. What was Jack's mean score for the four games? The mean is the average: Mean = (15 + 15 + 15 + 27)/4 Mean = 72/4 [B]Mean = 18[/B]

Jack's mother gave him 50 chocolates to give to his friends at his birthday party. He gave 3 chocola
Jack's mother gave him 50 chocolates to give to his friends at his birthday party. He gave 3 chocolates to each of his friends and still had 2 chocolates left. If Jack had 2 chocolates left, then the total given to his friends is: 50 - 2 = 48 Let f be the number of friends at his birthday party. Then we have: 3f = 48 [URL='https://www.mathcelebrity.com/1unk.php?num=3f%3D48&pl=Solve']Typing this equation into our search engine[/URL], we get: [B]f = 16[/B]

Jack's mother gave him 50 chocolates to give to his friends at his birthday party. He gave 3 chocola
Jack's mother gave him 50 chocolates to give to his friends at his birthday party. He gave 3 chocolates to each of his friends and still had 2 chocolates left. Let f be the number of Jacks's friends. We have the following equation to represent the chocolates: 3f + 2 = 50 To solve this equation for f, we [URL='https://www.mathcelebrity.com/1unk.php?num=3f%2B2%3D50&pl=Solve']type it in the math engine[/URL] and we get: f = [B]16[/B]

Jacob bought a car that loses 10% of its value each year. If the original cost of the car is n dolla
Jacob bought a car that loses 10% of its value each year. If the original cost of the car is n dollars, what is its value after 3 years? [LIST] [*]Year 1: 0.9*n = 0.9n [*]Year 2: 0.9 * 0.9n = 0.81n [*]Year 3: 0.9 * 0.81n = [B]0.729n[/B] [/LIST]

Jada scored 15 points in one basketball game and p points in another. Her two-game total is 34 point
Jada scored 15 points in one basketball game and p points in another. Her two-game total is 34 points The phrase [I]total[/I] means a sum, so we have the following equation: 15 + p = 34 To solve for p, we [URL='https://www.mathcelebrity.com/1unk.php?num=15%2Bp%3D34&pl=Solve']type this equation into our search engine [/URL]and we get: p = [B]19[/B]

Jake bought s shirts. they were $7 each. Write an equation to represent the total amount that Jake p
Jake bought s shirts. they were $7 each. Write an equation to represent the total amount that Jake paid for the shirts Since Amount = Price * Quantity, we have: [B]7s[/B]

Jake Callahan stars in the new TV series Stone Simons: Kid Astronaut. The day after the first episod
Jake Callahan stars in the new TV series Stone Simons: Kid Astronaut. The day after the first episode airs, Jake receives a bunch of fan mail. He splits all the letters into 3 equal stacks to open with his mom and his sister. Each stack contains 21 letters. Which equation can you use to find the number of letters n Jake receives? The number of letters n is represented by number of stacks (s) times letter per stack (l). We're given s = 3 and l = 21, so we have: n = 21(3) n = [B]63[/B]

Jake used 5 boxes to pack 43.5 kg of books. If the boxes each weighed the same and held 8 books, wh
Jake used 5 boxes to pack 43.5 kg of books. If the boxes each weighed the same and held 8 books, what did each book weigh? [U]Set up equations were w is the weight of each book:[/U] [LIST=1] [*]5 boxes * 8 books * w = 43.5 [*]40w = 43.5 [/LIST] [U]Divide each side by 40[/U] [B]w = 1.0875 kg[/B]

James Bond has a secret code. The code is 3 digits long and less than 160. The digits add to 10. Wha
James Bond has a secret code. The code is 3 digits long and less than 160. The digits add to 10. What is his secret code? less than 160 means 0 to 159 Working backwards with 1 in the hundreds place and 5 in the 10's place, we see that 1 + 5 + 4 = 10 [B]154[/B]

James has a weekly allowance of 5 plus 1.50 for each chore c he does
James has a weekly allowance of 5 plus 1.50 for each chore c he does We build the allowance function A(c) where c is each chore A(c) = cost per chore * c + Weekly Allowance Plugging in our numbers, we get: [B]A(c) = 1.50c + 5[/B]

James is four time as old as peter if their combined age is 30 how old is James.
James is four time as old as peter if their combined age is 30 how old is James. Let j be Jame's age. Let p be Peter's age. We're given: [LIST=1] [*]j = 4p [*]j + p = 30 [/LIST] Substitute (1) into (2) 4p + p = 30 Combine like terms: 5p = 30 [URL='https://www.mathcelebrity.com/1unk.php?num=5p%3D30&pl=Solve']Type 5p = 30 into our search engine[/URL], and we get p = 6. Plug p = 6 into equation (1) to get James's age, we get: j = 4(6) j = [B]24[/B]

James is ordering action figures online. Each action figure is $9. Shipping cost $10 per order. Jame
James is ordering action figures online. Each action figure is $9. Shipping cost $10 per order. James does not want to spend over $154. How many action figures can he order? Step 1: Subtract the cost of shipping from the spend $154 - $10 = $144 Step 2: Divide $144 to spend after shipping by $9 action figures 144/9 = [B]$16 action figures[/B]

jamie needs 3 cups of flour and 4 cups of sugar how many cups of sugar will she need if she uses 9 c
jamie needs 3 cups of flour and 4 cups of sugar how many cups of sugar will she need if she uses 9 cups of flour? Set up a proportion of flour/sugar: 3/4 = 9/x [URL='http://www.mathcelebrity.com/prop.php?num1=3&num2=9&den1=4&den2=x&propsign=%3D&pl=Calculate+missing+proportion+value']Cross multiply or enter that into the search engine[/URL] 3x = 36 [B]x = 12[/B]

Jamie spent $15.36 on several items at the store. he spent an equal amount on each item. if jamie sp
Jamie spent $15.36 on several items at the store. he spent an equal amount on each item. if jamie spent $1.92 on each item, how many items did he buy? Let x equal the number of items Jamie bought. We have: 1.92x = 15.36 Divide each side by 1.92 [B]x = 8[/B]

Jane did this calculation a. Add -12 b.subtract -9 c. Add 8 d. Subtract -2 the result is -5. What wa
Jane did this calculation a. Add -12 b.subtract -9 c. Add 8 d. Subtract -2 the result is -5. What was the original number? Let the original number be n. [LIST=1] [*]Add -12: n - 12 [*]Subtract -9: n - 12 - -9 = n - 12 + 9 [*]Add 8: n - 12 + 9 + 8 [*]Subtract - 2: n - 12 + 9 + 8 - -2 = n - 12 + 9 + 8 + 2 [*]The result is -5. So we build the following equation: [/LIST] n - 12 + 9 + 8 + 2 = -5 To solve this equation for n, we [URL='https://www.mathcelebrity.com/1unk.php?num=n-12%2B9%2B8%2B2%3D-5&pl=Solve']type it in our search engine[/URL] and we get: [B]n = -12[/B]

Jane has $7.50 to spend in the candy store. She likes lollipops and gumballs. Each lollipop costs
Jane has $7.50 to spend in the candy store. She likes lollipops and gumballs. Each lollipop costs $2.75, and each gumball costs $0.50. If Jane decides to buy 1 lollipop, then what is the greatest number of gumballs Jane can buy? A Subtract the cost of 1 lollipop: $7.50 - $2.75 = $4.75 Let the number of gumballs = g. We have: 0.50g = $4.75 [URL='https://www.mathcelebrity.com/1unk.php?num=0.50g%3D4.75&pl=Solve']Run this through the search engine[/URL] to get g = 9.5 The problem asks for the greatest number. So we round down to [B]9 gumballs[/B].

jane has 55$ to spend at cedar point. the admission price is 42$ and each soda is 4.25. write an ine
jane has 55$ to spend at cedar point. the admission price is 42$ and each soda is 4.25. write an inequality to show how many sodas he can buy. Let s be the number of sodas. Cost for the day is: Price per soda * s + Admission Price 4.25s + 42 We're told that Jane has 55, which means Jane cannot spend more than 55. Jane can spend up to or less than 55. We write this as an inequality using <= 55 [B]4.25s + 42 <= 55[/B] [B][/B] If the problems asks you to solve for s, we type it in our math engine and we get: Solve for [I]s[/I] in the inequality 4.25s + 42 ? 55 [SIZE=5][B]Step 1: Group constants:[/B][/SIZE] We need to group our constants 42 and 55. To do that, we subtract 42 from both sides 4.25s + 42 - 42 ? 55 - 42 [SIZE=5][B]Step 2: Cancel 42 on the left side:[/B][/SIZE] 4.25s ? 13 [SIZE=5][B]Step 3: Divide each side of the inequality by 4.25[/B][/SIZE] 4.25s/4.25 ? 13/4.25 [B]s ? 3.06[/B]

Jane is twice a old as Joel. If their ages total 63, how old is Joel?
Jane is twice a old as Joel. If their ages total 63, how old is Joel? Joel = j Jane = 2j j + 2j = 63 3j = 63 Divide each side by 3: 3j/3 = 63/j Cancel the 3's on the left side: j = [B]21[/B]

Janet drove 395 kilometers and the trip took 5 hours. How fast was Janet traveling?
Janet drove 395 kilometers and the trip took 5 hours. How fast was Janet traveling? Distance = Rate * Time We're given D = 395 and t = 5 We want Rate. We divide each side of the equation by time: Distance / Time = Rate * Time / Time Cancel the Time's on each side and we get: Rate = Distance / Time Plugging our numbers in, we get: Rate = 395/5 Rate = [B]79 kilometers[/B]

Janice is looking to buy a vacation home for $185,000 near her favorite southern beach. The formula
Janice is looking to buy a vacation home for $185,000 near her favorite southern beach. The formula to compute a mortgage payment, M, is shown below, where P is the principal amount of the loan, r is the monthly interest rate, and N is the number of monthly payments. Janice's bank offers a monthly interest rate of 0.325% for a 12-year mortgage. How many monthly payments must Janice make? 12 years * 12 months per year = [B]144 mortgage payments[/B]

Janice says that the sum of the measures of the interior angles of an octagon is 900°. Is Janice cor
Janice says that the sum of the measures of the interior angles of an octagon is 900°. Is Janice correct? Why or why not? She's [B]incorrect. [/B] The interior angle sum for a polygon is found with this formula: Interior Angle Sum = (sides - 2) x 180° Since an octagon has 8 sides, we have: Interior Angle Sum = (8 - 2) x 180° Interior Angle Sum = 6 x 180° Interior Angle sum = 1080°

jared bakes 2 apple pies. he cuts two pies into pieces. Each piece is 1/8 of a pie. Enter the number
jared bakes 2 apple pies. he cuts two pies into pieces. Each piece is 1/8 of a pie. Enter the number of pieces of pie jared cuts 1/8 of a pie per slice means there are 8 slices per pie 2 pies * 8 pieces per pie = [B]16 pieces[/B]

Jason has an equal number of nickels and dimes. The total value of his nickels and dimes is $2.25. H
Jason has an equal number of nickels and dimes. The total value of his nickels and dimes is $2.25. How many nickels does Jason have? Let the number of nickels be n Let the number of dimes be d We're given two equations: [LIST=1] [*]d = n [*]0.05n + 0.1d = 2.25 [/LIST] Substitute equation (1) for d into equation (2): 0.05n + 0.1n = 2.25 Solve for [I]n[/I] in the equation 0.05n + 0.1n = 2.25 [SIZE=5][B]Step 1: Group the n terms on the left hand side:[/B][/SIZE] (0.05 + 0.1)n = 0.15n [SIZE=5][B]Step 2: Form modified equation[/B][/SIZE] 0.15n = + 2.25 [SIZE=5][B]Step 3: Divide each side of the equation by 0.15[/B][/SIZE] 0.15n/0.15 = 2.25/0.15 n = [B]15[/B] [URL='https://www.mathcelebrity.com/1unk.php?num=0.05n%2B0.1n%3D2.25&pl=Solve']Source[/URL]

Jason is 9 miles ahead of Joe running at 5.5 miles per hour and Joe is running at the speed of 7 mil
[SIZE=6]Jason is 9 miles ahead of Joe running at 5.5 miles per hour and Joe is running at the speed of 7 miles per hour. How long does it take Joe to catch Jason? A. 3 hours B. 4 hours C. 6 hours D. 8 hours Distance formula is d = rt Jason's formula (Add 9 since he's ahead 9 miles): d = 5.5t + 9 Joe's formula: d = 7t Set both distance formulas equal to each other: 5.5t + 9 = 7t Subtract 5.5t from each side: 5.5t - 5.5t + 9 = 7t - 5.5t 1.5t = 9 Divide each side by 1.5: 1.5t/1.5 = 9/1.5 t = [B]6 hours[/B] [U]Check our work with t = 6[/U] Joe = 7(6) = 42 Jason = 5.5(6) + 9= 33 + 9 = 42 [MEDIA=youtube]qae3WCq9wzM[/MEDIA] [/SIZE]

Jason wrote a total of 8 pages over 2 hours. How many hours will Jason have to spend writing this we
Jason wrote a total of 8 pages over 2 hours. How many hours will Jason have to spend writing this week in order to have written a total of 12 pages? Assume the relationship is directly proportional. Set up a proportion of pages to hours 8 pages/2 hours = 12 pages/x hours enter 8/2 = 12/x into the [URL='http://www.mathcelebrity.com/prop.php?num1=8&num2=12&den1=2&den2=x&propsign=%3D&pl=Calculate+missing+proportion+value']search engine[/URL]: [B]x = 3[/B]

Jay earns S amount per day for working in a company. His total expenses per day is equal to the amou
Jay earns S amount per day for working in a company. His total expenses per day is equal to the amount E. Write an expression to show how much he earned per day in a month. Suppose he is working for 20 days per month. [LIST=1] [*]Each day, Jay earns a profit of S - E. [*]For one month (30 days), he earns 30(S - E) [*]For 20 working days in a month, he earns 20(S - E) [/LIST]

Jay has 5 paintings that he plans to display on a wall that only has 4 books. Nancy has 5 paintings
Jay has 5 paintings that he plans to display on a wall that only has 4 books. Nancy has 5 paintings that she plans to display on a wall with 5 hooks. Who has more possible ways to hang his/her paintings? Jay's ways: [URL='https://www.mathcelebrity.com/permutation.php?num=5&den=4&pl=Permutations']5 P 4 [/URL]= [B]120 [/B] Nancy's ways: [URL='https://www.mathcelebrity.com/permutation.php?num=5&den=5&pl=Permutations']5 P 5[/URL] = [B]120 Therefore, they have the same number of ways.[/B]

Jay purchased tickets for a concert. To place the order, a handling charge of $7 per ticket was cha
Jay purchased tickets for a concert. To place the order, a handling charge of $7 per ticket was charged. A sales tax of 4% was also charged on the ticket price and the handling charges. If the total charge for four tickets was $407.68, what was the ticket price? Round to the nearest dollar. with a ticket price of t, we have the total cost written as: 1.04 * (7*4 + 4t)= 407.68 Divide each side by 1.04 1.04 * (7*4 + 4t)/1.04= 407.68/1.04 Cancel the 1.04 on the left side and we get: 7*4 + 4t = 392 28 + 4t = 392 To solve this equation for t, we [URL='https://www.mathcelebrity.com/1unk.php?num=28%2B4t%3D392&pl=Solve']type it in our math engine[/URL] and we get: t = [B]91[/B]

Jayden spent $46.20 on 12 galllons of gasoline. What was the price per gallon?
Jayden spent $46.20 on 12 galllons of gasoline. What was the price per gallon? Price per gallon = Total spend / number of gallons Price per gallon = $46.20/12 Price per gallon = $[B]3.85[/B]

Jazmin is a hairdresser who rents a station in a salon for daily fee. The amount of money (m) Jazmin
Jazmin is a hairdresser who rents a station in a salon for daily fee. The amount of money (m) Jazmin makes from any number of haircuts (n) a day is described by the linear function m = 45n - 30 A) A haircut costs $30, and the station rent is $45 B) A haircut costs $45, and the station rent is $30. C) Jazmin must do 30 haircuts to pay the $45 rental fee. D) Jazmin deducts $30 from each $45 haircut for the station rent. [B]Answer B, since rent is only due once. Profit is Revenue - Cost[/B]

Jeff Bezos, who owns Amazon, has a net worth of approximately $143.1 billion (as of mid-2018). An em
Jeff Bezos, who owns Amazon, has a net worth of approximately $143.1 billion (as of mid-2018). An employee in the Amazon distribution center earns about $13 an hour. The estimated lifespan of the employee is 71 years. If the employee worked 24 hours a day, every day of the year from the moment of his birth, how many lifespans would it take for him to earn wages equivalent to Jeff Bezos' net worth? Round the answer to the nearest whole number. Calculate earnings per lifespan: Earnings per lifespan = lifespan in years * Annual Earnings Earnings per lifespan = 71 * 13 * 24 * 365 <-- (24 hours per day * 365 days per year) Earnings per lifespan = 8,085,480 Calculate the number of lifespans needed to match Jeff Bezos earnings: Number of lifespans = Jeff Bezos Net Worth / Earnings Per Lifespan Number of lifespans = 143,100,000,000 / 8,085,480 Number of lifespans = [B]17,698.39 ~ 17,699[/B]

Jeni sees 104 octopus legs in the aquarium how many octopuses are there ?
Jeni sees 104 octopus legs in the aquarium how many octopuses are there ? An octopus has 8 legs. So the total number of octupuses are: Total octopuses = Total legs / 8 Total octopuses = 104 / 8 Total octopuses = [B]13[/B]

Jennie and Alex both wanted to get a free ticket for a College Music concert. However, the concert s
Jennie and Alex both wanted to get a free ticket for a College Music concert. However, the concert staff told them the tickets were limited. Twenty people wanted to attend the concert but only 10 free tickets were left. So the concert center staff decided to use a lottery to decide who would receive the free tickets. What's the probability of Jennie and Alex both getting free tickets? 1/2 * 1/2 = 1/4 = [B]0.25[/B]

Jennifer has 26 less than triple the savings of Matthew. Matthew has saved 81. How much has Jennifer
Jennifer has 26 less than triple the savings of Matthew. Matthew has saved 81. How much has Jennifer saved? Let Jennifer's savings be j. We're given: j = 3(81) - 26 j = 243 - 26 j = [B]217[/B]

Jennifer is playing cards with her bestie when she draws a card from a pack of 25 cards numbered fro
Jennifer is playing cards with her bestie when she draws a card from a pack of 25 cards numbered from 1 to 25. What is the probability of drawing a number that is square? The squares from 1 - 25 less than or equal to 25 are as follows: [LIST=1] [*]1^2 = 1 [*]2^2 = 4 [*]3^2 = 9 [*]4^2 = 16 [*]5^2 = 25 [/LIST] So the following 5 cards are squares: {1, 4, 9, 16, 25} Therefore, our probability of drawing a square is: P(square) = Number of Squares / Number of Cards P(square) = 5/25 This fraction can be simplified. So [URL='https://www.mathcelebrity.com/fraction.php?frac1=5%2F25&frac2=3%2F8&pl=Simplify']we type in 5/25 into our search engine, choose simplify[/URL], and we get: P(square) = [B]1/5[/B]

Jennifer is twice as old as Peter. The difference between their ages is 15. What is Peters age
Jennifer is twice as old as Peter. The difference between their ages is 15. What is Peters age Let j be Jennifer's age Let p be Peter's age We're given two equations: [LIST=1] [*]j = 2p [*]j - p = 15 [/LIST] Substitute equation (1) into equation (2) for j 2p - p = 15 To solve for p, we [URL='https://www.mathcelebrity.com/1unk.php?num=2p-p%3D15&pl=Solve']type this equation into our calculation engine[/URL] and we get: p = [B]15[/B]

Jennifer spent $11.25 on ingredients for cookies shes making for the school bake sale. How many cook
Jennifer spent $11.25 on ingredients for cookies shes making for the school bake sale. How many cookies must she sale at $0.35 apiece to make profit? Let x be the number of cookies she makes. To break even, she must sell: 0.35x = 11.25 Use our [URL='http://www.mathcelebrity.com/1unk.php?num=0.35x%3D11.25&pl=Solve']equation calculator[/URL], and we get: x = 32.14 This means she must sell [B]33[/B] cookies to make a profit.

Jenny added $150 to her savings account in July. At the end if the month she had $500. How much did
Jenny added $150 to her savings account in July. At the end if the month she had $500. How much did she start with? Let the starting balance be s. A deposit means we added 150 to s to get 500. We set up this equation below: s + 150 = 500 To solve for s, we [URL='https://www.mathcelebrity.com/1unk.php?num=s%2B150%3D500&pl=Solve']type this equation into our search engine[/URL] and we get: s = 3[B]50[/B]

Jenny has $1200 and is spending $40 per week. Kelsey has $120 and is saving $50 a week. In how many
Jenny has $1200 and is spending $40 per week. Kelsey has $120 and is saving $50 a week. In how many weeks will Jenny and Kelsey have the same amount of money? Jenny: Let w be the number of weeks. Spending means we subtract, so we set up a balance equation B(w): B(w) = 1200 - 40w Kelsey: Let w be the number of weeks. Saving means we add, so we set up a balance equation B(w): B(w) = 120 + 50w When they have the same amount of money, we set the balance equations equal to each other: 1200 - 40w = 120 + 50w To solve for w, we [URL='https://www.mathcelebrity.com/1unk.php?num=1200-40w%3D120%2B50w&pl=Solve']type this equation into our search engine[/URL] and we get: w = [B]12[/B]

Jenny has $40 in her checking account. If she writes a check for $19 find her new account balance
Jenny has $40 in her checking account. If she writes a check for $19 find her new account balance Writing a check means we take out of the account, so we subtract: Balance = $40 - $19 Balance = [B]$21[/B]

Jenny makes 9 dollars for each hour of work. Write an equation to represent her total pay p after wo
Jenny makes 9 dollars for each hour of work. Write an equation to represent her total pay p after working h hours. Since Jenny makes 9 dollars for each hour of work, then her total pay (p) is her hourly rate times the number of hours worked: [B]p = 9h[/B]

Jenny threw the javelin 4 metres further than Angus but 5 metres less than Cameron. if the combined
Jenny threw the javelin 4 metres further than Angus but 5 metres less than Cameron. if the combined distance thrown by the 3 friends is 124 metres, how far did Angus throw the javelin? Assumptions and givens: [LIST] [*]Let a be the distance Angus threw the javelin [*]Let c be the distance Cameron threw the javelin [*]Let j be the distance Jenny threw the javelin [/LIST] We're given 3 equations: [LIST=1] [*]j = a + 4 [*]j = c - 5 [*]a + c + j = 124 [/LIST] Since j is the common variable in all 3 equations, let's rearrange equation (1) and equation (2) in terms of j as the dependent variable: [LIST=1] [*]a = j - 4 [*]c = j + 5 [*]a + c + j = 124 [/LIST] Now substitute equation (1) and equation (2) into equation (3) for a and c: j - 4 + j + 5 + j = 124 To solve this equation for j, we [URL='https://www.mathcelebrity.com/1unk.php?num=j-4%2Bj%2B5%2Bj%3D124&pl=Solve']type it in our math engine[/URL] and we get: j = 41 The question asks how far Angus (a) threw the javelin. Since we have Jenny's distance j = 41 and equation (1) has j and a together, let's substitute j = 41 into equation (1): a = 41 - 4 a = [B]37 meters[/B]

Jenny went shoe shopping. Now she has 5 more pairs than her brother. Together they have 25 pairs. Ho
Jenny went shoe shopping. Now she has 5 more pairs than her brother. Together they have 25 pairs. How many pairs does Jenny have and how many pairs does her brother have? [U]Let j be the number of shoes Jenny has and b be the number of s hoes her brother has. Set up 2 equations:[/U] (1) b + j = 25 (2) j = b + 5 [U]Substitute (2) into (1)[/U] b + (b + 5) = 25 [U]Group the b terms[/U] 2b + 5 = 25 [U]Subtract 5 from each side[/U] 2b = 20 [U]Divide each side by b[/U] [B]b = 10 [/B] [U]Substitute b = 10 into (2)[/U] j = 10 + 5 [B]j = 15[/B]

Jeremy can plant 10 trees in 4 hours. How many trees can he plant in 10 hours?
Jeremy can plant 10 trees in 4 hours. How many trees can he plant in 10 hours? Set up a proportion of trees planted to hours where t is the number of trees planted in 10 hours. 10/4 = t/10 [URL='https://www.mathcelebrity.com/prop.php?num1=10&num2=t&den1=4&den2=10&propsign=%3D&pl=Calculate+missing+proportion+value']Type this expression into the search engine[/URL] and we get [B]t = 25[/B]. This means Jeremy can plant 25 trees in 10 hours.

Jeremy ran 27 laps on a track that was 1/8 mile long. Jimmy ran 15 laps on a track that as 1/4 mile
Jeremy ran 27 laps on a track that was 1/8 mile long. Jimmy ran 15 laps on a track that as 1/4 mile long. who ran farther [U]Calculate Jeremy's distance:[/U] Distance = Laps * Track length Jeremy distance = 27 * 1/8 Jeremy distance = 27/8 [U]Calculate Jimmy's distance:[/U] Distance = Laps * Track length Jeremy distance = 15* 1/4 Jeremy distance = 15/4 [COLOR=#000000]Using our [URL='https://www.mathcelebrity.com/fraction.php?frac1=27/8&frac2=15/4&pl=Compare']fraction comparison calculator[/URL], we see that [B]Jimmy [/B]ran farther[/COLOR]

Jerry rolls a dice 300 times what is the estimated numbers the dice rolls on 6
Jerry rolls a dice 300 times what is the estimated numbers the dice rolls on 6 Expected Value = Rolls * Probability Since a 6 has a probability of 1/6, we have: Expected Value = 300 * 1/6 Expected Value = [B]50[/B]

Jerry, an electrician, worked 7 months out the year. What percent of the year did he work?
Jerry, an electrician, worked 7 months out the year. What percent of the year did he work? We know that there are 12 months in a year. Percentage worked = Months worked in a year / months in a year * 100% Percentage worked = 7/12 * 100% Percentage worked = 0.5833333 * 100% Multiplying by 100 means we shift the decimal place 2 spaces to the right: Percentage worked = [B]58.33%[/B]

Jerry’s Bakery makes 144 muffins daily. How many muffins do they make in 7 days? Explain.
Jerry’s Bakery makes 144 muffins daily. How many muffins do they make in 7 days? Explain. Total muffins = Muffins per day * number of days Total muffins = 144 * 7 Total muffins = [B]1,008[/B]

Jessica has 16 pairs of shoes. She buys 2 additional pair of shoes every month. What is the slope in
Jessica has 16 pairs of shoes. She buys 2 additional pair of shoes every month. What is the slope in this situation? Set up a graph where months is on the x-axis and number of shoes Jessica owns is on the y-axis. [LIST=1] [*]Month 1 = (1, 18) [*]Month 2 = (2, 20) [*]Month 3 = (3, 22) [*]Month 4 = (4, 24) [/LIST] You can see for every 1 unit move in x, we get a 2 unit move in y. Pick any of these 2 points, and [URL='https://www.mathcelebrity.com/slope.php?xone=3&yone=22&slope=+2%2F5&xtwo=4&ytwo=24&pl=You+entered+2+points']use our slope calculator[/URL] to get: Slope = [B]2[/B]

Jessica has at most to spend $40 and a pair of pants and shirts. She finds a pair of pants for $12.
Jessica has at most to spend $40 and a pair of pants and shirts. She finds a pair of pants for $12. Shirts cost $4 each. How many shirts can she buy? [U]Calculate the amount for shirts:[/U] Amount for shirts = Total Spend - Cost of pants Amount for shirts = 40 - 12 Amount for shirts = 28 Calculate the amount of t-shirts Jessica can buy: T-shirts to buy = Amount for shirts / cost per shirt T-shirts to buy = 28/4 T-shirts to buy = [B]7[/B]

Jessica tutors chemistry. For each hour that she tutors, she earns 30 dollars. Let E be her earnings
Jessica tutors chemistry. For each hour that she tutors, she earns 30 dollars. Let E be her earnings (in dollars) after tutoring for H hours. Write an equation relating E to H . Then use this equation to find Jessicas earnings after tutoring for 19 hours. Set up a function of h hours for tutoring: [B]E(h) = 30h[/B] We need to find E(19) E(19) = 30(19) E(19) = [B]570[/B]

Jessie invests $3345 in the stock market. Over the 3 years she has this invested she gets an average
Jessie invests $3345 in the stock market. Over the 3 years she has this invested she gets an average return of 7.8%. How much will her investment be worth after the 3 years? 7.8% = 0.078, so we use our compound interest formula to find our balance after 3 years. Using our [URL='https://www.mathcelebrity.com/compoundint.php?bal=3345&nval=3+&int=7.8&pl=Annually']compound interest balance calculator[/URL], we get: [B]$4,190.37[/B]

Jessie works in a hat shop for 4 hours per day. She worked a total of 592 hours over the past year.
Jessie works in a hat shop for 4 hours per day. She worked a total of 592 hours over the past year. How many days did she turn up for work? Days worked = Total Hours Worked / Hours worked per day Days worked = 592/4 Days worked = [B]148 days[/B]

Jethro wants a swimming pool in his backyard, so he digs a rectangular hole with dimensions 40 feet
Jethro wants a swimming pool in his backyard, so he digs a rectangular hole with dimensions 40 feet long, 20 feet wide, and 5 feet deep. How many cubic feet of water will the pool hold? This is a rectangular solid. The volume is l x w x h: V = 40 x 20 x 5 V = [B]4,000 cubic feet[/B]

Jill and Jack are getting vegetables from the Farmer's Market. Jill buys 12 carrots and 8 tomatoes f
Jill and Jack are getting vegetables from the Farmer's Market. Jill buys 12 carrots and 8 tomatoes for $34. Jack buys 10 carrots and 7 tomatoes for $29. How much does each carrot and each tomato cost? Let the cost of carrots be c and the cost of tomatoes be t. Since the total cost is price times quantity, We're given two equations: [LIST=1] [*]12c + 8t = 34 <-- Jill [*]10c + 7t = 29 <-- Jack [/LIST] We have a system of equations. We can solve this one of three ways: [LIST=1] [*][URL='https://www.mathcelebrity.com/simultaneous-equations.php?term1=12c+%2B+8t+%3D+34&term2=10c+%2B+7t+%3D+29&pl=Substitution']Substitution Method[/URL] [*][URL='https://www.mathcelebrity.com/simultaneous-equations.php?term1=12c+%2B+8t+%3D+34&term2=10c+%2B+7t+%3D+29&pl=Elimination']Elimination Method[/URL] [*][URL='https://www.mathcelebrity.com/simultaneous-equations.php?term1=12c+%2B+8t+%3D+34&term2=10c+%2B+7t+%3D+29&pl=Cramers+Method']Cramer's Rule[/URL] [/LIST] No matter what method we choose, we get: [LIST] [*][B]t = 2[/B] [*][B]c = 1.5[/B] [/LIST]

Jill made 122 muffins. She put them into 3 boxes and has two muffins left. How many are in each box
Jill made 122 muffins. She put them into 3 boxes and has two muffins left. How many are in each box if they all contain the same amount of muffins? Let m equal the number of muffins per box. We're told that we have 3 boxes and 2 muffins left after filling up all 3 boxes. 3m + 2 = 122 To solve for m, we subtract 2 from each side: 3m + 2 - 2 = 122 - 2 Cancel the 2's on the left side and we get: 3m = 120 Divide each side by 3 to isolate m: 3m/3 = 120/3 Cancel the 3's on the left side and we get: m = [B]40[/B]

Jim and his family are touring Chicago. They want to visit the Willis Tower, hang out at Navy Pier,
Jim and his family are touring Chicago. They want to visit the Willis Tower, hang out at Navy Pier, and shop on Michigan Avenue before their dinner reservations at 4:15 P.M. They plan to spend 1 hour and 25 minutes at the Willis Tower, 1 hour and 40 minutes at Navy Pier, and 1 hour and 40 minutes shopping. What is the latest time Jim's family can start their tour of Chicago and still make it to dinner on time? First thing we want is how much time is Jim's family spending on pre-dinner activities [LIST=1] [*]1 hour and 25 minutes at Willis Tower [*]1 hour and 40 minutes at Navy Pier [*]1 hour and 40 minutes shopping [/LIST] Add these all up and we get: 3 hours and 105 minutes 105 minutes = 60 + 45 3 + 1 hours = 4 hours and 45 minutes IF dinner reservations start at 4:15, the latest Jim's family can start their tour is: 4:15 pm and go back 4 hours and 45 minutes We go back 5 hours and we get 11:15 am and add 15 minutes to get [B]11:30 AM [/B] 4:15 pm and go back 4 hours to get 12:15 pm Now go back another 45 minutes and we get 11:30 am

Jim has $440 in his savings account and adds $12 per week to the account. At the same time, Rhonda h
Jim has $440 in his savings account and adds $12 per week to the account. At the same time, Rhonda has $260 in her savings account and adds $18 per week to the account. How long will it take Rhonda to have the same amount in her account as Jim? [U]Set up Jim's savings function S(w) where w is the number of weeks of savings:[/U] S(w) = Savings per week * w + Initial Savings S(w) = 12w + 440 [U]Set up Rhonda's savings function S(w) where w is the number of weeks of savings:[/U] S(w) = Savings per week * w + Initial Savings S(w) = 18w + 260 The problems asks for w where both savings functions equal each other: 12w + 440 = 18w + 260 To solve for w, we [URL='https://www.mathcelebrity.com/1unk.php?num=12w%2B440%3D18w%2B260&pl=Solve']type this equation into our math engine[/URL] and we get: w = [B]30[/B]

Jim is 9 years older than June. Alex is 8 years younger than June. If the total of their ages is 82,
Jim is 9 years older than June. Alex is 8 years younger than June. If the total of their ages is 82, how old is the eldest of them Let j be Jim's age, a be Alex's age, and u be June's age. We have 3 given equations: [LIST=1] [*]j + a + u = 82 [*]j = u + 9 [*]a = u - 8 [/LIST] Substitute (2) and (3) into (1) (u + 9) + (u - 8) + u = 82 Combine Like Terms: 3u + 1 = 82 [URL='https://www.mathcelebrity.com/1unk.php?num=3u%2B1%3D82&pl=Solve']Type this equation into the search engine[/URL], and we get u = 27. The eldest (oldest) of the 3 is Jim. So we have from equation (2) j = u + 9 j = 27 + 9 [B]j = 36[/B]

Jim was thinking of a number. Jim adds 20 to it, then doubles it and gets an answer of 99.2. What wa
Jim was thinking of a number. Jim adds 20 to it, then doubles it and gets an answer of 99.2. What was the original number? Start with x. Add 20 to it x + 20 Double it 2(x + 20) Set this equal to 99.2 2(x + 20) = 99.2 Divide each side by 2: x + 20 = 49.6 Subtract 20 from each side: x = [B]29.6[/B]

Jim works for his dad and earns $400 every week plus $22 for every chair (c) he sells. Write an equa
Jim works for his dad and earns $400 every week plus $22 for every chair (c) he sells. Write an equation that can be used to determine jims weekly salary (S) given the number of chairs (c) he sells. [B]S(c) = 400 + 22c[/B]

jimmy has 5 apples and beth has 8 apples how many apples do they have together
jimmy has 5 apples and beth has 8 apples how many apples do they have together [U]The word [I]together[/I] means we add:[/U] Total Apples = Jimmy's apples + Beth's apples Total Apples = 5 + 8 Total Apples = [B]13[/B]

Jimmy was given $16 for washing the dog.He now has $47. How much money did he start with?
Jimmy was given $16 for washing the dog. He now has $47. How much money did he start with? Let his starting money be s. We're told: s + 16 = 47 To solve for s, we [URL='https://www.mathcelebrity.com/1unk.php?num=s%2B16%3D47&pl=Solve']type this equation into our search engin[/URL]e and we get: s = [B]31[/B]

Jina's test score average decreased by 10 points this semester. Write a signed number to represent t
Jina's test score average decreased by 10 points this semester. Write a signed number to represent this change in average. Let A be the original average. The new average is: A + (-10)

Jinas final exam has true/false questions, worth 3 points each, and multiple choice questions, worth
Jinas final exam has true/false questions, worth 3 points each, and multiple choice questions, worth 4 points each. Let x be the number of true/false questions she gets correct, and let y be the number of multiple choice questions she gets correct. She needs at least 76 points on the exam to get an A in the class. Using the values and variables given, write an inequality describing this. At least means greater than or equal to, so we have: [B]3x + 4y >= 76[/B]

Joanie multiplied 0.78 times 0.34 and got the product 26.52. What error did she make
Joanie multiplied 0.78 times 0.34 and got the product 26.52. What error did she make [B]She didn't move the decimal point over 2 spots[/B]: 0.78 * 0.34 = 0.2652

Joaquin buys 3 dozen lightbulbs. After changing the lightbulbs in his house,he has 15 lightbulbs lef
Joaquin buys 3 dozen lightbulbs. After changing the lightbulbs in his house,he has 15 lightbulbs left. How many lightbulbs did he use? [URL='https://www.mathcelebrity.com/quantcon.php?quant=3&pl=Calculate&type=dozen']Type 3 dozen into the search engine[/URL]. We get 36 units. Now, if Joaquin has 15 lightbulbs left, we subtract 15 from 36: 36 - 15 = [B]21 lightbulbs used[/B]

Jocelyn invested $3,700 in an account paying an interest rate of 1.5% compounded continuously. Assum
Jocelyn invested $3,700 in an account paying an interest rate of 1.5% compounded continuously. Assuming no deposits or withdrawals are made, how much money would be in the account after 6 years? Using our [URL='https://www.mathcelebrity.com/simpint.php?av=&p=3700&int=1.5&t=6&pl=Continuous+Interest']continuous interest with balance calculator[/URL], we get: [B]$4,048.44[/B]

Jody is buying a scrapbook and sheets of designer paper. She has $40 and needs at least $18.25 to bu
Jody is buying a scrapbook and sheets of designer paper. She has $40 and needs at least $18.25 to buy the scrapbook. Each sheet of paper costs $0.34. How many sheets of paper can she buy? Set up a cost equation for the number of pieces of paper (p): 0.34p + 18.25 <= 40 <-- we have an inequality since we can't go over 40 [URL='https://www.mathcelebrity.com/1unk.php?num=0.34p%2B18.25%3C%3D40&pl=Solve']Type this inequality into our search engine[/URL] and we get: p <= 63.97 We round down, so we get p = [B]63[/B].

Joe buys 9 cds for the same price, he also buys a dvd for 20. His total bill is 119. Find the cost o
Joe buys 9 cds for the same price, he also buys a dvd for 20. His total bill is 119. Find the cost of one cd. [U]Let c be the cost of one CD. Set up the equation:[/U] 9c + 20 = 119 [U]Use the [URL='http://www.mathcelebrity.com/1unk.php?num=9c%2B20%3D119&pl=Solve']equation solver[/URL]:[/U] [B]c = 11[/B]

Joe had saved $264. He spent 3/8 of that to buy a camera. How much did the camera cost?
Joe had saved $264. He spent 3/8 of that to buy a camera. How much did the camera cost? [URL='https://www.mathcelebrity.com/fraction.php?frac1=264&frac2=3%2F8&pl=Multiply']264 *3/8[/URL] = [B]99[/B]

Joe is paid a 4% commission on all his sales in addition to a $500 per month salary. In May, his sal
Joe is paid a 4% commission on all his sales in addition to a $500 per month salary. In May, his sales were $100,235. How much money did he earn in May? [U]The commission and salary formula is:[/U] Earnings = Salary + Commission Percent * Sales Plugging in our numbers with 4% as 0.04, we get: Earnings = 500 + 0.04 * 100235 Earnings = 500 + 4009.40 Earnings = [B]4,509.40[/B]

Joe opens a bank account that starts with $20 and deposits $10 each week. Bria has a different accou
Joe opens a bank account that starts with $20 and deposits $10 each week. Bria has a different account that starts with $1000 but withdraws $15 each week. When will Joe and Bria have the same amount of money? Let w be the number of weeks. Deposits mean we add money and withdrawals mean we subtract money. [U]Joe's Balance function B(w) where w is the number of weeks:[/U] 20 + 10w [U]Bria's Balance function B(w) where w is the number of weeks:[/U] 1000 - 15w [U]The problem asks for when both balances will be the same. So we set them equal to each other and solve for w:[/U] 20 + 10w = 1000 - 15w To solve for w, we [URL='https://www.mathcelebrity.com/1unk.php?num=20%2B10w%3D1000-15w&pl=Solve']type this equation into our search engine[/URL] and we get: w = 39.2 We round up to full week and get: w = [B]40[/B]

joe plans to watch 3 movies each month. white an equation to represent the total number of movies n
joe plans to watch 3 movies each month. white an equation to represent the total number of movies n that he will watch in m months Build movie equation. 3 movies per month at m months means we multiply: [B]n = 3m[/B]

Joel bought 88 books. Some books cost $13 each and some cost $17 each. In all, he spent $128. Which
Joel bought 88 books. Some books cost $13 each and some cost $17 each. In all, he spent $128. Which system of linear equations represents the given situation? Let a be the number of the $13 book, and b equal the number of $17 books. We have the following system of linear equations: [LIST=1] [*][B]a + b = 88[/B] [*][B]13a + 17b = 128[/B] [/LIST] To solve this system, use our calculator for the following methods: [LIST] [*][URL='http://www.mathcelebrity.com/simultaneous-equations.php?term1=a+%2B+b+%3D+88&term2=13a+%2B+17b+%3D+128&pl=Substitution']Substitution[/URL] [*][URL='http://www.mathcelebrity.com/simultaneous-equations.php?term1=a+%2B+b+%3D+88&term2=13a+%2B+17b+%3D+128&pl=Elimination']Elimination[/URL] [*][URL='http://www.mathcelebrity.com/simultaneous-equations.php?term1=a+%2B+b+%3D+88&term2=13a+%2B+17b+%3D+128&pl=Cramers+Method']Cramers Method[/URL] [/LIST]

Joelle had $24 to spend on seven pencils. After buying them she had $10. How much did each pencil co
Joelle had $24 to spend on seven pencils. After buying them she had $10. How much did each pencil cost? Subtract the $10 left over from the $24 Joelle started with. $24 - $10 = $14 Therefore, Joelle spent $14 on seven pencils. Cost per pencil = Total Pencil Spend / Number of pencils Cost per pencil = 14 / 7 Cost per pencil = [B]$2[/B]

Joey and Romnick play in the same soccer team. Last Saturday, Romnick scored 3 more goals than Joey,
Joey and Romnick play in the same soccer team. Last Saturday, Romnick scored 3 more goals than Joey,but together they scored less than 9 goals. What are the possible number of goal Romnick scored? Let j be Joey's goals Let r by Romnick's goals We're given 1 equation and 1 inequality: [LIST=1] [*]r = j + 3 [*]r + j < 9 [/LIST] Rearranging equation 1 for j, we have: [LIST=1] [*]j = r - 3 [*]r + j < 9 [/LIST] Substitute equation (1) into inequality (2) for j: r + r - 3 < 9 2r - 3 < 9 [URL='https://www.mathcelebrity.com/1unk.php?num=2r-3%3C9&pl=Solve']Typing this inequality into our math engine[/URL], we get: [B]r < 6[/B]

Joey puts $1,000.00 into an account to use for school expenses. The account earns 12% interest, comp
Joey puts $1,000.00 into an account to use for school expenses. The account earns 12% interest, compounded annually. How much will be in the account after 6 years? Using our [URL='http://www.mathcelebrity.com/compoundint.php?bal=1000&nval=6&int=12&pl=Annually']balance calculator[/URL], we get [B]$1,973.82[/B]

Joey withdrew $125 from his savings account. After the withdrawal, his balance was $785. How much wa
Joey withdrew $125 from his savings account. After the withdrawal, his balance was $785. How much was in his account initially? [U]Withdrawal means he took money out, which means his initial balance is found by adding back the withdrawal:[/U] Initial Balance = Current Balance + Withdrawal Initial Balance = 785 + 125 Initial Balance = [B]910[/B]

John bought a painting for $600 and sold it for $648. Find the profit as a percentage of the cost.
John bought a painting for $600 and sold it for $648. Find the profit as a percentage of the cost. [U]Calculate the profit:[/U] Profit = Sale Price - Purchase price Profit = 648 - 600 Profit = 48 [U]Calculate Profit percentage of cost =[/U] Profit percentage of cost = 100% * Profit/cost Profit percentage of cost = 100% * 48 / 600 Profit percentage of cost = [B]8%[/B]

john had 27 sweets. he gave 2/3 of his sweets to mary and kept the rest for himself
john had 27 sweets. he gave 2/3 of his sweets to mary and kept the rest for himself. Mary has 2/3 * 27 = [B]18[/B] John has 27[B] [/B]- 18[B] = 9[/B]

John has 30 marbles, 18 of which are red and 12 of which are blue. Jane has 20 marbles, all of them
John has 30 marbles, 18 of which are red and 12 of which are blue. Jane has 20 marbles, all of them either red or blue. If the ratio of the red marbles to the blue marbles is the same for both John and Jane, then John has how many more blue marbles than Jane? John's red ratio = 18/30 Using a [URL='https://www.mathcelebrity.com/gcflcm.php?num1=18&num2=30&num3=&pl=GCF+and+LCM']GCF for (18, 30)[/URL], we get 6. Divide top and bottom of 18/30 by 6, we get 3/5 John's blue ratio is 12/30 Using a [URL='https://www.mathcelebrity.com/gcflcm.php?num1=12&num2=30&num3=&pl=GCF']GCF of (12, 30)[/URL], we get 6. Divide top and bottom of 12/30 by 6, we get 2/5 Use these same ratios for Jane, we get: Red: 3(20)/5 = 12 Blue: 20 - 12 = 8 Now the problem asks how many more blue marbles John has then Jane. We have 12 - 8 = [B]4[/B].

John has x number of marbles. His friend gave him 6 marbles more. Write an expression for the total
John has x number of marbles. His friend gave him 6 marbles more. Write an expression for the total number of marbles John now has. More means we add: [B]x + 6[/B]

John is paid a retainer of $550 a week as well as a 2% commission on sales made. Find his income for
John is paid a retainer of $550 a week as well as a 2% commission on sales made. Find his income for the week if in one week he sells cars worth of $80000 Set up the income function C(s) where s is the number of sales for a week. Since 2% can be written as 0.02, we have: I(s) = Retainer + 2% of sales I(s) = 550 + 0.02s The problem asks for a I(s) where s = 80,000: I(s) = 550 + 0.02(80000) I(s) = 550 + 1600 I(s) = [B]2150[/B]

John mows 3 lawns in 4 hours, Paul mows 5 lawns in 6 hours. Who mows faster?
John mows 3 lawns in 4 hours, Paul mows 5 lawns in 6 hours. Who mows faster? To see who mows faster, we set up fractions with a common denominator. You can see this by running this statement in the calculator: [URL='https://www.mathcelebrity.com/fraction.php?frac1=3/4&frac2=5/6&pl=Compare']3/4 or 5/6[/URL] You'll see that 5/6 is larger, so Paul mores more lawns per hour.

John read the first 114 pages of a novel, which was 3 pages less than 1/3
John read the first 114 pages of a novel, which was 3 pages less than 1/3 Set up the equation for the number of pages (p) in the novel 1/3p - 3 = 114 Add 3 to each side 1/3p = 117 Multiply each side by 3 [B]p = 351[/B]

John read the first 114 pages of a novel, which was 3 pages less than 1/3 of the novel.
John read the first 114 pages of a novel, which was 3 pages less than 1/3 of the novel. Let n be the number of pages in the novel. We have: 1/3n - 3 = 114 Multiply each side by 3: n - 9 = 342 Using our [URL='http://www.mathcelebrity.com/1unk.php?num=n-9%3D342&pl=Solve']equation solver[/URL], we get [B]n = 351[/B].

John spent $10.40 on 5 notebooks and 5 pens. Ariana spent $7.00 on 4 notebooks and 2 pens. What is t
John spent $10.40 on 5 notebooks and 5 pens. Ariana spent $7.00 on 4 notebooks and 2 pens. What is the ost of 1 notebook and what is the cost of 1 pen? Let the number of notebooks be n and the number of pens be p. We have two equations: [LIST=1] [*]5n + 5p = 10.40 [*]4n + 2p = 7 [/LIST] Using our [URL='http://www.mathcelebrity.com/simultaneous-equations.php?term1=5n+%2B+5p+%3D+10.40&term2=4n+%2B+2p+%3D+7&pl=Cramers+Method']simultaneous equation calculator[/URL], we have: [LIST] [*][B]n = 1.42[/B] [*][B]p = 0.66[/B] [/LIST]

John took 20,000 out of his retirement and reinvested it. He earned 4% for one investment and 5% on
John took 20,000 out of his retirement and reinvested it. He earned 4% for one investment and 5% on the other. How much did he invest in each if the total amount earned was 880? The first principal portion is x. Which means the second principal portion is 20,000 - x. We have: 0.04x + 0.05(20,000 - x) = 880 0.04x + 1,000 - 0.05x = 880 Group like terms: -0.01x + 1000 = 880 Using our [URL='http://www.mathcelebrity.com/1unk.php?num=-0.01x%2B1000%3D880&pl=Solve']equation solver[/URL], we get x = [B]12,000[/B]. Which means the other fund has 20,000 - 12,000 = [B]8,000[/B].

Johnny Rocket can run 300 meters in 90 seconds. If his speed remains constant, how far could he ru
Johnny Rocket can run 300 meters in 90 seconds. If his speed remains constant, how far could he run in 500 seconds? Round to one decimal place. Set up the distance equation: Distance = Rate * Time 300 = 90r Solving this equation for r, we [URL='https://www.mathcelebrity.com/1unk.php?num=300%3D90r&pl=Solve']type it in our search engine[/URL] and we get: r = 3.333 For 500 seconds, we set up our distance equation again: Distance = 500 * 3.333333 Distance = [B]1666.7 meters[/B]

Johns grade has 3 classrooms. Each classroom has 14 tables. Two students sit at each table about how
Johns grade has 3 classrooms. Each classroom has 14 tables. Two students sit at each table about how many students are there in all? 3 classrooms * 14 tables per classroom = 42 tables 2 students per table * 42 tables = 84 students

Joint Variation Equations
Free Joint Variation Equations Calculator - Given a joint variation (jointly proportional) of a variable between two other variables with a predefined set of conditions, this will create the joint variation equation and solve based on conditions. Also called combined variation.

Jon earned money baby-sitting. He spent 1/4 of the money on a guitar and then he gave 1/4 of what wa
Jon earned money baby-sitting. He spent 1/4 of the money on a guitar and then he gave 1/4 of what was left to charity. If he has $108 left, how much money did he start with? Calculate initial spend: Charity = 1/4 * 3/4 left = 3/16 [URL='https://www.mathcelebrity.com/fraction.php?frac1=1%2F4&frac2=3%2F16&pl=Add']1/4 + 3/16[/URL] = 7/16 This means he has 1 = 7/16 left 16/16 - 7/16 = 9/16 Let the starting amount be s: If he has 108 left, then we have [URL='https://www.mathcelebrity.com/proportion-calculator.php?num1=9s&num2=108&den1=16&den2=1&propsign=%3D&pl=Calculate+missing+proportion+value']9s/16 = 108[/URL] s =$[B]192[/B]

Jonathan earns a base salary of $1500 plus 10% of his sales each month. Raymond earns $1200 plus 15%
Jonathan earns a base salary of $1500 plus 10% of his sales each month. Raymond earns $1200 plus 15% of his sales each month. How much will Jonathan and Raymond have to sell in order to earn the same amount each month? [U]Step 1: Set up Jonathan's sales equation S(m) where m is the amount of sales made each month:[/U] S(m) = Commission percentage * m + Base Salary 10% written as a decimal is 0.1. We want decimals to solve equations easier. S(m) = 0.1m + 1500 [U]Step 2: Set up Raymond's sales equation S(m) where m is the amount of sales made each month:[/U] S(m) = Commission percentage * m + Base Salary 15% written as a decimal is 0.15. We want decimals to solve equations easier. S(m) = 0.15m + 1200 [U]The question asks what is m when both S(m)'s equal each other[/U]: The phrase [I]earn the same amount [/I]means we set Jonathan's and Raymond's sales equations equal to each other 0.1m + 1500 = 0.15m + 1200 We want to isolate m terms on one side of the equation. Subtract 1200 from each side: 0.1m + 1500 - 1200 = 0.15m + 1200 - 1200 Cancel the 1200's on the right side and we get: 0.1m - 300 = 0.15m Next, we subtract 0.1m from each side of the equation to isolate m 0.1m - 0.1m + 300 = 0.15m - 0.1m Cancel the 0.1m terms on the left side and we get: 300 = 0.05m Flip the statement since it's an equal sign to get the variable on the left side: 0.05m = 300 To solve for m, we divide each side of the equation by 0.05: 0.05m/0.05 = 300/0.05 Cancelling the 0.05 on the left side, we get: m = [B]6000[/B]

Jordan has already scored 153 points this basketball season. If he scores 17 points per game, which
Jordan has already scored 153 points this basketball season. If he scores 17 points per game, which inequality represents the number of addional games he needs to play in order to score at least 255 points for the season? Let g be the number of games Jordan plays. Total points per game is 17g. And he’s already scored 153. So we need 17g + 153 to be [I]at least [/I]255. The phrase at least means greater than or equal to, so we use the >= operator for our inequality: [B]17g + 153 >= 255[/B]

Jordan practices his trombone for 45 minutes each day. Write an expression for the number of minutes
Jordan practices his trombone for 45 minutes each day. Write an expression for the number of minutes Jordan practices she practices the trombone in d days. Let m = the number of minutes practiced. We ave: [B]m = 45d[/B]

Jose bought 2 movie tickets and a box of popcorn. The popcorn cost $6, and he spent a total of $24.
Jose bought 2 movie tickets and a box of popcorn. The popcorn cost $6, and he spent a total of $24. How much did each ticket cost? Subtract the cost of the popcorn: $24 - $6 = $18 2 movie tickets cost $18, so each movie ticket cost $18/2 = [B]$9[/B]

Jose bought a shirt for $25.00. The sales tax was 8%. If Jose paid with $40, what was his change?
Jose bought a shirt for $25.00. The sales tax was 8%. If Jose paid with $40, what was his change? Total Bill is 25 * 1.08 = $27 Change due = 40 - 27 = $[B]13[/B]

Jose earned 60 points on a game show. In the next round he lost 64 points then gained 12 points and
Jose earned 60 points on a game show. In the next round he lost 64 points then gained 12 points and at last lost 28 points. What was his score at the end of the show? Start with 60 points: 60 lose 64 means we subtract 64 from our points 60 - 64 = -4 Gained 12 means we add 12 to our points: -4 + 12 = 8 Lost 28 means we subtract 28 from our points: 8 - 28 = [B]-20 points[/B]

Jose has scored 556 points on his math tests so far this semester. To get an A for the semester, he
Jose has scored 556 points on his math tests so far this semester. To get an A for the semester, he must score at least 660 points. Write and solve an inequality to find the minimum number of points he must score on the remaining tests, n, in order to get an A. We want to know n, such that 556 + n >= 660. <-- We use >= symbol since at least means greater than or equal to. 556 + n >= 660 Use our [URL='http://www.mathcelebrity.com/1unk.php?num=556%2Bn%3E%3D660&pl=Solve']equation/inequality calculator[/URL], we get [B]n >= 104[/B]

Josh currently bench presses 150 lbs. He increases that amount by 10% a month for 3 months. About ho
Josh currently bench presses 150 lbs. He increases that amount by 10% a month for 3 months. About how much can he bench press now? We have 150(1.1)^3. We can also write this as 150(1.1)(1.1)(1.1). The 10% compounds. After 3 months, Josh benches 199.65 lbs, or approximately 200 lbs.

Jow buys 9 CD’s for the same price, and also a cassette tape for $9.45. His total bill was 118.89. W
Jow buys 9 CD’s for the same price, and also a cassette tape for $9.45. His total bill was 118.89. What was the cost of one CD? Let the price of each cd be c. We're given the equation: 9c + 9.45 = 118.89 [URL='https://www.mathcelebrity.com/1unk.php?num=9c%2B9.45%3D118.89&pl=Solve']We type this equation into our search engine[/URL] and we get: c = [B]12.16[/B]

JP's age is twice the age of Reyna. The sum of their ages does not exceed 51
JP's age is twice the age of Reyna. The sum of their ages does not exceed 51 Let JP's age be j. Let Reyna's age be r. We're given two expressions: [LIST=1] [*]w = 2r [*]r + w <= 51. ([I]Does not exceed means less than or equal to)[/I] [/LIST] We substitute (1) into (2) for w to get the inequality: r + 2r <= 51 To solve this inequality, we type it in our search engine and we get: [B]r <= 17[/B]

Juan has d dimes and q quarters in his pocket. The total value of the coins is less than $14.75. Whi
Juan has d dimes and q quarters in his pocket. The total value of the coins is less than $14.75. Which inequality models this situation? [U]Let d be the number of dimes and q be the number of quarters[/U] [B]0.1d + 0.25q < 14.75[/B]

Juan has d dimes and q quarters in his pocket. The total value of the coins is less than $14.75. Whi
Juan has d dimes and q quarters in his pocket. The total value of the coins is less than $14.75. Which inequality models this situation? Since dimes are worth $0.10 and quarters are worth $0.25, we have: [B]0.10d + 0.25q < 14.75[/B]

Juan is going on a flight to the beach. his luggage weighs 36 pounds. The bag weighs 4 pounds more t
Juan is going on a flight to the beach. his luggage weighs 36 pounds. The bag weighs 4 pounds more than the weight of 2 small bags of beach toys. Which equation can be used to find the weight in pounds of each bag of beach toys? Let b be the weight of each bag of beach toys. We're given the following relationship: 2b -4 = 36 To solve this equation for b, we [URL='https://www.mathcelebrity.com/1unk.php?num=2b-4%3D36&pl=Solve']type it in our math engine[/URL] and we get: b = [B]20[/B]

Juan runs out of gas in a city. He walks 30yards west and then 16 yards south looking for a gas stat
Juan runs out of gas in a city. He walks 30yards west and then 16 yards south looking for a gas station. How far is he from his starting point? Juan is located on a right triangle. We calculate the hypotenuse: 30^2 + 16^2 = Hypotenuse^2 900 + 256 = Hypotenuse^2 Hypotenuse^2 = 1156 Take the square root of each side: [B]Hypotenuse = 34 yards[/B]

Juan spent at most $2.50 on apples and oranges. He bought 5 apples at $0.36 each. What is the most h
Juan spent at most $2.50 on apples and oranges. He bought 5 apples at $0.36 each. What is the most he spent on oranges? Let a be spending apples and o be spending on oranges, we have: [LIST=1] [*]a + o <= 2.36 <-- At most means less than or equal to [*]a = 5 * 0.36 = 1.8 [/LIST] Substitute (2) into (1) 1.8 + o <= 2.36 Subtract 1.8 from each side [B]o <= 0.56[/B]

Julia has a bucket of water that weighs 10lbs. The total weight is 99% water. She leaves the bucke
Julia has a bucket of water that weighs 10lbs. The total weight is 99% water. She leaves the bucket outside overnight and some of the water evaporates, in the morning the water is only 98% of the total weight. What is the new weight? Setup the proportion: 0.99/10 = 0.98/w Using our [URL='http://www.mathcelebrity.com/prop.php?num1=0.99&num2=0.98&den1=10&den2=w&propsign=%3D&pl=Calculate+missing+proportion+value']proportion calculator[/URL], we get [B]w = 9.899 lbs[/B].

Julia owes 18.20 for the month of November. Her plan costs 9.00 for the first 600 text messages and
Julia owes 18.20 for the month of November. Her plan costs 9.00 for the first 600 text messages and .10 cents for additional texts. How many texts did she send out? Let m be the number of messages. We have a cost function of: C(m) = 9 + 0.1(m - 600) We are given C(m) = 18.20 18.20 = 9 + 0.1(m - 600) 18.20 = 9 + 0.1m - 60 Combine like terms: 18.20 = 0.1m - 51 Add 51 to each side 0.1m = 69.20 Divide each side by 0.1 [B]m = 692[/B]

Juliana answered 42 times correctly on an 80-item quiz, while her sister Angela answered 21 items co
Juliana answered 42 times correctly on an 80-item quiz, while her sister Angela answered 21 items correctly on a 40-item quiz. Do they have the same portion of correct answers? Let's compare based on correct answers to questions: Juliana = 42/80 = 0.525 Angela = 21/40 = 0.525 So yes, they do have the same portion of correct answers. But there's another way to solve this: [LIST=1] [*]Divide Juliana's the top and bottom of Juliana's fraction by 2. [*]We picked 2 as a GCF shown in our calculator. [*]Type [URL='https://www.mathcelebrity.com/gcflcm.php?num1=42&num2=80&num3=&pl=GCF']GCF of 42 and 80[/URL]. [/LIST] Divide top and bottom of Juliana's fraction by the GCF of 2 42/2 = 80/2 = 21/40 This ratio equals Angela's.

Julie has $300 to plan a dance. There is a one-time fee of $75 to reserve a room. It also costs $1.5
Julie has $300 to plan a dance. There is a one-time fee of $75 to reserve a room. It also costs $1.50 per person for food and drinks. What is the maximum number of people that can come to the dance? Let each person be p. We have the following relationship for cost: 1.50p + 75 <=300 We use the <= sign since we cannot go over the $300 budget. [URL='https://www.mathcelebrity.com/1unk.php?num=1.50p%2B75%3C%3D300&pl=Solve']We type this inequality into our search engine[/URL], and we get: p <= 150 Since we have the equal sign within the inequality, the maximum number of people that can come to the dance is [B]150.[/B]

Julie has $48 to spend at a carnival. The carnival charges $8 for admission and $5 per ride. What is
Julie has $48 to spend at a carnival. The carnival charges $8 for admission and $5 per ride. What is the maximum number of rides Julie can go on? Subtract admission charges, since that money is gone: $48 - $8 = $40 left over If rides cost $5, we can go on $40/$5 = [B]8 rides[/B] maximum.

Julie is making a documentary about how Boxerville residents see their town. If she talks to 7 peopl
Julie is making a documentary about how Boxerville residents see their town. If she talks to 7 people, and each interview lasts 4 minutes, how long will the film be? 7 people * 4 minutes each = [B]28 minutes[/B]

Julien spent 5 hours and 44 minutes mowing the lawn and 3 hours and 24 minutes trimming the hedge an
Julien spent 5 hours and 44 minutes mowing the lawn and 3 hours and 24 minutes trimming the hedge and some shrubs. How long did he work altogether? Add the minutes: 44 + 24 = 68 Step 1: 68 minutes is 1 hour and 8 minutes. So we take the 1 hour and add it to the 5 hours of mowing the lawn and 3 hours of trimming the hedge and we get: 5 + 3 + 1 = 9 hours We take the 8 minutes of Step 1 and we have: [B]9 hours and 8 minutes[/B]

Julio had $20 in his account. He made two withdrawals of $15 and $25, and then he deposits $28. What
Julio had $20 in his account. He made two withdrawals of $15 and $25, and then he deposits $28. What is his account balance now? Note: Balances add and Withdrawals subtract. So we have: 20 - 15 - 25 + 28 [B]8[/B]

Julio had a coin box that consisted of only quarters and dimes. The number of quarters was three tim
Julio had a coin box that consisted of only quarters and dimes. The number of quarters was three times the number of dimes. If the number of dimes is n, what is the value of coins in the coin box? Set up monetary value: [LIST] [*]Value of the dimes = 0.1n [*]Value of the quarters = 0.25 * 3n = 0.75n [/LIST] Add them together [B]0.85n[/B]

Julio has $150. Each week, he saves an additional $10. Write a function f(x) that models the total a
Julio has $150. Each week, he saves an additional $10. Write a function f(x) that models the total amount of money Julio has after x weeks f(x) = Savings per week * number of weeks + starting amount f(x) = [B]10x + 150[/B]

Julius Caesar was born and 100 BC and was 66 years old when he died in which year did he die?
Julius Caesar was born and 100 BC and was 66 years old when he died in which year did he die? BC means "Before Christ". On a timeline, it represents a negative number, where year 0 is the birth of Christ. So we have -100 + 66 = -34 -34 means [B]34 BC[/B].

July has 31 days how many seconds are there in july
July has 31 days how many seconds are there in July Using our [URL='https://www.mathcelebrity.com/timecon.php?quant=31&pl=Calculate&type=day']time conversion calculator[/URL], we get: 31 days = [B]2,678,400 seconds[/B]

Justin is older than Martina. The difference in their ages is 22 and the sum of their ages is 54. Wh
Justin is older than Martina. The difference in their ages is 22 and the sum of their ages is 54. What age is Martina? [U]Assumptions and givens:[/U] [LIST] [*]Let Justin's age be j [*]Let Martina's age be m [*]j > m ([I]since Justin is older than Martina[/I]) [/LIST] We're given the following equations : [LIST=1] [*]j - m = 22 [*]j + m = 54 [/LIST] Since the coefficients of m are opposites, we can take a shortcut using the [I]elimination method[/I] and add equation (1) to equation (2) (j + j) + (m - m) = 22 + 54 2j = 76 To solve for j, we [URL='https://www.mathcelebrity.com/1unk.php?num=2j%3D76&pl=Solve']type this equation into our math engine[/URL] and we get: j = 38 The question asks for Martina's age (m), so we can pick equation (1) or equation (2). Let's use equation (1): 38 - m = 22 To solve this equation for m, we [URL='https://www.mathcelebrity.com/1unk.php?num=38-m%3D22&pl=Solve']type it in our math engine[/URL] and we get: m = [B]16[/B]

k add 2 multiply by 6 then subtract 8
k add 2 multiply by 6 then subtract 8 k add 2: k + 2 Multiply by 6: 6(k + 2) Then subtract 8: [B]6(k + 2) - 8[/B]

k add d , multiply by e , then subtract f .
k add d , multiply by e , then subtract f . [LIST] [*]k add d: k + d [*]Multiply by e: e(k + d) [*]Then subtract f: [B]e(k + d) - f[/B] [/LIST]

k equals the sum of h and 23
The sum of h and 23 means we add: h + 23 k equals means we set our expression above equal to k h + 23 = k

k increased by the sum of 3 and j
k increased by the sum of 3 and j The sum of 3 and j 3 + j k increased by this sum: [B]k + 3 + j[/B]

K varies inversely with square root of m and directly with the cube of n.
K varies inversely with square root of m and directly with the cube of n. [LIST] [*]We take a constant c as our constant of proportionality. [*]The word inversely means we divide [*]The word directly means we multiply [/LIST] [B]k = cn^3/sqrt(m)[/B]

k varies jointly with m,n, p
k varies jointly with m,n, p The phrase [I]varies jointly[/I] means we have a constant c such that: [B]k= cmnp[/B]

k=g-a/5 for g
k=g-a/5 for g Add a/5 to each side; k + a/5 = g - a/5 + a/5 Cancel the a/5 terms on the right side, and we get: g = [B]k + a/5[/B]

Kaitlin is a software saleswoman. Let y represent her total pay (in dollars). Let x represent the nu
Kaitlin is a software saleswoman. Let y represent her total pay (in dollars). Let x represent the number of copies of Math is Fun she sells. Suppose that x and y are related by the equation 2500+110x=y. What is Kaitlin totalm pay if she doesnt sell any copies of Math is Fun? We want the value of y when x = 0. y = 2500 + 110(o) y = 2500 + 0 [B]y = 2500[/B]

Kamara has a square fence kennel area for her dogs in the backyard. The area of the kennel is 64 ft
Kamara has a square fence kennel area for her dogs in the backyard. The area of the kennel is 64 ft squared. What are the dimensions of the kennel? How many feet of fencing did she use? Explain. Area of a square with side length (s) is: A = s^2 Given A = 64, we have: s^2 = 64 [URL='https://www.mathcelebrity.com/radex.php?num=sqrt(64%2F1)&pl=Simplify+Radical+Expression']Typing this equation into our math engine[/URL], we get: s = 8 Which means the dimensions of the kennel are [B]8 x 8[/B]. How much fencing she used means perimeter. The perimeter P of a square with side length s is: P = 4s [URL='https://www.mathcelebrity.com/square.php?num=8&pl=Side&type=side&show_All=1']Given s = 8, we have[/URL]: P = 4 * 8 P = [B]32[/B]

Kamille is calculating the length of diagonal on a picture board and gets a solution of the square r
Kamille is calculating the length of diagonal on a picture board and gets a solution of the square root of 58. She needs to buy the ribbon to put across the diagonal of the board, so she estimates that she will need at least 60 inches of ribbon to cover the diagonal. Is she correct? Explain. [URL='https://www.mathcelebrity.com/powersq.php?num=sqrt%2858%29&pl=Calculate']The square root of 58 [/URL]has an answer between 7 and 8. So Kamille is [B]incorrect[/B]. She needs much less than 60 inches of ribbon. She needs less than 8 inches of ribbon.

Karen bought a bucket of popcorn at the movies for $5. She also bought some candy for $2 each. Karen
Karen bought a bucket of popcorn at the movies for $5. She also bought some candy for $2 each. Karen has to spend less than $15 on the popcorn and candy. Which inequality can be used to find c, the number of candies that Karen could have bought? Since the candy cost is the product of price and quantity, we have: 2c + 5 < 15 To solve this inequality for c, we [URL='https://www.mathcelebrity.com/1unk.php?num=2c%2B5%3C15&pl=Solve']type it in our math engine [/URL]and we get: [B]c < 5[/B]

Karen earns $20 per hour and already has $400 saved, and wants to save $1200. How many hours until b
Karen earns $20 per hour and already has $400 saved, and wants to save $1200. How many hours until bob gets his $1200 goal? Set up he savings function S(h) where h is the number of hours needed: S(h) = savings per hour * h + current savings amount S(h) = 20h + 400 The question asks for h when S(h) = 1200: 20h + 400 = 1200 To solve for h, we [URL='https://www.mathcelebrity.com/1unk.php?num=20h%2B400%3D1200&pl=Solve']type this equation into our search engine[/URL] and we get: h = [B]40[/B]

Karen wants to buy new shoes. There is a promotion for 3 pairs of sneakers for $450.75, how much wou
Karen wants to buy new shoes. There is a promotion for 3 pairs of sneakers for $450.75, how much would one pair of sneakers cost? Cost per sneaker = Total Cost / number of sneakers Cost per sneaker = 450.75/3 Cost per sneaker = [B]150.25[/B]

Karin has 3 to spend in the arcade. The game she likes costs 50c per play. What are the possible num
Karin has 3 to spend in the arcade. The game she likes costs 50c per play. What are the possible numbers of times that she can play? [U]Let x = the number of games Karin can play with her money[/U] 0.5x = 3 [U]Divide each side by 0.5[/U] [B]x = 6[/B]

Karleys bank account was negative $12.14. she then deposited $21.63. What was her account balance
Karleys bank account was negative $12.14. she then deposited $21.63. What was her account balance negative 12.14 can be written as -12.14 She then deposited 21.63 which means we add 21.63 to her bank account balance: +21.63 Final account balance is: -12.14 + 21.63 = [B]$9.49[/B]

Karmen just got hired to work at Walmart. She spent $15 on her new uniform and she gets paid $8 per
Karmen just got hired to work at Walmart. She spent $15 on her new uniform and she gets paid $8 per hour. Write an equation that represents how much money she profits after working for a certain number of hours. How many hours will she have to work for in order to buy a new snowboard ( which costs $450) Her profit equation P(h) where h is the number of hours worked is: [B]P(h) = 8h - 15[/B] Note: [I]We subtract 15 as the cost of Karmen's uniform. [/I] Next, we want to see how many hours Karmen must work to buy a new snowboard which costs $450. We set the profit equation equal to $450 8h - 15 = 450 [URL='https://www.mathcelebrity.com/1unk.php?num=8h-15%3D450&pl=Solve']Typing 8h - 15 = 450 into the search engine[/URL], we get h = 58.13. We round this up to 59 hours.

Kartek bought 86 pizzas for a school party. If there are 516 people at his school, how much pizza sh
Kartek bought 86 pizzas for a school party. If there are 516 people at his school, how much pizza should each person get? Setup unit slices: [URL='https://www.mathcelebrity.com/search.php?q=86%2F516']86 pizzas / 516 people[/URL] = [B]1/6 pizza per person[/B]

kate is twice as old as her sister mars. the sum of their ages is 24. find their ages.
kate is twice as old as her sister mars. the sum of their ages is 24. find their ages. Let k be Kate's age Let m be Mars's age We're given two equations: [LIST=1] [*]k = 2m. (Because twice means multiply by 2) [*]k + m = 24 [/LIST] Substitute equation (1) for k into equation (2): 2m + m = 24 T o solve for m, we [URL='https://www.mathcelebrity.com/1unk.php?num=2m%2Bm%3D24&pl=Solve']type this equation into our math engine[/URL]: m = [B]8 [/B] We want to solve for k using m= 8. Substitute this into equation 1 k = 2(8) k = [B]16 [/B] Check our work for equation 1 16 = 2 * 8 16 = 16 Check our work for equation 2 16 + 8 ? 24 24 = 24 [MEDIA=youtube]TJMTRYP-Ct8[/MEDIA]

Kate spent 1 more than Lauren, and together they spent 5
Kate spent 1 more than Lauren, and together they spent 5. Let k be the amount Kate spent, and l be the amount Lauren spent. We're given: [LIST=1] [*]k = l + 1 [*]k + l = 5 [/LIST] Substitute (1) into (2): (l + 1) + l = 5 Group like terms 2l + 1 = 5 [URL='https://www.mathcelebrity.com/1unk.php?num=2l%2B1%3D5&pl=Solve']Typing this equation into our search engine[/URL], we get: [B]l = 2[/B] Plug this into Equation (1), we get: k = 2 + 1 [B]k = 3 [/B] Kate Spent 3, and Lauren spent 2

Kate spent at most $2.50 on apples and oranges. She bought 5 apples at $0.36 each. What is the most
Kate spent at most $2.50 on apples and oranges. She bought 5 apples at $0.36 each. What is the most She spent on oranges [U]Assumptions and givens:[/U] [LIST] [*]Let a be the total cost of apples [*]Let o be the total cost of oranges [/LIST] The phrase [I]at most[/I] means less than or equal to, so we have: a + o <= 2.50 [U]Find the cost of apples (a)[/U] a = price per apple * quantity of apples a = 0.36 * 5 a = 1.8 Our new inequality with a = 1.8 is: 1.8 + o <= 2.50 [URL='https://www.mathcelebrity.com/1unk.php?num=1.8%2Bo%3C%3D2.50&pl=Solve']Typing this inequality into our search engine[/URL], we get: [B]o <= 0.7[/B]

Katie is twice as old as her sister Mara. The sum of their age is 24.
Let k = Katie's age and m = Mara's age. We have 2 equations: (1) k = 2m (2) k + m = 24 Substitute (1) into (2) (2m) + m = 24 Combine like terms: 3m = 24 Divide each side of the equation by 3 to isolate m m = 8 If m = 8, substituting into (1) or (2), we get k = 16. [MEDIA=youtube]Cu7gSgNkQPg[/MEDIA]

Kayla has $1500 in her bank account. She spends $150 each week. Write an equation in slope-intercept
Kayla has $1500 in her bank account. She spends $150 each week. Write an equation in slope-intercept form that represents the relationship between the amount in Kayla's bank account, A, and the number of weeks she has been spending, w [LIST] [*]Slope intercept form is written as A = mw + b [*]m = -150, since spending is a decrease [*]b = 1500, since this is what Kayla starts with when w = 0 [/LIST] [B]A = -150w + 1500[/B]

Kaylee had $197 in her savings now her savings is $429 . How much was her paycheck
Kaylee had $197 in her savings now her savings is $429 . How much was her paycheck Her paycheck equals the increase in savings from $197 to $429. We want the difference: Paycheck = Savings Now - Savings Before Paycheck Paycheck = $429 - $197 Paycheck = [B]$232[/B]

keisha is babysitting at 8$ per hour to earn money for a car. So far she has saved $1300. The car th
keisha is babysitting at 8$ per hour to earn money for a car. So far she has saved $1300. The car that keisha wants to buy costs at least $5440. How many hours does Keisha need to babysit to earn enough to buy the car Set up the Earning function E(h) where h is the number of hours Keisha needs to babysit: E(h) = 8h + 1300 The question asks for h when E(h) is at least 5440. The phrase [I]at least[/I] means an inequality, which is greater than or equal to. So we have: 8h + 1300 >= 5440 To solve this inequality, we [URL='https://www.mathcelebrity.com/1unk.php?num=8h%2B1300%3E%3D5440&pl=Solve']type it in our search engine[/URL] and we get: h >= [B]517.5[/B]

Keith has $500 in a savings account at the beginning of the summer. He wants to have at least $200 a
Keith has $500 in a savings account at the beginning of the summer. He wants to have at least $200 at the end of the summer. He withdraws $25 per week for food, clothing, and movie tickets. How many weeks can Keith withdraw money from his account. Keith's balance is written as B(w) where w is the number of weeks passed since the beginning of summer. We have: B(w) = 500 - 25w The problem asks for B(w) = 200, so we set 500 - 25w = 200. [URL='https://www.mathcelebrity.com/1unk.php?num=500-25w%3D200&pl=Solve']Typing 500 - 25w = 200 into the search engine[/URL], we get [B]w = 12[/B].

Keith has $500 in a savings account at the beginning of the summer. He wants to have at least $200 a
Keith has $500 in a savings account at the beginning of the summer. He wants to have at least $200 at the end of the summer. He withdraws $25 per week for food, clothing, and movie tickets. How many weeks can Keith withdraw money from his account Our account balance is: 500 - 25w where w is the number of weeks. We want to know the following for w: 500 - 25w = 200 [URL='https://www.mathcelebrity.com/1unk.php?num=500-25w%3D200&pl=Solve']Typing this equation into our search engine[/URL], we get: [B]w = 12[/B]

Keith is cutting two circular table tops out of a piece of plywood. the plywood is 4 feet by 8 feet
Keith is cutting two circular table tops out of a piece of plywood. the plywood is 4 feet by 8 feet and each table top has a diameter of 4 feet. If the price of a piece of plywood is $40, what is the value of the plywood that is wasted after the table tops are cut? Area of the plywood = 4 * 8 = 32 square feet [U]Calculate area of 1 round top[/U] Diameter = 2 Radius = Diameter/2 = 4/2 = 2 Area of each round top = pir^2 Area of each round top = 3.14 * 2 * 2 Area of each round top = 12.56 square feet [U]Calculate area of 2 round tops[/U] Area of 2 round tops = 12.56 + 12.56 Area of 2 round tops = 25.12 sq feet [U]Calculate wasted area:[/U] Wasted area = area of the plywood - area of 2 round tops Wasted area = 32 - 25.12 Wasted area = 6.88 sq feet [U]Calculate cost per square foot of plywood:[/U] Cost per sq foot of plywood = Price per plywood / area of the plywood Cost per sq foot of plywood = 40/32 Cost per sq foot of plywood = $1.25 [U]Calculate the value of the plywood:[/U] Value of the plywood = Wasted Area sq foot * Cost per sq foot of plywood Value of the plywood = 6.88 * 1.25 Value of the plywood = [B]$8.60[/B]

Keith is going to Renaissance Festival with $120 to pay for his admission, food and the cost of game
Keith is going to Renaissance Festival with $120 to pay for his admission, food and the cost of games. He spends a total of $85 on admission and food. Games cost $5 each. Which inequality models the maximum number of games Keith can play. Let the number of games be g. Keith can spend less than or equal to 120. So we have [B]5g + 85 <= 120 [/B] If we want to solve the inequality for g, we [URL='https://www.mathcelebrity.com/1unk.php?num=5g%2B85%3C%3D120&pl=Solve']type it in our search engine[/URL] and we have: g <= 7

kelko buys candy that costs $7 per pound. She will spend less than $84 on candy. What are the possib
kelko buys candy that costs $7 per pound. She will spend less than $84 on candy. What are the possible numbers of pounds she will buy. Let p be the number of pounds Kelko buys. p < 84/7 [B]p < 12[/B]

Kellie has only $5.25 to buy breakfast. She wants to buy as many carrot muffins as she can. Each muf
Kellie has only $5.25 to buy breakfast. She wants to buy as many carrot muffins as she can. Each muffin costs $0.75. What’s an equation? Let m be the number of muffins. Cost equals price * quantity, so we have: [B]0.75m = 5.25 [/B] To solve the equation for m, we [URL='https://www.mathcelebrity.com/1unk.php?num=0.75m%3D5.25&pl=Solve']type the equation into our search engine[/URL] and we get: m = [B]7[/B]

Kelly has 18 eggs stored in boxes. If the are 3 boxes, how many eggs must go in each box?
Kelly has 18 eggs stored in boxes. If the are 3 boxes, how many eggs must go in each box? 18 eggs per box / 3 boxes = [B]6 eggs per box[/B]

Kelly took clothes to the cleaners 3 times last month. First, she brought 4 shirts and 1 pair of sla
Kelly took clothes to the cleaners 3 times last month. First, she brought 4 shirts and 1 pair of slacks and paid11.45. Then she brought 5 shirts, 3 pairs of slacks, and 1 sports coat and paid 27.41. Finally, she brought 5 shirts and 1 sports coat and paid 16.94. How much was she charged for each shirt, each pair of slacks, and each sports coat? Let s be the cost of shirts, p be the cost of slacks, and c be the cost of sports coats. We're given: [LIST=1] [*]4s + p = 11.45 [*]5s + 3p + c = 27.41 [*]5s + c = 16.94 [/LIST] Rearrange (1) by subtracting 4s from each side: p = 11.45 - 4s Rearrange (3)by subtracting 5s from each side: c = 16.94 - 5s Take those rearranged equations, and plug them into (2): 5s + 3(11.45 - 4s) + (16.94 - 5s) = 27.41 Multiply through: 5s + 34.35 - 12s + 16.94 - 5s = 27.41 [URL='https://www.mathcelebrity.com/1unk.php?num=5s%2B34.35-12s%2B16.94-5s%3D27.41&pl=Solve']Group like terms using our equation calculator [/URL]and we get: [B]s = 1.99 [/B] <-- Shirt Cost Plug s = 1.99 into modified equation (1): p = 11.45 - 4(1.99) p = 11.45 - 7.96 [B]p = 3.49[/B] <-- Slacks Cost Plug s = 1.99 into modified equation (3): c = 16.94 - 5(1.99) c = 16.94 - 9.95 [B]c = 6.99[/B] <-- Sports Coat cost

Kelsey wants to buy a new video game. He has a $50 gift card and wants to spend less than $20 of
Kelsey wants to buy a new video game. He has a $50 gift card and wants to spend less than $20 of his own money. Which of the following amounts would Kelsey be willing to spend on a video game? Let x be the amount Kelsey will spend on a video game above 50. He will spend up to, but less than $20 above his $50 gift card. x < 50 + 20 [B]x < 70[/B]

Kendra has $20 in a savings account. The interest rate is 10%, compounded annually. To the nearest
Kendra has $20 in a savings account. The interest rate is 10%, compounded annually. To the nearest cent, how much will she have in 2 years? Using our [URL='https://www.mathcelebrity.com/compoundint.php?bal=20&nval=2&int=10&pl=Annually']balance with interest calculator[/URL], we get [B]$24.20[/B].

Kendra has $5.70 in quarters and nickels. If she has 12 more quarters than nickels, how many of each
Kendra has $5.70 in quarters and nickels. If she has 12 more quarters than nickels, how many of each coin does she have? Let n be the number of nickels and q be the number of quarters. We have: [LIST=1] [*]q = n + 12 [*]0.05n + 0.25q = 5.70 [/LIST] Substitute (1) into (2) 0.05n + 0.25(n + 12) = 5.70 0.05n + 0.25n + 3 = 5.70 Combine like terms: 0.3n + 3 = 5.70 Using our [URL='http://www.mathcelebrity.com/1unk.php?num=0.3n%2B3%3D5.70&pl=Solve']equation calculator[/URL], we get [B]n = 9[/B]. Substituting that back into (1), we get: q = 9 + 12 [B]q = 21[/B]

Kendra is half as old as Morgan and 3 years younger than Lizzie. The total of their ages is 39. How
Kendra is half as old as Morgan and 3 years younger than Lizzie. The total of their ages is 39. How old are they? Let k be Kendra's age, m be Morgan's age, and l be Lizzie's age. We're given: [LIST=1] [*]k = 0.5m [*]k = l - 3 [*]k + l + m = 39 [/LIST] Rearranging (1) by multiplying each side by 2, we have: m = 2k Rearranging (2) by adding 3 to each side, we have: l = k + 3 Substituting these new values into (3), we have: k + (k + 3) + (2k) = 39 Group like terms: (k + k + 2k) + 3 = 39 4k + 3 = 39 [URL='https://www.mathcelebrity.com/1unk.php?num=4k%2B3%3D39&pl=Solve']Type this equation into the search engine[/URL], and we get: [B]k = 9 [/B] Substitute this back into (1), we have: m = 2(9) [B]m = 18 [/B] Substitute this back into (2), we have: l = (9) + 3 [B][B]l = 12[/B][/B]

Kendrick set his watch 9 seconds behind, and it falls behind another 1 second everyday.How far behin
Kendrick set his watch 9 seconds behind, and it falls behind another 1 second everyday.How far behind is Kendrick's watch if he last set it 23 days ago? Seconds Behind = 9 seconds behind + 1 second everyday * 23 days Seconds Behind = 9 + 23 Seconds Behind = 32

Kent Realty Company had an annual loss of $63,408. What was the average loss per month?
Kent Realty Company had an annual loss of $63,408. What was the average loss per month? Convert years to months 1 year = 12 months 63,408/12 = [B]5,284 per month[/B]

Kerry asked a bank teller to cash 390 check using 20 bills and 50 bills. If the teller gave her a to
Kerry asked a bank teller to cash 390 check using 20 bills and 50 bills. If the teller gave her a total of 15 bills, how many of each type of bill did she receive? Let t = number of 20 bills and f = number of 50 bills. We have two equations. (1) 20t + 50f = 390 (2) t + f = 15 [U]Rearrange (2) into (3) for t, by subtracting f from each side:[/U] (3) t = 15 - f [U]Now substitute (3) into (1)[/U] 20(15 - f) + 50f = 390 300 - 20f + 50f = 390 [U]Combine f terms[/U] 300 + 30f = 390 [U]Subtract 300 from each side[/U] 30f = 90 [U]Divide each side by 30[/U] [B]f = 3[/B] [U]Substitute f = 3 into (3)[/U] t = 15 - 3 [B]t = 12[/B]

Kevin and randy have a jar containing 41 coins, all of which are either quarters or nickels. The tot
Kevin and randy have a jar containing 41 coins, all of which are either quarters or nickels. The total value of the jar is $7.85. How many of each type? Let d be dimes and q be quarters. Set up two equations from our givens: [LIST=1] [*]d + q = 41 [*]0.1d + 0.25q = 7.85 [/LIST] [U]Rearrange (1) by subtracting q from each side:[/U] (3) d = 41 - q [U]Now, substitute (3) into (2)[/U] 0.1(41 - q) + 0.25q = 7.85 4.1 - 0.1q + 0.25q = 7.85 [U]Combine q terms[/U] 0.15q + 4.1 = 7.85 [U]Using our [URL='http://www.mathcelebrity.com/1unk.php?num=0.15q%2B4.1%3D7.85&pl=Solve']equation calculator[/URL], we get:[/U] [B]q = 25[/B] [U]Substitute q = 25 into (3)[/U] d = 41 - 25 [B]d = 16[/B]

Kevin and Randy Muise have a jar containing 52 coins, all of which are either quarters or nickels.
Kevin and Randy Muise have a jar containing 52 coins, all of which are either quarters or nickels. The total value of the coins in the jar is $6.20. How many of each type of coin do they have? Let q be the number of quarters, and n be the number of nickels. We have: [LIST=1] [*]n + q = 52 [*]0.05n + 0.25q = 6.20 [/LIST] We can solve this system of equations three ways: [LIST] [*][URL='https://www.mathcelebrity.com/simultaneous-equations.php?term1=n+%2B+q+%3D+52&term2=0.05n+%2B+0.25q+%3D+6.20&pl=Substitution']Substitution Method[/URL] [*][URL='https://www.mathcelebrity.com/simultaneous-equations.php?term1=n+%2B+q+%3D+52&term2=0.05n+%2B+0.25q+%3D+6.20&pl=Elimination']Elimination Method[/URL] [*][URL='https://www.mathcelebrity.com/simultaneous-equations.php?term1=n+%2B+q+%3D+52&term2=0.05n+%2B+0.25q+%3D+6.20&pl=Cramers+Method']Cramers Rule[/URL] [/LIST] No matter what method we choose, we get the same answer: [LIST] [*][B]n = 34[/B] [*][B]q = 18[/B] [/LIST]

Kevin is 4 times old as Daniel and is also 6 years older than Daniel
Kevin is 4 times old as Daniel and is also 6 years older than Daniel. Let k be Kevin's age and d be Daniel's age. We have 2 equations: [LIST=1] [*]k = 4d [*]k = d + 6 [/LIST] Plug (1) into (2): 4d = d + 6 Subtract d from each side: 4d - d = d - d + 6 Cancel the d terms on the right side and simplify: 3d = 6 Divide each side by 3: 3d/3 = 6/3 Cancel the 3 on the left side: d = 2 Plug this back into equation (1): k = 4(2) k = 8 So Daniel is 2 years old and Kevin is 8 years old

Kevin ran 4 miles more than Steve ran. The sum of their distances is 26 miles. How far did Steve run
Kevin ran 4 miles more than Steve ran. The sum of their distances is 26 miles. How far did Steve run? The domain of the solution is: Let k be Kevin's miles ran Let s be Steve's miles ran We have 2 given equtaions: [LIST=1] [*]k = s + 4 [*]k + s = 26 [/LIST] Substitute (1) into (2) (s + 4) + s = 26 2s + 4 = 26 Plug this into our [URL='http://www.mathcelebrity.com/1unk.php?num=2s%2B4%3D26&pl=Solve']equation calculator[/URL] and we get s = 11

kevin saw the hit movie at the theater on sunday. On Monday, Kevin told 4 friends about the movie. T
Kevin saw the hit movie at the theater on sunday. On Monday, Kevin told 4 friends about the movie. The day after that, each of those friends told 4 more friends about the movie. If this pattern continues, how many people would have been told about the movie by Friday. Monday: 4 Tuesday: 4 x 4 = 16 Wednesday: 16 x 4 = 64 Thursday: 64 x 4 = 256 Friday: 256 x 4 = [B]1,024[/B]

Kierra had $35 to spend at the movies. If it was $11 to get in and snacks were 2$ each, how many sna
Kierra had $35 to spend at the movies. If it was $11 to get in and snacks were 2$ each, how many snacks could she buy? Subtract off cover charge: 35 - 11 = 24 Let s equal the number of snacks Kierra can buy. With each snack costing $2, we have the following equation: 2s = 24 Using our [URL='http://www.mathcelebrity.com/1unk.php?num=2s%3D24&pl=Solve']equation calculator[/URL], we have: [B]s = 12[/B]

Kiko is now 6 times as old as his sister. In 6 years, he will be 3 times as old as his sister. What
Kiko is now 6 times as old as his sister. In 6 years, he will be 3 times as old as his sister. What is their present age? Let k be Kiko's present age Let s be Kiko's sisters age. We're given two equations: [LIST=1] [*]k = 6s [*]k + 6 = 3(s + 6) [/LIST] To solve this system of equations, we substitute equation (1) into equation (2) for k: 6s + 6 = 3(s + 6) [URL='https://www.mathcelebrity.com/1unk.php?num=6s%2B6%3D3%28s%2B6%29&pl=Solve']Typing this equation into our math engine[/URL] to solve for s, we get: s = [B]4[/B] To solve for k, we substitute s = 4 into equation (1) above: k = 6 * 4 k = [B]24[/B]

kim and jason just had business cards made. kim’s printing company charged a one time setup fee of $
kim and jason just had business cards made. kim’s printing company charged a one time setup fee of $8 and then $20 per box of cards. jason,meanwhile ordered his online. they cost $8 per box. there was no setup fee, but he had to pay $20 to have his order shipped to his house. by coincidence, kim and jason ended up spending the same amount on their business cards. how many boxes did each buy? how much did each spend? Set up Kim's cost function C(b) where b is the number of boxes: C(b) = Cost per box * number of cards + Setup Fee + Shipping Fee C(b) = 20c + 8 + 0 Set up Jason's cost function C(b) where b is the number of boxes: C(b) = Cost per box * number of cards + Setup Fee + Shipping Fee C(b) = 8c + 0 + 20 Since Kim and Jason spent the same amount, set both cost equations equal to each other: 20c + 8 = 8c + 20 [URL='https://www.mathcelebrity.com/1unk.php?num=20c%2B8%3D8c%2B20&pl=Solve']Type this equation into our search engine[/URL] to solve for c, and we get: c = 1 How much did they spend? We pick either Kim's or Jason's cost equation since they spent the same, and plug in c = 1: Kim: C(1) = 20(1) + 8 C(1) = 20 + 8 C(1) = [B]28 [/B] Jason: C(1) = 8(1) + 20 C(1) = 8 + 20 C(1) = [B]28[/B]

Kim earns $30 for babysitting on Friday nights. She makes an average of $1.25 in tips per hour. Writ
Kim earns $30 for babysitting on Friday nights. She makes an average of $1.25 in tips per hour. Write the function of Kim's earnings, and solve for how much she would make after 3 hours. Set up the earnings equation E(h) where h is the number of hours. We have the function: E(h) = 1.25h + 30 The problem asks for E(3): E(3) = 1.25(3) + 30 E(3) = 4.75 + 30 E(3) = [B]$34.75[/B]

Kim read 95% of a book. What percent of the book was not yet read?
Kim read 95% of a book. What percent of the book was not yet read? The full book is 100%. So percent unread is: Percent Unread = 100% - Percent Read Percent Unread = 100% - 95% Percent Unread = [B]5%[/B]

kim wants to buy candy for 4 dollars a pound. if she wants to spend less than 20 dollars, how many b
kim wants to buy candy for 4 dollars a pound. if she wants to spend less than 20 dollars, how many bags can she buy Since cost = price * quantity, we have the following inequality with b as the number of bags: 4b < 20 To solve this inequality for b, we [URL='https://www.mathcelebrity.com/interval-notation-calculator.php?num=4b%3C20&pl=Show+Interval+Notation']type it in our search engine[/URL] and we get: [B]b < 5[/B]

Kim, Jenny, and Wendy are basketball players. Each plays a different position (guard, forward, and c
Kim, Jenny, and Wendy are basketball players. Each plays a different position (guard, forward, and center) and wears a different number (30, 32, and 35).Kim and number 30 are too small to play center. Number 35 is the center. Neither Kim nor Wendy is the forward. Who plays guard, and what uniform number does she wear? [LIST] [*]Kim does not play center [*]Kim does not play forward [*]Which means [B]Kim is the guard[/B] [*]Since Kim is not number 30, and she cannot be number 35 since Number 35 is the center, the only number left is [B]Number 32[/B] [/LIST] [B]Kim is the guard with number 32[/B]

Kimberly is taking three online classes during the summer. She spends 10 hours each week studying fo
Kimberly is taking three online classes during the summer. She spends 10 hours each week studying for her marketing class, 12 hours studying for her statistics class, and 8 hours studying for her business law class. What percent of her study time does she spend for her statistics class? The percentage equals hours spent on statistics divided by total hours spent studying for everything. [U]Calculate total study hours:[/U] Total Study Hours = Marketing Class Study Hours + Statistics Class Study Hours + Business Law Study Hours Total Study Hours = 10 + 8 + 12 Total Study Hours = [B]30[/B] [U]Calculate Statistics Study Hours Percentage:[/U] Statistics Study Hours Percentage = Statistics Class Study Hours / Total Study Hours Statistics Class Study Hours = 8/30 Using our [URL='https://www.mathcelebrity.com/perc.php?num=8&den=30&pcheck=1&num1=16&pct1=80&pct2=70&den1=80&idpct1=10&hltype=1&idpct2=90&pct=82&decimal=+65.236&astart=12&aend=20&wp1=20&wp2=30&pl=Calculate']fraction to decimal calculator[/URL], we get Statistics Class Study Hours = [B]26.67%[/B]

Kimberly takes 4 pages of notes during each hour of class. Write an equation that shows the relation
Kimberly takes 4 pages of notes during each hour of class. Write an equation that shows the relationship between the time in class x and the number of pages y. With x hours and y pages, our equation is: [B]y = 4x [/B]

Kimberly wants to become a member of the desert squad at a big catering company very badly, but she
Kimberly wants to become a member of the desert squad at a big catering company very badly, but she must pass three difficult tests to do so. On the first Terrifying Tiramisu test she scored a 68. On the second the challenging Chocalate-Sprinkled Creme Brulee she scored a 72. If kimberly needs an average of 60 on all three tests to become a member on the squad what is the lowest score she can make on her third and final test This is a missing average problem. Given 2 scores of 68, 72, what should be score number 3 in order to attain an average score of 60? [SIZE=5][B]Setup Average Equation:[/B][/SIZE] Average = (Sum of our 2 numbers + unknown score of [I]x)/[/I]Total Numbers 60 = (68 + 72 + x)/3 [SIZE=5][B]Cross Multiply[/B][/SIZE] 68 + 72 + x = 60 x 3 x + 140 = 180 [SIZE=5][B]Subtract 140 from both sides of the equation to isolate x:[/B][/SIZE] x + 140 - 140 = 180 - 140 x = [B]40[/B]

Kimi has 62 red peppers and g green peppers. Choose the expression that shows how many peppers Kimi
Kimi has 62 red peppers and g green peppers. Choose the expression that shows how many peppers Kimi has. We add to get the total peppers: [B]62 + g[/B]

Kinematic Equations
Free Kinematic Equations Calculator - Given the 5 inputs of the 4 kinematic equations, this will solve any of the equations it can based on your inputs for the kinematics.

Kinetic Energy
Free Kinetic Energy Calculator - Solves for any of the 3 items in the kinetic energy equation: Energy (e), Mass (m), and Velocity (v)

kira will spend less than 27 on gifts. so far, she has spent 12$. what are the possible additional a
kira will spend less than 27 on gifts. so far, she has spent 12$. what are the possible additional amounts she will spend? The key word in this problem is [I]less than[/I]. So we know this is an inequality. Let s be Kira's possible spend. We have: s + 12 < 27 To solve for s in this inequality, we subtract 12 from each side: s + 12 - 12 < 27 - 12 Cancel the 12's on the left side, and we get: [B]s < 15 [/B] [I]The summary here is Kira can spend anything up to [U]but not including[/U] 15[/I]

Kristen and Julia went skating. Julia skated 30 minutes longer than Kristen. If Julia skated for 55
Kristen and Julia went skating. Julia skated 30 minutes longer than Kristen. If Julia skated for 55 minutes, write and solve an equation to find how long Kristen skated Let j be the number of minutes Julia skates and k be the number of minutes Kristen skated. We have 2 equations: [B](1) j = k + 30 (2) j = 55[/B] [U]Plug (2) into (1)[/U] j = 55 + 30 [B]j = 85 minutes, or 1 hour and 25 minutes[/B]

Krutika was thinking of a number. Krutika doubles it and adds 8.7 to get an answer of 64.9. Form an
Krutika was thinking of a number. Krutika doubles it and adds 8.7 to get an answer of 64.9. Form an equation with x from the information. [LIST=1] [*]The number we start with is x. [*]Double it means we multiply by 2: 2x [*]Add 8.7: 2x + 8.7 [*][I]Get an answer[/I] means we have an equation, so we set (3) above equal to 64.9 [*][B]2x + 8.7 = 64.9[/B] [/LIST] If you want to solve for x, use our [URL='http://www.mathcelebrity.com/1unk.php?num=2x%2B8.7%3D64.9&pl=Solve']equation calculator[/URL].

Kunio puts $2,200.00 into savings bonds that pay a simple interest rate of 2.4%. How much money will
Kunio puts $2,200.00 into savings bonds that pay a simple interest rate of 2.4%. How much money will the bonds be worth at the end of 4 years? Using our [URL='http://www.mathcelebrity.com/simpint.php?av=&p=2200&int=2.4&t=4&pl=Simple+Interest']simple interest balance calculator[/URL], we his account will be worth [B]$2,411.20[/B] after 4 years

kyle baked 29 muffins. He placed p muffins each in 4 boxes and had 5 muffins left over. How many muf
kyle baked 29 muffins. He placed p muffins each in 4 boxes and had 5 muffins left over. How many muffins were in each box We set up the following equation: 4p + 5 = 29 To solve this equation for p, we [URL='https://www.mathcelebrity.com/1unk.php?num=4p%2B5%3D29&pl=Solve']type it in our math engine[/URL] and we get: p = [B]6[/B]

Kyle can walk ½ mile in ¼ of an hour. What is Kyle’s speed in miles per hour?
Kyle can walk ½ mile in ¼ of an hour. What is Kyle’s speed in miles per hour? We write this in terms of miles per hour as: 1/2 / 1/4 We want 1 for the denominator to represent an hour, so we multiply top and bottom of the fraction by 4: 4/2 / 4/4 2 / 1 [B]2 miles per hour[/B]

L is the set of letters in the word Mississippi
L is the set of letters in the word Mississippi We want only unique letters, so we have: [B]L = {I, M, P, S}[/B]

L'Hôpital's Rule
Free L'Hôpital's Rule Calculator - This lesson walks you through L'Hôpitals's Rule including the definition, pronunciation, notation, and examples

Lagrange Four Square Theorem (Bachet Conjecture)
Free Lagrange Four Square Theorem (Bachet Conjecture) Calculator - Builds the Lagrange Theorem Notation (Bachet Conjecture) for any natural number using the Sum of four squares.

Lamar had N record albums that he tried to sell at a garage sale for $5 each. If the number of recor
Lamar had N record albums that he tried to sell at a garage sale for $5 each. If the number of record albums he didn't sell is called Q, how much money did Lamar get from record album sales? Sales = Price * (Albums had - Albums sold) [B]Sales = 5(N - Q)[/B]

larger of 2 numbers is 12 more than the smaller number. if the sum of the 2 numbers is 74 find the 2
larger of 2 numbers is 12 more than the smaller number. if the sum of the 2 numbers is 74 find the 2 numbers Declare Variables for each number: [LIST] [*]Let l be the larger number [*]Let s be the smaller number [/LIST] We're given two equations: [LIST=1] [*]l = s + 12 [*]l + s = 74 [/LIST] Equation (1) already has l solved for. Substitute equation (1) into equation (2) for l: s + 12 + s = 74 Solve for [I]s[/I] in the equation s + 12 + s = 74 [SIZE=5][B]Step 1: Group the s terms on the left hand side:[/B][/SIZE] (1 + 1)s = 2s [SIZE=5][B]Step 2: Form modified equation[/B][/SIZE] 2s + 12 = + 74 [SIZE=5][B]Step 3: Group constants:[/B][/SIZE] We need to group our constants 12 and 74. To do that, we subtract 12 from both sides 2s + 12 - 12 = 74 - 12 [SIZE=5][B]Step 4: Cancel 12 on the left side:[/B][/SIZE] 2s = 62 [SIZE=5][B]Step 5: Divide each side of the equation by 2[/B][/SIZE] 2s/2 = 62/2 s = [B]31[/B] To solve for l, we substitute in s = 31 into equation (1): l = 31 + 12 l = [B]43[/B]

larger of 2 numbers is 4 more than the smaller. the sum of the 2 is 40. what is the larger number?
larger of 2 numbers is 4 more than the smaller. the sum of the 2 is 40. what is the larger number? Declare variables for the 2 numbers: [LIST] [*]Let l be the larger number [*]Let s be the smaller number [/LIST] We're given two equations: [LIST=1] [*]l = s + 4 [*]l + s = 40 [/LIST] To get this problem in terms of the larger number l, we rearrange equation (1) in terms of l. Subtract 4 from each side in equation (1) l - 4 = s + 4 - 4 Cancel the 4's and we get: s = l - 4 Our given equations are now: [LIST=1] [*]s = l - 4 [*]l + s = 40 [/LIST] Substitute equation (1) into equation (2) for s: l + l - 4 = 40 Grouping like terms for l, we get: 2l - 4 = 40 Add 4 to each side: 2l - 4 + 4 = 40 + 4 Cancelling the 4's on the left side, we get 2l = 44 Divide each side of the equation by 2 to isolate l: 2l/2 = 44/2 Cancel the 2's on the left side and we get: l = [B]22[/B]

Larry and his friend split the dinner bill evenly. They each paid $21.34. What was the cost of dinne
Larry and his friend split the dinner bill evenly. They each paid $21.34. What was the cost of dinner? Total Bill = Larry's portion + Friend's portion Total Bill = 21.34 + 21.34 Total Bill = [B]$42.68[/B]

Larry is buying new clothes for his return to school. He is buying shoes for $57 and shirts cost $15
Larry is buying new clothes for his return to school. He is buying shoes for $57 and shirts cost $15 each. He has $105 to spend. Which of the following can be solved to find the number of shirts he can afford? Let s be the number of shirts. Since shoes are a one-time fixed cost, we have: 15s + 57 = 105 We want to solve this equation for s. We [URL='https://www.mathcelebrity.com/1unk.php?num=15s%2B57%3D105&pl=Solve']type it in our math engine[/URL] and we get: s = [B]3.2 or 3 whole shirts[/B]

Larry is rolling two dice. His dad told him that he can skip doing the dishes that night unless he r
Larry is rolling two dice. His dad told him that he can skip doing the dishes that night unless he rolls double sixes. What is the probability that Larry will be able to skip doing the dishes? P(6, 6) = 1/6 * 1/6 = 1/36 P(Not 6,6) = 1 - 1/36 = [B]35/36[/B]

Larry Mitchell invested part of his $31,000 advance at 6% annual simple interest and the rest at 7%
Larry Mitchell invested part of his $31,000 advance at 6% annual simple interest and the rest at 7% annual simple interest. If the total yearly interest from both accounts was $2,090, find the amount invested at each rate. Let x be the amount invested at 6%. Then 31000 - x is invested at 7%. We have the following equation: 0.06x + (31000 - x)0.07 = 2090 Simplify: 0.06x + 2170 - 0.07x = 2090 Combine like Terms -0.01x + 2170 = 2090 Subtract 2170 from each side -0.01x = -80 Divide each side by -0.01 x = [B]8000 [/B]at 6% Which means at 7%, we have: 31000 - 8000 = [B]23,000[/B]

Last December at Dubai International Airport 1,309,738 passengers travelled through terminal 1 and 2
Last December at Dubai International Airport 1,309,738 passengers travelled through terminal 1 and 2,516,989 passengers through terminal 2. How many passengers travelled through terminal 1 and terminal 2 altogether? The word [I]altogether[/I] means we add Terminal 1 to Terminal 2: 1,309,738 + 2,516,989 = [B]3,826,727[/B]

Last month, a parking lot had 23 spaces in each of its rows. Recently, the lost was expanded, and 4
Last month, a parking lot had 23 spaces in each of its rows. Recently, the lost was expanded, and 4 spaces were added to each row. If the lot has 8 rows, how many spaces are there now? 23 spaces + 4 additional spaces = 27 spaces 27 spaces * 8 rows = [B]216 spaces[/B]

Last month, my saving account was balance was $1,000. since then, i spent x dollars from my saving
Last month, my saving account was balance was $1,000. since then, i spent x dollars from my saving Spending means reducing our balance, so we have a new balance of: [B]1000 - x[/B]

Last week at the business where you work, you sold 120 items. The business paid $1 per item and sol
Last week at the business where you work, you sold 120 items. The business paid $1 per item and sold them for $3 each. What profit did the business make from selling the 120 items? Let n be the number of items. We have the following equations: Cost Function C(n) = n For n = 120, we have C(120) = 120 Revenue Function R(n) = 3n For n = 120, we have R(120) = 3(120) = 360 Profit = Revenue - Cost Profit = 360 - 120 Profit = [B]240[/B]

Last week, a man worked 47 hours at Starbucks. Find his gross earnings for the week if he is paid
Last week, a man worked 47 hours at Starbucks. Find his gross earnings for the week if he is paid $7.80 per hour and earns time-and-a-half for all hours over 40. [U]Step 1: Calculate regular time pay up to 40 hours:[/U] Regular Pay = Hourly Wage * Hours up to 40 Regular Pay = $7.80 * 40 Regular Pay = $312 [U]Step 2: Calculate overtime hours above 40 hours:[/U] Overtime Hours = Hours Worked - 40 hours Overtime Hours = 47 - 40 Overtime Hours = 7 [U]Step 3: Calculate overtime pay above 40 hours:[/U] Overtime Pay = 1.5 * Hourly Rate * Overtime Hours Overtime Pay = 1.5 * $7.80 * 7 Overtime Pay = $81.90 [U]Step 4: Calculate Gross Earnings[/U] Gross Earnings = Regular Pay + Overtime Pay Gross Earnings = $312 + $81.90 = [B]$393.90 [URL='https://www.facebook.com/MathCelebrity/videos/10156751976078291/']FB Live Session[/URL][/B]

last week, bill drove 252 miles. This week, he drove m miles. Using m , write an expression for the
last week, bill drove 252 miles. This week, he drove m miles. Using m, write an expression for the total number of miles he drove in the two weeks We add the distance driven: [B]252 + m[/B]

Last year, Eric had $20,000 to invest. He invested some of it in an account that paid 10% simple int
Last year, Eric had $20,000 to invest. He invested some of it in an account that paid 10% simple interest per year, and he invested the rest in an account that paid 7% simple interest per year. After one year, he received a total of $1880 in interest. How much did he invest in each account? Using our [URL='http://www.mathcelebrity.com/split-fund-interest-calculator.php?p=20000&i1=10&i2=7&itot=1880&pl=Calculate']split fund interest calculator[/URL], we get: [LIST] [*][B]Fund 1 = 16,000[/B] [*][B]Fund 2 = 4,000[/B] [/LIST]

Last year, Greg biked 524 miles. This year, he biked m miles. Using m , write an expression for the
Last year, Greg biked 524 miles. This year, he biked m miles. Using m , write an expression for the total number of miles he biked. We add both years to get our algebraic expression of miles biked: [B]m + 524[/B]

Last year, Manuel had $10,000 to invest. He invested some of it in an account that paid 7% simple in
Last year, Manuel had $10,000 to invest. He invested some of it in an account that paid 7% simple interest per year, and he invested the rest in an account that paid 10% simple interest per year. After one year, he received a total of $730 in interest. How much did he invest in each account? The answer is $9,000 and $1,000 found on [URL='http://www.mathcelebrity.com/split-fund-interest-calculator.php?p=10000&i1=7&i2=10&itot=730&pl=Calculate']this calculator[/URL].

Last year, Maria biked M miles. This year, she biked 390 miles. Using m , write an expression for th
Last year, Maria biked M miles. This year, she biked 390 miles. Using m , write an expression for the total number of miles she biked. [U]Calculate Total miles biked[/U] Total miles biked = Last Year + This year Total miles biked = [B]m + 390[/B]

Last year, Miguel had $10,000 to invest. He invested some of it in an account that paid 5% simpl
Last year, Miguel had $10,000 to invest. He invested some of it in an account that paid 5% simple interest per year, and he invested the rest in an account that paid 10% simple interest per year. After one year, he received a total of $800 in interest. How much did he invest in each account? Using our [URL='http://www.mathcelebrity.com/split-fund-interest-calculator.php?p=10000&i1=5&i2=10&itot=800&pl=Calculate']split fund interest calculator[/URL], we get: [LIST] [*][B]4,000 in Fund 1 at 5%[/B] [*][B]6,000 in Fund 2 at 10%[/B] [/LIST]

Last year, the 6th grade had 200 students. This year the number decreased 35% How many students are
Last year, the 6th grade had 200 students. This year the number decreased 35% How many students are in this year's 6th grade class? [URL='https://www.mathcelebrity.com/percentoff.php?p1=&m=35&p2=200&pl=Calculate']200 decreased by 35%[/URL] is [B]130[/B]

Lattice Multiplication
Free Lattice Multiplication Calculator - Performs Lattice Multiplication or the Napiers Bones (Napier Rods) method of multiplication

Laura found a roll of fencing in her garage. She couldn't decide whether to fence in a square garden
Laura found a roll of fencing in her garage. She couldn't decide whether to fence in a square garden or a round garden with the fencing. Laura did some calculations and found that a circular garden would give her 1380 more square feet than a square garden. How many feet of fencing were in the roll that Laura found? (Round to the nearest foot.) Feet of fencing = n Perimeter of square garden = n Each side of square = n/4 Square garden's area = (n/4)^2 = n^2/16 Area of circle garden with circumference = n is: Circumference = pi * d n = pi * d Divide body tissues by pi: d = n/pi Radius = n/2pi Area = pi * n/2pi * n/2pi Area = pin^2/4pi^2 Reduce by canceling pi: n^2/4pi n^2/4 * 3.14 n^2/12.56 The problem says that the difference between the square's area and the circle's area is equal to 1380 square feet. Area of Circle - Area of Square = 1380 n^2/12.56 - n^2/16 = 1380 Common denominator = 200.96 (16n^2 - 12.56n^2)/200.96 = 1380 3.44n^2/200.96 = 1380 Cross multiply: 3.44n^2 = 277,324.8 n^2 = 80,617.7 n = 283.9 Nearest foot = [B]284[/B]

Laura has three errands to complete. She must wash the dishes, mow the lawn, and paint a fence. How
Laura has three errands to complete. She must wash the dishes, mow the lawn, and paint a fence. How many ways can Laura arrange the order of the three errands? 3! = 3 * 2 * 1 = [B]6 ways[/B]

Laura weighs 45 pounds more than her pet dog. When they are on the scale together, they weigh 85 pou
Laura weighs 45 pounds more than her pet dog. When they are on the scale together, they weigh 85 pounds. How much does Laura weigh? Let Laura weigh l and her dog weigh d. WE have: [LIST=1] [*]l = d + 45 [*]d + l = 85 [/LIST] Substitute equation (1) into Equation (2) for l: d + d + 45 = 85 Solve for [I]d[/I] in the equation d + d + 45 = 85 [SIZE=5][B]Step 1: Group the d terms on the left hand side:[/B][/SIZE] (1 + 1)d = 2d [SIZE=5][B]Step 2: Form modified equation[/B][/SIZE] 2d + 45 = + 85 [SIZE=5][B]Step 3: Group constants:[/B][/SIZE] We need to group our constants 45 and 85. To do that, we subtract 45 from both sides 2d + 45 - 45 = 85 - 45 [SIZE=5][B]Step 4: Cancel 45 on the left side:[/B][/SIZE] 2d = 40 [SIZE=5][B]Step 5: Divide each side of the equation by 2[/B][/SIZE] 2d/2 = 40/2 d = 20 From equation (1), we substitute d = 20: l = d + 45 l = 20 + 45 l = [B]65 pounds [URL='https://www.mathcelebrity.com/1unk.php?num=d%2Bd%2B45%3D85&pl=Solve']Source[/URL][/B]

Lauren invested $340 in an account paying an interest rate of 5.8% compounded monthly. Assuming no d
Lauren invested $340 in an account paying an interest rate of 5.8% compounded monthly. Assuming no deposits or withdrawals are made, how much money, to the nearest cent, would be in the account after 13 years? 13 years * 12 months per year = 156 compounding periods. [URL='https://www.mathcelebrity.com/compoundint.php?bal=340&nval=156&int=5.8&pl=Monthly']Using our compound interest balance calculator[/URL] with 156 for t, we get: $[B]721.35[/B]

Lauren's savings increased by 12 and is now 31
Lauren's savings increased by 12 and is now 31 [LIST] [*]Let Lauren's savings be s. [*]The phrase increased by means we add. [*]The phrase [I]is now[/I] means an equation. [*]We have an algebraic expression of: [/LIST] [B]s + 12 = 31 [/B] To solve for s, we [URL='https://www.mathcelebrity.com/1unk.php?num=s%2B12%3D31&pl=Solve']type this equation into our search engine[/URL] and we get: s = [B]19[/B]

Layla buys 2 1/2pounds of chocolate for 3.50 how much is she paying for a pound of chocolate
Layla buys 2 1/2pounds of chocolate for 3.50 how much is she paying for a pound of chocolate? [URL='https://www.mathcelebrity.com/fraction.php?frac1=2%261%2F2&frac2=3%2F8&pl=Simplify']Using our mixed fraction converter[/URL], 2&1/2 = 5/2 Cost per pound = 3.50 / 5/2 pounds Dividing by 5/2 is the same as multiplying by the reciprocal 2/5: 3.50 * 2/5 7/5 [B]$1.40 per pound[/B]

Leah is 12 years older than Anna. if the age of Anna is x, what is the age of Leah?
Leah is 12 years older than Anna. if the age of Anna is x, what is the age of Leah? Older means we add 12 to Anna's age. So if Anna's age is x, then Leah's age (l) is: l = [B]x + 12[/B]

Lebron James scored 288 points in 9 games this season. Assuming he continues to score at this consta
Lebron James scored 288 points in 9 games this season. Assuming he continues to score at this constant rate, write a linear equation that represents the scenario. 288 points / 9 games = 32 points per game Let g be the number of games Lebron plays. We build an equation for his season score: Lebron's Season Score = Points per game * number of games Lebron's Season Score = [B]32g[/B]

Leifs rich uncle decided to give him $1.00 the first day of Christmas and to double the amount each
Leifs rich uncle decided to give him $1.00 the first day of Christmas and to double the amount each subsequent day. How much money (in dollars) does he recieve after all 12 days of Christmas? Let's look at each day: [LIST=1] [*]1 [*]2 [*]4 [*]8 [*]16 [*]32 [*]64 [*]128 [*]256 [*]512 [*]1024 [*]2048 [/LIST] Total received: 1 + 2 + 4 + 8 + 16 + 32 + 64 + 128 + 256 + 512 + 1024 + 2048 = [B]4,095[/B]

Leilani can read 20 pages in 2 minutes. if she can maintain this page, how many pages can she read i
Leilani can read 20 pages in 2 minutes. if she can maintain this page, how many pages can she read in an hour? We know that 1 hour is 60 minutes. Let p be the number of pages Leilani can read in 1 hour (60 minutes) The read rate is constant, so we can build a proportion. 20 pages /2 minutes = p/60 We can cross multiply: Numerator 1 * Denominator 2 = Denominator 1 * Numerator 2 [SIZE=5][B]Solving for Numerator 2 we get:[/B][/SIZE] Numerator 2 = Numerator 1 * Denominator 2/Denominator 1 [SIZE=5][B]Evaluating and simplifying using your input values we get:[/B][/SIZE] p = 20 * 60/ 2 p = 1200/2 p = [B]600[/B]

Lena purchased a prepaid phone card for $15. Long distance calls cost 24 cents a minute using this
Lena purchased a prepaid phone card for $15. Long distance calls cost 24 cents a minute using this card. Lena used her card only once to make a long distance call. If the remaining credit on her card is $4.92, how many minutes did her call last? [U]Figure out how many minutes Lena used:[/U] Lena spent $15 - $4.92 = $10.08. [U]Now determine the amount of minutes[/U] $10.08/0.24 cents per minute = [B]42 minutes[/B]

Length (l) is the same as width (w) and their product is 64.
Length (l) is the same as width (w) and their product is 64. We're given 2 equations: [LIST=1] [*]lw = 64 [*]l = w [/LIST] Substitute equation (2) into equation (1): w * w = 64 w^2 = 64 [B]w = 8[/B] Since l = w, then [B]l = 8[/B]

Leo and Zach went to lunch at a cafe. They ordered a spinach salad for $6.55, a tuna sandwich for $4
Leo and Zach went to lunch at a cafe. They ordered a spinach salad for $6.55, a tuna sandwich for $4.75, and 2 glasses of lemonade for $0.85 each. The tax was $1.30. They gave the waiter $15.00. How much change should they have received? Change = Cash - Total Bill - Tax Change = $15 - ($6.55 + $4.75 + 2($0.85)) - $1.30 Change = $15 - ($6.55 + $4.75 + $1.70) - $1.30 Change = $15 - $13 - $1.30 Change = $15 - $14.30 Change = [B]$0.70 or 70 cents[/B]

Leonard earned $100 from a bonus plus $15 per day (d) at his job this week. Which of the following e
Leonard earned $100 from a bonus plus $15 per day (d) at his job this week. Which of the following expressions best represents Leonards income for the week? We set up an income function I(d), were d is the number of days Leonard works: [B]I(d) = 15d + 100 [/B] Each day, Leonard earns $15. Then we add on the $100 bonus

Leslie has 8 pencils. She has 9 fewer pencils than Michelle. How many pencils does Michelle have?
Let m = the number of pencils Michelle has. So, Leslie has m - 9 = 8. Add 9 to both sides: m = 17. So Michelle has 17 pencils, and Leslie has 8, which is 9 fewer than 17

Let A = (-4,5) and B = (1,3) Find the distance from A to B
Let A = (-4,5) and B = (1,3) Find the distance from A to B Using our [URL='https://www.mathcelebrity.com/slope.php?xone=-4&yone=5&slope=+&xtwo=1&ytwo=3&bvalue=+&pl=You+entered+2+points']distance between two points calculator[/URL], we get: [B]5.3852[/B]

Let A and B be independent events with P(A) = 0.52 and P(B) = 0.62. a. Calculate P(A ? B).
Let A and B be independent events with P(A) = 0.52 and P(B) = 0.62. a. Calculate P(A ? B). With independent events, the intersection probability is found by: P(A ? B) = P(A) * P(B) P(A ? B) = 0.52 * 0.62 P(A ? B) = [B]0.3224[/B]

Let A={a,b,c} and B={1,2,3} Compute A?B
Let A={a,b,c} and B={1,2,3} Compute A?B Union means all elements in either A or B, so we have: A?B = [B]{a,b,c,1,2,3}[/B]

Let f(x) = 3x - 6 and g(x)= -2x + 5. Which of the following is f(x) - g(x)?
Let f(x) = 3x - 6 and g(x)= -2x + 5. Which of the following is f(x) - g(x)? f(x) - g(x) = 3x - 6 - (-2x + 5) Distribute the negative sign where double negative equals a plus: f(x) - g(x) = 3x - 6 + 2x - 5 Combine like terms: f(x) - g(x) = (3 + 2)x - 6 - 5 f(x) - g(x) = [B]5x - 11[/B]

Let f(x)=5x and g(x)=2x + 1; find and simplify the following f(g(x))
Let f(x)=5x and g(x)=2x + 1; find and simplify the following f(g(x)) f(g(x)) = 5(g(x)) f(g(x)) = 5(2x + 1) f(g(x)) = [B]10x + 5[/B]

Let n be an integer. If n^2 is odd, then n is odd
Let n be an integer. If n^2 is odd, then n is odd Proof by contraposition: Suppose that n is even. Then we can write n = 2k n^2 = (2k)^2 = 4k^2 = 2(2k) so it is even [I]So an odd number can't be the square of an even number. So if an odd number is a square it must be the square of an odd number.[/I]

Let n be the middle number of three consecutive integers
Let n be the middle number of three consecutive integers This means: [LIST] [*]n is the second of three consecutive integers [*]The first consecutive integer is n - 1 [*]The third consecutive integer is n + 1 [/LIST] The sum is found by: n - 1 + n + n + 1 Simplifying, we get: (n + n + n) + 1 - 1 [B]3n[/B]

Let P(n) and S(n) denote the product and the sum, respectively, of the digits of the integer n. For
Let P(n) and S(n) denote the product and the sum, respectively, of the digits of the integer n. For example, P(23) = 6 and S(23) = 5. Suppose N is a two-digit number such that N = P(N) + S(N). What could N be? Is there more than one answer? For example, for 23 P(23) = 6 and S(23) = 5, but 23 could not be the N that we want since 23 <> 5 + 6 Let t = tens digit and o = ones digit P(n) = to S(n) = t + o P(n) + S(n) = to + t + o N = 10t + o Set them equal to each other N = P(N) + S(N) 10t + o = to + t + o o's cancel, so we have 10t = to + t Subtract t from each side, we have 9t = to Divide each side by t o = 9 So any two-digit number with 9 as the ones digit will work: [B]{19,29,39,49,59,69,79,89,99}[/B]

Let U be the set of all integers between ?3 and 3 (including ?3 and 3). Let A={?2,0,1,3}. Find Ac. G
Let U be the set of all integers between ?3 and 3 (including ?3 and 3). Let A={?2,0,1,3}. Find Ac. Give your answer in standard set notation Ac is anything not in A, but in U. So we have: Ac = [B]{-3, -1, 2}[/B]

Let x be an integer. If x is odd, then x^2 is odd
Let x be an integer. If x is odd, then x^2 is odd Proof: Let x be an odd number. This means that x = 2n + 1 where n is an integer. [U]Squaring x, we get:[/U] x^2 = (2n + 1)^2 = (2n + 1)(2n + 1) x^2 = 4n^2 + 4n + 1 x^2 = 2(2n^2 + 2n) + 1 2(2n^2 + 2n) is an even number since 2 multiplied by any integer is even So adding 1 is an odd number [MEDIA=youtube]GlzV80M33x0[/MEDIA]

Let x be the dog’s age in years. What is the dog’s age when he is thrice as old?
Let x be the dog’s age in years. What is the dog’s age when he is thrice as old? Thrice means triple, or multiply by 3. So we have the future age as: [B]3x[/B]

let x be the variable, an age that is at least 57 years old
let x be the variable, an age that is at least 57 years old At least means greater than or equal to x >= 57

Letter Arrangements in a Word
Free Letter Arrangements in a Word Calculator - Given a word, this determines the number of unique arrangements of letters in the word.

Levi invested $630 in an account paying an interest rate of 4.6% compounded daily. Assuming no depos
Levi invested $630 in an account paying an interest rate of 4.6% compounded daily. Assuming no deposits or withdrawals are made, how long would it take, to the nearest year, for the value of the account to reach $970? 3,425 days, per the [URL='http://www.mathcelebrity.com/compoundint.php?bal=630&nval=3425&int=4.6&pl=Daily']balance calculator[/URL].

Liam, a 19th century cowboy, carries an 1847 Colt single action 6 shooter revolver. So proficient is
Liam, a 19th century cowboy, carries an 1847 Colt single action 6 shooter revolver. So proficient is he with this weapon that when he fires all 6 shots in a row, the time between the first bullet and the last is 40 seconds. How long would it take him to fire 4 shots? We set up a proportion of shots to seconds where s is the number of seconds it takes to fire 4 shots: 6/40 = 4/s Using our [URL='https://www.mathcelebrity.com/prop.php?num1=6&num2=4&den1=40&den2=s&propsign=%3D&pl=Calculate+missing+proportion+value']proportion calculator[/URL], we get: s = [B]26.67[/B]

License plate that is made up of 4 letters followed by 2 numbers
License plate that is made up of 4 letters followed by 2 numbers Using the fundamental rule of counting, we have: 26 possible letters * 26 possible letters * 26 possible letters * 26 possible letters * 10 possible numbers * 10 possible numbers = [B]45,697,600 license plate combinations[/B]

license plate with 4 letter combinations and 3 number combinations
license plate with 4 letter combinations and 3 number combinations There are 26 total letters and 10 digits [0-9]. We have 26 C 4 * 10 C 3. [URL='http://www.mathcelebrity.com/permutation.php?num=26&den=4&pl=Combinations']26 C 4[/URL] = 14,950 [URL='http://www.mathcelebrity.com/permutation.php?num=10&den=3&pl=Combinations']10 C 3[/URL] = 120 Total license plate combinations: 14,950 * 120 = [B]1,794,000[/B]

License plates are made using 3 letters followed by 2 digits. How many plates can be made if repetit
License plates are made using 3 letters followed by 2 digits. How many plates can be made if repetition of letters and digits is allowed We have 26 letters A-Z and 10 possible digits 0-9. Using the fundamental rule of counting, we have: 26 * 26 * 26 * 10 * 10 = [B]1,757,600 possible choices[/B]

License plates are made using 3 letters followed by 3 digits. How many plates can be made of repetit
License plates are made using 3 letters followed by 3 digits. How many plates can be made of repetition of letters and digits is allowed We have 26 letters in the alphabet We have 10 digits [0-9] The problem asks for the following license plate scenario of Letters (L) and Digits (D) LLLDDD The number of plates we can make using L = 26 and D = 10 using the fundamental rule of counting is: Number of License Plates = 26 * 26 * 26 * 10 * 10 * 10 Number of License Plates = [B]17,576,000[/B]

Lilly has 10 peices of fruit. 5 of them are pears the rest are apples how many apples does she have?
Lilly has 10 peices of fruit. 5 of them are pears the rest are apples how many apples does she have? The 10 pieces are either pears [U]or[/U] apples 10 pieces of fruit - 5 pears = [B]5 apples[/B]

Lily needs an internet connectivity package for her firm. She has a choice between CIVISIN and GOMI
Lily needs an internet connectivity package for her firm. She has a choice between CIVISIN and GOMI with the following monthly billing policies. Each company's monthly billing policy has an initial operating fee and charge per megabyte. Operating Fee charge per Mb CIVSIN 29.95 0.14 GOMI 4.95 0.39 (i) Write down a system of equations to model the above situation (ii) At how many Mb is the monthly cost the same? What is the equal monthly cost of the two plans? (i) Set up a cost function C(m) for CIVSIN where m is the number of megabytes used: C(m) = charge per Mb * m + Operating Fee [B]C(m) = 0.14m + 29.95[/B] Set up a cost function C(m) for GOMI where m is the number of megabytes used: C(m) = charge per Mb * m + Operating Fee [B]C(m) = 0.39m + 4.95 [/B] (ii) At how many Mb is the monthly cost the same? Set both cost functions equal to each other: 0.14m + 29.95 = 0.39m + 4.95 We [URL='https://www.mathcelebrity.com/1unk.php?num=0.14m%2B29.95%3D0.39m%2B4.95&pl=Solve']type this equation into our search engine[/URL] and we get: m = [B]100[/B] (ii) What is the equal monthly cost of the two plans? CIVSIN - We want C(100) from above where m = 100 C(100) = 0.14(100) + 29.95 C(100) = 14 + 29.95 C(100) = [B]43.95[/B] GOMI - We want C(100) from above where m = 100 C(100) = 0.39(100) + 4.95 C(100) = 39 + 4.95 C(100) = [B]43.95[/B]

Lily put $750 in the bank if she earns 4% interest how much will she have in 5 years?
Lily put $750 in the bank if she earns 4% interest how much will she have in 5 years? We assume annual compounding, so [URL='https://www.mathcelebrity.com/compoundint.php?bal=750&nval=5&int=4&pl=Annually']using our balance with compound interest calculator[/URL], we have: [B]$912.49[/B]

Limit of a Function
Free Limit of a Function Calculator - This lesson walks you through what limit is, how to write limit notation, and limit theorems

Linda estimates that her business is growing at a rate of 6% per year. Her profits is 2002 were $30,
Linda estimates that her business is growing at a rate of 6% per year. Her profits is 2002 were $30,000. To the nearest hundred dollars, estimate her profits for 2011. Calculate the number of years of appreciation: Appreciation years = 2011 - 2002 Appreciation years = 9 So we want 30000 to grow for 9 years at 6%. We [URL='https://www.mathcelebrity.com/apprec-percent.php?num=30000togrowfor9yearsat6%.whatisthevalue&pl=Calculate']type this into our search engine[/URL] and we get: [B]$50,684.37[/B]

Linda takes classes at both Westside Community College and Pinewood Community College. At Westside,
Linda takes classes at both Westside Community College and Pinewood Community College. At Westside, class fees are $98 per credit hour, and at Pinewood, class fees are $115 per credit hour. Linda is taking a combined total of 18 credit hours at the two schools. Suppose that she is taking w credit hours at Westside. Write an expression for the combined total dollar amount she paid for her class fees. Let p be the number of credit hours at Pinewood. We have two equations: [LIST] [*]98w for Westside [*]115p at Pinewood [*]w + p = 18 [*]Total fees: [B]98w + 115p[/B] [/LIST]

Lindsey took a total of 8 quizzes over the course of 2 weeks. After attending 5 weeks of school this
Lindsey took a total of 8 quizzes over the course of 2 weeks. After attending 5 weeks of school this quarter, how many quizzes will Lindsey have taken in total? Assume the relationship is directly proportional. Since the relationship is directly proportional, set up a proportion of quizzes to weeks, where q is the number of quizzes Lindsey will take in 5 weeks: 8/2 = q/5 [URL='https://www.mathcelebrity.com/prop.php?num1=8&num2=q&den1=2&den2=5&propsign=%3D&pl=Calculate+missing+proportion+value']We type this proportion into our search engine[/URL], and we get: [B]q = 20 [/B] Another way to look at this is, Lindsey takes 8 quizzes over 2 weeks. This means she takes 4 per week since 8/2 = 4. So if she takes 4 quizzes per week, then in 5 weeks, she takes 4*5 = 20 quizzes.

Line Equation-Slope-Distance-Midpoint-Y intercept
Free Line Equation-Slope-Distance-Midpoint-Y intercept Calculator - Enter 2 points, and this calculates the following:
* Slope of the line (rise over run) and the line equation y = mx + b that joins the 2 points
* Midpoint of the two points
* Distance between the 2 points
* 2 remaining angles of the rignt triangle formed by the 2 points
* y intercept of the line equation
* Point-Slope Form
* Parametric Equations and Symmetric Equations

Or, if you are given a point on a line and the slope of the line including that point, this calculates the equation of that line and the y intercept of that line equation, and point-slope form.

Also allows for the entry of m and b to form the line equation

Line m passes through points (3, 16) and (8, 10). Line n is perpendicular to m. What is the slope of
Line m passes through points (3, 16) and (8, 10). Line n is perpendicular to m. What is the slope of line n? First, find the slope of the line m passing through points (3, 16) and (8, 10). [URL='https://www.mathcelebrity.com/slope.php?xone=3&yone=16&slope=+2%2F5&xtwo=8&ytwo=10&pl=You+entered+2+points']Typing the points into our search engine[/URL], we get a slope of: m = -6/5 If line n is perpendicular to m, then the slope of n is denote as: n = -1/m n = -1/-6/5 n = -1*-5/6 n = [B]5/6[/B]

Line m passes through points (7, 5) and (9, 10). Line n passes through points (3, 1) and (7, 10). Ar
Line m passes through points (7, 5) and (9, 10). Line n passes through points (3, 1) and (7, 10). Are line m and line n parallel or perpendicular [U]Slope of line m is:[/U] (y2 - y1)/(x2 - x1) (10 - 5)/(9 - 7) 5/2 [U]Slope of line n is:[/U] (y2 - y1)/(x2 - x1) (10 - 1)/(7 - 3) 9/4 Run 3 checks on the slopes: [LIST=1] [*]Lines that are parallel have equal slopes. Since 5/2 does not equal 9/4, these lines [B]are not parallel[/B] [*]Lines that are perpendicular have negative reciprocal slopes. Since 9/4 is not equal to -2/5 (the reciprocal of the slope of m), these lines [B]are not perpendicular[/B] [*][B]Therefore, since the lines are not parallel and not perpendicular[/B] [/LIST]

Linear Congruential Generator
Free Linear Congruential Generator Calculator - Using the linear congruential generator algorithm, this generates a list of random numbers based on your inputs

Linear Conversions
Free Linear Conversions Calculator - Converts to and from the following linear measurements for a given quantity:
Inches
Feet
Yards
Miles
Micrometer
Millimeters
Centimeters
Meters
Kilometers
Furlongs

Liquid Conversions
Free Liquid Conversions Calculator - Takes a liquid measurement as seen in things like recipes and performs the following conversions: ounces, pints, quarts, gallons, teaspoon (tsp), tablespoon (tbsp), microliters, milliliters, deciliters, kiloliters,liters, bushels, and cubic meters.

Lisa has $150 at most to spend on clothes. She wants to buy a pair of jeans for $58 and will spend t
Lisa has $150 at most to spend on clothes. She wants to buy a pair of jeans for $58 and will spend the rest on t-shirts that cost $14 each. Let the number of t-shirts be t. Lisa can spend up to, but not more than 150. We have the following inequality: 14t + 58 <= 150 To solve this inequality, we [URL='https://www.mathcelebrity.com/1unk.php?num=14j%2B58%3C%3D150&pl=Solve']type it in our search engine[/URL] and we get: t <= 6.57 To round to a whole number, we round down to [B]t = 6 [/B]

Lisa has 32 nickels this is one third of her coins how many coins does she have
Lisa has 32 nickels this is one third of her coins how many coins does she have Let x be the total amount of coins. we have: 32 = x/3 Cross multiply, we get: [B]x = 96[/B]

Lisa has 5 skirts, 10 blouses, and 4 jackets. How many 3-piece outfits can she put together assuming
Lisa has 5 skirts, 10 blouses, and 4 jackets. How many 3-piece outfits can she put together assuming any piece goes with any other? Using the fundamental rule of counting, we have: 5 * 10 * 4 = [B]200 different 3-piece outfits[/B]

Lisa wants to rent a boat and spend less than $52. The boat costs $7 per hour, and Lisa has a discou
Lisa wants to rent a boat and spend less than $52. The boat costs $7 per hour, and Lisa has a discount coupon for $4 off. What are the possible numbers of hours Lisa could rent the boat? Calculate discounted cost: Discounted cost = Full Cost - Coupon Discounted cost = 52 - 7 Discounted cost = 45 Since price equals rate * hours (h), and we want the inequality (less than) we have: 7h < 52 Using our [URL='https://www.mathcelebrity.com/interval-notation-calculator.php?num=7h%3C52&pl=Show+Interval+Notation']inequality calculator,[/URL] we see that: [B]h < 7.42[/B]

list the natural numbers less than 70 that are divisible by 8
list the natural numbers less than 70 that are divisible by 8 Natural numbers are {1, 2, 3, ... We want natural numbers less than 70 which are divisible by 8: [LIST] [*]8 * 1 = 8 [*]8 * 2 = 16 [*]8 * 3 = 24 [*]8 * 4 = 32 [*]8 * 5 = 40 [*]8 * 6 = 48 [*]8 * 7 = 56 [*]8 * 8 = 64 [/LIST] Our answer is: [B]{8, 16, 24, 32, 40, 48, 56, 64}[/B]

Littles Law
Free Littles Law Calculator - Given two out of the three inputs for Littles Law, Throughput (TH), Cycle Time (CT, and WIP, this solves for the third item.

Liz harold has a jar in her office that contains 47 coins. Some are pennies and the rest are dimes.
Liz harold has a jar in her office that contains 47 coins. Some are pennies and the rest are dimes. If the total value of the coins is 2.18, how many of each denomination does she have? [U]Set up two equations where p is the number of pennies and d is the number of dimes:[/U] (1) d + p = 47 (2) 0.1d + 0.01p = 2.18 [U]Rearrange (1) into (3) by solving for d[/U] (3) d = 47 - p [U]Substitute (3) into (2)[/U] 0.1(47 - p) + 0.01p = 2.18 4.7 - 0.1p + 0.01p = 2.18 [U]Group p terms[/U] 4.7 - 0.09p = 2.18 [U]Add 0.09p to both sides[/U] 0.09p + 2.18 = 4.7 [U]Subtract 2.18 from both sides[/U] 0.09p = 2.52 [U]Divide each side by 0.09[/U] [B]p = 28[/B] [U]Now substitute that back into (3)[/U] d =47 - 28 [B]d = 19[/B]

Local salesman receives a base salary of $650 monthly. He also receives a commission of 11% on all s
Local salesman receives a base salary of $650 monthly. He also receives a commission of 11% on all sales over $1500. How much would he have to sell in one month if he needed to have $3000 Let the Sales amount be s. We have: Sales over 1,500 is written as s - 1500 11% is also 0.11 as a decimal, so we have: 0.11(s - 1500) + 650 = 3000 Multiply through: 0.11s - 165 + 650 = 3500 0.11s + 485 = 3500 To solve this equation for s, [URL='https://www.mathcelebrity.com/1unk.php?num=0.11s%2B485%3D3500&pl=Solve']we type it in our search engine[/URL] and we get: s = [B]27,409.10[/B]

log5 = 0.699, log2 = 0.301. Use these values to evaluate log40
log5 = 0.699, log2 = 0.301. Use these values to evaluate log40. One of the logarithmic identities is: log(ab) = log(a) + log(b). Using the numbers 2 and 5, we somehow need to get to 40. [URL='http://www.mathcelebrity.com/factoriz.php?num=40&pl=Show+Factorization']List factors of 40[/URL]. On the link above, take a look at the bottom where it says prime factorization. We have: 40 = 2 x 2 x 2 x 5 Using our logarithmic identity, we have: log40 = log(2 x 2 x 2 x 5) Rewriting this using our identity, we have: log40 = log2 + log2 + log2 + log5 log40 = 0.301 + 0.301 + 0.301 + 0.699 log40 = [B]1.602 [MEDIA=youtube]qyG_Jkf9VDc[/MEDIA][/B]

Logan is 8 years older than 4 times the age of his nephew. Logan is 32 years old. How old is his nep
Logan is 8 years older than 4 times the age of his nephew. Logan is 32 years old. How old is his nephew? Let the age of Logan's nephew be n. We're given: 4n + 8 = 32 (Since [I]older[/I] means we add) To solve this equation for n, we [URL='https://www.mathcelebrity.com/1unk.php?num=4n%2B8%3D32&pl=Solve']type it into our search engine[/URL] and we get: [B]n = 6[/B]

Logarithms
Free Logarithms Calculator - Using the formula Log ab = e, this calculates the 3 pieces of a logarithm equation:
1) Base (b)
2) Exponent
3) Log Result
In addition, it converts
* Expand logarithmic expressions

Logarithms and Natural Logarithms and Eulers Constant (e)
Free Logarithms and Natural Logarithms and Eulers Constant (e) Calculator - This calculator does the following:
* Takes the Natural Log base e of a number x Ln(x) → logex
* Raises e to a power of y, ey
* Performs the change of base rule on logb(x)
* Solves equations in the form bcx = d where b, c, and d are constants and x is any variable a-z
* Solves equations in the form cedx=b where b, c, and d are constants, e is Eulers Constant = 2.71828182846, and x is any variable a-z
* Exponential form to logarithmic form for expressions such as 53 = 125 to logarithmic form
* Logarithmic form to exponential form for expressions such as Log5125 = 3


Logistic Map
Free Logistic Map Calculator - Given r, x0 and (n) trials, this will display the logistic map.

Lois is purchasing an annuity that will pay $5,000 annually for 20 years, with the first annuity pay
Lois is purchasing an annuity that will pay $5,000 annually for 20 years, with the first annuity payment made on the date of purchase. What is the value of the annuity on the purchase date given a discount rate of 7 percent? This is an annuity due, since the first payment is made on the date of purchase. Using our [URL='http://www.mathcelebrity.com/annimmpv.php?pv=&av=&pmt=5000&n=20&i=7&check1=2&pl=Calculate']present value of an annuity due calculator[/URL], we get [B]56,677.98[/B].

Look at this sequence: 53, 53, 40, 40, 27, 27, ... What number should come next?
Look at this sequence: 53, 53, 40, 40, 27, 27, ... What number should come next? This looks like a sequence where we subtract 13 and then 0, 13 and then 0 from the prior number. Since the last group of 27 repeated, our next number is found by subtracting 13: 27 - 13 = [B]14[/B]

Lorda is older than Kate. The sum of their ages is 30. The difference in their ages is 6. What are t
Lorda is older than Kate. The sum of their ages is 30. The difference in their ages is 6. What are their ages? Let Lorda's age be l. Let Kate's age be k. We're given two equations: [LIST=1] [*]l + k = 30 [*]l - k = 6 <-- Since Lorda is older [/LIST] Add the 2 equations together and we eliminate k: 2l = 36 [URL='https://www.mathcelebrity.com/1unk.php?num=2l%3D36&pl=Solve']Typing this equation into our search engine[/URL] and solving for l, we get: l = [B]18[/B] Now substitute l = 18 into equation 1: 18 + k = 30 [URL='https://www.mathcelebrity.com/1unk.php?num=18%2Bk%3D30&pl=Solve']Type this equation into our search engine[/URL] and solving for k, we get: k = [B]12[/B]

Lotto Drawing Probability
Free Lotto Drawing Probability Calculator - Given a lotto drawing with a Pick(x) out of (y) total choices, this calculates the probability of winning that lottery picking all (x) correct numbers.

Louis kept money through a hole inn his pocket. He started with 35 cents, lost 20 cents , put in 75
Louis kept money through a hole inn his pocket. He started with 35 cents, lost 20 cents , put in 75 cents , spent 43 cents, lost 16 cents again, and then put in 14 cents. How much change should there be in his pocket? The phrase [I]put in[/I] mean we add money to the total The phrases s[I]pent or lost[/I] mean we subtract 35 - 20 + 75 - 43 - 16 + 14 = [B]45 cents[/B]

Lucas has nickels,dimes,and quarters in the ratio 1:3:2. If 10 of Lucas coins are quarters, how many
Lucas has nickels,dimes,and quarters in the ratio 1:3:2. If 10 of Lucas coins are quarters, how many nickels and dimes does Lucas have? 1 + 3 + 2 = 6. Quarters account for 2/6 which is 1/3 of the total coin count. Let x be the total number of coins. We have: 1/3x = 10 Multiply each side by 3 x = 30 We have the following ratios and totals: [LIST] [*]Nickels: 1/6 * 30 = [B]5 nickels[/B] [*]Dimes: 3/6 * 30 = [B]15 dimes[/B] [*]Quarters: 2/6 * 30 = [B]10 quarters[/B] [/LIST]

Lucas is offered either 15% or $21 off his total shopping bill. How much would have to be spent to m
Lucas is offered either 15% or $21 off his total shopping bill. How much would have to be spent to make the 15% option the best one? Let the total bill be b. We have: 0.15b > 21 <-- Since 15% is 0.15 Using our [URL='http://www.mathcelebrity.com/interval-notation-calculator.php?num=0.15b%3E21&pl=Show+Interval+Notation']inequality calculator[/URL], we get [B]b>140[/B]. So any bill greater than $140 will make the 15% off option the best one, since the discount will be higher than $21.

Lucas Numbers
Free Lucas Numbers Calculator - Generates a list of the first 100 Lucas numbers.

Lucy has taken four tests in math class and has an average of 85. i. What score would she have to g
Lucy has taken four tests in math class and has an average of 85. i. What score would she have to get on her fifth test to have an average of 88? ii. Can she get an average of 90? Explain. i. She would need a perfect score of [B]100[/B] from our [URL='http://www.mathcelebrity.com/missingaverage.php?num=+81%2C83%2C87%2C89&avg=+88&pl=Calculate+Missing+Score']Missing Average Calculator[/URL] ii. [B]Impossible since we know from question i., a score of 100 only gets her to an 88. She cannot score higher than 100 on the fifth test, therefore, she cannot attain an average score of 90.[/B]

Lucy is thinking of a number. The number is greater than two hundred twenty-five. Her number is less
Lucy is thinking of a number. The number is greater than two hundred twenty-five. Her number is less than 2 hundreds, 2 tens, and 7 ones. What is Lucy's number? Let the number be n. n > 225 Also: n < 2(100) + 2(10) + 7(1) n < 200 + 20 + 7 n < 227 Combine these, we get: 225 < n < 227 Only one number satisfies this: n = [B]226 [MEDIA=youtube]-LFbAZFy13o[/MEDIA][/B]

Luke and 5 friends packed enough food for a 2-week canoe trip. If one extra person decided to go on
Luke and 5 friends packed enough food for a 2-week canoe trip. If one extra person decided to go on the trip at the last minute, how long will the food last? Luke and 5 friends = 6 people. 2 weeks = 14 days, so the food lasts 6 people * 14 days = 84 days One extra person on the trip means 6 + 1 = 7 people. 84 days of food / 7 people = [B]12 days[/B]

Luke and Dan's total debt is $72. If Luke's debt is three times Dan's debt, what is Dan's debt?
Luke and Dan's total debt is $72. If Luke's debt is three times Dan's debt, what is Dan's debt? Let Dan's debt be d. Let Luke's debt be l. We're given two equations: [LIST=1] [*]d + l = 72 [*]l = 3d [/LIST] Substitute equation (2) for l into equation (1): d + 3d = 72 Solve for [I]d[/I] in the equation d + 3d = 72 [SIZE=5][B]Step 1: Group the d terms on the left hand side:[/B][/SIZE] (1 + 3)d = 4d [SIZE=5][B]Step 2: Form modified equation[/B][/SIZE] 4d = + 72 [SIZE=5][B]Step 3: Divide each side of the equation by 4[/B][/SIZE] 4d/4 = 72/4 d = [B]18[/B]

Luke drove for n hours at 55 miles per hour. Luke's mother drove for n hours at a speed of 60 miles
Luke drove for n hours at 55 miles per hour. Luke's mother drove for n hours at a speed of 60 miles per hour. How much farther than Luke did his mother drive? Distance = Rate * Time [LIST] [*]Luke drove: 55n [*]Mom drove 60n [/LIST] Distance difference = 60n - 55n = [B]5n[/B]

Luna had $50 when she got to the carnival. After riding 12 rides she had $26. What was the price aft
Luna had $50 when she got to the carnival. After riding 12 rides she had $26. What was the price after each ride? After riding 12 rides, Lucy had $26. Which means she spent $50 - $26 = $24. $24 / 12 rides = [B]$2 per ride[/B].

Lunch meat A is 10.00 for 2 lbs or meat B for 6.00 for 1lb
Lunch meat A is 10.00 for 2 lbs or meat B for 6.00 for 1lb Determine unit cost: Unit Cost A = 10/2 lbs = 5 per lb Unit Cost B = 6/1lb = 6 per lb [B]Unit Cost A is less, so that is the better buy.[/B]

M deck of cards . Each deck has 52 cards . The total number of cards
M deck of cards . Each deck has 52 cards . The total number of cards. [B]52M[/B]

M decreased by the sum of 13 and the number P is less than 12
M decreased by the sum of 13 and the number P is less than 12 The sum of 13 and the number P 13 + P M decreased by the sum of 13 and the number P M - (13 + P) Less than 12 means we set this entire expression less than 12 as an inequality [B]M - (13 + P) < 12[/B]

M is halved, then 7 is added
M is halved, then 7 is added Take this algebraic expression in parts: [LIST] [*]M is halved. This means we divide M by 2: M/2 [*]Then 7 is added. We add 7 to M/2 [/LIST] [B]M/2 + 7[/B]

m is inversely proportional to the square of p-1 when p=4 m=5 find m when p=6
m is inversely proportional to the square of p-1 when p=4 and m=5. find m when p=6 Inversely proportional means there is a constant k such that: m = k/(p - 1)^2 When p = 4 and m = 5, we have: 5 = k/(4 - 1)^2 5 = k/3^2 5 = k/9 [U]Cross multiply:[/U] k = 45 [U]The problems asks for m when p = 6. And we also now know that k = 45. So plug in the numbers:[/U] m = k/(p - 1)^2 m = 45/(6 - 1)^2 m = 45/5^2 m = 45/25 m = [B]1.8[/B]

M is the midpoint of AB. Prove AB = 2AM
M is the midpoint of AB. Prove AB = 2AM M is the midpoint of AB (Given) AM = MB (Definition of Congruent Segments) AM + MB = AB (Segment Addition Postulate) AM + AM = AB (Substitution Property of Equality) 2AM = AB (Distributive property) [MEDIA=youtube]8BNo_4kvBzw[/MEDIA]

m is the midpoint of cf for points c(3,4) and f(9,8). Find MF
m is the midpoint of cf for points c(3,4) and f(9,8). Find MF Using our [URL='https://www.mathcelebrity.com/slope.php?xone=3&yone=4&slope=+2%2F5&xtwo=9&ytwo=8&pl=You+entered+2+points']line equation and midpoint calculator[/URL], we get: MF = [B](6, 6)[/B]

M is the set of integers that are greater than or equal to -1 and less than or equal to 2
M is the set of integers that are greater than or equal to -1 and less than or equal to 2 We include -1 on the left, and include 2 on the right [B]M = {-1, 0, 1, 1, 2)[/B]

M is the sum of a and its reciprocal
M is the sum of a and its reciprocal The reciprocal of a variable is 1 divided by the variable 1/a The sum of a and its reciprocal means we add: a + 1/a The phrase [I]is[/I] means an equation, so we set M equal to the sum of a + 1/a: [B]M = 1 + 1/a[/B]

m times the difference of 2p and 4r
m times the difference of 2p and 4r The difference of 2p and 4r: 2p - 4r m times the difference: [B]m(2p - 4r)[/B]

M/n = p-6 for m
M/n = p-6 for m Solve this literal equation by multiplying each side by n to isolate M: Mn/n = n(p - 6) Cancelling the n terms on the left side, we get: [B]M = n(p - 6)[/B]

m/x = k-6 for m
m/x = k-6 for m To solve this literal equation, multiply each side by x: x(m/x) = x(k - 6) The x's cancel on the left side, so we get: m = [B]x(k - 6)[/B]

m=u/k-r/k for k
m=u/k-r/k for k Multiply both sides by k to eliminate the k denominator: km = uk/k - rk/k Cancel the k's on the right side and we get km = u - r Divide each side by m: km/m = (u - r)/m Cancel the m on the left side: [B]k = (u - r)/m[/B]

m=y-b/x-t for y
m=y-b/x-t for y Add b/x + t to each side: m + b/x + t = y - b/x - t + b/x + t Cancel b/x terms and t terms on the right side to get: y = [B]m + b/x + t[/B]

Mackenzie baked 12 cookies with 2 scoops of flour. How many scoops of flour does Mackenzie need in o
Mackenzie baked 12 cookies with 2 scoops of flour. How many scoops of flour does Mackenzie need in order to bake 18 cookies? Assume the relationship is directly proportional. Set up a proportion of cookies to scoops with s as the number of scoops needed for 18 cookies: 12/2 = 18/s To solve for s, we [URL='https://www.mathcelebrity.com/prop.php?num1=12&num2=18&den1=2&den2=s&propsign=%3D&pl=Calculate+missing+proportion+value']type this proportion into our search engine[/URL] and we get: s = [B]3 [/B]

Madeline’s science quiz consists of 10 questions, all of which are true or false. How many different
Madeline’s science quiz consists of 10 questions, all of which are true or false. How many different choices for answering the 10 questions are possible? 2 ways of answering each True or False Question ^ (10 different ways to answer each question) 2^10 = [B]1,024 ways[/B]

Maggie earns $10 each hour she works at the pet store and $0.25 for each phone call she answers. Mag
Maggie earns $10 each hour she works at the pet store and $0.25 for each phone call she answers. Maggie answered 60 phone calls and earned $115 last week Set up an equation where c is the number of phone calls Maggie answers and h is the number of hours Maggie worked: 0.25c + 10h = 115 We're given c = 60, so we have: 0.25(60) + 10h = 115 15 + 10h = 115 We want to solve for h. So we[URL='https://www.mathcelebrity.com/1unk.php?num=15%2B10h%3D115&pl=Solve'] type this equation into our search engine[/URL] and we get: h = [B]10[/B]

maggie has two job offers. The first job offers to pay her $50 per week and 10 1/2 cents per flier.
maggie has two job offers. The first job offers to pay her $50 per week and 10 1/2 cents per flier. The second job offer will pay only $30 per week but gives 20 cents per flier. Write and solve an equation to find how many fliers must she deliver so that the two offers pay the same per week? Let the number of fliers be f. First job: 0.105f + 50 Second job: 20f + 30 Set them equal to each other: 0.105f + 50 = 20f + 30 [URL='https://www.mathcelebrity.com/1unk.php?num=0.105f%2B50%3D20f%2B30&pl=Solve']Typing this equation into our search engine[/URL], we get: [B]f = 1[/B]

Maggie is shopping for her friend's party. She has a budget of $40 to spend. She needs to get a bann
Maggie is shopping for her friend's party. She has a budget of $40 to spend. She needs to get a banner for $25 and candy necklaces that cost $1.25 each. Write an inequality for the budget. Let n be the necklaces. Since Maggie can spend [I]up to[/I] $40, we have the following inequality: [B]1.25n + 25 <=40 [/B] If you have to solve for n in the inequality, we [URL='https://www.mathcelebrity.com/1unk.php?num=1.25n%2B25%3C%3D40&pl=Solve']type it in our math engine[/URL] and we get: [B]n < = 12[/B]

Manuel can pay for his car insurance on a monthly basis, but if he pays an entire year's insurance i
Manuel can pay for his car insurance on a monthly basis, but if he pays an entire year's insurance in advance, he'll receive a $40 discount. His discounted bill for the year would then be $632. What is the monthly fee for his insurance? His full bill F, is denoted as: F - 40 = 632 [URL='https://www.mathcelebrity.com/1unk.php?num=f-40%3D632&pl=Solve']If we add 40 to each side[/URL], we get: F = [B]$672[/B]

MAPE - MPE - MAPD
Free MAPE - MPE - MAPD Calculator - Given a time series of actual and forecasted values, this determines the following:
* Mean Absolute Percentage Error (MAPE) also known as the Mean Absolute Percentage Deviation (MAPD)
* Symmetric Mean Absolute Percentage Error (sMAPE)
* Mean Absolute Percentage Error (MPE)

Marcela is having a presidential debate watching party with all of her friends, She will be making c
Marcela is having a presidential debate watching party with all of her friends, She will be making chicken wings and hot dogs. Each chicken wing costs $2 to make and each hot dog costs $3. She needs to spend at least $500. Marcela knows that she will make more than 50 chicken wings and hot dogs combined. She also knows that she will make less than 120 chicken wings and less that 100 hot dogs. What are her inequalities? Let c be the number of chicken wings and h be the number of hot dogs. Set up the given inequalities: [LIST=1] [*]c + h > 50 [I]Marcela knows that she will make more than 50 chicken wings and hot dogs combined.[/I] [*]2c + 3h >= 500 [I]She needs to spend at least $500[/I] [*]c < 120 [I]She also knows that she will make less than 120 chicken wings[/I] [*]h < 100 [I]and less that 100 hot dogs[/I] [/LIST]

Marco orders a large pizza, with a diameter of 14 inches. It is cut into 8 congruent pieces. what is
Marco orders a large pizza, with a diameter of 14 inches. It is cut into 8 congruent pieces. what is the area of one piece? A pizza is a circle. If the diameter of the pizza is 14 inches, the radius is 14/2 = 7 inches. Area of a circle is pi(r^2). With r = 7, we have: A =7^2(pi) A = 49pi Area of a slice of pizza is the area of the full pizza divided by 8 A(Slice) = [B]49pi/8[/B]

Marco takes 2 quizzes each week. Write an equation that shows the relationship between the number of
Marco takes 2 quizzes each week. Write an equation that shows the relationship between the number of weeks x and the total number of quizzes y. Write your answer as an equation with y first, followed by an equals sign. Our total quizzes equal 2 times the number of weeks (x): [B]y = 2x[/B]

Marcus bought w poster boards for $3 each
The total is $3 * w = 3w

Marcus drives a machine that paints lines along the highway. He needs to paint a line that is 9/10 o
Marcus drives a machine that paints lines along the highway. He needs to paint a line that is 9/10 of a mile long. He is 2/3 of the way done when he runs out of paint. What fraction of a mile has he painted? Marcus has painted 2/3 of 9/10. If we [URL='https://www.mathcelebrity.com/fraction.php?frac1=2%2F3&frac2=9%2F10&pl=Multiply']type 2/3 of 91/20 in our search engine[/URL], we get: [B]3/5[/B]

Margaret earns 15 an hour at her job. Each week she earns at least 450 and no more than 600. How man
Margaret earns 15 an hour at her job. Each week she earns at least 450 and no more than 600. How many hours does Margaret work each week? Let h be the hours worked We know that hourly rate * h equals total earnings. The phrases at least and no more than signify inequalities, so we have: 450 <= 15h <= 600 Divide each entry by 15: [B]30 <= h <= 40[/B] This means Margaret works at least 30 hours a week and no more than 40

Margin of Error from Confidence Interval
Free Margin of Error from Confidence Interval Calculator - Given a confidence interval, this determines the margin of error and sample mean.

Marginal propensity to save is 0.3. Calculate MPC
Marginal propensity to save is 0.3. Calculate MPC. MPC is Marginal Propensity to Consume. And MPS is Marginan Propensity to Save. The relational equation between the two is: MPC + MPS = 1 To get MPC, we have: MPC = 1 - MPS The problem gives us MPS = 0.3, so plug it into this modified MPC equation: MPC = 1 - 0.3 MPC = [B]0.7[/B]

Maria bought 7 boxes. A week later half of all her boxes were destroyed in a fire. There are now onl
Maria bought 7 boxes. A week later half of all her boxes were destroyed in a fire. There are now only 22 boxes left. With how many did she start? [U]Let x be the starting box number. We have:[/U] (x + 7)/2 = 22 [U]Cross multiply[/U] x + 7 = 44 [U]Subtract 7 from each side[/U] [B]x = 37[/B]

Maria bought seven boxes. A week later half of all her boxes were destroyed in a fire. There are now
Maria bought seven boxes. A week later half of all her boxes were destroyed in a fire. There are now only 22 boxes left. How many did she start with? Take this in parts [LIST=1] [*]Maria starts with b boxes. [*]She buys seven more. So she has b + 7 boxes [*]A week later, half of all her boxes are destroyed in a fire. Which means she's left with 1/2. (b + 7)/2 [*]Now she has 22 boxes. So we set (b + 7)/2 = 22 [/LIST] (b + 7)/2 = 22 Cross multiply: b + 7 = 22 * 2 b + 7 = 44 [URL='https://www.mathcelebrity.com/1unk.php?num=b%2B7%3D44&pl=Solve']Typing this equation into our search engine and solving for b[/URL], we get: [B]b = 37[/B]

Maria bought seven boxes. A week later half of all her boxes were destroyed in a fire. There are now
Maria bought seven boxes. A week later half of all her boxes were destroyed in a fire. There are now only 22 boxes left. With how many did she start? Let the number of boxes Maria started with be b. We're given the following pieces: [LIST] [*]She starts with b [*]She bought 7 boxes. So we add 7 to b: b + 7 [*]If half the boxes were destroyed, she's left with 1/2. So we divide (b + 7)/2 [*]Only 22 boxes left means we set (b + 7)/2 equal to 22 [/LIST] (b + 7)/2 = 22 Cross multiply: b + 7 = 22 * 2 b + 7 = 44 [URL='https://www.mathcelebrity.com/1unk.php?num=b%2B7%3D44&pl=Solve']Type this equation into our search engine[/URL] to solve for b and we get: b = [B]37[/B]

Maria called her sister long distance on Wednesday. The first 5 minutes cost $3, and each minute aft
Maria called her sister long distance on Wednesday. The first 5 minutes cost $3, and each minute after that cost $0.25. How much did it cost if they talked for 15 minutes? First 5 minutes: $3 If they talked 15 minutes, the additional charge past 5 minutes is: 0.25 * (15 - 5) 0.25 * 10 minutes = $2.5 We add this to the first 5 minutes: $3 + $2.5 = [B]$5.50[/B]

Maria is saving money to buy a bike that cost 133$. She has 42$ and will save an additional 7 each w
Maria is saving money to buy a bike that cost 133$. She has 42$ and will save an additional 7 each week. Set up an equation with w as the number of weeks. We want to find w such that: 7w + 42 = 133 [URL='https://www.mathcelebrity.com/1unk.php?num=7w%2B42%3D133&pl=Solve']Typing this equation into our search engine[/URL], we get: w = [B]13[/B]

Maria leaves her house and runs west for 6 m miles. She then turns North and runs 5 miles. Maria the
Maria leaves her house and runs west for 6 miles. She then turns North and runs 5 miles. Maria then travels east for 7 miles and then south for 5 miles. How far is Maria from her house now? Maria traveled the same distance north and south of 5 miles. These cancel each other out. Her 7 mile eastern trip compared to the 6 mile west trip represents a net difference of [B]1 mile[/B]

Maria paid a fee of $75 to the local golf course. For every 18 holes of goals it was $12.50. How muc
Maria paid a fee of $75 to the local golf course. For every 18 holes of goals it was $12.50. How much did Maria pay to play 36 holes of golf 36 holes is 2 * 18 hole rounds. [U]Maria's total cost for the trip is:[/U] Maria's Fee = Local Golf Course Fee + 2(18 hole round fee) Maria's Fee = 75 + 2(12.50) Maria's Fee = 75 +25 Maria's Fee = [B]$100[/B]

Maria runs each lap in 5 minutes. She will run less than 7 laps today. What are the possible numbers
Maria runs each lap in 5 minutes. She will run less than 7 laps today. What are the possible numbers of minutes she will run today? Total Time < Laps * minutes per laps Total Time < 7 * 5 [B]Total Time < 35[/B]

Marion Middle School has 600 students. Mike surveys a random sample of 30 students and finds that 7
Marion Middle School has 600 students. Mike surveys a random sample of 30 students and finds that 7 of them play a musical instrument. How many students at the school are likely to play a musical instrument? Set up a proportion of those that play musical instruments to total students where m is the amount of students in the 600 who play a musical instrument: 7/30 = m/600 [URL='https://www.mathcelebrity.com/prop.php?num1=7&num2=m&den1=30&den2=600&propsign=%3D&pl=Calculate+missing+proportion+value']Typing this proportion into our search engine[/URL], we get: m = [B]140[/B]

Marissa has 24 coins in quarters and nickels. She has 3 dollars. How many of the coins are quarters?
Let n be the number of nickels and q be the number of quarters. We have two equations: (1) n + q = 24 (2) 0.05n + 0.25q = 3 Rearrange (1) to solve for n in terms of q for another equation (3) (3) n = 24 - q Plug (3) into (2) 0.05(24 - q) + 0.25q = 3 Multiply through: 1.2 - 0.05q + 0.25q = 3 Combine q terms 0.2q + 1.2 = 3 Subtract 1.2 from each side: 0.2q = 1.8 Divide each side by 0.2 [B]q = 9[/B]

Marita's nose is 2 inches long and her head is 9 inches tall. Assume Mount Rushmore was carved using
Marita's nose is 2 inches long and her head is 9 inches tall. Assume Mount Rushmore was carved using the same ratio. If Teddy Roosevelt's head is 60 feet tall, how long should his nose be? Round to the nearest foot, if necessary. Set up a proportion/ratio of head height to nose height where n is the nose height for 60 feet head height: 9/2 = 60/n [U]Using our [URL='https://www.mathcelebrity.com/prop.php?num1=9&num2=60&den1=2&den2=n&propsign=%3D&pl=Calculate+missing+proportion+value']proportion calculator[/URL], we see that:[/U] n = [B]13.33 rounded to the nearest foot is 13 feet[/B]

Mark and Jennie are bowling. Jennie’s score is double Mark’s score. If the sum of their score is 171
Mark and Jennie are bowling. Jennie’s score is double Mark’s score. If the sum of their score is 171, find each person’s score by writing out an equation. Let Mark's score be m. Let Jennie's score be j. We're given two equations: [LIST=1] [*]j = 2m [*]j + m = 171 [/LIST] Substitute equation (1) into equation (2): 2m + m = 171 [URL='https://www.mathcelebrity.com/1unk.php?num=2m%2Bm%3D171&pl=Solve']Type this equation into our search engine[/URL] to solve for m: m = [B]57 [/B] To solve for j, we substitute m = 57 in equation (1) above: j = 2(57) j = [B]114[/B]

mark has a drawer full of 12 yellow baseball caps and 18 white baseball caps. What is the probabilit
mark has a drawer full of 12 yellow baseball caps and 18 white baseball caps. What is the probability of the next cap he chooses at random will be yellow? P(yellow) = yellow caps / Total caps P(yellow) = 12/(12 + 18) P(yellow) = 12/30 [URL='https://www.mathcelebrity.com/fraction.php?frac1=12.%2F30&frac2=3%2F8&pl=Simplify']Simplifying this fraction,[/URL] we get: P(yellow) = [B]2/5[/B]

Markup Markdown
Free Markup Markdown Calculator - Given the 3 items of a markup word problem, cost, markup percentage, and sale price, this solves for any one of the three given two of the items. This works as a markup calculator, markdown calculator.

Marla wants to rent a bike Green Lake Park has an entrance fee of $8 and charges $2 per hour for bik
Marla wants to rent a bike Green Lake Park has an entrance fee of $8 and charges $2 per hour for bike Oak Park has an entrance fee of $2 and charges $5 per hour for bike rentals she wants to know how many hours are friend will make the costs equal [U]Green Lake Park: Set up the cost function C(h) where h is the number of hours[/U] C(h) = Hourly Rental Rate * h + Entrance Fee C(h) = 2h + 8 [U]Oak Park: Set up the cost function C(h) where h is the number of hours[/U] C(h) = Hourly Rental Rate * h + Entrance Fee C(h) = 5h + 2 [U]Marla wants to know how many hours make the cost equal, so we set Green Lake Park's cost function equal to Oak Parks's cost function:[/U] 2h + 8 = 5h + 2 To solve for h, we [URL='https://www.mathcelebrity.com/1unk.php?num=2h%2B8%3D5h%2B2&pl=Solve']type this equation into our search engine[/URL] and we get: h = [B]2[/B]

Martha is 18 years older than Harry. Their ages add to 106. Write an equation and solve it to find t
Martha is 18 years older than Harry. Their ages add to 106. Write an equation and solve it to find the ages of Martha and Harry. Let m be Martha's age. Let h be Harry's age. We're given two equations: [LIST=1] [*]m = h + 18 [I](older means we add)[/I] [*]h + m = 106 [/LIST] Substitute equation (1) into equation (2) for m: h + h + 18 = 106 To solve for h, [URL='https://www.mathcelebrity.com/1unk.php?num=h%2Bh%2B18%3D106&pl=Solve']we type this equation into our search engine[/URL] and we get: h = [B]44[/B]

Martha's age 2/3 of her brother's age. Martha is 24 years old now. How old is her brother?
Martha's age 2/3 of her brother's age. Martha is 24 years old now. How old is her brother? Let her brother's age be b. We're given: 2b/3 = 24 To solve this proportion for b, [URL='https://www.mathcelebrity.com/prop.php?num1=2b&num2=24&den1=3&den2=1&propsign=%3D&pl=Calculate+missing+proportion+value']we type it in our search engine[/URL] and we get: b = [B]36[/B]

Martin buys b books at £10 each what is the total cost
Martin buys b books at £10 each what is the total cost Total Cost = Price * Quantity Total Cost = £10 * b Total Cost = [B]£10b[/B]

Marty is 3 years younger than 6 times his friend Warrens age. The sum of their ages is greater than
Marty is 3 years younger than 6 times his friend Warrens age. The sum of their ages is greater than 11. What is the youngest age Warren can be? Let m be Marty's age and w be Warren's age. We have two equations: (1) m = 6w - 3 (2) m + w > 11 Plug (1) into (2) 6w - 3 + w > 11 Combine w terms 7w - 3 > 11 Add 3 to each side 7w > 14 Divide each side by 7 w > 2 which means [B]w = 3[/B] as the youngest age.

Mary invested $800, part at 9% per annum and the rest at 12% per annum. After 1 year, the total inte
Mary invested $800, part at 9% per annum and the rest at 12% per annum. After 1 year, the total interest earned was $79.50. How much did she invest at each rate? Using our [URL='https://www.mathcelebrity.com/split-fund-interest-calculator.php?p=800&i1=9&i2=12&itot=79.50&pl=Calculate']split fund calculator[/URL], we get: [LIST] [*]Fund 1: $550 [*]Fund 2: $250 [/LIST]

Mary owns a store that sells computers. Her profit in dollars is represented by the function P(x) =
Mary owns a store that sells computers. Her profit in dollars is represented by the function P(x) = x^3 - 22x^2 - 240x, where x is the number of computers sold. Mary hopes to make a profit of at least $10,000 by the time she sells 36 computers. Explain whether Mary will meet her goal. Justify your reasoning. Calculate P(10): P(10) = 10^3 - 22(10)^2 - 240(10) P(10) = 1000 - 2200 - 2400 P(10) = -3600 Mary will [B]not[/B] meet her goal of making a profit of at least $10,000 when she sells 36 computers because her profit is in the negative.

Mary paid 1.97 for toothpaste and a bar of soap using a discount coupon if the toothpaste cost 1.29
Mary paid 1.97 for toothpaste and a bar of soap using a discount coupon if the toothpaste cost 1.29 and the song cost 83 cents. What is the value of the discount coupon? Find the full price package: 1.29 + 0.83 = 2.12 The value of the discount coupon is the money off, so: 2.12 - 1.97 = [B]0.15[/B]

Mary spent a total of $291.94 for a party. She spent $200.29 on food, plus an additional $30.55 for
Mary spent a total of $291.94 for a party. She spent $200.29 on food, plus an additional $30.55 for each hour of the party. How long was the party? First, figure out the remaining cost after food: 291.94 -200.29 = 91.65 91.65 / 30.55 per hour = 3 hours

Mary went bowling on the weekend. Each game cost $2.50, and the shoe rental $2.00. She spent $14.50
Mary went bowling on the weekend. Each game cost $2.50, and the shoe rental $2.00. She spent $14.50 total. How many games did she bowl? Set up the equation where g is the number of games. We add the shoe rental fee to the cost per games 2.5g + 2 = 14.50 To solve for g, we [URL='https://www.mathcelebrity.com/1unk.php?num=2.5g%2B2%3D14.50&pl=Solve']type this equation into our search engine[/URL] and we get: g = [B]5[/B]

Match each variable with a variable by placing the correct letter on each line.
Match each variable with a variable by placing the correct letter on each line. a) principal b) interest c) interest rate d) term/time 2 years 1.5% $995 $29.85 [B]Principal is $995 Interest is $29.85 since 995 * .0.15 * 2 = 29.85 Interest rate is 1.5% Term/time is 2 year[/B]s

Math Problem Library
Contains word problems and other long form problems with step-by-step solutions.

Math Problem Solving (Help Please)
A box in the shape of a rectangular prism is used in a movie scene. The base of the box measures 6 feet by 5 feet. The box has a volume of 195 cubic feet. The director hires an actor who is 6 feet 4 inches tall. Can the actor stand up straight in the box? Also I do need to show my work so please write down the work to solve this. Thanks!

Math Problem Solving (Help Please)
Volume of rectangular prism is: V = lwh Plugging in the numbers you gave: 195 = (6)(5)h 195 = 30h Divide each side by 30 h = 6.5 6.5 feet is 6 feet, 6 inches. This is 2 inches more than your actor, so [B]yes[/B], he will fit in the box standing up.

Math Written Assignment
Im sorta confused about this question? He has decided to remove all the old sod (grass), bring in a new 4 inch layer of topsoil, install new in-ground sprinklers, and reseed the lawn. He seems to think that he'll be able to save money by hauling loads of topsoil from the store himself in his pickup truck, rather than paying for delivery, but I don't think he's right. You're going to help us settle this. Here is (most of) the information you asked for: [LIST] [*]Is he redoing the whole yard or just the front? He's redoing the whole yard [*]How much topsoil does he need? I'm not sure, you'll have to figure that out. Remember he's putting a new 4 inch layer down over all the area currently covered by grass in the overhead picture above. [*]How big is the yard? I'm not sure, but you can probably estimate it using the overhead picture. [*]What kind of pickup truck does he drive? A 2003 Ford F-150 XL. [*]How much can the pickup carry? The truck bed is 80 inches long, 69 inches wide, and 20 inches tall. [*]How much is the delivery charge? $30 per truckload on top of the soil cost. Each truckload can deliver up to 18 cubic yards. [*]How much does the topsoil cost? $18 per cubic yard (sold in 1/4 yard increments). [*]How far is the soil store? It is 9 miles away. It takes about 20 minutes to drive there. [*]What gas mileage does the pickup truck get? It averages 17 miles to the gallon. [*]What is the current gas cost? Assume it's $3.79/gallon. [/LIST] Using this information, figure out whether my neighbor will save money by picking up the soil himself. Use the results of your calculations to guide your decision: would you recommend that my neighbor pick up the soil himself, or pay for delivery? Detail all your assumptions and calculations, and clearly write out your final conclusions.

Math Written Assignment
Do you have a picture? I need the measurements to calculate volume.

Math Written Assignment
[QUOTE="math_celebrity, post: 1040, member: 1"]Do you have a picture? I need the measurements to calculate volume.[/QUOTE] Yes here u go

Math Written Assignment
Do we have to subtract out the space covered by the house?

Math Written Assignment
[QUOTE="math_celebrity, post: 1046, member: 1"]Do we have to subtract out the space covered by the house?[/QUOTE] I think so

Math Written Assignment
The truck bed is 80 inches long, 69 inches wide, and 20 inches tall. So the total volume the truck can carry is: 80 x 69 x 20 = 110,400 cubic inches can be carried each time. Find out how many gallons in a full tank for the 2003 Ford F150. Then you calculate the amount of miles you can drive on a full trip.

Mathematical Constants and Identities
Free Mathematical Constants and Identities Calculator - Calculates and explains various mathematical constants such as:
* Gelfonds (Gelfond's) Constant
* Eulers Constant


Matilda needs at least $112 to buy an new dress. She has already saved $40. She earns $9 an hour bab
Matilda needs at least $112 to buy an new dress. She has already saved $40. She earns $9 an hour babysitting. Write and solve and inequality to find how many hours she will need to babysit to buy the dress. Subtract remaining amount needed after savings: 112 - 40 = 72 Let h be her hourly wages for babysitting. We have the equation: [B]9h = 72[/B] Divide each side by 9 [B]h = 8[/B]

Matrix Properties
Free Matrix Properties Calculator - Given a matrix |A|, this calculates the following items if they exist:
* Determinant = det(A)
* Inverse = A-1
* Transpose = AT
* Adjoint = adj(A)
* Eigen equation (characteristic polynomial) = det|λI - A|
* Trace = tr(A)
* Gauss-Jordan Elimination using Row Echelon and Reduced Row Echelon Form
* Dimensions of |A| m x n
* Order of a matrix
* Euclidean Norm ||A||
* Magic Sum if it exists
* Determines if |A| is an Exchange Matrix

Matt has $100 dollars in a checking account and deposits $20 per month. Ben has $80 in a checking ac
Matt has $100 dollars in a checking account and deposits $20 per month. Ben has $80 in a checking account and deposits $30 per month. Will the accounts ever be the same balance? explain Set up the Balance account B(m), where m is the number of months since the deposit. Matt: B(m) = 20m + 100 Ben: B(m) = 80 + 30m Set both balance equations equal to each other to see if they ever have the same balance: 20m + 100 = 80 + 30m To solve for m, [URL='https://www.mathcelebrity.com/1unk.php?num=20m%2B100%3D80%2B30m&pl=Solve']we type this equation into our search engine[/URL] and we get: m = [B]2 So yes, they will have the same balance at m = 2[/B]

Matthew has $3,000 in a savings account that earns 10% interest per year. How much will he have in 3
Matthew has $3,000 in a savings account that earns 10% interest per year. How much will he have in 3 years? Using our [URL='https://www.mathcelebrity.com/compoundint.php?bal=3000&nval=3&int=10&pl=Annually']compound interest with balance calculator[/URL], we get: [B]$3,993[/B]

Matthew works 45 hours at $22.10 per hour and 3 hours overtime at double time. Calculate his total e
Matthew works 45 hours at $22.10 per hour and 3 hours overtime at double time. Calculate his total earnings per week. If Matthew gets 3 hours overtime, then his regular time is 45 - 3 = 42 [U]Calculate regular hours earnings:[/U] Regular hours earnings = Hourly Rate * Regular hours worked Regular hours earnings = 22.10 * 42 Regular hours earnings = 928.20 [U]Calculate overtime hours earnings:[/U] Double time = twice the regular hourly ratre Overtime hours earnings = Hourly Rate * 2 * Overtime hours worked Overtime hours earnings = 22.10 * 2 * 3 Overtime hours earnings = 132.60 [U]Calculate total earnings:[/U] Total earnings = Regular hours earnings + Overtime hours earnings Total earnings = 928.20 + 132.60 Total earnings = [B]$1,060.80[/B]

Matthew's cat weighs 10 pounds more than his pet hamster. His dog weighs the same as his cat. If the
Matthew's cat weighs 10 pounds more than his pet hamster. His dog weighs the same as his cat. If the weight of all three pets is 35 pounds, ow much does his hamster weigh? Setup weights and relations: [LIST] [*]Hamster weight: w [*]Cat weight: w + 10 [*]Dog weight:w + 10 [/LIST] Add all the weights up: w + w + 10 + w + 10 = 35 Solve for [I]w[/I] in the equation w + w + 10 + w + 10 = 35 [SIZE=5][B]Step 1: Group the w terms on the left hand side:[/B][/SIZE] (1 + 1 + 1)w = 3w [SIZE=5][B]Step 2: Group the constant terms on the left hand side:[/B][/SIZE] 10 + 10 = 20 [SIZE=5][B]Step 3: Form modified equation[/B][/SIZE] 3w + 20 = + 35 [SIZE=5][B]Step 4: Group constants:[/B][/SIZE] We need to group our constants 20 and 35. To do that, we subtract 20 from both sides 3w + 20 - 20 = 35 - 20 [SIZE=5][B]Step 5: Cancel 20 on the left side:[/B][/SIZE] 3w = 15 [SIZE=5][B]Step 6: Divide each side of the equation by 3[/B][/SIZE] 3w/3 = 15/3 w =[B] 5[/B] [B] [URL='https://www.mathcelebrity.com/1unk.php?num=w%2Bw%2B10%2Bw%2B10%3D35&pl=Solve']Source[/URL][/B]

Matthew's pay increases by 20% each month. If his first pay is $450, determine the amount of his pay
Matthew's pay increases by 20% each month. If his first pay is $450, determine the amount of his pay in month 5. Let me be the number of months. We have a pay functionalists P(m) as: P(m) = Initial Pay * (1 + Increase %/100)^m With m = 5, initial pay = 450, and Increase % = 20, we have P(5) = 450 * (1.2)^5 P(5) = 450 * 2.48832 P(5) = [B]1,119.74[/B]

Max and Bob went to the store. Max bought 2 burgers and 2 drinks for $5.00 bob bought 3 burgers and
Max and Bob went to the store. Max bought 2 burgers and 2 drinks for $5.00. Bob bought 3 burgers and 1 drink for $5.50. How much is each burger and drink? [U]Set up the givens where b is the cost of a burger and d is the cost of a drink:[/U] Max: 2b + 2d = 5 Bob: 3b + d = 5.50 [U]Rearrange Bob's equation by subtracting 3b from each side[/U] (3) d = 5.50 - 3b [U]Now substitute that d equation back into Max's Equation[/U] 2b + 2(5.50 - 3b) = 5 2b + 11 - 6b = 5 [U]Combine b terms:[/U] -4b + 11 = 5 [U]Subtract 11 from each side[/U] -4b = -6 [U]Divide each side by -4[/U] b = 3/2 [B]b = $1.50[/B] [U]Now plug that back into equation (3):[/U] d = 5.50 - 3(1.50) d = 5.50 - 4.50 [B]d = $1.00[/B]

Max is 23 years younger than his father.Together their ages add up to 81.
Max is 23 years younger than his father.Together their ages add up to 81. Let Max's age be m, and his fathers' age be f. We're given: [LIST=1] [*]m = f - 23 <-- younger means less [*]m + f = 81 [/LIST] Substitute Equation (1) into (2): (f - 23) + f = 81 Combine like terms to form the equation below: 2f - 23 = 81 [URL='https://www.mathcelebrity.com/1unk.php?num=2f-23%3D81&pl=Solve']Typing this equation into our search engine[/URL], we get: [B]f = 52[/B] Substitute this into Equation (1): m = 52 - 23 [B]m = 29[/B]

Max sneezes every 5 minutes, Lina coughs every 6 minutes, and their dog barks every 3 minutes. If th
Max sneezes every 5 minutes, Lina coughs every 6 minutes, and their dog barks every 3 minutes. If there was sneezing, barking, and coughing at 3:15 PM, when is the next time that these three sounds will happen simultaneously? To find the next time the sounds happen simultaneously, we want to find the Least Common Multiple (LCM). [URL='https://www.mathcelebrity.com/gcflcm.php?num1=3&num2=5&num3=6&pl=LCM']Using our LCM Calculator[/URL], we find the least common multiple of 3, 5, and 6 is 30. The least common multiple gives us a common time where each sound reaches a "cycle". [LIST] [*]Dog: A bark every e minutes means the dog has 10 barks, with the 10th bark at 30 minutes after 3:15 [*]Max: A sneeze every 5 minutes means he has 6 sneezes, with the 6th sneeze at 30 minutes after 3:15 [*]Lisa: A cough every 6 minutes means she has 5 coughs, with the 5th cough at 30 minutes after 3:15 [/LIST] 30 minutes after 3:15 means we have: 3:15 + 30 = [B]3:45 PM[/B]

Max's age is 2 more than his fathers age divided by 4. Max is 13 years old. How old is his dad?
Max's age is 2 more than his fathers age divided by 4. Max is 13 years old. How old is his dad? Let Max's father be age f. We're given: (f + 2)/4 = 13 Cross Multiply: f + 2 = 52 [URL='https://www.mathcelebrity.com/1unk.php?num=f%2B2%3D52&pl=Solve']Typing this equation into the search engine[/URL], we get: f = [B]50[/B]

Mcnemar Test
Free Mcnemar Test Calculator - Given a 2 x 2 contingency table and a significance level, this will determine the test statistic, critical value, and hypothesis conclusion using a Mcnemar test.

mcubemultipliedbyntothefourthpower
mcubemultipliedbyntothefourthpower m cubed means we raise m to the 3rd power: m^3 n to the fourth power: n^4 Multiply both expressions together: [B]m^3n^4[/B]

Megan has $50 and saves $5.50 each week. Connor has $18.50 and saves $7.75 each week. After how many
Megan has $50 and saves $5.50 each week. Connor has $18.50 and saves $7.75 each week. After how many weeks will megan and connor have saved the same amount [U]Set up the Balance function B(w) where w is the number of weeks for Megan:[/U] B(w) = savings per week * w + Current Balance B(w) = 5.50w + 50 [U]Set up the Balance function B(w) where w is the number of weeks for Connor:[/U] B(w) = savings per week * w + Current Balance B(w) = 7.75w + 18.50 The problem asks for w when both B(w) are equal. So we set both B(w) equations equal to each other: 5.50w + 50 = 7.75w + 18.50 To solve this equation for w, we[URL='https://www.mathcelebrity.com/1unk.php?num=5.50w%2B50%3D7.75w%2B18.50&pl=Solve'] type it in our search engine[/URL] and we get: w = [B]14[/B]

Melinda is paid 17000 per year. She is also paid a sales commission of 5% of the value of her sales.
Melinda is paid 17000 per year. She is also paid a sales commission of 5% of the value of her sales. Last year she sold 344000 worth of products. What percent of her total income was her commission? Calculate Melinda's commission: 344,000 * 0.05 = 17,200 Calculate her total income for the year Total Income = Base Pay + Commission Total Income = 17,000 + 17,200 Total Income = 34,200 Calculate the percent of her income which is commission: Commission Income Percent = 100 * 17,200/34,200 Commission Income Percent = 100 * 0.5029 [B]Commission Income Percent = 50.29%[/B]

Melissa runs a landscaping business. She has equipment and fuel expenses of $264 per month. If she c
Melissa runs a landscaping business. She has equipment and fuel expenses of $264 per month. If she charges $53 for each lawn, how many lawns must she service to make a profit of at $800 a month? Melissa has a fixed cost of $264 per month in fuel. No variable cost is given. Our cost function is: C(x) = Fixed Cost + Variable Cost. With variable cost of 0, we have: C(x) = 264 The revenue per lawn is 53. So R(x) = 53x where x is the number of lawns. Now, profit is Revenue - Cost. Our profit function is: P(x) = 53x - 264 To make a profit of $800 per month, we set P(x) = 800. 53x - 264 = 800 Using our [URL='http://www.mathcelebrity.com/1unk.php?num=53x-264%3D800&pl=Solve']equation solver[/URL], we get: [B]x ~ 21 lawns[/B]

Men's heights are normally distributed with mean 69.0 inches and standard deviation 2.8 inches. Mimi
Men's heights are normally distributed with mean 69.0 inches and standard deviation 2.8 inches. Mimi is designing a plane with a height that allows 95% of the men to stand straight without bending in the plane. What is the minimum height of the plane? Using the [URL='http://www.mathcelebrity.com/probnormdist.php?xone=50&mean=69&stdev=2.8&n=1&pl=Empirical+Rule']empirical rule calculator[/URL], we have a [B]63.4[/B] minimum height.

Mental Models of Math Book
Calculation Domination: How Anybody Can Explode Their Math Scores Using the Mental Magic of Elon Musk and Warren Buffett. [MEDIA=youtube]RclG-k6itpk[/MEDIA] This audiobook shows you various mental models of the top 5% of math students. Mental models come from the following disciplines: Math Anxiety Science Problem Solving Probability Decision Making Metaphysics Persuasion Math Testing

Meryl can only take 4 out of 7 classes offered during the summer. How many different ways can she ch
Meryl can only take 4 out of 7 classes offered during the summer. How many different ways can she choose the classes she will take We want 7 choose 4, or 7C4: We [URL='https://www.mathcelebrity.com/permutation.php?num=7&den=4&pl=Combinations']type 7C4 into our search engine and we get[/URL]: 35

Method of Equated Time-Exact Method-Macaulay Duration-Volatility
Free Method of Equated Time-Exact Method-Macaulay Duration-Volatility Calculator - Given a set of cash flows at certain times, and a discount rate, this will calculate t using the equated time method and the exact method, as well as the macaulay duration and volatility

Michael invited 30 of his friends to his part and a third of guest arrived late how many arrived on
Michael invited 30 of his friends to his part and a third of guest arrived late how many arrived on time If 1/3 arrived late, then [URL='https://www.mathcelebrity.com/fraction.php?frac1=1&frac2=1%2F3&pl=Subtract']1 - 1/3[/URL] = 2/3 arrived on time Guests who arrived on time = 2/3 of 30 [URL='https://www.mathcelebrity.com/fraction.php?frac1=30&frac2=2/3&pl=Multiply']Guests who arrived on time[/URL] = [B]20[/B]

Michael is riding his bicycle. He rides 25.6 kilometers in 4 hours. What is his speed?
We need the speed of KM per hour. 25.6 km / 4 hours [U]Divide top and bottom by 4 to get km per hour[/U] [B]6.4km per hour[/B]

Michelle and Julie sold 65 cupcakes. If Julie sold 9 more cupcakes than Michelle, how many cupcakes
Michelle and Julie sold 65 cupcakes. If Julie sold 9 more cupcakes than Michelle, how many cupcakes did each of them sell? Let m = Michelle's cupcakes and j = Julie's cupcakes. We have two equations: m + j = 65 j = m + 9 Substituting, we get: m + (m + 9) = 65 Combine like terms, we get: 2m + 9 = 65 Subtract 9 from each side: 2m = 56 Divide each side by 2 to isolate m m = 28 If m = 28, then j = 28 + 9 = 37 So (m, j) = (28, 37)

Michelle and Natalie both went out to eat at a new restaurant. Michelle’s bill was $22.50, and she l
Michelle and Natalie both went out to eat at a new restaurant. Michelle’s bill was $22.50, and she left a 15% tip. Natalies bill was $24.25, and she left a 10% tip. Whose total bill was the greatest? [U]Michelles's total bill:[/U] Total Bill = Pre-Tax Bill * (1 + tax rate) Since 15% = 0.15, we have: Total Bill = 22.50 * 1.15 Total Bill = 25.88 [U]Natalie's total bill:[/U] Total Bill = Pre-Tax Bill * (1 + tax rate) Since 10% = 0.10, we have: Total Bill = 24.25 * 1.10 Total Bill = 26.68 [B]Natalie's[/B] total bill was the greatest.

Michelle can paint one car in 2 hours. It takes Tyler 3 hours to paint the same car while Colton tak
Michelle can paint one car in 2 hours. It takes Tyler 3 hours to paint the same car while Colton takes 6 hours to paint the car. If they all work together, how long will it take them to paint the car? Setup unit rates: [LIST] [*]Michelle can paint 1/2 of the car in one hour [*]Tyler can paint 1/3 of the car in one hour [*]Colton can paint 1/6 of the car in one hour [/LIST] In one hour using a combined effort, they can paint: 1/2 + 1/3 + 1/6 = 6/6 = 1 car in [B]one hour[/B].

Midpoint formula
Midpoint formula Given two points (x1, y1) and (x2, y2), the midpoint is found as the average distance between the 2 points: [LIST] [*]x value is: (x1 + x2)/2 [*]y value is: (y1 + y2)/2 [/LIST] So our midpoint is: ((x1 + x2)/2, (y1 + y2)/2)

Miguel has $80 in his bank and saves $2 a week. Jesse has $30 in his bank but saves $7 a week. In ho
Miguel has $80 in his bank and saves $2 a week. Jesse has $30 in his bank but saves $7 a week. In how many weeks will Jesse have more in his bank than Miguel? [U]Set up the Bank value B(w) for Miguel where w is the number of weeks[/U] B(w) = Savings Per week * w + Current Bank Balance B(w) = 2w + 80 [U]Set up the Bank value B(w) for Jesse where w is the number of weeks[/U] B(w) = Savings Per week * w + Current Bank Balance B(w) = 7w + 30 The problem asks when Jesse's account will be more than Miguel's. So we set up an inequality where: 7w + 30 > 2w + 80 To solve this inequality, we [URL='https://www.mathcelebrity.com/1unk.php?num=7w%2B30%3E2w%2B80&pl=Solve']type it in our search engine[/URL] and we get: [B]w > 10[/B]

Mike cut 2 acres of grass in 30 minutes on his tractor. Which proportion would determine how many ac
Mike cut 2 acres of grass in 30 minutes on his tractor. Which proportion would determine how many acres of grass Mike cut in 60 minutes? Let a be the number of acres of grass cut by Mike in 60 minutes. We have the following proportion: 2/30 = a/60 [URL='https://www.mathcelebrity.com/prop.php?num1=2&num2=a&den1=30&den2=60&propsign=%3D&pl=Calculate+missing+proportion+value']Typing this problem into our search engine[/URL], we get [B]a = 4[/B].

Mike received a penny on December 1st. On December 2nd he received 2 cents. December 3rd another 4 c
Mike received a penny on December 1st. On December 2nd he received 2 cents. December 3rd another 4 cents and December 4th he received 8 cents. If his money continues to double, how much will he earn on December 25th? We have 24 doubling times starting December 2 to December 25 0.01 * 2^24 0.01 * 16,777,216 [B]167,772.16[/B]

mike went to canalside with $40 to spend. he rented skates for $10 and paid $3 per hour to skate.wha
mike went to canalside with $40 to spend. he rented skates for $10 and paid $3 per hour to skate.what is the greatest number of hours Mike could have skated? Let h be the number of hours of skating. We have the cost function C(h): C(h) = Hourly skating rate * h + rental fee C(h) = 3h + 10 The problem asks for h when C(h) = 40: 3h + 10 = 40 To solve this equation for h, we [URL='https://www.mathcelebrity.com/1unk.php?num=3h%2B10%3D40&pl=Solve']type it in our search engine[/URL] and we get: h = [B]10[/B]

Mike works in a toy store. One week, he worked 38 hours and made $220. The next week, he received a
Mike works in a toy store. One week, he worked 38 hours and made $220. The next week, he received a raise, so when he worked 30 hours he made $180. How much was his raise (to the nearest cent)? First week, Mike earns the following in hours (h) 38h = 220 h = 5.79 [URL='https://www.mathcelebrity.com/1unk.php?num=38h%3D220&pl=Solve']using our equation calculator[/URL] We call this his old hourly salary Next week, Mike earns the following in hours (h) 30h = 180 h = 6 [URL='https://www.mathcelebrity.com/1unk.php?num=30h%3D180&pl=Solve']using our equation calculator[/URL] We call this his new hourly salary His raise is the difference between his current hourly salary and his old hourly salary: Raise = New Hourly Salary - Old Hourly Salary Raise = 6 - 5.79 Raise = [B]$0.21[/B] Mike got a 21 cent hourly raise

Mike writes a book and gets 15% royalty of total sales. He sells 50,000 books at a cost of $35 per b
Mike writes a book and gets 15% royalty of total sales. He sells 50,000 books at a cost of $35 per book. What is the royalty he receives? Remember to put the $ symbol in your answer. For example, if your answer is 10 dollars, write $10 in the answer box. [U]Calculate total sales:[/U] Total Sales = Number of Books * Price per book Total Sales = 50,000 * $35 Total Sales = $1,750,000 [U]Now calculate Mike's royalties:[/U] Royalties = Total Sales * Royalty Percent Royalties = $1,750,000 * 15% [URL='https://www.mathcelebrity.com/perc.php?num=+5&den=+8&num1=+16&pct1=+80&pct2=15&den1=1750000&pcheck=3&pct=+82&decimal=+65.236&astart=+12&aend=+20&wp1=20&wp2=30&pl=Calculate']Royalties[/URL] = [B]$262,500[/B]

Milan plans to watch 2 movies each month. Write an equation to represent the total number of movies
Milan plans to watch 2 movies each month. Write an equation to represent the total number of movies n that he will watch in m months. Number of movies equals movies per month times the number of months. So we have: [B]n = 2m[/B]

Military Phonetic Alphabet
Free Military Phonetic Alphabet Calculator - Translates a statement or phrase into the military phonetic alphabet

Mimi just started her tennis class three weeks ago. On average, she is able to return 20% of her opp
Mimi just started her tennis class three weeks ago. On average, she is able to return 20% of her opponent's serves. Assume her opponent serves 8 times. Show all work. Let X be the number of returns that Mimi gets. As we know, the distribution of X is a binomial probability distribution. a) What is the number of trials (n), probability of successes (p) and probability of failures (q), respectively? b) Find the probability that that she returns at least 1 of the 8 serves from her opponent. (c) How many serves can she expect to return? a) [B]n = 8 p = 0.2[/B] q = 1 - p q = 1 - 0.2 [B]q = 0.8 [/B] b) [B]0.4967[/B] on our [URL='http://www.mathcelebrity.com/binomial.php?n=+8&p=0.2&k=1&t=+5&pl=P%28X+>+k%29']binomial calculator[/URL] c) np = 8(0.2) = 1.6 ~ [B]2[/B] using the link above

Mimis math class starts at 9:00 A.M. and lasts for 1 hour and 55 minutes. After math class, Mimi has
Mimis math class starts at 9:00 A.M. and lasts for 1 hour and 55 minutes. After math class, Mimi has recess for 15 minutes. What time does Mimis recess end? [LIST=1] [*]Start at 9:00 AM [*]1 hour and 55 minutes of class puts us at 10:55 AM [*]Recess for 15 minutes puts us at [B]11:10 AM[/B] [/LIST] [B][/B] [LIST=1] [*]Another way to do this is work in whole hours and minute blocks [*]9:00 AM, add 1 hour that is 10:00 AM [*]55 minutes is 5 minutes less than 1 hour [*]So add another hour to 10:00 AM which is 11:00 AM [*]Subtract the 5 minutes is 10:55 AM [*]15 minutes is 5 minutes + 10 minutes [*]Add 5 minutes to 10:55AM is 11:00 [*]10 minutes added to this is [B]11:10 AM[/B] [/LIST]

Mindy and troy combined ate 9 pieces of the wedding cake. Mindy ate 3 pieces of cake and troy had 1
Mindy and troy combined ate 9 pieces of the wedding cake. Mindy ate 3 pieces of cake and troy had 1/4 of the total cake. Write an equation to determine how many pieces of cake (c) that were in total Let c be the total number of pieces of cake. Let m be the number of pieces Mindy ate. Let t be the number of pieces Troy ate. We have the following given equations: [LIST] [*]m + t = 9 [*]m = 3 [*]t = 1/4c [/LIST] Combining (2) and (3) into (1), we have: 3 + 1/4c = 9 Subtract 3 from each side: 1/4c = 6 Cross multiply: [B]c = 24 [MEDIA=youtube]aeqWQXr5f_Y[/MEDIA][/B]

Missing Average
Free Missing Average Calculator - Given a set of scores and an average, this calculates the next score necessary to attain that average

Modified Internal Rate of Return (MIRR)
Free Modified Internal Rate of Return (MIRR) Calculator - Given a set of positive/negative cash flows, a finance rate, and a reinvestment rate, this calculates the modified internal rate of return

Modified Payback Period
Free Modified Payback Period Calculator - Given a set of cash inflows, outflows, and a discount rate, this calculates the modified payback period.

Modular Exponentiation and Successive Squaring
Free Modular Exponentiation and Successive Squaring Calculator - Solves xn mod p using the following methods:
* Modular Exponentiation
* Successive Squaring

Molar Mass
Free Molar Mass Calculator - Calculates the molar mass of an element or solution.

Molly is making strawberry infused water. For each ounce of strawberry juice, she uses two times as
Molly is making strawberry infused water. For each ounce of strawberry juice, she uses three times as many ounces of water as juice. How many ounces of strawberry juice and how many ounces of water does she need to make 40 ounces of strawberry infused water? Let j be the ounces of strawberry juice and w be the ounces of water. We're given: [LIST=1] [*]j + w = 40 [*]w = 3j [/LIST] Substitute (2) into (1): j + 3j = 40 Combine like terms: 4j = 40 [URL='https://www.mathcelebrity.com/1unk.php?num=4j%3D40&pl=Solve']Typing this equation into our search engine[/URL], we get: [B]j = 10[/B] From equation (2), we substitute j = 2: w = 3(10) [B]w = 30 [/B] This means we have [B]10 ounces of juice[/B] and [B]30 ounces of water[/B] for a 40 ounce mix.

Moment of Inertia
Free Moment of Inertia Calculator - Calculates any of the 3 items from the Moment of Inertia equation, Inertia (I), Mass (M), and Length (L).

Money Multiplier
Free Money Multiplier Calculator - Given a reserve ratio and initial deposit amount, this calculates the money multiplier and displays the re-lending process table for a bank to other banks including reserves and loans.

Morse Code Translator
Free Morse Code Translator Calculator - Given a phrase with letters, numbers, and most punctuation symbols, the calculator will perform the following duties:
1) Translate that phrase to Morse Code.
2) Translate the Morse Code to a Dit-Dah message
3) Calculate the number of dots in the message
4) Calculate the number of dashes in the message

This also translates from Morse Code back to English.

Mortgage
Free Mortgage Calculator - Calculates the monthly payment, APY%, total value of payments, principal/interest/balance at a given time as well as an amortization table on a standard or interest only home or car loan with fixed interest rate. Handles amortized loans.

Mount McKinley in Alaska, the highest mountain in North America, is 20,320 feet above sea level. Dea
Mount McKinley in Alaska, the highest mountain in North America, is 20,320 feet above sea level. Death Valley, the lowest point, is 280 feet below sea level. What is the difference in height between Mount McKinley and Death Valley? Regarding height with respect to sea level... [LIST] [*]Above sea level is written as positive height [*]Below sea level is written as negative height [/LIST] So we have: +20,320 - -280 +20,320 + 280 [B]20,600[/B]

Mr rosenthal bought 15 computer disks and a carrying case for 28.50 if the carrying case cost 6.75 w
Mr rosenthal bought 15 computer disks and a carrying case for 28.50 if the carrying case cost 6.75 what was the cost of each disk Figure out how much he has left over after purchasing the carrying case: 28.50 - 6.75 = 21.75 Now, figure out the price per disk: 21.75/15 = [B]1.45[/B]

Mr turner sent his car to the workshop for repair work as well as to change 4 tires. Mr turner paid
Mr turner sent his car to the workshop for repair work as well as to change 4 tires. Mr turner paid $1035 in all. The repair work cost 5 times the price of each tire. The mechanic told Mr. turner that the repair work cost $500. Explain the mechanic’s mistake Let the cost for work be w. Let the cost for each tire be t. We're given; [LIST=1] [*]w = 5t [*]w + 4t = 1035 [/LIST] Substitute equation 1 into equation 2: (5t) + 4t = 1035 [URL='https://www.mathcelebrity.com/1unk.php?num=%285t%29%2B4t%3D1035&pl=Solve']Type this equation into our search engine[/URL], and we get: t = 115 Substitute this into equation (1): w = 5(115) w = [B]575[/B] The mechanic underestimated the work cost.

Mr. Chris’s new app “Tick-Tock” is the hottest thing to hit the app store since...ever. It costs $5
Mr. Chris’s new app “Tick-Tock” is the hottest thing to hit the app store since...ever. It costs $5 to buy the app and then $2.99 for each month that you subscribe (a bargain!). How much would it cost to use the app for one year? Write an equation to model this using the variable “m” to represent the number of months that you use the app. Set up the cost function C(m) where m is the number of months you subscribe: C(m) = Monthly Subscription Fee * months + Purchase fee [B]C(m) = 2.99m + 5[/B]

Mr. Crews goes to Publix and spends $81.25 on groceries. He pays the cashier with a hundred dollar b
Mr. Crews goes to Publix and spends $81.25 on groceries. He pays the cashier with a hundred dollar bill. How much change will Mr. Crews get back from the cashier? Using our [URL='https://www.mathcelebrity.com/changecounter.php?cash=100&bill=81.25&pl=Calculate+Change+Amount']change calculator[/URL], we get: [B]$18.75[/B]

Mr. Crimmins bought 15 apples and 15 oranges. Each apple cost $1.00, each orange cost $1.50. How muc
Mr. Crimmins bought 15 apples and 15 oranges. Each apple cost $1.00, each orange cost $1.50. How much more did he spend on oranges than apples? [U]Calculate apple spend:[/U] Apple Spend = Apple Cost * Number of Apples Apple Spend = $1.00 * 15 Apple Spend =[B] [/B]$15 [B][/B] [U]Calculate apple spend:[/U] Orange Spend = Orange Cost * Number of Oranges Orange Spend = $1.50 * 15 Orange Spend = $22.50 [B][/B] [U]Calculate the additional amount spent on oranges over apples:[/U] Additional Orange Spend = Orange Spend - Apple Spend Additional Orange Spend = $22.50 - $15.00 Additional Orange Spend = [B]$7.50[/B]

Mr. Demerath has a large collection of Hawaiian shirts. He currently has 42 Hawaiian shirts. He gets
Mr. Demerath has a large collection of Hawaiian shirts. He currently has 42 Hawaiian shirts. He gets 2 more every month. After how many months will Mr. Demerath have at least 65 Hawaiian shirts? We set up the function H(m) where m is the number of months that goes by. Mr. Demerath's shirts are found by: H(m) = 2m + 42 The problem asks for m when H(m) = 65. So we set H(m) = 65: 2m + 42 = 65 To solve this equation for m, we[URL='https://www.mathcelebrity.com/1unk.php?num=2m%2B42%3D65&pl=Solve'] type it in our search engine [/URL]and we get: m = [B]11.5[/B]

Mr. Elk is secretly a huge fan of Billie Eilish, and is saving up for front row seats. He puts $250
Mr. Elk is secretly a huge fan of Billie Eilish, and is saving up for front row seats. He puts $250 in the bank that has an interest rate of 8% compounded daily. After 4 years, Billie is finally hitting up NJ on her tour. How much money does Mr. Elk have in the bank? (rounded to the nearest cent) * 4 years = 365*4 days 4 years = 1,460 days. Using this number of compounding periods, we [URL='https://www.mathcelebrity.com/compoundint.php?bal=250&nval=1460&int=8&pl=Daily']plug this into our compound interest calculator[/URL] to get: [B]$344.27[/B]

Mr. Jimenez has a pool behind his house that needs to be fenced in. The backyard is an odd quadrilat
Mr. Jimenez has a pool behind his house that needs to be fenced in. The backyard is an odd quadrilateral shape and the pool encompasses the entire backyard. The four sides are 1818a, 77b, 1111a, and 1919b in length. How much fencing? (the length of the perimeter) would he need to enclose the pool? The perimeter P is found by adding all 4 sides: P = 1818a + 77b + 1111a + 1919b Group the a and b terms P = (1818 + 1111)a + (77 + 1919b) [B]P = 2929a + 1996b[/B]

Mr. Smith wants to spend less than $125 at a zoo. A ticket cost $7 he is taking 2 kids with him. Use
Mr. Smith wants to spend less than $125 at a zoo. A ticket cost $7 he is taking 2 kids with him. Use p to represent the other money he can spend there. 2 kids and Mr. Smith = 3 people. Total Ticket Cost is 3 people * 7 per ticket = 21 If he has 125 to spend, we have the following inequality using less than or equal to (<=) since he can spend up to or less than 125: p + 21 <= 125 Subtract 21 from each side: [B]p <= 104[/B]

Mr. Wilson wants to park his carin a parking garage that charges 3 per hour along with a flat fee of
Mr. Wilson wants to park his carin a parking garage that charges 3 per hour along with a flat fee of 6. If Mr. Wilson paid 54 to park in the garage, for how many hours did he park there? [U]Set up an equation, where f is the flat fee, and h is the number of hours parked:[/U] 3h + f = 54 [U]Substitute f = 6 into the equation:[/U] 3h + 6 = 54 [U]Using our [URL='http://www.mathcelebrity.com/1unk.php?num=3h%2B6%3D54&pl=Solve']equation solver[/URL], we get[/U] [B]h = 16[/B]

Mr. Winkle downloaded 34 more songs than Mrs. Winkle downloaded. Together they downloaded 220 song
Mr. Winkle downloaded 34 more songs than Mrs. Winkle downloaded. Together they downloaded 220 songs. How many songs did each download? Let x = Mr. Winkle downloads and y = Mrs. Winkle downloads. We then have x = y + 34 and x + y = 220. Substitute equation 1 into equation 2, we have: (y + 34) + y = 220 2y + 34 = 220 Subtract 34 from each side: 2y = 186 Divide each side by 2: y = 93 (Mrs. Winkle) x = 93 + 34 x = 127 (Mr. Winkle)

Mr.coulter has n calculators and mr Riley has 4 more calculators then him
Mr.coulter has n calculators and mr Riley has 4 more calculators then him More means add, so Riley's calculators are: [B]n + 4[/B]

Mrs. Evans has 120 crayons and 30 pieces of paper to give her students. What is the largest number o
Mrs. Evans has 120 crayons and 30 pieces of paper to give her students. What is the largest number of students she can have her class so that each student gets an equal number of crayons and equal number of paper? [URL='https://www.mathcelebrity.com/gcflcm.php?num1=30&num2=120&num3=&pl=GCF+and+LCM']Using our GCF calculator for the GCF(30, 120)[/URL], we get 30. So 30 people get the following: [B]30/30 = 1 piece of paper 120/30 = 4 crayons[/B]

Mrs. Lopez gave a homework assignment over summer vacation to read three books from the following li
Mrs. Lopez gave a homework assignment over summer vacation to read three books from the following list: a) Call of the Wild b) Wuthering Heights c) Death of a Salesman d) The Cartoon Book of Physics How many possible combinations of three books are there in the list of four books? We need to elimination those of the same order, so we use combinations: [URL='https://www.mathcelebrity.com/permutation.php?num=4&den=3&pl=Combinations']4C3[/URL] = [B]4[/B]

Mrs. Lowe charges $45 an hour with a $10 flat fee for tutoring. Mrs. Smith charges $40 an hour wit
Mrs. Lowe charges $45 an hour with a $10 flat fee for tutoring. Mrs. Smith charges $40 an hour with a $15 flat fee to tutor. Write an equation that represents the situation when the cost is the same to be tutored by Mrs. Lowe and Mrs. Smith. [U]Set up cost equation for Mrs. Lowe where h is the number of hours tutored:[/U] Cost = Hourly Rate * number of hours + flat fee Cost = 45h + 10 [U]Set up cost equation for Mrs. Smith where h is the number of hours tutored:[/U] Cost = Hourly Rate * number of hours + flat fee Cost = 40h + 15 [U]Set both cost equations equal to each other:[/U] 45h + 10 = 40h + 15 <-- This is our equation To solve for h if the problem asks, we [URL='https://www.mathcelebrity.com/1unk.php?num=45h%2B10%3D40h%2B15&pl=Solve']type this equation into our search engine[/URL] and we get: h = 1

Mrs. Mormon bought 2.5 pounds of grapes and paid $4.72. What was the cost per pound of the grapes?
Mrs. Mormon bought 2.5 pounds of grapes and paid $4.72. What was the cost per pound of the grapes? Cost per pound = Total Cost / pounds Cost per pound = $4.72 / 2.5 Cost per pound = [B]$1.89[/B]

Mrs. Taylor is making identical costumes for the dancers in the dance club. She uses 126 pink ribbon
Mrs. Taylor is making identical costumes for the dancers in the dance club. She uses 126 pink ribbons and 108 yellow ribbons. a) What is the maximum possible number of costumes she can make? b) How many pink and how many yellow ribbons are on each costume? a), we want the greatest common factor (GCF) of 108 and 126. [URL='https://www.mathcelebrity.com/gcflcm.php?num1=108&num2=126&num3=&pl=GCF+and+LCM']Using our GCF calculator[/URL] we get: [B]a) 18 costumes [/B] b) Pink Ribbons per costume = Total Pink Ribbons / GCF in question a Pink Ribbons per costume = 126/18 Pink Ribbons per costume = [B]7[/B] [B][/B] Yellow Ribbons per costume = Total Yellow Ribbons / GCF in question a Yellow Ribbons per costume = 108/18 Yellow Ribbons per costume = [B]6[/B]

Ms. Gonzales is investing $17000 at an annual interest rate of 6% compounded continuously. How much
Ms. Gonzales is investing $17000 at an annual interest rate of 6% compounded continuously. How much money will be in the account after 16 years? Round your answer to the nearest hundredth (two decimal places). Using our [URL='https://www.mathcelebrity.com/simpint.php?av=&p=17000&int=6&t=16&pl=Continuous+Interest']continuous interest calculator[/URL], we get: [B]44,398.84[/B]

Ms. Jeffers is splitting $975 among her three sons. If the oldest gets twice as much as the youngest
Ms. Jeffers is splitting $975 among her three sons. If the oldest gets twice as much as the youngest and the middle son gets $35 more than the youngest, how much does each boy get? Let 0 be the oldest son, m be the middle sun, and y be the youngest son. Set up our given equations [LIST] [*]o = 2y [*]m = y + 35 [*]o + m + y = 975 [/LIST] [U]Substitute the first and second equations into Equation 3[/U] 2y + y + 35 + y = 975 [U]Combine the y terms[/U] 4y + 35 = 975 Subtract 35 using our [URL='http://www.mathcelebrity.com/1unk.php?num=4y%2B35%3D975&pl=Solve']equation calculator[/URL] to solve and get [B]y = 235[/B] [U]Plug y = 235 into equation 2[/U] m = 235 + 35 [B]m = 270[/B] [U]Plug y = 235 into equation 2[/U] o = 2(235) [B]o = 470[/B]

Mt. Everest, the highest elevation in Asia, is 29,028 feet above sea level. The Dead Sea, the lowest
Mt. Everest, the highest elevation in Asia, is 29,028 feet above sea level. The Dead Sea, the lowest elevation, is 1,312 feet below sea level. What is the difference between these two elevations? Above sea level is listed as positive (+) Below sea level is listed as negative (-) We have: Difference = +29,028 - (-1312) Difference = 29028 + 1312 [URL='https://www.mathcelebrity.com/longdiv.php?num1=29028&num2=1312&pl=Add']Difference[/URL] = [B]30,340[/B]

Multifactorials
Free Multifactorials Calculator - Calculates the multifactorial n!(m)

Multinomial Distribution
Free Multinomial Distribution Calculator - Given a set of xi counts and a respective set of probabilities θi, this calculates the probability of those events occurring.

Multiplication Equality Property
Free Multiplication Equality Property Calculator - Demonstrates the Multiplication Equality Property Numerical Properties

Multiplication Property Of Inequality
Free Multiplication Property Of Inequality Calculator - Demonstrates the Multiplication Property Of Inequality Numerical Properties

Multiplicative Identity Property
Free Multiplicative Identity Property Calculator - Demonstrates the Multiplicative Identity property using a number. Numerical Properties

Multiplicative Inverse Property
Free Multiplicative Inverse Property Calculator - Demonstrates the Multiplicative Inverse property using a number. Numerical Properties

Multiply 0 by 3 and add 4
Multiply 0 by 3 and add 4 multiply 0 by 3: 0 * 3 Then add 4: [B]0 * 3 + 4 <--- [/B][I]This is our algebraic expression.[/I] If we want to evaluate this expression, we [URL='https://www.mathcelebrity.com/order-of-operations-calculator.php?num=0%2A3%2B4&pl=Perform+Order+of+Operations']type it in our search engine[/URL] and we get: [B]4[/B]

multiply 3 by the difference of u and t
multiply 3 by the difference of u and t Take this algebraic expression in parts: The difference of u and t means we subtract t from u u - t Multiply this difference by 3: [B]3(u - t)[/B]

Multiply 3w by the sum of v and 2u
Multiply 3w by the sum of v and 2u the sum of v and 2u: v + 2u Multiply 3w by the sum of v and 2u [B]3w(v + 2u)[/B]

multiply 5 and sum of twice of d and 10
multiply 5 and sum of twice of d and 10 Twice d means we multiply d by 2: 2d The sum of twice d and 10 means we add 2d to 10 2d + 10 We multiply this quantity by 5: [B]5(2d + 10)[/B]

multiply 9 by 3, subtract y from the result
multiply 9 by 3, subtract y from the result Multiply 9 by 3 9 * 3 Subtract y from the result [B]9 * 3 - y[/B]

multiply 9 by the quotient of 4 and z
multiply 9 by the quotient of 4 and z Quotient of 4 and z is written as: 4/z Multiply this quotient by 9: 9(4)/z Multiplying the top, we get: [B]36/z[/B]

multiply a number by 4 and then subtract the answer from 30
multiply a number by 4 and then subtract the answer from 30 The phrase [I]a number[/I] means an arbitrary variable, let's call it x. x Multiply this number by 4: 4x Subtract the answer from 30: [B]30 - 4x[/B]

Multiply a number by 6 and subtracting 6 gives the same result as multiplying the number by 3 and su
Multiply a number by 6 and subtracting 6 gives the same result as multiplying the number by 3 and subtracting 4. Find the number The phrase [I]a number [/I]means an arbitrary variable, let's call it x. multiply a number by 6 and subtract 6: 6x - 6 Multiply a number by 3 and subtract 4: 3x - 4 The phrase [I]gives the same result[/I] means an equation. So we set 6x - 6 equal to 3x - 4 6x - 6 = 3x - 4 To solve this equation for x, we type it in our search engine and we get: x = [B]2/3[/B]

Multiply by 19 no calculator shortcut
we start at the top and write 1 and add 2 for every line we go down: 19 * 1 = 1 19 * 2 = 3 19 * 3 = 5 19 * 4 = 7 19 * 5 = 9 19 * 6 = 11 19 * 7 = 13 19 * 8 = 15 19 * 9 = 17 19 * 10 = 19 Next, we start the bottom of the list. add on a 0 at the end, and for each line we go up, we add 1 more from the previous line 19 * 1 = 19 19 * 2 = 38 19 * 3 = 57 19 * 4 = 76 19 * 5 = 95 19 * 6 = 114 19 * 7 = 133 19 * 8 = 152 19 * 9 = 171 19 * 10 = 190 [MEDIA=youtube]3epAV3Bw0Qw[/MEDIA]

Multiply by 29 No Calculator Shortcut
we start at the top and write 2 and add 3 for every line we go down: 29 * 1 = 2 29 * 2 = 5 29 * 3 = 8 29 * 4 = 11 29 * 5 = 14 29 * 6 = 17 29 * 7 = 20 29 * 8 = 23 29 * 9 = 26 29 * 10 = 29 Next, we start the bottom of the list. add on a 0 at the end, and for each line we go up, we add 1 more from the previous line 29 * 1 = 29 29 * 2 = 58 29 * 3 = 87 29 * 4 = 116 29 * 5 = 145 29 * 6 = 174 29 * 7 = 203 29 * 8 = 232 29 * 9 = 261 29 * 10 = 290 [MEDIA=youtube]r6OI56rnezM[/MEDIA]

Multiply by 39 No Calculator Shortcut
we start at the top and write 3 and add 4 for every line we go down: 39 * 1 = 3 39 * 2 = 7 39 * 3 = 11 39 * 4 = 15 39 * 5 = 19 39 * 6 = 23 39 * 7 = 27 39 * 8 = 31 39 * 9 = 35 39 * 10 = 39 Next, we start the bottom of the list. add on a 0 at the end, and for each line we go up, we add 1 more from the previous line 39 * 1 = 39 39 * 2 = 78 39 * 3 = 117 39 * 4 = 156 39 * 5 = 195 39 * 6 = 234 39 * 7 = 273 39 * 8 = 312 39 * 9 = 351 39 * 10 = 390 [MEDIA=youtube]Z5vt6L8PolU[/MEDIA]

Multiply by 49 No Calculator Shortcut
we start at the top and write 4 and add 5 for every line we go down: 49 * 1 = 4 49 * 2 = 9 49 * 3 = 14 49 * 4 = 19 49 * 5 = 24 49 * 6 = 29 49 * 7 = 34 49 * 8 = 39 49 * 9 = 44 49 * 10 = 49 Next, we start the bottom of the list. add on a 0 at the end, and for each line we go up, we add 1 more from the previous line 49 * 1 = 49 49 * 2 = 98 49 * 3 = 147 49 * 4 = 196 49 * 5 = 245 49 * 6 = 294 49 * 7 = 343 49 * 8 = 392 49 * 9 = 441 49 * 10 = 490 [MEDIA=youtube]ETb-wJlBOms[/MEDIA]

Multiply c by five and square the answer
Multiply c by five and square the answer Multiply c by five: 5c Square the answer means we raise 5c to the power of 2: [B](5c)^2 [/B] This can also be written as [B]25c^2[/B]

Multiply Even Numbers by 5 No Calculator Shortcut
Take the number being multiplied by 5. Divide it in half Add a zero 14 * 5 Divide 14/2 = 7 Add a 0 --> 70 [MEDIA=youtube]lOJmx0Ygpz8[/MEDIA]

multiply k by 5.8, and then subtract 3.09 from the product
multiply k by 5.8, and then subtract 3.09 from the product Take this algebraic expression in pieces: [U]Multiply k by 5.8:[/U] 5.8k [U]Then subtract 3.09 from the product[/U] [B]5.8k - 3.09[/B]

multiply m by 2, then add n to the result
[U]Multiply m by 2[/U] 2m [U]Add n to the result[/U] [B]2m + n[/B]

multiply m by 5, double the result, then multiply 10 by what you have
multiply m by 5, double the result, then multiply 10 by what you have Take this algebraic expression in parts: [LIST] [*]Multiply m by 5: 5m [*]double the result means multiply 5m by 2: 2(5m) = 10m [*]Multiply 10 by what you have means multiply 10 by the result of 10m above: [/LIST] 10(10m) = [B]100m[/B]

multiply r by t, add the result to u, then multiply what you have by s
multiply r by t, add the result to u, then multiply what you have by s Take this algebraic expression in parts: [LIST=1] [*]Multiply r by t: rt [*]Add the result to u means we add rt to u: u + r [*]Multiply what you have by s. This means we take the result in #2, u + r, and multiply it by s: [/LIST] [B]s(u + r)[/B]

Multiply t by 4 then multiply s by the result
Multiply t by 4 then multiply s by the result Multiply t by 4 4t Then multiply s by the result [B]4st[/B]

multiply t by u, add the to v, then triple what you have
multiply t by u, add the to v, then triple what you have Multiply t by u: tu Add this to v: v + tu Then triple what you have - This means we multiply the expression above by 3: [B]3(v + tu)[/B]

Multiply the difference of 3 and q by p
Multiply the difference of 3 and q by p. Take this algebraic expression in pieces: [B][U]Step 1: The difference of 3 and q[/U][/B] The word [I]difference[/I] means we subtract the variable q from 3 3 - q [B][U]Step 2: Multiply the expression 3 - q by p:[/U] p(3 - q)[/B]

multiply the sum of 2p and q by3
multiply the sum of 2p and q by3 The sum of 2p and q: 2p + q Multiply the sum by 3: [B]3(2p + q)[/B]

Multiply two-digits numbers by 11 shortcut
[U]53 x 11[/U] Write 3 as the right most digit in 53 3 5 + 3 = 8. Write that to the left of 3 83 Then write the 5 as the left most digit 583 [U]35 x 11[/U] Write 5 as the right most digit in 35 5 3 + 5 = 8. Write that to the left of 5 85 Then write the 3 as the left most digit 385 [U]57 x 11[/U] Write the 7 in 57 as the leftmost digit 7 5 + 7 = 12 Write the 2 in 12 as the next digit 27 Since our last sum was 10 or greater, we add 1 to the 5 in 57 to get 6 627 [U]91 x 11[/U] Write the 1 in 91 as the leftmost digit 1 9 + 1 = 10 Write the 0 in 10 as the next digit 01 Since our last sum was 10 or greater, we add 1 to the 9 in 91 to get 10 1001 [MEDIA=youtube]Uzfj57veazA[/MEDIA]

multiply u by s, multiply the result by v, then multiply t
multiply u by s, multiply the result by v, then multiply t Take this algebraic expression in parts: [LIST] [*]Multiply u by s: us [*]Multiply the result by v: usv [*]Then multiply by t: [B]usvt[/B] [/LIST]

Multiplying a number by 6 is equal to the number increased by 9
Multiplying a number by 6 is equal to the number increased by 9. The phrase [I]a number[/I] means an arbitrary variable, let's call it x. Multiply it by 6 --> 6x We set this equal to the same number increased by 9. Increased by means we add: [B]6x = x + 9 <-- This is our algebraic expression [/B] To solve this equation, we [URL='https://www.mathcelebrity.com/1unk.php?num=6x%3Dx%2B9&pl=Solve']type it into the search engine [/URL]and get x = 1.8.

mx=ac/np for n
mx=ac/np for n Cross multiply: mnpx = ac Divide each side by mpx: mnpx/mpx = ac/mpx Cancel the mpx on the right side: n = [B]ac/mpx[/B]

My brother is x years old. I am 5 years older than him. Our combined age is 30 years old. How old is
My brother is x years old. I am 5 years older than him. Our combined age is 30 years old. How old is my brother Brother's age is x: I am 5 years older, meaning I'm x + 5: The combined age is found by adding: x + (x + 5) = 30 Group like terms: 2x + 5 = 30 To solve for x, [URL='https://www.mathcelebrity.com/1unk.php?num=2x%2B5%3D30&pl=Solve']type this equation into our search engine[/URL] and we get: x = [B]12.5[/B]

My dad’s annual income is $8460. What is the monthly income of my dad?
My dad’s annual income is $8460. What is the monthly income of my dad? Monthly Income = Annual Income / 12 Monthly Income = 8460/12 Monthly Income = [B]$705[/B]

My rent was 800.00 a month. My landlord raised my rent to 1,240.00. What percentage did he raise m
My rent was 800.00 a month. My landlord raised my rent to 1,240.00. What percentage did he raise my rent?. First, calculate the difference between the old and new rent: Difference = 1,240 - 800 = 440 Percentage increase = 440/800 [URL='https://www.mathcelebrity.com/perc.php?num=440&den=800&pcheck=1&num1=16&pct1=80&pct2=70&den1=80&idpct1=10&hltype=1&idpct2=90&pct=82&decimal=+65.236&astart=12&aend=20&wp1=20&wp2=30&pl=Calculate']Type 440/800 into the search engine, and choose the percent option[/URL] You get [B]55%[/B] increase.

My son is 9 less than 1/2 my age. If I am 34 how old is my son?
1/2 of the parent age is 34/2 = 17. 9 less than that is 17 - 9 = 8. The son is 8 years old. You can also write this as 1/2(34) - 9 --> 17 - 9 = 8.

n + .07n = $90.95
n + .07n = $90.95 Group like terms: 1.07n = $90.95 Solve for [I]n[/I] in the equation 1.07n = 90.95 [SIZE=5][B]Step 1: Divide each side of the equation by 1.07[/B][/SIZE] 1.07n/1.07 = 90.95/1.07 n = [B]85 [URL='https://www.mathcelebrity.com/1unk.php?num=1.07n%3D90.95&pl=Solve']Source[/URL][/B]

n + 2n + 3n + 4n = 2 + 3 + 4 + 5 + 6
n + 2n + 3n + 4n = 2 + 3 + 4 + 5 + 6 Solve for [I]n[/I] in the equation n + 2n + 3n + 4n = 2 + 3 + 4 + 5 + 6 [SIZE=5][B]Step 1: Group the n terms on the left hand side:[/B][/SIZE] (1 + 2 + 3 + 4)n = 10n [SIZE=5][B]Step 2: Group the constant terms on the right hand side:[/B][/SIZE] 2 + 3 + 4 + 5 + 6 = 20 [SIZE=5][B]Step 3: Form modified equation[/B][/SIZE] 10n = + 20 [SIZE=5][B]Step 4: Divide each side of the equation by 10[/B][/SIZE] 10n/10 = 20/10 n = [B]2[/B]

n + 9n - 8 - 5 = 2n + 3
n + 9n - 8 - 5 = 2n + 3 Solve for [I]n[/I] in the equation n + 9n - 8 - 5 = 2n + 3 [SIZE=5][B]Step 1: Group the n terms on the left hand side:[/B][/SIZE] (1 + 9)n = 10n [SIZE=5][B]Step 2: Group the constant terms on the left hand side:[/B][/SIZE] -8 - 5 = -13 [SIZE=5][B]Step 3: Form modified equation[/B][/SIZE] 10n - 13 = 2n + 3 [SIZE=5][B]Step 4: Group variables:[/B][/SIZE] We need to group our variables 10n and 2n. To do that, we subtract 2n from both sides 10n - 13 - 2n = 2n + 3 - 2n [SIZE=5][B]Step 5: Cancel 2n on the right side:[/B][/SIZE] 8n - 13 = 3 [SIZE=5][B]Step 6: Group constants:[/B][/SIZE] We need to group our constants -13 and 3. To do that, we add 13 to both sides 8n - 13 + 13 = 3 + 13 [SIZE=5][B]Step 7: Cancel 13 on the left side:[/B][/SIZE] 8n = 16 [SIZE=5][B]Step 8: Divide each side of the equation by 8[/B][/SIZE] 8n/8 = 16/8 n = [B]2[/B]

n + 9n - 90 = 0
n + 9n - 90 = 0 Solve for [I]n[/I] in the equation n + 9n - 90 = 0 [SIZE=5][B]Step 1: Group the n terms on the left hand side:[/B][/SIZE] (1 + 9)n = 10n [SIZE=5][B]Step 2: Form modified equation[/B][/SIZE] 10n - 90 = [SIZE=5][B]Step 3: Group constants:[/B][/SIZE] We need to group our constants -90 and 0. To do that, we add 90 to both sides 10n - 90 + 90 = 0 + 90 [SIZE=5][B]Step 4: Cancel 90 on the left side:[/B][/SIZE] 10n = 90 [SIZE=5][B]Step 5: Divide each side of the equation by 10[/B][/SIZE] 10n/10 = 90/10 n = [B]9[/B]

n - n = 10 - n
n - n = 10 - n Solve for [I]n[/I] in the equation n - n = 10 - n [SIZE=5][B]Step 1: Group the n terms on the left hand side:[/B][/SIZE] (1 - 1)n = 0n = 0 [SIZE=5][B]Step 2: Form modified equation[/B][/SIZE] = - n + 10 [SIZE=5][B]Step 3: Group variables:[/B][/SIZE] We need to group our variables and -n. To do that, we add n to both sides + n = -n + 10 + n [SIZE=5][B]Step 4: Cancel -n on the right side:[/B][/SIZE] n = [B]10[/B]

n = 3n - 1/2
n = 3n - 1/2 Solve for [I]n[/I] in the equation n = 3n - 1/2 [SIZE=5][B]Step 1: Group variables:[/B][/SIZE] We need to group our variables n and 3n. To do that, we subtract 3n from both sides n - 3n = 3n - 0.5 - 3n [SIZE=5][B]Step 2: Cancel 3n on the right side:[/B][/SIZE] -2n = -0.5 [SIZE=5][B]Step 3: Divide each side of the equation by -2[/B][/SIZE] -2n/-2 = -0.5/-2 n = [B]0.25 or 1/4[/B]

n = 5m^2d for d
n = 5m^2d for d Divide each side by 5m^2 to isolate d: n/5m^2 = 5m^2d/5m^2 Cancel the 5m^2 on the right side and we get: d = [B]n/5m^2[/B]

n = b + d^2a for a
n = b + d^2a for a Let's start by isolating the one term with the a variable. Subtract b from each side: n - b = b - b + d^2a Cancel the b terms on the right side and we get: n - b = d^2a With the a term isolated, let's divide each side of the equation by d^2: (n - b)/d^2 = d^2a/d^2 Cancel the d^2 on the right side, and we'll display this with the variable to solve on the left side: a = [B](n - b)/d^2 [MEDIA=youtube]BCEVsZmoKoQ[/MEDIA][/B]

n and m are congruent and supplementary. prove n and m are right angles
n and m are congruent and supplementary. prove n and m are right angles Given: [LIST] [*]n and m are congruent [*]n and m are supplementary [/LIST] If n and m are supplementary, that means we have the equation: m + n = 180 We're also given n and m are congruent, meaning they are equal. So we can substitute n = m into the supplementary equation: m + m = 180 To solve this equation for m, [URL='https://www.mathcelebrity.com/1unk.php?num=m%2Bm%3D180&pl=Solve']we type it in our search engine[/URL] and we get: m = 90 This means m = 90, n = 90, which means they are both right angles since by definition, a right angle is 90 degrees.

n increased by the difference between 10 times n and 9
n increased by the difference between 10 times n and 9 Take this algebraic expression in pieces: [LIST] [*]10 times n: 10n [*]The difference between 10 times n and 9: 10n - 9 [*]n increased by the difference...: [B]n + (10n - 9)[/B] [/LIST]

n is a factor of 16
n is a factor of 16 List out factor of 16: 1 * 16 2 * 8 4 * 4 From the list above, we take the 5 [U]unique[/U] factors and build a set for n: n = [B]{1, 2, 4, 8, 16}[/B]

n is equal to 135 less than the quantity 61 times n
n is equal to 135 less than the quantity 61 times n 61 times n: 61n 135 less than the quantity 61 times n 61n - 135 We set n equal to this expression: [B]n = 61n - 135[/B]

n is equal to the product of 7 and the sum of m and 6
n is equal to the product of 7 and the sum of m and 6 The sum of m and 6: m + 6 The product of 7 and this sum: 7(m + 6) We set this expression equal to n: [B]7(m + 6) = n[/B]

n is the sum of twenty-five and fifteen
n is the sum of twenty-five and fifteen The sum of twenty-five and fifteen: 25 + 15 The word [I]is[/I] means an equal to, so we set 25 + 15 equal to n: [B]n = 25 + 15 n = 40[/B]

n is tripled then decreased by 5
n is tripled then decreased by 5 n is tripled means we multiply n by 3: 3n Decreased by 5 means we subtract 5 from 3n: [B]3n - 5[/B]

N reduced by 2 is the same as Z increased by 7
N reduced by 2 is the same as Z increased by 7 [LIST] [*]N reduced by 2 means subtract --> n - 2 [*]z increased by 7 means add --> z + 7 [*][I]Is the same as[/I] means equal to, so we set these expressions equal to each other [*][B]n - 2 = z + 7[/B] [/LIST]

N squared multiplied by the difference of n and 3
N squared multiplied by the difference of n and 3 n squared means we raise n to the power of 2: n^2 The difference of n and 3 means we subtract 3 from n: n - 3 Now we multiply both terms together: [B]n^2(n - 3)[/B]

n subtract m, multiply by c, then add w
n subtract m, multiply by c, then add w Take this algebraic expression in pieces: [LIST] [*]n subtract m: n - m [*]multiply by c: c(n - m) [*]Then add w: [B]c(n - m) + w[/B] [/LIST]

n times 146, reduced by 94 is the same as h
n times 146, reduced by 94 is the same as h n time 146 146n Reduced by 94 146n - 94 Is the same as h means an equation: [B]146n - 94 = h[/B]

n=i*x+y for i
n=i*x+y for i This is a literal equation. Subtract y from each side of the equation: n - y = i*x + y - y The y's cancel on the right side, so we have: n - y = ix Divide each side of the equation by x, to isolate i (n - y)/x = ix/x The x's cancel on the right side, so we have: i = [B](n - y)/x[/B]

name five coins that equal 18 cents
name five coins that equal 18 cents Here are the five coins: [LIST] [*][B]1 dime = [/B]10 cents [*][B]1 nickel = [/B]5 cents [*][B]3 pennies = [/B]3 cents [*]5 coins = 10 + 5 + 3 = 18 cents [/LIST]

Name the property shown. 6 + 5 + 84 = 84 + 5 + 6
Name the property shown. 6 + 5 + 84 = 84 + 5 + 6 [B] Commutative property of addition[/B]

Nancy is 10 years less than 3 times her daughters age. If Nancy is 41 years old, how old is her daug
Nancy is 10 years less than 3 times her daughters age. If Nancy is 41 years old, how old is her daughter? Declare variables for each age: [LIST] [*]Let Nancy's age be n [*]Let her daughter's age be d [/LIST] We're given two equations: [LIST=1] [*]n = 3d - 10 [*]n = 41 [/LIST] We set 3d - 10 = 41 and solve for d: Solve for [I]d[/I] in the equation 3d - 10 = 41 [SIZE=5][B]Step 1: Group constants:[/B][/SIZE] We need to group our constants -10 and 41. To do that, we add 10 to both sides 3d - 10 + 10 = 41 + 10 [SIZE=5][B]Step 2: Cancel 10 on the left side:[/B][/SIZE] 3d = 51 [SIZE=5][B]Step 3: Divide each side of the equation by 3[/B][/SIZE] 3d/3 = 51/3 d = [B]17[/B]

Nancy shot a 16 on 4 holes of golf. At this rate, what can she expect her score to be if she plays 1
Nancy shot a 16 on 4 holes of golf. At this rate, what can she expect her score to be if she plays 18 holes? Round to the nearest whole number Set up a proportion of score to holes of golf where s is the score for 18 holes: 16/4 = s/18 To solve this proportion for s, we [URL='https://www.mathcelebrity.com/prop.php?num1=16&num2=s&den1=4&den2=18&propsign=%3D&pl=Calculate+missing+proportion+value']type it in our search engine[/URL] and we get: s = [B]72[/B]

Nancy started the year with $435 in the bank and is saving $25 a week. Shane started with $875 and i
Nancy started the year with $435 in the bank and is saving $25 a week. Shane started with $875 and is spending $15 a week. [I]When will they both have the same amount of money in the bank?[/I] [I][/I] Set up the Account equation A(w) where w is the number of weeks that pass. Nancy (we add since savings means she accumulates [B]more[/B]): A(w) = 25w + 435 Shane (we subtract since spending means he loses [B]more[/B]): A(w) = 875 - 15w Set both A(w) equations equal to each other to since we want to see what w is when the account are equal: 25w + 435 = 875 - 15w [URL='https://www.mathcelebrity.com/1unk.php?num=25w%2B435%3D875-15w&pl=Solve']Type this equation into our search engine to solve for w[/URL] and we get: w =[B] 11[/B]

nandita earned $224 last month. she earned $28 by selling cards at a craft fair and the rest of the
nandita earned $224 last month. she earned $28 by selling cards at a craft fair and the rest of the money by babysitting. Complete an equation that models the situation and can be used to determine x, the number of dollars nandita earned last month by babysitting. We know that: Babysitting + Card Sales = Total earnings Set up the equation where x is the dollars earned from babysitting: [B]x + 28 = 224[/B] To solve this equation for x, we [URL='https://www.mathcelebrity.com/1unk.php?num=x%2B28%3D224&pl=Solve']type it in our math engine[/URL] and we get: x = [B]196[/B]

Narda has $250 and Ding has $170. How much money must Narda give to Ding so that each of them will h
Narda has $250 and Ding has $170. How much money must Narda give to Ding so that each of them will have an equal amount of money? Find the difference of Narda and Ding's money: Difference = Narda - Ding Difference = 250 - 170 Difference = 80 Find half the difference: Half the difference = 80/2 Half the difference = 40 So Narda must give Ding [B]$40[/B] to have equal amounts: Narda's new total = 250 - 40 = 210 Ding's new total = 1760 + 40 = 210

Natalie made a deal with a farmer. She agreed to work for an entire year and in return, the farmer w
Natalie made a deal with a farmer. She agreed to work for an entire year and in return, the farmer would give her $10,200 plus a prize pig. After working for 5 months, Natalie decided to quit. The farmer determined that 5 months of work was equal to $3375 plus the pig. How much money was the pig worth? The value of a year's work is $10,200 plus a pig of unknown value. The farmer took away $6825 because Natalie worked 5 months. If Natalie worked 7 more months, she would have been paid the additional $6825. 6825/7 months work = $975 per month A full year's work is $975 * 12 = $11,700 Pig value = Full years work - payout Pig value = 11,700 - 10,200 Pig value = [B]1,500[/B]

Natural Logarithm Table
Free Natural Logarithm Table Calculator - Generates a natural logarithm table for the first (n) numbers rounded to (r) digits

natural numbers that are factors of 16
natural numbers that are factors of 16 Natural numbers are positive integers starting at 1. {1, 2, 3, 4, 5, 6, 7, 8, 9, 10, 11, 12, 13, 14, 15, 16} Of these, [URL='https://www.mathcelebrity.com/factoriz.php?num=16&pl=Show+Factorization']the only factors of 16[/URL] are: {[B]1, 2, 4, 8, 16}[/B]

Nava is 17 years older than Edward. the sum of Navas age and Edwards ages id 29. How old is Nava?
Nava is 17 years older than Edward. the sum of Navas age and Edwards ages id 29. How old is Nava? Let Nava's age be n and Edward's age be e. We have 2 equations: [LIST=1] [*]n = e + 17 [*]n + e = 29 [/LIST] Substitute (1) into (2) (e + 17) + e = 29 Group like terms: 2e + 17 = 29 Running this equation [URL='http://www.mathcelebrity.com/1unk.php?num=2e%2B17%3D29&pl=Solve']through our search engine[/URL], we get: e = 6 Substitute this into equation (1) n = 6 + 17 [B]n = 23[/B]

Need help on this question
Consider the recurrence relation T(n) =2 if n = 1, T(n?1) + 4n?2 if n > 1 (i) Derive the closed form expression f(n) for this recurrence relation. (ii) Prove that T(n) = f(n),?n ?N

Need help quickly! My math skills are escaping me!
If I have 13 participants attending new hire class. 3 of them did not pass, 10 passed successfully. What is the percentage of success? What is the ratio of success? I don't believe there is a ratio, I could be wrong. Probably so, math does not agree with me! Please help! Thank you!

Negative Binomial Distribution
Free Negative Binomial Distribution Calculator - Calculates the probability of the kth success on the xth try for a negative binomial distribution also known as the Pascal distribution.? ? It calculates the probability of exactly k successes, no more than k successes, and greater than k successes as well as the mean, variance, and standard deviation.

Nervous Speaker #1 says the word "um" 8 times each minute. Nervous Speaker #2 says the word "um" 10
[SIZE=5]Nervous Speaker #1 says the word "um" 8 times each minute. Nervous Speaker #2 says the word "um" 10 times each minute. Working together, how many minutes will it take them to say the word "um" 270 times? [/SIZE] [SIZE=4]In one minute, Nervous speaker 1 and 2 say "um" 8 + 10 = 18 times per minute. We want to know how many minutes it takes for both of them to say 270 "um"s. We divide 270/18 to get [B]15 minutes[/B][/SIZE]

Net Present Value (NPV) - Internal Rate of Return (IRR) - Profitability Index
Free Net Present Value (NPV) - Internal Rate of Return (IRR) - Profitability Index Calculator - Given a series of cash flows Ct at times t and a discount rate of (i), the calculator will determine the Net Present Value (NPV) at time 0, also known as the discounted cash flow model.
Profitability Index
Also determines an Internal Rate of Return (IRR) based on a series of cash flows. NPV Calculator

Newton Method
Free Newton Method Calculator - Calculates the square root of a positive integer using the Newton Method

Nia is trying to decide between two possible jobs. Job A pays $2000 a month with a 2% annual raise.
Nia is trying to decide between two possible jobs. Job A pays $2000 a month with a 2% annual raise. Job B pays 24,000 a year with a $500 annual raise. Write a function to represent the annual salary for Job A after x years. Write a function to represent the annual salary for Job B after x years. After how many years would Nia have a greater salary at Job A? Nia Job A salary at time t: S(t) $2,000 per month equals $24,000 per year. So we have S(t) = 24,000(1.o2)^t Nia Job B salary at time t: S(t) $24,000 per year. So we have S(t) = 24,000 + 500t We want to know t when Job A salary is greater than Job B Salary: 24,000(1.o2)^t > 24,000 + 500t Time | A | B 0 | 24000 | 24000 1 | 24480 | 24500 2 | 24969.6 | 25000 3 | 25468.99 | 25500 4 | 25978.37 | 26000 5 | 26497.94 | 26500 6 | 27027.9 | 27000 7 | 27568.46 | 27500 8 | 28119.83 | 28000 9 | 28682.22 | 28500 10 | 29255.87 | 29000 11 | 29840.98 | 29500 12 | 30437.8 | 30000 13 | 31046.56 | 30500

Nick is given $50 to spend on a vacation . He decides to spend $5 a day. Write an equation that show
Nick is given $50 to spend on a vacation . He decides to spend $5 a day. Write an equation that shows how much money Nick has after x amount of days. Set up the function M(x) where M(x) is the amount of money after x days. Since spending means a decrease, we subtract to get: [B]M(x) = 50 - 5x[/B]

Nick said that his sister is 4 times as old as his brother, and together their ages add to 20 comple
Nick said that his sister is 4 times as old as his brother, and together their ages add to 20 complete this equation to find his brothers age Let b be the brother's age and s be the sister's age. We're given two equations: [LIST=1] [*]s =4b [*]b + s = 20 [/LIST] Plug (1) into (2): b + 4b = 20 [URL='https://www.mathcelebrity.com/1unk.php?num=b%2B4b%3D20&pl=Solve']Type this equation into the search engine[/URL], and we get: [B]b = 4[/B]

Nicole is half as old as Donald. The sum of their ages is 72. How old is Nicole in years?
Nicole is half as old as Donald. The sum of their ages is 72. How old is Nicole in years? Let n be Nicole's age. Let d be Donald's age. We're given two equations: [LIST=1] [*]n = 0.5d [*]n + d = 72 [/LIST] Substitute equation (1) into (2): 0.5d + d = 72 1.5d = 72 [URL='https://www.mathcelebrity.com/1unk.php?num=1.5d%3D72&pl=Solve']Typing this equation into the search engine and solving for d[/URL], we get: d = [B]48[/B]

Nine less than the product of 2 and y is not less than 15
The product of 2 and y means we multiply 2y Nine less than that product means we subtract 9 2y - 9 Finally, the entire expression is not less than 15, which means 15 or more. We use greater than or equal to [B]2y - 9 >= 15 [/B] If you want to solve this inequality and write the interval notation, use [URL='http://www.mathcelebrity.com/1unk.php?num=2y-9%3E%3D15&pl=Solve']our calculator[/URL].

Nine times the sum of a number and 6
Nine times the sum of a number and 6 The phrase [I]a number[/I] means an arbitrary variable, let's call it x. x The sum of a number and 6 means we add 6 to x: x + 6 9 times the sum: [B]9(x + 6)[/B]

nine times x is twice the sum of x and five
nine times x is twice the sum of x and five Take this algebraic expression in 4 pieces: [U]Step 1: nine time x:[/U] 9x [U]Step 2: The sum of x and five means we add 5 to x:[/U] x + 5 [U]Step 3: The word [I]twice[/I] means we multiply the sum x + 5 by 2:[/U] 2(x + 5) [U]Step 4: The word [I]is[/I] means equal to, so we set 9x equal to 2(x + 5) to get our final algebraic expression of:[/U] [B]9x = 2(x + 5)[/B]

Nine workers are hired to harvest potatoes from a field. Each is given a plot which is 5x5 feet in s
Nine workers are hired to harvest potatoes from a field. Each is given a plot which is 5x5 feet in size. What is the total area of the field? Area of each plot is 5x5 = 25 square feet. Total area = Area per plot * number of plots Total area = 25 sq ft * 9 Total area = [B]225 sq ft[/B]

Ning prepared 16 kilograms of dough after working 4 hours. How many hours did Ning work if he prepar
Ning prepared 16 kilograms of dough after working 4 hours. How many hours did Ning work if he prepared 28 kilograms of dough? Assume the relationship is directly proportional. Set up a proportion of kilograms of dough to working hours. We have: 16/4 = 28/h where h is the number of hours worked. Typing this in our [URL='http://www.mathcelebrity.com/prop.php?num1=16&num2=28&den1=4&den2=h&propsign=%3D&pl=Calculate+missing+proportion+value']proportion calculator[/URL], we get [B]h = 7[/B].

Nio is 20 years old and his brother Miguel is 8 years old. How old was Miguel when Nio is only 15?
Nio is 20 years old and his brother Miguel is 8 years old. How old was Miguel when Nio is only 15? Nio is 20. 20 - 15 is 5 years ago. So Miguel's age 5 years ago is: 8 - 5 = [B]3[/B]

Noah scores 20 points. Mai’s score was 30 points. The mean for Noah’s, Mia’s, and Clare’s was 40 poi
Noah scores 20 points. Mai’s score was 30 points. The mean for Noah’s, Mia’s, and Clare’s was 40 points. What was Clare’s score? [URL='https://www.mathcelebrity.com/missingaverage.php?num=20%2C30&avg=40&pl=Calculate+Missing+Score']Using our missing average calculator[/URL], Claire's score was [B]70[/B].

Nominal Yield
Free Nominal Yield Calculator - Given an effective annual rate of interest based on a compounding period, this determines the nominal yield.

Nonagon
Free Nonagon Calculator - Calculates the side, perimeter, and area of a nonagon

Nonagonal Number
Free Nonagonal Number Calculator - This calculator determines the nth nonagonal number

Normal body temperature is 98.6 ? F. Write an inequality that describes the temperature
Normal body temperature is 98.6 ? F. Write an inequality that describes the temperature, T, of people with above normal temperatures. Above means greater than, so we set up the inequality: [B]T > 98.6 ?[/B]

Normal Distribution
Free Normal Distribution Calculator - Calculates the probability that a random variable is less than or greater than a value or between 2 values using the Normal Distribution z-score (z value) method (Central Limit Theorem).
Also calculates the Range of values for the 68-95-99.7 rule, or three-sigma rule, or empirical rule. Calculates z score probability

Norwood High Schools jazz band includes 33 trombone players and 27 trumpet players. Meanwhile, Lakew
Norwood High Schools jazz band includes 33 trombone players and 27 trumpet players. Meanwhile, Lakewood High Schools jazz band has 37 trombone players and 28 trumpet players. Which jazz band has a lower ratio of trombone to trumpet players? Norwood: 33 : 27, is 33 out of 60 = 55% Lakewood: 37 : 28 = 37/65 = 57% Since [B]Norwood[/B] is lower than Lakewood, they have the lower ratio or trombone to trumpet players

Notebooks cost $1.39 each. What are the possible numbers of notebooks that can be purchased with $10
Notebooks cost $1.39 each. What are the possible numbers of notebooks that can be purchased with $10? Let n be the number of notebooks you can purchase. We have the following inequality: 1.39n <= 10 Divide each side by 1.39 n <= 7.194 We want whole notebooks, we cannot buy fractions of notebooks, so we have: n <= 7 The question asks for the possible numbers of notebooks we can buy. This implies we buy at least 1, but our inequality says not more than 7. So our number set is: [B]N = {1, 2, 3, 4, 5, 6, 7}[/B]

Number Line Midpoint
Free Number Line Midpoint Calculator - Calculates a midpoint between 2 points on a number line or finds the second endpoint if one endpoint and midpoint are given.

Number of cents in q quarters is 275
Number of cents in q quarters is 275 Each quarter makes 25 cents. We write this as 0.25q. Now set this equal to 275 0.25q = 275 Typing this [URL='http://www.mathcelebrity.com/1unk.php?num=0.25q%3D275&pl=Solve']equation in the search engine[/URL], we get [B]q = 1,100[/B].

Number Property
Free Number Property Calculator - This calculator determines if an integer you entered has any of the following properties:
* Even Numbers or Odd Numbers (Parity Function or even-odd numbers)
* Evil Numbers or Odious Numbers
* Perfect Numbers, Abundant Numbers, or Deficient Numbers
* Triangular Numbers
* Prime Numbers or Composite Numbers
* Automorphic (Curious)
* Undulating Numbers
* Square Numbers
* Cube Numbers
* Palindrome Numbers
* Repunit Numbers
* Apocalyptic Power
* Pentagonal
* Tetrahedral (Pyramidal)
* Narcissistic (Plus Perfect)
* Catalan
* Repunit

numerator of a fraction is 5 less than its denominator. if 1 is added to the numerator and to the de
numerator of a fraction is 5 less than its denominator. if 1 is added to the numerator and to the denominator the new fraction is 2/3. find the fraction. Let n be the numerator. Let d be the denominator. We're given 2 equations: [LIST=1] [*]n = d - 5 [*](n + 1)/(d + 1) = 2/3 [/LIST] Substitute equation (1) into equation (2) for n: (d - 5 + 1) / (d + 1) = 2/3 (d - 4) / (d + 1) = 2/3 Cross multiply: 3(d - 4) = 2(d + 1) To solve this equation for d, we type it in our search engine and we get: d = 14 Substitute d = 14 into equation (1) to solve for n: n = 14 - 5 n = 9 Therefore, our fraction n/d is: [B]9/14[/B]

n^2 + 9 = 34
n^2 + 9 = 34 Subtract 9 from each side: n^2 + 9 - 9 = 34 - 9 n^2 = 25 Take the square root of each side: n = [B]5[/B]

n^2 - 1 = -99/100
n^2 - 1 = -99/100 Add 1 (100/100) to each side: n^2 - 1 + 1 = -99/100 + 100/100 Cancel the 1's on the left side: n^2 = 1/100 Take the square root of both sides: n = [B]1/10 or -1/10[/B]

n^2 = 1/4
n^2 = 1/4 Take the square root of each side: n = [B]1/2[/B]

n^2 = 6&1/4
n^2 = 6&1/4 [URL='https://www.mathcelebrity.com/fraction.php?frac1=6%261%2F4&frac2=3%2F8&pl=Simplify']6&1/4[/URL] = 25/4 n^2 = 25/4 Take the square root of each side: n = [B]5/2 or -5/2[/B]

n^2 = 64
n^2 = 64 Take the square root of each side: sqrt(n^2) = sqt(64) n = [B]8[/B]

n^2+n = odd
n^2+n = odd Factor n^2+n: n(n + 1) We have one of two scenarios: [LIST=1] [*]If n is odd, then n + 1 is even. The product of an odd and even number is an even number [*]If n is even, then n + 1 is odd. The product of an even and odd number is an even number [/LIST]

n^2-n = even
n^2-n = even Factor n^2-n: n(n - 1) We have one of two scenarios: [LIST=1] [*]If n is odd, then n - 1 is even. The product of an odd and even number is an even number [*]If n is even, then n - 1 is odd. The product of an even and odd number is an even number [/LIST]

N^2=5qd for d
N^2=5qd for d Divide each side by 5q to isolate d: N^2/5q = 5qd/5q Cancel 5q on the right side and we get: d = [B]N^2/5q[/B]

Oakdale School is sponsoring a canned food drive. In the first week of the drive, the students colle
Oakdale School is sponsoring a canned food drive. In the first week of the drive, the students collected 638 cans. They collected 698 cans in the second week and 758 cans in the third week. If the students continue to collect cans at this rate, in which week will they collect more than 1,000 cans? We have an arithmetic sequence where each successive term increases by 50. [URL='https://www.mathcelebrity.com/sequenceag.php?num=638%2C698%2C758&n=10&pl=Calculate+Series&a1=5&d=3']Using our sequence calculator[/URL], we find that week #8 is when the students cross 1,000 cans.

Oceanside Bike Rental Shop charges $15.00 plus $9.00 per hour for renting a bike. Dan paid $51.00 to
Oceanside Bike Rental Shop charges $15.00 plus $9.00 per hour for renting a bike. Dan paid $51.00 to rent a bike. How many hours was he hiking for? Set up the cost equation C(h) where h is the number of hours needed to rent the bike: C(h) = Cost per hour * h + rental charge Using our given numbers in the problem, we have: C(h) = 9h + 15 The problem asks for h, when C(h) = 51. 9h + 15 = 51 To solve for h, we [URL='https://www.mathcelebrity.com/1unk.php?num=9h%2B15%3D51&pl=Solve']plug this equation into our search engine[/URL] and we get: h = [B]4[/B]

Oceanside Bike Rental Shop charges 16 dollars plus 6 dollars an hour for renting a bike. Mary paid 5
Oceanside Bike Rental Shop charges 16 dollars plus 6 dollars an hour for renting a bike. Mary paid 58 dollars to rent a bike. How many hours did she pay to have the bike checked out ? Set up the cost function C(h) where h is the number of hours you rent the bike: C(h) = Hourly rental cost * h + initial rental charge C(h) = 6h + 16 Now the problem asks for h when C(h) = 58, so we set C(h) = 58: 6h + 16 = 58 To solve this equation for h, we [URL='https://www.mathcelebrity.com/1unk.php?num=6h%2B16%3D58&pl=Solve']type it in our math engine[/URL] and we get: h = [B]7 hours[/B]

Octagonal Number
Free Octagonal Number Calculator - This calculator determines the nth octagonal number

Odds Probability
Free Odds Probability Calculator - Given an odds prediction m:n of an event success, this calculates the probability that the event will occur or not occur

Odds Ratio
Free Odds Ratio Calculator - This calculator determines the odds ratio for 2 groups X and Y with success and failure for an outcome.

Of all smokers in particular district, 40% prefer brand A and 60% prefer brand B. Of those who prefe
Of all smokers in particular district, 40% prefer brand A and 60% prefer brand B. Of those who prefer brand A, 30% are female, and of those who prefer brand B, 40% are female. Q: What is the probability that a randomly selected smoker prefers brand A, given that the person selected is a female? P(F) = P(F|A)*P(A) + P(F|B)*P(B) P(F) = 0.3*0.4 + 0.4*0.6 = 0.36 So, 36% of all the smokers are female. You are looking for P(A|F) P(A|F) = P(A and F)/P(F) P(A|F) = (P(F|A)*P(A))/P(F) P(A|F) = (0.3 * 0.4)/0.36 P(A|F) = [B]0.33 or 33%[/B]

Of the 20 boats at the Mariana, 10 were from Massachusetts. What is the probability that a randomly
Of the 20 boats at the Mariana, 10 were from Massachusetts. What is the probability that a randomly selected boat will be from Massachusetts? P(Boat from Massachusetts) = Number of Massachusetts boats / Total Boats at the Mariana P(Boat from Massachusetts) = 10/20 [URL='https://www.mathcelebrity.com/fraction.php?frac1=10%2F20&frac2=3%2F8&pl=Simplify']Simplifying this fraction, we get[/URL]: P(Boat from Massachusetts) = [B]1/2[/B]

Of the 800 students of a school, 600 have traveled. What percentage of the students went on a trip?
Of the 800 students of a school, 600 have traveled. What percentage of the students went on a trip? Our percentage is found as 600/800. Simplifying by dividing top and bottom by 100, we have: 6/8 Divide top and bottom by 2, we get: 3/4 or [B]75% [/B] You can also type in the [URL='http://www.mathcelebrity.com/perc.php?num=600&den=800&pcheck=1&num1=+16&pct1=+80&pct2=+35&den1=+90&pct=+82&decimal=+65.236&astart=+12&aend=+20&wp1=20&wp2=30&pl=Calculate']search engine[/URL]: 600/800 as percent.

Of the worlds 7.5 billion people, 1.2 billion people live on less than 4 per day. Calculate the perc
Of the worlds 7.5 billion people, 1.2 billion people live on less than 4 per day. Calculate the percent of the worlds population who lives on less than 4 per day? We want the percentage 1.2/7.5. [URL='https://www.mathcelebrity.com/perc.php?num=1.2&den=7.5&pcheck=1&num1=16&pct1=80&pct2=70&den1=80&idpct1=10&hltype=1&idpct2=90&pct=82&decimal=+65.236&astart=12&aend=20&wp1=20&wp2=30&pl=Calculate']Type this fraction into our search engine[/URL], choose percentage, and we get: [B]16%[/B]

ohn read the first 114 pages of a novel, which was 3 pages less than1/3 of the novel
Let p be the novel pages. We have 1/3p - 3 = 114 Add 3 to each side 1/3p = 117 Multiply each side by 3 p = 351

Olga wrote all the natural numbers from 1 to k. Including 1 and k. How many numbers did she write?
Olga wrote all the natural numbers from 1 to k. Including 1 and k. How many numbers did she write? The formula for the number of numbers including A to B is: B - A + 1 With A = 1 and B = k, we have: k - 1 + 1 [B]k[/B]

Oliver and Julia deposit $1,000.00 into a savings account which earns 14% interest compounded contin
Oliver and Julia deposit $1,000.00 into a savings account which earns 14% interest compounded continuously. They want to use the money in the account to go on a trip in 3 years. How much will they be able to spend? Use the formula A=Pert, where A is the balance (final amount), P is the principal (starting amount), e is the base of natural logarithms (?2.71828), r is the interest rate expressed as a decimal, and t is the time in years. Round your answer to the nearest cent. [URL='https://www.mathcelebrity.com/simpint.php?av=&p=1000&int=3&t=14&pl=Continuous+Interest']Using our continuous interest calculator[/URL], we get: A = [B]1,521.96[/B]

Oliver earns $50 per day plus $7.50 for each package he delivers. If his paycheck for the first day
Oliver earns $50 per day plus $7.50 for each package he delivers. If his paycheck for the first day was $140, how many packages did he deliver that day? His total earnings per day are the Flat Fee of $50 plus $7.50 per package delivered. We have: 50 + 7.50p = 140 where p = the number of packages delivered Using our [URL='http://www.mathcelebrity.com/1unk.php?num=50%2B7.50p%3D140&pl=Solve']equation solver[/URL], we have: [B]p = 12[/B]

Oliver invests $1,000 at a fixed rate of 7% compounded monthly, when will his account reach $10,000?
Oliver invests $1,000 at a fixed rate of 7% compounded monthly, when will his account reach $10,000? 7% monthly is: 0.07/12 = .00583 So we have: 1000(1 + .00583)^m = 10000 divide each side by 1000; (1.00583)^m = 10 Take the natural log of both sides; LN (1.00583)^m = LN(10) Use the identity for natural logs and exponents: m * LN (1.00583) = 2.30258509299 0.00252458479m = 2.30258509299 m = 912.064867899 Round up to [B]913 months[/B]

Olivia bought 20 notebooks. Her cost for all of the notebooks was 2.00$. If each notebook cost the s
Olivia bought 20 notebooks. Her cost for all of the notebooks was 2.00$. If each notebook cost the same amount then how much did she pay for one 20 notebooks / 2 Divide top and bottom by 20: 1 notebook = 2/20 1 notebook = 1/10 1 notebook = [B]10 cents[/B]

Olivia spends 5 hours a day at school and sleeps for 9 hours a day. What fraction of the day does sh
Olivia spends 5 hours a day at school and sleeps for 9 hours a day. What fraction of the day does she have left for other activities? Write your answer as a fraction in its simplest form. Add up existing hours for school and sleep School + sleep = 5 + 9 = 14 hours Since there are 24 hours in a day, she has 24 - 14 = 10 hours remaining. The fraction we want is 10/24. But we can simplify this. Using our [URL='http://www.mathcelebrity.com/fraction.php?frac1=10%2F24&frac2=3%2F8&pl=Simplify']simplify fractions calculator[/URL], we get: [B]5/12[/B]

Omar mows lawns for $9.25 an hour. He spends $7.50 on gas for the mower. How much does he make if he
Omar mows lawns for $9.25 an hour. He spends $7.50 on gas for the mower. How much does he make if he works h hours? His revenue R(h) where h is the number of hours is denoted by: R(h) = Hourly Rate * h - Gas cost [B]R(h) = 9.25h - 7.50[/B]

Omar mows lawns for $9.25 per hour. He spends $7.50 on gas for the mower. How much does he make if h
Omar mows lawns for $9.25 per hour. He spends $7.50 on gas for the mower. How much does he make if he works h hours? We have the following profit equation: Profit = Revenue - Cost: Revenue = Hourly rate * number of hours [B]9.25h - 7.50[/B]

Omar's classroom has 2 closets. Each closet has 3 shelves. There are 5 backpacks on each shelf.
Omar's classroom has 2 closets. Each closet has 3 shelves. There are 5 backpacks on each shelf. 2 closets * 3 shelves per closet * 5 backpacks per shelf = [B]30 backpacks[/B]

On a Friday evening a pizza shop had orders for 4 pepperoni, 97 vegetable, and 335 cheese pizzas. If
On a Friday evening a pizza shop had orders for 4 pepperoni, 97 vegetable, and 335 cheese pizzas. If the 4 cooks each made an equal number of pizzas, how many pizzas did each cook make? Total Pizzas Made = 4 pepperoni + 97 vegetable + 335 cheese Total Pizzas Made = 436 Equal number of pizzas per cook = 436 pizzas / 4 cooks Equal number of pizzas per cook = [B]109[/B]

On a map, every 5 cm represents 250 kilometres. What distance would be represented by a 3 cm line?
On a map, every 5 cm represents 250 kilometres. What distance would be represented by a 3 cm line? We set up a proportion of map cm distance to kilometers where k is the kilometers represented by a 3cm line 5/250 = 3/k To solve this proportion for k, we [URL='https://www.mathcelebrity.com/prop.php?num1=5&num2=3&den1=250&den2=k&propsign=%3D&pl=Calculate+missing+proportion+value']type it in our search engine[/URL] and we get: k = [B]150[/B]

On a Math test, 12 students earned an A. This number is exactly 25% of the total number of students
On a Math test, 12 students earned an A. This number is exactly 25% of the total number of students in the class. How many students are in the class? Let the total number of students be s. Since 25% is 0.25 as a decimal, We have an equation: 0.25s = 12 [URL='https://www.mathcelebrity.com/1unk.php?num=0.25s%3D12&pl=Solve']Type this equation into our search engine[/URL], and we get: s = [B]48[/B]

On a particular road map, 1/2 inch represents 18 miles. About how many miles apart are 2 towns that
On a particular road map, 1/2 inch represents 18 miles. About how many miles apart are 2 towns that are 2 1/2 inches apart on this map? A) 18 B) 22 1/2 C) 36 D) 45 E) 90 Set up a proportion of inches to miles where m is the number of miles for 2 1/2 inches. Note: 1/2 = 0.5 and 2 1/2 = 2.5 0.5/18 = 2.5/m [URL='https://www.mathcelebrity.com/prop.php?num1=0.5&num2=2.5&den1=18&den2=m&propsign=%3D&pl=Calculate+missing+proportion+value']Typing this proportion into the search engine[/URL], we get: [B]m = 90 Answer E[/B]

On an algebra test, the highest grade was 42 points higher than the lowest grade. The sum of the two
On an algebra test, the highest grade was 42 points higher than the lowest grade. The sum of the two grades was 138. Find the lowest grade. [U]Let h be the highest grade and l be the lowest grade. Set up the given equations:[/U] (1) h = l + 42 (2) h + l = 138 [U]Substitute (1) into (2)[/U] l + 42 + l = 138 [U]Combine l terms[/U] 2l + 42 = 138 [U]Enter that equation into our [URL='http://www.mathcelebrity.com/1unk.php?num=2l%2B42%3D138&pl=Solve']equation calculator[/URL] to get[/U] [B]l = 48 [/B] [U]Substitute l = 48 into (1)[/U] h = 48 + 42 [B]h = 90[/B]

On her 10 mile trip to school, Jessica's car gets 50 mpg of gas. On her way home, her car gets 40 mi
On her 10 mile trip to school, Jessica's car gets 50 mpg of gas. On her way home, her car gets 40 miles per gallon. How many miles per gallon does Jessica's car get during the entire 20 mile trip? 50 miles each gallon for a 10 mile drive = 1/5 gallon 40 miles each gallon for a 10 mile drive = 1/4 gallon [URL='https://www.mathcelebrity.com/fraction.php?frac1=1%2F4&frac2=1%2F5&pl=Add']1/4 + 1/5[/URL] = 9/20 20 miles driven /9/20 gallon = 400/9 = [B]44.44 miles per gallon[/B]

On January 1st a town has 75,000 people and is growing exponentially by 3% every year. How many peop
On January 1st a town has 75,000 people and is growing exponentially by 3% every year. How many people will live there at the end of 10 years? [URL='https://www.mathcelebrity.com/population-growth-calculator.php?num=atownhasapopulationof75000andgrowsat3%everyyear.whatwillbethepopulationafter10years&pl=Calculate']Using our population growth calculator[/URL], we get: [B]100,794[/B]

On July 10, the high temperature in Phoenix, Arizona, was 109°, and the high temperature in Juneau,
On July 10, the high temperature in Phoenix, Arizona, was 109°, and the high temperature in Juneau, Alaska, was 63°. What was the difference between the temperature in Phoenix and the temperature in Juneau? Difference is found by subtracting the lower temperature from the higher temperature: [URL='https://www.mathcelebrity.com/longdiv.php?num1=109&num2=63&pl=Subtract']109 - 63 [/URL]= [B]46[/B]

On Melissa 6 birthday she gets a $2000 cd that earns 4% interest, compounded semiannual. If the cd m
On Melissa 6 birthday she gets a $2000 cd that earns 4% interest, compounded semiannual. If the cd matures on her 16th birthday, how much money will be available? Semiannual compounding means twice a year. With 16 - 6 = 10 years of compounding, we have: 10 x 2 = 20 semiannual periods. [URL='https://www.mathcelebrity.com/compoundint.php?bal=2000&nval=20&int=4&pl=Semi-Annually']Using our interest on balance calculator[/URL], we get: [B]$2,971.89[/B]

On Monday 208 student went on a trip to the zoo . All 5 buses were filled and 8 student had to trave
On Monday 208 student went on a trip to the zoo . All 5 buses were filled and 8 student had to travel in car . How many student were in each bus? Calculate the number of students who traveled by bus: Total bus Students = Total Students - Total Car Students Total bus Students = 208 - 8 Total bus Students = 200 Figure how the students per bus: Students per bus = Total Bus Students / Number of Filled Busses Students per bus = 200/5 Students per bus = [B]40[/B]

On Monday the office staff at your school paid 8.77 for 4 cups of coffee and 7 bagels. On Wednesday
On Monday the office staff at your school paid 8.77 for 4 cups of coffee and 7 bagels. On Wednesday they paid 15.80 for 8 cups of coffee and 14 bagels. Can you determine the cost of a bagel Let the number of cups of coffee be c Let the number of bagels be b. Since cost = Price * Quantity, we're given two equations: [LIST=1] [*]7b + 4c = 8.77 [*]14b + 8c = 15.80 [/LIST] We have a system of equations. We can solve this 3 ways: [LIST=1] [*][URL='https://www.mathcelebrity.com/simultaneous-equations.php?term1=7b+%2B+4c+%3D+8.77&term2=14b+%2B+8c+%3D+15.80&pl=Substitution']Substitution Method[/URL] [*][URL='https://www.mathcelebrity.com/simultaneous-equations.php?term1=7b+%2B+4c+%3D+8.77&term2=14b+%2B+8c+%3D+15.80&pl=Elimination']Elimination Method[/URL] [*][URL='https://www.mathcelebrity.com/simultaneous-equations.php?term1=7b+%2B+4c+%3D+8.77&term2=14b+%2B+8c+%3D+15.80&pl=Cramers+Method']Cramer's Rule[/URL] [/LIST] No matter which method we use, we get the same answer [LIST] [*]The system is inconsistent. Therefore, we have no answer. [/LIST]

On Monday, 417 students went on a trip to the zoo. All 7 buses were filled and 4 students had to tra
On Monday, 417 students went on a trip to the zoo. All 7 buses were filled and 4 students had to travel in cars. How many students were in each bus? 417 - 4 in cars = 413 413 students remaining / 7 cars = [B]59 students per bus[/B]

On the day of a child's birth, a deposit of $25,000 is made in a trust fund that pays 8.5% interest.
On the day of a child's birth, a deposit of $25,000 is made in a trust fund that pays 8.5% interest. Determine that balance in this account on the child's 25th birthday. Using our [URL='https://www.mathcelebrity.com/compoundint.php?bal=25000&nval=25&int=8.5&pl=Annually']compound interest calculator[/URL], we get: [B]192,169.06 [/B]

On the first day of school each student in the class of 26 will bring 4 writing books and 2 maths bo
On the first day of school each student in the class of 26 will bring 4 writing books and 2 maths books. How many books will they have altogether? Each student has 4 books plus 2 math books = 6 total books per student Calculate total books Total Books = Number of students * books per student Total Books = 26 * 6 Total Books = [B]156[/B]

On the first day of ticket sales the school sold 3 senior citizen tickets and 10 child tickets for a
On the first day of ticket sales the school sold 3 senior citizen tickets and 10 child tickets for a total of $82. The school took in $67 on the second day by selling 8 senior citizen tickets And 5 child tickets. What is the price of each ticket? Let the number of child tickets be c Let the number of senior citizen tickets be s We're given two equations: [LIST=1] [*]10c + 3s = 82 [*]5c + 8s = 67 [/LIST] We have a system of simultaneous equations. We can solve it using any one of 3 ways: [LIST] [*][URL='https://www.mathcelebrity.com/simultaneous-equations.php?term1=10c+%2B+3s+%3D+82&term2=5c+%2B+8s+%3D+67&pl=Substitution']Substitution Method[/URL] [*][URL='https://www.mathcelebrity.com/simultaneous-equations.php?term1=10c+%2B+3s+%3D+82&term2=5c+%2B+8s+%3D+67&pl=Elimination']Elimination Method[/URL] [*][URL='https://www.mathcelebrity.com/simultaneous-equations.php?term1=10c+%2B+3s+%3D+82&term2=5c+%2B+8s+%3D+67&pl=Cramers+Method']Cramer's Rule[/URL] [/LIST] No matter what method we choose, we get: [LIST] [*][B]c = 7[/B] [*][B]s = 4[/B] [/LIST]

On the math test, Ralph answered 17 out of 20 problems. What percent did he get right?
On the math test, Ralph answered 17 out of 20 problems. What percent did he get right? Using our [URL='http://www.mathcelebrity.com/perc.php?num=17&den=20&pcheck=1&num1=+16&pct1=+80&pct2=+35&den1=+90&pct=+82&decimal=+65.236&astart=+12&aend=+20&wp1=20&wp2=30&pl=Calculate']percentage calculator or entering the phrase 17 out of 20 in the search engine[/URL], we get: [B]85%[/B]

One bag of cat food will feed 3 cats for 40 days. If I buy 2 more cats, how long will on bag of cat
One bag of cat food will feed 3 cats for 40 days. If I buy 2 more cats, how long will on bag of cat food last? If 1 bag of cat food feeds 3 cats for 40 days, then 1 bag of cat food will feed 1 cat for 3 * 40 days = 120 days. Therefore, if I buy 2 more cats, I have 3 + 2 = 5 cats. 120 days / 5 cats = [B]24 days of food[/B]

One day a quarter of the class is absent and 21 children are present. How many children are there on
One day a quarter of the class is absent and 21 children are present. How many children are there on the class when no one is away? If 1/4 of the class is absent, this means that 1 - 1/4 is present. Since 1 = 4/4, we have 4/4 - 1/4 = 3/4 of the class is present. If the full size of the class is c, then we have 3/4c = 21 [URL='https://www.mathcelebrity.com/1unk.php?num=3%2F4c%3D21&pl=Solve']Typing 3/4c = 21 into the search engine[/URL], we get: [B]c = 28[/B]

One fifth of the square of a number
One fifth of the square of a number We have an algebraic expression. Let's break this into parts. [LIST=1] [*]The phrase [I]a number[/I] means an arbitrary variable, let's call it x [*]The square of a number means we raise it to the power of 2. So we have x^2 [*]One-fifth means we have a fraction, where we divide our x^2 in Step 2 by 5. So we get our final answer below: [/LIST] [B]x^2/5[/B]

One number exceeds another by 15. The sum of the numbers is 51. What are these numbers
One number exceeds another by 15. The sum of the numbers is 51. What are these numbers? Let the first number be x, and the second number be y. We're given two equations: [LIST=1] [*]x = y + 15 [*]x + y = 51 [/LIST] Plug (1) into (2) (y + 15) + y = 51 Combine like terms: 2y + 15 = 51 [URL='https://www.mathcelebrity.com/1unk.php?num=2y%2B15%3D51&pl=Solve']Plug this equation into the search engine[/URL] and we get: [B]y = 18[/B] Now plug this into (1) to get: x = 18 + 15 [B]x = 33[/B]

One number is 1/4 of another number. The sum of the two numbers is 25. Find the two numbers. Use a c
One number is 1/4 of another number. The sum of the two numbers is 25. Find the two numbers. Use a comma to separate your answers. Let the first number be x and the second number be y. We're given: [LIST=1] [*]x = 1/4y [*]x + y = 25 [/LIST] Substitute (1) into (2) 1/4y + y = 25 Since 1/4 = 0.25, we have: 0.25y + y = 25 [URL='https://www.mathcelebrity.com/1unk.php?num=0.25y%2By%3D25&pl=Solve']Type this equation into the search engine[/URL] to get: [B]y = 20 [/B] Now, substitute this into (1) to solve for x: x = 1/4y x = 1/4(20) [B]x = 5 [/B] The problem asks us to separate the answers by a comma. So we write this as: [B](x, y) = (5, 20)[/B]

One number is 1/5 of another number. The sum of the two numbers is 18. Find the two numbers.
One number is 1/5 of another number. The sum of the two numbers is 18. Find the two numbers. Let the two numbers be x and y. We're given: [LIST=1] [*]x = 1/5y [*]x + y = 18 [/LIST] Substitute (1) into (2): 1/5y + y = 18 1/5 = 0.2, so we have: 1.2y = 18 [URL='https://www.mathcelebrity.com/1unk.php?num=1.2y%3D18&pl=Solve']Type 1.2y = 18 into the search engine[/URL], and we get [B]y = 15[/B]. Which means from equation (1) that: x = 15/5 [B]x = 3 [/B] Our final answer is [B](x, y) = (3, 15)[/B]

One number is 3 times another. Their sum is 44.
One number is 3 times another. Their sum is 44. Let the first number be x, and the second number be y. We're given: [LIST=1] [*]x = 3y [*]x + y = 44 [/LIST] Substitute (1) into (2): 3y + y = 44 [URL='https://www.mathcelebrity.com/1unk.php?num=3y%2By%3D44&pl=Solve']Type this equation into the search engine[/URL], and we get: [B]y = 11[/B] Plug this into equation (1): x = 3(11) [B]x = 33[/B]

one number is 3 times as large as another. Their sum is 48. Find the numbers
one number is 3 times as large as another. Their sum is 48. Find the numbers Let the first number be x. Let the second number be y. We're given two equations: [LIST=1] [*]x = 3y [*]x + y = 48 [/LIST] Substitute equation (1) into equation (2): 3y + y = 48 To solve for y, [URL='https://www.mathcelebrity.com/1unk.php?num=3y%2By%3D48&pl=Solve']we type this equation into the search engine[/URL] and we get: [B]y = 12[/B] Now, plug y = 12 into equation (1) to solve for x: x = 3(12) [B]x = 36[/B]

One number is 4 times the other number
Let one number be x, and the other number be y [B]x = 4y[/B]

One number is 8 times another number. The numbers are both positive and have a difference of 70.
One number is 8 times another number. The numbers are both positive and have a difference of 70. Let the first number be x, the second number be y. We're given: [LIST=1] [*]x = 8y [*]x - y = 70 [/LIST] Substitute(1) into (2) 8y - y = 70 [URL='https://www.mathcelebrity.com/1unk.php?num=8y-y%3D70&pl=Solve']Plugging this equation into our search engine[/URL], we get: [B]y = 10[/B] <-- This is the smaller number Plug this into Equation (1), we get: x = 8(10) [B]x = 80 [/B] <-- This is the larger number

One number is equal to the square of another. Find the numbers if both are positive and their sum is
One number is equal to the square of another. Find the numbers if both are positive and their sum is 650 Let the number be n. Then the square is n^2. We're given: n^2 + n = 650 Subtract 650 from each side: n^2 + n - 650 = 0 We have a quadratic equation. [URL='https://www.mathcelebrity.com/quadratic.php?num=n%5E2%2Bn-650%3D0&pl=Solve+Quadratic+Equation&hintnum=+0']We type this into our search engine[/URL] and we get: n = 25 and n = -26 Since the equation asks for a positive solution, we use [B]n = 25[/B] as our first solution. the second solution is 25^2 = [B]625[/B]

one number is twice a second number. the sum of those numbers is 45
one number is twice a second number. the sum of those numbers is 45. Let the first number be x and the second number be y. We're given: [LIST=1] [*]x = 2y [*]x + y = 45 [/LIST] Substitute Equation (1) into Equation (2): 2y + y = 45 [URL='https://www.mathcelebrity.com/1unk.php?num=2y%2By%3D45&pl=Solve']Typing this equation into our search engine[/URL], we get: [B]y = 15[/B] Plug this into equation (1) to solve for x, and we get: x = 2(15) [B]x = 30[/B]

One positive number is one-fifth of another number. The difference between the two numbers is 192, f
One positive number is one-fifth of another number. The difference between the two numbers is 192, find the numbers. Let the first number be x and the second number be y. We're given two equations: [LIST=1] [*]x = y/5 [*]x + y = 192 [/LIST] Substitute equation 1 into equation 2: y/5 + y = 192 Since 1 equals 5/5, we rewrite our equation like this: y/5 = 5y/5 = 192 We have fractions with like denominators, so we add the numerators: (1 + 5)y/5 = 192 6y/5 = 192 [URL='https://www.mathcelebrity.com/prop.php?num1=6y&num2=192&den1=5&den2=1&propsign=%3D&pl=Calculate+missing+proportion+value']Type this equation into our search engine[/URL], and we get: [B]y = 160[/B] Substitute this value into equation 1: x = 160/5 x = [B]32[/B]

One third of the bagels in a bakery are sesame bagels. There are 72 sesame bagels.
One third of the bagels in a bakery are sesame bagels. There are 72 sesame bagels. Set up our equation where b is the number of total bagels 72 = b/3 Multiply each side by 3 [B]b = 216[/B]

one third of the sum of 4 and P
The sum of 4 and p is written as: 4 + p We then take 1/3 of that, or multiply: 1/3(4 + p)

One thousand people in. room decide to shake hands with every other person in the room. Instead of o
One thousand people in. room decide to shake hands with every other person in the room. Instead of one handshake per couple, each person must shake both of the hands of every person in the room with both his right and his left hand. (Tom will use his right hand to shake Dave's right hand and then Dave's left hand. Tom will then use his left hand to shake Dave's right hand and then Dave's left hand.) How many total handshakes will take place? 1000 people taken 2 at a time: [URL='https://www.mathcelebrity.com/permutation.php?num=1000&den=2&pl=Combinations']1000C2[/URL] = 499,500 But each group of 2 makes 4 unique handshakes: 499,500 * 4 = [B]1,998,000[/B]

one-fifth of forty-five
one-fifth of forty-five one-fifth is 1/4 forty-five is 45 When you see a fraction then the word of and then a number, it means you multiply: 1/5 * 45 45/5 [B]9[/B]

One-fourth the sum of m and p
One-fourth the sum of m and p Take this algebraic expression in parts: [LIST] [*]The sum of m and p means we add p to m: m + p [*]1/4 of the sum mean we divide m + p by 4 [/LIST] [B](m + p)/4[/B]

One-half a number is fifty
The phrase [I]a number[/I] means an arbitrary variable. We can pick any letter a-z except for i and e. Let's choose x. One-half a number means we divide x by2: x/2 The word [I]is[/I] means equal to. We set x/2 equal to 50 for our algebraic expression [B]x/2 = 50 [/B] If the problem asks us to solve for x, we cross multiply: x = 2 * 50 x = [B]100[/B]

One-half a number times fifteen
The phrase [I]a number[/I] means an arbitrary variable. We can pick any letter a-z except for i and e. Let's choose x. One-half a number means we multiply x by 1/2: x/2 Times fifteen means we multiply: [B]15x/2[/B]

One-half the sum of 5 and t
One-half the sum of 5 and t The sum of 5 and t: 5 + t One-half of this means we multiply 5 + t by 1/2 [B](5 + t)/2[/B]

One-third a number less two
The phrase [I]a number[/I] means an arbitrary variable. We can pick any letter a-z except for i and e. Let's choose x. One-third a number means we multiply x by 1/3: x/3 Less two means we subtract 2 [B]x/3 - 2[/B]

Opposite Numbers
Free Opposite Numbers Calculator - Given a positive or negative integer (n), this calculates the opposite number of n

opposite of twice the quotient of a and a
opposite of twice the quotient of a and a the quotient of a and a: a/a 1 Twice the quotient of a and a 2(1) 2 Opposite means multiply 2 by -1: -1 * 2 [B]-2[/B]

Orange Theory is currently offering a deal where you can buy a fitness pass for $100 and then each c
Orange Theory is currently offering a deal where you can buy a fitness pass for $100 and then each class is $13, otherwise it is $18 for each class. After how many classes is the total cost with the fitness pass the same as the total cost without the fitness pass? Let the number of classes be c. For the fitness pass plan, we have the total cost of: 13c + 100 For the flat rate plan, we have the total cost of: 18c The question asks for c when both plans are equal. So we set both costs equal and solve for c: 13c + 100 = 18c We [URL='https://www.mathcelebrity.com/1unk.php?num=13c%2B100%3D18c&pl=Solve']type this equation into our math engine[/URL] and we get: c = [B]20[/B]

Order of Operations
Free Order of Operations Calculator - Evaluates an expression using the order of operations, or PEMDAS or PEDMAS or BEDMAS or BODMAS

Ordered Pair
Free Ordered Pair Calculator - This calculator handles the following conversions:
* Ordered Pair Evaluation and symmetric points including the abcissa and ordinate
* Polar coordinates of (r,θ°) to Cartesian coordinates of (x,y)
* Cartesian coordinates of (x,y) to Polar coordinates of (r,θ°)
* Quadrant (I,II,III,IV) for the point entered.
* Equivalent Coordinates of a polar coordinate
* Rotate point 90°, 180°, or 270°
* reflect point over the x-axis
* reflect point over the y-axis
* reflect point over the origin

Ordering Numbers
Free Ordering Numbers Calculator - Given a list of numbers, this will order the list ascending (lowest to highest or least to greatest) or descending (highest to lowest or greatest to least)

Ordinal Number
Free Ordinal Number Calculator - This calculator determines the ordinal number of an integer

Oscar makes a large purchase at Home Depot and plans to rent one of its trucks to take his supplies
Oscar makes a large purchase at Home Depot and plans to rent one of its trucks to take his supplies home. The most he wants to spend on the truck is $56.00. If Home Depot charges $17.00 for the first 75 minutes and $5.00 for each additional 15 min, for how long can Oscar keep the truck and remain within his budget? Set up the cost equation C(m) where m is the number of minutes for rental: C(m) = 17 * min(m, 75) + max(0, 5(m - 75)) If Oscar uses the first 75 minutes, he spends $17. So he's left with: $56 - $17 = $38 $38 / $5 = 7 Remainder 3 We remove the remainder 3, since it's not a full 15 minute block. So Oscar can rent the truck for: 7 * 15 minute blocks = [B]105 minutes[/B]

our recipe calls for 2 eggs and 3 cups of sugar. if we want to use 5 eggs, how much sugar will we ne
Our recipe calls for 2 eggs and 3 cups of sugar. if we want to use 5 eggs, how much sugar will we need? Set up a relational proportion for eggs to cups of sugar where s is the number of cups of sugar we need for 5 eggs. 2/3 = 5/s [URL='https://www.mathcelebrity.com/prop.php?num1=2&num2=5&den1=3&den2=s&propsign=%3D&pl=Calculate+missing+proportion+value']Plugging this into the search engine[/URL], we get [B]7.5 cups of sugar[/B].

Out of 53 teachers 36 drink tea 18 drink coffee, 10 drink neither. how many drink both?
Out of 53 teachers 36 drink tea 18 drink coffee, 10 drink neither. how many drink both? Let T be tea drinkers Let C be coffee drinkers Let (T & C) be Tea & Coffee drinkers. And 53 are total. So we use the Union formula relation: C U T = C + T - (C & T) 53 = 18 + 36 - (C & T) C & T = 53 - (Not C & Not T) since we subtract people who don't drink coffee and don't drink tea C & T = 53 - 10 = 43 C U T = 18 + 36 - 43 C U T = [B]11[/B]

Out of the 485 Cookies for the bake sale, 2/5 were chocolate chip. Estimate the number of chocolate
Out of the 485 Cookies for the bake sale, 2/5 were chocolate chip. Estimate the number of chocolate chips We want 2/5 of 485. We [URL='https://www.mathcelebrity.com/fraction.php?frac1=485&frac2=2/5&pl=Multiply']type this in our search engine[/URL] and we get; [B]194[/B]

output is 3 times the input x
output is 3 times the input x Let output be y. We have: [B]y = 3x[/B]

p = i^2r for r
p = i^2r for r Divide each side of the equation by i^2 to isolate r: p/i^2 = i^2/ri^2 Cancel the i^2 on the right side and we get: r = [B]p/i^2[/B]

p decreased by 65 is the same as the total of f and 194
p decreased by 65 is the same as the total of f and 194 p decreased by 65 p - 65 The total of f and 194 f + 194 The phrase [I]is the same as[/I] means equal to, so we set the expressions above equal to each other [B]p - 65 = f + 194[/B]

P is the natural numbers that are factors of 25
P is the natural numbers that are factors of 25 we type in [I][URL='https://www.mathcelebrity.com/factoriz.php?num=25&pl=Show+Factorization']factor 25[/URL][/I] into our math engine and we get: {1, 5, 25} Since [U]all[/U] of these are natural numbers, our answer is: [B]{1, 5, 25}[/B]

P is twice the length plus twice the width
P is twice the length plus twice the width Let the length be l. Let the width be w. The phrase [I]twice[/I] means we multiply by 2. We have: [B]2l + 2w = P[/B]

p more than the square of q
p more than the square of q Take this algebraic expression in parts: Step 1: Square of q means raise q to the 2nd power: q^2 Step 2: The phrase [I]more[/I] means we add p to q^2 [B]q^2 + p[/B]

P varies directly as q and the square of r and inversely as s
P varies directly as q and the square of r and inversely as s There exists a constant k such that: p = kqr^2/s [I]Note: Direct variations multiply and inverse variations divide[/I]

p varies directly as the square of r and inversely as q and s and p = 40 when q = 5, r = 4 and s = 6
p varies directly as the square of r and inversely as q and s and p = 40 when q = 5, r = 4 and s = 6, what is the equation of variation? Two rules of variation: [LIST=1] [*]Varies directly means we multiply [*]Varies inversely means we divide [/LIST] There exists a constant k such that our initial equation of variation is: p = kr^2/qs [B][/B] With p = 40 when q = 5, r = 4 and s = 6, we have: 4^2k / 5 * 6 = 40 16k/30 = 40 Cross multiply: 16k = 40 * 30 16k = 1200 Using our [URL='https://www.mathcelebrity.com/1unk.php?num=16k%3D1200&pl=Solve']equation calculator[/URL], we get: k = [B]75[/B] So our final equation of variation is: [B]p = 75r^2/qs[/B]

p(t)=6t represent the number of people p(t) that a number of turkeys can feed at Thanksgiving. How m
p(t)=6t represent the number of people p(t) that a number of turkeys can feed at Thanksgiving. How many people can 6 turkeys feed? Plug in t = 6 p(6) = 6(6) p(6) = 36

p(x)=2x-5 find the domain
p(x)=2x-5 find the domain Using our[URL='http://www.mathcelebrity.com/function-calculator.php?num=2x-5&pl=Calculate'] function calculator[/URL]: [B]All real numbers[/B]

P-Hat Confidence Interval
Free P-Hat Confidence Interval Calculator - Given a large sized distribution, and a success amount for a certain criteria x, and a confidence percentage, this will calculate the confidence interval for that criteria.

p/q = f/q- f for f
p/q = f/q- f for f Isolate f in this literal equation. Factor out f on the right side: p/q = f(1/q - 1) Rewriting the term in parentheses, we get: p/q = f(1 - q)/q Cross multiply: f = pq/q(1 - q) Cancelling the q/q on the right side, we get: f = [B]p/(1 - q)[/B]

p/q=f/q-f for f
p/q=f/q-f for f To solve this literal equation for f, let's factor out f on the right side: p/q=f(1/q-1) Divide each side by (1/q - 1) p/(q(1/q - 1)) = f(1/q-1)/(1/q - 1) Cancelling the (1/q - 1) on the right side, we get: f = p/(1/q - 1) Rewriting this since (1/q -1) = (1 - q)/q since q/q = 1 we have: f = [B]pq/(1 - q)[/B]

P/v=nr/t for r
P/v=nr/t for r Cross multiply to solve this literal equation: Pt = nrv Divide each side of the equation by nv: Pt/nv = nrv/nv Cancel the nv's on the right side, we get: r = [B]Pt/nv[/B]

P=15+5d/11 for d
Subtract 15 from each side: 5d/11 = P - 15 Multiply each side by 11 5d = 11p - 165 Divide each side of the equation by d: d = (11p - 165) ------------ 5

Pablo is saving money to buy a game. So far he has saved $22, which is one-half of the total cost
Pablo is saving money to buy a game. So far he has saved $22, which is one-half of the total cost of the game. How much does the game cost? 22 is 1/2 of the cost, so multiply 22 * 2 to get the [B]full cost of $44[/B].

Pam has two part-time jobs. At one job, she works as a cashier and makes $8 per hour. At the second
Pam has two part-time jobs. At one job, she works as a cashier and makes $8 per hour. At the second job, she works as a tutor and makes$12 per hour. One week she worked 30 hours and made$268 . How many hours did she spend at each job? Let the cashier hours be c. Let the tutor hours be t. We're given 2 equations: [LIST=1] [*]c + t = 30 [*]8c + 12t = 268 [/LIST] To solve this system of equations, we can use 3 methods: [LIST] [*][URL='https://www.mathcelebrity.com/simultaneous-equations.php?term1=c+%2B+t+%3D+30&term2=8c+%2B+12t+%3D+268&pl=Substitution']Substitution Method[/URL] [*][URL='https://www.mathcelebrity.com/simultaneous-equations.php?term1=c+%2B+t+%3D+30&term2=8c+%2B+12t+%3D+268&pl=Elimination']Elimination Method[/URL] [*][URL='https://www.mathcelebrity.com/simultaneous-equations.php?term1=c+%2B+t+%3D+30&term2=8c+%2B+12t+%3D+268&pl=Cramers+Method']Cramer's Rule[/URL] [/LIST] No matter which method we use, we get the same answer: [LIST] [*]c = [B]23[/B] [*]t = [B]7[/B] [/LIST]

Par on a golf course is 72. If a golfer shot rounds of 74, 70, and 71 in a tournament, what will she
Par on a golf course is 72. If a golfer shot rounds of 74, 70, and 71 in a tournament, what will she need to shoot on the final round to average par? Par is the word for average in golf. We have a missing average problem. Using our [URL='http://www.mathcelebrity.com/missingaverage.php?num=74%2C70%2C71&avg=72&pl=Calculate+Missing+Score']missing average calculator[/URL], we need to shoot a [B]73[/B].

Parabolas
Free Parabolas Calculator - Determines the focus, directrix, and other related items for a parabola.

Parallel Resistors
Free Parallel Resistors Calculator - Given a set of parallel resistors, this calculates the total resistance in ohms, denoted Rt

Partial Quotient
Free Partial Quotient Calculator - Divides 2 numbers using the Partial Quotient

Pascal-Floyd-Leibniz Triangle
Free Pascal-Floyd-Leibniz Triangle Calculator - This generates the first (n) rows of the following triangles:
Pascal's Triangle
Leibniz's Harmonic Triangle
Floyd's Triangle

Pat starts reading at 1pm. He reads 5 pages in 15 minutes. If pat reads until 2:45 PM how many pages
Pat starts reading at 1pm. He reads 5 pages in 15 minutes. If pat reads until 2:45 PM how many pages has he read From 1PM to 2:45 PM is 1 hour and 45 minutes. Since 1 hour is 60 minutes, Pat reads 105 minutes. Calculate the 15 minute blocks: Blocks = Total Minutes / 15 Blocks = 105/15 Blocks = 7 Pat reads 5 pages for every 15 minute block. So we have: Total Pages Read = 5 pages * 7 blocks Total Pages Read = [B]35[/B]

Patricia has $425.82 in her checking account. How much does she have in her account after she makes
Patricia has $425.82 in her checking account. How much does she have in her account after she makes a deposit of $120.75 and a withdrawal of $185.90? Start with $425.82 Deposits mean we [B]add[/B] money to the bank account: 425.82 + 120.75 = 546.57 Our new balance is 546.57. Withdrawals mean we [B]subtract[/B] money from the bank account: 546.57 - 185.90 = [B]360.67[/B]

Paul can walk 15 steps in 5 minutes How long does it take Paul to walk 75 steps at the same speed
Paul can walk 15 steps in 5 minutes How long does it take Paul to walk 75 steps at the same speed Set up a proportion of steps to minutes where m is the number of minutes to walk 75 steps: 15/5 = 75/m To solve this proportion for m, we [URL='https://www.mathcelebrity.com/prop.php?num1=15&num2=75&den1=5&den2=m&propsign=%3D&pl=Calculate+missing+proportion+value']type it in our search engine[/URL] and we get: m = [B]25[/B]

Paul’s age is 7 years younger than half of Marina’s age. Express their ages.
Paul’s age is 7 years younger than half of Marina’s age. Express their ages. Assumptions: [LIST] [*]Let Paul's age be p [*]Let Marina's age be m [/LIST] Our expression is: [B]p = 1/2m - 7[/B]

Payback Period
Free Payback Period Calculator - Given a set of cash inflows and cash outflows at certain times, this determines the net cash flow, cumulative cash flow, and payback period

Pedro has r red peppers and 44 green peppers. Write an expression that shows how many peppers Pedro
Pedro has r red peppers and 44 green peppers. Write an expression that shows how many peppers Pedro has. Add them together: [B]r + 44[/B]

Penelope and Owen work at a furniture store. Penelope is paid $215 per week plus 3.5% of her total s
Penelope and Owen work at a furniture store. Penelope is paid $215 per week plus 3.5% of her total sales in dollars, xx, which can be represented by g(x)=215+0.035x. Owen is paid $242 per week plus 2.5% of his total sales in dollars, xx, which can be represented by f(x)=242+0.025x. Determine the value of xx, in dollars, that will make their weekly pay the same. Set the pay functions of Owen and Penelope equal to each other: 215+0.035x = 242+0.025x Using our [URL='http://www.mathcelebrity.com/1unk.php?num=215%2B0.035x%3D242%2B0.025x&pl=Solve']equation calculator[/URL], we get: [B]x = 2700[/B]

Penny bought a new car for $25,000. The value of the car has decreased in value at rate of 3% each
Penny bought a new car for $25,000. The value of the car has decreased in value at rate of 3% each year since. Let x = the number of years since 2010 and y = the value of the car. What will the value of the car be in 2020? Write the equation, using the variables above, that represents this situation and solve the problem, showing the calculation you did to get your solution. Round your answer to the nearest whole number. We have the equation y(x): y(x) = 25,000(0.97)^x <-- Since a 3 % decrease is the same as multiplying the starting value by 0.97 The problem asks for y(2020). So x = 2020 - 2010 = 10. y(10) = 25,000(0.97)^10 y(10) = 25,000(0.73742412689) y(10) = [B]18,435.60[/B]

Pentagonal Number
Free Pentagonal Number Calculator - This calculator determines the nth pentagonal number

Pentagons
Free Pentagons Calculator - Given a side length and an apothem, this calculates the perimeter and area of the pentagon.

People with a drivers license are at least 16 years old and no older than 85 years old
People with a drivers license are at least 16 years old and no older than 85 years old. Set up the inequality, where p represents the people: [LIST=1] [*]The phrase [I]at least[/I] means greater than or equal to. So we use the >= sign. 16 <= p [*]The phrase [I]no older than[/I] means less than or equal to. So we use the <= sign. p <= 85 [/LIST] Combine these inequalities, and we get: [B]16 <= p <= 85[/B] To see the interval notation for this inequality and all possible values, visit the [URL='https://www.mathcelebrity.com/interval-notation-calculator.php?num=16%3C%3Dp%3C%3D85&pl=Show+Interval+Notation']interval notation calculator[/URL].

Percent Error
Free Percent Error Calculator - Percentage error is the difference between an experimental measured value and a theoretical actual value

Percent Off Problem
Free Percent Off Problem Calculator - Given the 3 items of a percent word problem, Reduced Price, percent off, and full price, this solves for any one of the three given two of the items.

Percentage Change
Free Percentage Change Calculator - Calculates the percentage change between two values. Percentage Increase or Percentage Decrease

Percentage of Completion
Free Percentage of Completion Calculator - Given a sales price, total costs, and costs per period, this determines the gross profit to date using the percentage of completion method.

Percentage of the Pie Word Problem
Free Percentage of the Pie Word Problem Calculator - This takes two or three fractions of ownership in some good or object, and figures out what remaining fraction is left over.

Percentage-Decimal-Fraction Relations
Free Percentage-Decimal-Fraction Relations Calculator - Calculates the relational items between a fraction, a decimal (including repeating decimal and terminating decimal), a percentage, and the numerator and denominator piece of that fraction. Also calculates the percentage change going from one number to another or the amount increase or decrease of a percentage above/below a number. Round decimals. decimals into fractions

Percentile for Normal Distribution
Free Percentile for Normal Distribution Calculator - Given a mean, standard deviation, and a percentile range, this will calculate the percentile value.

Percentiles
Free Percentiles Calculator - Given a set of scores and a target score, this will determine the percentile of the target score using two different formulas.

Perimeter of a rectangle is 372 yards. If the length is 99 yards, what is the width?
Perimeter of a rectangle is 372 yards. If the length is 99 yards, what is the width? The perimeter P of a rectangle with length l and width w is: 2l + 2w = P We're given P = 372 and l = 99, so we have: 2(99) + 2w = 372 2w + 198 = 372 [SIZE=5][B]Step 1: Group constants:[/B][/SIZE] We need to group our constants 198 and 372. To do that, we subtract 198 from both sides 2w + 198 - 198 = 372 - 198 [SIZE=5][B]Step 2: Cancel 198 on the left side:[/B][/SIZE] 2w = 174 [SIZE=5][B]Step 3: Divide each side of the equation by 2[/B][/SIZE] 2w/2 = 174/2 w = [B]87[/B]

Periodic Table Items
Free Periodic Table Items Calculator - Shows details of all the elements on the periodic table including atomic weight, natural state.

Permutations and Combinations
Free Permutations and Combinations Calculator - Calculates the following:
Number of permutation(s) of n items arranged in r ways = nPr
Number of combination(s) of n items arranged in r unique ways = nCr including subsets of sets

Permutations with Replacement
Free Permutations with Replacement Calculator - Calculates the following:
How many permutations can we have from a sample of r elements from a set of n distinct objects where order does matter and replacements are not allowed?

Perpetuity
Free Perpetuity Calculator - Walks you through the definition of a perpetuity, the present value of a perpetuity immediate, and the present value of a perpetuity due.

Pet supplies makes a profit of $5.50 per bag, if the store wants to make a profit of no less than $5
Pet supplies makes a profit of $5.50 per bag, if the store wants to make a profit of no less than $5225, how many bags does it need to sell? 5.5ob >= $5,225 Divide each side of the inequality by $5.50 b >=9.5 bags, so round up to a whole number of 10 bags.

Peter has $500 in his savings account. He purchased an iPhone that charged him $75 for his activatio
Peter has $500 in his savings account. He purchased an iPhone that charged him $75 for his activation fee and $40 per month to use the service on the phone. Write an equation that models the number of months he can afford this phone. Let m be the number of months. Our equation is: [B]40m + 75 = 500 [/B] <-- This is the equation [URL='https://www.mathcelebrity.com/1unk.php?num=40m%2B75%3D500&pl=Solve']Type this equation into the search engine[/URL], and we get: m = [B]10.625[/B] Since it's complete months, it would be 10 months.

Peter is buying office supplies. He is able to buy 3 packages of paper and 4 staplers for $40, or he
Peter is buying office supplies. He is able to buy 3 packages of paper and 4 staplers for $40, or he is able to buy 5 packages of paper and 6 staplers for $62. How much does a package of paper cost? How much does a stapler cost? Let the cost of paper packages be p and the cost of staplers be s. We're given two equations: [LIST=1] [*]3p + 4s = 40 [*]5p + 6s = 62 [/LIST] We have a system of equations. We can solve this three ways: [LIST] [*][URL='https://www.mathcelebrity.com/simultaneous-equations.php?term1=3p+%2B+4s+%3D+40&term2=5p+%2B+6s+%3D+62&pl=Substitution']Substitution Method[/URL] [*][URL='https://www.mathcelebrity.com/simultaneous-equations.php?term1=3p+%2B+4s+%3D+40&term2=5p+%2B+6s+%3D+62&pl=Elimination']Elimination Method[/URL] [*][URL='https://www.mathcelebrity.com/simultaneous-equations.php?term1=3p+%2B+4s+%3D+40&term2=5p+%2B+6s+%3D+62&pl=Cramers+Method']Cramer's Rule[/URL] [/LIST] No matter what method we choose, we get the same answer: [LIST] [*][B]p = 4[/B] [*][B]s = 7[/B] [/LIST]

Peter was thinking of a number. Peter doubles it and adds 0.8 to get an answer of 31. Form an equati
Peter was thinking of a number. Peter doubles it and adds 0.8 to get an answer of 31. Form an equation with x from the information. Take this algebraic expression in parts, starting with the unknown number x: [LIST] [*]x [*][I]Double it [/I]means we multiply x by 2: 2x [*]Add 0.8: 2x + 0.8 [*]The phrase [I]to get an answer of[/I] means an equation. So we set 2x + 0.8 equal to 31 [/LIST] Build our final algebraic expression: [B]2x + 0.8 = 31[/B] [B][/B] If you have to solve for x, then we [URL='https://www.mathcelebrity.com/1unk.php?num=2x%2B0.8%3D31&pl=Solve']type this equation into our search engine[/URL] and we get: x = 15.1

Peter’s Lawn Mowing Service charges $10 per job and $0.20 per square yard. Peter earns $25 for a job
Peter’s Lawn Mowing Service charges $10 per job and $0.20 per square yard. Peter earns $25 for a job. Let y be the number of square yards. We have the following equation: 0.2y + 10 = 25 To solve for y, we[URL='https://www.mathcelebrity.com/1unk.php?num=0.2y%2B10%3D25&pl=Solve'] type this equation into our search engine [/URL]and we get: y = [B]75[/B]

Phone Number Translator
Free Phone Number Translator Calculator - Given a phone number with letters in it, this calculator will determine the numeric phone number for you to dial.

Phonetic Algorithms
Free Phonetic Algorithms Calculator - Given a name, this calculator translates a name to one of the following 3 phonetic algorithms:
* Soundex
* Metaphone
* New York State Identification and Intelligence System (NYSIIS)

Phonograms
Free Phonograms Calculator - Shows the 75 basic phonograms of the English language

Phyllis's mother has 6 pounds of candy to divide evenly among her 8 children. This is an average of
Phyllis's mother has 6 pounds of candy to divide evenly among her 8 children. This is an average of how many pounds per child? 6 pounds divide among 8 children can be represented as a fraction. We want to simplify this. So we use our [URL='https://www.mathcelebrity.com/fraction.php?frac1=6%2F8&frac2=3%2F8&pl=Simplify']fraction simplify calculator[/URL], and we get: 3 pounds per 4 children, or 0.75 pounds per child.

PI
Free PI Calculator - This calculator performs operations with PI and gives you other options for π related calculations.

Pi
Get a free pi coin. Use the link below: I am sending you 1?! Pi is a new digital currency developed by Stanford PhDs, with over 47 million members worldwide. To claim your Pi, follow this link [URL]https://minepi.com/mathcelebrity[/URL] and use my username (mathcelebrity) as your invitation code.--

Pick's Theorem
Free Pick's Theorem Calculator - This calculator determines the area of a simple polygon using interior points and boundary points using Pick's Theorem

Pixels Per Inch PPI
Free Pixels Per Inch PPI Calculator - This calculator determines the PPI from width, height, and diagonal in inches

Place Value
Free Place Value Calculator - Given a whole number or a decimal, the calculator will perform place number analysis on each place in your number.
For the whole and decimal portion, the calculator goes out to the 100 trillion mark.

Plane and Parametric Equations in R3
Free Plane and Parametric Equations in R3 Calculator - Given a vector A and a point (x,y,z), this will calculate the following items:
1) Plane Equation passing through (x,y,z) perpendicular to A
2) Parametric Equations of the Line L passing through the point (x,y,z) parallel to A


Plants are sold in packs of 4. A container holds 2 packs. There are 8 containers. How many plants?
Plants are sold in packs of 4. A container holds 2 packs. There are 8 containers. How many plants? 8 containers * 2 packs / 1 container * 4 plants / 1 pack = [B]64 plants[/B]

Pleasantburg has a population growth model of P(t)=at2+bt+P0 where P0 is the initial population. Sup
Pleasantburg has a population growth model of P(t)=at^2+bt+P0 where P0 is the initial population. Suppose that the future population of Pleasantburg t years after January 1, 2012, is described by the quadratic model P(t)=0.7t^2+6t+15,000. In what month and year will the population reach 19,200? Set P(t) = 19,200 0.7t^2+6t+15,000 = 19,200 Subtract 19,200 from each side: 0.7t^2+6t+4200 = 0 The Quadratic has irrational roots. So I set up a table below to run through the values. At t = 74, we pass 19,200. Which means we add 74 years to 2012: 2012 + 74 = [B]2086[/B] t 0.7t^2 6t Add 15000 Total 1 0.7 6 15000 15006.7 2 2.8 12 15000 15014.8 3 6.3 18 15000 15024.3 4 11.2 24 15000 15035.2 5 17.5 30 15000 15047.5 6 25.2 36 15000 15061.2 7 34.3 42 15000 15076.3 8 44.8 48 15000 15092.8 9 56.7 54 15000 15110.7 10 70 60 15000 15130 11 84.7 66 15000 15150.7 12 100.8 72 15000 15172.8 13 118.3 78 15000 15196.3 14 137.2 84 15000 15221.2 15 157.5 90 15000 15247.5 16 179.2 96 15000 15275.2 17 202.3 102 15000 15304.3 18 226.8 108 15000 15334.8 19 252.7 114 15000 15366.7 20 280 120 15000 15400 21 308.7 126 15000 15434.7 22 338.8 132 15000 15470.8 23 370.3 138 15000 15508.3 24 403.2 144 15000 15547.2 25 437.5 150 15000 15587.5 26 473.2 156 15000 15629.2 27 510.3 162 15000 15672.3 28 548.8 168 15000 15716.8 29 588.7 174 15000 15762.7 30 630 180 15000 15810 31 672.7 186 15000 15858.7 32 716.8 192 15000 15908.8 33 762.3 198 15000 15960.3 34 809.2 204 15000 16013.2 35 857.5 210 15000 16067.5 36 907.2 216 15000 16123.2 37 958.3 222 15000 16180.3 38 1010.8 228 15000 16238.8 39 1064.7 234 15000 16298.7 40 1120 240 15000 16360 41 1176.7 246 15000 16422.7 42 1234.8 252 15000 16486.8 43 1294.3 258 15000 16552.3 44 1355.2 264 15000 16619.2 45 1417.5 270 15000 16687.5 46 1481.2 276 15000 16757.2 47 1546.3 282 15000 16828.3 48 1612.8 288 15000 16900.8 49 1680.7 294 15000 16974.7 50 1750 300 15000 17050 51 1820.7 306 15000 17126.7 52 1892.8 312 15000 17204.8 53 1966.3 318 15000 17284.3 54 2041.2 324 15000 17365.2 55 2117.5 330 15000 17447.5 56 2195.2 336 15000 17531.2 57 2274.3 342 15000 17616.3 58 2354.8 348 15000 17702.8 59 2436.7 354 15000 17790.7 60 2520 360 15000 17880 61 2604.7 366 15000 17970.7 62 2690.8 372 15000 18062.8 63 2778.3 378 15000 18156.3 64 2867.2 384 15000 18251.2 65 2957.5 390 15000 18347.5 66 3049.2 396 15000 18445.2 67 3142.3 402 15000 18544.3 68 3236.8 408 15000 18644.8 69 3332.7 414 15000 18746.7 70 3430 420 15000 18850 71 3528.7 426 15000 18954.7 72 3628.8 432 15000 19060.8 73 3730.3 438 15000 19168.3 74 3833.2 444 15000 19277.2

please answer my second word problem
A tortoise is walking in the desert. It walks at a speed of 4 meters per minute for 6.4 meters. For how many minutes does it walk?

please answer this word problem
Two trains leave the station at the same time, one heading east and the other west. The eastbound train travels at 105 miles per hour. The westbound train travels at 85 miles per hour. How long will it take for the two trains to be 494 miles apart?

please answer this word problem
Time 1, distance apart is 105 + 85 = 190 So every hour, the distance between them is 190 * t where t is the number of hours. Set up our distance function: D(t) = 190t We want D(t) = 494 190t = 494 Divide each side by 190 [B]t = 2.6 hours[/B]

Please help me!! I don't understand!
I don't understand this word problem: If each of these shapes in Figure 1 were separated and filled with water, could the sphere that contains the cube hold all of the water? [I]Assume in the second image the corners of the cube touch the sphere so the diagonal from one corner of the cube to the opposite diagonal corner is the diameter of the sphere. [IMG]https://classroom.ucscout.org/courses/1170/files/191225/preview?verifier=mT7v59BhdVHalyprWq0KmBEItbf4CPWFqOgwoEa8[/IMG][IMG]https://classroom.ucscout.org/courses/1170/files/191494/preview?verifier=nsLscsxToebAVXTSYsoMr7rwIl536LrCJSDGPaHp[/IMG][/I] Could you guys help me please?

Please help me!! I don't understand!
Figure 1, we have a cone, cylinder, and cube. Let's get the volume of each Cone volume = pir^2h/3 radius = s/2 h = s Cone Volume = pi(s/2)^2(s)/3 Cone Volume = pis^3/12 Volume of cube = s^3 Volume of cylinder = pir^2h Volume of cylinder = pi(s/2)^2s Volume of cylinder = pis^3/2 But Figure 2 has no sizes? For sides, height, etc. So I cannot answer the question until I have that.

Please help!!
Find the value of |A| if: (1) |P(A)| = 4 (2) |B| = |A|+ 1 and |A×B| = 30 (3) |B| = |A|+ 2 and |P(B)|?|P(A)| = 24

Please help!!
It is a set theory question

Please help!!
(1) |P(A)| = 4 <-- Cardinality of the power set is 4, which means we have 2^n = 4.[B] |A| = 2 [/B] (2) |B| = |A|+ 1 and |A×B| = 30 |B| = 6 if [B]|A| = 5[/B] and |A x B| = 30 (3) |B| = |A|+ 2 and |P(B)|?|P(A)| = 24 Since |B| = |A|+ 2, we have: 2^(a + 2) - 2^a = 24 2^a(2^2 - 1) = 24 2^a(3) = 24 2^a = 8 [B]|A |= 3[/B] To check, we have |B| = |A| + 2 --> 3 + 2 = 5 So |P(B)| = 2^5 = 32 |P(A)| = 2^3 = 8 And 32 - 8 = 24

please solve the fifth word problem
Karen purchased a prepaid phone card for $20 . Long distance calls cost 11 cents a minute using this card. Karen used her card only once to make a long distance call. If the remaining credit on her card is $17.47 , how many minutes did her call last?

please solve the fifth word problem
Find what was used: Used Money = Prepaid original cost - Remaining Credit Used Money = 20 - 17.47 Used Money = 2.53 Using (m) as the number of minutes, we have the following cost equation: C(m) = 0.11m C(m) = 2.53, so we have: 0.11m = 2.53 Divide each side by 0.11 [B]m = 23[/B]

please solve the fourth word problem
The sum of three numbers is 105 . The first number is 5 less than the second. The third number is 3 times the second. What are the numbers?

please solve the fourth word problem
Let x be the first number, y be the second number, and z be the number. We have the following equations: [LIST=1] [*]x + y + z = 305 [*]x = y - 5 [*]z = 3y [/LIST] Substitute (2) and (3) into (1) (y - 5) + y + (3y) = 305 Combine like terms: 5y - 5 = 305 Use our [URL='http://www.mathcelebrity.com/1unk.php?num=5y-5%3D305&pl=Solve']equation solver[/URL] [B]y = 62 [/B] Substitute y = 62 into (3) z = 3(62) [B]z = 186 [/B] x = (62) - 5 [B]x = 57[/B]

please solve the third word problem
A Web music store offers two versions of a popular song. The size of the standard version is 2.7 megabytes (MB). The size of the high-quality version is 4.7 MB. Yesterday, the high-quality version was downloaded three times as often as the standard version. The total size downloaded for the two versions was 4200 MB. How many downloads of the standard version were there?

please solve the third word problem
A Web music store offers two versions of a popular song. The size of the standard version is 2.7 megabytes (MB). The size of the high-quality version is 4.7 MB. Yesterday, the high-quality version was downloaded three times as often as the standard version. The total size downloaded for the two versions was 4200 MB. How many downloads of the standard version were there? Let s be the standard version downloads and h be the high quality downloads. We have two equations: [LIST=1] [*]h = 3s [*]2.7s + 4.7h = 4200 [/LIST] Substitute (1) into (2) 2.7s + 4.7(3s) = 4200 2.7s + 14.1s = 4200 Combine like terms: 16.8s = 4200 Divide each side by 16.8 [B]s = 250[/B]

Point and a Line
Free Point and a Line Calculator - Enter any line equation and a 2 dimensional point.  The calculator will figure out if the point you entered lies on the line equation you entered. If the point does not lie on the line, the distance between the point and line will be calculated.

Point Estimate and Margin of Error
Free Point Estimate and Margin of Error Calculator - Given an upper bound and a lower bound and a sample size, this calculate the point estimate, margin of error.

Point P is located at -15 and point Q is located at 6 on a number line. Which value would represent
Point P is located at -15 and point Q is located at 6 on a number line. Which value would represent point T, the midpoint of PQ? Using our [URL='https://www.mathcelebrity.com/mptnline.php?ept1=-15&empt=&ept2=6&pl=Calculate+missing+Number+Line+item']midpoint calculator[/URL], we get: T = [B]-4.5[/B]

Points A, B, and C are collinear. Point B is between A and C. Find AB if AC=15 and BC=7.
Points A, B, and C are collinear. Point B is between A and C. Find AB if AC=15 and BC=7. Collinear means on the same line. By segment subtraction, we have: AB = AC - BC AB = 15 - 7 AB = [B]8[/B]

Poisson Distribution
Free Poisson Distribution Calculator - Calculates the probability of 3 separate events that follow a poisson distribution.
It calculates the probability of exactly k successes P(x = k)
No more than k successes P (x <= k)
Greater than k successes P(x >= k)
Each scenario also calculates the mean, variance, standard deviation, skewness, and kurtosis.
Calculates moment number t using the moment generating function

Polar Conics
Free Polar Conics Calculator - Given eccentricity (e), directrix (d), and angle θ, this determines the vertical and horizontal directrix polar equations.

Polygon Side
Free Polygon Side Calculator - Determines the sides of a polygon given an interior angle sum.

Polygons
Free Polygons Calculator - Using various input scenarios of a polygon such as side length, number of sides, apothem, and radius, this calculator determines Perimeter or a polygon and Area of the polygon. This also determines interior angles of a polygon and diagonals of a polygon as well as the total number of 1 vertex diagonals.

Polynomial
Free Polynomial Calculator - This calculator will take an expression without division signs and combine like terms.
It will also analyze an polynomial that you enter to identify constant, variables, and exponents. It determines the degree as well.

Pool Volume
Free Pool Volume Calculator - Given a round shaped pool, this calculates the volume (Capacity) in gallons of the pool when filled with water

population MEAN OF ENVIRONMENTAL SPECIALIST SALARY IS $62000.A RANDOM SAMPLE OF 45 SPECIALIST IS DRA
population MEAN OF ENVIRONMENTAL SPECIALIST SALARY IS $62000.A RANDOM SAMPLE OF 45 SPECIALIST IS DRAWN FROM THE POPULATION. WHAT IS THE LIKELIHOOD THAT THE MEAN SALARY SAMPLE IS $59000. ASSUME SIGMA IS $6000. Using our [URL='http://www.mathcelebrity.com/probnormdist.php?xone=59000&mean=62000&stdev=6000&n=45&pl=P%28X+%3C+Z%29']Z-Score calculator[/URL], we get the probability as [B]0.0004[/B].

porportion problems
Im not really good with proportion and rates word problems and I need some help with it in my homework If Leah walks 5 miles in 60 minutes, then Leah will walk how far in 110 minutes if she walks at the same speed the whole time? If necessary, round your answer to the nearest tenth of a mile. I wanna know how i get this answer and copy the formula. Please help me thank you.

porportion problems
Set up a proportion of miles to minutes where m is the number of miles walked in 110 minutes: 5/60 = m/110 Use our [URL='http://www.mathcelebrity.com/prop.php?num1=5&num2=m&den1=60&den2=110&propsign=%3D&pl=Calculate+missing+proportion+value']proportion calculator[/URL], we get: [B]m = 9.1667 miles[/B]

Portfolio Rate of Return
Free Portfolio Rate of Return Calculator - Given a portfolio of individual assets with returns and weights, this calculates the total portfolio rate of return.

Positive numbers less than 4
Update, this has been added to our shortcuts. You can type any expression in the form, positive numbers less than x where x is any integer. You can also type positive numbers greater than x where x is any integer. Same with less than or equal to and greater than or equal to.

Positivity Rate
Free Positivity Rate Calculator - This calculator determines the positivity rate using positive tests and total tests

Potato chips were $29.26 for 7 bags.Brand B was $25.38 for 6 bags which is the better buy
Potato chips were $29.26 for 7 bags.Brand B was $25.38 for 6 bags which is the better buy. Using our [URL='http://www.mathcelebrity.com/betterbuy.php?p1=29.26&p2=25.38&q1=7&q2=6&pl=Better+Buy']better buy calculator[/URL], we get: [LIST] [*]Potato Chips have a unit cost of $4.18 [*]Brand B has a unit cost of $4.23 [*]Since Potato Chips have a lower unit cost, [B]Potato Chips are the better buy[/B] [/LIST]

Pound of strawberries for $4.00. What is the price, in dollars, per ounce of strawberries?
Pound of strawberries for $4.00. What is the price, in dollars, per ounce of strawberries? 1 pound equals 16 ounces. So the pounds per ounce equals: $4.00/16 ounces Divide top and bottom by 16, we get: [B]$0.25 per ounce[/B]

power set for S= {b,c,f}
power set for S= {b,c,f} The [I]power set[/I] P is the set of all subsets of S including S and the empty set ?. Since S contains 3 terms, our Power Set should contain 2^3 = 8 items [URL='https://www.mathcelebrity.com/powerset.php?num=b%2Cc%2Cf&pl=Show+Power+Set+and+Partitions']Link to power set for this problem[/URL] P = [B]{{}, {b}, {c}, {f}, {b,c}, {b,f}, {c,f}, {b,c,f}}[/B]

Power Sets and Set Partitions
Free Power Sets and Set Partitions Calculator - Given a set S, this calculator will determine the power set for S and all the partitions of a set.

Powers Of
Free Powers Of Calculator - Determines the powers of a number from 1 to n.

pr=xf/y for r
pr=xf/y for r So for this literal equation, we divide each side of the equation by p to isolate r. pr/p = xf/yp Cancel the p's on the left side and we get: r = [B]xf/yp [MEDIA=youtube]6ekuN4H3mM4[/MEDIA][/B]

Predecessor
Free Predecessor Calculator - Calculates the predecessor number to a given number

Pressure Conversions
Free Pressure Conversions Calculator - This calculator converts between the following pressure measurements:
atmosphere
bar
dynes/cm2
in. Hg
in. water
kg/cm2
mbar
mtorr or micron Hg
Pa or N/m2
PSI or lb/in2
torr or mm Hg

Pressure Law
Free Pressure Law Calculator - This will solve for any of the 4 items in the Pressure Law equation, also known as Gay-Lussacs Law assuming constant volume
P1 ÷ T1 = P2 ÷ T2

Previously, an organization reported that teenagers spent 4.5 hours per week, on average, on the pho
Previously, an organization reported that teenagers spent 4.5 hours per week, on average, on the phone. The organization thinks that, currently, the mean is higher. Fifteen randomly chosen teenagers were asked how many hours per week they spend on the phone. The sample mean was 4.75 hours with a sample standard deviation of 2.0. Conduct a hypothesis test, [U][B]the Type I error is[/B][/U]: a. to conclude that the current mean hours per week is higher than 4.5, when in fact, it is higher
b. to conclude that the current mean hours per week is higher than 4.5, when in fact, it is the same
c. to conclude that the mean hours per week currently is 4.5, when in fact, it is higher
d. to conclude that the mean hours per week currently is no higher than 4.5, when in fact, it is not higher [B]b. to conclude that the current mean hours per week is higher than 4.5, when in fact, it is the same [/B] [I]A Type I error is when you reject the null hypothesis when it is in fact true[/I]

principal $3000, actual interest rate 5.6%, time 3 years. what is the balance after 3 years
principal $3000, actual interest rate 5.6%, time 3 years. what is the balance after 3 years [URL='https://www.mathcelebrity.com/compoundint.php?bal=3000&nval=3&int=5.6&pl=Annually']Using our compound interest calculator[/URL], we get a final balance of: [B]$3,532.75[/B]

PRIVATE SAT TUTORING - LIVE FACE-TO-FACE SKYPE TUTORING
PRIVATE SAT TUTORING - LIVE FACE-TO-FACE SKYPE TUTORING Schedule a free consultation: [URL]https://calendly.com/soflo-sat/celeb[/URL] Expert SAT & ACT Tutoring with a live person. SoFlo SAT Tutoring offers face to face test prep through Skype. We provide all curriculum and create a custom plan tailored to our student’s strengths and weaknesses. Our founder Adam Shlomi had an 800 in Reading and 770 in Math on the SAT — good for the 99th percentile on both sections, went to Georgetown University, and has been tutoring for five years. Every SAT expert scored at least a 1500/1600 on the SAT and comes from the country’s top schools like Princeton, Johns Hopkins, and Georgetown. After only 10 sessions our average student improves 120 points. Our success comes from the individual attention we give our students. Our strategies give them confidence to succeed, plus we coach them through the SAT by creating a structured study plan. Working with our expert tutors, our students achieve amazing SAT success. [QUOTE]Adam is the best tutor I've ever had! He really knew the material and took his time explaining concepts to me. He's also fun to sit down and study with, which is super important for me. I couldn't be happier with SoFlo SAT. -- Charlotte Forman, Bard College[/QUOTE] [QUOTE]Because of SoFlo SAT my score increased 8 points on the ACT. He pushed me and helped me reach my goals. That 8 point boost helped me earn thousands of dollars on scholarships! SoFlo has the best SAT Tutor in South Florida. -- Jake Samuels, University of Florida[/QUOTE] Schedule a free call today with Adam to set up SAT prep! [URL]https://calendly.com/soflo-sat/celeb[/URL]

Prizes hidden on a game board with 10 spaces. One prize is worth $100, another is worth $50, and tw
Imagine you are in a game show. Prizes hidden on a game board with 10 spaces. One prize is worth $100, another is worth $50, and two are worth $10. You have to pay $20 to the host if your choice is not correct. Let the random variable x be the winning (a) What is your expected winning in this game? (b) Determine the standard deviation of x. (Round the answer to two decimal places) (a) 100(0.1) + 50(0.1) + 10(0.2) - 20 = 10 + 5 + 2 - 20 = [B]-3[/B] (b) 3.3 using our [URL='http://www.mathcelebrity.com/statbasic.php?num1=+100,50,10&num2=+0.1,0.1,0.2&usep=usep&pl=Number+Set+Basics']standard deviation calculator[/URL]

Probability
Free Probability Calculator - This lesson walks you through the basics of probability like the probability definition, events, outcomes, experiments, and probability postulates

Probability (A U B U C)
Free Probability (A U B U C) Calculator - Calculates the probability of a union of a three event sample space, A, B, and C, as well as P(A), P(B), P(C), P(A ∩ B), P(A ∩ C), P(B ∩ C), P(A ∩ B ∩ C).

Probability (A U B)
Free Probability (A U B) Calculator - Given a 2 event sample space A and B, this calculates the probability of the following events:
P(A U B)
P(A)
P(B)
P(A ∩ B)

Probability of getting either a sum of 8 or at least one 4 in the roll of a pair dice
Sum of 8 equal to 5/36 shown [URL='http://www.mathcelebrity.com/2dice.php?gl=1&pl=8&opdice=1&rolist=+2%2C3%2C9%2C10&dby=2%2C3%2C5&ndby=4%2C5&montect=+100']here[/URL]. At least one 4 means one of three scenarios: [LIST=1] [*](4, not 4) = 1/6 * 5/6 = 5/36 [*](not 4, 4) = 5/6 * 1/6 = 5/36 [*](4, 4) = 1/6 * 1/6 = 1/36 [/LIST] The phrase "or", means we add both probabilities (sum of 8) and (at least one 4): 5/36 + (5/36 + 5/36 + 1/36) 16/36 Simplify by dividing each part of the fraction by 4 [B]4/9[/B]

Problems Involving Rational Expressions
When Joana Mae, Precious Jewels and Molly Anne work together, they finish installing a garden in 3 days. The job could be completed if Joana Mae worked 4 days alone and Molly Anne worked 10 days alone, or if Precious Jewels worked 5 days alone and Molly Anne worked 3 days alone. How many days would it take each worker, alone, to complete the garden?

Problems Involving Rational Expressions
We are given, using the word word problem combined formula, that: 1/j + 1/p + 1/m = 1/3 However, you state the hours working alone, but then ask how much it would take working alone. I'm confused on the last part. Can you clarify?

product of 8 and the sum of 6 and 3y
product of 8 and the sum of 6 and 3y the sum of 6 and 3y 6 + 3y product of 8 and the sum of 6 and 3y [B]8(6 + 3y)[/B]

product of a number and its reciprocal is increased by 7
product of a number and its reciprocal is increased by 7 The phrase [I]a number[/I] means an arbitrary variable, let's call it x: x Its reciprocal means we take the reciprocal of x: 1/x product of a number and its reciprocal: x * 1/x x/x The x's cancel giving us: 1 is increased by 7 means we add 7: 1 + 7 [B]8[/B]

Product of Consecutive Numbers
Free Product of Consecutive Numbers Calculator - Finds the product of (n) consecutive integers, even or odd as well. Examples include:
product of 2 consecutive integers
product of 2 consecutive numbers
product of 2 consecutive even integers
product of 2 consecutive odd integers
product of 2 consecutive even numbers
product of 2 consecutive odd numbers
product of two consecutive integers
product of two consecutive odd integers
product of two consecutive even integers
product of two consecutive numbers
product of two consecutive odd numbers
product of two consecutive even numbers
product of 3 consecutive integers
product of 3 consecutive numbers
product of 3 consecutive even integers
product of 3 consecutive odd integers
product of 3 consecutive even numbers
product of 3 consecutive odd numbers
product of three consecutive integers
product of three consecutive odd integers
product of three consecutive even integers
product of three consecutive numbers
product of three consecutive odd numbers
product of three consecutive even numbers
product of 4 consecutive integers
product of 4 consecutive numbers
product of 4 consecutive even integers
product of 4 consecutive odd integers
product of 4 consecutive even numbers
product of 4 consecutive odd numbers
product of four consecutive integers
product of four consecutive odd integers
product of four consecutive even integers
product of four consecutive numbers
product of four consecutive odd numbers
product of four consecutive even numbers
product of 5 consecutive integers
product of 5 consecutive numbers
product of 5 consecutive even integers
product of 5 consecutive odd integers
product of 5 consecutive even numbers
product of 5 consecutive odd numbers
product of five consecutive integers
product of five consecutive odd integers
product of five consecutive even integers
product of five consecutive numbers
product of five consecutive odd numbers
product of five consecutive even numbers


product of r plus 7 and 4
product of r plus 7 and 4 r plus 7 means we add 7 to r: r + 7 The product means we multiply the expression r + a 7 by 4: [B]4(r + 7)[/B]

product of x and y decreased by their sum
product of x and y decreased by their sum Product of x and y: xy Their sum: x + y Product of x and y decreased by their sum: [B]xy - (x + y)[/B]

Profit Equation
Free Profit Equation Calculator - Using the Profit Equation with inputs (Revenue-Cost-Profit-Tax), this determines the relevant output including gross proft, gross profit margin, net profit, and net profit margin.

Projectile Motion
Free Projectile Motion Calculator - Solves for time using a height and velocity of an object thrown up in the air

Proportion
Free Proportion Calculator - 1) Calculates the missing link of 2 equivalent proportions or ratios.
2) Also determines if two numerical proportions that you entered such as 1/10=6/12 are equivalent or not equivalent. Note: You can use all allowable operators such as =,<,≤,>,≥

Proportion Sample Size
Free Proportion Sample Size Calculator - This calculator determines a sample size to select to meet certain criteria related to a confidence percentage, reliability percentage, and a p value proportion. Simply enter your values not using percentage signs. This works whether p^ is known or not known.

Prove 0! = 1
Prove 0! = 1 Let n be a whole number, where n! represents the product of n and all integers below it through 1. The factorial formula for n is: n! = n · (n - 1) * (n - 2) * ... * 3 * 2 * 1 Written in partially expanded form, n! is: n! = n * (n - 1)! [U]Substitute n = 1 into this expression:[/U] n! = n * (n - 1)! 1! = 1 * (1 - 1)! 1! = 1 * (0)! For the expression to be true, 0! [U]must[/U] equal 1. Otherwise, 1! <> 1 which contradicts the equation above

Prove 0! = 1
[URL='https://www.mathcelebrity.com/proofs.php?num=prove0%21%3D1&pl=Prove']Prove 0! = 1[/URL] Let n be a whole number, where n! represents: The product of n and all integers below it through 1. The factorial formula for n is n! = n · (n - 1) · (n - 2) · ... · 3 · 2 · 1 Written in partially expanded form, n! is: n! = n · (n - 1)! [SIZE=5][B]Substitute n = 1 into this expression:[/B][/SIZE] n! = n · (n - 1)! 1! = 1 · (1 - 1)! 1! = 1 · (0)! For the expression to be true, 0! [U]must[/U] equal 1. Otherwise, 1! ? 1 which contradicts the equation above [MEDIA=youtube]wDgRgfj1cIs[/MEDIA]

Prove P(A’) = 1 - P(A)
Prove P(A’) = 1 - P(A) The sample space S contains an Event A and everything not A, called A' We know P(S) = 1 P(S) = P(A U A') P(A U A') = 1 P(A) + P(A') = 1 subtract P(A) from each side: P(A’) = 1 - P(A) [MEDIA=youtube]dNLl_8vejyE[/MEDIA]

Prove sqrt(2) is irrational
Use proof by contradiction. Assume sqrt(2) is rational. This means that sqrt(2) = p/q for some integers p and q, with q <>0. We assume p and q are in lowest terms. Square both side and we get: 2 = p^2/q^2 p^2 = 2q^2 This means p^2 must be an even number which means p is also even since the square of an odd number is odd. So we have p = 2k for some integer k. From this, it follows that: 2q^2 = p^2 = (2k)^2 = 4k^2 2q^2 = 4k^2 q^2 = 2k^2 q^2 is also even, therefore q must be even. So both p and q are even. This contradicts are assumption that p and q were in lowest terms. So sqrt(2) [B]cannot be rational. [MEDIA=youtube]tXoo9-8Ewq8[/MEDIA][/B]

Prove that the difference between alternate consecutive squares as always even
Take an integer n. The next alternate consecutive integer is n + 2 Subtract the difference of the squares: (n + 2)^2 - n^2 n^2 + 4n + 4 - n^2 n^2 terms cancel, we get: 4n + 4 Factor out a 4: 4(n + 1) If n is odd, n + 1 is even. 4 * even is always even If n is even, n + 1 is odd. 4 * odd is always odd Since both cases are even, we've proven our statement. [MEDIA=youtube]J_E9lR5qFY0[/MEDIA]

Prove that the difference of two consecutive cubes is never divisible by 3
Take two consecutive integers: n, n + 1 The difference of their cubes is: (n + 1)^3 - n^3 n^3 + 3n^2 + 3n + 1 - n^3 Cancel the n^3 3n^2 + 3n + 1 Factor out a 3 from the first 2 terms: 3(n^2 + n) + 1 The first two terms are always divisible by 3 but then the + 1 makes this expression not divisible by 3: 3(n^2 + n) + 1 = 1 (mod 3) [MEDIA=youtube]hFvJ3epqmyE[/MEDIA]

Prove the difference between two consecutive square numbers is always odd
Take an integer n. The next consecutive integer is n + 1 Subtract the difference of the squares: (n + 1)^2 - n^2 n^2 + 2n + 1 - n^2 n^2 terms cancel, we get: 2n + 1 2 is even. For n, if we use an even: we have even * even = Even Add 1 we have Odd 2 is even. For n, if we use an odd: we have even * odd = Even Add 1 we have Odd Since both cases are odd, we've proven our statement. [MEDIA=youtube]RAi0HbH5bqc[/MEDIA]

Prove the following statement for non-zero integers a, b, c, If a divides b and b divides c, then a
Prove the following statement for non-zero integers a, b, c, If a divides b and b divides c, then a divides c. If an integer a divides an integer b, then we have: b = ax for some non-zero integer x If an integer b divides an integer c, then we have: c = by for some non-zero integer y Since b = ax, we substitute this into c = by for b: c = axy We can write this as: c = a(xy) [LIST] [*]Since x and y are integers, then xy is also an integer. [*]Therefore, c is the product of some integer multiplied by a [*]This means a divides c [/LIST] [MEDIA=youtube]VUIUFAFFVU4[/MEDIA]

Prove the sum of any two rational numbers is rational
Take two integers, r and s. We can write r as a/b for integers a and b since a rational number can be written as a quotient of integers We can write s as c/d for integers c and d since a rational number can be written as a quotient of integers Add r and s: r + s = a/b + c/d With a common denominator bd, we have: r + s = (ad + bc)/bd Because a, b, c, and d are integers, ad + bc is an integer since rational numbers are closed under addition and multiplication. Since b and d are non-zero integers, bd is a non-zero integer. Since we have the quotient of 2 integers, r + s is a rational number. [MEDIA=youtube]0ugZSICt_bQ[/MEDIA]

Prove the sum of two odd numbers is even
Take two arbitrary integers, x and y We can express the odd integer x as 2a + 1 for some integer a We can express the odd integer y as 2b + 1 for some integer b x + y = 2a + 1 + 2b + 1 x + y = 2a + 2b + 2 Factor out a 2: x + y = 2(a + b + 1) Since 2 times any integer even or odd is always even, then [B]x + y by definition is even[/B]. [MEDIA=youtube]9A-qe4yZXYw[/MEDIA]

Prove there is no integer that is both even and odd
Let us take an integer x which is both even [I]and[/I] odd. [LIST] [*]As an even integer, we write x in the form 2m for some integer m [*]As an odd integer, we write x in the form 2n + 1 for some integer n [/LIST] Since both the even and odd integers are the same number, we set them equal to each other 2m = 2n + 1 Subtract 2n from each side: 2m - 2n = 1 Factor out a 2 on the left side: 2(m - n) = 1 By definition of divisibility, this means that 2 divides 1. But we know that the only two numbers which divide 1 are 1 and -1. Therefore, our original assumption that x was both even and odd must be false. [MEDIA=youtube]SMM9ubEVcLE[/MEDIA]

Put Call Parity
Free Put Call Parity Calculator - This solve for any of the 6 items in the put call parity for European options

Put Options and Call Options
Free Put Options and Call Options Calculator - Shows the basics of a call option and put option including intrinsic value, extrinsic value, and in/out/at the money

Put the number 123456789 exactly ones in the bubble so that each edge adds up to say number
Put the number 123456789 exactly ones in the bubble so that each edge adds up to say number [B] Each side adds up to 17 [IMG]https://www.mathcelebrity.com/images/triangle_sum_17.png[/IMG] [/B]

Puzzle Master
Free Puzzle Master Calculator - A link to our friends: Puzzle Master has a large and unique collection of brain teasers; puzzles for sale. In addition they also carry chess,mechanical banks, puzzle books, magic trick books, boomerangs, etc.

pv/t = ab/c for c
pv/t = ab/c for c Cross multiply: cpv = abt Divide each side of the equation by pv to isolate c: cpv/pv = abt/pv Cancel the pv terms on the left side and we get: c = [B]abt/pv[/B]

Pythagorean Theorem
Free Pythagorean Theorem Calculator - Figures out based on user entry the missing side or missing hypotenuse of a right triangle. In addition, the calculator shows the proof of the Pythagorean Theorem and then determines by numerical evaluation if the 2 sides and hypotenuse you entered are a right triangle using the Pythagorean Theorem

Pythagorean Theorem Trig Proofs
Free Pythagorean Theorem Trig Proofs Calculator - Shows the proof of 3 pythagorean theorem related identities using the angle θ:
Sin2(θ) + Cos2(θ) = 1
Tan2(θ) + 1 = Sec2(θ)
Sin(θ)/Cos(θ) = Tan(θ)

q increased by the difference between 18 times q and 5
q increased by the difference between 18 times q and 5 Take this algebraic expression in parts. 18 times q: 18q The difference between 18 times q and 5 means we subtract 5 from 18q: 18q - 5 q increased by the difference between 18 times q and 5 means we add 18q - 5 to q: q + (18q - 5) [B]q + 18q - 5[/B] IF we want to simplify, we group like terms: [B]19q - 5[/B]

Q is 5% less than P
The phrase is means equal to, so we have: Q = P - 5% 5% is written as 0.05, so we have: Q = P - 0.05

Q is a point on segment PR. If PQ = 2.7 and PR = 6.1, what is QR?
Q is a point on segment PR. If PQ = 2.7 and PR = 6.1, what is QR? From segment addition, we know that: PQ + QR = PR Plugging our given numbers in, we get: 2.7 + QR = 6.1 Subtract 2.7 from each side, and we get: 2.7 - 2.7 + QR = 6.1 - 2.7 Cancelling the 2.7 on the left side, we get: QR = [B]3.4[/B]

q is equal to 207 subtracted from the quantity 4 times q
q is equal to 207 subtracted from the quantity 4 times q 4 time q 4q 207 subtracted from 4 times q: 4q - 207 Set this equal to q: [B]4q - 207 = q [/B]<-- This is our algebraic expression To solve for q, [URL='https://www.mathcelebrity.com/1unk.php?num=4q-207%3Dq&pl=Solve']type this equation into the search engine[/URL]. We get: [B]q = 69[/B]

q to the 10th power subtracted from 100
q to the 10th power subtracted from 100 q to the 10th power: q^10 We subtract this from 100: [B]100 - q^10[/B]

q=c+d/5 for d
q=c+d/5 for d Subtract c from each side to solve this literal equation: q - c = c - c + d/5 Cancel the c's on the right side, we get d/5 = q - c Multiply each side by 5: 5d/5 = 5(q - c) Cancel the 5's on the left side, we get: [B]d = 5(q - c)[/B]

Quadrant
Free Quadrant Calculator - Describes the Quadrant and the details

Quadratic equation hacks using the discriminant
Quadratic equation hacks using the discriminant Solve x^2- 4x+ 5 using a discriminant: Discriminant is: Discriminant = b^2- 4ac Discriminant = (-4)^2 - 4(1)(5) Discriminant = 16 - 20 Discriminant = -4 When Discriminant < 0, the quadratic has [I][U]no solution [MEDIA=youtube]RogZ3430_8E[/MEDIA][/U][/I]

Quadratic Equations and Inequalities
Free Quadratic Equations and Inequalities Calculator - Solves for quadratic equations in the form ax2 + bx + c = 0. Also generates practice problems as well as hints for each problem.
* Solve using the quadratic formula and the discriminant Δ
* Complete the Square for the Quadratic
* Factor the Quadratic
* Y-Intercept
* Vertex (h,k) of the parabola formed by the quadratic where h is the Axis of Symmetry as well as the vertex form of the equation a(h - h)2 + k
* Concavity of the parabola formed by the quadratic
* Using the Rational Root Theorem (Rational Zero Theorem), the calculator will determine potential roots which can then be tested against the synthetic calculator.

Quadrilateral
Free Quadrilateral Calculator - Given 4 points entered, this determines the area using Brahmaguptas Formula and perimeter of the quadrilateral formed by the points as well as checking to see if the quadrilateral (quadrangle) is a parallelogram.

Quartic Equations
Free Quartic Equations Calculator - Solves quartic equations in the form ax4 + bx3 + cx2 + dx + e using the following methods:
1) Solve the long way for all roots and the discriminant Δ
2) Rational Root Theorem (Rational Zero Theorem) to solve for real roots followed by the synthetic div/quadratic method for the other imaginary roots if applicable.

quotient of the sum of 17 and x and y
quotient of the sum of 17 and x and y The sum of 17 and x means we add x to 17: 17 + x quotient of the sum of 17 and x and y means we divide 17 + x by y [B](17 + x)/y[/B]

quotient of the sum of 2 numbers and 6
quotient of the sum of 2 numbers and 6 The phrase [I]two numbers[/I] means we choose 2 arbitrary variables, let's call them x and y x, y The sum of 2 numbers: x + y quotient of the sum of 2 numbers and 6 [B](x + y)/6[/B]

quotient of the sum of 3 numbers and 3
quotient of the sum of 3 numbers and 3 The phrase [I]3 numbers[/I] means we choose 3 arbitrary variables: a, b,c The sum of the 3 numbers: a + b + c quotient of the sum of 3 numbers and 3 [B](a + b + c)/3[/B]

Quotient-Remainder Theorem
Free Quotient-Remainder Theorem Calculator - Given 2 positive integers n and d, this displays the quotient remainder theorem.

r decreased by the quotient of r and 3
r decreased by the quotient of r and 3 The quotient of r and 3 is: r/3 The phrase [I]decreased by[/I] means we subtract r/3 from r [B]r - r/3[/B]

r less than 164 is 248 more than the product of 216 and r
r less than 164 is 248 more than the product of 216 and r [U]r less than 164:[/U] 164 - r [U]The product of 216 and r:[/U] 216r [U]248 more than the product of 216 and r[/U] 216r + 248 [I]The word is means an equation, so we set 164 - r equal to 216r + 248[/I] [B]164 - r = 216r + 248[/B]

r squared plus the product of 3 and s plus 5
r squared plus the product of 3 and s plus 5 r squared means we raise r to the power of 2 r^2 The product of 3 and s means we multiply s by 3: 3s plus 5 means we add 3s + 5 R squared plus means we add r^2: [B]r^2 + 3s + 5[/B]

r varies directly with s and inversely with the square root of t
r varies directly with s and inversely with the square root of t Varies directly means we multiply Varies inversely means we divide There exists a constant k such that: [B]r = ks/sqrt(t)[/B]

Rachel borrowed 8000 at a rate of 10.5%, compounded monthly. Assuming she makes no payments, how muc
Rachel borrowed 8000 at a rate of 10.5%, compounded monthly. Assuming she makes no payments, how much will she owe after 4 years? [U]Convert annual amounts to monthly[/U] 4 years = 12 * 4 = 48 months i = .105/12 = 0.00875 monthly [U]Build our accumulation function A(t) where t is the time in months[/U] A(48) = 8,000 * (1.00875)^48 A(48) = 8,000 * 1.5192 A(48) = [B]12,153.60 [/B] [URL='http://www.mathcelebrity.com/compoundint.php?bal=8000&nval=48&int=10.5&pl=Monthly']You can also use the balance calculator[/URL]

Rachel buys some scarves that cost $10 each and 2 purses that cost $16 each. The cost of Rachel's to
Rachel buys some scarves that cost $10 each and 2 purses that cost $16 each. The cost of Rachel's total purchase is $62. What equation can be used to find n, the number of scarves that Rebecca buys Scarves Cost + Purses Cost = Total Cost [U]Calculate Scarves Cost[/U] Scarves cost = Cost per scarf * number of scarves Scarves cost = 10n [U]Calculate Purses Cost[/U] Purses cost = Cost per purse * number of purses Purses cost = 16 * 2 Purses cost = 32 Total Cost = 62. Plug in our numbers and values to the Total Cost equation : 10n + 32 = 62 Solve for [I]n[/I] in the equation 10n + 32 = 62 [SIZE=5][B]Step 1: Group constants:[/B][/SIZE] We need to group our constants 32 and 62. To do that, we subtract 32 from both sides 10n + 32 - 32 = 62 - 32 [SIZE=5][B]Step 2: Cancel 32 on the left side:[/B][/SIZE] 10n = 30 [SIZE=5][B]Step 3: Divide each side of the equation by 10[/B][/SIZE] 10n/10 = 30/10 n = [B]3[/B]

Rachel deposits $6000 into an account that pays simple interest at a rate of 6% per year. How much i
Rachel deposits $6000 into an account that pays simple interest at a rate of 6% per year. How much interest will she be paid in the first 4 years? Using our [URL='http://www.mathcelebrity.com/simpint.php?av=&p=6000&int=6&t=4&pl=Simple+Interest']simple interest calculator[/URL], we get interest paid of [B]$1,440[/B]

Rachel runs 2 miles during each track practice. Write an equation that shows the relationship betwe
Rachel runs 2 miles during each track practice. Write an equation that shows the relationship between the practices p and the distance d. Distance equals rate * practicdes, so we have: [B]d = 2p[/B]

Rachel runs each lap in 6 minutes. She will run less than 8 laps today. What are the possible number
Rachel runs each lap in 6 minutes. She will run less than 8 laps today. What are the possible numbers of minutes she will run today Less than means an inequality. 6 minutes per lap * 8 laps = 48 minutes. If m is the number of minutes Rachel runs, then we have: [B]m < 48[/B]

Rachel saved $200 and spends $25 each week. Roy just started saving $15 per week. At what week will
Rachel saved $200 and spends $25 each week. Roy just started saving $15 per week. At what week will they have the same amount? Let Rachel's account value R(w) where w is the number of weeks be: R(w) = 200 - 25w <-- We subtract -25w because she spends it every week, decreasing her balance. Let Roy's account value R(w) where w is the number of weeks be: R(w) = 15w Set them equal to each other: 200 - 25w = 15w To solve this equation, [URL='https://www.mathcelebrity.com/1unk.php?num=200-25w%3D15w&pl=Solve']we type it into our search engine[/URL] and get: [B]w = 5[/B]

Rachel works at a bookstore. On Tuesday, she sold twice as many books as she did on Monday. On Wedne
Rachel works at a bookstore. On Tuesday, she sold twice as many books as she did on Monday. On Wednesday, she sold 6 fewer books than she did on Tuesday. During the 3 days Rachel sold 19 books. Create an equation that can be used to find m, a number of books Rachel sold on Monday. Let me be the number of books Rachel sold on Monday. We're given Tuesday's book sales (t) and Wednesday's books sales (w) as: [LIST=1] [*]t = 2m [*]w = t - 6 [*]m + t + w = 19 [/LIST] Plug (1) and (2) into (3): Since t = 2m and w = t - 6 --> 2m - 6, we have: m + 2m + 2m - 6 = 19 Combine like terms: 5m - 6 = 19 [URL='https://www.mathcelebrity.com/1unk.php?num=5m-6%3D19&pl=Solve']Plugging this equation into our search engine[/URL], we get: [B]m = 5[/B]

Rafael is a software salesman. His base salary is $1900 , and he makes an additional $40 for every c
Rafael is a software salesman. His base salary is $1900 , and he makes an additional $40 for every copy of Math is Fun he sells. Let p represent his total pay (in dollars), and let c represent the number of copies of Math is Fun he sells. Write an equation relating to . Then use this equation to find his total pay if he sells 22 copies of Math is Fun. We want a sales function p where c is the number of copies of Math is Fun p = Price per sale * c + Base Salary [B]p = 40c + 1900 [/B] Now, we want to know Total pay if c = 22 p = 40(22) + 1900 p = 880 + 1900 p = [B]2780[/B]

raise 2 to the 10th power and divide k by the result
raise 2 to the 10th power and divide k by the result Raise 2 to the 10th power: 2^10 Divide k by the result: k / 2^10

Raise 3 to 9th power then multiply b by the result
Raise 3 to 9th power then multiply b by the result 3^9th power: 3^9 Multiply b by the result? [B]3^9 * b[/B]

raise 3 to the 4th power, subtract w from the result, then divide v by what you have
raise 3 to the 4th power, subtract w from the result, then divide v by what you have Raise 3 to the 4th power: 3^4 Simplified, this is 81 Subtract w from the result. We subtract w from 81: 81 - w Then divide v by what you have. We divide v by (81 -w) [B]v/(81 - w)[/B]

raise 3 to the 8th power, then divide the result by t
raise 3 to the 8th power, then divide the result by t 3 to the 8th power 3^8 Divide the result by t 3^8/t Now, if they want you to evaluate 3 to the 8th, you have: 6,561/t

raise 6 to the 4th power, add h to the result, then multiply what you have by 8
raise 6 to the 4th power, add h to the result, then multiply what you have by 8 Raise 6 to the 4th power: 6^4 add h to the result: 6^4 + h Then multiply what we have by 8: [B]8(6^4 + h)[/B]

Raise 9 to the 3rd power, subtract d from the result, then divide what you have by c
Raise 9 to the 3rd power, subtract d from the result, then divide what you have by c. This is an algebraic expression, let's take in parts (or chunks). Raise 9 to the 3rd power. This means we take 9, and raise it to an exponent of 3 9^3 Subtract d from the result, means we subtract d from 9^3 9^3 - d Now we divide 9^3 - d by c [B](9^3 - d) / c[/B]

Raise a to the 4th power then find the sum of the result and b
Raise a to the 4th power then find the sum of the result and b Raise a to the 4th: a^4 Then get the sum of this and b [B]a^4 + b[/B]

raise b to the 6th power then find the sum of the result and 4
raise b to the 6th power then find the sum of the result and 4 raise b to the 6th power b^6 raise b to the 6th power then find the sum of the result and 4 [B]b^6 + 4[/B]

raise c to the 2nd power, add the result to 8, then subtract what you have from d
raise c to the 2nd power, add the result to 8, then subtract what you have from d Raise c to the 2nd power: c^2 Add the result to 8: c^2 + 8 Subtract what you have from d: d - (c^2 + 8)

Raise c to the 7th power, divide the result by 4, then triple what you have
Raise c to the 7th power, divide the result by 4, then triple what you have. Take this algebraic expression in pieces. Raise c to the 7th power: c^7 Divide the result by 4, means we divide c^7 by 4 c^7 / 4 Triple what you have means multiply c^7 / 4 by 3 [B]3(c^7 / 4)[/B]

raise f to the 3rd power, then find the quotient of the result and g
raise f to the 3rd power, then find the quotient of the result and g Take this algebraic expression in two parts: [LIST=1] [*]Raise f to the 3rd power means we take f, and write it with an exponent of 3: f^3 [*]Find the quotient of the result and g. We take f^3, and divide it by g [/LIST] [B]f^3/g[/B]

raise f to the 8th power then multiply the result by g
f to the 8th power: f^8 Multiply the result by g (f^8) * g

Raise f to the 8th power, divide the result by 5, then multiply 10
Raise f to the 8th power, divide the result by 5, then multiply 10 f to the 8th power means we raise f to the power of 8 using an exponent: f^8 Divide f^8 by 5 (f^8)/5 Now multiply this by 10: 10(f^8)/5 We can simplify this algebraic expression by dividing 10/5 to get 2 on top: 2[B](f^8)[/B]

Raise F to the second power then divide G by the result
Raise F to the second power then divide G by the result F to the second power: F^2 Divide G by the result: [B]G/F^2[/B]

Raise p to the 5th power, then triple the result
Raise p to the 5th power, then triple the result Raise p to the 5th power: p^5 Triple the result [B]3p^5[/B]

Raise p to the 9th power, multiply the result by q, then divide what you have by r
Raise p to the 9th power, multiply the result by q, then divide what you have by r. Take this in steps: [LIST] [*]Raise p to the 9th power: p^9 [*]Multiply the result by q: qp^9 [*]Divide what you have (the result) by r: qp^9/r [/LIST] [B](qp^9)/r [MEDIA=youtube]I5PShTfas4Y[/MEDIA][/B]

raise q to the 5th power add the result to p then divide what you have by r
raise q to the 5th power add the result to p then divide what you have by r Take this algebraic expression in parts: [LIST] [*]Raise q to the 5th power: q^5 [*]Add the result to p: p + q^5 [*]Divide what you have by r. This means we take our result above and divide it by r: [/LIST] [B](p + q^5)/r[/B]

Raise q to the 5th power, then find the quotient of the result and r
Raise q to the 5th power, then find the quotient of the result and r q to the 5th power: q^5 Quotient of this and r [B]q^5/r[/B]

raise r to the 8th power then find the product of the result and 3
raise r to the 8th power then find the product of the result and 3 Raise r to the 8th power means we raise r with an exponent of 8: r^8 The product of the result and 3 means we muliply r^8 by 3 [B]3r^8[/B]

raise t to the 10th power, then find the quotient of the result and s
raise t to the 10th power, then find the quotient of the result and s Raise t to the 10th power means we use t as our variable and 10 as our exponent: t^10 The quotient means a fraction, where the numerator is t^10 and the denominator is s: [B]t^10/s[/B]

raise the difference of 8 and v to the 7th power
raise the difference of 8 and v to the 7th power Difference of 8 and v 8 - v To the 7th power [B](8 - v)^7[/B]

Raise the difference of V and 7 to the 10th
Raise the difference of V and 7 to the 10th The difference of V and 7: V - 7 Raise this to the 10th power: [B](V - 7)^10[/B]

Raise the sum of k and j to the second power
Raise the sum of k and j to the second power The sum of k and j is written as: k + j Raise the sum to the second power: [B](k + j)^2[/B]

Raise the sum of w and v to the 7
Raise the sum of w and v to the 7 The sum of w and v: w + v Raise the sum to the 7: [B](w + v)^7[/B]

raise v to the 9th power, then dividethe result by u
V to the 9th power means we use an exponent: v^9 Divide that result by u [B]v^9/u[/B]

raise x to the 10th power, then divide b by the result
raise x to the 10th power, then divide b by the result x to the 10th power x^10 Divide b by the result: [B]b/x^10[/B]

raise y to the 10th power, then find the quotient of the result and 2
y to the 10th power means we give y an exponent of 10 y^10 The quotient of y^10 and 2 is: y^10 ----- 2

raise z to the 2nd power, multiply 8 by the result then subtract what you have from 4
raise z to the 2nd power, multiply 8 by the result then subtract what you have from 4 Take this algebraic expression in pieces: [LIST] [*]Raise z to the 2nd power: z^2 [*]Multiply by 8: 8z^2 [*]Subtract what you have from 4: [/LIST] [B]4 - 8z^2[/B]

Random Sampling from the Normal Distribution
Free Random Sampling from the Normal Distribution Calculator - This performs hypothesis testing on a sample mean with critical value on a sample mean or calculates a probability that Z <= z or Z >= z using a random sample from a normal distribution.

Random Test
Free Random Test Calculator - Given a set of data and an α value, this determines the test statistic and accept/reject hypothesis based on randomness of a dataset.

Rates of Return
Free Rates of Return Calculator - Given a set of stock prices and dividends if applicable, this calculates the periodic rate of return and the logarithmic rate of return

ratio of the squares of t and u
ratio of the squares of t and u Ratio is also known as quotient in algebraic expression problems. The square of t means we raise t to the power of 2: t^2 The square of u means we raise u to the power of 2: u^2 ratio of the squares of t and u means we divide t^2 by u^2: [B]t^2/u^2[/B]

ratio of x cubed and the sum of y and 5
ratio of x cubed and the sum of y and 5 x cubed means we raise x to the power of 3: x^3 The sum of y and 5: y + 5 ratio of x cubed and the sum of y and 5 [B]x^3/(y + 5)[/B]

Rational Exponents - Fractional Indices
Free Rational Exponents - Fractional Indices Calculator - This calculator evaluates and simplifies a rational exponent expression in the form ab/c where a is any integer or any variable [a-z] while b and c are integers. Also evaluates the product of rational exponents

Rational Number Subtraction
Free Rational Number Subtraction Calculator - Subtracting 2 numbers, this shows an equivalent operations is adding the additive inverse. p - q = p + (-q)

Rational,Irrational,Natural,Integer Property
Free Rational,Irrational,Natural,Integer Property Calculator - This calculator takes a number, decimal, or square root, and checks to see if it has any of the following properties:
* Integer Numbers
* Natural Numbers
* Rational Numbers
* Irrational Numbers Handles questions like: Irrational or rational numbers Rational or irrational numbers rational and irrational numbers Rational number test Irrational number test Integer Test Natural Number Test

Ratios
Free Ratios Calculator - * Simplifies a ratio of a:b
* Given a ratio in the form a:b or a to b, and a total population amount, this calculator will determine the expected value of A and B from the ratio.


Ravi deposits $500 into an account that pays simple interest at a rate of 4% per year. How much inte
Ravi deposits $500 into an account that pays simple interest at a rate of 4% per year. How much interest will he be paid in the first 4 years? The formula for [U]interest[/U] using simple interest is: I = Prt where P = Principal, r = interest, and t = time. We're given P = 500, r =0.04, and t = 4. So we plug this in and get: I = 500(0.04)(4) I = [B]80[/B]

Reagan bought t T-shirts. The shirts came in 8 packages. Write an expression that shows how many T-s
Reagan bought t T-shirts. The shirts came in 8 packages. Write an expression that shows how many T-shirts were in each package. T-shirts per package = number of packages / number of t-shirts per package T-shirts per package = [B]8/t[/B]

Rearrange the following equation to make x the subject, and select the correct rearrangement from th
Rearrange the following equation to make x the subject, and select the correct rearrangement from the list below 3x + 2y 1 -------- = --- 4x + y 3 [LIST] [*]x = 7y/13 [*]x = 7y/5 [*]x = -7y [*]x = -3y [*]x = 3y/5 [*]x = -5y/13 [*]x = -y [/LIST] Cross multiply: 3(3x - 2y) = 4x + y Multiply the left side through 9x - 6y = 4x + y Subtract 4x from each side and add 6y to each side 5x = 7y Divide each side by 5 to isolate x, the subject of an equation is the variable to the left [B]x = 7y/5[/B]

rectangle abcd prove: triangle adc is congruent to triangle bcd
rectangle abcd prove: triangle adc is congruent to triangle bcd 1. Given: ABCD is a rectangle 2. AB = CD since opposite sides of rectangle are congruent 3. BC = AD since opposite sides of rectangle are congruent 4. AC = AC by the Reflexive Property of Equality 5. triangle ADC = triangle CBA by the Side-Side-Side (SSS) Property

Rectangles and Parallelograms
Free Rectangles and Parallelograms Calculator - Solve for Area, Perimeter, length, and width of a rectangle or parallelogram and also calculates the diagonal length as well as the circumradius and inradius.

Rectangular Number
Free Rectangular Number Calculator - This calculator determines the nth rectangular number

Reduce the quotient of 20 and a by 8
Reduce the quotient of 20 and a by 8 The quotient of 20 and a: 20/a Reduce this by 8: [B]20/a - 8[/B]

Reece made a deposite into an account that earns 8% simple interest. After 8 years reece has earned
Reece made a deposite into an account that earns 8% simple interest. After 8 years Reece has earned 400 dollars. How much was Reece's initial deposit? Simple interest formula: A = P(1 + it) where P is the amount of principal to be invested, i is the interest rate, t is the time, and A is the amount accumulated with interest. Plugging in our numbers, we get: 400 = P(1 + 0.08(8)) 400 = P(1 + 0.64) 400 = 1.64P 1.64P = 400 [URL='https://www.mathcelebrity.com/1unk.php?num=1.64p%3D400&pl=Solve']Typing this problem into our search engine[/URL], we get: P = [B]$243.90[/B]

Refer to a bag containing 13 red balls numbered 1-13 and 5 green balls numbered 14-18. You choose a
Refer to a bag containing 13 red balls numbered 1-13 and 5 green balls numbered 14-18. You choose a ball at random. a. What is the probability that you choose a red or even numbered ball? b. What is the probability you choose a green ball or a ball numbered less than 5? a. The phrase [I]or[/I] in probability means add. But we need to subtract even reds so we don't double count: We have 18 total balls, so this is our denonminator for our fractions. Red and Even balls are {2, 4, 6, 8, 10, 12} Our probability is: P(Red or Even) = P(Red) + P(Even) - P(Red and Even) P(Red or Even) = 13/18 + 9/18 - 6/18 P(Red or Even) = 16/18 Using our [URL='https://www.mathcelebrity.com/fraction.php?frac1=16%2F18&frac2=3%2F8&pl=Simplify']Fraction Simplify Calculator[/URL], we have: P(Red or Even) = [B]16/18[/B] [B][/B] b. The phrase [I]or[/I] in probability means add. But we need to subtract greens less than 5 so we don't double count: We have 18 total balls, so this is our denonminator for our fractions. Green and less than 5 does not exist, so we have no intersection Our probability is: P(Green or Less Than 5) = P(Green) + P(Less Than 5) - P(Green And Less Than 5) P(Green or Less Than 5) = 5/18 + 4/18 - 0 P(Green or Less Than 5) = 9/18 Using our [URL='https://www.mathcelebrity.com/fraction.php?frac1=9%2F18&frac2=3%2F8&pl=Simplify']Fraction Simplify Calculator[/URL], we have: P(Red or Even) = [B]1/2[/B]

Reference Angle
Free Reference Angle Calculator - Calculates the reference angle for a given angle. Also known as the positive acute angle.

Reflexive Property
Free Reflexive Property Calculator - Demonstrates the reflexive property of congruence using a number. Numerical Properties

Relative Error
Free Relative Error Calculator - Relative error is the absolute error divided by quantity

Renee sells 6 gifts in 20 minutes. How many might she sell in 4 hrs
Renee sells 6 gifts in 20 minutes. How many might she sell in 4 hrs What is 4 hours in minutes? 4 hours = 4 * 60 = 240 minutes. Now we are on a minutes to minutes basis, set up a proportion: 6/20 = x/240 where x is the number of gifts in 240 minutes (4 hours) Using our [URL='http://www.mathcelebrity.com/prop.php?num1=6&num2=x&den1=20&den2=240&propsign=%3D&pl=Calculate+missing+proportion+value']proportion calculator[/URL], we get: [B]x = 72[/B]

Rental canoes cost $30 plus $5 per house of use. Which expression gives the cost of renting a canoe
Rental canoes cost $30 plus $5 per house of use. Which expression gives the cost of renting a canoe for h hours [B]R = 30 + 5h[/B]

Represent the number of inches in 7 feet
Represent the number of inches in 7 feet We [URL='https://www.mathcelebrity.com/linearcon.php?quant=7&pl=Calculate&type=foot']type in 7 feet to our search engine and we get[/URL]: 7 feet = [B]84 inches[/B]

Researchers in Antarctica discovered a warm sea current under the glacier that is causing the glacie
Researchers in Antarctica discovered a warm sea current under the glacier that is causing the glacier to melt. The ice shelf of the glacier had a thickness of approximately 450 m when it was first discovered. The thickness of the ice shelf is decreasing at an average rate if 0.06 m per day. Which function can be used to find the thickness of the ice shelf in meters x days since the discovery? We want to build an function I(x) where x is the number of days since the ice shelf discovery. We start with 450 meters, and each day (x), the ice shelf loses 0.06m, which means we subtract this from 450. [B]I(x) = 450 - 0.06x[/B]

Resistor Color Codes
Free Resistor Color Codes Calculator - Given 3 Band level color codes and a tolerance color chosen, this calculates the resistance in ohms and the tolerance percentage

rewrite without an exponent :4^-2
rewrite without an exponent :4^-2 Since the exponent is negative, we have: 4^-2 = 1 / 4^2 4^-2 = [B]1 / 16[/B]

RGB and HEX conversions
Free RGB and HEX conversions Calculator - This converts HTML colors from RGB (Red, Green, Blue) to HEX and HEX to RGB (Red, Green, Blue) as well as calculate the Hue.

Rhombus
Free Rhombus Calculator - Given inputs of a rhombus, this calculates the following:
Perimeter of a Rhombus
Area of a Rhombus
Side of a Rhombus

Rhonda raised $245 for her softball team's fundraiser.She wants to raise no less than $455.Write and
Rhonda raised $245 for her softball team's fundraiser.She wants to raise no less than $455.Write and solve an inequality to determine how much more money Rhonda must raise to reach her goal. Let d represent the amount of money in dollars Rhonda must raise to reach her goal. The phrase [I]no less than[/I] is an inequality using the greater than or equal sign: d >= 455 - 245 d >= [B]210[/B]

Richard earns $2700 a month. He received a 3% raise. What is Richard's new annual salary? Remember 1
Richard earns $2700 a month. He received a 3% raise. What is Richard's new annual salary? Remember 12 months in 1 year $2,700 per month * 12 months = 32,400 per year. A 3% raise means the new salary is: 32,400 * 1.03 = [B]$33,372[/B]

Richard is thrice as old as Alvin. The sum of their ages is 52 years. Find their ages
Richard is thrice as old as Alvin. The sum of their ages is 52 years. Find their ages. Let r be Richard's age. And a be Alvin's age. We have: [LIST=1] [*]r = 3a [*]a + r = 52 [/LIST] Substitute (1) into (2) a + 3a = 52 Group like terms: 4a = 52 [URL='https://www.mathcelebrity.com/1unk.php?num=4a%3D52&pl=Solve']Typing this into the search engine[/URL], we get [B]a = 13[/B]. This means Richard is 3(13) = [B]39[/B]

Rick earns $8.50 per hour at his mothers house office. He plans on working 12.5 hours this week. How
Rick earns $8.50 per hour at his mothers house office. He plans on working 12.5 hours this week. How much money will rick earn. Total Earnings = Hourly Rate * Hours Worked Total Earnings = 8.50 * 12.5 Total Earnings = [B]$106.25[/B]

Rick sold a total of 75 books during the first 22 days of May. If he continues to sell books at the
Rick sold a total of 75 books during the first 22 days of May. If he continues to sell books at the same rate, how many books will he sell during the month of May? Set up a proportion of days to books where n is the number of books sold in May: 22/31 = 75/n Using our [URL='https://www.mathcelebrity.com/proportion-calculator.php?num1=22&num2=75&den1=31&den2=n&propsign=%3D&pl=Calculate+missing+proportion+value']proportion calculator[/URL] and rounding to the next integer, we get: n = [B]106[/B]

Ricky reads 20 pages in 50 minutes. How many minutes does it take him to read one page
Ricky reads 20 pages in 50 minutes. How many minutes does it take him to read one page Set up a proportion of pages per minute where m is the number of minutes to read one page: 20/50 = 1/m To solve this proportion for m, we [URL='https://www.mathcelebrity.com/prop.php?num1=20&num2=1&den1=50&den2=m&propsign=%3D&pl=Calculate+missing+proportion+value']type it in our search engine[/URL] and we get: m = [B]2.5[/B]

Rico was born 6 years after Nico. The sum of their age is 36. How old is Nico?
Rico was born 6 years after Nico. The sum of their age is 36. How old is Nico? Let Rico's age be r Let Nico's age be n We're given two equations: [LIST=1] [*]r = n + 6 [*]n + r = 36 [/LIST] We plug equation (1) into equation (2) for r: n + n + 6 = 36 To solve this equation for n, we [URL='https://www.mathcelebrity.com/1unk.php?num=n%2Bn%2B6%3D36&pl=Solve']type it in our search engine[/URL] and we get: [B]n = 15[/B]

Rigby and Eleanor's combined score on the systems of equations test was 181. Rigby scored 9 more poi
Rigby and Eleanor's combined score on the systems of equations test was 181. Rigby scored 9 more points than Eleanor. What were Eleanor and Rigby's scores? Let Rigby's score be r Let Eleanor's score be e We're given two equations: [LIST=1] [*]r = e + 9 [*]e + r = 181 [/LIST] Substitute equation (1) into equation (2): e + (e + 9) = 181 Group like terms: 2e + 9 = 181 To solve this equation for e, we [URL='https://www.mathcelebrity.com/1unk.php?num=2e%2B9%3D181&pl=Solve']type it in our search engine[/URL] and we get: e = [B]86[/B]

Right Triangles
Free Right Triangles Calculator - This solves for all the pieces of a right triangle based on given inputs using items like the sin ratio, cosine ratio, tangent ratio, and the Pythagorean Theorem as well as the inradius.

Riley is trying to raise money by selling key chains. each key chain costs $2.50. If riley is trying
Riley is trying to raise money by selling key chains. each key chain costs $2.50. If riley is trying to raise $60. How many key chains will he have to sell Let the number of key chains be k. We have the following equation: 2.50k = 60 To solve this equation for k, we [URL='https://www.mathcelebrity.com/1unk.php?num=2.50k%3D60&pl=Solve']type it in our search engine[/URL] and we get: k = [B]24[/B]

Rita has spent $16 so far on gifts. The additional amount she will spend will be less than $14. What
Rita has spent $16 so far on gifts. The additional amount she will spend will be less than $14. What are the possible total amounts she will spend? Rita will spend at least another cent on other gifts above the $16 she spent so far, but no more than $14. Also, the problem says less than 14. 16 + 14 is 30, so that is the top end of her spending. Let's say her remaining spending is s. Set up the inequality for possible spending values. [B]16 < s < 30[/B]

Rob has 40 coins, all dimes and quarters, worth $7.60. How many dimes and how many quarters does he
Rob has 40 coins, all dimes and quarters, worth $7.60. How many dimes and how many quarters does he have? We have two equations where d is the number of dimes and q is the number of quarters: [LIST=1] [*]d + q = 40 [*]0.1d + 0.25q = 7.60 [/LIST] Using our [URL='http://www.mathcelebrity.com/simultaneous-equations.php?term1=d+%2B+q+%3D+40&term2=0.1d+%2B+0.25q+%3D+7.60&pl=Cramers+Method']simultaneous equation calculator[/URL], we get: [B]d = 16 q = 24[/B]

Rob has scores of 73,75 and 79 on three exams. what does he need on the last exam to get an average
Rob has scores of 73,75 and 79 on three exams. what does he need on the last exam to get an average of no less than 80 Using our [URL='https://www.mathcelebrity.com/missingaverage.php?num=73%2C75%2C79&avg=80&pl=Calculate+Missing+Score']missing average calculator[/URL], we find the missing score must be: [B]93[/B]

Robert and Robert go to the movie theater and purchase refreshments for their friends. Robert spend
Robert and Robert go to the movie theater and purchase refreshments for their friends. Robert spends a total of $65.25 on 4 drinks and 9 bags of popcorn. Robert spends a total of $51.75 on 8 drinks and 3 bags of popcorn. Write a system of equations that can be used to find the price of one drink and the price of one bag of popcorn. Using these equations, determine and state the price of a bag of popcorn, to the nearest cent. Let d be the cost of each drink, and p be the price of each popcorn bag. We have 2 equations for our system of equations: [LIST=1] [*][B]4d + 9p = 65.25[/B] [*][B]8d + 3p = 51.75[/B] [/LIST] Using our [URL='http://www.mathcelebrity.com/simultaneous-equations.php?term1=4d+%2B+9p+%3D+65.25&term2=8d+%2B+3p+%3D+51.75&pl=Cramers+Method']system of equations calculator[/URL], we get: [LIST] [*]d = 4.5 [*][B]p = 5.25 <-- Since the problem asks for the cost of each popcorn bag[/B] [/LIST]

Robert buys 3 pounds of bananas at $0.50 per pound and 3 pounds of apples at $1.00 per pound. Which
Robert buys 3 pounds of bananas at $0.50 per pound and 3 pounds of apples at $1.00 per pound. Which of the following expressions represents the total cost of the fruit he bought (in dollars)? Total Cost of Fruit = Bananas in pounds * cost per banana pound + Apples in pounds * cost per apple pound Total Cost of Fruit = 3($0.50) + 3($1.00) Total Cost of Fruit = $1.50 + $3.00 Total Cost of Fruit = [B]$4.00[/B]

Robert has 45 dollars. He buys 6 tshirts and has 7 dollars left over. How much did each tshirt cost?
Let x be the price of one t-shirt. Set up an equation: 6 times the number of t-shirts plus 7 dollars left over get him to a total of 45 6x = 45 - 7 6x = 38 Divide each side by 6 [B]x = 6.33[/B]

Roberto has taken 17 photos photos are placed on each odd number page and the newspaper has 10 pages
Roberto has taken 17 photos photos are placed on each odd number page and the newspaper has 10 pages total. The pages with photographs will have 3 or 4 photos each. How many pages has 3 photos and how many pages have 4 photos? Odd pages are 1, 3, 5, 7, 9 17/5 = 3 with 2 remaining. So all 5 pages have 3 photos. Then with 2 left over, 2 pages get 4 photos. So 5 pages have [B]3 photos, and 2 pages have 2 photos[/B] 3(3) + 4(2) = 9 + 8 = 17

Roberto owns a trucking company. He charges $50 hook up fee and $2 per mile. How much to tow your ca
Roberto owns a trucking company. He charges $50 hook up fee and $2 per mile. How much to tow your car: 1mile , 2miles , 10miles ? The Cost Function C(m) where m is the number of miles is written as: C(m) = 2m + 50 The problem asks for C(1), C(2), and C(10) Calculate C(1) C(1) = 2(1) + 50 C(1) = 2 + 50 C(1) = [B]52[/B] Calculate C(2) C(2) = 2(2) + 50 C(2) = 4 + 50 C(2) = [B]54[/B] Calculate C(10) C(10) = 2(10) + 50 C(10) = 20 + 50 C(10) = [B]70[/B]

Rochelle deposits $4,000 in an IRA. What will be the value (in dollars) of her investment in 25 year
Rochelle deposits $4,000 in an IRA. What will be the value (in dollars) of her investment in 25 years if the investment is earning 8% per year and is compounded continuously? Using our [URL='https://www.mathcelebrity.com/simpint.php?av=&p=4000&int=8&t=25&pl=Continuous+Interest']continuous interest calculator[/URL], we get: [B]29,556.22[/B]

Roger has 9 white buttons. He has 3 fewer brown buttons than white buttons. Roger has 3 more gray bu
Roger has 9 white buttons. He has 3 fewer brown buttons than white buttons. Roger has 3 more gray buttons than brown buttons. How many buttons does Roger have in all? [LIST] [*]9 white buttons [*]3 fewer brown buttons than white buttons. So Brown = 9 white - 3 = 6 brown [*]Gray = 3 more than brown = 6 + 3 = 9 [/LIST] All together: 9 white + 6 brown + 9 gray = [B]24 buttons[/B]

Ronald scored 4 goals in his first soccer game. He then scored the same amount of goals in his next
Ronald scored 4 goals in his first soccer game. He then scored the same amount of goals in his next 3 games. If Ronald has 10 goals total, how many did he score in each game? For the remaining 3 games, he scored 10 - 4 = 6 goals. 6 goals divided by 3 games = [B]2 goals in each game[/B].

Ronnie, liza, vivien, and vina are classmates. They send each other a Valentines card. Find the numb
Ronnie, liza, vivien, and vina are classmates. They send each other a Valentines card. Find the number of Valentines cards they send altogether We've got 4 classmates. Which means each person sends 3 Valentine's cards (to everybody else in the class but themselves): 3 * 3 * 3 * 3 or 4 * 3 = 12 Valentine's cards.

Rosanne takes 190 milligrams of an antibiotic. Every hour, her body breaks down 50% of the drug. How
Rosanne takes 190 milligrams of an antibiotic. Every hour, her body breaks down 50% of the drug. How much will be left after 5 hours Let the antibiotic amount be A(h) where h is the amount of hours after ingestion. We have: A(h) = 190 * (1 - 0.5)^h A(h) = 190 * (0.5)^h The problem asks for A(5): A(5) = 190 * (0.5)^5 A(5) = 190 * 0.03125 A(5) = [B]5.9375 milligrams[/B]

Rose weighs 140 pounds and gains 10%. What percent of her new weight must she lose to get back to 14
Rose weighs 140 pounds and gains 10%. What percent of her new weight must she lose to get back to 140 pounds? Find her new weight after the 10% gain: New Weight = Starting Weight * (1 + 10%) Since 10% is 0.1, we have: New Weight = Starting Weight * (1 + 0.1) New Weight = Starting Weight * (1.1) Plug in our numbers: New Weight = 140 * (1.1) New Weight = 154 To get back to 140, Rose must lose 154 - 140 = 14 pounds. As a percentage of her new weight, [URL='https://www.mathcelebrity.com/perc.php?num=14&den=154&pcheck=1&num1=16&pct1=80&pct2=70&den1=80&idpct1=10&hltype=1&idpct2=90&pct=82&decimal=+65.236&astart=12&aend=20&wp1=20&wp2=30&pl=Calculate']we type 14/154 into our search engine[/URL], and get: [B]9.09% [/B] [I]We read this as, Rose must lose 9.09% of her current body weight of 154 pounds to get back to her starting weight of 140 pounds.[/I]

Roster Notation
Free Roster Notation Calculator - Given a set of numbers, this displays the roster notation

Roulette
Free Roulette Calculator - Calculates the probability for different bets on a roulette wheel including expected return on a monetary bet.

Roulette Cumulative Betting
Free Roulette Cumulative Betting Calculator - This calculator displays the probability and return grid for a roulette scenario where you play x games, betting y per number playing z numbers per game.

Round 12.4263 to the nearest cent
Round 12.4263 to the nearest cent [LIST] [*]Work from right to left [*]3 is not greater than 5, so 6 stays the same [*]6 is 5 or more, so we round .42 up to 43. [/LIST] [B]12.43[/B]

Rounding
Free Rounding Calculator - Rounds a number to the nearest number of your choice

rs+h^2=1 for h
rs+h^2=1 for h Subtract rs from each side to isolate h: rs - rs + h^2 = 1 - rs Cancel the rs on the left side: h^2 = 1 - rs Take the square root of each side: sqrt(h^2) = sqrt(1 - rs) [B]h = +- sqrt(1 -rs)[/B]

Rule of 72
Free Rule of 72 Calculator - Calculates how long it would take money to double (doubling time) using the rule of 72 interest approximation as well as showing the mathematical proof of the Rule of 72.

Rule of Succession
Free Rule of Succession Calculator - Given s successes in n independent trials, this calculates the probability that the next repetition is a success

Run Length Encoding
Free Run Length Encoding Calculator - Given a string, this will determine the run length encoding using repeating patterns of characters.

Running from the top of a flagpole to a hook in the ground there is a rope that is 9 meters long. If
Running from the top of a flagpole to a hook in the ground there is a rope that is 9 meters long. If the hook is 4 meters from the base of the flagpole, how tall is the flagpole? We have a right triangle, with hypotenuse of 9 and side of 4. [URL='https://www.mathcelebrity.com/pythag.php?side1input=&side2input=4&hypinput=9&pl=Solve+Missing+Side']Using our Pythagorean Theorem calculator[/URL], we get a flagpole height of [B]8.063[/B].

Ruth prepares 7 kilograms of dough every hour she works at the bakery. How much dough did Ruth prepa
Ruth prepares 7 kilograms of dough every hour she works at the bakery. How much dough did Ruth prepare if she worked for 5 hours? 7 kilograms of dough per hour * 5 hours of work = 35 kilograms of dough.

Ryan buys candy that costs $4 per pound. He will buy at least 12 pounds of candy. What are the possi
Ryan buys candy that costs $4 per pound. He will buy at least 12 pounds of candy. What are the possible amounts he will spend on candy? Clue for you: the phrase [I]at least[/I] means an inequality. Let s be the spend on candy. Cost = Price * quantity Cost = 4 * 12 Cost = 48 The phrase [I]at least[/I] means greater than or equal to: [B]s >= 48[/B]

s = tu^2 for u
s = tu^2 for u Divide each side by t u^2 = s/t Take the square root of each side [LIST] [*]u = sqrt(s/t) [*]u = -sqrt(s/t) [/LIST] We have two answers due to negative number squared is positive

s dollars saved and she adds d dollars per week for the next twelve weeks
s dollars saved and she adds d dollars per week for the next twelve weeks Total savings come from adding current savings plus weekly savings: [B]s + 12d[/B]

S equals the quotient of r and the sum of r and 8.
S equals the quotient of r and the sum of r and 8. A quotient means a fraction, so we have: [B]S = r/(r + 8)[/B]

S varies jointly with t cubed and v
S varies jointly with t cubed and v Varied jointly means there exists a constant k such that: [B]s = kt^3v[/B]

s=u^2t for t
s=u^2t for t Divide each side by u^2 to isolate t: u^2t/u^2 = s/u^2 Cancel the u^2 on the left side, we get: t = [B]s/u^2[/B]

s=w-10e/m for w
s=w-10e/m for w Add 10e/m to each side to isolate w: s + 10e/m = w - 10e/m + 10e/m Cancel the 10e/m on the right side, and we get: w = [B]s + 10e/m[/B]

Sadie and Connor both play soccer. Connor scored 2 times as many goals as Sadie. Together they score
Sadie and Connor both play soccer. Connor scored 2 times as many goals as Sadie. Together they scored 9 goals. Could Sadie have scored 4 goals? Why or why not? [U]Assumptions:[/U] [LIST] [*]Let Connor's goals be c [*]Let Sadie's goals be s [/LIST] We're given the following simultaneous equations: [LIST=1] [*]c = 2s [*]c + s = 9 [/LIST] We substitute equation (1) into equation (2) for c: 2s + s = 9 To solve the equation for s, we type it in our search equation and we get: s = [B]3[/B] So [U][B]no[/B][/U], Sadie could not have scored 4 goals since s = 3

Salary Converter
Free Salary Converter Calculator - This calculator converts an annual salary to the following measures:
* Monthly
* Weekly
* Daily
* Hourly
* Each Minute
* Each Second


sales 45,000 commission rate is 3.6% and salary is $275
sales 45,000 commission rate is 3.6% and salary is $275 Set up the commission function C(s) where s is the salary: C(s) = Commission * s + salary We're given: C(s) = 45,000, commission = 3.6%, which is 0.036 and salary = 275, so we have: 0.036s + 275 = 45000 To solve for s, we type this equation into our search engine and we get: s = [B]1,242,361.11[/B]

Sales Price Variance
Free Sales Price Variance Calculator - Calculates the Sales Price Variance and Total Variance for a group of products

Sales Tax
Free Sales Tax Calculator - Given a sales price and a total bill, this calculates the sales tax amount and sales tax percentage

Sales tax is currently 9.1%. Write an algebraic expression to represent the total amount paid for an
Sales tax is currently 9.1%. Write an algebraic expression to represent the total amount paid for an item that costs d dollars after tax is added to the purchase. We need to increase the price by 9.1%. Our expression is: [B]1.091d[/B]

Sales tax is directly proportional to cost. If the sales tax on a 46000 automobile is $240, what is
Sales tax is directly proportional to cost. If the sales tax on a 46000 automobile is $240, what is the sales tax on a $9000 automobile? Set up a proportion of sales tax to purchase price where s is the sales tax on a 9000 automobile: 240/46000 = s/9000 [URL='https://www.mathcelebrity.com/prop.php?num1=240&num2=s&den1=46000&den2=9000&propsign=%3D&pl=Calculate+missing+proportion+value']Typing this proportion into our search engine[/URL] and we get: s = [B]46.96[/B]

Sales Tax Question
Cost of an item is $55 the total cost is $58.30 what is the sales tax rate and amount of tax ? [URL='http://www.mathcelebrity.com/tax.php?p=55&tb=58.30&pl=Calculate+Tax']Answer[/URL]

Sally and Adam works a different job. Sally makes $5 per hour and Adam makes $4 per hour. They each
Sally and Adam works a different job. Sally makes $5 per hour and Adam makes $4 per hour. They each earn the same amount per week but Adam works 2 more hours. How many hours a week does Adam work? [LIST] [*]Let [I]s[/I] be the number of hours Sally works every week. [*]Let [I]a[/I] be the number of hours Adam works every week. [*]We are given: a = s + 2 [/LIST] Sally's weekly earnings: 5s Adam's weekly earnings: 4a Since they both earn the same amount each week, we set Sally's earnings equal to Adam's earnings: 5s = 4a But remember, we're given a = s + 2, so we substitute this into Adam's earnings: 5s = 4(s + 2) Multiply through on the right side: 5s = 4s + 8 <-- [URL='https://www.mathcelebrity.com/expand.php?term1=4%28s%2B2%29&pl=Expand']multiplying 4(s + 2)[/URL] [URL='https://www.mathcelebrity.com/1unk.php?num=5s%3D4s%2B8&pl=Solve']Typing this equation into the search engine[/URL], we get s = 8. The problem asks for Adam's earnings (a). We plug s = 8 into Adam's weekly hours: a = s + 2 a = 8 + 2 [B]a = 10[/B]

Sally can paint a room in 7 hours while it takes Steve 6 hours to paint the same room. How long woul
Sally can paint a room in 7 hours while it takes Steve 6 hours to paint the same room. How long would it take them to paint the room if they worked together? [URL='http://www.mathcelebrity.com/workcombine.php?w1=+7&w2=+6&pl=Calculate+Combined+Work+Time']Use our work word problem calculator[/URL] [B]3 hours and 13 minutes[/B]

Sally earns $19.25 per hour. This week she earned $616. Write a two step equation to represent the p
Sally earns $19.25 per hour. This week she earned $616. Write a two step equation to represent the problem Let hours be h. We're given: [B]19.25h = 616[/B]

Sally found 73 seashells on the beach, she gave Mary some of her seashells. She has 10 left. How man
Sally found 73 seashells on the beach, she gave Mary some of her seashells. She has 10 left. How many did she give to Mary? Let the number of seashells Sally gave away as g. We're given: 73 - g = 10 To solve this equation for g, we [URL='https://www.mathcelebrity.com/1unk.php?num=73-g%3D10&pl=Solve']type it in our search engine[/URL] and we get: g = [B]63[/B]

Sally is 4 years older than Mark. Twice Sally's age plus 5 times Mark's age is equal to 64.
Sally is 4 years older than Mark. Twice Sally's age plus 5 times Mark's age is equal to 64. Let Sally's age be s. Let Mark's age be m. We're given two equations: [LIST=1] [*]s = m + 4 [*]2s + 5m = 64 <-- [I]Since Twice means we multiply by 2[/I] [/LIST] Substitute equation (1) into equation (2): 2(m + 4) + 5m = 64 Multiply through: 2m + 8 + 5m = 64 Group like terms: (2 + 5)m + 8 = 64 7m + 8 = 64 [URL='https://www.mathcelebrity.com/1unk.php?num=7m%2B8%3D64&pl=Solve']Type this equation into the search engine[/URL] and we get: m = [B]8[/B]

Sally made 8 more shots then k
Sally made 8 more shots then k [B]k + 8[/B]

Salma purchased a prepaid phone card for 30. Long distance calls cost 9 cents a minute using this ca
Salma purchased a prepaid phone card for 30. Long distance calls cost 9 cents a minute using this card. Salma used her card only once to make a long distance call. If the remaining credit on her card is 28.38, how many minutes did her call last? [U]Set up the equation where m is the number of minutes used:[/U] 0.09m = 30 - 28.38 0.09m = 1.62 [U]Divide each side by 0.09[/U] [B]m = 18[/B]

Sam and Jeremy have ages that are consecutive odd integers. The product of their ages is 783. Which
Sam and Jeremy have ages that are consecutive odd integers. The product of their ages is 783. Which equation could be used to find Jeremy's age, j, if he is the younger man. Let Sam's age be s. Let' Jeremy's age be j. We're given: [LIST=1] [*]s = j + 2 <-- consecutive odd integers [*]sj = 783 [/LIST] Substitute (1) into (2): (j + 2)j = 783 j^2 + 2j = 783 Subtract 783 from each side: j^2 + 2j - 783 = 0 <-- This is the equation to find Jeremy's age. To solve this, [URL='https://www.mathcelebrity.com/quadratic.php?num=j%5E2%2B2j-783%3D0&pl=Solve+Quadratic+Equation&hintnum=+0']we type this quadratic equation into the search engine[/URL] and get: j = 27, j = -29. Since ages cannot be negative, we have: [B]j = 27[/B]

Sam bought 8 new basketball trading cards to add to his collection. The next day his dog ate half of
Sam bought 8 new basketball trading cards to add to his collection. The next day his dog ate half of his collection. There are now only 40 cards left. How many cards did Sam start with? Let the starting about of cards be s. Sam adds 8 new cards, so he has s + 8. Then the dog ate half, so he's left with half. Sam is left with 40 cards: (s + 8)/2 = 40 Cross multiply: s + 8 = 80 [URL='https://www.mathcelebrity.com/1unk.php?num=s%2B8%3D80&pl=Solve']Type s + 8 = 80 into the search engine[/URL], and we get [B]s = 72[/B]

Sam can pick 56 apples in 30 minutes. How many can he pick in 45 minutes?
Sam can pick 56 apples in 30 minutes. How many can he pick in 45 minutes? We set up a proportion of apples to minutes where a is the number of apples Sam can pick in 45 minutes. 56/30 = a/45 Using our math engine, we [URL='https://www.mathcelebrity.com/prop.php?num1=56&num2=a&den1=30&den2=45&propsign=%3D&pl=Calculate+missing+proportion+value']type this proportion into our search box[/URL] and get: a = [B]84 [MEDIA=youtube]tpNHh1jh3XE[/MEDIA][/B]

Sam finished 18 problems in one hour. How many hours will it take same to solve 80 problems
Sam finished 18 problems in one hour. How many hours will it take same to solve 80 problems Set up a proportion of problems to hours where h is the number of hours for 80 problems: 18/1 = 80/h To solve for h, we [URL='https://www.mathcelebrity.com/prop.php?num1=18&num2=80&den1=1&den2=h&propsign=%3D&pl=Calculate+missing+proportion+value']type this proportion into our search engine [/URL]and we get: h = [B]4.44[/B]

Sam had 120 teddy bears in his toy store. He sold 2/3 of them at $12 each. How much did he receive?
Sam had 120 teddy bears in his toy store. He sold 2/3 of them at $12 each. How much did he receive? Revenue = Price * Quantity 12 * 2/3 * 120 12 * 80 [B]960[/B]

Sam has $2.25 in quarters and dimes, and the total number of coins is 12. How many quarters and how
Sam has $2.25 in quarters and dimes, and the total number of coins is 12. How many quarters and how many dimes? Let d be the number of dimes. Let q be the number of quarters. We're given two equations: [LIST=1] [*]0.1d + 0.25q = 2.25 [*]d + q = 12 [/LIST] We have a simultaneous system of equations. We can solve this 3 ways: [LIST] [*][URL='https://www.mathcelebrity.com/simultaneous-equations.php?term1=0.1d+%2B+0.25q+%3D+2.25&term2=d+%2B+q+%3D+12&pl=Substitution']Substitution Method[/URL] [*][URL='https://www.mathcelebrity.com/simultaneous-equations.php?term1=0.1d+%2B+0.25q+%3D+2.25&term2=d+%2B+q+%3D+12&pl=Elimination']Elimination Method[/URL] [*][URL='https://www.mathcelebrity.com/simultaneous-equations.php?term1=0.1d+%2B+0.25q+%3D+2.25&term2=d+%2B+q+%3D+12&pl=Cramers+Method']Cramer's Rule[/URL] [/LIST] No matter which method we choose, we get the same answer: [LIST] [*][B]d = 5[/B] [*][B]q = 7[/B] [/LIST]

Sam has x model planes. Anton has 8 more planes than Sam does. How many model planes Does Anton have
Sam has x model planes. Anton has 8 more planes than Sam does. How many model planes Does Anton have? how many planes do they have together? Sam has x Anton has [B]x + 8[/B] since the word [I]more[/I] means we add The word [I]together[/I] means we add, so we have: Sam + Anton = x + x + 8 Grouping like terms, we have: Sam + Anton = [B]2x + 8[/B]

Sam invested $48,000, some at 6% interest and the rest at 10%. How much did he invest at each rate i
Sam invested $48,000, some at 6% interest and the rest at 10%. How much did he invest at each rate if he received $4,000 in interest in one year? Using our [URL='https://www.mathcelebrity.com/split-fund-interest-calculator.php?p=48000&i1=6&i2=10&itot=4000&pl=Calculate']split fund interest calculator[/URL], we get: [LIST] [*]Fund 1 @ 6% = [B]$20,000[/B] [*]Fund 2 @ 10% = [B]$28,000[/B] [/LIST]

Sam is 52 years old. This is 20 years less than 3 times the age of John. How old is John
Sam is 52 years old. This is 20 years less than 3 times the age of John. How old is John Let John's age be j. We're given the following equation: 3j - 20 = 52 ([I]Less than[/I] means we subtract) To solve for j, we [URL='https://www.mathcelebrity.com/1unk.php?num=3j-20%3D52&pl=Solve']type this equation into our search engine[/URL] and we get: j = [B]24[/B]

Sam is eating a Big Hamburger. The first bite was 20% of the Hamburger, the second bite was 20% of w
Sam is eating a Big Hamburger. The first bite was 20% of the Hamburger, the second bite was 20% of what is left and so every next bite is 20% of what is left. b Is it possible for Sam to eat it all if he will bite 20% of what it is left? [B]No, this will go on for infinity. [/B] The number gets closer to 0 but never hits 0.

Sam leaves school to go home. He walks 10 blocks North and then 8 blocks west. How far is John from
Sam leaves school to go home. He walks 10 blocks North and then 8 blocks west. How far is John from the school? Sam walked at a right angle. His distance from home to school is the hypotenuse. Using our [URL='https://www.mathcelebrity.com/pythag.php?side1input=8&side2input=10&hypinput=&pl=Solve+Missing+Side']Pythagorean theorem calculator[/URL], we get: [B]12.806 blocks[/B]

Sam needs to save $300 to buy a video game system. He is able to save $20 per week. How many weeks w
Sam needs to save $300 to buy a video game system. He is able to save $20 per week. How many weeks will it take till he can buy the video game system? Let w be the number of weeks. We have the following equation: 20w = 300 Using our [URL='http://www.mathcelebrity.com/1unk.php?num=20w%3D300&pl=Solve']equation solver[/URL], we get: [B]w = 15[/B]

Sam purchased n notebooks. They were 4 dollars each. Write an equation to represent the total cost c
Sam purchased n notebooks. They were 4 dollars each. Write an equation to represent the total cost c that Sam paid. Cost Function is: [B]c = 4n[/B] Or, using n as a function variable, we write: c(n) = 4n

Sam's plumbing service charges a $50 diagnostic fee and then $20 per hour. How much money does he ea
Sam's plumbing service charges a $50 diagnostic fee and then $20 per hour. How much money does he earn, m, when he shows up to your house to do a job that takes h hours [U]Set up the cost equation:[/U] m = Hourly Rate * h + service charge [U]Plugging in our numbers, we get:[/U] [B]m = 20h + 50[/B]

Sample Size Reliability for μ
Free Sample Size Reliability for μ Calculator - Given a population standard deviation σ, a reliability (confidence) value or percentage, and a variation, this will calculate the sample size necessary to make that test valid.

Sample Size Requirement for the Difference of Means
Free Sample Size Requirement for the Difference of Means Calculator - Given a population standard deviation 1 of σ1, a population standard deviation 2 of σ2 a reliability (confidence) value or percentage, and a variation, this will calculate the sample size necessary to make that test valid.

Sample Space Probability
Free Sample Space Probability Calculator - Given a sample space S and an Event Set E, this calculates the probability of the event set occuring.

Sandy has 10.0 books, Benny has 24.0 books, and Tim has 33.0 books. How many books do they have toge
Sandy has 10.0 books, Benny has 24.0 books, and Tim has 33.0 books. How many books do they have together? Total Books = Sandy's books + Benny's books + Tim's books Total Books = 10.0 + 24.0 + 33.0 Total Books = [B]67.0[/B]

Sara bought a gas-electric hybrid car. She traveled 481.25 miles and used 9.625 gallons of gas. How
Sara bought a gas-electric hybrid car. She traveled 481.25 miles and used 9.625 gallons of gas. How many miles did the hybrid car travel for each gallon of gas? 481.25/9.625 = [B]50 miles per gallon[/B]

Sara has a box of candies. In the box there are 8 pink candies, 7 purple candies and 5 blue candies.
Sara has a box of candies. In the box there are 8 pink candies, 7 purple candies and 5 blue candies. She takes one candy and records its color. She then puts it back in the box and draws another candy. What is the probability of taking out a pink candy followed by a blue candy? [B][U]Calculate the total number of candies:[/U][/B] Total candies = Pink + Purple + Blue Total candies = 8 + 7 + 5 Total candies = 20 [B][U]Calculate the probability of drawing one pink candy:[/U][/B] P(Pink) = 8/20 Using our [URL='https://www.mathcelebrity.com/fraction.php?frac1=8%2F20&frac2=3%2F8&pl=Simplify']fraction reduction calculator[/URL], we get: P(Pink) = 2/5 [B][U]Calculate the probability of drawing one blue candy:[/U][/B] P(Blue) = 5/20 <-- [I]20 options since Sara replaced her first draw[/I] Using our [URL='https://www.mathcelebrity.com/fraction.php?frac1=5%2F20&frac2=3%2F8&pl=Simplify']fraction reduction calculator[/URL], we get: P(Blue) = 1/4 The problem asks for the probability of a Pink followed by a Blue. Since each event is independent, we multiply: P(Pink, Blue) = P(Pink) * P(Blue) P(Pink, Blue) = 2/5 * 1/4 P(Pink, Blue) = 2/20 Using our [URL='https://www.mathcelebrity.com/fraction.php?frac1=2%2F20&frac2=3%2F8&pl=Simplify']fraction reduction calculator[/URL], we get: P(Pink, Blue) = [B]1/10 or 10%[/B]

Sara opened an account with $800 and withdrew $20 per week. Jordan opened an account with $500 and d
Sara opened an account with $800 and withdrew $20 per week. Jordan opened an account with $500 and deposited $30 per week. In how many weeks will their account be equal? Each week, Sara's account value is: 800 - 20w <-- Subtract because Sara withdraws money each week Each week, Jordan's account value is: 500 + 30w <-- Add because Jordan deposits money each week Set them equal to each other: 800 - 20w = 500 + 30w Using our [URL='http://www.mathcelebrity.com/1unk.php?num=800-20w%3D500%2B30w&pl=Solve']equation solver[/URL], we get w = 6. Check our work: 800 - 20(6) 800 - 120 680 500 + 30(6) 500 + 180 680

Sara wants to arrange the seven scrabble letters she has in every possible way so she can determine
Sara wants to arrange the seven scrabble letters she has in every possible way so she can determine if she has a 7-letter word. how many different ways are there for Sara to arrange all seven letters? 7! = 7 x 6 x 5 x 4 x 3 x 2 x 1 = [B]5,040 ways[/B]

Sarah has $250 in her account. She withdraws $25 per week. How many weeks can she withdraw money fro
Sarah has $250 in her account. She withdraws $25 per week. How many weeks can she withdraw money from her account and still have money left? Let w be the number of weeks. We have the following equation for the Balance after w weeks: B(w) = 250 - 25w [I]we subtract for withdrawals[/I] The ability to withdrawal money means have a positive or zero balance after withdrawal. So we set up the inequality below: 250 - 25w >= 0 To solve this inequality for w, we [URL='https://www.mathcelebrity.com/1unk.php?num=250-25w%3E%3D0&pl=Solve']type it in our search engine[/URL] and we get: w <= [B]10 So Sarah can withdrawal for up to 10 weeks[/B]

Sarah has 12 apples she divided them in 4 groups. How many are in each group?
Sarah has 12 apples she divided them in 4 groups. How many are in each group? 12 apples per group divided by 4 groups is written as: 12/4 So we have [B]3 groups[/B].

Sarah makes $9 per hour working at a daycare center and $12 per hour working at a restaurant. Next
Sarah makes $9 per hour working at a daycare center and $12 per hour working at a restaurant. Next week, Sarah is scheduled to work 8 hours at the daycare center. Which of the following inequalities represents the number of hours (h) that Sandra needs to work at the restaurant next week to earn at least $156 from these two jobs? Set up Sarah's earnings function E(h) where h is the hours Sarah must work at the restaurant: 12h + 9(8) >= 156 <-- The phrase [I]at least[/I] means greater than or equal to, so we set this up as an inequality. Also, the daycare earnings are $9 per hour * 8 hours Multiplying through and simplifying, we get: 12h + 72 >= 156 We [URL='https://www.mathcelebrity.com/1unk.php?num=12h%2B72%3E%3D156&pl=Solve']type this inequality into the search engine[/URL], and we get: [B]h>=7[/B]

Sarah rolls 2 fair dice and adds the results from each. Work out the probability of getting a total
Sarah rolls 2 fair dice and adds the results from each. Work out the probability of getting a total that is a factor of 6. Factors of 6 are {6, 12} [URL='https://www.mathcelebrity.com/2dice.php?gl=1&pl=6&opdice=1&rolist=+&dby=&ndby=&montect=+']P(Roll a 6)[/URL] = 5/36 [URL='https://www.mathcelebrity.com/2dice.php?gl=1&pl=12&opdice=1&rolist=+&dby=&ndby=&montect=+']P(Roll a 12)[/URL] = 1/36 P(Roll a 6 or Roll a 12) = P(Roll a 6) + P(Roll a 12) P(Roll a 6 or Roll a 12) = 5/36 + 1/36 P(Roll a 6 or Roll a 12) = 6/36 Using our [URL='https://www.mathcelebrity.com/fraction.php?frac1=6%2F36&frac2=3%2F8&pl=Simplify']fraction simplifier[/URL], we see that: P(Roll a 6 or Roll a 12) = [B]1/6[/B]

Sarah sells cookies. She has a base month salary of $500 and makes $50 for every cookie she sells. w
Sarah sells cookies. She has a base month salary of $500 and makes $50 for every cookie she sells. whats is the equation. Let S(c) be the equation for the money Sarah makes selling (c) cookies. We have: S(c) = Cost per cookies * c cookies + Base Salary [B]S(c) = 50c + 500[/B]

Sarah splits her 87 Pokémon cards into 9 piles. How many are left over?
Sarah splits her 87 Pokémon cards into 9 piles. How many are left over? We want the reminder of 87/9, so we t[URL='https://www.mathcelebrity.com/modulus.php?num=87mod9&pl=Calculate+Modulus']ype 87 mod 9 into our search engine and we get[/URL]: 87 mod 9 =[B] 6[/B]

Sarah starts with $300 in her savings account. She babysits and earns $30 a week to add to her accou
Sarah starts with $300 in her savings account. She babysits and earns $30 a week to add to her account. Write a linear equation to model this situation? Enter your answer in y=mx b form with no spaces. Let x be the number of hours Sarah baby sits. Then her account value y is: y = [B]30x + 300[/B]

Sarah’s house has 5 stories. If the first story has 10 windows, and the following stories have one l
Sarah’s house has 5 stories. If the first story has 10 windows, and the following stories have one less window than the story below it, how many windows does Sarah’s house have? [LIST=1] [*]Floor 1 has 10 windows [*]Floor 2 has 9 windows [*]Floor 3 has 8 windows [*]Floor 4 has 7 windows [*]Floor 5 has 6 windows [/LIST] 10 + 9 + 8 + 7 + 6 = [B]40 windows[/B]

SAT Practice Exam
This is a practice exam for the SAT (Standard Aptitude Test).

Savannah is a salesperson who sells computers at an electronics store. She makes a base pay of $90 e
Savannah is a salesperson who sells computers at an electronics store. She makes a base pay of $90 each day and is also paid a commission for each sale she makes. One day, Savannah sold 4 computers and was paid a total of $100. Write an equation for the function P(x), representing Savannah's total pay on a day on which she sells x computers. If base pay is $90 per day, then the total commission Savannah made for selling 4 computers is: Commission = Total Pay - Base Pay Commission = 100 - 90 Commission = $10 Assuming the commission for each computer is equal, we need to find the commission per computer: Commission per computer = Total Commission / Number of Computers Sold Commission per computer = 10/4 Commission per computer = $2.50 Now, we build the Total pay function P(x): Total Pay = Base Pay + Commission * Number of Computers sold [B]P(x) = 90 + 2.5x[/B]

Scientists are studying a cell that divides in half every 15 minutes. How many cells will there by a
Scientists are studying a cell that divides in half every 15 minutes. How many cells will there by after 2.5 hours? Divide 2.5 hours into 15 minute blocks. 2.5 hours = 2(60) + 0.5(60) minutes 2.5 hours = 120 + 30 minutes 2.5 hours = 150 minutes Now determine the amount of 15 minute blocks 150 minutes/15 minutes = 10 blocks or divisions [LIST] [*]We start with 1 cell at time 0, and double it every 15 minutes [*]We have A(0) = 1, we want A(10). [*]Our accumulation function is A(t) = A(0) * 2^t [/LIST] A(10) = 1 * 2^10 A(10) = [B]1024[/B]

Scott and Dylan both leave the park at the same time, but in opposite directions. If Scott travels 1
Scott and Dylan both leave the park at the same time, but in opposite directions. If Scott travels 12 mph and Dylan travels 19 mph, how long until they are 186 miles apart? Hour 1, they are 19 + 12 = 31 miles apart. So each hour, they get 31 miles more apart. When they are [URL='https://www.mathcelebrity.com/fraction.php?frac1=186%2F31&frac2=3%2F8&pl=Simplify']186 miles apart[/URL], we divide this by 31 miles apart per hour: 186/31 = [B]6 hours[/B]

Sean is helping his dad build a tiled walkway in their backyard. The walkway will be 606060 feet lon
Sean is helping his dad build a tiled walkway in their backyard. The walkway will be 60 feet long and 2 feet wide. The local hardware store sells tiles which are 2 by 2 feet and come in boxes of 6. There isn't a calculator for Rational Word Problems.

Sean is helping his dad build a tiled walkway in their backyard. The walkway will be 606060 feet lon
I assume you want to know how many tiles or how many boxes of tile they need? I'll do both: Since each tile covers the full 2 foot width of the walkway, we need to see how many tiles length wise we need. 60/2 = [B]30 tiles[/B] needed to cover the full walkway. Now, each box contains 6 tiles, which means we need 30 tiles/6 tiles per box = [B]5 boxes of tiles[/B]

Sections of a rail way are 66m in length. What is the length of 81 placed to end to end?
Sections of a rail way are 66m in length. What is the length of 81 placed to end to end? We have 81 sections x 66 meters per section = [B]5,346[/B]

Sectoral Balance
Free Sectoral Balance Calculator - Solves for any of the 6 inputs in the Sectoral Balance equation by Wynne Godley

Security Market Line and Treynor Ratio
Free Security Market Line and Treynor Ratio Calculator - Solves for any of the 4 items in the Security Market Line equation, Risk free rate, market return, Β, and expected return as well as calculate the Treynor Ratio.

Security Plus Exam
Exam answers and Study Guide for the Security Plus Exam

Select 6 bills from a combination of 5 different bills
We use the combination formula, 6 choose 5, or 6C5. Using our [URL='http://www.mathcelebrity.com/permutation.php?num=6&den=5&pl=Combinations']combinations calculator[/URL], or entering 6C6 into the search engine, we get [B]6 ways to select.[/B]

Selling a Business and Reinvesting Proceeds
If a business sells for $1,000,000 (hypothetically)and the proceeds are paid out over 5 years, using the following breakdown: 10% in the first year 15% in the second year 25% in years 3 through 5 Calculate the payouts: [LIST] [*]Year 1: 10% * $1,000,000 = $100,000 [*]Year 2: 15% * $1,000,000 = $150,000 [*]Year 3: 25% * $1,000,000 = $250,000 [*]Year 4: 25% * $1,000,000 = $250,000 [*]Year 5: 25% * $1,000,000 = $250,000 [/LIST] To check our work, add up our proceed payouts: $100,000 + $150,000 + $250,000 + $250,000 + $250,000 = $1,000,000

Sequences
Free Sequences Calculator - Given a function a(n) and a count of sequential terms you want to expand (n), this calcuator will determine the first (n) terms of your sequence, {a1, a2, ..., an}

Sergeant U has 360 rounds of ammunition to distribute to Lance Corporal (LCpl) F, Lance Corporal (LC
Sergeant U has 360 rounds of ammunition to distribute to Lance Corporal (LCpl) F, Lance Corporal (LCpl) M and Lance Corporal (LCpl) Z in the ratio 3:5:7. How many rounds did Lance Corporal (LCpl) M receive? Our ratio denominator is: 3 + 5 + 7 = 15 Lance Corporal (LCpl) M gets 5:15 of the ammunition. [URL='https://www.mathcelebrity.com/fraction.php?frac1=5%2F15&frac2=3%2F8&pl=Simplify']Using our fraction simplifier[/URL], we see that 5/15 = 1/3 So we take 360 rounds of ammunition times 1/3: 360/3 = [B]120[/B]

Serial numbers for a product are to be using 3 letters followed by 4 digits. The letters are to be t
Serial numbers for a product are to be using 3 letters followed by 4 digits. The letters are to be taken from the first 8 letters of the alphabet with no repeats. The digits are taken from numbers 0-9 with no repeats. How many serial numbers can be generated The serial number is organized with letters (L) and digits (D) like this: LLLDDDD Here's how we get the serial number: [LIST=1] [*]The first letter can be any of 8 letters A-H [*]The second letter can be any 7 of 8 letters A-H [*]The third letter can be any 6 of 8 letters A-H [*]The fourth digit can be any of 10 digits 0-9 [*]The fifth digit can be any 9 of 10 digits 0-9 [*]The sixth digit can be any 8 of 10 digits 0-9 [*]The seventh digit can be any 7 of 10 digits 0-9 [/LIST] We multiply all possibilities: 8 * 7 * 6 * 10 * 9 * 8 * 7 [B]1,693,440[/B]

Serial numbers for a product are to made using 4 letters followed by 4 numbers. If the letters are t
Serial numbers for a product are to made using 4 letters followed by 4 numbers. If the letters are to be taken from the first 5 letters of the alphabet with repeats possible and the numbers are taken from the digits 0 through 9 with no repeats, how many serial numbers can be generated? First 5 letters of the alphabet are {A, B, C, D, E} The 4 letters can be chosen as possible: 5 * 5 * 5 * 5 The number are not repeatable, so the 4 numbers can be chosen as: 10 * 9 * 8 * 7 since we have one less choice with each pick Grouping letters and numbers together, we have the following serial number combinations: 5 * 5 * 5 * 5 * 10 * 9 * 8 * 7 = [B]3,150,000[/B]

Set B is the set of distinct letters in the word GIFT
Set B is the set of distinct letters in the word GIFT Set B has 4 elements below: B = [B]{G, I, F, T}[/B]

Set C is the set of two-digit even numbers greater than 72 that do not contain the digit 8.
Set C is the set of two-digit even numbers greater than 72 that do not contain the digit 8. First, two-digit numbers mean anything less than 100. Let's, list out our two-digit even numbers greater than 72 but less than 100. C = {74, 76, 78, 80, 82, 84, 86, 88, 90, 92, 94, 96, 98} The problem asks for numbers that do not contain the digit 8. Let's remove those numbers from the list. C = {74, 76, [S]78[/S], [S]80, 82, 84, 86, 88[/S], 90, 92, 94, 96, [S]98[/S]} [B]C = {74, 76, 90, 92, 94, 96} [MEDIA=youtube]_O6nXX0V4zo[/MEDIA][/B]

Set C is the set of two-digit even numbers less than 56 that are divisible by 5
[U]Two digit Numbers less than 56:[/U] {10, 11, 12, ..., 55} [U]Two Digit Even Numbers of that Set:[/U] {10, 12, 14, ..., 54} [U]Two Digit Even numbers Divisible by 5[/U] [B]C = {10, 20, 30, 40, 50}[/B] [I]Note: Even means you can divide it by 2 with no remainder. Divisible by 5 means the number ends in 5 or 0. Since it is even numbers only, end in 0. [MEDIA=youtube]aQKLVxIB-p4[/MEDIA][/I]

Set D is the set of two-digit even numbers less than 67 that are divisible by 5
Set D is the set of two-digit even numbers less than 67 that are divisible by 5 two-digit numbers start at 10. Divisible by 5 means the last digit is either 0 or 5. But even numbers don't end in 5, so we take the two-digit numbers ending in 0: D = {[B]10, 20, 30, 40, 50, 60}[/B]

Set Notation
Free Set Notation Calculator - Given two number sets A and B, this determines the following:
* Union of A and B, denoted A U B
* Intersection of A and B, denoted A ∩ B
* Elements in A not in B, denoted A - B
* Elements in B not in A, denoted B - A
* Symmetric Difference A Δ B
* The Concatenation A · B
* The Cartesian Product A x B
* Cardinality of A = |A|
* Cardinality of B = |B|
* Jaccard Index J(A,B)
* Jaccard Distance Jσ(A,B)
* Dice's Coefficient
* If A is a subset of B
* If B is a subset of A

Set of all consonants in the word,'SECONDARY'
Set of all consonants in the word,'SECONDARY' Our set has 6 elements below: [B]{C, D, N, R, S, Y}[/B]

set of all letters in Australia
set of all letters in Australia We remove duplicate "a's" and treat A and a as the same letters. Our set S is: S = [B]{a, i, l, r, s, t, u}[/B] If we want to find the properties of this set, we visit our [URL='https://www.mathcelebrity.com/powerset.php?num=%7Ba%2Ci%2Cl%2Cr%2Cs%2Ct%2Cu%7D&pl=Show+Power+Set']set notation calculator[/URL].

Set of consonants in the word EDUCATION
Set of consonants in the word EDUCATION Consonants are all letters not vowels, so anything that is not {A, E, I, O, U} Our set of consonants in the word EDUCATION is: [B]{C, D, N, T}[/B]

set of days with the letter n
set of days with the letter n We have the set below: {Mo[B]n[/B]day, Wed[B]n[/B]esday, Su[B]n[/B]day}

set of vowels in the word ALGEBRA
set of vowels in the word ALGEBRA Vowels are A, E, I, O, U. For ALGEBRA, we have A, E, A We group the 2 A's together, so our answer is: [B]{A, E}[/B]

Set Theory Notation
Free Set Theory Notation Calculator - Evaluates and describes various set theory notation

Seth is constantly forgetting the combination to his lock. He has a lock with four dials. (Each ha 1
Seth is constantly forgetting the combination to his lock. He has a lock with four dials. (Each has 10 numbers 0-9). If Seth can try one lock combination per second, how many seconds will it take him to try every possible lock combination? Start with 0001, 0002, all the way to 9999 [URL='https://www.mathcelebrity.com/inclusnumwp.php?num1=0&num2=9999&pl=Count']When you do this[/URL], you get 10,000 combinations. One per second = 10,000 seconds

Seven less than 1/4 of a number is 9.
Seven less than 1/4 of a number is 9. The phrase [I]a number[/I] means an arbitrary variable, let's call it x. x 1/4 of a number means we multiply x by 1/4: x/4 Seven less than this means we subtract 7 from x/4: x/4 - 7 The word [I]is[/I] means an equation, so we set x/4 - 7 equal to 9: [B]x/4 - 7 = 9[/B]

Seven subtracted from the product of 3 and a number is greater than or equal to -26
Seven subtracted from the product of 3 and a number is greater than or equal to -26 [LIST=1] [*]A number means an arbitrary variable, let's call it x. [*]The product of 3 and a number is written as 3x [*]Seven subtracted from 3x is written as 3x - 7 [*]Finally, that entire expression is greater than or equal to -26: [B]3x - 7 >= - 26[/B] [/LIST]

Shalini gave 0.4 of her plums to her brother and 20% to her sister. She kept 16 for herself how many
Shalini gave 0.4 of her plums to her brother and 20% to her sister. She kept 16 for herself how many plums did she have at first? Let p be the number of plums Shalini started with. We have: [LIST] [*]0.4 given to her brother [*]20% which is 0.2 given away to her sister [*]What this means is she kept 1 - (0.4 + 0.2) = 1 - 0.6 = 0.4 for herself [/LIST] 0.4p = 16 Divide each side by 0.4 [B]p = 40[/B]

Shalini gave 0.4 of her plums to her brother and 20% to her sister. She kept 16 for herself. How man
Shalini gave 0.4 of her plums to her brother and 20% to her sister. She kept 16 for herself. How many plums did she have first? Let's convert everything to decimals. 20% = 0.2 So Shalini gave 0.4 + 0.2 = 0.6 of the plums away. Which means she has 1 = 0.6 = 0.4 or 40% left over. 40% represents 16 plums So our equation is 0.4p = 16 where p is the number of plums to start with Divide each side by 0.4 [B]p = 40[/B]

Shanice won 97 pieces of gum playing basketball at the county fair. At school she gave four to every
Shanice won 97 pieces of gum playing basketball at the county fair. At school she gave four to every student in her math class. She only has 5 remaining. How many students are in her class? Let the number of students be s. We have a situation described by the following equation: 4s + 5 = 97 <-- We add 5 since it's left over to get to 97 [URL='https://www.mathcelebrity.com/1unk.php?num=4s%2B5%3D97&pl=Solve']We type this equation into the search engine[/URL] and we get: s = [B]23[/B]

Shanti had 275 red beads and 3 tines as many blue beads as red beads. She used a total of 156 beads
Shanti had 275 red beads and 3 tines as many blue beads as red beads. She used a total of 156 beads to make a bracelet how many beads did she have left? Calculate Blue Beads: Blue Beads =3 * Red Beads Blue Beads = 3(275) Blue Beads = 825 Subtract off the beads Shanti used for the bracelet: 825 - 156 = [B]669[/B]

Sharon is 17 years old. The sum of the ages of Sharon and John is 70
Sharon is 17 years old. The sum of the ages of Sharon and John is 70. John's age is 70 - Sharon's age. John's age is 70 - 17 = [B]53[/B]

Sharpe Ratio
Free Sharpe Ratio Calculator - Calculates the Sharpe ratio given return on assets, risk free rate, and standard deviation

She earns $20 per hour as a carpenter and $25 per hour as a blacksmith, last week Giselle worked bot
She earns $20 per hour as a carpenter and $25 per hour as a blacksmith, last week Giselle worked both jobs for a total of 30 hours, and a total of $690. How long did Giselle work as a carpenter and how long did she work as a blacksmith? Assumptions: [LIST] [*]Let b be the number of hours Giselle worked as a blacksmith [*]Let c be the number of hours Giselle worked as a carpenter [/LIST] Givens: [LIST=1] [*]b + c = 30 [*]25b + 20c = 690 [/LIST] Rearrange equation (1) to solve for b by subtracting c from each side: [LIST=1] [*]b = 30 - c [*]25b + 20c = 690 [/LIST] Substitute equation (1) into equation (2) for b 25(30 - c) + 20c = 690 Multiply through: 750 - 25c + 20c = 690 To solve for c, we [URL='https://www.mathcelebrity.com/1unk.php?num=750-25c%2B20c%3D690&pl=Solve']type this equation into our search engine[/URL] and we get: c = [B]12 [/B] Now, we plug in c = 12 into modified equation (1) to solve for b: b = 30 - 12 b = [B]18[/B]

She ordered 6 large pizzas. Luckily, she had a coupon for 3 off each pizza. If the bill came to 38.9
She ordered 6 large pizzas. Luckily, she had a coupon for 3 off each pizza. If the bill came to 38.94, what was the price for a large pizza? [U]Determine additional amount the pizzas would have cost without the coupon[/U] 6 pizzas * 3 per pizza = 18 [U]Add 18 to our discount price of 38.94[/U] Full price for 6 large pizzas = 38.94 + 18 Full price for 6 large pizzas = 56.94 Let x = full price per pizza before the discount. Set up our equation: 6x = 56.94 Divide each side by 6 [B]x = $9.49[/B]

Sheila is giving $1,204 to her grandchildren. She has 14 and each is going to receive the same amoun
Sheila is giving $1,204 to her grandchildren. She has 14 and each is going to receive the same amount of money. How much will she give to each grandchild? If each grandchild gets the same amount of money, then they each get: 1,204/14 = [B]$86[/B]

Sheila loaded 21 trucks every 28 minutes. At this rate how long will it take to load 12 trucks
Sheila loaded 21 trucks every 28 minutes. At this rate how long will it take to load 12 trucks. Let m be the number of minutes it takes Sheila to load 12 trucks. We set up a proportion of trucks to minutes: 21/28 = 12/m [URL='https://www.mathcelebrity.com/prop.php?num1=21&num2=12&den1=28&den2=m&propsign=%3D&pl=Calculate+missing+proportion+value']Type this proportion into our search engine[/URL],and we get: m = [B]16[/B]

Sheila wants build a rectangular play space for her dog. She has 100 feet of fencing and she wants i
Sheila wants build a rectangular play space for her dog. She has 100 feet of fencing and she wants it to be 5 times as long as it is wide. What dimensions should the play area be? Sheila wants: [LIST=1] [*]l =5w [*]2l + 2w = 100 <-- Perimeter [/LIST] Substitute (1) into (2) 2(5w) + 2w = 100 10w + 2w = 100 12w = 100 Divide each side by 12 [B]w = 8.3333[/B] Which means l = 5(8.3333) -->[B] l = 41.6667[/B]

Shelby has already taken 31 quizzes during past quarters, and she expects to have 1 quiz during each
Shelby has already taken 31 quizzes during past quarters, and she expects to have 1 quiz during each week of this quarter. After attending 9 weeks of school this quarter, how many quizzes will Shelby have taken in total? [U]Calculate the latest quiz amounts:[/U] 1 quiz per week * 9 weeks = 9 quizzes. [U]Now add that to our starting amount of 31 quizzes[/U] 31 + 9 = [B]40 quizzes[/B]

Shen buys a pack of 9 towels for $24.30. Find the unit price in dollars per towel.
Shen buys a pack of 9 towels for $24.30. Find the unit price in dollars per towel. Unit Price = Total Price/Units Unit Price = 24.30/9 Unit Price = [B]2.7[/B]

Sherry is 31 years younger than her mom. The sum of their ages is 61. How old is Sherry?
Sherry is 31 years younger than her mom. The sum of their ages is 61. How old is Sherry? Let Sherry's age be s. Let the mom's age be m. We're given two equations: [LIST=1] [*]s = m - 31 [*]m + s = 61 [/LIST] Substitute equation (1) into equation (2) for s: m + m - 31 = 61 To solve for m, [URL='https://www.mathcelebrity.com/1unk.php?num=m%2Bm-31%3D61&pl=Solve']we type this equation into our search engine[/URL] and we get: m = 46 Now, we plug m = 46 into equation (1) to find Sherry's age s: s = 46 - 31 s = [B]15[/B]

Short Sale Yield Rate
Free Short Sale Yield Rate Calculator - Calculates the Yield Rate on a short sale of stock.

show how 6 people could share 4 sandwiches.
show how 6 people could share 4 sandwiches. [URL='https://www.mathcelebrity.com/search.php?q=4%2F6&x=0&y=0']4/6 = [/URL]2/3 Cut each of the 4 sandwiches in 3 pieces. this gives us 4 * 3 = 12 [B]Each person gets 2[/B] of the 12 slices because 12 * 2/3 = 6

Sierra borrows $310 from her brother to buy a lawn mower. She will repay $85 to start, and then anot
Sierra borrows $310 from her brother to buy a lawn mower. She will repay $85 to start, and then another $25 per week. A. Write an equation that can be used to determine w, the number of weeks it will take for Sierra to repay the entire amount. Let w be the number of weeks. We have the equation: 25w + 85 = 310 [URL='https://www.mathcelebrity.com/1unk.php?num=25w%2B85%3D310&pl=Solve']Type this equation into the search engine[/URL], and we get: w = [B]9[/B]

Sieve of Eratosthenes
Free Sieve of Eratosthenes Calculator - Using the Sieve of Eratosthenes algorithm, this will show how many prime numbers are less than a number (n).

Sigmoid Function
Free Sigmoid Function Calculator - Calculates the Sigmoid Function S(x) given an x value

Sign Test
Free Sign Test Calculator - This will determine whether to accept or reject a null hypothesis based on a number set, mean value, alternative hypothesis, and a significance level using the Sign Test.

Signed Integer Operations
Free Signed Integer Operations Calculator - This performs a string of signed integer operations, either all addition and subtraction, or all multiplication and division.

Simple and Compound and Continuous Interest
Free Simple and Compound and Continuous Interest Calculator - Calculates any of the four parameters of the simple interest formula or compound interest formula or continuous compound formula
1) Principal
2) Accumulated Value (Future Value)
3) Interest
4) Time.

Simple Discount and Compound Discount
Free Simple Discount and Compound Discount Calculator - Given a principal value, interest rate, and time, this calculates the Accumulated Value using Simple Discount and Compound Discount

Simplify 2/sqrt(5)
To do this, we need rationalize the denominator. This means get rid of the radical: Multiply top and bottom by sqrt(5) 2sqrt(5)/sqrt(5) * sqrt(5) sqrt(5) * sqrt(5) = sqrt(25) so we have: 2sqrt(5)/sqrt(25) sqrt(25) = 5, so we have: [B]2sqrt(5)/5[/B] [MEDIA=youtube]jearVN9LhBE[/MEDIA]

Simplify 6x^2y^3(2x^2y)^3
Simplify the monomial in parentheses: 2^3x^2*3y^3 8x^6y^3 Now we update the multiplication: 6x^2y^3(8x^6y^3) 6*8 x^(2 + 6)y^(3 + 3) [B]48x^8y^6[/B] [MEDIA=youtube]rmuDx027gL4[/MEDIA]

Simplify 7sqrt(3) - sqrt(12)
7sqrt(3) is broken down. sqrt(12) is not broken down. Let's find all the factors of 12 and see if we stumble on a perfect square: [LIST] [*]1 * 12 [*]2 * 6 [*]3 * 4 [/LIST] 4 is a perfect square, since sqrt(4) = 2. So sqrt(12) = sqrt(3 * 4) We pull the sqrt(4) = 2 outside the radical and rewrite our problem as: 7sqrt(3) - 2sqrt(3) These are like terms, so we have: (7 - 2)sqrt(3) [B]5sqrt(3) [/B] [MEDIA=youtube]ljXVXWnKiWY[/MEDIA]

Simplify sin(x) * cos(x) * tan(x) * csc(x) * sec(x) * cot(x)
We know the following: [LIST] [*]csc(x) = 1/sin(x) [*]sec(x) = 1/cos(x) [*]cot(x) = 1/tan(x) [/LIST] We can rewrite our original expression as: sin(x) * cos(x) * tan(x)/sin(x) * cos(x) * tan(x) Everything cancels and we are left with [B]1[/B] [MEDIA=youtube]MsL-Ni4Hen4[/MEDIA]

Simplify sin^2(x)/(1 - sin^2(x))
We know from the pythagorean theorem: [SIZE=5][B]sin^2(x) + cos^2(x) = 1[/B] [B]Subtract sin^2(x) from each side and we get:[/B] [B][B]cos^2(x) = 1 - [B]sin^2(x)[/B][/B][/B] [B][B][B]We can rewrite our original expression as:[/B][/B][/B] [B][B][B]sin^2(x)/cos^2(x)[/B][/B][/B] [/SIZE] [B][B][B][SIZE=5]But this expression [/SIZE][SIZE=4]equals[/SIZE][SIZE=5] tan^2(x)[/SIZE][/B][/B][/B] [MEDIA=youtube]zqYg0VRq5Ak[/MEDIA]

Simultaneous Equations
Free Simultaneous Equations Calculator - Solves a system of simultaneous equations with 2 unknowns using the following 3 methods:
1) Substitution Method (Direct Substitution)
2) Elimination Method
3) Cramers Method or Cramers Rule Pick any 3 of the methods to solve the systems of equations 2 equations 2 unknowns

sin(x)cot(x)
sin(x)cot(x) We know that cot(x) = cos(x)/sin(x), so we rewrite this as: sin(x)cos(x)/sin(x) The sin(x) terms cancel and we get: [B]cos(x)[/B]

Since pounds are smaller than tons, i need to ______ the number of pounds by _____
Since pounds are smaller than tons, i need to ______ the number of pounds by _____ [B]Divide[/B] the number of pounds by [B]2,000[/B]

Sine Wave
Free Sine Wave Calculator - Solves for any of the 3 items of the Sine Wave: Peak Value, Average Value, and RMS value given 1 input.

Sinking Fund Depreciation Method
Free Sinking Fund Depreciation Method Calculator - Using the Sinking Fund method of Depreciation, this calculator determines the following:
* Depreciation at time t (Dt)
* Asset Value (A)
* Salvage Value (S)
* Book Value at time t (Bt)

Six friends went out to dinner. Each person ordered the same dinner, which costs $15.85. The friends
Six friends went out to dinner. Each person ordered the same dinner, which costs $15.85. The friends left a combined tip of $14. What was the total of the bill and tip? When all six friends eat the same meal, we calculate the total meal bill before the tip: Total meal bill = Cost per Meal * Number of Friends Total meal bill = 15.85 * 6 Total meal bill = $95.10 Calculate the Total bill and Tip: Total Bill and Tip = Total Meal Bill + Tip Total Bill and Tip = $95.10 + $14 Total Bill and Tip = [B]$109.10[/B]

Six is the principal square root of 36
Six is the principal square root of 36 The two square roots of 36 are: [LIST] [*]+6 [*]-6 [/LIST] The positive square root is known as the principal square root, therefore, this is [B]true[/B].

Six Less than the total of three times a number and negative eight
Six Less than the total of three times a number and negative eight. Let's take this in pieces: Three times a number = 3x The total of this and negative eight means we subtract eight 3x - 8 Six Less than this total means we subtract 6 3x - 8 - 6 Simplify by combining like terms: [B]3x - 14[/B]

Six less than twice a number is at least -1 and at most 1
First, the phrase [I]a number[/I] means we choose an arbitrary variable, let's call it x. Twice a number means we multiply it by 2. 2x Six less than that means we subtract 6 2x - 6 Now, the last piece, we set up an inequality. At least -1 means greater than or equal to 1. At most 1 means less than or equal to 1. Notice, for both points, we include the number. -1 <= 2x - 6 <= 1

Six ounces of canned fish in oil has 396 calories. How many calories does 1 ounce have?
Set up a proportion of ounces to calories: 6/396 = 1/x <-- x is the amount of calories in one ounce Run this through the [URL='http://www.mathcelebrity.com/prop.php?num1=6&num2=1&den1=396&den2=x&propsign=%3D&pl=Calculate+missing+proportion+value']proportion calculator[/URL]: [B]x = 66[/B]

Six Years ago, 12.2% of registered births were to teenage mothers. A sociologist believes that the
Six Years ago, 12.2% of registered births were to teenage mothers. A sociologist believes that the percentage has decreased since then. (a) Which of the following is the hypothesis to be conducted? A. H0: p = 0.122, H1 p > 0.122 B. H0: p = 0.122, H1 p <> 0.122 C. H0: p = 0.122, H1 p < 0.122 (b) Which of the following is a Type I error? A. The sociologist rejects the hypothesis that the percentage of births to teenage mothers is 12.2%, when the true percentage is less than 12.2% B. The sociologist fails to reject the hypothesis that the percentage of births to teenage mothers is 12.2%, when the true percentage is less than 12.2% C. The sociologist rejects the hypothesis that the percentage of births to teenage mothers is 12.2%, when it is the true percentage. c) Which of the following is a Type II error? A. The sociologist rejects the hypothesis that the percentage of births to teenage mothers is 12.2%, when it is the true percentage B. The sociologist fails to reject the hypothesis that the percentage of births to teenage mothers is 12.2%, when it is the true percentage C. The sociologist fails to reject the hypothesis that the percentage of births to teenage mothers is 12.2%, when the true percentage is less than 12.2% (a) [B]C H0: p = 0.122, H1: p < 0.122[/B] because a null hypothesis should take the opposite of what is being assumed. So the assumption is that nothing has changed while the hypothesis is that the rate has decreased. (b) [B]C.[/B] The sociologist rejects the hypothesis that the percentage of births to teenage mothers is 12.2%, when it is the true percentage. Type I Error is rejecting the null hypothesis when it is true c) [B]C.[/B] The sociologist fails to reject the hypothesis that the percentage of births to teenage mothers is 12.2%, when the true percentage is less than 12.2% Type II Error is accepting the null hypothesis when it is false.

Sixteen subtracted from five times a number equals the number plus four
The phrase [I]a number[/I] means an arbitrary variable. We can pick any letter a-z except for i and e. Let's choose x. 5 times a number 5x Sixteen subtracted from five times a number 5x - 16 the number plus 4: x + 4 Equals means we set 5x - 16 equals to x + 4 for our algebraic expression: [B]5x - 16 = x + 4[/B] [B][/B] If you have to solve for x, [URL='https://www.mathcelebrity.com/1unk.php?num=5x-16%3Dx%2B4&pl=Solve']type this expression into our math solver[/URL] and we get: x = [B]5[/B]

Sixty-six to the x th power
Sixty-six to the x th power We raise 66 to the x power: [B]66^x[/B]

slope is 0 and whose y-intercept is 9.
slope is 0 and whose y-intercept is 9. The standard line equation is y = mx + b where m is the slope and b is the y-intercept is b. Plugging in our numbers, we get: y = 0x + 9 y = [B]9[/B]

Small pizzas were $3 and large pizzas were $5. To feed the throng, it was necessary to spend $475 fo
Small pizzas were $3 and large pizzas were $5. To feed the throng, it was necessary to spend $475 for 125 pizzas. How many small pizzas were purchased? Let s be the number of small pizzas and l be the number of large pizzas. We have two given equations: [LIST=1] [*]l + s = 125 [*]3s + 5l = 475 [/LIST] Using our [URL='http://www.mathcelebrity.com/simultaneous-equations.php?term1=l+%2B+s+%3D+125&term2=3s+%2B+5l+%3D+475&pl=Cramers+Method']simultaneous equation calculator[/URL], we get [B]s = 75[/B]:

Soda cans are sold in a local store for 50 cents each. The factory has $900 in fixed costs plus 25 c
Soda cans are sold in a local store for 50 cents each. The factory has $900 in fixed costs plus 25 cents of additional expense for each soda can made. Assuming all soda cans manufactured can be sold, find the break-even point. Calculate the revenue function R(c) where s is the number of sodas sold: R(s) = Sale Price * number of units sold R(s) = 50s Calculate the cost function C(s) where s is the number of sodas sold: C(s) = Variable Cost * s + Fixed Cost C(s) = 0.25s + 900 Our break-even point is found by setting R(s) = C(s): 0.25s + 900 = 50s We [URL='https://www.mathcelebrity.com/1unk.php?num=0.25s%2B900%3D50s&pl=Solve']type this equation into our search engine[/URL] and we get: s = [B]18.09[/B]

Solve 100 / 1/2
Solve 100 / 1/2 Dividing by a fraction is the same as multiplying by the reciprocal of the fraction: [URL='https://www.mathcelebrity.com/fraction.php?frac1=1%2F2&pl=Reciprocal']Reciprocal of 1/2[/URL] = 2 100 * 2 = [B]200[/B]

Solve 11 - 1/2y = 3 + 6x for y
Solve 11 - 1/2y = 3 + 6x for y Subtract 11 from each side so we can isolate the y term: 11 -11 - 1/2y = 3 + 6x - 11 Cancelling the 11's on the left side, we get: -1/2y = 6x - 8 <-- Since 3 - 11 = -8 Multiply both sides of the equation by -2 to remove the -1/2 on the left side: -2(-1/2)y = -2(6x - 8) Simplifying, we get: y = [B]-12x + 16 [MEDIA=youtube]38uwIaj88Lw[/MEDIA][/B]

Solve a= (a + b + c + d)/4 for c
Solve a= (a + b + c + d)/4 for c Cross multiply: 4a = a + b + c + d Subtract a + b+ d from each side to isolate c: 4a - a - b - d = a + b + c + d - a - b - d Canceling the a, b, and d from the right side, we get: c = [B]3a - b - d [/B]

Solve for h. rs + h^2 = l
Solve for h. rs + h^2 = l [U]Subtract rs from each side to isolate h:[/U] rs - rs + h^2 = l - rs [U]Cancel the rs terms on the left side, and we get:[/U] h^2 = l - rs [U]Take the square root of each side:[/U] h = [B]sqrt(l - rs)[/B]

Solve for x
Expand the right side: 1/3x + 1/2 = 6/4x - 10 Simplify as 6/4 is 3/2 x/3 + 1/2 = 3x/2 - 10 Common denominator of 2 and 3 is 6. So we have: 2x/6 + 1/2 = 9x/6 -10 Subtract 2x/6 from each side 7x/6 - 10 = 1/2 Add 10 to each side. 10 is 20/2 7x/6 = 21/2 Using our [URL='http://www.mathcelebrity.com/prop.php?num1=7x&num2=21&den1=6&den2=2&propsign=%3D&pl=Calculate+missing+proportion+value']proportion calculator[/URL], we get: [B]x = 9[/B]

Solve for x
[IMG]https://mathcelebrity.com/community/data/attachments/0/supp-angles.jpg[/IMG] The angle with measurements of 148 degrees lies on a straight line, which means it's supplementanry angle is: 180 - 148 = 32 Since the angle of 2x - 16 and 32 lie on a straight line, their angle sum equals 180: 2x + 16 + 32 = 180 To solve this equation for x, we [URL='https://www.mathcelebrity.com/1unk.php?num=2x%2B16%2B32%3D180&pl=Solve']type it in our math engine [/URL]and we get: x = [B]66[/B]

Solve mgh=1/2mv^2+1/2(2/5)mr^2(v^2/r^2) for v
Solve mgh=1/2mv^2+1/2(2/5)mr^2(v^2/r^2) for v 1/2(2/5) = 1/5 since the 2's cancel r^2/r^2 = 1 So we simplify, and get: mgh=1/2mv^2+1/5(mv^2) for v Divide each side by m, so m's cancel in each term on the left and right side: gh = 1/2v^2 + 1/5(v^2) Combine like terms for v^2 on the right side: 1/2 + 1/5 = 7/10 from our [URL='https://www.mathcelebrity.com/fraction.php?frac1=1%2F2&frac2=1%2F5&pl=Add']fraction calculator[/URL] So we have: gh = 7v^2/10 Multiply each side by 10: 10gh = 7v^2 Now divide each side by 7 10gh/7 = v^2 Take the square root of each side: [B]v = sqrt(10gh/7)[/B]

Solve Problem
based on a sample of size 41, a 95% confidence interval for the mean score of all students,µ, on an aptitude test is from 60 to 66. Find the margin of error

Solve Problem
A sample of 71 college students yields a mean annual income of $3595. Assuming that ? = $898, find the margin of error in estimating µ at the 99% level of confidence

Solve the problem
a confidence interval for a population mean has a margin of error of 0.081. Determine the length of the confidence interval

Solving word problems with the matrix method?
Hello everyone. I am stuck on a work question that we are required to solve using the matrix (or Gauss-Jordan) method. [CENTER]"A car rental company wants to buy 100 new cars. Compact cars cost $12,000 each, intermediate size cars cost $18,000 each, full size cars cost $24,000 each, and the company has a budget of $1,500,000. If they purchase twice as many compact cars as intermediate sized cars, determine the number of cars of each type that they buy, assuming they spend the entire budget." [/CENTER] I am fairly certain that I could solve this easily, except I cannot figure out the proper three equations that correspond to this question. I someone could help me figure them out, it would be greatly appreciated!

Solving word problems with the matrix method?
Let c be the cost of compact cars, i be the cost of intermediate cards, and f be the cost of full-size cars. We have the following equations: [LIST] [*]c + i + f = 100 [*]12,000c + 18,000i + 24,000 f = 1,500,000 [*]c = 2i [/LIST]

Some History teachers at Richmond High School are purchasing tickets for students and their adult ch
Some History teachers at Richmond High School are purchasing tickets for students and their adult chaperones to go on a field trip to a nearby museum. For her class, Mrs. Yang bought 30 student tickets and 30 adult tickets, which cost a total of $750. Mr. Alexander spent $682, getting 28 student tickets and 27 adult tickets. What is the price for each type of ticket? Let the number of adult tickets be a Let the number of student tickets be s We're given two equations: [LIST=1] [*]30a + 30s = 750 [*]27a + 28s = 682 [/LIST] To solve the simultaneous equations, we can use any of three methods below: [LIST] [*][URL='https://www.mathcelebrity.com/simultaneous-equations.php?term1=30a+%2B+30s+%3D+750&term2=27a+%2B+28s+%3D+682&pl=Substitution']Substitution Method[/URL] [*][URL='https://www.mathcelebrity.com/simultaneous-equations.php?term1=30a+%2B+30s+%3D+750&term2=27a+%2B+28s+%3D+682&pl=Elimination']Elimination Method[/URL] [*][URL='https://www.mathcelebrity.com/simultaneous-equations.php?term1=30a+%2B+30s+%3D+750&term2=27a+%2B+28s+%3D+682&pl=Cramers+Method']Cramer's Rule[/URL] [/LIST] No matter what method we use, we get the same answers: [LIST] [*][B]a = 18[/B] [*][B]s = 7[/B] [/LIST]

Some hot dogs come in packages of 8. Why would a baker of hot dog buns package 7 hot dog buns to a p
Some hot dogs come in packages of 8. Why would a baker of hot dog buns package 7 hot dog buns to a package For customers that like to have matching hot dogs and buns, consider this scenario. For the first round, you have one extra hot dog. Now you buy a hot dog buns package. You're over 6 buns. This continues... We want to see when packaging and hot dogs math. Find the least common multiple of 7 and 8 so that packages match. [URL='https://www.mathcelebrity.com/gcflcm.php?num1=7&num2=8&num3=&pl=LCM']LCM(7, 8[/URL][I][URL='https://www.mathcelebrity.com/gcflcm.php?num1=7&num2=8&num3=&pl=LCM']) [/URL]= 56[/I]

Some scientists believe that there are 10^87 atoms in the entire universe. The number googolplex is
Some scientists believe that there are 10^87 atoms in the entire universe. The number googolplex is a 1 followed by a googol of zeros. If each atom in the universe is used as a zero, how many universes would you need in order to have enough zeros to write out completely the number googolplex? 10^100 zeros in the entire googolplex and 10^87 atoms in the universe 10^100 / 10^87 = [B]10^13 times as many zeros in the googolplex as there are atoms in the universe[/B]

Sonia visited a park in California that had redwood trees. When Sonia asked how tall a certain large
Sonia visited a park in California that had redwood trees. When Sonia asked how tall a certain large redwood tree was, the ranger said that he wouldn't tell its height, but would give Sonia a clue. How tall is the redwood tree Sonia asked about? Sonia said the tree is 64 times my height. The tree is also 112 feet taller than the tree next to it. The two trees plus my height total 597.5 feet. [LIST] [*]Rangers's height = n [*]Tree height = 64n [*]Smaller tree height = 64n - 112 [*]Total height = 64n - 112 + 64n = 597.5 [/LIST] Solve for [I]n[/I] in the equation 64n - 112 + 64n = 597.5 [SIZE=5][B]Step 1: Group the n terms on the left hand side:[/B][/SIZE] (64 + 64)n = 128n [SIZE=5][B]Step 2: Form modified equation[/B][/SIZE] 128n - 112 = + 597.5 [SIZE=5][B]Step 3: Group constants:[/B][/SIZE] We need to group our constants -112 and 597.5. To do that, we add 112 to both sides 128n - 112 + 112 = 597.5 + 112 [SIZE=5][B]Step 4: Cancel 112 on the left side:[/B][/SIZE] 128n = 709.5 [SIZE=5][B]Step 5: Divide each side of the equation by 128[/B][/SIZE] 128n/128 = 709.5/128 n = 5.54296875 Tree height = 64 * ranger height Tree height = 64 * 5.54296875 Tree height = [B]354.75 feet[/B]

Sophie and Claire are having a foot race. Claire is given a 100-foot head-start. If Sophie is runn
Sophie and Claire are having a foot race. Claire is given a 100-foot head-start. If Sophie is running at 5 feet per second and Claire is running at 3 feet per second. i. After how many seconds will Sophie catch Claire? ii. If the race is 500 feet, who wins? i. Sophie's distance formula is given as D = 5s Claire's distance formula is given as D = 3s + 100 Set them equal to each other 5s = 3s + 100 Subtract 3s from both sides: 2s = 100 Divide each side by 2 [B]s = 50[/B] ii. [B]Sophie since after 50 seconds, she takes the lead and never gives it back.[/B]

Sound takes 5 seconds to go one mile. Clark is standing near a rock wall and when he shouts, it take
Sound takes 5 seconds to go one mile. Clark is standing near a rock wall and when he shouts, it takes 20 seconds for the echo to reach his ears. How far away is the rock wall? The sound makes a round trip from Clark to the wall back to Clark. 20 seconds / 5 seconds per mile = 4 miles 4 miles / 2 for round trip = [B]2 miles[/B]

Sound travels about 340 m/s. The function d(t) = 340t give the distance d(t),in meters., that sound
Sound travels about 340 m/s. The function d(t) = 340t give the distance d(t),in meters., that sound travel in T seconds. How far goes sound traveling 59s? What we want is d(59) d(59) = 340m/s(59s) = [B]20,060m[/B]

Special Triangles: Isosceles and 30-60-90
Free Special Triangles: Isosceles and 30-60-90 Calculator - Given an Isosceles triangle (45-45-90) or 30-60-90 right triangle, the calculator will solve the 2 remaining sides of the triangle given one side entered.

Speed Conversions
Free Speed Conversions Calculator - This converts between the following speed measurements:
* mph
* ft/s
* km/h
* m/sec

spent $19.05. ended with $7.45. how much did you start with?
spent $19.05. ended with $7.45. how much did you start with? Let s be the amount we started with. We're given: s - 19.05 = 7.45 To solve this equation for s, we t[URL='https://www.mathcelebrity.com/1unk.php?num=s-19.05%3D7.45&pl=Solve']ype it in our math engine [/URL]and we get: [B]s = 26.5[/B]

Split Fund Interest
Free Split Fund Interest Calculator - Given an initial principal amount, interest rate on Fund 1, interest rate on Fund 2, and a total interest paid, calculates the amount invested in each fund.

Sports Pool Generator
Free Sports Pool Generator Calculator - This generator produces the following two sports (office) pools with shuffled scoring numbers (0 - 9):
1) Blank Sports Pool: This button generates a blank sports pool grid with shuffled numbers
2) Sports Pool with Names: This sports pool allows you to enter up to 100 names which will be randomly dropped into the blank grid boxes from Option 1 above.

This is easily copied and pasted into a program like Microsoft Word so that you can format it to your liking.

Sports radio stations numbered 220 in 1996. The number of sports radio stations has since increased
Sports radio stations numbered 220 in 1996. The number of sports radio stations has since increased by approximately 14.3% per year. Write an equation for the number of sports radio stations for t years after 1996. If the trend continues, predict the number of sports radio stations in 2015. Equation - where t is the number of years after 1996: R(t) = 220(1.143)^t We Want R(t) for 2015 t = 2015 - 1996 = 19 R(19) = 220(1.143)^19 R(19) = 220 * 12.672969 [B]R(19) = 2788.05 ~ 2,788[/B]

SportStation.Store - #1 Sports Equipment Online Store!
SportStation.Store is one of America's top retailers of name brand sporting goods and accessories. SportStation.Store product mix includes athletic shoes, apparel and accessories, as well as a broad selection of outdoor and athletic equipment for team sports, fitness, tennis, golf, winter and summer recreation and roller sports. SportStation.Store offering FREE Shipping worldwide! Without minimum orders! sports accessories,shoes,sporting goods store,sporting goods,sports shoes,sport shop,fitness equipment,gym accessories,dunham's sporting goods,gear sport,modell's sporting goods,sports gear,sports equipment,sporting goods nyc,sports equipment stores,sports items,daves sporting goods,disks sporting goods,sportswear online,5 sporting goods,vans sporting goods,sporting goods san diego,shields sporting goods,sporting goods denver,good sports,sports equipment shop,champs sporting goods,online sports shop,discount sporting goods,fitness accessories,sporting goods shop,sports products,online sports store,louisville sporting goods,olympia sporting goods,allen sporting goods,womens sports gear,sporting goods boulder,sporting goods suppliers,sporting goods coupons,cardiff sports gear,sports car accessories,gears sports,sporting goods uk,bell's sporting goods,sporting goods brooklyn,university sporting goods,sporting goods memphis,sell sports equipment,sporting goods oakland,decks sporting goods,sporting goods calgary,turf sporting goods,sports equipment for sale,western sporting goods,sporting goods companies,sports4good,gear for sports sale,sports equipment organizer,kids sporting goods,riverside sporting goods,allsports equipment,nashville sporting goods,sporting goods phoenix,temples sporting goods,sporting goods chicago,russell sporting goods,tennessee sporting goods,sports equipment logos,sporting goods sarasota,sporting goods miami,sporting goods modesto,sporting goods kalamazoo,pictures of sports equipment,cheap sports equipment,duluth sporting goods,outdoor sports equipment,world sporting goods,sporting goods mn,sporting goods regina,sporting goods knoxville,sporting goods greensboro,sporting goods fort myers,sporting goods saskatoon,sporting goods pensacola,sporting goods new york,maui sporting goods,sports equipment london,michigan sporting goods,phoenix sporting goods,sports and goods,kent sporting goods,buffalo sporting goods,discount sports gear,pats sporting goods,williams sporting goods,sports gear shop,sporting goods tampa,sporting goods green bay,sporting goods fort wayne,sporting goods pittsburgh,sporting goods hawaii,sporting goods raleigh,brown bear sporting goods,sports equipment brands,sports equipment images,sports equipment of toronto,dallas sporting goods,sporting goods dallas,cincinnati sporting goods,keeper sports products,sporting goods fort worth,sporting goods ottawa,sports and equipment,football sports equipment,madison sporting goods,sale sporting goods,american sports store,sporting goods atlanta,sports headgear,sporting goods san jose,home sport equipment,stitches athletic gear,sports equipment ireland,sports gear stores,sporting goods portland,sporting goods canada,discount sports equipment,sports accessories near me,buy sports equipment,durham sporting goods,sports equipment canada,sporting goods everett,kids sports gear,sports gear uk,sporting goods houston,fargo sporting goods,sporting goods omaha,cheap sporting goods,sporting goods outlet,dog sport equipment,sports gear sale,sports supply store,sports equipment perth,sports accessories shop,online sporting goods stores,sports items online,sporting goods online,sports accessories online,buy sports goods online,exercise accessories,sports equipment online,sport accessories store,sports gear online,sports equipment uk,legends sporting goods,sporting goods scottsdale,kids sports equipment,fencing sport equipment,sports day equipment,sport trac accessories,gopher sports equipment,discount sporting goods stores online,sporting goods website,graham sporting goods,sporting goods nj,national sporting goods,team sports gear,sporting goods retailers,custom sports gear,bass sporting goods,davis sporting goods,sporting goods utah,sports equipment toronto,franklin sporting goods,big sporting goods,sporting goods seattle,sporting goods reno,pasadena sporting goods,sports equipment suppliers,sporting goods tulsa,toddler sports equipment,athletic accessories,nebraska sporting goods,az sporting goods,sporting goods texas,concord sporting goods,outdoor sport accessories,sporting goods colorado,eastern sporting goods,sports equipment seattle,sporting goods online sales,gear for sports inc,sporting goods olympia,porter sports equipment,running sport accessories,sporting goods ontario,jackson sporting goods,spartan sports equipment,huntington sporting goods,seattle sports gear,sports equipment reviews,disco sporting goods,sporting goods pennsylvania,sports equipment accessories,spartan athletic products,franklin sports products,martins sporting goods,sporting goods shopping,shop sporting goods online,sporting goods market,cooper sports equipment,huntsville sporting goods,sporting goods oregon,winter sporting goods,sports equipment nyc,traps sporting goods,sportsman sporting goods,sports equipment clothing,american sports equipment,sporting goods careers,sports equipment boston,mountain sporting goods,sporting goods ohio,sporting goods houston tx,sporting goods detroit,georgetown sporting goods,sporting goods columbia,sporting goods vendors,auburn sporting goods,sports sporting goods,mountain sports equipment,bargain sports equipment,sporting goods california,watkins sporting goods,quest sporting goods,york sport equipment,copeland's sporting goods,augusta sporting goods,elite sports gear,birmingham sporting goods,sporting goods raleigh nc,prince sporting goods,sports equipment outlet,online sporting goods sales,sports team accessories,california sporting goods,high gear sports,bulk sporting goods,sports accessories website,sporting goods pasadena,sporting goods toledo,sporting goods for cheap,discount sporting goods catalog,sport b accessories,sports gear outlet,sporting goods florida,sporting goods cleveland,majestic sporting goods,discount sporting goods website,sports coaching equipment,youth sports equipment,sports gear organizer,sporting goods online retailers,sporting clays equipment,online purchase of sports goods,lifetime sports equipment,electronic sports equipment,girls sports gear,soccer sports equipment,sports fitness accessories,sports accessories online purchase,sporty accessories,buy sporting goods,navy sports gear,chicago sports gear,sporting goods storage,champion sports equipment,sports accessories online store,best sports accessories,men's athletic gear,sporting goods sites,team sports equipment,sports equipment sites,marcy sports equipment,sports equipment websites,personalized sports gear,sports items online purchase,sports accessories online shopping,sporting goods online shopping,buy sports accessories online,buy online sports goods,sports gear websites,online sports equipment stores,american sporting goods stores,fitness accessories online,best online sporting goods store,online sports accessories store,cheap sporting goods online,online shopping sites for sports goods,mp15 sport accessories,free sports equipment,online sporting goods retailers,indoor sports equipment,buy sports accessories,sports gear storage,cheap sports accessories,sports therapy equipment,fitness sports equipment,mens sports accessories,kids sports accessories,cold weather sports gear,discount sporting goods stores,where to buy sporting goods,womens sports accessories,compression sports gear,redhead sporting goods,bulk sports equipment,smart sport accessories,women's athletic accessories,snow sports gear,cool sports accessories,sports equipment repair,baby sports gear,sports equipment catalog,sports accessories brands,ramsey sporting goods,best sports equipment,sports equipment usa,sporting goods franchise,duke athletic products,sports equipment india,sports equipment shoes,best sporting goods websites,philadelphia sports gear,closeout sporting goods,sports equipment winnipeg,century sporting goods,school sports gear,sports equipment ottawa,sporting goods accessories,snow sports equipment,retro sports gear,new sports equipment,all sports gear,sports equipment and accessories,cheap sports merchandise,sports product design,youth sports gear,sporting goods catalog,sports gear and accessories,sports equipment adelaide,best online sporting goods,shop sport equipment,sports and fitness accessories,toddler sports gear,sports day accessories,sports accessories store near me,sports running accessories,bike sporting goods,kids athletic gear,sports accessories uk,clearance sporting goods,pink sports gear,all sports accessories,athletic speed equipment,sports equipment retailers,cleats sporting goods,aquatic sports equipment,pool sports equipment

sqrt(12)sqrt(6)/sqrt(8)
sqrt(12)sqrt(6)/sqrt(8) Simplified, we have: sqrt(72)/sqrt(8) The top can be written as sqrt(8 * 9) which is sqrt(8) * sqrt(9) So we have: sqrt(8) * sqrt(9)/sqrt(8) Cancelling the sqrt(8), we have: [S]sqrt(8)[/S] * sqrt(9)/[S]sqrt(8)[/S] sqrt(9) =[B] 3[/B]

Square Number
Free Square Number Calculator - This calculator determines the nth square number

Square root of 9136 divided by 43
Square root of 9136 divided by 43 First, [URL='https://www.mathcelebrity.com/powersq.php?num=sqrt%289136%29&pl=Calculate']take the square root of 9136 in our calculator[/URL]: 4 * sqrt(571) Now divide this by 43: [B]4 * sqrt(571) / 43[/B]

square root of the sum of 2 variables
square root of the sum of 2 variables The phrase [I]2 variables[/I] means we choose 2 arbitrary variables, let's call them x and y: x, y The sum of 2 variables means we add: x + y Square root of the sum of 2 variables is written as: [B]sqrt(x + y)[/B]

square root of x times the square root of y
square root of x times the square root of y square root of x: sqrt(x) square root of y: sqrt(y) square root of x times the square root of y [B]sqrt(x) * sqrt(y)[/B]

Square Root Table
Free Square Root Table Calculator - Generates a square root table for the first (n) numbers rounded to (r) digits

Square Roots and Exponents
Free Square Roots and Exponents Calculator - Given a number (n), or a fraction (n/m), and/or an exponent (x), or product of up to 5 radicals, this determines the following:
* The square root of n denoted as √n
* The square root of the fraction n/m denoted as √n/m
* n raised to the xth power denoted as nx (Write without exponents)
* n raised to the xth power raised to the yth power denoted as (nx)y (Write without exponents)
* Product of up to 5 square roots: √abcde
* Write a numeric expression such as 8x8x8x8x8 in exponential form

Squaring a number equals 5 times that number
Squaring a number equals 5 times that number. The phrase [I]a number[/I] means an arbitrary variable, let's call it x. Squaring this number: x^2 5 times this number means we multiply by 5: 5x The phrase [I]equals[/I] means we set both expressions equal to each other: [B]x^2 = 5x [/B] <-- This is our algebraic expression If you want to solve for x, then we subtract 5x from each side: x^2 - 5x = 5x - 5x Cancel the 5x on the right side, leaving us with 0: x^2 - 5x = 0 Factor out x: x(x - 5) So we get x = 0 or [B]x = 5[/B]

Stacy and Travis are rock climbing. Stacys rope is 4 feet shorter than 3 times the length of Travis
Stacy and Travis are rock climbing. Stacys rope is 4 feet shorter than 3 times the length of Travis rope. Stacys rope is 23 feet long. Write and solve an equation to find the length t of Travis rope. Let Stacy's rope be s. Travis's rope be t. We have: s = 3t - 4 s = 23 So [B]3t - 4 = 23 [/B] [URL='http://www.mathcelebrity.com/1unk.php?num=3t-4%3D23&pl=Solve']Paste this equation into our search engine[/URL] to get [B]t = 9[/B].

Stacy sells art prints for $12 each. Her expenses are $2.50 per print, plus $38 for equipment. How m
Stacy sells art prints for $12 each. Her expenses are $2.50 per print, plus $38 for equipment. How many prints must she sell for her revenue to equal her expenses? Let the art prints be p Cost function is 38 + 2p Revenue function is 12p Set cost equal to revenue 12p = 38 + 2p Subtract 2p from each side 10p = 38 Using our [URL='http://www.mathcelebrity.com/1unk.php?num=10p%3D38&pl=Solve']equation calculator[/URL] gives us [B]p = 3.8[/B]

standard deviation of 545 dollars. Find the sample size needed to have a confidence level of 95% and
standard deviation of 545 dollars. Find the sample size needed to have a confidence level of 95% and margin of error 128

standard deviation of 545 dollars. Find the sample size needed to have a confidence level of 95% and
Standard Error (margin of Error) = Standard Deviation / sqrt(n) 128 = 545/sqrt(n) Cross multiply: 128sqrt(n) = 545 Divide by 128 sqrt(n) = 4.2578125 Square both sides: [B]n = 18.1289672852 But we need an integer, so the answer is 19[/B]

Standard Normal Distribution
Free Standard Normal Distribution Calculator - Givena normal distribution z-score critical value, this will generate the probability. Uses the NORMSDIST Excel function.

Standard Notation
Free Standard Notation Calculator - Calculates the following:
* The numeric notation of word notation
* Standard notation of expanded notation

Standard Notation
Free Standard Notation Calculator - Displays the Standard notation of expanded notation

Stanley bought a ruler and a yardstick for $1.25. If the yardstick cost 45 cents more than the ruler
Stanley bought a ruler and a yardstick for $1.25. If the yardstick cost 45 cents more than the ruler, what was the cost of the yardstick? Let r be the cost of the ruler Let y be the cost of the yardstick We're given 2 equations: [LIST=1] [*]r + y = 1.25 [*]y = r + 0.45 [/LIST] Substitute equation (2) into equation (1) for y r + r + 0.45 = 1.25 Solve for [I]r[/I] in the equation r + r + 0.45 = 1.25 [SIZE=5][B]Step 1: Group the r terms on the left hand side:[/B][/SIZE] (1 + 1)r = 2r [SIZE=5][B]Step 2: Form modified equation[/B][/SIZE] 2r + 0.45 = + 1.25 [SIZE=5][B]Step 3: Group constants:[/B][/SIZE] We need to group our constants 0.45 and 1.25. To do that, we subtract 0.45 from both sides 2r + 0.45 - 0.45 = 1.25 - 0.45 [SIZE=5][B]Step 4: Cancel 0.45 on the left side:[/B][/SIZE] 2r = 0.8 [SIZE=5][B]Step 5: Divide each side of the equation by 2[/B][/SIZE] 2r/2 = 0.8/2 r = 0.4 Substitute r = 0.4 into equation (2) above: y = r + 0.45 y = 0.4 + 0.45 r = [B]0.85 [URL='https://www.mathcelebrity.com/1unk.php?num=r%2Br%2B0.45%3D1.25&pl=Solve']Source[/URL][/B]

Stanley earns $1160 a month. He spends $540 every month and saves the rest. How much will he save in
Stanley earns $1160 a month. He spends $540 every month and saves the rest. How much will he save in 4 years? [U]Calculate savings amount per month:[/U] Savings amount per month = Earnings - Spend Savings amount per month = 1160 - 540 Savings amount per month = 620 [U]Convert years to months[/U] 4 years = 12 * 4 months 4 years = 48 months [U]Calculate total savings:[/U] Total Savings = Savings per month * number of months saved Total Savings = 620 * 48 Total Savings = [B]$29,760 [MEDIA=youtube]sbzRra8dSFs[/MEDIA][/B]

Start with q. Multiply by p. Add 3. Divide A
Start with q. Multiply by p. Add 3. Divide A Start with q: q Multiply by p: pq Add 3: pq + 3 Divide A means divide by A. We wrap pq + 3 in parentheses to divide by the sum (pq + 3)/A

Start with t , add 6, divide by 2, then subtract 5.
Start with t , add 6, divide by 2, then subtract 5. Start with t: t Add 6: t + 6 Divide by 2: (t + 6)/2 [I]Add parentheses because we're dividing the [U]quantity[/U] by 2 [/I] Then subtract 5: [B](t + 6)/2 - 5[/B]

Start with t and cube it.
Start with t and cube it. Cubing a variable means raising it to the power of 3: [B]t^3[/B]

Start with x , subtract 6, then times by 3.
Start with x , subtract 6, then times by 3. We start with x: x Subtract 6: x - 6 The phrase [I]times by[/I] means we multiply (x - 6) by 3 [B]3(x - 6) [/B] <-- This is our algebraic expression If the problem asks you to multiply through, then you'd have: 3x - 18

States that begin with the letter C
States that begin with the letter C Our set has 3 elements: [B]{California, Colorado, Connecticut}[/B]

Stella buys one carton of juice. The carton holds 130 fl oz. She fills 5 glasses with juice from the
Stella buys one carton of juice. The carton holds 130 fl oz. She fills 5 glasses with juice from the carton. Each glass holds 8 fl oz. How much fl oz is left in the carton? First, we find out how much juice was poured into glasses from the carton: Poured Juice Weight = Glasses of Juice * Ounces per glass Poured JuiceWeight = 5 * 8 Poured JuiceWeight = 40 Next, we subtract the poured out juice weight from the full carton weight to find out the leftover amount: Leftover Juice = Carton Weight when full - Poured Juice Weight Leftover Juice = 130 - 40 Leftover Juice = [B]90 oz[/B]

Stephanie and her sister go bowling every weekend and have been keeping track of their wins for the
Stephanie and her sister go bowling every weekend and have been keeping track of their wins for the last couple months. So far, Stephanie has won 8 out the total 18 games that they have played. if Stephanie wishes to have an 80% winning record, how many games in a row will Stephanie have to win, without losing? Track each game the percentage [LIST=1] [*]8 out of 18 = 44.44% [*]9 out of 19 = 47.37% [*]10 out of 20 = 50% [*]11 out of 21 = 52.38% [*]12 out of 22 = 54.55% [*]13 out of 23 = 56.52% [*]14 out of 24 = 58.33% [*]15 out of 25 = 60% [*]16 out of 26 = 61.54% [*]17 out of 27 = 62.96% [*]18 out of 28 = 64.29% [*]19 out of 29 = 65.52% [*]20 out of 30 = 66.67% [*]21 out of 31 = 67.74% [*]22 out of 32 = 68.75% [*]23 out of 33 = 69.7% [*]24 out of 34 = 70.59% [*]25 out of 35 = 71.43% [*]26 out of 36 = 72.22% [*]27 out of 37 = 72.97% [*]28 out of 38 = 73.68% [*]29 out of 39 = 74.36% [*]30 out of 40 = 75% [*]31 out of 41 = 75.61% [*]32 out of 42 = 76.19% [*]33 out of 43 = 76.74% [*]34 out of 44 = 77.27% [*]35 out of 45 = 77.78% [*]36 out of 46 = 78.26% [*]37 out of 47 = 78.72% [*]38 out of 48 = 79.17% [*]39 out of 49 = 79.59% [*][B]40 out of 50 = 80%[/B] [/LIST] [B]So our answer is 32 games in a row[/B]

Stephanie spent $6.95 on these 12 chocolates. What was the cost of each chocolate? Give your answer
Stephanie spent $6.95 on these 12 chocolates. What was the cost of each chocolate? Give your answer to the nearest 5 cents $6.95/12 chocolates Divide top and bottom by 12 to get the cost per one chocolate: $6.95/12 = 0.58 cents per chocolate The problem asks us to round to the nearest [I]5 cents[/I]. 5 * 11 = 55 5 * 12 = 60 Since 58 cents is closer to 60, we have [B]60 cents[/B] as our answer

Steve had $200 in his bank account. He made a deposit of $75 and then made a withdrawal of $90. How
Steve had $200 in his bank account. He made a deposit of $75 and then made a withdrawal of $90. How much money does Steve have in his account now? We add deposits 200 + 75 = 275 We subtract withdrawals 275 - 90 = [B]185[/B]

Steve Has Overdrawn His Checking Account By $27. His Bank Charged Him $15 For An Overdraft Fee Then
Steve Has Overdrawn His Checking Account By $27. His Bank Charged Him $15 For An Overdraft Fee Then He Quickly Deposited $100. What Is His Current Balance? [LIST=1] [*]Overdrawn means money he doesn't have, so we go into the negative. Start with -27. [*]A bank charge of $15 means he goes in the negative another $15, so -27 - 15 = -42 [*]Then he deposits $100, so his balance is: $100 - 42 = [B]$58[/B] [/LIST]

Steve woke up and it was -12 Fahrenheit outside the weatherman said it was supposed to warm up to 20
Steve woke up and it was -12 Fahrenheit outside the weatherman said it was supposed to warm up to 20 degrees. how many degrees will the temperature increase We start with a temperature of -12F Warming up means we [U][B]add[/B][/U] degrees to the original temperature. -12 + 20 = [B]+8F[/B]

Steven has some money. If he spends $9, then he will have 3/5 of the amount he started with.
Steven has some money. If he spends $9, then he will have 3/5 of the amount he started with. Let the amount Steven started with be s. We're given: s - 9 = 3s/5 Multiply each side through by 5 to eliminate the fraction: 5(s - 9) = 5(3s/5) Cancel the 5's on the right side and we get: 5s - 45 = 3s To solve for s, we [URL='https://www.mathcelebrity.com/1unk.php?num=5s-45%3D3s&pl=Solve']type this equation into our search engine[/URL] and we get: s = [B]22.5[/B]

Stock A is worth 4.5. Stock B is worth 8.0. Stock C is worth 10.0. She purchased half as many shares
Stock A is worth 4.5. Stock B is worth 8.0. Stock C is worth 10.0. She purchased half as many shares of B as A and half as many shares of C as B. If her investments are worth 660, how many shares of each stock does she own? Let s be the number of shares in Stock A. We have: [LIST=1] [*]A: 4.5s [*]B: 8s/2 = 4s [*]C: 10s/4 = 2.5s [/LIST] Value equation: 4.5s + 4s + 2.5s = 660 Combining like terms: 11s = 660 Using the [URL='http://www.mathcelebrity.com/1unk.php?num=11s%3D660&pl=Solve']equation calculator[/URL], we get [B]s = 60[/B] for Stock A Stock B shares is equal to 1/2A = [B]30[/B] Stock C shares is equal to 1/2B = [B]15[/B]

Stop Your Math Homework at THIS moment
Use the Ziegarnik Effect to find the perfect breaking point for your math homework. [MEDIA=youtube]PtllyMvRE6M[/MEDIA]

Stopping-Braking Distance for a Car
Free Stopping-Braking Distance for a Car Calculator - Calculates the estimated stopping distance of a vehicle given a speed in miles per hour (mph)

Storm Center 7 was tracking a cold front approaching Cedarburg. Before it rolled in, the temperature
Storm Center 7 was tracking a cold front approaching Cedarburg. Before it rolled in, the temperature was – 7°F. Then, the temperature decreased by 9°F. What was the temperature after the cold front rolled in? Using signed integers, we start with 7 below or -7 -7 The temperature decreased by 9 which means we subtract: -7 - 9 or -7 + (-9) [B]-16°F or 16 below 0 [MEDIA=youtube]oJjEhkdnTxA[/MEDIA][/B]

Straight Line Depreciation
Free Straight Line Depreciation Calculator - Solves for Depreciation Charge, Asset Value, Salvage Value, Time, N, and Book Value using the Straight Line Method.

Strain
Free Strain Calculator - Solves for any of the 3 items in the strain equation: Change in Length, Strain, and Original Length

Strategy then Tactics
[URL]https://soundcloud.com/mathcelebrity/organic-seo-part-1-strategy-then-tactics[/URL]

Stress
Free Stress Calculator - Solves for any of the 3 items in the stress formula: Stress, Force, and Area

String Comparison Algorithms
Free String Comparison Algorithms Calculator - Given two strings A and B, this calculates the following items:
1) Similar Text Pair Ranking Score
2) Levenshtein (Edit Distance).

Stuart traveled n miles at a speed of 72 miles per hour. How many seconds did it take Stuart to trav
Stuart traveled n miles at a speed of 72 miles per hour. How many seconds did it take Stuart to travel the n miles? Distance = Rate * Time Time = Distance/Rate Time = n/72 hours 3600 seconds per hour so we have: 3600n/72 [B]50n[/B]

Student-t Distribution Critical Values
Free Student-t Distribution Critical Values Calculator - Given an α value and degrees of freedom, this calculates the right-tailed test and left-tailed test critical values for the Student-t Distribution

students at East Central High School earned $384 selling car washes. They want to make $2690 for a c
Students at East Central High School earned $384 selling car washes. They want to make $2690 for a club trip. What percent of their goal has been reached? 384/2690 = [B]14.28%[/B] using our [URL='http://www.mathcelebrity.com/perc.php?num=384&den=2690&pcheck=1&num1=16&pct1=80&pct2=70&den1=80&idpct1=10&hltype=1&idpct2=90&pct=82&decimal=+65.236&astart=12&aend=20&wp1=20&wp2=30&pl=Calculate']percentage-decimal calculator[/URL]

Students stuff envelopes for extra money. Their initial cost to obtain the information for the job w
Students stuff envelopes for extra money. Their initial cost to obtain the information for the job was $140. Each envelope costs $0.02 and they get paid $0.03per envelope stuffed. Let x represent the number of envelopes stuffed. (a) Express the cost C as a function of x. (b) Express the revenue R as a function of x. (c) Determine analytically the value of x for which revenue equals cost. a) Cost Function [B]C(x) = 140 + 0.02x[/B] b) Revenue Function [B]R(x) = 0.03x[/B] c) Set R(x) = C(x) 140 + 0.02x = 0.03x Using our [URL='http://www.mathcelebrity.com/1unk.php?num=140%2B0.02x%3D0.03x&pl=Solve']equation solver[/URL], we get x = [B]14,000[/B]

Substitute the given values into given formula and solve for the unknown variable. S=4LW + 2 WH; S=
Substitute the given values into given formula and solve for the unknown variable. S = 4LW + 2 WH; S= 144, L= 8, W= 4. H= S = 4LW + 2 WH Substituting our given values, we have: 144 = 4(8)(4) + 2(4)H 144 = 128 + 8H Using our [URL='http://www.mathcelebrity.com/1unk.php?num=128%2B8h%3D144&pl=Solve']equation calculator[/URL], we get: [B]H = 2[/B]

Subtract 12 from the square sum of w and v
Sum of w and v: w + v Square that sum (w + v)^2 Subtract 12 from the squared sum (w + v)^2 - 12

Subtract 4 from the sum of 2x and 5y
Subtract 4 from the sum of 2x and 5y. The sum of 2x and 5y means we add both terms: 2x + 5y Subtract 4 from this sum to get our algebraic expression: [B](2x + 5y) - 4[/B]

subtract 5 from the sum of 3x and 8y
subtract 5 from the sum of 3x and 8y Take this algebraic expression in parts: [U]The sum of 3x and 8y means we add 8y to 3x:[/U] 3x + 8y [U]Subtract 5 from this sum above:[/U] [B]3x + 8y - 5[/B]

Subtract 7 from p, then multiply 5 by the result
Subtract 7 from p, then multiply 5 by the result. Subtract 7 from p p - 7 Multiply 5 by the result: [B]5(p - 7)[/B]

Subtract b from the sum of a and 10
Subtract b from the sum of a and 10 The sum of a and 10 a + 10 Subtract b from this [B]a + 10 - b[/B]

subtract half of a number from 10
subtract half of a number from 10 The phrase [I]a number[/I] means an arbitrary variable, let's call it x: x half of a number means we divide x by 2: x/2 subtract half of a number from 10 [B]10 - x/2[/B]

subtract q from r, then subtract 6 from the result
subtract q from r, then subtract 6 from the result Subtract q from r r - q Then subtract 6 from the results [B](r - q) - 6[/B]

subtract s from q, subtract the result from r, then double what you have
subtract s from q, subtract the result from r, then double what you have Subtract s from q: q - s Subtract the result from r: r - (q - s) Then double what you have: [B]2(r - (q - s))[/B]

subtract the difference of t and s from 8
subtract the difference of t and s from 8 The difference of t and s: t - s Subtract this from 8: 8 - (t - s)

subtract the product of 5 and x from 7
subtract the product of 5 and x from 7 The product of 5 and x means we multiply 5 by x: 5x We subtract this product, 5x, from 7 [B]7 - 5x[/B]

Subtract the quotient of m and 7 from 4
Subtract the quotient of m and 7 from 4 The quotient of m and 7 means we add divide m by 7 m/7 Subtract this quotient from 4 [B]4 - m/7[/B]

subtract w from u, triple the result, then multiply v by what you have
subtract w from u, triple the result, then multiply v by what you have Take this algebraic expression in 3 parts: [U]1) subtract w from u:[/U] u - w [U]2) Triple the result means we multiply u - w by 3:[/U] 3(u - w) [U]3) Multiply v by what you have. [I]What you have[/I] means the result from step 2:[/U] [B]3v(u - w)[/B]

subtract w from v, add the result to u, then triple what you have
subtract w from v, add the result to u, then triple what you have Take this algebraic expression in parts: [LIST=1] [*]Subtract w from v: v - w [*]Add the result to u (the result is #1): u + v - w [*]Triple what you have. This means multiply the result in #2 by 3: [/LIST] [B]3(u + v - w)[/B]

Subtracting 9s shortcut
Subtracting 9s shortcut Add the digits of the larger number [LIST] [*]10 - 9 = 1 + 0 = 1 [*]11 - 9 = 1 + 1 = 2 [*]12 - 9 = 1 + 2 = 3 [*]13 - 9 = 1 + 3 = 4 [*]14 - 9 = 1 + 4= 5 [*]15 - 9=. 1 + 5 = 6 [*]16 - 9 = 1 + 6= 7 [*]17 - 9= 1 + 7 = 8 [*]18 - 9 = 1 + 8 = 9 [*]19 - 9 = 1 + 9 = 10 [/LIST] [MEDIA=youtube]YOHcJ6UG1D8[/MEDIA]

Subtraction Equality Property
Free Subtraction Equality Property Calculator - Demonstrates the Subtraction Equality Property Numerical Properties

Subtraction Property Of Inequality
Free Subtraction Property Of Inequality Calculator - Demonstrates the Subtraction Property Of Inequality Numerical Properties

Success in a binomial event is .15 what is the probability of failure?
Success in a binomial event is .15 what is the probability of failure? Success is represented as p. p = 0.15. The probability of failure q, is written as q = 1 - p q = 1 - 0.15 [B]q = 0.85[/B]

Successor
Free Successor Calculator - Calculates the successor number to a given number

Sue has $25,000 to invest. She deposits some in stocks and the rest in annuities. If the stocks are
Sue has $25,000 to invest. She deposits some in stocks and the rest in annuities. If the stocks are at a rate of 6% and the annuities are at a rate of 3% and Sue wants to earn $1200 by the end of the year, find how much Sue deposited into each. Using our [URL='https://www.mathcelebrity.com/split-fund-interest-calculator.php?p=25000&i1=6&i2=3&itot=1200&pl=Calculate']split fund interest calculator[/URL], we get: [LIST] [*][B]15,000 in stocks[/B] [*][B]10,000 in annuities[/B] [/LIST]

sum of 3 consecutive odd integers equals 1 hundred 17
sum of 3 consecutive odd integers equals 1 hundred 17 The sum of 3 consecutive odd numbers equals 117. What are the 3 odd numbers? 1) Set up an equation where our [I]odd numbers[/I] are n, n + 2, n + 4 2) We increment by 2 for each number since we have [I]odd numbers[/I]. 3) We set this sum of consecutive [I]odd numbers[/I] equal to 117 n + (n + 2) + (n + 4) = 117 [SIZE=5][B]Simplify this equation by grouping variables and constants together:[/B][/SIZE] (n + n + n) + 2 + 4 = 117 3n + 6 = 117 [SIZE=5][B]Subtract 6 from each side to isolate 3n:[/B][/SIZE] 3n + 6 - 6 = 117 - 6 [SIZE=5][B]Cancel the 6 on the left side and we get:[/B][/SIZE] 3n + [S]6[/S] - [S]6[/S] = 117 - 6 3n = 111 [SIZE=5][B]Divide each side of the equation by 3 to isolate n:[/B][/SIZE] 3n/3 = 111/3 [SIZE=5][B]Cancel the 3 on the left side:[/B][/SIZE] [S]3[/S]n/[S]3 [/S]= 111/3 n = 37 Call this n1, so we find our other 2 numbers n2 = n1 + 2 n2 = 37 + 2 n2 = 39 n3 = n2 + 2 n3 = 39 + 2 n3 = 41 [SIZE=5][B]List out the 3 consecutive odd numbers[/B][/SIZE] ([B]37, 39, 41[/B]) 37 ? 1st number, or the Smallest, Minimum, Least Value 39 ? 2nd number 41 ? 3rd or the Largest, Maximum, Highest Value

sum of 5 times h and twice g is equal to 23
sum of 5 times h and twice g is equal to 23 Take this [U]algebraic expressions[/U] problem in pieces. Step 1: 5 times h: 5h Step 2: Twice g means we multiply g by 2: 2g Step 3: sum of 5 times h and twice g means we add 2g to 5h 5h + 2g Step 4: The phrase [I]is equal to[/I] means an equation, so we set 5h + 2g equal to 23: [B]5h + 2g = 23[/B]

sum of a number and 7 is subtracted from 15 the result is 6.
Sum of a number and 7 is subtracted from 15 the result is 6. The phrase [I]a number[/I] means an arbitrary variable, let's call it x. We take this expression in pieces. Sum of a number and 7 x + 7 Subtracted from 15 15 - (x + 7) The result is means an equation, so we set this expression above equal to 6 [B]15 - (x + 7) = 6 <-- This is our algebraic expression[/B] If the problem asks you to solve for x, we Group like terms 15 - x - 7 = 6 8 - x = 6 [URL='https://www.mathcelebrity.com/1unk.php?num=8-x%3D6&pl=Solve']Type 8 - x = 6 into the search engine[/URL], and we get [B]x = 2[/B]

Sum of a number and it's reciprocal is 6. What is the number?
Sum of a number and it's reciprocal is 6. What is the number? Let the number be n. The reciprocal is 1/n. The word [I]is[/I] means an equation, so we set n + 1/n equal to 6 n + 1/n = 6 Multiply each side by n to remove the fraction: n^2 + 1 = 6n Subtract 6n from each side: [B]n^2 - 6n + 1 = 0 [/B]<-- This is our algebraic expression If the problem asks you to solve for n, then you [URL='https://www.mathcelebrity.com/quadratic.php?num=n%5E2-6n%2B1%3D0&pl=Solve+Quadratic+Equation&hintnum=+0']type this quadratic equation into our search engine[/URL].

Sum of Consecutive Numbers
Free Sum of Consecutive Numbers Calculator - Finds the sum of (n) consecutive integers, even or odd as well. Examples include:
sum of 2 consecutive integers
sum of 2 consecutive numbers
sum of 2 consecutive even integers
sum of 2 consecutive odd integers
sum of 2 consecutive even numbers
sum of 2 consecutive odd numbers
sum of two consecutive integers
sum of two consecutive odd integers
sum of two consecutive even integers
sum of two consecutive numbers
sum of two consecutive odd numbers
sum of two consecutive even numbers
sum of 3 consecutive integers
sum of 3 consecutive numbers
sum of 3 consecutive even integers
sum of 3 consecutive odd integers
sum of 3 consecutive even numbers
sum of 3 consecutive odd numbers
sum of three consecutive integers
sum of three consecutive odd integers
sum of three consecutive even integers
sum of three consecutive numbers
sum of three consecutive odd numbers
sum of three consecutive even numbers
sum of 4 consecutive integers
sum of 4 consecutive numbers
sum of 4 consecutive even integers
sum of 4 consecutive odd integers
sum of 4 consecutive even numbers
sum of 4 consecutive odd numbers
sum of four consecutive integers
sum of four consecutive odd integers
sum of four consecutive even integers
sum of four consecutive numbers
sum of four consecutive odd numbers
sum of four consecutive even numbers
sum of 5 consecutive integers
sum of 5 consecutive numbers
sum of 5 consecutive even integers
sum of 5 consecutive odd integers
sum of 5 consecutive even numbers
sum of 5 consecutive odd numbers
sum of five consecutive integers
sum of five consecutive odd integers
sum of five consecutive even integers
sum of five consecutive numbers
sum of five consecutive odd numbers
sum of five consecutive even numbers


Sum of N and its next consecutive even integer is 65
Sum of N and its next consecutive even integer is 65 Next even consecutive integer is N + 2. We have N + (N + 2) = 65. Combine like terms, we have 2N + 2 = 65 [URL='http://www.mathcelebrity.com/1unk.php?num=2n%2B2%3D65&pl=Solve']Running this problem through the search engine[/URL], we get n = 31.5. Meaning this problem is impossible, it cannot be done. n is not an integer, and neither is the next consecutive even integer.

sum of the cube of x and half of y
sum of the cube of x and half of y The cube of x means we raise x to the 3rd power: x^3 half of y means we divide y by 2: y/2 sum of the cube of x and half of y means we add y/2 to x^3 [B]x^3 + y/2[/B]

Sum of the First (n) Numbers
Free Sum of the First (n) Numbers Calculator - Determines the sum of the first (n)
* Whole Numbers
* Natural Numbers
* Even Numbers
* Odd Numbers
* Square Numbers
* Cube Numbers
* Fourth Power Numbers

sum of the squares of u and v
sum of the squares of u and v The square of u means we raise u to the power of 2 u^2 The square of v means we raise v to the power of 2 v^2 The sum means we add v^2 to u^2: [B]u^2 + v^2[/B]

Sum of the Years Digits (SOYD) Depreciation
Free Sum of the Years Digits (SOYD) Depreciation Calculator - Solves for Depreciation Charge, Asset Value, and Book Value using the Sum of the Years Digits Method

sum of twice w and 3 times l
sum of twice w and 3 times l Twice w means we multiply w by 2: 2w 3 times l: 3l When we see the phrase [I]sum of[/I], we add: [B]2w + 3l[/B]

Sum of two consecutive numbers is always odd
Sum of two consecutive numbers is always odd Definition: [LIST] [*]A number which can be written in the form of 2 m where m is an integer, is called an even integer. [*]A number which can be written in the form of 2 m + 1 where m is an integer, is called an odd integer. [/LIST] Take two consecutive integers, one even, and one odd: 2n and 2n + 1 Now add them 2n + (2n+ 1) = 4n + 1 = 2(2 n) + 1 The sum is of the form 2n + 1 (2n is an integer because the product of two integers is an integer) Therefore, the sum of two consecutive integers is an odd number.

Sum to Product and Product to Sum Formulas
Free Sum to Product and Product to Sum Formulas Calculator - Given two angles in degrees of u and v, this determines the following:
* Sin(u) ± Sin(v)
* Cos(u) ± Cos(v)
* Sin(u)Sin(v)
* Cos(u)Cos(v)
* Sin(u)Cos(v)
* Cos(u)Sin(v)
* Sin(u + v)
* Sin(u - v)
* Cos(u + v)
* Cos(u - v)
* Tan(u + v)
* Tan(u - v)

Sun Shadow
Free Sun Shadow Calculator - This solves for various components and scenarios of the sun shadow problem

Super Snack, a convenience store, charges $4.35 for a large chicken sandwich and two large colas. Fo
Super Snack, a convenience store, charges $4.35 for a large chicken sandwich and two large colas. For a large chicken sandwich and a large cola, they charge $4.00. How much are the Super Snack large chicken sandwiches? The difference between the orders is $0.35 and 1 large cola. Therefore, 1 large cola = $0.35. And if we use the first order of one large chicken sandwich and one large cola, we get: Large Chicken Sandwich + 0.35 = 4.35 Subtract 0.35 from each side, and we get: Large Chicken Sandwich = $[B]4.00[/B]

SuperFit Gym charges $14 per month, as well as a one-time membership fee of $25 to join. After how m
SuperFit Gym charges $14 per month, as well as a one-time membership fee of $25 to join. After how many months will I spend a total of $165? [U]Let the number of months be m. We have a total spend T of:[/U] cost per month * m + one-time membership fee = T [U]Plugging in our numbers, we get:[/U] 14m + 25 = 165 To solve this equation for m, we [URL='https://www.mathcelebrity.com/1unk.php?num=14m%2B25%3D165&pl=Solve']type it in our search engine[/URL] and we get: m = [B]10[/B]

Suppose $10000 is invested in a savings account paying 8% interest per year , after 5 years how much
Suppose $10000 is invested in a savings account paying 8% interest per year , after 5 years how much would be in the account compounded continuously Using our [URL='http://www.mathcelebrity.com/simpint.php?av=&p=10000&int=8&t=5&pl=Continuous+Interest']continuous compounding calculator[/URL], we get 14,918.25

Suppose 5% of a package of balloons are yellow balloons. Then what percentage of balloons are a diff
Suppose 5% of a package of balloons are yellow balloons. Then what percentage of balloons are a different color than yellow? Our total possibilities of any color balloon is 100%. So we have: Non-yellow balloons = 100% - yellow balloons percent Non-yellow balloons = 100% - 5% Non-yellow balloons = [B]95%[/B]

Suppose a city's population is 740,000. If the population grows by 12,620 per year, find the populat
Suppose a city's population is 740,000. If the population grows by 12,620 per year, find the population of the city in 7 years Set up the population function P(y) where y is the number of years since now: P(y) = Current population + Growth per year * y Plugging in our numbers at y = 7, we get: P(7) = 740000 + 12620(7) P(7) = 740000 + 88340 P(7) = [B]828,340[/B]

Suppose a city's population is 740,000. If the population grows by 12,620 per year, find the populat
Suppose a city's population is 740,000. If the population grows by 12,620 per year, find the population of the city in 7 years. We setup the population function P(y) where y is the number of years of population growth, g is the growth per year, and P(0) is the original population. P(y) = P(0) + gy Plugging in our numbers of y = 7, g = 12,620, and P(0) = 740,000, we have: P(7) = 740,000 + 12,620 * 7 P(7) = 740,000 + 88,340 P(7) = [B]828,340[/B]

Suppose a firm producing light bulbs wants to know if it can claim that its light bulbs it produces
Suppose a firm producing light bulbs wants to know if it can claim that its light bulbs it produces last 1,000 burning hours (u). To do this, the firm takes a random sample of 100 bulbs and find its average life time (X=980 hrs) and the sample standard deviation s = 80 hrs. If the firm wants to conduct the test at the 1% of significance, what's you final suggestion? (i..e, Should the producer accept the Ho that its light bulbs have a 1,000 burning hrs. at the 1% level of significance?) Ho: u = 1,000 hours. Ha: u <> 1,000 hours. [URL='http://www.mathcelebrity.com/mean_hypothesis.php?xbar=+980&n=+100&stdev=+80&ptype==&mean=+1000&alpha=+0.01&pl=Mean+Hypothesis+Testing']Perform a hypothesis test of the mean[/URL] [B]Yes, accept null hypothesis[/B]

Suppose Briley has 10 coins in quarters and dimes and has a total of 1.45. How many of each coin doe
Suppose Briley has 10 coins in quarters and dimes and has a total of 1.45. How many of each coin does she have? Set up two equations where d is the number of dimes and q is the number of quarters: (1) d + q = 10 (2) 0.1d + 0.25q = 1.45 Rearrange (1) into (3) to solve for d (3) d = 10 - q Now plug (3) into (2) 0.1(10 - q) + 0.25q = 1.45 Multiply through: 1 - 0.1q + 0.25q = 1.45 Combine q terms 0.15q + 1 = 1.45 Subtract 1 from each side 0.15q = 0.45 Divide each side by 0.15 [B]q = 3[/B] Plug our q = 3 value into (3) d = 10 - 3 [B]d = 7[/B]

Suppose Rocky Mountain have 72 centimeters of snow. Warmer weather is melting at the rate of 5.8 cen
Suppose Rocky Mountain have 72 centimeters of snow. Warmer weather is melting at the rate of 5.8 centimeters a day. If snow continues to melt at this rate, after seven days of warm weather, how much snow will be left? Snow remaining = Starting snow - melt rate * days Snow remaining = 72 - 5.8(7) Snow remaining = 72 - 40.6 Snow remaining = [B]31.4 cm[/B]

Suppose that 17 inches of wire costs 51 cents at the same rate, how many inches of wire can be bough
Suppose that 17 inches of wire costs 51 cents at the same rate, how many inches of wire can be bought for 42 cents? Set up a proportion of inches of wire to cost, were w equals the inches of wire at 42 cents. We have: 17/51 = w/42 [URL='https://www.mathcelebrity.com/prop.php?num1=17&num2=w&den1=51&den2=42&propsign=%3D&pl=Calculate+missing+proportion+value']Type this proportion into our search engine[/URL], we get: [B]w = 14[/B]

Suppose that 18% of people own dogs. If you pick two people at random, what is the probability that
Suppose that 18% of people own dogs. If you pick two people at random, what is the probability that they both own a dog? Since each person is independent of the others, we have: P(Person 1 has a dog and person 2 have a dog) = P(person 1 has a dog) * P(person 2 has a dog) P(Person 1 has a dog and person 2 have a dog) = 0.18 * 0.18 P(Person 1 has a dog and person 2 have a dog) = [B]0.0324 or 3.24%[/B]

Suppose that Candidates A and B have moderate political positions, while Candidate C is quite libera
Suppose that Candidates A and B have moderate political positions, while Candidate C is quite liberal. Voter opinions about the candidates are as follows. 35% want A as their first choice, but would also approve of B. 31% want B as their first choice, but would also approve of A. 20% want B as their first choice, and approve of neither A nor C. 10% want C as their first choice, and approve of neither A nor B. [LIST=1] [*]If all voters could vote only for their first choice, which candidate would win by plurality? [*]Which candidate wins by an approval vote? [/LIST] [U]Plurality Voting:[/U] [LIST] [*]A: 35% [*]B: 31% + 20% = 51% [*]C: 10% [/LIST] [B]Candidate B wins[/B] using the plurality voting method and a majority [U]Approval Voting:[/U] [LIST] [*]A: 35% + 31% = 2 approvals [*]B: 35% + 31% + 20% = 3 approvals [*]C: 10% = 1 approval [/LIST] Therefore, [B]Candidate B wins[/B] using the approval voting method

Suppose that h(x) varies directly with x and h(x)=44 when x = 2. What is h(x) when x = 1.5?
Suppose that h(x) varies directly with x and h(x)=44 when x = 2. What is h(x) when x = 1.5? Direct variation means we set up an equation: h(x) = kx where k is the constant of variation. For h(x) = 44 when x = 2, we have: 2k = 44 [URL='https://www.mathcelebrity.com/1unk.php?num=2k%3D44&pl=Solve']Type this equation into our search engine[/URL], we get: k = 22 The question asks for h(x) when x = 1.5. So we set up our variation equation, knowing that k = 22. kx = h(x) With k = 22 and x = 1.5, we get: 22(1.5) = h(x) h(x) = [B]33[/B]

Suppose that J and K are on the number line. If JK=9 and J lies at 4 where could K be located?
Suppose that J and K are on the number line. If JK=9 and J lies at 4 where could K be located? We'd need 9 spaces to the right of 4 or 9 spaces to the left of 4 to have JK be 9. To the right: K = 4 + 9 K = [B]13[/B] K = 4 - 9 K = [B]-5[/B]

Suppose that previously collected traffic data indicate that, during the afternoon rush hour, an ave
Suppose that previously collected traffic data indicate that, during the afternoon rush hour, an average of 4 cars arrive at a toll bridge each second. If it is assumed that cars arrive randomly, and can thus be modeled with Poisson distribution, what is the probability that in the next second, [U][B]NO[/B][/U] cars will arrive? Use the [I]Poisson Distribution[/I] with λ = 4 and x = 0 Using the [URL='http://www.mathcelebrity.com/poisson.php?n=+10&p=+0.4&k=+0&t=+3&pl=P%28X+=+k%29']Poisson Distribution calculator[/URL], we get P(0; 4) = [B]0.0183[/B]

Suppose that Sn = 3 + 1/3 + 1/9 + ... + 1/3(n-2)
Suppose that Sn = 3 + 1/3 + 1/9 + ... + 1/3(n-2) a) Find S10 and S? b) If the common difference in an arithmetic sequence is twice the first term, show that Sn/Sm = n^2/m^2 a) Sum of the geometric sequence is a = 3 and r = 1/3 (a(1 - r)^n)/(1 - r) (3(1 - 1/3)^9)/(1 - 1/3) [B]S10 = 4.499771376[/B] For infinity, as n goes to infinity, the numerator goes to 1 so we have [B]S? = 3(1)/2/3 = 4.5[/B] b) Sum of an arithmetic sequence formula is below: n(a1 + an)/2 an = a1 + (n - 1)2a1 since d = 2a1 n(a1 + a1 + (n - 1)2a1)/2 (2a1n + n^2 - 2a1n)/2 n^2/2 For Sm m(a1 + am)/2 am = a1 + (m - 1)2a1 since d = 2a1 m(a1 + 1 + (m - 1)2a1)/2 (2a1m + m^2 - 2a1m)/2 m^2/2 Sn/Sm = n^2/m^2 (cancel the 2's) S10/S1 = 10^2/1^2 We know S1 = 3 So we have 100(3)/1 [B]S10 = 300[/B]

Suppose that the distance of fly balls hit to the outfield (in baseball) is normally distributed wit
Suppose that the distance of fly balls hit to the outfield (in baseball) is normally distributed with a mean of 250 feet and a standard deviation of 50 feet. We randomly sample 49 fly balls. a. If X = average distance in feet for 49 fly balls, then X ~ _______(_______,_______)
b. What is the probability that the 49 balls traveled an average of less than 240 feet? Sketch the graph. Scale the horizontal axis for X. Shade the region corresponding to the probability. Find the probability.
c. Find the 80th percentile of the distribution of the average of 49 fly balls a. N(250, 50/sqrt(49)) = [B]0.42074[/B] b. Calculate Z-score and probability = 0.08 shown [URL='http://www.mathcelebrity.com/probnormdist.php?xone=+240&mean=+250&stdev=+7.14&n=+1&pl=P%28X+%3C+Z%29']here[/URL] c. Inverse of normal distribution(0.8) = 0.8416. Use NORMSINV(0.8) [URL='http://www.mathcelebrity.com/zcritical.php?a=0.8&pl=Calculate+Critical+Z+Value']calculator[/URL] Using the Z-score formula, we have 0.8416 = (x - 250)/50 x = [B]292.08[/B]

Suppose that the distance of fly balls hit to the outfield (in baseball) is normally distributed wit
Suppose that the distance of fly balls hit to the outfield (in baseball) is normally distributed with a mean of 250 feet and a standard deviation of 50 feet. a. If X = distance in feet for a fly ball, then X ~ b. If one fly ball is randomly chosen from this distribution, what is the probability that this ball traveled fewer than 220 feet? Sketch the graph. Scale the horizontal axis X. Shade the region corresponding to the probability. Find the probability. c. Find the 80th percentile of the distribution of fly balls. Sketch the graph, and write the probability statement. a. [B]N(250, 50/sqrt(1))[/B] b. Calculate [URL='http://www.mathcelebrity.com/probnormdist.php?xone=+220&mean=250&stdev=50&n=+1&pl=P%28X+%3C+Z%29']z-score[/URL] Z = -0.6 and P(Z < -0.6) = [B]0.274253[/B] c. Inverse of normal distribution(0.8) = 0.8416 using NORMSINV(0.8) [URL='http://www.mathcelebrity.com/zcritical.php?a=0.8&pl=Calculate+Critical+Z+Value']calculator[/URL] Z-score formula: 0.8416 = (x - 250)/50
x = [B]292.08[/B]

Suppose that the manager of the Commerce Bank at Glassboro determines that 40% of all depositors hav
Suppose that the manager of the Commerce Bank at Glassboro determines that 40% of all depositors have a multiple accounts at the bank. If you, as a branch manager, select a random sample of 200 depositors, what is the probability that the sample proportion of depositors with multiple accounts is between 35% and 50%? [URL='http://www.mathcelebrity.com/proportion_hypothesis.php?x=50&n=+100&ptype==&p=+0.4&alpha=+0.05&pl=Proportion+Hypothesis+Testing']50% proportion probability[/URL]: z = 2.04124145232 [URL='http://www.mathcelebrity.com/proportion_hypothesis.php?x=+35&n=+100&ptype==&p=+0.4&alpha=+0.05&pl=Proportion+Hypothesis+Testing']35% proportion probability[/URL]: z = -1.02062072616 Now use the [URL='http://www.mathcelebrity.com/zscore.php?z=p%28-1.02062072616

Suppose that the weight (in pounds) of an airplane is a linear function of the amount of fuel (in ga
Suppose that the weight (in pounds) of an airplane is a linear function of the amount of fuel (in gallons) in its tank. When carrying 20 gallons of fuel, the airplane weighs 2012 pounds. When carrying 55 gallons of fuel, it weighs 2208 pounds. How much does the airplane weigh if it is carrying 65 gallons of fuel? Linear functions are written in the form of one dependent variable and one independent variable. Using g as the number of gallons and W(g) as the weight, we have: W(g) = gx + c where c is a constant We are given: [LIST] [*]W(20) = 2012 [*]W(55) = 2208 [/LIST] We want to know W(65) Using our givens, we have: W(20) = 20x + c = 2012 W(55) = 55x + c = 2208 Rearranging both equations, we have: c = 2012 - 20x c = 2208 - 55x Set them both equal to each other: 2012 - 20x = 2208 - 55x Add 55x to each side: 35x + 2012 = 2208 Using our [URL='http://www.mathcelebrity.com/1unk.php?num=35x%2B2012%3D2208&pl=Solve']equation solver[/URL], we see that x is 5.6 Plugging x = 5.6 back into the first equation, we get: c = 2012 - 20(5.6) c = 2012 - 112 c = 2900 Now that we have all our pieces, find W(65) W(65) = 65(5.6) + 2900 W(65) = 264 + 2900 W(65) = [B]3264[/B]

Suppose that you have just purchased a car for $40,000. Historically, the car depreciates by 8% each
Suppose that you have just purchased a car for $40,000. Historically, the car depreciates by 8% each year, so that next year the car is worth $40000(.92). What will the value of the car be after you have owned it for three years? Book Value B(t) at time t is B(t) = 40,000(1-0.08)^t or B(t) = 40,000(0.92)^t At t = 3 we have: B(3) = 40,000(0.92)^3 B(3) = 40,000 * 0.778688 B(3) = [B]31,147.52[/B]

Suppose two number cubes are rolled. What is the probability of rolling a sum greater than 8?
List out the sums greater than 8: (4, 5) (4, 6) (5, 5) (5, 6) (6, 6) (5, 4) (6, 4) (6, 5) Since there are 6 * 6 = 36 total outcomes, we have the probability of the sum greater than 8 as: 8/36 = 2/9

Suppose we need 4 eggs to make a cake. If there are 24 eggs, write an inequality representing the po
Suppose we need 4 eggs to make a cake. If there are 24 eggs, write an inequality representing the possible number of cakes we can make. Set up a proportion of eggs to cakes where c is the number of cakes per 24 eggs: 4/1 <= 24/c [URL='https://www.mathcelebrity.com/prop.php?num1=4&num2=24&den1=1&den2=c&propsign=%3C&pl=Calculate+missing+proportion+value']Typing this proportion inequality into our search engine[/URL], we get: [B]c <= 6[/B]

Suppose x is a natural number. When you divide x by 7 you get a quotient of q and a remainder of 6.
Suppose x is a natural number. When you divide x by 7 you get a quotient of q and a remainder of 6. When you divide x by 11 you get the same quotient but a remainder of 2. Find x. [U]Use the quotient remainder theorem[/U] A = B * Q + R where 0 ? R < B where R is the remainder when you divide A by B Plugging in our numbers for Equation 1 we have: [LIST] [*]A = x [*]B = 7 [*]Q = q [*]R = 6 [*]x = 7 * q + 6 [/LIST] Plugging in our numbers for Equation 2 we have: [LIST] [*]A = x [*]B = 11 [*]Q = q [*]R = 2 [*]x = 11 * q + 2 [/LIST] Set both x values equal to each other: 7q + 6 = 11q + 2 Using our [URL='http://www.mathcelebrity.com/1unk.php?num=7q%2B6%3D11q%2B2&pl=Solve']equation calculator[/URL], we get: q = 1 Plug q = 1 into the first quotient remainder theorem equation, and we get: x = 7(1) + 6 x = 7 + 6 [B]x = 13[/B] Plug q = 1 into the second quotient remainder theorem equation, and we get: x = 11(1) + 2 x = 11 + 2 [B]x = 13[/B]

Suppose you are asked to choose three movies. If there are 20 different movies, in how many ways can
Suppose you are asked to choose three movies. If there are 20 different movies, in how many ways can the three movies be chosen? We want unique combinations, so we have 20 choose 3: We [URL='https://www.mathcelebrity.com/permutation.php?num=20&den=3&pl=Combinations']type 20C3 into our search engine[/URL] and we get: 20C3 = [B]1,140[/B]

Suppose you deposit $1000 in a college fund that pays 7.2% interest compounded monthly. Find the acc
Suppose you deposit $1000 in a college fund that pays 7.2% interest compounded monthly. Find the account balance after 12 years. Round your answer to two decimal places. Using our[URL='https://www.mathcelebrity.com/compoundint.php?bal=1000&nval=12&int=7.2&pl=Monthly'] compound interest balance calculator[/URL], we get: [B]$1,074.42[/B]

Suppose you deposit $3000 in an account paying 2% annual interest, compounded continuously. Use A=Pe
Suppose you deposit $3000 in an account paying 2% annual interest, compounded continuously. Use A=Pert to find the balance after 5 years. A = $3,000 * e^0.02(5) A = $3,000 * e^0.1 A = $3,000 * 1.105171 A = [B]$3,315.51[/B]

Suppose you deposited $1200 in an account paying a compound interest rate of 6.25% quarterly, what w
Suppose you deposited $1200 in an account paying a compound interest rate of 6.25% quarterly, what would the account balance be after 10 years? [URL='https://www.mathcelebrity.com/compoundint.php?bal=1200&nval=40&int=6.25&pl=Quarterly']Using our compound interest with balance calculator[/URL], we get: [B]$2,231.09[/B]

Suppose you have $28.00 in your bank account and start saving $18.25 every week. Your friend has $16
Suppose you have $28.00 in your bank account and start saving $18.25 every week. Your friend has $161.00 in his account and is withdrawing $15 every week. When will your account balances be the same? Set up savings and withdrawal equations where w is the number of weeks. B(w) is the current balance [LIST] [*]You --> B(w) = 18.25w + 28 [*]Your friend --> B(w) = 161 - 15w [/LIST] Set them equal to each other 18.25w + 28 = 161 - 15w [URL='http://www.mathcelebrity.com/1unk.php?num=18.25w%2B28%3D161-15w&pl=Solve']Type that problem into the search engine[/URL], and you get [B]w = 4[/B].

Suppose you invest $1600 at an annual interest rate of 4.6% compounded continuously. How much will
Suppose you invest $1600 at an annual interest rate of 4.6% compounded continuously. How much will you have in the account after 4 years? Using our [URL='http://www.mathcelebrity.com/simpint.php?av=&p=1600&int=4.6&t=4&pl=Continuous+Interest']continuous compound calculator[/URL], we get $1,923.23

Suppose you secured your phone using a passcode. Later, you realized that you forgot the 6-digit cod
Suppose you secured your phone using a passcode. Later, you realized that you forgot the 6-digit code. You only remembered that the code contains the digits 1, 2,3, 4,5 and 6. How many possible codes can there be? 6 possible digits, 1-6 and the code is 6-digits long. So we have: 6 * 6 * 6 * 6 * 6 * 6 = 6^6 = [B]46,656 possible codes[/B]

Suppose you write a book. The printer charges $4 per book to print it, and you spend 5500 on adverti
Suppose you write a book. The printer charges $4 per book to print it, and you spend 5500 on advertising. You sell the book for $15 a copy. How many copies must you sell to break even. Profit per book is: P = 15 - 4 P = 11 We want to know the number of books (b) such that: 11b = 5500 <-- Breakeven means cost equals revenue [URL='https://www.mathcelebrity.com/1unk.php?num=11b%3D5500&pl=Solve']Typing this equation into the search engine[/URL], we get: b = [B]500[/B]

Survival Rates
Free Survival Rates Calculator - Given a set of times and survival population counts, the calculator will determine the following:
Survival Population lx
Mortality Population dx
Survival Probability px
Mortality Probability qx
In addition, the calculator will determine the probability of survival from tx to tx + n

Susan makes and sells purses. The purses cost her $15 each to make, and she sells them for $30 each.
Susan makes and sells purses. The purses cost her $15 each to make, and she sells them for $30 each. This Saturday, she is renting a booth at a craft fair for $50. Write an equation that can be used to find the number of purses Susan must sell to make a profit of $295 Set up the cost function C(p) where p is the number of purses: C(p) = Cost per purse * p + Booth Rental C(p) = 15p + 50 Set up the revenue function R(p) where p is the number of purses: R(p) = Sale price * p R(p) = 30p Set up the profit function which is R(p) - C(p) equal to 295 30p - (15p + 50) = 295 To solve this equation, [URL='https://www.mathcelebrity.com/1unk.php?num=30p-%2815p%2B50%29%3D295&pl=Solve']we type it into our search engine[/URL] and we get: p = [B]23[/B]

Susan works as a tutor for $14 an hour and as a waitress for $13 an hour. This month, she worked a c
Susan works as a tutor for $14 an hour and as a waitress for $13 an hour. This month, she worked a combined total of 104 hours at her two jobs. Let t be the number of hours Susan worked as a tutor this month. Write an expression for the combined total dollar amount she earned this month. Let t be the number of hours for math tutoring and w be the number of hours for waitressing. We're given: [LIST=1] [*]t + w = 104 [*]14t + 13w = D <-- Combined total dollar amount [/LIST]

Susie bought 15 pairs of shoes last year for an avarage of 30$ per pair. She sold each pair for 1/3
Susie bought 15 pairs of shoes last year for an avarage of 30$ per pair. She sold each pair for 1/3 of the avagrage price at a consignment shop. How much money did she make at the consigment shop? Calculate average price: 1/3 the average price is $30/3 = $10 Total money made: Pairs of Shoes * Average Price 15 * 10 = [B]$150[/B]

Symmetric Property
Free Symmetric Property Calculator - Demonstrates the Symmetric property using a number. Numerical Properties

Synthetic Division
Free Synthetic Division Calculator - Using Ruffinis Rule, this performs synthetic division by dividing a polynomial with a maximum degree of 6 by a term (x ± c) where c is a constant root using the factor theorem. The calculator returns a quotient answer that includes a remainder if applicable. Also known as the Rational Zero Theorem

T = mg - mf for f
T = mg - mf for f Subtract mg from each side: T - mg = mg - mg - mf Cancel the mg on the right side and we get: T - mg = -mf Multiply each side by -1: -(T - mg) = -(-mf) mg - T = mf Now Divide each side by m to isolate f: (mg - T)/m = mf/m Cancel the m on the right side and we get: f = [B](mg - T)/m[/B]

t varies directly with the square of r and inversely with w
t varies directly with the square of r and inversely with w There exists a constant k such that: [B]t = kr^2/w[/B] [I]Directly means multiply and inversely means divide[/I]

T-Bill
Free T-Bill Calculator - Calculates any of the four items of the T-Bill (Treasury Bill or TBill) formula:
1) Price (P)
2) Face Value (F)
3) Number of Weeks (w)
4) Yield Rate (y)

T-shirts sell for $19.97 and cost $14.02 to produce. Which equation represents p, the profit, in ter
T-shirts sell for $19.97 and cost $14.02 to produce. Which equation represents p, the profit, in terms of x, the number of t-shirts sold? A) p = $19.97x - $14.02 B) p = x($19.97 - $14.02) C) p = $19.97 + $14.02x D) p = x($19.97 + $14.02) [B]B) p = x($19.97 - $14.02)[/B] [B][/B] [LIST] [*]Profit is Revenue - Cost [*]Each shirt x generates a profit of 19.97 - 14.02 [/LIST]

Take a look at the following sums: 1 = 1 1 + 3 = 4 1 + 3 + 5 = 9 1 + 3 + 5 + 7 = 16 1 + 3 + 5 + 7 +
Take a look at the following sums: 1 = 1 1 + 3 = 4 1 + 3 + 5 = 9 1 + 3 + 5 + 7 = 16 1 + 3 + 5 + 7 + 9 = 25 a. Come up with a conjecture about the sum when you add the first n odd numbers. For example, when you added the first 5 odd numbers (1 + 3 + 5 + 7 + 9), what did you get? What if wanted to add the first 10 odd numbers? Or 100? b. Can you think of a geometric interpretation of this pattern? If you start with one square and add on three more, what can you make? If you now have 4 squares and add on 5 more, what can you make? c. Is there a similar pattern for adding the first n even numbers? 2 = 2 2 + 4 = 6 2 + 4 + 6 = 12 2 + 4 + 6 + 8 = 20 a. The formula is [B]n^2[/B]. The sum of the first 10 odd numbers is [B]100[/B] seen on our s[URL='http://www.mathcelebrity.com/sumofthefirst.php?num=10&pl=Odd+Numbers']um of the first calculator[/URL] The sum of the first 100 odd numbers is [B]10,000[/B] seen on our [URL='http://www.mathcelebrity.com/sumofthefirst.php?num=100&pl=Odd+Numbers']sum of the first calculator[/URL] b. Geometric is 1, 4, 9 which is our [B]n^2[/B] c. The sum of the first n even numbers is denoted as [B]n(n + 1)[/B] seen here for the [URL='http://www.mathcelebrity.com/sumofthefirst.php?num=+10&pl=Even+Numbers']first 10 numbers[/URL]

take away 1 from the cube of e
The cube of e is e^3. Take away 1 means subtract 1 e^3 - 1

take away the product of 12 and p from 25
take away the product of 12 and p from 25 The product of 12 and p means we multiply 12 by p: 12p Take away this product means we subtract 12p from 25: [B]25 - 12p[/B]

Tally Marks
Free Tally Marks Calculator - Shows the tally mark representation (tallies) for a positive integer.

Tamara can proofread 16 pages in 8 minutes. How many minutes will it take her to proofread 108 pages
Tamara can proofread 16 pages in 8 minutes. How many minutes will it take her to proofread 108 pages Set up a proportion of pages to minutes: 16 pages/8 minutes = 108 pages / p minutes We want to solve for p. Type [I][URL='https://www.mathcelebrity.com/prop.php?num1=16&num2=108&den1=8&den2=p&propsign=%3D&pl=Calculate+missing+proportion+value']16/8 = 108/p[/URL][/I] into the search engine. We get p = [B]54 minutes[/B]

Tamira and her 3 friends spent a total of $37 for a large pizza and 4 sodas. Each soda cost $2. Whic
Tamira and her 3 friends spent a total of $37 for a large pizza and 4 sodas. Each soda cost $2. Which equation can be used to find p, the cost of the pizza? We add the cost of the pizza (p) to the 4 sodas @ $2 each to get 37 p + 4(2) = 37 [B]p + 8 = 37 (This is the equation) [/B] If the problem asks you to solve for p, then we [URL='https://www.mathcelebrity.com/1unk.php?num=p%2B8%3D37&pl=Solve']type the equation above into our search engine[/URL] and we get: p = [B]29[/B]

tammy earns $18000 salary with 4% comission on sales. How much should she sell to earn $55,000 total
tammy earns $18000 salary with 4% comission on sales. How much should she sell to earn $55,000 total We have a commission equation below: Sales * Commission percent = Salary We're given 4% commission percent and 55,000 salary. With 4% as 0.04, we have: Sales * 0.04 = 55,000 Divide each side of the equation by 0.04, and we get: Sales = [B]1,375,000[/B]

Target Heart Rate
Free Target Heart Rate Calculator - Given an age, this calculator determines the following 5 target heart rate zones:
Healthy Heart Zone (Warm up) 50 - 60%
Fitness Zone (Fat Burning) 60 - 70%
Aerobic Zone (Endurance Training) 70 - 80%
Anaerobic Zone (Performance Training) 80 - 90%
Red Line (Maximum Effort) 90 - 100%

Tarzan looked at 48 websites in 4 hours. At that rate, how many would he look at in 10 minutes?
Tarzan looked at 48 websites in 4 hours. At that rate, how many would he look at in 10 minutes? 48 websites per hour / 4 hours = 12 websites / hour Since an hour is 60 minutes, we have 12 / websites per 60 minutes = w / 10 minutes or [URL='https://www.mathcelebrity.com/prop.php?num1=12&num2=w&den1=60&den2=10&propsign=%3D&pl=Calculate+missing+proportion+value']12/60 = w/10[/URL] Solving the proportion in our calculator above, we get [B]w = 2[/B]

Taylor is playing a game using a die and a spinner. The spinner is divided into 4 equal parts with c
Taylor is playing a game using a die and a spinner. The spinner is divided into 4 equal parts with colors green, red, yellow, and purple. Taylor rolls the die and spins the spinner. What is the probability the die shows a 2 and the spinner lands on purple? Probability of rolling a 2 on the die is 1/6 Probability of getting a purple on the spinner is 1/4 Since each event is independent, our joint probability is: P(2 on the die and Purple on the spinner) = P(2 on the die) x P(Purple on the Spinner) P(2 on the die and Purple on the spinner) = 1/6 x 1/4 P(2 on the die and Purple on the spinner) = [B]1/24[/B]

Ted tossed a number cube and rolled a die. How many possible outcomes are there?
Ted tossed a number cube and rolled a die. How many possible outcomes are there? A number cube has 6 possible outcomes A die has 6 possible outcomes. We have 6 * 6 = [B]36 possible outcomes[/B].

Temperature Change
Free Temperature Change Calculator - This calculator determines the total temperature change

Temperature Conversions
Free Temperature Conversions Calculator - Performs the following temperature conversion measurements
* Fahrenheit (F)
* Celsius (C)
* Kelvin (K)
* Rankine (R)
* Newton (N)
* Reaumur


Ten people are competing for the title of "Best Singer in the World". There will be a 1st place and
Ten people are competing for the title of "Best Singer in the World". There will be a 1st place and a 2nd place awarded. How many different ways can the 1st and 2nd place be awarded?: We have a combinations problem of 10 choose 2. Using our [URL='https://www.mathcelebrity.com/permutation.php?num=10&den=2&pl=Permutations']permutations calculator[/URL], we see that: 10 P 2 = [B]90 ways[/B]

Ten subtracted from the product of 9 and a number is less than ?24
Ten subtracted from the product of 9 and a number is less than ?24. A number means an arbitrary variable, let's call it x x The product of 9 and a number: 9x Ten subtracted from that 9x - 10 Finally, is less than means we set our entire expression less than -24 [B]9x - 10 < -24[/B]

Ten times the sum of twice a number and six
The phrase [I]a number[/I] means an arbitrary variable. We can pick any letter a-z except for i and e. Let's choose x. Twice a number means we multiply x by 2: 2x The sum of twice a number and 6: 2x + 6 Ten times the sum of twice a number and six [B]10(2x + 6)[/B]

Terry recorded the temperature every hour from 8 AM to 1 PM. The temperature at 8 AM was 19?. The te
Terry recorded the temperature every hour from 8 AM to 1 PM. The temperature at 8 AM was 19?. The temperature dropped 4? every hour. What was the temperature at 1 PM? Group of answer choices 1 degree Set up our temperature function T(h) where h is the number of hours since 8 AM: T(h) = 19 - 4h <-- We subtract 4h since each hour, the temperature drops 4 degrees The questions asks for the temperature at 1PM. We need to figure out how many hours pass since 8 AM: 8 AM to 12 PM is 4 hours 12 PM to 1 PM is 1 hour Total time is 5 hours So we want T(5): T(5) = 19 - 4(5) T(5) = 19 - 20 T(5) = [B]-1?[/B]

The 4/7 part of a number is 84 . What is the number?
The 4/7 part of a number is 84 . What is the number? We multiply 4/7 * 84. 7 goes into 84 12 times, so we have: 4 * 12 = [B]48[/B]

the absolute value of a number is its _____ from 0
the absolute value of a number is its _____ from 0 The answer is [B]distance[/B]. As an example: 2 and -2 are 2 units away from 0.

the absolute value of the difference 6 and k
the absolute value of the difference 6 and k The difference of 6 and k means we subtract k from 6: 6 - k Take the absolute value: [B]|6 - k|[/B]

The admission fee at an amusement park is $1.50 for children and $4 for adults. On a certain day, 32
The admission fee at an amusement park is $1.50 for children and $4 for adults. On a certain day, 327 people entered the park , and the admission fee collected totaled 978.00 dollars . How many children and how many adults were admitted? Let the number of children's tickets be c. Let the number of adult tickets be a. We're given two equations: [LIST=1] [*]a + c = 327 [*]4a + 1.50c = 978 [/LIST] We can solve this system of equation 3 ways: [LIST] [*][URL='https://www.mathcelebrity.com/simultaneous-equations.php?term1=a+%2B+c+%3D+327&term2=4a+%2B+1.50c+%3D+978&pl=Substitution']Substitution Method[/URL] [*][URL='https://www.mathcelebrity.com/simultaneous-equations.php?term1=a+%2B+c+%3D+327&term2=4a+%2B+1.50c+%3D+978&pl=Elimination']Elimination Method[/URL] [*][URL='https://www.mathcelebrity.com/simultaneous-equations.php?term1=a+%2B+c+%3D+327&term2=4a+%2B+1.50c+%3D+978&pl=Cramers+Method']Cramer's Rule[/URL] [/LIST] No matter which method we choose, we get the same answers: [LIST] [*][B]a = 195[/B] [*][B]c = 132[/B] [/LIST]

The admission fee at an amusement park is $1.50 for children and $4.00 for adults. On a certain day,
The admission fee at an amusement park is $1.50 for children and $4.00 for adults. On a certain day, 281 people entered the park, and the admission fees collected totaled $784 . How many children and how many adults were admitted? Let c be the number of children and a be the number of adults. We have two equations: [LIST=1] [*]a + c = 281 [*]4a + 1.5c = 784 [/LIST] Using our [URL='http://www.mathcelebrity.com/simultaneous-equations.php?term1=a%2Bc%3D281&term2=4a+%2B+1.5c+%3D+784&pl=Cramers+Method']simultaneous equations calculator[/URL], we get: [LIST] [*][B]a = 145[/B] [*][B]c = 136[/B] [/LIST]

The age of a woman 15 years ago
The age of a woman 15 years ago Let the woman's current age be a. 15 years ago means we subtract 15 from a: [B]a - 15[/B]

The age of denver 3 years ago if he is x years old now
The age of denver 3 years ago if he is x years old now 3 years ago means we subtract: [B]x - 3[/B]

The age of three sister are consecutive intergers the sum of their age is 45 what is their ages
The age of three sister are consecutive intergers the sum of their age is 45 what is their ages Type this into the search engine: [URL='https://www.mathcelebrity.com/sum-of-consecutive-numbers.php?num=thesumofthreeconsecutivenumbersis45&pl=Calculate']The sum of three consecutive numbers is 45[/URL]. We get [B]14, 15, 16[/B].

The age of woman 15 years ago
The age of woman 15 years ago Let a be the woman's age today. 15 years ago means we subtract 15 from a: [B]a - 15[/B]

The ages of three siblings are all consecutive integers. The sum of of their ages is 39.
The ages of three siblings are all consecutive integers. The sum of of their ages is 39. Let the age of the youngest sibling be n. This means the second sibling is n + 1. This means the oldest/third sibling is n + 2. So what we want is the[URL='https://www.mathcelebrity.com/sum-of-consecutive-numbers.php?num=sumof3consecutiveintegersequalto39&pl=Calculate'] sum of 3 consecutive integers equal to 39[/URL]. We type this command into our search engine. We get: n = 12. So the youngest sibling is [B]12[/B]. The next sibling is 12 + 1 = [B]13[/B] The oldest/third sibling is 12 + 2 = [B]14[/B]

The anti-inflammation drug Advil has a half-life of 2 hours. That is, the amount of the drug present
The anti-inflammation drug Advil has a half-life of 2 hours. That is, the amount of the drug present in the body is halved every two hours. What fraction of the initial amount of the drug will be left in the body after 4 hours? [LIST] [*]At time 0, we have 100% [*]At time 2, we have 100% * 1/2 = 50% or 1/2 [*]At time 4, we have 1/2 * 1/2 = [B]1/4[/B] [/LIST]

The area of a desert in Africa is 12 times the area of a desert in Asia. If the area of a desert in
The area of a desert in Africa is 12 times the area of a desert in Asia. If the area of a desert in Asia is Y square miles, express the area of a desert in Africa as an algebraic expression in Y. [B]Africa Area = 12Y[/B]

The arithmetic mean (average) of 17, 26, 42, and 59 is equal to the arithmetic mean of 19 and N. Wha
The arithmetic mean (average) of 17, 26, 42, and 59 is equal to the arithmetic mean of 19 and N. What is the value of N ? Average of the first number set is [URL='https://www.mathcelebrity.com/statbasic.php?num1=17%2C26%2C42%2C59&num2=+0.2%2C0.4%2C0.6%2C0.8%2C0.9&pl=Number+Set+Basics']using our average calculator[/URL] is: 36 Now, the mean (average) or 19 and N is found by adding them together an dividing by 2: (19 + N)/2 Since both number sets have equal means, we set (19 + N)/2 equal to 36: (19 + N)/2 = 36 Cross multiply: 19 + N = 36 * 2 19 + n = 72 To solve for n, we [URL='https://www.mathcelebrity.com/1unk.php?num=19%2Bn%3D72&pl=Solve']type this equation into our search engine[/URL] and we get: n = [B]53[/B]

The astronomer found that 0.017 at the stars examined were red dwarfs. if 29,000 stars or examined,
The astronomer found that 0.017 at the stars examined were red dwarfs. if 29,000 stars or examined, how many were not red dwarfs? [U]If 0.017 were red dwarfs, then we have a not red dwarfs percent of:[/U] Not red dwarfs pct = 1 - red dwarfs Not red dwarfs pct = 1 - 0.017 Not red dwarfs pct = 0.983 [U]For 29,000 stars, we have[/U] Not red dwarfs = Starts * not red dwarfs pct Not red dwarfs = 29000 * 0.983 Not red dwarfs = [B]28,507[/B]

The auditorium can hold a maximum of 150 people
The auditorium can hold a maximum of 150 people We want an inequality for the number of people (p) in the auditorium. The word [I]maximum[/I] means [I]no more than[/I] or [I]less than or equal to[/I]. So we have: [B]p <= 150[/B]

The auto repair shop took 2.5 hours to repair Victoria’s car. The cost of parts was $93, and the tot
The auto repair shop took 2.5 hours to repair Victoria’s car. The cost of parts was $93, and the total bill was $248. What is the shops charge per hour. Calculate Labor Cost: Labor Cost = Total bill - Parts Labor Cost = $248 - $93 Labor Cost = $155 Calculate labor hourly rate: Labor Hourly Rate = Labor Cost / Number of Labor Hours Labor Hourly Rate = 155/2.5 Labor Hourly Rate = [B]$62[/B]

The average age of 15 men is 25 years. What is their total age in years?
The average age of 15 men is 25 years. What is their total age in years? Average Age = Total Ages/Total Men 25 = Total Ages / 15 Cross multiply and we get: Total Ages = 15 * 25 Total Ages = [B]375[/B]

The average cost of printing a book in a publishing company is c(x) = 5.5x+kx , where x is the numbe
The average cost of printing a book in a publishing company is c(x) = 5.5x+kx , where x is the number of books printed that day and k is a constant. Find k, if on the day when 200 were printed the average cost was $9 per book. We are given: c(200) = 9, so we have: 9 = 5.5(200) + k(200) 200k + 1100 = 9 Using our [URL='http://www.mathcelebrity.com/1unk.php?num=200k%2B1100%3D9&pl=Solve']equation solver[/URL], we get: [B]k = -5.455[/B]

The average height of a family of 6 is 6 feet. After the demise of the mother, the average height re
The average height of a family of 6 is 6 feet. After the demise of the mother, the average height remained the same. What is the height of the mother? [LIST] [*]Let the height of the family without the mom be f. Let the height of the mother be m. [*]Averages mean we add the heights and divide by the number of people who were measured. [/LIST] We're given two equations: [LIST=1] [*](f + m)/6 = 6 [*]f/5 = 6 [/LIST] Cross multiplying equation (2), we get: f = 5 * 6 f = 30 Plug f = 30 into equation (1), we get: (30 + m)/6 = 6 Cross multiplying, we get: m + 30 = 6 * 6 m + 30 = 36 To solve this equation for m, we [URL='https://www.mathcelebrity.com/1unk.php?num=m%2B30%3D36&pl=Solve']type it in our search engine[/URL] and we get: m = [B]6[/B] [SIZE=3][FONT=Arial][COLOR=rgb(34, 34, 34)][/COLOR][/FONT][/SIZE]

The average of 16 and x is 21. Find x.
The average of 16 and x is 21. Find x. The average of 2 numbers is the sum of the 2 numbers divided by 2. So we have: (16 + x)/2 = 21 Cross multiply: 16 + x = 21*2 16 + x = 42 To solve this equation, [URL='https://www.mathcelebrity.com/1unk.php?num=16%2Bx%3D42&pl=Solve']we type this expression into the search engine[/URL] and get [B]x = 26[/B]. Check our work by restating our answer: The average of 16 and 26 is 21. TRUE.

The average of 171 and x?
The average of 171 and x? The phrase [I]average[/I] means add up all the items in the number set, divided by the count of items in the number set. Our number set in this case is {171, x} which has 2 elements. Therefore, our average is: [B](171 + x)/2[/B]

The average of 20 numbers is 18 while the average of 18 numbers is 20. What is the average of the 38
The average of 20 numbers is 18 while the average of 18 numbers is 20. What is the average of the 38 numbers? The average of averages is found by getting the sum of both groups of numbers and dividing by the count of numbers. Calculate the sum of the first group of numbers S1: Average = S1 / n1 18 = S1 / 20 S1 = 20 * 18 S1 =360 Calculate the sum of the second group of numbers S2: Average = S2 / n2 20 = S2 / 18 S2 = 18 * 20 S2 =360 Our average of averages is found by the following: A = (S1 + S2)/(n1 + n2) A = (360 + 360)/(20 + 18) A = 720/38 [B]A = 18.947[/B]

The average of a number and double the number is 25.5
Let x equal "a number". Double the number is 2x. The average is (x + 2x)/2 Combine the terms in the numerator: 3x/2 The word [I]is[/I] means equal to, so we set 3x/2 equal to 25.5 3x/2 = 25.5 Cross multiply the 2: 3x = 51 Divide each side by 3 [B]x = 17[/B]

the average of eighty-five and a number m is ninety
the average of eighty-five and a number m is ninety Average of 2 numbers means we add both numbers and divide by 2: (85 + m)/2 = 90 Cross multiply: m + 85 = 90 * 2 m + 85 = 180 To solve this equation for m, we [URL='https://www.mathcelebrity.com/1unk.php?num=m%2B85%3D180&pl=Solve']type it in our math engine [/URL]and we get: m = [B]95[/B]

The average of manny three tests is an 84. What must he get on a 4th test to raise his average to a
The average of manny three tests is an 84. What must he get on a 4th test to raise his average to a 87? [URL='http://www.mathcelebrity.com/missingaverage.php?num=84%2C84%2C84&avg=87&pl=Calculate+Missing+Score']This is a missing average problem, use our missing average calculator[/URL] His 4th test must be [B]96[/B]

the average of two numbers x and y
the average of two numbers x and y Average is the sum divided by the count: Sum: x + y We have 2 numbers, so we divide (x + y) by 2 [B](x + y)/2[/B]

The average precipitation for the first 7 months of the year is 19.32 inches with a standard deviati
The average precipitation for the first 7 months of the year is 19.32 inches with a standard deviation of 2.4 inches. Assume that the average precipitation is normally distributed. a. What is the probability that a randomly selected year will have precipitation greater than 18 inches for the first 7 months? b. What is the average precipitation of 5 randomly selected years for the first 7 months? c. What is the probability of 5 randomly selected years will have an average precipitation greater than 18 inches for the first 7 months? [URL='https://www.mathcelebrity.com/probnormdist.php?xone=18&mean=19.32&stdev=2.4&n=1&pl=P%28X+%3E+Z%29']For a. we set up our z-score for[/URL]: P(X>18) = 0.7088 b. We assume the average precipitation of 5 [I]randomly[/I] selected years for the first 7 months is the population mean ? = 19.32 c. [URL='https://www.mathcelebrity.com/probnormdist.php?xone=18&mean=19.32&stdev=2.4&n=5&pl=P%28X+%3E+Z%29']P(X > 18 with n = 5)[/URL] = 0.8907

the average, a, is at least 85
the average, a, is at least 85 At least is an inequality. It also means greater than or equal to, so we have: [B]a >= 85[/B]

the balance of an account after $40 withdrawal
the balance of an account after $40 withdrawal Let the balance be b. A withdrawal means a [U]reduction[/U][I] in the balance[/I]. So we have [B]b - 40[/B]

The base of a triangle with a height of 7 units is represented by the formula b=2/7A. The base of th
The base of a triangle with a height of 7 units is represented by the formula b=2/7A. The base of the triangle is less than 10 units. Write and solve an inequality that represents the possible area A of the triangle We're given: b=2/7A We're also told that b is less than 10. So we have: 2/7A < 10 2A/7 < 10 Cross multiply: 2A < 7 * 10 2A < 70 Divide each side of the inequality by 2 to isolate A 2A/2 < 70/2 Cancel the 2's on the left side and we get: A < [B]35[/B]

The baseball coach bought 2 new baseballs for $1 each. The basketball coach bought 7 new basketballs
The baseball coach bought 2 new baseballs for $1 each. The basketball coach bought 7 new basketballs for $10 each. How much more did the basketball coach spend than the baseball coach? [U]Baseball coach spend:[/U] Spend = Number of baseballs * cost per baseball Spend= 2 * $1 Spend = $2 [U]Basketball coach spend:[/U] Spend = Number of basketballs * cost per basketball Spend= 7 * $10 Spend = $70 [U]Calculate the difference in spend:[/U] Difference = Basketball coach spend - Baseball coach spend Difference= $70 - $2 Difference= [B]$68[/B]

The basketball coach bought 8 new basketballs for 2$ each. The baseball coach bought 8 new baseballs
the basketball coach bought 8 new basketballs for 2$ each. the baseball coach bought 8 new baseballs for 25$ each. how much more did the basketball coach spend than the baseball coach. [LIST] [*]Basketball Coach Spend: 8 basketballs * $2 = $16 [*]Baseball Coach Spend: 8 baseballs * $25 = $200 [/LIST] The Baseball Coach spent $200 - $16 = [B]$184[/B] more than the Basketball coach

The basketball team is selling candy as a fundraiser. A regular candy bar cost 0.75 and a king sized
The basketball team is selling candy as a fundraiser. A regular candy bar cost 0.75 and a king sized candy bar costs 1.50. In the first week of the sales the team made 36.00. Exactly 12 regular sized bars were sold that week. How many king size are left? Let r be the number of regular bars and k be the number of king size bars. Set up our equations: [LIST=1] [*]0.75r + 1.5k = 36 [*]r = 12 [/LIST] [U]Substitute (2) into (1)[/U] 0.75(12) + 1.5k = 36 9 + 1.5k = 36 [U]Use our equation solver, we get:[/U] [B]k = 18[/B]

The bigger of 2 numbers in 5 larger than the smaller. Twice the smaller, increased by, twice the lar
The bigger of 2 numbers in 5 larger than the smaller. Twice the smaller, increased by, twice the larger, is equal to 50. Find each number. Let the big number be b. Let the small number be s. We're given two equations: [LIST=1] [*]b = s + 5 [*]2s + 2b = 50 [/LIST] Substitute equation (1) into equation (2) 2s + 2(s + 5) = 50 [URL='https://www.mathcelebrity.com/1unk.php?num=2s%2B2%28s%2B5%29%3D50&pl=Solve']Type this equation into our search engine[/URL], and we get: [B]s = 10[/B] Now substitute s = 10 into equation (1) to solve for b: b = 10 + 5 [B]b = 15[/B]

The bill for the repair of a car was $294. The cost of parts was $129, and labor charge was $15 per
The bill for the repair of a car was $294. The cost of parts was $129, and labor charge was $15 per hour. How many hours did it take to repair the car? Write a sentence as your answer. Let h be the number of hours. We have: 15h + 129 = 294 To solve this equation for h, we [URL='https://www.mathcelebrity.com/1unk.php?num=15h%2B129%3D294&pl=Solve']type it in the search engine [/URL]and we get: h = [B]11[/B]

The bill from your plumber was $134. The cost for labor was $32 per hour. The cost materials was $46
The bill from your plumber was $134. The cost for labor was $32 per hour. The cost materials was $46. How many hours did the plumber work? Set up the cost equation where h is the number of hours worked: 32h + 46 = 134 [URL='https://www.mathcelebrity.com/1unk.php?num=32h%2B46%3D134&pl=Solve']Typing this equation into our search engine[/URL], we get [B]h = 2.75[/B].

The blue star publishing company produces daily "Star news". It costs $1200 per day to operate regar
The blue star publishing company produces daily "Star news". It costs $1200 per day to operate regardless of whether any newspaper are published. It costs 0.20 to publish each newspaper. Each daily newspaper has $850 worth of advertising and each newspaper is sold for $.30. Find the number of newspaper required to be sold each day for the Blue Star company to 'break even'. I.e all costs are covered. Build our cost function where n is the number of newspapers sold: C(n) = 1200+ 0.2n Now build the revenue function: R(n) = 850 + 0.3n Break even is where cost and revenue are equal, so set C(n) = R(n) 1200+ 0.2n = 850 + 0.3n Using our [URL='http://www.mathcelebrity.com/1unk.php?num=1200%2B0.2n%3D850%2B0.3n&pl=Solve']equation solver[/URL], we get: [B]n = 3,500[/B]

the book is 11 inches long, 11 inches wide, and 2 inches thick. find the volume of the book
the book is 11 inches long, 11 inches wide, and 2 inches thick. find the volume of the book The book is a rectangular solid, so our Volume (V) is: V = l * w * h V = 11 * 11 * 2 V = [B]242 cubic inches[/B]

The book Shelly has 235 pages. she has read 110 pages. If she reads 25 pages from now on, how many m
The book Shelly has 235 pages. she has read 110 pages. If she reads 25 pages from now on, how many more days will it take her to complete the book? Subtract the pages read to get the unread pages: 235 - 110 = 125 unread pages Now figure out how many days, reading 25 pages per day, to read 125 pages 125/25 = [B]5 days[/B]

The brand manager for a brand of toothpaste must plan a campaign designed to increase brand recognit
The brand manager for a brand of toothpaste must plan a campaign designed to increase brand recognition. He wants to first determine the percentage of adults who have heard of the bran. How many adults must he survey in order to be 90% confident that his estimate is within seven percentage points of the true population percentage? [IMG]https://ci5.googleusercontent.com/proxy/kc6cjrLvUq64guMaArhfiSR0mOnTrBwB9iFM9u9VaZ5YYn86CSDWXr1FNyqxylwytHdbQ3iYsUDnavt-zvt-OK0=s0-d-e1-ft#http://latex.codecogs.com/gif.latex?%5Chat%20p[/IMG] = 0.5 1 - [IMG]https://ci5.googleusercontent.com/proxy/kc6cjrLvUq64guMaArhfiSR0mOnTrBwB9iFM9u9VaZ5YYn86CSDWXr1FNyqxylwytHdbQ3iYsUDnavt-zvt-OK0=s0-d-e1-ft#http://latex.codecogs.com/gif.latex?%5Chat%20p[/IMG] = 0.5 margin of error (E) = 0.07 At 90% confidence level the t is, alpha = 1 - 90% alpha = 1 - 0.90 alpha = 0.10 alpha / 2 = 0.10 / 2 = 0.05 Zalpha/2 = Z0.05 = 1.645 sample size = n = (Z[IMG]https://ci4.googleusercontent.com/proxy/mwhpkw3aM19oMNA4tbO_0OdMXEHt9juW214BnNpz4kjXubiVJgwolO7CLbmWXXoSVjDPE_T0CGeUxNungBjN=s0-d-e1-ft#http://latex.codecogs.com/gif.latex?%5Calpha[/IMG] / 2 / E )2 * [IMG]https://ci5.googleusercontent.com/proxy/kc6cjrLvUq64guMaArhfiSR0mOnTrBwB9iFM9u9VaZ5YYn86CSDWXr1FNyqxylwytHdbQ3iYsUDnavt-zvt-OK0=s0-d-e1-ft#http://latex.codecogs.com/gif.latex?%5Chat%20p[/IMG] * (1 - [IMG]https://ci5.googleusercontent.com/proxy/kc6cjrLvUq64guMaArhfiSR0mOnTrBwB9iFM9u9VaZ5YYn86CSDWXr1FNyqxylwytHdbQ3iYsUDnavt-zvt-OK0=s0-d-e1-ft#http://latex.codecogs.com/gif.latex?%5Chat%20p[/IMG] ) = (1.645 / 0.07)^2 *0.5*0.5 23.5^2 * 0.5 * 0.5 552.25 * 0.5 * 0.5 = 138.06 [B]sample size = 138[/B] [I]He must survey 138 adults in order to be 90% confident that his estimate is within seven percentage points of the true population percentage.[/I]

The button on Alice's shirt has a diameter of 8 millimeters. What is the button's radius?
The button on Alice's shirt has a diameter of 8 millimeters. What is the button's radius? Radius = Diameter / 2 Radius = 8/2 Radius = [B]4[/B]

The buyer of a lot pays P10,000 every month for 10 years. If the money is 8% compounded annually, ho
The buyer of a lot pays P10,000 every month for 10 years. If the money is 8% compounded annually, how much is the cash value of the lot? (use j= 0.006434, n=120) Using our [URL='https://www.mathcelebrity.com/compoundint.php?bal=10000&nval=120&int=8&pl=Monthly']compound interest calculator[/URL], we get: [B]22,196.40[/B]

The cafe needs 170 onions. there are 3 onions in each bag. How many bags should the cafe buy?
The cafe needs 170 onions. there are 3 onions in each bag. How many bags should the cafe buy? Bags needed = Total Onions / Onions per bag Bags needed = 170/3 Bags needed = 56.67 ~ [B]57 bags[/B]

The Canucks lost 6 of their first 24 games. At this rate how many would the lose in an 84 game sched
The Canucks lost 6 of their first 24 games. At this rate how many would the lose in an 84 game schedule? Set up a proportion of losses to games where l is the number of losses for 84 games: 6/24 = l/84 [URL='https://www.mathcelebrity.com/prop.php?num1=6&num2=l&den1=24&den2=84&propsign=%3D&pl=Calculate+missing+proportion+value']Typing this proportion into our search engine[/URL], we get: l = [B]21[/B]

The chance of a soldier being an enemy spy is .0005. Out of 10,000 soldiers, how many of them are ex
The chance of a soldier being an enemy spy is .0005. Out of 10,000 soldiers, how many of them are expected to be spies? Expected Spies = Probability of being a spy * Total Soldiers Expected Spies = 0.0005 * 10000 Expected Spies = [B]5[/B]

The charge to rent a trailer is $30 for up to 2 hours plus $9 per additional hour or portion of an
The charge to rent a trailer is $30 for up to 2 hours plus $9 per additional hour or portion of an hour. Find the cost to rent a trailer for 2.4 hours, 3 hours, and 8.5 hours. Set up the cost function C(h), where h is the number of hours to rent the trailer. We have, for any hours greater than 2: C(h) = 30 + 9(h - 2) Simplified, we have: C(h) = 9h - 18 + 30 C(h) = 9h + 12 The question asks for C(2.4), C(3), and C(8.5) [U]Find C(2.4)[/U] C(2.4) = 9(2.4) + 12 C(2.4) = 21.6 + 12 C(2.4) = [B]33.6 [/B] [U]Find C(3)[/U] C(3) = 9(3) + 12 C(3) = 27 + 12 C(2.4) = [B][B]39[/B][/B] [U]Find C(8.5)[/U] C(8.5) = 9(8.5) + 12 C(8.5) = 76.5 + 12 C(8.5) = [B]88.5[/B]

The circle has an arc measure of 180 degrees
The circle has an arc measure of 180 degrees - True or False. False. A Circle has an arc measure of 360 degrees. A few vital facts about arcs measures, also called central angles: [LIST=1] [*]An arc measure [I]< [/I]180° is a minor arc. [*]An arc measure [I]> [/I]180° is a major arc. [*]An arc measure [I]= [/I]180° is a semicircle. [*]An arc measure [I]= 36[/I]0° is a circle. [/LIST]

The club uses the function S(t) = -4,500t + 54,000 to determine the salvage S(t) of a fertilizer ble
The club uses the function S(t) = -4,500t + 54,000 to determine the salvage S(t) of a fertilizer blender t years after its purchase. How long will it take the blender to depreciate completely? Complete depreciation means the salvage value is 0. So S(t) = 0. We need to find t to make S(t) = 0 -4,500t + 54,000 = 0 Subtract 54,000 from each side -4,500t = -54,000 Divide each side by -4,500 [B]t = 12[/B]

The coach of a hockey team is holding tryouts and can take only 2 more players for the team. There a
The coach of a hockey team is holding tryouts and can take only 2 more players for the team. There are 5 players trying out. How many different groups of 2 players could possibly be chosen? We want 5C2. Using our [URL='http://www.mathcelebrity.com/permutation.php?num=5&den=2&pl=Combinations']combinations calculator[/URL], we get: [B]10[/B] possible combinations of 2 player groups

The coach writes the batting order on a piece of paper. How many different ways could the list be wr
The coach writes the batting order on a piece of paper. How many different ways could the list be written? We have 9 people in a line up. The total lineups are shown by: 9 * 8 * 7 * ... * 2 * 1 Or, 9!. [URL='https://www.mathcelebrity.com/factorial.php?num=9!&pl=Calculate+factorial']Typing 9! in our search engine[/URL] and we get [B]362,880[/B]

The coefficient of determination is found by taking the square root of the coefficient of correlatio
The coefficient of determination is found by taking the square root of the coefficient of correlation. True or False [B]FALSE[/B] - It is found by squaring the coefficient of correlation

The construction crew has less than 7 days to complete the road repairs. Let d represent the number
The construction crew has less than 7 days to complete the road repairs. Let d represent the number of days left to complete the road repairs. Less than means we don't include 7: [B]d < 7 [/B]

The cost for parking at a parking garage is 2.25 plus an additional 1.50 for each hour. What is the
The cost for parking at a parking garage is 2.25 plus an additional 1.50 for each hour. What is the total cost to park for 5 hours? Set up our equation where C is cost and h is the number of hours used to park C = 1.5h + 2.25 With h = 5, we have: C = 1.5(5) + 2.25 C = 7.5 + 2.25 C = 9.75

The cost of $9 movie tickets for you and three friends
The cost of $9 movie tickets for you and three friends You and three friends is 4 people. Cost per Person = Total Cost / Total People Cost per Person = $9/4 Cost per Person = [B]$2.25[/B]

the cost of 12 notebooks at x pesos each
the cost of 12 notebooks at x pesos each Cost = quantity * price Cost = [B]12x[/B]

The cost of 25 apples is less than $9.50. The cost of 12 apples is more than 3.60. What are the poss
The cost of 25 apples is less than $9.50. The cost of 12 apples is more than 3.60. What are the possible prices of one apple? Let a be the price of each apple. We're given 2 inequalities: [LIST=1] [*]25a < 9.50 [*]12a > 3.60 [/LIST] [URL='https://www.mathcelebrity.com/interval-notation-calculator.php?num=25a%3C9.50&pl=Show+Interval+Notation']Typing 25a < 9.50 into our search engine[/URL], we get a < 0.38 [URL='https://www.mathcelebrity.com/interval-notation-calculator.php?num=12a%3E3.60&pl=Show+Interval+Notation']Typing 12a > 360 into our search engine[/URL], we get a > 0.3 Therefore, the possible prices a of one apple are expressed as the inequality: [B]0.3 < a < 0.38[/B]

the cost of 3 notebooks at m dollars each
the cost of 3 notebooks at m dollars each Cost = Quantity x Price Cost = [B]3m[/B]

the cost of 7 CD at d$ each
the cost of 7 CD at d$ each The cost is price * quantity [B]7d[/B]

the cost of a buffet at a restaurant is different for adults and kids. the bill for 2 adults and 3 k
the cost of a buffet at a restaurant is different for adults and kids. the bill for 2 adults and 3 kids is $51. the bill for 3 adults and 1 kid is $45. what is the cost per adult and per kid? Let the cost for each adult be a Let the cost for each kid be k We're given two equations: [LIST=1] [*]2a + 3k = 51 [*]3a + k = 45 [/LIST] To solve this simultaneous set of equations, we can use three methods: [LIST] [*][URL='https://www.mathcelebrity.com/simultaneous-equations.php?term1=2a+%2B+3k+%3D+51&term2=3a+%2B+k+%3D+45&pl=Substitution']Substitution Method[/URL] [*][URL='https://www.mathcelebrity.com/simultaneous-equations.php?term1=2a+%2B+3k+%3D+51&term2=3a+%2B+k+%3D+45&pl=Elimination']Elimination Method[/URL] [*][URL='https://www.mathcelebrity.com/simultaneous-equations.php?term1=2a+%2B+3k+%3D+51&term2=3a+%2B+k+%3D+45&pl=Cramers+Method']Cramer's Rule[/URL] [/LIST] No matter which method we use, we get the same answer: [LIST] [*]a = [B]12[/B] [*]k = [B]9[/B] [/LIST]

The cost of a field trip is $220 plus $7 per student. If the school can spend at most $500, how many
The cost of a field trip is $220 plus $7 per student. If the school can spend at most $500, how many students can go on the field trip? Set up the inequality where s is the number of students: C(s) = 220 + 7s We want C(s) <= 500, since at most means no more than 220 + 7s <= 500 Using our [URL='http://www.mathcelebrity.com/1unk.php?num=220%2B7s%3C%3D500&pl=Solve']inequality calculator[/URL], we get: [B]s <= 40[/B]

The cost of a gallon of milk (m) is .50 more than 5 times the cost of a gallon of water (w). If a ga
The cost of a gallon of milk (m) is .50 more than 5 times the cost of a gallon of water (w). If a gallon of milk cost 3.75, what is the cost of a gallon of water? We're given: m = 5w + 0.50 m = $3.75 Set them equal to each other: 5w + 0.50 = 3.75 [URL='https://www.mathcelebrity.com/1unk.php?num=5w%2B0.50%3D3.75&pl=Solve']Typing this equation into our search engine[/URL], we get: [B]w = 0.65[/B]

The cost of a taxi ride is $1.2 for the first mile and $0.85 for each additional mile or part thereo
The cost of a taxi ride is $1.2 for the first mile and $0.85 for each additional mile or part thereof. Find the maximum distance we can ride if we have $20.75. We set up the cost function C(m) where m is the number of miles: C(m) = Cost per mile after first mile * m + Cost of first mile C(m) = 0.8(m - 1) + 1.2 C(m) = 0.8m - 0.8 + 1.2 C(m) = 0.8m - 0.4 We want to know m when C(m) = 20.75 0.8m - 0.4 = 20.75 [URL='https://www.mathcelebrity.com/1unk.php?num=0.8m-0.4%3D20.75&pl=Solve']Typing this equation into our math engine[/URL], we get: m = 26.4375 The maximum distance we can ride in full miles is [B]26 miles[/B]

the cost of b books at p dollars each
the cost of b books at p dollars each Cost = Price * Quantity, so we have: Cost = [B]pb[/B]

the cost of d drinks at $2 each and 5 pies at $n each
the cost of d drinks at $2 each and 5 pies at $n each Total cost = Price * Quantity: Total cost = [B]2d + 5n[/B]

The cost of having a plumber spend h hours at
The cost of having a plumber spend h hours at your house if the plumber charges $60 for coming to the house and $70 per hour labor: We have a fixed cost of $60 plus the variable cost of $70h. We add both for our total cost C(h): [B]C(h) = $70h + 60[/B]

The cost of hiring a car for a day is $60 plus 0.25 cents per kilometer. Michelle travels 750 kilome
The cost of hiring a car for a day is $60 plus 0.25 cents per kilometer. Michelle travels 750 kilometers. What is her total cost Set up the cost function C(k) where k is the number of kilometers traveled: C(k) = 60 + 0.25k The problem asks for C(750) C(750) = 60 + 0.25(750) C(750) = 60 + 187.5 C(750) = [B]247.5[/B]

The cost of purchasing a hockey stick and puck if the stick costs 6 less than twice the cost of the
The cost of purchasing a hockey stick and puck if the stick costs 6 less than twice the cost of the puck. Let the hockey stick cost h, and puck cost p. Twice the cost of the puck means we multiply p by 2: 2p 6 less than this means we subtract 6: h = 2p - 6 [B][/B] The total cost of the hockey stick and puck is: p + 2p - 6 [B]3p - 6[/B]

The cost of renting a rototiller is $19.50 for the first hour and $7.95 for each additional hour. Ho
The cost of renting a rototiller is $19.50 for the first hour and $7.95 for each additional hour. How long can a person have the rototiller if the cost must be less than $95? Setup the inequality: $19.50 + $7.95x < $95 Subtract 19.50 from both sides: 7.95x < 75.50 Divide each side of the inequality by 7.95 to isolate x x < 9.5 The next lowest integer is 9. So we take 9 + the first hour of renting to get [B]10 total hours[/B]. Check our work: $7.95 * 9.5 + $19.50 $71.55 + $19.50 = $91.05

The cost of tuition at Johnson Community College is $160 per credit hour. Each student also has to p
The cost of tuition at Johnson Community College is $160 per credit hour. Each student also has to pay $50 in fees. Model the cost, C, for x credit hours taken. Set up cost equation, where h is the number of credit hours: [B]C = 50 + 160h[/B]

the cost of x concert tickets if one concert ticket costs $97
the cost of x concert tickets if one concert ticket costs $97 The cost function C(x), where x is the number of concert tickets is: [B]C(x) = 97x[/B]

The cost of x ice cream if one ice cream cost $9 and the fixed cost is $8142
The cost of x ice cream if one ice cream cost $9 and the fixed cost is $8142 Cost function is C(x) is: C(x) = Cost per ice cream * number of ice creams + Fixed Cost C(x) = [B]9x + 8142[/B]

The cost of x movies if each movie cost $20
The cost of x movies if each movie cost $20 Cost = Price * Quantity, so we have: Cost = [B]20x[/B]

The cost of X pounds of cheese or $6.75 a pound
The cost of X pounds of cheese or $6.75 a pound Total Cost = Unit Cost * units Total Cost = [B]$6.75 * x[/B]

the cost of x pounds of pork at $4.10 a pound
the cost of x pounds of pork at $4.10 a pound Set up the cost function for pounds of pork: [B]C(x) = 4.10x[/B]

The cost of x textbooks if one textbook costs $140
The cost of x textbooks if one textbook costs $140. Set up a cost function where x is the number of textbooks: [B]C(x) = 140x[/B]

The cost to rent a boat is $10. There is also charge of $2 for each person. Which expresion represen
The cost to rent a boat is $10. There is also charge of $2 for each person. Which expresion represents the total cost to rent a boat for p persons? The cost function includes a fixed cost of $10 plus a variable cost of 2 persons for p persons: [B]C(p) = 2p + 10[/B]

The cost to rent a construction crane is 450 per day plus 150 per hour. What is the maximum number o
The cost to rent a construction crane is 450 per day plus 150 per hour. What is the maximum number of hours the crane can be used each day if the rental cost is not to exceed 1650 per day? Set up the cost function where h is the number of hours: C(h) = 150h + 450 We want C(h) <= 1650: 150h + 450 <= 1650 Using our [URL='http://www.mathcelebrity.com/1unk.php?num=150h%2B450%3C%3D1650&pl=Solve']equation/inequality solver[/URL], we get: [B]h <= 8[/B]

the cube of c decreased by a^2
the cube of c decreased by a^2 The cube of means we raise the variable c to the power of 3: c^3 The phrase [I]decreased by[/I] means we subtract: [B]c^3 - a^2[/B]

The cube of g plus the square of m
The cube of g plus the square of m The cube of g means we raise g to the 3rd power: g^3 The square of m means we raise m to the 2nd power: m^2 The word [I]plus[/I] means we add them both [B]g^3 + m^2[/B]

the cube of t is less than 12
the cube of t is less than 12 The cube of t t^3 is less than 12, we use the operator < t^3 < 12

The cube of the difference of 5 times the square of y and 7 divided by the square of 2 times y
The cube of the difference of 5 times the square of y and 7 divided by the square of 2 times y Take this in algebraic expression in parts: [U]Term 1[/U] [LIST] [*]The square of y means we raise y to the 2nd power: y^2 [*]5 times the square of y: 5y^2 [/LIST] [U]Term 2[/U] [LIST] [*]2 times y: 2y [*]The square of 2 times y: (2y)^2 = 4y^2 [*]7 divide by the square of 2 times y: 7/4y^2 [/LIST] [U]The difference of these terms is written as Term 1 minus Term 2:[/U] [LIST] [*]5y^2/4y^2 [/LIST] [U]The cube of the difference means we raise the difference to the power of 3:[/U] [B](5y^2/4y^2)^3[/B]

the cube of the difference of 5 times x and 4
the cube of the difference of 5 times x and 4 Take this algebraic expression in pieces: 5 times x: 5x The difference of 5x and 4 means we subtract 4 from 5x: 5x - 4 We want to cube this difference, which means we raise the difference to the power of 3. [B](5x - 4)^3[/B]

the cube of the product of 3 and x
the cube of the product of 3 and x The product of 3 and x: 3x Cube this product means raise it to the power of 3: (3x)^3 = [B]27x^3[/B]

the cube of the sum of 2a and 3b
the cube of the sum of 2a and 3b Sum of 2a and 3b: (2a + 3b) The cube of the sum mean we raise the sum to the power of 3: [B](2a + 3b)^3[/B]

The cube of x is less than 15
The cube of x is less than 15 The cube of x means we raise x to the 3rd power: x^3 Less than 15 means we setup the following inequality [B]x^3 < 15[/B]

The dance committee of pine bluff middle school earns $72 from a bake sale and will earn $4 for each
The dance committee of pine bluff middle school earns $72 from a bake sale and will earn $4 for each ticket sold they sell to the Spring Fling dance. The dance will cost $400 Let t be the number of tickets sold. We have a Revenue function R(t): R(t) = 4t + 72 We want to know t such that R(t) = 400. So we set R(t) = 400: 4t + 72 = 400 [URL='https://www.mathcelebrity.com/1unk.php?num=4t%2B72%3D400&pl=Solve']Typing this equation into our search engine[/URL], we get: [B]t = 82[/B]

The denominator of a fraction is 4 more than the numerator. If 4 is added to the numerator and 7 is
The denominator of a fraction is 4 more than the numerator. If 4 is added to the numerator and 7 is added to the denominator, the value of the fraction is 1/2. Find the original fraction. Let the original fraction be n/d. We're given: [LIST=1] [*]d = n + 4 [*](n + 4) / (d + 7) = 1/2 [/LIST] Cross multiply Equation 2: 2(n + 4) = d + 7 2n + 8 = d + 7 Now substitute equation (1) into tihs: 2n + 8 = (n + 4) + 7 2n + 8 = n + 11 [URL='https://www.mathcelebrity.com/1unk.php?num=2n%2B8%3Dn%2B11&pl=Solve']Type this equation into our search engine[/URL], and we get: n = 3 This means from equation (1), that: d = 3 + 4 d = 7 So our original fraction n/d = [B]3/7[/B]

The Devil is in the Details
[URL]https://soundcloud.com/mathcelebrity/organic-seo-part-7-urls-the-devil-is-in-the-details[/URL]

The diagonal of a rectangle is 10 inches long and the height of the rectangle is 8 inches. What is t
Draw this rectangle and you'll see we have a pythagorean theorem equation. a^2 + b^2= c^2 b = 8 and c= 10 a^2 + 8^2 = 10^2 a^2 + 64 = 100 Subtract 64 from each side: a^2 = 36 a= 6 Therefore, perimeter P is: P = 2l + 2w P = 2(6) + 2(8) P = 12 + 16 P = [B]28[/B] [MEDIA=youtube]8lcpRet3r18[/MEDIA]

The difference between 2 numbers is 108. 6 times the smaller is equal to 2 more than the larger. Wh?
The difference between 2 numbers is 108. 6 times the smaller is equal to 2 more than the larger. What are the numbers? Let the smaller number be x. Let the larger number be y. We're given: [LIST=1] [*]y - x = 108 [*]6x = y + 2 [/LIST] Rearrange (1) by adding x to each side: [LIST=1] [*]y = x + 108 [/LIST] Substitute this into (2): 6x = x + 108 + 2 Combine like terms 6x = x +110 Subtract x from each side: 5x = 110 [URL='https://www.mathcelebrity.com/1unk.php?num=5x%3D110&pl=Solve']Plugging this equation into our search engine[/URL], we get: x = [B]22[/B]

The difference between 3 times x and 4
[U]3 times x:[/U] 3x [U]The difference between 3x and 4 means we subtract:[/U] 3x - 4

the difference between 7 times a number and 9 less than a number
the difference between 7 times a number and 9 less than a number The phrase [I]a number[/I] means an arbitrary variable, let's call it x. 7 times a number means we multiply x by 7 7x 9 less than a number means we subtract 9 from x x - 9 The difference between the two expressions means we subtract (x - 9) from 7x 7x - (x - 9) Simplifying this, we have: 7x - x + 9 Grouping like terms, we get: [B]6x + 9[/B]

The difference between a and b is 10
The difference between a and b is 10. The problem asks for an algebraic expression. Let's take each piece one by one: [I]Difference between[/I] means we subtract: a - b The phrase [I]is [/I]means an equation, so we set a - b equal to 10 [B]a - b = 10[/B]

The difference between A and B is no less than 30
The difference between A and B is no less than 30 The difference between means we subtract. No less than means greater than or equal to, so we have the following inequality; [B]A - B >= 30[/B]

the difference between A and B is no less than 30.
the difference between A and B is no less than 30. The difference between a and b: a - b The phrase [I]no less than[/I] means an inequality. You can also say this as [I]greater than or equal to[/I]. [B]a - b >= 30[/B]

The difference between a number and 9 is 27. Find that number
The difference between a number and 9 is 27. Find that number The phrase [I]a number[/I] means an arbitrary variable, let's call it x: x The difference between a number and 9 x - 9 The word [I]is[/I] means equal to, so we set x - 9 equal to 27: x - 9 = 27 To solve this equation for x, we [URL='https://www.mathcelebrity.com/1unk.php?num=x-9%3D27&pl=Solve']type it in our math engine[/URL] and we get: x = [B]36[/B]

The difference between sixty-four and y
The difference between sixty-four and y The difference between means we subtract y from 64: [B]64 - y[/B]

The difference between the opposite of a number and 6.
The difference between the opposite of a number and 6. The phrase [I]a number means[/I] an arbitrary variable, let's call it x. x The opposite of a number means we multiply by x by -1 -x The phrase [I]the difference between[/I] means we subtract 6 from -x: [B]-x - 6[/B]

The difference between the product of 4 and a number and the square of a number
The difference between the product of 4 and a number and the square of a number The phrase [I]a number[/I] means an arbitrary variable, let's call it x. The product of 4 and a number: 4x The square of a number means we raise x to the power of 2: x^2 The difference between the product of 4 and a number and the square of a number: [B]4x - x^2[/B]

The difference between the quotient of x and y, and twice z
The difference between the quotient of x and y, and twice z The quotient of x and y means we divide x by y: x/y Twice z means we multiply z by 2: 2z The difference between the quotient of x and y, and twice z means we subtract 2z from x/y [B]x/y - 2z[/B]

The difference between the square of b and the total of b and 9
The difference between the square of b and the total of b and 9 The square of b means we raise b to the power of 2: b^2 The total of b and 9 means we add 9 to b: b + 9 The difference means we subtract: [B]b^2 - (b + 9)[/B]

The difference between the square of b and the total of d and g
The difference between the square of b and the total of d and g Square of b means we raise b to the 2nd power: b^2 Total of d and g: d + g The difference between the square of b and the total of d and g [B]b^2 - (d + g)[/B]

The difference between the squares of two consecutive numbers is 141. Find the numbers
The difference between the squares of two consecutive numbers is 141. Find the numbers Take two consecutive numbers: n- 1 and n Given a difference (d) between the squares of two consecutive numbers, the shortcut for this is: 2n - 1 = d Proof of this: n^2- (n - 1)^2 = d n^2 - (n^2 - 2n + 1) = d n^2 - n^2 + 2n - 1 = d 2n - 1 = d Given d = 141, we have 2n - 1 = 141 Add 1 to each side: 2n = 142 Divide each side by 2: 2n/2 = 142/2 n = [B]71[/B] Therefore, n - 1 = [B]70 Our two consecutive numbers are (70, 71)[/B] Check your work 70^2 = 4900 71^2 = 5041 Difference = 5041 - 4900 Difference = 141 [MEDIA=youtube]vZJtZyYWIFQ[/MEDIA]

the difference between triple a number and double a number
the difference between triple a number and double a number The phrase [I]a number[/I] means an arbitrary variable, let's call it x. Triple a number means we multiply x by 3: 3x Double a number means we multiply x by 2: 2x The difference means we subtract 2x from 3x: 3x - 2x Simplifying like terms, we have: (3 - 2)x = [B]x[/B]

The difference between two numbers is 25. The smaller number is 1/6th of the larger number. What is
The difference between two numbers is 25. The smaller number is 1/6th of the larger number. What is the value of the smaller number Let the smaller number be s. Let the larger number be l. We're given two equations: [LIST=1] [*]l - s = 25 [*]s = l/6 [/LIST] Plug in equation (2) into equation (1): l - l/6 = 25 Multiply each side of the equation by 6 to remove the fraction: 6l - l = 150 To solve for l, we [URL='https://www.mathcelebrity.com/1unk.php?num=6l-l%3D150&pl=Solve']type this equation into our search engine[/URL] and we get: l = 30 To solve for s, we plug in l = 30 into equation (2) above: s = 30/6 [B]s = 5[/B]

The difference between two numbers is 96. One number is 9 times the other. What are the numbers?
The difference between two numbers is 96. One number is 9 times the other. What are the numbers? Let x be the first number Let y be the second number We're given two equations: [LIST=1] [*]x - y = 96 [*]x = 9y [/LIST] Substitute equation (2) into equation (1) for x 9y - y = 96 [URL='https://www.mathcelebrity.com/1unk.php?num=9y-y%3D96&pl=Solve']Plugging this equation into our math engine[/URL], we get: y = [B]12 [/B] If y = 12, then we plug this into equation 2: x = 9(12) x = [B]108[/B]

The difference between two positive numbers is 5 and the square of their sum is 169
The difference between two positive numbers is 5 and the square of their sum is 169. Let the two positive numbers be a and b. We have the following equations: [LIST=1] [*]a - b = 5 [*](a + b)^2 = 169 [*]Rearrange (1) by adding b to each side. We have a = b + 5 [/LIST] Now substitute (3) into (2): (b + 5 + b)^2 = 169 (2b + 5)^2 = 169 [URL='https://www.mathcelebrity.com/community/forums/calculator-requests.7/create-thread']Run (2b + 5)^2 through our search engine[/URL], and you get: 4b^2 + 20b + 25 Set this equal to 169 above: 4b^2 + 20b + 25 = 169 [URL='https://www.mathcelebrity.com/quadratic.php?num=4b%5E2%2B20b%2B25%3D169&pl=Solve+Quadratic+Equation&hintnum=+0']Run that quadratic equation in our search engine[/URL], and you get: b = (-9, 4) But the problem asks for [I]positive[/I] numbers. So [B]b = 4[/B] is one of our solutions. Substitute b = 4 into equation (1) above, and we get: a - [I]b[/I] = 5 [URL='https://www.mathcelebrity.com/1unk.php?num=a-4%3D5&pl=Solve']a - 4 = 5[/URL] [B]a = 9 [/B] Therefore, we have [B](a, b) = (9, 4)[/B]

The difference in Julies height and 9 is 48 letting j be Julie's height
The difference in Julies height and 9 is 48 letting j be Julie's height Step 1: If Julie's height is represented with the variable j, then we subtract 9 from j since the phrase [I]difference[/I] means we subtract: j - 9 Step 2: The word [I]is[/I] means an equation, so we set j - 9 equal to 48 for our final algebraic expression: [B]j - 9 = 48[/B]

The difference of 100 and x is 57
The difference of 100 and x means we subtract x from 100: 100 - x Is means equal to, so we set our expression above equal to 57 [B]100 - x = 57 [/B] If you want to solve for x, use our [URL='http://www.mathcelebrity.com/1unk.php?num=100-x%3D57&pl=Solve']equation calculator[/URL]

The difference of 2 positive numbers is 54. The quotient obtained on dividing the 1 by the other is
The difference of 2 positive numbers is 54. The quotient obtained on dividing the 1 by the other is 4. Find the numbers. Let the numbers be x and y. We have: [LIST] [*]x - y = 54 [*]x/y = 4 [*]Cross multiply x/y = 4 to get x = 4y [*]Now substitute x = 4y into the first equation [*](4y) - y = 54 [*]3y = 54 [*]Divide each side by 3 [*][B]y = 18[/B] [*]If x = 4y, then x = 4(18) [*][B]x = 72[/B] [/LIST]

The difference of 25 and a number added to triple another number
The difference of 25 and a number added to triple another number The phrase [I]a number [/I]means an arbitrary variable, let's call it x: x The difference of 25 and a number means we subtract x from 25: 25 - x The phrase [I]another number[/I] means a different arbitrary variable, let's call it y: y Triple another number means we multiply y by 3: 3y The phrase [I]added to[/I] means we add 25 - x to 3y [B]25 - x + 3y[/B]

the difference of 4 and the quotient of 18 and a number
the difference of 4 and the quotient of 18 and a number The phrase [I]a number[/I] means an arbitrary variable, let's call it x. The quotient of 18 and a number means we divide 18 by the variable x. 18/x The difference of 4 and the quotient above means we subtract 18/x from 4: [B]4 - 18/x[/B]

the difference of 5 and the cube of the sum of x and y
the difference of 5 and the cube of the sum of x and y The sum of x and y: x + y The cube of the sum of x and y means we raise x + y to the 3rd power: (x + y)^3 The difference of 5 and the cube of the sum of x and y [B]5 - (x + y)^3[/B]

The difference of 6 and the sum a and b
The difference of 6 and the sum a and b The sum of a and b means we add b to a: a + b The difference of 6 and the sum of a and b: [B]6 - (a + b)[/B]

The difference of 9 and the sum of x and 4
The difference of 9 and the sum of x and 4 The sum of x and 4: x + 4 The difference of 9 and the sum of x and 4: [B]9 - (x + 4)[/B]

The difference of a number and 6 is the same as 5 times the sum of the number and 2. What is the num
The difference of a number and 6 is the same as 5 times the sum of the number and 2. What is the number? We have two expressions: [U]Expression 1: [I]The difference of a number and 6[/I][/U] The phrase [I]a number[/I] means an arbitrary variable, let's call it x. The difference of a number and 6 means we subtract 6 from x: x - 6 [U]Expression 2: [I]5 times the sum of the number and 2[/I][/U] The phrase [I]a number[/I] means an arbitrary variable, let's call it x. The sum of a number and 2 means we add 2 to x: x + 2 5 times the sum means we multiply x + 2 by 5 5(x + 2) [U]For the last step, we evaluate the expression [I]is the same as[/I][/U] This means equal to, so we set x - 6 equal to 5(x + 2) [B]x - 6 = 5(x + 2)[/B]

The difference of a number times 3 and 6 is equal to 7 . Use the variable w for the unknown n
The difference of a number times 3 and 6 is equal to 7 . Use the variable w for the unknown number. The phrase a number uses the variable w. 3 times w is written as 3w The difference of 3w and 6 is written as 3w - 6 Set this equal to 7 [B]3w - 6 = 7 [/B] This is our algebraic expression. To solve this equation for w, we [URL='http://www.mathcelebrity.com/1unk.php?num=3w-6%3D7&pl=Solve']type the algebraic expression into our search engine[/URL].

The difference of five and five y is the same as eight and two y
The difference of five and five y 5 - 5y eight and two y 8 + 2y The phrase [I]is the same as[/I] means equal to. Set 5 - 5y equal to 8 + 2y for our final algebraic expression [B]5 - 5y = 8 + 2y[/B] [B][/B] If the problem asks you to solve for y: Add 5y to each side: 5 = 8 + 7y Subtract 8 from each side: 7y = -3 Divide each side by 7: [B]y= -3/7[/B]

The difference of twice a number and 4 is at least -27
The difference of twice a number and 4 is at least -27. The phrase [I]a number[/I] means an arbitrary variable, let's call it x. x Twice a number means multiply the number by 2 2x [I]and 4[/I] means we add 4 to our expression: 2x + 4 [I]Is at least[/I] means an inequality. In this case, it's greater than or equal to: [B]2x + 4 >= -27 [/B] To solve this inequality, [URL='https://www.mathcelebrity.com/1unk.php?num=2x%2B4%3E%3D-27&pl=Solve']type it in the search engine[/URL].

The difference of twice a number and 6 is at most 28
The difference of twice a number and 6 is at most 28 This is an algebraic expression. Let's take it in parts: [LIST=1] [*]The phrase [I]a number[/I], means an arbitrary variable, let's call it x [*]Twice this number means we multiply x by 2: 2x [*][I]The difference of[/I] means subtract, so we subtract 6 to 2x: 2x - 6 [*][I]Is at most [/I]means less than or equal to, so we create an inequality where 2x - 6 is less than or equal to 28, using the <= sign [/LIST] [B]2x - 6 <= 28 [/B] If you wish to solve this inequality, [URL='https://www.mathcelebrity.com/1unk.php?num=2x-6%3C%3D28&pl=Solve']click this link[/URL].

the difference of twice a number and 8 is at most -30
the difference of twice a number and 8 is at most -30. The phrase [I]a number[/I] means an arbitrary variable, let's call it x. Twice this number means we multiply by 2, so we have 2x. We take the difference of 2x and 8, meaning we subtract 8: 2x - 8 Finally, the phrase [I]at most[/I] means an inequality, also known as less than or equal to: [B]2x - 8 <= 30 <-- This is our algebraic expression [/B] To solve this, we [URL='https://www.mathcelebrity.com/1unk.php?num=2x-8%3C%3D30&pl=Solve']type it into the search engine[/URL] and get x <= 19.

The difference of twice a number and 9 is less than 22
The difference of twice a number and 9 is less than 22 The phrase a number, means an arbitrary variable, let's call it x. x Twice a number 2x The difference of twice a number and 9 2x - 9 Is less than 22 [B]2x - 9 < 22[/B]

The difference of two numbers is 12 and their mean is 15. Find the two numbers
The difference of two numbers is 12 and their mean is 15. Find the two numbers. Let the two numbers be x and y. We're given: [LIST=1] [*]x - y = 12 [*](x + y)/2 = 15. <-- Mean is an average [/LIST] Rearrange equation 1 by adding y to each side: x - y + y = y + 12 Cancelling the y's on the left side, we get: x = y + 12 Now substitute this into equation 2: (y + 12 + y)/2 = 15 Cross multiply: y + 12 + y = 30 Group like terms for y: 2y + 12 = 30 [URL='https://www.mathcelebrity.com/1unk.php?num=2y%2B12%3D30&pl=Solve']Typing this equation into our search engine[/URL], we get: [B]y = 9[/B] Now substitute this into modified equation 1: x = y + 12 x = 9 + 12 [B]x = 21[/B]

The difference of two numbers is 720. The smaller of the numbers is 119. What is the other number?
The difference of two numbers is 720. The smaller of the numbers is 119. What is the other number? Let the larger number be l. We're given: l - 119 = 720 [URL='https://www.mathcelebrity.com/1unk.php?num=l-119%3D720&pl=Solve']We type this equation into the search engine[/URL] and we get: l = [B]839[/B]

the difference of x and 5 is 2 times of x
the difference of x and 5 is 2 times of x The difference of x and 5 means we subtract 5 from x x - 5 The word [I]is[/I] means an equation, so we set x - 5 equal to 2 times x [B]x - 5 = 2x[/B]

The difference of x and x squared
The difference of x and x squared We subtract x^2 from x: [B]x - x^2[/B]

the difference of x and y added to twice the sum of a and b
the difference of x and y added to twice the sum of a and b Take this algebraic expression in parts: [LIST] [*]The difference of x and y: x - y [*]The sum of a and b: a + b [*]Twice the sum of a and b means we multiply a + b by 2: 2(a + b) [*]The phrase [I]added to[/I] means we add: [/LIST] [B]x - y + 2(a + b)[/B]

The difference when (10x - 6y) is subtracted from (7x - 4y) In simplest form
The difference when (10x - 6y) is subtracted from (7x - 4y) In simplest form (7x - 4y) - (10x - 6y) 7x - 4y - 10x - -6y 7x - 4y - 10x + 6y (7 - 10)x + (-4 + 6)y [B]-3x + 2y[/B]

The dimensions of a rectangle are 30 cm and 18 cm. When its length decreased by x cm and its width i
The dimensions of a rectangle are 30 cm and 18 cm. When its length decreased by x cm and its width is increased by x cm, its area is increased by 35 sq. cm. a. Express the new length and the new width in terms of x. b. Express the new area of the rectangle in terms of x. c. Find the value of x. Calculate the current area. Using our [URL='https://www.mathcelebrity.com/rectangle.php?l=30&w=18&a=&p=&pl=Calculate+Rectangle']rectangle calculator with length = 30 and width = 18[/URL], we get: A = 540 a) Decrease length by x and increase width by x, and we get: [LIST] [*]length = [B]30 - x[/B] [*]width = [B]18 + x[/B] [/LIST] b) Our new area using the lw = A formula is: (30 - x)(18 + x) = 540 + 35 Multiplying through and simplifying, we get: 540 - 18x + 30x - x^2 = 575 [B]-x^2 + 12x + 540 = 575[/B] c) We have a quadratic equation. To solve this, [URL='https://www.mathcelebrity.com/quadratic.php?num=-x%5E2%2B12x%2B540%3D575&pl=Solve+Quadratic+Equation&hintnum=+0']we type it in our search engine, choose solve[/URL], and we get: [B]x = 5 or x = 7[/B] Trying x = 5, we get: A = (30 - 5)(18 + 5) A = 25 * 23 A = 575 Now let's try x = 7: A = (30 - 7)(18 + 7) A = 23 * 25 A = 575 They both check out. So we can have

The distance between consecutive bases is 90 feet. An outfielder catches the ball on the third base
The distance between consecutive bases is 90 feet. An outfielder catches the ball on the third base line about 40 feet behind third base. How far would the outfielder have to throw the ball to first base? We have a right triangle. From home base to third base is 90 feet. We add another 40 feet to the outfielder behind third base to get: 90 + 40 = 130 The distance from home to first is 90 feet. Our hypotenuse is the distance from the outfielder to first base. [URL='https://www.mathcelebrity.com/pythag.php?side1input=130&side2input=90&hypinput=&pl=Solve+Missing+Side']Using our Pythagorean theorem calculator[/URL], we get: d = [B]158.11 feet[/B]

The distance between X and 8 is less than 14
Distance implies the positive difference between 2 points. Therefore, we use absolute value: |x - 8| < 14 Note, we use less than since 14 is not included.

The distance to the nearest exit door is less than 100 feet. Use d to represent the distance (in fee
The distance to the nearest exit door is less than 100 feet. Use d to represent the distance (in feet) to the nearest exit door. Less than means we use the < sign: [B]d < 100[/B]

The distance traveled in t hours by a car traveling at 65 miles per hour
The distance traveled in t hours by a car traveling at 65 miles per hour. Distance = Rate * Time Distance = 65 mph * t hours Distance = [B]65t[/B]

The distribution of actual weights of 8 oz chocolate bars produced by a certain machine is normal wi
The distribution of actual weights of 8 oz chocolate bars produced by a certain machine is normal with µ=8.1 ounces and ?=0.1 ounces. A sample of 5 of these chocolate bars is selected. What is the probability that their average weight is less than 8 ounces? Calculate Z score and probability using [URL='http://www.mathcelebrity.com/probnormdist.php?xone=8&mean=8.1&stdev=0.1&n=5&pl=P%28X+%3C+Z%29']our calculator[/URL]: Z = -2.236 P(X < -2.236) = [B]0.012545[/B]

The doctor told Anna that tomorrow she will have to check her blood pressure every 15 minutes from 1
The doctor told Anna that tomorrow she will have to check her blood pressure every 15 minutes from 10:00 AM to 4:00 PM. How many times does she have to take her blood pressure? 10:00 A.M. to 4:00 P.M. is 6 hours. Each hour is 60 minutes 60 minutes divided by 15 minutes equals 4 blood pressure checks per hour. 4 blood pressure checks per hour * 6 hours = [B]24 blood pressure checks[/B]

The domain of a relation is all even negative integers greater than -9. The range y of the relation
The domain of a relation is all even negative integers greater than -9. The range y of the relation is the set formed by adding 4 to the numbers in the domain. Write the relation as a table of values and as an equation. The domain is even negative integers greater than -9: {-8, -6, -4, -2} Add 4 to each x for the range: {-8 + 4 = -4, -6 + 4 = -2. -4 + 4 = 0, -2 + 4 = 2} For ordered pairs, we have: (-8, -4) (-6, -2) (-4, 0) (-2, 2) The equation can be written: y = x + 4 on the domain (x | x is an even number where -8 <= x <= -2)

The doubling time of a population of flies is 8 hours by what factor does a population increase in 2
The doubling time of a population of flies is 8 hours. a) By what factor does a population increase in 24 hours? b) By what factor does the population increase in 2 weeks? a) If the population doubles every 8 hours, then it doubles 3 times in 24 hours, since 24/8 = 3. So 2 * 3 = 6. The increase factor is [B]6[/B] b) Since a) is one full day, we now need to know how much it doubles in 14 days, which is 2 weeks. We take our factor of 6 for each day, and multiply it by 14: 14 * 6 = [B]84[/B]

The earth makes a full rotation in 24 hours. Jupiter takes approximately 58 days, 15 h, and 30 min t
The earth makes a full rotation in 24 hours. Jupiter takes approximately 58 days, 15 h, and 30 min to complete a full rotation. How many hours does it take Jupiter to complete a full rotation? Show your work using the correct conversion factors. Convert 58 days, 15 h, and 30 min to hours. [LIST=1] [*]Type [URL='https://www.mathcelebrity.com/timecon.php?quant=58&pl=Calculate&type=day']58 days[/URL] into the search engine to get 1,392 hours. [*]Add 15 hours to get 1,392 + 15 = 2,007 hours [*]Now convert the 30 min to hours. [URL='https://www.mathcelebrity.com/timecon.php?quant=30&pl=Calculate&type=minute']Type 30 minutes into the search engine[/URL] to get 0.5 hours [*]Add up (1), (2), and (3) to get 1,392 + 15 + 0.5 = [B]2007.5[/B] hours for a full rotation. [/LIST]

the elevation of this lake is -513 if you are standing 442 above the lake what is your elevation
the elevation of this lake is -513 if you are standing 442 above the lake what is your elevation Standing above means we add: -513 + 442 = -[B]71[/B]

The enrollment at a gymnastics academy increased 120% from 2016 to 2017. The enrollment in 2017 was
The enrollment at a gymnastics academy increased 120% from 2016 to 2017. The enrollment in 2017 was 210. What is 2016's enrollment? We take 2017's enrollment of 210 and divide by 1.2 since 120% is 1.2 as a multiplier: 2016 enrollment = 2017 enrollment / 1.2 2016 enrollment =210/1.2 2016 enrollment = [B]175[/B]

The enrollment at High School R has been increasing by 20 students per year. High School R currently
The enrollment at High School R has been increasing by 20 students per year. High School R currently has 200 students. High School T has 400 students and is decreasing 30 students per year. When will the two school have the same enrollment of students? Set up the Enrollment function E(y) where y is the number of years. [U]High School R:[/U] [I]Increasing[/I] means we add E(y) = 200 + 20y [U]High School T:[/U] [I]Decreasing[/I] means we subtract E(y) = 400 - 30y When the two schools have the same enrollment, we set the E(y) functions equal to each other 200 + 20y = 400 - 30y To solve this equation for y, we [URL='https://www.mathcelebrity.com/1unk.php?num=200%2B20y%3D400-30y&pl=Solve']type it in our search engine[/URL] and we get: y = [B]4[/B]

The entrance fee to the national park is $30. A campsite fee is $15 per night. Write an equation to
The entrance fee to the national park is $30. A campsite fee is $15 per night. Write an equation to represent the situation. Let n be the number of nights. We have a cost (C) of: C = Cost per night * n + entrance fee C = [B]15n + 50[/B]

the equation of a line is y = mx + 4. find m if the line passes through (-5,0)
the equation of a line is y = mx + 4. find m if the line passes through (-5,0) Plug in our numbers of x = -5, and y = 0: -5m + 4 = 0 To solve for m, we [URL='https://www.mathcelebrity.com/1unk.php?num=-5m%2B4%3D0&pl=Solve']plug in this equation into our search engine[/URL] and we get: [B]m = 0.8 or 4/5[/B] so our line equation becomes: [B]y = 4/5x + 4[/B]

The expression (5x - 2)/(x + 3) is equivalent to which of the following?
The expression (5x - 2)/(x + 3) is equivalent to which of the following? [LIST] [*]A) (5 - 2)/3 [*]B) 5 - 2/3 [*]C) 5 - (2/(x + 3)) [*]D) 5 - (17/(x + 3)) [/LIST] Let's start with an integer x = 2. Plug that into our original expression, and we get: (5(2) - 2)(2 + 3) (10 - 2)/5 8/5 So what we do next is, take x = 2, and plug it into answer choices A-D, and see which one results in 8/5 A) 3/3 = 1 <-- Nope B) Since 5 is 15/3, we have 15/3 - 2/3 = 13/3 which is over 4, so Nope C) 5 - (2/(2 + 3)) = 5 - (2/5). Since 5 is 25/5, we have 25/5 - 2/5 = 23/5. <-- Nope D) 5 - (17/(2 + 3)) = 5 - 17/5. Since 5 is 25/5, we have 25/5 - 17/5 = 8/5 <-- YES Since 8/5 = 8/5, our answer is [B]D) 5 - (17/(x + 3))[/B]

The famous Concorde jet travelled at a speed of 2000km/h for two and a half hours. Do you think it c
The famous Concorde jet travelled at a speed of 2000km/h for two and a half hours. Do you think it could make it to its destination which is 5500km away on time Calculate the total distance traveled @ 2000km/h for 2.5 hours: d = rt d = 2000 * 2.5 d = 5,000 km The answer is [B]no, it cannot make the destination[/B].

The famous mathematician Pythagoras founded the Mathematical Brotherhood in 530 BC. About how many y
The famous mathematician Pythagoras founded the Mathematical Brotherhood in 530 BC. About how many years ago did this happen? BC means before year 0. So we take the current year, which at the time of this post, is 2021. We [U]add[/U] 530 years to that since BC is before year 0, and we get: 2021 + 530 = [B]2551 years ago[/B]

The fastest student in gym class runs 50 meters in 7.4 seconds. The slowest time in the class was 4.
The fastest student in gym class runs 50 meters in 7.4 seconds. The slowest time in the class was 4.36 seconds slower than the fastest time. Slowest time = 7.4 - 4.36 Slowest time = [B]3.04[/B]

The first group orders 3 pizzas and 4 drinks for $33.50. The second group orders 5 pizzas and 6 drin
The first group orders 3 pizzas and 4 drinks for $33.50. The second group orders 5 pizzas and 6 drinks for $54. Find the cost for each pizza and each drink Assumptions: [LIST] [*]Let the cost of each pizza be p [*]Let the cost of each drink be d [/LIST] Givens: [LIST=1] [*]4d + 3p = 33.50 [*]6d + 5p = 54 [/LIST] We have a simultaneous group of equations. To solve this, we can use 3 methods: [LIST] [*][URL='https://www.mathcelebrity.com/simultaneous-equations.php?term1=4d+%2B+3p+%3D+33.50&term2=6d+%2B+5p+%3D+54&pl=Substitution']Substitution Method[/URL] [*][URL='https://www.mathcelebrity.com/simultaneous-equations.php?term1=4d+%2B+3p+%3D+33.50&term2=6d+%2B+5p+%3D+54&pl=Elimination']Elimination Method[/URL] [*][URL='https://www.mathcelebrity.com/simultaneous-equations.php?term1=4d+%2B+3p+%3D+33.50&term2=6d+%2B+5p+%3D+54&pl=Cramers+Method']Cramer's Rule[/URL] [/LIST] No matter what method we use, we get the same answer: [LIST] [*]d = [B]$2.75[/B] [*]p = [B]$7.5[/B] [/LIST]

The first plan has $14 monthly fee and charges an additional $.14 for each minute of calls. The seco
The first plan has $14 monthly fee and charges an additional $.14 for each minute of calls. The second plan had a $21 monthly fee and charges an additional $.10 for each minute of calls. For how many minutes of calls will the cost of the two plans be equal? Set up the cost equation C(m) for the first plan, where m is the amount of minutes you use C(m) = 0.14m + 14 Set up the cost equation C(m) for the second plan, where m is the amount of minutes you use C(m) = 0.10m + 21 Set them equal to each other: 0.14m + 14 = 0.10m + 21 [URL='https://www.mathcelebrity.com/1unk.php?num=0.14m%2B14%3D0.10m%2B21&pl=Solve']Typing this equation into our search engine[/URL], we get: m = [B]175[/B]

The first significant digit in any number must be 1, 2, 3, 4, 5, 6, 7, 8, or 9. It was discovered t
The first significant digit in any number must be 1, 2, 3, 4, 5, 6, 7, 8, or 9. It was discovered that first digits do not occur with equal frequency. Probabilities of occurrence to the first digit in a number are shown in the accompanying table. The probability distribution is now known as Benford's Law. For example, the following distribution represents the first digits in 231 allegedly fraudulent checks written to a bogus company by an employee attempting to embezzle funds from his employer. Digit, Probability 1, 0.301 2, 0.176 3, 0.125 4, 0.097 5, 0.079 6, 0.067 7, 0.058 8, 0.051 9, 0.046 [B][U]Fradulent Checks[/U][/B] Digit, Frequency 1, 36 2, 32 3, 45 4, 20 5, 24 6, 36 7, 15 8, 16 9, 7 Complete parts (a) and (b). (a) Using the level of significance α = 0.05, test whether the first digits in the allegedly fraudulent checks obey Benford's Law. Do the first digits obey the Benford's Law?
Yes or No Based on the results of part (a), could one think that the employe is guilty of embezzlement? Yes or No Show frequency percentages Digit Fraud Probability Benford Probability 1 0.156 0.301 2 0.139 0.176 3 0.195 0.125 4 0.087 0.097 5 0.104 0.079 6 0.156 0.067 7 0.065 0.058 8 0.069 0.051 9 0.03 0.046 Take the difference between the 2 values, divide it by the Benford's Value. Sum up the squares to get the Test Stat of 2.725281277 Critical Value Excel: =CHIINV(0.95,8) = 2.733 Since test stat is less than critical value, we cannot reject, so [B]YES[/B], it does obey Benford's Law and [B]NO[/B], there is not enough evidence to suggest the employee is guilty of embezzlement.

The fixed costs to produce a certain product are 15,000 and the variable costs are $12.00 per item.
The fixed costs to produce a certain product are 15,000 and the variable costs are $12.00 per item. The revenue for a certain product is $27.00 each. If the company sells x products, then what is the revenue equation? R(x) = Revenue per item x number of products sold [B]R(x) = 27x[/B]

The flu is starting to hit Lanberry. Currently, there are 894 people infected, and that number is gr
The flu is starting to hit Lanberry. Currently, there are 894 people infected, and that number is growing at a rate of 5% per day. Overall, how many people will have gotten the flu in 5 days? Our exponential equation for the Flu at day (d) is: F(d) = Initial Flu cases * (1 + growth rate)^d Plugging in d = 5, growth rate of 5% or 0.05, and initial flu cases of 894 we have: F(5) = 894 * (1 + 0.05)^5 F(5) = 894 * (1.05)^5 F(5) = 894 * 1.2762815625 F(5) = [B]1141[/B]

The fraction has a value of 3/5. The sum of the numerator and the denominator was 40. What was the f
The fraction has a value of 3/5. The sum of the numerator and the denominator was 40. What was the fraction? We're given two equations with a fraction with numerator (n) and denominator (d): [LIST=1] [*]n + d = 40 [*]n/d = 3/5 [/LIST] Cross multiply equation 2, we get: 5n = 3d Divide each side by 5: 5n/5 = 3d/5 n = 3d/5 Substitute this into equation 1: 3d/5 + d = 40 Multiply through both sides of the equation by 5: 5(3d/5) = 5d = 40 * 5 3d + 5d =200 To solve this equation for d, we [URL='https://www.mathcelebrity.com/1unk.php?num=3d%2B5d%3D200&pl=Solve']type it in our search engine and we get[/URL]: d = [B]25 [/B] Now substitute that back into equation 1: n + 25 = 40 Using [URL='https://www.mathcelebrity.com/1unk.php?num=n%2B25%3D40&pl=Solve']our equation solver again[/URL], we get: n = [B]15[/B]

the fuel tank of a jet used gas at a constant rate of 300 gallons for each hour of flight. the tank
the fuel tank of a jet used gas at a constant rate of 300 gallons for each hour of flight. the tank can hold a maximum of 2400 gallons of gas. write an equation representing the amount of fuel left in the tank as a function of the number of hours spent flying. We have an equation F(h) where h is the number of hours since the flight took off: [B]F(h) = 2400 - 300h[/B]

The function f(x) = e^x(x - 3) has a critical point at x =
The function f(x) = e^x(x - 3) has a critical point at x = The critical point is where the derivative equals 0. We multiply through for f(x) to get: f(x) = xe^x - 3e^x Using the product rule on the first term f'g + fg', we get: f'(x) = xe^x + e^x - 3e^x f'(x) = xe^x -2e^x f'(x) = e^x(x - 2) We want f'(x) = 0 e^x(x - 2) = 0 When [B]x = 2[/B], then f'(x) = 0

The function f(x) = x^3 - 48x has a local minimum at x = and a local maximum at x = ?
The function f(x) = x^3 - 48x has a local minimum at x = and a local maximum at x = ? f'(x) = 3x^2 - 48 Set this equal to 0: 3x^2 - 48 = 0 Add 48 to each side: 3x^2 = 48 Divide each side by 3: x^2 = 16 Therefore, x = -4, 4 Test f(4) f(4) = 4^3 - 48(4) f(4) = 64 - 192 f(4) = [B]-128 <-- Local minimum[/B] Test f(-4) f(-4) = -4^3 - 48(-4) f(-4) = -64 + 192 f(-4) = [B]128 <-- Local maximum[/B]

The function P(x) = -30x^2 + 360x + 785 models the profit, P(x), earned by a theatre owner on the ba
The function P(x) = -30x^2 + 360x + 785 models the profit, P(x), earned by a theatre owner on the basis of a ticket price, x. Both the profit and the ticket price are in dollars. What is the maximum profit, and how much should the tickets cost? Take the [URL='http://www.mathcelebrity.com/dfii.php?term1=-30x%5E2+%2B+360x+%2B+785&fpt=0&ptarget1=0&ptarget2=0&itarget=0%2C1&starget=0%2C1&nsimp=8&pl=1st+Derivative']derivative of the profit function[/URL]: P'(x) = -60x + 360 We find the maximum when we set the profit derivative equal to 0 -60x + 360 = 0 Subtract 360 from both sides: -60x = -360 Divide each side by -60 [B]x = 6 <-- This is the ticket price to maximize profit[/B] Substitute x = 6 into the profit equation: P(6) = -30(6)^2 + 360(6) + 785 P(6) = -1080 + 2160 + 785 [B]P(6) = 1865[/B]

The girl’s hockey team won 6 games, lost 3 games, and tied 2 games. What fraction of games did they
The girl’s hockey team won 6 games, lost 3 games, and tied 2 games. What fraction of games did they win? Win Fraction = Won Games / Total Games Played Win Fraction = 6 / (6 + 3 + 2) Win Fraction = [B]6/11[/B]

The graph of a polynomial f(x) = (2x - 3)(x - 4)(x + 3) has x-intercepts at 3 values. What are they?
The graph of a polynomial f(x) = (2x - 3)(x - 4)(x + 3) has x-intercepts at 3 values. What are they? A few things to note: [LIST] [*]X-intercepts are found when y (or f(x)) is 0. [*]On the right side, we have 3 monomials. [*]Therefore, y or f(x) could be 0 when [U]any[/U] of these monomials is 0 [/LIST] The 3 monomials are: [LIST=1] [*]2x - 3 = 0 [*]x - 4 = 0 [*]x + 3 = 0 [/LIST] Find all 3 x-intercepts: [LIST=1] [*]2x - 3 = 0. [URL='https://www.mathcelebrity.com/1unk.php?num=2x-3%3D0&pl=Solve']Using our equation calculator[/URL], we see that x = [B]3/2 or 1.5[/B] [*]x - 4 = 0 [URL='https://www.mathcelebrity.com/1unk.php?num=x-4%3D0&pl=Solve']Using our equation calculator[/URL], we see that x = [B]4[/B] [*]x + 3 = 0 [URL='https://www.mathcelebrity.com/1unk.php?num=x%2B3%3D0&pl=Solve']Using our equation calculator[/URL], we see that x = [B]-3[/B] [/LIST] So our 3 x-intercepts are: x = [B]{-3, 3/2, 4}[/B]

The graph shows the average length (in inches) of a newborn baby over the course of its first 15 mon
The graph shows the average length (in inches) of a newborn baby over the course of its first 15 months. Interpret the RATE OF CHANGE of the graph. [IMG]http://www.mathcelebrity.com/community/data/attachments/0/rate-of-change-wp.jpg[/IMG] Looking at our graph, we have a straight line. For straight lines, rate of change [U][I]equals[/I][/U] slope. Looking at a few points, we have: (0, 20), (12, 30) Using our [URL='https://www.mathcelebrity.com/slope.php?xone=0&yone=20&slope=+2%2F5&xtwo=12&ytwo=30&pl=You+entered+2+points']slope calculator for these 2 points[/URL], we get a slope (rate of change) of: [B]5/6[/B]

the grass in jamie’s yard grew 16 centimeters in 10 days. how many days did it take for the grass to
the grass in jamie’s yard grew 16 centimeters in 10 days. how many days did it take for the grass to grow 1 centimeter We set up a proportion of centimeters to days where d is the number of days it takes for the grass to grow 1 centimeter: 16/10 = 1/d To solve this proportion for d, [URL='https://www.mathcelebrity.com/prop.php?num1=16&num2=1&den1=10&den2=d&propsign=%3D&pl=Calculate+missing+proportion+value']we type it in our search engine[/URL] and we get: d = [B]0.625 or 5/8[/B]

The half-life of a radioactive substance is 24 hours and there are 100 grams initially. What is the
The half-life of a radioactive substance is 24 hours and there are 100 grams initially. What is the amount of substance remaining after one week? Using our [URL='https://www.mathcelebrity.com/halflife.php?x=100&t=+0&h=1&t1=7&pl=Calculate+Half+Life+Problem']half life calculator[/URL] converting to days since 24 hours is 1 day and one week is 7 days, we have: [B]0.78125[/B]

The half-life of radioactive kind of manganese is 6 days. How much will be left 18 days, if you star
The half-life of radioactive kind of manganese is 6 days. How much will be left 18 days, if you start with 80 grams of it? Using our [URL='http://www.mathcelebrity.com/halflife.php?x=80&t=+0&h=6&t1=18&pl=Calculate+Half+Life+Problem']half-life calculator,[/URL] we get size [B]10[/B]. [MEDIA=youtube]-_FsUz_ZhXM[/MEDIA]

The height of an object t seconds after it is dropped from a height of 300 meters is s(t)=-4.9t^2 +3
The height of an object t seconds after it is dropped from a height of 300 meters is s(t)=-4.9t^2 +300. Find the average velocity of the object during the first 3 seconds? (b) Use the Mean value Theorem to verify that at some time during the first 3 seconds of the fall the instantaneous velocity equals the average velocity. Find that time. Average Velocity: [ f(3) - f(0) ] / ( 3 - 0 ) Calculate f(3): f(3) = -4.9(3^2) + 300 f(3) = -4.9(9) + 300 f(3) = -44.1 + 300 f(3) = 255.9 Calculate f(0): f(0) = -4.9(0^2) + 300 f(0) = 0 + 300 f(0) = 300 So we have average velocity: Average velocity = (255.9 - 300)/(3 - 0) Average velocity = -44.1/3 Average velocity = -[B]14.7 [/B] Velocity is the first derivative of position s(t)=-4.9t^2 +300 s'(t) = -9.8t So we set velocity equal to average velocity: -9.8t = -14.7 Divide each side by -9.8 to solve for t, we get [B]t = 1.5[/B]

The Henson family cleaned out all their drawers. They found 47 black pens and 39 blue pens. They als
The Henson family cleaned out all their drawers. They found 47 black pens and 39 blue pens. They also found 6 pens in other colors. How many pens did they find in all? The phrase [I]in all[/I] means we add, so we have: Total pens = Black Pens + Blue Pens + Other color pens Total pens = 47 + 39 + 6 Total pens = [B]92[/B]

The holy bible contains 525 pages. A believer reads 10 pages during weekends and 15 pages during wee
The holy bible contains 525 pages. A believer reads 10 pages during weekends and 15 pages during weekdays. How long will it take him to finish reading the bible? Take one 7 day week: 15 + 10 = 25 pages 525 pages/25 pages = [B]21 weeks[/B]

The hourly wages of employees at Rowan have a mean wage rate of $10 per hour with a standard deviati
The hourly wages of employees at Rowan have a mean wage rate of $10 per hour with a standard deviation of $1.20. What is the probability the mean hourly wage of a random sample of 36 employees will be larger than $10.50? Assume the company has a total of 1,000 employees Using our [URL='http://www.mathcelebrity.com/probnormdist.php?xone=10.5&mean=10&stdev=1.2&n=36&pl=P%28X+>+Z%29']normal distribution calculator[/URL], we get P(x > 10.5) = [B]0.00621[/B]

The income i is directly proportional to working hours h
The income i is directly proportional to working hours h The phrase [I]directly proportional[/I] means there exists a constant k such that: [B]I = kh[/B]

the initial deposit in a bank account was $6000 and it has an annual interest rate of 4.5%. Find the
the initial deposit in a bank account was $6000 and it has an annual interest rate of 4.5%. Find the amount of money in the bank after 3 years Using our [URL='https://www.mathcelebrity.com/compoundint.php?bal=6000&nval=4.5&int=3&pl=Annually']balance and interest calculator[/URL], we get: [B]$6,853.60[/B]

The IQ scores are normally distributed with a mean of 100 and a standard deviation of 15. a) What i
The IQ scores are normally distributed with a mean of 100 and a standard deviation of 15. a) What is the probability that a randomly person has an IQ between 85 and 115? b) Find the 90th percentile of the IQ distribution c) If a random sample of 100 people is selected, what is the standard deviation of the sample mean? a) [B]68%[/B] from the [URL='http://www.mathcelebrity.com/probnormdist.php?xone=50&mean=100&stdev=15&n=1&pl=Empirical+Rule']empirical rule calculator[/URL] b) P(z) = 0.90. so z = 1.28152 using Excel NORMSINV(0.9)
(X - 100)/10 = 1.21852 X = [B]113[/B] rounded up c) Sample standard deviation is the population standard deviation divided by the square root of the sample size 15/sqrt(100) = 15/10 =[B] 1.5[/B]

The January record high temperature in Austin, Texas is 92 degrees. The January record low temperatu
The January record high temperature in Austin, Texas is 92 degrees. The January record low temperature is -1 degree. What is the difference between the record high and low temperatures? Difference = High - Low Difference = 92 - -1 Difference = 92 + 1. <-- Since minus negative equals positive Difference = [B]93 degrees[/B]

The jimenez family inherited land that was purchased for $50,000 in 1967. The value of the land incr
The jimenez family inherited land that was purchased for $50,000 in 1967. The value of the land increased by approximately 4% per year. What is the approximate value of the land by the year 2016? 1967 to 2016 is 49 years. So we have 341,667.47 using our [URL='http://www.mathcelebrity.com/simpint.php?av=&p=50000&int=4&t=49&pl=Compound+Interest']compound interest calculator[/URL].

The Lakers recently scored 81 points. Their points came from 2 and 3 point baskets. If they made 39
The Lakers recently scored 81 points. Their points came from 2 and 3 point baskets. If they made 39 baskets total, how many of each basket did they make? Let x = 2 point baskets and y = 3 point baskets. We have the following given equations: [LIST=1] [*]x + y = 39 [*]2x + 3y = 81 [/LIST] Using our [URL='http://www.mathcelebrity.com/simultaneous-equations.php?term1=x%2By%3D39&term2=2x+%2B+3y+%3D+81&pl=Cramers+Method']simultaneous equations calculator[/URL], we get: [B]x = 36 <-- 2 point baskets y = 3 <-- 3[B] point baskets [/B][/B] To confirm our work: [LIST=1] [*]36 + 3 = 39 [*]2(36) + 3(3) = 72 + 9 = 81 [/LIST]

The Lakewood library has $8,040 to buy science magazines. If each magazine costs $3, how many magazi
The Lakewood library has $8,040 to buy science magazines. If each magazine costs $3, how many magazines will the library be able to buy? Let number of magazines be m. We know that: Cost per magazine * m = Total Cost We're given Total Cost = 8040 and Cost per magazine = 3, so we have 3m = 8040 To solve this equation for m, we [URL='https://www.mathcelebrity.com/1unk.php?num=3m%3D8040&pl=Solve']type it in our math engine[/URL] and we get: m = [B]2680[/B]

The larger number b exceeds the smaller number c by 45.
The larger number b exceeds the smaller number c by 45. Exceeds means greater than or more than, so we have: [B]b = c + 45[/B]

The larger of 2 numbers is 1 more than 3 times the smaller number
The larger of 2 numbers is 1 more than 3 times the smaller number. Let the larger number be l. Let the smaller number be s. The algebraic expression is: 3 times the smaller number is written as: 3s 1 more than that means we add 1 3s + 1 Our final algebraic expression uses the word [I]is[/I] meaning an equation. So we set l equal to 3s + 1 [B]l = 3s + 1[/B]

The largest American flag ever flown had a perimeter of 1,520 feet and a length of 505 feet. Find th
The largest American flag ever flown had a perimeter of 1,520 feet and a length of 505 feet. Find the width of the flag. for a rectangle, the Perimeter P is given by: P = 2l + 2w P[URL='https://www.mathcelebrity.com/rectangle.php?l=505&w=&a=&p=1520&pl=Calculate+Rectangle']lugging in our numbers for Perimeter and width into our rectangle calculator[/URL], we get: l =[B] 255[/B]

the left and right page numbers of an open book are two consecutive integers whose number is 235 fin
the left and right page numbers of an open book are two consecutive integers whose number is 235 find the page numbers Using our [URL='https://www.mathcelebrity.com/consecintwp.php?pl=Sum&num=+235']consecutive integer calculator[/URL], we get: [B]117, 118[/B]

The left and right page numbers of an open book are two consecutive integers whose sum is 403. Find
The left and right page numbers of an open book are two consecutive integers whose sum is 403. Find these page numbers. Page numbers left and right are consecutive integers. So we want to find a number n and n + 1 where: n + n + 1 = 403 Combining like terms, we get: 2n + 1 = 403 Typing that equation into our search engine, we get: [B]n = 201[/B] This is our left hand page. Our right hand page is: 201 + 1 = [B]202[/B]

The length of a rectangle is 6 less than twice the width. If the perimeter is 60 inches, what are th
The length of a rectangle is 6 less than twice the width. If the perimeter is 60 inches, what are the dimensions? Set up 2 equations given P = 2l + 2w: [LIST=1] [*]l = 2w - 6 [*]2l + 2w = 60 [/LIST] Substitute (1) into (2) for l: 2(2w - 6) + 2w = 60 4w - 12 + 2w = 60 6w - 12 = 60 To solve for w, [URL='https://www.mathcelebrity.com/1unk.php?num=6w-12%3D60&pl=Solve']type this into our math solver [/URL]and we get: w = [B]12 [/B] To solve for l, substitute w = 12 into (1) l = 2(12) - 6 l = 24 - 6 l = [B]18[/B]

The length of a rectangle is equal to triple the width. Find the length of the rectangle if the peri
The length of a rectangle is equal to triple the width. Find the length of the rectangle if the perimeter is 80 inches. The perimeter (P) of a rectangle is: 2l + 2w = P We're given two equations: [LIST=1] [*]l = 3w [*]2l + 2w = 80 [/LIST] We substitute equation 1 into equation 2 for l: 2(3w) + 2w = 80 6w + 2w = 80 To solve this equation for w, we [URL='https://www.mathcelebrity.com/1unk.php?num=6w%2B2w%3D80&pl=Solve']type it in our search engine[/URL] and we get: w = 10 To solve for the length (l), we substitute w = 10 into equation 1 above: l = 3(10) l = [B]30[/B]

The length of a rectangle is three times its width.If the perimeter is 80 feet, what are the dimensi
The length of a rectangle is three times its width.If the perimeter is 80 feet, what are the dimensions? We're given 2 equations: [LIST=1] [*]l = 3w [*]P = 80 = 2l + 2w = 80 [/LIST] Substitute (1) into (2) for l: 2(3w) + 2w = 80 6w + 2w = 80 8w = 80 Divide each side by 8: 8w/8 = 80/8 w = [B]10 [/B] Substitute w = 10 into (1) l = 3(10) l = [B]30[/B]

The length of a rectangular building is 6 feet less than 3 times the width. The perimeter is 120 fee
The length of a rectangular building is 6 feet less than 3 times the width. The perimeter is 120 feet. Find the width and length of the building. P = 2l + 2w Since P = 120, we have: (1) 2l + 2w = 120 We are also given: (2) l = 3w - 6 Substitute equation (2) into equation (1) 2(3w - 6) + 2w = 120 Multiply through: 6w - 12 + 2w = 120 Combine like terms: 8w - 12 = 120 Add 12 to each side: 8w = 132 Divide each side by 8 to isolate w: w =16.5 Now substitute w into equation (2) l = 3(16.5) - 6 l = 49.5 - 6 l = 43.5 So (l, w) = (43.5, 16.5)

The length of a rectangular building is 6 feet less than 3 times the width. The perimeter is 120 fee
The length of a rectangular building is 6 feet less than 3 times the width. The perimeter is 120 feet. Find the width and length of the building. Using our [URL='http://www.mathcelebrity.com/rectangle-word-problems.php?t1=perimeter&v1=120&t2=length&v2=6&op=less&v3=3&t4=times&t5=width&pl=Calculate']rectangular word problem calculator[/URL], we have: [LIST] [*][B]l = 43.5[/B] [*][B]w = 16.5[/B] [/LIST]

The length of a rectangular building is 6 feet less than 3 times the width. The perimeter is 120 fee
The length of a rectangular building is 6 feet less than 3 times the width. The perimeter is 120 feet. Find the width and length of the building. Using our [URL='http://www.mathcelebrity.com/rectangle-word-problems.php?t1=perimeter&v1=120&t2=length&v2=6&op=less&v3=3&t4=times&t5=width&pl=Calculate']rectangle word problem calculator[/URL], we get: [LIST] [*][B]w = 16.5[/B] [*][B]l = 43.5[/B] [/LIST]

the length of a rectangular map is 15 inches and the perimeter is 50 inches. Find the width
The length of a rectangular map is 15 inches and the perimeter is 50 inches. Find the width. Using our r[URL='http://www.mathcelebrity.com/rectangle.php?l=3&w=&a=&p=50&pl=Calculate+Rectangle']ectangle solver[/URL], we get [B]w = 10[/B].

The length of a train car is 50.6 feet. This is 5.8 feet less than 6 times the width. What is the wi
The length of a train car is 50.6 feet. This is 5.8 feet less than 6 times the width. What is the width? 5.8 feet less than 6 times the width is an algebraic expression: 6w - 5.8 We set this equal to the length of 50.6 6w - 5.8 = 50.6 Add 5.8 to each side: 6w - 5.8 + 5.8 = 50.6 + 5.8 Cancel the 5.8 on the left side: 6w = 56.4 Divide each side by 6: 6w/6 = 56.4/6 [URL='http://www.mathcelebrity.com/1unk.php?num=6w-5.8%3D50.6&pl=Solve']Typing this problem into the search engine[/URL], we get [B]w = 9.4[/B]. [MEDIA=youtube]gfM-d_Ej728[/MEDIA]

The length of a wooden frame is 1 foot longer than its width and its area is equal to 12ft²
The length of a wooden frame is 1 foot longer than its width and its area is equal to 12ft² The frame is a rectangle. The area of a rectangle is A = lw. So were given: [LIST=1] [*]l = w + 1 [*]lw = 12 [/LIST] Substitute equation (1) into equation (2) for l: (w + 1) * w = 12 Multiply through and simplify: w^2 + w = 12 We have a quadratic equation. To solve for w, we type this equation into our search engine and we get two solutions: w = 3 w = -4 Since width cannot be negative, we choose the positive result and have: w = [B]3[/B] To solve for length, we plug w = 3 into equation (1) above and get: l = 3 + 1 l = [B]4[/B]

The length of Sally’s garden is 4 meters greater than 3 times the width. The perimeter of her garden
The length of Sally’s garden is 4 meters greater than 3 times the width. The perimeter of her garden is 72 meters. Find the dimensions of Sally’s garden. Gardens have a rectangle shape. Perimeter of a rectangle is 2l + 2w. We're given: [LIST=1] [*]l = 3w + 4 [I](3 times the width Plus 4 since greater means add)[/I] [*]2l + 2w = 72 [/LIST] We substitute equation (1) into equation (2) for l: 2(3w + 4) + 2w = 72 Multiply through and simplify: 6w + 8 + 2w = 72 (6 +2)w + 8 = 72 8w + 8 = 72 To solve this equation for w, we [URL='https://www.mathcelebrity.com/1unk.php?num=8w%2B8%3D72&pl=Solve']type it in our search engine[/URL] and we get: w = [B]8 [/B] To solve for l, we substitute w = 8 above into Equation (1): l = 3(8) + 4 l = 24 + 4 l = [B]28[/B]

The length of Sally’s garden is 4 meters greater than 3 times the width. The perimeter of her garden
The length of Sally’s garden is 4 meters greater than 3 times the width. The perimeter of her garden is 72 meters A garden is a rectangle, which has perimeter P of: P = 2l + 2w With P = 72, we have: 2l + 2w = 72 We're also given: l = 3w + 4 We substitute this into the perimeter equation for l: 2(3w + 4) + 2w = 72 6w + 8 + 2w = 72 To solve this equation for w, we t[URL='https://www.mathcelebrity.com/1unk.php?num=6w%2B8%2B2w%3D72&pl=Solve']ype it in our search engine[/URL] and we get: w =[B] 8[/B] Now, to solve for l, we substitute w = 8 into our length equation above: l = 3(8) + 4 l = 24 + 4 l = [B]28[/B]

The length of the flag is 2 cm less than 7 times the width. The perimeter is 60cm. Find the length a
The length of the flag is 2 cm less than 7 times the width. The perimeter is 60cm. Find the length and width. A flag is a rectangle shape. So we have the following equations Since P = 2l + 2w, we have 2l + 2w = 60 l = 7w - 2 Substitute Equation 1 into Equation 2: 2(7w -2) + 2w = 60 14w - 4 + 2w = 60 16w - 4 = 60 Add 4 to each side 16w = 64 Divide each side by 16 to isolate w w = 4 Which means l = 7(4) - 2 = 28 - 2 = 26

The letters that form the word ALGEBRA are placed in a bowl. What is the probability of choosing a l
The letters that form the word ALGEBRA are placed in a bowl. What is the probability of choosing a letter that is not “A”? ALGEBRA has 7 letters Of the 7 letters, we have 2 A's. So we have 7 - 2 = 5 letters which are not A P(Not A) = Letters not A / Total letters P(Not A) = [B]5/7[/B]

The library received 1,125 new books. 45 books fit on each shelf. How many shelves are needed for th
The library received 1,125 new books. 45 books fit on each shelf. How many shelves are needed for the new books? Shelves needed = Total Books / Number of Shelves Shelves needed = 1,125/45 Shelves needed = [B]25[/B]

The longest bridge in america is 1700 ft long. Write an inequality that describes the length of ever
The longest bridge in america is 1700 ft long. Write an inequality that describes the length of every bridge. Let the bridge length be b. Since no bridge will ever be greater than 1700 ft, we have: [B]b <= 1700[/B]

The lowest score that is greater than 25% of the scores
The lowest score that is greater than 25% of the scores. [B]This is defined as the 25th percentile.[/B]

the lowest temperature on may 15 is 2/3 as warm as the warmest temperature on may 15. the lowest tem
the lowest temperature on may 15 is 2/3 as warm as the warmest temperature on may 15. the lowest temperature on may 15 is 60F what is the warmest temperature on may 15? Set up an equation where w is the warmest temperature on May 15: 60 = 2/3w [URL='https://www.mathcelebrity.com/1unk.php?num=60%3D2%2F3w&pl=Solve']Type this equation into our search engine[/URL], and we get: w = [B]90[/B]

The margarita is one of the most common tequila-based cocktails, made with tequila, triple sec, and
The margarita is one of the most common tequila-based cocktails, made with tequila, triple sec, and lime juice, often served with salt on the glass rim. A manager at a local bar is concerned that the bartender is not using the correct amounts of the three ingredients in more than 50% of margaritas. He secretly observed the bartender and found that he used the CORRECT amounts in only 9 out of the 39 margaritas in the sample. Use the critical value approach to test if the manager's suspicion is justified at ? = 0.10. Let p represent the proportion of all margaritas made by the bartender that have INCORRECT amounts of the three ingredients. Use Table 1. a. Select the null and the alternative hypotheses. [B]H0: p ? 0.50; HA: p > 0.50[/B] [B][/B] b. Calculate the sample proportion. (Round your answer to 3 decimal places.) 9/39 = [B]0.231 [/B] c. Calculate the value of test statistic. (Round your intermediate calculations to 4 decimal places and final answer to 2 decimal places.) Using our [URL='http://www.mathcelebrity.com/proportion_hypothesis.php?x=9&n=39&ptype=%3C&p=+0.5&alpha=+0.10&pl=Proportion+Hypothesis+Testing']proportion hypothesis calculator[/URL], we get: [B]Test Stat = -3.36[/B] [B][/B] d. Calculate the critical value. (Round your answer to 2 decimal places.) Using the link above, we get a critical value of [B]1.2816 [/B] e. What is the conclusion? [B]The manager’s suspicion is not justified since the value of the test statistic does not fall in the rejection region. Do not reject H0[/B] [B][/B]

The marianas trench is 10415m below the sea level. Directly above it a helicopter is hovering 3200m
The marianas trench is 10415m below the sea level. Directly above it a helicopter is hovering 3200m above sea level. How far is the helicopter from the trench? Below sea level is negative: -10415 Above sea level is positive: +3200 The distance is found by: +3200 - -10415 +3200 + 10415 [B]13,615[/B]

The mean age of 10 women in an office is 30 years old. The mean age of 10 men in an office is 29 yea
The mean age of 10 women in an office is 30 years old. The mean age of 10 men in an office is 29 years old. What is the mean age (nearest year) of all the people in the office? Mean is another word for [U]average[/U]. Mean age of women = Sum of all ages women / number of women We're told mean age of women is 30, so we have: Sum of all ages women / 10 = 30 Cross multiply, and we get: Sum of all ages of women = 30 * 10 Sum of all ages of women = 300 Mean age of men = Sum of all ages men / number of men We're told mean age of men is 29, so we have: Sum of all ages men / 10 = 29 Cross multiply, and we get: Sum of all ages of men = 29 * 10 Sum of all ages of men = 290 [U]Calculate mean age (nearest year) of all the people in the office:[/U] mean age of all the people in the office = Sum of all ages of people in the office (men and women) / Total number of people in the office mean age of all the people in the office = (300 + 290) / (10 + 10) mean age of all the people in the office = 590 / 20 mean age of all the people in the office = 29.5 The question asks for nearest year. Since this is a decimal, we round down to either 29 or up to 30. Because the decimal is greater or equal to 0.5 (halfway), we round [U]up[/U] to [B]30[/B]

The mean age of 5 people in a room is 28 years. A person enters the room. The mean age is now 32. W
The mean age of 5 people in a room is 28 years. A person enters the room. The mean age is now 32. What is the age of the person who entered the room? The sum of the 5 people's scores is S. We know: S/5 = 28 Cross multiply: S = 140 We're told that: (140 + a)/6 = 32 Cross multiply: 140 + a = 192 [URL='https://www.mathcelebrity.com/1unk.php?num=140%2Ba%3D192&pl=Solve']Type this equation into our search engine[/URL], we get: a = [B]52[/B]

The mean age of 5 people in a room is 32 years. A person enters the room. The mean age is now 40. Wh
The mean age of 5 people in a room is 32 years. A person enters the room. The mean age is now 40. What is the age of the person who entered the room? Mean = Sum of Ages in Years / Number of People 32 = Sum of Ages in Years / 5 Cross multiply: Sum of Ages in Years = 32 * 5 Sum of Ages in Years = 160 Calculate new mean after the next person enters the room. New Mean = (Sum of Ages in Years + New person's age) / (5 + 1) Given a new Mean of 40, we have: 40 = (160 + New person's age) / 6 Cross multiply: New Person's Age + 160 = 40 * 6 New Person's Age + 160 = 240 Let the new person's age be n. We have: n + 160 = 240 To solve for n, [URL='https://www.mathcelebrity.com/1unk.php?num=n%2B160%3D240&pl=Solve']we type this equation into our search engine[/URL] and we get: n = [B]80[/B]

The mean age of 5 people in a room is 38 years. A person enters the room. The mean age is now 39. Wh
The mean age of 5 people in a room is 38 years. A person enters the room. The mean age is now 39. What is the age of the person who entered the room? The mean formulas is denoted as: Mean = Sum of Ages / Total People We're given Mean = 38 and Total People = 5, so we plug in our numbers: 28 = Sum of Ages / 5 Cross multiply, and we get: Sum of Ages = 28 * 5 Sum of Ages = 140 One more person enters the room. The mean age is now 39. Set up our Mean formula: Mean = Sum of Ages / Total People With a new Mean of 39 and (5 + 1) = 6 people, we have: 39 = Sum of Ages / 6 But the new sum of Ages is the old sum of ages for 5 people plus the new age (a): Sum of Ages = 140 + a So we have: 29 = (140 + a)/6 Cross multiply: 140 + a = 29 * 6 140 + a = 174 To solve for a, [URL='https://www.mathcelebrity.com/1unk.php?num=140%2Ba%3D174&pl=Solve']we type this equation into our search engine[/URL] and we get: a = [B]34[/B]

The mean height of a class of 20 children is 1.27 the mean height of 12 boys in the class is 1.29 wh
The mean height of a class of 20 children is 1.27 the mean height of 12 boys in the class is 1.29 what is the mean height of the girls in the class? The mean of sums is the sum of means. So we have: Total Height / 20 = 1.27 Cross multiplying, we get: Total Height = 20 * 1.27 Total Height = 25.4 Boys Height / 12 = 1.29 Cross multiplying, we get: Boys Height = 12 * 1.29 Boys Height = 15.48 The Problem asks for mean height for girls. The formula is: Girls Height / # of Girls = Mean of Girls Height # of Girls = Total children - # of boys # of Girls = 20 - 12 # of Girls = 8 Girls Height = Total Height - Boys Height Girls Height = 25.4 - 15.48 Girls Height = 9.92 Plugging this into the Mean of girls height, we get: 9.92 /8 = [B]1.24[/B]

the mean of 12 scores is 8.8 . what is the sum of the scores ?
the mean of 12 scores is 8.8 . what is the sum of the scores ? The Mean is denoted as: Mean = Sum / count We're given: 8.8 = Sum / 12 Cross multiply and we get: Sum = 8.8*12 Sum = [B]105.6[/B]

The mean of 3 numbers is 20. Two of the numbers are 21, and 35. What is the 3rd number?
The mean of 3 numbers is 20. Two of the numbers are 21, and 35. What is the 3rd number? The mean of 3 numbers is the sum of 3 numbers divided by 3. Let the 3rd number be n. We have: Mean = (21 + 35 + n) / 3 The Mean is given as 20, so we have: 20 = (n + 56) / 3 Cross multiply: n + 56 = 20 * 3 n + 56 = 60 To solve for n, we [URL='https://www.mathcelebrity.com/1unk.php?num=n%2B56%3D60&pl=Solve']type this number in our search engine [/URL]and we get: n = [B]4[/B]

The mean of two numbers is 49.1. The first number is 18.3. What is the second number
The mean of two numbers is 49.1. The first number is 18.3. What is the second number We call the second number n. Since the mean is an average, in this case 2 numbers, we have: (18.3 + n)/2 = 49.1 Cross multiply: 18.3 + n = 98.2 [URL='https://www.mathcelebrity.com/1unk.php?num=18.3%2Bn%3D98.2&pl=Solve']Typing this equation into our search engine[/URL], we get: [B]n = 79.9[/B]

the midpoint between m and n
the midpoint between m and n The [I]midpoint is halfway between[/I] m and n: [B](m + n)/2[/B]

The minimum daily requirement of vitamin C for 14 year olds is at least 50 milligrams per day. An av
The minimum daily requirement of vitamin C for 14 year olds is at least 50 milligrams per day. An average sized apple contains 6 milligrams of vitamin C. How many apples would a person have to eat each day to satisfy this requirement? Let a be the number of apples required. The phrase [I]at least[/I] means greater than or equal to, so we have the inequality: 6a >= 50 To solve this inequality, we [URL='https://www.mathcelebrity.com/interval-notation-calculator.php?num=6a%3E%3D50&pl=Show+Interval+Notation']type it in our math engine[/URL] and we get: [B]a >= 8.3333 apples or rounded up to a full number, we get 9 apples[/B]

The monthly earnings of a group of business students are are normally distributed with a standard de
The monthly earnings of a group of business students are are normally distributed with a standard deviation of 545 dollars. A researcher wants to estimate the mean monthly earnings of all business students. Find the sample size needed to have a confidence level of 95% and a margin of error of 128 dollars.

The monthly earnings of a group of business students are are normally distributed with a standard de
The monthly earnings of a group of business students are are normally distributed with a standard deviation of 545 dollars. A researcher wants to estimate the mean monthly earnings of all business students. Find the sample size needed to have a confidence level of 95% and a margin of error of 128 dollars.

The monthly earnings of a group of business students are are normally distributed with a standard de
[URL]http://mathcelebrity.com/community/threads/standard-deviation-of-545-dollars-find-the-sample-size-needed-to-have-a-confidence-level-of-95-and.450/[/URL]

The monthly rental for an apartment is $412.50. How much would the rent be for one year?
The monthly rental for an apartment is $412.50. How much would the rent be for one year? Since there are 12 months in a year, we have: Yearly Rent = Monthly Rent * 12 Yearly Rent = $412.50 * 12 Yearly Rent = [B]$4,950[/B]

The moon's diameter is 2,159 miles. What is the surface area of the moon? Round to the nearest mile.
The moon's diameter is 2,159 miles. What is the surface area of the moon? Round to the nearest mile. The moon is a sphere. So our Surface Area formula is: S =4pir^2 If diameter is 2,159, then radius is 2,159/2 = 1079.5. Plug this into the Surface Area of a sphere formula: S = 4 * pi * 1079.5^2 S = 4 * pi *1165320.25 S = 4661281 pi S = [B]14,643,846.15 square miles[/B]

The negative of the sum of C and D is equal to the difference of the negative of C and D
The negative of the sum of C and D is equal to the difference of the negative of C and D The negative of the sum of C and D means -1 times the sum of C and D -(C + D) Distribute the negative sign: -C - D the difference of the negative of C and D means we subtract D from negative C -C - D So this statement is [B]true[/B] since -C - D = -C - D

The next number in the series 2,5,11,20,32,47, is
The next number in the series 2,5,11,20,32,47, is [LIST] [*]2 + 3 = 5 [*]5 + 6 = 11 [*]11 + 9 = 20 [*]20 + 12 = 32 [*]32 + 15 = 47 [/LIST] Notice the addition pattern: 3, 6, 9, 12, 15 This means our next term is: 47 + (15 + 3) 47 + 18 [B]65 [MEDIA=youtube]mAj3tqXUbZs[/MEDIA][/B]

The next number in the series 38 36 30 28 22 is
The next number in the series 38 36 30 28 22 is Notice the change of factors. Subtract 2, Subtract 6, Subtract 2, Subtract 6. So the next number should subtract 2. 22 - 2 = [B]20 [MEDIA=youtube]x7SHk_6-aok[/MEDIA][/B]

The NJ state education department finds that in a random sample of 100 persons who attended college,
The NJ state education department finds that in a random sample of 100 persons who attended college, 40 received a college degree. What's the 95% confidence interval for the proportion of college graduates out of all the persons who attended college? [URL='http://www.mathcelebrity.com/propconf.php?bign=100&smalln=40&conf=95&pl=Proportion+Confidence+Interval']Proportion Confidence Interval Test[/URL] 0.304 < p < 0.496 --> [B]30.4% < p < 49.6%[/B]

The noise level of an ambulance siren is 10 decibels louder than that of a car horn. If d represents
The noise level of an ambulance siren is 10 decibels louder than that of a car horn. If d represents the noise level, in decibels, of a car horn, express the noise level of an ambulance siren in terms of the noise level of a car horn. Let s be the level of the ambulance siren. [B]s = d + 10[/B]

The number -2.34 can be found between which two integers
The number -2.34 can be found between which two integers We want to take the integer of -2.34 which is -2 Since -2.34 is less than 0, we subtract 1: -2 -1 = -3 Therefore, -2.34 lies between [B]-2 and -3[/B] -3 < -2.34 < -2

The number of days in t weeks and 5 days
The number of days in t weeks and 5 days Each week has 7 days, so we have [B]d = 7t + 5[/B]

the number of minutes in h hours and 32 minutes
the number of minutes in h hours and 32 minutes 60 minutes in each hour, so we have: [B]60h + 32[/B]

the number of minutes in h hours and 49 minutes
the number of minutes in h hours and 49 minutes 1 hour = 60 minutes so we have h hours = 60h minutes Add this to 49 minutes [B]60h + 49[/B]

the number of students allowed on a trip cannot be greater than 85 let s represent the number of stu
the number of students allowed on a trip cannot be greater than 85 let s represent the number of students x <= 85

The Oakdale High School Speech and Debate Club hosted its annual car wash fundraiser. Each club memb
The Oakdale High School Speech and Debate Club hosted its annual car wash fundraiser. Each club member brought a bottle of car wash soap, so there were 8 total bottles. 6 of the bottles contained orange soap. If a club member randomly selects 5 bottles to pour into the first soap bucket, what is the probability that all of them contain orange soap? This is assumed to be draw without replacement, so we have: [LIST=1] [*]Draw 1: 6/8 [*]Draw 2: 5/7 [*]Draw 3: 4/6 [*]Draw 4: 3/5 [*]Draw 5: 2/4 [/LIST] Since they are independent events, we multiply: 6/8 * 5/7 * 4/6 * 3/5 * 2/4 (6 * 5 * 4 * 3 * 2)/(8 * 7 * 6 * 5 * 4) 720/6720 [B]0.1071[/B]

The observation which occurs most frequently in a sample is the
The observation which occurs most frequently in a sample is the [B]mode[/B]

The ones digit of a two-digit number is three, while the tens digit is four.
The ones digit of a two-digit number is three, while the tens digit is four. We write this as tens digit ones digit: [B]43[/B]

The opposite of the difference of h and 5
The opposite of the difference of h and 5 The difference of h and 5 h - 5 The opposite of the difference of h and 5 means we multiply the difference of h and 5 by -1: -(h - 5) Distribute the negative sign: [B]5 - h[/B]

The ordered pairs (8,12), (16, 24), (x, 21), (26, y ) represent a proportional relationship. Find th
The ordered pairs (8,12), (16, 24), (x, 21), (26, y ) represent a proportional relationship. Find the value of x and the value of y. 12/8 = 1.5 24/16 = 1.5 So we have our proportion; y/x = 1.5 or 3/2 [U]For (x, 21), we have:[/U] 21/x = 3/2 [URL='https://www.mathcelebrity.com/prop.php?num1=21&num2=3&den1=x&den2=2&propsign=%3D&pl=Calculate+missing+proportion+value']Type this proportion into our search engine[/URL] and we get: x = [B]14[/B] For (26, y), we have: y/26 = 3/2 [URL='https://www.mathcelebrity.com/prop.php?num1=y&num2=3&den1=26&den2=2&propsign=%3D&pl=Calculate+missing+proportion+value']Type this proportion into our search engine[/URL] and we get; y = [B]39[/B]

The original price of a computer was $895.00. Eleanor had a 25% off coupon which she was able to us
The original price of a computer was $895.00. Eleanor had a 25% off coupon which she was able to use to make the purchase. If sales tax of 6.5% was added after the discount was taken, how much did Eleanor pay altogether for the computer? First, apply the discount: $895 * 25% = $223.75 $895 - $223.75 = $671.25 Now, apply sales tax of 6.5% to this discount price of $671.25 $671.25 * 1.065 = [B]$714.88[/B]

the output is double the input
the output is double the input Double means multiply by 2. So this means a function with input of x and output of y such that: [B]y = 2x[/B]

The Palafoxes make $3,840 a month. They spend $1,600 for rent. What fraction of their income goes to
The Palafoxes make $3,840 a month. They spend $1,600 for rent. What fraction of their income goes to rent? Rent Payment Fraction = Rent Payment / Total Income Rent Payment Fraction = 1600 / 3840 Our greatest common factor of 1600 and 3840 is 320. So if we divide 1600 and 3840 by 320, we get: Rent Payment Fraction = [B]5/12 [MEDIA=youtube]DsXk6AKT18M[/MEDIA][/B]

The patient recovery time from a particular surgical procedure is normally distributed with a mean o
The patient recovery time from a particular surgical procedure is normally distributed with a mean of 5.3 days and a standard deviation of 2.1 days. What is the median recovery time? a. 2.7 b. 5.3 c. 7.4 d. 2.1 [B]b. 5.3 (mean, median, and mode are all the same in a normal distribution)[/B]

The perfect square less than 30
The perfect square less than 30 We know that: [LIST] [*]5^2 = 25 [*]6^ = 36 [/LIST] So our answer is [B]5[/B]

The perimeter of a bedroom door is 28 feet. It is 4 feet wide. How tall is it?
The perimeter of a bedroom door is 28 feet. It is 4 feet wide. How tall is it? Using our[URL='https://www.mathcelebrity.com/rectangle.php?l=&w=4&a=&p=28&pl=Calculate+Rectangle'] rectangle calculator[/URL], we get: l = [B]10[/B]

The perimeter of a college basketball court is 102 meters and the length is twice as long as the wid
The perimeter of a college basketball court is 102 meters and the length is twice as long as the width. What are the length and width? A basketball court is a rectangle. The perimeter P is: P = 2l + 2w We're also given l = 2w and P = 102. Plug these into the perimeter formula: 2(2w) + 2w = 102 4w + 2w = 102 6w = 102 [URL='https://www.mathcelebrity.com/1unk.php?num=6w%3D102&pl=Solve']Typing this equation into our calculator[/URL], we get: [B]w = 17[/B] Plug this into the l = 2w formula, we get: l = 2(17) [B]l = 34[/B]

The perimeter of a garden is 70 meters. Find its actual dimensions if its length is 5 meters longer
The perimeter of a garden is 70 meters. Find its actual dimensions if its length is 5 meters longer than twice its width. Let w be the width, and l be the length. We have: P = l + w. Since P = 70, we have: [LIST=1] [*]l + w = 70 [*]l = 2w + 5 [/LIST] Plug (2) into (1) 2w + 5 + w = 70 Group like terms: 3w + 5 = 70 Using our [URL='https://www.mathcelebrity.com/1unk.php?num=3w%2B5%3D70&pl=Solve']equation calculator[/URL], we get [B]w = 21.66667[/B]. Which means length is: l = 2(21.6667) + 5 l = 43.33333 + 5 [B]l = 48.3333[/B]

The perimeter of a poster is 20 feet. The poster is 6 feet tall. How wide is it?
The perimeter of a poster is 20 feet. The poster is 6 feet tall. How wide is it? [U]Assumptions and givens:[/U] [LIST] [*]The poster has a rectangle shape [*]l = 6 [*]P = 20 [*]The perimeter of a rectangle (P) is: 2l + 2w = P [/LIST] Plugging in our l and P values, we get: 2(6) + 2w = 20 Multiplying through and simplifying, we get: 12 + 2w = 20 To solve for w, we [URL='https://www.mathcelebrity.com/1unk.php?num=12%2B2w%3D20&pl=Solve']type this equation into our search engine [/URL]and we get: w = [B]4[/B]

The perimeter of a rectangle is 400 meters. The length is 15 meters less than 4 times the width. Fin
The perimeter of a rectangle is 400 meters. The length is 15 meters less than 4 times the width. Find the length and the width of the rectangle. l = 4w - 15 Perimeter = 2l + 2w Substitute, we get: 400 = 2(4w - 15) + 2w 400 = 8w - 30 + 2w 10w - 30 = 400 Add 30 to each side 10w = 370 Divide each side by 10 to isolate w w = 37 Plug that back into our original equation to find l l = 4(37) - 15 l = 148 - 15 l = 133 So we have (l, w) = (37, 133)

The perimeter of a rectangle parking lot is 340 m. If the length of the parking lot is 97 m, what is
The perimeter of a rectangle parking lot is 340 m. If the length of the parking lot is 97 m, what is it’s width? The formula for a rectangles perimeter P, is: P = 2l + 2w where l is the length and w is the width. Plugging in our P = 340 and l = 97, we have: 2(97) + 2w = 340 Multiply through, we get: 2w + 194 = 340 [URL='https://www.mathcelebrity.com/1unk.php?num=2w%2B194%3D340&pl=Solve']Type this equation into our search engine[/URL], we get: [B]w = 73[/B]

The perimeter of a rectangular backyard is 162 feet. It is 52 feet long. How wide is it?
The perimeter of a rectangular backyard is 162 feet. It is 52 feet long. How wide is it? We [URL='https://www.mathcelebrity.com/rectangle.php?l=52&w=&a=&p=162&pl=Calculate+Rectangle']use our rectangle solver to solve for w[/URL]. We get: [B]w = 29[/B]

The perimeter of a rectangular bakery is 204 feet. It is 66 feet long. How wide is it?
The perimeter of a rectangular bakery is 204 feet. It is 66 feet long. How wide is it? Set up the perimeter equation: 2l + 2w = P Given P = 204 and l = 66, we have: 2(66) + 2w = 204 2w + 132 = 204 Using our [URL='http://www.mathcelebrity.com/1unk.php?num=2w%2B132%3D204&pl=Solve']equation solver,[/URL] we get w = [B]36[/B].

The perimeter of a rectangular field is 220 yd. the length is 30 yd longer than the width. Find the
The perimeter of a rectangular field is 220 yd. the length is 30 yd longer than the width. Find the dimensions We are given the following equations: [LIST=1] [*]220 = 2l + 2w [*]l = w + 30 [/LIST] Plug (1) into (2) 2(w + 30) + 2w = 220 2w + 60 + 2w = 220 Combine like terms: 4w + 60 = 220 [URL='https://www.mathcelebrity.com/1unk.php?num=4w%2B60%3D220&pl=Solve']Plug 4w + 60 = 220 into the search engine[/URL], and we get [B]w = 40[/B]. Now plug w = 40 into equation (2) l = 40 + 30 [B]l = 70[/B]

The perimeter of a rectangular field is 250 yards. If the length of the field is 69 yards, what is
The perimeter of a rectangular field is 250 yards. If the length of the field is 69 yards, what is its width? Set up the rectangle perimeter equation: P = 2l + 2w For l = 69 and P = 250, we have: 250= 2(69) + 2w 250 = 138 + 2w Using our [URL='http://www.mathcelebrity.com/1unk.php?num=2w%2B138%3D250&pl=Solve']equation solver[/URL], we get: [B]w = 56 [/B]

The perimeter of a rectangular field is 300m. If the width of the field is 59m, what is it’s length
The perimeter of a rectangular field is 300m. If the width of the field is 59m, what is it’s length? Set up the perimeter (P) of a rectangle equation given length (l) and width (w): 2l + 2w = P We're given P = 300 and w = 59. Plug these into the perimeter equation: 2l + 2(59) = 300 2l + 118 = 300 [URL='https://www.mathcelebrity.com/1unk.php?num=2l%2B118%3D300&pl=Solve']Typing this equation into our search engine[/URL], we get: [B]l = 91[/B]

The perimeter of a rectangular notecard is 16 inches. The notecard is 5 inches wide. How tall is it?
The perimeter of a rectangular notecard is 16 inches. The notecard is 5 inches wide. How tall is it? Perimeter of a rectangle P is: P = 2l + 2w We have: 2l + 2w = 16 We are given w = 5, so we have: 2l + 2(5) = 16 2l + 10 = 16 [URL='https://www.mathcelebrity.com/1unk.php?num=2l%2B10%3D16&pl=Solve']Plugging this into our equation calculator[/URL], we get [B]l = 3[/B].

The perimeter of a rectangular outdoor patio is 54 ft. The length is 3 ft greater than the width. Wh
The perimeter of a rectangular outdoor patio is 54 ft. The length is 3 ft greater than the width. What are the dimensions of the patio? Perimeter of a rectangle is: P = 2l + 2w We're given l = w + 3 and P = 54. So plug this into our perimeter formula: 54= 2(w + 3) + 2w 54 = 2w + 6 + 2w Combine like terms: 4w + 6 = 54 [URL='https://www.mathcelebrity.com/1unk.php?num=4w%2B6%3D54&pl=Solve']Typing this equation into our search engine[/URL], we get: [B]w = 12[/B] Plug this into our l = w + 3 formula: l = 12 + 3 [B]l = 15[/B]

The perimeter of a rectangular parking lot is 258 meters. If the length of the parking lot is 71, wh
The perimeter of a rectangular parking lot is 258 meters. If the length of the parking lot is 71, what is its width? The perimeter for a rectangle (P) is given as: 2l + 2w = P We're given P = 258 and l = 71. Plug these values in: 2(71) + 2w = 258 142 + 2w = 258 [URL='https://www.mathcelebrity.com/1unk.php?num=142%2B2w%3D258&pl=Solve']Typing this equation into our search engine[/URL], we get: [B]w = 58[/B]

The perimeter of a rectangular shelf is 60 inches. The shelf is 7 inches deep. How wide is it?
The perimeter of a rectangular shelf is 60 inches. The shelf is 7 inches deep. How wide is it? The perimeter for a rectangle is given below: P = 2l + 2w We're given l = 7 and P = 60. Plug this into the perimeter formula: 60 = 2(7) + 2w 60 = 14 + 2w Rewritten, it's 2w + 14 = 60. [URL='https://www.mathcelebrity.com/1unk.php?num=2w%2B14%3D60&pl=Solve']Typing this equation into our search engine[/URL], we get [B]w = 23[/B].

The perimeter of a square with side a
The perimeter of a square with side a Perimeter of a square is 4s where s is the side length. With s = a, we have: P = [B]4a[/B]

The perpendicular height of a right-angled triangle is 70 mm longer than the base. Find the perimete
The perpendicular height of a right-angled triangle is 70 mm longer than the base. Find the perimeter of the triangle if its area is 3000. [LIST] [*]h = b + 70 [*]A = 1/2bh = 3000 [/LIST] Substitute the height equation into the area equation 1/2b(b + 70) = 3000 Multiply each side by 2 b^2 + 70b = 6000 Subtract 6000 from each side: b^2 + 70b - 6000 = 0 Using our [URL='http://www.mathcelebrity.com/quadratic.php?num=b%5E2%2B70b-6000%3D0&pl=Solve+Quadratic+Equation&hintnum=+0']quadratic calculator[/URL], we get: b = 50 and b = -120 Since the base cannot be negative, we use b = 50. If b = 50, then h = 50 + 70 = 120 The perimeter is b + h + hypotenuse Using the [URL='http://www.mathcelebrity.com/righttriangle.php?angle_a=&a=70&angle_b=&b=50&c=&pl=Calculate+Right+Triangle']right-triangle calculator[/URL], we get hypotenuse = 86.02 Adding up all 3 for the perimeter: 50 + 70 + 86.02 = [B]206.02[/B]

The phone company charges Rachel 12 cents per minute for her long distance calls. A discount company
The phone company charges Rachel 12 cents per minute for her long distance calls. A discount company called Rachel and offered her long distance service for 1/2 cent per minute, but will charge a $46 monthly fee. How many minutes per month must Rachel talk on the phone to make the discount a better deal? Minutes Rachel talks = m Current plan cost = 0.12m New plan cost = 0.005m + 46 Set new plan equal to current plan: 0.005m + 46 = 0.12m Solve for [I]m[/I] in the equation 0.005m + 46 = 0.12m [SIZE=5][B]Step 1: Group variables:[/B][/SIZE] We need to group our variables 0.005m and 0.12m. To do that, we subtract 0.12m from both sides 0.005m + 46 - 0.12m = 0.12m - 0.12m [SIZE=5][B]Step 2: Cancel 0.12m on the right side:[/B][/SIZE] -0.115m + 46 = 0 [SIZE=5][B]Step 3: Group constants:[/B][/SIZE] We need to group our constants 46 and 0. To do that, we subtract 46 from both sides -0.115m + 46 - 46 = 0 - 46 [SIZE=5][B]Step 4: Cancel 46 on the left side:[/B][/SIZE] -0.115m = -46 [SIZE=5][B]Step 5: Divide each side of the equation by -0.115[/B][/SIZE] -0.115m/-0.115 = -46/-0.115 m = [B]400 She must talk over 400 minutes for the new plan to be a better deal [URL='https://www.mathcelebrity.com/1unk.php?num=0.005m%2B46%3D0.12m&pl=Solve']Source[/URL][/B]

The pieces of a 500 piece puzzle are stored in three containers. 220 pieces are in the first contain
The pieces of a 500 piece puzzle are stored in three containers. 220 pieces are in the first container and 180 pieces are in the second container. What percentage of the pieces is in the third container? [U]Calculate the number of pieces in the 3rd container:[/U] Pieces in container 3 = Total Puzzle Pieces - Pieces in container 2 - Pieces in container 1 Pieces in container 3 = 500 - 220 - 180 Pieces in container 3 = 100 Calculate the percentage of pieces in the 3rd container: Percentage of pieces in container 3 = 100% * Pieces in container 3 / Total puzzle pieces Percentage of pieces in container 3 = 100% * 100 / 500 Percentage of pieces in container 3 = 100% * 0.2 Percentage of pieces in container 3 = [B]20%[/B]

The place value of 3 in 16.534 is
The place value of 3 in 16.534 is We [URL='https://www.mathcelebrity.com/placevalue.php?num=16.534&pl=Show+Place+Value']type in 16.534 into our search engine, choose place value[/URL], and we get: 3 is the [B]hundredths digit[/B]

The planets in the solar system as set G
The planets in the solar system as set G. Since Pluto was removed as a planet, we have the following set G with 8 elements: G = [B]{Mercury, Venus, Earth, Mars, Jupiter, Saturn, Uranus, Neptune}[/B]

THE PLAYER CHOSE 20 OUT OF 70 NUMBERS IN A GAME OF CHANCE. ...WHEN THE SHOW BEGIN,THE BANKER WILL
THE PLAYER CHOSE 20 OUT OF 70 NUMBERS IN A GAME OF CHANCE. ...WHEN THE SHOW BEGIN,THE BANKER WILL THEN RAFFLE OR DO A DRAW WHERE IN THE BANKER PICKS AS WELL 20 OUT OF 70 NUMBERS. .....NOW HERES THE TRICK, FOR YOU TO BEAT THE BANKER .YOUR CHOSEN 20 NUMBERS SHOULD NOT MATCH ANY OF THE BANKER 20 0UT OF 70 NUMBERS THAT HAD BEEN DRAWS IN THE GAME OF SHOW. IF THE 20 NUMBERS YOU HAVE ARE TOTALLY DIFFERENT FROM THE BANKERS 20 NUMBERS DRAWN THEN YOU WIN THE PRICE. Banker Draw Numbers not matching Total numbers Probability Probability Decimal Cumulative Probability 1 50 70 50/70 0.7142857143 0.7142857143 2 49 69 49/69 0.7101449275 0.5072463768 3 48 68 48/68 0.7058823529 0.358056266 4 47 67 47/67 0.7014925373 0.2511737985 5 46 66 46/66 0.696969697 0.1750605262 6 45 65 45/65 0.6923076923 0.1211957489 7 44 64 44/64 0.6875 0.0833220774 8 43 63 43/63 0.6825396825 0.05687062425 9 42 62 42/62 0.6774193548 0.03852526159 10 41 61 41/61 0.6721311475 0.02589402828 11 40 60 40/60 0.6666666667 0.01726268552 12 39 59 39/59 0.6610169492 0.01141092772 13 38 58 38/58 0.6551724138 0.007476125057 14 37 57 37/57 0.649122807 0.004852923282 15 36 56 36/56 0.6428571429 0.003119736396 16 35 55 35/55 0.6363636364 0.001985286797 17 34 54 34/54 0.6296296296 0.001249995391 18 33 53 33/53 0.6226415094 0.000778299017 19 32 52 32/52 0.6153846154 0.0004789532412 20 31 51 31/51 0.6078431373 [B]0.0002911284407 [/B]

The plumber added an extra $35 to her bill what is the algebraic phrase
The plumber added an extra $35 to her bill what is the algebraic phrase Added $35 to a bill (b) is: [B]b + 35[/B]

The point (1,5) is a solution to the equation 2y - x = 9
The point (1,5) is a solution to the equation 2y - x = 9 [B]Yes[/B], because: 2(5) - 1 ? 9 10 - 1 ? 9 9 = 9

The points -5, -24 and 5,r lie on a line with slope 4. Find the missing coordinate r. Slope = (y2 -
The points -5, -24 and 5,r lie on a line with slope 4. Find the missing coordinate r. Slope = (y2 - y1)/(x2 - x1) Plugging in our numbers, we get: 4 = (r - -24)/(5 - -5) 4 = (r +24)/10 Cross multiply: r + 24 = 40 Subtract 24 from each side: [B]r = 16[/B]

The points 6,4 and 9,r lie on a line with slope 3. Find the missing coordinate r.
The points 6,4 and 9,r lie on a line with slope 3. Find the missing coordinate r. Slope = (y2 - y1)/(x2 - x1) Plugging in our numbers, we get: 3 = (r - 4)/(9 - 6) 3 = (r - 4)/3 Cross multiply: r - 4 = 9 Add 4 to each side: [B]r = 13[/B]

The polynomial function P(x) = 75x - 87,000 models the relationship between the number of computer
The polynomial function P(x) = 75x - 87,000 models the relationship between the number of computer briefcases x that a company sells and the profit the company makes, P(x). Find P (4000), the profit from selling 4000 computer briefcases. Plug in 4,000 for x: P(4000) = 75(4000) - 87,000 P(4000) = 300,000- 87,000 P(4000) = [B]213,000[/B]

The population of a town doubles every 12 years. If the population in 1945 was 11,005 people, what w
The population of a town doubles every 12 years. If the population in 1945 was 11,005 people, what was the population in 1981? Calculate the difference in years: Difference = 1981 - 1945 Difference = 36 Calculate doubling periods: Doubling periods = Total years / Doubling time Doubling periods = 36/12 Doubling periods = 3 Population = Initial Population * 2^doubling periods Population = 11005 * 2^3 Population = 11005 * 8 Population = [B]88,040[/B]

The population of a town is currently 22,000. This represents an increase of 40% from the population
The population of a town is currently 22,000. This represents an increase of 40% from the population 5 years ago. Find the population of the town 5 years ago. Round to the nearest whole number if necessary. To get the population 5 years ago, we'd discount the current population of 22,000 by 40%. We can write a 40% discount as 1.4. Population 5 years ago = 22,000/1.4 Population 5 years ago = 15,714.29 Rounding to the nearest whole number, we get [B]15,714[/B]

The population of goats on a particular nature reserve t years after the initial population was sett
The population of goats on a particular nature reserve t years after the initial population was settled is modeled by p(t) = 4000 - 3000e^-0.2t. How many goats were initially present? [U]Initially present means at time 0. Substituting t = 0, p(0), we get:[/U] p(0) = 4000 - 3000e^-0.2(0) p(0) = 4000 - 3000e^0 p(0) = 4000 - 3000(1) p(0) = 4000 - 3000 [B]p(0) = 1000[/B]

The population of Kansas is two-fifths the population of California
The population of Kansas is two-fifths the population of California Assumptions: [LIST] [*]Let p be the population of Kansas [*]Let c be the population of California [/LIST] We have: [B]k = 2c/5[/B]

The population of Westport was 43,000 at the beginning of 1980 and has steadily decreased by 1% per
The population of Westport was 43,000 at the beginning of 1980 and has steadily decreased by 1% per year since. Write an expression that shows the population of Westport at the beginning of 1994 and solve. 1994 - 1980 = 14 years. Using our [URL='https://www.mathcelebrity.com/population-growth-calculator.php?num=thepopulationofwestportwas43000hassteadilydecreasedby1%for14years&pl=Calculate']population calculator[/URL], we get: [B]37,356[/B]

The price of a baseball glove is no more than $38.95
The price of a baseball glove is no more than $38.95. Let p be the price of the baseball glove. The phrase "no more than" means less than or equal to. Our inequality is: p <= $38.95

The price of a cheap backpack is $15 less than an expensive backpack. When Emily bought both, she pa
The price of a cheap backpack is $15 less than an expensive backpack. When Emily bought both, she paid $75. What is the cost of the cheap backpack? backpack cost = b Cheap backpack = b - 15 The total of both items equals 75: b + b - 15 = 75 Solve for [I]b[/I] in the equation b + b - 15 = 75 [SIZE=5][B]Step 1: Group the b terms on the left hand side:[/B][/SIZE] (1 + 1)b = 2b [SIZE=5][B]Step 2: Form modified equation[/B][/SIZE] 2b - 15 = + 75 [SIZE=5][B]Step 3: Group constants:[/B][/SIZE] We need to group our constants -15 and 75. To do that, we add 15 to both sides 2b - 15 + 15 = 75 + 15 [SIZE=5][B]Step 4: Cancel 15 on the left side:[/B][/SIZE] 2b = 90 [SIZE=5][B]Step 5: Divide each side of the equation by 2[/B][/SIZE] 2b/2 = 90/2 b = 45 Cheap backpack = 45 - 15 = [B]30 [URL='https://www.mathcelebrity.com/1unk.php?num=b%2Bb-15%3D75&pl=Solve']Source[/URL][/B]

The price of a gallon of gasoline is $3.15. The price when Ryan’s mother started driving was 1/7 of
The price of a gallon of gasoline is $3.15. The price when Ryan’s mother started driving was 1/7 of the current price. What was the price of gasoline when Ryan’s mother started driving? $3.15/7 = [B]$0.45[/B]

the price of a remote control helicopeter is $34.40. a remote control boat costs 4/5 the price of th
the price of a remote control helicopter is $34.40. a remote control boat costs 4/5 the price of the helicopter. sales tax on the remote control boat is 8%.what is the price of the remote control boat, including sales tax? round your answer to the nearest penny 4/5 of 34.40 = $27.52 Add sales tax: 27.52(1.08) = [B]$29.72[/B]

The price p of a gym’s membership is $30 for an enrollment fee and $12 per week w to be a member. W
The price p of a gym’s membership is $30 for an enrollment fee and $12 per week w to be a member. What is the cost to be a member for 5 weeks? Set up the cost function C(w) C(w) = 12w + 30 The problem asks for C(5) C(5) = 12(5) + 30 C(5) = 60 + 30 C(5) = [B]90[/B]

the price p per gallon g
the price p per gallon g Price per gallon = Price / Gallons Price per gallon = [B]p/g[/B]

The principal randomly selected six students to take an aptitude test. Their scores were: 87.4 86.9
The principal randomly selected six students to take an aptitude test. Their scores were: 87.4 86.9 89.9 78.3 75.1 70.6 Determine a 90% confidence interval for the mean score for all students.

The principal randomly selected six students to take an aptitude test. Their scores were: 87.4 86.9
First, determine the [URL='http://www.mathcelebrity.com/statbasic.php?num1=87.4%2C86.9%2C89.9%2C78.3%2C75.1%2C70.6&num2=+0.2%2C0.4%2C0.6%2C0.8%2C0.9&pl=Number+Set+Basics']mean and standard deviation[/URL] for the [I]sample[/I] Mean = 81.3667 SD = 7.803 Next, use our [URL='http://www.mathcelebrity.com/normconf.php?n=6&xbar=81.3667&stdev=7.803&conf=90&rdig=4&pl=Small+Sample']confidence interval for the mean calculator[/URL] with these values and n = 6 [B]74.9478 < u < 87.7856[/B]

The probability of failing to reject a false null hypothesis is ____
The probability of failing to reject a false null hypothesis is ____ a. ? b. 1 - ? c. 1 - ? d. ? [B]d. ?[/B]

The product 18 And q
The product 18 And q [B]18q[/B]

the product of 2 less than a number and 7 is 13
the product of 2 less than a number and 7 is 13 Take this algebraic expression in [U]4 parts[/U]: Part 1 - The phrase [I]a number[/I] means an arbitrary variable, let's call it x. x Part 2 - 2 less than a number means we subtract 2 from x x - 2 Part 3 - The phrase [I]product[/I] means we multiply x - 2 by 7 7(x - 2) Part 4 - The phrase [I]is[/I] means an equation, so we set 7(x - 2) equal to 13 [B]7(x - 2) = 13[/B]

the product of 3 and the sum of m and 2n
the product of 3 and the sum of m and 2n The sum of m and 2n means we add 2n to m: m + 2n The product of 3 means we multiply the sum m + 2n by 3: [B]3(m + 2n)[/B]

the product of 8 and 15 more than a number
the product of 8 and 15 more than a number. Take this algebraic expression in pieces. The phrase [I]a number[/I] means an arbitrary variable, let's call it x. x 15 more than x means we add 15 to x: x + 15 The product of 8 and 15 more than a number means we multiply 8 by x + 15 [B]8(x + 15)[/B]

The product of 8 and a number k is greater than 4 and no more than 16
Let's take this by pieces. The product of 8 and a number k is written as: 8k. Since it's greater than 4, but not more than 16, we include this in the middle of an inequality statement. 4 < 8k <= 16 Notice no more than has an equal sign, it means less than or equal to. Greater does not include an equal sign.

the product of a number and 15 is not less than 15
the product of a number and 15 is not less than 15 The phrase [I]a number[/I] means an arbitrary variable. Let's call it x. x the product of a number and 15 means we multiply x by 15 15x The phrase [I]not less than[/I] means greater than or equal to. We set 15x greater than prequel to 15 [B]15x >= 15 <-- This is our algebraic expression [/B] [U]If the problem asks you to solve for x:[/U] Divide each side by 15: 15x/15 >= 15/15 [B]x >= 1[/B]

The product of a number and its square is less than 8
Let the number be x. Let the square be x^2. So we have (x)(x^2) = x^3 < 8 Take the cube root of this, we get x = 2

The product of a number b and 3 is no less than 12.
The product of a number b and 3 is no less than 12. A number b is just written as b. So we have: The product of b and 3 is no less than 12. take this in parts: [LIST] [*]The product of b and 3: 3b [*]The phrase [I]is no less than[/I] means an inequality, so we have greater than or equal to. We set 3b greater than or equal to 12 [/LIST] [B]3b >= 12[/B]

the product of k and 70, minus 15
the product of k and 70, minus 15 Take this algebraic expression in pieces: The product of k and 70 means we multiply 70 times k 70k The word [I]minus[/I] means we subtract 15 from 70k [B]70k - 15[/B]

The product of the 2 numbers x and y
The product of the 2 numbers x and y The phrase [I]product [/I]means we multiply the two variables, x and y. [B]xy[/B]

The product of two numbers less than 20 ( let a and b are the numbers)
The product means we multiply the two numbers: [B]ab < 20[/B]

The product of two positive numbers is 96. Determine the two numbers if one is 4 more than the other
The product of two positive numbers is 96. Determine the two numbers if one is 4 more than the other. Let the 2 numbers be x and y. We have: [LIST=1] [*]xy = 96 [*]x = y - 4 [/LIST] [U]Substitute (2) into (1)[/U] (y - 4)y = 96 y^2 - 4y = 96 [U]Subtract 96 from both sides:[/U] y^2 - 4y - 96 = 0 [U]Factoring using our quadratic calculator, we get:[/U] (y - 12)(y + 8) So y = 12 and y = -8. Since the problem states positive numbers, we use [B]y = 12[/B]. Substituting y = 12 into (2), we get: x = 12 - 4 [B]x = 8[/B] [B]We have (x, y) = (8, 12)[/B]

The product of x and 7 is not greater than 21
The product of x and 7 is not greater than 21 The product of x and 7: 7x Is not greater than means less than or equal to, so we have our algebraic expression: 7x <= 21 If you want to solve this inequality and interval notation, use our [URL='http://www.mathcelebrity.com/interval-notation-calculator.php?num=7x%3C%3D21&pl=Show+Interval+Notation']calculator[/URL].

The product of x and u is not greater than 21
The product of x and u is not greater than 21 The product of x and u xu Not greater than means less than or equal to: xu <= 21

The property taxes on a boat were $375. What was the tax rate if the boat was valued at $75,000?
The property taxes on a boat were $375. What was the tax rate if the boat was valued at $75,000? Tax Rate = Tax Amount / Purchase Price Tax Rate = 375 / 75,000 Tax Rate = 0.005 Tax Rates are generally expressed in percentages, so the percentage = 0.005 * 100 = [B]0.5%[/B].

The property taxes on a business office were $960. What was the tax rate if the business office was
The property taxes on a business office were $960. What was the tax rate if the business office was valued at $80,000? Tax Rate = 100% * Tax Amount / Office Value Tax Rate = 100% * 960 / 80000 Tax Rate = 100% * 0.012 Tax Rate = [B]1.2%[/B]

The property taxes on a house were $810. What was the tax rate if the house was valued at $90,000
The property taxes on a house were $810. What was the tax rate if the house was valued at $90,000 Tax rate = Property Tax Amount/House Value Tax rate = 810/90000 [B]Tax Rate = 0.009, or as a percentage, 0.9%[/B]

The quantity x minus y divided by 4
The quantity x minus y divided by 4 The quantity x minus y x - y The quantity x minus y divided by 4 [B](x - y)/4[/B]

The quotient of 2 and the sum of a number and 1
The quotient of 2 and the sum of a number and 1. The phrase [I]a number[/I] represents an arbitrary variable, let's call it x. The sum of a number and 1 is written as: x + 1 The word [I]quotient[/I] means a fraction. So we divide 2 by x + 1 2 -------- ( x + 1)

the quotient of 3 and u is equal to 52 divided by u
the quotient of 3 and u is equal to 52 divided by u Take this algebraic expression in 3 parts: [LIST=1] [*]The quotient of 3 and u means we divide 3 by u: 3/u [*]52 divided by u means we divide 52 by u: 52/u [*]The phrase [I]is equal to[/I] means an equation, so we set (1) equal to (2) [/LIST] [B]3/u = 52/u[/B]

The quotient of 3 plus y and 12 minus x
The quotient of 3 plus y and 12 minus x 3 plus y: 3 + y 12 minus x 12 - x The quotient of 3 plus y and 12 minus x: [B](3 + y)/(12 - x)[/B]

the quotient of 4 more than a number and 7 is 10
the quotient of 4 more than a number and 7 is 10 Take this algebraic expression in pieces: The phrase [I]a number[/I] means an arbitrary variable, let's call it x. x 4 more than a number means we add 4 to x: x + 4 The quotient of 4 more than a number and 7 means we divide x + 4 by 7 (x + 4)/7 The word [I]is[/I] means an equation, so we set (x + 4)/7 equal to 10 [B](x + 4)/7 = 10[/B]

The quotient of 49 and n squared
n squared is written as n to the power of 2, n^2 We have a fraction, where 49 is the numerator, and n^2 is the denominator 49 ----- n^2

the quotient of 7 and the total of 5 and x
the quotient of 7 and the total of 5 and x The total of 5 and x 5 + x the quotient of 7 and the total of 5 and x [B]7/(5 + x)[/B]

the quotient of 77 and x
the quotient of 77 and x Quotient means we have a fraction where 77 is the numerator and x is the denominator: [B]77/x[/B]

the quotient of 8 and the difference of x and m
The difference of x and m means we subtract: x - m Quotient means a fraction. 8 is the numerator, and x - m is the denominator: [B] 8 ------ x - m[/B]

The quotient of 9-x and twice x
The quotient of 9-x and twice x Twice x means we multiply x by 2: 2x The quotient of 9 - x and twice x is formed by the fraction: [B](9 - x)/2x[/B]

the quotient of a number and twice another number
the quotient of a number and twice another number The phrase[I] a number [/I]means an arbitrary variable, let's call it x. The phrase[I] another number [/I]means another arbitrary variable, let's call it y. Twice means we multiply y by 2:2y The quotient means we divide x by 2y: [B]x/2y[/B]

the quotient of a variable and 7
the quotient of a variable and 7. A variable means an arbitrary number, let's call it x. A quotient means a fraction, where x is the numerator and 7 is the denominator: [B] x --- 7[/B]

the quotient of c and d diminished by 5y
the quotient of c and d diminished by 5y the quotient of c and d c/d the quotient of c and d diminished by 5y [B]c/d - 5y[/B]

the quotient of d and 182 is the same as w minus 137
The quotient of d and 182 is the same as w minus 137 Take this algebraic expression in 3 parts: The quotient of d and 182 d/182 w minus 137 w - 137 The phrase [I]is the same as[/I] means we set d/182 equal to w - 137 [B]d/182 = w - 137[/B]

The quotient of m and -2 is greater than 24
We write the quotient: m/-2, but move the negative sign to the top. -m/2 Next, greater than 24 means we use the > sign -m/2 > 24

the quotient of m and the sum of n and p.
the quotient of m and the sum of n and p. The sum of n and p means we add p to n: n + p The quotient means a fraction, so we divide m by (n + p) [B]m/(n + p)[/B]

the quotient of m squared and a squared
the quotient of m squared and a squared [U]m squared means we raise m to the power of 2:[/U] m^2 [U]a squared means we raise a to the power of 2:[/U] a^2 [U]The [I]quotient[/I] means we divide m^2 by a^2:[/U] [B]m^2/a^2[/B]

The quotient of t and 12 is the sum of s and r.
The quotient of t and 12 is the sum of s and r. Step 1: The quotient of t and 12 is: t/12 Step 2: The Sum of s and r is s + r Step 3: The word [I]is[/I] means equal to, so we set t/12 equal to s + r [B]t/12 = s + r[/B]

the quotient of the cube of a number x and 5
the quotient of the cube of a number x and 5 [LIST] [*]A number means an arbitrary variable, let's call it x [*]The cube of a number means raise it to the 3rd power, so we have x^3 [*]Quotient means we have a fraction, so our numerator is x^3, and our denominator is 5 [/LIST] [B]x^3 ---- 5[/B]

The quotient of the quantity of x plus y and 3
The quotient of the quantity of x plus y and 3 Quantity x plus y x + y Quotient of this and 3 [B](x + y)/3[/B]

the quotient of the sum and difference of c and d
the quotient of the sum and difference of c and d The sum of c and d: c + d The difference of c and d: c - d the quotient of the sum and difference of c and d [B](c + d)/(c - d)[/B]

the quotient of triple m and n squared
the quotient of triple m and n squared Triple m means we multiply m by 3: 3m n squared means we raise n to the 2nd power: n^2 The quotient is formed as follows: [B]3m/n^2[/B]

The quotient of x and 5 decreased by 7
The quotient of x and 5 decreased by 7 The quotient of x and 5 x/5 Decreased by 7 [B]x/5 - 7[/B]

the quotient of x and y is equal to the sum of a and b
the quotient of x and y is equal to the sum of a and b The quotient of x and y: x/y The sum of a and b: a + b The phrase [I]is equal to[/I] means an equation, so we set x/y equal to a + b: [B]x/y = a + b[/B]

The Radio City Music Hall is selling tickets to Kayla’s premiere at the Rockettes. On the first day
The Radio City Music Hall is selling tickets to Kayla’s premiere at the Rockettes. On the first day of ticket sales they sold 3 senior citizen tickets and 9 child tickets for a total of $75. It took in $67 on the second day by selling 8 senior citizen tickets and 5 child tickets. What is the price of each senior citizen ticket and each child ticket? Let the cost of child tickets be c Let the cost of senior tickets be s Since revenue = cost * quantity, we're given two equations: [LIST=1] [*]9c + 3s = 75 [*]5c + 8s = 67 [/LIST] To solve this simultaneous group of equations, we can use 3 methods: [LIST] [*][URL='https://www.mathcelebrity.com/simultaneous-equations.php?term1=9c+%2B+3s+%3D+75&term2=5c+%2B+8s+%3D+67&pl=Substitution']Substitution Method[/URL] [*][URL='https://www.mathcelebrity.com/simultaneous-equations.php?term1=9c+%2B+3s+%3D+75&term2=5c+%2B+8s+%3D+67&pl=Elimination']Elimination Method[/URL] [*][URL='https://www.mathcelebrity.com/simultaneous-equations.php?term1=9c+%2B+3s+%3D+75&term2=5c+%2B+8s+%3D+67&pl=Cramers+Method']Cramer's Rule[/URL] [/LIST] No matter which method we use, we get the same answer: [LIST] [*][B]c = 7[/B] [*][B]s = 4[/B] [/LIST]

The ratio between the sum of a and b and the difference of a and b is equal to 5.
The ratio between the sum of a and b and the difference of a and b is equal to 5. The sum of a and b: a + b The difference of a and b: a - b The ratio between the sum of a and b and the difference of a and b (a + b)/(a - b) The ratio between the sum of a and b and the difference of a and b is equal to 5. [B](a + b)/(a - b) = 5[/B]

the ratio of 50 and a number added to the quotient of a number and 10
the ratio of 50 and a number added to the quotient of a number and 10 Take this algebraic expression in parts. The phrase [I]a number[/I] means an arbitrary variable, let's call it x. x The ratio of 50 and x means we divide by 50 by x 50/x The quotient of a number and 10 means we have a fraction: x/10 The phrase [I]added to[/I] means we add 50/x to x/10 [B]50/x + x/10[/B]

the ratio of a number x and 4 added to 2
the ratio of a number x and 4 added to 2 Take this algebraic expression in parts. The phrase [I]a number[/I] means an arbitrary variable, let's call it x. x The ratio of this number and 4 means we have a fraction: x/4 The phrase [I]added to[/I] means we add 2 to x/4 [B]x/4 + 2[/B]

The ratio of adults to children at the beach is 4:3. If there are a total of 56 people how many are
The ratio of adults to children at the beach is 4:3. If there are a total of 56 people how many are adults? How many are children? Using our [URL='http://www.mathcelebrity.com/ratio.php?simpratio=100%3A350&rs=4%3A3&rtot=56&pl=Calculate+Ratio']ratio calculator[/URL], we get: [LIST] [*][B]32 adults[/B] [*][B]24 children[/B] [/LIST]

The ratio of girls to boys is 14 girls to 12 boys. If there are 6 boys, how many girls are there?
The ratio of girls to boys is 14 girls to 12 boys. If there are 6 boys, how many girls are there? Set up a proportion of girls to boys: 14/12 = g/6 Using our [URL='http://www.mathcelebrity.com/prop.php?num1=14&num2=g&den1=12&den2=6&propsign=%3D&pl=Calculate+missing+proportion+value']proportion calculator[/URL]: [B]g = 7[/B]

The ratio of men to women working for a company is 5 to 3 . If there are 75 men working for the
The ratio of men to women working for a company is 5 to 3 . If there are 75 men working for the company, what is the total number of employees? We read this as a proportion, of men to women. 5/3 = 75/w where w is the number of women for 75 men. Entering this expression into our [URL='http://www.mathcelebrity.com/prop.php?num1=5&num2=75&propsign=%3D&den1=3&den2=w&pl=Calculate+missing+proportion+value']proportion calculator[/URL], we get [B]w = 45[/B].

The ratio of men to women working for a company is 3 to 4. If there are 81 men working for the compa
The ratio of men to women working for a company is 3 to 4. If there are 81 men working for the company, what is the total number of employees? Men to women is 3:4. Set up a proportion where w is the number of women: 3/4 = 81/w Using our [URL='http://www.mathcelebrity.com/prop.php?num1=3&num2=81&den1=4&den2=w&propsign=%3D&pl=Calculate+missing+proportion+value']proportion calculator[/URL], we get w = 108. The problem asks for total employees, so we add men and women: Total Employees = Men + Women Total Employees = 81 + 108 Total Employees = [B]189[/B]

The ratio of men to women working for a company is 4 to 7. If there are 319 employees total, how man
The ratio of men to women working for a company is 4 to 7. If there are 319 employees total, how many men work for the company? [B]116[/B] using our [URL='http://www.mathcelebrity.com/ratio.php?simpratio=100%3A350&rs=4%3A7&rtot=319&pl=Calculate+Ratio']ratio calculator[/URL]

the ratio of ten to a number
the ratio of ten to a number The phrase [I]a number[/I] means an arbitrary variable, let's call it x. The ratio of 10 and this number x is written as: [B]10/x[/B]

The ratio of the measures of the 3 angles of a triangle is 1:2:3. What is the measure of the largest
The ratio of the measures of the 3 angles of a triangle is 1:2:3. What is the measure of the largest angle in degrees? Let the smallest angle be x. Then we have 3 angles based on the ratio: x, 2x, 3x We know the sum of the angles of a triangle equals 180. So we have: x + 2x + 3x = 180 6x = 180 Divide each side by 6: 6x/6 = 180/6 x = 30 The largest angle is 3(30) = [B]90 [MEDIA=youtube]l8Lc6YtK9dg[/MEDIA][/B]

The ratio of the number of carabaos, goats, and cows in a farm is 5:1:2. If there are 48 animals of
The ratio of the number of carabaos, goats, and cows in a farm is 5:1:2. If there are 48 animals of these kinds in his backyard how many of them are goats Calculate total ratio: 5 + 1 + 2 = 8 Multiply fractional portion of goats by total animals in the backyard. 1/8 * 48 = [B]6 goats[/B]

the ratio of twice c to d
the ratio of twice c to d Twice c means we multiply c by 2: 2c The ratio is formed by the quotient: [B]2c/d[/B]

the ratio of yellow to red balloons is 2:1 respectively. if there are 7 red balloons, how many yello
the ratio of yellow to red balloons is 2:1 respectively. if there are 7 red balloons, how many yellow balloons are there? 7 red balloons means we have twice as many yellow balloons. So 7 * 2 = [B]14[/B]. Written as a proportion, of yellow to red, we have: 2/1 = y/7 where y is the number of yellow balloons. [URL='https://www.mathcelebrity.com/prop.php?num1=2&num2=y&den1=1&den2=7&propsign=%3D&pl=Calculate+missing+proportion+value']Run this proportion through our search engine[/URL] to get [B]y = 14[/B].

the reciprocal of the product a and b
the reciprocal of the product a and b Take this algebraic expression in pieces: The product a and b means we multiply a times b ab The [I]reciprocal[/I] means we take 1 over ab [B]1/ab[/B]

The recommended daily calcium intake for a 20-year-old is 1,000 milligrams (mg). One cup of milk con
The recommended daily calcium intake for a 20-year-old is 1,000 milligrams (mg). One cup of milk contains 299 mg of calcium and one cup of juice contains 261 mg of calcium. Which of the following inequalities represents the possible number of cups of milk [I]m[/I] and cups of juice [I]j[/I] a 20-year-old could drink in a day to meet or exceed the recommended daily calcium intake from these drinks alone? Total calcium = Milk calcium + Juice Calcium Calculate Milk Calcium: Milk Calcium = 299m where m is the number of cups of milk Calculate Juice Calcium: Juice Calcium = 261j where j is the number of cups of juice The phrase [I]meet or exceed[/I] means greater than or equal to, so we have an inequality, where Total Calcium is greater than or equal to 1000. So we write our inequality as: Milk calcium + Juice Calcium >= Total Calcium [B]299m + 261j >= 1000[/B]

The regular cost of a guitar is n. On Saturdays, all guitars are 15% off. What is the price of the g
The regular cost of a guitar is n. On Saturdays, all guitars are 15% off. What is the price of the guitar on Saturday? 15% = 0.15 If we take that price off, we have: 1 - 0.15 = 0.85 So the cost is [B]0.85n[/B]

The regular price for a television is Q dollars. Each Saturday televisions are 20% off (The discount
The regular price for a television is Q dollars. Each Saturday televisions are 20% off (The discount is .2Q). What is the price of a television on Saturday in terms of Q? Q = Regular Price .2Q = Discount Discounted Price = Q - .2Q = [B]0.8Q[/B]

The regular price of a shirt was $19.00, but it is on sale for $13.30. What is the percent that the
The regular price of a shirt was $19.00, but it is on sale for $13.30. What is the percent that the shirt has been discounted? Using our [URL='http://www.mathcelebrity.com/markup.php?p1=19&m=&p2=++13.30&pl=Calculate']markdown calculator[/URL], we get a 30% markdown, or sale.

The relief time provided by a standard dose of a popular children’s allergy medicine averages 7.9
The relief time provided by a standard dose of a popular children’s allergy medicine averages 7.9 hours with a standard deviation of 2.2 hours. Use Table 1. a. Determine the percentage of children who experience relief for less than 6.4 hours if the relief time follows a normal distribution. (Round your answer to 2 decimal places.) Using our [URL='http://www.mathcelebrity.com/probnormdist.php?xone=6.4&mean=7.9&stdev=2.2&n=1&pl=P%28X+%3C+Z%29']normal distribution calculator[/URL], we get Answer = [B]0.25[/B]

The rent for an apartment is $6600 per year and increases at a rate of 4% each year. Find the rent o
The rent for an apartment is $6600 per year and increases at a rate of 4% each year. Find the rent of the apartment after 5 years. Round your answer to the nearest penny. Our Rent R(y) where y is the number of years since now is: R(y) = 6600 * (1.04)^y <-- Since 4% is 0.04 The problem asks for R(5): R(5) = 6600 * (1.04)^5 R(5) = 6600 * 1.2166529024 R(5) = [B]8,029.91[/B]

The residents of a city voted on whether to raise property taxes. The ratio of yes votes to no votes
The residents of a city voted on whether to raise property taxes. The ratio of yes votes to no votes was 6 to 5. If there were 4570 no votes, what was the total number of votes? Set up a proportion where y is the number of yes votes to 4570 no votes 6/5 = y/4570 Using our [URL='http://www.mathcelebrity.com/prop.php?num1=6&num2=y&den1=5&den2=4570&propsign=%3D&pl=Calculate+missing+proportion+value']proportion calculator,[/URL] we get: [B]y = 5484[/B]

The residents of a city voted on whether to raise property taxes. The ratio of yes votes to no votes
The residents of a city voted on whether to raise property taxes. The ratio of yes votes to no votes was 4 to 3 . If there were 2958 no votes, what was the total number of votes? Set up a ratio of yes to no votes 4/3 = x/2958 Using our [URL='http://www.mathcelebrity.com/prop.php?num1=4&num2=x&den1=3&den2=2958&propsign=%3D&pl=Calculate+missing+proportion+value']proportion calculator[/URL], we get x = 3,944 for yes votes. Adding yes votes and no votes together to get total votes, we get: Total Votes = Yes Votes + No Votes Total Votes = 3,944 + 2,958 Total Votes = [B]6,902[/B]

the result of increasing n by six
the result of increasing n by six Increasing means we add: [B]n + 6[/B]

the result of quadrupling a number is 80
the result of quadrupling a number is 80 Let our number be x. Quadrupling any number means multiplying it by 4. We have: 4x = 80 [URL='https://www.mathcelebrity.com/1unk.php?num=4x%3D80&pl=Solve']Typing this problem into our search engine[/URL], we get: [B]x = 20[/B]

The revenue for selling x candles is given by f(x)=12x. The teams profit is $40 less than 80% of the
The revenue for selling x candles is given by f(x)=12x. The teams profit is $40 less than 80% of the revenue of selling x candles. write a function g to model the profit. Profit = Revenue - Cost We are given the revenue function f(x) = 12x. We are told the profit is 0.8(Revenue) - 40. Our profit function P(x) is: P(x) = 0.8(12x) - 40 Simplifying, we have: [B]P(x) = 9.6x - 40[/B]

The sale price of an item that is discounted by 20% of its list price L
The sale price of an item that is discounted by 20% of its list price L S = L - 20%/100 * L S = L - 0.20L [B]S = 0.8L[/B]

The sales price of a new compact disc player is $210 at a local discount store. At the store where t
The sales price of a new compact disc player is $210 at a local discount store. At the store where the sale is going on, each new cd is on sale for $11. If Kyle purchases a player and some cds for $243 how many cds did he purchase? If Kyle bought the player, he has 243 - 210 = 33 left over. Each cd is 11, so set up an equation to see how many CDs he bought: 11x = 33 Divide each side by 11 [B]x = 3[/B]

The sales price s of a pair of shoes plus 4% sales tax
The sales price s of a pair of shoes plus 4% sales tax Total price is s(1 + 0.04) or [B]s(1.04)[/B]

The sales tax for an item was $21.50 and it cost $430 before tax. Find the sales tax rate. Write you
The sales tax for an item was $21.50 and it cost $430 before tax. Find the sales tax rate. Write your answer as a percentage. Sales tax percentage is: 21.50/430 = 0.05 To get a percentage, multiply the decimal by 100 0.05 * 100 = [B]5%[/B]

The sales tax on a computer was $33.60. If the sales tax rate is 7%, how much did the computer cost
The sales tax on a computer was $33.60. If the sales tax rate is 7%, how much did the computer cost without tax? Let the cost of the computer be c. We have: 0.07c = 33.60 Solve for [I]c[/I] in the equation 0.07c = 33.60 [SIZE=5][B]Step 1: Divide each side of the equation by 0.07[/B][/SIZE] 0.07c/0.07 = 33.60/0.07 c = $[B]480[/B] [URL='https://www.mathcelebrity.com/1unk.php?num=0.07c%3D33.60&pl=Solve']Source[/URL]

The sales tax rate in a city is 7.27%. How much sales tax is charged on a purchase of 5 headphones
The sales tax rate in a city is 7.27%. How much sales tax is charged on a purchase of 5 headphones at $47.44 each? What is the total price? [U]First, calculate the pre-tax price:[/U] Pre-tax price = Price per headphone * Number of Headphones Pre-tax price = $47.44 * 5 Pre-tax price = $237.20 Now calculate the tax amount: Tax Amount = Pre-Tax Price * (Tax Rate / 100) Tax Amount = $237.20 * 7.27/100 Tax Amount = $237.20 * 0.0727 Tax Amount = [B]$17.24 [/B] Calculate the total price: Total Price = Pre-Tax price + Tax Amount Total Price = $237.20 + $17.24 Total Price = [B]$254.44[/B]

the sample space for a coin being tossed twice
the sample space for a coin being tossed twice Since each toss results in 2 outcomes, we have 2^2 = 4 possible events in the sample space: [LIST=1] [*]H,H [*]H,T [*]T,H [*]T,T [/LIST]

The scale of a map shows that 1/2 inch is equal to 3/4 of a mile. How many inches on a map would be
The scale of a map shows that 1/2 inch is equal to 3/4 of a mile. How many inches on a map would be equal to 3 miles? Set up a proportion of scale to actual distance 1/2 / 3/4 = x/3 4/3 = x/3 Cross multiply: 3x = 12 Divide each side by 3: 3x/3 = 12/3 x = [B]4 (1/2 inch sections) or 2 inches[/B]

The scale on a map is 1 inch = 60 miles. If two cities are 75 miles apart, how far apart are they on
The scale on a map is 1 inch = 60 miles. If two cities are 75 miles apart, how far apart are they on the map? Set up a proportion of inches to miles where n is the number of inches for 75 miles 1 inch/60 miles = n/75 Using our [URL='https://www.mathcelebrity.com/proportion-calculator.php?num1=1&num2=n&den1=60&den2=75&propsign=%3D&pl=Calculate+missing+proportion+value']proportion calculator[/URL], we get: n = [B]1.25 inches[/B]

The school council began the year with a $600 credit to their account, but they spent $2,000 on new
The school council began the year with a $600 credit to their account, but they spent $2,000 on new books for classrooms. How much must the PTA earn through fundraising to break even? +600 - 2000 = -1,400. Break even means no profit or loss. So the PTA must earn [B]1,400 [/B]to break even on the -1,400

The school is selling potted plants as a fundraiser. Kara sold 12 ferns and 8 ivy plants for 260.00.
The school is selling potted plants as a fundraiser. Kara sold 12 ferns and 8 ivy plants for 260.00. Paul sold 15 ivy plants and 6 ferns for 240. What’s the selling price of each plant. Let the cost of each fern be f Let the cost of each ivy plant be I We're given: [LIST=1] [*]12f + 8i = 260 [*]15i + 6f = 240 [/LIST] To solve this system of equations, we can use 3 methods: [LIST] [*][URL='https://www.mathcelebrity.com/simultaneous-equations.php?term1=12f+%2B+8i+%3D+260&term2=15f+%2B+6i+%3D+240&pl=Substitution']Substitution Method[/URL] [*][URL='https://www.mathcelebrity.com/simultaneous-equations.php?term1=12f+%2B+8i+%3D+260&term2=15f+%2B+6i+%3D+240&pl=Elimination']Elimination Method[/URL] [*][URL='https://www.mathcelebrity.com/simultaneous-equations.php?term1=12f+%2B+8i+%3D+260&term2=15f+%2B+6i+%3D+240&pl=Cramers+Method']Cramer's Rule[/URL] [/LIST] No matter which method we choose, we get the same answer: [LIST] [*][B]f = 7.5[/B] [*][B]i= 21.25[/B] [/LIST]

The school yearbook costs $15 per book to produce with an overhead of $5500. The yearbook sells for
The school yearbook costs $15 per book to produce with an overhead of $5500. The yearbook sells for $40. Write a cost and revenue function and determine the break-even point. [U]Calculate cost function C(b) with b as the number of books:[/U] C(b) = Cost per book * b + Overhead [B]C(b) = 15b + 5500[/B] [U]Calculate Revenue Function R(b) with b as the number of books:[/U] R(b) = Sales Price per book * b [B]R(b) = 40b[/B] [U]Calculate break even function E(b):[/U] Break-even Point = Revenue - Cost Break-even Point = R(b) - C(b) Break-even Point = 40b - 15b - 5500 Break-even Point = 25b - 5500 [U]Calculate break even point:[/U] Break-even point is where E(b) = 0. So we set 25b - 5500 equal to 0 25b - 5500 = 0 To solve for b, we [URL='https://www.mathcelebrity.com/1unk.php?num=25b-5500%3D0&pl=Solve']type this equation into our search engine[/URL] and we get: [B]b = 220[/B]

The science club charges 4.50 per car at their car wash. Write and solve and inequality to find how
The science club charges 4.50 per car at their car wash. Write and solve and inequality to find how many cars they have to wash to earn at least 300 Let x be the number of cars they wash. Set up our inequality. Note, at least 300 means 300 or greater, so we use greater than or equal to. [U]Inequality:[/U] [B]4.50x >= 300 [/B] [U]So solve for x, divide each side by 4[/U] [B]x >= 66.67[/B]

The senior class at high school A and high school B planned separate trips to the state fair. There
The senior class at high school A and high school B planned separate trips to the state fair. There senior class and high school A rented and filled 10 vans and 6 buses with 276 students. High school B rented and filled 5 vans and 2 buses with 117 students. Every van had the same number of students in them as did the buses. How many students can a van carry?? How many students can a bus carry?? Let b be the number of students a bus can carry. Let v be the number of students a van can carry. We're given: [LIST=1] [*]High School A: 10v + 6b = 276 [*]High School B: 5v + 2b = 117 [/LIST] We have a system of equations. We can solve this 3 ways: [LIST] [*][URL='https://www.mathcelebrity.com/simultaneous-equations.php?term1=10v+%2B+6b+%3D+276&term2=5v+%2B+2b+%3D+117&pl=Substitution']Substitution Method[/URL] [*][URL='https://www.mathcelebrity.com/simultaneous-equations.php?term1=10v+%2B+6b+%3D+276&term2=5v+%2B+2b+%3D+117&pl=Elimination']Elimination Method[/URL] [*][URL='https://www.mathcelebrity.com/simultaneous-equations.php?term1=10v+%2B+6b+%3D+276&term2=5v+%2B+2b+%3D+117&pl=Cramers+Method']Cramer's Rule[/URL] [/LIST] No matter which method we choose, we get: [LIST] [*][B]b = 21[/B] [*][B]v = 15[/B] [/LIST]

The set of all letters in the word p lus is
The set of all letters in the word p lus is The cardinality of this set is 4 with the elements below: [B]{p, l, u, s}[/B]

The set of all letters in the word true is
The set of all letters in the word true is: We have [B]{t, r, u, e}[/B]

The set of all odd numbers between 10 and 30
The set of all odd numbers between 10 and 30 [B]{11, 13, 15, 17, 19, 21, 23, 25, 27, 29}[/B]

The set of months of a year ending with the letters “ber”.
The set of months of a year ending with the letters “ber”. We build set S below: [B]S = {September, October, November, December}[/B] The cardinality of S, denoted |S|, is the number of items in S: [B]|S| = 4[/B]

The set of months that contain less than 30 days
The set of months that contain less than 30 days. Let M be the set. Only February has less than 30 days out of the 12 months. [B]M = {February}[/B]

the set of natural numbers less than 7 that are divisible by 3
the set of natural numbers less than 7 that are divisible by 3 Natural Numbers less than 7 {1, 2, 3, 4, 5, 6} Only 2 of them are divisible by 3. Divisible means the number is divided evenly, with no remainder: [B]{3, 6}[/B]

The sides of a triangle are consecutive numbers. If the perimeter of the triangle is 240 m, find the
The sides of a triangle are consecutive numbers. If the perimeter of the triangle is 240 m, find the length of each side Let the first side be n. Next side which is consecutive is n + 1 Next side which is consecutive is n + 1 + 1 = n + 2 So we have the sum of 3 consecutive numbers is 240. We type in [I][URL='https://www.mathcelebrity.com/sum-of-consecutive-numbers.php?num=sumof3consecutivenumbersis240&pl=Calculate']sum of 3 consecutive numbers is 240[/URL][/I] into our search engine and we get: [B]79, 80, 81[/B]

The singular form of the word "dice" is "die". Tom was throwing a six-sided die. The first time he t
The singular form of the word "dice" is "die". Tom was throwing a six-sided die. The first time he threw, he got a three; the second time he threw, he got a three again. What's the probability of getting a three at the third time? Since all trials are independent: 1/6 * 1/6 * 1/6 = [B]1/216[/B]

The slope of a line is 7/6. What is the slope of any line parallel to this line?
The slope of a line is 7/6. What is the slope of any line parallel to this line? Parallel lines have the same slope, because they never touch. So the slope of the parallel line is [B]7/6[/B]

The slope of a roof is called its pitch. The Parthenon, an ancient Greek temple, has a roof with a r
The slope of a roof is called its pitch. The Parthenon, an ancient Greek temple, has a roof with a rise of 3.6 meters and a run of 12 meters. What is the pitch of the roof? Enter your answer in the box. Slope is rise over run. Slope = 3.6/12 Slope = [B]0.3[/B]

The sound from a thunderstorm travels approximately 1/5 of a mile in one second. How far will the so
The sound from a thunderstorm travels approximately 1/5 of a mile in one second. How far will the sound travel in 18.6 seconds? 1/5 mile per second * 18.6 seconds = [B]3.72 miles[/B]

The square of a number added to its reciprocal
The square of a number added to its reciprocal The phrase [I]a number [/I]means an arbitrary variable, let's call it x. the square of x mean we raise x to the power of 2. It's written as: x^2 The reciprocal of x is 1/x We add these together to get our final algebraic expression: [B]x^2 + 1/x [MEDIA=youtube]ZHut58-AoDU[/MEDIA][/B]

The square of a number increased by 7 is 23
The square of a number increased by 7 is 23 The phrase [I]a number [/I]means an arbitrary variable, let's call it x. x The square of a number means we raise x to the power of 2: x^2 [I]Increased by[/I] means we add 7 to x^2 x^2 + 7 The word [I]is[/I] means an equation. So we set x^2 + 7 equal to 23: [B]x^2 + 7 = 23[/B]

The square of a number is always nonnegative.
The square of a number is always nonnegative. This is true, and here is why: Suppose you have a positive number n. n^2 = n * n A positive times a positive is a positive Suppose you have a negative number -n (-n)^2 = -n * -n = n^2 A negative times a negative is a positive.

The square of a number is positive
The square of a number is positive N ca be positive or negative, so test both scenarios: Take a positive number n. n^2 = n^2 * n^2 or Positive * Positive which is positive Take a negative number n (-n)^2 = -n * -n or Negative * Negative which is positive (-n)^2 = n^2

The Square of a positive integer is equal to the sum of the integer and 12. Find the integer
The Square of a positive integer is equal to the sum of the integer and 12. Find the integer Let the integer be x. [LIST] [*]The sum of the integer and 12 is written as x + 12. [*]The square of a positive integer is written as x^2. [/LIST] We set these equal to each other: x^2 = x + 12 Subtract x + 12 from each side: x^2 - x - 12 = 0 We have a quadratic function. [URL='https://www.mathcelebrity.com/quadratic.php?num=x%5E2-x-12%3D0&pl=Solve+Quadratic+Equation&hintnum=+0']Run it through our search engine[/URL] and we get x = 3 and x = -4. The problem asks for a positive integer, so we have [B]x = 3[/B]

The square of a positive integer minus twice its consecutive integer is equal to 22. find the intege
The square of a positive integer minus twice its consecutive integer is equal to 22. Find the integers. Let x = the original positive integer. We have: [LIST] [*]Consecutive integer is x + 1 [*]x^2 - 2(x + 1) = 22 [/LIST] Multiply through: x^2 - 2x - 2 = 22 Subtract 22 from each side: x^2 - 2x - 24 = 0 Using our [URL='http://www.mathcelebrity.com/quadratic.php?num=x%5E2-2x-24%3D0&pl=Solve+Quadratic+Equation&hintnum=+0']quadratic calculator[/URL], we get: x = 6 and x = -4 Since the problem states [U]positive integers[/U], we use: x = 6 and x + 1 = 7 [B](6, 7)[/B]

The square of the difference of a number and 4
The square of the difference of a number and 4 A number means an arbitrary variable, let's call it x The difference of a number and 4: x - 4 The square of this difference: [B](x - 4)^2[/B]

The square of the difference of n and 2, increased by twice n
The square of the difference of n and 2, increased by twice n The difference of n and 2: n - 2 The square of the difference of n and 2 means we raise (n - 2) to the 2nd power: (n - 2)^2 Twice n means we multiply n by 2: 2n The square of the difference of n and 2, increased by twice n [B](n - 2)^2 + 2n[/B]

The square of the radius r
The square of the radius r The square means you raise r to the power of 2: [B]r^2[/B]

the square of the sum of 2a and 3b
the square of the sum of 2a and 3b the sum of 2a and 3b 2a + 3b The square of this sum means we raise 2a + 3b to the 2nd power: [B](2a + 3b)^2[/B]

the square of the sum of p and 5
the square of the sum of p and 5 The sum of p and 5 p + 5 Square this sum: [B](p + 5)^2[/B]

The square of the sum of twice a number x and y
The square of the sum of twice a number x and y Take this in algebraic expression in 3 parts: [LIST=1] [*]Twice a number x means we multiply x by 2: 2x [*]The sum of twice a number x and y means we add y to 2x above: 2x + y [*]The square of the sum means we raise the sum (2x + y) to the second power below: [/LIST] [B](2x + y)^2[/B]

the square of the sum of two numbers
the square of the sum of two numbers Let the first number be x. Let the second number be y. The sum is: x + y Now we square that sum by raising the sum to a power of 2: [B](x + y)^2[/B]

the square of the sum of x and y is less than 20
the square of the sum of x and y is less than 20 The sum of x and y means we add y to x: x + y the square of the sum of x and y means we raise the term x + y to the 2nd power: (x + y)^2 The phrase [I]is less than[/I] means an inequality, so we write this as follows: [B](x + y)^2 < 20[/B]

the square root of twice a number is 4 less than the number
Write this out, let the number be x. sqrt(2x) = x - 4 since 4 less means subtract Square each side: sqrt(2x)^2 = (x - 4)^2 2x = x^2 - 8x + 16 Subtract 2x from both sides x^2 - 10x + 16 = 0 Using the [URL='http://www.mathcelebrity.com/quadratic.php?num=x%5E2+-+10x+%2B+16+%3D+0&pl=Solve+Quadratic+Equation&hintnum=0']quadratic calculator[/URL], we get two potential solutions x = (2, 8) Well, 2 does not work, since sqrt(2*2) = 2 which is not 4 less than 2 However, 8 does work: sqrt(2*8) = sqrt(16) = 4, which is 4 less than the number 8.

The store is selling apples for $0.49 per pound. Write a function to model the cost of "p" pounds of
The store is selling apples for $0.49 per pound. Write a function to model the cost of "p" pounds of apples. Let p be the pounds of apples. Our cost function is: [B]C(p) = 0.49p[/B]

The sum is greater than 7, the sum is divisible by 2
The sum is greater than 7, the sum is divisible by 2 2 dice sum greater than 7 means 8, 9, 10, 11, 12. Now take this set, and intersect it with sums divisible by 2. [B]8, 10, 12[/B]

The sum of -4x^2 - 5x + 7 and 2x^2 + 8x - 11 can be written in the form ax^2 + bx + c, where a, b, a
The sum of -4x^2 - 5x + 7 and 2x^2 + 8x - 11 can be written in the form ax^2 + bx + c, where a, b, and c are constants. What is the value of a + b + c? The sum means we add the polynomials together. We do this by adding the like terms: -4x^2 - 5x + 7 + 2x^2 + 8x - 11 (-4 +2)x^2 + (-5 + 8)x +(7 - 11) -2x^2 + 3x - 4 We have (a, b, c) = (-2, 3, -4) The question asks for a + b + c a + b + c = -2 + 3 - 4 a + b + c = [B]-3[/B]

The sum of 13 and twice janelles age
Let Janelle's age be the variable a. So twice Janelle's age is denoted as 2a. We want the sum of 13 and 2a. Sum means add. 13 + 2a or 2a + 13

the sum of 16 and twice julies savings use the variable j to represent julies savings
The sum of 16 and twice julies savings use the variable j to represent julies savings Twice Julie's savings: 2j The sum of 16 and twice Julie's savings: [B]2j + 16[/B]

the sum of 16 squared and a number x
the sum of 16 squared and a number x 16 squared: 16^2 The sum of this and a number x [B]x + 16^2[/B]

The sum of 2 and w is less than or equal to 27.
The sum of 2 and w is less than or equal to 27. Take this algebraic expression in parts: [LIST] [*]The sum of 2 and w: 2 + w [*]The phrase [I]less than or equal to[/I] means an inequality, using the <= sign. [/LIST] [B]2 + w <= 27[/B]

The sum of 2 consecutive numbers is 3 less than 3 times the first number. What are the numbers?
The sum of 2 consecutive numbers is 3 less than 3 times the first number. What are the numbers? Let the first number be x. And the second number be y. We're given: [LIST=1] [*]y = x + 1 [*]x + y = 3x - 3 (less 3 means subtract 3) [/LIST] Substitute (1) into (2): x + x + 1 = 3x - 3 Combine like terms: 2x + 1 = 3x - 3 [URL='https://www.mathcelebrity.com/1unk.php?num=2x%2B1%3D3x-3&pl=Solve']Type this equation into the search engine[/URL], we get: x = 4 Substituting x = 4 into equation 1: y = 4 + 1 y = 5 So (x, y) = [B](4, 5)[/B]

The sum of 2 numbers is 18. 3 times the greater number exceeds 4 times the smaller number by 5. Find
The sum of 2 numbers is 18. 3 times the greater number exceeds 4 times the smaller number by 5. Find the numbers. Let the first number be x. The second number is y. We have: [LIST=1] [*]x + y = 18 [*]3x = 4y + 5 [/LIST] Rearrange (2), by subtracting 4y from each side: 3x - 4y = 5 So we have a system of equations: [LIST=1] [*]x + y = 18 [*]3x - 4y = 5 [/LIST] Using our [URL='https://www.mathcelebrity.com/simultaneous-equations.php?term1=x+%2B+y+%3D+18&term2=3x+-+4y+%3D+5&pl=Cramers+Method']simultaneous equations calculator[/URL], we get: [B]x = 11 y = 7[/B]

the sum of 2 numbers is 5. 5 times the larger number plus 4 times the smaller number is 37. Find the
the sum of 2 numbers is 5. 5 times the larger number plus 4 times the smaller number is 37. Find the numbers Let the first small number be x. Let the second larger number be y. We're given: [LIST=1] [*]x + y = 5 [*]5y + 4x = 37 [/LIST] We can solve this 3 ways, using the following methods: [LIST=1] [*][URL='https://www.mathcelebrity.com/simultaneous-equations.php?term1=x+%2B+y%3D5&term2=5y+%2B+4x+%3D+37&pl=Substitution']Substitution Method[/URL] [*][URL='https://www.mathcelebrity.com/simultaneous-equations.php?term1=x+%2B+y%3D5&term2=5y+%2B+4x+%3D+37&pl=Elimination']Elimination Method[/URL] [*][URL='https://www.mathcelebrity.com/simultaneous-equations.php?term1=x+%2B+y%3D5&term2=5y+%2B+4x+%3D+37&pl=Cramers+Method']Cramer's Rule[/URL] [/LIST] No matter what method we choose, we get: [B]x = -12 y = 17 [/B] Let's check our work using equation 1: -12 + 17 ? 5 5 = 5 <-- Check Let's check our work using equation 2: 5(17) + 4(-12) ? 37 85 - 48 ? 37 37 = 37 <-- Check

The sum of 2 numbers is 60. The larger number is thrice the smaller
The sum of 2 numbers is 60. The larger number is thrice the smaller. Let the 2 numbers be x and y, where x is the smaller number and y is the larger number. We are given: [LIST=1] [*]x + y = 60 [*]y = 3x [/LIST] Substitute (2) into (1): x + (3x) = 60 Combine like terms: 4x = 60 [URL='https://www.mathcelebrity.com/1unk.php?num=4x%3D60&pl=Solve']Type 4x = 60 into our search engine[/URL], and you get [B]x = 15[/B]. Substituting x = 15 into Equation (2) above, we get: y = 3(15) [B]y = 45 [/B] Check our work in Equation (1): 15 + 45 ? 60 60 = 60 Check our work in Equation (2): 45 ? 15(3) 45 = 45 The numbers check out, so our answer is [B](x, y) = (15, 45)[/B]

The sum of 2 numbers is 70. The difference of these numbers is 24. Write and solve a system of equat
The sum of 2 numbers is 70. The difference of these numbers is 24. Write and solve a system of equations to determine the numbers. Let the two numbers be x and y. We have the following equations: [LIST=1] [*]x + y = 70 [*]x - y = 24 [/LIST] Add (1) to (2): 2x = 94 Divide each side by 2 [B]x = 47[/B] Plug this into (1) 47 + y = 70 Subtract 47 from each side, we have: [B]y = 23[/B]

the sum of 2 times a number and -2, added to 4 times a number
the sum of 2 times a number and -2, added to 4 times a number. The phrase, [I]a number[/I], means an arbitrary variable, let's call it x. 2 times a number 2x The sum of means add, so we add -2, which is the same as subtracting 2 2x - 2 Now, we add 4 times x 2x - 2 + 4x Combining like terms, we have: (2 + 4)x - 2 [B]6x - 2[/B]

the sum of 2 times x and 3 times y diminished by 12
the sum of 2 times x and 3 times y diminished by 12 2 times x: 2x 3 times y: 3y the sum of 2 times x and 3 times y 2x + 3y the sum of 2 times x and 3 times y diminished by 12 [B]2x + 3y - 12[/B]

The sum of 2 times x and 5 times y is 7
The sum of 2 times x and 5 times y is 7 2 times x: 2x 5 times y: 5y The sum of 2 times x and 5 times y: 2x + 5y The word [I]is[/I] means equal to, so we set 2x + 5y equal to 7: [B]2x + 5y = 7[/B]

the sum of 23 and victor age is 59
the sum of 23 and victor age is 59 Let's Victor's age be a. The sum of 23 and Victor's age (a) mean we add a to 23: 23 + a The word [I]is[/I] means an equation, so we set 23 + a equal to 59: [B]23 + a = 59[/B] <-- This is our algebraic expression Now if the problem asks you to take it a step further and solve this for a, [URL='https://www.mathcelebrity.com/1unk.php?num=23%2Ba%3D59&pl=Solve']we type this equation into our search engine[/URL] and we get: [B]a = 36[/B]

The sum of 24 and twice Chau’s score . Use c to represent Chau’s score.
The sum of 24 and twice Chau’s score . Use c to represent Chau’s score. Twice Chau's score means we multiply his score of c by 2: 2c The sum of 24 and 2c means we add: [B]24 + 2c[/B]

The sum of 2x and y is at least 20
The sum of 2x and y is at least 20 The sum of 2x and y: 2x + y The phrase [I]is at least[/I] means an inequality. We write this as >= or greater than or equal to: [B]2x + y >= 20[/B]

the sum of 3 and 2x is 10
the sum of 3 and 2x is 10 The sum of 3 and 2x means we add 2x to 3: 3 + 2x The word [I]is[/I] means an equation, so we set 3 + 2x equal to 10 [B]3 + 2x = 10[/B]

The sum of 3 consecutive integers is greater than 30.
The sum of 3 consecutive integers is greater than 30. Let the first consecutive integer be n The second consecutive integer is n + 1 The third consecutive integer is n + 2 The sum is written as: n + n + 1 + n + 2 Combine like terms: (n + n + n) + (1 + 2) 3n + 3 The phrase [I]greater than[/I] means an inequality, which we write as: [B]3n + 3 > 30[/B]

the sum of 3 consecutive natural numbers, the first of which is n
the sum of 3 consecutive natural numbers, the first of which is n Natural numbers are counting numbers, so we the following expression: n + (n + 1) + (n + 2) Combine n terms and constants: (n + n + n) + (1 + 2) [B]3n + 3 Also expressed as 3(n + 1)[/B]

the sum of 3 consecutive natural numbers, the first of which is n
the sum of 3 consecutive natural numbers, the first of which is n We have: n + (n + 1) + (n + 2) Grouping like terms, we have: [B]3n + 3[/B]

The sum of 3 consecutive natural numbers, the first of which is n
The sum of 3 consecutive natural numbers, the first of which is n. We have 3 numbers: n, n + 1, and n + 2 Add them up: n + (n + 1) + (n + 2) Group like terms: [B]3n + 3[/B]

the sum of 3 numbers
Since no variable name is defined, we pick 3 arbitrary variables. Let's pick x, y, and z. The sum of 3 numbers means we add them together: x + y + z

the sum of 3 numbers
Let's choose 3 arbitrary variables, w, x, and y. We add them up: [B]w + x + y[/B]

the sum of 3 numbers a, b, and c
the sum of 3 numbers a, b, and c [B]a + b + c[/B]

the sum of 3 numbers divided by its product
the sum of 3 numbers divided by its product The phrase [I]3 numbers[/I] means we choose [I]3[/I] arbitrary variables. Let's call them x, y, z. The sum of of these 3 numbers is: x + y + z The phrase [I]its product[/I] means we multiply all 3 arbitrary variables together: xyz Now, the phrase [I]divided by[/I] means we divide x + y + z by xyz: [B](x + y + z)/xyz[/B]

The sum of 3 times the square of a number and negative 7
The sum of 3 times the square of a number and negative 7 [U]The phrase [I]a number[/I] means an arbitrary variable, let's call it x:[/U] x [U]The square of a number means we raise x to the power of 2:[/U] x^2 [U]3 times the square of a number:[/U] 3x^2 [U]The sum of 3 times the square of a number and negative 7[/U] [B]3x^2 - 7[/B]

The sum of 3, 7, and a number amounts to 16
The sum of 3, 7, and a number amounts to 16 Let the number be n. A sum means we add. We're given: 3 + 7 + n = 16 Grouping like terms, we get: n + 10 = 16 [URL='https://www.mathcelebrity.com/1unk.php?num=n%2B10%3D16&pl=Solve']Typing this equation into our search engine[/URL], we get: n = [B]6 [/B]

The sum of 3h and k divided by 2
The sum of 3h and k divided by 2 The sum of 3h and k 3h + k Divided by 2: [B](3h + k)/2[/B]

The sum of 3w and 5 cubed
The sum of 3w and 5 cubed The sum of 3w and 5: 3w + 5 The word [I]cubed[/I] means we raise 3w + 5 to the power 3: [B](3w + 5)^3[/B]

The sum of 4 and x is multiplied by 5. The result is then taken away from 16
The sum of 4 and x is multiplied by 5. The result is then taken away from 16. Take this algebraic expression in 3 parts: [U]Part 1: The sum of 4 and x:[/U] 4 + x [U]Part 2: Multiplied by 5:[/U] 5(4 + x) [U]Part 3: The result is then taken away from 16:[/U] [B]16 - 5(4 + x)[/B]

the sum of 4 and x split into 5 equal parts
the sum of 4 and x split into 5 equal parts The sum of x and 4 means we add 4 to x: x + 4 Whenever you see the phrase [I]split into[/I], think of divide or divided by: [B](x + 4)/5[/B]

The sum of 5 and 2x is at most 27
The sum of 5 and 2x is at most 27 The sum of 5 and 2x means we add 2x to 5: 5 + 2x The phrase [I]at most[/I] means less than or equal to, so we have an inequality where 5 + 2x is less than or equal to 27 [B]5 + 2x <= 27[/B]

the sum of 5 and y is less than or equal to -21
the sum of 5 and y is less than or equal to -21 Take this algebraic expression in parts: The sum of 5 and y means we add y to 5 5 + y The phrase [I]less than or equal to[/I] -21 means an inequality. We use the <= sign to relate 5 + y to -21 [B]5 + y <= -21[/B]

The sum of 5 odd consecutive numbers is 145
The sum of 5 odd consecutive numbers is 145. Let the first odd number be n. We have the other 4 odd numbers denoted as: [LIST] [*]n + 2 [*]n + 4 [*]n + 6 [*]n + 8 [/LIST] Add them all together n + (n + 2) + (n + 4) + (n + 6) + (n + 8) The sum of the 5 odd consecutive numbers equals 145 n + (n + 2) + (n + 4) + (n + 6) + (n + 8) = 145 Combine like terms: 5n + 20 = 145 Using our [URL='http://www.mathcelebrity.com/1unk.php?num=5n%2B20%3D145&pl=Solve']equation solver[/URL], we get [B]n = 25[/B]. Using our other 4 consecutive odd numbers above, we get: [LIST] [*]27 [*]29 [*]31 [*]33 [/LIST] Adding the sum up, we get: 25 + 27 + 29 + 31 + 33 = 145. So our 5 odd consecutive number added to get 145 are [B]{25, 27, 29, 31, 33}[/B]. [MEDIA=youtube]0T2PDuQIIwI[/MEDIA]

the sum of 5 times p and 10
the sum of 5 times p and 10 5 times p 5p and 10 means add 10 [B]5p + 10[/B]

The sum of 5, 8, and a number amounts to 19. Find the number.
The sum of 5, 8, and a number amounts to 19. Find the number. We represent [I]a number[/I] with the variable "x". We write our problem as: 5 + 8 + x = 19 13 + x = 19 [URL='https://www.mathcelebrity.com/1unk.php?num=13%2Bx%3D19&pl=Solve']Type this problem into our calculator[/URL], and we get [B]x = 6[/B].

The sum of 5x and 2x is at least 70
[I]Is at least [/I]means greater than or equal to: 5x + 2x >= 70 If we combine like terms, we have: 7x >=70 We can further simplify by dividing each side of the inequality by 7 x >=10 If you want the interval notation for that, use the [URL='http://www.mathcelebrity.com/interval-notation-calculator.php?num=x%3E%3D10&pl=Show+Interval+Notation']interval notation calculator[/URL].

the sum of 6 and 7, plus 5 times a number, is -12
the sum of 6 and 7, plus 5 times a number, is -12 The sum of 6 and 7 means we add the two numbers: 6 + 7 This evaluates to 13 Next, we take 5 times a number. The phrase [I]a number[/I] means an arbitrary variable, let's call it x. So we multiply x by 5: 5x The first two words say [I]the sum[/I], so we add 13 and 5x 13 + 5x The word [I]is[/I] means an equation, so we set 13 + 5x equal to -12 [B]13 + 5x = -12[/B] <-- This is our algebraic expression If the problem asks you to take it a step further and solve for x, then you [URL='https://www.mathcelebrity.com/1unk.php?num=13%2B5x%3D-12&pl=Solve']type this algebraic expression into our search engine[/URL] and you get: [B]x = -5[/B]

The sum of 6 times a number and -8, added to 3 times a number
The sum of 6 times a number and -8, added to 3 times a number The phrase "a number", means an arbitrary variable, let's call it x. 6 times a number: 6x And means we add, so we have 6x - 8 Added to 3 times a number 6x - 8 + 3x Combine like terms: [B]9x - 8[/B]

the sum of 7 times y and 3 is equal to 2
the sum of 7 times y and 3 is equal to 2 7 times y: 7y The sum of 7 times y and 3 means we add 3 to 7y 7y + 3 The phrase [I]is equal to[/I] means an equation, so we set 7y + 3 equal to 2 [B]7y + 3 = 2[/B]

The sum of 80 and 40 is divided by 5
The sum of 80 and 40 is divided by 5 The sum of 80 and 40: 80 + 40 Divided by 5: [B](80 + 40)/5[/B]

The sum of 9 and victors age is 55
The sum of 9 and victors age is 55 Let v be Victor's age. We have the algebraic expression: [B]v + 9 = 55 [/B] If you want to solve or v, use our [URL='http://www.mathcelebrity.com/1unk.php?num=v%2B9%3D55&pl=Solve']equation calculator[/URL].

The sum of a and b added with the quotient of x and y.
The sum of a and b added with the quotient of x and y. The sum of a and b is written as: a + b The quotient of x and y is written as: x/y The phrase [I]added with[/I] means we add x/y to a + b: [B]a + b + x/y[/B]

The sum of a and b divided by their product
The sum of a and b divided by their product The sum of a and b means we add b to a: a + b The product of a and b means we multiply a by b: ab To get our final algebraic expression, we divide the sum (a + b) by the product ab: [B](a + b)/ab[/B]

the sum of a and b minus 4 is 12
the sum of a and b minus 4 is 12 the sum of a and b a + b the sum of a and b minus 4 a + b - 4 The word [I]is[/I] means equal to, so we set a + b - 4 equal to 12: a + b - 4 = 12

the sum of a and b, divided by the product of c and d
the sum of a and b, divided by the product of c and d The sum of a and b, means we add b to a a + b The product of c and d means we multiply c by d cd Divided by means we divide a + b by cd [B](a + b)/cd[/B]

the sum of a and its reciprocal
the sum of a and its reciprocal The reciprocal of a is 1/a, so the sum is: [B]a + 1/a[/B]

the sum of a number and 16 is e
A number means an arbitrary variable, let's call it x. The sum of x and 16 means we add: x + 16 Is, means equal to, so we set x + 16 = e x + 16 = e

The sum of a number and 34 times the number
The sum of a number and 34 times the number The phrase [I]a number[/I] means an arbitrary variable. Let's call it x. x 34 times the number: 34x The sum of a number and 34 times the number means we add both terms together: x + 34x

The sum of a number and 5 all divided by 2 is 17
The sum of a number and 5 all divided by 2 is 17 The phrase [I]a number[/I] means an arbitrary variable, let's call it x x The sum of a number and 5: x + 5 All divided by 2: (x + 5)/2 The word [I]is[/I] means equal to, so we set (x + 5)/2 equal to 17: [B](x + 5)/2 = 17[/B]

The sum of a number and 5 divided by 8
The sum of a number and 5 divided by 8. Let's take this algebraic expression in parts. Part 1: The phrase [I]a number[/I] means an arbitrary variable, let's call it x. Part 2: The sum of a number and 5 means we add 5 to the number x x + 5 Part 3: Next, we divide this expression by 8 [B](x + 5)/8[/B]

the sum of a number and its reciprocal is 5/2
the sum of a number and its reciprocal is 5/2 The phrase [I]a number[/I] means an arbitrary variable, let's call it x. The reciprocal of the number means 1/x. The sum means we add them: x + 1/x The word [I]is[/I] means an equation, so we set x + 1/x equal to 52 [B]x + 1/x = 52[/B]

The sum of a number and its reciprocal is 72
The sum of a number and its reciprocal is 72 The phrase [I]a number[/I] means an arbitrary variable, let's call it x x The reciprocal of the number is written as: 1/x The sum of a number and its reciprocal means we add: x + 1/x The word [I]is[/I] means an equation, so we set x + 1/x equal to 72 [B]x + 1/x = 72[/B]

The sum of a number and its reciprocal is five.
The sum of a number and its reciprocal is five. The phrase [I]a number[/I] means an arbitrary variable. Let's call it x. The reciprocal of the number is 1/x. The sum means we add them together: x + 1/x The word [I]is[/I] means an equation, so we set x + 1/x equal to 5 [B]x + 1/x = 5[/B]

The sum of a number and its square is 72. find the numbers?
The sum of a number and its square is 72. find the numbers? Let the number be n. We have: n^2 + n = 72 Subtract 72 from each side: n^2 + n - 72 = 0 Using our [URL='http://www.mathcelebrity.com/quadratic.php?num=n%5E2%2Bn-72%3D0&pl=Solve+Quadratic+Equation&hintnum=+0']quadratic calculator[/URL], we have: [B]n = 8 or n = -9 [/B] Since the numbers do not state positive or negative, these are the two solutions.

the sum of a number and itself is 8
A number means an arbitrary variable, let's call it x. The sum of a number and itself means adding the number to itself x + x Simplified, we have 2x The word is means equal to, so we have an algebraic expression of: [B]2x= 8 [/B] IF you need to solve this equation, divide each side by 2 [B]x = 4[/B]

The sum of a number and twice its reciprocal is 3
The sum of a number and twice its reciprocal is 3 the phrase [I]a number[/I] means an arbitrary variable, let's call it x. x The reciprocal of a number means we take 1 over that number: 1/x Twice the reciprocal means we multiply 1/x by 2: 2/x The sum of a number and twice its reciprocal x + 2/x The word [I]is[/I] means equal to, so we set x + 2/x equal to 3 [B]x + 2/x = 3[/B]

The sum of a number b and 3 is greater than 4 and no more than 16
The sum of a number b and 3 is greater than 4 and no more than 16 The sum of a number b and 3: b + 3 Greater than 4 and no more than 16 means we have a combo inequality: [LIST] [*]Greater than 4 means we use a > sign [*]No more than 16 means less than or equal to, so <= [/LIST] [B]4 < b + 3 <= 16[/B]

the sum of a number divided by 8 and 3 equals 6
"A Number" means an arbitrary variable, let's call it x. x divide d by 8 is written as a quotient x/8 The sum of x/8 and 3 means we add: x/8 + 3 Finally, equals means we have an equation, so we set our expression above equal to 6 x/8 + 3 = 6

The sum Of a number squared and 14
The sum Of a number squared and 14. A number means an arbitrary variable, let's call it x. Squared means we raise x to the 2nd power: x^2 The sum means we add x^2 to 14 to get our algebraic expression below: [B]x^2 + 14[/B]

the sum of a number times 3 and 30 is less than 17
the sum of a number times 3 and 30 is less than 17 A number is denoted as an arbitrary variable, let's call it x. x Times 3 means we multiply x by 3: 3x The sum of a number times 3 and 30 means we add 30 to 3x above 3x + 30 Is less than 17 means we have an inequality, so we set 3x + 30 less than 17 3x + 30 < 17 To solve for x and see the interval notation, use [URL='http://www.mathcelebrity.com/1unk.php?num=3x%2B30%3C17&pl=Solve']our calculator[/URL]:

the sum of b cubed and five
the sum of b cubed and five b cubed b^3 the sum of this and five [B]b^3 + 5[/B]

the sum of d and twice g
Twice g means we multiply g by 2. 2g The sum of d and twice g means we add: d + 2g

The sum of d and v, all multiplied by 8
The sum of d and v, all multiplied by 8 This is an algebraic expression. The sum of d and v: d + v Multiply this sum by 8: [B]8(d + v)[/B]

the sum of doubling a number and 100 which totals to 160
the sum of doubling a number and 100 which totals to 160 Take this algebraic expression in pieces: [LIST=1] [*]Let the number be n. [*]Double it, means we multiply n by 2: 2n [*]The sum of this and 100 means we add 100 to 2n: 2n + 100 [*]The phrase [I]which totals[/I] means we set 2n + 100 equal to 160 [/LIST] [B]2n + 100 = 160[/B] <-- This is our algebraic expression If the question asks you to solve for n, then we [URL='https://www.mathcelebrity.com/1unk.php?num=2n%2B100%3D160&pl=Solve']type this equation into our search engine[/URL] and we get: [B]n = 30[/B]

the sum of five -sixths of m and 7
the sum of five -sixths of m and 7 five-sixths of m: 5/6m The word [I]and[/I] means we add, so we have: [B]5/6m + 7[/B]

The sum of five and twice a number is 17
The sum of five and twice a number is 17 [U]The phrase a number means an arbitrary variable, let's call it x[/U] x [U]Twice a number means we multiply x by 2:[/U] 2x [U]The sum of five and twice a number means we add 5 to 2x:[/U] 2x + 5 [U]The phrase [I]is[/I] means an equation, so we set 2x + 5 equal to 17 to get our algebraic expression[/U] [B]2x + 5 = 17[/B] [B][/B] As a bonus, if the problem asks you to solve for x, we [URL='https://www.mathcelebrity.com/1unk.php?num=2x%2B5%3D17&pl=Solve']type in this algebraic expression into our math engine[/URL] and we get: x = 6

The sum of Jocelyn and Joseph's age is 40. In 5 years, Joseph will be twice as Jocelyn's present age
The sum of Jocelyn and Joseph's age is 40. In 5 years, Joseph will be twice as Jocelyn's present age. How old are they now? Let Jocelyn's age be a Let Joseph's age be b. We're given two equations: [LIST=1] [*]a + b = 40 [*]2(a + 5) = b + 5 [/LIST] We rearrange equation (1) in terms of a to get: [LIST=1] [*]a = 40 - b [*]2a = b + 5 [/LIST] Substitute equation (1) into equation (2) for a: 2(40 - b) = b + 5 80 - 2b = b + 5 To solve this equation for b, we [URL='https://www.mathcelebrity.com/1unk.php?num=80-2b%3Db%2B5&pl=Solve']type it in our search engine[/URL] and we get: [B]b (Joseph's age) = 25[/B] Now, substitute b = 25 into equation (1) to solve for a: a = 40 - 25 [B]a (Jocelyn's age) = 15[/B]

The sum of Juan’s age and Sara’s age is 33 yrs. If Sara is 15 yrs old, how old is Juan?
The sum of Juan’s age and Sara’s age is 33 yrs. If Sara is 15 yrs old, how old is Juan? Let j be Juan's age and s be Sara's age. We have the following equations: [LIST=1] [*]j + s = 33 [*]s = 15 [/LIST] Substitute (2) into (1) j + 15 = 33 Using our [URL='http://www.mathcelebrity.com/1unk.php?num=j%2B15%3D33&pl=Solve']equation solver[/URL], we get[B] j = 18[/B]

The sum of m and 3 divided by the difference of m minus 3
The sum of m and 3 divided by the difference of m minus 3. Sum of m and 3: m + 3 Difference of m minus 3 m - 3 Take a quotient of these expressions: [B]m + 3 ------- m - 3[/B]

The sum of Mr. Adams and Mrs. Benson's age is 55. The difference is 3. What are their ages?
The sum of Mr. Adams and Mrs. Benson's age is 55. The difference is 3. What are their ages? [U]Givens[/U] [LIST] [*]Let Mr. Adam's age be a [*]Let Mrs. Benson's age be b [*]We're given two equations where [I]sum[/I] means we add and [I]difference[/I] means we subtract: [/LIST] [LIST=1] [*]a + b = 55 [*]a - b = 3 [/LIST] Since we have opposite coefficients for b, we can take a shortcut and add equation 1 to equation 2: (a + a) + (b - b) = 55 + 3 Combining like terms and simplifying, we get: 2a = 58 To solve this equation for a, we [URL='https://www.mathcelebrity.com/1unk.php?num=2a%3D58&pl=Solve']type it in our search engine[/URL] and we get: a = [B]29 [/B] If a = 29, then we plug this into equation (1) to get: 29 + b = 55 b = 55 - 29 b = [B]26 [MEDIA=youtube]WwkpNqPvHs8[/MEDIA][/B]

the sum of n and twice n is 12
Twice n means we multiply n by 2 2n The sum of n and twice n means we add n + 2n The word [I]is[/I] means equal to, so we set that expression above equal to 12 n + 2n = 12 Combine like terms: 3n = 12 Divide each side of the equation by 3 to isolate n n = 4 Check our work Twice n is 2*4 = 8 Add that to n = 4 8 + 4 12

The sum of six times a number and 1 is equal to five times the number. Find the number.
The sum of six times a number and 1 is equal to five times the number. Find the number. The phrase [I]a number[/I] means an arbitrary variable, let's call it x. 6 times a number is written as: 6x the sum of six times a number and 1 is written as: 6x + 1 Five times the number is written as: 5x The phrase [I]is equal to[/I] means an equation, so we set 6x + 1 equal to 5x: 6x + 1 = 5x [URL='https://www.mathcelebrity.com/1unk.php?num=6x%2B1%3D5x&pl=Solve']Plugging this into our search engine[/URL], we get: x = [B]-1[/B]

The sum of the ages of levi and renee is 89 years. 7 years ago levi's age was 4 times renees age. Ho
The sum of the ages of levi and renee is 89 years. 7 years ago levi's age was 4 times renees age. How old is Levi now? Let Levi's current age be l. Let Renee's current age be r. Were given two equations: [LIST=1] [*]l + r = 89 [*]l - 7 = 4(r - 7) [/LIST] Simplify equation 2 by multiplying through: [LIST=1] [*]l + r = 89 [*]l - 7 = 4r - 28 [/LIST] Rearrange equation 1 to solve for r and isolate l by subtracting l from each side: [LIST=1] [*]r = 89 - l [*]l - 7 = 4r - 28 [/LIST] Now substitute equation (1) into equation (2): l - 7 = 4(89 - l) - 28 l - 7 = 356 - 4l - 28 l - 7 = 328 - 4l To solve for l, we [URL='https://www.mathcelebrity.com/1unk.php?num=l-7%3D328-4l&pl=Solve']type the equation into our search engine[/URL] and we get: l = [B]67[/B]

the sum of the cube of a number and 12
the sum of the cube of a number and 12 The phrase [I]a number[/I] means an arbitrary variable, let's call it x. The cube of a number means we raise x to the power of 3: x^3 Finally, we take the sum of x^3 and 12. Meaning, we add 12 to x^3. This is our final algebraic expression. [B]x^3 + 12[/B]

The sum of the digits of a 2 digit number is 10. The value of the number is four more than 15 times
The sum of the digits of a 2 digit number is 10. The value of the number is four more than 15 times the unit digit. Find the number. Let the digits be (x)(y) where t is the tens digit, and o is the ones digit. We're given: [LIST=1] [*]x + y = 10 [*]10x+ y = 15y + 4 [/LIST] Simplifying Equation (2) by subtracting y from each side, we get: 10x = 14y + 4 Rearranging equation (1), we get: x = 10 - y Substitute this into simplified equation (2): 10(10 - y) = 14y + 4 100 - 10y = 14y + 4 [URL='https://www.mathcelebrity.com/1unk.php?num=100-10y%3D14y%2B4&pl=Solve']Typing this equation into our search engine[/URL], we get: y = 4 Plug this into rearranged equation (1), we get: x = 10 - 4 x = 6 So our number xy is [B]64[/B]. Let's check our work against equation (1): 6 + 4 ? 10 10 = 10 Let's check our work against equation (2): 10(6)+ 4 ? 15(4) + 4 60 + 4 ? 60 + 4 64 = 64

The sum of the digits of a certain two-digit number is 16. Reversing its digits increases the number
The sum of the digits of a certain two-digit number is 16. Reversing its digits increases the number by 18. What is the number? Let x and (16-x) represent the original ten and units digits respectively Reversing its digits increases the number by 18 Set up the relational equation [10x + (16-x)] + 18 = 10(16 - x) + x Solving for x 9x + 34 = 160 - 9x Combine like terms 18x = 126 Divide each side of the equation by 18 18x/18 = 126/18 x = 7 So 16 - x = 16 - 7 = 9 The first number is 79, the other number is 97. and 97 - 79 = 18 so we match up. The number in our answer is [B]79[/B]

The sum of the length l and 17
The sum of the length l and 17 The word [I]sum[/I] means we add: [B]l + 17[/B]

The sum of the measures of two exterior angles of a triangle is 205. What is the measure of the thir
The sum of the measures of two exterior angles of a triangle is 205. What is the measure of the third exterior angle? The sum of exterior angles for a triangle is 360. To find the third exterior angle, we take 360 - 205 = [B]155[/B].

The sum of the product and quotient of the numbers x and y
The sum of the product and quotient of the numbers x and y the product of the numbers x and y xy The quotient of the numbers x and y x/y The sum of the product and quotient of the numbers x and y [B]xy + x/y [MEDIA=youtube]0bzv8aEIF0I[/MEDIA][/B]

the sum of the reciprocal of x and the reciprocal of y
the sum of the reciprocal of x and the reciprocal of y Reciprocal of x means 1 over x: 1/x Reciprocal of y means 1 over y: 1/y The sum means we add the two reciprocals together: [B]1/x + 1/y[/B]

The sum of the reciprocals of x and y
The sum of the reciprocals of x and y The reciprocal of a variable is found by taking 1 over the variable. [LIST] [*]Reciprocal of x = 1/x [*]Reciprocal of y = 1/y [/LIST] The sum means we add the reciprocals together [B]1/x + 1/y[/B]

The sum of the square of a number and 7 times a number
The sum of the square of a number and 7 times a number The phrase [I]a number[/I] means an arbitrary variable, let's call it x. x Square the number: x^2 7 times the number means we multiply x by 7: 7x The sum means we add x^2 and 7x [B]x^2 + 7x[/B]

the sum of the squares of a and b
the sum of the squares of a and b Square of a means we raise a to the 2nd power: a^2 Square of b means we raise b to the 2nd power: b^2 The sum of squares means we add these terms together to get our algebraic expression: [B]a^2 + b^2[/B]

The sum of the squares of c and d is 25
The sum of the squares of c and d is 25 The square of c means we we raise c to the power of 2: c^2 The square of d means we we raise d to the power of 2: d^2 The sum of the squares of c and d means we add d^2 to c^2: c^2 + d^2 The word [I]is[/I] means equal to, so we set c^2 + d^2 equal to 25: [B]c^2 + d^2 = 25[/B]

The sum of the squares of two consecutive positive integers is 61. Find these two numbers.
The sum of the squares of two consecutive positive integers is 61. Find these two numbers. Let the 2 consecutive integers be x and x + 1. We have: x^2 + (x + 1)^2 = 61 Simplify: x^2 + x^2 + 2x + 1 = 61 2x^2 + 2x + 1 = 61 Subtract 61 from each side: 2x^2 + 2x - 60 = 0 Divide each side by 2 x^2 + x - 30 Using our [URL='http://www.mathcelebrity.com/quadratic.php?num=x%5E2%2Bx-30&pl=Solve+Quadratic+Equation&hintnum=+0']quadratic equation calculator[/URL], we get: x = 5 and x = -6 The question asks for [I]positive integers[/I], so we use [B]x = 5. [/B]This means the other number is [B]6[/B].

The sum of the sum of x and z and the difference of y and z
The sum of the sum of x and z and the difference of y and z Take this algebraic expression in 3 parts: Step 1: The sum of x and z is written as: x + z Step 2: The difference of y and z is written as: y - z Step 3: the sum of the sum and difference is written as: x + z + (y - z) x + z + y - z Cancelling the z terms, we get: [B]x + y [MEDIA=youtube]bmoZXImYCrg[/MEDIA][/B]

The sum of three consecutive integers is 42
Let the 3 integers be x, y, and z. y = x + 1 z = y + 1, or x + 2. Set up our equation: x + (x + 1) + (x + 2) = 42 Group our variables and constants: (x + x + x) + (1 + 2) = 42 3x + 3 = 42 Subtract 3 from each side: 3x = 39 Divide each side of the equation by 3: [B]x = 13 So y = x + 1 = 14 z = x + 2 = 15 (x,y,z) = (13,14,15)[/B]

The sum of three numbers is 171. The second number is 1/2 of the first and the third is 3/4 of the f
The sum of three numbers is 171. The second number is 1/2 of the first and the third is 3/4 of the first. Find the numbers. We have three numbers, x, y, and z. [LIST=1] [*]x + y + z = 171 [*]y = 1/2x [*]z = 3/4x [/LIST] Substitute (2) and (3) into (1) x + 1/2x + 3/4x = 171 Use a common denominator of 4 for each x term 4x/4 + 2x/4 + 3x/4 = 171 (4 + 2 + 3)x/4 = 171 9x/4 = 171 [URL='https://www.mathcelebrity.com/prop.php?num1=9x&num2=171&den1=4&den2=1&propsign=%3D&pl=Calculate+missing+proportion+value']Plug this equation into our search engine[/URL], and we get [B]x = 76[/B] So y = 1/2(76) --> [B]y = 38[/B] Then z = 3/4(76) --> [B]z = 57[/B]

The Sum of three times a number and 18 is -39. Find the number
The Sum of three times a number and 18 is -39. Find the number. A number means an arbitrary variable, let us call it x. Three times x: 3x The sum of this and 18: 3x + 18 Is means equal to, so we set 3x + 18 = -39 3x + 18 = -39 Using our [URL='http://www.mathcelebrity.com/1unk.php?num=3x%2B18%3D-39&pl=Solve']equation solver[/URL], we get [B]x = -19[/B]

The sum of three times a number and twelve
The phrase [I]a number[/I] means an arbitrary variable. We can pick any letter a-z except for i and e. Let's choose x. 3 times a number: 3x The sum of three times a number and twelve means we add 12 to 3x: [B]3x + 12[/B]

the sum of twice a and b
the sum of twice a and b Twice a means multiply a by 2 2a The sum of means add Twice a to b [B]2a + b[/B]

The sum of twice an integer and 3 times the next consecutive integer is 48
The sum of twice an integer and 3 times the next consecutive integer is 48 Let the first integer be n This means the next consecutive integer is n + 1 Twice an integer means we multiply n by 2: 2n 3 times the next consecutive integer means we multiply (n + 1) by 3 3(n + 1) The sum of these is: 2n + 3(n + 1) The word [I]is[/I] means equal to, so we set 2n + 3(n + 1) equal to 48: 2n + 3(n + 1) = 48 Solve for [I]n[/I] in the equation 2n + 3(n + 1) = 48 We first need to simplify the expression removing parentheses Simplify 3(n + 1): Distribute the 3 to each term in (n+1) 3 * n = (3 * 1)n = 3n 3 * 1 = (3 * 1) = 3 Our Total expanded term is 3n + 3 Our updated term to work with is 2n + 3n + 3 = 48 We first need to simplify the expression removing parentheses Our updated term to work with is 2n + 3n + 3 = 48 [SIZE=5][B]Step 1: Group the n terms on the left hand side:[/B][/SIZE] (2 + 3)n = 5n [SIZE=5][B]Step 2: Form modified equation[/B][/SIZE] 5n + 3 = + 48 [SIZE=5][B]Step 3: Group constants:[/B][/SIZE] We need to group our constants 3 and 48. To do that, we subtract 3 from both sides 5n + 3 - 3 = 48 - 3 [SIZE=5][B]Step 4: Cancel 3 on the left side:[/B][/SIZE] 5n = 45 [SIZE=5][B]Step 5: Divide each side of the equation by 5[/B][/SIZE] 5n/5 = 45/5 Cancel the 5's on the left side and we get: n = [B]9[/B]

The sum of two consecutive integers if n is the first integer.
The sum of two consecutive integers if n is the first integer. consecutive means immediately after, so we have: n n + 1 [U]The sum is written as:[/U] n + n + 1 [U]Grouping like terms, we have:[/U] (n + n) + 1 [B]2n + 1[/B]

The sum of two consecutive integers plus 18 is 123
The sum of two consecutive integers plus 18 is 123. Let our first integer be n and our next integer be n + 1. We have: n + (n + 1) + 18 = 123 Group like terms to get our algebraic expression: 2n + 19 = 123 If we want to solve the algebraic expression using our [URL='http://www.mathcelebrity.com/1unk.php?num=2n%2B19%3D123&pl=Solve']equation solver[/URL], we get n = 52. This means the next integer is 52 + 1 = 53

The sum of two numbers is 231. The larger is twice the smaller. What are the numbers?
Let x be the larger number. Let y be the smaller number. We're given two equations: [LIST=1] [*]x + y = 231 [*]x = 2y [/LIST] Substitute (2) into (1) for x: 2y + y = 231 3y = 231 [URL='https://www.mathcelebrity.com/1unk.php?num=3y%3D231&pl=Solve']Type this into our math solver[/URL] and get y = 77 This means x is: x = 2(77) x = [B]154[/B]

The sum of two numbers multiplied by 9
Choose two variables as arbitrary numbers, let's say x and y [U]The sum of x and y is:[/U] x + y [U]Multiply that by 9[/U] [B]9(x + y)[/B]

The sum of two y and the quantity of three plus y plus twice the quantity two y minus two equals fif
The sum of two y and the quantity of three plus y plus twice the quantity two y minus two equals fifteen The sum of two y and the quantity of three plus y 2y + (3 + y) twice the quantity two y minus two 2(2y - 2) The sum of two y and the quantity of three plus y plus twice the quantity two y minus two 2y + (3 + y) + 2(2y - 2) Equals 15 to get our algebraic expression [B]2y + (3 + y) + 2(2y - 2) = 15[/B] [B][/B] If the problem asks you to solve for yL 2y + 3 + y + 4y - 4 = 15 Group like terms: 7y - 1 = 15 Add 1 each side: 7y = 16 Divide each side by 7: y = [B]16/7[/B]

The sum of two y and three is the same as the difference of three y and one
The sum of two y and three 2y + 3 the difference of three y and one 3y - 1 is the same as means equal to. Set 2y + 3 equal to 3y - 1 for our final algebraic expression: [B]2y + 3 = 3y - 1[/B] [B][/B] If the problem asks you to solve for y, subtract 2y from each side: 3 = y - 1 Add 1 to each side: y = [B]4[/B]

The sum of two-fifths and f is one-half.
The sum of two-fifths and f is one-half. We write two-fifths as 2/5. The sum of two-fifths and f is written by adding f to two-fifths using the + sign: 2/5 + f one-half is written as 1/2 The word [I]is[/I] means equals, so we set up an equation where 2/5 + f equal to 1/2 [B]2/5 + f = 1/2[/B]

the sum of w and v divided by their difference
the sum of w and v divided by their difference the sum of w and v: w + v their difference: w - v the sum of w and v divided by their difference [B](w + v)/(w - v)[/B]

The sum of x and 10 equals the sum of 2 times x and 12
The sum of x and 10 equals the sum of 2 times x and 12 The sum of x and 10 means we add 10 to x: x + 10 2 times x means we multiply x by 2: 2x the sum of 2 times x and 12 means we add 12 to 2x: 2x + 12 The sum of x and 10 equals the sum of 2 times x and 12: x + 10 + (2x + 12) Distribute the parentheses, and we get: x + 10 + 2x + 12 Group like terms: (1 + 2)x + 10 + 12 [B]3x + 22[/B]

the sum of x and 3 is divided by 2
the sum of x and 3 is divided by 2 The sum of x and 3 x + 3 Divide this expression by 2 (x + 3)/2

the sum of X and 3 is divided by 2
the sum of X and 3 is divided by 2 The sum of X and 3 X + 3 Is divided by 2 [B](X + 3)/2[/B]

the sum of x and 96 equals half of x
the sum of x and 96 equals half of x half of x means we divide x by 2: x/2 The sum of x and 96: x + 96 The phrase equals means we set x + 96 equal to x/2: [B]x + 96 = x/2[/B]

the sum of x and its cube
the sum of x and its cube The cube of x means we raise x to the power of 3: x^3 The sum of x and it's cube means we add x^3 to x: [B]x + x^3[/B]

the sum of x and its reciprocal
the sum of x and its reciprocal The reciprocal of x is found by dividing 1 by x: 1/x the sum of x and its reciprocal means we add 1/x to x: [B]x + 1/x[/B]

The sum of x and one half of x
The sum of x and one half of x To write this algebraic expression correctly, we have (1 + 1/2)x To get common denominators, we write 1 as 2/2. So we have: (2/2 + 1/2)x [B]3/2x[/B]

The sum of x and twice y
The sum of x and twice y Twice y 2y The sum of x and twice y x + 2y

The sum of x and twice y is equal to m.
The sum of x and twice y is equal to m. Twice y means we multiply y by 2: 2y The sum of x and twice y: x + 2y The phrase [I]is equal to[/I] means an equation, so we set x + 2y equal to m [B]x + 2y = m[/B]

The sum of x and y doubled
The sum of x and y doubled The sum of x and y: x + y Double it [B]2(x + y)[/B]

The sum of x and y is at most 10
The sum of x and y is at most 10 The sum of x and y: x + y Is at most 10 means we have an inequality, at most means 10 or less, so less than or equal to [B]x + y <= 10[/B]

The sum of x and y is multiplied by 6.
The sum of x and y is multiplied by 6. [LIST] [*]The sum of x and y: x + y [*]Multiply the sum by 6: [/LIST] [B]6(x + y)[/B]

the sum of x squared plus y squared
the sum of x squared plus y squared x squared means we raise x to the power of 2: x^2 y squared means we raise y to the power of 2: y^2 The sum means we add both terms together: [B]x^2 + y^2[/B]

The sum of y and z decreased by the difference of m and n
The sum of y and z decreased by the difference of m and n. Take this algebraic expression in 3 parts: [LIST=1] [*]The sum of y and z means we add z to y: y + z [*]The difference of m and n means we subtract n from m: m - n [*]The phrase [I]decreased by[/I] means we subtract the quantity (m - n) from the sum (y + z) [/LIST] [B](y + z) - (m - n)[/B]

The tax in a state is 5%. If Eric paid $4.60 tax for a bike, what was the cost of the bike before sa
The tax in a state is 5%. If Eric paid $4.60 tax for a bike, what was the cost of the bike before sales tax was added? 4.60 / 0.05 = [B]$92[/B]

The tax on 1 dollar is 7 cents. What is the tax on 5 dollars?
The tax on 1 dollar is 7 cents. What is the tax on 5 dollars? Two ways you can do this: [LIST=1] [*]Every 1 dollar has 7 cents, so every 5 dollars has (1 * 5) = (7 * 5) = [B]35 cents[/B] [*]7 cents is 7% of 1 dollar. So 7% of 5 dollars is [B]35 cents[/B] [/LIST]

The teacher is handing out note cards to her students. She gave 20 note cards to the first student,
The teacher is handing out note cards to her students. She gave 20 note cards to the first student, 30 note cards to the second student, 40 note cards to the third student, and 50 note cards to the fourth student. If this pattern continues, how many note cards will the teacher give to the fifth student? [LIST] [*]Student 1 has 20 [*]Student 2 has 30 [*]Student 3 has 40 [*]Student 4 has 50 [/LIST] The teacher adds 10 note cards to each student. Or, if we want to put in a sequence formula, we have: S(n) = 10 + 10n where n is the student number Simplified, we write this as: S(n) = 10(1 + n) The question asks for S(5) S(5) = 10(1 + 5) S(5) = 10(6) [B]S(5) = 60 [/B] If we wanted to simply add 10 and not use a sequence formula, we see that S(4) = 50. So S(5) = S(4) + 10 S(5) = 50 + 10 [B]S(5) = 60[/B]

The team A scored 13 more points than Team B. The total of their score was 47. How many points did t
The team A scored 13 more points than Team B. The total of their score was 47. How many points did team A score? Let a be the amount of points A scored, and b be the amount of points B scored. We're given: [LIST=1] [*]a = b + 13 [*]a + b = 47 [/LIST] Plug (1) into (2) (b + 13) + b = 47 Combine like terms: 2b + 13 = 47 [URL='https://www.mathcelebrity.com/1unk.php?num=2b%2B13%3D47&pl=Solve']Typing this equation into our search engine[/URL], we get: b = 17 Now plug this into (1): a = 17 + 13 a = [B]30[/B]

The temp
The temperature of a solution was -23C. After adding a substance to the solution, the temperature after adding the substance to the solution was 133C. What is the difference between the temperature of the solution before and after adding the substance Using our [URL='https://www.mathcelebrity.com/temp-change.php?num=thetemperatureofasolutionwas-23c.afteraddingasubstancetothesolutionthetemperaturefe133c.whatisthedifferencebetweenthetemperatureofthesolutionbeforeandafteraddingthesubstance%3E&pl=Calculate+Temp+Change']temperature difference calculator[/URL], we get: [B]156C[/B]

The temperature dropped 2 every hours for 6 hours. What was the total number of degrees the temperat
The temperature dropped 2 every hours for 6 hours. What was the total number of degrees the temperature changed in the 6 hours 2 degrees drop per hour * 6 hours = [B]12 degree drop[/B]

The temperature in Chicago was five (5) degrees Celsius in the morning and is expected to drop by as
The temperature in Chicago was five (5) degrees Celsius in the morning and is expected to drop by as much as 12 degrees during the day. What is the lowest temperature in Chicago for the day? We start with 5 celsius A drop in temperature means we subtract 5 - 12 = [B]-7 or 7 degrees below zero[/B]

The temperature in Minneapolis changed from -7 fahrenheit at 6am to 7 fahrenheit at noon. What was t
The temperature in Minneapolis changed from -7 fahrenheit at 6am to 7 fahrenheit at noon. What was the difference between high and low temperatures? Difference = High Temp - Low Temp Difference = 7 - -7 Difference = [B]14[/B]

The temperature inside the lab refrigerator is no more than 35 . Use t to represent the temperature
The temperature inside the lab refrigerator is no more than 35 . Use t to represent the temperature (in ) of the refrigerator. The phrase [I]no more than[/I] means less than or equal to. We have this inequality: [B]t <= 35[/B]

The temperature is 68. What is the temperature in degrees Celsius
The temperature is 68. What is the temperature in degrees Celsius We type [URL='https://www.mathcelebrity.com/temperature.php?temp=68&type=Fahrenheit&pl=Convert']68 degrees Fahrenheit into our search engine[/URL] and we get: [B]20 Celsius[/B]

The temperature was 7? below zero. The temperature drops by 6?. What is the temperature now
The temperature was 7? below zero. The temperature drops by 6?. What is the temperature now Below zero means negative. A drop means we subtract, so we have: [LIST] [*]7 below zero = -7 [*]Drops by 6 = -6 [*]-7 - 6 = [B]-13[/B] [/LIST]

The temperature when Spencer arrived at school was a very chilly – 4°F. By the time school got out,
The temperature when Spencer arrived at school was a very chilly – 4°F. By the time school got out, the temperature had risen 13°F. What was the temperature when school got out? -4 + 13 = [B]9 degrees[/B]

The theater can hold 250 people 50 seats are empty how many people are there
The theater can hold 250 people 50 seats are empty how many people are there 250 - 50 empty seats = [B]200 people in the theater[/B].

The top part of the tree is 3 times as long as the trunk which equation can tulia use to find t the
The top part of the tree is 3 times as long as the trunk which equation can tulia use to find t the length of the trunk Let p be the top part of the tree. We have p = 3t. Divide by 3, we get t = [B]p/3[/B]

The total age of three cousins is 48. Suresh is half as old as Hakima and 4 years older than Saad. h
The total age of three cousins is 48. Suresh is half as old as Hakima and 4 years older than Saad. How old are the cousins? Let a be Suresh's age, h be Hakima's age, and c be Saad's age. We're given: [LIST=1] [*]a + h + c = 48 [*]a = 0.5h [*]a = c + 4 [/LIST] To isolate equations in terms of Suresh's age (a), let's do the following: [LIST] [*]Rewriting (3) by subtracting 4 from each side, we get c = a - 4 [*]Rewriting (2) by multiply each side by 2, we have h = 2a [/LIST] We have a new system of equations: [LIST=1] [*]a + h + c = 48 [*]h = 2a [*]c = a - 4 [/LIST] Plug (2) and (3) into (1) a + (2a) + (a - 4) = 48 Group like terms: (1 + 2 + 1)a - 4 = 48 4a - 4 = 48 [URL='https://www.mathcelebrity.com/1unk.php?num=4a-4%3D48&pl=Solve']Typing this equation into our search engine[/URL], we get: [B]a = 13 [/B]<-- Suresh's age This means that Hakima's age, from modified equation (2) above is: h = 2(13) [B]h = 26[/B] <-- Hakima's age This means that Saad's age, from modified equation (3) above is: c = 13 - 4 [B]c = 9[/B] <-- Saad's age [B] [/B]

The total cost for 9 bracelets, including shipping was $72. The shipping charge was $9. Define your
The total cost for 9 bracelets, including shipping was $72. The shipping charge was $9. Define your variable and write an equation that models the cost of each bracelet. We set up a cost function as fixed cost plus total cost. Fixed cost is the shipping charge of $9. So we have the following cost function where n is the cost of the bracelets: C(b) = nb + 9 We are given C(9) = 72 and b = 9 9n + 9 = 72 [URL='https://www.mathcelebrity.com/1unk.php?num=9n%2B9%3D72&pl=Solve']Run this through our equation calculator[/URL], and we get [B]n = 7[/B].

The total cost of 100 dresses is $1,500.00. The mark-up is estimated at 20% of the unit cost, the pr
The total cost of 100 dresses is $1,500.00. The mark-up is estimated at 20% of the unit cost, the price of a single dress using the cost-plus method will be The phrase [I]unit cost[/I] means price per one unit. [U]Unit cost for one dress is:[/U] Price of dresses / Number of dresses 1500/100 15 Each dress cost $15 which is the unit cost [U]Cost plus method:[/U] Cost plus price = Unit price + Unit price * markup Cost plus price = 15 + 15 * 20% Cost plus price = 15 + 3 Cost plus price = [B]$18 [MEDIA=youtube]H9rOp592y5s[/MEDIA][/B]

The total cost of producing x units for which the fixed cost are $2500 and the cost per unit $20
The total cost of producing x units for which the fixed cost are $2500 and the cost per unit $20 Total Cost = Cost per Unit * Units + Fixed Cost Total Cost = [B]20x + 2500[/B]

The total cost of producing x units for which the fixed costs are $2900 and the cost per unit is $25
The total cost of producing x units for which the fixed costs are $2900 and the cost per unit is $25 [U]Set up the cost function:[/U] Cost function = Fixed Cost + Variable Cost per Unit * Number of Units [U]Plug in Fixed Cost = 2900 and Cost per Unit = $25[/U] [B]C(x) = 2900 + 25x [MEDIA=youtube]77PiD-VADjM[/MEDIA][/B]

The total cost to fix your bike is $45 the parts cost $10 and the labor cost seven dollars per hour
The total cost to fix your bike is $45 the parts cost $10 and the labor cost seven dollars per hour how many hours were there: Set up a cost function where h is the number of hours: 7h + 10 = 45 To solve for h, we t[URL='https://www.mathcelebrity.com/1unk.php?num=7h%2B10%3D45&pl=Solve']ype this equation into our search engine[/URL] and we get: h = [B]5[/B]

the total number of fish if you had 8 and bought 4 more
the total number of fish if you had 8 and bought 4 more If you have 8, and buy 4 more, this means you add. So we have: 8 fish + 4 fish = [B]12 fish[/B].

the total of 3 times the cube of u and the square of u
the total of 3 times the cube of u and the square of u [U]The cube of u means we raise u to the power of 3:[/U] u^3 [U]The square of u means we raise u to the power of 2:[/U] u^2 The total of both of these is found by adding them together: [B]u^3 + u^2[/B]

the total of a and 352 equals a divided by 195
the total of a and 352 equals a divided by 195 Take this algebraic expression in 3 parts: [LIST=1] [*]The total of a and 352 means we add 352 to a: a + 352 [*]a divided by 195: a/195 [*]The phrase [I]equals[/I] means we set (1) equal to (2) to get our final algebraic expression: [/LIST] [B]a + 352 = a/195[/B]

The total of z and 12 multiplied by the difference of 9 and y
The total of z and 12 multiplied by the difference of 9 and y The total of z and 12: z + 12 The difference of 9 and y: 9 - y Now we multiply z + 12 by 9 - y: [B](z + 12)(9 - y)[/B]

The United States Department of Agriculture reports that 18% of Americans are now on food stamps. If
The United States Department of Agriculture reports that 18% of Americans are now on food stamps. If there are 250,000,000 Americans, how many are on food stamps? Since 18% is 0.18, we have: 250,000 * 0.18 = 45,000

the university of california tuition in 1990 was $951 and tuition has been increasing by a rate of 2
the university of california tuition in 1990 was $951 and tuition has been increasing by a rate of 26% each year, what is the exponential formula Let y be the number of years since 1990. We have the formula T(y): [B]T(y) = 951 * 1.26^y[/B]

The value of 3 times the quantity of 4 + x is greater than 6 less than x.
The value of 3 times the quantity of 4 + x is greater than 6 less than x. 3 times the quantity 4 + x 3(4 + x) 6 less than x x - 6 3 times the quantity 4 + x is greater than x - 6 [B]3(4 + x) > x - 6[/B]

The value of a company van is $15,000 and decreased at a rate of 4% each year. Approximate how much
The value of a company van is $15,000 and decreased at a rate of 4% each year. Approximate how much the van will be worth in 7 years. Each year, the van is worth 100% - 4% = 96%, or 0.96. We have the Book value equation: B(t) = 15000(0.96)^t where t is the time in years from now. The problem asks for B(7): B(7) = 15000(0.96)^7 B(7) = 15000(0.7514474781) B(7) = [B]11,271.71[/B]

The value of a stock begins at $0.07 and increases by $0.02 each month. Enter an equation representi
The value of a stock begins at $0.07 and increases by $0.02 each month. Enter an equation representing the value of the stock v in any month m. Set up our equation v(m): [B]v(m) = 0.07 + 0.02m[/B]

The value of all the quarters and dimes in a parking meter is $18. There are twice as many quarters
The value of all the quarters and dimes in a parking meter is $18. There are twice as many quarters as dimes. What is the total number of dimes in the parking meter? Let q be the number of quarters. Let d be the number of dimes. We're given: [LIST=1] [*]q = 2d [*]0.10d + 0.25q = 18 [/LIST] Substitute (1) into (2): 0.10d + 0.25(2d) = 18 0.10d + 0.5d = 18 [URL='https://www.mathcelebrity.com/1unk.php?num=0.10d%2B0.5d%3D18&pl=Solve']Type this equation into our search engine[/URL], and we get [B]d = 30[/B].

The volleyball team and the wrestling team at Clarksville High School are having a joint car wash t
The volleyball team and the wrestling team at Clarksville High School are having a joint car wash today, and they are splitting the revenues. The volleyball team gets $4 per car. In addition, they have already brought in $81 from past fundraisers. The wrestling team has raised $85 in the past, and they are making $2 per car today. After washing a certain number of cars together, each team will have raised the same amount in total. What will that total be? How many cars will that take? Set up the earnings equation for the volleyball team where w is the number of cars washed: E = Price per cars washed * w + past fundraisers E = 4w + 81 Set up the earnings equation for the wrestling team where w is the number of cars washed: E = Price per cars washed * w + past fundraisers E = 2w + 85 If the raised the same amount in total, set both earnings equations equal to each other: 4w + 81 = 2w + 85 Solve for [I]w[/I] in the equation 4w + 81 = 2w + 85 [SIZE=5][B]Step 1: Group variables:[/B][/SIZE] We need to group our variables 4w and 2w. To do that, we subtract 2w from both sides 4w + 81 - 2w = 2w + 85 - 2w [SIZE=5][B]Step 2: Cancel 2w on the right side:[/B][/SIZE] 2w + 81 = 85 [SIZE=5][B]Step 3: Group constants:[/B][/SIZE] We need to group our constants 81 and 85. To do that, we subtract 81 from both sides 2w + 81 - 81 = 85 - 81 [SIZE=5][B]Step 4: Cancel 81 on the left side:[/B][/SIZE] 2w = 4 [SIZE=5][B]Step 5: Divide each side of the equation by 2[/B][/SIZE] 2w/2 = 4/2 w = [B]2 <-- How many cars it will take [/B] To get the total earnings, we take either the volleyball or wrestling team's Earnings equation and plug in w = 2: E = 4(2) + 81 E = 8 + 81 E = [B]89 <-- Total Earnings[/B]

The Waist to Hip Ratio (WHR) is commonly expressed as a decimal. WHR has been shown to be a good pre
The Waist to Hip Ratio (WHR) is commonly expressed as a decimal. WHR has been shown to be a good predictor of possible cardiovascular problems in both men and women. If Jonia has a WHR greater than 1, she is at “high risk” for cardiovascular problems. Jonia’s waist measurement is 42 inches and her hip measurement 2 inches less. Jonia's WHR: WHR = W/H WHR = 42/(42 - 2) WHR =4 2/40 WHR = [B]1.5 which is high risk[/B]

The weight of a 9.5-inch by 6-inch paperback book published by Leaden Publications, Inc., is 16.2 oz
The weight of a 9.5-inch by 6-inch paperback book published by Leaden Publications, Inc., is 16.2 oz. The standard deviation is 2.9 oz. What is the probability that the average weight of a sample of 33 such books is less than 15.89 oz? Using our [URL='http://www.mathcelebrity.com/probnormdist.php?xone=15.89&mean=16.2&stdev=2.9&n=33&pl=P%28X+%3C+Z%29']z-score calculator[/URL], we get: [B]0.271[/B]

The weight of water is approximately 2 pounds 3 ounces per litre. How much will 8 litres of water we
The weight of water is approximately 2 pounds 3 ounces per litre. How much will 8 litres of water weigh? First, convert 2 pounds 3 ounces to ounces. 16 ounces to a pound, so we have: 2(16) + 3 32 + 3 35 ounces for one liter For 8 litres, we have: 35 * 8 = 280 ounces Now, convert that back to pounds 280/16 = [B]17.5 pounds, or 17 pounds, 8 ounces.[/B]

The width of a rectangle is fixed at 4cm. For what lengths will the area be less than 86 cm^2
The width of a rectangle is fixed at 4cm. For what lengths will the area be less than 86 cm^2 The Area (A) of a rectangle is given by: A = lw With an area of [I]less than[/I] 86 and a width of 4, we have the following inequality: 4l < 86 To solve for l, we [URL='https://www.mathcelebrity.com/interval-notation-calculator.php?num=4l%3C86&pl=Show+Interval+Notation']type this inequality into our search engine[/URL] and we get: [B]l < 21.5[/B]

The world record for the mile in the year 1865 was held by Richard Webster of England when he comple
The world record for the mile in the year 1865 was held by Richard Webster of England when he completed a mile in 4 minutes and 36.5 seconds. The world record in 1999 was set by Hicham El Guerrouj when he ran a mile in 3 minutes and 43.13 seconds. If both men ran the mile together, how many feet behind would Richard Webster be when Hichem El Guerrouj crossed the finish line? Change times to seconds: [LIST] [*]4 minutes and 36.5 seconds = 4*60 + 36.5 = 240 + 36.5 = 276.5 seconds [*]3 minute and 43.13 seconds = 3*60 + 43.13 = 180 + 43.13 = 223.13 seconds [/LIST] Now, find the distance Richard Webster travelled in 3 minutes and 43.13 seconds which is when Hiram El Guerrouj crossed the finish line. 1 mile = 5280 feet: Set up a proportion of distance in feet to seconds where n is the distance Richard Webster travelled 5280/276.5 = n/223.13 Using our [URL='https://www.mathcelebrity.com/proportion-calculator.php?num1=5280&num2=n&den1=276.5&den2=223.13&propsign=%3D&pl=Calculate+missing+proportion+value']proportion calculator,[/URL] we get: n = 4260.85 feet Distance difference = 5280 - 4260.85 = [B]1019.15 feet[/B]

The ———— of 5 and 7 is 12
The ———— of 5 and 7 is 12 [B](sum) or (total) since 5 + 7 = 12[/B]

Theodore invests $17,000 at 9% simple interest for 1 year. How much is in the account at the end of
Theodore invests $17,000 at 9% simple interest for 1 year. How much is in the account at the end of the 1 year period. Using our [URL='http://www.mathcelebrity.com/simpint.php?av=&p=17000&int=9&t=1&pl=Simple+Interest']balance calculator with simple interest[/URL], we have: [B]18,530[/B]

there are $4.20 in nickel and quarters. There are 6 more nickels than quarters there. How many coins
there are $4.20 in nickel and quarters. There are 6 more nickels than quarters there. How many coins of each are there We're given two equations: [LIST=1] [*]n = q + 6 [*]0.05n + 0.25q = 4.2 [/LIST] Substitute equation (1) into equation (2): 0.05(q + 6) + 0.25q = 4.2 Multiply through and simplify: 0.05q + 0.3 + 0.25q 0.3q + 0.3 = 4.2 To solve for q, we [URL='https://www.mathcelebrity.com/1unk.php?num=0.3q%2B0.3%3D4.2&pl=Solve']type this equation into the search engine[/URL] and we get: q = [B]13 [/B] To solve for n, we plug in q = 13 into equation (1): n = 13 + 6 n = [B]19[/B]

There are 1 carrot, 3 onions, and 2 potatoes in a sink. What fraction of the vegetables are onions
There are 1 carrot, 3 onions, and 2 potatoes in a sink. What fraction of the vegetables are onions? We have 1 + 3 + 2 = 6 total vegetables. Which means we have 3/6 onions. But, we can reduce this fraction. [URL='https://www.mathcelebrity.com/fraction.php?frac1=3%2F6&frac2=3%2F8&pl=Simplify']Simplifying 3/6 using our fraction simplifier[/URL], we get 1/2.

There are 10 more cars(c) than jeeps(j)
There are 10 more cars(c) than jeeps(j) [B]c = j + 10[/B]

There are 10 true or false questions on a test. You do not know the answer to 4 of the questions, so
There are 10 true or false questions on a test. You do not know the answer to 4 of the questions, so you guess. What is the probability that you will get all 4 answers right? Probability you guess right is 1/2 or 0.5. Since each event is independent of the other events, we multiply 1/2 4 times: 1/2 * 1/2 * 1/2 * 1/2 = [B]1/16[/B]

There are 100 people in a sport centre. 67 people use the gym. 62 people use the swimming pool. 5
There are 100 people in a sport centre. 67 people use the gym. 62 people use the swimming pool. 56 people use the track. 38 people use the gym and the pool. 31 people use the pool and the track. 33 people use the gym and the track. 16 people use all three facilities. A person is selected at random. What is the probability that this person doesn't use any facility? WE use the compound probability formula for 3 events: [LIST=1] [*]Gym use (G) [*]Swimming pool use (S) [*]Track (T) [/LIST] P(G U S U T) = P(G) + P(S) + P(T) - P(G Intersection S) - P(G Intersection T) - P(S Intersection T) + P(G Intersection S Intersection T) [LIST] [*]Note: U means Union (Or) and Intersection means (And) [/LIST] Plugging our numbers in: P(G U S U T) = 67/100 + 62/100 + 56/100 - 38/100 - 31/100 - 33/100 + 16/100 P(G U S U T) = (67 + 62 + 56 - 38 - 31 - 33 + 16)/100 P(G U S U T) = 99/100 or 0.99 What this says is, the probability that somebody uses at any of the 3 facilities is 99/100. The problem asks for none of the 3 facilities, or P(G U S U T)' P(G U S U T)' = 1 - P(G U S U T) P(G U S U T)' = 1 - 99/100 P(G U S U T)' = 100/100 - 99/100 P(G U S U T)' = [B]1/100 or 0.1[/B]

There are 100 teachers in a school of 3300 students find the ratio of number of teachers to the numb
There are 100 teachers in a school of 3300 students find the ratio of number of teachers to the number of students. The Ratio is 100/3300. Divide top and bottom by 100: 1/330 or [B]1:33 [/B] You can also this into the search engine: [URL='https://www.mathcelebrity.com/ratio.php?simpratio=100%3A3300&rs=+7%3A5&rtot=+36&ab=+7%3A3&bc=+2%3A5&pl=Simplify+Ratio']Ratio of 100 to 3300[/URL].

There are 1000 juniors in a college. Among the 1000 juniors, 200 students are taking STAT200, and 10
There are 1000 juniors in a college. Among the 1000 juniors, 200 students are taking STAT200, and 100 students are taking PSYC300. There are 50 students taking both courses. a) What is the probability that a randomly selected junior is taking at least one of these two courses? b) What is the probability that a randomly selected junior is taking PSYC300, given that he/she is taking STAT200? a) P(A U B) = P(A) + P(B) - P(A ? B) = 0.2 + 0.1 - 0.05 = [B]0.25[/B] b) P(SYC|STAT) = P(STAT ? SYC)/P(STAT) = 0.05/0.2 = [B]0.25[/B]

There are 11 kids at a birthday party. If there are 6 girls and 5 boys at the party, what fraction o
There are 11 kids at a birthday party. If there are 6 girls and 5 boys at the party, what fraction of the kids are boys? Boys fraction = [B]5/11[/B]

There are 113 identical plastic chips numbered 1 through 113 in a box. What is the probability of re
There are 113 identical plastic chips numbered 1 through 113 in a box. What is the probability of reaching into the box and randomly drawing a chip number that is greater than 44? We want 45, 46, … 113 The formula to get inclusive number count between and including 2 numbers is: Total numbers = L - S + 1 Total numbers = 113 - 45 + 1 Total numbers = 69 That is 69 possible numbers. We draw this out of a total of 113 [B]P(Number > 44) = 69/113 [B]P(Number > 44) [/B]= 0.610619 [MEDIA=youtube]BLBVcpdHqXU[/MEDIA][/B]

There are 12 eggs in a dozen. Write an algebraic expression for the number of eggs in d dozen.
There are 12 eggs in a dozen. Write an algebraic expression for the number of eggs in d dozen. [B]12d[/B]

There are 12 inches per foot. How many inches are there in 14 feet?
There are 12 inches per foot. How many inches are there in 14 feet? Two ways to solve this. Plug in [URL='http://www.mathcelebrity.com/linearcon.php?quant=14&pl=Calculate&type=foot']14 feet [/URL]into the search engine to get [B]168 inches.[/B] Or, we do proportions: 12 inches / 1 foot * 14 feet = 12 * 14 = 168 inches per 14 feet.

there are 120 calories in 3/4 cup serving of cereal. How many Calories are there in 6 cups of cereal
120/3/4 = x/6 Cross multiply: 0.75x = 720 Divide each side of the equation by 0.75 [B]x = 960[/B]

There are 13 animals in the barn. some are chickens and some are pigs. there are 40 legs in all. How
There are 13 animals in the barn. some are chickens and some are pigs. there are 40 legs in all. How many of each animal are there? Chickens have 2 legs, pigs have 4 legs. Let c be the number of chickens and p be the number of pigs. Set up our givens: (1) c + p = 13 (2) 2c + 4p = 40 [U]Rearrange (1) to solve for c by subtracting p from both sides:[/U] (3) c = 13 - p [U]Substitute (3) into (2)[/U] 2(13 - p) + 4p = 40 26 - 2p + 4p = 40 [U]Combine p terms[/U] 2p + 26 = 40 [U]Subtract 26 from each side:[/U] 2p = 14 [U]Divide each side by 2[/U] [B]p = 7[/B] [U]Substitute p = 7 into (3)[/U] c = 13 - 7 [B]c = 6[/B] For a shortcut, you could use our [URL='http://www.mathcelebrity.com/simultaneous-equations.php?term1=c+%2B+p+%3D+13&term2=2c+%2B+4p+%3D+40&pl=Cramers+Method']simultaneous equations calculator[/URL]

There are 144 people in an audience. The ratio of adults to children is 5 to 3. How many are adults?
There are 144 people in an audience. The ratio of adults to children is 5 to 3. How many are adults? We set up an equation to represent this: 5x + 3x = 144 [URL='https://www.mathcelebrity.com/1unk.php?num=5x%2B3x%3D144&pl=Solve']Typing this equation into our search engine[/URL], we get: x = 18 This means we have: Adults = 5(18) [B]Adults = 90[/B] Children = 3(18) [B]Children = 54[/B]

There are 144 people in the audience. The ratio of adults to children is 5 to 3. How many adults are
[SIZE=6]There are 144 people in the audience. The ratio of adults to children is 5 to 3. How many adults are there? Let x be the number of people, we have: 5x + 3x = 144 [/SIZE] [URL='https://www.mathcelebrity.com/1unk.php?num=5x%2B3x%3D144&pl=Solve'][SIZE=6]Typing this problem in our search[/SIZE][/URL][SIZE=6][URL='https://www.mathcelebrity.com/1unk.php?num=5x%2B3x%3D144&pl=Solve'] engine[/URL], we get x = 18. Which means we have 5(18) = [B]90 adults[/B][/SIZE]

There are 15 cards, numbered 1 through 15. If you pick a card, what is the probability that you choo
There are 15 cards, numbered 1 through 15. If you pick a card, what is the probability that you choose an odd number or a two? We want the P(odd) or P(2). P(odd) = 1, 3, 5, 7, 9, 11, 13, 15 = 8/15 P(2) = 1/15 Add them both: 8/15 + 1/15 = 9/15 Simplified, we get [B]3/5[/B].

There are 15 houses in a neighborhood. Nine of the houses have 6 people in them. The remaining house
There are 15 houses in a neighborhood. Nine of the houses have 6 people in them. The remaining houses have 4 people in them. How many people are in a neighborhood. 9 houses * 6 people per house = 54 people The remaining houses equal 15 total houses - 9 houses = 6 houses 6 houses remaining times 4 people in each house = 24 people 54 people + 24 people = [B]78 people in the neighborhood[/B]

There are 2 chalkboards in your classroom. If each chalkboard needs 2 pieces of chalk, how many piec
There are 2 chalkboards in your classroom. If each chalkboard needs 2 pieces of chalk, how many pieces do you need in total? Total chalk pieces = Number of Chalkboards * Chalk pieces per chalkboard Total chalk pieces = 2 * 2 Total chalk pieces = [B]4[/B]

There are 2 consecutive integers. Twice the first increased by the second yields 16. What are the nu
There are 2 consecutive integers. Twice the first increased by the second yields 16. What are the numbers? Let x be the first integer. y = x + 1 is the next integer. We have the following givens: [LIST=1] [*]2x + y = 16 [*]y = x + 1 [/LIST] Substitute (2) into (1) 2x + (x + 1) = 16 Combine x terms 3x + 1 = 16 Subtract 1 from each side 3x = 15 Divide each side by 3 [B]x = 5[/B] So the other integer is 5 + 1 = [B]6[/B]

There are 2 piles of papers on a desk. Each pile has the same number of papers. There are 12 papers
There are 2 piles of papers on a desk. Each pile has the same number of papers. There are 12 papers in all on the desk. How many papers are in each pile? 12 papers on the desk / 2 piles of papers Divide top and bottom by 2 [B]6 papers per pile.[/B]

There are 2.5 servings in a can of tuna fish. how many servings are there in 7 cans?
There are 2.5 servings in a can of tuna fish. how many servings are there in 7 cans? Total Servings = Servings per can * number of cans Total Servings = 2.5 * 7 Total Servings = [B]17.5 servings[/B]

There are 24 competitors in a cycling race. How many different selections are possible for first and
There are 24 competitors in a cycling race. How many different selections are possible for first and second place? We want unique combinations, so we have: [URL='https://www.mathcelebrity.com/permutation.php?num=24&den=2&pl=Combinations']24 C 2[/URL] = [B]276[/B]

There are 24 hours in a day and scientists tell us that we should sleep for 3/8 of the day. How much
There are 24 hours in a day and scientists tell us that we should sleep for 3/8 of the day. How much time should we spend sleeping? Multiply 24 hours per day * 3/8 day Since 24/8 = 3, we have: 3 * 3 = [B]9 hours of sleep[/B].

There are 24 hours in a day and scientists tell us that we should sleep for 3/8 of the day. How much
There are 24 hours in a day and scientists tell us that we should sleep for 3/8 of the day. How much time should we spend sleeping? 3/8 of the day means we take 3/8 of 24 also written as: 3/8 * 24 We [URL='https://www.mathcelebrity.com/fraction.php?frac1=3%2F8&frac2=24&pl=Multiply']type this expression into our search engine [/URL]and get: [B]9 hours[/B]

There are 24 students in a class. Three new students joined the class. Work out the percentage chang
There are 24 students in a class. Three new students joined the class. Work out the percentage change in the number of students in the class. We want to know how much an increase of 3 people is in a class of 24: 3/24 Using [URL='https://www.mathcelebrity.com/perc.php?num=3&den=24&pcheck=1&num1=16&pct1=80&pct2=70&den1=80&idpct1=10&hltype=1&idpct2=90&pct=82&decimal=+65.236&astart=12&aend=20&wp1=20&wp2=30&pl=Calculate']our percentage/decimal calculator[/URL], we get: [B]12.5% increase[/B]

There are 250 boys and 150 girls in a school, if 60% of the boys and 40% of the girls play football,
There are 250 boys and 150 girls in a school, if 60% of the boys and 40% of the girls play football, what percentage of the school play football [U]First calculate total students:[/U] Total students = Boys + Girls Total students = 250 + 150 Total students = 400 [U]Calculate the boys that play football:[/U] Boys playing football = 60% * 250 [URL='https://www.mathcelebrity.com/perc.php?num=+5&den=+8&num1=+16&pct1=+80&pct2=60&den1=250&pcheck=3&pct=+82&decimal=+65.236&astart=+12&aend=+20&wp1=20&wp2=30&pl=Calculate']Boys playing football [/URL]= 150 [U]Calculate the girls that play football:[/U] Girls playing football = 40% * 150 [URL='https://www.mathcelebrity.com/perc.php?num=+5&den=+8&num1=+16&pct1=+80&pct2=40&den1=150&pcheck=3&pct=+82&decimal=+65.236&astart=+12&aend=+20&wp1=20&wp2=30&pl=Calculate']Girls playing football[/URL][URL='https://www.mathcelebrity.com/perc.php?num=+5&den=+8&num1=+16&pct1=+80&pct2=60&den1=250&pcheck=3&pct=+82&decimal=+65.236&astart=+12&aend=+20&wp1=20&wp2=30&pl=Calculate'] [/URL]= 60 [U]Calculate total people playing football[/U]: Total people playing football = Boys playing football + Girls playing football Total people playing football = 150 + 60 Total people playing football = 210 Calculate percentage of the school playing football (P): P = 100% * Total people playing football / Total Students P = 100% * [URL='https://www.mathcelebrity.com/longdiv.php?num1=210&num2=400&pl=Long%20Division%20%28Decimals%29']210/400[/URL] P = 100% * 0.525 P = [B]52.5%[/B]

There are 3,742,450 men, 3,177,805 women and 21,508 children in a village. Find the total population
There are 3,742,450 men, 3,177,805 women and 21,508 children in a village. Find the total population of the village. Total population = Men + Women + Children Total population = 3,742,450 + 3,177,805 + 21,508 Total population = [B]6,941,763[/B]

There are 30 students in a classroom. Eighteen students read A Wrinkle in Time while 22 children rea
There are 30 students in a classroom. Eighteen students read [I]A Wrinkle in Time[/I] while 22 children read [I]The Hobbit[/I]. If all children read at least one of the books, how many read both books? 30 - 18 = 12 students read the Hobbit only 30 - (12 + 8) = [B]10 students who read both[/B]

There are 32 ears of corn for 16 people how many ears of corn can each person eat?
There are 32 ears of corn for 16 people how many ears of corn can each person eat? Ears of corn per person = Ears of Corn / Number of people Ears of corn per person = 32/16 Ears of corn per person = [B]2[/B]

There are 32 female performers in a dance recital. The ratio of men to women is 3:8. How many men ar
There are 32 female performers in a dance recital. The ratio of men to women is 3:8. How many men are in the dance recital? 3:8 = x:32 3/8 = x/32 Cross multiply: 8x = 96 Divide each side by 8 x = 12 Check our work: 12:32 Divide each part by 4 12/4 = 3 and 32/4 = 8 so we have 3:8 :)

There are 32 students in a class. Nine of those students are women. What percent are men
There are 32 students in a class. Nine of those students are women. What percent are men [U]Find the number of male students:[/U] Males = Total Students - Females Males = 32 - 9 Males = 23 [U]Calculate percentage of males:[/U] Percentage of males = 100% * Males / Total Students Percentage of males = 100% * 23 / 32 Percentage of males = 100% * 0.71875 Percentage of males = 71.88% [URL='https://www.mathcelebrity.com/perc.php?num=23&den=32&pcheck=1&num1=16&pct1=80&pct2=70&den1=80&idpct1=10&hltype=1&idpct2=90&pct=82&decimal=+65.236&astart=12&aend=20&wp1=20&wp2=30&pl=Calculate']See this link as well[/URL]

There are 33 students in an Algebra I class. There are 7 fewer girls than boys. How many girls are i
There are 33 students in an Algebra I class. There are 7 fewer girls than boys. How many girls are in the class? Let b be the number of boys and g be the number of girls. We are given 2 equations: [LIST=1] [*]g = b - 7 [*]b + g = 33 [/LIST] Substitute (1) into (2): b + (b - 7) = 33 Combine like terms: 2b - 7 = 33 [URL='https://www.mathcelebrity.com/1unk.php?num=2b-7%3D33&pl=Solve']Typing this equation into our search engine[/URL], we get b = 20. Since the problem asks for how many girls (g) we have, we substitute b = 20 into Equation (1): g = 20 - 7 [B]g = 13[/B]

There are 35 peanuts in every bowl. How many peanuts are there in 5 bowls? How about in 7 bowls?
There are 35 peanuts in every bowl. How many peanuts are there in 5 bowls? How about in 7 bowls? [LIST] [*]5 bowls: 35 peanuts per bowl * 5 bowls = [B]175 peanuts[/B] [*]7 bowls: 35 peanuts per bowl * 7 bowls = [B]245 peanuts[/B] [/LIST]

There are 35 peanuts in every bowl. How many peanuts are there in 5 bowls? How about in 7 bowls?
There are 35 peanuts in every bowl. How many peanuts are there in 5 bowls? How about in 7 bowls? [LIST] [*]35 peanuts per bowl x 5 bowls = [B]175 peanuts[/B] [*]35 peanuts per bowl x 7 bowls = [B]245 peanuts[/B] [/LIST]

There are 377 baseball teams at the tournament and each team has 228 players. How many players are a
There are 377 baseball teams at the tournament and each team has 228 players. How many players are at the tournament? Key words are "There are", "each team", and "how many". We multiply teams by players per team to get number of players. 377 * 228 = [B]85,956[/B]

There are 4 fewer peaches than lemons on a table. If there are x lemons, how many peaches are there?
There are 4 fewer peaches than lemons on a table. If there are x lemons, how many peaches are there? Fewer means subtract: [B]x - 4[/B]

There are 4 people at a party. Each person brings one gift for each other person. What is the total
There are 4 people at a party. Each person brings one gift for each other person. What is the total number of gifts at the party? Each person brings 3 gifts, 1 for each person other than themselves. 4 people x 3 gifts each = [B]12 total gifts[/B]

There are 40 grams in 5 prunes. How much gram of weight is in 34 prunes
There are 40 grams in 5 prunes. How much gram of weight is in 34 prunes? Set up a proportion of grams to prunes where g is the number of grams in 34 prunes: 40/5 = g/34 [URL='https://www.mathcelebrity.com/prop.php?num1=40&num2=g&den1=5&den2=34&propsign=%3D&pl=Calculate+missing+proportion+value']Typing this proportion of 40/5 = g/34 into our search engine[/URL], we get: [B]g = 272[/B]

There are 4064 calories in 8 pints of strawberry icecream. How many calories are ther in each pint o
There are 4064 calories in 8 pints of strawberry ice cream. How many calories are there in each pint of strawberry ice cream? Set up a proportion using x as the number of calories in 1 pint of ice cream. 4064/8 = x/1 Using our [URL='http://www.mathcelebrity.com/prop.php?num1=4064&num2=x&den1=8&den2=1&propsign=%3D&pl=Calculate+missing+proportion+value']proportion calculator[/URL], we get: x = [B]508[/B]

There are 480 calories per hour. An officer swims 1.5 hours for 30 days. How many calories is that?
There are 480 calories per hour. An officer swims 1.5 hours for 30 days. How many calories is that? 1.5 hours per day times 30 days = 45 total hours. 480 calories per hour times 45 total hours = [B]21,600 total calories[/B].

There are 5 2/3 cups of milk. How many thirds are there?
Convert the mixed fraction [URL='http://www.mathcelebrity.com/fraction.php?frac1=5%262%2F3&frac2=3%2F8&pl=Simplify']using the calculator[/URL] to get 17/3.

There are 5 orange books, 12 black books, and 8 tan books on Mr. Johnsons bookshelf. Calculate the p
There are 5 orange books, 12 black books, and 8 tan books on Mr. Johnsons bookshelf. Calculate the probability of randomly selecting a black book and then a tan book without replacement. Write your answer as a percent. P(black book first draw) P(black book first draw) = 12 black / (5 orange + 12 black + 8 tan) P(black book first draw) = 12 / 25 P(tan book second draw) P(tan book second draw) = 8 tan / (5 orange + 11 black + 8 tan) <-- 11 black because we already drew one black P(tan book second draw) = 8 / 24 Using our fraction reduction calculator, this simplifies to 1/3 Since each draw is independent, we multiply both probabilities: P(black book first draw, tan book second draw) = 12/25 * 1/3 P(black book first draw, tan book second draw) = 12/75 P(black book first draw, tan book second draw) = [B]16%[/B]

There are 5 pencil-cases on the desk. Each pencil-case contains at least 10 pencils, but not more th
[SIZE=4]There are 5 pencil-cases on the desk. Each pencil-case contains at least 10 pencils, but not more than 14 pencils. Which of the following could be the total number of pencils in all 5 cases? A) 35 B) 45 C) 65 D) 75 [U]Determine the minimum amount of pencils (At least means greater than or equal to):[/U] Minimum Amount of pencils = Cases * Min Quantity Minimum Amount of pencils = 5 * 10 Minimum Amount of pencils = 50 [SIZE=4][U]Determine the maximum amount of pencils (Not more than means less than or equal to):[/U] Maximum Amount of pencils = Cases * Min Quantity Maximum Amount of pencils = 5 * 14 Maximum Amount of pencils = 70[/SIZE] So our range of pencils (p) is: 50 <= p <= 70 Now take a look at our answer choices. The only answer which fits in this inequality range is [B]C, 65[/B]. [B][/B][/SIZE]

There are 5 red and 4 black balls in a box. If you pick out 2 balls without replacement, what is the
There are 5 red and 4 black balls in a box. If you pick out 2 balls without replacement, what is the probability of getiing at least one red ball? First list out our sample space. At least one means 1 or 2 red balls, so we have 3 possible draws: [LIST=1] [*]Red, Black [*]Black, Red [*]Red, Red [/LIST] List out the probabilities: [LIST=1] [*]Red (5/9) * Black (4/8) = 5/18 [*]Black (4/9) * Red (5/8) = 5/18 [*]Red (5/9) * Red (4/8) = 5/18 [/LIST] Add these up: 3(5)/18 = [B]5/6[/B]

There are 50 pairs of pants. One-half of the pants are black. One-fifth of the pants are tan. How ma
There are 50 pairs of pants. One-half of the pants are black. One-fifth of the pants are tan. How many pairs of pants are not black or tan. First, determine what fraction of pants are black and tan: 1/2 + 1/5 Using our [URL='https://www.mathcelebrity.com/fraction.php?frac1=1%2F2&frac2=1%2F5&pl=Add']fraction addition calculator[/URL], we get 7/10. So the rest of the pants are 1 - 7/10. 1 can be written as 10/10. So we have 10/10 - 7/10 = 3/10 3/10 * 50 = 150/10 = [B]15[/B]

There are 6 women and 5 men in a department. How many ways can a committee of 2 women and 2 men be s
There are 6 women and 5 men in a department. How many ways can a committee of 2 women and 2 men be selected? We want 6C2 * 5C2 using combinations. [LIST] [*]Using our [URL='http://www.mathcelebrity.com/permutation.php?num=6&den=2&pl=Combinations']combinations calculator[/URL], 6C2 = 15 [*]Using our [URL='http://www.mathcelebrity.com/permutation.php?num=5&den=2&pl=Combinations']combinations calculator[/URL], 5C2 = 10 [/LIST] Our answer is 15 * 10 = [B]150[/B]

There are 60 rocks placed equally into 3 jars . How many rocks are in each jar?
There are 60 rocks placed equally into 3 jars . How many rocks are in each jar? 60 rocks / 3 jars = [B]20 rocks per jar[/B]

There are 60 students in a class. Three-fourths of them are girls. How many boys are there
There are 60 students in a class. Three-fourths of them are girls. How many boys are there In the class, we have either boys or girls. Total students = 60 3/4 of 60 = 60 * 3/4 Since 60/4 = 15, we have: 15 * 3 = 45 girls Boys = 60 - girls Boys = 60 - 45 Boys = [B]15[/B]

There are 63 students in middle school chorus. There are 11 more boys than girls. How many boys x an
There are 63 students in middle school chorus. There are 11 more boys than girls. How many boys x and girls y are in the chorus? Set up equations: [LIST=1] [*]x + y = 63 [*]x = y + 11 [/LIST] Substitute (1) into (2) y + 11 + y = 63 2y + 11 = 63 Use our [URL='http://www.mathcelebrity.com/1unk.php?num=2y%2B11%3D63&pl=Solve']equation solver[/URL]: [B]y = 26[/B]

There are 64 members in the history club. 11 less than half of the members are girls. How many membe
There are 64 members in the history club. 11 less than half of the members are girls. How many members are boys? Set up two equations where b = the number of boys and g = the number of girls [LIST=1] [*]b + g = 64 [*]1/2(b + g) - 11 = g [/LIST] Substitute (1) for b + g into (2) 1/2(64) - 11 = g 32 - 11 = g [B]g = 21[/B] Substitute g = 21 into (1) b + 21 = 64 Using our [URL='http://www.mathcelebrity.com/1unk.php?num=b%2B21%3D64&pl=Solve']equation calculator[/URL], we get: [B]b = 43[/B]

There are 7 more jeeps than vans.
There are 7 more jeeps than vans. [U]Define variables[/U] [LIST] [*]Let j be the number of jeeps [*]Let v be the number of vans [/LIST] 7 more jeeps than vans means we add 7 to the number of vans: [B]j = v + 7[/B]

There are 72 boys and 90 girls on the Math team For the next math competition Mr Johnson would like
There are 72 boys and 90 girls on the Math team For the next math competition Mr Johnson would like to arrange all of the students in equal rows with only girls or boys in each row with only girls or boys in each row. What is the greatest number of students that can be put in each row? To find the maximum number (n) of boys or girls in each row, we want the GCF (Greatest Common Factor) of 72 and 90. [URL='https://www.mathcelebrity.com/gcflcm.php?num1=72&num2=90&num3=&pl=GCF+and+LCM']Using our GCF calculator for GCF(72,90)[/URL], we get 18. [LIST] [*]72 boys divided by 18 = [B]4 rows of boys[/B] [*]90 girls divided by 18 = [B]5 rows of girls[/B] [/LIST]

There are 76 milligrams of cholesterol in a 3.2 ounce serving of lobster. How much cholesterol is in
There are 76 milligrams of cholesterol in a 3.2 ounce serving of lobster. How much cholesterol is in a 6 ounce serving? Let x equal the amount of cholesterol in milligrams for a 6 ounce service. Set up a proportion: 76/3.2 = x/6 Using our [URL='http://www.mathcelebrity.com/prop.php?num1=76&num2=x&den1=3.2&den2=6&propsign=%3D&pl=Calculate+missing+proportion+value']proportion calculator[/URL] by plugging that expression into the search engine, we get x = 142.5

There are 8 lions, 4 tigers, 5 cheetahs, 6 giraffes, 7 hippos, and 78 monkeys at the City Zoo. If ea
There are 8 lions, 4 tigers, 5 cheetahs, 6 giraffes, 7 hippos, and 78 monkeys at the City Zoo. If each of the 4 zookeepers feeds the same number of animals, how many animals does each zookeeper feed? Calculate Total Animals: 8 + 4 + 5 + 6 + 7 + 78 = 108 Now divide 108 animals equally into 4 zookeepers 108/4 = [B]27 animals each zookeeper will feed[/B]

There are 812 students in a school. There are 36 more girls than boys. How many girls are there?
[SIZE=6]There are 812 students in a school. There are 36 more girls than boys. How many girls are there? Let b be boys Let g be girls We're given two equations:[/SIZE] [LIST=1] [*][SIZE=6]b + g = 812[/SIZE] [*][SIZE=6]g = b + 36[/SIZE] [/LIST] [SIZE=6]Rearrange equation 2 to subtract b from each side: [/SIZE] [LIST=1] [SIZE=6] [LIST][*]b + g = 812[/LIST] [LIST][*]-b + g = 36[/LIST][/SIZE] [/LIST] [SIZE=6]Add equation (1) to equation (2): b - b + 2g = 812 + 36 The b's cancel: 2g = 848 Divide each side by 2: 2g/2 = 848/2 g = [B]424[/B] [B][/B] To find b, we put g= 424 into equation 1: b + 424 = 812 b = 812 - 424 b = [B]388[/B] [MEDIA=youtube]JO1b7qVwWoI[/MEDIA] [/SIZE]

There are 85 students in a class, 40 of them like math,31 of them like science, 12 of them like both
There are 85 students in a class, 40 of them like math,31 of them like science, 12 of them like both, how many don't like either. We have the following equation: Total Students = Students who like math + students who like science - students who like both + students who don't like neither. Plug in our knowns, we get: 85 = 40 + 31 - 12 + Students who don't like neither 85 = 59 + Students who don't like neither Subtract 59 from each side, we get: Students who don't like neither = 85 - 59 Students who don't like neither = [B]26[/B]

there are 9 apples in each box. how many apples are in 6 boxes
there are 9 apples in each box. how many apples are in 6 boxes Total apples = Number of boxes * apples per box Total apples = 6 * 9 Total apples = [B]54[/B]

There are 9 buses, each of these buses is taking 35 people to a game. Another bus is taking 7 people
There are 9 buses, each of these buses is taking 35 people to a game. Another bus is taking 7 people. How many people are these buses taking to the game? We have 9 buses * 35 people each + another bus with 7 more people: 9(35) + 7 315 + 7 [B]322 people[/B]

There are cows and chickens in a barn along with a three-legged dog named Tripod. If there are twice
There are cows and chickens in a barn along with a three-legged dog named Tripod. If there are twice as many chickens as cows, how many legs are there in the barn. (Call the number of cows n.) Number of cows = n Number of cows legs = 4n Number of chickens = 2n Number of chicken legs = 2*2n = 4n Tripod legs = 3 Total legs = 4n + 4n + 3 [B]8n + 3[/B]

There are five consecutive numbers and the smallest is called n. What is the largest number called?
There are five consecutive numbers and the smallest is called n. What is the largest number called? List out consecutive numbers. Each consecutive number is found by adding 1 to the prior number [LIST=1] [*]n [*]n + 1 [*]n + 2 [*]n + 3 [*][B]n + 4[/B] [/LIST]

There are only horses and ducks on a farm. There are 80 animals in all and the number of ducks is ca
There are only horses and ducks on a farm. There are 80 animals in all and the number of ducks is called n. How many horse legs are there on the farm? Number of duck legs = 2 legs * n ducks = 2n legs Number of horses = 80 - n Legs per horse = 4 Total horse legs = 4(80 - n) = [B]320 - 4n[/B]

there are some red counters and some yellow counters in the ratio 1:5. There are 20 yellow counters
There are some red counters and some yellow counters in the ratio 1:5. There are 20 yellow counters in the bag. Set up a proportion where x is the amount of red counters to 20 yellow counters 1/5 = x/20 Enter that in the search engine and our [URL='http://www.mathcelebrity.com/prop.php?num1=1&num2=x&den1=5&den2=20&propsign=%3D&pl=Calculate+missing+proportion+value']proportion calculator[/URL] gives us: [B]x = 4[/B]

There are t pencils in a pack. I buy 4 packs. How many pencils?
There are t pencils in a pack. I buy 4 packs. How many pencils? [B]4t[/B]

There are thrice as many girls (g) as there are boys (b)
There are thrice as many girls (g) as there are boys (b) Thrice means we multiply by 3, so we have the following algebraic expression: [B]g = 3b[/B]

There are two bells in the school. Bell A rings every 2 minutes. Bell B rings every 3 minutes. If bo
There are two bells in the school. Bell A rings every 2 minutes. Bell B rings every 3 minutes. If both bells ring together at 8.02 p.m., when will they ring together again? Using our[URL='http://www.mathcelebrity.com/gcflcm.php?num1=2&num2=3&num3=&pl=LCM'] least common multiple calculator,[/URL] we find the LCM(2, 3) = 6. Which means the next time both bells ring together is 6 minutes from now. 8:02 p.m. + 6 minutes = [B]8:08 p.m.[/B]

There are two containers. One holds exactly 7 quarts and the other holds exactly 9 quarts. There are
There are two containers. One holds exactly 7 quarts and the other holds exactly 9 quarts. There are no markings on the containers that allow you to know when they contain one, two, three, four, five, six or eight quarts. You have a tub full of water and you can fill and empty the 7 and 9 quart container however you wish. How can you end up with exactly 8 quarts in the 9 quart container? [LIST=1] [*]Fill the 7-quart and pour it into the 9-quart [*]Fill the 7-quart and pour 2 quarts into the 9-quart. The 9-quart is filled and 5 quarts are remaining in the 7-quart [*]Empty the 9-quart [*]Pour the remaining 5 quarts that are in the 7-quart into the 9-quart [*]Fill the 7-quart and pour 4 quarts into the 9-quart, which will fill it. 3 quarts are remaining in the 7-quart [*]Empty the 9-quart [*]Pour the 3 quarts that are remaining in the 7-quart into the 9-quart container [*]Fill the 7-quart and pour 6 quarts into the 9-quart. This will fill it and leave 1 quart remaining in the 7-quart container [*]Empty the 9-quart [*]Pour the 1 quart from the 7-quart into the 9-quart [*]Fill the 7-quart and pour it into the 9-quart. There are now 8 quarts in the 9-quart container [/LIST]

There are two numbers. The sum of 4 times the first number and 3 times the second number is 24. The
There are two numbers. The sum of 4 times the first number and 3 times the second number is 24. The difference between 2 times the first number and 3 times the second number is 24. Find the two numbers. Let the first number be x and the second number be y. We have 2 equations: [LIST=1] [*]4x + 3y = 24 [*]2x - 3y = 24 [/LIST] Without doing anything else, we can add the 2 equations together to eliminate the y term: (4x + 2x) + (3y - 3y) = (24 + 24) 6x = 48 Divide each side by 6: [B]x = 8 [/B] Substitute this into equation (1) 4(8) + 3y = 24 32 + 3y = 24 [URL='https://www.mathcelebrity.com/1unk.php?num=32%2B3y%3D24&pl=Solve']Type 32 + 3y = 24 into our search engine[/URL] and we get [B]y = 2.6667[/B].

There are y horses and z chickens in a barn. How many legs are there in the barn?
There are y horses and z chickens in a barn. How many legs are there in the barn? [U]For total legs, we have:[/U] Total Legs = Horse Legs + Chicken Legs Total Legs = Legs per horse * number of horses + Legs per chicken * number of chickens [U]Horses have 4 legs and chickens have 2 legs. We have y horses and z chickens. Plugging this in, we have:[/U] Total Legs = [B]4y + 2z[/B]

There is a bag filled with 3 blue, 4 red and 5 green marbles. A marble is taken at random from the
There is a bag filled with 3 blue, 4 red and 5 green marbles. A marble is taken at random from the bag, the colour is noted and then it is not replaced. Another marble is taken at random. What is the probability of getting exactly 1 green? Calculate Total marbles Total marbles = Blue + Red + Green Total marbles = 3 + 4 + 5 Total marbles = 12 Probability of a green = 5/12 Probability of not green = 1 - 5/12 = 7/12 To get exactly one green in two draws, we either get a green, not green, or a not green, green [U]First Draw Green, Second Draw Not Green[/U] [LIST] [*]1st draw: Probability of a green = 5/12 [*]2nd draw: Probability of not green = 7/11 <-- 11 since we did not replace the first marble [*]To get the probability of the event, since each draw is independent, we multiply both probabilities [*]Probability of the event is (5/12) * (7/11) = 35/132 [/LIST] [U]First Draw Not Green, Second Draw Not Green[/U] [LIST] [*]1st draw: Probability of not a green = 7/12 [*]2nd draw: Probability of not green = 5/11 <-- 11 since we did not replace the first marble [*]To get the probability of the event, since each draw is independent, we multiply both probabilities [*]Probability of the event is (7/12) * (5/11) = 35/132 [/LIST] To get the probability of exactly one green, we add both of the events: First Draw Green, Second Draw Not Green + First Draw Not Green, Second Draw Not Green 35/132 + 35/132 = 70/132 [URL='https://www.mathcelebrity.com/fraction.php?frac1=70%2F132&frac2=3%2F8&pl=Simplify']Using our fraction simplify calculator[/URL], we get: [B]35/66[/B]

There is a bag filled with 4 blue, 3 red and 5 green marbles. A marble is taken at random from the
There is a bag filled with 4 blue, 3 red and 5 green marbles. A marble is taken at random from the bag, the colour is noted and then it is replaced. Another marble is taken at random. What is the probability of getting 2 blues? We have (4 blue + 3 red + 5 green) = 12 total marbles With replacement, the probability of getting one blue is 4/12 = 1/3 Since each draw is independent of the last, the probability of Blue, Blue = 1/3 * 1/3 = [B]1/9[/B]

There is a bag filled with 5 blue, 6 red and 2 green marbles. A marble is taken at random from the b
There is a bag filled with 5 blue, 6 red and 2 green marbles. A marble is taken at random from the bag, the colour is noted and then it is replaced. Another marble is taken at random. What is the probability of getting exactly 1 blue? Find the total number of marbles in the bag: Total marbles = 5 blue + 6 red + 2 green Total marbles = 13 The problem asks for exactly one blue in 2 draws [I]with replacement[/I]. Which means you could draw as follows: Blue, Not Blue Not Blue, Blue The probability of drawing a blue is 5/13, since we replace the marbles in the bag each time. The probability of not drawing a blue is (6 + 2)/13 = 8/13 And since each of the 2 draws are independent of each other, we multiply the probability of each draw: Blue, Not Blue = 5/13 * 8/13 =40/169 Not Blue, Blue = 8/13 * 5/13 = 40/169 We add both probabilities since they both count under our scenario: 40/169 + 40/169 = 80/169 Checking our [URL='https://www.mathcelebrity.com/fraction.php?frac1=80%2F169&frac2=3%2F8&pl=Simplify']fraction simplification calculator[/URL], we see you cannot simplify this fraction anymore. So our probability stated in terms of a fraction is 80/169 [URL='https://www.mathcelebrity.com/perc.php?num=80&den=169&pcheck=1&num1=16&pct1=80&pct2=70&den1=80&idpct1=10&hltype=1&idpct2=90&pct=82&decimal=+65.236&astart=12&aend=20&wp1=20&wp2=30&pl=Calculate']Stated in terms of a decimal[/URL], it's 0.4734

There is a ratio of 5 girls to 3 boys in the chorus. There are 24 boys in the chorus.How many girls
There is a ratio of 5 girls to 3 boys in the chorus. There are 24 boys in the chorus.How many girls are in the chorus? Set up a proportion of girls to boys: 5/3 = g/24 where g is the number of girls for 24 boys. Typing 5/3 = g/24 into the [URL='http://www.mathcelebrity.com/prop.php?num1=5&num2=g&den1=3&den2=24&propsign=%3D&pl=Calculate+missing+proportion+value']math tutoring calculator[/URL] gives us [B]g = 40[/B]. [MEDIA=youtube]c-xshqvfvig[/MEDIA]

There is a sales tax of $15 on an item that costs $153 before tax. A second item costs $81.60 before
There is a sales tax of $15 on an item that costs $153 before tax. A second item costs $81.60 before tax. What is the sales tax on the second item? We assume the goods are bought in the same store, so tax rates are the same: Tax Rate = Tax Amount / Cost before tax Tax Rate = 15/153 Tax Rate = 0.098 or 9.8% Calculate sales tax on the second item Sales Tax = Cost before Tax * Tax Rate Sales Tax = 81.60 * 0.098 Sales Tax = 7.9968 We round to 2 decimals for dollars and cents and we get: Sales Tax = [B]$8.00[/B]

There is a sales tax of $4 on an item that cost $54 before tax. The sales tax on a second item is $1
There is a sales tax of $4 on an item that cost $54 before tax. The sales tax on a second item is $14. How much does the second item cost before tax? Sales Tax on First Item = Tax Amount / Before Tax Sale Amount Sales Tax on First Item = 4/54 Sales Tax on First Item = 0.07407407407 For the second item, let the before tax sale amount be b. We have: 0.07407407407b = 14 To solve this equation for b, we [URL='https://www.mathcelebrity.com/1unk.php?num=0.07407407407b%3D14&pl=Solve']type it in our search engine[/URL] and we get: b = [B]189[/B]

There is a sales tax of $5 on an item that costs $51 before tax. A second item costs $173.40 before
There is a sales tax of $5 on an item that costs $51 before tax. A second item costs $173.40 before tax. What is the sales tax on the second item? Calculate the sales tax percent using the first item: Sales Tax Decimal = 100% * Sales Tax / Pre-Tax Bill Sales Tax Decimal = 100% * 5/51 Sales Tax Decimal = 0.098 Calculate the sales tax on the second item: Sales Tax = Pre-Tax bill * (1 + Sales Tax) Sales Tax = $173.40 (1 + 0.098) Sales Taax = $173.40 * 1.098 Sales Tax = [B]$190.39[/B]

There is a stack of 10 cards, each given a different number from 1 to 10. suppose we select a card r
There is a stack of 10 cards, each given a different number from 1 to 10. Suppose we select a card randomly from the stack, replace it, and then randomly select another card. What is the probability that the first card is an odd number and the second card is greater than 7. First Event: P(1, 3, 5, 7, 9) = 5/10 = 1/2 or 0.5 Second Event: P(8, 9, 10) = 3/10 or 0.3 Probability of both events since each is independent is 1/2 * 3/10 = 3/20 = [B]0.15 or 15%[/B]

There is an escalator that is 1090.3 feet long and drops a vertical distance of 193.4 feet. What is
There is an escalator that is 1090.3 feet long and drops a vertical distance of 193.4 feet. What is its angle of depression? The sin of the angle A is the length of the opposite side / hypotenuse. sin(A) = Opposite / Hypotenuse sin(A) = 193.4 / 1090/3 sin(A) = 0.1774 [URL='https://www.mathcelebrity.com/anglebasic.php?entry=0.1774&pl=arcsin']We want the arcsin(0.1774)[/URL]. [B]A = 10.1284[/B]

There was 35 balloons at the beginning of a party. By the end of the party, n of them had popped. Us
There was 35 balloons at the beginning of a party. By the end of the party, n of them had popped. Using n, write an expression for the number of balloons that were left. We start with 35, we take away or subtract n that popped. We're left with: [B]35 - n[/B]

There were 175 tickets sold for the upcoming event in the gym. If students tickets cost $5 and adult
There were 175 tickets sold for the upcoming event in the gym. If students tickets cost $5 and adult tickets are $8, tell me how many tickets were sold if gate receipts totaled $1028? Let s be the number of student tickets and a be the number of adult tickets. We are given: a + s = 175 8a + 5s = 1028 There are 3 ways to solve this, all of which give us: [B]a = 51 s = 124 [/B] [URL='https://www.mathcelebrity.com/simultaneous-equations.php?term1=a+%2B+s+%3D+175&term2=8a+%2B+5s+%3D+1028&pl=Substitution']Substitution Method[/URL] [URL='https://www.mathcelebrity.com/simultaneous-equations.php?term1=a+%2B+s+%3D+175&term2=8a+%2B+5s+%3D+1028&pl=Elimination']Elimination Method[/URL] [URL='https://www.mathcelebrity.com/simultaneous-equations.php?term1=a+%2B+s+%3D+175&term2=8a+%2B+5s+%3D+1028&pl=Cramers+Method']Cramers Method[/URL]

There were 286,200 graphic designer jobs in a country in 2010. It has been projected that there will
There were 286,200 graphic designer jobs in a country in 2010. It has been projected that there will be 312,500 graphic designer jobs in 2020. (a) Using the data, find the number of graphic designer jobs as a linear function of the year. [B][U]Figure out the linear change from 2010 to 2020[/U][/B] Number of years = 2020 - 2010 Number of years = 10 [B][U]Figure out the number of graphic designer job increases:[/U][/B] Number of graphic designer job increases = 312,500 - 286,200 Number of graphic designer job increases = 26,300 [B][U]Figure out the number of graphic designer jobs added per year[/U][/B] Graphic designer jobs added per year = Total Number of Graphic Designer jobs added / Number of Years Graphic designer jobs added per year = 26,300 / 10 Graphic designer jobs added per year = 2,630 [U][B]Build the linear function for graphic designer jobs G(y) where y is the year:[/B][/U] G(y) = 286,200 + 2,630(y - 2010) [B][U]Multiply through and simplify:[/U][/B] G(y) = 286,200 + 2,630(y - 2010) G(y) = 286,200 + 2,630y - 5,286,300 [B]G(y) = 2,630y - 5,000,100[/B]

There were 500 balloons in 2 buckets. Rahul bursted 350 balloons of 1st bucket and Raghav bursted 4
There were 500 balloons in 2 buckets. Rahul bursted 350 balloons of 1st bucket and Raghav bursted 4 by 5 of the balloons of 2nd bucket. Who bursted more balloons and how many did Raghav burst? If Rahul bursted 350 balloons, then we have: Remaining Balloons = 500 - 350 Remaining Balloons = 150 Raghav bursted [URL='https://www.mathcelebrity.com/fraction.php?frac1=150&frac2=4/5&pl=Multiply']4/5 * 150[/URL] = [B]120[/B] Since 350 for Rahul > 120 for Raghav, [B]Rahul bursted more balloons[/B]

thesumof9andanumber
We denoted a number using the arbitrary variable "x". The sum of 9 and x is written: x + 9 or 9 + x

Thin Lens Distance
Free Thin Lens Distance Calculator - Given two out of three items in the thin lens equation, this solves for the third.

Think of a number. Double the number. Subtract 6 from the result and divide the answer by 2. The quo
Think of a number. Double the number. Subtract 6 from the result and divide the answer by 2. The quotient will be 20. What is the number Let's call our number n. Double the number means we multiply n by 2: 2n Subtract 6 from the result means we subtract 6 from 2n: 2n - 6 Divide the answer by 2: (2n - 6)/2 We can simplify this as n - 3 The quotient will be 20. This means the simplified term above is set equal to 20: [B]n - 3 = 20 [/B] <-- This is our algebraic expression If you want to take it a step further, and solve for n in the algebraic expression above, we [URL='https://www.mathcelebrity.com/1unk.php?num=n-3%3D20&pl=Solve']type this expression into our calculator[/URL], and get: n = 23

Thirty is half of the sum of 4 and a number
Thirty is half of the sum of 4 and a number. The phrase [I]a number[/I] represents an arbitrary variable, let's call it x. The sum of 4 and a number: 4 + x Half of this sum means we divide by 2: (4 + x)/2 Set this equal to 30: [B](4 + x)/2 = 30[/B] <-- This is our algebraic expression

three coins are tossed.how many different ways can they fall?
three coins are tossed.how many different ways can they fall? [URL='https://www.mathcelebrity.com/cointoss.php?hts=+HTHTHH&hct=+2&tct=+1&fct=+5>=no+more+than&nmnl=+2&htpick=heads&tossct=3&montect=3&calc=5&pl=Calculate+Probability']8 outcomes using our coin toss calculator[/URL]

Three days before the day after tomorrow is Monday. What day is today?
Three days before the day after tomorrow is Monday. What day is today? List out days of the week: Sunday, Monday, Tuesday, Wednesday, Thursday, Friday, Saturday Three days before the day after tomorrow means we start 2 days from now, and go back three days to get to Monday. Which means it's a +1 day gain. Monday + 1 = [B]Tuesday[/B]

Three good friends are in the same algebra class, their scores on a recent test are three consecutiv
Three good friends are in the same algebra class, their scores on a recent test are three consecutive odd integers whose sum is 273. Find the score In our search engine, we type in [URL='https://www.mathcelebrity.com/sum-of-consecutive-numbers.php?num=3consecutiveintegerswhosesumis273&pl=Calculate']3 consecutive integers whose sum is 273[/URL] and we get: [B]90, 91, 92[/B]

Three more than 2x is greater than or equal to 1 and less than or equal to 11
This is a double inequality. Let's take it by pieces: Three more than 2x is denoted as 2x + 3. We add since we see the phrase, [I]more than[/I]. Because it's greater than or equal to 1, we have: 1 <= 2x + 3 Finally, that same phrase is [U]also[/U] less than or equal to 11. 2x + 3 <= 11. Piecing these two inequalities together, we have: 1 <= 2x + 3 <= 11.

Three more than 2x is greater than or equal to 1 and less than or equal to 11
This is a two-part inequality. Let's take it piece by piece. Three more than 2x means we add. 2x + 3 It's greater than or equal to 1, denoted below: 1 <= 2x + 3 It's also less than or equal to 11, denoted below 2x + 3 <= 11 Piece these two inequalities together: 1 <= 2x + 3 <= 11

Three ordinary dice are rolled. What is the probability that the results are all less than 5
Three ordinary dice are rolled. What is the probability that the results are all less than 5 Calculate individual die probabilities: [LIST] [*]Die 1 P(x < 5) = 4/6 = 2/3 [*]Die 2 P(x < 5) = 4/6 = 2/3 [*]Die 3 P(x < 5) = 4/6 = 2/3 [/LIST] Since each roll is independent, we have: P(Die 1 < 5, Die 2 < 5, Die 3 < 5) = 2/3 * 2/3 * 2/3 P(Die 1 < 5, Die 2 < 5, Die 3 < 5) = [B]8/27[/B]

Three out of every 7 houses in a neighborhood are painted white. There are 224 houses in the neighbo
Three out of every 7 houses in a neighborhood are painted white. There are 224 houses in the neighborhood. How many houses are white? [URL='https://www.mathcelebrity.com/fraction.php?frac1=224&frac2=3/7&pl=Multiply']3/7 of 224[/URL] = [B]96 houses are white[/B]

Three people can pick all the apples from 5 trees in 5 hours. How long will it take 5 people to pick
Three people can pick all the apples from 5 trees in 5 hours. How long will it take 5 people to pick all the apples from the 5 trees? Three people * 5 hours= 15 hours 15 hours / 5 people = [B]3 hours[/B]

Three people went to lunch and bought a large meal which they all split. The total cost, including t
Three people went to lunch and bought a large meal which they all split. The total cost, including tip, was $30. Each person paid $10 to the waitress and started to leave the restaurant. As they left, the waitress came running up to them with five dollars saying that she made a mistake and that the meal and tip should have cost only $25. The waitress then gave each person one dollar, but didn't know how to split the remaining two dollars. They told her to keep the extra two dollars as an additional tip. When the people started talking about what had just happened, they started getting confused. They had each paid $10 for the meal and received one dollar back, so they each really paid $9 for the meal for a total of $27. Add the two dollars of extra tip and the total is $29. Where did the extra one dollar go? [B]The missing dollar is not really missing. The cost of the meal is really $27. The $25 plus the extra two dollar tip was given to the waitress -- $27 What we have is the cost ($27) plus the refund ($3) = $30. The $30 that was originally paid is accounted for as follows: Restaurant + regular waitress tip: $25 Three people: $3 (refund) Waitress: $2 (extra tip) $25 + $3 + $2 = $30[/B]

Three tennis balls each have a radius of 2 inches. They are put into a 12 inch high cylinder with a
Three tennis balls each have a radius of 2 inches. They are put into a 12 inch high cylinder with a 4 inch diameter. What is the volume of the space remaining in the cylinder? Volume of each ball is 4/3 ?r^3 V = 4/3 * 3.1415 * 2^3 V = 1.33 * 3.1415 * 8 = 33.41 cubic inches The volume of 3 balls is: V = 3(33.41) V = 100.23 Volume of the cylinder is area of circle times height: V = 3.14 * 2 * 2 * 1 = 150.72 Volume of remaining space is: V = Volume of cylinder - Volume of 3 balls V = 150.72 - 100.23 V = [B]50.49[/B]

Three x is five less than twice x
Twice x means we multiply x by 2: 2x five less than twice x 2x - 5 Three x 3x The word [I]is[/I] means equal to. Set 2x - 5 equal to 3x for our algebraic expression: [B]2x - 5 = 3x [/B] If the problem asks you to solve for x, subtract 2x from each side [B]x = -5[/B]

thrice the sum of x y and z
thrice the sum of x y and z The sum of x, y, and z x + y + z Thrice the sum means multiply by 3 [B]3(x + y + z)[/B]

thrice the sum of x y and z
thrice the sum of x y and z The sum of x, y, and z means we add all 3 variables together: x + y + z The word [I]thrice[/I] means we multiply the sum of x, y, and z by 3: 3(x + y +z)

thrice the sum of x y and z
thrice the sum of x y and z The sum of x, y, and z: x + y + z Thrice means multiply the sum by 3: [B]3(x + y + z)[/B]

Tickets for a concert were priced at $8 for students and $10 for nonstudents. There were 1340 ticket
Tickets for a concert were priced at $8 for students and $10 for nonstudents. There were 1340 tickets sold for a total of $12,200. How many student tickets were sold? Let s be the number of student tickets and n be the number of nonstudent tickets: [LIST=1] [*]n + s = 1340 [*]10n + 8s = 12200 [/LIST] Use our [URL='http://www.mathcelebrity.com/simultaneous-equations.php?term1=n+%2B+s+%3D+1340&term2=10n+%2B+8s+%3D+12200&pl=Cramers+Method']simultaneous equation calculator[/URL]: n = 740 [B]s = 600[/B]

Tickets to the amusement park cost $12 for adults and $8 for kids. Write on algebraic expression to
Tickets to the amusement park cost $12 for adults and $8 for kids. Write on algebraic expression to show the cost of a adult and k kids Since cost = price * quantity, we have: [B]12a + 8k[/B]

Tiffany is 59 years old. The sum of the ages of Tiffany and Maria is 91. How old is Maria?
Tiffany is 59 years old. The sum of the ages of Tiffany and Maria is 91. How old is Maria? Tiffany + Maria = 91 59 + Maria = 91 Subtract 59 from each side Maria = 91 - 59 [B]Maria = 32[/B]

Tim caught 6 ladybugs. Ali caught 7 and Jon caught 5. The children put some of their ladybugs in a b
Tim caught 6 ladybugs. Ali caught 7 and Jon caught 5. The children put some of their ladybugs in a big jar. The jar then had 8 ladybugs. How many ladybugs were not placed in the jar? Add up the total ladybugs caught: Total ladybugs caught = Tim's ladybugs + Ali's ladybugs + Jon's ladybugs Total ladybugs caught = 6 + 7 + 5 Total ladybugs caught = 18 If the jar has 8 ladybugs, then the amount not in the jar is: Amount of ladybugs not in the jar = Total ladybugs caught - Amount of ladybugs in the jar Amount of ladybugs not in the jar = 18 - 8 Amount of ladybugs not in the jar = [B]10[/B]

Time and Distance
charlie leaves home going 40 miles per hour. When charlie is 9 miles from home, Danny starts after charlie from the same place, going 55 miles per hour. How long does it take Danny to catch up charlie?

Time and Distance
Let h be the number of hours that pass when Charlie starts. We have the following equations: [LIST] [*]Charlie: D = 40h + 9 [*]Danny: D = 55h [/LIST] Set them equal to each other: 40h + 9 = 55h Subtract 40h from both sides: 15h = 9 h = 3/5 [B]3/5 of an hour = 3(60)/5 = 36 minutes[/B]

Time and Distance
Thank you so much [QUOTE="math_celebrity, post: 1003, member: 1"]Let h be the number of hours that pass when Charlie starts. We have the following equations: [LIST] [*]Charlie: D = 40h + 9 [*]Danny: D = 55h [/LIST] Set them equal to each other: 40h + 9 = 55h Subtract 40h from both sides: 15h = 9 h = 3/5 [B]3/5 of an hour = 3(60)/5 = 36 minutes[/B][/QUOTE]

Time Weighted Interest Method
Free Time Weighted Interest Method Calculator - Solves for Interest Rate based on 2 annual asset value events other than beginning or ending value using the Time Weighted Method

Time Zone Converter
Free Time Zone Converter Calculator - Converts a time from one time zone (timezone) to another.

Tina's mom made brownies. When tinas friend came over they ate 1/3 of the brownies. Her sister ate 2
Tina's mom made brownies. When tinas friend came over they ate 1/3 of the brownies. Her sister ate 2 and her dad ate 4. If there are 26 brownies left. How many did her mom make Let b denote the number of brownies Tina's mom made. We're given: b - 1/3b - 2 - 4 = 26 Combining like terms, we have: 2b/3 - 6 = 26 Add 6 to each side, we get: 2b/3 = 32 To solve this equation for b, we [URL='https://www.mathcelebrity.com/prop.php?num1=2b&num2=32&den1=3&den2=1&propsign=%3D&pl=Calculate+missing+proportion+value']type it in our math engine[/URL] and we get: b = [B]48[/B]

Tip
Free Tip Calculator - Calculates the total bill with Tip and how much each person owes if the bill is split evenly. Shows the amount of tip per person.

To be a member of world fitness gym, it costs $60 flat fee and $30 per month. Maria has paid a total
To be a member of world fitness gym, it costs $60 flat fee and $30 per month. Maria has paid a total of $210 for her gym membership so far. How long has Maria been a member to the gym? The cost function C(m) where m is the number of months for the gym membership is: C(m) = 30m + 60 We're given that C(m) = 210 for Maria. We want to know the number of months (m) that Maria has been a member. With C(m) = 210, we have: 30m + 60 =210 To solve this equation, [URL='https://www.mathcelebrity.com/1unk.php?num=30m%2B60%3D210&pl=Solve']we type it in our search engine[/URL] and we get: m = [B]5[/B]

To buy a minivan you can pay $12,500 cash or put down $5000 and make 24 monthly payments of $698.05.
To buy a minivan you can pay $12,500 cash or put down $5000 and make 24 monthly payments of $698.05. How much would you save by paying cash? [U]Calculate the total amount with payments:[/U] Total Amount with payments = Payment Amount * Total Payments Total Amount with payments = $698.05 * 24 Total Amount with payments = $16,753.20 [U]Calculate the total amount saved by paying cash:[/U] Savings by paying cash = Total Amount with payments - Cash Payment Savings by paying cash = $16,753.20 - $12,500 Savings by paying cash = [B]$4,253.20[/B]

To convert from Celsius to Fahrenheit, multiply by 1.8 and add 32. Write a formula to describe this
To convert from Celsius to Fahrenheit, multiply by 1.8 and add 32. Write a formula to describe this relationship. Given C as Celsius and F as Fahrenheit, we have the following equation: [B]F = 1.8C + 32[/B]

To create an entry code, you must first choose 2 letters and then, 4 single-digit numbers. How ma
To create an entry code, you must first choose 2 letters and then, 4 single-digit numbers. How many different entry codes can you create? List total combinations using the product of all possibilities: 26 letters (A - Z) * 26 letters (A - Z) * 10 digits (0-9) * 10 digits (0-9) * 10 digits (0-9) * 10 digits (0-9) [B]6,760,000 entry codes [MEDIA=youtube]Y23EGnVuU7I[/MEDIA][/B]

To make an international telephone call, you need the code for the country you are calling. The code
To make an international telephone call, you need the code for the country you are calling. The code for country A, country B, and C are three consecutive integers whose sum is 90. Find the code for each country. If they are three consecutive integers, then we have: [LIST=1] [*]B = A + 1 [*]C = B + 1, which means C = A + 2 [*]A + B + C = 90 [/LIST] Substitute (1) and (2) into (3) A + (A + 1) + (A + 2) = 90 Combine like terms 3A + 3 = 90 Using our [URL='http://www.mathcelebrity.com/1unk.php?num=3a%2B3%3D90&pl=Solve']equation calculator[/URL], we get: [B]A = 29[/B] Which means: [LIST] [*]B = A + 1 [*]B = 29 + 1 [*][B]B = 30[/B] [*]C = A + 2 [*]C = 29 + 2 [*][B]C = 31[/B] [/LIST] So we have [B](A, B, C) = (29, 30, 31)[/B]

To rent a building for a school dance, Ava paid 120 plus 2.50 for each student. To attend the school
To rent a building for a school dance, Ava paid 120 plus 2.50 for each student. To attend the school all together Ava paid 325. How many students attended the dance? Let the number of students be s. We're given 2.50s + 120 = 325 [URL='https://www.mathcelebrity.com/1unk.php?num=2.50s%2B120%3D325&pl=Solve']Type this equation into our search engine[/URL], and we get: s = [B]82[/B]

To rent a car it costs $12 per day and $0.50 per kilometer traveled. If a car were rented for 5 days
To rent a car it costs $12 per day and $0.50 per kilometer traveled. If a car were rented for 5 days and the charge was $110.00, how many kilometers was the car driven? Using days as d and kilometers as k, we have our cost equation: Rental Charge = $12d + 0.5k We're given Rental Charge = 110 and d = 5, so we plug this in: 110 = 12(5) + 0.5k 110 = 60 + 0.5k [URL='https://www.mathcelebrity.com/1unk.php?num=60%2B0.5k%3D110&pl=Solve']Plugging this into our equation calculator[/URL], we get: [B]k = 100[/B]

To ship a package with UPS, the cost will be $7 for the first pound and $0.20 for each additional po
To ship a package with UPS, the cost will be $7 for the first pound and $0.20 for each additional pound. To ship a package with FedEx, the cost will be $5 for the first pound and $0.30 for each additional pound. How many pounds will it take for UPS and FedEx to cost the same? If you needed to ship a package that weighs 8 lbs, which shipping company would you choose and how much would you pay? [U]UPS: Set up the cost function C(p) where p is the number of pounds:[/U] C(p) = Number of pounds over 1 * cost per pounds + first pound C(p) = 0.2(p - 1) + 7 [U]FedEx: Set up the cost function C(p) where p is the number of pounds:[/U] C(p) = Number of pounds over 1 * cost per pounds + first pound C(p) = 0.3(p - 1) + 5 [U]When will the costs equal each other? Set the cost functions equal to each other:[/U] 0.2(p - 1) + 7 = 0.3(p - 1) + 5 0.2p - 0.2 + 7 = 0.3p - 0.3 + 5 0.2p + 6.8 = 0.3p + 4.7 To solve this equation for p, we [URL='https://www.mathcelebrity.com/1unk.php?num=0.2p%2B6.8%3D0.3p%2B4.7&pl=Solve']type it in our search engine[/URL] and we get: p = [B]21 So at 21 pounds, both UPS and FedEx costs are equal [/B] Now, find out which shipping company has a better rate at 8 pounds: [U]UPS:[/U] C(8) = 0.2(8 - 1) + 7 C(8) = 0.2(7) + 7 C(8) = 1.4 + 7 C(8) = 8.4 [U]FedEx:[/U] C(8) = 0.3(8 - 1) + 5 C(8) = 0.3(7) + 5 C(8) = 2.1 + 5 C(8) = [B]7.1[/B] [B]Therefore, FedEx is the better cost at 8 pounds since the cost is lower[/B] [B][/B]

Today a car is valued at $42000. the value is expected to decrease at a rate of 8% each year. what i
Today a car is valued at $42000. the value is expected to decrease at a rate of 8% each year. what is the value of the car expected to be 6 years from now. Depreciation at 8% per year means it retains (100% - 8%) = 92% of it's value. We set up our depreciation function D(t), where t is the number of years from right now. D(t) = $42,000(0.92)^t The problem asks for D(6): D(6) = $42,000(0.92)^6 D(6) = $42,000(0.606355) D(6) = [B]$25,466.91[/B]

Today is my birthday! Four-fifths of my current age is greater than three-quarters of my age one yea
Today is my birthday! Four-fifths of my current age is greater than three-quarters of my age one year from now. Given that my age is an integer number of years, what is the smallest my age could be? Let my current age be a. We're given: 4/5a > 3/4(a + 1) Multiply through on the right side: 4a/5 > 3a/4 + 3/4 Let's remove fractions by multiply through by 5: 5(4a/5) > 5(3a/4) + 5(3/4) 4a > 15a/4 + 15/4 Now let's remove the other fractions by multiply through by 4: 4(4a) > 4(15a/4) + 4(15/4) 16a > 15a + 15 [URL='https://www.mathcelebrity.com/1unk.php?num=16a%3E15a%2B15&pl=Solve']Typing this inequality into our search engine[/URL], we get: a > 15 This means the smallest [I]integer age[/I] which the problem asks for is: 15 + 1 = [B]16[/B]

Todd bought 5 ice cream sandwiches for $3.75. Bryce bought one ice cream sandwich for $1.00. Who got
Todd bought 5 ice cream sandwiches for $3.75. Bryce bought one ice cream sandwich for $1.00. Who got the better deal? Todd's unit cost is found by: Todd's Unit Cost = Total Price / Total Ice Cream Sandwiches Todd's Unit Cost = $3.75/5 Todd's Unit Cost = $0.75 Bryce's unit cost is $1.00 per ice cream sandwich, so [B]Todd got the better deal.[/B]

todd has a bag of 150 pieces of candy. every weekday he eats 2 pieces and 3 pieces on weekends until
todd has a bag of 150 pieces of candy. every weekday he eats 2 pieces and 3 pieces on weekends until he has no more left. So each week, he eats 2*5 + 3*2 = 10 + 6 = 16 pieces 16 per week * 9 weeks = 144 pieces 150 - 144 = 6 pieces left 3 weekdays * 2 pieces per weekday = 6. So, Todd ate all the candy in 9 weeks, 3 days.

Together Sarah and Jasmine have collected 37 bugs. If Sarah collected 18 bugs, how many did Jasmine
Together Sarah and Jasmine have collected 37 bugs. If Sarah collected 18 bugs, how many did Jasmine collect? 37 - 18 = [B]19[/B]

Tom and Damian are shooting free throws. If Tom makes 7 free throws in 9 attempts, and Damian makes
Tom and Damian are shooting free throws. If Tom makes 7 free throws in 9 attempts, and Damian makes 5 free throws in 6 attempts, who has the higher relative performance? Answer this by calculating and comparing the free throw percentages of Tom and Damian. Tom makes 7/9. [URL='https://www.mathcelebrity.com/perc.php?num=7&den=9&pcheck=1&num1=16&pct1=80&pct2=70&den1=80&idpct1=10&hltype=1&idpct2=90&pct=82&decimal=+65.236&astart=12&aend=20&wp1=20&wp2=30&pl=Calculate']Using our percentage and decimal calculator[/URL], he makes 77.78% of the free throws. Damian makes 5/6. [URL='https://www.mathcelebrity.com/perc.php?num=+5&den=+6&pcheck=1&num1=16&pct1=80&pct2=70&den1=80&idpct1=10&hltype=1&idpct2=90&pct=82&decimal=+65.236&astart=12&aend=20&wp1=20&wp2=30&pof1=&pof2=&pl=Calculate']Using our percentage and decimal calculator[/URL], he makes 83.33% of the free throws. [B]Damian[/B] has a better free throw percentage.

Tom has a collection 21 CDs and Nita has a collection of 14 CDs. Tom is adding 3 cds a month to his
Tom has a collection 21 CDs and Nita has a collection of 14 CDs. Tom is adding 3 cds a month to his collection while Nita is adding 4 CDs a month to her collection. Find the number of months after which they will have the same number of CDs? Set up growth equations for the CDs where c = number of cds after m months Tom: c = 21 + 3m Nita: c = 14 + 4m Set the c equations equal to each other 21 + 3m = 14 + 4m Using our [URL='http://www.mathcelebrity.com/1unk.php?num=21%2B3m%3D14%2B4m&pl=Solve']equation calculator[/URL], we get [B]m = 7[/B]

Tom has x nickles. Express the value of the nickles in cents
Tom has x nickles. Express the value of the nickles in cents Since each nickel is worth 0.05, we have: [B]0.05x[/B]

Tom is 2 years older than Sue and Bill is twice as old as Tom. If you add all their ages and subtra
Tom is 2 years older than Sue and Bill is twice as old as Tom. If you add all their ages and subtract 2, the sum is 20. How old is Bill? Let t be Tom's age., s be Sue's age, and b be Bill's age. We have the following equations: [LIST=1] [*]t = s + 2 [*]b = 2t [*]s + t + b - 2 = 20 [/LIST] Get (2) in terms of s (2) b = 2(s + 2) <-- using (1), substitute for t So we have (3) rewritten with substitution as: s + (s + 2) + 2(s + 2) - 2 = 20 s + (s + 2) + 2s + 4 - 2 = 20 Group like terms: (s + s + 2s) + (2 + 4 - 2) = 20 4s + 4 = 20 Run this through our [URL='https://www.mathcelebrity.com/1unk.php?num=4s%2B4%3D20&pl=Solve']equation calculator [/URL]to get s = 4 Above, we had b = 2(s + 2) Substituting s = 4, we get: 2(4 + 2) = 2(6) = [B]12 Bill is 12 years old[/B]

Tom is the deli manager at a grocery store. He needs to schedule employees to staff the deli departm
Tom is the deli manager at a grocery store. He needs to schedule employees to staff the deli department at least 260 person-hours per week. Tom has one part-time employeewho works 20 hours per week. Each full-time employee works 40 hours per week. Write an inequality to determine n, the number of full-time employees Tom must schedule, so that his employees will work at least 260 person-hours per week. Set up the inequality: [LIST] [*]Add the part-timer's hours of 20 [*]Full time hours is 40 times n employees [*]At least means greater than or equal to, so we use the >= sign [/LIST] [B]40n + 20 >= 260[/B]

Tomás is a salesperson who earns a monthly salary of $2250 plus a 3% commission on the total amount
Tomás is a salesperson who earns a monthly salary of $2250 plus a 3% commission on the total amount of his sales. What were his sales last month if he earned a total of $4500? Let total sales be s. We're given the following earnings equation: 0.03s + 2250 = 4500 To solve for s, we [URL='https://www.mathcelebrity.com/1unk.php?num=0.03s%2B2250%3D4500&pl=Solve']type this equation into our search engine[/URL] and we get: s = [B]75,000[/B]

Tony has 6 cds that he is giving away. He lets his best friend choose 3 of the 6 cds. How many gr
Tony has 6 cds that he is giving away. He lets his best friend choose 3 of the 6 cds. How many groups of 3 cds are possible? This problem asks for [I]unique[/I] combinations. We want 6 choose 3, or 6C3. Go to the [URL='https://www.mathcelebrity.com/permutation.php?num=6&den=3&pl=Combinations']search engine, and type in 6C3[/URL], we get [B]20[/B] possible groups.

Torus
Free Torus Calculator - Calculates the volume of a torus and surface area of a torus given major radius and minor radius.

total of a number and the square of a number
total of a number and the square of a number The phrase [I]a number[/I] means an arbitrary variable, let's call it x. The square of a number means we raise x to the power of 2. x^2 The total means we add x squared to x: [B]x + x^2[/B]

Total Revenue
Free Total Revenue Calculator - Given a quantity, price, and item, this calculates the total revenue.

Trade Cost
Free Trade Cost Calculator - Calculates the saved hours under the electrician/carpenter model of specializing in jobs as well as opportunity cost.

Transitive Property of Equality
Free Transitive Property of Equality Calculator - Demonstrates the Transitive property of equality using a number. Numerical Properties

translate the product of -1 and a number in mathematics expression
translate the product of -1 and a number in mathematics expression The phrase [I]a number[/I] means an arbitrary variable. Let's call it x. The product of -1 and the number; [B]-x[/B]

Translate the sentence into an inequality. Twice y is less than 21.
Translate the sentence into an inequality. Twice y is less than 21. Twice y 2y Is less than 21 means we have an inequality: [B]2y < 21[/B]

Translate this phrase into an algebraic expression. 57 decreased by twice Mais savings Use the varia
Translate this phrase into an algebraic expression. 57 decreased by twice Mais savings Use the variable m to represent Mais savings. Twice means multiply by 2 2m 57 decreased by means subtract 2m from 57 [B]57 - 2m[/B]

Translate this phrase into an algebraic expression. 58 decreased by twice Gails age. Use the variabl
Translate this phrase into an algebraic expression. 58 decreased by twice Gails age. Use the variable g to represent Gails age. Twice Gail's age: 2g 58 decreased by twice Gail's age [B]58 - 2g[/B]

Translate this sentence into an equation. 43 is the sum of 17 and Gregs age. Use the variable g to
Translate this sentence into an equation. 43 is the sum of 17 and Gregs age. Use the variable g to represent Gregs age. The sum of 17 and Greg's age: g + 17 The word [I]is[/I] means equal to, so we set g + 17 equal to 43 [B]g + 17 = 43[/B] <-- This is our algebraic expression If you want to solve this equation for g, use our [URL='http://www.mathcelebrity.com/1unk.php?num=g%2B17%3D43&pl=Solve']equation calculator[/URL]. [B]g = 26[/B]

Translate this sentence into an equation. 48 is the difference of Ritas age and 11 . Use the variabl
Translate this sentence into an equation. 48 is the difference of Ritas age and 11 . Use the variable r to represent Ritas age. The difference of Rita's age and 11 is written: r - 11 The phrase [I]is[/I] means equal to, so we set r - 11 equal to 48 r - 11 = 48

Translate this sentence into an equation. 49 is the difference of Diegos age and 17. Use the variabl
Translate this sentence into an equation. 49 is the difference of Diegos age and 17. Use the variable d to represent Diegos age. The difference means we subtract, so we have d as Diego's age minus 17 d - 17 The word "is" means an equation, so we set d - 17 equal to 49 [B]d - 17 = 49[/B]

Translate this sentence into an equation. The difference of Maliks age and 15 is 63 Use the variable
Translate this sentence into an equation. The difference of Maliks age and 15 is 63 Use the variable m to represent Malik's age. [B]m - 15 = 63 [/B] To solve this equation, use our [URL='http://www.mathcelebrity.com/1unk.php?num=m-15%3D63&pl=Solve']equation calculator[/URL].

Translate to an inequality. The cost is smaller than $94,000
Translate to an inequality. The cost is smaller than $94,000. Let the cost be c. We have: [B]c<94,000[/B]

Trapezoids
Free Trapezoids Calculator - This calculator determines the following items for a trapezoid based on given inputs:
* Area of trapezoid
* Perimeter of a Trapezoid

Triangle Coordinate Items
Free Triangle Coordinate Items Calculator - Enter 3 points for the vertices of a triangle, and this will calculate the area of that triangle and the centroid.

Triangle Inequality
Free Triangle Inequality Calculator - This calculator displays 2 scenarios
1) Enter 3 sides of a triangle, and it will determine if the side lengths satisfy the properties of the triangle inequality and form a triangle
2) Enter 2 sides of a triangle, and this will determine an acceptable range for the length of the 3rd side of a triangle so that the 3rd side respects the Triangle Inequality.

Triangle KLM has vertices at . k(-2,-2), l(10,-2), m(4,4) What type of triangle is KLM?
Triangle KLM has vertices at . k(-2,-2), l(10,-2), m(4,4) What type of triangle is KLM? [URL='https://www.mathcelebrity.com/slope.php?xone=-2&yone=-2&slope=+2%2F5&xtwo=10&ytwo=-2&pl=You+entered+2+points']Side 1: KL[/URL] = 12 [URL='https://www.mathcelebrity.com/slope.php?xone=10&yone=-2&slope=+2%2F5&xtwo=4&ytwo=4&pl=You+entered+2+points']Side 2: LM[/URL] = 8.4853 [URL='https://www.mathcelebrity.com/slope.php?xone=-2&yone=2&slope=+2%2F5&xtwo=4&ytwo=4&pl=You+entered+2+points']Side 3: KM[/URL] = 6.3246 Then, we want to find the type of triangle. Using our [URL='https://www.mathcelebrity.com/tribasic.php?side1input=12&side2input=8.4853&side3input=6.3246&angle1input=&angle2input=&angle3input=&pl=Solve+Triangle']triangle solver with our 3 sides[/URL], we get: [B]Obtuse, Scalene[/B]

Triangle Solver and Classify Triangles
Free Triangle Solver and Classify Triangles Calculator - Solves a triangle including area using the following solving methods
Side-Angle-Side (SAS) Side Angle Side
Angle-Side-Angle (ASA) Angle Side Angle
Side-Side-Angle (SSA) Side Angle Side
Side-Side-Side (SSS) Side Side Side
Area (A) is solved using Herons Formula
Law of Sines
Law of Cosines

Also classifies triangles based on sides and angles entered.

triangle sum theorem
The triangle sum theorem states the sum of the three angles in a triangle equals 180 degrees. So if you're given two angles and need too find the 3rd angle, add the 2 known angles up, and subtract them from 180 to get the 3rd angle measure.

Triangle with perimeter
A triangle with a perimeter of 120. What degree are the three sides?

Triangles
Free Triangles Calculator - This lesson walks you through the basics of a triangle and shows you triangle types like acute, right, obtuse, scalene, isosceles, equilateral.

Triangular Number
Free Triangular Number Calculator - This calculator determines the nth triangular number. Generates composite numbers.

Trichotomy Property
Free Trichotomy Property Calculator - Demonstrates the Trichotomy Property with 2 numbers. Numerical Properties

Trig Measurement
Free Trig Measurement Calculator - Given an angle θ, this calculates the following measurements:
Sin(θ) = Sine
Cos(θ) = Cosine
Tan(θ) = Tangent
Csc(θ) = Cosecant
Sec(θ) = Secant
Cot(θ) = Cotangent
Arcsin(x) = θ = Arcsine
Arccos(x) = θ = Arccosine
Arctan(x) =θ = Arctangent
Also converts between Degrees and Radians and Gradians
Coterminal Angles as well as determine if it is acute, obtuse, or right angle. For acute angles, a cofunction will be determined. Also shows the trigonometry function unit circle

Trigonometry Relations
Free Trigonometry Relations Calculator - Calculates various trigonometry measurements (sin,cos,tan,csc,sec,cot) given other measurements that you enter.

Trimmed Mean and Winsorized Mean
Free Trimmed Mean and Winsorized Mean Calculator - Given a number set and a trimmed mean percentage, this will calculate the trimmed mean (truncated mean) or winsorized mean.

Trinomials
Free Trinomials Calculator - Checks to see if equations in the trinomial form ax2 + bx + c are a perfect square as well as completing the square from equations in the form ax2 + bx + ?. Also shows you a perfect square trinomial

triple 5, raise the result to the 10th power, then divide p by what you have
triple 5, raise the result to the 10th power, then divide p by what you have Triple 5, means multiply 5 by 3 3 * 5 --> Simplified, this is 15 Raise the result to the 10th power, means we raise 15 to the 10 power: 15^10 Then divide it by p: [B]15^10/p[/B]

triple a number and another number
triple a number and another number The phrase [I]a number[/I] means an arbitrary variable, let's call it x: x Triple a number means we multiply x by 3: 3x The phrase [I]another number[/I] means another arbitrary variable, let's call it y: y The word [I]and[/I] means we add y to 3x: [B]3x + y[/B]

triple c divide the result by a
triple c divide the result by a Take this algebraic expression in pieces. Triple c means we multiply the variable c by 3 3c Divide the result by a, means we take 3c, and divide by a [B]3c/a[/B]

triple c, multiply the result by a, then subtract b
triple c, multiply the result by a, then subtract b Triple c means we multiply c by 3: 3c Multiply the result by a means we multiply 3c by a 3ac Then, we subtract b from 3ac: [B]3ac - b[/B]

triple h then raise the result to the 8th power
triple h then raise the result to the 8th power [U]Triple h means we multiply h by 3:[/U] 3h [U]Raise the result to the 8th power:[/U] [B](3h)^8[/B]

triple s add the result to q then divide what you have by r
triple s add the result to q then divide what you have by r. Triple s means multiply s by 3: 3s Add the result to q: 3s + q Divide what you have by r: [B](3s + q)/r[/B]

triple the sum of 36 and 6 then add 4
triple the sum of 36 and 6 then add 4 Take this algebraic expression in parts: The sum of 36 and 6: 36 + 6 Triple the sum means we multiply the sum by 3: 3(36 + 6) Then add 4: [B]3(36 + 6) + 4[/B] If the problem asks you to simplify the algebraic expression, we have: 3(42) + 4 126 + 4 [B]130[/B]

Triple the sum of 4 and y
The sum of 4 and y is written as (4 + y) Triple that means we multiply that entire sum by 3. 3(4 + y)

Triple the sum of 7 and m
The sum of 7 and m is written as 7 + m Triple that means multiply by 3: 3(7 + m)

triple the sum of b and c
triple the sum of b and c The sum of b and c b + c Triple this sum 3(b + c)

triple the sum of y and six
The sum of y and six is denoted as: y + 6 We triple that sum by multiplying it by 3 3(y + 6)

triple the value of c plus 3 is 84
Triple the value of c means we multiply c by 3 3c Plus 3 means we add 3 3c + 3 Is, means equal to, so we set our expression equal to 84 [B]3c +3 = 84 [/B] If you want to solve that equation, use our [URL='http://www.mathcelebrity.com/1unk.php?num=3c%2B3%3D84&pl=Solve']equation solver[/URL]: c = 27

tripled square of the difference of a and b
The difference of a and b is written as: a - b Square the difference means raise the difference to the power of 2 (a - b)^2 Triple this expression means multiply by 3: [B]3(a - b)^2[/B]

Tristan is building a slide for his kids. The ladder is 6 feet tall and the slide is 10 feet long. W
Tristan is building a slide for his kids. The ladder is 6 feet tall and the slide is 10 feet long. What is the distance between the ladder and the bottom of the slide? The answer is 8. We have a 3-4-5 triangle. But it's scaled by 2. 3 * 2 = 6 5 * 2 = 10 (hypotenuse-slide) 4 * 2 = [B]8[/B]

Tristian bought an orange that was 1/5 pound. He cut the orange into 8 slices. How much does each sl
Tristian bought an orange that was 1/5 pound. He cut the orange into 8 slices. How much does each slice weigh? 1/5 pound / 8 slices = 1/5 * 1/8 = [B]1/40 pound[/B]

True or False (a) The normal distribution curve is always symmetric to its mean. (b) If the variance
True or False (a) The normal distribution curve is always symmetric to its mean. (b) If the variance from a data set is zero, then all the observations in this data set are identical. (c) P(A AND Ac)=1, where Ac is the complement of A. (d) In a hypothesis testing, if the p-value is less than the significance level ?, we do not have sufficient evidence to reject the null hypothesis. (e) The volume of milk in a jug of milk is 128 oz. The value 128 is from a discrete data set. [B](a) True, it's a bell curve symmetric about the mean (b) True, variance measures how far a set of numbers is spread out. A variance of zero indicates that all the values are identical (c) True. P(A) is the probability of an event and P(Ac) is the complement of the event, or any event that is not A. So either A happens or it does not. It covers all possible events in a sample space. (d) False, we have sufficient evidence to reject H0. (e) False. Volume can be a decimal or fractional. There are multiple values between 127 and 128. So it's continuous.[/B]

True or False: The standard deviation of the chi-square distribution is twice the mean.
True or False: The standard deviation of the chi-square distribution is twice the mean. [B]False[/B], the variance is twice the mean. Mean is k, Variance is 2k

Trump stamps sold at $1.25 and Obama stamps sold at $2 . How many of each stamp was sold if 700 stam
Trump stamps sold at $1.25 and Obama stamps sold at $2 . How many of each stamp was sold if 700 stamps were sold making $1250 Let o be the number of Obama stamps. Let t be the number of Trump stamps. We have two equations: [LIST=1] [*]o + t = 700 [*]2o + 1.25t = 1250 [/LIST] Using our [URL='http://www.mathcelebrity.com/simultaneous-equations.php?term1=o%2Bt%3D700&term2=2o%2B1.25t%3D1250&pl=Cramers+Method']simultaneous equations calculator[/URL], we get: [B]o = 500 t = 200[/B]

Twelve friends went to a movie theater. Because the movie was boring, they decided to figure out how
Twelve friends went to a movie theater. Because the movie was boring, they decided to figure out how many different ways they could sit in the 12 seats. How many different permutations are there for these 12 friends? 12 taken 12 at a time is written as: [URL='https://www.mathcelebrity.com/permutation.php?num=12&den=12&pl=Permutations']12P12[/URL] = [B]479,001,600[/B]

Twelve students tried out for the football team. Coach Moorhead only has 5 openings. In how many way
Twelve students tried out for the football team. Coach Moorhead only has 5 openings. In how many ways, can he pick 5 of the 12 students to be on the team? We use the combinations formula. We can write this as 12C5. [URL='https://www.mathcelebrity.com/permutation.php?num=12&den=5&pl=Combinations']Type this into our search engine[/URL] and we get: [B]792 ways[/B]

Twenty-five is nine more than four times a number
The phrase [I]a number[/I] means an arbitrary variable. We can pick any letter a-z except for i and e. Let's choose x. Four times a number: 4x nine more than four times a numbrer 4x + 9 The phrase [I]is[/I] means equal to. We set 4x + 9 equal to 25 as our algebraic expression: [B]4x + 9 = 25 [/B] If the problem asks you to solve for x, we [URL='https://www.mathcelebrity.com/1unk.php?num=4x%2B9%3D25&pl=Solve']type it in our math solver[/URL] and get: x = [B]4[/B]

Twenty-five is the same as ten added to twice a number
The phrase [I]a number[/I] means an arbitrary variable. We can pick any letter a-z except for i and e. Let's choose x. Twice a number means we multiply x by 2: 2x ten added to twice a number 2x + 10 The phrase [I]is the same as [/I]means equal to. Set 25 equal to 2x + 10 to get our algebraic expression [B]25 = 2x + 10 [/B] If the problem asks you to solve for x, [URL='https://www.mathcelebrity.com/1unk.php?num=25%3D2x%2B10&pl=Solve']type it in our math solver [/URL]and get x = [B]7.5[/B]

twenty-nine to the w
twenty-nine to the w We take 29 and raise it to the w power: [B]29^w[/B]

Twice a first number decreased by a second number is 16. The first number increased by 3 times the s
Twice a first number decreased by a second number is 16. The first number increased by 3 times the second number is 1. Find the numbers. Let the first number be x and the second number be y. We're given: [LIST=1] [*]2x - y = 16 [*]x + 3y = 1 [/LIST] Using our simultaneous equations calculator, you can solve this 3 ways: [LIST] [*][URL='https://www.mathcelebrity.com/simultaneous-equations.php?term1=2x+-+y+%3D+16&term2=x+%2B+3y+%3D+1&pl=Substitution']Substitution Method[/URL] [*][URL='https://www.mathcelebrity.com/simultaneous-equations.php?term1=2x+-+y+%3D+16&term2=x+%2B+3y+%3D+1&pl=Elimination']Elimination Method[/URL] [*][URL='https://www.mathcelebrity.com/simultaneous-equations.php?term1=2x+-+y+%3D+16&term2=x+%2B+3y+%3D+1&pl=Cramers+Method']Cramers Rule[/URL] [/LIST] No matter what method we use, we get the same answers: [B]x = 7 y = -2 (x, y) = (7, -2) [/B] Let's check our work in equation 1: 2(7) - -2 ? 16 14 + 2 ? 16 16 = 16 <-- Check Let's check our work in equation 2: 7 + 3(-2) ? 1 7 - 6 ? 1 1 = 1 <-- Check

Twice a number decreased by eight is zero
The phrase [I]a number[/I] means an arbitrary variable. We can pick any letter a-z except for i and e. Let's choose x. Twice a number: 2x decreased by eight 2x - 8 [I]is [/I]means equal to. Set 2x - 8 equal to zero for our algebraic expression: [B]2x - 8 = 0 [/B] If the problem asks you to solve for x, add 8 to each side: 2x = 8 Divide each side by 2: x= [B]4[/B]

Twice a number decreased by six
The phrase [I]a number[/I] means an arbitrary variable. We can pick any letter a-z except for i and e. Let's choose x. Twice a number means we multiply x by 2: 2x Decreased by six means we subtract 6 from 2x: [B]2x - 6[/B]

twice a number subtracted from the square root of the same number
twice a number subtracted from the square root of the same number The phrase [I]a number[/I] means an arbitrary variable, let's call it x: x Twice a number means we multiply x by 2: 2x Square root of the same number: sqrt(x) twice a number subtracted from the square root of the same number [B]sqrt(x) - 2x[/B]

twice the difference between x and 28 is 3 times a number
twice the difference between x and 28 is 3 times a number The difference between x and 28: x - 28 Twice the difference means we multiply x - 28 by 2: 2(x - 28) The phrase [I]a number[/I] means an arbitrary variable, let's call it x x 3 times a number: 3x The word [I]is[/I] means equal to, so we set 2(x - 28) equal to 3x: [B]2(x - 28) = 3x[/B]

twice the difference of a number and 3 is equal to 3 times the sum of a number and 2
twice the difference of a number and 3 is equal to 3 times the sum of a number and 2. We've got 2 algebraic expressions here. Let's take them in parts. Left side algebraic expression: twice the difference of a number and 3 [LIST] [*]The phrase [I]a number[/I] means an arbitrary variable, let's call it x. [*]The word [I]difference[/I] means we subtract 3 from the variable x [*]x - 3 [*]Twice this difference means we multiply (x - 3) by 2 [*]2(x - 3) [/LIST] Right side algebraic expression: 3 times the sum of a number and 2 [LIST] [*]The phrase [I]a number[/I] means an arbitrary variable, let's call it x. [*]The word [I]sum[/I] means we add 2 to the variable x [*]x + 2 [*]3 times the sum means we multiply (x + 2) by 3 [*]3(x + 2) [/LIST] Now, we have both algebraic expressions, the problem says [I]is equal to[/I] This means we have an equation, where we set the left side algebraic expression equal to the right side algebraic expression using the equal sign (=) to get our answer [B]2(x - 3) = 3(x + 2)[/B]

twice the difference of a number and 55 is equal to 3 times the sum of a number and 8
twice the difference of a number and 55 is equal to 3 times the sum of a number and 8 Take this algebraic expression in pieces. Left side: The phrase [I]a number[/I] means an arbitrary variable, let's call it x. x The difference of this number and 55 means we subtract 55 from x x - 55 Twice the difference means we multiply x - 55 by 2 2(x - 55) Right side: The phrase [I]a number[/I] means an arbitrary variable, let's call it x. x The sum of a number and 8 means we add 8 to x x + 8 3 times the sum means we multiply x + 8 by 3 3(x + 8) Now that we have the left and right side of the expressions, we see the phrase [I]is equal to[/I]. This means an equation, so we set the left side equal to the right side: [B]2(x - 55) = 3(x + 8)[/B]

twice the product of p q and r
twice the product of p q and r The product of p q and r means we multiply all 3 variables together: pqr The word [I]twice[/I] means we multiply pqr by 2: [B]2pqr[/B]

twice the product of p q and r
twice the product of p q and r The product of p q and r: pqr Twice means we multiply pqr by 2: [B]2pqr[/B]

Twice the quantity of seven plus x is the same as the difference of x and seven
seven plus x 7 + x Twice the quantity of seven plus x 2(7 + x) Difference of x and seven x - 7 The phrase [I]is the same as[/I] means equal to. This is our algebraic expression: [B]2(7 + x) = x - 7 [/B] If the problem asks you to solve for x, distribute 2 on the left side: 14 + 2x = x - 7 Subtract x from the right side 14 + x = -7 Subtract 14 from each side [B]x = -21[/B]

Twice the quantity of two y and six
two y and six 2y + 6 Twice the quantity: [B]2(2y + 6)[/B]

twice the square of the product of x and y
twice the square of the product of x and y Take this algebraic expression in pieces: [LIST] [*]The product of x and y means we multiply x and y: xy [*]The square of the product means we raise xy to the power of 2: (xy)^2 = x^2y^2 [*]Twice the square means we multiply the square by 2: [B]2x^2y^2[/B] [/LIST]

twice the square of the product of x and y
twice the square of the product of x and y [LIST] [*]The product of x and y: xy [*]The square of the product means we raise xy to the power of 2: (xy)^2 [*]Twice the square means we multiply by 2 [/LIST] [B]2(xy)^2 or 2x^2y^2[/B]

twice the square root of a number increased by 5 is 23
twice the square root of a number increased by 5 is 23 The phrase [I]a number[/I] means an arbitrary variable, let's call it x: x The square root of a number means we raise x to the 1/2 power: sqrt(x) the square root of a number increased by 5 means we add 5 to sqrt(x): sqrt(x) + 5 twice the square root of a number increased by 5 means we multiply sqrt(x) + 5 by 2: 2(sqrt(x) + 5) The phrase [I]is 23[/I] means we set 2(sqrt(x) + 5) equal to 23: [B]2(sqrt(x) + 5) = 23[/B]

twice the sum of a and b is thrice c
twice the sum of a and b is thrice c The sum of a and b: a + b twice the sum of a and b means we multiply the sum of a and b by 2: 2(a + b) Thrice c means we multiply c by 3: 3c The word [I]is[/I] means equal to, so we set 2(a + b) equal to 3c: [B]2(a + b) = 3c [MEDIA=youtube]G_D4b8Jv89Q[/MEDIA][/B]

Twice the sum of a number and 6 is equal to three times the difference of the number and 3. Find the
[SIZE=6]Twice the sum of a number and 6 is equal to three times the difference of the number and 3. Find the number. The phrase [/SIZE][I][SIZE=7]a number[/SIZE][/I][SIZE=6] means an arbitrary variable, let's call it x. The sum of a number and 6 means we add 6 to x: x + 6 Twice the sum of a number and 6 means we multiply x + 6 by 2: 2(x + 6) the difference of the number and 3 means we subtract 3 from x x - 3 three times the difference of the number and 3 means we multiply x - 3 by 3: 3(x- 3) The word [I]is[/I] means we set 2(x + 6) equal to 3(x - 3) 2(x + 6) = 3(x - 3) Use the distributive property to multiply through: 2x + 12 = 3x - 9 Subtract 2x from each side: 2x - 2x + 12 = 3x - 2x - 9 x - 9 = 12 Add 9 to each side: x - 9 + 9 = 12 + 9 x = [B]21[/B] [B][/B] [B][MEDIA=youtube]CeZl_oZnSiw[/MEDIA][/B][/SIZE]

Twice x increased by the cube of y equals z
Twice x increased by the cube of y equals z [LIST] [*]Twice x means we multiply x by 2: 2x [*]Increased this by the cube of y which is y^3. So we have 2x + y^3 [*]Now, we set this entire expression equal to z: 2x + y^3 = z [/LIST]

Two coins are flipped 2 times. Calculate the total outcomes of these coins.
Two coins are flipped 2 times. Calculate the total outcomes of these coins. 2 coins * 2 outcomes per coin = 4 possible outcomes [LIST=1] [*][B]H,H[/B] [*][B]H,T[/B] [*][B]T,H[/B] [*][B]T,T[/B] [/LIST]

Two Column Proof
Free Two Column Proof Calculator - Shows you the details behind a two column proof including the five parts and examples

Two consecutive even integers that equal 126
Two consecutive even integers that equal 126 Let the first integer equal x. So the next even integer must be x + 2. The sum which is equal to 126 is written as x + (x + 2) = 126 Simplify: 2x + 2 = 126 Using our [URL='http://www.mathcelebrity.com/1unk.php?num=2x%2B2%3D126&pl=Solve']equation calculator,[/URL] we get: x = 62 This means the next consecutive even integer is 62 = 2 = 64. So our two even consecutive integers with a sum of 126 are [B](62, 64)[/B]

Two dice are rolled. Enter the size of the set that corresponds to the event that both dice are odd.
Two dice are rolled. Enter the size of the set that corresponds to the event that both dice are odd. If dice 1 is odd, then we have the following face values: {1, 3, 5} If dice 2 is odd, then we have the following face values: {1, 3, 5} [URL='https://www.mathcelebrity.com/2dice.php?gl=1&opdice=1&pl=Both+Odd&rolist=+2%2C3%2C9%2C10&dby=2%2C3%2C5&ndby=4%2C5&montect=+100']From this 2 dice odds face link[/URL], we see that the size of the set is 9. [LIST=1] [*]{1, 1} [*]{1, 3} [*]{1, 5} [*]{3, 1} [*]{3, 3} [*]{3, 5} [*]{5, 1} [*]{5, 3} [*]{5, 5} [/LIST]

Two fifths of the sum of 8 and b
Two fifths of the sum of 8 and b The sum of 8 and b 8 + b Two fifths of this sum: [B]2/5(8 + b)[/B]

two mechanics worked on a car. the first mechanic worked for 10 hours, and the second mechanic worke
two mechanics worked on a car. the first mechanic worked for 10 hours, and the second mechanic worked for 5 hours. together they charged a total of 1225. what was the rate charged per hour by each mechanic if the sum of the two rates was 170 per hour? Set up two equations: (1) 10x + 5y = 1225 (2) x + y = 170 Rearrange (2) x = 170 - y Substitute that into (1) 10(170 - y) + 5y = 1225 1700 - 10y + 5y = 1225 1700 - 5y = 1225 Move 5y to the other side 5y + 1225 = 1700 Subtract 1225 from each side 5y =475 Divide each side by 5 [B]y = 95[/B] Which means x = 170 - 95, [B]x = 75[/B]

Two mechanics worked on a car. the first mechanic worked for 5 hours snd the second mechanic worked
Two mechanics worked on a car. the first mechanic worked for 5 hours snd the second mechanic worked for 15 hours. Together they charged a total of $2375. What was the rate charged per hour by each mechanic if the sum of the two rates was $235 per hour? Setup equations where x is the rate of the first mechanic and y is the rate of the second mechanic: [LIST] [*]5x + 15y = 2375 [*]x + y = 235 [/LIST] Using Cramers method with our [URL='http://www.mathcelebrity.com/simultaneous-equations.php?term1=5x+%2B+15y+%3D+2375&term2=x+%2B+y+%3D+235&pl=Cramers+Method']simultaneous equations calculator[/URL], we get: [LIST] [*][B]x = 115[/B] [*][B]y = 120[/B] [/LIST]

Two numbers have a sum of 20. Determine the lowest possible sum of their squares.
Two numbers have a sum of 20. Determine the lowest possible sum of their squares. If sum of two numbers is 20, let one number be x. Then the other number would be 20 - x. The sum of their squares is: x^2+(20 - x)^2 Expand this and we get: x^2 + 400 - 40x + x^2 Combine like terms: 2x^2 - 40x + 400 Rewrite this: 2(x^2 - 20x + 100 - 100) + 400 2(x - 10)^2 - 200 + 400 2(x?10)^2 + 200 The sum of squares of two numbers is sum of two positive numbers, one of which is a constant of 200. The other number, 2(x - 10)^2, can change according to the value of x. The least value could be 0, when x=10 Therefore, the minimum value of sum of squares of two numbers is 0 + 200 = 200 when x = 10. If x = 10, then the other number is 20 - 10 = 10.

Two numbers have a sum of 20. If one number is p, express the other in terms of p.
Two numbers have a sum of 20. If one number is p, express the other in terms of p. If the sum is 20 and one number is p, then let the other number be q. We have: p + q = 20 We want q, so we subtract p from each side: [B]q = 20 - p[/B]

Two numbers have a sum of 59. If one number is q, express the other number on terms of q
Two numbers have a sum of 59. If one number is q, express the other number on terms of q The other number is [B]59 - q[/B]. Add them together, you get q + (59 - q) = 59.

two numbers have an average of 2100 and one number is $425 more than the other number. What are the
two numbers have an average of 2100 and one number is $425 more than the other number. What are the numbers Let the first number be x and the second number be y. We're given two equations: [LIST=1] [*](x + y)/2 = 2100 (Average) [*]y = x + 425 [/LIST] Rearrange equation (1) by cross multiplying x + y = 2 * 2100 x + y = 4200 So we have our revised set of equations: [LIST=1] [*]x + y = 4200 [*]y = x + 425 [/LIST] Substituting equation (2) into equation (1) for y, we get: x + (x + 425) = 4200 Combining like terms, we get: 2x + 425 = 4200 Using our [URL='https://www.mathcelebrity.com/1unk.php?num=2x%2B425%3D4200&pl=Solve']equation solver[/URL], we get: x = [B]1887.5[/B] Which means using equation (2), we get y = 1887.5 + 425 y = [B]2312.5[/B]

Two numbers have the sum of 40 if one number is P express the other in terms of P
Two numbers have the sum of 40 if one number is P express the other in terms of P We write this as P + (40 - P) = 40 So the other number is [B]40 - P[/B]

Two numbers that total 44 and have a difference of 6
Two numbers that total 44 and have a difference of 6. Let the two numbers be x and y. We're given the following equations: [LIST=1] [*]x + y = 44 <-- Total means a sum [*]x - y = 6 [/LIST] Add the two equations together: (x + x) + (y - y) = 44 + 6 Cancelling the y terms, we have: 2x = 50 [URL='https://www.mathcelebrity.com/1unk.php?num=2x%3D50&pl=Solve']Typing this equation into the search engine[/URL], we get: [B]x = 25 [/B] Rearranging equation (2) above, we get: y = x - 6 Substituting x = 25 into this, we get: y = 25 - 6 [B]y = 19[/B]

Two numbers total 12, and their differences is 20. Find the two numbers.
Two numbers total 12, and their differences is 20. Find the two numbers. Let the first number be x. Let the second number be y. We're given two equations: [LIST=1] [*]x + y = 12 [*]x - y = 20 [/LIST] Since we have y coefficients of (-1 and 1) that cancel, we add the two equations together: (x + x) + (y - y) = 12 + 20 The y terms cancel, so we have: 2x = 32 [URL='https://www.mathcelebrity.com/1unk.php?num=2x%3D32&pl=Solve']Type this equation into our search engine[/URL] and we get: x = [B]16[/B] Substitute this value of x = 16 back into equation 1: 16 + y = 12 [URL='https://www.mathcelebrity.com/1unk.php?num=16%2By%3D12&pl=Solve']Typing this equation into our search engine[/URL], we get: y = [B]-4 [/B] Now, let's check our work for both equations: [LIST=1] [*]16 - 4 = 12 [*]16 - -4 --> 16 + 4 = 20 [/LIST] So these both check out. (x, y) = ([B]16, -4)[/B]

Two numbers total 50 and have a difference of 28. Find the two numbers.
Two numbers total 50 and have a difference of 28. Find the two numbers. Using x and y as our two numbers, we write the following 2 equations: [LIST=1] [*]x + y = 50 [*]x - y = 28 [/LIST] Add the 2 rows: 2x = 78 Divide each side by 2: [B]x = 39[/B] If x = 39, then from (1), we have y = 50 - 39 [B]y = 11[/B]

Two numbers total 83 and have a difference of 17 find the two numbers
Let the numbers be x and y. Set up our givens: [LIST=1] [*]x + y = 83 [*]x - y = 17 [/LIST] [U]Add equation (1) to equation (2)[/U] x + x + y - y = 83 + 17 [U]The y-terms cancel out:[/U] 2x = 100 [U]Divide each side by 2:[/U] 2x/2= 100/2 x = [B]50[/B] [U] Plug x = 50 into equation (1)[/U] 50 + y = 83 [U]Subtract 50 from each side:[/U] 50 - 50 + y = 83 - 50 [U]Cancel the 50 on the left side:[/U] y = [B]33 [/B] So our two numbers (x, y) = (33, 50) [MEDIA=youtube]jajO043ChUM[/MEDIA]

two pages that face each other in a book have a sum of 569
two pages that face each other in a book have a sum of 569 Pages that face each other are consecutive. Let the first page be p. The second page is p + 1. Their sum is: p + p + 1 = 569 [URL='https://www.mathcelebrity.com/1unk.php?num=p%2Bp%2B1%3D569&pl=Solve']Type this equation into our search engine to solve for p[/URL], and we get: p = 284 This means p + 1 = 284 + 1 = 285 So the pages that face each other having a sum of 569 are: [B]284, 285[/B]

Two Step equation word problems.
Maria bought seven boxes. A week later half of all her boxes were destroyed in a fire. There are now 22 boxes left. With how many did she start?

Two Step equation word problems.
[URL]http://www.mathcelebrity.com/community/threads/maria-bought-7-boxes-a-week-later-half-of-all-her-boxes-were-destroyed-in-a-fire-there-are-now-onl.348/[/URL]

two thirds of a number is no more than -10
two thirds of a number is no more than -10 The phrase [I]a number[/I] means an arbitrary variable, let's call it x: x Two thirds of a number mean we multiply x by 2/3: 2x/3 The phrase [I]no more than[/I] -10 means less than or equal to -10, so we have an inequality: [B]2x/3 <= -10[/B]

two thirds of the sum of p and 2
The sum of p and 2 is written as p + 2. 2/3 of the sum is written as: 2(p + 2) --------- 3

two unbiased dice are thrown. find the probability that the total number on the dice is greater than
two unbiased dice are thrown. find the probability that the total number on the dice is greater than 10 [URL='http://www.mathcelebrity.com/2dice.php?gl=2&pl=10&opdice=1&rolist=+2%2C3%2C9%2C10&dby=2%2C3%2C5&ndby=4%2C5&montect=+100']From our 2 dice calculator[/URL]: We have (5,6),(6,5),(6,6) P(Sum) > 10 is [B]1/12[/B]

Two years of local internet service costs 685, including the installation fee of 85. What is the mon
Two years of local internet service costs 685, including the installation fee of 85. What is the monthly fee? Subtract the installation fee of 85 from the total cost of 685 to get the service cost only: 685 - 85 = 600 Now, divide that by 24 months in 2 years to get a per month fee 600/24 = [B]25 per month[/B]

two-thirds the difference of c and d
two-thirds the difference of c and d The difference of c and d: c - d two-thirds the difference means we multiply c - d by 2/3: [B]2(c - d)/3[/B]

Ty uses his blocks to build towers of 10 blocks each. There are 14 towers and five blocks left over
Ty uses his blocks to build towers of 10 blocks each. There are 14 towers and five blocks left over. Total Blocks = Tower Blocks + Left Over Blocks [U]First, calculate the number of tower blocks:[/U] Tower Blocks = Towers * Blocks per Tower Tower Blocks = 14 * 10 Tower Blocks = 140 [U]We have 5 left over blocks, so we calculate our total blocks:[/U] Total Blocks = 140 + 5 Total Blocks = [B]145[/B]

Tyler has a meal account with $1200 in it to start the school year. Each week he spends $21 on food
Tyler has a meal account with $1200 in it to start the school year. Each week he spends $21 on food a.) write an equation that relates the amount in the account (a) with the number of (w) weeks b.) How many weeks will it take until Tyler runs out of money? [U]Part a) where w is the number of weeks[/U] a = Initial account value - weekly spend * w ([I]we subtract because Tyler spends)[/I] a = [B]1200 - 21w [/B] [U]Part b)[/U] We want to know the number of weeks it takes where a = 0. So we have: 1200 - 21w = 0 To solve for w, we [URL='https://www.mathcelebrity.com/1unk.php?num=1200-21w%3D0&pl=Solve']type this equation into our search engine[/URL] and we get: w = 57.14 weeks The problem asks for when he runs out of money, so we round up to [B]58 whole weeks[/B]

Tyrese’s sister is 41 inches tall. A ride at the amusement park states that riders must be at least
Tyrese’s sister is 41 inches tall. A ride at the amusement park states that riders must be at least 52 inches tall to ride. Which statements describe how much taller Tyrese’s sister must be to ride? Let h be the required additional height. The phrase [I]at least[/I] means an inequality, using the >= sign, so we have: h + 41 >= 52 If we want another way to express this, we [URL='https://www.mathcelebrity.com/1unk.php?num=h%2B41%3E%3D52&pl=Solve']type this inequality into our math engine[/URL] and we get: [B]h >= 11[/B]

Tyrone re sells 3 pairs of Yeezys and a pair of Nikes for 250$. Nucci re sells a pair of Yeezys and
Tyrone re sells 3 pairs of Yeezys and a pair of Nikes for 250$. Nucci re sells a pair of Yeezys and Nikes for 150$ How much does a pair of Yeezys cost? Let y be the cost of Yeezy's and n be the cost of Nike's. We're given two equations: [LIST=1] [*]3y + n = 250 [*]y + n = 150 [/LIST] We have a system of equations, and we can solve it using one of three ways: [LIST] [*][URL='https://www.mathcelebrity.com/simultaneous-equations.php?term1=3y+%2B+n+%3D+250&term2=y+%2B+n+%3D+150&pl=Substitution']Substitution Method[/URL] [*][URL='https://www.mathcelebrity.com/simultaneous-equations.php?term1=3y+%2B+n+%3D+250&term2=y+%2B+n+%3D+150&pl=Elimination']Elimination Method[/URL] [*][URL='https://www.mathcelebrity.com/simultaneous-equations.php?term1=3y+%2B+n+%3D+250&term2=y+%2B+n+%3D+150&pl=Cramers+Method']Cramer's Rule[/URL] [/LIST] No matter what method we choose, we get: [LIST] [*][B]n = 100[/B] [*][B]y = 50[/B] [/LIST]

u cubed equals nine
u cubed equals nine u cubed means we raise u to the 3rd power: u^3 We set this equal to 9: [B]u^3 = 9[/B]

u more than the quotient of 8 and 5
u more than the quotient of 8 and 5 The quotient of 8 and 5: 8/5 u more means we add u 8/5 + u

u varies jointly as q and the square of m
u varies jointly as q and the square of m Varies jointly means we multiply. There exists a constant k such that: [B]u = kqm^2[/B]

u=ak/b for a
Cross multiply: ub = ak Divide each side of the equation by k to isolate a: a = ub/k [MEDIA=youtube]A3NW3Y68iNY[/MEDIA]

Uniform Distribution
Free Uniform Distribution Calculator - This calculates the following items for a uniform distribution
* Probability Density Function (PDF) ƒ(x)
* Cumulative Distribution Function (CDF) F(x)
* Mean, Variance, and Standard Deviation
Calculates moment number t using the moment generating function

Unit Cost
Free Unit Cost Calculator - Calculates the unit cost based on a price and a quantity

Unit Fraction
Free Unit Fraction Calculator - Determines the unit fraction for a fraction.

Units of Output (Service Output) Depreciation
Free Units of Output (Service Output) Depreciation Calculator - Given an asset value, salvage value, production units, and units per period, this calculates the depreciation per period using the units of output depreciation (service output depreciation)

Unknown Number
Free Unknown Number Calculator - Determines the unknown number needed to make an equation true.

Unlock Your Hidden Math Genius with Hypnosis
This hypnosis works on your subconscious by removing limiting beliefs for math. The goal for this hypnosis will be to remove anything causing you math anxiety. Since nature abhors a vacuum, the limiting beliefs fade away and you're left with positive intentions and ease to improve math grades. Unlock Your Hidden Math Genius with Hypnosis [MEDIA=youtube]gxzMKQq38QU[/MEDIA]

Use k as the constant of variation. L varies jointly as u and the square root of v.
Use k as the constant of variation. L varies jointly as u and the square root of v. Since u and v vary jointly, we multiply by the constant of variation k: [B]l = ku * sqrt(v)[/B]

Use the definite integral to find the area between the x-axis and the function f(x)= x^2-x-12 over t
Use the definite integral to find the area between the x-axis and the function f(x)= x^2-x-12 over the interval [ -5, 10]. Using our [URL='http://www.mathcelebrity.com/dfii.php?term1=x%5E2-x-12&fpt=0&ptarget1=0&ptarget2=0&itarget=-5%2C10&starget=0%2C1&nsimp=8&pl=Integral']integral calculator[/URL], we get: [B]157.5[/B]

Use the information below to determine the weight of 500 gallons of water. a) There are 1.057 quart
Use the information below to determine the weight of 500 gallons of water. a) There are 1.057 quarts in a liter and 4 quarts in a gallon b) A cubic decimeter of water is a liter of water c) A cubic decimeter of water weighs one kilogram d) There are 2.2 pounds in a kilogram [LIST] [*]500 gallons = 2000 quarts [*]2000 quarts / 1.057 quarts in a liter = 1892.15 liters [*]1892.15 liters weight 1892.15 kilograms [*]1892.15 kilograms x 2.2 pounds = [B]4163 pounds[/B] [/LIST]

Utility and Cost Utility Ratio
Free Utility and Cost Utility Ratio Calculator - Given 2 methods with a set of utilities and weights/probabilities, this will calculate the utility for each method, as well as the total utility using the additive method, as well as the Cost Utility Ratio

v equals 66 decreased by d
The calculator is updated to handle shortcuts like these.

v is equal to the product of 7 and the sum of u and 6
v is equal to the product of 7 and the sum of u and 6 [LIST] [*]Sum of u and 6: u + 6 [*]the product of 7 and the sum of u and 6: 7(u + 6) [*]We set this expression equal to v: [/LIST] [B]v = 7(u + 6)[/B]

V ? E + F = 2 for e
V ? E + F = 2 for e To solve this literal equation, we want to isolate e. Add E to both sides: V ? E + F + E = 2 + E The E's cancel on the left side, so we have: V + F = 2 + E Subtract 2 from each side: V + F - 2 = 2 + E + 2 The 2's cancel on the right side, so we have: E = [B]V + F - 2[/B]

Vacation is 72 days long. What percent of the entire year is summer vacation ?
Vacation is 72 days long. What percent of the entire year is summer vacation ? Vacation day Percent = 100% * Vacation Days / Total Days in the year Vacation day Percent = 100% * 72/365 Vacation day Percent = 100% * [URL='https://www.mathcelebrity.com/perc.php?num=72&den=365&pcheck=1&num1=16&pct1=80&pct2=70&den1=80&idpct1=10&hltype=1&idpct2=90&pct=82&decimal=+65.236&astart=12&aend=20&wp1=20&wp2=30&pl=Calculate']0.1973[/URL] Vacation day Percent = [B]19.73%[/B]

Van needs to enter a formula into a spreadsheet to show the outputs of an arithmetic sequence that s
Van needs to enter a formula into a spreadsheet to show the outputs of an arithmetic sequence that starts with 13 and continues to add seven to each output. For now, van needs to know what the 15th output will be. Complete the steps needed to determine the 15th term in sequence. Given a first term a1 of 13 and a change amount of 7, expand the series The explicit formula for an [I]arithmetic series[/I] is an = a1 + (n - 1)d d represents the common difference between each term, an - an - 1 Looking at all the terms, we see the common difference is 7, and we have a1 = 13 Therefore, our explicit formula is an = 13 + 7(n - 1) If n = 15, then we plug it into our explicit formula above: an = 13 + 7(n - 1) a(15) = 15 + 7(15 - 1) a(15) = 15 + 7 * 14 a(15) = 15 + 98 a(15) = [B]113[/B]

Vanessa is packing her bags for her vacation. She has 5 unique action figures, but only 3 fit in her
Vanessa is packing her bags for her vacation. She has 5 unique action figures, but only 3 fit in her bag. How many different groups of 3 action figures can she take? The key word here is [U]different[/U]. This means combinations. We use our [URL='http://www.mathcelebrity.com/permutation.php?num=5&den=3&pl=Combinations']combinations calculator[/URL] to find 5 C 3 which equals [B]10[/B].

Variation Equations
Free Variation Equations Calculator - This calculator solves the following direct variation equations and inverse variation equations below:
* y varies directly as x
* y varies inversely as x
* y varies directly as the square of x
* y varies directly as the cube of x
* y varies directly as the square root of x
* y varies inversely as the square of x
* y varies inversely as the cube of x
* y varies inversely as the square root of x

Vectors
Free Vectors Calculator - Given 2 vectors A and B, this calculates:
* Length (magnitude) of A = ||A||
* Length (magnitude) of B = ||B||
* Sum of A and B = A + B (addition)
* Difference of A and B = A - B (subtraction)
* Dot Product of vectors A and B = A x B
A ÷ B (division)
* Distance between A and B = AB
* Angle between A and B = θ
* Unit Vector U of A.
* Determines the relationship between A and B to see if they are orthogonal (perpendicular), same direction, or parallel (includes parallel planes).
* Cauchy-Schwarz Inequality
* The orthogonal projection of A on to B, projBA and and the vector component of A orthogonal to B → A - projBA
Also calculates the horizontal component and vertical component of a 2-D vector.

Vendor Discount Effective Rate of Interest
Free Vendor Discount Effective Rate of Interest Calculator - Calculates the effective rate of interest earned from a vendor discount for a prepayment of a balance within a certain amount of days for a percentage discount

Venn Diagram
Free Venn Diagram Calculator - This lesson walks you through what a Venn diagram is, the Venn diagram for A union B, A intersection B, and A Complement.

Venn Diagram (2 circles)
Free Venn Diagram (2 circles) Calculator - Given two circles A and B with an intersection piece of C, this will calculate all relevant probabilities of the Venn Diagram.

Vice Versa Percentage Method
x% of y is the same as y% of x Example 1: 18% of 10 is the same as 10% of 18 10% is easy because we move the decimal one place left to get [B]1.8[/B] Example 2: 40% of 50 is the same as 50% of 40 50% is easy because we cut a number in half 40/2 = [B]20 [MEDIA=youtube]aiKsvYWEo0c[/MEDIA][/B]

Victoria is 4 years older than her neighbor. The sum of their ages is no more than 14 years.
Victoria is 4 years older than her neighbor. The sum of their ages is no more than 14 years. Let Victoria's age be v. And her neighbor's age be n. We're given: [LIST=1] [*]v = n + 4 [*]v + n <=14 <-- no more than means less than or equal to [/LIST] Substitute Equation (1) into Inequality (2): (n + 4) + n <= 14 Combine like terms: 2n + 4 <= 14 [URL='https://www.mathcelebrity.com/1unk.php?num=2n%2B4%3C%3D14&pl=Solve']Typing this inequality into our search engine[/URL], we get: n <= 5 Substituting this into inequality (2): v + 5 <= 14 [URL='https://www.mathcelebrity.com/1unk.php?num=v%2B5%3C%3D14&pl=Solve']Typing this inequality into our search engine[/URL], we get: [B]v <= 9[/B]

Video store movie rental plans. Plan A 25 membership fee plus 1.25 for movie. Plan B 40 for unlimite
Video store movie rental plans. Plan A 25 membership fee plus 1.25 for movie. Plan B 40 for unlimited rentals. What number of movies rentals is plan B less than plan A? Let x equal the number of movies rented and C the cost for rentals Plan A: C = 1.25x + 25 Plan B: C = 40 Set up the inequality: 1.25x + 25 > 40 Subtract 25 from each side: 1.25x > 15 Divide each side of the inequality by 1.25 x > 12 So [B]13[/B] rentals or more make Plan B less than Plan A.

Vinculum
Free Vinculum Calculator - Shows you the various examples and definition of a vinculum

vw^2+y=x for w
vw^2+y=x for w This is an algebraic expression. Subtract y from each side: vw^2 + y - y = x - y The y's cancel on the left side, so we're left with: vw^2 = x - y Divide each side by v w^2 = (x - y)/v Take the square root of each side: w = [B]Sqrt((x - y)/v)[/B]

Walking Distance (Pedometer)
Free Walking Distance (Pedometer) Calculator - Given a number of steps and a distance per stride in feet, this calculator will determine how far you walk in other linear measurements.

Wan bought 2 salad rolls and 1 bottle of orange juice. If each salad roll costs x cents and the oran
Wan bought 2 salad rolls and 1 bottle of orange juice. If each salad roll costs x cents and the orange juice costs $1.50, write the expression for the total cost (in cents) for the food and drink The cost C is: C = 2x + 1.50(1) Simplify: [B]C = 2x + 1.50[/B]

Warren was making $100,000 per year. His boss said that he was going to cut his salary 25%, but that
Warren was making $100,000 per year. His boss said that he was going to cut his salary 25%, but that Warren shouldn't worry because he would be given a 25% raise the next day. How much will Warren's salary be after the 25% cut and 25% raise? Cut salary: 100,000 * 0.75 = 75,000 New salary after raise: 75,000 * 1.25 = [B]93,750[/B]

Water flows from tank A to tank B at the rate of 2 litres per minute.
Water flows from tank A to tank B at the rate of 2 litres per minute. Initially tank A has 200 litres in it and tank B has 100 Litres in it. Water drains from tank B at 0.5 litres per minute. After how many minutes are there equal volumes of water in the 2 tanks? Write an equation and solve it.

Water flows from tank A to tank B at the rate of 2 litres per minute.
[QUOTE="Jahn, post: 78, member: 5"]Water flows from tank A to tank B at the rate of 2 litres per minute. Initially tank A has 200 litres in it and tank B has 100 Litres in it. Water drains from tank B at 0.5 litres per minute. After how many minutes are there equal volumes of water in the 2 tanks? Write an equation and solve it.[/QUOTE] Tank A: V = 200 - 2x Tank B: V = 100 - 0.5x Where x is the number of minutes passed. Set them equal to each other 200 - 2x = 100 - 0.5x Subtract 100 from each side: 100 - 2x = -0.5x Add 2x to each side: 1.5x = 100 Divide each side of the equation by x: x = 66.66666667

Wayne’s widget world sells widgets to stores for $10.20 each (wholesale price). A local store marks
Wayne’s widget world sells widgets to stores for $10.20 each (wholesale price). A local store marks them up $6.79. What is the retail price at this store? [I]Note: Markup means we add to the wholesale price. [/I] Calculate Retail Price: Retail Price = Wholesale Price + Markup Amount Retail Price = $10.20 + $6.79 Retail Price = [B]$16.99[/B]

Weight Conversions
Free Weight Conversions Calculator - This calculator converts between the following weight measurements:
* Ounces (oz.)
* Pounds (lb.)
* Tons
* Milligrams (mg.)
* Grams
* Kilograms (kg.)
* Centigrams (cg.)
* Micrograms (mcg.)
* Stone

Weighted Average Cost of Capital (WACC) and Capital Asset Pricing Model (CAPM)
Free Weighted Average Cost of Capital (WACC) and Capital Asset Pricing Model (CAPM) Calculator - Calculates the Weighted Average Cost of Capital (WACC) and also calculates the return on equity if not given using the Capital Asset Pricing Model (CAPM) using debt and other inputs such as Beta and risk free rate.

Wendy is paid $7.50 per hour plus a bonus of $80 each week. Last week Wendy earned $312.50. How many
Wendy is paid $7.50 per hour plus a bonus of $80 each week. Last week Wendy earned $312.50. How many hours did Wendy work last week? Setup the earnings equation with h hours: 7.5h + 80 = 312.50 Solve for [I]h[/I] in the equation 7.5h + 80 = 312.50 [SIZE=5][B]Step 1: Group constants:[/B][/SIZE] We need to group our constants 80 and 312.50. To do that, we subtract 80 from both sides 7.5h + 80 - 80 = 312.50 - 80 [SIZE=5][B]Step 2: Cancel 80 on the left side:[/B][/SIZE] 7.5h = 232.5 [SIZE=5][B]Step 3: Divide each side of the equation by 7.5[/B][/SIZE] 7.5h/7.5 = 232.5/7.5 h = [B]31 [URL='https://www.mathcelebrity.com/1unk.php?num=7.5h%2B80%3D312.50&pl=Solve']Source[/URL][/B]

What can we conclude if the coefficient of determination is 0.94?
What can we conclude if the coefficient of determination is 0.94? [LIST] [*]Strength of relationship is 0.94 [*]Direction of relationship is positive [*]94% of total variation of one variable(y) is explained by variation in the other variable(x). [*]All of the above are correct [/LIST] [B]94% of total variation of one variable(y) is explained by variation in the other variable(x)[/B]. The coefficient of determination explains ratio of explained variation to the total variation.

what does the digit 7 in 65.47 stand for
what does the digit 7 in 65.47 stand for To the right of the decimal place, moving left to right, we have: 4 is the tenths place [B]7 is the hundredths place[/B]

What does y=f(x) mean
What does y=f(x) mean It means y = a function of the variable x. x is the independent variable and y is the dependent variable. f(x) means a function in terms of x

What fraction lies exactly halfway between 2/3 and 3/4?
What fraction lies exactly halfway between 2/3 and 3/4? A) 3/5 B) 5/6 C) 7/12 D) 9/16 E) 17/24 Halfway means taking the average, which is dividing the sum of the fractions by 2 for 2 fractions: 1/2(2/3 + 3/4) 1/2(2/3) + 1/2(3/4) 1/3 + 3/8 We need common denominators, so [URL='https://www.mathcelebrity.com/fraction.php?frac1=1%2F3&frac2=3%2F8&pl=Add']we type this fraction sum into our search engine[/URL] and get: [B]17/24 - Answer E[/B]

what integer is tripled when 9 is added to 3 fourths of it?
what integer is tripled when 9 is added to 3 fourths of it? Let the integer be n. Tripling an integer means multiplying it by 3. We're given: 3n = 3n/4 + 9 Since 3 = 12/4, we have: 12n/4 = 3n/4 + 9 Subtract 3n/4 from each side: 9n/4 = 9 [URL='https://www.mathcelebrity.com/prop.php?num1=9n&num2=9&den1=4&den2=1&propsign=%3D&pl=Calculate+missing+proportion+value']Typing this equation into the search engine[/URL], we get: [B]n = 4[/B]

what is 828 rounded to the nearest hundred
what is 828 rounded to the nearest hundred 828 lies between 800 and 900. The halfway point is 850, so 828 rounds down to [B]800[/B]

What is a Fraction
Free What is a Fraction Calculator - This lesson walks you through what a fraction is and the different parts of a fraction.

What is a Function
Free What is a Function Calculator - This lesson walks you through what a function is, how to write a function, the part of a function, and how to evaluate the outputs of a function.
This lesson also shows you the domain and range of a function. This lesson shows you the y-intercept of a function and the x-intercept of a function. Also shows Relation and function

What is a Line
Free What is a Line Calculator - This lesson walks you through what a line is and the various implications of a line in geometry

What is a Perpendicular Bisector
Free What is a Perpendicular Bisector Calculator - This lesson walks you through what a perpendicular bisector is and the various properties of the segment it bisects and the angles formed by the bisection

What is a Point
Free What is a Point Calculator - This lesson walks you through what a point is and the various implications of a point in geometry

What is a Ray
Free What is a Ray Calculator - This lesson walks you through what a Ray is and the various implications of a ray in geometry

What is a Segment
Free What is a Segment Calculator - This lesson walks you through what a segment is and the various implications of a segment in geometry including the midpoint of a segment.

What is a Variable
Free What is a Variable Calculator - This lesson walks you through what a variable is and how to use it. Also demonstrates the let statement.

what is a well defined set
what is a well defined set? A well defined set is with no ambiguity or confusion about what belongs to the set. Think of it as a collection of distinct objects: Examples: [LIST] [*]Set of the first 5 even numbers: {2, 4, 6, 8, 10} [*]Set of weekend days: {Saturday, Sunday} [/LIST]

What is an Exponent
Free What is an Exponent Calculator - This lesson walks you through what an exponent is, the product rule for exponents, the quotient rule for exponents, the 0 power rule, the power of a power rule for exponents

What is my IP address?
Free What is my IP address? Calculator - This shows you your IP address of the device you use as well as a definition of IP addresses

What is the 1000th term in the series 0, 7, 14, 21, … ?
Map this out as a function with term number n and value 1, 0 2, 7 3, 14 4, 21 The values jump by 7, but they do so as the n - 1 term. We have the series formula S(n) = 7(n - 1) The problem asks for S(1000) S(1000) = 7(1000 - 1) S(1000) = 7(999) S(1000) = [B]6,993[/B] [MEDIA=youtube]ZF10Ec29XKo[/MEDIA]

What is the 7th number in the following pattern: 3.2, 4.4, 5.6, 6.8, ...
What is the 7th number in the following pattern: 3.2, 4.4, 5.6, 6.8, ... This is an arithmetic sequence with an increase amount of 1.2. Each term S(n) is found by adding 1.2 to the prior term. S(1) = 3.2 S(2) = 3.2 + 1.2 = 4.4 S(3) = 4.4 + 1.2 = 5.6 S(4) = 5.6 + 1.2 = 6.8 S(5) = 6.8 + 1.2 = 8.0 S(6) = 8.0 + 1.2 = 9.2 S(7) = 9.2 + 1.2 = [B]10.4[/B]

What is the annual nominal rate compounded daily for a bond that has an annual yield of 5.4%? Round
What is the annual nominal rate compounded daily for a bond that has an annual yield of 5.4%? Round to three decimal places. Use a 365 day year. [U]Set up the accumulation equation:[/U] (1+i)^365 = 1.054 [U]Take the natural log of each side[/U] 365 * Ln(1 + i) = 1.054 Ln(1 + i) = 0.000144089 [U]Use each side as a exponent to eulers constant e[/U] (1 + i) = e^0.000144089 1 + i = 1.000144099 i = 0.000144099 or [B].0144099%[/B]

What is the area of a triangular parking lot with a width of 200m and a length of 100m?What is the a
What is the area of a triangular parking lot with a width of 200m and a length of 100m? Area of a Triangle = bh/2 Plugging in our numbers, we get: Area of Parking Lot = 200(100)/2 Area of Parking Lot = 100 * 100 Area of Parking Lot = [B]10,000 sq meters[/B]

What is the average of 7 consecutive numbers if the smallest number is called n?
What is the average of 7 consecutive numbers if the smallest number is called n? [LIST] [*]First number = n [*]Second number = n + 1 [*]Third number = n + 2 [*]Fourth number = n + 3 [*]Fifth number = n + 4 [*]Sixth number = n + 5 [*]Seventh number = n + 6 [/LIST] Average = Sum of all numbers / Total numbers Average = (n + n + 1 + n + 2 + n + 3 + n + 4 + n + 5 + n + 6)/7 Average = 7n + 21/7 Factor out a 7 from the top: 7(n + 3)/7 Cancel the 7's: [B]n + 3[/B]

What is the base 10 number 100 in base 7?
What is the base 10 number 100 in base 7? Using our [URL='https://www.mathcelebrity.com/binary.php?num=100&check1=7&bchoice=7&pl=Convert']base change calculator[/URL], we see that: 100 in base 10 = [B]202[/B] in base 7

What is the correct translation of; 8 increased by a number is 10?
What is the correct translation of; 8 increased by a number is 10? We [URL='https://www.mathcelebrity.com/community/forums/calculator-requests.7/create-thread']type in [I]8 increased by a number is 10[/I] into our search engine[/URL] and we get: [B]8 + a = 10[/B]

what is the cost of 3 books at p cents and 4 pens at q cents each?
what is the cost of 3 books at p cents and 4 pens at q cents each? Cost = Price * Quantity. [B]3p + 4q[/B]

What is the formula for the area of a circle?
What is the formula for the area of a circle? Given a radius r, we have Area (A) of: [B]A = ?r^2[/B]

What is the formula for the circumference of a circle?
What is the formula for the circumference of a circle? Given radius r and diameter d, the circumference C is: [B]C = 2?r or ?d[/B]

What is the formula for the volume of a cylinder?
What is the formula for the volume of a cylinder? The Volume (V) of a cylinder with radius (r) and height (h) is: [B]V = ?r^2h[/B]

What is the inverse of dividing by 3
What is the inverse of dividing by 3 [B]Multiplying by 3[/B] Suppose we have 2 divided by 3: 2/3 To undo this operation to get to 2 again, we'd multiply by 3: 2/3 * 3 = 2

What is the Least Common Multipole of 3,4,5?
What is the Least Common Multipole of 3,4,5? Using our l[URL='https://www.mathcelebrity.com/gcflcm.php?num1=3&num2=4&num3=5&pl=LCM']east common multiple calculator[/URL], we get: [B]60[/B]

What is the missing number? 0, 1, 1, 2, 3, 5, 8, __, 21, 34
We can't add a common number of multiply a common number to solve this. But what happens if we start at the 3rd term and add the 2 prior terms? [LIST] [*]0 + 1= 1 [*]1 + 1 = 2 [*]1 + 2 = 3 [*]3 + 5 = 8 [*]5 + 8 = [B]13[/B] [/LIST] [B]This is known as the Fibonacci Sequence. [MEDIA=youtube]9SZ1TJGwyLw[/MEDIA][/B]

What is the number of days in w weeks and d days?
What is the number of days in w weeks and d days? Since a week is 7 days, we have a number of days of: [B]7w + d[/B]

What is the probability of drawing an ace from a deck of 52 cards?
What is the probability of drawing an ace from a deck of 52 cards? With 4 Aces in the deck, the probability we draw an Ace is: 4/52 Simplifying this fraction, we get [B]1/13[/B]

What is the probability of picking a P from the word hippopotamus?
What is the probability of picking a P from the word hippopotamus? There are 3 P's in a word with 12 characters. So our probability is 3/12. [URL='https://www.mathcelebrity.com/fraction.php?frac1=3%2F12&frac2=3%2F8&pl=Simplify']Using our fraction simplify calculator[/URL], this simplifies to 1/4. Written as a decimal, it's 0.25. So our answer is [B]1/4 or 0.25[/B]

What is the probability of rolling 12, 5 times in a row?
The only way you can roll a 12 with two dice is 6 and 6. Since each die roll is independent, we have: [LIST] [*]P(12) = P(6) * P(6) [*]P(12) = 1/6 * 1/6 [*]P(12) = 1/36. [/LIST] Now, what is the probability we roll a 12 five times in a row? The same rules apply, each new roll is independent of the last, so we multiply: [LIST] [*]P(12, 12, 12, 12, 12) = 1/36 * 1/36 * 1/36 * 1/36 * /36 [*]P(12, 12, 12, 12, 12) = [B]1/60,466,176[/B] or [B]1.65381717e-8[/B] [/LIST]

What is the probability that a month chosen at random has less than 31 days?
What is the probability that a month chosen at random has less than 31 days? Months with 31 days: [LIST=1] [*]January [*]March [*]May [*]July [*]August [*]October [*]December [/LIST] 7 months out of 12 have 31 days, so our probability is [B]7/12[/B]

what is the probabilty of tossing two coins and both landing on heads
what is the probabilty of tossing two coins and both landing on heads We want P(HH). We type in [URL='https://www.mathcelebrity.com/cointoss.php?hts=HH&hct=+2&tct=+1&fct=+5>=no+more+than&nmnl=+2&htpick=tails&calc=1&montect=500&pl=Calculate+Probability']HH into our search engine[/URL] and we get: P(HH) = [B]0.25 or 1/4[/B]

What is the range of possible values for a coefficient of correlation?
What is the range of possible values for a coefficient of correlation? [B]The range is -1.0 to +1.0[/B]

What is the ratio 18b^2 to 45b written in simplest for
What is the ratio 18b^2 to 45b written in simplest for Using our [URL='https://www.mathcelebrity.com/monomial.php?num1=+%286xy%5E3%29%5E4&num2=+%283y%5E4%29%5E5%288x%5E6y%5E3%29&num3=18b%5E2%2F45b&pl=Divide']monomial calculator[/URL], we see that 18b^2/45b is [B]2b/5[/B]

What is the ratio of the area of a circle to the area of a square drawn around that circle? Express
What is the ratio of the area of a circle to the area of a square drawn around that circle? Express your answer in terms of pi. Area of a circle = pir^2 area of a square = (2r)^2 = 4r^2 Ratio = pir^2/4r^2 Ratio = [B]pi/4[/B]

What is the ratio of vowels to consonants in the word RAINBOW
What is the ratio of vowels to consonants in the word RAINBOW Vowels (3): A, I, O Consonants (4): R, N, B, W Ratio of vowels to consonants: [B]3:4[/B]

What is the sample space for a 10 sided die?
What is the sample space for a 10 sided die? Sample space means the set of all possible outcomes. For a 10-sided die, we have: [B]{1, 2, 3, 4, 5, 6, 7, 8, 9, 10}[/B]

What is the simple interest accrued from a $500 investment at 7% interest for 5 years?
What is the simple interest accrued from a $500 investment at 7% interest for 5 years? Using our [URL='http://www.mathcelebrity.com/simpint.php?av=&p=500&int=7&t=5&pl=Simple+Interest']simple interest balance calculator[/URL], we get $175 in simple interest earned.

What is the slope of the line through (1,9) and (5,3)
What is the slope of the line through (1,9) and (5,3) [URL='https://www.mathcelebrity.com/slope.php?xone=1&yone=9&slope=+2%2F5&xtwo=5&ytwo=3&pl=You+entered+2+points']We run this through our slope calculator[/URL], and get an initial slope of 6/4. But this is not in simplest form. So we type 6/4 into our calculator, and s[URL='https://www.mathcelebrity.com/fraction.php?frac1=6%2F4&frac2=3%2F8&pl=Simplify']elect the simplify option[/URL]. We get [B]3/2[/B]

What is the sum of a number x and y raised to the power of two in algebraic expression
What is the sum of a number x and y raised to the power of two in an algebraic expression? The sum of a number x and y: x + y Raise this to the power of 2 (x + y)^2

What is the sum of four consecutive multiples of 5?
What is the sum of four consecutive multiples of 5? First number = n Second number = n + 5 Third number = n + 10 Fourth number = n + 15 Add them together: (n + n + n + n) + (5 + 10 + 15) [B]4n + 30[/B]

What is the value of an unknown number of nickels expressed in cents?
What is the value of an unknown number of nickels expressed in cents? 1 nickel = 5 cents n nickels = [B]5n[/B] cents

What is the value of n quarters expressed as dollars?
What is the value of n quarters expressed as dollars? dollars = quarters/4 [B]n/4 or 0.25n[/B]

What is the weight of a cubic meter of water? Express your answer in kilograms?
What is the weight of a cubic meter of water? Express your answer in kilograms? 1 kilogram per cubic decimeter and 1000 cubic decimeters in a cubic meter = [B]1000 kilograms[/B]

What is the weight of a liter of water expressed in kilograms?
What is the weight of a liter of water expressed in kilograms? the answer is [B]1 kilogram[/B]

What is the X coordinate of the point (6, 19)
What is the X coordinate of the point (6, 19) Using our [URL='https://www.mathcelebrity.com/ordered-pair.php?num=6%2C19&pl=Show+Detail']ordered pair calculator[/URL], we see that the x coordinate is [B]6[/B]

What number is half between 1.24 and 1.8?
What number is half between 1.24 and 1.8? Halfway between two points is called the midpoint. Using out [URL='http://www.mathcelebrity.com/mptnline.php?ept1=1.24&empt=&ept2=1.8&pl=Calculate+missing+Number+Line+item']midpoint calculator[/URL], we get 1.52:

What number when multiplied by four exceeds itself by 42?
What number when multiplied by four exceeds itself by 42? Let the number be n. We have: 4n = n + 42 Subtract n from each side: 3n = 42 Divide each side by 3 [B]n = 14[/B]

What pair of consecutive integers gives the following: 7 times the smaller is less than 6 times the
What pair of consecutive integers gives the following: 7 times the smaller is less than 6 times the larger? Let x and y be consecutive integers, where y = x + 1 We have 7x < 6y as our inequality. Substituting x, y = x + 1, we have: 7x < 6(x + 1) 7x < 6x + 6 Subtracting x from each side, we have: x < 6, so y = 6 + 1 = 7 (x, y) = (6, 7)

What pair of factors of -28 has a sum of -3
What pair of factors of -28 has a sum of -3? We type in [I]factor -28[/I] into our search engine. Scrolling down the list of factor sums, we see: -7 + 4 = -3 So our answer is [B](-7, 4)[/B]

WHAT SHOULD BE SUBTRACTED FROM -9876 TO OBTAIN -9512
WHAT SHOULD BE SUBTRACTED FROM -9876 TO OBTAIN -9512 We set up an arbitrary number x. Subtracted from is written as -9876 - x The phrase [I]to obtain[/I] means an equation, so we set -9876 - x equal to -9512 -9876 - x = -9512 To solve for x, we [URL='https://www.mathcelebrity.com/1unk.php?num=-9876-x%3D-9512&pl=Solve']type this equation into our search engine[/URL] and we get: x = [B]364[/B]

what’s the probability of rolling a 5 and then rolling a number less then 2
what’s the probability of rolling a 5 and then rolling a number less then 2 [U]Roll a 5:[/U] There's only one 5 on a six sided die P(X = 5) = 1/6 A number less than 2 is only 1: P(X < 2) = P(X = 1) P(X = 1) = 1/6 Since each event is independent, we multiply: P(X = 5) * P(X = 1) = 1/6 * 1/6 P(X = 5) * P(X = 1) = [B]1/36[/B]

When 20 is subtracted from 3 times a certain number, the result is 43
A certain number means an arbitrary variable, let's call it x x 3 times x 3x 20 is subtracted from 3 time x 3x - 20 The result is means equal to, so we set 3x - 20 equal to 43 for our algebraic expression [B]3x - 20 = 43 [/B] If you need to solve this, use our [URL='http://www.mathcelebrity.com/1unk.php?num=3x-20%3D43&pl=Solve']equation calculator[/URL]: [B]x = 21[/B]

When 28 is subtracted from the square of a number, the result is 3 times the number. Find the negati
When 28 is subtracted from the square of a number, the result is 3 times the number. Find the negative solution. Let the number be n. Square of a number: n^2 28 is subtracted from the square of a number: n^2 - 28 3 times the number: 3n [I]The result is[/I] mean an equation, so we set n^2 - 28 = 3n n^2 - 28 = 3n Subtract 3n from each side: n^2 - 3n - 28 = 3n - 3n The right side cancels to 0, so we have: n^2 - 3n - 28 = 0 This is a quadratic equation in standard form, so we [URL='https://www.mathcelebrity.com/quadratic.php?num=n%5E2-3n-28%3D0&pl=Solve+Quadratic+Equation&hintnum=+0']use our quadratic calculator[/URL] to solve: We get two solutions for n: n = (-4, 7) The question asks for the negative solution, so our answer is: [B]n = -4[/B]

When 3 consecutive positive integers are multiplied, the product is 16 times the sum of the 3 intege
When 3 consecutive positive integers are multiplied, the product is 16 times the sum of the 3 integers. What is the difference of the product minus the sum? Let the 3 consecutive positive integers be: [LIST=1] [*]x [*]x + 1 [*]x + 2 [/LIST] The product is: x(x + 1)(x + 2) The sum is: x + x + 1 + x + 2 = 3x + 3 We're told the product is equivalent to: x(x + 1)(x + 2) = 16(3x + 3) x(x + 1)(x + 2) = 16 * 3(x + 1) Divide each side by (x + 1) x(x + 2) = 48 x^2 + 2x = 48 x^2 + 2x - 48 = 0 Now subtract the sum from the product: x^2 + 2x - 48 - (3x + 3) [B]x^2 - x - 51[/B]

When 39 is added to a number, the result is 40 times the number. Find the number. Let n be the unkn
When 39 is added to a number, the result is 40 times the number. Find the number. Let n be the unknown number. Write the translated equation below. [LIST=1] [*]39 added to a number is written as n + 39 [*]40 times the number is written as 40n [*]The result is means we have an equation, so set (1) equal to (2) [/LIST] n+ 39 = 40n Running [URL='http://www.mathcelebrity.com/1unk.php?num=n%2B39%3D40n&pl=Solve']n + 39 = 40n through the search engine[/URL], we get[B] n = 1[/B].

When 4 is subtracted from the square of a number, the result is 3 times the number. Find the positiv
When 4 is subtracted from the square of a number, the result is 3 times the number. Find the positive solution. Let the number be n. We have: n^2 - 4 = 3n Subtract 3n from each side: n^2 - 3n - 4 = 0 [URL='https://www.mathcelebrity.com/quadratic.php?num=n%5E2-3n-4%3D0&pl=Solve+Quadratic+Equation&hintnum=+0']Typing this quadratic equation into the search engine[/URL], we get: n = (-1, 4) The problem asks for the positive solution, so we get [B]n = 4[/B].

When 4 times a number is increased by 40, the answer is the same as when 100 is decreased by the num
When 4 times a number is increased by 40, the answer is the same as when 100 is decreased by the number. Find the number The phrase [I]a number, [/I]means an arbitrary variable, let's call it "x". 4 times a number, increased by 40, means we multiply 4 times x, and then add 40 4x + 40 100 decreased by the number means we subtract x from 100 100 - x The problem tells us both of these expressions are the same, so we set them equal to each other: 4x + 40 = 100 - x Add x to each side: 4x + x + 40 = 100 - x + x The x's cancel on the right side, so we have: 5x + 40 = 100 [URL='https://www.mathcelebrity.com/1unk.php?num=5x%2B40%3D100&pl=Solve']Typing this equation into the search engine[/URL], we get [B]x = 12[/B].

When 4 times a number is increased by 40, the answer is the same as when 100 is decreased by the num
When 4 times a number is increased by 40, the answer is the same as when 100 is decreased by the number The phrase [I]a number[/I] means an arbitrary variable, let's call it x. x 4 times a number means we multiply x by 4: 4x Increased by 40 means we add 40 to 4x: 4x + 40 100 decreased by the number means we subtract x from 100: 100 - x The phrase [I]is the same as[/I] means equal to, so we set 4x + 40 equal to 100 - x 4x + 40 = 100 - x Solve for [I]x[/I] in the equation 4x + 40 = 100 - x [SIZE=5][B]Step 1: Group variables:[/B][/SIZE] We need to group our variables 4x and -x. To do that, we add x to both sides 4x + 40 + x = -x + 100 + x [SIZE=5][B]Step 2: Cancel -x on the right side:[/B][/SIZE] 5x + 40 = 100 [SIZE=5][B]Step 3: Group constants:[/B][/SIZE] We need to group our constants 40 and 100. To do that, we subtract 40 from both sides 5x + 40 - 40 = 100 - 40 [SIZE=5][B]Step 4: Cancel 40 on the left side:[/B][/SIZE] 5x = 60 [SIZE=5][B]Step 5: Divide each side of the equation by 5[/B][/SIZE] 5x/5 = 60/5 x = [B]12[/B] Check our work for x = 12: 4(12) + 40 ? 100 - 12 48 + 40 ? 100 - 12 88 = 88

When 54 is subtracted from the square of a number, the result is 3 times the number.
When 54 is subtracted from the square of a number, the result is 3 times the number. This is an algebraic expression. Let's take it in parts. The phrase [I]a number[/I] means an arbitrary variable, let's call it "x". x Square the number, means raise it to the 2nd power: x^2 Subtract 54: x^2 - 54 The phrase [I]the result[/I] means an equation, so we set x^2 - 54 equal to 3 [B]x^2 - 54 = 3[/B]

When 9 is subtracted from 5 times a number ,the result is 31
When 9 is subtracted from 5 times a number ,the result is 31 A number means an arbitrary variable, let's call it x. 5 times this number is written as 5x. 9 subtracted from this is written as 5x - 9 [I]The result[/I] means we have an equation, so we set [B]5x - 9 = 31[/B]. This is our algebraic expression. Now if we want to solve for x, we [URL='http://www.mathcelebrity.com/1unk.php?num=5x-9%3D31&pl=Solve']plug this equation into the search engine [/URL]and get [B]x = 8[/B].

When a circle's radius triples, what happens to its area?
When a circle's radius triples, what happens to its area? A = ?r^2 When r = 3r, then we have: a = ?(3r)^2 A = 9(?r^2) This means Area increases by [B]9x [MEDIA=youtube]j5aqShSh4uE[/MEDIA][/B]

When a dog noticed a fox, they were 60 meters apart. The dog immediately started to chase the fox at
When a dog noticed a fox, they were 60 meters apart. The dog immediately started to chase the fox at a speed of 750 meters per minute. The fox started to run away at a speed of 720 meters per minute. How soon will the dog catch the fox? The dog sits a position p. Distance = Rate x Time The dogs distance in minutes is D = 720t The fox sits at position p + 60 Distance = Rate x Time The fox's distance in minutes is D = 750t - 60 <-- Subtract 60 since the fox is already ahead 60 meters. We want to know when their distance (location) is the same. So we set both distance equations equal to each other: 720t = 750t - 60 [URL='https://www.mathcelebrity.com/1unk.php?num=720t%3D750t-60&pl=Solve']Using our equation calculator[/URL], we get [B]t = 2[/B]. Let's check our work: Dog's distance is 720(2) = 1440 Fox's distance is 750(2) - 60 = 1,440

When a number is doubled, the result is 36
When a number is doubled, the result is 36 Let the number be n. We have: 2n = 36 Using our [URL='http://www.mathcelebrity.com/1unk.php?num=2n%3D36&pl=Solve']equation solver[/URL], we see that [B]n = 18[/B]

When a number is doubled, the result is 36
Excited to announce these types of algebraic expressions can be [URL='http://www.mathcelebrity.com/algexpress.php?num=whenanumberisdoubled,theresultis36&pl=Write+Expression']typed directly in our search engine[/URL].

When an alligator is born it is about 8 inches long each year they grow 12 inches determine the age
When an alligator is born it is about 8 inches long each year they grow 12 inches determine the age and years of 116 inch alligator? Calculate inches to grow to get to 116 116 - 8 = 108 Now figure out how many years it takes growing at 12 inches per year, using y as years 12y = 108 Using our [URL='http://www.mathcelebrity.com/1unk.php?num=12y%3D108&pl=Solve']equation calculator[/URL], we get: [B]y = 9[/B]

When Esteban left for college, his parents decided to give him an allowance of $100 every 4 weeks. T
When Esteban left for college, his parents decided to give him an allowance of $100 every 4 weeks. They told Esteban that he could decide how he wanted raises to his allowance determined. Choice #1 - A raise of $10 every 4 weeks Choice #2 - A raise of $1.50 each week What choice should Esteban pick? Choice 1: [LIST] [*]First 4 weeks = $100 [*]Weeks 5 - 8 = $110 [*]Weeks -9-12 = $120 [*]Total = $330 [/LIST] Choice 2: [LIST] [*]1st week = $25 [*]2nd week = $26.50 [*]3rd week = $28 [*]4th week - $29.50 [*]5th week = $31.00 [*]6th week = $32.50 [*]7th week = $34.00 [*]8th week = $35.50 [*]9th week = $37.00 [*]10th week = $38.50 [*]11th week = $40.00 [*]12th week = $41.50 [*]Total = [B]$399[/B] [/LIST] [B]Choice 2 is the better option[/B]

When finding the power of a power, you _____________________ the exponents
When finding the power of a power, you _____________________ the exponents [B]Multiply [/B] Example: (a^b)^c = a^bc

When five people are playing a game called hearts, each person is dealt ten cards and the two remain
When five people are playing a game called hearts, each person is dealt ten cards and the two remaining cards are put face down on a table. Because of the rules of the game, it is very important to know the probability of either of the two cards being a heart. What is the probability that at least one card is a heart? Probability that first card is not a heart is 3/4 since 4 suits in the deck, hearts are 1/4 of the deck. Since we don't replace cards, the probability of the next card drawn without a heart is (13*3 - 1)/51 = 38/51 Probability of both cards not being hearts is found by multiplying both individual probabilities: 3/4 * 38/51 = 114/204 Having at least one heart is found by subtracting this from 1 which is 204/204: 204/204 - 114/204 = 90/204 [URL='https://www.mathcelebrity.com/search.php?q=90%2F204&x=0&y=0']This reduces to[/URL] [B]15/34[/B]

When m = 120 , the value of .10m + 54 is 66. Explain what this means in the context of this car rent
When m = 120 , the value of .10m + 54 is 66. Explain what this means in the context of this car rental company. This means 0.10 is the [B]per-mileage charge[/B] and $54 is the flat rate or rental fee

When Mike uses a riding mower, it takes him 3 hours to mow his lawn. When he uses a push mower, it t
When Mike uses a riding mower, it takes him 3 hours to mow his lawn. When he uses a push mower, it takes him 6 hours to mow the lawn. (His sister also can mow the lawn with the push mower in 6 hours.). Mike wanted to get the lawn mowed as quickly as possible, so he paid his sister $10 to mow with the push mower while he used the riding mower. How long will it take Mike an his sister to mow the lawn if they worked together? Mike can mow 1/3 of the lawn in an hour. Mike's sister can mow 1/6 the lawn in an hour. together, they can mow [URL='https://www.mathcelebrity.com/fraction.php?frac1=1%2F3&frac2=1%2F6&pl=Add']1/3 + 1/6 [/URL]= 1/2 of the lawn in one hour. Which means it would take [B]2 hours [/B](2 * 1/2) = 1 to mow the full lawn.

When Ms. Thelma turned on her oven, the temperature inside was 70 degrees F. The temperature began t
When Ms. Thelma turned on her oven, the temperature inside was 70 degrees F. The temperature began to rise at a rate of 20 degrees per minute. How Long did it take for the oven to reach 350 degrees F? Figure out how many degrees we have left: 350 - 70 = 280 Let m = minutes 20m = 280 Divide each side by m [B]m = 14[/B]

When ringing up a customer, a cashier needs 3 seconds to scan each item and 9 seconds to process the
When ringing up a customer, a cashier needs 3 seconds to scan each item and 9 seconds to process the payment. Let m represent the number of items and s represent the number of seconds to ring up a customer. Build our equation R(m): [B]R(m) = ms + 9[/B]

When the circumference of a circle is increased from 10 pi inches to 15 pi inches, by how many inche
When the circumference of a circle is increased from 10 pi inches to 15 pi inches, by how many inches is the radius increased? C = 2pir Smaller circle: 2pir = 10pi Divide each side by 2pi: r = 5 Larger circle: 2pir = 15pi Divide each side by 2pi: r = 7.5 Difference = 7.5 - 5 = [B]2.5 or 2&1/2 [MEDIA=youtube]HvMNNffcv78[/MEDIA][/B]

When the drain is closed, a swimming pool can be filled in 4 hours. When the drain is opened, it tak
When the drain is closed, a swimming pool can be filled in 4 hours. When the drain is opened, it takes 5 hours to empty the pool. The pool is being filled, but the drain was accidentally left open. How long until the pool is completely filled? Set up unit fill rates per hour: [LIST] [*]1/4 of the pool is filled each hour [*]1/5 of the pool is drained away each hour [/LIST] The amount left over after an hour of filling minus draining is: [URL='https://www.mathcelebrity.com/fraction.php?frac1=1%2F4&frac2=1%2F5&pl=Subtract']1/4 - 1/5[/URL] = 1/20 Therefore, it take [B]20 hours to fill the pool[/B]

When the side of a square is doubled in length, its area increases by 432 square inches. What is the
When the side of a square is doubled in length, its area increases by 432 square inches. What is the size of the original square? Original square side length is s Area = s^2 Double the side lengths to 2s New area = (2s)^2 = 4s^2 Setup the difference relation: 4s^2 - s^2 = 432 3s^2 = 432 Divide each side by 3: 3s^2/3 = 432/3 s^2 = 144 s = [B]12[/B]

When the water is turned on, a bathtub will be filled in 12 minutes. When the drain is opened it tak
When the water is turned on, a bathtub will be filled in 12 minutes. When the drain is opened it takes 20 minutes for the tub to drain. If the water is turned on and the drain is left open, how long until the tub is filled? In one hour, the faucet will fill 5 tubs since 12 * 5 = 60 minutes. In one hour, the drain will empty 3 tubs since 20 * 3 = 60 minutes The difference is 2 tubs filled per hour. Therefore, we have 1 tub filled in [B]1/2 hour or 30 minutes[/B]

When twice a number is reduced by 15 you get 95 what is the number
When twice a number is reduced by 15 you get 95 what is the number? The phrase [I]a number[/I] means an arbitrary variable, let's call it x. x [I]Twice[/I] x means we multiply x by 2 2x [I]Reduced by[/I] 15 means we subtract 15 2x - 15 [I]You get[/I] means equal to, as in an equation. Set 2x - 15 = 95 2x - 15 = 95 <-- This is our algebraic expression. [URL='https://www.mathcelebrity.com/1unk.php?num=2x-15%3D95&pl=Solve']Type 2x - 15 = 95 into the search engine[/URL] and we get [B]x = 55[/B].

When you conduct a hypothesis testing, at which of the following P-value, you feel more confident to
When you conduct a hypothesis testing, at which of the following P-value, you feel more confident to reject the null hypothesis? a. 0.05 b. 0.01 c. 0.95 d. 0.03 [B]b. 0.01[/B] [I]The lower the p value, the more confident you are about rejecting the null hypothesis.[/I]

which is a better deal, 8 pens for $6.16 or 7 pens for $5.46
which is a better deal, 8 pens for $6.16 or 7 pens for $5.46 Calculate unit cost for each deal: [LIST=1] [*]6.16/8 = 0.77 per pen [*]5.46/7 = 0.78 per pen [/LIST] [B]So deal #1, $6.16 for 8 pens is the better deal [/B]since each pen costs less than the other deal

Which number is larger? 1.00987 or 1.01
Which number is larger? 1.00987 or 1.01 1.01 can be written as: 1 & 1,000/100,000 1.00987 can be written as: 1 & 987/100,000 Since 1,000 > 987, then [B]1.01 is larger [URL='https://www.mathcelebrity.com/compare.php?num1=1.00987&num2=1.01&pl=Compare']Source[/URL][/B]

which number is the same distance from 0 on the number line as 4
which number is the same distance from 0 on the number line as 4 We use absolute value for distance. Since 4 is 4 units right of 0 on the number line, we can also move 4 units left of 0 on the number line and we land on [B]-4[/B]

Which of the following can increase power?
Which of the following can increase power? a. Increasing standard deviation b. Decreasing standard deviation c. Increasing both means but keeping the difference between the means constant d. Increasing both means but making the difference between the means smaller [B]b. Decreasing standard deviation[/B] [LIST=1] [*]Power increases if the standard deviation is smaller. [*]If the difference between the means is bigger, the power is bigger. [*]Sample size increase also increases power [/LIST]

Which of the following could reduce the rate of Type I error? a. Making the significant level from
Which of the following could reduce the rate of Type I error? a. Making the significant level from 0.01 to 0.05 b. Making the significant level from 0.05 to 0.01 c. Increase the Β level d. Increase the power [B]a. Making the significant level from 0.01 to 0.05[/B] [I]This widens the space under the graph and makes the test less strict.[/I]

Which of the following descriptions of confidence interval is correct? (Select all that apply) a. I
Which of the following descriptions of confidence interval is correct? (Select all that apply) a. If a 99% confidence interval contains 0, then the 95% confidence interval contains 0 b. If a 95% confidence interval contains 0, then the 99% confidence interval contains 0 c. If a 99% confidence interval contains 1, then the 95% confidence interval contains 1 d. If a 95% confidence interval contains 1, then the 99% confidence interval contains 1 [B]a. If a 99% confidence interval contains 0, then the 95% confidence interval contains 0 c. If a 99% confidence interval contains 1, then the 95% confidence interval contains 1 [/B] [I]The lower the confidence interval, the wider the range, so if a higher confidence interval contains a point, a lower confidence interval will contain that point as well.[/I]

Which of the following descriptions of null hypothesis are correct? (Select all that apply) a. A nu
Which of the following descriptions of null hypothesis are correct? (Select all that apply) a. A null hypothesis is a hypothesis tested in significance testing. b. The parameter of a null hypothesis is commonly 0. c. The aim of all research is to prove the null hypothesis is true d. Researchers can reject the null hypothesis if the P-value is above 0.05 [B]a. A null hypothesis is a hypothesis tested in significance testing. [/B] [I]b. is false because a parameter can be anything we choose it to be c. is false because our aim is to disprove or fail to reject the null hypothesis d. is false since a p-value [U]below[/U] 0.05 is often the rejection level.[/I]

Which of the following equations represents a line that is parallel to the line with equation y = -3
Which of the following equations represents a line that is parallel to the line with equation y = -3x + 4? A) 6x + 2y = 15 B) 3x - y = 7 C) 2x - 3y = 6 D) x + 3y = 1 Parallel lines have the same slope, so we're looking for a line with a slope of 3, in the form y = mx + b. For this case, we want a line with a slope of -3, like our given line. If we rearrange A) by subtracting 6x from each side, we get: 2y = -6x + 15 Divide each side by 2, we get: y = -3x + 15/2 This line is in the form y = mx + b, where m = -3. So our answer is [B]A[/B].

Which of the following involves making pairwise comparisons? a. Comparing the standard deviation of
Which of the following involves making pairwise comparisons? a. Comparing the standard deviation of GRE grades between two states b. Comparing the variance of the amount of soda consumed by boys and girls in a high school c. Comparing the mean weight between children in grades 2, 3, 4 and 5 d. Comparing the number of restaurants in New York and Boston [B]c. Comparing the mean weight between children in grades 2, 3, 4 and 5[/B] Pairwise comparison generally refers to any process of comparing entities in pairs to judge which of each entity is preferred, or has a greater amount of some quantitative property

Which of the following is equivalent to 3(2x + 1)(4x + 1)?
Which of the following is equivalent to 3(2x + 1)(4x + 1)? [LIST] [*]A) 45x [*]B) 24x^2 + 3 [*]C) 24x^2 + 18x + 3 [*]D) 18x^2 + 6 [/LIST] First, [URL='https://www.mathcelebrity.com/binomult.php?term1=2x%2B1&term2=4x%2B1&pl=Expand+Product+of+2+Binomials+using+FOIL']multiply the binomials[/URL]: We get 8x^2 + 6x + 1 Now multiply this polynomial by 3: 3(8x^2 + 6x + 1) = [B]24x^2 + 18x + 3, answer C[/B]

Which of the following is equivalent to 9^3/4?
Which of the following is equivalent to 9^3/4? a) 9^1/3 b) 9 ^ 1/4 c) sqrt(3) d) 3 * sqrt(3) Since 9 is 3^2, we have 3^(3*2/4) which is 3^6/4 Since 6/4 is 3/2, we have: 3^(3/2) Since 3/2 is 1 + 1/2, we have: 3^1*sqrt(3) [B]3*sqrt(3) or option D. [MEDIA=youtube]Uq544xLphiM[/MEDIA][/B]

Which of the following is equivalent to the sum of the expression a^2 - 1 and a + 1?
Which of the following is equivalent to the sum of the expression a^2 - 1 and a + 1? A) a^2 + a B) a^3 - 1 C) 2a^2 D) a^3 a^2 - 1 + a + 1 The 1's cancel, so we're left with: [B]a^2 + a - Answer A[/B]

Which of the following is NOT TRUE about the distribution for averages?
Which of the following is NOT TRUE about the distribution for averages? a. The mean, median, and mode are equal. b. The area under the curve is one. c. The curve never touches the x-axis. d. The curve is skewed to the right. Answer is d, the curve is skewed to the right For a normal distribution: [LIST] [*] The area under the curve for a standard normal distribution equals 1 [*] Mean media mode are equal [*] Never touches the x-axis since in theory, all events have some probability of occuring [/LIST]

Which of the following is the probability that a green marble will be selected from a bag containing
Which of the following is the probability that a green marble will be selected from a bag containing 9 red marbles 6 blue marbles 7 green marbles and 11 yellow marbles if one is selected randomly? Total marbles in the bag: 9 red + 6 blue + 7 green + 11 yellow = 33 P(Green) = Green Marbles / Total Marbles P(Green) = [B]7/33[/B]

Which of the following is the probability that subjects do not have the disease, but the test result
Which of the following is the probability that subjects do not have the disease, but the test result is positive? a. Miss rate b. False positive rate c. Base rate d. Disease rate [B]b. [URL='http://sites.stat.psu.edu/~lsimon/stat250/sp00/solutions/misc/diagtests.htm']False positive rate[/URL][/B]

Which of the following shows the numbers in ascending order? 2/3,0.68,67%,4/5
Which of the following shows the numbers in ascending order? 2/3,0.68,67%,4/5 A. 67%, 0.68, 2/3, 4/5 B. 67%, 0.68, 4/5, 2/3 C. 0.68, 67%, 2/3, 4/5 D. 2/3, 67%, 0.68, 4/5 Convert all of these to decimals: [LIST] [*]2/3 = 0.666666 [*]0.68 = 0.68 [*]67% = 0.67 [*]4/5= 0.8 [/LIST] Order these ascending, and you get answer D [B]D. 2/3, 67%, 0.68, 4/5[/B] [MEDIA=youtube]ABnPvvZhv6k[/MEDIA]

Which of the followings can increase the value of t? (select all the apply) a. Increase the standar
Which of the followings can increase the value of t? (select all the apply) a. Increase the standard deviation of difference scores b. Decrease the standard deviation of difference scores c. Increase the difference between means d. Decrease the difference between means [B]b. Decrease the standard deviation of difference scores c. Increase the difference between means[/B] [I]Increase numerator or decrease denominator of the t-value formula[/I]

Which of the followings is the definition of power? a. Power is the probability of rejecting a null
Which of the followings is the definition of power? a. Power is the probability of rejecting a null hypothesis b. Power is the probability of accepting a null hypothesis c. Power is the probability of accepting a false null hypothesis d. Power is the probability of rejecting a false null hypothesis [B]d. Power is the probability of rejecting a false null hypothesis[/B]

Which phrase is represented by the equation p equals kv
Which phrase is represented by the equation p equals kv [B]p varies directly as v[/B]

while scuba diving jerey rose directly toward the surface of the water at a constant velocity for 2.
while scuba diving jerey rose directly toward the surface of the water at a constant velocity for 2.0 minutes. he rose 9.0 meters in that time. what was his velocity? 9 meters / 2 minutes = [B]4.5 meters / minute[/B]

Whitney has already baked 2 cakes, and she can bake 1 cake with each additional stick of butter she
Whitney has already baked 2 cakes, and she can bake 1 cake with each additional stick of butter she buys. Write an equation that shows the relationship between the number of additional sticks of butter s and the number of cakes c. [LIST] [*]Let c, the number of cakes, be represented by f(s) where s are the number of sticks of butter. [*]We already have 2 cakes to start, and each additional stick of butter gets us one more cake. [/LIST] f(s) = 1s + 2 Simplify, since 1s is just s [B]f(s) = s + 2[/B]

Wilbie had candy to give to his 3 children. He first took 5 pieces and evenly divided the rest among
Wilbie had candy to give to his 3 children. He first took 5 pieces and evenly divided the rest among each child. Each child received 3 pieces. With how many pieces did he start? Let the starting candy amount be c. We're given: (c - 5)/3 = 3 Cross multiply: c - 5 = 3*3 c - 5 = 9 [URL='https://www.mathcelebrity.com/1unk.php?num=c-5%3D9&pl=Solve']Type this equation into the search engine[/URL], and we get: c = 14

Wind Chill Factor
Free Wind Chill Factor Calculator - This calculator determines the wind chill factor given a temperature in F° and a wind speed in miles per hour (mph). Simply enter your temperature and wind speed and press the button

Winnie earns an annual salary of $55,117. If she pays $3,715 a year in taxes and receives a paycheck
Winnie earns an annual salary of $55,117. If she pays $3,715 a year in taxes and receives a paycheck every other week, how much does Winnie receive from each paycheck? Subtract the taxes to get Winnie's Total net pay: Total Net Pay = Annual Salary - Annual Taxes Total Net Pay =$55,117 - $3,715 Total Net Pay = $51,402 Now, if Winnie gets paid every other week, and there are 52 weeks in a year, then she gets paid 26 times. Calculate single paycheck amount Single Paycheck Amount = Total Net Pay / 26 payments Single Paycheck Amount = $51,402 / 26 Single Paycheck Amount = [B]$1,977[/B]

Woorank Certification
Exam answers and study guide for the Woorank Certification exam

Word Notation
Free Word Notation Calculator - Calculates the following:
* The word notation of a number of numeric expression

Word Problem
Suppose the consumption of electricity grows at 5.3% per year, compounded continuously. Find the number of years before the use of electricity has tripled. Round to the nearest hundredth.

Word Problem Help
A man is three times as old as his son was at the time when the father was twice as old as his son will ne two years from now. Find the present ages of each person.

Word Problem Help
I'm confused. Is there a father, grandfather, and son involved in this problem?

Word Problem Help
I am confused too it is okay because I turned in some random stuff but the teacher explained in class so I know now

Work
Free Work Calculator - Solves for any of the 3 variables, Work (W), Force (F) and Distance (d) in the work formula

Work Word Problems
Free Work Word Problems Calculator - Given Person or Object A doing a job in (r) units of time and Person or Object B doing a job in (s) units of time, this calculates how long it would take if they combined to do the job.

Write a fraction with a denominator of 9. The fraction should be less than 1/2
Write a fraction with a denominator of 9. The fraction should be less than 1/2 Let n be the numerator. We have: n/9 < 1/2 multiply each side by 9: 9n/9 < 9/2 n < 9/2 Examples are 8/2, 7/2, 6/2, 5/2, 4/2, 3/2,

Write a model that utilizes all three explanatory variables with no interaction or quadratic terms.
Write a model that utilizes all three explanatory variables with no interaction or quadratic terms. Choose the correct answer below. A. y i = B0 + B1x1 + B2x2 + B3x3 + e i B. y i = B0 + B1x1 + B2x2 + B3x3x2 + e i C. y i = B1x1 + B2x2 + B3x3 + ei D. None of the above equations satisfy all of the conditions [B]A. y i = B0 + B1x1 + B2x2 + B3x3 + e i[/B]

Write a system of equations to describe the situation below, solve using any method, and fill in the
Write a system of equations to describe the situation below, solve using any method, and fill in the blanks. Hugo is going to send some flowers to his wife. Somerville Florist charges $2 per rose, plus $39 for the vase. Dwaynes Flowers, in contrast, charges $3 per rose and $10 for the vase. If Hugo orders the bouquet with a certain number of roses, the cost will be the same with either flower shop. What would the total cost be? How many roses would there be? Let r be the number of roses and C(r) be the cost function. The vase is a one-time cost. Somerville Florist: C(r) = 2r + 39 Dwaynes Flowers C(r) = 3r + 10 Set them equal to each other: 2r + 39 = 3r + 10 Using our [URL='http://www.mathcelebrity.com/1unk.php?num=2r%2B39%3D3r%2B10&pl=Solve']equation calculator[/URL], we get: [B]r = 29[/B]

Write an algebraic expression for 8 multiplied by the result of u reduced by 11.
Write an algebraic expression for 8 multiplied by the result of u reduced by 11. u [I]reduced by[/I] 11 Reduced by means subtract 11 from u. So we have: u - 11 We multiply this expression by 8 to get our algebraic expression of: [B]8(u - 11)[/B]

Write an equation in slope-intercept form for the line with slope 4 and y-intercept -7
Write an equation in slope-intercept form for the line with slope 4 and y-intercept -7 The standard equation for slope (m) and y-intercept (b) is given as: y = mx + b We're given m = 4 and y-intercept = -7, so we have: [B]y = 4x - 7[/B]

Write an equation that relates the quantities. G varies jointly with t and q and inversely with the
Write an equation that relates the quantities. G varies jointly with t and q and inversely with the cube of w . The constant of proportionality is 8.25 . [LIST] [*]Varies jointly or directly means we multiply [*]Varies inversely means divide [*]The cube of w means we raise w to the 3rd power: w^3 [/LIST] Using k = 8.25 as our constant of proportionality, we have: [B]g = 8.25qt/w^3[/B]

Write an expression for the amount of money in p pennies plus 7 dollars.
Write an expression for the amount of money in p pennies plus 7 dollars. Each penny is worth 0.01, so we have: [B]0.01p + 7d[/B]

Write in set builder form {all possible numbers formed by any two of the digits 1 2 5}
Write in set builder form {all possible numbers formed by any two of the digits 1 2 5} With 3 numbers, we got [URL='https://www.mathcelebrity.com/factorial.php?num=3!&pl=Calculate+factorial']3! = 6[/URL] possible numbers formed by the two digits [LIST=1] [*]12 [*]15 [*]21 [*]25 [*]51 [*]52 [/LIST] In set builder notation, we write this as: {x : x ? {12, 15, 21, 25, 51, 52}) x such that x is a element of the set {12, 15, 21, 25, 51, 52}

Write p times p times p times p times p times p and n index form
Write p times p times p times p times p times p and n index form We have p times itself 6 times, so the index form is: [B]p^6[/B]

Write the interval (2,5) in set builder notation
Write the interval (2,5) in set builder notation It's a closed interval, so [URL='https://www.mathcelebrity.com/interval-notation-calculator.php?num=%5B2%2C5%5D&pl=Show+Interval+Notation']we type in [2,5] into the search engine[/URL], and we get: [B]{x|2<= x <= 5}[/B]

Write the sentence as an equation. 19 is equal to c less than 321
Write the sentence as an equation. 19 is equal to c less than 321 c less than 321: 321 - c The phrase [I]is equal to[/I] means an equation, so we set 321 - c equal to 19: [B]321 - c = 19[/B]

Write the verbal expression for: 9x
Write the verbal expression for: 9x Using our [URL='http://www.mathcelebrity.com/verbalphrase.php?num=9x&pl=Verbal+Phrase']verbal expression calculator[/URL], we get either of the following: [LIST] [*][B]9 times x[/B] [*][B]9 multiplied by x[/B] [/LIST]

Write the verbal expression for: 9x
You can now add shortcuts in the search engine for this.

Write these times as 24 hours times: 1:00 pm 8:10 am 4:45 pm 10:12 pm Write these times as 12 hours
Write these times as 24 hours times: 1:00 pm 8:10 am 4:45 pm 10:12 pm Write these times as 12 hours time: 15:45 7:12 20:38 12:01 Write these times as 24 hours times (any time on or after 1:00 pm, we add 1 to the 12 noon marker: 1:00 pm = 12 + 1 = 13:00 8:10 am = 8:10 <-- since not past 12 noon 4:45 pm = 4 hours and 45 minutes past 12 noon, so we have 16:45 10:12 pm = 10 hours and 12 minutes past 12 noon, so we have 22:12 Write these times as 12 hours time: 15:45 = 15:45 - 12 = 3:45 PM 7:12 = 7:12, not past noon, so 7;12 am 20:38 = 20:38 - 12 = 8:38 PM 12:01 = 12:01 pm

writing and solving equations
My daughter is having issues with a math problem for her homework. she tells me that I am doing it wrong but I am getting the correct answer... Can you please look at it and see if i am correct? The problem is: A painter charges $15.35 per hour, plus an additional amount for supplies. If he made $141.73 on a job where he worked 4 hours, how much did the supplies cost? I have the equation as: $15.35 * 4 = $141.73 - x ... I got the answer of $80.33 for supplies She is telling me that the teacher is wanting her to do the PEMDAS backwards but that is not working out for her and I am not understanding that at all. Any suggestions would help out Thanks, Tina

writing and solving equations
Your answer is correct. Here is how I set up the profit equation where h is the hours worked and x is the supply cost: P(h) = 15.35h + x We know P(4) = 141.73 P(4) = 15.35(4) + x 141.73 = 15.35(4) + x Simplify 141.73 = 61.4 + x Subtract 61.4 from each side: [B]x = 80.33[/B]

x + 8y/4 = 9y for x
x + 8y/4 = 9y for x Step 1: Isolate x by subtracting 8y/4 from each side: x + 8y/4 - 8y/4 = 9y - 8y/4 Cancel 8y/4 on the left side: x =[B] 9y - 8y/4 [MEDIA=youtube]5NLDNw_T8GU[/MEDIA][/B]

x - 5y = 6 for x
x - 5y = 6 for x Add 5y to each side to solve this literal equation for x. x - 5y + 5y = 6 + 5y Cancel the 5y on each side, we get: x = [B]6 + 5y[/B]

x add y, multiply by z then subtract d
x add y, multiply by z then subtract d Take this algebraic expression in pieces: [LIST] [*]x add y: x + y [*]multiply by z: z(x + y) [*]Subtract d: [B]z(x + y) - d[/B] [/LIST]

X bisects WY. XY=32 and WY=2x. Find x and WY
X bisects WY. XY=32 and WY=2x. Find x and WY\ Bisects means split into two equal parts. So we have: XY = 32 WX = XY If XY = 32, then: WY = 2 * 32 =[B] 64[/B] So x = [B]32[/B]

x cubed plus x squared decreased by 7
x cubed plus x squared decreased by 7 [U]x cubed means we raise x to the power of 3:[/U] x^3 [U]x squared means we raise x to the power of 2:[/U] x^2 [U]x cubed plus x squared[/U] x^3 + x^2 [U]Decreased by 7:[/U] [B]x^3 + x^2 - 7[/B]

x is a multiple of 6 and 1 ? x ? 16
x is a multiple of 6 and 1 ? x ? 16. We want multiples of 6 between 1 and 16. We start with 6. Another multiple of 6 is 12 The next multiple of 6 is 18, which is out side the range of 1 ? x ? 16. So our number set is [B]x = {6, 12}[/B]

X is a natural number greater than 6
I saw this ticket come through today. The answer is x > 6. Natural numbers are positive numbers not 0. So 1, 2, 3, ... Let me build this shortcut into the calculator. Also, here is the[URL='http://www.mathcelebrity.com/interval-notation-calculator.php?num=x%3E6&pl=Show+Interval+Notation'] interval notation[/URL] for that expression.

x is a woman who served as US president before 2000
x is a woman who served as US president before 2000 No woman US presidents before 2000, so we have the empty set. [B]x = {}[/B]

X is at least as large as 4
X is at least as large as 4. This is an algebraic expression, where the phrase [I]at least as large as[/I] means greater than or equal to: [B]x >=4[/B]

X is not between 6 and 12
You can now type in to the search engine directly, X is not between 6 and 12

x is smaller than 9 and greater than 4
x is smaller than 9 and greater than 4 Written as: 4 < x < 9 To display the interval notation, use our [URL='http://www.mathcelebrity.com/interval-notation-calculator.php?num=4%3Cx%3C9&pl=Show+Interval+Notation']interval notation calculator[/URL].

X is the speed limit is a maximum 65 mph
X is the speed limit is a maximum 65 mph A maximum of means less than or equal to. Or, no more than. So we have the inequality: [B]X <= 65[/B]

x is tripled and then 2 is added
x is tripled and then 2 is added Tripled means multiply x by 3: 3x Then add 2 to this: [B]3x + 2[/B]

X plus 9 is equal to 3 times x minus 4
X plus 9 is equal to 3 times x minus 4 x plus 9: x + 9 3 times x minus 4: 3x - 4 The phrase [I]is equal to[/I] means an equation, so we set x + 9 equal to 3x - 4: [B]x + 9 = 3x - 4[/B]

x squared plus a minus b
x squared plus a minus b x squared means we raise x to the power of 2: x^2 Plus a: x^2 + a Minus b: [B]x^2 + a - b[/B]

x squared times the difference of x and y
x squared times the difference of x and y x squared means we raise x to the power of 2: x^2 The difference of x and y x - y x squared times the difference of x and y [B]x^2(x - y)[/B]

X to the 9th is less than or equal to 38
X to the 9th is less than or equal to 38: x to the 9th means 9th power: x^9 We set this less than or equal 38: [B]x^9 <= 38[/B]

X varies directly with the cube root of y when x=1 y=27. Calculate y when x=4
X varies directly with the cube root of y when x=1 y=27. Calculate y when x=4 Varies directly means there is a constant k such that: x = ky^(1/3) When x = 1 and y = 27, we have: 27^1/3(k) = 1 3k = 1 To solve for k, we[URL='https://www.mathcelebrity.com/1unk.php?num=3k%3D1&pl=Solve'] type in our equation into our search engine[/URL] and we get: k = 1/3 Now, the problem asks for y when x = 4. We use our variation equation above with k = 1/3 and x = 4: 4 = y^(1/3)/3 Cross multiply: y^(1/3) = 4 * 3 y^(1/3) =12 Cube each side: y^(1/3)^3 = 12^3 y = [B]1728[/B]

x-m=n+p, for x
x-m=n+p, for x Add m to each side: x - m + m = n + p + m Cancelling the m's on each side: x = [B]n + p + m[/B]

x/3 - g = a for x
x/3 - g = a for x Add g to each side so we can isolate the x term: x/3 - g + g = a + g Cancel the g terms on the left side and we get: x/3 = a + g Multiply each side by 3 to isolate x: 3(x/3) = 3(a + g) Cancelling the 3's on the left side, we get: x = [B]3(a + g)[/B]

x/5-7=2q for x
x/5-7=2q for x Add 7 to each side: x/5 -7 + 7 = 2q + 7 Cancel the 7's on the left side, we get: x/5 = 2q + 7 Cross multiply the 5: x = 5(2q + 7) x = [B]10q + 35[/B]

x/r - h = 4 for x
x/r - h = 4 for x Add h to each side: x/r - h + h = h + 4 Cancel the h's on the left side, we get: x/r = h + 4 Multiply each side by r to isolate x: xr/r = r(h + 4) Cancel the r's on the left side, we get: x = [B]r(h + 4)[/B]

x/y + 9 = n for x
x/y + 9 = n for x Subtract 9 from each side to isolate the x term: x/y + 9 - 9 = n - 9 Cancel the 9's on the left side and we get: x/y = n - 9 Because we have a fraction on the left side, we can cross multiply the denominator y by n - 9 [B]x =[/B] [B]y(n - 9)[/B]

x/y + 9 = n for y
x/y + 9 = n for y First, subtract 9 from each side to isolate the y term: x/y + 9 - 9 = n - 9 Cancel the 9's on the left side, and we get: x/y = n - 9 Cross multiply: x = y(n - 9) Divide each side by (n - 9): x/(n - 9) = y(n - 9)/(n - 9) Cancel the (n - 9) on the right side, and we get: y = [B]x/(n - 9)[/B]

x/y = z - 8 for x
x/y = z - 8 for x Multiply each side by y to isolate x: y*(x/y) = y(z - 8) The y's cancel out on the left side, so we have: x = [B]y(z - 8)[/B]

x/y = z - 8 for x
x/y = z - 8 for x We start by seeing that x is isolated. To remove y from the left side, we multiply each side of the equation by y: xy/y = y(z - 8) Cancelling y on the left side, we get our answer of: x = [B]y(z - 8) [MEDIA=youtube]_HNyGlnnQdQ[/MEDIA][/B]

Xavier has $132 to buy a video game. Each game costs $12. Write an equation to find the number of ga
Xavier has $132 to buy a video game. Each game costs $12. Write an equation to find the number of games Xavier can purchase. Let g be the number of games, we have a cost function C(g) C(g) = 12g We want to find g such that C(g) = 132 12g = 132 Divide each side by 12 [B]g = 11[/B]

Xaviers birthday party costs $3 for every guest he invites. If there are 8 guests, how much money wi
Xaviers birthday party costs $3 for every guest he invites. If there are 8 guests, how much money will Xaviers birthday party cost Cost = Amount per guest * number of guest Cost = 3 * 8 Cost = [B]24[/B]

Y add z then divide by x
Y add z then divide by x y add z: y + z Then divide by x means we divide the sum (y + z) by x [B](y + z)/x[/B]

y is the sum of twice a number and 3
y is the sum of twice a number and 3 The phrase [I]a number[/I] means an arbitrary variable, let's call it x. x twice a number means we multiply x by 2: 2x the sum of twice a number and 3: 2x + 3 The word [I]is[/I] means equal to, so we set 2x + 3 equal to y [B]y = 2x + 3[/B]

y minus 10 is equal to the product of y and 8
y minus 10 is equal to the product of y and 8. Take this algebraic expression in 3 parts: Part 1: y minus 10 Subtract 10 from the variable y y - 10 Part 2: The product of y and 8 We multiply 8 by the variable y 8y Part 3: The phrase [I]is equal to[/I] means an equation, so we set y - 10 equal to 8y [B]y - 10 = 8y[/B]

y varies directly as the reciprocal of x
y varies directly as the reciprocal of x The reciprocal of x is written as: 1/x The phrase [I]varies directly[/I] means there exists a constant k such that [B]y = k/x[/B]

y varies directly as x and inversely as i
y varies directly as x and inversely as I Note: Direct variation means we multiply. Inverse variation means we divide. There exists a constant k such that: [B]y = kx/i[/B]

y/2+c=d for y
Multiply each side by 2 to isolate y. y +2c = 2d Subtract 2c from each side of the equation: y = 2d - 2c This can also be written y = 2(d - c)

Yael worked out at a gym for 2 hours. Her workout consisted of stretching for 21 minutes,jogging for
Yael worked out at a gym for 2 hours. Her workout consisted of stretching for 21 minutes,jogging for 45 minutes, and lifting weights for the remaining amount of time. What percentage of Yael’s workout was spent lifting weights? Each hour is 60 minutes, so we have 2 * 60 = 120 minutes of workout time for Yael. We subtract off the stretching and jogging time to get the time Yael lifted weights: 120 - 21 - 45 = 54 minutes

Yahtzee-1st Roll
Free Yahtzee-1st Roll Calculator - Calculates the probability of various scoring hands in the game of Yahtzee on the 1st roll of the dice.

Yandex Direct Exam
Exam answers and Study Guide for the Yandex Direct Exam

Yesterday a car rental agency rented 4 convertibles and 30 other vehicles. Considering this data, ho
Yesterday a car rental agency rented 4 convertibles and 30 other vehicles. Considering this data, how many of the first 68 vehicles rented today should you expect to be convertibles? 30 other vehicles + 4 convertibles = 34 cars 34 * 2 = 68 30 * 2 other vehicles + 4 * 2 convertibles = 68 cars 60 other vehicles and [B]8 convertibles[/B]

Yesterday, Boris had 144 baseball cards. Today, he got m more. Using m, write an expression for the
Yesterday, Boris had 144 baseball cards. Today, he got m more. Using m, write an expression for the total number of baseball cards he has now. 144 and m more means we add [B]144 + m[/B]

Yesterday, Kareem had n baseball cards. Today, he got 9 more. Using n, write an expression for the t
Yesterday, Kareem had n baseball cards. Today, he got 9 more. Using n, write an expression for the total number of baseball cards he has now. 9 more means we add 9 to n [B]n + 9[/B]

Yesterday, there were 100 problems assigned for math homework. Andrew got 20 problems correct and 80
Yesterday, there were 100 problems assigned for math homework. Andrew got 20 problems correct and 80 problems incorrect. What percentage did Andrew get correct? Correct Problems = 20/100 [URL='https://www.mathcelebrity.com/perc.php?num=20&den=100&pcheck=1&num1=16&pct1=80&pct2=70&den1=80&idpct1=10&hltype=1&idpct2=90&pct=82&decimal=+65.236&astart=12&aend=20&wp1=20&wp2=30&pl=Calculate']Using our percentage and decimal calculator[/URL], we get [B]20%[/B]

Yesterday, there were 72 problems assigned for math homework. Austen got 18 problems correct and 54
Yesterday, there were 72 problems assigned for math homework. Austen got 18 problems correct and 54 problems incorrect. What percentage did Austen get correct? [URL='https://www.mathcelebrity.com/perc.php?num=18&den=72&pcheck=1&num1=+16&pct1=+80&pct2=+35&den1=+90&pct=+82&decimal=+65.236&astart=+12&aend=+20&wp1=20&wp2=30&pl=Calculate']18/72 as a percentage[/URL] is [B]25%[/B]

Yolanda had 9 cookies in a jar. She then bought 6 bags of 13 cookies each. How many cookies does she
Yolanda had 9 cookies in a jar. She then bought 6 bags of 13 cookies each. How many cookies does she have now? 6 bags * 13 cookies each = 78 cookies. 78 cookies added to the original 9 in the jar = [B]87 cookies[/B].

Yolanda is riding her bicycle. She rides for 5 hours at a speed of 12.5 kilometers per hours. For ho
Yolanda is riding her bicycle. She rides for 5 hours at a speed of 12.5 kilometers per hours. For how many kilometers does she ride? This is a distance problem, where distance = rate * time. We are given time of 5 hours, at a rate of 12.5km/hour. Using our [URL='http://www.mathcelebrity.com/drt.php?d=+&r=12.5&t=5&pl=Calculate+the+missing+Item+from+D%3DRT']distance calculator[/URL], we get D = [B]62.5km[/B].

Yolanda runs each lap in 7 minutes. She will run less than 35 minutes today. What are the possible n
Yolanda runs each lap in 7 minutes. She will run less than 35 minutes today. What are the possible numbers of laps she will run today? 7 minutes per lap must be less than 35 minutes. Let l be the number of laps 7l < 35 Divide each side by 7 [B]l < 5[/B]

Yolanda wants to rent a boat and spend less than $41. The boat costs $8 per hour, and Yolanda has a
Yolanda wants to rent a boat and spend less than $41. The boat costs $8 per hour, and Yolanda has a discount coupon for $7 off. What are the possible numbers of hours Yolanda could rent the boat? A few things to build this problem: [LIST=1] [*]Discount subtracts from our total [*]Cost = Hourly rate * hours [*]Less than means an inequality using the < sign [/LIST] Our inequality is: 8h - 7 < 41 To solve this inequality for h, we [URL='https://www.mathcelebrity.com/1unk.php?num=8h-7%3C41&pl=Solve']type it in our math engine[/URL] and we get: h < [B]6[/B]

Yosemite National Park charges $7 per person for an all day admission to the park. The total cost fo
Yosemite National Park charges $7 per person for an all day admission to the park. The total cost for n people to go to the park all day is given by the expression 7n. 8 friends go to the park on Saturday. What is the total cost of admission? We want to evaluate f(n) = 7n for n = 8 f(8) = 7(8) = [B]56[/B]

you and 5 friends go to a concert. how many different ways can you sit in the assigned seats
You and 5 friends go to a concert. how many different ways can you sit in the assigned seats? With 6 possible seats, the [URL='https://www.mathcelebrity.com/factorial.php?num=6!&pl=Calculate+factorial']number of unique arrangements is[/URL]: 6! = 6 x 5 x 4 x 3 x 2 x 1 = [B]720[/B]

You and 6 friends went out for pizza. When the total amount was split, each person paid $3. How much
You and 6 friends went out for pizza. When the total amount was split, each person paid $3. How much was the total bill? You and 6 friends means 7 total people. If the bill was split, everybody paid the same. We have: 7 people * $3 per person = [B]$21 total bill[/B]

You and a friend collect acorns from a field. After g minutes u have collected(10 + 2g) acorns and y
You and a friend collect acorns from a field. After g minutes u have collected(10 + 2g) acorns and your friend has collected (5g - 2) acorns. How many total acorns have you and your friend collected Add both acorn collections together: (10 + 2g) + (5g - 2) Group like terms: (5 + 2)g + 10 - 2 [B]7g + 8[/B]

You and a friend want to start a business and design t-shirts. You decide to sell your shirts for $1
You and a friend want to start a business and design t-shirts. You decide to sell your shirts for $15 each and you paid $6.50 a piece plus a $50 set-up fee and $25 for shipping. How many shirts do you have to sell to break even? Round to the nearest whole number. [U]Step 1: Calculate Your Cost Function C(s) where s is the number of t-shirts[/U] C(s) = Cost per Shirt * (s) Shirts + Set-up Fee + Shipping C(s) = $6.50s + $50 + $25 C(s) = $6.50s + 75 [U]Step 2: Calculate Your Revenue Function R(s) where s is the number of t-shirts[/U] R(s) = Price Per Shirt * (s) Shirts R(s) = $15s [U]Step 3: Calculate Break-Even Point[/U] Break Even is where Cost = Revenue. Set C(s) = R(s) $6.50s + 75 = $15s [U]Step 4: Subtract 6.5s from each side[/U] 8.50s = 75 [U]Step 5: Solve for s[/U] [URL='https://www.mathcelebrity.com/1unk.php?num=8.50s%3D75&pl=Solve']Run this through our equation calculator[/URL] to get s = 8.824. We round up to the next integer to get [B]s = 9[/B]. [B][URL='https://www.facebook.com/MathCelebrity/videos/10156751976078291/']FB Live Session[/URL][/B]

You and five friends are planning a trip. You want to keep the cost less than $85 per person.
You and five friends are planning a trip. You want to keep the cost less than $85 per person. You and five friends means you have: 5 + 1 = 6 people. If you want a cost per person less than 85, then we have a cost c such that: [B]c/6 < 85[/B]

you and michael have the sum of 19.75. if michael has 8.25 how much more do you have
you and michael have the sum of 19.75. if michael has 8.25 how much more do you have If you and Michael have 19.75, and Michael has 8.25, then you have: 19.75 - 8.25 = 11.50 Overage/Excess than Michael = Your Money - Michael's money Overage/Excess than Michael = 11.50 - 8.25 Overage/Excess than Michael = [B]3.25[/B]

You and some friends are going to the fair. Each ticket for a ride costs $0.75. If n is the number o
You and some friends are going to the fair. Each ticket for a ride costs $0.75. If n is the number of tickets purchased, write an expression that gives the total cost of buying n tickets. We know cost = Price * Quantity, so we have: Cost of buying n tickets = [B]0.75n[/B]

You and two friends share 7 cookies equally. How many cookies do you each get?
You and two friends share 7 cookies equally. How many cookies do you each get? 7/3 = 2 with a remainder of 1/3 So everybody gets 2 whole cookies, and they split the last cookie into 1/3. [B]2 & 1/3[/B]

You and your friend are playing a number-guessing game. You ask your friend to think of a positive n
You and your friend are playing a number-guessing game. You ask your friend to think of a positive number, square the number, multiply the result by 2, and then add three. If your friend's final answer is 53, what was the original number chosen? Let n be our original number. Square the number means we raise n to the power of 2: n^2 Multiply the result by 2: 2n^2 And then add three: 2n^2 + 3 If the friend's final answer is 53, this means we set 2n^2 + 3 equal to 53: 2n^2 + 3 = 53 To solve for n, we subtract 3 from each side, to isolate the n term: 2n^2 + 3 - 3 = 53 - 3 Cancel the 3's on the left side, and we get: 2n^2 = 50 Divide each side of the equation by 2: 2n^2/2 = 50/2 Cancel the 2's, we get: n^2 = 25 Take the square root of 25 n = +-sqrt(25) n = +-5 We are told the number is positive, so we discard the negative square root and get: n = [B]5[/B]

You and your friend are saving for a vacation. You start with the same amount and save for the same
You and your friend are saving for a vacation. You start with the same amount and save for the same number of weeks. You save 75 per week, and your friend saves 50 per week. When vacation time comes, you have 950, and your friend has 800. How much did you start with, and for how many weeks did you save? [U]Let w be the number of weeks. Set up two equations where s is the starting amount:[/U] (1) s + 75w =950 (2) s + 50w = 800 [U]Rearrange (1) into (3) to solve for s by subtracting 75w[/U] (3) s = 950 - 75w [U]Rearrange (2) into (4) to solve for s by subtracting 50w[/U] (4) s = 800 - 50w [U]Set (3) and (4) equal to each other so solve for w[/U] 950 - 75w = 800 - 50w [U]Add 75w to each side, and subtract 950 from each side:[/U] 25w = 150 [U]Divide each side by w[/U] [B]w = 6[/B] Now plug w = 6 into (3) s = 950 - 75(6) s = 950 - 450 [B]s = 500[/B]

You are baking muffins for your class. There are 17 total students in your class and you have baked
You are baking muffins for your class. There are 17 total students in your class and you have baked 5 muffins. Write and solve an equation to find the additional number x of muffins you need to bake in order to have 2 muffins for each student. Write your equation so that the units on each side of the equation are muffins per student. 2 muffins per student = 17*2 = 34 muffins. We have an equation with a given 5 muffins, how much do we need (x) to get to 34 muffins (2 per student): x + 5 = 34 To solve for x, we type this equation into our search engine and we get: x = [B]29[/B]

You are baking muffins for your class. There are 17 total students in your class and you have baked
You are baking muffins for your class. There are 17 total students in your class and you have baked 5 muffins. Write and solve an equation to find the additional number x of muffins you need to bake in order to have 2 muffins for each student. Write your equation so that the units on each side of the equation are muffins per student. [U]Calculate total muffins:[/U] Total muffins = 2 muffins per student * 17 students Total muffins = 34 [U]Set up the equation using x for muffins:[/U] [B]x + 5 = 34 [/B] [U]To Solve this equation for x, we [URL='https://www.mathcelebrity.com/1unk.php?num=x%2B5%3D34&pl=Solve']type it in our search engine[/URL] and we get:[/U] x = [B]29 [/B]

You are buying boxes of cookies at a bakery. Each box of cookies costs $4. In the equation below, c
You are buying boxes of cookies at a bakery. Each box of cookies costs $4. In the equation below, c represents the number of boxes of cookies you buy, and d represents the amount the cookies will cost you (in dollars). The relationship between these two variables can be expressed by the following equation: d=4c. Identify the dependent and independent variables. [B]The variable d is dependent, and c is independent since the value of d is determined by c.[/B]

You are comparing the costs of producing shoes at two different manufacturers. Company 1 charges $5
You are comparing the costs of producing shoes at two different manufacturers. Company 1 charges $5 per pair of shoes plus a $650 flat fee. Company 2 charges $4 per pair of shoes plus a $700 flat fee. How many pairs of shoes are produced when the total costs for both companies are equal? Let s be the number of shoes. We have two equations: (1) C = 5s + 650 (2) C = 4s + 700 Set the costs equal to each other 5s + 650 = 4s + 700 Subtract 4s from each side s + 650 = 700 Subtract 650 from each side [B]s =50[/B]

You are given a choice of taking the simple interest on $100,000 invested for 5 years at a rate of 2
You are given a choice of taking the simple interest on $100,000 invested for 5 years at a rate of 2% or the interest on $100,000 invested for 5 years at an interest rate of 2% compounded daily. Which investment earns the greater amount of interest? Give the difference between the amounts of interest earned by the two investments [URL='http://www.mathcelebrity.com/simpint.php?av=&p=100000&int=2&t=5&pl=Simple+Interest']Simple interest balance after 5 years[/URL] at 2% is $110,000. [URL='http://www.mathcelebrity.com/compoundint.php?bal=100000&nval=1825&int=2&pl=Daily']Daily compounded interest for 5 years[/URL] at 2% is 365 days per year * 5 years = 1,825 days = [B]$110,516.79 Compound interest earns more by $110,516.79 - $110,000 = $516.79[/B]

You are heading to Cedar Point for the day. It costs $50 to get in to the park and each ride costs $
You are heading to Cedar Point for the day. It costs $50 to get in to the park and each ride costs $2 for a ticket. Write an expression for the total cost of going to Cedar Point where r is the number of rides. Set up the cost equation C(r): C(r) = Cost per ride * r rides + Park Fee [B]C(r) = 2r + 50[/B]

You are making identical gift bags using 24 candles and 36 bottles of lotion. What is the greatest n
You are making identical gift bags using 24 candles and 36 bottles of lotion. What is the greatest number of gift bags you can make with no items left over? We take the greatest common factor [URL='https://www.mathcelebrity.com/gcflcm.php?num1=24&num2=36&num3=&pl=GCF+and+LCM']GCF (24, 36) = 12[/URL] So we have a ratio of 24/12 = 2 candles and 36/12 = 3 bottles of lotion per bag giving us [B]12 bags[/B].

You are offered two different sales jobs. The first company offers a straight commission of 6% of th
You are offered two different sales jobs. The first company offers a straight commission of 6% of the sales. The second company offers a salary of $330 per week plus 2% of the sales. How much would you have to sell in a week in order for the straight commission offer to be at least as good? Let s be the sales and C be the weekly commission for each sales job. We have the following equations: [LIST=1] [*]C = 0.06s [*]C = 330 + 0.02s [/LIST] Set them equal to each other: 0.06s = 330 + 0.02s Subtract 0.02s from each side: 0.04s = 330 Using our [URL='http://www.mathcelebrity.com/1unk.php?num=0.04s%3D330&pl=Solve']equation solver[/URL], we get [B]s = 8,250[/B]

You are parking your car in a garage. The first hour is free but every additional hour is 2 dollars.
You are parking your car in a garage. The first hour is free but every additional hour is 2 dollars. You parked for 3.25 hours. What is the cost? [U]Calculate the number of paid hours:[/U] Paid Hours = Total Hours - 1 (since first hour is free) Paid Hours = 3.25 - 1 Paid Hours = 2.25 [U]Calculate the total cost:[/U] Total Cost = Hourly Rate * Paid Hours Total Cost = 2 * 2.25 Paid Hours = [B]$4.50[/B]

You are purchasing carpeting for an office building. The space to be carpeted is 30 feet by 50 feet.
You are purchasing carpeting for an office building. The space to be carpeted is 30 feet by 50 feet. Company A charges $2.99 per square foot plus a $200 installation charge. Company B charges $19.99 per square yard plus a $500 installation charge. What is the best deal? Did you notice the word snuck in on this problem? Company B is given in square [I][B]yards[/B][/I], not feet. Let's convert their price to square feet to match company A. [U]Company B conversion:[/U] Since we have 1 square yard = 3 feet * 3 feet = 9 square feet, we need to solve the following proportion: $19.99/square yard * 1 square yard/9 feet = $19.99 square yard / 9 feet = $2.22 / square foot. Now, let's set up the cost equations C(s) for each Company in square feet (s) [LIST] [*]Company A: C(s) = 200 + 2.99s [*]Company B: C(s) = 500 + 2.22s [/LIST] The problem asks for s = 30 feet * 50 feet = 1500 square feet. So we want to calculate C(1500) [U]Company A:[/U] C(1500) = 200 + 2.99(1500) C(1500) = 200 + 4485 C(1500) = 4685 [U]Company B:[/U] C(1500) = 500 + 2.22(1500) C(1500) = 500 + 3330 C(1500) = 3830 Since [B]Company B[/B] has the lower cost per square foot, they are the better buy.

You are researching the price of DVD players. You found an average price of $58.80. One DVD player c
You are researching the price of DVD players. You found an average price of $58.80. One DVD player costs $56 and another costs $62. Find the price of the third DVD player. We want to find n, such that n makes the average of the 3 DVD players $58.80. [URL='https://www.mathcelebrity.com/missingaverage.php?num=56%2C62&avg=58.80&pl=Calculate+Missing+Score']Using our missing average calculator[/URL], we get the price of the 3rd DVD player is $58.40.

You are selling fertilizer to female farmers in Ghana. There are 22,600,000 people in Ghana, and 60%
You are selling fertilizer to female farmers in Ghana. There are 22,600,000 people in Ghana, and 60% are of working age. Within that working-age group, women account for 53%. Of the working-age females, 42% of them are employed in farming. What is the total number of potential customers for your fertilizer? [U]Our sample population is found by this product:[/U] Female farmers of working age in Ghana = Total people in Ghana *[I] Working Age[/I] * Women of working Age * Farmers Since 60% = 0.6, 53% = 0.53, and 42% = 0.42, we have Female farmers of working age in Ghana = 22,600,000 * 0.6 * 0.53 * 0.42 Female farmers of working age in Ghana = [B]3,018,456[/B]

You are using a spinner with the numbers 1-10 on it. Find the probability that the pointer will sto
You are using a spinner with the numbers 1-10 on it. Find the probability that the pointer will stop on an odd number or a number less than 4. We want P(odd number) or P(n<4). [LIST] [*]Odd numbers are {1, 3, 5, 7, 9} [*]n < 4 is {1, 2, 3} [/LIST] We want the union of these 2 sets: {1, 2, 3, 5, 7, 9} We have 6 possible pointers in a set of 10. [B]6/10 = 3/5 = 0.6 or 60%[/B]

You borrowed $25 from your friend. You paid him back in full after 6 months. He charged $2 for inter
You borrowed $25 from your friend. You paid him back in full after 6 months. He charged $2 for interest. What was the annual simple interest rate that he charged you? Use the formula: I = Prt. We have I = 2, P = 25, t = 0.5 2 = 25(r)0.5 Divide each side by 0.5 4 = 25r Divide each side by 25 r = 4/25 [B]r = 0.16[/B] As a percentage, this is [B]16%[/B]

You bought a box of pens for $5 and some notepads for $3 each. You spent a total of $26.
You bought a box of pens for $5 and some notepads for $3 each. You spent a total of $26. Let p be the number of pens and n be the number of notepads. Our equation is: 5[B]p + 3n = 26[/B]

You bought a magazine for $3 and some candy bars for $2 each. You spent a total of $19. How many can
You bought a magazine for $3 and some candy bars for $2 each. You spent a total of $19. How many candy bars did you buy? Calculate the candy bar spend: Candy bar spend = Total spend - magazine spend Candy bar spend = 19 - 3 Candy bar spend = 16 Calculate number of candy bars: Number of candy bars = Candy Bar Spend / Cost per candy bar Number of candy bars = 16/2 Number of candy bars = [B]$8[/B]

You bought a magazine for $5 and four erasers. You spent a total of $25. How much did each eraser co
You bought a magazine for $5 and four erasers. You spent a total of $25. How much did each eraser cost? Subtract the cost of the magazine from what you spent: $25 - $5 = $20. If you spent $20 on 4 erasers, we divide 20/4 = [B]$5 per eraser[/B]

You bought a magazine for $5 and four erasers. You spent a total of $25. How much did each eraser co
You bought a magazine for $5 and four erasers. You spent a total of $25. How much did each eraser cost? [U]Calculate the cost of all erasers:[/U] Cost of all erasers = Total Spend - Magazine Cost Cost of all erasers = 25 - 5 Cost of all erasers = 20 [U]Calculate the cost per eraser:[/U] Cost per eraser = Cost of all erasers / Total erasers Cost per eraser = 20 / 4 Cost per eraser = Cost of all erasers / Total erasers Cost per eraser = [B]$5[/B]

You bought a magazine for $5 and four erasers. You spent a total of $25.how much did each eraser cos
You bought a magazine for $5 and four erasers. You spent a total of $25.how much did each eraser cost? [U]Calculate the cost of the erasers:[/U] Cost of erasers = Total Spend - Magazine Cost of erasers = 25 - 5 Cost of erasers = 20 [U]Calculate the cost per eraser:[/U] Cost per eraser = Cost of erasers / Number of erasers Cost per eraser = 20/4 Cost per eraser = [B]$5[/B]

You but a sandwich for $2.25, an a apple for $0.55, and a bottle of juice for $1.85 what is the tota
You but a sandwich for $2.25, an a apple for $0.55, and a bottle of juice for $1.85 what is the total cost [U]Calculate total cost:[/U] Total cost = Sandwich cost + apple cost + juice cost Total cost = 2.25 + 0.55 + 1.85 Total cost = [B]$4.65[/B]

You buy a book that is 540 pages long. You can read about 30 pages per hour. How long does it take y
You buy a book that is 540 pages long. You can read about 30 pages per hour. How long does it take you to read the book? 540 pages / 30 pages per hour = [B]18 hours[/B]

You buy a container of cat litter for $12.25 and a bag of cat food for x dollars. The total purchase
You buy a container of cat litter for $12.25 and a bag of cat food for x dollars. The total purchase is $19.08, which includes 6% sales tax. Write and solve an equation to find the cost of the cat food. Our purchase includes at cat litter and cat food. Adding those together, we're given: 12.25 + x = 19.08 To solve for x, [URL='https://www.mathcelebrity.com/1unk.php?num=12.25%2Bx%3D19.08&pl=Solve']we type this equation into our search engine[/URL], and we get: x = 6.83 Since the cat food includes sales tax, we need to remove the sales tax of 6% to get the original purchase price. Original purchase price = After tax price / (1 + tax rate) Original purchase price = 6.83/1.06 Original purchase price = [B]$6.44[/B]

You buy a house for $130,000. It appreciates 6% per year. How much is it worth in 10 years
You buy a house for $130,000. It appreciates 6% per year. How much is it worth in 10 years The accumulated value in n years for the house is: A(n) = 130,000(1.06)^n We want A(10) A(10) = 130,000(1.06)^10 A(10) =130,000*1.79084769654 A(10) = [B]232,810.20[/B]

You can afford monthly deposits of $270 into an account that pays 3.0% compounded monthly. How long
You can afford monthly deposits of $270 into an account that pays 3.0% compounded monthly. How long will it be until you have $11,100 to buy a boat. Round to the next higher month. [U]Set up our accumulation expression:[/U] 270(1.03)^n = 11100 1.03^n = 41.1111111 [U]Take the natural log of both sides[/U] n * Ln(1.03) = 41.1111111 n = 3.71627843/0.0295588 n = 125.72 so round up to [B]126[/B]

You can get 2 different moving companies to help you move. The first one charges $150 up front then
You can get 2 different moving companies to help you move. The first one charges $150 up front then $38 an hour. The second one charges $230 then $30 an hour, at what exact time will Both companies cost the same [U]Company 1: We set up the cost equation C(h) where h is the number of hours[/U] C(h) = Hourly Rate * h + up front charge C(h) = 38h + 150 [U]Company 2: We set up the cost equation C(h) where h is the number of hours[/U] C(h) = Hourly Rate * h + up front charge C(h) = 30h + 230 The question asks for h when both cost equations C(h) are equal. So we set both C(h) equations equal to other: 38h + 150 = 30h + 230 To solve for h, we [URL='https://www.mathcelebrity.com/1unk.php?num=38h%2B150%3D30h%2B230&pl=Solve']type this equation into our search engine [/URL]and we get: h = [B]10[/B]

You can pay a daily entrance fee of $3 or purchase a membership for the 12 week summer season for $8
You can pay a daily entrance fee of $3 or purchase a membership for the 12 week summer season for $82 and pay only $1 per day to swim. How many days would you have to swim to make the membership worthwhile? Set up cost equations: Daily entrance fee: 3d where d is the number of days of membership Membership fee 82 + 1d Set them equal to each other 82 + 1d = 3d Subtract d from each side: 2d = 82 Divide each side by 2 [B]d = 41[/B]

You can spend at most $35. If you buy 5 tickets, how much can you spend on each ticket
You can spend at most $35. If you buy 5 tickets, how much can you spend on each ticket We're given the number of tickets as 5. We know cost = price * quantity Let p = price The phrase [B]at most[/B] means less than or equal to, so we have: 5p <= 35 [URL='https://www.mathcelebrity.com/interval-notation-calculator.php?num=5p%3C%3D35&pl=Show+Interval+Notation']Plugging this inequality into our search engine[/URL], we have: [B]p <= 7[/B]

You choose an alpha level of .01 and then analyze your data.
(a) What is the probability that
You choose an alpha level of .01 and then analyze your data. (a) What is the probability that you will make a Type I error given that the null hypothesis is true? (b) What is the probability that you will make a Type I error given that the null hypothesis is false. [B](a) 0.01. Instead, α is the probability of a Type I error given that the null hypothesis is true. If the null hypothesis is false, then it is impossible to make a Type I error.[/B] [B](b) Impossible Instead, α is the probability of a Type I error given that the null hypothesis is true. If the null hypothesis is false, then it is impossible to make a Type I error.[/B]

You collect a total of 75 in donations from three people. The three donations are in the ratio 4:4:7
You collect a total of 75 in donations from three people. The three donations are in the ratio 4:4:7. How much is each of the smaller donations? 4 + 4 + 7 = 15 Each person's donation ratio is: [LIST=1] [*]Donation 1 is 4/15 of 75 [*]Donation 2 is 4/15 of 75 [*]Donation 3 is 7/15 of 75 [/LIST] 4/15(75) = 5 * 4 = 20 7/15(75) = 5 * 7 = 35 Each person's donation amount is: [LIST=1] [*][B]$20[/B] [*][B]$20[/B] [*][B]$35[/B] [/LIST] Check out work: 20 + 20 + 35 = 75!

You collect stamps. You give steven 21 stamps. At the end youbhave 3. How many stamps did you start
You collect stamps. You give steven 21 stamps. At the end youbhave 3. How many stamps did you start with? You start with s stamps. s You give Steven 21. Giving means you subtract from your total: s - 21 You have 3 left s - 21 = 3 To solve this equation for s, we t[URL='https://www.mathcelebrity.com/1unk.php?num=s-21%3D3&pl=Solve']ype it in our math engine[/URL] and we get: s = [B]24[/B]

You conduct 50,000 tests, 1500 people test positive, what's the positivity rate?
You conduct 50,000 tests, 1500 people test positive, what's the positivity rate? [U]Our Positivity Rate formula is below:[/U] Positivity Rate = 100% * positive tests / Total tests [U]Plugging in our numbers from the problem, we get:[/U] Positivity Rate = 100% * 1500/50000 Positivity Rate = 100% * 0.03 Positivity Rate = [B]3%[/B]

You deposit $150 into an account that yields 2% interest compounded quarterly. How much money will
You deposit $150 into an account that yields 2% interest compounded quarterly. How much money will you have after 5 years? 2% per year compounded quarterly equals 2/4 = 0.5% per quarter. 5 years * 4 quarter per year = 20 quarters of compounding. Using our [URL='https://www.mathcelebrity.com/compoundint.php?bal=150&nval=20&int=2&pl=Quarterly']balance calculator[/URL], we get [B]$165.73[/B] in the account after 20 years.

You deposit $1600 in a bank account. Find the balance after 3 years if the account pays 1.75% annual
You deposit $1600 in a bank account. Find the balance after 3 years if the account pays 1.75% annual interest compounded monthly Using our [URL='https://www.mathcelebrity.com/compoundint.php?bal=1600&nval=36&int=1.75&pl=Monthly']compound interest calculator with 3 years = 36 months[/URL], we get: [B]1,686.18[/B]

You deposit $2000 in an account that earns simple interest at an annual rate of 4%. How long must yo
You deposit $2000 in an account that earns simple interest at an annual rate of 4%. How long must you leave the money in the account to earn $500 in interest? The simple interest formula for the accumulated balance is: Prt = I We are given P = 2,000, r = 0.04, and I = 500. 2000(0.04)t = 500 80t = 500 Divide each side by 80 t = [B]6.25 years [MEDIA=youtube]Myz0FZgwZpk[/MEDIA][/B]

you deposit $2000 in an account that pays 3% annual interest. Find the balance after 10 years if the
you deposit $2000 in an account that pays 3% annual interest. Find the balance after 10 years if the interest is compounded quarterly. Please give your answer to 2 decimal places. Using our [URL='https://www.mathcelebrity.com/compoundint.php?bal=2000&nval=40&int=3&pl=Quarterly']compound interest calculator, with 10 * 4 = 40 quarters[/URL], we have: [B]$2,696.70[/B]

You deposit $750 in an account that earns 5% interest compounded quarterly. Show and solve a functio
You deposit $750 in an account that earns 5% interest compounded quarterly. Show and solve a function that represents the balance after 4 years. The Accumulated Value (A) of a Balance B, with an interest rate per compounding period (i) for n periods is: A = B(1 + i)^n [U]Givens[/U] [LIST] [*]4 years of quarters = 4 * 4 = 16 quarters. So this is t. [*]Interest per quarter = 5/4 = 1.25% [*]Initial Balance (B) = 750. [/LIST] Using our [URL='https://www.mathcelebrity.com/compoundint.php?bal=750&nval=16&int=5&pl=Quarterly']compound balance interest calculator[/URL], we get the accumulated value A: [B]$914.92[/B]

You deposit $8500 in an account that pays 1.78% annual interest. Find the balance after 10 years whe
You deposit $8500 in an account that pays 1.78% annual interest. Find the balance after 10 years when the interest is compounded monthly. 10 years * 12 months per year = 120 months. Using our [URL='https://www.mathcelebrity.com/compoundint.php?bal=8500&nval=120&int=1.781&pl=Monthly']compound interest calculator[/URL], we get a balance of: [B]$10,155.69[/B]

you draw a card at random from a deck that contains 3 black cards and 7 red cards what is the probab
you draw a card at random from a deck that contains 3 black cards and 7 red cards what is the probability of you drawing a black card Total cards = 3 black + 7 red Total cards = 10 P(Black) = Black cards / Total Cards P(Black) = [B]3/10 or 0.3[/B]

You earn $7 for every ? hour you cut the grass. How much money do you make for 3 hours?
You earn $7 for every ? hour you cut the grass. How much money do you make for 3 hours? 3 hours / 1/3 hour = 9 (1/3 blocks) So we have: 7 * 9 = [B]63[/B]

You earned $141 last week babysitting and cleaning. You earned $5 per hour babysitting and $7 per ho
You earned $141 last week babysitting and cleaning. You earned $5 per hour babysitting and $7 per hour cleaning. You worked 9 more hours babysitting than cleaning. How many hours did you work last week? Let b be the hours of babysitting and c be the hours of cleaning. We're given two equations: [LIST=1] [*]b = c + 9 [*]5b + 7c = 141 [/LIST] Substitute equation (1) into (2): 5(c + 9) + 7c = 141 Multiply through: 5c + 45 + 7c = 141 Combine like terms: 12c + 45 = 141 [URL='https://www.mathcelebrity.com/1unk.php?num=12c%2B45%3D141&pl=Solve']Typing this equation into our search engine[/URL], we get: c = 8 Now substitute this value of c back into Equation (1): b = 8 + 9 b = 17 The total hours worked (t) is: t = b + c t = 17 + 8 t = [B]25[/B]

You fly x miles and then drive y miles. How many miles did you travel?
You fly x miles and then drive y miles. How many miles did you travel? Total miles traveled = fly miles + drive miles Total miles traveled = [B]x + y[/B]

You get paid $8 an hour. You make $35 in tips. You make $167.00 in a week. How many hours did you wo
You get paid $8 an hour. You make $35 in tips. You make $167.00 in a week. How many hours did you work? To figure out the hours worked, we first need the amount of hourly wages made: Hourly Wages = Total Wages - Tips Hourly Wages = $167 - $35 Hourly Wages = $132 Calculate Hours Worked Hours Worked = Hourly Wages / Hourly Rate Hourly Worked = $132 / $8 Hourly Worked = [B]16.5[/B]

You go to your favorite restaurant. The bill for the food is $74.26. The tax on the bill will be 9%.
You go to your favorite restaurant. The bill for the food is $74.26. The tax on the bill will be 9%. You are planning on giving a tip on that total amount (bill and tax together) of 20%. What is your final bill, all taxes and tips included? [U]Calculate the after tax amount:[/U] After tax amount = Bill * (1 + Tax Rate) Since 9% = 0.09, we have: After tax amount = 74.26 * (1 + 0.09) After tax amount = 74.26 * 1.09 After tax amount = 80.94 [U]Calculate the Tip amount:[/U] Tip amount = After tax amount * tip percent Since 20% = 0.2, we have: Tip amount = 80.94 * 0.20 Tip amount = 16.19 [U]Calculate the final bill:[/U] Final Bill = After Tax Amount + Tip Amount Final Bill = 80.94 + 16.19 Final Bill = [B]97.13[/B]

You had $21 to spend on three raffle tickets. After buying them you have $12. How much does each raf
You had $21 to spend on three raffle tickets. After buying them you have $12. How much does each raffle ticket cost? If we have $12 left, we spent: Total Raffle Spend = Total Spend - LeftOver spend Total Raffle Spend = $21 - $12 Total Raffle Spend = $9 Calculate the cost per raffle ticket: Cost per raffle ticket = Total Raffle Spend / Cost Per raffle ticket Cost per raffle ticket = $9/3 Cost per raffle ticket = [B]$3[/B]

You had $22 to spend on 8 notebooks after buying them you had $6
If you have $6 left over, then 8 notebooks cost $22 - $6 = $16. 8 notebooks = $16 Divide each side of the equation by 8 Each notebook is $2

You have $10.00 to spend on tacos. Each taco costs $0.50. Write and solve an inequality that explain
You have $10.00 to spend on tacos. Each taco costs $0.50. Write and solve an inequality that explains how many tacos you can buy. Let's start with t as the number of tacos. We know that cost = price * quantity, so we have the following inequality for our taco spend: [B]0.5t <= 10 [/B] Divide each side of the inequality by 0.5 to isolate t: 0.5t/0.5 <= 10/0.5 Cancel the 0.5 on the left side and we get: t <= [B]20 [MEDIA=youtube]yy51EsGi1nM[/MEDIA][/B]

You have $110 saved in your bank account. You want to save $15 every week. Let x be the amount of we
You have $110 saved in your bank account. You want to save $15 every week. Let x be the amount of weeks and y be the total amount saved. Savings mean we add to the bank balance, so we have: [B]y = 15x + 110[/B]

You have $140 in a savings account and save $10 per week. Your friend has $95 in a savings account a
You have $140 in a savings account and save $10 per week. Your friend has $95 in a savings account and saves $19 per week. How many weeks will it take for you and your friend to have the same balance? [U]Set up the savings account S(w) for you where w is the number of weeks[/U] S(w) = 140 + 10w [U]Set up the savings account S(w) for your friend where w is the number of weeks[/U] S(w) = 95 + 19w The problem asks for the number of weeks (w) when the balances are the same. So set both equations equal to each other: 140 + 10w = 95 + 19w To solve this equation for w, [URL='https://www.mathcelebrity.com/1unk.php?num=140%2B10w%3D95%2B19w&pl=Solve']we type it in the search engine[/URL] and get: w = [B]5[/B]

You have $20 to spend on a taxi fare. The ride costs $5 plus $2.50 per kilometer.
You have $20 to spend on a taxi fare. The ride costs $5 plus $2.50 per kilometer. Let k be the number of kilometers. Total Cost = Cost per kilometer * number of kilometers + Fixed Cost With k for kilometers, 2.5 as cost per kilometer, and 5 as fixed cost, and 20 on total cost, we have: 2.5k + 5 = 20 To solve this equation for k, we [URL='https://www.mathcelebrity.com/1unk.php?num=2.5k%2B5%3D20&pl=Solve']type it in our math engine [/URL]and we get k = [B]6[/B]

You have $20 to spend on taxi fare. The ride costs $5 plus $2.50 per kilometer. Write the inequality
You have $20 to spend on taxi fare. The ride costs $5 plus $2.50 per kilometer. Write the inequality. Let k be the number of kilometers. We want our total to be $20 [I]or less. [/I]We have the following inequality: [B]2.50k + 5 <= 20[/B]

You have $250,000 in an IRA (Individual Retirement Account) at the time you retire. You have the op
You have $250,000 in an IRA (Individual Retirement Account) at the time you retire. You have the option of investing this money in two funds: Fund A pays 5.4% annually and Fund B pays 7.9% annually. How should you divide your money between fund Fund A and Fund B to produce an annual interest income of $14,750? You should invest $______in Fund A and $___________in Fund B. Equation is x(.079) + (250,000 - x).054 = 14,750 .025x + 13,500 = 14,750 .025x = 1,250 [B]x = 50,000 for Fund A[/B] So at 5.4%, we have 250,000 - 50,000 = [B]200,000[/B] for the other fund B.

You have $6.50 to make copies. It cost $0.45. Write and solve an equality that represents the number
You have $6.50 to make copies. It cost $0.45. Write and solve an equality that represents the number of copies Hoow many exact copies can you make? Let the number of copies be c. We have: 0.45c = 6.50 [URL='https://www.mathcelebrity.com/1unk.php?num=0.45c%3D6.50&pl=Solve']Type this equation into our search engine[/URL] and we get: c = 14.444 We round down and say we can make 14 copies. [B]c = 14[/B] Now, if the problem asks you for an [I]inequality[/I], we want to see how many copies we can make without exceeding our $6.50 spend. So it's less than or equal to: [B]c <= 14[/B]

You have $80. Jeans cost $29 and shirts cost $12. Mom told you to buy one pair of jeans and use the
You have $80. Jeans cost $29 and shirts cost $12. Mom told you to buy one pair of jeans and use the rest of the money to buy shirts. Find the inequality. Let j be the number of jeans. Let s be the number of shirts. We are given: [LIST] [*]Mom told you to buy one pair of jeans. So we have $80 to start with - $29 for 1 pair of jeans = $51 left over [/LIST] Now, since shirts cost $12 each, and our total number of shirts we can buy is s, our inequality is [B]12s <= 51[/B]. We want to find the s that makes this inequality true. [URL='https://www.mathcelebrity.com/interval-notation-calculator.php?num=12s%3C%3D51&pl=Show+Interval+Notation']Run this statement through our calculator[/URL], and we get s <= 4.25. But, we need s to be an integer, so we have s <= 4.

You have 240 grams of a radioactive kind of tellurium. How much will be left after 210 minutes if it
You have 240 grams of a radioactive kind of tellurium. How much will be left after 210 minutes if its half-life is 70 minutes? if the half life is 70 minutes, then we have 210/70 = 3 half life cycles. So the first half-life is 240 * 1/2 = 120 The second half life is 120 * 1/2 = 60 The third half life is 60 * 1/2 = [B]30 grams[/B]

You have 30 DONUTS. 1/6 of them are Boston Cream. 2/5 of them are Maple. 3/10 of them are Chocolate
You have 30 DONUTS. 1/6 of them are Boston Cream. 2/5 of them are Maple. 3/10 of them are Chocolate Dip and 1/3 are Sprinkled. IS THIS EVEN POSSIBLE?? How many donuts OVER or UNDER am I? (Show your work and use EQUIVALENTS.) We use 30 as our common denominator. Let's get [I]equivalent fraction[/I]s for each donut type with a denominator of 30: [LIST] [*][URL='https://www.mathcelebrity.com/equivalent-fractions.php?num=1%2F6&pl=Equivalent+Fractions']1/6[/URL] = 5/30 [*][URL='https://www.mathcelebrity.com/equivalent-fractions.php?num=2%2F5&pl=Equivalent+Fractions']2/5 [/URL]= 12/30 [*][URL='https://www.mathcelebrity.com/equivalent-fractions.php?num=3%2F10&pl=Equivalent+Fractions']3/10[/URL] = 9/30 [*][URL='https://www.mathcelebrity.com/equivalent-fractions.php?num=1%2F3&pl=Equivalent+Fractions']1/3[/URL] = 10/30 [/LIST] Add up our numerators of the common denominator of 30: 5 + 12 + 9 + 10 = 36 So our fraction is 36/30. This makes our scenario [B]impossible[/B]. Fractions of the donut should add up to 1. Which would mean our numerators need to sum to 1 or less. Since 36 > 30, this scenario is [B]impossible.[/B]

You have 36 marbles that are red, white and blue. If 12 of the marbles are red, and 13 of the marble
You have 36 marbles that are red, white and blue. If 12 of the marbles are red, and 13 of the marbles are blue, what fraction of marbles are white? [U]Calculate the number of white marbles:[/U] Number of white marbles = Total marbles - Red marbles - Blue marbles Number of white marbles = 36 - 12 - 13 Number of white marbles = 11 [U]Calculate the fraction of white marbles:[/U] Fraction of white marbles =Number of white marbles / Total marbles Fraction of white marbles = [B]11/36[/B]

You have a total of 42 math and science problems for homework. You have 10 more math problems than s
You have a total of 42 math and science problems for homework. You have 10 more math problems than science problems. How many problems do you have in each subject? Let m be the math problems and s be the science problems. We have two equations: (1) m + s = 42 (2) m = s + 10 Substitute (2) into (1) (s + 10) + s = 42 Combine like terms 2s + 10 = 42 Subtract 10 from each side 2s = 32 Divide each side by 2 [B]s = 16[/B] So that means m = 16 + 10 --> [B]m = 26 (m, s) = (26, 16)[/B]

You have read 100 pages of a 250 book. You have 3 days left to finish the book. How many pages will
You have read 100 pages of a 250 book. You have 3 days left to finish the book. How many pages will you need to reach each day in order to be able to finish the book? Calculate remaining pages to read: Remaining Pages = 250 - 100 = 150 Now, calculate pages per day Pages per day = Remaining Pages/Days left Pages per day = 150/3 Pages per day = [B]50[/B]

You have read a 247 page book for a class and decide to read 18 pages a night. How many pages are le
You have read a 247 page book for a class and decide to read 18 pages a night. How many pages are left in the book if you have been reading for n nights? Set up the remaining pages read function R(n). We have: [B]R(n) = 247 - 18n[/B]

You have saved $50 over the last two weeks and decide to treat yourself by buying some new clothes.
You have saved $50 over the last two weeks and decide to treat yourself by buying some new clothes. You go to the store and find two shirts and three pairs of jeans you like. The two shirts are buy-one-get-one half off, at $22.35 each. The three pairs of jeans are buy-two-get-one-free, at $23.70. Tax Rate for Harmonized Sales Tax is 13% a. What would be the total for the two shirts (don’t forget to include taxes)? b. What would be the total for the three pairs of jeans (don’t forget to include taxes)? c. Which would you buy and why? a. Half of 22.35 is 11.18 So two shirts cost: 22.35 + 11.18 = 33.53 Cost with Tax of 13% is: 33.53 * 1.13 = [B]37.89 [/B] b. Three pairs of jeans are calculated by cost of 1 pair times 2 jeans and you get the third one free: 23.70 * 2 = 47.40 Cost with Tax of 13% is: 47.40 * 1.13 = [B]53.56 [/B] c. Calculate unit cost, which is cost per item Unit cost of Shirts = 37.89 / 2 = [B]18.95[/B] Unit cost of Jeans = 53.56 / 3 = [B]17.85 Buy the jeans since they have a lower unit cost[/B]

You have to pay 29 a month until you reach 850 how many months will that take
You have to pay 29 a month until you reach 850 how many months will that take. Let m be the number of months. We set up the inequality: 29m > = 850 <-- We want to know when we meet or exceed 850, so we use greater than or equal to [URL='https://www.mathcelebrity.com/interval-notation-calculator.php?num=29m%3E%3D850&pl=Show+Interval+Notation']Type this inequality into our search engine[/URL], and we get: m >= 29.31 We round up to the next integer month, to get [B]m = 30[/B].

You invest $1,300 in an account that has an annual interest rate of 5%, compounded annually. How muc
You invest $1,300 in an account that has an annual interest rate of 5%, compounded annually. How much money will be in the account after 10 years? Using our [URL='http://www.mathcelebrity.com/compoundint.php?bal=1300&nval=10&int=5&pl=Annually']compound interest balance calculator[/URL], we get: [B]$2,117.56[/B]

You like to shovel snow in winter. You made them pay 7 dollars for every driveway you shoveled and e
You like to shovel snow in winter. You made them pay 7 dollars for every driveway you shoveled and earned 42 dollars. How many driveways did you shovel? Driveways shoveled = Total Money / Dollars per Driveway Driveways shoveled = 42/7 [URL='https://www.mathcelebrity.com/fraction.php?frac1=42%2F7&frac2=3%2F8&pl=Simplify']Driveways shoveled[/URL] = [B]6[/B]

you must be 65 or older to join inequality
you must be 65 or older to join inequality Let a be the age. 65 or older means greater than or equal to 65: [B]a >=65[/B]

You need to hire a catering company to serve meals to guests at a wedding reception. Company A charg
You need to hire a catering company to serve meals to guests at a wedding reception. Company A charges $500 plus $20 per guest. Company B charges $800 plus $16 per guest. For how many guests are the total costs the same at both companies? Set up the Cost equations for both companies where g is the number of guests: [LIST] [*]C(a) = 20g + 500 [*]C(b) = 16g + 800 [/LIST] Set each equation equal to each other and use our [URL='http://www.mathcelebrity.com/1unk.php?num=20g%2B500%3D16g%2B800&pl=Solve']equation solver[/URL] to get: [B]g = 75[/B]

You open a hat stand in the mall with an initial start-up cost of $1500 plus 50 cents for every hat
You open a hat stand in the mall with an initial start-up cost of $1500 plus 50 cents for every hat you stock your booth with. a) What is your cost function? Set up the cost function C(h) where h is the number of hats you stock: C(h) = Cost per hat * h hats + Start Up Cost [B]C(h) = 0.5h + 1500[/B]

You open up a savings account. Your initial deposit is $300. You plan to add in $50 per month to sav
You open up a savings account. Your initial deposit is $300. You plan to add in $50 per month to save up for college. Write an equation to represent the situation. Let m be the number of months. We have a Savings account function S(m): S(m) = Monthly deposit * number of months + Initial Deposit [B]S(m) = 50m + 300[/B]

You owe $25 to a friend. You have paid back $12 but asked for another $8. How much do you owe?
You owe $25 to a friend. You have paid back $12 but asked for another $8. How much do you owe? You pay back 12, so your balance is: -25 + 12 = -13 or you owe 13 You ask for (Borrow) another $8 -13 - 8 = [B]-21 or you owe 21[/B]

You pay 510.00 to rent a storage unit for 3 months the total cost includes an initial deposit plus a
You pay 510.00 to rent a storage unit for 3 months the total cost includes an initial deposit plus a monthly fee of 160.00. Write and equation that represents your total cost Y in dollars after X months. Set up the cost function Y where x is the number of months you rent [B]Y = 160x + 510[/B]

You practice the piano for 30 minutes each day. Write and solve an equation to find the total time t
You practice the piano for 30 minutes each day. Write and solve an equation to find the total time t you spend practicing the piano in a week. Since there is 7 days in a week, we have: t = 30 * 7 [B]t = 210[/B]

You prepare 18 scoops of dog food for 6 dogs, and prepare 24 scoops of dog food for 8 dogs. What is
You prepare 18 scoops of dog food for 6 dogs, and prepare 24 scoops of dog food for 8 dogs. What is the constant of proportionality for the amount of dog food to the number of dogs? How many scoops of dog food should you prepare for 9 dogs? 18/6 = 24/8 = 3 as the constant of proportionality for the amount of dog food to the number of dogs. What this means is for every dog, we give them 3 scoops of food. So for 9 dogs, we give 9 dogs * 3 scoops of food per dog = 27 scoops

You purchase a car for $23,000. The car depreciates at a rate of 15% per year. Determine the value
You purchase a car for $23,000. The car depreciates at a rate of 15% per year. Determine the value of the car after 7 years. Round your answer to the nearest cent. Set up the Depreciation equation: D(t) = 23,000/(1.15)^t We want D(7) D(7) = 23,000/(1.15)^7 D(7) = 23,000/2.66002 D(7) = [B]8,646.55[/B]

You purchase a new car for $35,000. The value of the car depreciates at a rate of 8.5% per year. If
You purchase a new car for $35,000. The value of the car depreciates at a rate of 8.5% per year. If the rate of decrease continues, what is the value of your car in 5 years? Set up the depreciation function D(t), where t is the time in years from purchase. We have: D(t) = 35,000(1 - 0.085)^t Simplified, a decrease of 8.5% means it retains 91.5% of it's value each year, so we have: D(t) = 35,000(0.915)^t The problem asks for D(5) D(5) = 35,000(0.915)^5 D(5) = 35,000(0.64136531607) D(5) = $[B]22,447.79[/B]

You put $5500 in a bond fund which has an annual yield of 4.8%. How much interest will be earned in
You put $5500 in a bond fund which has an annual yield of 4.8%. How much interest will be earned in 23 years? Build the accumulation of principal. We multiply 5,500 times 1.048 raised to the 23rd power. Future Value = 5,500 (1.048)^23 Future Value =5,500(2.93974392046) Future Value = 16,168.59 The question asks for interest earned, so we find this below: Interest Earned = Future Value - Principal Interest Earned = 16,168.59 - 5,500 Interest Earned = [B]10,668.59[/B]

You receive 9 text messages in 12 minutes. What is the rate of text messages per hour?
You receive 9 text messages in 12 minutes. What is the rate of text messages per hour? Set up a proportion of text messages to minutes. Remember, there are 60 minutes in an hour, so we have: 9/12 = t/60 where t is the number of text messages in 60 minutes (1 hour) [URL='https://www.mathcelebrity.com/prop.php?num1=9&num2=t&den1=12&den2=60&propsign=%3D&pl=Calculate+missing+proportion+value']Typing this into the search engine[/URL], we get [B]t = 45[/B].

You rent skates for $5 and pay $1 an hour for skating per person. Write an equation.
You rent skates for $5 and pay $1 an hour for skating per person. Write an equation. Let the number of hours be h. Our cost function C(h) is: C(h) = Cost per hour * hourly rate + rental fee Plugging in our numbers, we get: [B]C(h) = h + 5[/B]

You roll a red die and a green die. What is the size of the sample space of all possible outcomes of
You roll a red die and a green die. What is the size of the sample space of all possible outcomes of rolling these two dice, given that the red die shows an even number and the green die shows an odd number greater than 1? [LIST] [*]Red Die Sample Space {2, 4, 6} [*]Green Die Sample Space {3, 5} [*]Total Sample Space {(2, 3), (2, 5), (4, 3), (4, 5), (6, 3), (6, 5)} [*]The sie of this is 6 elements. [/LIST]

You roll a standard, fair, 5-sided die and see what number you get. Find the sample space of this ex
You roll a standard, fair, 5-sided die and see what number you get. Find the sample space of this experiment. Write your answer using { } symbols, and write your values in order with a comma but no spaces between Sample Space: [B]{1,2,3,4,5}[/B]

You roll two six-sided dice. What is the probability that the sum is less than 13?
You roll two six-sided dice. What is the probability that the sum is less than 13? The probability is [B]1, or 100%[/B], since the maximum sum of two six-sided dice is 12.

you save $35 a week for a year. How much do you have at the end of the year?
you save $35 a week for a year. How much do you have at the end of the year? We know that 1 year = 52 weeks $35 per week * 52 weeks = [B]$1,820 saved for the year[/B]

You spend $91 shopping for new clothes. You spend $24 for a pair of jeans and 35$ for a pair of shoe
You spend $91 shopping for new clothes. You spend $24 for a pair of jeans and 35$ for a pair of shoes. You also buy 4 shirts that cost d dollars. How much is each shirt? Subtract the cost of the jeans and shoes to get the cost of the shirts: Cost of shirts = Shopping Spend - Cost of Jeans - Cost of Shoes Cost of shirts = $91 - $24 - $35 Cost of shirts = $32 We're given the cost of each shirt is s, and we bought 4 shirts. Therefore, we have: 4s = 32 [URL='https://www.mathcelebrity.com/1unk.php?num=4s%3D32&pl=Solve']Type this equation into the search engine[/URL], and we get the cost of each shirt s = [B]$8[/B]

You split $1,500 between two savings accounts. Account A pays 5% annual interest and Account B pays
You split $1,500 between two savings accounts. Account A pays 5% annual interest and Account B pays 4% annual interest. After one year, you have earned a total of $69.50 in interest. How much money did you invest in each account. Explain. Let a be the amount you invest in Account A. So this means you invested 1500 - A in account B. We have the following equation: 05a + (1500 - a).04 = 69.50 Simplifying, we get: 0.05a + 1560 - 0.04a = 69.50 0.01a + 60 = 69.50 Using our [URL='http://www.mathcelebrity.com/1unk.php?num=0.01a%2B60%3D69.50&pl=Solve']equation solver[/URL], we get: [B]a = 950[/B] So this means Account B is b = 1500 - 950 = [B]550[/B]

you start at a point on the number line and move 4 units left. If you are now at 10, then what was y
you start at a point on the number line and move 4 units left. If you are now at 10, then what was your original point? Work backwards. If we're at 10, and we moved left, this means we add 4 to get back to our starting point: 10 + 4 = [B]14[/B]

You start reading on page 342 and end on 531. How many pages did you read?
You start reading on page 342 and end on 531. How many pages did you read? The pages read formula is: Pages Read = End Page - Start Page + 1 Pages Read = 531 - 342 + 1 Pages Read = [B]190[/B]

you start with 150$ in year bank account if you save $28 a year with equation would model your savin
you start with 150$ in year bank account if you save $28 a year with equation would model your savings find equation. We create a savings function S(y) where y is the number of years since the start. S(y) = Savings per year * y + initial savings [B]S(y) = 28y + 150[/B]

You started the day with 3 gallons of water. If there are 4 cups in a quart and 4 quarts in a gallon
You started the day with 3 gallons of water. If there are 4 cups in a quart and 4 quarts in a gallon, how many cups of water did you start with? [LIST] [*]1 gallon = 4 quarts [*]So 3 gallons = 4 * 3 = 12 quarts [*]12 quarts * 4 cups per quart = [B]48 cups of water[/B] [/LIST]

You started this year with $491 saved and you continue to save an additional $11 per month. Write an
You started this year with $491 saved and you continue to save an additional $11 per month. Write an algebraic expression to represent the total amount of money saved after m months. Set up a savings function for m months [B]S(m) = 491 + 11m[/B]

You throw two dice. The red dice is fair but on the blue dice the probability of a 1=15%, probabilit
You throw two dice. The red dice is fair but on the blue dice the probability of a 1=15%, probability of a 2 is 25%, and the probability of any other number is 15%. What is the probability of getting 4? Possible Rolls with a sum of 4: [LIST] [*]R = 1, B = 3 [*]R = 2, B = 2 [*]R = 3, B = 1 [/LIST] Probabilities: [LIST] [*]R = 1, B = 3 = 1/6 * 15/100 = 15/600 = 1/40 = 0.025 [*]R = 2, B = 2 = 1/6 * 25/100 = 25/600 = 1/24 = 0.041667 [*]R = 3, B = 1 = 1/6 * 15/100= 15/600 = 1/40 = 0.025 [/LIST] We add all three probabilities up to get: 0.025 + 0.025 + 0.014667 = [B]0.09166667[/B]

You use 4 gallons of water on 30 plants in your garden. At that rate, how much water will it take to
You use 4 gallons of water on 30 plants in your garden. At that rate, how much water will it take to water 45 plants? Set up a proportion of gallons to plants: 4/30 = x/45 where x is the gallons of water needed for 45 plants. Use our [URL='http://www.mathcelebrity.com/prop.php?num1=4&num2=x&den1=30&den2=45&propsign=%3D&pl=Calculate+missing+proportion+value']proportion calculator[/URL]: [B]x = 6[/B]

You want to put 520 quarters in coin wrappers. You need one wrapper for every $10 in quarters. Write
You want to put 520 quarters in coin wrappers. You need one wrapper for every $10 in quarters. Write an equation you can use to find how many wrappers w you need First, calculate the number of quarters in $10: Quarters in $10 = Value of Quarters / Cost per quarter Quarters in $10 = 10/0.25 Quarters in $10 = 40 Now find out how many wrappers we need with each wrapper holding 40 quarters: Number of wrappers = Total quarters / Quarters per wrapper Number of wrappers =520/40 Number of wrappers = [B]13[/B]

You went to the State Fair and spent $20. If cotton candy costs $2 and a soda pop costs $1. Which eq
You went to the State Fair and spent $20. If cotton candy costs $2 and a soda pop costs $1. Which equation represents the relation between the number of cotton candy (c) and soda pops (s) you can buy? Our total cost for 20 at the state fair is: Cost of Cotton Candy + Cost of Soda = 20 We know that price = cost * quantity, so we have: 2c + 1s = 20 Since 1s is written as s, we have: [B]2c + s = 20[/B]

You were able to send 30 snapchat stories in 9 minutes. At this rate, how many snapchat stories can
You were able to send 30 snapchat stories in 9 minutes. At this rate, how many snapchat stories can you send in 21 minutes? Set up a proportion of stories to minutes where s is the number of Snapchat stories you can send in 21 minutes: 30/9 = s/21 To solve this proportion for s, we [URL='https://www.mathcelebrity.com/prop.php?num1=30&num2=s&den1=9&den2=21&propsign=%3D&pl=Calculate+missing+proportion+value']type it in our math engine[/URL] and we get: s = [B]70[/B]

You work for a remote manufacturing plant and have been asked to provide some data about the cost of
You work for a remote manufacturing plant and have been asked to provide some data about the cost of specific amounts of remote each remote, r, costs $3 to make, in addition to $2000 for labor. Write an expression to represent the total cost of manufacturing a remote. Then, use the expression to answer the following question. What is the cost of producing 2000 remote controls? We've got 2 questions here. Question 1: We want the cost function C(r) where r is the number of remotes: C(r) = Variable Cost per unit * r units + Fixed Cost (labor) [B]C(r) = 3r + 2000 [/B] Question 2: What is the cost of producing 2000 remote controls. In this case, r = 2000, so we want C(2000) C(2000) = 3(2000) + 2000 C(2000) = 6000 + 2000 C(2000) = [B]$8000[/B]

Youngs Modulus-Stress-Strain
Free Youngs Modulus-Stress-Strain Calculator - Calculates any of the 3 items in the Youngs Modulus equation with stress and strain.

Your 14 gallon tank was filled for $32.06. What was the cost per gallon?
Your 14 gallon tank was filled for $32.06. What was the cost per gallon? Cost per gallon = Total Cost / Gallons in Tank Cost per gallon = 32.06 / 14 Cost per gallon = [B]2.29[/B]

Your bill for dinner, including a 7.25% sales tax, was $49.95. You want to leave a 15% tip on the co
Your bill for dinner, including a 7.25% sales tax, was $49.95. You want to leave a 15% tip on the cost of the dinner before the sales tax. Find the amount of the tip to the nearest dollar. Find the pretax cost: 49.95/1.0725 = 46.57 Now, add 15% tip to the pretax bill: 46.57(1.15) = [B]$53.56[/B]

Your brother has $2000 saved for a vacation. His airplane ticket is $637. Write and solve an inequal
Your brother has $2000 saved for a vacation. His airplane ticket is $637. Write and solve an inequality to find how much he can spend for everything else. Let x be the amount your brother can spend. Subtracting the cost of the plane ticket from savings, we have: x <= 2000 - 637 [B]x <= 1,363[/B]

your classmate asserted that x^2 - 4x - 12 and 12 - 4x - x^2 has the same factors is your classmate
your classmate asserted that x^2 - 4x - 12 and 12 - 4x - x^2 has the same factors is your classmate correct Factor x^2 - 4x - 12 using binomials: (x + 2)(x - 6) Therefore, factors are x = -2, x = 6 Factor 12 - 4x - x^2 -(x - 6)(x + 2) Therefore, factors are x = -2, x = -6 Because they don't have two matching factors, your classmate is [B]incorrect.[/B]

Your clothes washer stopped working during the spin cycle and you need to get a person in to fix the
Your clothes washer stopped working during the spin cycle and you need to get a person in to fix the washer. Company A costs $20 for the visit and $15 for every hour the person is there to fix the problem. Company B costs $40 for the visit and $5 for every hour the person is there to fix the problem. When would Company B be cheaper than Company A? Set up the cost functions: [LIST] [*]Company A: C(h) = 15h + 20 [*]Company B: C(h) = 5h + 40 [/LIST] Set them equal to each other: 15h + 20 = 5h + 40 Using our [URL='http://www.mathcelebrity.com/1unk.php?num=15h%2B20%3D5h%2B40&pl=Solve']equation solver[/URL], we get h = 2. With [B]h = 3[/B] and beyond, Company B becomes cheaper than Company A.

Your friend deposits 9500$ in an investment account that earns 2.1% annual interest find the balance
Your friend deposits 9500$ in an investment account that earns 2.1% annual interest find the balance after 11 years when the interest is compounded quarterly 11 years * 4 quarters per year = 44 quarters Using our [URL='https://www.mathcelebrity.com/compoundint.php?bal=9500&nval=44&int=2.1&pl=Quarterly']compound interest with balance calculator[/URL], we have: [B]11,961.43[/B]

Your friends in class want you to make a run to the vending machine for the whole group. Everyone pi
Your friends in class want you to make a run to the vending machine for the whole group. Everyone pitched in to make a total of $12.50 to buy snacks. The fruit drinks are $1.50 and the chips are $1.00. Your friends want you to buy a total of 10 items. How many drinks and how many chips were you able to purchase? Let c be the number of chips. Let f be the number of fruit drinks. We're given two equations: [LIST=1] [*]c + f = 10 [*]c + 1.5f = 12.50 [/LIST] Rearrange equation 1 by subtracting f from both sides: [LIST=1] [*]c = 10 - f [*]c + 1.5f = 12.50 [/LIST] Substitute equation (1) into equation (2): 10 - f + 1.5f = 12.50 To solve for f, we [URL='https://www.mathcelebrity.com/1unk.php?num=10-f%2B1.5f%3D12.50&pl=Solve']type this equation into our search engine[/URL] and we get: [B]f = 5[/B] Now, substitute this f = 5 value back into modified equation (1) above: c = 10 - 5 [B]c = 5[/B]

Your grade must be at least 60 to pass this class
Your grade must be at least 60 to pass this class Assumptions and givens: [LIST] [*]The phrase [I]at least[/I] means greater than or equal to. [*]Let g be your grade [/LIST] We have: [B]g >= 60[/B]

Your grandfather gave you $12,000 a a graduation present. You decided to do the responsible thing an
Your grandfather gave you $12,000 a a graduation present. You decided to do the responsible thing and invest it. Your bank has a interest rate of 6.5%. How much money will you have after 10 years if the interest is compounded monthly? Using our [URL='https://www.mathcelebrity.com/compoundint.php?bal=12000&nval=120&int=6.5&pl=Monthly']compound interest calculator[/URL], we have 10 years * 12 months = 120 months. [B]$22,946.21[/B]

Your job pays you $7 per hour. What is the algebraic expression if you worked h hours?
Your job pays you $7 per hour. What is the algebraic expression if you worked h hours? If your pay is rate times hours, we have: [B]7h[/B]

Your mother gave you $13.32 With which to buy a present. This covered 3/5 of the cost. How much did
Your mother gave you $13.32 With which to buy a present. This covered 3/5 of the cost. How much did the present cost Let the present cost p. We set up the equation we're given: 3/5p = 13.32 [URL='https://www.mathcelebrity.com/1unk.php?num=3%2F5p%3D13.32&pl=Solve']Type this equation into our search engine[/URL] and we get: p = [B]$22.20[/B]

Your profit for mowing lawns this week is $24. You are paid $8 per hour and you paid $40 for gas for
Your profit for mowing lawns this week is $24. You are paid $8 per hour and you paid $40 for gas for the lawn mower. How many hours did you work this week? We know profit from the equation below: Revenue - Cost = Profit We're given Profit as 42, so we have: Revenue - Cost = 42 Let hours worked be h. We have revenue as: Revenue = 8h Cost = 40, so we plug these into profit to get: 8h - 40 = 42 To solve this equation for h, we [URL='https://www.mathcelebrity.com/1unk.php?num=8h-40%3D42&pl=Solve']plug this equation into our math engine[/URL] and get: h = [B]10.25[/B]

Your salary after a 5% increase if your salary before the increase was s
Your salary after a 5% increase if your salary before the increase was s. If we start with s, and get a 5% increase, we will have s + 0.05s. Factor our s: [B]s(1.05) or 1.05s[/B]

Your salary after a 9% salary increase if your salary before the increase was s
Your salary after a 9% salary increase if your salary before the increase was s 9% increase means we multiply s by 1.09 [B]1.09s[/B]

your starting salary at a new company is 45000. Each year you receive a 2% raise. How long will it t
your starting salary at a new company is 45000. Each year you receive a 2% raise. How long will it take you to make $80000? Let y be the number of years of compounding the 2% raise. Since 2% as a decimal is 0.02, we have the following equation for compounding the salary: 45000 * (1.02)^y = 80000 Divide each side by 45000: (1.02)^y = 1.77777777778 To solve this equation for y, we [URL='https://www.mathcelebrity.com/natlog.php?num=1.02%5Ey%3D1.77777777778&pl=Calculate']type it in our search engine[/URL] and we get: y = [B]29.05[/B] [B]Or just over 29 years[/B]

Youre setting sales goals for next month. You base your goals on previous average sales. The actual
Youre setting sales goals for next month. You base your goals on previous average sales. The actual sales for the same month for the last four years have been 24 units, 30 units, 23 units, and 27 units. What is the average number of units you can expect to sell next month? Find the average sales for the last four years: Average Sales = Total Sales / 4 Average Sales = (24 + 30 + 23 + 27) / 4 Average Sales = 104 / 4 Average Sales = [B]26 units[/B]

z , subtract 5 then times by 3
z , subtract 5 then times by 3 Take this algebraic expression two parts: [LIST] [*]z subtract 5: z - 5 [*][I]Then times by 3[/I] means we multiply the expression z - 5 by 3 [/LIST] [B]3(z - 5)[/B]

z fewer than the difference of 5 and y
z fewer than the difference of 5 and y Take this algebraic expression in parts: The difference of 5 and y means we subtract y from 5 5 - y z fewer than this difference means we subtract z from 5 - y [B]5 - y - z[/B]

z is directly proportional to the square of x and y
z is directly proportional to the square of x and y Directly proportional means there exists a constant k such that: z = [B]kx^2y [MEDIA=youtube]J3ByZkcX38E[/MEDIA][/B]

z is jointly proportional to the square of x and the cube of y
z is jointly proportional to the square of x and the cube of y The square of x means we raise x to the power of 2: x^2 The cube of y means we raise y to the power of 3: y^3 The phrase [I]jointly proportional[/I] means we have a constant k such that: [B]z = kx^2y^3[/B]

Z Score Lookup
Free Z Score Lookup Calculator - Given a Z-score probability statement from the list below, this will determine the probability using the normal distribution z-table.
* P(z < a)
* P(z <= a)
* P(z > a)
* P(z >= a)
* P(a < z < b) Calculates z score probability

z varies directly with x and inversely with y
z varies directly with x and inversely with y [LIST] [*]The phrase directly means we multiply. [*]The phrase inversely means we divide [*]Variation means there exists a constant k such that: [/LIST] [B]z = kx/y[/B]

z varies inversely as the square of t. if z=4 when t=2, find z when t is 10
z varies inversely as the square of t. if z=4 when t=2, find z when t is 10 Varies inversely means there exists a constant k such that: z = k/t^2 If z = 4 when t = 2, we have: 4 = k/2^2 4 = k/4 Cross multiply and we get: k = 4 * 4 k = 16 Now the problem asks to find z when t is 10: z = k/t^2 z = 16/10^2 z = 16/100 z = [B]0.16[/B]

z varies inversely with w, x, and y
z varies inversely with w, x, and y Inversely means their exists a constant k such that: [B]z = k/wxy[/B]

Z varies jointly as the 4th power of x and the 5th power of y
Z varies jointly as the 4th power of x and the 5th power of y The 4th power of x means we raise x to the power of 4: x^4 The 5th power of y means we raise y to the power of 5: y^5 The phrase [I]varies jointly[/I] means we have a constant k such that: z = [B]kx^4y^5[/B]

z varies jointly as x and y. If z=3 when x=3 and y=15, find z when x=6 and y=9
z varies jointly as x and y. If z=3 when x=3 and y=15, find z when x=6 and y=9 Varies jointly means there exists a constant k such that: z = kxy We're given z = 3 when x = 3 and y = 15, so we have: 3 = 15 * 3 * k 3 = 45k Using our [URL='https://www.mathcelebrity.com/1unk.php?num=3%3D45k&pl=Solve']equation solver,[/URL] we see that: k = 1/15 So our joint variation equation is: z = xy/15 Then we're asked to find z when x = 6 and y = 9 z = 6 * 9 / 15 z = 54/15 [URL='https://www.mathcelebrity.com/search.php?q=54%2F15&x=0&y=0']z =[/URL] [B]18/5[/B]

z/w=x+z/x+y for z
z/w=x+z/x+y for z This is a literal equation. Let's isolate z on one side. Subtract z/x from each side. z/w - z/x = x + y Factor our z on the left side: z(1/w - 1/x) = x + y Divide each side by (1/w - 1/x) z = x + y/(1/w - 1/x) To remove reciprocals in the denominator, we rewrite 1/w - 1/x with a common denominator xw (x - w)/xw Then multiply x + y by the reciprocal z = [B](x + y)xw/(x - w)[/B]

z=m-x+y, for x
z=m-x+y, for x This is a literal equation. Let's add subtract (m + y) from each side: z - (m + y) = m - x + y - (m + y) The m + y terms cancel on the right side, so we have: z - m - y = -x Multiply each side by -1 to isolate x: -1(z - m - y) = -(-x) x = [B]m + y - z[/B]

Zach can read 7 pages of a book in 5 minutes. At this rate, how long will it take him to read the en
Zach can read 7 pages of a book in 5 minutes. At this rate, how long will it take him to read the entire 175 page book? Set up a proportion of pages to minutes where m is the number of minutes needed to read 175 pages: 7/5 = 175/m To solve this proportion, we [URL='https://www.mathcelebrity.com/prop.php?num1=7&num2=175&den1=5&den2=m&propsign=%3D&pl=Calculate+missing+proportion+value']type it in our search engine [/URL]and we get: m = [B]125 minutes or 2 hours and 5 minutes[/B]

Zachary has 25 country music CDs, which are one-fifth of his CD collection. How many CDs does Zachar
Zachary has 25 country music CDs, which are one-fifth of his CD collection. How many CDs does Zachary have? Let the number of Zachary's CD's be: 25 * 1/5 = 5 country music CD's

Zalika thinks of a number. She subtracts 6 then multiplies the result by 5. The answer is the same a
Zalika thinks of a number. She subtracts 6 then multiplies the result by 5. The answer is the same as subtracting 5 from the number then multiplying by 4. The phrase [I]a number[/I] means an arbitrary variable, let's call it x. We're given two expressions in relation to this number (x): [U]She subtracts 6 then multiplies the result by 5[/U] [LIST] [*]Subtract 6: x - 6 [*]Multiply the result by 5: 5(x - 6) [/LIST] [U]She subtracts 5 from the number then multiplying by 4[/U] [LIST] [*]Subtract 6: x - 5 [*]Multiply the result by 5: 4(x - 5) [/LIST] Finally, the expression [I]is the same as[/I] means an equation, so we set the first expression equal to the second expression to make the following equation: 5(x - 6) = 4(x - 5) Now, let's solve the equation for x. To do this, we [URL='https://www.mathcelebrity.com/1unk.php?num=5%28x-6%29%3D4%28x-5%29&pl=Solve']type this equation into our search engine [/URL]and we get: x = [B]10[/B]

Zero Multiplication Property
Free Zero Multiplication Property Calculator - Demonstrates the Zero Multiplication property using a number. Also called the Zero Product Property. Numerical Properties

Zero-Coupon Bond Price
Free Zero-Coupon Bond Price Calculator - This calculator calculates the price of a zero-coupon bond given a face value, yield rate, and term.

Zoey invested $230 in an account paying an interest rate of 6.3% compounded daily. Assuming no depos
Zoey invested $230 in an account paying an interest rate of 6.3% compounded daily. Assuming no deposits or withdrawals are made, how much money, to the nearest hundred dollars, would be in the account after 12 years? Using our [URL='https://www.mathcelebrity.com/compoundint.php?bal=230&nval=4380&int=6.3&pl=Daily']compound interest calculator with 12*365 = 4380 for days,[/URL] we have a balance of: [B]$489.81[/B]

Zombies are doubling every 2 days. If two people are turned into zombies today, how long will it tak
Zombies are doubling every 2 days. If two people are turned into zombies today, how long will it take for the population of about 600,000 to turn into zombies? Let d = every 2 days. We set up the exponential equation 2 * 2^d = 600,000 Divide each side by 2: 2^d = 300000 To solve this equation for d, we [URL='https://www.mathcelebrity.com/natlog.php?num=2%5Ed%3D300000&pl=Calculate']type it in our math engine[/URL] and we get d = 18.19 (2 day periods) 18.19 * days per period = 36.38 total days Most problems like this ask you to round to full days, so we round up to [B]37 days[/B].

zy-dm=ky/t for y
zy-dm=ky/t for y Isolate terms with y to solve this literal equation. Subtract zy from each side: zy - dm - zy = ky/t - zy Cancel the zy terms on the left side, we get: -dm = ky/t - zy Factor out y: y(k/t - z) = -dm Divide each side by (k/t - z) y = -dm/(k/t - z) (k/t - z) can be rewritten as (k - tz)/t We multiply -dm by the reciprocal of this quotient to get our simplified literal equation: y = [B]-dmt/(k - tz)[/B]

Zyrelle is now 20 years older than her sister. Find the present age of Zyrelle
Zyrelle is now 20 years older than her sister. Find the present age of Zyrelle Let Zyrelle's age be z. Let her sister's age be s. Older means we add, so we have: [B]z = s + 20[/B]

___is the probability of a Type II error; and ___ is the probability of correctly rejecting a false
___is the probability of a Type II error; and ___ is the probability of correctly rejecting a false null hypothesis. a. 1 - ?; ? b. ?; 1 - ?; c. ?; ?; d. ?; ? [B]b. ?; 1 - ?;[/B] [LIST] [*]H0 is true = Correct Decision 1 - ? Confidence Level = Size of a Test ? = Type I Error [*]Ho is false = Type II Error ? = Correct Decision 1 - ? = Power of a Test [/LIST]

? = 5, ? = 4 ; calculate P(0 < x < 8)
? = 5, ? = 4 ; calculate P(0 < x < 8) This is the same as P(x < 8) - P(x < 0). P(x < 8) [URL='https://www.mathcelebrity.com/probnormdist.php?xone=8&mean=5&stdev=4&n=1&pl=P%28X+%3C+Z%29']using our calculator[/URL] is 0.773373 P(x < 0) [URL='https://www.mathcelebrity.com/probnormdist.php?xone=0&mean=5&stdev=4&n=1&pl=P%28X+%3C+Z%29']using our calculator[/URL] is 0.10565 So we have 0.773373 - 0.10565 = [B]0.667723[/B]

“The shortest person in a class is 55 inches tall and tallest person in that class is 75 inches tall
The shortest person in a class is 55 inches tall and tallest person in that class is 75 inches tall. write an absolute value equation that requires the minimum and maximum height. Use X to represent heights. We write our inequality as: [B]55 <= X <= 75[/B]